Download as pdf or txt
Download as pdf or txt
You are on page 1of 1132

−ν

(σ  ∂τ ∂τ yz ∂σ∇z ⋅ σρ+ ρb = 0
y
[ σ
y +σ
)]  ∂x ∂y ∂z



E z =

3
 y −ν

X l
−µ

2X
+ + + b =
(σ +


xz


y +σ
0 ε = 1 3 ,33
)]

X l 
z
 ∂x ∂y ∂z

2
3× −
[

2
]
z
σ 10 5 r r r

0
z =

l
t (nˆ ) =∇σ⋅⋅σn

3
x
σ )

l 2 3 
(σ + −µ
ˆ + ρb = 0
3

2
X

,3 323x×1 a − λ −r4ax 2 − λ − 4ax 2 z − ν σ


E
( =−
z

−µ

2X


2

nˆ )∂σ = 0∂τ 2 x1 a ∂τ γ
2

2,31
µ X

]
X 2

x σ 10 −5 (

z )= xz y ρ= 1 = x +σ
−µ l

t = σ n
ˆ
3

0 8× −
− 4ax 2 2ax1 − λx +
)]
x

SOLVING PROBLEMS
xy
+ + b =
l2

−2  − 4 ax 2 ax 1x τ
− λ 0 10 5

−µ
y =

2 3
∂ ∂ ∂

µ X 0
G

2X
2
, 3  x y z y = −
8⇒× (2 x1−a − λ )(2ax1 − λ )− 16
x
+σ 1 x(22ax γa=−0 λ )(2ax − λ )2−,16
2 2
4,
)]  ∂τ xy ∂σ⇒ ∂1τ xyzy = ρ1 1 2 x1 a5−×xλ120a −5−=40ax 2 348 × −

X 2
2 2
10 5  ∂σ x ∂τ xy ∂τ

−µ l

3
y =
 0

ε −4,13 ( )  (4ax 2+) ⇒ +


y
+G2bτy = 0 − 2 =0 10 6 + +

0

l2


⇒ − λ 2
= 2
x =
2 x a ∂y (γ2 x∂1xaz − λ ) =xy(4=ax02 )x41 ax 2 2ax1 − λ
48 ×
[ ∂x ∂x ∂y ∂
1

0

=
E σx −10 −6
0 5 − dx1 t
(  τ  λ
⇒ ∂τ 1yz + ∂σ
= 2x x =
a − 1 4 ax ∫
⇒ (2λx11 a==−2 xλd1)( a − 41ax−2λ )− 16 x 2 a = 0 
 ∂τ xy ∂σ y ∂τ
BY MEANS∫ OF
y 2 2
2 x1 a − λ = ±4∂ax


⇒ νσ 1 2
τ=ρxb±4ax t 2ax⇒

0
ε 2 +  ⇒2

ij
ε
y = 1
xz
λ = x a −z
G λ
+
+ = = 0 ⇒ X1 x1 ε   + +
[ − yλ + σ− 4ax2 ∂x x =l2n1


)] ∂y2 2 x∂1za 4ax 2 0 ⇒ (λ =−02λx )a2 + ( ) [ x1  ∂x ∂y ∂x
t 1 y z 2
σ 2 x a = 4 ax  
∫ dt E
1
⇒ y −ν−x(4ax 2 2ax1 −3λ,3
z = = 0 x 2 x 21 a 1 4 ax 2 2  σ  = t
E
(
2
dx2 Xx −  ∂τ

z = 1 ln 1σ  33 × ∫ =

t
ε λ 1 1=2 xν1 aσ− 4ax 2 x1 =

 +
∂τ yz ∂σ
+
[ d xz
x =+ σ t = 1
σ 

1 2 x a z ) =2ax1
(  t )( ] ) 10 −52 2 t (nˆ ) = σ ⋅ nˆ X⇒
r x22 x1 a − λ = ±4ax 2 ⇒  y
[ y +σ
)]  X ∂ 1xexp t ∂y ∂
CONTINUUM MECHANICS
⇒ X − λ ⇒ − λ − = 
16 x a 0 0 2t + 3 ⇒ E σλlyn2−=x22 x1 a + 4axz2 = 3
E z −
νσ  1 (
2

−2 2xx1 =a −X λ 2− 4ax
dt  ( = ln
ν
(
x =

ε = 1 ,33
x (+2σx1 a − λ )2 = (,43ax 1182×)2 1 exp t X3
 X2 σ

2t=+ 13
3
⇒ ⇒

3× −
] ) ( )
2
=0
)] [ x + σ 2t +

z
10 5 r

1 
τ l n  x  − 4 ax 0 − 2 ax − λ σ ) 3 − t(

 ==ln−4
l 2 

l
(
5
G xy = y n
 E
( =
2
z −ν
,3 +⇒2 (⇒ λ−1∇=λ ⋅)(
2
2 x1 a − r4axr2
1
2

l 2 
z

−2 3
) ( )
0

2,5 ×⇒ X
2


3

+ 1ρ−b λ=)0− 16 x 22 a 2 = 0∇ ⋅ σ +γxρb



σ
X

10 2−5x21a − λ = ±24t4ax
r r
2 3 

8 3× 2−xln1−aλ = 2σ2xax
0

,31
3

1
X

l 2 3  = = 1
0 + x

8
+ σ
)] ×


1 a 4 ax
−µ

y
2X

x

G τxy = x1 0 6 23 2 ⇒ τ 10 −5
−µ

2X


 1 2
⇒ (2 x1 a − λ ) = (4ax 2x ) G xy = =−
X

2 y


0 dx
X

2 = X γ 4,3
l

2,5 ×

X l 
∫ 2
−µ

t
48 ×

2
 l2 3

= 1
µ X 0
2X

1 r r
∫ 1
−µ

0
= + x
2 3

3
λ 1 = 23x1 a −14ax 2
2 y
⋅ −
0+ ρb = 0
µ X 0
2X

τx
X
10 − 6

l 2 3 
dt τ ∇ σ 5

2
X1 x1 ⇒ ⇒  2 x a − λ = ±4ax ⇒
2

y =
ln x1 1  ∂σ x + ∂2τ xy +λ∂τ2 xz= 2+xρ1 ab+γ∂=4σax Gτ xy = x1 2 x1 a − λ − 4a
x x 0x= 2+1 Eduardo
−µ ∂τ0xz dρx W. V. Chaves
2X
X 2


3

0 x2 ∂
µ X 2 µ X

0

3
0

2

  = t 

xy
+ + 1b = 0
0

l2

t

 ∂⇒ ∂y ∂z  y − 4ax 2 − 2λax1
l

dx2  X1 
∫ x ∂zX x =x dt ∫
0

∂x τ∂xy

t
−µ

2 x1 a

−µ

2 3


0


µ X 0

=
2X

x

dt = G ⇒
ij =

y =  x⇒ (

X 2 x2  ∂τ xy r ∂σ1 y X1∂eτxyzp t 1
− λ2 )(2a2
 ij =


l
1

+ ρby ∂=τ xy 0 n x−1 a4ax


0 + ∂σ y + ∂τ xyz2 +∂ρ  2
 00

r
X 2

0
3



ε

ε0 2t +13  x ∇ ⋅ σ + ρ+b = 0 + σbx =∂0τ xy 2∂τx1xza −λρ −=4tax


1

0

l

3
ε
2

= l n ε 
[   x =∂x1 ∂y ∂x z d 
l 2 3

yt+ + X
+ b1 =20x a −⇒
x3 = X x
([ ) ∫ ∂z4ax 2 ⇒x2(ax (2λx1λa=)−02 λ=
0

∂x z x2∂=
2

2

 2
l
2X

 ∂x ∂y

E σx −  X 2  = ln 2σ d∂ty ⇒1−
l


ij = 

−µ

x −


0

(
3

(( )
X

t +∂3τ x
X 2 

νσ ∂τE ln ∂σ3z + ρb  ∂=2τx0xz1 a − ∂λτ yz − 4∂ax


1
 xz +x − yzν− σ
X2
ε σ z2 2 ∂ρτ xy0 2t ∂+σ3y ∂⇒ ⇒ (2−x−λ1 aλ=)−±

y + σ εy =  1∂
+ += 0b z += 0 + ⇒τ(yz2 x+1 aln ρ−b⇒λx)( −λ4
µ

y = 1
2 3

[

X l 

 =2x01ax
µ X 0
2X

+ +
 0

)] [ ∂y y ∂+zε ⇒ 22 a1
(

z
x3 = X
ε

x 

)] [
σ r1 ∂rxx
0

σ ∂ ∂ =
3

σ, x σy∂τ−xy ∂τ∇xz ⋅ σxρz+=ρ=b = 0− 42 ax = 2X y 2ax 2 1 −z λ 3∂x  X  ln


y

z = ∂y ∂x
X 2

E y − ∂3E
2

ν(σ ⇒ (z2 x1 a − λ2)2⇒= 2(4xaxa 22−)t 2λ+


l
3

l 2 3

ε2 x=1 a 1− λ − 4ax 3 +3 ×∂y −(σ ∂zr nˆ + + = σ +


−µ

b
2X

ν 3 t
0


l2

2
 3 0
[
2
x =+0σ εz = 1∂x
( E
x ,3⇒3x 3(−2 νx a − λ3)(2ax − ∂λτ)− 16 x∂τ2 ayz2 = ∂0σ
1 1
=

−µ

] [
z
1 5 x +t ε ⋅ × 1σ − ρ
z )=
 0
0

1 
σ λ
]
− 4Eax 2 2ax1 − λ σ
( )
= σ n
ˆ 1 r xz
+ 2
+ z
+ b =
 σ∂)x⋅2nˆ ∂y ⇒∂z2 x1 a − λz = ±−4λ
0
2∂τσxyz − ∂σ y ∂τ yzyz =) = σ ⇒ (2 x a y− +λ )σ2 t= (4ax
1 0 5
[

( =
( )
z −ν
)]
−µ

− ⇒
2 3

− 4aλ
( E ax
0
2X

γ

σ +ρ Eb−y2=,y30−
= ε(12x x=1 a 1− λ )(2axx 1 +−σλ )γ−xy 16= x 221a 2,∂3=108 +×ν∂σx +
2 x1 a
0

2
z =
ij

xy ⇒ + 1 2

µ X

)] ε∂x∂=z σ x1 ∂τ xy 18 ×∂(τσxz − ρ ν
X 2

3

,33 λ = 2 x a − 4ax − 4ax


[


−µ l

)] [ x 10y−5 σ

τ  3 ×1 2 ax


σ)2 = (4ax )2 = −G τx + + 10x ++5 b = 0
0

l2

⇒G(2 xx1yaE=− 2λ

z
]
y 1 r 2 1
4,3 ∂yτ = 2 ∂τ yz ∂yσ ∂E=x −ρ4σ ∂y ⇒ 2∂xz1 a − σλz =)x±=4ax 2 ⇒ λ10=−52 x a t+(nˆ4)ax
2
= σ ⋅n


x5 − ν
(

γ ˆ
2 ⇒ (2 x1 a − λ )(2a

l 2  
,
0

×
(
10 σ−γ5y xy+ =λ1 =  8 ×−1 +,5− ×1 γ+−  ∂τ+ b,z3z∂4−σ=8ν×0 σ ∂τ
2
xy = ε1

4  2

2
−2

23 X  
xz z

0
1
y = 1
[

3
ij =

X ll 2  3 
,31
τ
)]
σ 2x1∂ax 04ax6∂2y 0 xy5 =∂z 1 xy

y 10 + ρ
0

G⇒ x2yEx1=a σ− λx1 = ±4ax 2 ε⇒ Gz τ= − 8 ⇒ (2 x1 a − λ )2 =


)] ×1 −5
1 yz
σ + by = 0
ε

 τ + + x 6

−µ


2
γ 0 y −dxν γ t x =λ12 =xy32,=x10a +x14ax 2 0

1(σ [ ∂ ∂ ∂

0
=−

[ ∫
xy = ε1 3 G x x y x y
=xxy+d= 1E σ 33 × 1dx1γ− = t r (nˆ ) y = 2,5 ×

l 2 3 
X
(∫ ∫

2
z 00 − µ 0l 2 2 X  
z = 1 4,3
τ ∫ ]
− µµ
σ −

2X
y =dtt ∂τ=xzσ ⋅ n 4


G xyE = σ 1 1 ε Gz )τ⇒ x t 0 1 ∂ τ ∂ σ
 µ X 2 l 2 3
ν 0 + b = 0 8 × 10 − 6
0
2

x
 5
x ˆ 1 ρ ⇒ 2 x1 a − λ = ±4
xy −= ln 2Xxσ
X −
x
0

x111 a21 − λ − λ− 4τax


( 0y = 1 =
µ X3

yz
⇒ +

0 z −x2 ν  + 5z
X

− µ2 XX 

[ +x a − 4 ax x  l 22 X3332
)] l

γ 0   G 2
= n 
dxσ
2 ,31 Xx2− γ = tσ y 0 
 ∂y−xλ = 0 ∂1=y 0  ∂=z
0 z

−µ

1 x 2
σ 1
µ

x

xy = 1 2
+
∫ )
σ E y − ν ×1d1x02xy−−524=ax
]
8 ⇒
= 1
0

4 ax 2 ax dx  t t


X l 

t

(σ⇒+ = t−1λ2τdt3,323ax3x11−×=−λXλX−1xeXx1p1 =t2r 2dt= ⇒ x1 = X ∇ ⋅ σ + ρbr = 0r


x z

X

G τxy =X 2 x2 =εz = y d1t = −


2
2


l
− µ l ij =  


X 2


0
3

X 2x x2(2σx1 aG )( =)(12ax10)1−−λ516
X

l
l 2 3 

[
3
X 2 

 x 1 ex p t
3

γ 2,5 4,⇒
] ⇒ (2 x a x−y2λax ) x16 2 a(n xˆ )2 a02 =⇒
µ X 0

ε 3  0
2X

⋅ 0ˆ
l2
0 2t +
x3 = X× 10 −5 E 3σz − 48 ×x1ln − x2 z)0 =21t−+ 23 0 ⇒1x2 2 ln x0t2 2 = σ n ln 1  =
3 1 −

−µ

2X

xy = 1

l  

−µ

(
X l 

( )∫
ε

(620⇒
rx1 a=−lnλ2), = (24ax 2 )dx 2 t  ε
2

G τxy = x1 γx3y = 1 ν σ 3⇒ 
r0=XX
( )
23  (2 x1 a −2 3tλ1+)8 = (4ax22 ) X  = ln  X 1  t
0  

0

x = 1
[
3


X



ij 2=
l 2 3 

∇ σ + ρ b =
0

+
( )∫


2

)]
−µ

× σ
2 3

0 dx σ =
( )
3 − 2 + x
µ X 0

=
0

x d
2X


1X 20 lxn−52λ31 = 2 x1 a − 4ax 2 3 − ln t

t
−µ


2X

τ X1 εE
[(

2
2= 1 e tx − 
0

t 1
x
X l 

x1=a2−−
=

p
⇒ ⇒ 0 λ21t⇒  ∂σ x ⇒∂τ xy εy ∂=τ xz1 x ρ= σν σ 
=+2 x1 a −⇒4ax 2  3

=0 = G 1
1

y
x =
2

2,5 ×  ⇒ 2⇒ λ=±
X 2

G τxy = X1 γx1 dt y ⇒ x41,a34− λ4=ax±x24ax


l

3 x

 l
[
X

x n  x+ = X +σ 2bEx = 0x −y +
l 2 3 

λ = +

2+ =
8 × −3 = X 3 2 λ22 x=1 a2 x1 4aax X 22 2   = +ε
(
 0

2
0

0 x2 xy = 01
l2

ij

(

−µ

2 2
ε
3

ln1 0x1−5 

24 ax
X

 Xt 2+1∂x ln x 2∂=ty +1 2 ∂εE ν


µ

10 6
)
2X
−µ

τ  
[ ( )[ z 2 y t−+
0

=
2

y = 31 ν1(σ

=


2
X

dx2 Gt xy = x∂1 σ  X∂1 τxy t ∂τ ⇒ 3


X 2

ε 3σ− ln

−µ l

γ 0 dxx + t  + xz + ρbx == 0  ∂τ xy ∂σE


3

x1 a − 4ax 2 zy = ∂x1τ −
∫ [ ( 3 σ + 

= d
∫ ρ
l2

+ σ ν+Eσ b y y⇒ =−0ν xσ


−µ

 1
 0

X 2 yx2=
1 t
3

x
ij

yz
µ X 0

ε
ε

+
2X

X1 eεxp t

x 1x
 (σ2
∂ x = ∂
∫ y ∂ z r r
∇ ⋅ σ + ρb = 0  ∂x y = ∂1y εEz ∂x=z z 1− ν
[ [ ( )]
0 τ2t + x = 1
x1 a + 4ax 2 X ⇒x  dt
[ y +σ
2
X 2

x3 = X G xy = 30 1 ∂τ1xy ln0∂σx2y  ⇒ x  σ γ σ σ


l

∂ τ
( l n  E x −
(  ∂τ E∂xyτ = y1−∂σE σ x + σ= 3x
X 2 − − 20 3  

= ln yz + ρb 1 = =0
0

( )]
2

εz xz= +1 yzγG+ τxν(zσ+ ρzb −


 0

 ν
+ +
) ( )[
x2
ε  νσ y ,

dx X  εt
µ Xµ l X 2  

σ
l

3
−µ

x = 1
∫ ∂2x t ∂y2  ∂x z 2t +X31y − y = ⇒ z =0
[
0

1
[
− µ 2lX32   

+
=

 l + =
 n 3 σ⇒ x1 r= Xr e tσz )]  ∂xγ σ∂y xy = ∂z1 2, σ
]
2

=
∫ 5 ×z ) =− x + σ
y y x
3 

dt
E σx −
0

∇ ⋅ σ y+−ρb = 01 xp
2X  

X 2 x2
X

( = Exy = z 1−
(

∂ τ E τ
 0

ν ∂ τ ∂ σ 1 −
ij

ν(xσ2 = X 2 γ ,33 ν
l 2 3 

⇒ + ρlnbz ε= 3 τx εσGxx =+xy1 = 2 0 5 2,3


ε σ  xz 0+ t + 3 + 2 yz
−µ

z xz 20= 1
y = 1
[ y + σ x3 = X∂
[( + = 3 1× γ G [)]

)]   σ 3 t + 1 γ τ10 x−y5 = 1t=(nˆ0) E =σσdy σ


,5
ˆ ×t 10 −5
x
∂y ∂z
]
r
2

2 y
 3x  X 2 E= lnσ 2t + ⋅
x y x
E σy − )
() ( )
n
1
X

z =
∫ (
l

z −ν 3 − = G y = xy1 = τx x =
1 −−ν
ν(σ
z
ln 3 γ −2,3
µ Xl 2 0 3

x x
3,33 2ε,G
0


2X

ε r= 1r γ σ = x1= 4d,3
tσ4yt8+×
µ X 22 X3

τ 5
xy =y ×=10 X−15 01
1
[ 3 × 1 −5
[ ∫ x
y
ρz b = 0 x +σ xy = 1 xτ + σ xy = 118 γG
  µ X


]
r
z )=
t = σ ⋅n  ∂σ
)]
∂ × d x
32

0 n ∂τ 1x0y −=5 21 0 E x2 σ
E σz −
( ˆ )
ˆ

G τxy = x ∂+σ x + ∂τ+xyy +=xz∂−τ+xzρb+xG ρ=bx20τx==y X


01

( ∫ = d
l

xy
y −ν
ij =  

3
0

−µl 2

x1 τ
(

νσ −2 = x
ε  0 0

ε t
l2

0 G d 1σ
γ ∂ 2 ∂ ∂ γ 4 0
∫ t + = x
2
,3 x
  5∂x× , y z ,
xy∂z=341 z3 = x1 1
X t
∫ [∫ d 1
x x
= 1 x +σ 18 × ∂y y
0 x2 = xdt0+ σ

1

2
xy = 1

t

)] 8
0  0

τ 10 −5  ∂τ 1
∂σ

0 5 ∂τ × 1 −6 =Xσ x
G τxy = xyx1∂τ+xy y ∂+σ y yz∂τ+yzρGb yτρx=y 00= γ X1 x1E 2z d−t2 ν d0⇒
ij =

∫( ∫
y =
∂G
x22t +t z
τ xy xy ∂=τ xz2, ρ −4

ε

ε
 0

, ε γ 0 + + + b =0 σ = l3nd

x = 1 ∂x d∂x1 ∂yt ∂ ∂x z ∂


5+× b x− = 0 348
[ [ 0 x 3 = X
0
1 + xy x2= 1 X 2 x2x + σ 
∂y ∂z 1 0 5 × 10 − 6 E σ σ x = 1 x y = 1  ∫ x
 ∂Xτ  x ∂=τ dt ∂σ ∫
y x
y
dx2 τ t )] 0 y2t + X
G τxy = x1 ∫
3
x −ν G τ
z

∂σ y 0∂τ yz dx ρ t ε ε 1
y =
E x −( (
σν σ x y = ∂ τ ⇒
0  x2  ∂τ+xz +0 +yz + ∂+σ z bl+nzρ=bx10γ=xy 0=X 2 1x2
1 xz 1 yz z ρ  G =xy = xdt =
∫ 23,5 ×X 3
τx ∂y
+ ∫
∂Xx1 z x1
+1 =b y = 0
dt ∫ E
y = 1
σ σ [ [ −
y +σ
y +
z σ)] )]∫
=

 ∂ x
dx2∂x t ∂y ∂ y ∂ z ∂z  X 1
 z  = t
 x3G= ⇒ τx 0 2 t + 3 10⇒ −5
ln
y = ⇒ ε x  E yν(−σ 3,3=33 = X 3y = x1 =x1

y z d
0 l n z 1=ε 1 ν (+σ X x32,3 ×33
t γ 0 X
dx1 expt t
[ ∫
0
∂τ ∂x   1  1
II MECÁNICA DEL MEDIO CONTINUO
Nomenclature III

Solving Problems by means of


Continuum Mechanics
EDUARDO WALTER VIEIRA CHAVES
IV MECÁNICA DEL MEDIO CONTINUO
Presentación

Presentation

er

s
transf

Co
anic
nv s
ec
tio h
ulic
mec
n- a
dr
at

dif y
e

fu H
H

sio
Soil

n s
eam
B
Flu

s
Structure Plate
s
ids
x

Flu

lids
So

IBVP and Solution Strategies


Rigid Body Motion
Constitutive equations

Fundamental equations of C.M.

Stress

Continuum kinematics

Tensors
VI MECÁNICA DEL MEDIO CONTINUO
Abbreviations VII

Contents

Contents
VIII MECÁNICA DEL MEDIO CONTINUO
Abbreviations

Abbreviations

IBVP Initial Boundary Value Problem


BVP Boundary Value Problem
FEM Finite Element Method
BEM Boundary Element Method
FDM Finite Difference Method
C.M. Continuum Mechanics
iff if and only if

Latin

i.e. id est that is


et al. et alii and the others
e.g. exempli gratia for example
etc. et cetera and so on
Q.E.D. Quod Erat Demonstrandum which had to be demonstrated
v., vs. versus versus
viz. vidilicet namely
Operators and Symbols

Operators and Symbols

• +•
〈•〉 = Macaulay bracket
2
• Euclidian norm of •
Tr (•) trace of (•)
(•) T transpose of (•)
(•) −1 inverse of (•)
(•) −T inverse of the transpose of (•)
(•) sym symmetric part of (•)
(•) skew antisymmetric (skew-symmetric) part of (•)
(•) sph spherical part of (•)
(•) dev deviatoric part of (•)
• module of •
[[•]] jump of •
⋅ scalar product
det(•) ≡ • determinant of (•)
D•
≡ •& material time derivative of (•)
Dt
cof (•) cofactor of • ;
Adj(•) adjugate of (•)
Tr (•) trace of (•)
: double scalar product (or double contraction or double dot product)
∇2 Scalar differential operator
⊗ tensorial product
∇ • ≡ grad(•) gradient of •
∇ ⋅ • ≡ div (•) divergence of •
∧ vector product (or cross product)
I • , II • , III • first, second and third principal invariants of the tensor •
r vector

•ˆ unity vector
1 Second-order unit tensor
I fourth-order unit tensor
I sym ≡ I symmetric fourth-order unit tensor
SI-Units

SI-Units

length m - meter electric current A - ampere


mass kg - kilogram amount of substance mol - mole
time s - second luminous intensity cd - candela
temperature K - kelvin

m
velocity energy, work, heat J = Nm - Joules
s
m J
acceleration power ≡ W watt
s2 s
energy J = Nm - Joules
force N - Newton permeability m2
N
pressure, stress Pa ≡ 2 - Pascal dynamic viscosity Pa × s
m
1 kg
frequency ≡ Hz Hertz mass flux
s m2s
thermal W J
energy flux
conductivity mK m2s
kg J
mass density energy density
m3 m3

Prefix Symbol 10 n Prefix Symbol 10 n


pico p 10 −12 kilo k 10 3
nano η 10 −9 Mega M 10 6
micro µ 10 −6 Giga G 10 9
mili m 10 −3 Tera T 1012
centi c 10 −2
deci d 10
XII SOLVING PROBLEMS BY MEANS OF CONTINUUM MECHANICS

Physical Constants

m3
Newtonian constant of gravitation: G = 6.67384 × 10 −11
kg s 2
m m
Speed of light in vacuum: c = 299 792 458 ≈ 300 000 000
s s

Absolute zero (temperature): T = 0 K = −273.15º C


Nomenclature

Nomenclature
r r r r m
A( X , t ) ≡ a ( X , t ) Acceleration (reference configuration)
s2
A Transformation matrix
r r m
a ( x, t ) Acceleration (current configuration)
s2
Continuum medium in the reference configuration at
B0
t=0
Continuum medium in the current configuration at
B time t
∂B Boundary of B
r r N
b( x , t ) Body force (per unit mass)
m3
b Left deformation Cauchy-Green tensor, Finger
deformation tensor
B Piola deformation tensor
J
B Entropy created inside
sK
J
b Local entropy per unit mass per unit time
kg s K
Ce Elasticity tensor Pa
[C ] Elasticity matrix (Voigt notation) Pa
C in
Inelasticity tensor Pa
c Cauchy deformation tensor
Cv Calor específico a volumen constante
Cp Calor específico a presión constante
c Cohesion Pa
mol
cc Solute concentration
m3
C Right deformation Cauchy-Green tensor
m
DV Dilation
m
D Rate-of-Deformation tensor
r
dA Area element vector in the reference configuration m2
r
da Area element vector in the current configuration m2
dV Volume element m3
XIV SOLVING PROBLEMS BY MEANS OF CONTINUUM MECHANICS

E Green-Lagrange strain tensor, or Lagrangian finite m


strain tensor, or Green-St_Venant strain tensor m
m
e Almansi strain tensor, or Eulerian finite strain tensor
m
E Young’s modulus, or elastic modulus Pa
ê i Cartesian basis in symbolic notation
ˆi , ˆj, kˆ Cartesian basis
m
F Deformation gradient (pseudo-tensor)
m
G Shear modulus Pa
H Biot strain tensor
J
H Total entropy
K
r kgm 2
HO Angular momentum = Js
s
m3
J Jacobian determinant
m3
r m
J ( X , t) Material displacement gradient tensor
m
r m
j ( x, t ) Spatial displacement gradient tensor
m
r mol
J Diffusion tensor
m2s
W J
K Thermal conductivity tensor =
mK smK
K Kinetic energy J
r kg m
L Linear momentum
s
m
l Spatial velocity gradient
sm
m Mass kg
M Mandel stress tensor Pa
Outward unit normal to the boundary (current
n̂ configuration)
Outward unit normal to the boundary (reference
N̂ configuration)
r N
p Body force (per unit volume)
m3
P First Piola-Kirchhoff stress tensor Pa
p Thermodynamic pressure Pa
r r J
q( x , t ) Cauchy heat flux (non-convective vector)
m2s
Q Orthogonal tensor
Q Thermal work J
r
r ( x, t ) J
Radiant heat constant, or heat source (per unit mass)
kg s
NOMENCLATURE XV

R Orthogonal tensor of polar decomposition


S Second Piola-Kirchhoff stress tensor Pa
r J
s Entropy flux
kg s m 2
T Biot stress tensor Pa
r ˆ r
t (n) ( x , t , nˆ ) Traction vector (current configuration) Pa
r (Nˆ )
t0 Traction pseudo-vector (reference configuration) Pa
r
T ( x, t ) Temperature K
t Time s
t0 ≡ t = 0 Initial time s
J
U& Rate of change of the internal energy =W
s
J
u Specific internal energy
kg
r r
u( x , t ) Displacement vector (Eulerian) m
r r
u( X , t ) Displacement vector (Lagrangian) m
r
U( X , t ) Right stretch tensor, or Lagrangian stretch tensor, or
material stretch tensor
r
V ( x, t ) Left stretch tensor, or Eulerian stretch tensor, or
spatial stretch tensor
r r r r m
V ( X , t) ≡ v ( X , t) Velocity (reference configuration)
s
r r m
v ( x, t ) Velocity (current configuration)
s
Spin tensor, rate-of-rotation tensor, or vorticity m rad
W =
tensor ms s
J
w int Stress power =W
s
r
X Vector position (material coordinate) m
r
x Vector position (spatial coordinate) m
1
α Coefficient of thermal expansion
K
δ ij Kronecker delta
ε1 , ε 2 , ε 3 Principal strains (infinitesimal strain)
m
ε Unit Extension
m
 ijk Permutation symbol, or Levi-Civita tensor
components
Linear dilatation (volume ratio) (small deformation m
εV
regime) m
m
ε Infinitesimal strain tensor
m
J
η Specific entropy
kg K
κ Bulk modulus Pa
XVI SOLVING PROBLEMS BY MEANS OF CONTINUUM MECHANICS

m2
κ Thermal diffusivity
s
m
λ Stretch
m
λ, µ Lamé constants Pa
ν Poisson’s ratio
kg
ρS Solute mass density
m3
kg
ρf Fluid mass density
m3
r kg
ρ 0 (X ) Mass density (reference configuration)
m3
r kg
ρ ( x, t ) Mass density (current configuration)
m3
1 m3
Specific volume
ρ kg
σ Cauchy stress tensor, or true stress tensor Pa
r
σN Normal traction vector Pa
r
σS Tangential traction vector Pa
σm Mean stress Pa
σ1 , σ 2 , σ 3 Principal stresses Pa
r
σ oct Normal octahedral vector Pa
r
τ oct Tangential octahedral vector Pa
τ max Maximum shear stress
τ Kirchhoff stress tensor Pa
φ Angle of internal friction
J
ψ Helmholtz free energy, specific (per unit mass)
kg
J
Ψ Helmholtz free energy (per unit volume)
m3
J
Ψ (ε ) = Ψ e Strain energy density
m3
Solving Problems by Menas of Continuum Mechanics

Useful Formulas

Some Trigonometric Identities


sin(θ ± φ ) = sin(θ ) cos(φ ) ± cos(θ ) sin(φ )
cos(θ ± φ ) = cos(θ ) sin(φ ) m sin(θ ) sin(φ )
1
cos(θ ) cos(φ ) = [cos(θ + φ ) + cos(θ − φ )]
2
1
sin(θ ) sin(φ ) = [cos(θ − φ ) − cos(θ + φ )]
2
1
sin(θ ) cos(φ ) = [sin(θ + φ ) + sin(θ − φ )]
2
1
cos 2 (φ ) = [1 + cos(2θ )]
2
1
sin 2 (θ ) = [1 − cos(2θ )]
2
θ +φ  θ −φ 
cos(θ ) + cos(φ ) = 2 cos  cos 
 2   2 
θ +φ  φ −θ 
cos(θ ) − cos(φ ) = 2 sin   sin  
 2   2 
θ ±φ  θ m φ 
sin(θ ) ± sin(φ ) = 2 sin   cos 
 2   2 
cos 2 (θ ) + sin 2 (φ ) = 1
sin(θ )
tan(θ ) =
cos(θ )
1
sec(θ ) =
cos(θ )
1 cos(θ )
cot(θ ) = =
tan(θ ) sin(θ )
sec2 (θ ) + tan 2 (φ ) = 1

sin( x) 1 − cos( x)
lim =1 ; lim =0
x →0 x x →0 x

List of trigonometric identity


XVIII SOLVING PROBLEMS BY MEANS OF CONTINUUM MECHANICS

http://en.wikipedia.org/wiki/Trigonometric_identity

Some Series Expansions


∂f 1 ∂2 f 2 1 ∂3 f
f ( x) = f (a ) + ( x − a) + ( x − a ) + ( x − a ) 3 + L (Taylor’s series)
∂x 2! ∂x 2 3! ∂x 3
n(n − 1) 2
(1 + x) n = 1 + nx + x +L ; ( x < 1) (binomial series)
2!
1 1
exp x = 1 + x + x 2 + x 3 + L
2! 3!
1 3 1 5
Ln(1 + x) = x − x + x − L
3! 5!
1 1
cos( x) = 1 − x 2 + x 4 − L
2! 4!
1 3 1 5
sin( x) = x − x + x − L
3! 5!
1 2 1 4
cosh( x) = 1 + x + x + L
2! 4!
1 1
sinh( x) = x + x 3 + x 5 + L
3! 5!
1 1  π
tan( x) = x + x 3 + x 5 + L x < 
3 15  2

Some Derivatives
d d x d 1 d 1
(exp x ) = exp x ; (a ) = Ln(a) a x ; [Ln( x)] = ; [log a ( x)] =
dx dx dx x dx xLn(a )
d 1 ∂f ( x)
[Ln( f ( x))] =
dx f ( x) ∂x
where e ≡ exp stands for exponential and Ln for natural logarithm, where it fulfils:
Ln(exp x ) = x and exp Ln( x ) = x

d d d
[sin( x)] = cos( x) ; [cos( x)] = − sin( x) ; [tan( x)] = sec 2 ( x)
dx dx dx
d 1 d −1 d 1
[arcsin( x)] = ; [arccos(x)] = ; [arctan(x)] =
dx 1 − x2 dx 1 − x2 dx 1 + x2

List of derivatives
http://en.wikipedia.org/wiki/List_of_derivatives

Some Integrals
USEFUL FORMULAS XIX

∂f ( x)
∫ exp dx = exp ∫
x x
; exp f ( x ) dx = exp f ( x )
∂x
1
∫ x dx = Ln( x) ; ∫ Ln( x)dx = xLn( x) − x + C
where e ≡ exp stands for exponential and Ln for natural logarithm.

du u
∫ 2
a −u
= sin −1   + C
2 a
du 1 u
∫ 2
a +u 2
= tan −1   + C
a a
du 1  u 
∫u u −a2 2
=
a
sec −1   + C
 a 

List of integrals
http://en.wikipedia.org/wiki/List_of_integrals

Some Function Solutions


Quadratic function
solution − b ± b 2 − 4ac
ax 2 + bx + c = 0  → x= ( a ≠ 0)
2a

Ruffini’s rule
http://en.wikipedia.org/wiki/Ruffini%27s_rule

Expressions related to the circle:

Equation of the circle: ( x1 − a ) 2 + ( x2 − b) 2 ≤ r 2


Area enclosed by a circumference: A = πr 2
Length of circumference: C = 2πr
x2

r
θ
b

a
x1

The relationship ds = rdθ holds, where ds is the infinitesimal arc length.


XX SOLVING PROBLEMS BY MEANS OF CONTINUUM MECHANICS

Expressions related to the ellipse:


x2

r
x
b
θ f2

f1 x1
b

a a

r p
Equation of the ellipse: x = r =
1 + e cos θ
a 2 − b2 p2
Eccentricity: e = ; 0 < e < 1 , where a 2 = holds.
a2 (1 − e 2 ) 2
Area enclosed by an ellipse: A = πab .
1 Tensors
The indicial notation was introduced by ‘Einstein (1916, sec.
5), who later jested to a friend, "I have made a great discovery
in mathematics; I have suppressed the summation sign every
time that the summation must be made over an index which
occurs twice..." (Kollros 1956; Pais 1982, p. 216). ‘
Ref. (Wolfram MathWorld (Einstein Summation))

1.1 Vectors, Indicial Notation


Problem 1.1
r r
Let a and b be arbitrary vectors. Prove that the following relationship is true:
r r r r r r r r r r
(a ∧ b) ⋅ (a ∧ b) = (a ⋅ a)(b ⋅ b) − (a ⋅ b) 2
Solution:
r r r r r r 2 r r
(a ∧ b) ⋅ (a ∧ b) = a ∧ b = a b sin θ ( 2
)
r 2 r 2 r 2 r 2 r 2 r 2 r r 2
= a b sin 2 θ = a b 1 − cos 2 θ = a b − a ( ) 2
b cos 2 θ
r 2 r 2 r r
( 2 r 2 r 2 r r
= a b − a b cos θ = a b − (a ⋅ b) 2 )
r r r r r r
= (a ⋅ a)(b ⋅ b) − (a ⋅ b) 2
r r r 2 r r r 2
where we have taking into account that a ⋅ a = a and b ⋅ b = b .

Problem 1.2
r r r r
Show that: if c = a + b , the module of c can be expressed by means of the following
relationship:
r r 2 r r r 2
c = a + 2 a b cos β + b
r r
where β is the angle formed by the vectors a and b , (see Figure 1.1(a)).
Solution:
Starting from the module definition of a vector it fulfills that:
r r 2 r r r r r r r r r r r r
a + b = (a + b) ⋅ (a + b) = a ⋅ a + a ⋅ b + b ⋅ a + b ⋅ b

r r r r r r 2 r r r r
Taking into account that a ⋅ a = a , b ⋅ b = b and a ⋅ b = b ⋅ a (commutative), we can
2

conclude that:
2 SOLVING PROBLEMS BY MEANS OF CONTINUUM MECHANICS

r r 2 r r r r r r r r r2 r r r 2 r2 r r r 2
a + b = a ⋅ a + a ⋅ b + b ⋅ a + b ⋅ b = a + 2a ⋅ b + b = a + 2 a b cos β + b

r r r 2 r r r 2
with which we prove a + b = a + 2 a b cos β + b . Then, it is easy to show that
r r r r r
2 r 2
a−b = − 2 a b cos β + b . Note also that when β = 0º ⇒ cos( β ) = 1 and the
a
r r r r
equation a + b = a + b holds, (see Figure 1.1(b)).

a) r b) β = 0º
b r
r b
a
β r r r r r
a a+b = a + b

Figure 1.1
r r 2 r r r r 2
NOTE: Starting from the equation a + b = a + 2a ⋅ b + b
2
we can conclude that the
r r 2
value a + b is maximum when β = 0º holds, then
r r 2 r 2 r r r 2 r2 r r r 2 r r
a + b = a + 2a ⋅ b + b = a + 2 a b + b = a + b ( )
2

r r r r
Then, for any value of 0º < β ≤ 180 º the outcome a + b will be less than a + b . Then, the
r r r r
inequality a + b ≤ a + b holds, (see Figure 1.2):
r r r r r
c = a+b ≤ a + b

r
b r r r
c = a+b
r
b
r
a

Figure 1.2
r r r r r r
In a similar fashion we can show that a ≤ c + b and b ≤ a + c which is known as the
triangle inequality, (see Figure 1.3).

a<c+b
c b<a+c
b
c<a+b

Figure 1.3

University of Castilla-La Mancha Draft By: Eduardo W. V. Chaves (2016)


Ciudad Real - Spain
1 TENSORS 3

Problem 1.3
1
Given the following functions σ(ε) = Eε and ψ(ε) = Eε 2 , demonstrate whether these
2
functions show a linear transformation or not.
Solution:
σ(ε 1 + ε 2 ) = E [ε1 + ε 2 ] = Eε 1 + Eε 2 = σ(ε1 ) + σ(ε 2 ) (linear transformation)

σ( ε)
σ (ε 1 + ε 2 ) = σ (ε 1 ) + σ ( ε 2 )

σ (ε 2 )

σ (ε 1 )

ε1 ε2 ε1 + ε 2 ε

Figure 1.4
1
The function ψ (ε) = Eε 2 does not show a linear transformation because the condition
2
ψ (ε1 + ε 2 ) = ψ (ε1 ) + ψ (ε 2 ) has not been satisfied:
1 1 1 1 1
ψ (ε1 + ε 2 ) = E[ε1 + ε 2 ]2 = E[ε12 + 2ε1ε 2 + ε 22 ] = Eε12 + Eε 22 + E 2ε1ε 2
2 2 2 2 2
= ψ (ε1 ) + ψ (ε 2 ) + Eε1ε 2 ≠ ψ (ε1 ) + ψ (ε 2 )

ψ ( ε)

ψ (ε1 + ε 2 )

ψ (ε1 ) + ψ (ε 2 )
ψ (ε 2 )

ψ (ε1 )

ε1 ε2 ε1 + ε 2 ε

Figure 1.5

University of Castilla-La Mancha Draft By: Eduardo W. V. Chaves (2016)


Ciudad Real - Spain
4 SOLVING PROBLEMS BY MEANS OF CONTINUUM MECHANICS

Problem 1.4
Consider the points: A(1,3,1) , B (2,−1,1) , C (0,1,3) and D(1,2,4 ) , defined in the Cartesian
coordinate system.
→ →
1) Find the parallelogram area defined by AB and AC ; 2) Find the volume of the
→ → → → →
parallelepiped defined by AB , AC and AD ; 3) Find the projection vector of AB onto BC .
Solution:
→ →
1) Firstly we calculate the vectors AB and AC :
r → → →
a = AB = OB − OA = (2ˆi − 1ˆj + 1kˆ ) − (1ˆi + 3ˆj + 1kˆ ) = 1ˆi − 4ˆj + 0kˆ
r → → →
b = AC = OC − OA = (0ˆi + 1ˆj + 3kˆ ) − (1ˆi + 3ˆj + 1kˆ ) = −1ˆi − 2ˆj + 2kˆ
Next, we evaluate the vector product as follows:
ˆi ˆj kˆ
r r
a∧b= 1 − 4 0 = ( −8)ˆi − 2ˆj + ( −6)kˆ
−1 − 2 2

Then, the parallelogram area can be obtained by using the following definition:
r r
A = a ∧ b = (−8) 2 + (−2) 2 + ( −6) 2 = 104


2) Next, we can evaluate the vector AD as:
r → → →
c = AD = OD − OA = (1ˆi + 2ˆj + 4kˆ ) − (1ˆi + 3ˆj + 1kˆ ) = 0ˆi − 1ˆj + 3kˆ
we can obtain the volume of the parallelepiped as follows:
r r r r r r
V (a, b, c ) = c ⋅ (a ∧ b) = (0ˆi − 1ˆj + 3kˆ ) ⋅ ( −8ˆi − 2ˆj − 6kˆ ) = 0 + 2 − 18 = 16


3) The BC vector can be calculated as:
→ → →
BC = OC − OB = (0ˆi + 1ˆj + 3kˆ ) − ( 2ˆi − 1ˆj + 1kˆ ) = −2ˆi + 2ˆj + 2kˆ
→ →
Hence, it is possible to evaluate the projection vector of AB onto BC as follows:
→ →
→ BC ⋅ AB → ( −2ˆi + 2ˆj + 2kˆ ) ⋅ (1ˆi − 4ˆj + 0kˆ )
proj BC AB =
→ BC = ( −2ˆi + 2ˆj + 2kˆ )
BC

⋅4
BC

( −2ˆi + 2ˆj + 2kˆ ) ⋅ ( −2ˆi + 2ˆj + 2kˆ )
1
42 3
→ 2
BC
( −2 − 8 + 0 ) 5 5 5
= ( −2ˆi + 2ˆj + 2kˆ ) = ˆi − ˆj − kˆ
( 4 + 4 + 4) 3 3 3

Problem 1.5
Rewrite the following equations using indicial notation:
1) a1 x1 x 3 + a 2 x 2 x 3 + a 3 x3 x 3
Solution: a i xi x 3 (i = 1,2,3)

University of Castilla-La Mancha Draft By: Eduardo W. V. Chaves (2016)


Ciudad Real - Spain
1 TENSORS 5

2) x1 x1 + x2 x2
Solution: xi x i (i = 1,2)

 a11 x + a12 y + a13 z = b x



3) a 21 x + a 22 y + a 23 z = b y

 a 31 x + a 32 y + a 33 z = b z
Solution:
 a11 x1 + a12 x 2 + a13 x 3 = b1  a1 j x j = b1
 
dummy free

a 21 x1 + a 22 x 2 + a 23 x 3 = b2
index j
→ a 2 j x j = b2 index
i → a ij x j = bi
a x + a x + a x = b 
 31 1 32 2 33 3 3  a 3 j x j = b3
As we can appreciate in this problem, the use of the indicial notation means that the equation
becomes very concise. In many cases, if algebraic operation do not use indicial or tensorial
notation they become almost impossible to deal with due to the large number of terms
involved.

Problem 1.6
Show that:
a) δ 3 p v p = v3 ; b) δ 3i A ji = A j 3 ; c) δ ij  ijk ; d) δ i 2 δ j 3 Aij .
Solution:
The Kronecker delta components are:
δ 11 δ 12 δ 13  1 0 0
δ ij = δ 21 δ 22 δ 23  = 0 1 0 (1.1)
δ 31 δ 32 δ 33  0 0 1
a) The expression ( δ 3 p v p ) has no free index, then the result is a scalar:
δ 3 p v p = δ 31v1 + δ 32 v 2 + δ 33 v 3 = v3 (1.2)
b) The expression δ 3i A ji has one free index ( j ), then the result is a vector:
δ 3i A ji = δ 31 A j1 + δ 32 A j 2 + δ 33 A j 3 = A j 3 (1.3)
c) The expression δ ij  ijk has one free index ( k ), then the result is a vector:
δ ij  ijk = δ 1 j 1 jk + δ 2 j  2 jk + δ 3 j  3 jk
123 1424 3 123
δ 1111k + δ 21 21k + δ 31 31k
+ + + (1.4)
δ 12 12 k + δ 22  22 k + δ 32  32 k
+ + +
δ 13 13k + δ 23  23k + δ 33  33k
thus δ ij  ijk = 0 k is the null vector. Note that δ ij  ijk =  iik = 11k +  22 k +  33k = 0 k .
d)

University of Castilla-La Mancha Draft By: Eduardo W. V. Chaves (2016)


Ciudad Real - Spain
6 SOLVING PROBLEMS BY MEANS OF CONTINUUM MECHANICS

δ i 2 δ j 3 Aij = A23 (1.5)

Problem 1.7
Expand the equation: Aij x i x j (i, j = 1,2,3)

Solution: The indices i, j are dummy indices, and indicate index summation and there is no
free index in the expression Aij x i x j , therefore the result is a scalar. So, we expand first the
dummy index i and later the index j to obtain:
expanding i
Aij x i x j   → A1 j x1 x j + A2 j x 2 x j + A3 j x 3 x j
1
424 3 1 424 3 1 424 3
A11 x1 x1 A21 x 2 x1 A31 x 3 x1
+ + +
expanding j

A12 x1 x 2 A22 x 2 x 2 A32 x 3 x 2


+ + +
A13 x1 x 3 A23 x 2 x 3 A33 x 3 x 3
Rearranging the terms we obtain:
Aij x i x j = A11 x1 x1 + A12 x1 x 2 + A13 x1 x3 + A21 x 2 x1 + A22 x 2 x 2 +
A23 x 2 x3 + A31 x3 x1 + A32 x 3 x 2 + A33 x 3 x3

Problem 1.8
Obtain the numerical value of:
1) δ ii δ jj
Solution: δ ii δ jj = (δ 11 + δ 22 + δ 33 )(δ 11 + δ 22 + δ 33 ) = 3 × 3 = 9
2) δ α1δ αγ δ γ1
Solution: δ α1δ αγ δ γ1 = δ γ1δ γ1 = δ 11 = 1
NOTE: Note that the following algebraic operation is incorrect δ γ1δ γ1 ≠ δ γγ = 3 ≠ δ 11 = 1 ,
since what must be replaced is the repeated index, not the number.
Problem 1.9
a) Prove the following is true  ijk  pjk = 2δ ip ,  ijk  ijk = 6 and  ijk a j a k = 0 i . b) Obtain the
numerical value of  ijk δ 2 j δ 3k δ 1i .
Solution: a) Using the equation  ijk  pqk = δ ip δ jq − δ iq δ jp , and by substituting q for j , we
obtain:
 ijk  pjk = δ ip δ jj − δ ij δ jp = δ ip 3 − δ ip = 2δ ip
Based on the above result, it is straight forward to check that:
 ijk  ijk = 2δ ii = 6
Note that  ijk = − ikj , i.e. it is antisymmetric in jk and also note that a j a k is a symmetric
second-order tensor. So, as we know, the double scalar product between a symmetric and an
antisymmetric second-order tensors is zero, thus:

University of Castilla-La Mancha Draft By: Eduardo W. V. Chaves (2016)


Ciudad Real - Spain
1 TENSORS 7

r r r r
ijk a j ak = ijk (a ⊗ a ) jk = 0i = (a ∧ a )i = 0i
b)  ijk δ 2 j δ 3k δ 1i = 123 = 1
Problem 1.10
Get the value of the following expressions:
a)  ijk δ i1δ j 2 δ 3k
b)  ijk  pqk = δ ip δ jq − δ iq δ jp for the following cases:
b.1) i = 1, j = q = 2, p = 3
b.2) i = q = 1, j = p = 2
c) ( ijk A jp c p A kq c q + δ i1 )( ist A sa c a A tb c b + δ i1 )
where  ijk is the permutation symbol, (see Figure 1.6), and δ ij is the Kronecker delta.
Reminder: Permutation symbol

 ijk =  jki =  kij


ijk = 1  ijk = − ikj = − kji = − jik
1 i

ijk = −1
3 2 k j

k =3
j =1
i =1 0 j=2
j =3
k =2 i=2 1
j =1 -1 0
i=3 0
i =1 0 j=2 0 0
j =3
k =1 0 0
i=2 0
j =1 0 -1
i=3 0
i =1 0 j=2 1 0  ij 3
j =3
i=2 0 0
0 0 0
i=3 0
0 1  ij 2
-1
0
 ij1

Figure 1.6
Solution:
a)  ijk δ i1δ j 2 δ 3k = 123 = 1 ;
b.1) 12 k  32 k = 121 321 + 122  322 + 123 323 = 0 × (−1) + 0 × 0 + 0 × 0 = 0
b.2) 12 k  21k = 121 211 + 122  212 + 123 213 = 0 × 0 + 0 × 0 + 1 × ( −1) = −1

University of Castilla-La Mancha Draft By: Eduardo W. V. Chaves (2016)


Ciudad Real - Spain
8 SOLVING PROBLEMS BY MEANS OF CONTINUUM MECHANICS

c) Note that the result of A jp c p = b j is a vector, and also note that the following is true
r r
 ijk A jp c p A kq c q = [( A ⋅ c) ∧ ( A ⋅ c )]i = (b ∧ b) i = 0 i , with which we can obtain:
r r

( ijk A jp c p A kq c q + δ i1 )( ist A sa c a A tb c b + δ i1 ) = δ i1δ i1 = δ 11 = 1

NOTE 1: The second-order tensor  ijk wk can easily be obtained as follows:


ijk wk = ij1w1 + ij 2 w2 + ij 3 w3
0 0 0  0 0 − 1  0 1 0  0 w3 − w2 
 
= w1 0 0 1 + w2 0 0 0  + w3 − 1 0 0 = − w3
  0 w1 
0 − 1 0 1 0 0   0 0 0  w2 − w1 0 

NOTE 2: The relationship eˆ i ∧ eˆ j can also be expressed in terms of ijk , since:


r
 eˆ 1 ∧ eˆ 1 eˆ 1 ∧ eˆ 2 eˆ 1 ∧ eˆ 3   0 eˆ 3 − eˆ 2 
   r 
eˆ i ∧ eˆ j = eˆ 2 ∧ eˆ 1 eˆ 2 ∧ eˆ 2 eˆ 2 ∧ eˆ 3  = − eˆ 3 0 eˆ 1 
r
 eˆ 3 ∧ eˆ 1 eˆ 3 ∧ eˆ 2 eˆ 3 ∧ eˆ 3   eˆ 2 − eˆ 1 0 
   
r r r r r r r
0 0 0   0 0 − eˆ 2   0 eˆ 3 0 
r r  r r r   r r
= 0 0 eˆ 1  +  0 0 0  +  − eˆ 3 0 0 
r r r r r r r
0 − eˆ 0  eˆ 2 0 0   0 0 0
 1     
0 0 0  0 0 − 1  0 1 0
= 0 0 1 e1 + 0 0 0  e 2 +  − 1 0 0 eˆ 3
  ˆ   ˆ
0 − 1 0 1 0 0   0 0 0
=  ij1eˆ 1 +  ij 2 eˆ 2 +  ij 3 eˆ 3
eˆ i ∧ eˆ j =  ijk eˆ k =  kij eˆ k

Note also that ijk is antisymmetric tensor in ij , since  ijk = − jik , (see Figure 1.6).
Problem 1.11
r
Write in indicial notation: a) the modulus of the vector a ; b) cos θ , where θ is the angle
r r
formed by the vectors a and b .
Solution:
δ ij
r r r r
= a ⋅ a = a i eˆ i ⋅ a j eˆ j = a i a j δ ij = a i a i = a j a j
2
a ⇒ a = ai ai
r
thus, it is also true that b = b i b i .
r r r r
By definition a ⋅ b = a b cos θ where:
r
a ⋅ b = a i eˆ i ⋅ b j eˆ j = a i b j δ ij = a i b i = a j b j

Taking into account that the index cannot appear more than twice in a term of the expression,
we can express cos θ as follows:
r r
a⋅b a jb j
cos θ = r r =
a b ai ai b k b k

University of Castilla-La Mancha Draft By: Eduardo W. V. Chaves (2016)


Ciudad Real - Spain
1 TENSORS 9

Problem 1.12
Show the Schwarz inequality:
r r r r
a⋅b ≤ a b Schwarz inequality (1.6)

Solution:
Let us consider a scalar α , then the following is true:
r r2 r r r r r r r r r r r r
aα − b = (aα − b) ⋅ (aα − b) = a ⋅ aα 2 − a ⋅ bα − b ⋅ aα + b ⋅ b ≥ 0
1424 3
≥0
r2 r r r2
= a α 2 − 2 a ⋅ bα + b ≥ 0
r r
r 2 2 r r r 2 a⋅b
where we define f (α ) = a α − 2a ⋅ bα + b ≥ 0 . Now, for the case when α = r 2 we can
a
obtain:
 r r   r r 
2
r r
 ( a ⋅ b )  r 2  (a ⋅ b )  r r (a ⋅ b) r 2
f α = r 2  = a  r 2  − 2(a ⋅ b) r 2 + b ≥ 0
 a   a  a
  
r r r r r r r r
r 2 (a ⋅ b) 2 r r ( a ⋅ b) r 2 ( a ⋅ b ) 2 ( a ⋅ b) 2 r 2
= a r 4 − 2( a ⋅ b) r 2 + b = r 2 − 2 r 2 + b ≥ 0
a a a a
r r 2
(a ⋅ b ) r 2
=− r 2 + b ≥0
a

r r 2
r 2 (a
⇒ b ≥ r 2
⋅ b) ⇒
r 2 r 2 r r
a b ≥ ( a ⋅ b) 2 ⇒
r r r r
a b ≥ a⋅b
a
Q.E.D.

Alternative solution
r r r r r r
Taking in account that 0 ≤ cos θ ≤ 1 we obtain a ⋅ b = a b cos θ ≤ a b , thus we can
r r r r
conclude that a ⋅ b ≤ a b .

Problem 1.13
r r r r
Rewrite the expression (a ∧ b) ⋅ (c ∧ d) without using the vector product symbol.
r r r r
Solution: The vector product (a ∧ b) can be expressed as (a ∧ b) = a j eˆ j ∧ b k eˆ k =  ijk a j b k eˆ i .
r r r r
Likewise, it is possible to express (c ∧ d) as (c ∧ d) =  nlm c l d m eˆ n , thus:
r r r r
(a ∧ b) ⋅ (c ∧ d) =  ijk a j b k eˆ i ) ⋅ ( nlm c l d m eˆ n ) =  ijk  nlm a j b k c l d m eˆ i ⋅ eˆ n
=  ijk  nlm a j b k c l d mδ in =  ijk  ilm a j b k c l d m

University of Castilla-La Mancha Draft By: Eduardo W. V. Chaves (2016)


Ciudad Real - Spain
10 SOLVING PROBLEMS BY MEANS OF CONTINUUM MECHANICS

Taking into account that  ijk  ilm =  jki  lmi and by applying the equation
 jki  lmi = δ jl δ km − δ jm δ kl =  jki  ilm , we obtain:
 ijk  ilm a j b k c l dm = (δ jlδ km − δ jmδ kl ) a j b k c l d m = al b m c l dm − a mb l c l dm
r r r r
Since a l c l = (a ⋅ c ) and b m d m = (b ⋅ d) holds true, we can conclude that:
r r r r r r r r r r r r
(a ∧ b) ⋅ (c ∧ d) = (a ⋅ c )(b ⋅ d) − (a ⋅ d)(b ⋅ c )
r r r r
Therefore, it is also valid when a = c and b = d , thus:
r r r r r r 2 r r r r r r r r r 2 r 2 r r
(a ∧ b) ⋅ (a ∧ b) = a ∧ b = (a ⋅ a)(b ⋅ b) − (a ⋅ b)(b ⋅ a) = a b − (a ⋅ b) 2

which is the same equation obtained in Problem 1.1.


r r r r
NOTE: We can start from the above equation to show a ∧ b = a b sin θ , i.e.:
r r
a∧b
2 r
= a
2 r 2 r r r
b − (a ⋅ b ) 2 = a
2 r 2 r r
(
b − a b cos θ ) = ar
2 2 r 2 r
b (1 − cos 2 θ ) = a
2 r 2
b sin 2 θ
r r r r
⇒ a ∧ b = a b sin θ
r r r r r r
Note that 0 ≤ sin θ ≤ 1 , with that we can prove that a ∧ b = a b sin θ ≤ a b , thus
r r r r
a∧b ≤ a b

Problem 1.14
Show that:
a)  ijk a i a j b k = 0 ;
r r
b)  ijk (a k b 3δ i1δ j 2 + a j b 2 δ i1δ k 3 + a i b1δ j 2 δ k 3 ) = a ⋅ b ;
c) Aij A ji is an invariant.
Solution:
a)  ijk a i a j b k =  ij1a i a j b1 +  ij 2 a i a j b 2 +  ij 3 a i a j b 3 . The term  ij1a i a j b1 can be evaluated as
follows:
 ij1a i a j b1 = 1 j1 a1 a j b1 +  2 j1a 2 a j b1 +  3 j1a 3 a j b1
= 111 a1 a1b 1 +  211 a 2 a1b1 +  311 a 3 a1b 1 +
+ 121 a1 a 2 b 1 +  221 a 2 a 2 b 1 +  321 a 3 a 2 b1 +
+ 131 a1 a 3 b1 +  231 a 2 a 3b 1 +  331 a 3 a 3 b1
=  321 a 3 a 2 b 1 +  231 a 2 a 3b 1 = −a 3 a 2 b 1 + a 2 a 3b 1
=0
In the same way we can obtain  ij 2 a i a j b 2 =  ij 3 a i a j b 3 = 0 .
b)
 ijk a k b 3 δ i1δ j 2 +  ijk a j b 2 δ i1δ k 3 +  ijk a i b1δ j 2 δ k 3 =
r r
12 k a k b 3 +  1 j 3 a j b 2 +  i 23 a i b1 = a 3b 3 + a 2 b 2 + a1b1 = a i b i = a ⋅ b

University of Castilla-La Mancha Draft By: Eduardo W. V. Chaves (2016)


Ciudad Real - Spain
1 TENSORS 11

Problem 1.15
r r r r r r r r r r r r
Prove that (a ∧ b) ∧ (c ∧ d) = c [d ⋅ (a ∧ b)] − d [c ⋅ (a ∧ b)]
Solution: Expressing the correct equality term in indicial notation we obtain:
{cr [dr ⋅ (ar ∧ br )] − dr [cr ⋅ (ar ∧ br )] } p = c p [di ( ijk a j b k )] − d p [c i ( ijk a j b k )]

⇒  ijk a j b k c p di −  ijk a j b k c i d p ⇒  ijk a j b k (c p di − c i d p )


Using the Kronecker delta the above equation becomes:
⇒  ijk a j b k (δ pm c m d n δ ni − δ im c m d nδ np ) ⇒ ( ijk a j b k ) c m d n (δ pm δ ni − δ imδ np )
and by applying the equation δ pm δ ni − δ im δ np =  pil  mnl , the above equation can be rewritten
as follows:
⇒ ( ijk a j b k ) c m dn ( pil  mnl ) ⇒  pil [( ijk a j b k )( mnl c m dn )]
r r r r
Since  ijk a j b k and  mnl c m d n represent the components of (a ∧ b) and (c ∧ d) , respectively,
we can conclude that:
r r r r
 pil [( ijk a j b k )( mnl c m d n )] = [(a ∧ b) ∧ (c ∧ d)] p

Problem 1.16
r r r r
Let a , b , c be linearly independent vectors, and v be a vector, demonstrate that:
r r r r r
v = αa + β b + γ c ≠ 0 components
  → v i = αa i + β b i + γ c i ≠ 0 i
where the scalars α , β , γ are given by:
 ijk v i b j c k  ijk a i v j c k  ijk a i b j v k
α= ; β= ; γ=
 pqr a p b q c r  pqr a p b q c r  pqr a p b q c r
b) Given three linearly independent vectors, show that: when interchanging two rows or two
r r r
columns the sign of the determinant a ⋅ (b ∧ c ) changes.
r r r
Solution: a) The scalar product made up of v and ( b ∧ c ) becomes:
r r r
r r r r r r r r r r r r v ⋅ (b ∧ c )
v ⋅ (b ∧ c ) = αa ⋅ (b ∧ c ) + β b ⋅ (b ∧ c ) + γ c ⋅ (b ∧ c ) ⇒ α= r r r
14243 14243
=0 =0
a ⋅ (b ∧ c )

which is the same as:


v1 v2 v3 v1 b1 c1
b1 b2 b3 v2 b2 c2
c1 c2 c3 v3 b3 c3  ijk v i b j c k
α= = =
a1 a2 a3 a1 b1 c1  pqr a p b q c r
b1 b2 b3 a2 b2 c2
c1 c2 c3 a3 b3 c3

One can obtain the parameters β and γ in a similar fashion, i.e.:

University of Castilla-La Mancha Draft By: Eduardo W. V. Chaves (2016)


Ciudad Real - Spain
12 SOLVING PROBLEMS BY MEANS OF CONTINUUM MECHANICS

r r r r r r r r r r r r
v ⋅ (a ∧ c ) = α a ⋅ (a ∧ c ) + β b ⋅ ( a ∧ c ) + γ c ⋅ ( a ∧ c )
14243 14243
=0 =0
r r r r r r
v ⋅ (a ∧ c )  ijk v i a j c k −  jik a j v i c k a ⋅ (v ∧ c)
⇒β = r r r = = = r r r
b ⋅ (a ∧ c )  pqr b p a q c r −  qpr a q b p c r a ⋅ (b ∧ c )
r r r r r r r r r r r r
v ⋅ (a ∧ b ) = α a ⋅ ( a ∧ b ) + β b ⋅ ( a ∧ b ) + γ c ⋅ (a ∧ b )
14243 14243
=0 =0
r r r r r r
v ⋅ (a ∧ b)  ijk v i a j b k  jki a j b k v i a ⋅ (b ∧ v )
⇒γ = r r r = = = r r r
c ⋅ (a ∧ b)  pqr c p a q b r  qrp a q b r c p a ⋅ (b ∧ c )
r
NOTE 1: We can restructure the v -components as follows:
 v 1   a1 b1 c 1  α   a1 b1 c 1   z1 
     
v i = v 2  = a 2 b2 c 2  β  = a 2 b2 c 2  z 2  = B ij z j
v  a b3 c 3  γ   a 3 b3 c 3  z 3 
 3  3
where we have denoted by z1 = α , z 2 = β , z 3 = γ , in which:
v1 b1 c1 a1 v1 c1
v2 b2 c2 a2 v2 c2
 vb c v b3 c3 B (1)  av c a v3 c3 B (2)
α =z1 = ijk i j k = 3 = ; β =z 2 = ijk i j k = 3 =
 pqr a p b q c r a1 b1 c1 B  pqr a p b q c r a1 b1 c1 B
a2 b2 c2 a2 b2 c2
a3 b3 c3 a3 b3 c3

a1 b1 v1
a2 b2 v2
 ab v a b3 v3 B (3)
γ =z 3 = ijk i j k = 3 =
 pqr a p b q c r a1 b 1 c1 B
a2 b2 c2
a3 b3 c3

where B (i ) is the determinant of the resulting matrix by replacing the column (i) of the
r
matrix B by the v -components. With that we can state that:
B (i )
Given v i = B ij z j ⇒ zi = Cramer’s rule
B

NOTE 2: Although we have demonstrated for 3 × 3 matrix, this procedure is also valid for
matrices of n-dimensions, which is known, in the literature, as Cramer’s Rule.
NOTE 3: The solution ( z i ) is possible if B ≠ 0 .

NOTE 4: If v i = 0 i we have B ij z j = 0 i and B (i ) = 0 i , with that according to Cramer’s rule


we have:
z i B = B (i ) = 0 i

Note that the non-trivial solution z i ≠ 0 i is only possible if and only if B = 0 , (see Problem
1.50).

University of Castilla-La Mancha Draft By: Eduardo W. V. Chaves (2016)


Ciudad Real - Spain
1 TENSORS 13

r r r r r r
b) The determinant defined by a ⋅ (b ∧ c ) = [a, b, c ] in indicial notation becomes  ijk a i b j c k ,
and by taking into account the permutation symbol properties, (see Figure 1.7), we can
conclude that:
a1 a2 a3
 ijk ai b j c k = b1 b 2 b3
c1 c 2 c3
a1 a2 a3
−  ikj ai b j c k = − c1 c2 c3
b1 b 2 b3
b1 b 2 b3
 jki ai b j c k = c1 c2 c3
a1 a2 a3
r r r r r r r r r
ijk aib j c k = [a, b, c ] = −ikj aib j c k = −[a, c , b] =  jki aib j c k = [b, c , a]
 ijk =  jki =  kij
 ijk = − ikj = − kji = − jik
i

k j

Figure 1.7

Problem 1.17
a) Show that
r r r r r r r r r r r r r r
a ∧ (b ∧ c ) = (a ⋅ c ) b − (a ⋅ b ) c = (b ⊗ c − c ⊗ b) ⋅ a
r r r r r r r r
a ∧ (b ∧ a) = [(a ⋅ a)1 − a ⊗ a] ⋅ b
r r r r
b) Obtain the explicit component of the tensor [(a ⋅ a)1 − a ⊗ a] .
Solution:
r r r r r
a) Taking into account that (d) i = (b ∧ c ) i =  ijk b j c k and that (a ∧ d) q =  qjk b j c k , we obtain:
r r r
[a ∧ (b ∧ c )]r =  rsi a s ( ijk b j c k )
=  rsi  ijk a s b j c k =  rsi  jki a sb j c k
= (δ rjδ sk − δ rkδ sj ) a s b j c k
= δ rjδ sk a s b j c k − δ rkδ sj a sb j c k = a sb r c s − a sb s c r
= ak b r c k − a jb j c r = (b r c s − b s c r )a s
r r r r r r r r r
= b r ( a ⋅ c ) − c r (a ⋅ b ) = [(b ⊗ c − c ⊗ b) ⋅ a]r
r r r r r r
= [b (a ⋅ c ) − c (a ⋅ b)]r
With that we conclude that:

University of Castilla-La Mancha Draft By: Eduardo W. V. Chaves (2016)


Ciudad Real - Spain
14 SOLVING PROBLEMS BY MEANS OF CONTINUUM MECHANICS

r r r r r r r r r r r r r r
a ∧ (b ∧ c ) = (a ⋅ c ) b − (a ⋅ b) c = (b ⊗ c − c ⊗ b ) ⋅ a
Note that it is also true that:
r r r r r r r r r r r r r r r r r r
a ∧ (b ∧ c ) = (b ⊗ c − c ⊗ b) ⋅ a = [(a ⋅ c )1 − c ⊗ a] ⋅ b = [b ⊗ a − (a ⋅ b)1] ⋅ c
r r
In the particular case when a = c we obtain:
r r r
[a ∧ (b ∧ a)]r = (a k a k )b r − (a j b j )a r = (a j a j )b pδ rp − (a j b pδ jp ) a r

= [(a j a j )δ rp − (a jδ jp )a r ] b p = [(a j a j )δ rp − a p a r ] b p
r r
{ r r r
= [ (a ⋅ a)1 − a ⊗ a ] ⋅ b r }
r r r r
b) The components of [(a ⋅ a)1 − a ⊗ a] can be obtained as follows:
1 0 0   a 1 a 1 a1 a 2 a1 a 3 
r r r r
[(a ⋅ a)1 − a ⊗ a] ij = (a k a k )δ ij − ai a j = (a12 + a 22 + a 32 ) 01 0 − a1 a 2 a2a2 a1 a 3 
0 0 1  a1 a 3 a1 a 3 a 3 a 3 
(a 22 + a 32 ) − a1 a 2 − a1 a 3 
 2 2 
=  − a1 a 2 (a1 + a 3 ) − a1 a 3 
 − a1a 3 − a1a 3 (a12 + a 22 ) 

Problem 1.18
Show the Jacobi identity:
r r r r r r r r r r
a ∧ (b ∧ c ) + b ∧ (c ∧ a) + c ∧ (a ∧ b) = 0
r r r r r r r r r
Solution: By means of Problem 1.17 in which a ∧ (b ∧ c ) = (a ⋅ c ) b − (a ⋅ b) c was proven, we
can obtain that:
r r r r r r r r r r r r r r r r r r
b ∧ (c ∧ a) = (b ⋅ a) c − (b ⋅ c ) a c ∧ (a ∧ b) = (c ⋅ b ) a − (c ⋅ a) b
;
r r r r r r r r
Then, by considering that the dot product is commutative, i.e. (a ⋅ c ) = (c ⋅ a) , (a ⋅ b) = (b ⋅ a) ,
r r r r
(b ⋅ c ) = (c ⋅ b) , we can conclude that:
r r r r r r
(a ⋅ c ) b − a ⋅ b c ( )
r r r+ r r r r
r r r r r r r r r
( ) ( )
a ∧ (b ∧ c ) + b ∧ (c ∧ a) + c ∧ (a ∧ b) = b ⋅ a c − b ⋅ c a = 0
r r r+ r r r
c ⋅ b a − (c ⋅ a ) b( )
1.2 Algebraic Operations with Higher Order Tensors

Problem 1.19
Define the order of the tensors represented by their Cartesian components: v i , Φ ijk , Fijj , ε ij ,
C ijkl , σ ij . Determine the number of components in tensor C .

Solution: The order of the tensor is given by the number of free indices, so it follows that:

University of Castilla-La Mancha Draft By: Eduardo W. V. Chaves (2016)


Ciudad Real - Spain
1 TENSORS 15

r r
First-order tensor (vector): v , F ; Second-order tensor: ε , σ ; Third-order tensor: Φ ;
Fourth-order tensor: C
The number of tensor components is given by the maximum index range value, i.e.
i, j , k , l = 1,2,3 , to the power of the number of free indices which is equal to 4 in the case of
C ijkl . Thus, the number of independent components in C is given by:

3 4 = (i = 3) × ( j = 3) × (k = 3) × (l = 3) = 81
The fourth-order tensor C ijkl has 81 components.

Problem 1.20
r r r r r r r r r r r r r r
Show that a) (a ⊗ b) ⋅ c = (b ⋅ c ) a ; b) (a ⊗ b) ⋅ (c ⊗ d) = (b ⋅ c ) a ⊗ d
Solution:
a) We express the vector in the Cartesian basis and be aware that we cannot repeat index more
than twice:
r r r r r r r r r
(a ⊗ b ) ⋅ c = (ai eˆ i ⊗ b j eˆ j ) ⋅ c k eˆ k = ai eˆ i b j c kδ jk = (b k c k )ai eˆ i = (b ⋅ c )a ≡ (b ⋅ c ) ⊗ a
r r r r
b) The expression (a ⊗ b) ⋅ (c ⊗ d) , which is a second-order tensor, can be expressed in
indicial notation as follows:
r r r r r r r r
[(a ⊗ b) ⋅ (c ⊗ d)] ij = (a ⊗ b) ik (c ⊗ d) kj = (a i b k )(c k d j ) = ai b k c k d j = b k c k ai d j
r r r
= (b k c k )(a i d j ) = (b ⋅ c )(a ⊗ d) ij
123
scalar

Problem 1.21
Expand and simplify the expression A ij xi x j when a) A ij = A ji ; b) A ij = − A ji .
Solution:
By expanding A ij xi x j (scalar) we can obtain:
A ij xi x j = A 1 j x1 x j + A 2 j x 2 x j + A 3 j x3 x j =
= A 11 x1 x1 + A 21 x 2 x 1 + A 31 x3 x 1 +
(1.7)
A 12 x1 x 2 + A 22 x 2 x 2 + A 32 x 3 x 2 +
A 13 x1 x 3 + A 23 x 2 x 3 + A 33 x3 x 3
a) If A ij = A ji (symmetry) we have

A ij xi x j = A11x12 + 2A12 x1x 2 +2 A13 x1x 3 + A 22 x22 + 2A 23 x2 x3 + A 33 x32 (1.8)


b) If A ij = − A ji we have A11 = − A11 = 0 , A 22 = − A 22 = 0 , A 33 = − A 33 = 0 , A12 = − A 21 ,
A13 = − A 31 , A 23 = − A 32 , with that the equation (1.7) becomes
A ij xi x j = 0 (1.9)
NOTE: As we will see later, when A ij = − A ji holds we said that the tensor is antisymmetric.
And also note that for the case (b) the following is true:

University of Castilla-La Mancha Draft By: Eduardo W. V. Chaves (2016)


Ciudad Real - Spain
16 SOLVING PROBLEMS BY MEANS OF CONTINUUM MECHANICS

r r r r r r
A ij xi x j = x ⋅ A ⋅ x = A : ( x ⊗ x ) = A skew : ( x ⊗ x ) sym = 0 (1.10)
r r r r
That is, if A = A skew is an antisymmetric and ( x ⊗ x ) = ( x ⊗ x ) sym is a symmetric tensor, the
double scalar product between them is always equal to zero.

Problem 1.22
Let ε and T be second-order tensors, whose Cartesian components are:
 5 2 4  3 1 2
ε ij =  − 1 2 1  ; Tij = 4 2 1  (1.11)
 4 3 6 1 3 8 

Obtain T : ε .
Solution:
T : ε = Tij ε ij (1.12)
Tij ε ij = T1 j ε1 j + T2 j ε 2 j + T3 j ε 3 j
123 123 123
T11ε11 + T21ε 21 + T31ε 31
+ + + (1.13)
T12 ε12 + T22 ε 22 + T32 ε 32
+ + +
T13 ε13 + T23 ε 23 + T33 ε 33
thus,
Tij ε ij = 5 × 3 + 2 × 1 + 4 × 2 + (−1) × 4 + 2 × 2 + 1 × 1 + 4 × 1 + 3 × 3 + 6 × 8 = 87 (1.14)

Problem 1.23
Given the B tensor components:
3 2 4
B ij = 1 5 3 (1.15)
5 7 9

Obtain:
a) C ij = B ik B kj ; b) D ij = B ik B jk ; c) E ij = B ki B kj ; d) C ii , D ii , E ii
Solution:
3 2 4 3 2 4  31 44 54 
C = B ⋅B ⇒ C ij = B ik B kj = 1 5 3 1 5 3 =  23 48 46  (1.16)
5 7 9 5 7 9  67 108 122
T
3 2 4  3 2 4   29 25 65 
D = B ⋅ BT ⇒ D ij = B ik B jk    
= 1 5 3 1 5 3 =  25 35 67  (1.17)
5 7 9 5 7 9  65 67 155

University of Castilla-La Mancha Draft By: Eduardo W. V. Chaves (2016)


Ciudad Real - Spain
1 TENSORS 17

T
3 2 4 3 2 4 35 46 60 
E = BT ⋅ B ⇒ E ij = B kiB kj = 1 5 3 1 5 3 =  46 78 86  (1.18)
5 7 9  5 7 9  60 86 106

Then:
Tr (C) = Tr (B ⋅ B) = C ii = C11 + C 22 + C 33 = 31 + 48 + 122 = 201
Tr (D) = Tr (B ⋅ BT ) = D ii = D11 + D 22 + D 33 = 29 + 35 + 155 = 219 (1.19)
Tr (E) = Tr (B T
⋅ B) = E ii = E11 + E 22 + E 33 = 35 + 78 + 106 = 219

NOTE: The numerical value for B : B is:


B : B = B ijB ij = B11B11+B 21B 21+B 31B 31+ B12B12 +B 22B 22 +B 32B 32 + B13B13 +B 23B 23 +B 33B 33 = 219

With that we can verify the following is true: Tr (B ⋅ BT ) = Tr (BT ⋅ B) = B : B = 219 , which is a
trace property.

Problem 1.24
Given the B second-order tensor components:
1 0 2
B ij = 0 1 2
3 0 3

Obtain: a) B kk b) B ij B ij c) B jk B kj
Solution:
a) B kk = B 11 + B 22 + B 33 = 1 + 1 + 3 = 5
b) B ij B ij = B 1 j B 1 j + B 2 jB 2 j + B 3 jB 3 j
123 123 123
B 11B 11 + B 21B 21 + B 31B 31
+ + +
B 12B 12 + B 22B 22 + B 32B 32
+ + +
B 13B 13 + B 23B 23 + B 33B 33
which the result is:
B ij B ij = 1 × 1 + 0 × 0 + 2 × 2 + 0 × 0 + 1 × 1 + 2 × 2 + 3 × 3 + 0 × 0 + 3 × 3 = 28

c) B jk B kj = B 1k B k1 + B 2k B k 2 + B 3k B k 3
123 123 123
B 11B 11 + B 21B 12 + B 31B 13
+ + +
B 12B 21 + B 22B 22 + B 32B 23
+ + +
B 13B 31 + B 23B 32 + B 33B 33

University of Castilla-La Mancha Draft By: Eduardo W. V. Chaves (2016)


Ciudad Real - Spain
18 SOLVING PROBLEMS BY MEANS OF CONTINUUM MECHANICS

B jk B kj = B 11B 11 + B 22B 22 + B 33B 33 + 2B 21B 12 + 2B 31B 13 + 2B 32B 23


= 1 × 1 + 1 × 1 + 3 × 3 + 2(0 × 0) + 2(3 × 2 ) + 2(0 × 2 ) = 23

Problem 1.25
The D tensor is given by the algebraic operation D = A : B . Obtain the order of the tensor
D and its components for the following cases:
 2 3 2  2 3 1
a) when Aij = 4 1 1  ; Bij = 1 2 1
1 1 5 1 2 5

 7 13 14 13 9 17 
b) when Aik B kj = 11 18 11 ; Aik B jk = 15 9 13 
16 27 31 18 12 32

Solution:
a) Note that if A and B are second-order tensors, the D = A : B is a scalar (zeroth-order
tensor), so, we only have one component:
A : B = 2 × 2 + 3 × 3 + 2 × 1 + 4 × 1 + 1 × 2 + 1 × 1 + 1 × 1 + 1 × 2 + 5 × 5 = 50
b) Taking into account that Tr ( A ⋅ B T ) = Tr ( AT ⋅ B) = A : B and Aik B jk = A ⋅ B T , we can
conclude that A : B = Tr ( A ⋅ B T ) = 13 + 9 + 32 = 54 .
Problem 1.26
Let us consider the following second-order tensor T = Tr ( E )1 + ( F : E ) E which in indicial
notation is Tij = E kk δ ij + ( Fkp E kp ) E ij . If the components of E and F are given by:

 2 1 4 4 3 1
E ij = 1 5 0 ; Fij =  2 0 3
2 0 1   2 0 0

a) Obtain the T tensor components. b) Are T and E coaxial tensors? Prove it.
Solution:
Next, we obtain the following scalars:
Tr ( E ) = 2 + 5 + 1 = 8
F : E = 2 × 4 + 1 × 3 + 4 × 1 + 1 × 2 + 5 × 0 + 0 × 3 + 2 × 2 + 0 × 0 + 1 × 0 = 21
Then
1 0 0  2 1 4 50 21 84 
 
Tij = 80 1 0 + 211 5 0 =  21 113 0 
0 0 1  2 0 1   42 0 29

Two tensors are coaxial when they have the same eigenvectors or when the relationship
T ⋅ E = E ⋅ T holds:

University of Castilla-La Mancha Draft By: Eduardo W. V. Chaves (2016)


Ciudad Real - Spain
1 TENSORS 19

50 21 84   2 1 4 289 155 284


(T ⋅ E )ij = Tik Ekj =  21 113 0  1 5 0 = 155 586 84 
 42 0 29  2 0 1  142 42 197 
2 1 4 50 21 84  289 155 284
( E ⋅ T )ij = Eik Tkj = 1 5 0  21 113 0  = 155 586 84 
 2 0 1  42 0 29 142 42 197 
with that we can conclude that T and E are coaxial tensors.
Problem 1.27
Obtain the result of the following algebraic operations: I : I , I : I , I : I , I : I , I : I , I : I ,
I : I , I : I , I sym : I sym , I sym : I , I : I sym , where

I = 1⊗1 = I ijkl eˆ i ⊗ eˆ j ⊗ eˆ k ⊗ eˆ l where I ijkl = δ ik δ jl (1.20)


I = 1⊗1 = I ijkl eˆ i ⊗ eˆ j ⊗ eˆ k ⊗ eˆ l where I ijkl = δ il δ jk (1.21)

I = 1 ⊗ 1 = I ijkl eˆ i ⊗ eˆ j ⊗ eˆ k ⊗ eˆ l where I ijkl = δ ij δ kl (1.22)

Solution:
(I : I ) ijkl = I ijpq I pqkl = δ ip δ jq δ pk δ ql = δ ik δ jl = I ijkl

( I : I ) ijkl = I ijpq I pqkl = δ iq δ jp δ pl δ qk = δ ik δ jl = I ijkl

( I : I ) ijkl = I ijpq I pqkl = δ ij δ pq δ pq δ kl = δ qq δ ij δ kl = 3I ijkl

( I : I ) ijkl = I ijpq I pqkl = δ iq δ jp δ pk δ ql = δ il δ jk = I ijkl

(I : I ) ijkl = I ijpq I pqkl = δ ip δ jq δ pl δ qk = δ il δ jk = I ijkl

(I : I ) ijkl = I ijpq I pqkl = δ ip δ jq δ pq δ kl = δ iq δ jq δ kl = δ ij δ kl = I ijkl

( I : I ) ijkl = I ijpq I pqkl = δ iq δ jp δ pq δ kl = δ iq δ jq δ kl = δ ij δ kl = I ijkl

We summarize the above in tensorial notation as follows:


I : I = (1⊗1) : (1⊗1) = 1⊗1 = I

I : I = (1⊗1) : (1⊗1) = 1⊗1 = I

I : I = (1 ⊗ 1) : (1 ⊗ 1) = 3(1 ⊗ 1) = 3I

I : I = (1⊗1) : (1⊗1) = 1⊗1 = I

I : I = (1⊗1) : (1⊗1) = 1⊗1 = I

I : I = (1⊗1) : (1 ⊗ 1) = 1 ⊗ 1 = I

I : I = (1⊗1) : (1 ⊗ 1) = 1 ⊗ 1 = I

Taking into account the definition I sym =


1
2
( 1
) ( )
I + I = 1⊗1 + 1⊗1 , we can conclude that:
2

University of Castilla-La Mancha Draft By: Eduardo W. V. Chaves (2016)


Ciudad Real - Spain
20 SOLVING PROBLEMS BY MEANS OF CONTINUUM MECHANICS

I sym : I sym =
1
4
( )(
1⊗1 + 1⊗1 : 1⊗1 + 1⊗1 )
1
[( ) ( ) (
= 1⊗1 : 1⊗1 + 1⊗1 : 1⊗1 + 1⊗1 : 1⊗1 + (1⊗1 : 1⊗1)
4
) ]
1
[
= 1⊗1 + 1⊗1 + 1⊗1 + 1⊗1
4
]
1
(
= 1⊗1 + 1⊗1
2
)
= I sym

I sym : (1 ⊗ 1) = I sym : I =
1
2
( 1
2
) ( 1
) ( )
I + I : I = I : I + I : I = I + I = I =1 ⊗1
2
(1 ⊗ 1) : I sym = I : I sym
1
2
(
1
2
) ( 1
= I : I + I = I :I + I : I = I + I = I =1 ⊗1
2
) ( )
1.3 Tensor Transpose

Problem 1.28
Let A , B and C be arbitrary second-order tensors. Show that:
A : (B ⋅ C ) = (B T ⋅ A ) : C = ( A ⋅ C T ) : B
Solution: Expressing the term A : (B ⋅ C ) in indicial notation we obtain:
δ kp

A : (B ⋅ C ) = A ij eˆ i ⊗ eˆ j : (B lk eˆ l ⊗ eˆ k ⋅ C pq eˆ p ⊗ eˆ q ) = A ij B lk C pq eˆ i ⊗ eˆ j : (δ kp eˆ l ⊗ eˆ q )

δ il

A : (B ⋅ C ) = A ij B lk C pqδ kp eˆ i ⊗ eˆ j : (eˆ l ⊗ eˆ q )

δ jq

A : (B ⋅ C ) = A ijB lk C pqδ kpδ ilδ jq = A ij B ik C kj

Note that, when we are dealing with indicial notation the position of the terms does not
matter, i.e.:
A ij B ik C kj = B ik A ij C kj = A ij C kj B ik

We can now observe that the algebraic operation B ik A ij is equivalent to the components of
the second-order tensor (B T ⋅ A ) kj , thus,

B ik A ij C kj = (B T ⋅ A ) kj C kj = (B T ⋅ A ) : C .

Likewise, we can state that A ij C kj B ik = ( A ⋅ C T ) : B .

University of Castilla-La Mancha Draft By: Eduardo W. V. Chaves (2016)


Ciudad Real - Spain
1 TENSORS 21

Problem 1.29
r r
Let u , v be vectors and A be a second-order tensor. Show that the following relationship
holds:
r r r r
u⋅ AT ⋅ v = v ⋅ A ⋅u
Solution:
r r r r
u ⋅ AT ⋅ v = v ⋅ A ⋅u
u i eˆ i ⋅ A jl eˆ l ⊗ eˆ j ⋅ v k eˆ k = v k eˆ k ⋅ A jl eˆ j ⊗ eˆ l ⋅ u i eˆ i
u i A jl δ il v k δ jk = v k δ kj A jl u i δ il
u l A jl v j = v j A jl u l

1.4 Symmetry and Antisymmetry

Problem 1.30
Show that σ : W = 0 is always true when σ is a symmetric second-order tensor and W is an
antisymmetric second-order tensor.
Solution:
σ : W = σ ij (eˆ i ⊗ eˆ j ) : Wlk (eˆ l ⊗ eˆ k ) = σ ij Wlk δ il δ jk = σ ij Wij (scalar)

Thus,
σ ij Wij = σ1 j W1 j + σ 2 j W2 j + σ 3 j W3 j
123 1424 3 1
424 3
σ11W11 σ21W21 σ31W31
+ + +
σ12 W12 σ 22 W22 σ32 W32
+ + +
σ13W13 σ23W23 σ33W33

Taking into account the characteristics of a symmetric and an antisymmetric tensor, i.e.
σ12 = σ 21 , σ 31 = σ13 , σ 32 = σ 23 , and W11 = W22 = W33 = 0 , W21 = − W12 , W31 = − W13 ,
W32 = − W23 , the equation above becomes:

σ :W =0
Q.E.D.

Problem 1.31
r r r r r
Show that a) M ⋅ Q ⋅ M = M ⋅ Q sym ⋅ M ; b) A : B = A sym : B sym + A skew : B skew where M is a
vector, and Q , A , B are arbitrary second-order tensors; c) Show that the relationship
 ijk T jk = 0 i holds, where T is symmetric, i.e. Tij = T ji .
Solution:
r r r r r r r r
a) M ⋅ Q ⋅ M = M ⋅ (Q sym + Q skew ) ⋅ M = M ⋅ Q sym ⋅ M + M ⋅ Q skew ⋅ M

University of Castilla-La Mancha Draft By: Eduardo W. V. Chaves (2016)


Ciudad Real - Spain
22 SOLVING PROBLEMS BY MEANS OF CONTINUUM MECHANICS

r r r r
Since the relation M ⋅ Q skew ⋅ M = Q skew : (1
M ⊗ M) = 0 holds, it follows that:
424 3
symmetric tensor
r r r r
M ⋅ Q ⋅ M = M ⋅ Q sym ⋅ M
r r
NOTE: We can make the geometric interpretation of M ⋅ Q skew ⋅ M = 0 as follows. Note that
r r r r r r r r
the algebraic operation Q skew ⋅ M = q (M) is a vector, thus M ⋅ Q skew ⋅ M = M ⋅ q (M) = 0 , which
r r r
implies that M and q (M) are orthogonal vectors. With that we conclude that: the projection of
r r r
an antisymmetric second-order tensor according to the direction ( M ) is the vector ( q (M) )
r
which is orthogonal to M , (see Figure 1.8).
r
Q ⋅M r r r
q (M ) ⋅ M = 0

r r r r
M q (M) = Q skew ⋅ M r
M

Figure 1.8
b)
A :B = ( A sym + A skew ) : (B sym + B skew )
= A sym : B sym + 1
A sym
42
skew
: B43 A skew
+1 B sym + A skew : B skew
42: 43
=0 =0
= A sym : B sym + A skew : B skew
Then, it is also valid that:
A : B sym = A sym : B sym ; A : B skew = A skew : B skew
Q.E.D.
c)
 ijk T jk =  ij1 T j1 +  ij 2 T j 2 +  ij 3 T j 3 = 0 i
=  i11 T11 +  i 21 T21 +  i 31 T31 +  i12 T12 +  i 22 T22 +  i 32 T32 +  i13 T13 +  i 23 T23 +  i 33 T33
=  i 21 T21 +  i 31 T31 +  i12 T12 +  i 32 T32 +  i13 T13 +  i 23 T23 = 0 i
Then, the vector components are:
i =1 ⇒ 1 jk T jk = 132 T32 + 123 T23 = − T32 + T23 = 0 ⇒ T32 = T23
i=2 ⇒  2 jk T jk =  231 T31 +  213 T13 = T31 − T13 = 0 ⇒ T31 = T13
i=3 ⇒  3 jk T jk =  321 T21 +  312 T12 = − T21 + T12 = 0 ⇒ T21 = T12
with that we have shown that: if  ijk T jk = 0 i holds this implies that T is symmetric, i.e.
T = TT .

University of Castilla-La Mancha Draft By: Eduardo W. V. Chaves (2016)


Ciudad Real - Spain
1 TENSORS 23

Problem 1.32
Given a second-order tensor A in which the components of the symmetric part is known in
the Cartesian system:
 4 2 0
A ijsym =  2 1 0
0 0 3

ˆ , where the unit vector components are Nˆ = [1 0 0] .


ˆ ⋅ A ⋅N
Obtain N i

Solution:
ˆ ⋅ A ⋅N
In Problem 1.31 it was shown that N ˆ ⋅ A sym ⋅ N
ˆ =N ˆ with that we can obtain:

 4 2 0  1 
ˆ ⋅ A ⋅N
N ˆ ⋅A
ˆ =N sym
⋅ Nˆ = Ni A ijsymN j = [1 0 0]  2 1 0 0 = 4
0 0 3 0

Problem 1.33
Let W be an antisymmetric tensor. a) Show that W ⋅ W is a symmetric second-order tensor.
b) Also show that (W T ⋅ W ⋅ W ) : 1 = 0 .
Solution:
a) If we show that (W ⋅ W ) skew = 0 holds, then we prove that W ⋅ W is symmetric.

(W ⋅ W ) skew =
1
2
[ 1
2
] [ 1
( W ⋅ W) − (W ⋅ W )T = (W ⋅ W) − WT ⋅ WT = [(W ⋅ W) − W ⋅ W ] = 0
2
]
where we have applied the antisymmetric tensor property W = −W T .
Alternative solutions a) The tensor A is symmetric if A = A T , so, if we can show that
W ⋅ W = ( W ⋅ W )T , the tensor ( W ⋅ W ) is symmetric. Taking into account the definition of
antisymmetric tensor, W = − W T , we can obtain:
W ⋅ W = ( − W T ) ⋅ ( − W T ) = W T ⋅ W T = ( W ⋅ W )T
We can also check the symmetry by means of the tensor components:
 0 W12 W13   0 W12 W13 
( W ⋅ W ) ij =  − W12 0 W23   − W12 0 W23 
 − W13 − W23 0   − W13 − W23 0 
− W122
− W132 − W13 W23 W12 W23 
 
=  − W13 W23 − W122− 2
W23 − W12 W13 
 W12 W23 − W12 W13 2 
− W132 − W23
 
b) Let us try to write the term (W T ⋅ W ⋅ W ) : 1 (scalar) in indicial notation. Note that the
tensor (WT ⋅ W ⋅ W) is a second-order tensor, since:

University of Castilla-La Mancha Draft By: Eduardo W. V. Chaves (2016)


Ciudad Real - Spain
24 SOLVING PROBLEMS BY MEANS OF CONTINUUM MECHANICS

δ jl δ kp

W T ⋅ W ⋅ W = W ji eˆ i ⊗ eˆ j ⋅ Wlk eˆ l ⊗ eˆ k ⋅ W pq eˆ p ⊗ eˆ q = W ji Wlk W pqδ jlδ kp eˆ i ⊗ e q


= W ji W jk Wkq eˆ i ⊗ e q

and
δ ir

( W T ⋅ W ⋅ W ) : 1 = ( W ji W jk Wkq eˆ i ⊗ eˆ q ) : (δ rs eˆ r ⊗ eˆ s ) = W ji W jk Wkqδ rsδ irδ qs = W ji W jk Wki

δ qs
Note that the following is true:
(W T ⋅ W ⋅ W ) : 1 = (W T ⋅ W ⋅ W) iq δ iq = (W ji W jk Wkq )δ iq = W ji (W jk Wki ) = W : (W ⋅ W) = 0

In Problem 1.43 we will show that T : 1 = Tr (T ) , where T is a second-order tensor, so,


(W T ⋅ W ⋅ W ) : 1 = Tr (W T ⋅ W ⋅ W) = Tr[W T ⋅ (W ⋅ W )] = (W T )T : (W ⋅ W ) = W : (W ⋅ W)

where we have applied the trace property Tr ( A ⋅ B T ) = Tr ( AT ⋅ B) = A : B . Note also that


⇒ W
{ : (W ⋅ W ) = 0
Antisymmetric
1424 3
Symmetric

As we have seen in Problem 1.30, the double scalar product between a symmetric tensor
(W ⋅ W ) and an antisymmetric tensor ( W ) is zero.

Problem 1.34
1
Let B be a second-order tensor such that B pq =  pqs a s with a i =  ijk B jk . Prove that B is an
2
antisymmetric tensor.
Solution:
1  1 1
B pq =  pqs a s =  pqs   sjk B jk  =  pqs  sjk B jk =  pqs  jks B jk
2  2 2
Taking into account the relationship  pqs  jks = δ pj δ qk − δ pk δ qj , the above equation can be
rewritten as follows:
1 1 1 1
B pq =  pqs jksB jk = (δ pjδ qk − δ pkδ qj )B jk = (δ pjδ qk B jk − δ pkδ qjB jk ) = (B pq − B qp ) = B skew
pq
2 2 2 2
Alternative solution:
Taking into account that B qp =  qps a s and  pqs = − qps , we can conclude that:

B pq =  pqs a s = − qps a s = −B qp ⇒ B = −B T (antisymmetric)

University of Castilla-La Mancha Draft By: Eduardo W. V. Chaves (2016)


Ciudad Real - Spain
1 TENSORS 25

Problem 1.35
Show that the tensor A skew ⋅ A sym + A sym ⋅ A skew is an antisymmetric tensor.
Solution: Denoting by B = A skew ⋅ A sym + A sym ⋅ A skew , and by taking into account that
A skew = −(A skew ) T , A sym = (A sym ) T , we can conclude that:

B = A skew ⋅ A sym + A sym ⋅ A skew = A skew ⋅ A sym − A sym ⋅ ( A skew )T = A skew ⋅ A sym − ( A skew ⋅ A sym )T
= 2( A skew ⋅ A sym ) skew

Problem 1.36
r
Let T be an arbitrary second-order tensor, and n be a vector. Check if the relationship
r r
n ⋅ T = T ⋅ n is valid.
Solution:
r r
n ⋅ T = n i eˆ i ⋅ Tkl (eˆ k ⊗ eˆ l ) T ⋅ n = Tlk (eˆ l ⊗ eˆ k ) ⋅ n i eˆ i
= n i Tkl δ ik eˆ l = n i Tlk δ ki eˆ l
and
= n k Tkl eˆ l = n k Tlk eˆ l
= (n1 T1l + n 2 T2 l + n 3 T3l )eˆ l = (n1 Tl1 + n 2 Tl 2 + n 3 Tl 3 )eˆ l
With the above we can prove that n k Tkl ≠ n k Tlk , then:
r r
n⋅ T ≠ T ⋅n
r r
If T is a symmetric tensor, it follows that the relationship n ⋅ T sym = T sym ⋅ n holds.
Problem 1.37
r r r
Obtain the axial vector w associated with the antisymmetric second-order tensor ( x ⊗ a ) skew .
r
Solution: Let z be an arbitrary vector, it then holds that:
r r r r r
( x ⊗ a ) skew ⋅ z = w ∧ z
r r r
where w is the axial vector associated with ( x ⊗ a ) skew . Using the definition of an
antisymmetric tensor:
r r 1 r r
[r r 1 r r r r
]
( x ⊗ a ) skew = ( x ⊗ a ) − ( x ⊗ a )T = [ x ⊗ a − a ⊗ x ]
2 2
r r skew r r r
and by replacing it with ( x ⊗ a ) ⋅ z = w ∧ z , we obtain:
1 r r r r r r r
[x ⊗ a − a ⊗ x ] ⋅ z = w ∧ z ⇒ [xr ⊗ ar − ar ⊗ xr ] ⋅ zr = 2wr ∧ zr
2
r r r r r r r r
By using the equation [x ⊗ a − a ⊗ x ] ⋅ z = z ∧ ( x ∧ a ) , (see Problem 1.17), the above
equation becomes:
[xr ⊗ ar − ar ⊗ xr ] ⋅ zr = zr ∧ ( xr ∧ ar ) = (ar ∧ xr ) ∧ zr = 2wr ∧ zr
with the above we can conclude that:
r 1 r r r r
w = (a ∧ x ) is the axial vector associated with ( x ⊗ a ) skew
2

University of Castilla-La Mancha Draft By: Eduardo W. V. Chaves (2016)


Ciudad Real - Spain
26 SOLVING PROBLEMS BY MEANS OF CONTINUUM MECHANICS

Problem 1.38
Let us consider two symmetric tensors W (1) and W ( 2) , and their axial vectors represented
r r
respectively by w (1) and w ( 2) . Show that:
r r r r
W (1) ⋅ W ( 2 ) = ( w ( 2 ) ⊗ w (1) ) − ( w (1) ⋅ w ( 2) )1
r r
Tr[W (1) ⋅ W ( 2 ) ] = −2( w (1) ⋅ w ( 2) )
Solution: By means of the antisymmetric tensor properties, we can obtain that:
r r r
W (1) ⋅ a = w (1) ∧ a
r r r
a ⋅ W (1) = −a ∧ w (1)
T
r r r
r r r and W ( 2) ⋅ a = w ( 2 ) ∧ a
− a ⋅ W (1) = −a ∧ w (1)
r r r
a ⋅ W (1) = a ∧ w (1)
r r
Then, by applying the dot product (a ⋅ W (1) ) ⋅ ( W ( 2) ⋅ a) we obtain:
r r r r r r
(a ⋅ W (1) ) ⋅ ( W ( 2 ) ⋅ a) = (a ∧ w (1) ) ⋅ ( w ( 2 ) ∧ a)
We will continue the development in indicial notation:
(a i Wij(1) )(W jk(1) a k ) = ( ijk a j wk(1) )( ipq w (p2 ) a q )
[ ]
a i (Wij(1) W (jk1) )a k = a j ( ijk  ipq wk(1) w (p2 ) )a q = a j (δ jp δ kq − δ jq δ kp ) wk(1) w (p2) a q
[
= a j δ jp δ kq wk(1) w (p2 ) − δ jq δ kp wk(1) w (p2) a q]
=aj [w (1) ( 2 )
q wj − δ jq wk(1) wk( 2) a q]
In tensorial notation the above equation becomes:
r
[ ]
r r r
[ r r r
a ⋅ W (1) ⋅ W ( 2 ) ⋅ a = a ⋅ ( w ( 2 ) ⊗ w (1) ) − ( w (1) ⋅ w ( 2 ) )1 ] ⋅ ar
r r r r
With that we can conclude that W (1) ⋅ W ( 2) = ( w ( 2) ⊗ w (1) ) − ( w (1) ⋅ w ( 2) )1 .
b)

[ ] r
[ r r r r
] r
[ r r
]
Tr W (1) ⋅ W ( 2 ) = Tr ( w ( 2 ) ⊗ w (1) ) − ( w (1) ⋅ w ( 2 ) )1 = Tr ( w ( 2) ⊗ w (1) ) − Tr ( w (1) ⋅ w ( 2) )1 [ ]
r r r r r r
= ( w ( 2) ⋅ w (1) ) − ( w (1) ⋅ w ( 2 ) ) 1
Tr21] = −2( w (1) ⋅ w ( 2 ) )
[3
=3

Alternative solution
In this alternative solution we use the tensor components in which it fulfills:
 0 W12(1) W13(1)   0 − w3(1) w2(1) 
 (1)   (1) 
Wij(1) = − W12(1) 0 W23  =  w3 0 − w1(1) 
− W (1) − W12(1) 0   − w2(1) w1(1) 0 
 12

 0 W12( 2 ) W13( 2 )   0 − w3( 2 ) w2( 2 ) 


 (2)   (2) 
Wij( 2 ) = − W12( 2 ) 0 W23  =  w3 0 − w1( 2 ) 
− W ( 2 ) − W12( 2 ) 0  − w2( 2) w1( 2 ) 0 
 12

With that we can obtain:

University of Castilla-La Mancha Draft By: Eduardo W. V. Chaves (2016)


Ciudad Real - Spain
1 TENSORS 27

 0 − w3(1) w2(1)   0 − w3( 2 ) w2( 2 ) 


[W (1)
⋅ W ( 2) ]ij = Wik(1) Wkj( 2 )

=  w3(1) 0

− w1(1)   w3( 2 ) 0

− w1( 2 ) 
− w (1) w1(1) 0  − w2( 2) w1( 2 ) 0 
 2
− w3(1) w3( 2 ) − w2(1) w2( 2 ) w2(1) w1( 2 ) w3(1) w1( 2 ) 
 
⇒ Wik(1) Wkj( 2 ) = w1(1) w2( 2) − w3( 2 ) w3(1) − w1(1) w1( 2 ) w2( 2) w3(1) 
 w3( 2 ) w1(1) w3( 2 ) w2(1) − (1) ( 2 )
w2 w2 − w1 w1  ( 2 ) (1) 

In the term (11) of the above matrix we sum and subtract the term w1( 2) w1(1) , in the term (22)
we sum and subtract the term w2( 2) w2(1) and in the term (33) we add and subtract the term
w3( 2 ) w3(1) , so,

 w1( 2 ) w1(1) w1( 2 ) w2(1) w1( 2 ) w3(1) 


 
Wik(1) Wkj( 2 ) =  w2( 2 ) w1(1) w2( 2 ) w2(1) w2( 2 ) w3(1)  +
 w( 2 ) w(1) w3( 2 ) w2(1) w3( 2 ) w3(1) 
 3 1
 − w1(1) w1( 2 ) − w2(1) w2( 2 ) − w3(1) w3( 2 ) 0 0 
 
+ 0 − w1(1) w1( 2 ) − w2(1) w2( 2 ) − w3(1) w3( 2 ) 0 
 0 0 (1) ( 2 ) (1) ( 2 ) (1) ( 2 ) 
− w1 w1 − w2 w2 − w3 w3 

which is the same as:
Wik(1) Wkj( 2 ) = wi( 2) w (j1) − ( w1(1) w1( 2) + w2(1) w2( 2) + w3(1) w3( 2) )δ ij = wi( 2) w (j1) − ( wk(1) wk( 2) )δ ij

NOTE: The alternative solution by means of components was made only as a check. The
reader must give priority to the solution via indicial or tensorial notation, since the solution via
components is not always so simple to obtain.

1.5 Cofactor. Adjugate. Inverse. Particular Tensors.


Determinant

Problem 1.39
r r r r
Show that Tr (a ⊗ b) = a ⋅ b .
Solution:
r r r r
Tr (a ⊗ b) = Tr[(a i eˆ i ) ⊗ (b j eˆ j )] = ai b j Tr[eˆ i ⊗ eˆ j ] = ai b j (eˆ i ⋅ eˆ j ) = ai b jδ ij = a i b i = a ⋅ b
r r
= ai eˆ i ⋅ b j eˆ j = a ⋅ b

Problem 1.40
1
Given Tij = λE kk δ ij + 2µ Eij , W = Tij E ij , and P = Tij Tij , show that:
2
λ
W = µE : E + [Tr( E )]2 and P = 4µ 2 E : E + λ (3λ + 4µ )[Tr ( E )]
2

2
Solution 1: (Indicial notation)

University of Castilla-La Mancha Draft By: Eduardo W. V. Chaves (2016)


Ciudad Real - Spain
28 SOLVING PROBLEMS BY MEANS OF CONTINUUM MECHANICS

1
2
1
2
( 1
2
) ( 1
) (
W = Tij E ij = λE kk δ ij + 2µ E ij E ij = λE kk δ ij E ij + 2µ E ij E ij = λE kk E ii + 2µ E ij E ij
2
)
λ
since E kk = E ii = Tr (E ) and Eij Eij = E : E , we can conclude that W = µ E : E + [Tr( E )]2 .
2
( )(
P = TijTij = λEkkδ ij + 2 µEij λEqqδ ij + 2 µEij )
= λEkkδ ijλEqqδ ij + λEkkδ ij 2 µEij + 2 µEijλEqqδ ij + 2 µEij 2 µEij
= λ 2 Ekkδ ii Eqq + 2 µλEkk Eii + 2 µλEii Eqq + 4 µ 2 Eij Eij = 3λ 2 Ekk Eqq + 4 µλEkk Eii + 4 µ 2 Eij Eij
= λ(3λ + 4 µ )Ekk Eqq + 4 µ 2 Eij Eij

With that we show that P = 4 µ 2 E : E + λ(3λ + 4 µ )[ Tr ( E )]2 .


Solution 2: (Tensorial notation)
In tensorial notation we obtain:
1
T = λTr ( E )1 + 2µ E , W = T : E , and P = T : T
2
Then
1 1 1 1
W = T : E = (λTr ( E )1 + 2 µE ) : E = (λTr ( E )1 : E + 2 µE : E ) = (λTr ( E ) Tr ( E ) + 2 µE : E )
2 2 2 2
λ
= [Tr( E )]2 + µE : E
2
P = T : T = (λTr ( E )1 + 2 µE ) : (λTr ( E )1 + 2 µE )
= [λTr ( E )] 1 : 1 + 2 µλTr ( E ) 1 : E + 2 µλTr ( E ) {
E : 1 + (2 µ ) 2 E : E
2
{ {
=3 = Tr ( E ) = Tr ( E )

= 3λ [Tr ( E )] + 4 µλ[Tr ( E )] + 4 µ E : E = λ(3λ + 4 µ )[Tr ( E )] + 4 µ 2 E : E


2 2 2 2 2

Problem 1.41
Let σ ij be the second-order tensor components which are a function of ε ij , σ ij = σ ij (ε ij ) , and
is given by:
σ ij = λε kk δ ij + 2µ ε ij Tensorial
 → σ = λTr (ε )1 + 2µ ε

where λ and µ are scalars. Starting with the above equation, obtain an expression for ε ij in
function of σ ij , i.e. ε ij = ε ij (σ ij ) . Express the result in indicial and tensorial notation.
Solution:
Indicial notation Tensorial notation
σ ij = λε kk δ ij + 2µ ε ij σ = λTr (ε )1 + 2µ ε
⇒ 2µ ε ij = σ ij − λε kk δ ij ⇒ 2µ ε = σ − λTr (ε )1
1 λ 1 λ
⇒ ε ij = σ ij − ε kk δ ij ⇒ε= σ− Tr (ε )1
2µ 2µ 2µ 2µ

Next, we need to obtain the following trace ε kk , to do this we obtain the trace of σ ij :

University of Castilla-La Mancha Draft By: Eduardo W. V. Chaves (2016)


Ciudad Real - Spain
1 TENSORS 29

Indicial notation Tensorial notation


σ ij = λε kk δ ij + 2µ ε ij (i = j )
⇒ σ ii = λε kk δ ii + 2µ ε ii = λε kk 3 + 2µ ε kk σ : 1 = λTr (ε )1 : 1 + 2µ ε : 1
⇒ σ kk = (3λ + 2µ )ε kk Tr (σ ) = λTr (ε )3 + 2µ Tr (ε )
1 1
⇒ ε kk = σ kk ⇒ Tr (ε ) = Tr (σ )
(3λ + 2µ ) (3λ + 2µ )

Then
Indicial notation Tensorial notation
1 λ 1 λ
ε ij = σ ij − ε kk δ ij ε= σ− Tr (ε )1
2µ 2µ 2µ 2µ
1 λ 1 1 λ
= σ ij − σ kk δ ij = σ− Tr (σ )1
2µ 2µ (3λ + 2µ ) 2µ 2 µ (3λ + 2 µ )

Problem 1.42
Let T be a second-order tensor. Show that:
( T m )T = ( T T ) m and Tr ( T T ) m = Tr ( T m ) .
Solution:
( T m )T = ( T ⋅ T L T )T = T T ⋅ T T L T T = ( T T ) m

For the second demonstration we can use the trace property Tr ( T T ) = Tr ( T ) , thus:
Tr ( T T ) m = Tr ( T m )T = Tr ( T m )

Problem 1.43
Show that T : 1 = Tr (T ) , where T is an arbitrary second-order tensor.
Solution:
T : 1 = Tij eˆ i ⊗ eˆ j : δ kl eˆ k ⊗ eˆ l = Tijδ klδ ikδ jl = Tijδ ij = Tii = T jj = Tr ( T )

Problem 1.44
Show that if σ and D are second-order tensors, the following relationship is valid:
σ ⋅ ⋅ D = Tr (σ ⋅ D )
Solution: We start with the following definition:
σ ⋅ ⋅ D = σ ij D ji
= σ kj D jlδ ikδ il = σ kj D jlδ lk
= σ kj D jl δ lk
123
( σ⋅D )
kl
= (σ ⋅ D) kl δ lk = (σ ⋅ D) kk = (σ ⋅ D) ll
= Tr (σ ⋅ D)
An alternative demonstration would be:

University of Castilla-La Mancha Draft By: Eduardo W. V. Chaves (2016)


Ciudad Real - Spain
30 SOLVING PROBLEMS BY MEANS OF CONTINUUM MECHANICS

σ ⋅ ⋅ D = σijD ji = σ ijD jkδ ik = (σ ⋅ D ) : 1 = Tr (σ ⋅ D )

Problem 1.45
Show that A  tpq =  rjk A rt A jp A kq .
Solution:
We start with the following definition:
A =  rjk A r1 A j 2 A k 3 ⇒ A  tpq =  rjk  tpq A r1A j 2 A k 3 (1.23)
and also taking into account that the term  rjk  tpq can be replaced by:

δ rt δ rp δ rq
 rjk tpq = δ jt δ jp δ jq = δ rtδ jpδ kq + δ rpδ jqδ kt + δ rqδ jtδ kp − δ rqδ jpδ kt − δ jqδ kpδ rt − δ kqδ jtδ rp
δ kt δ kp δ kq
(1.24)
Then, by substituting (1.24) into (1.23) we can obtain:
A  tpq = A t1A p 2 A q 3 + A p1A q 2 A t 3 + A q1A t 2 A p 3 − A q1A p 2 A t 3 − A t1A q 2 A p 3 − A p1A t 2 A q 3
( ) ( ) ( )
= A t1 1 jk A pj A qk + A t 2  2 jk A pj A qk + A t 3  3 jk A pj A qk =  rjk A rt A jp A kq =  rjk A tr A pj A qk

NOTE: Let us consider that C = A ⋅ B ( C ij = A ik B kj ), then we can obtain


C = A ⋅ B =  rjk C r1C j 2 C k 3 =  rjk [A ⋅ B ]r1 [A ⋅ B ] j 2 [A ⋅ B ]k 3 =  rjk ( A rt B t1 )( A jpB p 2 )( A kqB q 3 )
=  rjk A rt A jp A kqB t1B p 2B q 3 = A  tpq B t1B p 2B q 3 = A B

So, we have shown that A ⋅ B = A B .


Alternative solution:
δ 1t δ 1 p δ 1q δ 1t δ 2t δ 3t
Considering that  tpq = δ 2t δ 2 p δ 2 q = δ 1 p δ 2 p δ 3 p and that A ⋅ B = A B we can
δ 3 t δ 3 p δ 3 q δ 1q δ 2 q δ 3 q
obtain:
δ 1t δ 2t δ 3t A11 A12 A13  δ 1t δ 2t δ 3t   A11 A12 A13 
 
A tpq =  rjk tpq A r1A j 2 A k 3 = δ 1 p δ 2 p δ 3 p A 21 A 22 A 23 = δ 1 p δ 2 p δ 3 p   A 21 A 22 A 23 
δ 1q δ 2 q δ 3q A 31 A 32 A 33 δ 1q δ 2 q δ 3q   A 31 A 32 A 33 
Note that δ 1t A11 + δ 2t A 21 + δ 3t A 31 = δ st A s1 = A t1 , with that we can obtain:
 A t1 At2 At3 
 
A  tpq =  rjk  tpq A r1A j 2 A k 3 =  A p1 A p2 A p 3  =  rjk A tr A pj A qk
 A q1 A q2 A q 3 

Problem 1.46
1
Show that A =  rjk  tpq A rt A jp A kq .
6

University of Castilla-La Mancha Draft By: Eduardo W. V. Chaves (2016)


Ciudad Real - Spain
1 TENSORS 31

Solution:
Starting with the definition A  tpq =  rjk A rt A jp A kq , (see Problem 1.45), and by multiplying
both sides of the equation by  tpq , we obtain:

A  tpq  tpq =  rjk  tpq A rt A jp A kq (1.25)


Note that  tpq  tpq = δ tt δ pp − δ tp δ tp = δ tt δ pp − δ tt = 6 . Then, the relationship (1.25) becomes:
1
A =  rjk  tpq A rt A jp A kq
6
Problem 1.47
Show the following property:
r r r r r r r r r r r r
(B ⋅ a) ⋅ (b ∧ c ) − (B ⋅ b) ⋅ (a ∧ c ) + (B ⋅ c ) ⋅ (a ∧ b) = Tr (B)[a ⋅ (b ∧ c )] (1.26)

Solution:
Expressing in Voigt notation the left side of the above equation we obtain:
r r r
 ijk (B ⋅ a) i b j c k −  ijk (B ⋅ b) i a j c k +  ijk (B ⋅ c ) i a j b k =
⇒=  ijk [(B i1a1 + B i 2 a 2 + B i 3 a 3 )b j c k − (B i1b1 + B i 2 b 2 + B i 3b 3 )a j c k +
+ (B i1 c 1 + B i 2 c 2 + B i 3 c 3 )a j b k ]
⇒=  ijk [(B i1a1b j c k + B i 2 a 2 b j c k + B i 3 a 3b j c k ) − (B i1b1 a j c k + B i 2 b 2 a j c k + B i 3b 3 a j c k ) +
+ (B i1 c 1 a j b k + B i 2 c 2 a j b k + B i 3 c 3 a j b k )]
⇒=  ijk [B i1 (a1b j c k − b1a j c k + c 1a j b k ) + B i 2 (a 2 b j c k − b 2 a j c k + c 2 a j b k ) +
+ B i 3 (a 3b j c k − b 3 a j c k + c 3 a j b k )]

⇒= (1 jk B11 +  2 jk B 21 +  3 jk B 31 )(a1b j c k − b1a j c k + c1a j b k ) +


(1 jk B12 +  2 jk B 22 +  3 jk B 32 )(a 2b j c k − b 2 a j c k + c 2 a j b k ) + (1.27)
(1 jk B13 +  2 jk B 23 +  3 jk B 33 )(a 3b j c k − b 3 a j c k + c 3a j b k )

Note that:
a1 a 2 a3
1 jk (a1b j c k − b1a j c k + c 1a j b k ) = b1 b 2 b 3 =  ijk a i b j c k
c1 c2 c3

 2 jk (a1b j c k − b1a j c k + c 1a j b k ) =  3 jk (a1b j c k − b1a j c k + c 1a j b k ) = 0


whereby the equation in (1.27) becomes:
r r r
B11ijk aib j c k + B 22ijk aib j c k + B 33ijk aib j c k = (B11 + B 22 + B 33 )ijk aib j c k = Tr (B)[a ⋅ (b ∧ c)]
Q.E.D.
Note also that:
r r r r r r r r r r r r
(BT ⋅ a) ⋅ (b ∧ c ) − (BT ⋅ b) ⋅ (a ∧ c ) + (BT ⋅ c ) ⋅ (a ∧ b) = Tr (B)[a ⋅ (b ∧ c)]

since Tr (B T ) = Tr (B) ≡ I B . It is also valid the following:

University of Castilla-La Mancha Draft By: Eduardo W. V. Chaves (2016)


Ciudad Real - Spain
32 SOLVING PROBLEMS BY MEANS OF CONTINUUM MECHANICS

r r r r r r r r r r r r
(B ⋅ a) ⋅ (b ∧ c ) + a ⋅ ((B ⋅ b) ∧ c ) + a ⋅ (b ∧ (B ⋅ c )) = Tr (B)[a ⋅ (b ∧ c )]
r r r r r r r r r r r r (1.28)
⇒ [(B ⋅ a), b, c ] + [a, (B ⋅ b), c ] + [a, b, (B ⋅ c )] = IB [a, b, c]

Problem 1.48
Show the following property:
r r r r r r
( A ⋅ a) ⋅ [( A ⋅ b) ∧ ( A ⋅ c )] = det ( A )[a ⋅ (b ∧ c)] (1.29)
r r r
where A is a non-singular second order tensor, and a , b and c are linearly independent
vectors.
Solution:
A non-singular tensor ⇒ det( A ) ≡ A ≠ 0 .
r r r r r r
a , b , c linearly independent vectors ⇒ a ⋅ (b ∧ c) ≠ 0 .
r r r
We express the scalar triple product in indicial notation, i.e. a ⋅ (b ∧ c ) = ijk aib j c k , and by
multiply both sides of this equation by the determinant of A we obtain:
r r r
a ⋅ (b ∧ c ) A = ijk aib j c k A

It was proven in Problem 1.45 that A  ijk =  pqr A pi A qj A rk , thus:


r r r
a ⋅ (b ∧ c ) A =  ijk a i b j c k A =  pqr A pi A qj A rk a i b j c k =  pqr ( A pi a i )( A qj b j )( A rk c k )
r r r r
[r
=  pqr ( A ⋅ a) p ( A ⋅ b) q ( A ⋅ c ) r = ( A ⋅ a) ⋅ ( A ⋅ b) ∧ ( A ⋅ c )
r
]
Problem 1.49
r r
Let a , b be arbitrary vectors and α , µ be scalars. Show that:
r r r r
det ( µ 1 + α a ⊗ b) = µ 3 + µ 2α a ⋅ b (1.30)

Solution: The determinant of A is given by A =  ijk A i1 A j 2 A k 3 . If we denote by


A ij µδ ij + α a i b j , thus, A i1 = µδ i1 + α a i b 1 , A j 2 = µδ j 2 + αa j b 2 , A k 3 = δ k 3 + α a k b 3 , then
the equation in (1.30) can be rewritten as:
r r
det ( µ 1 + α a ⊗ b) = ijk (µδ i1 + α aib1 ) µδ ( j2 )
+ α a jb 2 (µδ k 3 + α ak b3 ) (1.31)
By developing the equation (1.31), we obtain:
r r
[
det ( µ 1 + αa ⊗ b) = ijk µ 3δ i1δ j 2δ k 3 + µ 2αa k b3δ i1δ j2 + µ 2αa jb 2δ i1δ k 3 + µ 2αaib1δ j 2δ k 3 +
+ µα 2a jb 2a k b3δ i1 + µα 2ai a k b1b3δ j2 + µα 2ai a jb1b 2δ k 3 + α 3ai a j a k b1b 2b3 ]
Note that: µ 3  ijk δ i1δ j 2 δ k 3 = µ 3 123 = µ 3 ,

University of Castilla-La Mancha Draft By: Eduardo W. V. Chaves (2016)


Ciudad Real - Spain
1 TENSORS 33

µ 2 α ( ijk a k b 3δ i1δ j 2 +  ijk a j b 2 δ i1δ k 3 +  ijk a i b1δ j 2 δ k 3 ) =


r r
µ 2 α (12 k a k b 3 + 1 j 3 a j b 2 +  i 23 a i b1 ) = µ 2 α (a 3b 3 + a 2 b 2 + a1b1 ) = µ 2 α (a k b k ) = µ 2 α (a ⋅ b)

 ijk a i a k b1b 3 δ j 2 =  i 2 k a i a k b1b 3 = a1a 3b1b 3 − a 3 a1b1b 3 = 0


 ijk a i a j b1b 2 δ k 3 =  ij 3 a i a j b1b 2 = 123 a1a 2 b1b 2 −  213 a 2 a1b1b 2 = 0
 ijk a i a j a k b1b 2 b 3 = 0
Notice that, there was no need to expand the terms  ijk a i a k b1b 3δ j 2 ,  ijk a i a j b1b 2 δ k 3 , and
 ijk a i a j a k b1b 2 b 3 to realize that these terms equal zero, since
r r
 ijk a i a k b1b 3 δ j 2 = (a ∧ a) j b1b 3 δ j 2 = 0 , similarly for other terms.
Taking into account the above considerations we can prove that:
r r r r
det ( µ 1 + α a ⊗ b) = µ 3 + µ 2α a ⋅ b
For the particular case when µ = 1 the above equation becomes:
r r r r
det (1 + α a ⊗ b) = 1 + α a ⋅ b
r r
Then, it is simple to prove that det (α a ⊗ b) = 0 , since
r r
det (αa ⊗ b) = α 3ijk ai a j ak b1b 2b3 = α 3b1b 2b3 [a ⋅ (a ∧ a)] = 0
r r r

NOTE: We can extrapolate the equation in (1.30) in such a way that:

det ( µI sym + α A ⊗ B) = µ 3 + µ 2α A : B (1.32)

where I sym is the symmetric fourth-order unit tensor, A and B are second-order tensors.
Note that det ( I sym ) = (1) 3 + (1) 2 (0)(0 : 0 ) = 1 and det (1 ⊗ 1) = (0) 3 + (0) 2 (1)(1 : 1) = 0 .

Problem 1.50
r r
Let A be an arbitrary second-order tensor. Show that there is a nonzero vector n ≠ 0 so that
r r
A ⋅ n = 0 if and only if det ( A ) = 0 , Chadwick (1976).
r r
Solution: Firstly, we show that, if det ( A ) ≡ A = 0 ⇒ n ≠ 0 . Secondly, we show that, if
r r
n ≠ 0 ⇒ det ( A ) ≡ A = 0 .
r r r
We assume that det ( A ) ≡ A = 0 , and we choose an arbitrary basis {f , g, h} (linearly
independent), then:

(
r r r
) r r
[ r
]
f ⋅ g ∧ h A = ( A ⋅ f ) ⋅ ( A ⋅ g) ∧ ( A ⋅ h) , (see Problem 1.48)

Due to the fact that det ( A ) ≡ A = 0 , the implication is that:


r r r
[
( A ⋅ f ) ⋅ ( A ⋅ g) ∧ ( A ⋅ h) = 0 ]
r r r
Thus, we can conclude that the vectors ( A ⋅ f ) , ( A ⋅ g) , ( A ⋅ h) , are linearly dependent. This
implies that there are nonzero scalars α , β , γ so that:

University of Castilla-La Mancha Draft By: Eduardo W. V. Chaves (2016)


Ciudad Real - Spain
34 SOLVING PROBLEMS BY MEANS OF CONTINUUM MECHANICS

r r r r
(r r
α ( A ⋅ f ) + β ( A ⋅ g) + γ ( A ⋅ h) = 0 ⇒ A ⋅ αf + β g + γh = 0 ⇒ A ⋅ n = 0
r
) r r r

r r r r r r r r
where n = αf + β g + γh ≠ 0 since {f , g, h} is linearly independent.
r r r
Now we choose two vectors k , m , which are linearly independent to n . Once more, we
apply definition:
r r r r
k ⋅ (m ∧ n) A = ( A ⋅ k ) ⋅ [( A ⋅ m) ∧ ( A ⋅ n)]
r r

r r r r r r r r
Considering that A ⋅ n = 0 , and k ⋅ (m ∧ n) ≠ 0 owing to the fact that k , m , n are linearly
independent, we can conclude that:
r r r
k ⋅ (m ∧ n) A = 0 ⇒ A =0
14243
≠0

Problem 1.51
Let F be an arbitrary second-order tensor. Show that the resulting tensors C = F T ⋅ F and
b = F ⋅ F T are symmetric tensors and semi-positive definite tensors. Also check in what condition are
C and b positive definite tensors.
Solution: Symmetry:
C T = (F T ⋅ F )T = F T ⋅ (F T )T = F T ⋅ F = C
b T = (F ⋅ F T ) T = (F T )T ⋅ F T = F ⋅ F T = b

Thus, we have shown that C = F T ⋅ F and b = F ⋅ F T are symmetric tensors.


To prove that the tensors C = F T ⋅ F and b = F ⋅ F T are semi-positive definite tensors, we
start with the definition of a semi-positive definite tensor, i.e., a tensor A is semi-positive
r
definite if xˆ ⋅ A ⋅ xˆ ≥ 0 holds, for all xˆ ≠ 0 . Thus:
xˆ ⋅ ( F T ⋅ F ) ⋅ xˆ = F ⋅ xˆ ⋅ F ⋅ xˆ xˆ ⋅ ( F ⋅ F T ) ⋅ xˆ = xˆ ⋅ F ⋅ F T ⋅ xˆ
= ( F ⋅ xˆ ) ⋅ ( F ⋅ xˆ ) = ( F T ⋅ xˆ ) ⋅ ( F T ⋅ xˆ )
2
= F ⋅ xˆ = F T ⋅ xˆ ≥ 0
2
≥0

Or in indicial notation:
x i C ij x j = x i ( Fki Fkj ) x j x i bij x j = x i ( Fik F jk ) x j
= ( Fki x i )( Fkj x j ) = ( Fik x i )( F jk x j )
2 2
= Fki x i ≥0 = Fik x i ≥0

Thus, we proved that C = F T ⋅ F and b = F ⋅ F T are semi-positive definite tensors. Note that
r r
xˆ ⋅ C ⋅ xˆ = F ⋅ xˆ equals zero, when xˆ ≠ 0 , if F ⋅ xˆ = 0 . Furthermore, by definition
2

r r
F ⋅ xˆ = 0 with xˆ ≠ 0 if and only if det ( F ) = 0 , (see Problem 1.50). Then, the tensors
C = F T ⋅ F and b = F ⋅ F T are positive definite if and only if det ( F ) ≠ 0 .

University of Castilla-La Mancha Draft By: Eduardo W. V. Chaves (2016)


Ciudad Real - Spain
1 TENSORS 35

Problem 1.52
r r r r r r
Let dX (1) , dX ( 2) , dX (3) , dx (1) , dx ( 2) , dx (3) be vectors, and they are related to each other as
r r r r r r
follows dx (1) = F ⋅ dX (1) , dx ( 2) = F ⋅ dX ( 2) , dx (3) = F ⋅ dX (3) , where F is a non-singular
second-order tensor and ∃F −1 . a.1) Considering dV = dx (1) ⋅ (dx ( 2) ∧ dx (3) ) ≠ 0 and
r r r
r r r
dV0 = dX (1) ⋅ (dX ( 2 ) ∧ dX (3) ) ≠ 0 , obtain a relationship between the scalars dV and dV0 in
r r r r
terms of F . a.2) Obtain the relationship between c = dX ( 2) ∧ dX (3) ≠ 0 and
r r r r
c * = dx ( 2 ) ∧ dx ( 3 ) ≠ 0 .
Solution
a.1) Taking into account the problem statement it fulfills that:
r r r r r
[ r
dV = dx (1) ⋅ (dx ( 2 ) ∧ dx (3) ) = ( F ⋅ dX (1) ) ⋅ ( F ⋅ dX ( 2 ) ) ∧ ( F ⋅ dX (3) ) ]
r r r
In Problem 1.48 it was proven that a ⋅ (b ∧ c) A = ( A ⋅ a) ⋅ ( A ⋅ b) ∧ ( A ⋅ c ) , so
r
[ r r
]
r r r r r
[ r
a ⋅ (b ∧ c) A = ( A ⋅ a) ⋅ ( A ⋅ b) ∧ ( A ⋅ c ) ]
r r r r r
[ r
⇒ dX (1) ⋅ (dX ( 2 ) ∧ dX (3) ) F = ( F ⋅ dX (1) ) ⋅ ( F ⋅ dX ( 2 ) ) ∧ ( F ⋅ dX (3) ) ]
With that we conclude that:
r r r r
[ r r r
] r
[ r
dV = dx (1) ⋅ (dx ( 2 ) ∧ dx (3) ) = ( F ⋅ dX (1) ) ⋅ ( F ⋅ dX ( 2 ) ) ∧ ( F ⋅ dX (3) ) = F dX (1) ⋅ (dX ( 2 ) ∧ dX (3) ) ]
thus
dV = F dV0 (1.33)

a.2) Since exist the inverse of F , i.e. ∃F −1 , we can obtain that


r r r r r r
dx (1) = F ⋅ dX (1) ⇒ F −1 ⋅ dx (1) = F −1 ⋅ F ⋅ dX (1) = 1 ⋅ dX (1) = dX (1)
And by taking into account the equation (1.33) we can obtain:
r r r r r r
dV = F dV0 ⇒ dx (1) ⋅ (dx ( 2 ) ∧ dx (3) ) = F dX (1) ⋅ [dX ( 2 ) ∧ dX (3) ]
r r r r r r
⇒ dx (1) ⋅ ( dx ( 2 ) ∧ dx (3) ) = F ( F −1 ⋅ dx (1) ) ⋅ [dX ( 2 ) ∧ dX (3) ]
r r r r
( r r
⇒ dx (1) ⋅ ( dx ( 2 ) ∧ dx (3) ) = dx (1) ⋅ F F −T ⋅ [dX ( 2) ∧ dX (3) ] )
r r r r r r
⇒ ( dx ( 2 ) ∧ dx (3) ) = F F −T ⋅ [dX ( 2 ) ∧ dX (3) ] ⇒ c * = F F −T ⋅ c
r r
NOTE 1: Note that c * ≠ F ⋅ c . We can rewrite the above equation as follows
r r r
[ r
dx ( 2 ) ∧ dx (3) = F F −T ⋅ dX ( 2 ) ∧ dX (3) ] ⇒
r r r
[ r
( F ⋅ dX ( 2 ) ) ∧ ( F ⋅ dX (3) ) = F F −T ⋅ dX ( 2 ) ∧ dX (3) ]
The tensor F F −T is known as the cofactor of F , i.e. cof ( F ) = F F −T with this we define
the inverse of a tensor:
cof ( F ) = F F −T ⇒ [ F F −T ]T = [cof ( F )]T ⇒ F F −1 = [cof ( F )]T
1 1
⇒ F −1 = [cof ( F )]T = [adj( F )]
F F

University of Castilla-La Mancha Draft By: Eduardo W. V. Chaves (2016)


Ciudad Real - Spain
36 SOLVING PROBLEMS BY MEANS OF CONTINUUM MECHANICS

r r r
F dx (1) = F ⋅ dX (1)
dX (1)
r r r
r c * = dx ( 2 ) ∧ dx ( 3 )
dX ( 3 ) r r
r r r c* ≠ F ⋅c
c = dX ( 2) ∧ dX (3) r r
c * = [cof ( F )] ⋅ c r r
dx ( 3) = F ⋅ dX ( 3)
dV = F dV0
r r r
dX ( 2 ) dx ( 2) = F ⋅ dX ( 2)

r r r
dV0 = dX (1) ⋅ (dX ( 2) ∧ dX ( 3) ) ≠ 0 r r r
dV = dx (1) ⋅ ( dx ( 2) ∧ dx ( 3) ) ≠ 0
−1
F
Figure 1.9
r r r
NOTE 2: Let us suppose now that F = A ⋅ B , and let us consider three vectors a ⋅ (b ∧ c ) ≠ 0 ,
r r r r r r
and a * = B ⋅ a , b * = B ⋅ b , c * = B ⋅ c , thus by apply the previous definitions we can state:
r r r r
[ r r
F a ⋅ (b ∧ c ) = ( F ⋅ a ) ⋅ ( F ⋅ b ) ∧ ( F ⋅ c ) ] r
[ r r
= ( A ⋅ B ⋅ a) ⋅ ( A ⋅ B ⋅ b) ∧ ( A ⋅ B ⋅ c ) ]
r
[ r r
= ( A ⋅ a* ) ⋅ ( A ⋅ b*) ∧ ( A ⋅ c * ) ]
r r r r
[ r r
= A a * ⋅ (b * ∧ c * ) = A ( B ⋅ a ) ⋅ ( B ⋅ b ) ∧ ( B ⋅ c ) ]
r r r
= A B a ⋅ (b ∧ c )

With that we can conclude that: if F = A ⋅ B then F = A ⋅ B = A B .


Problem 1.53
Let A and B be orthogonal tensors, show that the tensor C = A ⋅ B is also an orthogonal
tensor.
Solution: By definition, a tensor is orthogonal if C −1 = C T holds:
C −1 = ( A ⋅ B) −1 = B −1 ⋅ A −1 = B T ⋅ A T = ( A ⋅ B) T = C T
Q.E.D.

Problem 1.54
Show that adj( A ⋅ B) = adj(B) ⋅ adj( A ) and cof( A ⋅ B) = [cof( A )] ⋅ [cof(B)] .
Solution:
Based on the definition of the inverse of a tensor we can say that:

B −1 ⋅ A −1 =
[adj(B)] ⋅ [adj(A )]
B A
⇒ A B B −1 ⋅ A −1 = [adj(B)] ⋅ [adj( A )] = [cof(B)] ⋅ [cof( A)]T
T

⇒ A B (A ⋅ B ) = [adj(B)] ⋅ [adj( A )] =
−1
( [cof(A)]⋅ [cof(B)] ) T
(1.34)

⇒AB
[adj(A ⋅ B)] = [adj(B)] ⋅ [adj(A)] = ([cof(A)] ⋅ [cof(B)])T
A ⋅B
⇒ adj( A ⋅ B) = [adj(B)] ⋅ [adj( A )] = ([cof( A )] ⋅ [cof(B)])
T

Q.E.D.

University of Castilla-La Mancha Draft By: Eduardo W. V. Chaves (2016)


Ciudad Real - Spain
1 TENSORS 37

where we have used the property A ⋅ B = A B . Also taking into account the definition of
adjugate and cofactor we can conclude that:
adj( A ⋅ B) = ([cof( A ⋅ B)]) = ([cof( A )]⋅ [cof(B)]) [cof(A ⋅ B)] = [cof(A)]⋅ [cof(B)]
T T
⇒ (1.35)

Problem 1.55
Show that:
r r r r
( A ⋅ a) ∧ ( A ⋅ b) = [cof( A )] ⋅ (a ∧ b) (1.36)

Solution:
Starting from the equation A  tpq =  rjk A rt A jp A kq , (see Problem 1.45), and by multiply both
sides by a t b p , we obtain:
A  tpq a t b p =  rjk A rt A jp A kq a t b p =  rjk ( A rt a t )( A jp b p ) A kq

Multiplying both sides by A −qs1 we obtain:

A  tpq a t b p A −qs1 =  rjk ( A rt a t )( A jp b p ) A kq A −qs1 =  rjk ( A rt a t )( A jp b p )δ ks =  rjs ( A rt a t )( A jp b p )

−1
[cof ( A )] sq
Note that A qs = holds, whereby the above equation becomes:
A

−1
[cof ( A )] sq
A  tpq a t b p A qs = A  tpq a t b p = [cof ( A )] sq  tpq a t b p =  rjs ( A rt a t )( A jp b p )
A
r r r r
⇒ [cof( A )] ⋅ (a ∧ b) = ( A ⋅ a) ∧ ( A ⋅ b)

Problem 1.56
Show that:
r
[ r r
] r r
[ r r
] r r
[ ]
r r r
a ⋅ ( A ⋅ b) ∧ ( A ⋅ c ) + ( A ⋅ a) ⋅ b ∧ ( A ⋅ c ) + ( A ⋅ a) ⋅ ( A ⋅ b) ∧ c = Tr ([cof ( A )]) a ⋅ (b ∧ c ) [ ]
(1.37)
Solution:
r r r r
In Problem 1.55 it was demonstrated that [cof( A )] ⋅ (a ∧ b) = ( A ⋅ a) ∧ ( A ⋅ b) , thus the
following relationships hold:
r r r r
[r r
a ⋅ [cof( A )] ⋅ (b ∧ c ) = a ⋅ ( A ⋅ b) ∧ ( A ⋅ c ) ]
r r r r
− b ⋅ [cof( A )] ⋅ (a ∧ c ) = −b ⋅ [( A ⋅ a) ∧ ( A ⋅ c ) ] = ( A ⋅ a) ⋅ b ∧ ( A ⋅ c )
r r r r r
[ ]
r r r r
[r r
]r r
c ⋅ [cof( A )] ⋅ (a ∧ b) = c ⋅ ( A ⋅ a) ∧ ( A ⋅ b) = ( A ⋅ a) ⋅ ( A ⋅ b) ∧ c
r
[ ]
Summing the three above equations we obtain:
r r r r r r r r r
a ⋅ [cof( A )] ⋅ (b ∧ c ) − b ⋅ [cof( A )] ⋅ (a ∧ c ) + c ⋅ [cof( A )] ⋅ (a ∧ b) =
r
[ r r
]r r
[
r r
] r r
= a ⋅ ( A ⋅ b) ∧ ( A ⋅ c ) + ( A ⋅ a) ⋅ b ∧ ( A ⋅ c ) + ( A ⋅ a) ⋅ ( A ⋅ b) ∧ c [ ]

University of Castilla-La Mancha Draft By: Eduardo W. V. Chaves (2016)


Ciudad Real - Spain
38 SOLVING PROBLEMS BY MEANS OF CONTINUUM MECHANICS

According to Problem 1.47 the following is true:


([cof( A )] ⋅ ar )⋅ (b ∧ cr ) − ([cof( A )] ⋅ b)⋅ (ar ∧ cr ) + ([cof( A )] ⋅ cr )⋅ (ar ∧ b) = Tr ([cof( A )]r)[cr ⋅ (ar ∧ b)]
T
r T
r T
r r
r r
= II A [c ⋅ (a ∧ b )]
where II A = Tr [cof( A )] is the second principal invariant of A , thus:
r
[ r r
] r r
[ r
] r
[r r
]
r r r
a ⋅ ( A ⋅ b) ∧ ( A ⋅ c ) + ( A ⋅ a) ⋅ b ∧ ( A ⋅ c ) + ( A ⋅ a) ⋅ ( A ⋅ b) ∧ c = II A a ⋅ (b ∧ c ) [ ]
NOTE 1: We can summarize that:
r r r r
[ r r r r
] [ r
] [
r r r
( A ⋅ a) ⋅ (b ∧ c) + a ⋅ ( A ⋅ b) ∧ c ) + a ⋅ b ∧ ( A ⋅ c ) = I A a ⋅ (b ∧ c ) ] (see Problem 1.47) (1.38)

a ⋅ [( A ⋅ b) ∧ ( A ⋅ c )] + ( A ⋅ a) ⋅ [b ∧ ( A ⋅ c )] + ( A ⋅ a) ⋅ [( A ⋅ b) ∧ c ] = II [a ⋅ (b ∧ c )]
r r r r r r r r r r r r
A
(1.39)

( A ⋅ a) ⋅ [( A ⋅ b) ∧ ( A ⋅ c )] = III [a ⋅ (b ∧ c )] (see Problem 1.48)


r r r r r r
A
(1.40)
r r r r r r
where I A = Tr (A ) , II A = Tr ([cof( A )]) , III A = det (A ) . Using the notation a ⋅ (b ∧ c ) ≡ [a, b, c ] ,
the above equations can also be written as follows:
r r r r r r r r r r r r
[( A ⋅ a), b, c ] + [a, ( A ⋅ b), c] + [a, b, ( A ⋅ c )] = I A [a, b, c ]
r r r r r r r r r r r r
[a, ( A ⋅ b), ( A ⋅ c )] + [( A ⋅ a), b, ( A ⋅ c )] + [( A ⋅ a), ( A ⋅ b), c ] = II A [a, b, c]
r r r r r r
[( A ⋅ a), ( A ⋅ b), ( A ⋅ c )] = III A [a, b, c ]
r r r r r r
NOTE 2: If we consider three linearly independent vectors [a ⋅ (b ∧ c )] ≡ [a, b, c ] ≠ 0 , and
three vectors such as:
r r r r r r
f = α 1a + α 2 b + α 3 c f  α
 r α 2 α 3  ar 
r  r   1  
β 2 β 3  b
r r
g = β 1 a + β 2 b + β 3 c ⇒ g = β 1 (1.41)
r r r r  r  γ r
γ 2 γ 3  c 
h = γ 1 a + γ 2 b + γ 3 c h   1  
And according to Cramer’s rule, (see Problem 1.16), the following relationships are true:
r r r r r r r r r r r r
f ⋅ (b ∧ c ) [ f , b, c ] [a, f , c ] [a, b, f ]
α1 = r r r ≡ r r r ; α2 = r r r ; α3 = r r r
a ⋅ (b ∧ c ) [a, b, c ] [a, b, c ] [a, b, c ]
r r r r r r r r r
[g, b, c ] [a, g, c ] [a, b, g]
β1 = r r r ; β2 = r r r ; β3 = r r r
[a, b, c ] [a, b, c ] [a, b, c ]
r r r r r r r r r
[h, b, c ] [a, h, c ] [a, b, h]
γ1 = r r r ; γ2 = r r r ; γ3 = r r r
[a, b, c ] [a, b, c ] [a, b, c ]
r r r r r r
By performing the triple scalar product [ f ⋅ (g ∧ h)] ≡ [f , g, h] , we can obtain:

University of Castilla-La Mancha Draft By: Eduardo W. V. Chaves (2016)


Ciudad Real - Spain
1 TENSORS 39

r r r α 1 α2 α3 
β r r r
[ f ⋅ (g ∧ h)] =  1 β 2 β 3  [a, b, c ]
 γ 1 γ 2 γ 3 
r r r r r r r r r
[ f , b, c ] [a, f , c ] [a, b, f ] 
1  r r r r r r r r r  r r r r r r
= r r r [g , b, c ] [a, g, c ] [a, b, g] [a, b, c ] = P [a, b, c ]
[a, b, c ]  r r r r r r r r r
[h, b, c ] [a, h, c ] [a, b, h]
 

where
r r r r r r r r r
α 1 α 2 α 3  [ f , b, c ] [a, f , c ] [a, b, f ] 
1  r r r r r r r r r 
P = β 1 β 2 β 3  = r r r [g, b, c ] [a, g, c ] [a, b, g] (1.42)
r r r r r r r r r
 γ 1 γ 2 γ 3  [a, b, c ] [h, b, c ] [a, h, c ] [a, b, h]
 
r r r r r r
For the case when f = A ⋅ a , g = A ⋅ b , h = A ⋅ c , the principal invariants of P are:
1
( r r r r r r r r r
I P = Tr ( P ) = r r r [ A ⋅ a, b, c] + [a, A ⋅ b, c] + [a, b, A ⋅ c ] = I A
[a, b, c]
)
r r r r r r r r r r r r r r r r r r
1  [a , A ⋅ b, c ] [a, b, A ⋅ b] [ A ⋅ a, b, c] [a, b, A ⋅ a] [ A ⋅ a, b, c ] [a, A ⋅ a, c ] 
II P = r r r  r r r r r r + r r r r r r + r r r r r r
([a, b, c ]) 2  [a, A ⋅ c, c] [a, b, A ⋅ c] [ A ⋅ c, b, c ] [a, b, A ⋅ c] [ A ⋅ b, b, c] [a, A ⋅ b, c] 
= II A
III P = III A = det (A )
NOTE 3: Let us consider the Cartesian system where
r r
a = a1eˆ 1 + a 2 eˆ 2 + a 3 eˆ 3 a  a a2 a 3  eˆ 1 
 r  r   1  
b = b1eˆ 1 + b 2 eˆ 2 + b 3 eˆ 3 ⇒ b  = b1 b2 b 3  eˆ 2 
r cr  c c2 c 3  eˆ 3 
c = c 1eˆ 1 + c 2 eˆ 2 + c 3 eˆ 3    1  
r r r
Also let us consider that f = eˆ 1 , g = eˆ 2 , h = eˆ 3 , so, taking into account the above equation
and the equation in (1.41) we can conclude that:
r r
f  α α 2 α 3  ar  1 0 0  eˆ 1  α 1 α 2 α 3  a1 a 2 a 3  eˆ 1 
 r   1   0 1 0  eˆ  = β  
g  =  β 1 β 2 β 3  b ⇒   2   1 β 2 β 3  b1 b 2 b 3  eˆ 2 
 r  γ r
γ 2 γ 3  c  0 0 1  eˆ 3   γ 1 γ 2 γ 3   c 1 c 2 c 3  eˆ 3 
h   1    
thus
−1
α 1 α 2 α 3   a 1 a 2 a 3  1 0 0  α 1 α 2 α 3   a1 a 2 a 3 
β β 2 β 3  b1 b 2 b 3  = 0 1 0 ⇒ β β 2 β 3  = b1 b 2 b 3 
 1  1
 γ 1 γ 2 γ 3   c 1 c 2 c 3  0 0 1  γ 1 γ 2 γ 3  c 1 c 2 c 3 
With that we can obtain the inverse of a tensor. Let us consider the tensor A where the
components are:
 A 11 A12 A 13  a1 a 2 a3 
r r r
A ij =  A 21 A 22 A 23  = b1 b 2 b 3  ⇒ A = [a, b, c ]
 A 31 A 32 A 33   c1 c 2 c 3 

University of Castilla-La Mancha Draft By: Eduardo W. V. Chaves (2016)


Ciudad Real - Spain
40 SOLVING PROBLEMS BY MEANS OF CONTINUUM MECHANICS

Then, the inverse P = A −1 , (see equation (1.42)), becomes:


r r r r r r r r r
[ f , b, c ] [a, f , c ] [a, b, f ] 
1  r r r r r r r r r 
A −1 = r r r [g, b, c ] [a, g, c ] [a, b, g]
[a, b, c ]  r r r r r r r r r
[h, b, c ] [a, h, c ] [a, b, h]
 
 1 0 0 a1 a2 a3 a1 a 3 
a2

 b1 b2 b3 1 0 0 b1 b 2 b 3 
   b2 b3 a2 a3 a2 a3 
 c1 c2 c3 c1 c2 c3 1 0 0   − 
   c2 c3 c2 c3 b2 b3 
 0 1 0 a1 a2 a3 a1 a 2 a 3   
1   1  b1 b3 a1 a3 a1 a 3 
= b1 b2 b3 0 1 0 b1 b 2 b 3  = − −
A  A  c1 c3 c1 c3 b1 b 3 
 c1 c2 c3 c1 c2 c3 0 1 0   
   b1 b2 a1 a2 a1 a 2 
   − 
 0 0 1 a1 a2 a3 a1 a 2 a 3   c 1 c2 c1 c2 b1 b 2 

 b1 b2 b3 0 0 1 b1 b 2 b 3 
 
c c2 c3 c1 c2 c3 0 0 1 
 1 
1
Taking into account that A −1 = [cof( A )]T = 1 [adj( A )] , we can conclude that:
A A
T
 b b3 a2 a3 a2a3   b b3 b1 b 3 b1 b 2 
 2 −   2 − 
 c2 c3 c2 c3 b2 b3   c2 c3 c1 c3 c1 c 2 
   
b b3 a1 a3 a1 a3  a a3 a1 a3 a1 a 2 
[cof( A )]ij =  − 1 −  =  − 2 −
c c3 c1 c3 b1 b 3 c c3 c1 c3 c1 c 2 
 1   2 
 b1 b2 a1 a2 a1 a 2   a2 a3 a1 a3 a1 a 2 
 −   − 
 c1 c2 c1 c2 b1 b 2 
  b 2 b3 b1 b 3 b1 b 2 

Note that the coefficient of the above matrix, [cof(A )]ij , can be obtained by solving the
determinant of the resulting matrix by removing the i th row and the j th column, which result
we multiply by (−1) i + j , for example:

a1 a 2 a3
b b3
[cof(A)]12 = (−1) 1+ 2
b1 b 2 b3 = − 1
c1 c 3
c1 c2 c3

Problem 1.57
Given the scalars I C , II C , III C in terms of the scalars I E , II E , III E :
I C = 2I E + 3

 II C = 4 I E + 4 II E + 3 (1.43)

 III C = 2 I E + 4 II E + 8 III E + 1

University of Castilla-La Mancha Draft By: Eduardo W. V. Chaves (2016)


Ciudad Real - Spain
1 TENSORS 41

Obtain the reverse form of the above equations, i.e. obtain I E , II E , III E in terms of I C , II C ,
III C .
Solution:
The equations in (1.43) can be restructured as follows:
 I C   2 0 0  I E  3  2 0 0   I E   I C  3
        4 4 0   II  =  II  − 3
 II C  =  4 4 0  II E  + 3 ⇒   E   C   
 III   2 4 8   III  1   2 4 8   III E   III C  1 
 C   E   
−1 −1
 2 0 0  2 0 0   I E   2 0 0    I C  3 
     
⇒  4 4 0  4 4 0   II E  =  4 4 0    II C  − 3 
 2 4 8   2 4 8   III E   2 4 8    III C  1  
−1
 I E  2 0 0  I C − 3 
     
 II E  =  4 4 0   II C − 3 
 III   2 4 8   III − 1
 E    C 
where
T
 4 0 4 0 4 4 
 −   1 
−1  4 8 2 8 2 4 
 2 0 0
2 0 0   −1 
1 1  0 0 2 0 2 0 1
A −1
=  4 4 0  = [cof( A ) ]T = − − =  0
A 64  4 8 2 8 2 4  2 4 
 2 4 8     1 −1 1
 0 0 2 0 2 0   8
 −  8 8 
 4 0 4 0 4 4 

with that the scalars I E , II E , III E can be obtained as follows:

 1  1 
0 0  2 ( I C − 3) 
 I C − 3   2
−1
I −3  
 I E   2 0 0
     −1 1   C  1



 II E  =  4 4 0  II C − 3  =  0   II C − 3  =  ( −2 I C + II C + 3) 
 III   2 4 8   III − 1  2 4 
III − 1 4 
 E    C   1 −1 1  C  1 
 8 8 8   8 ( I C − II C + III C − 1) 
 

1.6 Additive Decomposition of Tensors

Problem 1.58
Find a fourth-order tensor P so that P : A = A dev , where A is a second-order tensor.
Solution: Taking into account the additive decomposition into spherical and deviatoric parts,
we obtain:
Tr ( A ) Tr ( A )
A = A sph + A dev = 1 + A dev ⇒ A dev = A − 1
3 3
By definition the fourth-order tensors are:
I = 1⊗1 = δ ik δ jl eˆ i ⊗ eˆ j ⊗ eˆ k ⊗ eˆ l = I ijkl eˆ i ⊗ eˆ j ⊗ eˆ k ⊗ eˆ l (1.44)

University of Castilla-La Mancha Draft By: Eduardo W. V. Chaves (2016)


Ciudad Real - Spain
42 SOLVING PROBLEMS BY MEANS OF CONTINUUM MECHANICS

I = 1⊗1 = δ il δ jk eˆ i ⊗ eˆ j ⊗ eˆ k ⊗ eˆ l = I ijkl eˆ i ⊗ eˆ j ⊗ eˆ k ⊗ eˆ l (1.45)


I = 1 ⊗ 1 = δ ij δ kl eˆ i ⊗ eˆ j ⊗ eˆ k ⊗ eˆ l = I ijkl eˆ i ⊗ eˆ j ⊗ eˆ k ⊗ eˆ l (1.46)
where it holds that:
( )(
I : A = δ ik δ jl eˆ i ⊗ eˆ j ⊗ eˆ k ⊗ eˆ l : A pq eˆ p ⊗ eˆ q )
(
= δ ik δ jl A pq δ kp δ lq eˆ i ⊗ eˆ j )
(
= δ ik δ jl A kl eˆ i ⊗ eˆ j ) (1.47)
(
= A ij eˆ i ⊗ eˆ j )
=A
( )(
I : A = δ ij δ kl eˆ i ⊗ eˆ j ⊗ eˆ k ⊗ eˆ l : A pq eˆ p ⊗ eˆ q )
(
= δ ij δ kl A pq δ kp δ lq eˆ i ⊗ eˆ j )
(
= δ ij δ kl A kl eˆ i ⊗ eˆ j ) (1.48)
(
= A kk δ ij eˆ i ⊗ eˆ j )
= Tr ( A )1
Referring to the definition of fourth-order unit tensors seen in (1.47), and (1.48), where the
relations I : A = Tr ( A )1 and I : A = A hold, we can now state:
Tr ( A ) 1  1   1 
A dev = A − 1 = I : A − I : A =  I − I  : A =  I − 1 ⊗ 1 : A
3 3  3   3 
Therefore, we can conclude that:
1
P = I − 1 ⊗1
3
The tensor P is known as a fourth-order projection tensor, Holzapfel(2000).

1.7 Transformation Law for Tensor Components. Invariants.

Problem 1.59
Under the base transformation eˆ ′i = a ij eˆ j and by considering that the second-order tensor
components in this new base are given by:
Tij′ = a ik a jl Tkl

Show that:
a) Tii′ = Tkk = Tr (T ) ; b) Tij′ T ′ji = Tkl Tlk ; c) det ( T ′) = det ( T )
Solution:
Note that the transformation matrix aij = A is an orthogonal matrix, i.e. A −1 = A T , so the
relationship A T A = 1 (aki akj = δ ij ) hold.
=j
a) Tij′ = aik a jl Tkl i→ Tii′ = aik ail Tkl = δ kl Tkl = Tkk = Tll

b) Tij′ T ′ji = (a ik a jl Tkl )(a jp a iq T pq ) = a ik a iq a jl a jp Tkl T pq = δ kq δ lp Tkl T pq = Tqp T pq = Tkl Tlk


123 123
=δ kq =δ lp

University of Castilla-La Mancha Draft By: Eduardo W. V. Chaves (2016)


Ciudad Real - Spain
1 TENSORS 43

with that we show that Tr ( T 2 ) = Tr ( T ⋅ T ) = Tij T ji


c) det ( Tij′ ) = det(a ik a jl Tkl ) = det (a ik )det (a jl )det ( Tkl ) = det( Tkl )
1
424
31424
3
=1 =1

we have just shown that Tkk = Tr (T ) , Tkl Tlk = Tr ( T ⋅ T ) and det (T ) are invariants.

Problem 1.60
Let T be a symmetric second-order tensor and I T , II T , III T be scalars, where:

I T = Tr ( T ) = Tii ; II T =
1
2
{2
I T − Tr ( T 2 ) } ; III T = det ( T )

Show that I T , II T , III T are invariant with a change of basis.


Solution:
a) Taking into account the transformation law for the second-order tensor components
Tij′ = a ik a jl Tkl or in matrix form T ′ = A T A T . Then, Tii′ is:

Tii′ = a ik a il Tkl = δ kl Tkl = Tkk = I T


Hence we have proved that I T is independent of the adopted system.
b) To prove that II T is an invariant, one only needs to show that Tr ( T 2 ) is one also, since
2
I T is already an invariant.

Tr ( T ′ 2 ) = Tr ( T ′ ⋅ T ′) = T ′ : T ′ = Tij′ Tij′ = ( a ik a jl Tkl )( a ip a jq T pq )


= a ik a ip a jl a jq Tkl T pq
123 123
δ kp δ lq
= T pl T pl
= T : T = Tr ( T ⋅ T ) = Tr ( T 2 )

c) Matrix form: det ( T ′) = det (T ′) = det (A T A T ) = det


1
424
(A )det ( T )det (A T ) = det ( T )
3 1424 3
=1 =1

Problem 1.61
Show that the following relations are invariants:
C12 + C 22 + C 32 ; C13 + C 23 + C 33 ; C14 + C 24 + C 34
where C1 , C 2 , C 3 are the eigenvalues of the second-order tensor C .
Solution:
Recall that in the principal space of C the following is true:
C1 0 0  I C = C1 + C 2 + C3

Cij′ =  0 C 2 0  ⇒  II C = C 2 C3 + C1C3 + C1C 2
 0 0 C3   III = C C C
 C 1 2 3

Any combination of invariants is also an invariant, so, on this basis, we can try to express the
above expressions in terms of their principal invariants.
2
1444
(
424444 3
)
I C2 = (C1 + C 2 + C3 ) = C12 + C 22 + C 32 + 2 C1 C 2 + C1 C3 + C 2 C3 ⇒ C12 + C 22 + C 32 = I C2 − 2 II C
II C

University of Castilla-La Mancha Draft By: Eduardo W. V. Chaves (2016)


Ciudad Real - Spain
44 SOLVING PROBLEMS BY MEANS OF CONTINUUM MECHANICS

So, we have proved that C12 + C 22 + C 32 is an invariant. Similarly, we can obtain the other
relationships, so, we summarize:
C1 + C 2 + C3 = I C
C12 + C 22 + C32 = I C2 − 2 II C
C13 + C 23 + C33 = I C3 − 3 II C I C + 3 III C
C14 + C 24 + C34 = I C4 − 4 II C I C2 + 4 III C I C + 2 II C2
C15 + C 25 + C35 = I C5 − 5 II C I C3 + 5 III C I C2 + 5 II C2 I C − 5 III C II C

The following is also true:


( ) ( ) (
C1n+1 + C 2n+1 + C3n +1 = C1n + C 2n + C3n I C − C1 C 2n −1 + C3n−1 − C 2 C1n−1 + C3n−1 − C3 C1n −1 + C 2n−1 ) ( )
Problem 1.62
Show that, if a symmetric second-order tensor T has three different real eigenvalues
(λ 1 ≠ λ 2 ≠ λ 3 ) , the principal space of T is formed by an orthonormal basis.
Solution:
Consider a symmetric second-order tensor T . By the definition of eigenvalues, given in
T ⋅ nˆ ( a ) = λ a nˆ ( a ) , if λ 1 , λ 2 , λ 3 are the eigenvalues of T , then it follows that:

T ⋅ nˆ (1) = λ1nˆ (1) ; T ⋅ nˆ ( 2 ) = λ 2nˆ ( 2 ) ; T ⋅ nˆ (3) = λ 3nˆ (3) (1.49)


Applying the dot product between n̂ ( 2) and T ⋅ nˆ (1) = λ 1nˆ (1) , and the dot product between n̂ (1)
and T ⋅ nˆ ( 2) = λ 2 nˆ ( 2) we obtain:
nˆ ( 2 ) ⋅ T ⋅ nˆ (1) = λ1nˆ ( 2 ) ⋅ nˆ (1) ; nˆ (1) ⋅ T ⋅ nˆ ( 2 ) = λ 2nˆ (1) ⋅ nˆ ( 2) (1.50)
Since T is symmetric, it holds that nˆ ( 2) ⋅ T ⋅ nˆ (1) = nˆ (1) ⋅ T ⋅ nˆ ( 2) , so:
λ 1nˆ ( 2) ⋅ nˆ (1) = λ 2 nˆ (1) ⋅ nˆ ( 2 ) = λ 2 nˆ ( 2 ) ⋅ nˆ (1) (1.51)
⇒ (λ1 − λ 2 ) nˆ (1)
⋅ nˆ (2)
=0 (1.52)
To satisfy the equation (1.52), with λ1 ≠ λ 2 ≠ 0 , the following must be true:
nˆ (1) ⋅ nˆ ( 2 ) = 0 (1.53)
Similarly, it is possible to show that nˆ (1) ⋅ nˆ (3) = 0 and nˆ ( 2) ⋅ nˆ (3) = 0 and then we can conclude
that the eigenvectors are mutually orthogonal, and constitute an orthogonal basis, where the
transformation matrix between systems is:
 nˆ (1)   nˆ 1(1) nˆ (21) nˆ 3(1) 
   
A = nˆ ( 2)  = nˆ 1( 2) nˆ (22 ) nˆ 3( 2 )  (1.54)
nˆ (3)  nˆ (3) nˆ (23) nˆ (33) 
   1
NOTE: If the tensor is not symmetric the principal space not necessarily is orthogonal.

Problem 1.63
Obtain the components of T ′ , given by the transformation:
T′ = A ⋅ T ⋅ AT

University of Castilla-La Mancha Draft By: Eduardo W. V. Chaves (2016)


Ciudad Real - Spain
1 TENSORS 45

where the components of T and A are shown, respectively, as Tij and a ij . Afterwards, given
that a ij are the components of the transformation matrix, represent graphically the
components of the tensors T and T ′ on both systems.
Solution: The expression T ′ = A ⋅ T ⋅ A T in symbolic notation is given by:
′ (eˆ a ⊗ eˆ b ) = ars (eˆ r ⊗ eˆ s ) ⋅ T pq (eˆ p ⊗ eˆ q ) ⋅ akl (eˆ l ⊗ eˆ k ) = ars T pq aklδ spδ ql (eˆ r ⊗ eˆ k )
Tab
= arp T pq akq (eˆ r ⊗ eˆ k )

To obtain the components of T ′ one only need make the double scalar product with the basis
(eˆ i ⊗ eˆ j ) , the result of which is:

′ (eˆ a ⊗ eˆ b ) : (eˆ i ⊗ eˆ j ) = arp T pq akq (eˆ r ⊗ eˆ k ) : (eˆ i ⊗ eˆ j )


Tab
′ δ aiδ bj = arp T pq akqδ riδ kj
Tab ⇒ Tij′ = aip T pq a jq

The above equation is shown in matrix notation as:


T ′ = A T A T inverse
→ T = A −1 T ′ A −T
Since A is an orthogonal matrix, it holds that A T = A −1 . Thus, T = A T T ′ A . The
graphical representation of the tensor components in both systems can be seen in Figure 1.10.

T ′ = A T AT x3′

T33


T23

T32 ′
T22
x3
T33 T13′


T31 T12′ x2′

T13 ′
T21
T23 T11′
T32
T31 T22

T12
T21 x2 x1′
T11

x1
T = AT T ′ A
Figure 1.10: Transformation law for the second-order tensor components.

University of Castilla-La Mancha Draft By: Eduardo W. V. Chaves (2016)


Ciudad Real - Spain
46 SOLVING PROBLEMS BY MEANS OF CONTINUUM MECHANICS

Problem 1.64
Let T be a second-order tensor whose components in the Cartesian system (x1 , x 2 , x3 ) are
given by:
 3 − 1 0
(T )ij = Tij = T =  − 1 3 0 
 0 0 1 

Given that the transformation matrix between two systems, (x1 , x 2 , x3 ) - (x1′ , x 2′ , x 3′ ) , is:
 
 0 0 1
 2 2 
A= 0
 2 2 
 2 2 
− 0
 2 2 
Obtain the tensor components Tij in the new coordinate system (x1′ , x 2′ , x 3′ ) .
Solution: The transformation law for second-order tensor components is Tij′ = aik a jl Tkl .
To enable the previous calculation to be carried out in matrix form we use:
[ ]
Tij′ = [a i k ] [Tk l ] a l j
T

Thus
T ′ = A T AT

 0  2 2
0 1 0 − 
  2 2 
 2   3 − 1 0  
2   2 2 
T ′=  0   − 1 3 0  0
 2 2   2 2 
 2 2   0 0 1   
− 0 1 0 0 
 2 2   
On carrying out the operation of the previous matrices we now have:
1 0 0 
T ′ = 0 2 0 
0 0 4 

University of Castilla-La Mancha Draft By: Eduardo W. V. Chaves (2016)


Ciudad Real - Spain
1 TENSORS 47

Problem 1.65
Find the transformation matrix between the systems: x, y , z and x′′′, y′′′, z′′′ . These systems are
represented in Figure 1.11.

z = z′
z ′′ = z ′′′

y ′′′
β
γ y ′ = y ′′
α y

x α x′

γ
x ′′′
x ′′
Figure 1.11: Rotation.
Solution: Note that: if we have an initial space and successive transformations until the final
space, the transformation law from the initial space to the final space is formed by the product
of the transformations in the opposite direction. That is, we place in the final space and we
follow opposite direction of the arrows until the initial space, (see Figure 1.12).

r
a′
A r
a′′

B −1
r
a A −1
C −1 C
initial
space
r current
CBA a′′′
space

A −1B −1 C −1 = (CBA ) −1 iforthogonal


  s → A T B T C T = (CBA ) T

Figure 1.12
The coordinate system x′′′, y′′′, z′′′ can be obtained by different combinations of rotations as
follows:

University of Castilla-La Mancha Draft By: Eduardo W. V. Chaves (2016)


Ciudad Real - Spain
48 SOLVING PROBLEMS BY MEANS OF CONTINUUM MECHANICS

♦ Rotation along the z -axis, (see Figure 1.13).

z = z′ from x, y , z to x′, y′, z′


with 0 ≤ α ≤ 360 º
 cos α sin α 0
A = − sin α cos α 0
y′  0 0 1 
α y
y
α x′
y ′′
x x′
α
x

Figure 1.13

♦ Rotation along the y′ -axis, (see Figure 1.14).

z = z′ from x′, y′, z′ to x′′, y′′, z′′


z ′′
cos β 0 − sin β 
B =  0 1 0 
 sin β 0 cos β 
β y ′ = y ′′
z = z′ z ′′
α y

x α x′
x′
β
with 0 ≤ β ≤ 180 º
x ′′
x ′′

Figure 1.14
♦ Rotation along the z ′′ -axis, (see Figure 1.15)
z = z′ from x′′, y′′, z′′ to x′′′, y′′′, z′′′
with 0 ≤ γ ≤ 360º
z ′′ = z ′′′
 cos γ sin γ 0
C = − sin γ cos γ 0 
y ′′′  0 0 1 
β
γ y ′ = y ′′
α y
y ′′
α x′ x′′′
x
y ′′′
γ γ
x ′′′ x′′

x ′′

Figure 1.15

University of Castilla-La Mancha Draft By: Eduardo W. V. Chaves (2016)


Ciudad Real - Spain
1 TENSORS 49

The transformation matrix from ( x, y , z ) to ( x′′′, y′′′, z′′′ ) is given by


D = CBA
After multiplying the matrices, we obtain:
 (cos α cos β cos γ − sin α sin γ ) (sin α cos β cos γ + cos α sin γ ) − sin β cos γ 
D = ( − cos α cos β sin γ − sin α cos γ ) (− sin α cos β sin γ + cos α cos γ ) sin β sin γ 

 cos α sin β sin α sin β cos β 

The angles α, β , γ are known as Euler angles and were introduced by Leonhard Euler to
describe the orientation of a rigid body motion, which is discussed in Chapter 4.

Problem 1.66
If a ij represent the components of the base transformation matrix, show that the following
equations are true:
a11
2 2
+ a12 + a132
=1 a11
2 2
+ a 21 2
+ a 31 =1
 2 2 2
 2 2 2
a 21 + a 22 + a 23 = 1 a12 + a 22 + a 32 = 1
 2 2 2  2 2 2
a 31 + a 32 + a 33 = 1 a13 + a 23 + a 33 = 1
 or 
a11 a 21 + a12 a 22 + a13 a 23 = 0 a11 a12 + a 21 a 22 + a 31 a 32 = 0
 
a 21 a31 + a 22 a 32 + a 23 a 33 = 0 a12 a13 + a 22 a 23 + a 32 a 33 = 0
a a + a a + a a = 0 a a + a a + a a = 0
 11 31 12 32 13 33  11 13 21 23 31 33

Solution:
We start from the principle that the basis transformation matrix is an orthogonal matrix, i.e.
a ik a jk = a ki a kj = δ ij . Then:

(i = 1, j = 1) 2
a11 2
+ a12 2
+ a13 =1
 2 2 2
(i = 2, j = 2) a 21 + a 22 + a 23 =1
 2 2 2
(i = 3, j = 3) a31 + a 32 + a 33 =1
a ik a jk = a i1 a j1 + a i 2 a j 2 + a i 3 a j 3 = δ ij ⇒ 
(i = 1, j = 2) a11 a 21 + a12 a 22 + a13 a 23 = 0

(i = 2, j = 3) a 21 a 31 + a 22 a 32 + a 23 a 33 = 0
(i = 1, j = 3) a11 a 31 + a12 a 32 + a13 a 33 = 0

Alternative solution:
 a11 a12 a13   a11 a 21 a 31  1 0 0 
AA = 1 T
⇒ a a 22 a 23   a12 a 22 a 32  = 0 1 0 
 21
 a 31 a 32 a 33   a13 a 23 a 33  0 0 1 

Performing the matrix multiplication we obtain:


 2
a11 2
+ a12 2
+ a13 a11 a 21 + a12 a 22 + a13 a 23 a11 a 31 + a12 a 32 + a13 a 33  1 0 0 
 
 a11 a 21 + a12 a 22 + a13 a 23
2
a 21 2
+ a 22 2
+ a 23 a 21 a 31 + a 22 a 32 + a 23 a 33  = 0 1 0 
a a + a a + a a a 21 a 31 + a 22 a 32 + a 23 a 33 2
a 31 2
+ a 32 2
+ a 33  0 0 1 
 11 31 12 32 13 33   

University of Castilla-La Mancha Draft By: Eduardo W. V. Chaves (2016)


Ciudad Real - Spain
50 SOLVING PROBLEMS BY MEANS OF CONTINUUM MECHANICS

Problem 1.67
r r
a) Obtain the transformation matrix, a ij , from the system OX to the system ox , (see Figure
1.16). The plane defined by the triangle 1 − 2 − 3 is lying on the plane x2 − x3 . b) Obtain the
triangle area in terms of the node coordinates.

x1 x3
3 ( X 1(3) , X 2(3) , X 3(3) )


x2

2 ( X 1( 2 ) , X 2( 2 ) , X 3( 2 ) )
X3
o
1 ( X 1(1) , X 2(1) , X 3(1) )
ê 3
O
ê1 ê 2 X2

X1

Figure 1.16

Solution:
a) The unit vector associated with the direction 12 is given by:
^ (12)1 ˆ (12)2 ˆ (12)3 ˆ
12 = e1 + e2 + e3 = a21eˆ 1 + a22eˆ 2 + a23eˆ 3
12 12 12

where
12 = ( X 1( 2) − X 1(1) )eˆ 1 + ( X 2( 2) − X 2(1) )eˆ 2 + ( X 3( 2) − X 3(1) )eˆ 3

12 = ( X 1( 2) − X 1(1) ) 2 + ( X 2( 2) − X 2(1) )2 + ( X 3( 2) − X 3(1) )2

^ ^ ^
The unit vector nˆ // ox1 can be obtained by nˆ = 12∧ 13 . And the unit vector 13 is given by:

^ (13)1 ˆ (13)2 ˆ (13)3 ˆ


13 = e1 + e2 + e3 = s1eˆ 1 + s2eˆ 2 + s3eˆ 3
13 13 13

where
13 = ( X 1(3) − X 1(1) )eˆ 1 + ( X 2(3) − X 2(1) )eˆ 2 + ( X 3(3) − X 3(1) )eˆ 3

13 = ( X 1(3) − X 1(1) )2 + ( X 2(3) − X 2(1) )2 + ( X 3(3) − X 3(1) ) 2

Then, we can calculate:

University of Castilla-La Mancha Draft By: Eduardo W. V. Chaves (2016)


Ciudad Real - Spain
1 TENSORS 51

eˆ 1 eˆ 2 eˆ 3
^ ^
nˆ = 12∧ 13 = a21 a22 a23 = (a22 s3 − a23s2 )eˆ 1 + (a23s1 − a21s3 )eˆ 2 + (a21s2 − a22 s1 )eˆ 3
s1 s2 s3
⇒ nˆ = a11eˆ 1 + a12eˆ 2 + a13eˆ 3

The unit vector associated with the direction ox3 can be calculated as follows:
eˆ 1 eˆ 2 eˆ 3
^ ^
ox3 = nˆ ∧ 12 = a11 a12 a13 = (a12 a23 − a13a22 )eˆ 1 + (a13a21 − a11a23 )eˆ 2 + (a11a22 − a12 a21 )eˆ 3
a21 a22 a23
^
⇒ ox3 = a31eˆ 1 + a32eˆ 2 + a33eˆ 3
r r
Then, the transformation matrix from the system OX to the system ox is given by
 a11 a12 a13 
A = aij =  a21 a22 a23 
 a31 a32 a33 

b) The area vector, formed by 12 and 13 , can be obtained by the means of the cross product
as follows:
eˆ 1 eˆ 2 eˆ 3
r (12) 2 (12)3 ˆ (12)1 (12)3 ˆ (12)1 (12) 2 ˆ
A = (12 ∧ 13) = (12)1 (12) 2 (12)3 = e1 − e2 + e3
(13) 2 (13)3 (13)1 (13)3 (13)1 (13) 2
(13)1 (13) 2 (13)3
= A1eˆ 1 + A2eˆ 2 + A3eˆ 3
And the triangle area is defined by
1 r 1 1
AT = A = (12 ∧ 13) = A12 + A22 + A32
2 2 2
2 2 2
1  (12) 2 (12)3   (12)1 (12)3   (12)1 (12) 2 
= + − +
2  (13) 2 (13)3   (13)1 (13)3   (13)1 (13) 2 
     
1
= [(12) 2 (13)3 − (12)3 (13) 2 ]2 + [(12)1 (13)3 − (12)3 (13)1 ]2 + [(12)1 (13) 2 − (12) 2 (13)1 ]2
2
where the components of the vectors 12 and 13 are
12 = ( X 1( 2 ) − X 1(1) )eˆ 1 + ( X 2( 2 ) − X 2(1) )eˆ 2 + ( X 3( 2) − X 3(1) )eˆ 3
13 = ( X 1(3) − X 1(1) )eˆ 1 + ( X 2(3) − X 2(1) )eˆ 2 + ( X 3(3) − X 3(1) )eˆ 3

University of Castilla-La Mancha Draft By: Eduardo W. V. Chaves (2016)


Ciudad Real - Spain
52 SOLVING PROBLEMS BY MEANS OF CONTINUUM MECHANICS

1.8 Eigenvalues, Eigenvectors, Orthogonal Transformation

Problem 1.68
Let Q be a proper orthogonal tensor, and E be an arbitrary second-order tensor. Show that
the eigenvalues of E do not change with the following orthogonal transformation:
E* = Q ⋅ E ⋅ QT
Solution:
We will take into account that the eigenvalues of E are obtained by solving the determinant
E − λ1 = 0 , and that the spherical tensor is the same for any transformation
1 = Q ⋅ λ1 ⋅ Q T = 1 * , then:

0 = det (E* − λ1* ) 0 = det (E *ij − λδ *ij )


= det (Q ⋅ E ⋅ QT − Q ⋅ λ1 ⋅ Q T ) = det (Q ik E kp Q jp − λQ ik Q jpδ kp )
= det[Q ⋅ (E − λ1) ⋅ Q ] T
= det[Q ik (E kp − λδ kp )Q jp ]
T
= det (Q) det (E − λ1) det (Q ) = det (Q ik )det (E kp − λδ kp ) det (Q jp )
123 1424 3
1 1
= det (E kp − λδ kp )
= det (E − λ1)

Thus, we have proved that E and E * have the same eigenvalues.

Problem 1.69
Let A be a second-order tensor and Q be an orthogonal tensor. If the orthogonal
transformation law to A is given by A * = Q ⋅ A ⋅ Q T , show that A 2 = Q ⋅ A 2 ⋅ Q T .
*

Solution:
A 2 = A* ⋅ A* ( A 2 ) ij = ( A * ⋅ A * ) ij = A *ik A *kj
* *

= ( Q ⋅ A ⋅ Q T ) ⋅ (Q ⋅ A ⋅ Q T ) = (Q ip A pr Q kr )(Q ks A st Q jt )
= Q⋅A ⋅Q ⋅Q⋅A ⋅Q
T T
= Q ip A pr Q kr Q ks A st Q jt
123 123
=1
=δ rs
= Q ⋅ A ⋅ A ⋅ QT
= Q ip A pr δ rs A st Q jt = Q ip A ps A st Q jt
= Q ⋅ A 2 ⋅ QT
= Q ip ( A ⋅ A ) pt Q jt
= (Q ⋅ A 2 ⋅ Q T ) ij

Problem 1.70
Given the tensor components:
5 3 3 
Tij = 2 6 3
2 2 4

a) Obtain the principal invariants of T , i.e. obtain I T , II T and III T ;

University of Castilla-La Mancha Draft By: Eduardo W. V. Chaves (2016)


Ciudad Real - Spain
1 TENSORS 53

b) Obtain the characteristic polynomial associated with T ;


c) If λ 1 , λ 2 and λ 3 are the eigenvalues of T and λ 1 = 10 . Obtain λ 2 and λ 3 > 2 .
Solution:
a) The principal invariants of T are:
I T = Tr ( T ) = 5 + 6 + 4 = 15

6 3 5 3 5 3
II T = + + = 56
2 4 2 4 2 6
III T = det ( T ) = 60
b) The characteristic polynomial can be obtained by solving the determinant:
5−λ 3 3
2 6−λ 3 =0 ⇒ λ3 − λ2 I T + λ II T − III T = 0
2 2 4−λ
thus:
λ3 − 15λ2 + 56λ − 60 = 0
c) In the principal space the following is true:
λ 1 = 10 0 0 
Tij′ =  0 λ2 0 

 0 0 λ 3 > 2
where the principal invariants are
I T = Tr ( T ) = λ 1 + λ 2 + λ 3 = 15 ⇒ λ2 + λ3 = 5
III T = det ( T ) = λ 1λ 2 λ 3 = 60 ⇒ λ 2λ3 = 6
By combining these two equations we can obtain:
λ 2λ3 = 6  2
λ(31) = 3
 ⇒ (5 − λ 3 )λ 3 = 6 ⇒ λ 3 − 5λ 3 + 6 = 0 ⇒  (2)
λ 2 + λ 3 = 5 λ 3 = 2

We discard the solution λ(32) = 2 , thus λ 3 = 3 . Then:


10 0 0
Tij′ =  0 2 0
 0 0 3

 I T = Tr ( T ) = 10 + 2 + 3 = 15

In this space we can check that  II T = 2 × 3 + 10 × 3 + 10 × 2 = 56
 III = det ( T ) = 10 × 2 × 3 = 60
 T

Problem 1.71
Find the principal values and directions of the second-order tensor T , where the Cartesian
components of T are:

University of Castilla-La Mancha Draft By: Eduardo W. V. Chaves (2016)


Ciudad Real - Spain
54 SOLVING PROBLEMS BY MEANS OF CONTINUUM MECHANICS

 3 − 1 0
(T )ij = Tij = T =  − 1 3 0 
 0 0 1 

Solution: We need to find nontrivial solutions for ( Tij − λδ ij ) nˆ j = 0 i , which are constrained by
nˆ j nˆ j = 1 (unit vector). As we have seen, the nontrivial solution requires that:

Tij − λδ ij = 0

Explicitly, the above equation is:


T11 − λ T12 T13 3 − λ −1 0
T21 T22 − λ T23 = −1 3 − λ 0 =0
T31 T32 T33 − λ 0 0 1− λ

Developing the above determinant, we can obtain the cubic equation:


(1 − λ )[(3 − λ ) 2 − 1] = 0 ⇒ λ3 − 7λ2 + 14λ − 8 = 0
We could have obtained the characteristic equation directly in terms of invariants:
I T = Tr ( Tij ) = Tii = T11 + T22 + T33 = 7

T T23 T11 T13 T11 T12


II T =
1
2
( )
Tii T jj − Tij Tij = 22
T32 T33
+
T31 T33
+
T21 T22
= 14

III T = Tij =  ijk Ti1 T j 2 Tk 3 = 8

Then, the characteristic equation becomes:


λ3 − λ2 I T + λ II T − III T = 0 → λ3 − 7λ2 + 14λ − 8 = 0
On solving the cubic equation we obtain three real roots, namely:
λ 1 = 1; λ 2 = 2; λ3 = 4
We can also verify that:
I T = λ1 + λ 2 + λ 3 = 1 + 2 + 4 = 7 ✓
II T = λ 1 λ 2 + λ 2 λ 3 + λ 3 λ 1 = 1 × 2 + 2 × 4 + 4 × 1 = 14 ✓
III T = λ 1 λ 2 λ 3 = 8 ✓
Thus, we can see that the invariants are the same as those evaluated previously.
Principal directions:
Each eigenvalue, λ i , is associated with a corresponding eigenvector, nˆ (i ) . We can use the
equation ( Tij − λδ ij ) nˆ j = 0 i to obtain the principal directions.
Œ λ1 = 1
3 − λ 1 −1 0   n 1  3 − 1 − 1 0   n1   0 
 −1 3 − λ1 0  n 2  =  − 1 3 − 1 0  n 2  = 0 
   

 0 0 1 − λ 1  n 3   0 0 1 − 1 n 3  0 

These become the following system of equations:

University of Castilla-La Mancha Draft By: Eduardo W. V. Chaves (2016)


Ciudad Real - Spain
1 TENSORS 55

2n1 − n 2 = 0 
  ⇒ n1 = n 2 = 0
− n1 + 2n 2 = 0
0n = 0
 3
n i n i = n12 + n 22 + n 32 = 1

Then we can conclude that: λ1 = 1 ⇒ nˆ i(1) = [0 0 ± 1] .


This solution could have been directly determined by the specific features of the T matrix.
As the terms T13 = T23 = T31 = T32 = 0 imply that T33 = 1 is already a principal value, then,
consequently, the original direction is a principal direction.
λ2 = 2

3 − λ 2 −1 0   n 1  3 − 2 − 1 0   n1  0 
 −1 3 − λ2 0  n  =  − 1 3 − 2 0  n 2  = 0 
  2  
 0 0 1 − λ 2  n 3   0 0 1 − 2  n 3  0 

n1 − n 2 = 0 ⇒ n1 = n 2

 − n1 + n 2 = 0
− n = 0
 3
The first two equations are linearly dependent, after which we need an additional equation:
1
n i n i = n12 + n 22 + n 32 = 1 ⇒ 2n12 = 1 ⇒ n1 = ±
2
Thus:
 1 1 
λ2 = 2 ⇒ nˆ i( 2 ) =  ± ± 0
 2 2 
λ3 = 4

3 − λ 3 −1 0   n 1  3 − 4 − 1 0   n1   0 
 −1 3 − λ3    
0  n 2  =  − 1 3 − 4 0  n 2  = 0 

 0 0 1 − λ 3  n 3   0 0 1 − 4  n 3  0 

 − n1 − n 2 = 0 
  ⇒ n1 = −n 2
 − n1 − n 2 = 0 
− 3n = 0
 3

1
n i n i = n12 + n 22 + n 32 = 1 ⇒ 2n 22 = 1 ⇒ n 2 = ±
2
Then:
 1 1 
λ3 = 4 ⇒ nˆ i(3) =  m ± 0
 2 2 
Afterwards, we summarize the eigenvalues and eigenvectors of T :

University of Castilla-La Mancha Draft By: Eduardo W. V. Chaves (2016)


Ciudad Real - Spain
56 SOLVING PROBLEMS BY MEANS OF CONTINUUM MECHANICS

λ1 = 1 ⇒ nˆ i(1) = [0 0 ± 1]

 1 1 
λ2 = 2 ⇒ nˆ i( 2) = ± ± 0
 2 2 

 1 1 
λ3 = 4 ⇒ nˆ i(3) = m ± 0
 2 2 
NOTE 1: The tensor components of this problem are the same as those used in Problem
1.64. Additionally, we can verify that the eigenvectors make up the transformation matrix, A ,
between the original system, ( x1 , x2 , x3 ) , and the principal space, ( x1′ , x′2 , x3′ ) , (see Problem
1.64).

Problem 1.72
Let Q be a proper orthogonal tensor a) show that Q has one real eigenvalue and equals to 1 .
b) Also show that Q can be represented by means of the angle θ as follows:
Q = pˆ ⊗ pˆ + cos θ(qˆ ⊗ qˆ + rˆ ⊗ rˆ ) − sin θ(qˆ ⊗ rˆ − rˆ ⊗ qˆ )

where p̂ , q̂ , r̂ , are unit vectors which form an orthonormal basis, where p̂ is the direction
associated with the eigenvalue λ = 1 , i.e. p̂ is an eigenvector of Q . c) Obtain the principal
r
invariants of Q in function of the angle θ . d) Given a vector position x , obtain the new
r
vector originated by the orthogonal transformation Q ⋅ x in the space formed by p̂ , q̂ .
Solution:
a) Taking into account the definition of the orthogonal tensor we can state that:
QT ⋅ Q = 1 ⇒ QT ⋅ Q − QT = 1 − QT ⇒ QT ⋅ (Q − 1) = −(QT − 1)
⇒ QT ⋅ (Q − 1) = −(Q − 1)T
Then we obtain the determinant of the two previous tensors:
det[QT ⋅ (Q − 1)] = det[−(Q − 1)T ] = (−1) 3 det[(Q − 1)T ]
T T
⇒ det
142 3]det[(Q − 1)] = −det[(Q − 1) ] = −det[Q − 1]
[Q4 ⇒ det[Q − 1] = −det[Q − 1]
= detQ =1

where we have used the following determinant properties: det[αA ] = α 3det[ A ] ,


det[ A T ] = det[ A ] , det[ A ⋅ B] = det[ A ]det[B] . The unique scalar which satisfies the expression
above is zero, then:
det[Q − 1] = 0
Taking into account the definition of eigenvalue, det[Q − λ1] = 0 , we can conclude that when
λ = 1 it fulfills det[Q − 1] = 0 , then λ = 1 is eigenvalue of Q . Hence, there is a direction
(eigenvector) satisfying that Q ⋅ eˆ 1* = λeˆ 1* = eˆ 1* .
b) We consider that the vectors pˆ ≡ eˆ 1* , qˆ ≡ eˆ *2 , rˆ ≡ eˆ *3 form an orthonormal basis.

University of Castilla-La Mancha Draft By: Eduardo W. V. Chaves (2016)


Ciudad Real - Spain
1 TENSORS 57

ê 3
eˆ 1* ≡ pˆ
qˆ ≡ eˆ *2
Q ⋅ eˆ 1* = eˆ 1*

ê 2

ê1 rˆ ≡ eˆ *3

Figure 1.17

The symbolic representation of the tensor Q in the basis ê1* , ê *2 , ê *3 is given by:
Q = Q*ij eˆ *i ⊗ eˆ *j
* ˆ*
= Q11e1 ⊗ eˆ 1* + Q12
* ˆ*
e1 ⊗ eˆ *2 + Q13
* ˆ*
e1 ⊗ eˆ *3 + Q*21eˆ *2 ⊗ eˆ 1* + Q*22 eˆ *2 ⊗ eˆ *2 + Q*23eˆ *2 ⊗ eˆ *3 + (1.55)
Q*31eˆ *3 ⊗ eˆ 1* + Q*32 eˆ *3 ⊗ eˆ *2 + Q*33eˆ *3 ⊗ eˆ *3

Taking into account that ê1* is eigenvector of Q and is associated with the eigenvalue λ = 1 , it
holds that Q ⋅ eˆ 1* = λeˆ 1* = eˆ 1* . In addition, making the projection of Q , given by (1.55),
according to direction ê1* , we obtain:
Q ⋅ eˆ 1* = eˆ 1*
Q ⋅ eˆ 1* = [ Q11
* ˆ*
e1 ⊗ eˆ 1* + Q12
* ˆ*
e1 ⊗ eˆ *2 + Q13
* ˆ*
e1 ⊗ eˆ *3 + Q*21eˆ *2 ⊗ eˆ 1* + Q*22 eˆ *2 ⊗ eˆ *2 + Q*23eˆ *2 ⊗ eˆ *3 +
Q*31eˆ *3 ⊗ eˆ 1* + Q*32 eˆ *3 ⊗ eˆ *2 + Q*33eˆ *3 ⊗ eˆ *3 ]⋅ eˆ 1*
* ˆ*
= Q11e1 + Q*21eˆ *2 + Q*31eˆ *3
*
with that we conclude that Q11 = 1 , Q *21 = 0 , Q *31 = 0 .
Remember that two coaxial tensors have the same principal directions (eigenvectors). A tensor
and its inverse are coaxial tensors, then if Q −1 = Q T , this implies that Q T and Q are coaxial
tensors, and ê1* is also principal direction of Q T , then it fulfills that:
QT ⋅eˆ 1* = eˆ 1*
QT ⋅eˆ 1* = [ Q11
* ˆ*
e1 ⊗ eˆ 1* + Q*21eˆ 1* ⊗ eˆ *2 + Q*31eˆ 1* ⊗ eˆ *3 + Q12
* ˆ*
e 2 ⊗ eˆ 1* + Q*22 eˆ *2 ⊗ eˆ *2 + Q*32 eˆ *2 ⊗ eˆ *3 +
* ˆ*
Q13 e 3 ⊗ eˆ 1* + Q*23eˆ *3 ⊗ eˆ *2 + Q*33eˆ *3 ⊗ eˆ *3 ]⋅ eˆ 1*
* ˆ* * ˆ* * ˆ*
= Q11e1 + Q12 e 2 + Q13 e3
* * *
with that we conclude that Q11 = 1 , Q12 = 0 , Q13 = 0 . Then, the equation (1.55) becomes:

Q = eˆ 1* ⊗ eˆ 1* + Q *22 eˆ *2 ⊗ eˆ *2 + Q *23 eˆ *2 ⊗ eˆ *3 + Q *32 eˆ *3 ⊗ eˆ *2 + Q *33 eˆ *3 ⊗ eˆ *3 (1.56)


In matrix form, the components of Q in the basis eˆ *i , (see Figure 1.18), are given by:
1 0 0 
Q *ij = 0 Q *22 Q *23 
0 Q *32 Q *33 

University of Castilla-La Mancha Draft By: Eduardo W. V. Chaves (2016)


Ciudad Real - Spain
58 SOLVING PROBLEMS BY MEANS OF CONTINUUM MECHANICS

x 2*
Q *22

*
Q11 =1
1 0 0 
Q *32
Q *ij = 0 Q *22 Q *23 
0 Q *32 Q *33  Q *23 x1*

Q *33

x3*

Figure 1.18

Once again we apply the orthogonality condition Q T ⋅ Q = Q ⋅ Q T = 1 that in the space eˆ *i can
be represented by means of components as follows:
1 0 0  1 0 0  1 0 0
= δ ij
Q*ki Q*kj ⇒ 0 Q * Q*32  0 Q*22
Q*23  = 0 1 0
 22
0 Q*23 Q*33  0 Q*32
Q*33  0 0 1
(1.57)
1 0 0  1 0 0

⇒ 0 [(Q 22 ) + (Q 32 ) ] [Q 22 Q 23 + Q 32 Q 33 ] = 0 1 0
* 2 * 2 * * * * 

0 [Q*22 Q*23 + Q*32 Q*33 ] [(Q*33 ) 2 + (Q*23 ) 2 ]  0 0 1

The determinant of a proper orthogonal tensor is det (Q) = +1 , thus


1 0 0
0 Q*22 Q*23 = 1 ⇒ Q*22 Q*33 − Q*23Q*32 = 1 (1.58)
0 Q*32 Q*33
Taking into account (1.57) and (1.58) we obtain the following set of equations:
(Q *22 ) 2 + (Q *32 ) 2 = 1 cos 2 θ + sin 2 θ = 1
 * * * * 
Q 22 Q 23 + Q 32 Q 33 = 0 cos θ(− sin θ) + sin θ cos θ = 0
 * 2 * 2  2 2
(Q 33 ) + (Q 23 ) = 1 cos θ + sin θ = 1
 * * * * cos θ cos θ − (− sin θ)(sin θ) = 1
Q 22 Q 33 − Q 23 Q 32 = 1 
whereupon we have demonstrated the existence of an angle that meets the above conditions:
1 0 0  1 0 0 
Q *ij = 0 Q *22 Q 32  = 0 cos θ − sin θ
*  
(1.59)
0 Q *23 Q *33  0 sin θ cos θ 
Returning to the equation (1.56), and taking into account (1.59), we conclude that:
Q = eˆ 1* ⊗ eˆ 1* + (cos θ) eˆ *2 ⊗ eˆ *2 + (− sin θ)eˆ *2 ⊗ eˆ *3 + (sin θ)eˆ *3 ⊗ eˆ *2 + (cos θ) eˆ *3 ⊗ eˆ *3
[ ] [
= eˆ 1* ⊗ eˆ 1* + cos θ eˆ *2 ⊗ eˆ *2 + eˆ *3 ⊗ eˆ *3 − sin θ eˆ *2 ⊗ eˆ *3 − eˆ *3 ⊗ eˆ *2 ]

University of Castilla-La Mancha Draft By: Eduardo W. V. Chaves (2016)


Ciudad Real - Spain
1 TENSORS 59

Considering that pˆ ≡ eˆ 1* , qˆ ≡ eˆ *2 , rˆ ≡ eˆ *3 , we show that:


Q = pˆ ⊗ pˆ + cos θ(qˆ ⊗ qˆ + rˆ ⊗ rˆ ) − sin θ(qˆ ⊗ rˆ − rˆ ⊗ qˆ )
It is interesting to note that the additive decomposition of Q in an antisymmetric and a
symmetric part, in the space eˆ *i , is:
1 0 0  0 0 0 
Q*ij 
sym
= 0 cos θ 0  ; Q*ij 
skew
= 0 0 − sin θ
0 0 cos θ 0 sin θ 0 
1444 4244443 14444 4244444 3
[pˆ ⊗pˆ + cos θ(qˆ ⊗qˆ +rˆ ⊗rˆ )]ij [ − sin θ(qˆ ⊗rˆ −rˆ ⊗qˆ )]ij

skew
Note that the format of Q *ij has the same format as the antisymmetric tensor ( W ) in the
space defined by the axial vector:
0 0 0 
Wij* = 0 0 − ω 
0 ω 0 
where ω is the magnitude of the axial vector.
c) By means of (1.59) it is easy to show that I Q = II Q = 1 + 2 cos θ , III Q = 1 .
r
d) We represent the vector x by means its components and the basis p̂ , q̂ , r̂ , as follows:
r
x = ppˆ + qqˆ + rrˆ .
r r r
Then, it fulfills that: x ⋅ pˆ = ( ppˆ + qqˆ + rrˆ ) ⋅ pˆ = p ; x ⋅ qˆ = q ; x ⋅ rˆ = r
Thus, (see Figure 1.19), it holds that:
r
[
~ = Q ⋅ xr = pˆ ⊗ pˆ + cos θ(qˆ ⊗ qˆ + rˆ ⊗ rˆ ) − sin θ(qˆ ⊗ rˆ − rˆ ⊗ qˆ )
x ]⋅ [ppˆ + qqˆ + rrˆ ]
= ppˆ + (q cos θ − r sin θ)qˆ + (r cos θ + q sin θ)rˆ

eˆ 1* ≡ pˆ r
x
qˆ ≡ eˆ *2
O r r
~
x =Q⋅ x

rˆ ≡ eˆ *3

Figure 1.19

University of Castilla-La Mancha Draft By: Eduardo W. V. Chaves (2016)


Ciudad Real - Spain
60 SOLVING PROBLEMS BY MEANS OF CONTINUUM MECHANICS

Problem 1.73
r r r r
Let us consider the tensorial transformations p ′ = U ⋅ p and p ′′ = R ⋅ p ′ , where R is an
orthogonal tensor and U is a second-order tensor with U ⋅ U −1 = 1 , i.e. ∃ U −1 . Obtain the
r r
transformation law between p and p ′′ , (see Figure 1.20).

r R
U p′

r
r p ′′
p
?
Figure 1.20
Solution:
Taking into account that R −1 = R T (orthogonal tensor), we can guarantee that the inverse of
r r
R exists, and considering that p ′′ = R ⋅ p ′ we can obtain:
r r r r r r r
p′′ = R ⋅ p′ ⇒ R −1 ⋅ p′′ = R −1 ⋅ R ⋅ p′ ⇒ R −1 ⋅ p′′ = 1 ⋅ p′ = p′
r r r r
Substituting p ′ = R −1 ⋅ p ′′ into p ′ = U ⋅ p we obtain:
r r r r
p′ = U ⋅ p p′ = U ⋅ p
r r r r
⇒ R −1 ⋅ p ′′ = U ⋅ p ⇒ R −1 ⋅ p ′′ = U ⋅ p
r r r r
⇒ R ⋅ R −1 ⋅ p ′′ = R ⋅ U ⋅ p ⇒ U −1 ⋅ R −1 ⋅ p ′′ = U −1 ⋅ U ⋅ p (1.60)
r r r r r
⇒ 1 ⋅ p ′′ = R ⋅ U ⋅ p ⇒ (R ⋅ U) −1 ⋅ p ′′ = 1 ⋅ p = p
r r r r
⇒ p ′′ = (R ⋅ U) ⋅ p ⇒ p = (R ⋅ U) −1 ⋅ p ′′
Or in indicial notation:
p ′i = U ij p j p ′i = U ij p j
⇒ R ij−1p ′′j = U ij p j ⇒ R ij−1p ′′j = U ij p j
⇒ R ki R ij−1p ′′j = R ki U ij p j ⇒ U ki−1R ij−1p ′′j = U ki−1U ij p j (1.61)
⇒ δ kj p ′′j = R ki U ij p j ⇒ (R ki U ij ) −1 p ′′j = δ kj p j = p k
⇒ p ′k′ = (R ki U ij )p j ⇒ p k = (R ki U ij ) −1 p ′′j

And the graphical representation is presented in Figure 1.21.

University of Castilla-La Mancha Draft By: Eduardo W. V. Chaves (2016)


Ciudad Real - Spain
1 TENSORS 61

r
U p′ R

U −1
R −1 = R T
r r
p (R ⋅ U) p ′′

(R ⋅ U) −1 = U −1 ⋅ R T

Figure 1.21

1.9 Spectral Representation of Tensors

Problem 1.74
Let w be an antisymmetric second-order tensor and V be a positive definite symmetric
tensor whose spectral representation is given by:
3
V= ∑λ
a =1
a nˆ ( a ) ⊗ nˆ ( a )

Show that the antisymmetric tensor w can be represented by:


3
w = ∑ w ab nˆ (a ) ⊗ nˆ (b)
a ,b =1
a ≠b

Also show the following is true:


3
w ⋅ V − V ⋅ w = ∑ w ab (λ b − λ a ) nˆ ( a ) ⊗ nˆ (b)
a ,b =1
a ≠b

Solution:
It is true that
 
w ⋅1 = w ⋅  ∑ nˆ ( a) ⊗ nˆ ( a)  = ∑ w ⋅ nˆ ( a) ⊗ nˆ ( a ) = ∑ (wr ∧ nˆ (a ) )⊗ nˆ ( a) = ∑ wb (nˆ (b) ∧ nˆ ( a ) )⊗ nˆ (a )
3 3 3 3

 a =1  a =1 a =1 a ,b =1
r r
where we have applied the antisymmetric tensor property w ⋅ nˆ = w ∧ nˆ , where w is the axial
vector associated with w . Expanding the above equation, we obtain:
w = wb (nˆ (b) ∧ nˆ (1) ) ⊗ nˆ (1) + wb (nˆ (b) ∧ nˆ ( 2) ) ⊗ nˆ ( 2) + wb (nˆ (b) ∧ nˆ (3) ) ⊗ nˆ (3) =
( ) ( )
= w1 nˆ (1) ∧ nˆ (1) ⊗ nˆ (1) + w2 nˆ ( 2) ∧ nˆ (1) ⊗ nˆ (1) + w3 nˆ (3) ∧ nˆ (1) ⊗ nˆ (1) + ( )
+ w (nˆ1
(1)
∧ nˆ (2)
) ⊗ nˆ (2)
+ w2 nˆ ( ( 2)
∧ nˆ ( 2)
) ⊗ nˆ ( 2)
+ w3 nˆ ( ( 3)
∧ nˆ ( 2)
) ⊗ nˆ ( 2)
+
+ w (nˆ1
(1)
∧ nˆ ( 3)
) ⊗ nˆ ( 3)
+ w2 (nˆ ( 2)
∧ nˆ ( 3)
) ⊗ nˆ
( 3)
+ w3 (nˆ ( 3)
∧ nˆ ( 3)
) ⊗ nˆ( 3)

By simplifying the above equation we can obtain:

University of Castilla-La Mancha Draft By: Eduardo W. V. Chaves (2016)


Ciudad Real - Spain
62 SOLVING PROBLEMS BY MEANS OF CONTINUUM MECHANICS

w = −w2 nˆ (3) ⊗ nˆ (1) + w3 nˆ ( 2) ⊗ nˆ (1) + w1 nˆ (3) ⊗ nˆ ( 2) − w3 nˆ (1) ⊗ nˆ ( 2) + −w1 nˆ ( 2) ⊗ nˆ (3) + w2 nˆ (1) ⊗ nˆ (3)
Taking into account that w1 = −w 23 = w 32 , w2 = w13 = −w 31 , w3 = −w12 = w 21 , the above
equation becomes:
w = w31 nˆ (3) ⊗ nˆ (1) + w21 nˆ ( 2) ⊗ nˆ (1) + w32 nˆ (3) ⊗ nˆ ( 2) + w12 nˆ (1) ⊗ nˆ ( 2) +
+ w 23 nˆ ( 2 ) ⊗ nˆ (3) + w13 nˆ (1) ⊗ nˆ (3)
which is the same as:
3
w = ∑ w ab nˆ (a ) ⊗ nˆ (b)
a ,b =1
a ≠b

The terms w⋅V and V ⋅ w can be expressed as follows:


 3 
   3 

w ⋅ V =  w ab n ⊗ n  ⋅  λ b nˆ (b) ⊗ nˆ (b) 
 a ,b =1
ˆ (a) ˆ (b )

  b =1 

 a≠b 
3 3
= ∑ λ b w ab nˆ ( a ) ⊗ nˆ (b ) ⋅ nˆ (b ) ⊗ nˆ (b ) =
a ,b =1
∑λ w
a ,b =1
b ab nˆ ( a ) ⊗ nˆ (b )
a ≠b a≠b

and
 
 3   3  3


V ⋅ w =  λ a nˆ ⊗ nˆ  ⋅ 
 a =1
( a ) ( a )

  a ,b =1

w ab nˆ ⊗ nˆ  = λ a w ab nˆ ( a) ⊗ nˆ (b)
( a ) ( b )

 a ,b =1

 a≠b  a≠b
Then,
 3   3 
    3


 a ,b =1
(a) (b )

  a ,b =1
(a) ˆ (b )

w ⋅ V − V ⋅ w =  λbwab n ⊗ n  −  λ awab n ⊗ n  = wab (λb − λ a ) nˆ (a ) ⊗ nˆ (b)
ˆ ˆ ˆ
 a ,b =1

 a ≠b   a ≠b  a≠b
Similarly, it is possible to show that:
3
w ⋅ V 2 − V 2 ⋅ w = ∑ w ab (λ2b − λ2a ) nˆ ( a ) ⊗ nˆ (b)
a ,b =1
a ≠b

Problem 1.75
Let C be a positive definite tensor, whose Cartesian components are given by:
2 0 1 
C ij = 0 4 0
1 0 2

Obtain the following tensors: a) C 2 ; b) U = C . c) Check if the tensors C and U are coaxial.
Solution:
Note that the tensors C 2 and U = C are coaxial with the tensor C . By means of the
spectral representation of C :

University of Castilla-La Mancha Draft By: Eduardo W. V. Chaves (2016)


Ciudad Real - Spain
1 TENSORS 63

3
C= ∑ γ Nˆ
a =1
a
(a) ˆ (a )
⊗N

ˆ ( a ) are the eigenvectors of C , we can obtain:


where γ a are the eigenvalues of C , and N
3 3
C2 = ∑
a =1
ˆ (a) ⊗ N
γ 2a N ˆ (a) ; U= C = ∑
a =1
ˆ (a) ⊗ N
γaN ˆ (a ) (1.62)

Calculation of the eigenvalues and eigenvectors of the tensor C .


Due to the structure of the C tensor components we already know one eigenvalue γ 2 = 4
which is associated with the principal direction Nˆ i( 2) = [0 ± 1 0] . To calculate the remaining
eigenvalues is sufficient to solve the following characteristic determinant:
2−γ 1 γ 1 = 2 − 1 = 1
=0 ⇒ (2 − γ ) 2 = 1 ⇒ (2 − γ ) = ±1 ⇒ 
1 2−γ γ 3 = 2 + 1 = 3
Associated with the eigenvalue γ 1 = 1 we have the following eigenvector:

2 − γ 1 1  Nˆ 1(1)  0 1 1 Nˆ 1(1)  0


 1  = ⇒ 1 1  ˆ (1)  = 0 ⇒ Nˆ 1(1) = −Nˆ 3(1)
 2 − γ 1  Nˆ 3(1)  0   N 3   

with the restriction Nˆ i(1) Nˆ i(1) = 1 , thus


ˆ (1)Nˆ (1) + N
N ˆ (1)Nˆ (1) + N
ˆ (1)Nˆ (1) = 1
1 1 2 2 3 3

⇒ Nˆ 1(1)N
ˆ (1) + Nˆ (1)Nˆ (1) = 1 ⇒ ˆ (1) = ± 1
N ⇒ Nˆ 3(1) = −Nˆ 1(1) = m
1
1 1 1 1
2 2
Associated with the eigenvalue γ 3 = 3 we have the following eigenvector:

2 − γ 3 1  Nˆ 1(3)  0 − 1 1  Nˆ (3)  0


 1  = ⇒  1 − 1  ˆ (3)  = 0
1
⇒ Nˆ 1(3) = Nˆ 3(3)
 2 − γ 3  Nˆ (33)  0   N 3   

with the restriction Nˆ i(3) Nˆ i(3) = 1 , thus


ˆ (3)N
N ˆ (3) + Nˆ (3)Nˆ (3) + Nˆ (3)N
ˆ ( 3) = 1
1 1 2 2 3 3

⇒ Nˆ 1(3)Nˆ 1(3) + Nˆ 1(3)N


ˆ ( 3) = 1 ⇒ ˆ ( 3) = ± 1
N ⇒ ˆ ( 3) = ± 1
Nˆ 3(3) = N
1 1 1
2 2
Summarizing we have:

ˆ (1) = ± 1 1   1 1 
γ1 = 1 ⇒ N i  0 m  0 −
 2 2   
  2 2
γ2 = 4 ⇒ ˆN ( 2 ) = [0 ± 1 0] Transforma
  tion Matrix
 → A= 0 1 0 
i 
1   1 1 
γ3 = 3 ⇒ Nˆ ( 3) =  ± 1
0 ±
 0
i    2 2 
 2 2   

Then it holds that:


C′ = A C AT ⇒ C = AT C′ A
In the principal space we have:

University of Castilla-La Mancha Draft By: Eduardo W. V. Chaves (2016)


Ciudad Real - Spain
64 SOLVING PROBLEMS BY MEANS OF CONTINUUM MECHANICS

 12 0 0  1 0 0
 2  
Cij′ =  0 4
2
0  = 0 16 0
1 0 0   0 0 3 2  0 0 9 
    
Cij′ = 0 4 0 ⇒ 
  1 0 0  1 0 0
0 0 3
U′ = C ′ =  0  
0 
 ij ij  4 0  = 0 2
  0 0 3  0 0 3 
  
Be aware that the above operation can only be done in the principal space, (see Figure 1.22).
Note also that the tensor C is a positive definite tensor, so, its eigenvalues are positive. In the
original space we have the following components:
T
 1 1   1 1 
 0 − 0 −
 2 2  1 0 0  2 2  5 0 4
Cij = A C ′ A = 
2 T 2
0 1 0  0 16 0  0 1 0  = 0 16 0 (1.63)
1 1    1 1  
 0 0 0 9  0  4 0 5
 2 2   2 2 
 

Note that this result could have been obtained easily by means of the operation C 2 = C ⋅ C ,
which in components becomes:
 2 0 1   2 0 1   5 0 4
C ij2 = C ik C kj = 0 4 0 0 4 0 = 0 16 0
1 0 2 1 0 2  4 0 5

Similarly, we can obtain the components of U in the original Cartesian system:


T
 1 1   1 1   3 +1 3 − 1
 0 −  1 0 0  0 −   0 
 2 2 2 2  2 2 
U ij =  0 1 0  0 2 0  0 1 0 = 0 2 0 
1 1    1 1   3 −1
 0 0 0 3   0 3 + 1
0

 2 2  
 2 2   2 2 

c) The tensors C and U are coaxial, since they have the same eigenvectors, (see equation
(1.62)). Note also that the eigenvalues of U were obtained in the principal space of C . We
can also verify that C and U are coaxial by means of C ⋅ U = U ⋅ C , i.e.:
 3 +1 3 − 1
2 0 1  0  3.098 0 2.098
 2 2  
C ik U kj = 0 4 0   0 2 0 = 0 8 0 
1 0 2  3 − 1 3 + 1 2.098 0 3.098
 2 0  
 2 

 3 +1 3 − 1
 0  2 0 1  3.098 0 2.098
 2 2 
U ik C kj =  0 2 0  0 4 0 =  0 8 0 
 3 − 1 3 + 1  1 0 2 2.098 0 3.098
 2 0    
 2 

University of Castilla-La Mancha Draft By: Eduardo W. V. Chaves (2016)


Ciudad Real - Spain
1 TENSORS 65

x3′
C ′ = A C AT C3

C2 x′2
x3
A

C 33

C 23
C13 C1
C 23
Principal space
C13
C12 C 22
x2 x1′
C12
C11 C1 0 0
AT
Cij′ =  0 C2 0 
x1  0 0 C3 
C = AT C′ A
 C1 0 0 
C = A T C′ A  
Cij′ =  0 C2 0 
C 2 = A T C ′2 A  0 0 C3 
 

Figure 1.22

Problem 1.76
Let C be a symmetric second-order tensor and R a proper orthogonal tensor. The
components of these tensors, in the Cartesian system, are given by:
 0 0 1

2 0 1  
2 2
Cij = 0 4 0  ; R ij =  0
 2 2 
1 0 2  
2 2
− 0
 2 2 
a) Obtain the following tensors: a.1) C 8 ; a2) U = C .
b) Obtain the principal invariants of C .
c) Taking into account that the tensors b and C are related to each other by the following
proper orthogonal transformation C = R T ⋅ b ⋅ R , obtain the third principal invariant of b .
Solution:
 3281 0 3280

a) Answer: C =  0 8
65536 0  , (see Problem 1.75).
3280 0 3281
b) The principal invariants of C :
I C = Tr (C ij ) = C ii = C11 + C 22 + C 33 = 8

II C =
1
2
(C ii C jj − C ij C ij = )
4 0 2 1 2 0
+ +
0 2 1 2 0 4
= 19 ;

University of Castilla-La Mancha Draft By: Eduardo W. V. Chaves (2016)


Ciudad Real - Spain
66 SOLVING PROBLEMS BY MEANS OF CONTINUUM MECHANICS

III C = C =  ijk C i1C j 2 C k 3 = 12 .

c) Taking into account the determinant property, the third principal invariant of b can be
expressed as follows:
C ≡ det (C ) = det (R T ⋅ b ⋅ R ) = det (R T )det (b)det (R ) = det (b) = III b = 12
1424 3 123
= +1 = +1

NOTE: If R is an orthogonal tensor ( R −1 = R T ) and the equation C = R T ⋅ b ⋅ R holds, this


implies that C and b have the same eigenvalues γ (aC ) = γ (ab) = γ a , (see Problem 1.68), so, they
have also the same characteristic equation:
IC = I b
3 2 3 2 
λ − λ I C + λ II C − III C = λ − λ I b + λ II b − III b = 0 ⇒  II C = II b
 III = III
 C b

Note also that


C = RT ⋅ b ⋅ R ⇒ R ⋅ C ⋅ RT = R
12⋅ R3T ⋅ b ⋅ R
12⋅ R3T = b
=1 =1

3
And if we start from the spectral representation C = ∑ γ aN
ˆ (a) ⊗ N
ˆ ( a ) and by considering
a =1

b = R ⋅ C ⋅ R we can obtain:
T

b = R ⋅ C ⋅ RT
 3 ˆ ( a )  ⋅ R T =
3 3
b = R ⋅  γ aN
 a =1

ˆ (a) ⊗ N


∑ γ R ⋅ Nˆ
a =1
a
(a) ˆ (a) ⋅ RT =
⊗N ∑ γ R ⋅ Nˆ
a =1
a
(a)
⊗ R ⋅N
ˆ (a )

3
= ∑ γ nˆ
a =1
a
(a )
⊗ nˆ ( a )

where nˆ ( a ) = R ⋅ N
ˆ ( a ) are the eigenvectors of b .

Problem 1.77
Let S be a symmetric second-order tensor with det (S ) ≠ 0 . Considering that S has two
equal eigenvalues, i.e. S 2 = S 3 and S1 ≠ S 2 , show that S can be represented by:
S = S 1nˆ (1) ⊗ nˆ (1) + S 2 (1 − nˆ (1) ⊗ nˆ (1) )

where n̂ (1) is the eigenvector of S associated with the eigenvalue S1 , 1 is the second-order
unit tensor.
Solution: We start from the spectral representation of S :
3
S= ∑ S nˆ
a =1
a
(a)
⊗ nˆ ( a ) = S 1nˆ (1) ⊗ nˆ (1) + S 2 nˆ ( 2) ⊗ nˆ ( 2) + S 3nˆ (3) ⊗ nˆ (3)
(1.64)
= S 1nˆ (1)
⊗ nˆ (1)
+ S 2 (nˆ ( 2)
⊗ nˆ ( 2)
+ nˆ ( 3)
⊗ nˆ ) ( 3)

Remember that 1 is a spherical tensor, whereby any direction is a principal direction. Based
on this principle, we adopt the principal space of S to make the spectral representation of 1 ,
i.e.:

University of Castilla-La Mancha Draft By: Eduardo W. V. Chaves (2016)


Ciudad Real - Spain
1 TENSORS 67

3
1= ∑ nˆ
a =1
(a)
⊗ nˆ ( a ) = nˆ (1) ⊗ nˆ (1) + nˆ ( 2 ) ⊗ nˆ ( 2 ) + nˆ (3) ⊗ nˆ (3)
(1.65)
⇒ nˆ ( 2)
⊗ nˆ ( 2)
+ nˆ ( 3)
⊗ nˆ ( 3)
= 1 − nˆ (1)
⊗ nˆ (1)

By substituting the above equation into (1.64) we obtain:


S = S1nˆ (1) ⊗ nˆ (1) + S 2 (nˆ ( 2 ) ⊗ nˆ ( 2) + nˆ (3) ⊗ nˆ (3) ) = S1nˆ (1) ⊗ nˆ (1) + S 2 (1 − nˆ (1) ⊗ nˆ (1) )
Q.E.D.
1.10 Cayley-Hamilton Theorem

Problem 1.78
Let T be an arbitrary second-order tensor, show the Cayley-Hamilton theorem, which states
that any tensor satisfies its own characteristic equation.
Solution:
We start from the characteristic equation of the tensor: λ3 − λ2 I T + λ II T − III T = 0 , which
fulfils for each eigenvalue λ 1 , λ 2 , λ 3 , then:
λ31 − λ21 I T + λ 1 II T − III T = 0
λ32 − λ22 I T + λ 2 II T − III T = 0
λ33 − λ23 I T + λ 3 II T − III T = 0
Restructuring the above equations in matrix form we obtain:
λ31 0 0  λ21 0 0 λ 1 0 0 1 0 0  0 0 0 
     
0 λ32 0−0 λ22 0 I T +  0 λ2 0  II T − 0 1 0 III T = 0 0 0
 
0 0 λ33   0 0 λ23  0 0 λ 3  0 0 1  0 0 0 (1.66)
 
Tij′ 3 − Tij′ 2 I T + Tij′ II T − III T δ ij = 0 ij

Note that in the principal space of T the following relationships are true:
λ 1 0 0
 
Tij′ =  0 λ2 0
0 0 λ 3 

λ 1 0 0  λ 1 0 0  λ21 0 0
    
Tij′ = Tik′ Tkj′ =  0
2
λ2 0  0 λ2 0 =0 λ22 0
0 0 λ 3   0 0 λ 3   0 0 λ23 

λ 1 0 0  λ 1 0 0  λ 1 0 0  λ31 0 0
     
Tij′ = Tik′ Tkp
3
′ T ′pj =  0 λ2 0  0 λ2 0  0 λ2 0 =0 λ32 0
0 0 λ 3   0 0 λ 3   0 0 λ 3   0 0 λ33 

The component transformation law between spaces for a second-order tensor is
Tij′ = Fik Tkp F pj−1 , where Fij is the transformation matrix from the original space ( Tij ) to the
principal space ( Tij′ ). Note also that the relationships Tij′ 2 = Fik Tkp2 F pj−1 and Tij′ 3 = Fik Tkp3 F pj−1
hold, (see Problem 1.69). With that the equation in (1.66) can be rewritten as follows:

University of Castilla-La Mancha Draft By: Eduardo W. V. Chaves (2016)


Ciudad Real - Spain
68 SOLVING PROBLEMS BY MEANS OF CONTINUUM MECHANICS

Tij′ 3 − Tij′ 2 I T + Tij′ II T − III T δ ij = 0 ij


3
⇒ Fik Tkp F pj−1 − Fik Tkp2 F pj−1 I T + Fik Tkp F pj−1 II T − III T Fik δ kp F pj−1 = 0 ij
(
⇒ Fik Tkp3 − Tkp2 I T + Tkp II T − III T δ kp F pj−1 = 0 ij )
⇒ Fsi−1 Fik ( 3
Tkp − Tkp2 I T )
+ Tkp II T − III T δ kp F pj−1 F jt = Fsi−1 0 ij F jt = 0 st
⇒δ 3
(
sk Tkp − Tkp2 I T )
+ Tkp II T − III T δ kp δ pt = Fsi−1 0 ij F jt = 0 st
⇒ Tst3 − Tst2 I T + Tst II T − III T δ st = 0 st
3 2
⇒ T − T I T + T II T − III T 1 = 0
Q.E.D.

Alternative solution:
In Problem 1.56 (NOTE 1) we have summarized that:
r r r r r r r r r r r r
[( A ⋅ a), b, c] + [a, ( A ⋅ b), c ] + [a, b, ( A ⋅ c )] = I A [a, b, c]
r r r r r r r r r r r r
[a, ( A ⋅ b), ( A ⋅ c )] + [( A ⋅ a), b, ( A ⋅ c )] + [( A ⋅ a), ( A ⋅ b), c ] = II A [a, b, c]
r r r r r r
[( A ⋅ a), ( A ⋅ b), ( A ⋅ c )] = III A [a, b, c ]
r r r r r r r r r r r r
where [a, b, c ] ≡ a ⋅ (b ∧ c ) ≠ 0 holds with a ≠ 0 , b ≠ 0 , c ≠ 0 . Now if we consider that the
r r r
vector a is given by a = A ⋅ f we can obtain:
r r r r r r r r r r r r
[( A ⋅ a), b, c] + [a, ( A ⋅ b), c ] + [a, b, ( A ⋅ c)] = I A [a, b, c ]
r r r r r r r r r r r r
⇒ [( A ⋅ A ⋅ f ), b, c ] + [( A ⋅ f ), ( A ⋅ b), c ] + [( A ⋅ f ), b, ( A ⋅ c )] = I A [( A ⋅ f ), b, c ]
r r r r r r r r r r r r (1.67)
⇒ [( A 2 ⋅ f ), b, c ] + [( A ⋅ f ), ( A ⋅ b), c] + [( A ⋅ f ), b, ( A ⋅ c)] = I A [( A ⋅ f ), b, c ]
r r r r r r r r r r r r
⇒ [( A 2 ⋅ f ), b, c ] − I A [( A ⋅ f ), b, c ] = −[( A ⋅ f ), ( A ⋅ b), c ] − [( A ⋅ f ), b, ( A ⋅ c)]
According to the definition of II A it is also true that:
r r r r r r r r r r r r
[f , ( A ⋅ b), ( A ⋅ c )] + [( A ⋅ f ), b, ( A ⋅ c )] + [( A ⋅ f ), ( A ⋅ b), c ] = II A [f , b, c ]
r r r r r r r r r r r r
⇒ [f , ( A ⋅ b), ( A ⋅ c )] − II A [ f , b, c ] = −[( A ⋅ f ), b, ( A ⋅ c )] − [( A ⋅ f ), ( A ⋅ b), c]
Taking into account the above equation into the equation (1.67) we can obtain:
r r r r r r r r r r r r
[( A 2 ⋅ f ), b, c ] − I A [( A ⋅ f ), b, c ] = −[( A ⋅ f ), ( A ⋅ b), c] − [( A ⋅ f ), b, ( A ⋅ c )]
r r r r r r r r r r r r
⇒ [( A 2 ⋅ f ), b, c ] − I A [( A ⋅ f ), b, c] = [f , ( A ⋅ b), ( A ⋅ c )] − II A [ f , b, c ]
r r r r r r r r r r r r
⇒ [( A 2 ⋅ f ), b, c ] − I A [( A ⋅ f ), b, c ] + II A [f , b, c ] − [f , ( A ⋅ b), ( A ⋅ c )] = 0
r r r r r r r r r r r
[
⇒ ( A 2 ⋅ f ) ⋅ (b ∧ c ) − I A ( A ⋅ f ) ⋅ (b ∧ c) + II A f ⋅ (b ∧ c ) − f ⋅ ( A ⋅ b) ∧ ( A ⋅ c ) = 0
r
]
r r r r
In Problem 1.55 we have shown that ( A ⋅ b) ∧ ( A ⋅ c) = [cof( A )]⋅ (b ∧ c ) holds, then the
above equation becomes
r r r r r r
r r r
r r r r
r r r r
r
[ r
( A 2 ⋅ f ) ⋅ (b ∧ c ) − I A ( A ⋅ f ) ⋅ (b ∧ c) + II A f ⋅ (b ∧ c ) − f ⋅ ( A ⋅ b) ∧ ( A ⋅ c ) = 0
r r r r r
]
⇒ ( A 2 ⋅ f ) ⋅ (b ∧ c ) − I A ( A ⋅ f ) ⋅ (b ∧ c) + II A f ⋅ (b ∧ c) − f ⋅ [cof( A )]⋅ (b ∧ c) = 0
{ r r r r r r
⇒ ( A 2 ⋅ f ) − I A ( A ⋅ f ) + II A f − f ⋅ [cof( A )] ⋅ (b ∧ c ) = 0 }

University of Castilla-La Mancha Draft By: Eduardo W. V. Chaves (2016)


Ciudad Real - Spain
1 TENSORS 69

r r r r r r r r
Note that the vectors ( A 2 ⋅ f ) , ( A ⋅ f ) , f ≠ 0 , (f ⋅ [cof(A )]) are not orthogonal to (b ∧ c ) ≠ 0 ,
so, we can conclude that
r r r r r
⇒ ( A 2 ⋅ f ) − I A ( A ⋅ f ) + II A f − f ⋅ [cof( A )] = 0
r r r r r
⇒ A 2 ⋅ f − I A A ⋅ f + II A 1 ⋅ f − [cof( A )] ⋅ f = 0
T

{ }
r r
⇒ A 2 − I A A + II A 1 − [cof( A )] ⋅ f = 0
T

r
⇒ A 2 − I A A + II A 1 − [cof( A )] = 0
T

Using the definition A A −1 = [cof(A )]T , the above equation becomes


r
A 2 − I A A + II A 1 − [cof( A )] = 0
T

r
⇒ A 2 − I A A + II A 1 − A A −1 = 0
r
⇒ A 2 ⋅ A − I A A ⋅ A + II A 1 ⋅ A − A A −1 ⋅ A = 0 ⋅ A
r
⇒ A 3 − I A A 2 + II A A − A 1 = 0
Q.E.D.

Problem 1.79
Based on the Cayley-Hamilton theorem, find the inverse of a tensor T in terms of tensor
power.
Solution: The Cayley-Hamilton theorem states that:
T 3 − T 2 I T + T II T − III T 1 = 0

Carrying out the dot product between the previous equation and the tensor T −1 , we obtain:
T 3 ⋅ T −1 − T 2 ⋅ T −1I T + T ⋅ T −1 II T − III T 1 ⋅ T −1 = 0 ⋅ T −1

T 2 − TI T + 1 II T − III T T −1 = 0 ⇒ T −1 =
1
III T
(T 2 − I T T + II T 1 )

Problem 1.80
Check the Cayley-Hamilton theorem by using a second-order tensor whose Cartesian
components are given by:
5 0 0 
T = 0 2 0
0 0 1 

Solution:
The Cayley-Hamilton theorem states that:
T 3 − T 2 I T + T II T − III T 1 = 0
where I T = 5 + 2 + 1 = 8 , II T = 10 + 2 + 5 = 17 , III T = 10 , and
5 3 0 0  125 0 0  5 2 0 0  25 0 0 
   
T 3
=0 23 0  =  0 8 0  ; T 2
=0 22 0  =  0 4 0 
0 0 1   0 0 1  0 0 1   0 0 1 
 
By applying the Cayley-Hamilton theorem, we can verify that the following is true:

University of Castilla-La Mancha Draft By: Eduardo W. V. Chaves (2016)


Ciudad Real - Spain
70 SOLVING PROBLEMS BY MEANS OF CONTINUUM MECHANICS

125 0 0  25 0 0 5 0 0  1 0 0   0 0 0 
 0 8 0 − 8  0 4 0 + 17 0 2 0  − 10 0 1 0 = 0 0 0 
         
 0 0 1   0 0 1  0 0 1  0 0 1  0 0 0 

Problem 1.81
Given the matrix P which is represented by its components Pij (i, j = 1,2,3,4) . a) Obtain the
inverse of P , b) the invariants, y c) the characteristic equation. Consider that:
1 2 3 1 1 0 0 0
2 2 1 2 0 1 0 0 
P= and 1=
4 1 5 3 0 0 1 0
   
3 1 2 4 0 0 0 1
Solution:
By applying the Cayley-Hamilton theorem we can obtain:
P 4 + P 3 I1 + P 2 I 2 + P I 3 + I 41 = 0
(
⇒ P P 3 + P 2 I1 + P I 2 + 1 I 3 + I 41 = 0 )
⇒ P (P (P 2
) )
+ P I1 + 1 I 2 + 1 I 3 + I 41 = 0
   
  
4 4 2 443
( 
)
⇒ P  P  P P + 1 I 1 + 1 I 2  + 1 I 3  + I 41 = 0
1
   
  C1  
 

4 4 2
(44 3

)
⇒ P  P C1 + 1 I 2 + 1 I 3  + I 4 1 = 0
1
 
 C2 
(
⇒ P C2 + 1 I 3 + I 4 1 = 0 )
⇒ C3 + I 4 1 = 0
where we have denoted by:
C0 = P
(
C1 = P C0 + 1 I 1 )
C2 = P (C 1 +1 I ) 2

C3 = P (C 2 +1 I ) 3

We can obtain the trace of C3 + I 41 = 0 as follows:


Tr (C3 + I 41) = Tr (0 )
− Tr (C3 )
⇒ Tr (C3 ) + Tr ( I 41) = Tr (C3 ) + I 4 Tr (1) = Tr (C3 ) + 4 I 4 = 0 ⇒ I4 =
4
Similarly, we can define that:
− Tr (C2 ) − Tr (C1 ) − Tr (C0 )
I3 = ; I2 = ; I1 =
3 2 1
With that we can obtain:

University of Castilla-La Mancha Draft By: Eduardo W. V. Chaves (2016)


Ciudad Real - Spain
1 TENSORS 71

− Tr (C0 )
I1 = = −(1 + 2 + 5 + 4) = −12
1
thus we evaluate the matrix C1 = P C0 + 1 I 1 : ( )
1 2 3 1   1 2 3 1 1 0 0 0   8 − 14 − 14 6 
2   
2 1 2   2 
2 1 2 0 
1 0 0    − 8 − 13 5 − 7 
C1 =  − 12  =
4 1 5 3   4 1 5 3 0 0 1 0   − 13 6 − 16 − 3 
       
3 1 2 4   3 1 2 4 0 0 0 1    − 11 2 4 − 21

− Tr (C1 ) − (8 − 13 − 16 − 21) − (−42)


I2 = = = = 21
2 2 2
In turn we can obtain C2 = P C1 + 1 I 2 ( )
1 2 3 1   8 − 14 − 14 6  1 0 0 0   − 37 22 15 − 17 
2    0   
2 1 2   − 8 − 13 5 −7 1 0 0   7 − 2 − 5 − 5 
C2 =  + 21 =
4 1 5 3   − 13 6 − 16 − 3  0 0 1 0    10 − 12 − 14 2 
       
3 1 2 4   − 11 2 4 − 21 0 0 0 1   9 − 14 − 11 5 

− Tr (C2 ) − ( −37 − 2 − 14 + 5) − ( −48)


I3 = = = = 16
3 3 3
In turn we can obtain C3 = P C2 + 1 I 3 ( )
1 2 3 1    − 37 22 15 − 17  1 0 0 0   32 0 0 0 
2    0 
2 1 2   7 −2 −5 −5 1 0 0    0 32 0 0 
C3 =  + 16  =
4 1 5 3   10 − 12 − 14 2  0 0 1 0    0 0 32 0 
       
3 1 2 4   9 − 14 − 11 5  0 0 0 1    0 0 0 32 

− Tr (C3 ) − 4(32)
I4 = = = −32 = det (P )
4 4
Then, the characteristic equation becomes:
P 4 + P 3 I1 + P 2 I 2 + P I 3 + I 41 = 0 ⇒ P 4 − 12P 3 + 21P 2 + 16P − 321 = 0
The characteristic equation coefficients could have been obtained by evaluates the
determinant:
1− λ 2 3 1
2 2−λ 1 2
det (P − λ1) ≡ P − λ1 = =0
4 1 5−λ 3
3 1 2 4−λ
c) The inverse can be obtained by starting from:
P (C2 + 1 I 3 ) + I 4 1 = 0
( )
⇒ P −1 P C2 + 1 I 3 + I 4 P −11 = 0 ⇒ ( )
⇒ C2 + 1 I 3 + I 4 P −1 = 0

⇒ P −1 = −
1
I4
(
C2 + 1 I 3 ) ⇔ P −1 =
1
det (P )
adj[P ] ∴ (
adj[P ] = − C2 + 1 I 3 )
thus:

University of Castilla-La Mancha Draft By: Eduardo W. V. Chaves (2016)


Ciudad Real - Spain
72 SOLVING PROBLEMS BY MEANS OF CONTINUUM MECHANICS

  − 37 22 15 − 17  1 0 0 0  − 21 22 15 − 17 
  0   
1  7 −2 −5 −5  1 0 0  1  7 14 − 5 − 5 
P −1 =−  + 16 =
(−32)   10 − 12 − 14 2  0 0 1 0  32  10 − 12 2 2 

 9     
 − 14 − 11 5  0 0 0 1   9 − 14 − 11 21 

NOTE 1: The procedure performed previously, in the literature, is called Faddeev-Leverrier


method.
Note that the inverse can also be obtained by using the same procedure as the one used in the
equation (1.42), i.e.:

 1 0 0 0 1 2 3 1 1 2 3 1 1 2 3 1 
     
 2 2 1 2 1 0 0 0 2 2 1 2 2 2 1 2

 4 1 5 3 4 1 5 3 1 0 0 0 4 1 5 3 
 3 3 3 1 
 1 2 4  1 2 4  1 2 4  0 0 0
 
  0 1 0 0  1 2 3 1  1 2 3 1  1 2 3 1

 2 2 1 2 0 1 0 0 2 2 1 2 2 2 1 2 
 4 1 5 3 4 1 5 3 0 1 0 0 4 1 5 3   21 −22 −15 17 
  
−1 3
 2 4
3
 2 4
3
 2 4
0
 1 0 0

 32  1 1 1  =  −1   −7 −14 5 5 
     32  −10 
 0 0 1 0  1 2 3 1  1 2 3 1  1 2 3 1   12 −2 −2
   −9
 2 2 1 2 0 0 1 0 2 2 1 2 2 2 1 2
  14 11 −21 
 4 1 5 3 4 1 5 3 0 0 1 0 4 1 5 3 
 3 3 3 0 
 1 2 4  1 2 4  1 2 4  0 1 0
 
  0 0 0 1  1 2 3 1  1 2 3 1  1 2 3 1

 2 2 1 2 0 0 0 1 2 2 1 2 2 2 1 2 
 4 1 5 3 4 1 5 3 0 0 0 1 4 1 5 3 
 3 3 3 0 
  1 2 4  1 2 4  1 2 4  0 0 1 

University of Castilla-La Mancha Draft By: Eduardo W. V. Chaves (2016)


Ciudad Real - Spain
1 TENSORS 73

NOTE 2: We can also obtain the characteristic coefficients by means of the following
procedure. Considering P 4 − P 3 I 1 + P 2 I 2 − P I 3 + I 4 1 = 0
The last coefficient is I 4 = det (P ) = −32 .
The coefficient I 3 is obtained by the sum of the determinants of the resulting matrices by
eliminating one row and one column associated with the main diagonal, i.e.

1 2 3 1  1 2 3 1  1 2 3 1  1 2 3 1
2 2 1 2 2 2 1 2 2 2 1 2 2 2 1 2
I3 =  + + +
4 1 5 3  4 1 5 3  4 1 5 3  4 1 5 3
       
3 1 2 4  3 1 2 4  3 1 2 4  3 1 2 4
2 1 2 1 3 1  1 2 1  1 2 3
= 1 5 3 +  4 5 3 +  2 2 2 +  2 2 1 = −16
1 2 4  3 2 4  3 1 4  4 2 5

The coefficient I 2 is obtained by the sum of the determinants of the resulting matrices by
eliminating two rows and two columns associated with the main diagonal, i.e.
1 2 3 1 1 2 3 1 1 2 3 1
2 2 1 2 2 2 1 2 2 2 1 2
I2 =  +  +  +
4 1 5 3 4 1 5 3 4 1 5 3
     
3 1 2 4 3 1 2 4 3 1 2 4
1 2 3 1  1 2 3 1
2 2 1 2 2 2 1 2
+  + +
4 1 5 3  4 1 5 3
   
3 1 2 4  3 1 2 4
1 2 3 1
2 2 1 2
+ 
4 1 5 3
 
3 1 2 4
5 3  2 2 2 1 1 1  1 3 1 2
=  +   +   +   +  +   = 21
 2 4 1 4 1 5 3 4  4 5  2 2
The coefficient I 1 is obtained by the sum of the determinants of the resulting matrices by
eliminating three rows and three columns associated with the main diagonal, i.e.
1 2 3 1  1 2 3 1  1 2 3 1  1 2 3 1
2 2 1 2  2 2 1 2 2 2 1 2 2 2 1 2
I1 =  + + +
4 1 5 3  4 1 5 3  4 1 5 3  4 1 5 3
       
3 1 2 4  3 1 2 4  3 1 2 4  3 1 2 4
= [4] + [1] + [2] + 5 = 12 = Tr (P )

University of Castilla-La Mancha Draft By: Eduardo W. V. Chaves (2016)


Ciudad Real - Spain
74 SOLVING PROBLEMS BY MEANS OF CONTINUUM MECHANICS

Problem 1.82
Let A be a second-order tensor, show that:

a) II A =
1
2
{
( I A ) 2 − Tr ( A 2 ) }
b) det ( A ) =
1
6
{
[Tr (A )]3 + 2 Tr( A 3 ) − 3Tr(A ) Tr(A 2 ) }
Solution:
a) It was shown in Problem 1.79 that III A A −1 = ( A 2 − AI A + 1 II A ) , then, by applying the
double scalar product with the second-order unit tensor we obtain:
( )
III A A −1 : 1 = A 2 − AI A + 1 II A : 1 = A 2 : 1 − A : 1 I A + 1 : 1 II A
−1
III A Tr ( A ) = Tr ( A ) − Tr ( A ) I A + Tr (1) II A = Tr ( A 2 ) − ( I A ) 2 + 3 II A
2

Taking into account the inverse of a tensor A −1 =


[cof ( A )]T , we can conclude that:
III A


III A Tr ( A −1 ) = Tr ( III A A −1 ) = Tr  III A
[cof ( A )]T 
( )
 = Tr [cof ( A )]T = Tr ([cof ( A ) ]) = II A
 III A 
 
With that, we can obtain:
III A Tr ( A −1 ) = II A = Tr ( A 2 ) − ( I A ) 2 + 3 II A ⇒ II A − 3 II A = Tr ( A 2 ) − ( I A ) 2

⇒ II A =
1
2
{
( I A ) 2 − Tr ( A 2 ) }
b) We start from the Cayley-Hamilton theorem, which states that any tensor satisfies its own
characteristic equation, i.e.:
A 3 − A 2 I A + AII A − III A 1 = 0 (1.68)

where I A = [Tr (A )] , II A =
1
2
{ }
[Tr( A )]2 − Tr( A 2 ) and III A = det (A ) are the principal invariants

of A . Applying the double scalar product between the second-order unit tensor ( 1 ) and the
equation in (1.68) we obtain:
A 3 : 1 − A 2 : 1 I A + A : 1 II A − III A 1 : 1 = 0 : 1
Tr ( A 3 ) − Tr ( A 2 ) I A + Tr ( A ) II A − III A [ Tr (1)] = [ Tr (0 )]

Tr ( A 3 ) − Tr ( A 2 ) Tr ( A ) + Tr ( A )
1
2
{
[ Tr ( A )]2 − Tr ( A 2 ) − III A 3 = 0}
1 1
Tr ( A 3 ) − Tr ( A 2 ) Tr ( A ) + [ Tr ( A )]3 − Tr ( A ) Tr ( A 2 ) − III A 3 = 0
2 2
1
2
{2 Tr ( A 3 ) − 3 Tr ( A 2 ) Tr ( A ) + [ Tr ( A )]3 − III A 3 = 0}
with which we obtain:

III A = det ( A ) =
1
6
{
[Tr( A )]3 + 2 Tr(A 3 ) − 3Tr( A 2 ) Tr(A ) }
or in indicial notation:

III A = det ( A ) =
1
6
{
A ii A jj A kk + 2 A ij A jk A ki − 3A ij A ji A kk }

University of Castilla-La Mancha Draft By: Eduardo W. V. Chaves (2016)


Ciudad Real - Spain
1 TENSORS 75

NOTE: It is interesting to note that the principal invariants of A are formed by the three
fundamental invariants of a second-order tensor, namely Tr (A ) , Tr ( A 2 ) , Tr ( A 3 ) , i.e.:
I A = Tr ( A )

II A =
1
2
{
[ Tr ( A )]2 − Tr ( A 2 ) }
III A = det ( A ) =
1
6
{
[Tr ( A )]3 + 2 Tr ( A 3 ) − 3 Tr ( A 2 ) Tr ( A ) }

Problem 1.83

Show that II T = III T Tr ( T −1 ) , where II T =


1
2
{
[Tr( T )]2 − Tr(T 2 ) } is the second principal
invariant of T , and III T is the third principal invariant (the determinant of T ).
Solution:
1
It was shown in Problem 1.79 that T −1 = ( T 2 − TI T + 1 II T ) , then, by applying the double
III T
scalar product with the second-order unit tensor we obtain:

T −1 : 1 =
1
III T
( )
T 2 − TI T + 1 II T : 1 =
1
III T
(
T 2 : 1 − T : 1 I T + 1 : 1 II T )
Tr ( T −1 ) =
1
III T
(
Tr ( T 2 ) − Tr ( T ) I T + Tr (1) II T )
⇒ III T Tr ( T −1 ) = Tr ( T 2 ) − I T2 + 3 II T ⇒ III T Tr ( T −1 ) = II T
14243
= −2 II T

Problem 1.84
Show that:
r r
r r 1 β (c ⊗ b )
(α1 + β c ⊗ b) −1 = 1 − r r
α α (α + β c ⋅ b)
r r
where c and b are vectors, 1 is the second-order unit tensor, and α and β are scalars.
Solution:
r r
Let us consider that T = (α1 + β c ⊗ b) , and the inverse of a tensor obtained in Problem
1.79:

T −1 =
1
III T
(
T 2 − TI T + 1 II T ) (1.69)

Next, we obtain T 2 :
r r r r
T 2 = T ⋅ T = (α1 + β c ⊗ b) ⋅ (α1 + β c ⊗ b)
r r r r r r r r
= α 2 1 ⋅ 1 + αβ 1 ⋅ (c ⊗ b) + αβ (c ⊗ b) ⋅ 1 + β 2 (c ⊗ b) ⋅ (c ⊗ b)
r r r r r r r r
where it fulfills that (c ⊗ b) ⋅ (c ⊗ b) = (c ⋅ b)(c ⊗ b) , (see Problem 1.20). Then, the above
equation can be rewritten as follows:

University of Castilla-La Mancha Draft By: Eduardo W. V. Chaves (2016)


Ciudad Real - Spain
76 SOLVING PROBLEMS BY MEANS OF CONTINUUM MECHANICS

r r r r r r
T 2 = α 2 1 + 2αβ (c ⊗ b) + β 2 (c ⋅ b)(c ⊗ b)
and its trace is given by:
r r r r r r r r r r r r
Tr ( T 2 ) = Tr[α 2 1 + 2αβ (c ⊗ b) + β 2 (c ⋅ b)(c ⊗ b)] = α 2 Tr (1) + 2αβTr (c ⊗ b) + β 2 (c ⋅ b) Tr (c ⊗ b)
r r r r r r r r r r
= 3α 2 + 2αβ (c ⋅ b) + β 2 (c ⋅ b)(c ⋅ b) = 3α 2 + 2αβ (c ⋅ b) + β 2 (c ⋅ b) 2
Next, we calculate the principal invariants of T
r r r r r r
I T = Tr (α1 + β c ⊗ b) = αTr (1) + β Tr (c ⊗ b) = 3α + β (c ⋅ b)
r r r r r r
( I T ) 2 = [3α + β (c ⋅ b)]2 = 9α 2 + 6 β (c ⋅ b) + β 2 (c ⋅ b) 2

II T =
1 2
2
{ 1
} { r r r r r r r r
I T − Tr ( T 2 ) = 9α 2 + 6 β (c ⋅ b) + β 2 (c ⋅ b) 2 − [3α 2 + 2αβ (c ⋅ b) + β 2 (c ⋅ b ) 2 ]
2
}
r r
= 3α + 2αβ (c ⋅ b)
2

r r r r
III T = det (α1 + β c ⊗ b) = α 3 + α 2 β c ⋅ b , (see Problem 1.49).
Then, the equation in (1.69) becomes:
III T T −1 = T 2 − I T T + II T 1
r r r r r r
= α 2 1 + 2αβ (c ⊗ b ) + β 2 (c ⋅ b)(c ⊗ b )
[ r r
] r r
[
r r
− 3α + β (c ⋅ b ) (α 1 + β c ⊗ b) + 3α 2 + 2αβ (c ⋅ b) 1 ]
r r r r r r r r r r
= α 2 1 + 2αβ (c ⊗ b ) + β 2 (c ⋅ b)(c ⊗ b ) − 3α 2 1 − 3αβ (c ⊗ b ) − αβ (c ⋅ b)1 (1.70)
r r r r r r
− β 2 (c ⋅ b)(c ⊗ b ) + 3α 2 1 + 2αβ (c ⋅ b)1
r r r r r r r r
= 1α 2 + αβ (c ⋅ b)1 − αβ (c ⊗ b) = (α 2 + αβ c ⋅ b )1 − αβ (c ⊗ b)
1 r r r r
= (α 3 + α 2 β c ⋅ b)1 − αβ (c ⊗ b) = [adj( T )] = [cof ( T )]
T

α
r r r r
Taking into account that T = (α1 + β c ⊗ b) , III T = α 3 + α 2 β c ⋅ b , the above equation
becomes:
r r r r
−1 1 III T αβ (c ⊗ b) 1 αβ (c ⊗ b)
T = 1− = 1− r r (1.71)
α III T III T α (α 3 + α 2 β c ⋅ b )
or:
r r 1 β r r
(α1 + β c ⊗ b) −1 = 1 − r r (c ⊗ b) Tensorial notation (1.72)
α α (α + β c ⋅ b)
1 β
(α δ ij + β c i b j ) −1 = δ ij − (c b ) Indicial notation (1.73)
α α (α + β c k b k ) i j

[α[1] + β [{c}{b} ] ] T −1
=
1
α
[ 1] −
β
α (α + β {c} {b})
T
[
{c}{b}T ] Matrix notation (1.74)

NOTE 1: The above equation is also valid for matrices of n-dimensions.


In the particular case when α = 1 , β = 1 , we can obtain:
r r
r r −1 (c ⊗ b)
(1 + c ⊗ b) = 1 − r r (1.75)
1+ c ⋅b

University of Castilla-La Mancha Draft By: Eduardo W. V. Chaves (2016)


Ciudad Real - Spain
1 TENSORS 77

NOTE 2: The equation in (1.72) or in (1.70) can be rewritten as follows:


r r 1 β r r
T −1 = (α 1 + β c ⊗ b) −1 = 1 − r r (c ⊗ b )
α α (α + β c ⋅ b)
=
1
r [ r r r r
r (α 2 + αβ c ⋅ b)1 − βα (c ⊗ b) =
(α + α β c ⋅ b)
3 2
1
]
det ( T )
[adj( T )]
with that we can conclude that:
r r r r r r
adj(α1 + β c ⊗ b) = (α 2 + αβ c ⋅ b)1 − βα (c ⊗ b)
NOTE 3: We can extend the equation in (1.72) such that:
1 β
(α I sym + β A ⊗ B ) −1 = I sym − ( A ⊗ B)
α α (α + β A : B )
where we now have that I sym is the symmetric fourth-order unit tensor, A and B are second-
order tensors, and α and β are scalars. With that it is easy to show that ( I sym ) −1 = I sym .

Problem 1.85
r r 1 β r r
Taking into account that (α 1 + β c ⊗ b) −1 = 1− r r (c ⊗ b) , (see Problem 1.84),
α α (α + β c ⋅ b)
show that:
r r 1
(α A + β a ⊗ b) −1 = A −1 −
α
r
β
r
α (α + β b ⋅ A −1 ⋅ a)
[ r r
( A −1 ⋅ a) ⊗ (b ⋅ A −1 ) ] (1.76)

r r
where a and b are vectors, A is a second-order tensor, with det ( A ) ≠ 0 ( ∃A −1 ), and α and
β are scalars.
Solution:
r r
Note that the expression (αA + β a ⊗ b) can be rewritten as follows:
r r r r r r r r
(αA + β a ⊗ b) = (αA ⋅ 1 + β 1 ⋅ a ⊗ b) = (αA ⋅ 1 + β ( A ⋅ A −1 ) ⋅ a ⊗ b) = A ⋅ (α1 + β A −1 ⋅ a ⊗ b)

Using the inverse property such as ( A ⋅ B ) −1 = B −1 ⋅ A −1 , we can obtain:


r r
[ r r −1
] r r
(αA + β a ⊗ b) −1 = A ⋅ (α1 + β A −1 ⋅ a ⊗ b) = (α1 + β A −1 ⋅ a ⊗ b) −1 ⋅ A −1
r
Note that the result of the algebraic operation A −1 ⋅ a is a vector in which we denote
r r
c = A −1 ⋅ a , with that we rewrite the above equation as follows:
r r r r r r
(αA + β a ⊗ b) −1 = (α 1 + β A −1 ⋅ a ⊗ b) −1 ⋅ A −1 = (α 1 + β c ⊗ b) −1 ⋅ A −1
1 β r r  −1 1 β r r
r r (c ⊗ b)  ⋅ A = 1 ⋅ A − r r (c ⊗ b) ⋅ A
−1 −1
=  1−
α α (α + β c ⋅ b)  α α (α + β c ⋅ b)
β r r β r
1 1 −1 −1 r
= A −1 − r r c ⊗b⋅A = A − r r ( A ⋅ a) ⊗ (b ⋅ A )
−1 −1
α α (α + β c ⋅ b) α α (α + β c ⋅ b)
1 β r r
= A −1 − r
−1 r
( A −1 ⋅ a) ⊗ (b ⋅ A −1 )
α α (α + β b ⋅ A ⋅ a)
The above equation in indicial notation becomes:

University of Castilla-La Mancha Draft By: Eduardo W. V. Chaves (2016)


Ciudad Real - Spain
78 SOLVING PROBLEMS BY MEANS OF CONTINUUM MECHANICS

1 β
(αA ij + β a i b j ) −1 = A ij−1 − ( A ik−1 a k )(b s A −sj1 )
α α (α + β b p A −pq1 a q )
r r r r
The reader should be aware here with the algebraic operation ( A −1 ⋅ a) ⋅ b ≠ A −1 ⋅ ( a ⋅ b )
14243
,
Invalid Expression
the latter has no consistency, since we cannot have a scalar product (contraction) with the
r r
scalar (a ⋅ b) . We can check this fact by means of indicial notation
r r r
c ⋅ b = c i b i = ( A −1 ⋅ a) i b i = A ik−1 a k b i , then, the possible expressions tensorial notation are
r r
( A −1 ⋅ a) ⋅ b = b i A ik−1a k = a k A ik−1b i = A ik−1b i a k = A ik−1 a k b i = A ik−1a k b i .
14
r 24 3 1 424 3r 1 42r4 3 1424 3r 1424 3r
r r r r r
b⋅A −1⋅a a⋅A −T ⋅b A −1:(b⊗a) A −T :( a⊗b ) A −1:( a⊗b )T

NOTE 1: For the particular case when α = 1 , β = 1 , we fall back on the Sherman-Morrison
formula:
r r
r r −1 ( A −1 ⋅ a) ⊗ (b ⋅ A −1 ) Sherman-Morrison formula
−1
( A + a ⊗ b) = A − r r (1.77)
1 + b ⋅ A −1 ⋅ a (tensorial notation)

The above equation in matrix notation becomes

{ }{
 [ A]−1{a} {b}T [ A]−1 T  } Sherman-Morrison formula
[[ A] + [{a}{b} ] ]
T −1 −1
= [ A] −  T −1
1 + {b} [ A] {a}

(matrix notation)
(1.78)

r r r r
NOTE 2: Note that if (αA + β a ⊗ b) = A ⋅ (α1 + β A −1 ⋅ a ⊗ b) , the determinant is defined
as follows:
r r
[ r r
] r r
det (αA + β a ⊗ b) = det A ⋅ (α1 + β A −1 ⋅ a ⊗ b) = det [A ]det (α1 + β A −1 ⋅ a ⊗ b)
r
[ ]
r
= det [A ](α 3 + α 2 β b ⋅ A −1 ⋅ a)
with that, the equation in (1.76) can be rewritten as follows:
r r
(αA + β a ⊗ b) −1 =
1
γ
{ A (α 2
r r r r
[
+ αβ b ⋅ A −1 ⋅ a) A −1 − A αβ ( A −1 ⋅ a) ⊗ (b ⋅ A −1 ) ]}
r r r r
with γ = det (αA + β a ⊗ b) = A (α 3 + α 2 β b ⋅ A −1 ⋅ a) . (1.79)
with that we conclude that:
r r
{ r r r r
adj(αA + β a ⊗ b ) = A (α 2 + αβ b ⋅ A −1 ⋅ a) A −1 − A αβ ( A −1 ⋅ a) ⊗ (b ⋅ A −1 ) [ ]}
NOTE 3: We can extend the equation in (1.76) such that:

(α D + β A ⊗ B ) −1 =
1
α
D −1 −
α (α + β B : D
β
−1
: A)
[(D −1
: A ) ⊗ (B : D −1 ) ] (1.80)

where we now have that D is a fourth-order tensor, A and B are second-order tensors, and
α and β are scalars.
Note that:

University of Castilla-La Mancha Draft By: Eduardo W. V. Chaves (2016)


Ciudad Real - Spain
1 TENSORS 79

(αD + β A ⊗ B ) −1 =
1
γ
{ D (α 2
[
+ αβ B : D −1 : A )D −1 − D αβ (D −1 : A ) ⊗ (B : D −1 ) ]}
with γ = det (αD + β A ⊗ B ) = D (α 3 + α 2 β B : D −1 : A ) . (1.81)
where we can conclude that:
det (αD + β A ⊗ B ) = det (D )(α 3 + α 2 β B : D −1 : A ) (1.82)
{ [
adj(α D + β A ⊗ B ) = D (α 2 + αβ B : D −1 : A )D −1 − αβ D (D −1 : A ) ⊗ (B : D −1 ) ]} (1.83)

Problem 1.86
r r r r
a) Let C = (α1 + β a ⊗ b + γ c ⊗ d) be a second-order tensor. Show that:
r r r r r r r r
[r r r r
α1 + β a ⊗ b + γ c ⊗ d = α 3 + α 2 γ (c ⋅ d) + α 2 β (a ⋅ b) + αβγ (a ⋅ b)(c ⋅ d) − (a ⋅ d)(b ⋅ c )
r r r r
]
(1.84)
r r r r r r r r
where α 1 + β a ⊗ b + γ c ⊗ d ≡ det (α 1 + β a ⊗ b + γ c ⊗ d) represents the determinant of the
r r r r
tensor C . b) For the particular case when α = 1 , d = a , c = b , show that:
r r r r r r r r 2
det (1 + β a ⊗ b + γ b ⊗ a) = 1 + (β + γ )(a ⋅ b) − βγ a ∧ b (1.85)

Solution:
r r r r
We define an auxiliary tensor D = α1 + β a ⊗ b and in turn we have C = (D + γ c ⊗ d) .
According to Problem 1.85, (see equation (1.76)), it holds that:
r r r r
det (D + γ c ⊗ d) = D (1 + γ d ⋅ D −1 ⋅ c ) , where:
r r r r
det (D) ≡ D = det (α1 + β a ⊗ b ) = α 3 + α 2 β (a ⋅ b) and
r r 1 β r r
(D ) −1 = (α1 + β a ⊗ b) −1 = 1 − r r (a ⊗ b)
α α (α + β a ⋅ b )
With that, we can obtain that:
r r r r
det (D + γ c ⊗ d) = D (1 + γ d ⋅ D −1 ⋅ c )

[ r r  r 1
]
= α 3 + α 2 β (a ⋅ b ) (1 + γ d ⋅  1 −
 α
β
r r
α (α + β a ⋅ b )
(
r r  r
a ⊗ b)  ⋅ c 
 
   

[ r r 
= α 3 + α 2 β (a ⋅ b ) 1 + γ ] 1 r r
 d ⋅1 ⋅ c −
α
β
r
r r r r 
r d ⋅ (a ⊗ b) ⋅ c  
α (α + β a ⋅ b ) 
  

[ r r  1 r r
= α 3 + α 2 β (a ⋅ b ) 1 + γ
 d⋅c −
α
] β
r r
α (α + β a ⋅ b )
(
r r
d ⋅ a ) ⊗ (b
r r 
⋅ c )  
   

[ r r  γ r r
]
= α 3 + α 2 β (a ⋅ b ) 1 + (c ⋅ d) −
αβγ
r
r r r r 
r (a ⋅ d)(b ⋅ c ) 
α 2 (α + β a ⋅ b )
 α 

University of Castilla-La Mancha Draft By: Eduardo W. V. Chaves (2016)


Ciudad Real - Spain
80 SOLVING PROBLEMS BY MEANS OF CONTINUUM MECHANICS

r r r r r r r r r r r r
Note that (d ⋅ a) ⊗ (b ⋅ c ) = (a ⋅ d) ⊗ (b ⋅ c ) ≡ (a ⋅ d)(b ⋅ c ) .
123 123
scalar scalar

r r
[ r r  γ r r 
] r r 
det (D + γ c ⊗ d) = α 3 + α 2 β (a ⋅ b ) 1 + (c ⋅ d)  − α 3 + α 2 β (a ⋅ b)  2 [ αβγ r r r r r 
r ]
(a ⋅ d)(b ⋅ c ) 
 α  α (α + β a ⋅ b ) 

[ r r  γ r
 α
r
]

r r r r
= α 3 + α 2 β (a ⋅ b ) 1 + (c ⋅ d)  − αβγ (a ⋅ d)(b ⋅ c )

3
[2 r r 2 r r r r r r r r r r
= α + α γ (c ⋅ d) + α β (a ⋅ b) + αβγ (a ⋅ b)(c ⋅ d) − αβγ (a ⋅ d)(b ⋅ c ) ]
Then:
r r r r r r r r r r r r r r r r
det (α 1 + β a ⊗ b + γ c ⊗ d) = α 3 + α 2γ (c ⋅ d) + α 2 β (a ⋅ b ) + αβγ [( a ⋅ b )( c ⋅ d) − (a ⋅ d)(b ⋅ c )]
with that we can show the equation in (1.84).
r r r r
For the particular case when d = a , c = b , we have:
r rr r r r r r r r r r r r r r
c ⊗ d) = α 3 + α 2γ (c ⋅ d) + α 2 β (a ⋅ b) − αβγ [(a ⋅ d)(b ⋅ c ) − (a ⋅ b)(c ⋅ d)]
det (α 1 + β a ⊗ b + γ
r rr r r r r r r r r r r r r r
b ⊗ a) = α 3 + α 2γ (b ⋅ a) + α 2 β (a ⋅ b) − αβγ [(a ⋅ a)(b ⋅ b) − (a ⋅ b)(b ⋅ a)]
det (α 1 + β a ⊗ b + γ
r r r r r r r r r r
= α 3 + α 2 ( β + γ )(a ⋅ b) − αβγ [(a ⋅ a)(b ⋅ b) − (a ⋅ b)(a ⋅ b)]
r r 2 r 2 r 2 r r
In Problem 1.1 we have shown that a ∧ b = a b − (a ⋅ b) 2 holds, thus:
r r r r r r r r 2
det (α1 + β a ⊗ b + γ b ⊗ a) = α 3 + α 2 (β + γ )(a ⋅ b) − αβγ a ∧ b

For the particular case when α = 1 we can obtain:


r r r r r r r r 2
det (1 + β a ⊗ b + γ b ⊗ a) = 1 + (β + γ )(a ⋅ b) − βγ a ∧ b

Problem 1.87
r r r r
a) Obtain the inverse of the tensor C = (α1 + β a ⊗ b + γ c ⊗ d) .
r r r r
p⊗ p (B ⋅ q) ⊗ (B ⋅ q)
b.1) Given the second-order tensor D = B + r r − r r where B = B T and
p⋅q q ⋅ B ⋅q
∃B −1 , show that:
r r r r
( p ⋅ q + p ⋅ B −1 ⋅ p ) r r
D −1
=B + −1
r r 2 [q ⊗ q ] − r 2 r [qr ⊗ ( B −1 ⋅ pr )]sym (1.86)
( p ⋅ q) ( p ⋅ q)

b.2) If B is a positive definite tensor, obtain the conditions under which D is a non-singular
tensor.
Solution:
r r r r
Denoting by A = (α1 + β a ⊗ b) we can obtain C = ( A + γ c ⊗ d) , and by taking into account
r r 1
(α A + β a ⊗ b) −1 = A −1 −
α
r
β
r
α (α + β b ⋅ A −1 ⋅ a)
r
[ r
( A −1 ⋅ a) ⊗ (b ⋅ A −1 ) ] (1.87)

which was obtained in Problem 1.85, (see equation (1.76)), thus


r r
( A + γ c ⊗ d) −1 = A −1 − r
γ
r
(1 + γ d ⋅ A −1 ⋅ c )
[ r r
( A −1 ⋅ c ) ⊗ (d ⋅ A −1 ) ] (1.88)

University of Castilla-La Mancha Draft By: Eduardo W. V. Chaves (2016)


Ciudad Real - Spain
1 TENSORS 81

It was shown in Problem 1.84 that:


r r
r r 1 β (c ⊗ b )
(α1 + β c ⊗ b) −1 = 1 − r r (1.89)
α α (α + β c ⋅ b)
With that we can obtain:
r r
r r 1 β (a ⊗ b)
A −1
= (α1 + β a ⊗ b) −1 = 1 − r r
α α (α + β a ⋅ b )
Furthermore, we have
r r r r r r
r 1 β (a ⊗ b)  r 1 r β (a ⊗ b) r 1 r β (b ⋅ c ) r
A ⋅c =
−1  1− r r ⋅c = 1⋅c − r r ⋅c = c − r r a
α α (α + β a ⋅ b)  α α (α + β a ⋅ b ) α α (α + β a ⋅ b )

r r r r r r
r r 1 β ( a ⊗ b )  1 r r β ( a ⊗ b ) 1 r β ( d ⋅ a) r
d⋅ A = d⋅ 1 −
−1
r r  = d ⋅1 − d ⋅ r r = d− r r b
α α ( α + β a ⋅ b )  α α ( α + β a ⋅ b ) α α ( α + β a ⋅ b)
 
With that we conclude that
r r r r r r r r r r r r r r
(α 1 + β a ⊗ b + γ c ⊗ d) −1 = θ (1) 1 + θ ( 2 ) (a ⊗ b ) + θ ( 3) [θ (1) c + θ ( 2 ) (b ⋅ c )a] ⊗ [θ (1) d + θ ( 2 ) (a ⋅ d)b]
(1.90)
where
1
θ (1) =
α
−β
θ ( 2) = r r
α (α + β a ⋅ b )
−γ
θ ( 3) = r r
(1 + γ d ⋅ A −1 ⋅ c )
r r 1 r r β r r r r
d ⋅ A −1 ⋅ c = (d ⋅ c ) − r r ( d ⋅ a )(b ⋅ c )
α α (α + β a ⋅ b )
NOTE: The equation in (1.90) is also valid for matrices of n-dimensions.
b.1) We can rewrite the tensor D as follows:
r r r r r r r r
p⊗ p (B ⋅ q) ⊗ (B ⋅ q) p⊗ p (B ⋅ q) ⊗ (B ⋅ q)
D = B ⋅1 + 1 ⋅ r r − 1 ⋅ r r = B ⋅ 1 + ( B ⋅ B −1
) ⋅ r r − ( B ⋅ B −1
) ⋅ r r
p⋅q q ⋅ B ⋅q p⋅q q ⋅ B ⋅q
r r r r r r r r
 −1 p ⊗ p −1 ( B ⋅ q ) ⊗ ( B ⋅ q )   ( B −1 ⋅ p) ⊗ p ( B −1 ⋅ B ⋅ q ) ⊗ ( B ⋅ q ) 
= B ⋅ 1 + B ⋅ r r − B ⋅ r r  = B ⋅ 1 + r r − r r 
 p⋅q q ⋅ B ⋅q   p⋅q q ⋅ B ⋅q 
−1 r r r r
 ( B ⋅ p) ⊗ p q ⊗ ( B ⋅ q ) 
= B ⋅ 1 + r r − r r 
 p⋅q q ⋅ B ⋅q 
and by denoting by
r r r r r r r r 1 −1
a = ( B −1 ⋅ p) ; b=p ; c =q ; d = (B ⋅ q) ; β= r r ; γ= r r
p⋅q q ⋅B ⋅q
Then, we can rewrite D as follows

University of Castilla-La Mancha Draft By: Eduardo W. V. Chaves (2016)


Ciudad Real - Spain
82 SOLVING PROBLEMS BY MEANS OF CONTINUUM MECHANICS

r r
[ r r
D = B ⋅ 1 + β a ⊗ b + γc ⊗ d = B ⋅ C ⇒ ]
D −1 = ( B ⋅ C ) −1 = C −1 ⋅ B −1
r r r r
where C = [1 + βa ⊗ b + γc ⊗ d ] . The inverse of C can be obtained via subsection (a) with
α = 1 . Moreover, we have:
θ (1) = 1 ,
−β −β −1 1 −1
θ ( 2) = r r = r r = r r = r r r r
α (α + β a ⋅ b) (1 + β a ⋅ b) p ⋅ q (1 + r 1 r ( B −1 ⋅ pr ) ⋅ pr ) ( p ⋅ q + p ⋅ B −1 ⋅ p)
p⋅q
r r 1 r r β r r r r
d ⋅ A −1 ⋅ c = (d ⋅ c ) − r r (a ⋅ d )(b ⋅ c )
α α (α + β a ⋅ b )
r r −1 r r r r
= (( B ⋅ q ) ⋅ q ) + r r r −1 r
(( B −1 ⋅ p) ⋅ ( B ⋅ q ) )( p ⋅ q )
( p ⋅ q + p ⋅ B ⋅ p)
r r r r
r r − ( p ⋅ ( B −T ⋅ B ) ⋅ q ) ( p ⋅ q )
= q ⋅B ⋅q + r r r r
( p ⋅ q + p ⋅ B −1 ⋅ p)
−γ 1 1
θ ( 3) = r
−1 r
= r r r r r r
(1 + γ d ⋅ A ⋅ c ) q ⋅ B ⋅ q  −1 r r − ( p ⋅ ( B −T ⋅ B ) ⋅ q ) ( p ⋅ q ) 
1 + r
 r q ⋅B⋅q + r r r −1 r


 q ⋅ B ⋅ q ( p ⋅ q + p ⋅ B ⋅ p ) 
r r r −1 r
( p ⋅ q + p ⋅ B ⋅ p)
= r r r r
( p ⋅ ( B −T ⋅ B ) ⋅ q ) ( p ⋅ q )
r r r r
−1 ( p ⋅ q + p ⋅ B −1 ⋅ p ) −1
θ ( 2 ) θ (3) = r r r −1 r r r r r = r r r r
( p ⋅ q + p ⋅ B ⋅ p) ( p ⋅ ( B ⋅ B ) ⋅ q ) ( p ⋅ q ) ( p ⋅ ( B ⋅ B ) ⋅ q ) ( p ⋅ q )
−T −T

r r −1 −1 r r
θ ( 2 ) θ ( 3) (a ⋅ d ) = r r r r (( B ⋅ p) ⋅ ( B ⋅ q ))
( p ⋅ (B −T
⋅ B) ⋅ q) ( p ⋅ q )
−1 r r −1
r r r ( p ⋅ (B ⋅ B) ⋅ q ) = r r
−T
= r
( p ⋅ ( B −T ⋅ B) ⋅ q) ( p ⋅ q ) ( p ⋅ q)
r r −1 r r −1
θ ( 2 ) θ ( 3) (b ⋅ c ) = r r r r ( p ⋅ q) = r r
( p ⋅ (B −T
⋅ B) ⋅ q ) ( p ⋅ q ) ( p ⋅ (B ⋅ B) ⋅ q )
−T

r r r r −1 −1 r r r r
θ ( 2 ) θ (3) (b ⋅ c )(a ⋅ d ) = r r r r (( B ⋅ p) ⋅ ( B ⋅ q ))( p ⋅ q ) = −1
( p ⋅ (B −T
⋅ B) ⋅ q ) ( p ⋅ q )
The equation in (1.90) becomes:
r r r
[ r r r r
] [ r r r
C −1 = 1 + θ ( 2 ) (a ⊗ b ) + θ ( 3) c + θ ( 2 ) (b ⋅ c )a ⊗ d + θ ( 2 ) (a ⋅ d )b ]
r r
[ ]r r
[ ] r r r r
[ ] r r r r
C −1 = 1 + θ ( 2 ) a ⊗ b + θ ( 3) c ⊗ d + θ ( 3)θ ( 2 ) (a ⋅ d ) c ⊗ b + θ ( 3)θ ( 2 ) (b ⋅ c ) a ⊗ d [ ]
r r r r r r
+ θ ( 3)θ (22 ) (b ⋅ c )(a ⋅ d ) a ⊗ b [ ]
{ r r r r r r
}[ ]r r
[ ] r r r r
C −1 = 1 + θ ( 2 ) + θ ( 3)θ (22 ) (b ⋅ c )(a ⋅ d ) a ⊗ b + θ ( 3) c ⊗ d + θ ( 3)θ ( 2 ) (a ⋅ d ) c ⊗ b [ ]
r r r r
+ θ ( 3)θ ( 2 ) (b ⋅ c ) a ⊗ d [ ]
Note that: {θ ( 2 ) + θ (3) θ (22 ) (b ⋅ c )(a ⋅ d )}= θ ( 2 ) {1 + θ (3) θ ( 2 ) (b ⋅ c )(a ⋅ d )}= θ ( 2 ) {1 − 1} = 0 , thus
r r r r r r r r

r r
[ ] r r r r
[ ] r r r r
C −1 = 1 + θ ( 3) c ⊗ d + θ ( 3) θ ( 2 ) (a ⋅ d ) c ⊗ b + θ ( 3) θ ( 2 ) (b ⋅ c ) a ⊗ d [ ]

University of Castilla-La Mancha Draft By: Eduardo W. V. Chaves (2016)


Ciudad Real - Spain
1 TENSORS 83

[r r
] r r r r
[ ] r r r r
C −1 = 1 + θ ( 3) c ⊗ d + θ ( 3)θ ( 2 ) (a ⋅ d ) c ⊗ b + θ ( 3)θ ( 2 ) (b ⋅ c ) a ⊗ d [ ]
r r r r
( p ⋅ q + p ⋅ B −1 ⋅ p ) r r −1 r r
=1+ r r r r [q ⊗ ( B ⋅ q ) ] + r r [q ⊗ p ]
( p ⋅ (B ⋅ B) ⋅ q ) ( p ⋅ q )
−T
( p ⋅ q)

+ r
−1 −1 r
[
r ( B ⋅ p) ⊗ ( B ⋅ q )
( p ⋅ (B ⋅ B) ⋅ q)
−T
r
]
With that, we can obtain:
D −1 = C −1 ⋅ B −1
r r r r
 ( p ⋅ q + p ⋅ B −1 ⋅ p) r r −1 r r
= 1 + r r r r [q ⊗ ( B ⋅ q )] + r r [q ⊗ p ]
 ( p ⋅ (B ⋅ B) ⋅ q ) ( p ⋅ q )
−T
( p ⋅ q)
+ r
−1
( p ⋅ (B ⋅ B) ⋅ q )
−T
−1 r
[
r  −1
r ( B ⋅ p) ⊗ ( B ⋅ q )  ⋅ B ]

r r r −1 r
( p ⋅ q + p ⋅ B ⋅ p) r r −1 r r
= B −1 + r r r r [q ⊗ ( B ⋅ q )]⋅ B + r r [q ⊗ p ]⋅ B
−1 −1
( p ⋅ (B ⋅ B) ⋅ q ) ( p ⋅ q)
−T
( p ⋅ q)

+ r
−1 −1 r r
[
r ( B ⋅ p) ⊗ ( B ⋅ q ) ⋅ B
( p ⋅ (B ⋅ B) ⋅ q )
−T
−1
]
Note that:
{[qr ⊗ ( B ⋅ qr )]⋅ B } = [qr ⊗ ( B ⋅ qr )] B = [q ( B ⋅ qr ) ]B
−1
ij ik
−1
kj i k
−1
kj [ ]
= q i ( B kp q p ) B kj−1 = q i ( B kp B kj−1 q p )
= [q ⊗ ( B ⋅ B ) ⋅ q ) ]
r −T r
ij

[( B ⋅ pr ) ⊗ ( B ⋅ qr )]⋅ B = ( B ⋅ pr ) ⊗ ( B ⋅ B ) ⋅ qr
−1 −1 −1 −T

Now, if we consider the symmetry of B , i.e. B = B T , we obtain:


r r r r
( p ⋅ q + p ⋅ B −1 ⋅ p ) r r
D −1 = B −1 + r r r r
( p ⋅ q) ( p ⋅ q)
[q ⊗ q ] + r− 1r
( p ⋅ q)
[qr ⊗ ( pr ⋅ B )] + ( pr−⋅1qr ) [( B
−1 −1
⋅ p) ⊗ q ]
r r

{[qr ⊗ ( pr ⋅ B )] + [( B }
r r r r
( p ⋅ q + p ⋅ B −1 ⋅ p ) r r
[q ⊗ q ] + r− 1r ⋅ p) ⊗ q ]
r r
= B −1 + r r r r
−1 −1
( p ⋅ q) ( p ⋅ q) ( p ⋅ q)
r r r −1 r
( p ⋅ q + p ⋅ B ⋅ p) r r
= B −1 + r r 2
( p ⋅ q)
[q ⊗ q ] − r 2 r
( p ⋅ q)
[qr ⊗ ( B −1
⋅ p)]
r sym

r r r
Note that, due to the symmetry of B , it holds that p ⋅ B −1 = B −1 ⋅ p = s , and B −T ⋅ B = 1 .
b.2) A tensor is non-singular if det (D ) ≠ 0 . By using the equation obtained previously we get:

[ r r r r
D = B ⋅ 1 + β a ⊗ b + γc ⊗ d ]
⇒ ( [ r r r r
]) r r r r
det (D) = det B ⋅ 1 + β a ⊗ b + γ c ⊗ d = det ( B )det 1 + β a ⊗ b + γ c ⊗ d [ ]
Note that det ( B ) > 0 , since B is a positive definite tensor. Then, the condition under which
r r r r
D is a non-singular tensor is det[1 + βa ⊗ b + γc ⊗ d ] ≠ 0 . By using the determinant
expression obtained in Problem 1.86 we can obtain:
r r r r r r r r r r
[
r r r r
det (α1 + β a ⊗ b + γ c ⊗ d ) = α 3 + α 2 γ (c ⋅ d ) − αβγ (a ⋅ d )( b ⋅ c ) − (a ⋅ b )(c ⋅ d ) ]
r r r r r r r r r r r r r r r r
where α = 1 , a ⋅ b = ( B −1 ⋅ p ) ⋅ p = p ⋅ B −1 ⋅ p , a ⋅ d = ( B −1 ⋅ p ) ⋅ ( B ⋅ q ) = p ⋅ q , b ⋅ c = p ⋅ q
r r r r r r r r −1 r r
c ⋅ d = q ⋅ ( B ⋅ q ) = q ⋅ B ⋅ q , γ (c ⋅ d ) = r r q ⋅ B ⋅ q = −1 ,
q ⋅B ⋅q

University of Castilla-La Mancha Draft By: Eduardo W. V. Chaves (2016)


Ciudad Real - Spain
84 SOLVING PROBLEMS BY MEANS OF CONTINUUM MECHANICS

βγ [(a ⋅ d )(b ⋅ c ) − (a ⋅ b )(c ⋅ d ) ] = r r r [ ]


r r r r r r r r 1 −1 r r r r r −1 r r r
r ( p ⋅ q )( p ⋅ q ) − ( p ⋅ B ⋅ p )( q ⋅ B ⋅ q )
p⋅q q ⋅B ⋅q
Thus:

[
r r r r
det 1 + β a ⊗ b + γ c ⊗ d = r r r
1
]
( p ⋅ q )(q ⋅ B ⋅ q )
r r r r
[ r −1 r r r
r ( p ⋅ q )( p ⋅ q ) − ( p ⋅ B ⋅ p )(q ⋅ B ⋅ q ) ≠ 0 ]
r r r r r r r r
Then, the conditions are: p ≠ 0 , q ≠ 0 , ( p ⋅ q ) ≠ 0 , i.e. p and q can not be orthogonal
vectors. Another condition that must be met is:
r r r r r r r r
( p ⋅ q )( p ⋅ q ) − ( p ⋅ B −1 ⋅ p )(q ⋅ B ⋅ q ) ≠ 0
142 4 43 4 144424443
>0 >0
r r
Note that by the fact that B is positive definite tensor, the scalar (q ⋅ B ⋅ q ) > 0 is always
r r r r
positive for any vector q ≠ 0 . The same apply to ( p ⋅ B −1 ⋅ p ) > 0 , since, if the tensor is
positive definite so is its inverse. Note also that D is a positive definite tensor if
r r r r r r r r
( p ⋅ q ) 2 > ( p ⋅ B −1 ⋅ p )(q ⋅ B ⋅ q ) and ( p ⋅ q ) > 0 . These two conditions can be replaced by
r r r r r r
( p ⋅ q ) > ( p ⋅ B −1 ⋅ p )(q ⋅ B ⋅ q ) .

Problem 1.88
Let A = A (τ) and τ be a second-order tensor and a scalar respectively, show that:

dA  dA
= A Tr ⋅ A −1  (1.91)
dτ  dτ 

Solution:
In Problem 1.82 and in Problem 1.79, we have demonstrated, respectively, that:

III A = det ( A ) = A =
1
6
{
[Tr( A )]3 + 2 Tr(A 3 ) − 3Tr( A 2 ) Tr(A ) } (1.92)

III A A −1 = A 2 − AI A + II A 1 (1.93)

where I A = Tr (A ) , II A =
1
2
{ }
[Tr ( A )]2 − Tr ( A 2 ) .
Note also that the following derivatives are true:
d [I A ] d [Tr ( A )] d [A kk ] d [A ik δ ik ] d [A ik ] dA  dA 
= = = = δ ik = : 1 = Tr  
dτ dτ dτ dτ dτ dτ  dτ 
[
d Tr ( A 2 ) ]
 d (A 2 ) 
= Tr 

 = Tr  2A ⋅
dA   dA 
= 2 Tr  A ⋅
dτ  dτ 
 dτ   dτ  

[
d Tr ( A 3 ) ]
= 3Tr  A 2 ⋅
dA 
dτ  dτ 
Taking the derivative of (1.92) with respect to τ we can obtain:

University of Castilla-La Mancha Draft By: Eduardo W. V. Chaves (2016)


Ciudad Real - Spain
1 TENSORS 85

d ( III A ) 1 d

=
6 dτ
{
[Tr( A )]3 + 2 Tr(A 3 ) − 3Tr( A 2 ) Tr(A ) }
1
= 3[Tr ( A )]
2 d [Tr ( A )]
+2
[
d Tr ( A 3 )
−3
] [
d Tr ( A 2 ) ]
Tr ( A ) − 3 Tr ( A 2 )
d [Tr ( A )]

6 dt dτ dτ dτ 
1  dA   2 dA   dA   dA  
3[Tr ( A )] Tr   + 6 Tr  A ⋅ − 6 Tr  A ⋅
2
=   Tr ( A ) − 3Tr ( A 2 ) Tr   
6  dτ   dτ   dτ   dτ  

= Tr  A 2 ⋅

dA 

dτ 
 dA 
− Tr  A ⋅


dτ 
1
2
{
2  dA 
Tr ( A ) + [Tr ( A )] − Tr ( A 2 ) Tr  
 dτ 
}
or
d ( III A )  dA   dA   dA 
= Tr  A 2 ⋅  − Tr  A ⋅  I A + II A Tr   (1.94)
dτ  dτ   dτ   dτ 
dA
Taking the scalar product of the equation in (1.93) with , we can obtain:

III A A −1 ⋅
dA

(
= A 2 − A I A + II A 1 ⋅
dA

)= A2 ⋅
dA

−A⋅
dA

I A + II A
dA

and the trace of the above equation is given by:
 dA   2 dA dA dA 
Tr  A −1 ⋅  III A = Tr  A ⋅ −A⋅ I A + II A 
 d τ   d τ d τ dτ 
(1.95)
 dA   dA   dA 
= Tr  A 2 ⋅  − Tr  A ⋅  I A + Tr   II A
 d τ   d τ   dτ 
Comparing equations (1.94) and (1.95), we can conclude that:
 dA   dA
⋅ A −1 
d ( III A )
= III A Tr  A −1 ⋅  = III A Tr 
dτ  dτ   dτ 

1.11 Isotropic and Anisotropic Tensors

Problem 1.89
Let C be a fourth-order tensor, whose components are
C ijkl = λδ ij δ kl + µδ ik δ jl + γδ il δ jk (1.96)
where δ ij are the second-order unit tensor components, and λ , µ and γ are scalar.
a) What kind of symmetry has the tensor C ? b) What conditions must be met to guarantee
the symmetry of C ?
Solution:
The tensor has major symmetry whether C ijkl = C klij holds. Taking into account the equation
in (1.96), we can conclude that the tensor has major symmetry since
C klij = λδ kl δ ij + µδ ki δ lj + γδ kj δ li = C ijkl

We check now if the tensor has minor symmetry, e.g. C ijkl = C ijlk

University of Castilla-La Mancha Draft By: Eduardo W. V. Chaves (2016)


Ciudad Real - Spain
86 SOLVING PROBLEMS BY MEANS OF CONTINUUM MECHANICS

C ijlk = λδ ij δ lk + µδ il δ jk + γδ ik δ jl ≠ C ijkl

We can easily verify this by the fact that when i = 2 , j = 1 , k = 1 , l = 2 , we have


C ijkl = C 2112 = λδ 21δ 12 + µδ 21δ 12 + γδ 22 δ 11 = γ

C ijlk = C 2121 = λδ 21δ 21 + µδ 22 δ 11 + γδ 21δ 12 = µ

Then, the tensor C has minor symmetry if and only if µ = γ , with that we can obtain:
C ijkl = λδ ij δ kl + µ (δ ik δ jl + δ il δ jk )

Note that δ ij δ kl has major and minor symmetry, while the tensors δ ik δ jl , δ il δ jk are not
symmetric. Note also that (δ ik δ jl + δ il δ jk ) = 2I ijkl
sym
.

Problem 1.90
Let C be a fourth-order tensor, whose components are given by:
C ijkl = λδ ij δ kl + µ (δ ik δ jl + δ il δ jk ) (1.97)
where λ , µ are constant real numbers. Show that C is isotropic.
Solution:
Applying the transformation law for the fourth-order tensor components:
C ′ijkl = a im a jn a kp a lq C mnpq (1.98)

and by replacing the relation C mnpq = λδ mnδ pq + µ (δ mpδ nq + δ mqδ np ) into the above equation,
we obtain:
[
C′ijkl = aim a jn akp alq λδ mnδ pq + µ (δ mpδ nq + δ mqδ np ) ]
= λaim a jn akp alqδ mnδ pq + µ ( aim a jn akp alqδ mpδ nq + aim a jn akp alqδ mqδ np )
(1.99)
= λain a jn akq alq + µ ( aip a jq akp alq + aiq a jn akn alq )
= λδ ijδ kl + µ (δ ikδ jl + δ ilδ jk ) = C ijkl

which proves that C is an isotropic tensor, i.e. the C -components do not change for any
transformation basis.

Problem 1.91
Let C be a symmetric isotropic fourth-order tensor which is represented by its components
as follows:
(
C ijkl = λδ ij δ kl + µ δ ik δ jl + δ il δ jk ) (indicial notation)
C = λ1 ⊗ 1 + 2µ I (tensorial notation)
where λ and µ are scalars, 1 is the second-order unit tensor, I is the symmetric fourth-
order unit tensor, i.e. I ≡ I sym .
a) Given a symmetric second-order tensor ε , obtain an expression for σ knowing that
σ = C : ε . Express the result in indicial and tensorial notation.
b) Show that σ and ε have the same eigenvectors, i.e. the same principal directions.

University of Castilla-La Mancha Draft By: Eduardo W. V. Chaves (2016)


Ciudad Real - Spain
1 TENSORS 87

c) If γ σ are the eigenvalues (principal values) of σ , obtain the eigenvalues of ε in terms of


γσ .
Solution:
a)
Tensorial notation Indicial notation
σij = Cijkl ε kl
σ = C :ε [
= λδ ijδ kl + µ (δ ikδ jl ]
+ δ ilδ jk ) ε kl
= (λ1 ⊗ 1 + 2 µI ) : ε = λδ ijδ kl ε kl + µ (δ ikδ jl ε kl + δ ilδ jk ε kl )
= λ1 ⊗ 1 : ε + 2 µ I{

{ = λδ ij ε kk + µ (ε ij + ε ji )
Tr (ε ) ε sym = ε
= λTr (ε )1 + 2 µε = λδ ij ε kk + 2 µ (ε ijsym )
= λδ ij ε kk + 2 µεij

where we have considered the symmetry of the tensor ε = ε T .


b) and c) Starting from the definition of eigenvalues ( γ σ )-eigenvectors ( n̂ ) of the tensor σ :
σ ⋅ nˆ = γ σ nˆ
and by substituting the value of σ obtained previously we can obtain:
(λTr(ε)1 + 2 µ ε )⋅ nˆ = γ σ nˆ
⇒ λTr (ε )1 ⋅ nˆ + 2 µ ε ⋅ nˆ = γ σ nˆ ⇒ λTr (ε )nˆ + 2 µ ε ⋅ nˆ = γ σ nˆ
⇒ 2 µ ε ⋅ nˆ = γ σ nˆ − λTr (ε )nˆ ⇒ 2 µε ⋅ nˆ = (γ σ − λTr (ε ) )nˆ
 γ − λTr (ε ) 
⇒ ε ⋅ nˆ =  σ  nˆ
 2µ 
⇒ ε ⋅ nˆ = γ ε nˆ
Note that the last equation is the definition of eigenvalue-eigenvector of ε . With that we
conclude that σ and ε have the same eigenvectors (they are coaxial). And the eigenvalues of
ε can be obtained as follows:
γ σ − λTr (ε )
γε =

If we denote by γ ε(1) = ε1 , γ (ε2) = ε 2 , γ ε(3) = ε 3 and γ σ(1) = σ1 , γ σ( 2) = σ 2 , γ σ(3) = σ 3 . The explicit


form of the above relationship is given by:
ε 1 0 0 σ1 0 0  1 0 0
0 ε2 0= 1 
0 σ2 0  − λTr (ε ) 0 1 0
 2µ   2µ  
 0 0 
ε3  
0 0 σ 3  0 0 1

σ1 0 0  1 0 0 ε 1 0 0
where it is also true that  0 σ2 0  = λTr (ε ) 0 1 0 + 2µ  0
    ε2 0 
 0 0 σ 3  0 0 1  0 0 ε 3 

University of Castilla-La Mancha Draft By: Eduardo W. V. Chaves (2016)


Ciudad Real - Spain
88 SOLVING PROBLEMS BY MEANS OF CONTINUUM MECHANICS

Problem 1.92
a) Obtain the inverse of the fourth-order tensor C = 2µ I + λ1 ⊗ 1 where I ≡ I sym is the
symmetric fourth-order unit tensor, 1 is the second-order unit tensor, and µ > 0 and λ are

scalars. b) Obtain the determinant of C . In addition, if we consider that λ = ,
(1 + ν )(1 − 2ν )
E
µ= , find the possible values for E and ν in order to guarantee that the tensor C is
2(1 + ν )
positive definite. c) Obtain also the reciprocal of the equation σ = C : ε in function of µ > 0 ,
λ , where σ and ε are symmetric second-order tensors.
Solution:
a) We use the equation obtained in (1.80):

(α D + β A ⊗ B ) −1 =
1
α
D −1 −
α (α + β B : D
β
−1
: A)
[(D −1
: A ) ⊗ (B : D −1 ) ]
By denoting by D = I , A = B = 1 , α = 2µ , β = λ , the above equation can be rewritten as
follows:

C −1 = ( 2µ I + λ1 ⊗ 1) −1 =
1 −1

I −
λ
2µ ( 2µ + λ 1 : I −1 : 1)
(I −1 : 1) ⊗ (1 : I −1 ) [ ]
Remember that it holds that I −1 = I and (I −1 : 1) = I : 1 = 1 . Then we can obtain the scalar
value of 1 : I −1 : 1 = 1 : I : 1 = 1 : 1 = Tr (1) = 3 . We also express in indicial notation:
1 1
1 : I −1 : 1 = 1 : I : 1 = δ ij I ijkl
sym
δ kl =δ ij (δ ik δ jl + δ il δ jk )δ kl = (δ ij δ ik δ jl δ kl + δ ij δ il δ jk δ kl )
2 2
1
= (δ jj +δ jj )=3
2
Resulting that:
1 λ
C −1 = ( 2µ I + λ1 ⊗ 1) −1 = I− (1 ⊗ 1)
2µ 2µ ( 2µ + 3λ )

Let us check whether C : C −1 = I sym ≡ I holds or not:


 1 λ 
C : C −1 = ( 2µ I + λ1 ⊗ 1) :  I− (1 ⊗ 1) 
 2µ 2µ ( 2µ + 3λ ) 
2µ 2µλ λ λ2
C : C −1 = ( I:I − I : (1 ⊗ 1) + (1 ⊗ 1) : I − (1 ⊗ 1) : (1 ⊗ 1)
2µ 2µ ( 2µ + 3λ ) 2µ 2µ ( 2µ + 3λ )
According to Problem 1.27 it fulfills that I : I = I , I : (1 ⊗ 1) = (1 ⊗ 1) : I = 1 ⊗ 1 , and
(1 ⊗ 1) : (1 ⊗ 1) = 3(1 ⊗ 1) . With that we can obtain:

 − 2µλ λ 3λ 2 
C : C −1 = I +  + − (1 ⊗ 1) = I
µ ( 2µ + 3λ ) 2µ 2µ ( 2µ + 3λ ) 
124 44444424444444 3
=0

b) We can use directly the equation (1.32), (see Problem 1.49):


det (αI sym + βA ⊗ B) = α 3 + α 2 β A : B

and by denoting by α = 2µ , β = λ , A = B = 1 we can conclude that:

University of Castilla-La Mancha Draft By: Eduardo W. V. Chaves (2016)


Ciudad Real - Spain
1 TENSORS 89

det (2µ I + λ1 ⊗ 1) = (2µ ) 3 + (2µ ) 2 λ 1 : 1 = (2µ ) 3 + (2µ ) 2 λ 3 = (2µ ) 2 (2µ + 3λ )

The tensor C is definite positive if the eigenvalues are positive numbers, i.e.:
E
µ >0⇒ µ = >0
2(1 + ν )
E Eν E
2 µ + 3λ > 0 ⇒ 2 +3 = >0
2(1 + ν ) (1 + ν )(1 − 2ν ) (1 − 2ν )
Denoting by y1 = (1 + ν ) ≠ 0 , y2 = (1 − 2ν ) ≠ 0 , we can conclude that:
 E > 0  E > 0
 
E E   y1 > 0 E E  y 2 > 0
µ= = >0 ⇒  ; 2 µ + 3λ = = >0 ⇒ 
2(1 + ν ) 2 y1  E < 0 (1 − 2ν ) y 2  E < 0
 y < 0  y < 0
 1  2
The above conditions must fulfill simultaneously. Then, by means of Figure 1.23 we can
conclude that E > 0 and − 1 < ν < 0.5 .

y (ν )

y2 = (1 − 2ν ) ≠ 0

zone not feasible y1 = (1 + ν ) ≠ 0

zone not feasible

1
ν ≠ −1
( y 2 > 0 ⇒ E > 0) E >0 ( y 1 > 0 ⇒ E > 0)

( y 1 < 0 ⇒ E < 0) ( y 2 < 0 ⇒ E < 0)


ν
ν ≠ 0 .5

Figure 1.23
c)
σ = C:ε ⇒ C −1 : σ = C −1 : C : ε ⇒ C −1 : σ = I sym : ε = ε sym = ε
⇒ ε = C −1 : σ
 1 λ  1 λ
⇒ε= I− 1 ⊗ 1 : σ = I:σ − 1 ⊗1:σ
 2µ 2µ ( 2µ + 3λ )  2µ 2µ ( 2µ + 3λ )
1 λ
⇒ε= σ− Tr (σ )1
2µ 2µ ( 2µ + 3λ )
The transformation between the two hyperspaces can be appreciated in Figure 1.24. It is also
worth reviewing Problem 1.41.

University of Castilla-La Mancha Draft By: Eduardo W. V. Chaves (2016)


Ciudad Real - Spain
90 SOLVING PROBLEMS BY MEANS OF CONTINUUM MECHANICS

x3 C x3
ε 33 σ 33
σ = C:ε

ε13 σ13
ε 23 ε 32 σ 23 σ 32
ε 31 ε 22 σ 31 σ 22

ε12 σ12
ε 21 x2 σ 21 x2
ε11 σ11

x1 x1
ε = C −1 : σ

C −1

Figure 1.24

Problem 1.93
ˆ ) be a second-order tensor, which is known as the elastic acoustic tensor, and is
Let Q e (N
defined as follows:
Q e (N ˆ ⋅ Ce ⋅N
ˆ) =N ˆ

where N̂ is the unit vector and C e is the isotropic symmetric fourth-order tensor and given
by C e = λ (1 ⊗ 1) + 2µ I , whose components are: C ijkl
e
= λδ ij δ kl + µ (δ ik δ jl + δ il δ jk ) . Obtain
the components of the elastic acoustic tensor.
Solution:
Using symbolic notation we can obtain:
Q e (N ˆ ⋅ Ce ⋅N
ˆ) =N ˆ = Nˆ eˆ
i i ( )⋅ (C e ˆ
pqrs e p ⊗ eˆ q ⊗ eˆ r ⊗ eˆ s )⋅ (Nˆ eˆ )
j j

( )
= Nˆ i C epqrs Nˆ j δ ip δ sj eˆ q ⊗ eˆ r = N
ˆ C e Nˆ eˆ ⊗ eˆ
p pqrs s q r( )
ˆ ) are:
Then, the components of Q e (N
ˆ Ce Nˆ = N
Qe qr = N p pqrs s p pq rs [
ˆ λδ δ + µ (δ δ + δ δ ) Nˆ
pr qs ps qr s ]
ˆ N
= λδ pqδ rsN ˆ ˆ ˆ ˆ ˆ ˆ ˆ ˆ ˆ ˆ ˆ
p s + µ (N pδ prδ qsNs + N pδ psδ qrNs ) = λNqNr + µ (NrNq + Nsδ qrNs )

Note that N̂ is the unit vector, then Nˆ s Nˆ s = 1 holds. With that we can obtain:
ˆ Nˆ
Q e qr = µδ qr + (λ + µ )N tensorial
 → ˆ ) = µ 1 + (λ + µ )N
Q e (N ˆ ⊗N
ˆ
q r

Problem 1.94
Let Q be a symmetric second-order tensor and given by:
ˆ ) = µ 1 + (λ + µ )N
Q (N ˆ ⊗N
ˆ

University of Castilla-La Mancha Draft By: Eduardo W. V. Chaves (2016)


Ciudad Real - Spain
1 TENSORS 91

where λ and µ are scalars, and N̂ is the unit vector.


ˆ ) and determine the restrictions on λ and µ in order to
a) Obtain the eigenvalues of Q (N
ˆ ) , i.e. ∃ Q −1 .
guarantee the existence of Q −1 (N
Eν E
b) Taking into account that λ = , µ= , determine the possible values of
(1 + ν )(1 − 2ν ) 2(1 + ν )
ˆ ) is a positive definite tensor.
( E ,ν ) with which Q (N
ˆ).
c) Obtain the inverse of Q (N
Solution:
r r
a) It was shown in Problem 1.49 that, given the vectors a and b it holds that:
r r r r
det ( β 1 + αa ⊗ b) = β 3 + β 2αa ⋅ b
The eigenvalues can be determined by means of the characteristic determinant
det (Q − γ1) = 0 , where γ i are the eigenvalues of Q . Then:
ˆ ⊗N
det[ µ 1 + (λ + µ )N ˆ − γ1] = 0 ⇒ ˆ ⊗N
det[( µ − γ )1 + (λ + µ )N ˆ]=0

Denoting by β = (µ − γ ) and α = (λ + µ ) we conclude that:


ˆ ⊗N
det[( µ − γ )1 + (λ + µ )N ˆ]=0 ⇒ ˆ ⋅N
( µ − γ )3 + ( µ − γ ) 2 (λ + µ )N ˆ =0
{
=1

( µ − γ ) [( µ − γ ) + (λ + µ )] = 0
2
⇒ ( µ − γ ) [(λ + 2 µ ) − γ ] = 0
2

The above characteristic equation has the following solutions:


  γ1 = µ
( µ − γ ) = 0 ⇒ 
2
( µ − γ ) [(λ + 2 µ ) − γ ] = 0
2 solution
 
→  γ 2 = µ
[(λ + 2 µ ) − γ ] = 0 ⇒ γ = (λ + 2 µ )
 3

In the principal space of Q , the components of Q are:


µ 0 0 
Qij′ =  0 µ 0 

 0 0 (λ + 2µ )
The inverse of Q exits if the determinant of Q is non-zero:
µ ≠ 0
Q = µ 2 (λ + 2 µ ) ≠ 0 ⇒ 
λ + 2 µ ≠ 0 ⇒ λ ≠ −2 µ

b) A tensor is definite positive if its eigenvalues are greater than zero, then:
 E
 µ = 2(1 + ν ) > 0


λ + 2 µ = Eν E E (1 − ν )
+2 = >0
 (1 + ν )(1 − 2ν ) 2(1 + ν ) (−2ν 2 − ν + 1)

(1 + ν ) ≠ 0 ⇒ ν ≠ −1

We check that  ν ≠ −1
(−2ν − ν + 1) ≠ 0 ⇒ ν ≠ 0.5
2

 

University of Castilla-La Mancha Draft By: Eduardo W. V. Chaves (2016)


Ciudad Real - Spain
92 SOLVING PROBLEMS BY MEANS OF CONTINUUM MECHANICS

Denoting by y1 = (1 + ν ) ≠ 0 , y2 = (1 − ν ) ≠ 0 , y3 = (−2ν 2 − ν + 1) ≠ 0 , (see Figure 1.25), we can


rewrite the restrictions as follows:
  E > 0
 
 E  y1 > 0
µ = 2 y > 0 ⇒  E < 0
 1 
  y < 0
 1

   y2 , y3 > 0
 E > 0 ⇒ 
λ + 2µ = Ey 2 > 0 ⇒   y2 , y3 < 0
 y3   y 2 > 0, y 3 < 0
  E <0⇒
   y 2 < 0, y 3 > 0
with which we obtain:
E > 0 ⇒ ν ⊂ ]− 1 ; 0.5[ ∪ ] 1 ; ∞ [

E < 0 ⇒ ν ⊂ ]− ∞ ; − 1[

y2 = (1 − ν ) ≠ 0 y (ν )

y1 = (1 + ν ) ≠ 0
zone not feasible

1 ν =1
ν = −1

E<0 E >0 E >0

ν
ν = 0 .5

y3 = (−2ν 2 − ν + 1) ≠ 0

Figure 1.25
ˆ ) -components in the principal space of Q (N
c) The inverse of the Q (N ˆ ) are given by:

1 
 0 0 
µ 0 0  µ 
1 1
Qij′ =  0 µ 0 
 inverse
→ Qij′ −1 =0 0  ∴ Q −1 =
 µ  µ (λ + 2µ )
2
 0 0 (λ + 2µ )  1 
0 0 
 (λ + 2µ ) 

University of Castilla-La Mancha Draft By: Eduardo W. V. Chaves (2016)


Ciudad Real - Spain
1 TENSORS 93

ˆ ) −1 are Q ′ −1 = Q ′ −1 = 1 1
Then, the eigenvalues of Q (N , Q3′ −1 = . Recall that a tensor
1 2
µ (λ + 2µ )
and its inverse share the same principal space, i.e. they are coaxial tensors. Moreover, we can
ˆ ) −1 as follows:
express the spectral representation of Q (N
3
Q −1 = ∑Q
a =1
−1 ˆ ( a )
a N
ˆ ( a ) = Q −1N
⊗N 1
ˆ (1) ⊗ N
ˆ (1) + Q −1N
2
ˆ (2) ⊗ N
ˆ ( 2 ) + Q −1N
3
ˆ ( 3) ⊗ N
ˆ (3)

ˆ (1) ⊗ N
= Q1−1 (N ˆ (1) + Nˆ ( 2) ⊗ Nˆ ( 2 ) ) + Q −1N
ˆ ( 3) ⊗ N
ˆ (3) = Q −1 (1 − N
ˆ ( 3) ⊗ N
ˆ (3) ) + Q −1N
ˆ ( 3) ⊗ N
ˆ ( 3)
3 1 3

= Q −1 (1 − N ˆ ⊗N ˆ ) + Q −1N ˆ ⊗N ˆ
1 3

ˆ ( 3) = N
where we have considered that N ˆ . It is interesting to see Problem 1.77. Then:

ˆ ⊗N
Q −1 = Q1−1 (1 − N ˆ ) + Q −1N ˆ = 1 (1 − N
ˆ ⊗N ˆ ⊗N
ˆ)+ 1 ˆ ⊗N
N ˆ
3
µ (λ + 2 µ )

=
1
1−
1 ˆ ˆ
N⊗N+
1 ˆ = 1 1 −  1 −
ˆ ⊗N
N
1 ˆ ˆ
N⊗N
µ µ (λ + 2 µ ) µ  µ (λ + 2 µ ) 

1  λ+µ  ˆ ˆ
= 1 −  N⊗N
µ  µ (λ + 2 µ ) 
ˆ ⋅ Ce ⋅N
Note that Q −1 = N (
ˆ −1 ≠ N
ˆ ⋅ C e −1 ⋅ N )
ˆ , where C e −1 = 1 I −

λ
2µ ( 2µ + 3λ )
(1 ⊗ 1) . We

ˆ ⋅C e −1
evaluate the tensor Qinv = N ⋅ Nˆ :
ˆ  1 1 (δ δ + δ δ ) − λ 
⇒ (Qinv ) jk = Nˆ i Cijkl
−1 ˆ
Nl = Ni ik jl il jk δ ijδ kl Nˆ l
 2µ 2 2 µ (2 µ + 3λ ) 
1 1 ˆ λ
⇒ (Qinv ) jk = (Niδ ikδ jlNˆ l + Nˆ iδ ilδ jk Nˆ l ) − Nˆ iδ ijδ klNˆ l
2µ 2 2 µ (2 µ + 3λ )
1 ˆ ˆ λ ˆ Nˆ = 1 δ  1 λ ˆ ˆ
⇒ (Qinv ) jk = (NkN j + Nˆ lNˆ lδ jk ) − N j k jk +  − N jNk
4µ 2 µ (2 µ + 3λ ) 4µ  4 µ 2 µ (2 µ + 3λ ) 
1  2µ + λ ˆ ˆ
⇒ (Qinv ) jk = δ jk +  N jNk
4µ  4 µ ( 2 µ + 3λ ) 
Thus:
1  2µ + λ ˆ ˆ
Qinv = 1 +   N ⊗ N
4µ  4µ (2µ + 3λ ) 
Note that µ ≠ 0 and (2µ + 3λ ) ≠ 0 , and moreover, these conditions are the same as those to
guarantee that ∃C −1 , (see Problem 1.92).

University of Castilla-La Mancha Draft By: Eduardo W. V. Chaves (2016)


Ciudad Real - Spain
94 SOLVING PROBLEMS BY MEANS OF CONTINUUM MECHANICS

1.12 Polar Decomposition

Problem 1.95
Let us consider that F has inverse ( det ( F ) ≠ 0 ) and that can be multiplicatively decomposed
as:
F = Q ⋅U = V ⋅Q
ˆ ( a ) , and V has the eigenvalues
If U has the eigenvalues λ a associated with the eigenvectors N
µ a associated with the eigenvectors nˆ ( a ) , show that:
µ a = λa
ˆ ( a ) and nˆ ( a ) .
Obtain also the relationship between the eigenvectors N
Solution:
By using the definition of F we can obtain the following relationship:
QT ⋅ F = QT ⋅ Q ⋅U = QT ⋅ V ⋅ Q ⇒ QT ⋅ F = U = QT ⋅ V ⋅ Q
and by considering the definition of eigenvalue-eigenvector of U we can obtain:
U⋅N ˆ (a ) = λ N
a
ˆ (a)

QT ⋅ V ⋅ Q ⋅ N
ˆ (a ) = λ N
a
ˆ (a) (the index here does not indicate summation)
Q ⋅ QT ⋅ V ⋅ Q ⋅ N
ˆ (a ) = λ Q ⋅ N
a
ˆ (a)
123
1

thus,
V ⋅ Q ⋅N
ˆ (a) = λ Q ⋅ N
a
ˆ (a)

V ⋅ nˆ ( a ) = λ a nˆ ( a )

where we have assumed that nˆ ( a ) = Q ⋅ N


ˆ ( a ) . Furthermore, by comparing the two definitions
of eigenvalue-eigenvector of the tensors U and V , we can verify that the they have the same
eigenvalues but different eigenvectors and they are related to each other by the orthogonal
transformation nˆ ( a ) = Q ⋅ N
ˆ (a) .

1.13 Spherical and Deviatoric Tensors


Problem 1.96
Let σ be a symmetric second-order tensor, and s ≡ σ dev be a deviatoric tensor. Prove that
∂s
s: = s . Also show that σ and σ dev are coaxial tensors.
∂σ
Solution: First, we make use of the definition of a deviatoric tensor:
Iσ Iσ
σ = σ sph + σ dev = σ sph + s = 1+s ⇒ s=σ − 1.
3 3
Afterwards we calculate:

University of Castilla-La Mancha Draft By: Eduardo W. V. Chaves (2016)


Ciudad Real - Spain
1 TENSORS 95

 I 
∂ σ − σ 1 
∂s 3  ∂[σ ] 1 ∂[ I σ ]
=  = − 1
∂σ ∂σ ∂σ 3 ∂σ
which in indicial notation is:
∂s ij ∂σij 1 ∂[ I σ ] 1
= − δ ij = δ ikδ jl − δ klδ ij
∂σ kl ∂σ kl 3 ∂σ kl 3
Therefore
∂s ij  1  1 1 ∂s
s ij = s ij δ ik δ jl − δ kl δ ij  = s ij δ ik δ jl − s ij δ kl δ ij = s kl − δ kl s ii = s kl ⇒ s: =s
∂σ kl  3  3 3 { ∂σ
=0

To show that two tensors are coaxial, we must prove that σ dev ⋅ σ = σ ⋅ σ dev :

σ ⋅ σ dev = σ ⋅ (σ − σ sph ) = σ ⋅ σ − σ ⋅ σ sph = σ ⋅ σ − σ ⋅ 1
3
I I  I 
= σ ⋅ σ − σ ⋅ σ 1 = σ ⋅ σ − σ 1 ⋅ σ =  σ − σ 1  ⋅ σ = σ dev ⋅ σ
3 3  3 

Therefore, we have shown that σ and σ dev are coaxial tensors. In other words, they have the
same principal directions (eigenvectors).

Problem 1.97
1 1
Consider that J = [det (b )] 2 = ( III b ) 2 , where b is a symmetric second-order tensor, i.e. b = b T .
Obtain the partial derivatives of J and ln(J ) with respect to b .
Solution:
 1

∂ ( III b ) 2 
∂J −1 ∂ III −1
  1( 1 1 1 1
= = III b ) 2 b
= ( III b ) 2 III b b −T = ( III b ) 2 b −1 = J b −1
∂b ∂b 2 ∂b 2 2 2
  1 

∂ Ln III b 2  
∂[Ln( J )]   1 ∂ III b 1 −1
⇒ =  = = b
∂b ∂b 2 III b ∂b 2

1.14 Voigt Notation

Problem 1.98
a) Write the equation σ = C : ε in Voigt notation, where C = λ1 ⊗ 1 + 2µ I is the isotropic
symmetric fourth-order tensor, and the tensors σ and ε are structured according to Voigt
notation as follows:

University of Castilla-La Mancha Draft By: Eduardo W. V. Chaves (2016)


Ciudad Real - Spain
96 SOLVING PROBLEMS BY MEANS OF CONTINUUM MECHANICS

 σ11   ε11 
σ  ε 
 22   22 
 σ33   ε33 
{σ } =   ; {ε } =  
 σ12   2ε12 
σ23   2ε 23 
   
 σ13   2ε13 

b) Write the equation ε = C −1 : σ in Voigt notation, where the tensor C −1 , (see Problem
1.92), is given by.
1 λ
C −1 = I− 1⊗1
2µ 2µ ( 2µ + 3λ )
Solution:
We write the equation σ = (λ1 ⊗ 1 + 2µ I ) : ε in indicial notation:
 1
σij = λδ ijδ kl + 2 µ (δ ikδ
2
jl

[
+ δ ilδ jk ) ε kl = λδ ijδ kl + µ (δ ikδ jl ]
+ δ ilδ jk ) ε kl
 
The second-order unit tensor in Voigt notation is:
1
1
1 0 0  
  1
δ ij = 0 1 0 →{δ} =  
Voigt

0 0 1 0


0
 
0
Then, the term (1 ⊗ 1)ij = δ ij δ kl in Voigt notation becomes:

1  1 1 1 0 0 0
1  1 1 1 0 0 0
  
Iijkl []1  1
= δ ijδ kl → I =  [1 1 1 0 0 0] = 
Voigt 1 1 0 0 0
 = {δ}{δ}
T

0  0 0 0 0 0 0
0  0 0 0 0 0 0
   
0 0 0 0 0 0 0
1
The symmetric fourth-order unit tensor Iijkl = (δ ikδ jl + δ ilδ jk ) in Voigt notation is:
2
 I1111 I1122 I1133 I1112 I1123 I1113  1 0 0 0 0 0
I  1 0 0 0 0 
 2211 I 2222 I 2233 I 2212 I 2223 I 2213  0
I 3311 I 3322 I 3333 I 3312 I 3323 I 3313  0 0 1 0 0 0
I ijkl →[I ] = 
Voigt = 
 I1211 I1222 I1233 I1212 I1223 I1213  0 0 0 12 0 0 
I 2311 I 2322 I 2333 I 2312 I 2323 I 2313  0 0 0 0 12 0 
   
 I1311 I1322 I1333 I1312 I1323 I1313  0 0 0 0 0 12 

With these, we can conclude that C = λ1 ⊗ 1 + 2µ I in Voigt notation becomes:

University of Castilla-La Mancha Draft By: Eduardo W. V. Chaves (2016)


Ciudad Real - Spain
1 TENSORS 97

1 1 1 0 0 0 1 0 0 0 0 0  λ + 2µ λ λ 0 0 0
1  0  
 1 1 0 0 0  1 0 0 0 0
 λ λ + 2µ λ 0 0 0 
1 1 1 0 0 0 0 0 1 0 0 0   λ λ λ + 2µ 0 0 0
[C ] = λ   + 2µ  = 
0 0 0 0 0 0 0
1
0 0 2 0 0  0 0 0 µ 0 0
0 0 0 0 0 0  0 1
0 0 0 2 0   0 0 0 0 µ 0
     
0 0 0 0 0 0 0
1
0 0 0 0 2   0 0 0 0 0 µ 
thus
 σ11  λ + 2 µ λ λ 0 0 0   ε11 
σ   λ λ + 2µ λ 0 0 0   ε 22 
 22  
 σ 33   λ λ λ + 2 µ 0 0 0   ε 33 
σ = (λ1 ⊗ 1 + 2 µI ) : ε Voigt
→  =  
 σ12   0 0 0 µ 0 0   2ε12 
σ 23   0 0 0 0 µ 0  2ε 23 
    
 σ13   0 0 0 0 0 µ   2ε13 
14444444444 4244444444444 3

{σ } = [C ]{ε }
b)
ε = C −1 : σ
 1 λ  1 λ
⇒ε= I− 1 ⊗ 1 : σ = I:σ − 1 ⊗1:σ
 2µ 2µ ( 2µ + 3λ )  2µ 2µ ( 2µ + 3λ )
1 λ
⇒ε= σ− Tr (σ )1
2µ 2µ ( 2µ + 3λ )
1 λ
⇒ ε ij = σ ij − σ kk δ ij
2µ 2µ ( 2µ + 3λ )
Note that:
1 λ  µ +λ  λ
ε11 = σ11 − (σ11 + σ 22 + σ 33 )δ 11 =  σ11 − (σ 22 + σ 33 )
2µ 2 µ ( 2 µ + 3λ )  µ ( 2 µ + 3λ )  2 µ ( 2 µ + 3λ )
1 λ  µ +λ  λ
ε 22 = σ 22 − (σ11 + σ 22 + σ 33 )δ 22 =  σ 22 − (σ11 + σ 33 )
2µ 2 µ ( 2 µ + 3λ )  µ ( 2 µ + 3λ )  2 µ ( 2 µ + 3λ )
1 λ  µ +λ  λ
ε 33 = σ 33 − (σ11 + σ 22 + σ 33 )δ 33 =  σ 33 − (σ11 + σ 22 )
2µ 2 µ ( 2 µ + 3λ )  µ ( 2 µ + 3λ )  2 µ ( 2 µ + 3λ )
1 λ 1 1
ε12 = σ12 − (σ kk )δ 12 = σ12 ⇒ 2ε12 = σ12
2µ 2 µ ( 2 µ + 3λ ) {
=0
2µ µ
1
2ε 23 = σ 23
µ
1
2ε13 = σ13
µ
Restructuring the above in Voigt notation we can obtain:

University of Castilla-La Mancha Draft By: Eduardo W. V. Chaves (2016)


Ciudad Real - Spain
98 SOLVING PROBLEMS BY MEANS OF CONTINUUM MECHANICS

 µ +λ −λ −λ 
 µ ( 2 µ + 3λ ) 0 0 0
2 µ ( 2 µ + 3λ ) 2 µ ( 2 µ + 3λ )
 
 −λ µ +λ −λ
ε11   0 0 0  σ11 

   2 µ ( 2 µ + 3λ ) µ ( 2 µ + 3λ ) 2 µ ( 2 µ + 3λ )
 σ 22 
ε 22   −λ −λ µ +λ
0 0 0   
ε 33   2 µ ( 2 µ + 3λ ) 2 µ ( 2 µ + 3λ ) µ ( 2 µ + 3λ )  σ 33 
 = 1  
2ε12   0 0 0 0 0  σ12 
2ε 23   µ  σ 23 
   1  
2ε13   0 0 0 0 0  σ13 
µ 
 1 
 0 0 0 0 0
 µ 
{ε } = [ C ] −1 {σ }

Problem 1.99
r
Let T ( x , t ) be a symmetric second-order tensor, which is expressed in terms of the position
r
( x ) and time (t ) . Also, bear in mind that the tensor components, along direction x3 , are
equal to zero, i.e. T13 = T23 = T33 = 0 .
r
NOTE: We define T ( x , t ) as a field tensor, i.e. the value of T depends on position and time.
r r
If the tensor is independent of any one direction at all points ( x ) , e.g. if T ( x , t ) is
independent of the x3 -direction, (see Figure 1.26), the problem becomes a two-dimensional
problem (plane state) so that the problem is greatly simplified.

2D
x2 T T12 
Tij =  11
 T12 T22 
x2
T22
T12 T22

T12 T12
T11 T11
T11
x1
T12
x1
T22
x3

Figure 1.26: A two-dimensional problem (2D).

a) Obtain T11′ , T22


′ , T12′ in the new reference system ( x1′ − x ′2 ) defined in Figure 1.27.
b) Obtain the value of θ so that θ corresponds to the principal direction of T , and also find
an equation for the principal values of T .
c) Evaluate the values of Tij′ , (i, j = 1,2) , when T11 = 1 , T22 = 2 , T12 = −4 and θ = 45º . Also,
obtain the principal values and principal directions.

University of Castilla-La Mancha Draft By: Eduardo W. V. Chaves (2016)


Ciudad Real - Spain
1 TENSORS 99

d) Draw a graph that shows the relationship between θ and components T11′ , T22
′ and T12′ ,
and in which the angle varies from 0º to 360º .
Use the Voigt Notation, and express the results in terms of 2θ .

x2
 a11 a12 0  cos θ sin θ 0
x 2′ x1′ a ij = a 21 a 22 0 =  − sin θ cos θ 0
 0 0 1  0 0 1

θ
x1

Figure 1.27: A two-dimensional problem (2D).


Solution:
a) Here we can apply the transformation law in Voigt notation {T ′} = [M] {T } , where
 T11′   T11 
T′  T 
 22   22 
T′  T 
{T ′} =  33′  ; {T } =  33 
 T12   T12 
 T23
′   T23 
   
 T13′   T13 
 a11 2 a12 2 a13 2 2a11 a12 2a12 a13 2a11 a13 
 2 2 2 
 a 21 a 22 a 23 2a 21 a 22 2a 22 a 23 2a 21 a 23 
 2
a 32 2 a 33 2 
[M] =  a 31 2a 31 a 32 2a 32 a 33 2a 31 a 33

 a 21 a11 a 22 a12 a13 a 23 (a11 a 22 + a12 a 21 ) (a13 a 22 + a12 a 23 ) (a13 a 21 + a11 a 23 )
a a a 32 a 22 a 33 a 23 (a 31 a 22 + a 32 a 21 ) (a 33 a 22 + a 32 a 23 ) (a 33 a 21 + a 31 a 23 )
 31 21
 a 31 a11 a 32 a12 a 33 a13 (a 31 a12 + a 32 a11 ) (a 33 a12 + a 32 a13 ) (a 33 a11 + a 31 a13 )
For the particular case shown in Figure 1.27, the transformation matrix [M] , after eliminate
the role and column associated with the x3 -direction, becomes:

 T11′   a11   T11 


2 2
a12 2a11 a12
T′  =  a 2 2  
 22   21 a 22 2a 21 a 22   T22  (1.100)
 T12′  a 21 a11 a 22 a12 a11 a 22 + a12 a 21   T12 
 
The transformation matrix, a ij , in the plane, can be evaluated in terms of a single parameter,
θ:
 a11 a12 a13   cos θ sin θ 0
a ij = a 21 a 22 a 23  =  − sin θ cos θ 0 (1.101)
 a 31 a 32 a 33   0 0 1

University of Castilla-La Mancha Draft By: Eduardo W. V. Chaves (2016)


Ciudad Real - Spain
100 SOLVING PROBLEMS BY MEANS OF CONTINUUM MECHANICS

T ′ = A T AT
x2

x 2′ ′
T22
T22 T12′ T11′

T12
T11 T11 x1′
P
P P
T11′ θ
T12 T12′ x1
x1 ′
T22
T22

T = AT T ′ A
Figure 1.28: Transformation law for (2D) tensor components.

By substituting the matrix components a ij given in (1.101) into (1.100) we obtain:

 T11′   cos θ sin 2 θ 2 cos θ sin θ   T11 


2

T′  =  
 22   sin θ
2
cos 2 θ − 2 sin θ cos θ   T22  (1.102)
 T12′   − sin θ cos θ cos θ sin θ cos 2 θ − sin 2 θ   T12 

Making use of the following
trigonometric identities, 2 cos θ sin θ = sin 2θ ,
1 − cos 2θ 1 + cos 2θ
cos 2 θ − sin 2 θ = cos 2θ , sin 2 θ = , cos2 θ = , the equation (1.102) becomes:
2 2
 1 + cos 2θ   1 − cos 2θ  
    sin 2θ 
 T11′   2   2    T11 
T′  =   1 − cos 2θ   1 + cos 2θ   
 22      − sin 2θ   T22 
2   2 
 T12′     T12 

 − sin 2θ   sin 2θ  
   cos 2θ
  2   2  
Explicitly, the above components are given by:
  1 + cos 2θ   1 − cos 2θ 
 T11′ =   T11 +   T22 + T12 sin 2θ
  2   2 
  1 − cos 2θ   1 + cos 2θ 
′ =
 T22  T11 +   T22 − T12 sin 2θ
  2   2 
  sin 2θ   sin 2θ 
 T12′ =  −  T11 +   T22 + T12 cos 2θ
  2   2 
Rearranging the previous equation we can obtain:

University of Castilla-La Mancha Draft By: Eduardo W. V. Chaves (2016)


Ciudad Real - Spain
1 TENSORS 101

  T11 + T22   T11 − T22 


 T11′ =  +  cos 2θ + T12 sin 2θ
  2   2 
  T + T22   T11 − T22 
′ =  11
 T22 −  cos 2θ − T12 sin 2θ (1.103)
  2   2 
  T − T22 
 T12′ = − 11  sin 2θ + T12 cos 2θ
  2 

b) Recalling that the principal directions are characterized by the lack of any tangential
components, i.e. Tij = 0 if i ≠ j , in order to find the principal directions for the plane case, we
let T12′ = 0 , hence:
 T − T22   T − T22 
T12′ = − 11  sin 2θ + T12 cos 2θ = 0 ⇒  11  sin 2θ = T12 cos 2θ
 2   2 
sin 2θ 2 T12 2 T12
⇒ = ⇒ tan(2θ ) =
cos 2θ T11 − T22 T11 − T22
Then, the angle corresponding to the principal direction is:

1  2 T12 
θ = arctan  (1.104)
2  T11 − T22 

To find the principal values (eigenvalues) we must solve the following characteristic equation:
T11 − T T12
T12 T22 − T
=0 ⇒ ( )
T 2 − T ( T11 + T22 ) + T11 T22 − T122 = 0

And by evaluating the quadratic equation we obtain:

T(1, 2 ) =
− [− ( T11 + T22 )] ± [− (T11 + T22 )]2 (
− 4(1) T11 T22 − T122 )
2(1)

=
T11 + T22
±
[(T11 + T22 )] 2
(
− 4 T11 T22 − T122 )
2 4
By rearranging the above equation we obtain the principal values for the two-dimensional case
as:

2
T11 + T22  T − T22  (1.105)
T(1, 2 ) = ±  11  + T122
2  2 

c) We directly apply equation (1.103) to evaluate the values of the components Tij′ , (i, j = 1,2) ,
where T11 = 1 , T22 = 2 , T12 = −4 and θ = 45º , i.e.:

University of Castilla-La Mancha Draft By: Eduardo W. V. Chaves (2016)


Ciudad Real - Spain
102 SOLVING PROBLEMS BY MEANS OF CONTINUUM MECHANICS

 1 + 2  1 − 2 
 T11′ =  2  +  2  cos 90º −4 sin 90º = −2.5
    
 1 + 2  1 − 2 
′ =
 T22 −  cos 90º +4 sin 90º = 5.5
  2   2 
 1 − 2 
 T12′ = −  sin 90º −4 cos 90º = 0.5
  2 
And the angle corresponding to the principal direction is:
1  2 T12  2 × (−4)
θ = arctan  = ⇒ (θ = 41.4375º )
2  T11 − T22  1− 2
r
The principal values of T ( x , t ) can be evaluated as follows:
2
T11 + T22  T − T22   T1 = 5.5311
T(1, 2 ) = ±  11  + T122 ⇒ 
2  2   T2 = −2.5311
d) By referring to equation in (1.103) and by varying θ from 0º to 360º , we can obtain
different values of T11′ , T22
′ , T12′ , which are illustrated in the following graph:

x1′
T1
θ = 41.437 º
T2

x1′
θ = 131.437º
8 σ2
Components

T1 = 5.5311

T22
6

T22 2
T12′
T11
0
0 50 100 150 200 250 300 350
θ
-2
45º x1′ T11′

T12 -4 T2 = −2.5311
θ = 86.437º

-6

TS max = 4.0311

Figure 1.29

University of Castilla-La Mancha Draft By: Eduardo W. V. Chaves (2016)


Ciudad Real - Spain
1 TENSORS 103

Problem 1.100
Obtain the principal values (eigenvalues) and the principal directions (eigenvectors) of the
symmetric part of T , whose components in the Cartesian system are given by:
5 1 
Tij =   (i, j = 1,2)
3 4
Solution:
The symmetric part of the tensor is given by:
5 2
Tijsym =
1
(Tij + T ji =  ) 
2  2 4
The principal values:
5−λ 2
=0 ⇒ λ2 − 9λ + 16 = 0
2 4−λ
The solution of the quadratic equation is given by:
− 9 ± (−9) 2 − 4 × (1) × (16) λ 1 ≡ T1 = 6.5615
λ (1, 2 ) = ⇒
2 ×1 λ 2 ≡ T2 = 2.4385
We can draw the Mohr circle (2D) of the tensor T sym :

TSsym

(T11sym , T12sym )


TII = 2.4385 TI = 6.5615 T Nsym

Figure 1.30

For the plane case, the principal direction can be obtained by means of the equation:
2 T12sym 2× 2
tan(2θ ) = = =4 ⇒ θ = 37.982º
T11sym − T22 sym
5−4

University of Castilla-La Mancha Draft By: Eduardo W. V. Chaves (2016)


Ciudad Real - Spain
104 SOLVING PROBLEMS BY MEANS OF CONTINUUM MECHANICS

1.15 Tensor Fields

Sir Isaac Newton Gottfried Wilhelm Leibniz


(1642 - 1727) (1646 - 1716)
Source: Wikipedia

Problem 1.101
Find the gradient of the function f ( x1 , x 2 ) = cos( x1 ) + exp x1x2 at the point ( x1 = 0, x 2 = 1) .
Solution: By definition, the gradient of a scalar function is given by:
∂f ˆ ∂f ˆ
∇ xr f = e1 + e2
∂x1 ∂x 2
∂f ∂f
where: = − sin( x1 ) + x 2 exp x1x2 ; = x1 exp x1x2
∂x1 ∂x 2

∇ xr f ( x1 , x 2 ) = [ − sin( x1 ) + x2 exp x1x2 ]eˆ 1 + [ x1exp x1x2 ]eˆ 2 ⇒ ∇ xr f (0,1) = [1]eˆ 1 + [0]eˆ 2 = 2eˆ 1

Problem 1.102
r
Let v and ϕ be, respectively, vector and scalar, and twice continuously differentiable, by
using indicial notation, show that:
r r
a) ∇ xr ⋅ (∇ xr ∧ v ) = 0
b) ∇ xr ⋅ (∇ xr ϕ ) = ∇ xr 2ϕ
c) ∇ xr (φµ ) = µ (∇ xrφ ) + φ (∇ xr µ )
r r r
d) ∇ xr ⋅ (φv ) = (∇ xrφ ) ⋅ v + φ (∇ xr ⋅ v )
e) ∇ xr ⋅ ( A ⋅ B) = (∇ xr A ) : B + A ⋅ (∇ xr ⋅ B) ( A and B are second-order tensors)
Solution:
a) Considering that
r r ∂ (•)
∇ xr ∧ v =  ijk v k , j ê i and ∇ xr ⋅ (•) = ⋅ ê l (1.106)
∂xl
then
r r ∂ ∂ ∂
∇ xr ⋅ (∇ xr ∧ v ) = ( ijk vk , j eˆ i ) ⋅ eˆ l = ( ijk vk , jδ il ) = ( ljk vk , j ) =  ljk vk , jl (1.107)
∂xl ∂xl ∂xl

University of Castilla-La Mancha Draft By: Eduardo W. V. Chaves (2016)


Ciudad Real - Spain
1 TENSORS 105

Note that  ljk is an antisymmetric tensor in lj and vk , jl is a symmetric tensor in lj , thus:

 ljk v k , jl = 0 (1.108)
b)
∂ ∂ ∂ϕ , j
∇ xr ⋅ (∇ xrϕ ) = (ϕ ,i eˆ i ) ⋅ eˆ j = (ϕ ,iδ ij ) = = ϕ , jj
∂x j ∂x j ∂x j
(1.109)
∂  ∂ϕ  ∂ 2ϕ
=  = = ∇ xr ϕ
2

∂x j  ∂x j  ∂x 2
  j

c)
[∇ xr (φµ )]i
= (φµ ) ,i = φ ,i µ + φµ ,i = µ [∇ xr φ ]i + φ [∇ xr µ ]i (1.110)
r
d) The result of ∇ xr ⋅ (φv ) is a scalar which can be expressed as follows:
r
∇ xr ⋅ (φv ) = (φv i ) ,i = φ ,i v i + φv i ,i
r r
= (∇ xr φ ) ⋅ v + φ (∇ xr ⋅ v )
e) Considering that ( A ⋅ B) ij = A ik B kj , [∇ xr ⋅ ( A ⋅ B)]i = ( A ⋅ B) ij , j = ( A ik B kj ) , j , thus
( A ik B kj ), j = A ik , jB kj + A ik B kj , j = [(∇ xr A ) : B ]i + [A ⋅ (∇ xr ⋅ B)]i

Problem 1.103
r r r r r r
Let a and b be vectors. Show that the following identity ∇ xr ⋅ (a + b) = ∇ xr ⋅ a + ∇ xr ⋅ b holds.
Solution:
r r ∂ r r
Observing that a = a j eˆ j , b = b k eˆ k , ∇ xr = ê i , we can express ∇ xr ⋅ (a + b) as follows:
∂x i

∂ (a j eˆ j + b k eˆ k ) ∂a j ∂b k ∂a ∂b r r
⋅ eˆ i = eˆ j ⋅ eˆ i + eˆ k ⋅ eˆ i = i + i = ∇ xr ⋅ a + ∇ xr ⋅ b
∂x i ∂x i ∂x i ∂x i ∂x i
Alternative solution: Working directly with indicial notation we obtain:
r r r r
∇ xr ⋅ (a + b) = (ai + b i ),i = ai ,i + b i ,i = ∇ xr ⋅ a + ∇ xr ⋅ b

Problem 1.104
r r
Find the components of (∇ xr a) ⋅ b .
r r ∂
Solution: Bearing in mind that a = a j eˆ j , b = b k eˆ k , ∇ xr = ê i ( i = 1,2,3 ), the following is
∂x i
true:
r r  ∂ (a j eˆ j )   ∂a j  ∂a j ∂a j
(∇ xr a) ⋅ b =  ⊗ eˆ i  ⋅ (b k eˆ k ) =  eˆ j ⊗ eˆ i  ⋅ (b k eˆ k ) = b k δ ik eˆ j = b k eˆ j
 ∂x i  ∂ x ∂ x ∂ x
   i  i k

Expanding the dummy index k , we obtain:


∂a j ∂a j ∂a j ∂a j
bk = b1 + b2 + b3
∂x k ∂x1 ∂x 2 ∂x 3

University of Castilla-La Mancha Draft By: Eduardo W. V. Chaves (2016)


Ciudad Real - Spain
106 SOLVING PROBLEMS BY MEANS OF CONTINUUM MECHANICS

Thus,
∂a1 ∂a ∂a
j =1 ⇒ b1 + b2 1 + b3 1
∂x1 ∂x 2 ∂x 3
∂a 2 ∂a ∂a
j = 2 ⇒ b1 + b2 2 + b3 2
∂x1 ∂x 2 ∂x 3
∂a 3 ∂a ∂a
j = 3 ⇒ b1 + b 2 3 + b3 3
∂x1 ∂x 2 ∂x 3

Problem 1.105
Prove that the following relationship is valid:
r
q 1 r 1 r
∇ xr ⋅   = ∇ xr ⋅ q − 2 q ⋅ ∇ xr T
T  T T
r r r
where q( x , t ) is an arbitrary vector field, and T ( x , t ) is a scalar field.
Solution:
r
 q  ∂  qi   qi  1 1 1 r 1 r
∇ xr ⋅   =   ≡   = q i ,i − 2 q i T,i = ∇ xr ⋅ q − 2 q ⋅ ∇ xr T (scalar)
 T  ∂x i  T   T  ,i T T T T

Problem 1.106
Show that:
r r r r r r
a) rot (λa) ≡ ∇ xr ∧ (λa) = λ(∇ xr ∧ a) + (∇ xr λ ∧ a) (1.111)
r r r r r r r r r r r
b) ∇ xr ∧ (a ∧ b) = (∇ xr ⋅ b)a − (∇ xr ⋅ a)b + (∇ xr a) ⋅ b − (∇ xr b) ⋅ a (1.112)
r r r r r
c) ∇ xr ∧ (∇ xr ∧ a) = ∇ xr (∇ xr ⋅ a) − ∇ 2xr a (1.113)

d) ∇ xr ⋅ (ψ∇ xrφ ) = ψ∇ 2xrφ + (∇ xrψ ) ⋅ (∇ xrφ ) (1.114)

Solution:
r r
a) The result of the algebraic operation ∇ xr ∧ (λa) is a vector, whose components are given
by:
r r
[∇ xr ∧ (λa)]i = ijk (λak ), j
= ijk (λ, j ak + λak , j )
= ijk λak , j ijk λ, j ak (1.115)
r r
= λ(∇ x ∧ a)i ijk (∇ x λ) j ak
r r
r r r
= λ(∇ xr ∧ a)i (∇ xr λ ∧ a)i
r r r r r r
with that we check the identity: rot (λa) = ∇ xr ∧ (λa) = λ(∇ xr ∧ a) + (∇ xr λ ∧ a) .
r r r r
The components of the vector product (a ∧ b) are given by (a ∧ b) k =  kij a i b j . Then:
r r r
[∇ xr ∧ (a ∧ b)] l = lpk ( kij aib j ), p =  kij lpk (ai , pb j + aib j , p ) (1.116)

University of Castilla-La Mancha Draft By: Eduardo W. V. Chaves (2016)


Ciudad Real - Spain
1 TENSORS 107

b) Considering that  kij =  ijk , the result of  ijk  lpk = δ il δ jp − δ ip δ jl and by substituting into
the above equation we can obtain:
r r r
[∇ xr ∧ (a ∧ b)] l =  kij lpk (ai , pb j + aib j , p )
= (δ ilδ jp − δ ipδ jl )(ai , pb j + aib j , p )
(1.117)
= δ ilδ jp ai , pb j − δ ipδ jl ai , pb j + δ ilδ jp aib j , p − δ ipδ jl aib j , p
= al , pb p − a p , pbl + alb p , p − a pbl , p
r r r r r r r r
Note that [(∇ xr a) ⋅ b] l = al , pb p , [(∇ xr ⋅ a)b]l = a p , pbl , [(∇ xr ⋅ b)a]l = al b p , p , [(∇ xr b) ⋅ a]l = a pbl , p .
r r r r
c) The components of (∇ xr ∧ a) are given by (∇ xr ∧ a) i =  ijk a k , j . Then:
123
ci
r r r
[∇ xr ∧ (∇ xr ∧ a)]q =  qli c i ,l
=  qli (ijk ak , j ),l (1.118)
=  qli ijk ak , jl

Considering that  qli  ijk =  qli  jki = δ qj δ lk − δ qk δ lj , the above equation becomes:
r r r
[∇ xr ∧ (∇ xr ∧ a)] q =  qli ijk ak , jl = (δ qjδ lk − δ qkδ lj )ak , jl = δ qjδ lk ak , jl − δ qkδ lj ak , jl = ak , kq − aq ,ll
r r
Note that [∇ xr (∇ xr ⋅ a)]q = a k , kq and [∇ 2xr a]q = aq ,ll .
d)
∇ xr ⋅ (φ∇ xrψ ) = (φψ ,i ),i = φψ ,ii + φ ,iψ ,i = φ∇ 2xrψ + (∇ xrφ ) ⋅ (∇ xrψ ) (1.119)
where φ and ψ are scalar functions.
Another interesting identity originating from the above equation is:
(1) ∇ xr ⋅ (φ∇ xrψ ) = φ∇ 2xrψ + (∇ xrφ ) ⋅ (∇ xrψ )
(1.120)
(2) ∇ xr ⋅ (ψ∇ xrφ ) = ψ∇ 2xrφ + (∇ xrψ ) ⋅ (∇ xrφ ) (1)
Subtracting the two previous identities, (1) − (2) , we can obtain:
∇ xr ⋅ (φ∇ xrψ ) − ∇ xr ⋅ (ψ∇ xrφ ) = φ∇ 2xrψ − ψ∇ 2xrφ
(1.121)
⇒ ∇ xr ⋅ (φ∇ xrψ − ψ∇ xrφ ) = φ∇ 2xrψ − ψ∇ 2xrφ

Problem 1.107
Let φ be a scalar field which is independent of x1 . Show that the following relationship is true
r r ∂φ − ∂φ
(∇ xr ∧ τ ) ⋅ eˆ 1 = −∇ 2xrφ , where the vector field τ is given by τ 1 = 0 , τ 2 = , and τ 3 = .
∂x3 ∂x2
Solution:
According to the problem statement we have that φ = φ ( x2 , x3 ) . Then, the Laplacian of φ
becomes:

University of Castilla-La Mancha Draft By: Eduardo W. V. Chaves (2016)


Ciudad Real - Spain
108 SOLVING PROBLEMS BY MEANS OF CONTINUUM MECHANICS

∂ 2φ ∂ 2φ ∂ 2φ ∂ 2φ ∂ 2φ
∇ 2xrφ = ∇ xr ⋅ (∇ xrφ ) = φ ,ii = φ ,11 + φ , 22 + φ ,33 = + + = +
∂x12 ∂x22 ∂x32 ∂x22 ∂x32
{
=0
r
Next we calculate the components of the vector (∇ xr ∧ τ ) :
ijk ê i

r ∂ ˆ ∂τ k
∇ xr ∧ τ = e j ∧ τ k eˆ k = ijk eˆ i = ijkτ k , j eˆ i
∂x j ∂x j
thus, the components are:
r
(∇ xr ∧ τ )i = ijkτ k , j = i12τ 2,1 + i13τ 3,1 + i 21τ 1, 2 + i 23τ 3, 2 + i 31τ 1,3 + i 32τ 2,3
(i = 1) ⇒ 123τ 3, 2 + 132τ 2,3 = τ 3, 2 − τ 2,3
r 
(∇ xr ∧ τ )i = (i = 1) ⇒  213τ 3,1 +  231τ 1,3 = τ 1,3 − τ 3,1

(i = 1) ⇒ 312τ 2,1 + 321τ 1, 2 = τ 2,1 − τ 1, 2
 ∂τ 3 ∂τ 2   ∂τ 3  − ∂φ  ∂τ 2  ∂φ     ∂ φ ∂ φ 
2 2

 −     −     −  2 + 2 
τ 3, 2 − τ 2,3   ∂x2 ∂x3   ∂x2  ∂x2  ∂x3  ∂x3     ∂x2 ∂x3 
r   ∂τ ∂τ    
(∇ xr ∧ τ )i = τ 1,3 − τ 3,1  =  1 − 3  =  0 = 0 
∂x ∂x1    
τ 2,1 − τ 1, 2   3 
  ∂ τ ∂τ    
 2
− 1  0 0
   
 ∂x1 ∂x2     
r  ∂ 2φ ∂ 2φ  r
Then, (∇ xr ∧ τ ) ⋅ eˆ 1 = − 2 + 2  . With that we show (∇ xr ∧ τ ) ⋅ eˆ 1 = −∇ 2xrφ .
 ∂x2 ∂x3 

Problem 1.108
r r r
Let φ be a scalar field, and u be a vector field. a) Show that ∇ xr ⋅ (∇ xr ∧ v ) = 0 and
r r
∇ xr ∧ (∇ xrφ ) = 0 .
r r r r r r r r r r r r r
b) Show that ∇ xr ∧ [(∇ xr ∧ v ) ∧ v ] = (∇ xr ⋅ v )(∇ xr ∧ v ) + [∇ xr (∇ xr ∧ v )] ⋅ v − (∇ xr v ) ⋅ (∇ xr ∧ v ) ;
r r r r r r r
c) Referring ω = ∇ xr ∧ v , show that ∇ xr ∧ (∇ 2xr v ) = ∇ 2xr (∇ xr ∧ v ) = ∇ 2xr ω .
Solution:
r r
Regarding that: ∇ xr ∧ v =  ijk v k , j ê i
r r ∂ ∂ ∂
∇ xr ⋅ (∇ xr ∧ v ) = ( ijk vk , j eˆ i ) ⋅ eˆ l =  ijk (vk , j ) δ il = ijk (vk , j ) = ijk vk , ji
∂xl ∂xl ∂xi
r
The second derivative of v is symmetrical with ij , i.e. v k , ji = v k ,ij , while  ijk is antisymmetric
with ij , i.e.,  ijk = − jik , thus:
 ijk v k , ji =  ij1v1, ji +  ij 2 v 2, ji +  ij 3 v3, ji = 0
Note that  ij1v1, ji = 0 since the double scalar product between a symmetric and an
antisymmetric tensor is zero.
Likewise, we can show that:

University of Castilla-La Mancha Draft By: Eduardo W. V. Chaves (2016)


Ciudad Real - Spain
1 TENSORS 109

r r
∇ xr ∧ (∇ xrφ ) = ijkφ ,kj eˆ i = 0i eˆ i = 0
r r r
b) Denoting by ω = ∇ xr ∧ v we obtain:
r r r r r r r
∇ xr ∧ [(∇ xr ∧ v ) ∧ v ] = ∇ xr ∧ ( ω ∧ v )
Observing the equation in (1.112), it holds that:
r r r r r r r r r r r
∇ xr ∧ (ω ∧ v ) = (∇ xr ⋅ v ) ω − (∇ xr ⋅ ω)v + (∇ xr ω) ⋅ v − (∇ xr v ) ⋅ ω
r r r
Note that ∇ xr ⋅ ω = ∇ xr ⋅ (∇ xr ∧ v ) = 0 . Then, we can draw the conclusion that:
r r r r r r r r r
∇ xr ∧ (ω ∧ v ) = (∇ xr ⋅ v )ω + (∇ xr ω) ⋅ v − (∇ xr v ) ⋅ ω
r r r r r r r r r
= (∇ xr ⋅ v )(∇ xr ∧ v ) + [∇ xr (∇ xr ∧ v )] ⋅ v − (∇ xr v ) ⋅ (∇ xr ∧ v )
c) Observing the equation in (1.113) we obtain:
r r r r r r r r
∇ 2xr v = ∇ xr (∇ xr ⋅ v ) − ∇ xr ∧ (∇ xr ∧ v ) = ∇ xr (∇ xr ⋅ v ) − ∇ xr ∧ ω
Applying the curl to the above equation we obtain:
r r r r r r r
∇ xr ∧ (∇ 2xr v ) = ∇ xr ∧ [∇ xr (∇ xr ⋅ v )] − ∇ xr ∧ (∇ xr ∧ ω)
144 42r 444 3
=0
r r r
Referring once again to the equation in (1.113) to express the term ∇ xr ∧ (∇ xr ∧ ω) :
r r r r r r r r r r
∇ xr ∧ (∇ 2xr v ) = −∇ xr ∧ (∇ xr ∧ ω) = −∇ xr (∇ xr ⋅ ω ) + ∇ 2xr ω = −∇ xr [∇ xr ⋅ (∇ xr ∧ v )] + ∇ 2xr ω
144244 3
=0
r r
= ∇ 2xr (∇ xr ∧ v )

Problem 1.109
Show that:
r r r r r r r r r r r r
a) ∇ xr ⋅ (a ∧ b) = (∇ xr ∧ a) ⋅ b + a ⋅ (∇ xr ∧ b) ≡ rot (a) ⋅ b + a ⋅ rot (b) (1.122)

Solution:
r r
The expression ∇ xr ⋅ (a ∧ b) is a scalar which can be expressed as follows:
r r r r r r r r
∇ xr ⋅ (a ∧ b) = ( ijk a j b k ) ,i =  ijk a j ,i b k +  ijk b k ,i a j = (∇ xr ∧ a) ⋅ b + a ⋅ (∇ xr ∧ b)
1r 2r3 1 23
r r
(∇ ∧ a) k (∇ ∧b ) j

Problem 1.110
a) Let T be an arbitrary second-order tensor, obtain the symbolic notation in Cartesian basis
r r r r
for: a.1) (∇ xr ∧ T ) , a.2) (∇ xr ∧ T )T , a.3) (∇ xr ∧ T T ) , and a.4) (∇ xr ∧ T T )T . a.5) Considering that
r
c is a constant vector, show that:
r r r r r r
∇ xr ∧ ( T ⋅ c ) = (∇ xr ∧ T ) ⋅ c = c ⋅ [∇ xr ∧ T ]T
r r
b) Obtain the symbolic notation of ∇ xr ∧ (∇ xr ∧ ε )T .

University of Castilla-La Mancha Draft By: Eduardo W. V. Chaves (2016)


Ciudad Real - Spain
110 SOLVING PROBLEMS BY MEANS OF CONTINUUM MECHANICS

r
∂u r r
c) Consider the second-order tensor F = r + 1 , prove that c.1) ∇ xr ∧ (∇ xr ∧ F )T = 0 and
∂x
r T
r
∇ x ∧ F = 0 ; c.2) Obtain the explicit components of ∇ xr ∧ F .
r

Solution:
r ∂ ˆ ∂Tqj
a.1) (∇ xr ∧ T ) = e p ∧ Tqj (eˆ q ⊗ eˆ j ) = eˆ p ∧ eˆ q ⊗ eˆ j = Tqj , p  ipq eˆ i ⊗ eˆ j = ipq Tqj , p eˆ i ⊗ eˆ j
∂x p ∂x p
r
a.2) (∇ xr ∧ T )T = ipq Tqj , p eˆ j ⊗ eˆ i =  jpq Tqi , p eˆ i ⊗ eˆ j
r ∂ ˆ ∂T jq
a.3) (∇ xr ∧ T T ) = e p ∧ T jq (eˆ q ⊗ eˆ j ) = eˆ p ∧ eˆ q ⊗ eˆ j = ipq T jq , p eˆ i ⊗ eˆ j
∂x p ∂x p
r
a.4) (∇ xr ∧ T T )T =  jpq Tiq , p eˆ i ⊗ eˆ j

where we have considered the definition eˆ j ∧ eˆ k =  ijk eˆ i .


r r
a.5) Let us considere that a = T ⋅ c = ( Tqj c j )eˆ q = a q eˆ q , thus:
r r r r ∂ ˆ ∂a q ∂a
∇ xr ∧ ( T ⋅ c) = ∇ xr ∧ a = e p ∧ a q eˆ q =  ipq eˆ i =  ipq q eˆ i =  ipq a q , p eˆ i
∂x p ∂x p ∂x p
⇒  ipq a q , p eˆ i =  ipq ( Tqj c j ) , p eˆ i =  ipq Tqj , p c j eˆ i +  ipq Tqj c j , p eˆ i =  ipq Tqj , p c j eˆ i
{
=0 jp

r ∂c j
where we have considered that c is constant, i.e. c j , p = = 0 jp .
∂x p
r
Note that  ipq Tqj , p are the components of (∇ xr ∧ T )ij , (see (a.1)), thus
r r r r r r r
∇ xr ∧ ( T ⋅ c) =  ipq Tqj , p c j eˆ i = (∇ xr ∧ T ) ij c j eˆ i = [(∇ xr ∧ T ) ⋅ c]i eˆ i = [c ⋅ (∇ xr ∧ T )T ]i eˆ i
r r
(∇ xr ∧ T ) ⋅ c = ipq Tqj , p eˆ i ⊗ eˆ j ⋅ c k eˆ k =  ipq Tqj , p c k eˆ iδ jk =  ipq Tqj , p c j eˆ i
r
b) We have already shown that (∇ xr ∧ ε ) = ipq εqj , p eˆ i ⊗ eˆ j , thus
r r ∂ ˆ ∂ε qj , p
∇ xr ∧ (∇ xr ∧ ε )T = e s ∧ ( ipq ε qj , p eˆ j ⊗ eˆ i ) =  ipq eˆ s ∧ (eˆ j ⊗ eˆ i ) =  ipq  tsj ε qj , ps eˆ t ⊗ eˆ i
∂x s ∂xs
= (− iqp )(− tjs ) ε qj , ps eˆ t ⊗ eˆ i =  iqp  tjs ε qj , ps eˆ t ⊗ eˆ i =  qpi  jst ε qj , ps eˆ t ⊗ eˆ i

Note that:
r r ∂ ˆ ∂ε qj , p
∇ xr ∧ (∇ xr ∧ ε ) = e s ∧ ( ipq ε qj , p eˆ i ⊗ eˆ j ) =  ipq eˆ s ∧ (eˆ i ⊗ eˆ j ) =  ipq ε qj , ps  tsi eˆ t ⊗ eˆ j
∂x s ∂xs
=  its  ipq ε qj , ps eˆ t ⊗ eˆ j = (δ tpδ sq − δ tqδ sp ) ε qj , ps eˆ t ⊗ eˆ j
= (δ tpδ sq ε qj , ps − δ tqδ sp ε qj , ps )eˆ t ⊗ eˆ j = ( ε sj ,ts − εtj , ss )eˆ t ⊗ eˆ j
r r r
r r  ∂ u  r  ∂u  r r  ∂ u  r
c.1) Note that ∇ xr ∧ F = ∇ xr ∧  r + 1  = ∇ xr ∧  r  + ∇ xr ∧ (1) = ∇ xr ∧  r  = ∇ xr ∧ J , where
 ∂x   ∂x   ∂x 
r
∂u ∂u r r
we have denoted by J = r . Taking into account εqj = J qj = q = uq , j into ∇ xr ∧ (∇ xr ∧ ε )T
∂x ∂x j
we can obtain:

University of Castilla-La Mancha Draft By: Eduardo W. V. Chaves (2016)


Ciudad Real - Spain
1 TENSORS 111

r r
∇ xr ∧ (∇ xr ∧ J )T = iqp  tjs J qj , ps eˆ t ⊗ eˆ i = iqp  tjsuq , jps eˆ t ⊗ eˆ i

Note that uq , jps = uq , pjs = uq , psj , i.e. it is symmetric in js , and the tensor tjs = − tsj is
r r
antisymmetric in js , so tjsuq , jps = 0tqp , and ∇ xr ∧ (∇ xr ∧ F ) T = 0 ti eˆ t ⊗ eˆ i = 0 .
Alternative solution:
Taking into account that
δ it δ ij δ is
iqp tjs = δ qt δ qj δ qs = δ itδ qjδ ps + δ ijδ qsδ pt + δ isδ pjδ qt − δ isδ qjδ pt − δ qsδ pjδ it − δ psδ qtδ ij
δ pt δ pj δ ps
then
 iqp  tjs Fqj , ps = (δ itδ qjδ ps + δ ijδ qsδ pt + δ isδ pjδ qt − δ isδ qjδ pt − δ qsδ pjδ it − δ psδ qtδ ij ) u q , jps
= δ itδ qjδ ps u q , jps + δ ijδ qsδ pt u q , jps + δ isδ pjδ qt u q , jps − δ isδ qjδ pt u q , jps − δ qsδ pjδ it u q , jps − δ psδ qtδ ij u q , jps
= δ it u j , jss + u s ,its + ut , ppi − u j , jti − δ it u s , pps − ut ,ipp = 0ti

Note that δ it u j , jss = δ it u p , pss = δ it u p ,ssp = δ it us , pps , u s ,its = u j ,itj = u j , jti , ut , ppi = ut ,ipp .
r
We express ∇ xr ∧ J T in indicial notation:
r ∂ ˆ ∂J qj
∇ xr ∧ J T = e p ∧ J qj (eˆ j ⊗ eˆ q ) = eˆ p ∧ eˆ j ⊗ eˆ q = J qj , p ipj eˆ i ⊗ eˆ q
∂x p ∂x p
=  ipj J qj , p eˆ i ⊗ eˆ q = ipj uq , jp eˆ i ⊗ eˆ q = 0 ip eˆ i ⊗ eˆ q

Note that uq , jp = uq , pj is symmetric in jp meanwhile ipj = − ijp is antisymmetric in jp .


r
c.2) We express ∇ xr ∧ J in indicial notation, (see item (a.1)):
r
∇ xr ∧ J =  ipq J qj , p eˆ i ⊗ eˆ j =  ipq u q , jp eˆ i ⊗ eˆ j

Expanding the term ipquq , jp we can obtain:


ipquq, jp =  ip1u1, jp + ip 2u2, jp +  ip 3u3, jp
1
424
3 1424 3 1424 3
i11u1, j1 +  i12u2, j1 + i13u3, j1
+ + +
i 21u1, j 2 +  i 22u2, j 2 + i 23u3, j 2
+ + +
i 31u1, j 3 + i 32u2, j 3 + ip 3u3, j 3

thus,
u3,12 − u2,13 u3, 22 − u2, 23 u3,32 − u2,33   J 31, 2 − J 21,3 J 32, 2 − J 22,3 J 33, 2 − J 23,3 
r    
(∇ xr ∧ J )ij =  u1,13 − u3,11 u1, 23 − u3, 21 u1,33 − u3,31  =  J 11,3 − J 31,1 J 12,3 − J 32,1 J 13,3 − J 33,1 
 u2,11 − u1,12 u2, 21 − u1, 22 u2,31 − u1,32   J 21,1 − J 11, 2 J 22,1 − J 12, 2 J 23,1 − J 13, 2 
  
Note that

University of Castilla-La Mancha Draft By: Eduardo W. V. Chaves (2016)


Ciudad Real - Spain
112 SOLVING PROBLEMS BY MEANS OF CONTINUUM MECHANICS

 J 13, 2 − J 12,3 J 23, 2 − J 22,3 J 33, 2 − J 32,3  u1,32 − u1, 23 u2,32 − u2, 23 u3,32 − u3, 23 
r    
(∇ xr ∧ J )ij =  J 11,3 − J 13,1
T
J 21,3 − J 23,1 J 31,3 − J 33,1  =  u1,13 − u1,31 u2,13 − u2,31 u3,13 − u3,31 
 J 12,1 − J 11, 2 J 22,1 − J 21, 2 J 32,1 − J 31, 2  u1, 21 − u1,12 u2, 21 − u2,12 u3, 21 − u3,12 

= 0ij

Note that, if
1 r r 1r r 1r r
ε= (J + J T ) ⇒ ∇ xr ∧ (∇ xr ∧ ε )T = ∇ xr ∧ (∇ xr ∧ J )T + ∇ xr ∧ ( ∇ xr ∧ J T )T = 0
2 2 1442443 2 1424 3
=0 =0
r
∂u
where J = r .
∂x
Problem 1.111
r r
Let a and v be vectors, show that
r r r r r r
(∇ xr ∧ v ) ∧ a = [∇ xr v − (∇ xr v )T ] ⋅ a
Solution:
r r r r r
If we consider (∇ xr ∧ v ) i = ijk vk , j , then [(∇ xr ∧ v ) ∧ a]s =  sip  ijk vk , j a p . Note also that the
relationship  sip ijk =  psi  jki = δ pjδ sk − δ pkδ sj holds, then
r r r
[(∇ xr ∧ v ) ∧ a]s =  sip  ijk vk , j a p = (δ pjδ sk − δ pkδ sj )vk , j a p = (δ pjδ sk vk , j − δ pkδ sj vk , j )a p
{ r r
= (v s , p − v p , s )a p = [∇ xr v − (∇ xr v )T ] ⋅ a
r
} s

Alternative solution:
r r r r
If we denote by l = ∇ xr v , then [∇ xr v − (∇ xr v )T ] = 2(∇ xr v ) skew = 2 l skew . Note that the axial
r r r
vector associated with the antisymmetric tensor (∇ xr v ) skew = (v ⊗ ∇ xr ) skew is the vector
r 1 r r
ϕ = (∇ xr ∧ v ) , (see Problem 1.37). If we recall the property of an antisymmetric tensor
2
r r r r
(∇ xr v ) skew ⋅ a = ϕ ∧ a , we can conclude that
r r r r 1 rr r r 1 r r r
(∇ xr v ) skew ⋅ a = ϕ ∧ a ⇒ [∇ x v − (∇ xr v )T ] ⋅ a = (∇ xr ∧ v ) ∧ a
2 2
r rT r r r r
⇒ [∇ x v − (∇ x v ) ] ⋅ a = (∇ x ∧ v ) ∧ a
r r r

Problem 1.112
r r r r r r
Let u = u( x ) be a vector field. By means of components of u , a) show that ∇ 2xr u = ∇ xr (∇ xr ⋅ u)
r r r r r r r r r r
when ∇ xr ∧ (∇ xr ∧ u) = 0 , b) show that ∇ 2xr u = −∇ xr ∧ (∇ xr ∧ u) when ∇ xr (∇ xr ⋅ u) = 0 .
Solution:
We have proven in Problem 1.106 that the following is true:
r r r r r
∇ xr ∧ (∇ xr ∧ a) = ∇ xr (∇ xr ⋅ a) − ∇ 2xr a indicial
 → ilq qjk ak , jl = a j , ji − ai , jj
Then, we can obtain
r r r r r r
∇ xr ⋅ (∇ xr u) ≡ ∇ 2xr u = ∇ xr (∇ xr ⋅ u) − ∇ xr ∧ (∇ xr ∧ u) indicial
 → ui , jj = u j , ji − ilq  qjk uk , jl

Then, it is easy to verify that:

University of Castilla-La Mancha Draft By: Eduardo W. V. Chaves (2016)


Ciudad Real - Spain
1 TENSORS 113

r r r r r r r r
a) ∇ xr ⋅ (∇ xr u) ≡ ∇ 2xr u = ∇ xr (∇ xr ⋅ u) − ∇ xr ∧ (∇ xr ∧ u) ⇒ ∇ 2xr u = ∇ xr (∇ xr ⋅ u)
1442r 44 3
=0

Components:
ui , jj = u j , ji ⇒ ui ,11 + ui , 22 + ui ,33 = u1,1i + u 2, 2i + u3,3i
 u1,11 + u1, 22 + u1,33 = u1,11 + u 2, 21 + u3,31 u1, 22 + u1,33 = u 2, 21 + u3,31
  (1.123)
⇒ u 2,11 + u 2, 22 + u2,33 = u1,12 + u2, 22 + u3,32 ⇒ u 2,11 + u 2,33 = u1,12 + u3,32
u + u + u = u + u + u u + u
 3,11 3, 22 3,33 1,13 2 , 23 3,33  3,11 3, 22 = u1,13 + u 2 , 23

Note that in the Cartesian basis we have:


r
u = u i eˆ i = u1 eˆ 1 + u 2 eˆ 2 + u 3 eˆ 3
r r r r  ∂u ∂u   ∂u ∂u   ∂u ∂u 
(∇ xr ∧ u) ≡ rot (u) = (rot (u) )i eˆ i =  3 − 2 eˆ 1 +  1 − 3 eˆ 2 +  2 − 1 eˆ 3
1∂4
x 2 ∂x 3 
42r44 3 1∂4
x 3 ∂x1 
42r443 1∂4
x1 ∂x 2 
42r443
= (rot (u ) )1 = (rot (u) )2 = (rot (u) )3
r r r r r r
r r r  ∂ (rot (u) )3 ∂ (rot (u) )2   ∂ (rot (u) )1 ∂ (rot (u) )3   ∂ (rot (u) )2 ∂ (rot (u) )1 

∇ x ∧ (∇ x ∧ u) = 
r r −  ˆ
e1 +   −  ˆ 
e 2 +  − eˆ 3
 ∂x2 ∂x3   ∂x3 ∂x1   ∂x1 ∂x2 
r r
 ∂ (rot (u) )3 ∂ (rot (u) )2   ∂  ∂u2 − ∂u1  − ∂  ∂u1 − ∂u3 
 −   ∂x  ∂x ∂x  ∂x  ∂x ∂x1 

 ∂x ∂ x  2  2  3
r 
2 3 1 3

 ∂ (rot (u) )1 ∂ (rot (u) )3   ∂  ∂u3 ∂u2  ∂  ∂u2 ∂u1 


r
r r r
[∇ xr ∧ (∇ xr ∧ u)] i =  − =  −  −  − 
∂ x ∂ x ∂x  ∂x ∂x  ∂x  ∂x ∂x 
 r  
3 1 3 2 3 1 1 2
 ∂ (rot (ur
) )2 ∂ (rot (u) )1 
 −   ∂  ∂u1 − ∂u3  − ∂  ∂u3 − ∂u2 
 ∂x1 ∂x2   ∂x  ∂x   
 1  3 ∂x1  ∂x2  ∂x2 ∂x3 
u2,12 − u1, 22 − u1,33 + u3,13 
 
= u3, 23 − u2,33 − u2,11 + u1, 21 
u − u − u + u 
 1,31 3,11 3, 22 2, 32 
r r r r
If we are considering that ∇ xr ∧ (∇ xr ∧ u) = 0 then:
u2,12 − u1, 22 − u1,33 + u3,13  0 u2,12 + u3,13 = u1, 22 + u1,33
r r r     
[∇ xr ∧ (∇ xr ∧ u)] i = u3, 23 − u2,33 − u2,11 + u1, 21  = 0 ⇒ u3, 23 + u1, 21 = u2,33 + u2,11
u − u − u + u  0 u + u = u + u
 1,31 3,11 3, 22 2 ,32     1,31 2,32 3,11 3, 22

which are the same conditions as those presented in equation (1.123).


r r r r r r r r r r
b) ∇ xr ⋅ (∇ xru) ≡ ∇ 2xr u = ∇ xr (∇ xr ⋅ u) − ∇ xr ∧ (∇ xr ∧ u) ⇒ ∇ 2xr u = −∇ xr ∧ (∇ xr ∧ u)
142r 43
=0

Components
u1,11 + u1, 22 + u1,33 = −(u 2,12 − u1, 22 − u1,33 + u3,13 )

ui , jj = − ilq  qjk u k , jl ⇒ u 2,11 + u2, 22 + u2,33 = −(u3, 23 − u2,33 − u 2,11 + u1, 21 ) (1.124)
u + u + u = −(u − u − u + u )
 3,11 3, 22 3, 33 1,31 3,11 3, 22 2 , 32

r r
And if we consider ∇ xr (∇ xr ⋅ u) = 0 we can obtain:

University of Castilla-La Mancha Draft By: Eduardo W. V. Chaves (2016)


Ciudad Real - Spain
114 SOLVING PROBLEMS BY MEANS OF CONTINUUM MECHANICS

u1,11 + u2, 21 + u3,31 = 0 u3,31 + u2, 21 = −u1,11


r  
[∇ xr (∇ xr ⋅ u)]i = u1,1i + u2, 2i + u3,3i = 0i ⇒ u1,12 + u2, 22 + u3,32 = 0 ⇒ u1,12 + u3,32 = −u2, 22
u + u + u = 0 u + u = −u
 1,13 2, 23 3,33  1,13 2, 23 3,33

If we replace the above equations into (1.124) we prove that the equality holds.

Problem 1.113
r
Let σ be a second-order tensor field, and a be a vector field. Show the identities:
r r r
a) ∇ xr ⋅ (a ∧ σ ) =  : [(∇ xr a) ⋅ σ T ] + a ∧ (∇ xr ⋅ σ ) (1.125)
r r r r r
b) ∇ xr ⋅ (σ ∧ a) = a ⋅[∇ xr ∧ σ T ] − σ ⋅ [∇ xr ∧ a] (1.126)

where  is the Levi-Civita tensor (third-order tensor).


Solution:
a)
r
a ∧ σ = ai eˆ i ∧ σ jk eˆ j ⊗ eˆ k = σ jk ai  pij eˆ p ⊗ eˆ k
r ∂ ∂ ∂
⇒ ∇ xr ⋅ (a ∧ σ ) = (σ jk ai  pij eˆ p ⊗ eˆ k ) ⋅ eˆ q = (σ jk ai  pij eˆ p )δ kq = (σ jk ai  pij eˆ p )
∂xq ∂xq ∂xk
r
⇒ ∇ xr ⋅ (a ∧ σ ) = (σ jk ai  pij ),k eˆ p = ( pij σ jk ,k ai +  pij σ jk ai ,k )eˆ p
r r
Note that  pij σ jk ,k ai =  pij (∇ xr ⋅ σ ) j ai =  pij (a)i (∇ xr ⋅ σ ) j = [a ∧ (∇ xr ⋅ σ )] p

and  pij σ jk ai ,k =  pij σ jk (∇ xr a)ik =  pij (∇ xr a)ik σ jk =  pij [(∇ xr a) ⋅ σ T ]ij = { : [(∇ xr a) ⋅ σ T ]}p
r r r r

with that we show the equation in (1.125).


r r
Note that when a = x the equation (1.125) becomes:
r r r r r
∇ xr ⋅ ( x ∧ σ ) =  : [(∇ xr x ) ⋅ σ T ] + x ∧ (∇ xr ⋅ σ ) =  : [1 ⋅ σ T ] + x ∧ (∇ xr ⋅ σ ) =  : σ T + x ∧ (∇ xr ⋅ σ )
b)
r
σ ∧ a = σ jk eˆ j ⊗ eˆ k ∧ ai eˆ i = σ jk ai  pki eˆ j ⊗ eˆ p
r ∂ ∂ ∂
⇒ ∇ xr ⋅ (σ ∧ a) = (σ jk ai  pki eˆ j ⊗ eˆ p ) ⋅ eˆ q = (σ jk ai  pki eˆ j )δ pq = (σ jk ai  pki eˆ j )
∂xq ∂xq ∂x p
r
⇒ ∇ xr ⋅ (σ ∧ a) = (σ jk ai  pki ), p eˆ j = ( pki σ jk , p ai +  pki σ jk ai , p )eˆ j

Note that
 pki σ jk , p ai = ipk σ jk , p ai = [∇ xr ∧ σ T ] ij ai = {a ⋅[∇ xr ∧ σ T ]}j
r r r

 pki σ jk ai , p =  kip ai , p σ jk = − kpi ai , p σ jk = −(∇ xr ∧ a) k σ jk = −{σ ⋅ [∇ xr ∧ a]}j


r r r r

Problem 1.114
r r r r
Consider that ∇ xr ⋅ σ + p = q , where σ is a second-order tensor field, and p and q are vector
r r
fields. The equation ∇ xr ⋅ σ + p = q fulfills at any point of the volume V which is delimitated
by surface S . Show that, if the following equation:

University of Castilla-La Mancha Draft By: Eduardo W. V. Chaves (2016)


Ciudad Real - Spain
1 TENSORS 115

r r r r r r

V

x ∧ pdV + x ∧ t * dS = x ∧ qdV
S

V
r
is also valid, then σ = σ T holds. Consider that t * = σ ⋅ nˆ where n̂ is the outward pointing
unit normal to surface S .
Solution:
r r r r r r

V

x ∧ pdV + x ∧ t * dS = x ∧ qdV
S

V
r r r r r
⇒ x ∧ pdV + x ∧ (σ ⋅ nˆ )dS = x ∧ qdV
∫ ∫ ∫
V S V
r r r r
Note that ( x ∧ t * ) i =  ijk x j t *k =  ijk x j (σ ⋅ nˆ ) k =  ijk x j σ kp nˆ p = ( x ∧ σ ) ip nˆ p = ( x ∧ σ ) ⋅ nˆ , with
that we can obtain:
r r r r r
⇒ x ∧ pdV + ( x ∧ σ ) ⋅ nˆ dS = x ∧ qdV
∫ ∫ ∫
V S V

By applying the divergence theorem to the surface integral we can obtain:


r r r r r
⇒ x ∧ pdV + ∇ xr ⋅ ( x ∧ σ )dV = x ∧ qdV
∫ ∫ ∫
V V V
r r
It was proven in Problem 1.113 that ∇ xr ⋅ ( x ∧ σ ) =  : σ T + x ∧ (∇ xr ⋅ σ ) , and by replacing it
into the above equation we can obtain:
r r r r r
⇒ x ∧ pdV + [ : σ T + x ∧ (∇ xr ⋅ σ )]dV = x ∧ qdV
∫ ∫ ∫
V V V
r r r r r r
⇒ [ x ∧ p +  : σ T + x ∧ (∇ xr ⋅ σ ) − x ∧ q]dV = 0

V
r r r r
⇒ { x ∧ [(∇ xr ⋅ σ ) + p − q] +  : σ T }dV = 0
∫ 144 42r 444 3
V =0
r

T
⇒  : σ dV = 0
V
r
If ( : σ T ) if valid for the whole volume it is also valid locally, so  : σ T = 0 . Note that
( : σ T ) i =  ijk σ kj = − ikj σ kj = 0i , i.e. the tensor  is antisymmetric in kj , since the double
scalar product between a symmetric and an antisymmetric tensor is zero, then we prove that
σ is symmetric, i.e. σ T = σ . We can also prove that by means of components:
1 jk σ kj = 0 ⇒ 123 σ 32 132 σ 23 = 0 ⇒ σ 32 σ 23 = 0 ⇒ σ 32 σ 23

 2 jk σ kj = 0 ⇒  213 σ 31  231σ13 = 0 ⇒ − σ 31 σ13 = 0 ⇒ σ 31 σ13

 3 jk σ kj = 0 ⇒  312 σ 21  321σ12 = 0 ⇒ σ 21 σ12 = 0 ⇒ σ 21 σ12

Problem 1.115
a) Show that
r r r r r r r
∇ xr ∧ {[∇ xr ∧ ε ]T ∧ x} = {∇ xr ∧ [∇ xr ∧ ε ]T } ∧ x − (⋅ : (∇ xr ε )) 1 + [∇ xr ∧ ε ]
r
where ε is a second-order tensor, x is the vector position,  is the Levi-Civita tensor (third-
order tensor), and 1 is the second-order unit tensor. b) Simplify the above equation by
considering that ε = ε T is symmetric second-order tensor, i.e. ε = ε T .

University of Castilla-La Mancha Draft By: Eduardo W. V. Chaves (2016)


Ciudad Real - Spain
116 SOLVING PROBLEMS BY MEANS OF CONTINUUM MECHANICS

Solution:
r r  
T

r ∂  ∂ 
T
∇ xr ∧ {[∇ xr ∧ ε ] ∧ x} = eˆ i ∧  eˆ j ∧ ε pq eˆ p ⊗ eˆ q  ∧ x k eˆ k 
∂xi ∂
 j x  
 ∂ε 
T

∂ ˆ  pq
ˆ ˆ ˆ 
= e i ∧   tjp e t ⊗ e q  ∧ x k e k 
∂xi  ∂x j  

Applying the transpose property we can obtain:


r r r ∂ ˆ  ∂ε pq   ∂  ∂ε pq 
∇ xr ∧ {[∇ xr ∧ ε ]T ∧ x} = ei ∧  tjp eˆ q ⊗ eˆ t  ∧ xk eˆ k  = eˆ i ∧  xk tjp  stk eˆ q ⊗ eˆ s 
∂xi  ∂x j   ∂xi  ∂x j 

∂  ∂ε pq  ∂  ∂ε pq  ˆ
= xk tjp  stk  niq eˆ n ⊗ eˆ s =  niqtjp  stk xk e n ⊗ eˆ s
∂xi  ∂x j 
 ∂xi  ∂x j 

 ∂  ∂ε pq  ∂ε 
=  niq tjp  stk    xk + ∂xk pq eˆ n ⊗ eˆ s
 ∂xi  ∂x j  ∂xi ∂x j 
  
 ∂ 2ε pq ∂ε pq 
=  niq tjp  stk  x + δ ki eˆ n ⊗ eˆ s
 ∂xi ∂x j k ∂ x 
 j 

 ∂ 2ε pq ∂ε 

=  niq tjp  stk xk +  nkqtjp  stk pq eˆ n ⊗ eˆ s
 ∂xi ∂x j ∂x j 

∂ε pq
Note that the term  nkq tjp  stk =  nkq tjp  stk ε pq , j = − nqk tjp  stk ε pq , j can be expressed as
∂x j
follows:
 nkq tjp  stk ε pq , j = − nqk tjp  stk ε pq , j = −(δ nsδ qt − δ ntδ qs )tjp ε pq , j = −δ nsδ qt tjp ε pq , j + δ ntδ qs tjp ε pq , j
= −δ ns tjp ε pt , j +  njp ε ps , j = − ptj ε pt , jδ ns +  njp ε ps , j
= − ptj ε pt , jδ ns +  njp ε ps , j = −( ptj ε pt ), j δ ns +  njp ε ps , j
r r
= −(⋅ : (∇ xr ε ))δ ns + [∇ xr ∧ ε ]ns = −(∇ xr ⋅ ( : ε ))δ ns + [∇ xr ∧ ε ]ns
r r
= {−(⋅ : (∇ xr ε )) 1 + [∇ xr ∧ ε ]}ns = {−(∇ xr ⋅ ( : ε )) 1 + [∇ xr ∧ ε ]}ns
and
∂ 2 ε pq  r r r
 niq  tjp  stk xk =  niq  tjp  stk ε pq ,ij xk =  {∇ xr ∧ [∇ xr ∧ ε ]T } ∧ x 
∂xi ∂x j  
  ns

b) If ε = ε T we can show that ⋅ : (∇ xr ε ) = ∇ xr ⋅ ( : ε ) = 0 , then


r r r r r r r
∇ xr ∧ {[∇ xr ∧ ε ]T ∧ x} = {∇ xr ∧ [∇ xr ∧ ε ]T } ∧ x + [∇ xr ∧ ε ]

Problem 1.116
r r r r r
Let v be a vector field in function of x , i.e. v = v ( x ) , whose components are given by:

University of Castilla-La Mancha Draft By: Eduardo W. V. Chaves (2016)


Ciudad Real - Spain
1 TENSORS 117

v1 = x1 − 5 x 2 + 2 x3

v 2 = 5 x1 + x 2 − 3 x3
v = −2 x + 3 x + x
 3 1 2 3
r r
a) Obtain the gradient of v ; b) Obtain (∇ xr v ) : 1 ; c) Apply the additive decomposition of the
r
tensor ∇ xr v into a symmetric and antisymmetric parts; d) Obtain the axial vector associated
r
with the antisymmetric tensor (∇ xr v ) skew .
Solution: a)
 ∂v1 ∂v1 ∂v1 
 
r  ∂x1 ∂x2 ∂x3   1 − 5 2 
r ∂v components ∂vi  ∂v2 ∂v2 ∂v2  
1 − 3
r
∇ xr v = r   →(∇ xr v )ij = vi , j = = = 5
∂x ∂x j  ∂x1 ∂x2 ∂x3  
   1 
 ∂v3 ∂v3 ∂v3   − 2 3
 ∂x1 ∂x2 ∂x3 
r r
b) (∇ xr v ) : 1 = Tr (∇ xr v ) = 1 + 1 + 1 = 3
r r r
c) ∇ xr v = (∇ xr v ) sym + (∇ xr v ) skew =
1
[ r r 1
] [ r r
(∇ xr v ) + (∇ xr v ) T + (∇ xr v ) − (∇ xr v ) T
2 44424443 1 2 44424443
]
1 r r
=(∇ xr v ) sym =(∇ xr v ) skew
r r
Then, the components of (∇ xr v ) sym and (∇ xr v ) skew are given, respectively, by:
1 0 0  0 −5 2 
1  ∂vi ∂v j    1  ∂vi ∂v j  
0 − 3
r sym r skew
[(∇ xr v ) ]ij =  +  = 0 1 0 ; [(∇ xr v ) ]ij =  − = 5
2  ∂x j ∂xi   2  ∂x j ∂xi  
0 0 1 − 2 3 0 
d) Remember that
 1  ∂v1 ∂v2  1  ∂v1 ∂v3  
 0  −   − 
 2  ∂x2 ∂x1  2  ∂x3 ∂x1  
r  1  ∂v2 ∂v1  1  ∂v2 ∂v3 
,j =  
(W)ij ≡ [(∇ xr v ) skew ]ij ≡ viskew  −  0  − 
 2  ∂x1 ∂x2  2  ∂x3 ∂x2 
 1  ∂v ∂v  1  ∂v3 ∂v2   (1.127)
  3 − 1   −  0 
 2  ∂x1 ∂x3  2  ∂x2 ∂x3  
 0 W12 W13   0 W12 W13   0
w2  − w3
= W21 0 W23  =  − W12
0 0 W23  =  w3
− w1 
 W31 W32 0   − W13
− W23 w1 0  − w2
0 
r
where w1 , w2 , w3 are the components of the axial vector w associated with the
r
antisymmetric tensor W ≡ (∇ xr v ) skew , then, to the proposed problem we have:
 0 w2   0 − 5 2 
− w3 w1 = 3
 w 
 30 − w1  =  5 0 − 3 ⇒  w2 = 2
− w2
w1 0   − 2 3 0  w = 5
 3
r
The axial vector, in the Cartesian basis, is w = 3eˆ 1 + 2eˆ 2 + 5eˆ 3 .

University of Castilla-La Mancha Draft By: Eduardo W. V. Chaves (2016)


Ciudad Real - Spain
118 SOLVING PROBLEMS BY MEANS OF CONTINUUM MECHANICS

1 r r
Alternative solution d) In Problem 1.37 where we have shown that (a ∧ x ) is the axial vector
2
r r
associated with the antisymmetric tensor ( x ⊗ a ) skew . Then, the axial vector associated with
r r r r 1 r r
the antisymmetric tensor (∇ xr v ) skew = [(v ) ⊗ (∇ xr )]skew is the vector w = (∇ xr ∧ v ) , thus
2
eˆ 1 eˆ 2 eˆ 3
r 1 ∂ ∂ ∂ 1  ∂v ∂v   ∂v ∂v   ∂v ∂v  
w= =  3 − 2 eˆ 1 −  3 − 1 eˆ 2 +  2 − 1 eˆ 3 
2 ∂x1 ∂x2 ∂x3 2  ∂x2 ∂x3   ∂x1 ∂x3   ∂x1 ∂x2  
v1 v2 v3
1
= [(3 − (−3) )eˆ 1 − ((−2) − (2) )eˆ 2 + (5 − (−5) )eˆ 3 ] = 3eˆ 1 + 2eˆ 2 + 5eˆ 3
2
Problem 1.117
r r r
Let l = ∇ xr v be a second-order tensor. Considering that D = (∇ xr v ) sym and W = (∇ xr v ) skew ,
r r
show that W ⋅ D + D ⋅ W = 2(D ⋅ W ) skew = [(∇ xr v ) ⋅ (∇ xr v )]skew = ( l ⋅ l ) skew .
Solution:
In Problem 1.35 we have shown that: given an arbitrary second-order tensor l it fulfills that
l
skew
⋅l sym
+l sym
⋅l skew
= 2( l skew
⋅l sym skew
)

Then, W ⋅ D + D ⋅ W = 2(D ⋅ W ) skew holds. Taking into account the definition of symmetry and
antisymmetry, i.e. D =
1
2
[
l +l
T
] , W = 12 [l − l ] , we can conclude that:
T

2
W ⋅ D + D ⋅ W = 2(D ⋅ W ) skew =
4
[
(l + l T ) ⋅ (l − l T )
skew 1
= l ⋅l + l ⋅l
2
] [ T
−l T
⋅l −l T
⋅l T ]
skew

1
= 1 [
l ⋅l − l ⋅l
T
2 4442444
T skew 1
]
3 + 2 l ⋅l − l ⋅l
T T skew
[ ]
=0
1
[
= l ⋅ l − ( l ⋅ l )T
2
] skew
=
1
2
[
2( l ⋅ l ) skew ]
skew r r
= ( l ⋅ l ) skew = (∇ xr v ⋅ ∇ xr v ) skew

Obs.: Note that the resulting tensor l ⋅l T −l T


⋅ l is a symmetric one, since:
(l ⋅ l T
−l T
⋅ l )T = l ⋅l T
−l T
⋅l
Problem 1.118
Consider the scalar J = F ≡ det (F ) and an arbitrary second-order tensor given by
r dF
l = ∇ xr v = F& ⋅ F −1 , where F& ≡ represents the time derivative of F . Show that the
dt
following is true:

d(J ) & r
≡ J = J (∇ xr ⋅ v ) (1.128)
dt

 dA dA
Solution: In Problem 1.88 we have shown that = A Tr ⋅ A −1  holds, where A = A (τ)
dτ  dτ 
is an arbitrary second-order tensor and τ a scalar. Making A = F and τ = t , we can obtain:

University of Castilla-La Mancha Draft By: Eduardo W. V. Chaves (2016)


Ciudad Real - Spain
1 TENSORS 119

d F dJ  dF
⋅ F −1  = J Tr (F& ⋅ F −1 ) = J Tr(l ) = J Tr(l
r r
= = F Tr sym
) = J Tr (∇ xr v ) = J (∇ xr ⋅ v )
dt dt  dt 
Alternative solution:
In Problem 1.45 we have shown that given a second-order tensor F the relationship
F  tpq =  rjk Frt F jp Fkq holds, and if we take the time derivative of it we can obtain:

DF D
tpq = ( rjk Frt F jp Fkq ) =  rjk F&rt F jp Fkq +  rjk Frt F& jp Fkq +  rjk Frt F jp F&kq (1.129)
Dt Dt
According to the problem statement we have l = F& ⋅ F −1 ⇒ F& = l ⋅ F , with that the
following relations F&rt = l rs Fst , F& jp = l js Fsp and F&kq = l ks Fsq hold, and the equation in (1.129)
can be rewritten as follows:
DF
 tpq =  rjk F&rt F jp Fkq +  rjk Frt F& jp Fkq +  rjk Frt F jp F&kq
Dt
=  rjk l rs Fst F jp Fkq +  rjk Frt l js Fsp Fkq +  rjk Frt F jp l ks Fsq

We multiply both sides of the above equation by ut v p w q we can obtain:


DF
 tpq ut v p w q =  rjk l rs Fst F jp Fkqut v p w q +  rjk Frt l js Fsp Fkqut v p w q +  rjk Frt F jp l ks Fsqut v p w q
Dt
=  rjk ( l rs Fst ut )( F jp v p )( Fkq w q ) +  rjk ( Frt ut )( l js Fsp v p )( Fkq w q )
+  rjk ( Frt ut )( F jp v p )( l ks Fsq w q )
=  rjk ( l rs a s )(b j )(c k ) +  rjk (a r )( l jsb s )(c k ) +  rjk (a r )(b j )( l ks c s )

where we have denoted by a s = Fst ut , b j = F jp v p , c s = Fsq w q . The above equation in tensorial


notation becomes:
DF r r r r r r r r r r r r r r r
u ⋅ ( v ∧ w ) = ( l ⋅ a) ⋅ (b ∧ c ) + a ⋅ [( l ⋅ b) ∧ c ] + a ⋅ [b ∧ ( l ⋅ c )] = Tr ( l )[a ⋅ (b ∧ c )]
Dt
where we have used the property of trace, (see Problem 1.47). The above equation can also
be written as follows:
DF r r r r r r r r r r r r
u ⋅ ( v ∧ w ) = Tr ( l )[a ⋅ (b ∧ c )] = Tr ( l ){( F ⋅ u) ⋅ [( F ⋅ v ) ∧ ( F ⋅ w )]} = Tr ( l ) F u ⋅ ( v ∧ w )
Dt
where we have used the property of determinant, (see Problem 1.48), with that we conclude
DF
that = Tr ( l ) F .
Dt
Problem 1.119
r
Let us consider a vector field represented by the unit vector field bˆ ( x ) , (see Figure 1.31).
r r r
Obtain the second-order projection tensor P such that p = P ⋅ u holds, where u is an
r r
arbitrary vector and p is orthogonal to the field defined by bˆ ( x ) .

University of Castilla-La Mancha Draft By: Eduardo W. V. Chaves (2016)


Ciudad Real - Spain
120 SOLVING PROBLEMS BY MEANS OF CONTINUUM MECHANICS

r
bˆ ( x )

Figure 1.31: Vector field.


Solution:
The proposed problem is represented in Figure 1.32.

r ˆ r r r
u⊥ b = p = P ⋅ u u
r r ˆ
a = u // b

r
bˆ ( x )

Figure 1.32

r r r r
And, by considering the vector summation we obtain u = a + p . In addition, the vector a can
r r r r
be obtained by means of the projection of u onto the direction b̂ : a = a bˆ = (u ⋅ bˆ ) bˆ , note
r r r
also that a = (u ⋅ bˆ ) bˆ = u ⋅ (bˆ ⊗ bˆ ) . With that we can obtain:
r r r
p = u−a p i = ui − ai
r r r r
= u − (u ⋅ bˆ ) bˆ = u − (u ⋅ bˆ ) ⊗ bˆ = u i − (u k bˆ k )bˆ i
r r
= 1 ⋅ u − (bˆ ⊗ bˆ ) ⋅ u = u δ − u bˆ bˆ
k ik k k i
r
= [1 − (bˆ ⊗ bˆ )] ⋅ u = (δ ik − bˆ k bˆ i )u k
r
= P ⋅u = Pik u k
Thus, we conclude that the projection second-order tensor is given by:
P = 1 − bˆ ⊗ bˆ
The same result could have been obtained by means of vector product, (see Figure 1.33).
r r
Taking into account that a ∧ (b ∧ a) = [(a ⋅ a)1 − a ⊗ a]⋅ b , (see Problem 1.17), we can obtain
r r r r r r

r
[
bˆ ∧ (u ∧ bˆ ) = (bˆ ⋅ bˆ )1 − bˆ ⊗ bˆ ] ⋅ ur = [1 − bˆ ⊗ bˆ ] ⋅ ur = pr .
Then we can present a vector as follows:
r r ˆ r ˆ r r
u = u// b + u⊥b = (bˆ ⊗ bˆ ) ⋅ u + [1 − (bˆ ⊗ bˆ )] ⋅ u
r r r r
where u// b = (bˆ ⊗ bˆ ) ⋅ u is the vector parallel to b̂ -direction and u⊥b = [1 − (bˆ ⊗ bˆ )] ⋅ u is the
ˆ ˆ

perpendicular one.

University of Castilla-La Mancha Draft By: Eduardo W. V. Chaves (2016)


Ciudad Real - Spain
1 TENSORS 121

r
u ∧ bˆ
r
bˆ ( x )
r
u

r
bˆ ∧ (u ∧ bˆ )
Figure 1.33

Problem 1.120
r r
Given a vector field v ( x ) , show that the following relationship holds:
r r 1 r r r
(∇ xr v ) ⋅ v = ∇ xr (v 2 ) − v ∧ (∇ xr ∧ v )
2
r r r r
where v = v is the module of v , so v 2 = v ⋅ v .
Solution:
1 r 2 1 r r 1 1 r r
Note that [∇ x (v )]i = [∇ xr (v ⋅ v )]i = (vk vk ),i = (vk ,i vk + vk vk ,i ) = vk vk ,i = (v ⋅ ∇ xr v )i .
2 2 2 2
r r
At one point of the vector field v , we consider a plane normal to v and recalling that the
r
projection of a second-order tensor onto a direction ( v ) is a vector which does not necessary
r r r
have the same direction as ( v ), with that we represent the following vectors (∇ xr v ) ⋅ v and
r r
v ⋅ (∇ xr v ) :

r r
∇ xr v ⋅ v

r r r
r c⊥v
(∇ xr ∧ v ) r
r r
r c ⊥ (∇ xr ∧ v )
v r r
v ⋅ ∇ xr v

r r r r
c = v ∧ (∇ xr ∧ v )

Figure 1.34

Note that, by means of summation of vectors, (see Figure 1.34), we can obtain:
r r r r r r r r r r r r r r r
(∇ xr v ) ⋅ v + c = v ⋅ (∇ xr v ) ⇒ c = v ⋅ (∇ xr v ) − (∇ xr v ) ⋅ v ⇒ c = v ⋅ (∇ xr v ) − v ⋅ (∇ xr v ) T
r r r r r r
⇒ c = v ⋅ ((∇ xr v ) − (∇ xr v ) T ) = v ⋅ 2(∇ xr v ) skew

University of Castilla-La Mancha Draft By: Eduardo W. V. Chaves (2016)


Ciudad Real - Spain
122 SOLVING PROBLEMS BY MEANS OF CONTINUUM MECHANICS

r r
If we consider that w is the axial vector associated with the antisymmetric tensor (∇ xr v ) skew , it
r r r r r r r r
fulfills that: (∇ xr v ) skew ⋅ v = w ∧ v ⇒ v ⋅ (∇ xr v ) skew = v ∧ w . In addition, the relationship
r r r r
rot (v ) ≡ ∇ xr ∧ v = 2 w holds. Then,
r r r r r r r r
c = v ⋅ 2(∇ xr v ) skew = v ∧ 2 w = v ∧ (∇ xr ∧ v ) (1.130)
with that we conclude that:
r r r r r r r r r r
(∇ xr v ) ⋅ v + c = v ⋅ (∇ xr v ) ⇒ (∇ xr v ) ⋅ v = v ⋅ (∇ xr v ) − c
r r 1 r r r
⇒ (∇ xr v ) ⋅ v = ∇ xr (v 2 ) − v ∧ (∇ xr ∧ v )
2
r r r
It is interesting to note that: when (∇ xr v ) is a symmetric tensor, i.e. (∇ xr v ) = (∇ xr v ) sym , the
r r r r r r r r r r
following is fulfilled (∇ xr v ) skew = 0 , c = 0 , (∇ xr ∧ v ) = 0 , and (∇ xr v ) ⋅ v = v ⋅ (∇ xr v ) has the
r
same direction as v , (see Figure 1.35 (a)).
r r r r r r r
When (∇ xr v ) = (∇ xr v ) skew we have that c = v ⋅ 2(∇ xr v ) skew = 2v ⋅ (∇ xr v ) , (see equation (1.130)).
r r r r r
With that, v ⋅ (∇ xr v ) = −(∇ xr v ) ⋅ v holds, and the vector v is perpendicular to the vector
r r
(∇ xr ∧ v ) , (see Figure 1.35(b)).

r r r r r r
a) (∇ xr v ) = (∇ xr v ) sym b) (∇ xr v ) = (∇ xr v ) skew c⊥v
r r r
c ⊥ (∇ xr ∧ v )
r r
∇ xr v ⋅ v
r r r r r
v ⋅ (∇ xr v ) = (∇ xr v ) ⋅ v v

r r r
v (∇ xr ∧ v )

r r
v ⋅ ∇ xr v

r r r
(∇ xr ∧ v ) = 0 r r r r r r
c = v ∧ (∇ xr ∧ v ) = 2v ⋅ (∇ xr v )

Figure 1.35
Alternative solution:
r r r r r r r r r
∇ xr v ⋅ v = ((∇ xr v ) sym + (∇ xr v ) skew ) ⋅ v = (∇ xr v ) sym ⋅ v + (∇ xr v ) skew ⋅ v
r r r r r r r r
= (∇ xr v ) sym ⋅ v + (∇ xr v ) skew ⋅ v + ((∇ xr v ) skew ⋅ v − (∇ xr v ) skew ⋅ v )
r r r r r r
= ((∇ xr v ) sym ⋅ v − (∇ xr v ) skew ⋅ v ) + 2(∇ xr v ) skew ⋅ v
1 r
[ r r r r
] r
= (∇ xr v + (∇ xr v ) T ) − (∇ xr v − (∇ xr v ) T ) ⋅ v + 2(∇ xr v ) skew ⋅ v
2
r

1 r r r r r r r r
= (2(∇ xr v ) T ) ⋅ v + 2(∇ xr v ) skew ⋅ v = v ⋅ (∇ xr v ) + 2(∇ xr v ) skew ⋅ v
2
1 r 2 r rr r
= ∇ x (v ) − v ∧ (∇ x ∧ v )
2
r r r r r r r r r
Remember that (∇ xr v skew ) T = −(∇ xr v ) skew , thus 2(∇ xr v ) skew ⋅ v = −v ⋅ 2(∇ xr v ) skew = −v ∧ (∇ xr ∧ v )

University of Castilla-La Mancha Draft By: Eduardo W. V. Chaves (2016)


Ciudad Real - Spain
1 TENSORS 123

Problem 1.121
r r r
Let u( x ) be a stationary vector field. a) Obtain the components of the differential du . b)
r r
Now, consider that u( x ) represents a displacement field, and is independent of x3 . With
these conditions, graphically illustrate the displacement field in the differential area element
dx1 dx 2 .
Solution: According to the differential and gradient definitions, the following equations are true
r r r r r r r r r
du ≡ u( x + dx ) − u( x ) and du = (∇ xr u) ⋅ dx , (see Figure 1.36).

r r r r r r
r r r u( x ) dx u( x + dx )
r r r
du ≡ u( x + dx ) − u( x )
r r r
du = (∇ xr u) ⋅ dx
r
x2 x r r
x + dx

x1
x3

Figure 1.36
Thus, the components are defined as:

 ∂u1 ∂u1 ∂u1   ∂u1 ∂u ∂u


  du1 = dx1 + 1 dx 2 + 1 dx3
∂x1 ∂x 2 ∂x3
 du1   ∂x1 ∂x2 ∂x3   dx  
∂u ∂u ∂u 2 ∂u 2   
1
 ∂u 2 ∂u ∂u
du i = i dx j ⇒ du 2  =  2 dx2 ⇒  du 2 = dx1 + 2 dx 2 + 2 dx3
∂x j ∂x ∂x2 ∂x3     ∂x1 ∂x 2 ∂x3
 du 3   1  

 u3 ∂u3 ∂u 3   dx3   ∂u ∂u ∂u
 ∂x1 ∂x2 ∂x3  du 3 = 3 dx1 + 3 dx 2 + 3 dx 3
 ∂x1 ∂x 2 ∂x3
with
du1 = u1 ( x1 + dx1 , x 2 + dx 2 , x3 + dx3 ) − u1 ( x1 , x 2 , x3 )

du 2 = u 2 ( x1 + dx1 , x 2 + dx 2 , x3 + dx3 ) − u 2 ( x1 , x 2 , x3 )
du = u ( x + dx , x + dx , x + dx ) − u ( x , x , x )
 3 3 1 1 2 2 3 3 3 1 2 3

As the field is independent of x3 , the displacement field in the differential area element is
defined as:
 ∂u1 ∂u1
du1 = u1 ( x1 + dx1 , x 2 + dx 2 ) − u1 ( x1 , x 2 ) = ∂x dx1 + ∂x dx 2
 1 2

du = u ( x + dx , x + dx ) − u ( x , x ) = ∂u 2 dx + ∂u 2 dx
 2 2 1 1 2 2 2 1 2
∂x1
1
∂x 2
2

or:
 ∂u1 ∂u1
u1 ( x1 + dx1 , x 2 + dx 2 ) = u1 ( x1 , x 2 ) + ∂x dx1 + ∂x dx 2
 1 2

u ( x + dx , x + dx ) = u ( x , x ) + 2 dx + 2 dx∂ u ∂u
 2 1 1 2 2 2 1 2
∂x1
1
∂x 2
2

University of Castilla-La Mancha Draft By: Eduardo W. V. Chaves (2016)


Ciudad Real - Spain
124 SOLVING PROBLEMS BY MEANS OF CONTINUUM MECHANICS

Note that the above equation is equivalent to the Taylor series expansion taking into account
only up to linear terms. The representation of the displacement field in the differential area
element is shown in Figure 1.37.

∂u 2 ∂u 2 ∂u
u2 + dx 2 u2 + dx1 + 2 dx 2
∂x 2 ∂x1 ∂x 2

( x1 , x 2 + dx 2 ) ( x1 + dx1 , x 2 + dx 2 )

∂u1 ∂u1 ∂u
u1 + dx 2 u1 + dx1 + 1 dx 2
∂x 2 ∂x1 ∂x 2
r
du
dx 2

∂u 2
u2 + dx1
(u 2 ) ∂x1

( x1 , x 2 ) ( x1 + dx1 , x 2 )
x2 ∂u1
(u1 ) u1 + dx1
∂x1
dx1
x1
144444444444444444424444444444444444443

=
644444444444444444474444444444444444448
x 2 ,u 2
∂u1
dx2
∂x2
∂u 2
u2 + dx2
∂x2 B′

B B B′

dx 2 dx 2
+
O′ A′ A′
∂u 2
u2 A
dx1
O A ∂x1
dx1 O′
dx1
u1

∂u1
u1 + dx1
∂x1
x1 ,u1

Figure 1.37: Displacement field in the differential area element.

University of Castilla-La Mancha Draft By: Eduardo W. V. Chaves (2016)


Ciudad Real - Spain
1 TENSORS 125

Problem 1.122
r r
Given a second-order tensor field T ( x ) . Show that: if there is no source of the field T ( x ) it
r r
fulfills that the divergence of T ( x ) is equal to zero, i.e. ∇ xr ⋅ T = 0 . For the demonstration,
consider the tensor field in a differential volume element dV = dx1 dx 2 dx 3 in the Cartesian
system.
Solution:
r
Let us set the tensor field T ( x ) in the differential volume element. For this purpose, we start
r
from the definition of the differential of T ( x ) which is defined by means of gradient as
follows:
r r r
dT ≡ T ( x + dx ) − T ( x ) r r r r r r r r
r  ⇒ T ( x + dx ) − T ( x ) = (∇ xr T ) ⋅ dx ⇒ T ( x + dx ) = T ( x ) + (∇ xr T ) ⋅ dx
dT = (∇ xr T ) ⋅ dx 
The above equation in indicial notation becomes:
r r r
Tij ( x + dx ) = Tij ( x ) + Tij , k dx k
r
= Tij ( x ) + Tij ,1 dx1 + Tij , 2 dx 2 + Tij ,3 dx 3
r ∂Tij ∂Tij ∂Tij
= Tij ( x ) + dx1 + dx 2 + dx 3
∂x1 ∂x 2 ∂x 3
r r
The representation of the field components Tij ( x + dx ) can be appreciated in Figure 1.38.
r ∂Ti1
Note that on the face normal to x1 + dx1 act the components Ti1 ( x ) + dx1 , since
∂x1
according our convention, the first index indicate the direction in which points out and the
second index indicates the normal plane.
r r
Once established the tensor field componets Tij ( x + dx ) in the differential volume element,
r r
we apply the total balance of the tensor field components Tij ( x + dx ) according to the
directions x1 , x 2 , x3 .
r r
Total balance of Tij ( x + dx ) in dV according to x1 -direction is equal to zero (there is no
source):
 ∂T   ∂T   ∂T 
 T11 + 11 dx1  dx 2 dx3 +  T13 + 13 dx 3 dx1 dx 2 +  T12 + 12 dx 2 dx1 dx3 − T11 dx 2 dx3
 ∂x1   ∂x3   ∂x 2 
− T13 dx1 dx 2 − T12 dx1 dx3 = 0
By simplifying the above equation we can obtain:
∂T11 ∂T ∂T
dx1 dx 2 dx3 + 13 dx3 dx1 dx 2 + 12 dx 2 dx1 dx3 = 0
∂x1 ∂x3 ∂x 2
∂T11 ∂T12 ∂T13
⇒ + + =0
∂x1 ∂x 2 ∂x3
Similarly, according to the directions x 2 and x3 we will obtain, respectively:
∂T21 ∂T22 ∂T23 ∂T31 ∂T32 ∂T33
+ + =0 and + + =0
∂x1 ∂x 2 ∂x3 ∂x1 ∂x 2 ∂x3

University of Castilla-La Mancha Draft By: Eduardo W. V. Chaves (2016)


Ciudad Real - Spain
126 SOLVING PROBLEMS BY MEANS OF CONTINUUM MECHANICS

x3 Rear face
T11
∂T33 T21
T33 + dx3
Rear face ∂x3
∂T23
T23 + dx3
∂x3 T31
∂T13
T13 + dx3
∂x3 ∂T32 dx 3
T32 + dx2
∂x2
T12
T22 ∂T31
T31 + dx1 ∂T22
T22 + dx2
∂x1 ∂T12 ∂x2 x2
T12 + dx2
∂x2
T32
∂T21
T21 + dx1 dx1
∂T11 ∂x1
T11 + dx1
∂x1

T13
x1 T23
Rear face

T33

dx 2

Figure 1.38: Tensor field components in the differential volume element.

Then, we have the following set of equations that must be met simultaneously:
 ∂T11 ∂T12 ∂T13
 + + =0
 ∂x1 ∂x 2 ∂x3
 T11,1 + T12, 2 + T13,3 = 0  T1 j , j = 0
 ∂T21 ∂T22 ∂T23  
 + + =0 ⇒  T21,1 + T22, 2 + T23,3 = 0 ⇒  T2 j , j = 0 ⇒ Tij , j = 0 i
 ∂x1 ∂x 2 ∂x3  
 ∂T31 ∂T32 ∂T33  T31,1 + T32, 2 + T33,3 = 0  T3 j , j = 0
 + + =0
 ∂x1 ∂x 2 ∂x3
Thus, we have shown that in the absence of source, the divergence is zero:
r
Tij , j = 0 i ⇔ (∇ xr ⋅ T ) i = 0 i tensorial
  → ∇ xr ⋅ T = 0

NOTE 1: If we have a tensor field, the tensor order of the source is a minor order of the
tensor, e.g. the source of a vector field is represented by a scalar field.
NOTE 2: If the divergence of a tensor field is positive we have a source, on the contrary if
the divergence is negative we have a sink.

University of Castilla-La Mancha Draft By: Eduardo W. V. Chaves (2016)


Ciudad Real - Spain
1 TENSORS 127

Problem 1.123
Show that:
[(∇ xr T ) ⋅ ur ] ⋅ ar = [∇ xr ( T ⋅ ar )] ⋅ ur (1.131)
r r r r r
where T = T (x ) is a second-order tensor field, u = u( x ) is a vector field, and a an arbitrary
r
vector (independent of x ).
r r
Solution: Note that the term [(∇ xr T ) ⋅ u]⋅ a is a vector, which in indicial notation becomes:
[ ] [ ]
{[(∇ xr T ) ⋅ ur ]⋅ ar}i = [(∇ xr T ) ⋅ ur ]ik (ar )k = (∇ xr T )ikp u p ak = Tik , pu p ak = Tik , pu p ak (1.132)
r r
Now we express the term [∇ xr (T ⋅ a )] ⋅ u in indicial notation:
r r r
( T ⋅ a )i = Tik ak  → [∇ xr ( T ⋅ a )] ij = ( T ⋅ a )i , j = ( Tik ak ), j
gradient

r
⇒ [∇ xr ( T ⋅ a )] ij = ( Tik ak ), j = Tik , j ak + Tik ak , j = Tik , j ak
{
=0 k , j (1.133)
r r
[
or ⇒ [∇ xr ( T ⋅ a )] ij = a ⋅ (∇ xr T T ) ] ij 123
r r
[
+ [ T ⋅ (∇ xr a )]ij = a ⋅ (∇ xr T T ) ] ij
= Tik , j ak
=0
r r
where we have considered that a is independent of (x ) . If we apply the scalar product
r
between the above equation and u we obtain:
{ }
{[∇ xr ( T ⋅ ar )]⋅ ur}i = [∇ xr ( T ⋅ ar )]ij u j = Tik , j a k u j = Tik , p u p a k (1.134)
If we compare (1.132) with (1.134) we show (1.131).
r r r r
Not that, if a = a (x ) depends on x and according to the equation in (1.133) we can conclude
r r r r
that [∇ xr (T ⋅ a )] ij = ( Tik ak ), j = Tik , j ak + Tik ak , j ⇒ [∇ xr ( T ⋅ a )] ij = [a ⋅ (∇ xr T T )] ij + [ T ⋅ (∇ xr a )] ij .
Problem 1.124
r r r
Show that if the magnitude of a vector, ω = ω(t ) , is constant with time, this implies that ω is
r

orthogonal to at any time t .
dt
Solution:
r r r
We start from the definition of the magnitude of a vector, where ω = ω ⋅ ω holds, thus:
2

r 2 r r r r r r
d( ω ) d ( ω ⋅ ω) d ( ω) r r d ( ω) r d ( ω) r dω
= = ⋅ω + ω⋅ = 2ω ⋅ =0 ⇒ ω⊥
dt dt dt dt dt dt
NOTE: A particular case of this problem is the circular motion, (see Figure 1.39).
r r
v x = constant  r
r  r dx
r r dx ⇒ x ⊥
x v=  dt
dt 

Figure 1.39: Circular motion.

University of Castilla-La Mancha Draft By: Eduardo W. V. Chaves (2016)


Ciudad Real - Spain
128 SOLVING PROBLEMS BY MEANS OF CONTINUUM MECHANICS

1.15.1 Theorems Involving Integrals

Problem 1.125
r
Check the divergence theorem (Gauss theorem) for the vector field F whose Cartesian
components are given by Fi = xi + ( x32 − x 3 )δ i 3 . Consider the boundary defined by the
cylinder x12 + x 22 ≤ 1 , 0 ≤ x3 ≤ 1 .
Solution:
The divergence theorem states that:
r r
∫ ∇ xr ⋅ F dV = F ⋅ nˆ dS

V S

where n̂ is the normal to the surface and points outwards.

S (2)
n̂ ( 2 )
x12 + x 22 ≤ 1 x3

r r
r =1 r S (1)

h =1 r n̂ (1)
x

x2

x1 n̂ (3) S ( 3)

Figure 1.40
r
Calculation of ∫ ∇ xr ⋅ F dV :
V
r
[ ]
∇ xr ⋅ F = Fi ,i = xi + ( x32 − x3 )δ i 3 ,i = xi ,i + ( x32 − x3 ),i δ i 3 = δ ii + ( x32 − x3 ),3 = 3 + (2 x3 − 1) = 2 x3 + 2

Thus
x3 =1
r
∫ ∇ xr ⋅ F dV = (2 x3 + 2) dV =
∫ ∫A x ∫=(02 x 3 ∫
+ 2)dx3 dA = 3 dA = 3(πr 2 ) = 3π
V V 3 A

where A is the area defined by the circle x12 + x 22 ≤ 1 .


r
Calculation of ∫ F ⋅ nˆ dS
S

We decompose the boundary in three areas, namely: S (1) , S ( 2) , S (3) , (see Figure 1.40), then

University of Castilla-La Mancha Draft By: Eduardo W. V. Chaves (2016)


Ciudad Real - Spain
1 TENSORS 129

r r r r
∫ F ⋅ nˆ dS = ∫ F ⋅ nˆ (1) dS (1) + ∫ F ⋅ nˆ ( 2 ) dS ( 2 ) + ∫ F ⋅ nˆ (3) dS (3)
S S ( 1) S ( 2) S ( 3)
r
The components of F are: F1 = x1 + ( x32 − x3 )δ 13 = x1 , F2 = x 2 , F3 = x3 + ( x32 − x 3 )δ 33 = x32 .
r r
The representation of F in the Cartesian basis is given by: F = x1eˆ 1 + x 2 eˆ 2 + x32 eˆ 3 . The
normal for each surface are defined as follows:
r 1
nˆ (1) // r ⇒ nˆ (1) = ( x1 eˆ 1 + x 2 eˆ 2 ) ; nˆ ( 2 ) = eˆ 3 ; nˆ (3) = −eˆ 3
x12 + x 22

On the surface S (1) it holds that:


r 1
∫ F ⋅ nˆ (1) dS (1) = ∫ ( x eˆ 1 1 + x 2 eˆ 2 + x 32 eˆ 3 ) ⋅ ( x1 eˆ 1 + x 2 eˆ 2 )dS (1)
S ( 1)
S ( 1) x12 + x 22
x12 + x 22
∫ dS (1) = ∫ 1dS
(1)
= = 2πrh = 2π
S ( 1) x12 + x 22 S (1)

where we have considered the cylinder area ( 2πrh = 2π ).


On the surface S ( 2) it holds that x3 = 1 :
r
∫ F ⋅ nˆ ∫ ( x eˆ + x 2 eˆ 2 + 1eˆ 3 ) ⋅ (eˆ 3 ) dS ( 2 ) = ∫ 1dS
( 2)
dS ( 2 ) = 1 1
(2)
= πr 2 = π
S (2) S (2) S (2)

where we have considered the circle area ( πr 2 = π ).


On the surface S (3) , it holds that x3 = 0 :
r
∫ F ⋅ nˆ ∫ ( x eˆ + x 2 eˆ 2 + 0eˆ 3 ) ⋅ (−eˆ 3 )dS (3) = ∫ 0dS
( 3)
dS (3) = 1 1
( 3)
=0
( 3) ( 3) (3)
S S S
r r r r
with that: ∫ F ⋅ nˆ dS = ∫ F ⋅ nˆ (1) dS (1) + ∫ F ⋅ nˆ ( 2 ) dS ( 2 ) + ∫ F ⋅ nˆ
( 3)
dS (3) = 3π
S S (1 ) S (2) S ( 3)
r r
With that we check the divergence theorem: ∫ ∇ xr ⋅ F dV = ∫ F ⋅ nˆ dS = 3π .
V S

Problem 1.126
Let Ω be a domain bounded by Γ as shown in Figure 1.41. Further consider that m is a
second-order tensor field and ω is a scalar field. Show that the following relationship holds:

∫ [m : (∇

x (∇ x
r r ω ))]dΩ = ∫Γ [(∇ ω ) ⋅ m]⋅ nˆ dΓ − Ω∫ [(∇
r
x
r
x ⋅ m) ⋅ ∇ xrω ]dΩ

∫ [m ω ,

ij ij ∫
] dΩ = (ω ,i mij )nˆ j dΓ − [mij , jω ,i ] dΩ
Γ

University of Castilla-La Mancha Draft By: Eduardo W. V. Chaves (2016)


Ciudad Real - Spain
130 SOLVING PROBLEMS BY MEANS OF CONTINUUM MECHANICS

Ω n̂
x2

x1 Γ
Figure 1.41

Solution:
We could directly apply the definition of integration by parts to demonstrate the above
relationship. But, here we will start with the definition of the divergence theorem. That is,
r
given a tensor field v , it is true that:
r r
∫∇ r
x ⋅ v dΩ = ∫ v ⋅ nˆ dΓ indicial
 → ∫ v j , j dΩ = ∫ v j nˆ j dΓ
Ω Γ Ω Γ
r
Observing that the tensor v can be represented by the result of the algebraic operation
r
v = ∇ xr ω ⋅ m and the equivalent in indicial notation is v j = ω , i m ij , and by substituting it in
the above equation we obtain:

∫v

j, j ∫
dΩ = v jnˆ j dΓ
Γ
∫ ∫
⇒ [ω ,i mij ], j dΩ = ω ,i mijnˆ j dΓ
Ω Γ

∫ ∫
⇒ [ω ,ij mij + ω ,i mij , j ] dΩ = ω ,i mijnˆ j dΓ
Ω Γ

∫ ∫ ∫
⇒ [ω ,ij mij ] dΩ = ω ,i mijnˆ j dΓ − [ω ,i mij , j ] dΩ
Ω Γ Ω
The above equation in tensorial notation becomes:

∫Ω [m : (∇ x (∇ x
r r ω ))]dΩ = ∫Γ [(∇ r
x ω ) ⋅ m]⋅ nˆ dΓ − ∫ [∇ xr ω ⋅ (∇ xr ⋅ m)]dΩ

NOTE: Consider now the domain defined by the volume V , which is bounded by the
r
surface S with the outward unit normal to the surface n̂ . If N is a vector field and T is a
scalar field, it is also true that:

∫ N T,
V
i ij ∫
S

dV = N iT ,i nˆ j dS − N i , jT ,i dV
V

r r r
⇒ N ⋅ (∇ xr (∇ xrT ))dV = (∇ xrT ⋅ N ) ⊗ n̂dS − ∇ xrT ⋅ ∇ xr N dV
∫ ∫ ∫
V S V

where we have directly applied the definition of integration by parts.

University of Castilla-La Mancha Draft By: Eduardo W. V. Chaves (2016)


Ciudad Real - Spain
1 TENSORS 131

Problem 1.127
r r r r
Let b be a vector field, which is defined as b = ∇ xr ∧ v . Show that:

∫ λb nˆ
S
i i ∫
d S = λ, i b i dV
V
r
where λ = λ( x ) represents a scalar field.
r r r
Solution 1: The Cartesian components of b = ∇ xr ∧ v are represented by b i =  ijk v k , j and by
substituting them in the above surface integral we obtain:

∫ λb nˆ
S
i i ∫
dS = λ ijk v k , j nˆ i dS
S

Applying the divergence theorem we obtain:

∫ λb nˆ
S
i i ∫
dS = λ ijk v k , j nˆ i dS
S

= ( ijk λv k , j ), i dV
V


= ( ijk λ, i v k , j +  ijk λv k , ji ) dV
V


= (λ, i  ijk v k , j + λ  ijk v k , ji ) dV = λ, i b i dV
V
1424 3 1424 3 ∫
V
bi 0

Solution 2:

∫ λb nˆ
S
i i ∫
dS = (λb i ), i dV = (λ, i b i + λb i , i ) dV
V

V

note that b i =  ijk v k , j ⇒ b i ,i =  ijk v k , ji =  ijk v k ,ij = 0

∫ λb nˆ
S
i i ∫
dS = λ, i b i dV = λ, i  ijk v k , j dV
V V

Problem 1.128
Let V be a volume domain which is delimited by surface S . a) Show that:
r r
∫ ( x ⊗ nˆ + nˆ ⊗ x) dS = 2V 1
S
(1.135)

where n̂ is the outward unit vector to surface S . b) Show also that:


r r
∫ (∇ r
x ⋅ σ ) ⊗ x dV = ∫ (σ ⋅ nˆ ) ⊗ x dS − σ dV ∫ ∫σ ik , k ∫ ∫
x j dV = σ ik n̂ k x j dS − σ ij dV
V S V V S V

and
r r
∫ x ⊗ (∇ r
x ⋅ σ ) dV = x ⊗ (σ ⋅ nˆ ) dS − σ T dV
∫ ∫ ∫x σi jk , k ∫ ∫
dV = xi σ jk n̂ k dS − σ ji dV
V S V V S V

where σ is an arbitrary second-order tensor field.

University of Castilla-La Mancha Draft By: Eduardo W. V. Chaves (2016)


Ciudad Real - Spain
132 SOLVING PROBLEMS BY MEANS OF CONTINUUM MECHANICS

S r
dS = n̂dS
x2 n̂
V dS
B

r
x

x1
x3

Figure 1.42
Solution:
a) Considering only the first term of the integrand in (1.135), we can obtain:
r r r
∫ ( x ⊗ nˆ ) dS = ∫ ( x ⊗ 1 ⋅ nˆ ) dS = ∫ ( x ⊗ 1) ⋅ nˆ dS
S S S

By applying the divergence theorem we can obtain:


r r r
∫ ( x ⊗ nˆ ) dS = ∫ ( x ⊗ 1) ⋅ nˆ dS = ∫ ∇
S S V
r
x ⋅ ( x ⊗ 1) dV

We will continue the development in indicial notation:

∫ x nˆ
S
i j ∫
dS = x iδ jk nˆ k dS = (δ
S

V
jk xi ) ,k dV = (δ ∫
V
jk , k x i +δ jk xi ,k ) dV

Taking into account that δ jk ,k = 0 j , xi ,k = δ ik , we can conclude that:

r
∫ x n̂ dS = δ ∫ dV = δ ∫ dV = δ ∫ ( x ⊗ nˆ ) dS = V 1
T
i j ji ji jiV = V1 (1.136)
S V V S

r
Similarly, we can conclude that ∫ (nˆ ⊗ x ) dS = V 1 . With that the following is true:
S

r r
∫ ( x ⊗ nˆ + nˆ ⊗ x) dS = 2V 1
S

b) Note that the following is true


( x j σ ik ) ,k = x j ,k σ ik + x j σ ik ,k ⇒ x j σ ik , k = ( x j σ ik ) ,k − σ ij
{
=δ jk
r r
⇒ (∇ xr ⋅ σ ) ⊗ x = ∇ xr ⋅ (σ ⊗ x ) − σ
with that we can obtain:

University of Castilla-La Mancha Draft By: Eduardo W. V. Chaves (2016)


Ciudad Real - Spain
1 TENSORS 133

∫ (∇ r
x
r r
⋅ σ ) ⊗ x dV = ∫ ∇ xr ⋅ (σ ⊗ x ) dV − ∫ σ dV ∫x σ
V
j ik , k ∫
V

dV = ( x j σ ik ) ,k dV − σ ij dV
V
V V V


r r
(∇ xr ⋅ σ ) ⊗ x dV = (σ ⊗ x ) ⋅ nˆ dS − σ dV
∫ ∫ ∫x σ
V
j ik , k ∫
S

dV = x j σ ik nˆ k dS − σ ij dV
V
V S V

r
= (σ ⋅ nˆ ) ⊗ x dS − σ dV
∫ ∫ ∫
S

= (σ ik nˆ k ) x j dS − σ ij dV
V
S V

where we have applied the divergence theorem to the first integral on the right side of
equation.
Taking into account that
[(∇ xr ⋅ σ ) ⊗ xr ]T = [∇ xr ⋅ (σ ⊗ xr ) − σ ]T
r r T
⇒ x ⊗ (∇ xr ⋅ σ ) = [∇ xr ⋅ (σ ⊗ x )] − σ T xi σ jk ,k = ( xi σ jk ) ,k − σ ji

we can obtain:

r r
∫ x ⊗ (∇ r
x ⋅ σ ) dV = ∫ [∇ xr ⋅ (σ ⊗ x )]T ∫
dV − σ T dV ∫x σ i jk , k ∫ ∫
dV = ( xi σ jk ) , k dV − σ ji dV
V V V V V V

r r
∫ x ⊗ (∇ r
x ⋅ σ ) dV = ∫ ( x ⊗ σ ) ⋅ nˆ dS − ∫ σ T dV ∫x σ i jk , k ∫ ∫
dV = ( xi σ jk )nˆ k dS − σ ji dV
V S V V S V

r
= x ⊗ (σ ⋅ nˆ ) dS − σ T dV
∫ ∫ ∫ ∫
= xi (σ jk nˆ k ) dS − σ ji dV
S V S V

NOTE: If we obtain the trace of the equation (1.136) we can also obtain:
r r
∫ x n̂
S
i i dS = δ jiδ jiV = δ iiV ∫ ( x ⊗ nˆ ) : 1 dS = ∫ ( x ⋅ nˆ ) dS = V 1 : 1
S S
(1.137)

If we are dealing with a three dimensional case (3D) the trace δ ii = 3 , and if we are dealing
with two dimensional case (2D) we have that δ ii = 2 . With that we can conclude that:

r
∫ x nˆ
S
i i dS = 3V ∫ ( x ⋅ nˆ ) dS = 3V
S
(3D case)

and
r
∫Γ x nˆ
i i dΓ = 2 A ∫Γ ( x ⋅ nˆ ) dΓ = 2 A (2D case)

University of Castilla-La Mancha Draft By: Eduardo W. V. Chaves (2016)


Ciudad Real - Spain
134 SOLVING PROBLEMS BY MEANS OF CONTINUUM MECHANICS

Ω n̂
x2

A : area of the domain Ω


Γ
x1

Figure 1.43: Two dimensional case – 2D.

Problem 1.129
GM
Let φ be a scalar field which is given by φ = − r , where G and M are scalars and
a
r r r
constants, and a is the magnitude of the vector a ≠ 0 . a) Obtain the gradient of φ . b)
r r
Obtain the gradient of φ for the particular case when a = x and draw the field ∇ xr φ in the
Cartesian space.
Solution:
 − GM   
(∇ xr φ ),i ≡  ∂φr  ≡ φ ,i =  r
 a
 = −GM  − 1  ( ar ) ,i
 r
 a 2
(1.138)
 ∂x  ,i   ,i  
r
The term ( a ),i can be expressed as follows:

r  r r 1 −1
1 r r 2 r r 1 r r 2
−1
( a ) ,i =  ( a ⋅ a )  = (a ⋅ a ) ( a ⋅ a ) ,i = ( a ⋅ a ) ( a k a k ) ,i
2

  ,i 2 2
−1 −1
1 r r r r 1
= (a ⋅ a ) 2 ( a k ,i a k + a k a k ,i ) = (a ⋅ a ) 2 ( a k ,i a k ) = r ( a k , i a k )
2 a
or in indicial notation:
r 1 r r
∇ xr ( a ) = r (a ⋅ ∇ xr a ) (1.139)
a

Then, the equation (1.138) becomes:


 −1  r  1  1
(∇ xrφ ),i ≡  ∂φr  GM GM r r
≡ φ ,i = −GM  r 2  ( a ),i = GM  r 2  r (ak ,i ak ) = r 3 (ak ,i ak ) = r 3 (a ⋅ ∇ xr a )i
   
 ∂x ,i  a   a 
a a a
r
r a
Moreover, considering that the unit vector according to the direction a is given by aˆ = r ,
a
we can obtain:

(∇ xr φ ),i = GM r r
r GM r
r
r 3 (a ⋅ ∇ x a ) i = r 2 (aˆ ⋅ ∇ x a ) i (1.140)
a a
r r
b) For the particular case when a = x we have:

University of Castilla-La Mancha Draft By: Eduardo W. V. Chaves (2016)


Ciudad Real - Spain
1 TENSORS 135

r 1 1 1 r
( x ) ,i = r ( x k ,i x k ) = r (δ ki x k ) = r ( xi ) where r = x = x12 + x 22 + x32
x x x
or in tensorial notation:
r 1 r r 1 r 1 r
∇ xr ( x ) = r ( x ⋅ ∇ xr x ) = r ( x ⋅ 1) = r ( x ) = xˆ
x x x

whereupon
 − GM    r
(∇ xr φ )i ≡  ∂φr  ≡ φ ,i =  r
 x
 = −GM  − 1
  xr 2
 ( x ) = GM ( xr )
 ,i r3 i (1.141)
 ∂x  i   ,i   x

or in tensorial notation:
 − GM  GM r GM
∇ xr φ = ∇ xr  r = x = r 2 xˆ (1.142)
 x  xr 3 x
 
r
Note that the vector field ∇ xr φ is radial, i.e. it is normal to the spheres defined by x and
r 2
decreases with x = r 2 , (see Figure 1.44). The equation (1.142) can also be written as
follows:
 − GM  GM ˆ ∂  − GM  ˆ ∂φ(r ) ˆ
∇φ = ∇  = 2 r=  r = r = φ ′(r )rˆ (1.143)
 r  r ∂r  r  ∂r

x3

Spheres

x̂ ∇ xr φ
xˆ = 1

r
x

r x2
b
∇ xr φ
∇φ

x1
∇ xr φ

Figure 1.44

University of Castilla-La Mancha Draft By: Eduardo W. V. Chaves (2016)


Ciudad Real - Spain
136 SOLVING PROBLEMS BY MEANS OF CONTINUUM MECHANICS

GM
NOTE: The function φ = − r represents the gravitational potential which has the
x
r m3
following property b = −∇ xr φ , (see Figure 1.44), where G = 6.67384 × 10 −11 is the
kg s 2
gravitational constant, M is the total mass of the planet. We check the units:
 GM  m3 kg kg m m N m J
[φ ] =  − r  = 2
= 2 = = (Unit of energy per unit mass)
 x  kg s m s kg kg kg (specific energy)

r  ∂φ  J Nm kgm m
[b] = [−∇ xrφ ] =  r  = = = 2 = 2 (Unit of force per unit mass)
 ∂x  m kg m kg s kg s (unit of acceleration)

r r r r
It is interesting to check that ∇ xr ∧ b = ∇ xr ∧ [−∇ xrφ ] = 0 , (see Problem 1.108).
r
We can obtain b on the Earth surface by means of
r GM
b = −∇ xr φ = − r 2 xˆ
x

where the total mass of Earth is M ≈ 5.98 × 10 24 kg and the approximate radius is
R ≈ 6.37 × 10 6 m , with that we obtain
r GM GM
b = − r 2 xˆ = − 2 xˆ ≈ −9.82 xˆ
x R
r m
and its module is denoted by g = b ≈ 9.82 .
s2
r
Adopting that the system x ′ has its origin at the center of mass of the body M , and invoking
r r
the Newton’s second law ( F = ma ), we can obtain the force that act in a body ( m ) due to the
r
gravitational field b = −∇ xr φ :
r r r GMm
F = ma = mb = − r 2 xˆ ′ (1.144)
x′
r
We can express the above equation in a generic system ( x ), (see Figure 1.45).
r
Then, for the system x the force is given by:
r r r
GMm ( x ( m) − x ( M ) )
F ( mM ) = − r r Newton’s law of “universal” gravitation (1.145)
r r 2
x ( m) − x ( M )
x ( m) − x ( M )
r
where we have adopted the nomenclature F (mM ) to indicate the force in m due to the
influence of M . Note also that in M we have the same force in direction and magnitude, but
r
of opposite sense to F (mM ) .

University of Castilla-La Mancha Draft By: Eduardo W. V. Chaves (2016)


Ciudad Real - Spain
1 TENSORS 137

x 2′

M x1′
r
r x′ r
x3′ F (Mm) F (mM )
m

r
x (M ) r
x (m )
x2 r r r
x ( M ) + x ′ = x ( m)
r r r
⇒ x ′ = x (m) − x (M )

x1
x3

Figure 1.45

Problem 1.130
1 r
Consider that φ = where r = x = x12 + x 22 + x32 :
r
a) Show that:
r r ∂ 2φ ∂ 2φ ∂ 2φ
∇ xr ⋅ [∇ xrφ ( x − 0)] ≡ ∇ 2φ ≡ 2 + 2 + 2 = 0 Laplace equation (1.146)
∂x1 ∂x2 ∂x3
r r r r
for r ≠ 0 . We use the nomenclature [∇ xrφ ( x − 0)] to indicate that the origin ( x = 0 ) is not
included.
b) Given a closed surface S containing the origin, show that:

∫ (∇ φ ) ⋅ nˆ dS = −4π
S
r
x (1.147)

where n̂ is the outward unit vector to surface.


Solution:
It was obtained in Problem 1.129 that
 − GM  GM r GM
∇ xr φ = ∇ xr  r = x = r 2 xˆ (1.148)
 x  xr 3 x
 
Denoting by GM = −1 we obtain:
 1  −1 r −1
∇ xr φ = ∇ xr  r  = r 3 x = r 2 xˆ (1.149)
 x  x x
 
or in indicial notation:

University of Castilla-La Mancha Draft By: Eduardo W. V. Chaves (2016)


Ciudad Real - Spain
138 SOLVING PROBLEMS BY MEANS OF CONTINUUM MECHANICS

 −1 r −1
(∇ xr φ ) i =  r 3 x  = r 3 xi (1.150)
 x  x
 i
By calculating the divergence of the previous relationship we can obtain:
−x  x  1  x  −3 r 
∇ xr ⋅ (∇ xrφ ) = φ , ii =  r 3i  = − ri ,i3 − xi  r 3  = − ri ,i3 − xi  r 4 ( x ), i  (1.151)
 x  x  x  x  x 
 , i  ,i
r 1 r
In Problem 1.129 it was shown that ∇ xr ( x ) = r ( x ) , in addition, note that xi ,i = δ ii = 3 ,
x
with that we can obtain:

3  −3 r  3  −3 x  3 3x x
∇ xr ⋅ (∇ xrφ ) = − r 3 − xi  r 4 ( x ),i  = − r 3 − xi  r 4 ri  = − r 3 + ri 5i
x  x  x  x x  x x
r2 (1.152)
3 3x
=− r 3 + r 5 =0
x x

c) We adopt an arbitrary sphere of radius r , whose surface area is 4πr 2 . Then:


 −1  −1 −1 −1 −1
∫ (∇ φ ) ⋅ nˆ dS = ∫ 
r
x r2
S x
xˆ  ⋅ nˆ dS = r 2 xˆ ⋅ nˆ dS = r 2 dS = 2 × ( Area) = 2 × (4πr 2 ) = −4π
 x S
∫ x S
∫ r r
S 
(1.153)
Note that xˆ ⋅ nˆ = 1 since for the sphere it holds that xˆ // nˆ .
It is interesting to note that by means of the divergence theorem it fulfills that:

∫∇ r
x ⋅ [∇ xrφ ]dV = ∫ (∇ xrφ ) ⋅ nˆ dS ∫φ ,ii dV ∫
= φ ,i ni dS (1.154)
V S V S

r r r r
We have shown that ∇ xr ⋅ [∇ xrφ ( x − 0)] = 0 , but that only apply to x ≠ 0 (the origin is not
included). That is, taking into account the result in (1.153), the result in (1.154) has consistency
r r
if at the point x = 0 there is a sink and equal to ( − 4π ). With that, it is very intuitive to
conclude that any closed surface that does not contain the origin the following holds

S
∫ (∇ xrφ ) ⋅ nˆ dS = 0 , (see Parker (2003)).

Problem 1.131
a) Show that:

∫ (∇φ ) ⋅ nˆ dS = 4πGM (r )
S
(1.155)

 − GM 
where φ =   is the gravitational potential, and M (r ) is the total mass contained into
 r 
the sphere whose radius is r , and S -surface represents the sphere boundary.
b) Consider a sphere of radius r = a which represents a planet. Obtain the total mass of the
planet in function of the mass density ρ = ρ (r ) .

University of Castilla-La Mancha Draft By: Eduardo W. V. Chaves (2016)


Ciudad Real - Spain
1 TENSORS 139

c) Obtain the gravitational potential for r < a and r ≥ a . In this section, consider that the
mass density is uniform in the planet ρ = ρ 0
Solution:
a) In Problem 1.130 we have shown that:
  1 
∫ (∇ φ ) ⋅ nˆ dS = ∫ ∇ r  ⋅ nˆ dS = −4π
S S
(1.156)

By multiply both sides of the equation by GM (r ) we can obtain:


  1    − GM (r )  
− GM (r ) ∇    ⋅ nˆ dS = 4πGM (r )
∫ ⇒ ∫ ∇   ⋅ nˆ dS = 4πGM (r )
r
S   
r 
S (1.157)
⇒ ∫ [∇φ ]⋅ nˆ dS = 4πGM (r )
S

b)

Spherical planet

r=a

ρ (r )
r

Figure 1.46

The total mass is obtained as follows:


M = ρ (r )dV
V
(1.158)

Note that Vsphere = 43 πr 3 ⇒ dV = 43 π3r 2 dr = 4πr 2 dr . Then:


r =a


M = ρ (r )dV = ∫ ρ (r )4πr
2
dr (1.159)
V r =0

c) Remember that in Problem 1.129, (see equation (1.143)), we have obtained that
r  − GM  GM ˆ ∂  − GM  ˆ ∂φ(r ) ˆ
∇φ = −b = ∇  = 2 r =  r = r = φ ′(r )rˆ (1.160)
 r  r ∂r  r  ∂r
By using the equation in (1.157) we can obtain:

University of Castilla-La Mancha Draft By: Eduardo W. V. Chaves (2016)


Ciudad Real - Spain
140 SOLVING PROBLEMS BY MEANS OF CONTINUUM MECHANICS

∫ [∇φ ]⋅ nˆ dS = 4πGM (r )
S
r
⇒ − b ⋅ nˆ dS = φ ′(r ) rˆ ⋅ nˆ dS = φ ′(r ) dS = φ ′( r )(4πr 2 ) = 4πGM (r )
∫ ∫ ∫
123 (1.161)
S S =1 S

GM (r )
⇒ φ ′(r )r 2 = GM (r ) ⇒ φ ′(r ) =
r2
where M (r ) = Vρ 0 = 43 πr 3 ρ 0 . Then:
GM (r ) 4Gπρ 0 dφ ( r ) 4Gπρ 0 4Gπρ 0
φ ′(r ) = = r ⇒ = r ⇒ ⇒ dφ ( r ) = rdr (1.162)
r2 3 dr 3 3
By integrating the above equation we can obtain:
4Gπρ 0 4Gπρ 0 r 2 2Gπρ 0 2
∫ dφ = ∫ 3
rdr ⇒ φ (r ) =
3 2
+C ⇒ φ (1) (r ) =
3
r +C (1.163)

where we have denoted φ (1) ( r ) = φ(r ) for r < a . For values r ≥ a the gravitational potential is
given by
− GM − 4Gπa 3 ρ 0
φ= = = φ (2) for r≥a (1.164)
r 3r
where M is the total mass of the planet whose value is M = Vρ 0 = 43 πa 3 ρ 0 . Note that the
potential φ has to be continuous in r = a , (see Parker (2003)), thus:
2Gπρ 0 2 − 4Gπa 3 ρ 0
φ (1) (r = a ) = φ ( 2) ( r = a ) ⇒
a +C =
3 3a
(1.165)
− 2Gπa ρ 0 − 2Gπa ρ 0 4 3 − 2GM 3 − 3MG
3 3
⇒C = = = =
a a 34 a 4 2a
With that the equation (1.163) becomes
2Gπρ 0 2 2Gπρ 0 2 3MG MG 2 3MG MG  r 2 3
φ (1) (r ) = r +C = r − = 3r − = 2  2 −  (1.166)
3 3 2a 2a 2a 2a  2a 2
We summarize the gravitational potential, (see Figure 1.47 and Figure 1.48), as follows:
 MG  r 2 3
φ(r ) =  −  for r<a
2 
 2a  2a 2
2
 (1.167)
 MG
φ(r ) = r
for r≥a

University of Castilla-La Mancha Draft By: Eduardo W. V. Chaves (2016)


Ciudad Real - Spain
1 TENSORS 141

φ (r )

Planet surface

− MG
a
inflection point

− 3MG
2a

Figure 1.47: Gravitational potential vs. radius.

Figure 1.48: Gravitational potential (Ref. Wikipedia: “Gravitational potential”).

Problem 1.132
a) Show that the orbit of a planet takes place on a plane. b) Prove the Kepler’s laws of
planetary motion:
b.1) First Law: The orbit of a planet is described by an ellipse, with the Sun at one of the foci
of the ellipse;
b.2) Second Law: The vector position from the Sun to the planet describes an area at a constant
rate;
b.3) Third Law: If T (orbital period) represents the time required for the planet to perform a
full elliptical orbit, whose major axis of the ellipse is 2a , the relationship T 2 = κa 3 holds,
where κ is a constant.

University of Castilla-La Mancha Draft By: Eduardo W. V. Chaves (2016)


Ciudad Real - Spain
142 SOLVING PROBLEMS BY MEANS OF CONTINUUM MECHANICS

Reminder: Expressions related to the ellipse, (see Figure 1.49):


r p
Equation of the ellipse: x = r =
1 + e cos θ

a 2 − b2 p2
Eccentricity: e = ; 0 < e < 1 , where a 2 = holds.
a2 (1 − e 2 ) 2
Area enclosed by an ellipse: A = πab

x2

r
x
b
θ f2

f1 x1
b

a a

Figure 1.49
Solution:

M - Mass of Sun
m - Mass of the planet
r
x
xˆ = r
x
x2 r r r
r dx r d 2 x dv
v= , a= 2 =
x3 r r r dt dt dt
r c = x∧v
dx r r
=v a // xˆ
dt

r r r
x F // a
θ t=0
Sun r
h x1

Figure 1.50: Orbit of the planet.

r
a) To show that the orbit takes place on a plane, we must prove that the vector ( c ) normal to
r r
the plane which is defined by the vectors x̂ and v does not change with time, i.e. c is
constant, (see Figure 1.50).

University of Castilla-La Mancha Draft By: Eduardo W. V. Chaves (2016)


Ciudad Real - Spain
1 TENSORS 143

We recall the equation (1.144) obtained in Problem 1.132:


r r r GMm r GM
F = ma = mb = − r 2 xˆ ; a = − r 2 xˆ (1.168)
x x
r r r
Next, we obtain the rate of change of c = x ∧ v :
r
dc d r r d r r r d r r r r r r
= ( x ∧ v ) = ( x ) ∧ v + x ∧ (v ) = v12
∧3v + 1∧3
x2 a =0
dt dt dt dt r
=0
r
=0
r r r
Thus we have shown that the vector c = x ∧ v does not change with time, which implies that
the orbit takes place on a plane.
b.1) First Law
Since the planet’s orbit is performed on a plane, we take x1 − x 2 as the plane of the orbit, then
r
the vector c has the same direction as x3 , (see Figure 1.50).
r
We express c in term of x̂ :
r r
r dx d r d( x ) r dxˆ
v= = ( x xˆ ) = xˆ + x
dt dt dt dt
and
r r
r r r r  d( x ) r dxˆ  r d( x ) r 2 dxˆ r 2 dxˆ
c = x ∧ v = ( x xˆ ) ∧  xˆ + x = x
 ∧3xˆ + x xˆ ∧
xˆ 2
1 = x xˆ ∧
 dt dt  dt r
=0
dt dt

r GM r r
Taking into account that a = − r 2
xˆ , we calculate the vector a ∧ c which has the same
x
r r r r
direction as v , i.e. (a ∧ c ) // v :

r r  GM   r 2 dxˆ   dxˆ   dxˆ dxˆ 


a ∧ c =  − r 2 xˆ  ∧  x xˆ ∧  = −GM xˆ ∧  xˆ ∧  = −GM ( xˆ ⋅ ) xˆ − ( xˆ ⋅ xˆ ) 
 x   dt   dt   dt dt 
 
dxˆ
= GM
dt
r r r r r r r r r
where we have used the property a ∧ (b ∧ c) = (a ⋅ c )b − (a ⋅ b)c , (see Problem 1.17). Note
dxˆ dxˆ
also that it holds that xˆ ⋅ = 0 since xˆ ⊥ , and xˆ ⋅ xˆ = xˆ = 1 . Considering that GM is a
2

dt dt
constant, the following is true:
r r dxˆ d (GM xˆ )
a ∧ c = GM =
dt dt
r
Since the vector c does not change with time, the following is true:
r r r
r r dv r d ( v ∧ c )
a∧c = ∧c =
dt dt
Thus
r r
d (v ∧ c ) d (GM xˆ )
=
dt dt
Integrating over time the above equation we can obtain:
r r r
v ∧ c = GM xˆ + h

University of Castilla-La Mancha Draft By: Eduardo W. V. Chaves (2016)


Ciudad Real - Spain
144 SOLVING PROBLEMS BY MEANS OF CONTINUUM MECHANICS

r r
where h is constant vector of integration and is independent of time. Note that h is located
r r
on the plane x1 − x 2 , since (v ∧ c ) and x̂ are also on the plane x1 − x 2 , (see Figure 1.50).
We calculate:
r r
h ⋅ xˆ = h xˆ cos θ = h cos θ
r
where we have denoted by h = h . Then:
rr r r r r r r r
= c ⋅ c = ( x ∧ v ) ⋅ c = (v ∧ c ) ⋅ x
2
c2 = c
( r r
) r r r r r
= GM xˆ + h ⋅ ( x xˆ ) = x GM xˆ ⋅ xˆ + x h ⋅ xˆ = x GM + x h cos θ
r
= x (GM + h cos θ) = r (GM + h cos θ)
r
where we have considered that r = x . Then, we can obtain the following equation of the
ellipse:
c2
c2 GM p
⇒r= = =
(GM + h cos θ) (GM + h cos θ ) 1 + e cos θ
GM
where we have considered that:
c2 h
p= and e= (1.169)
GM GM
b.2) Second Law

r 1 r r 1 r r
A = x ∧ ∆S ∆
S →0
→ dA = x ∧ ds
2 2
x2
∆S

A
r
x

x1

Figure 1.51

r
The rate of change of dA becomes:
r r r r r
D(dA) 1 D( x ∧ ds ) 1 D( x ) r 1 r D ( ds )
= = ∧ ds + x ∧
Dt 2 Dt 2 Dt 2 Dt
r
1 D( x ) r 1r r 1r
= ∧ ds + x ∧ v = c (constant)
2 1Dt
4243 2 2
r
=0

University of Castilla-La Mancha Draft By: Eduardo W. V. Chaves (2016)


Ciudad Real - Spain
1 TENSORS 145

and its magnitude:


r
D (dA) D(dA) 1 r 1
= = c = c
Dt Dt 2 2

NOTE: As a consequence of second law it follows that if the areas of two sectors are equal,
the time required to perform their paths are equal, that is, according to Figure 1.52 as the areas
of the sectors OCD and EFO are equal the times to perform C → D and E → F are equal.
As result, when the planet is closer to the Sun its velocity is greater than when it is far.

sector EFO sector OCD


E
D
O
A A

C
F

Figure 1.52: Orbit of the planet.

b.3) Third Law


If T is the total time for a complete orbit (orbital period), we can obtain:
T T
D (dA) 1 1
A=
0
∫ Dt
dt =
0
2 ∫
c dt = cT
2

1
Taking into account the area enclosed by the ellipse: A = πab , we conclude that cT = πab ,
2
thus:
2πab 4π 2 a 2 b 2
T= ⇒ T2 = (1.170)
c c2
Considering the equation of the eccentricity, we can obtain:
a 2 − b2
e= ⇒ b2 = a 2 − a 2e2 ⇒ b 2 = a 2 (1 − e 2 )
a2
p2 p
and taking into account a 2 = 2 2
⇒a= 2
⇒ (1 − e 2 )a = p into the above equation,
(1 − e ) (1 − e )
we can obtain:
b2
b 2 = a 2 (1 − e 2 ) ⇒ b 2 = ap ⇒ p=
a
Whereby the equation (1.170) can be rewritten as follows:

University of Castilla-La Mancha Draft By: Eduardo W. V. Chaves (2016)


Ciudad Real - Spain
146 SOLVING PROBLEMS BY MEANS OF CONTINUUM MECHANICS

4π 2 a 2 b 2 4π 2 a 2 ab 2 4π 2 a 3 p 4π 2 3
T2 = 2
= 2
= 2
= a = κ a3 (1.171)
c c a c GM
p 1
where we have considered that 2
= , (see equation (1.169)).
c GM

COMPLEMENTARY NOTE 1

Geometrical Properties of Curves


Let us consider the curve defined by the function y = y (x) , (see Figure 1.53), we denote by:
dy ( x)
First derivative: ≡ y′ ≡ y, x (tangent of the curve at a point)
dx
d 2 y ( x)
Second derivative: ≡ y′′ ≡ y, xx .
dx 2
Infinitesimal arc length ds :
According to Figure 1.53 we can obtain:
2
∆x 2 (∆x 2 + ∆y 2 )  ∆y 
∆s = ∆x 2 + ∆y 2 = (∆x 2 + ∆y 2 ) = ∆x = 1 +   ∆x
 ∆x 
2 2
∆x ∆x
Then, we define the differential arc-length element as follows:
 2  2
[ ]
1
  ∆y    dy 
ds = lim  1 +   ∆x  = 1 +   dx ≡ 1 + ( y ′) 2 dx = 1 + ( y ′) 2 2 dx
∆x →0  ∆x    dx 
 

[ ]
1
ds
⇒ = 1 + ( y ′) 2 2
dx

y ( x) ∆s
y ∆y

∆x

∆s 2 = ∆x 2 + ∆y 2
y′
1

Figure 1.53

University of Castilla-La Mancha Draft By: Eduardo W. V. Chaves (2016)


Ciudad Real - Spain
1 TENSORS 147

Curvature
Curvature measures how quickly the direction of ŝ changes with respect to a change in arc
length s , where ŝ is the unit vector according to the ( y ′) -direction, (see Figure 1.57). So, we
define the vector curvature as follows:
r dsˆ r dsˆ
κ= curvature
 → κ= κ =
ds ds
where κ( x) is the curvature of the curve at point x .
dy
Let us consider that there is an angle ψ such as tan(ψ ) = ≡ y′ and if we differentiate with
dx
respect to x we obtain:
d [tan(ψ )] dψ
d  dy  d
  = [tan(ψ )] =
dx  dx  dx dψ dx
= sec 2 (ψ )

dx
= 1 + tan 2 (ψ )

dx
[ ]
dψ   dy   dψ
[ ] [ ]
2
d2y dψ
⇒ 2
≡ y ′
′ = 1 + tan 2
(ψ ) = 1 +    = 1 + ( y′) 2
dx dx   dx   dx dx
dψ y′′
⇒ =
dx [1 + ( y′) 2 ]
The curvature can be obtained as follows:
dψ dψ dx y′′ 1 y′′
κ= = = =
ds dx ds [1 + ( y′) 2 ] 1 3
[1 + ( y′) 2 ] 2 [1 + ( y′) 2 ] 2
dx 1
where = 1
holds.
ds
[1 + ( y′) 2 ] 2
Note that the curvature of a circumference is constant, (see Figure 1.54):

κ=
ds
= ⇒ κds = dψ integratin
 g → ∫ κds = ∫ dψ
(1.172)
2π 1

⇒ κ ds = dψ ∫ ⇒ ⇒ κ2πr = 2π ⇒ κ= =
2πr r
where ( 2πr ) is the length of the circumference of radius r .
If we consider Figure 1.54 we can conclude that the curvature of the circumference of radius
r (1) is greater than the circumference of radius r ( 2 ) :
1 1
r (1) < r ( 2 ) ⇒ (1)
> ⇒ κ (1) > κ ( 2 )
r r ( 2)

University of Castilla-La Mancha Draft By: Eduardo W. V. Chaves (2016)


Ciudad Real - Spain
148 SOLVING PROBLEMS BY MEANS OF CONTINUUM MECHANICS

κ(1) > κ( 2 )

∆ψ

r ( 2) ∆ψ

∆ψ
r
κ (1)
r ( 2) r (1)
κ

Figure 1.54

By considering the curvature, (see equation (1.172)), we can obtain:


B

∫ ∫
κds = dψ = ψ B − ψ A ≡ ∆ψ B _ A
A

Then, the area defined in Figure 1.55 can be obtained by Area = ∫ κds = ψ B − ψ A ≡ ∆ψ B _ A .

∆ψ B _ A

Area = κds = ψ B − ψ A ≡ ∆ψ B _ A

A
κ
κA Area

ψA B

κB ψB

Figure 1.55

University of Castilla-La Mancha Draft By: Eduardo W. V. Chaves (2016)


Ciudad Real - Spain
1 TENSORS 149

For example, let us consider a circumference of radius r , and the variation of angle from A
to B can be obtained as follows Area = ψ B − ψ A ≡ ∆ψ B _ A , (see Figure 1.56):

π
1 ∆ψ B _ A =

Area = κds = [(2πr ) κ]
4
B 2

1 1 π
= [(2πr ) ] =
4 r 2
= ψ B − ψ A ≡ ∆ψ B _ A
r

Figure 1.56

The Curvature Vector is defined as follows


r dŝ
κ=
ds
where ŝ is the unit vector (tangent to the curve), and if we use the unit vector properties we
can conclude that:
2 d (sˆ ⋅ sˆ ) d (1)
sˆ = 1 sˆ = sˆ ⋅ sˆ = 1 ⇒ = =0
ds ds
d (sˆ ) ˆ ˆ d (sˆ ) dsˆ ˆ r r
⇒ ⋅s + s⋅ =2 ⋅s =0 ⇒ κ ⋅ sˆ = 0 ⇒ κ ⊥ sˆ
ds ds ds

y
y ( x) ŝ

dŝ

ds dŝ
ds

Figure 1.57

University of Castilla-La Mancha Draft By: Eduardo W. V. Chaves (2016)


Ciudad Real - Spain
150 SOLVING PROBLEMS BY MEANS OF CONTINUUM MECHANICS

Geometrical relations among coordinate increments


r r
The transformation matrix from the system x to the system n̂ - ŝ (which is denoted by x ),
(see Figure 1.58), is given by:
 cosα sin α   nˆ 1 nˆ 2 
aij =  = 
 − sin α cosα  − nˆ 2 nˆ 1 
Then, it fulfils that:
dxi = aij dx j ⇒ dxi = a ji dx j
T
 dx1   nˆ 1 nˆ 2   0   nˆ 1 − nˆ 2   0  − nˆ 2 ds 
 = ˆ   =   =  
dx2  − n 2 nˆ 1  ds  nˆ 2 nˆ 1  ds   nˆ 1ds 
With that we can obtain:
 dx2 
nˆ 1   ds  dx j
 ˆ  =  dx  ⇒ nˆ i =  3ij
n 2  − 1  ds
 ds 
where  kij is the permutation symbol.

dx j
x2 x2 nˆ i =  3ij (i, j = 1,2)
x2 x1 ds
 dx2 
ŝ n̂ nˆ 1   ds 
nˆ i =   =  
nˆ 2  − dx1 
α  ds 
dx2 ds x1  kij - permutation symbol
ds - arc-length
− dx1

x1

Figure 1.58

University of Castilla-La Mancha Draft By: Eduardo W. V. Chaves (2016)


Ciudad Real - Spain
1 TENSORS 151

COMPLEMENTARY NOTE 2
Geometrical Center (Centroid – C.Geo.)
Let us consider V = ∫ dV the volume delimited by the surface S , and also let us consider the
V
r r
systems x and x ′ , (see Figure 1.59). By means of vector summation we can obtain:
r r r
x = x + x′
By integrate over the volume we can obtain:
r r r r r

V

xdV = ( x + x ′)dV = xdV + x ′dV
V

V

V
r
The volume centroid is the point ( x (V ) ) where the following equation fulfils:
r r r r

V
∫ x ′dV = 0 ⇒ x = x (V )

Then, the centroid can be calculated as follows:


r r
r r r r r r ∫ xdV ∫ xdV
∫ ∫ ∫ ∫ ∫ x (V ) = V V
xdV = x (V ) dV + x ′dV = x (V ) dV = x (V ) dV ⇒ =
V V V
12r3 V V ∫ dV
V
V
=0

x3′

S x′2
dV
x3
r
x2 x′
r x1′
x

r
x (V )

x1

Figure 1.59

The components of the volume centroid ( x1(V ) , x 2(V ) , x3(V ) ) can be obtained as follows:

∫ x dV 1 ∫ x dV 2 ∫ x dV 3

x1(V ) = V
; x 2(V ) = V
; x3(V ) = V
Volume Centroid (1.173)

V
dV ∫
V
dV ∫
V
dV

University of Castilla-La Mancha Draft By: Eduardo W. V. Chaves (2016)


Ciudad Real - Spain
152 SOLVING PROBLEMS BY MEANS OF CONTINUUM MECHANICS

where

∫ x dV
V
1 is the first moment of the volume about the x1 axis;

∫ x dV
V
2 is the first moment of the volume about the x 2 axis;

∫ x dV
V
3 is the first moment of the volume about the x 3 axis.

Note that, if the volume is given by V = V ( A) + V ( B ) then the following is true:


r r
r ∫ xdV ∫ xdV r r
x (V ) = V =V ⇒ V x (V ) = xdV ∫
∫ dV
V
V V

r r r r r r A r B
⇒ V x (V ) = xdV = ∫ ∫ xdV = ∫ xdV ( A) + ∫ xdV ( B ) = V ( A) x (V ) + V ( B ) x (V )
V V ( A ) +V ( B ) V ( A) V (B)

where we have used the definitions:


r r
∫ xdV ∫ xdV
( A) (B)

r A ( A) r B (B)
x (V ) = V ; x (V ) = V
∫ dV ( A) ∫ dV
( B)

V ( A) V (B)

In a same fashion to the derivation of the equations in (1.173), we can also define the area
centroid as follows:

∫ x dA 1 ∫ x dA 2 ∫ x dA 3

x1( A) = A
; x 2( A) = A
; x3( A) = A
Area centroid (1.174)

A
dA ∫
A
dA ∫
A
dA

and the line centroid:

∫ x dL 1 ∫ x dL 2 ∫ x dL 3

x1( L ) = L
; x 2( L ) = L
; x3( L ) = L
Line centroid (1.175)
∫ dL
L
∫ dL
L
∫ dL
L

Center of the Scalar Field of a Domain


r
Let us consider the scalar field ( φ = φ ( x ) ), by means of vector summation we can write:
r r r r r r
φx = φx (V _ φ ) + φx ′′ ∫ φxdV = ∫ φx dV + ∫ φx ′′dV
(V _ φ )
Integratin
 g →
V V V
r
The center of the scalar field φ = φ ( x ) delimited by the domain V is defined by:
r r
φx ′′dV = 0 ∫
V

With that we can conclude that:

University of Castilla-La Mancha Draft By: Eduardo W. V. Chaves (2016)


Ciudad Real - Spain
1 TENSORS 153

r r
r r (V _ φ ) r ∫ φxdV ∫ φxdV
∫ φxdV = ∫ φx dV ⇒ x (V _ φ ) = V =V
V V ∫ φdV
V
V (φ )

whose components are:

∫ φx dV 1 ∫ φx dV 2 ∫ φx dV 3

x1(V _ φ ) = V
; x 2(V _ φ ) = V
; x 3(V _ φ ) = V

∫ φdV
V
∫ φdV
V
∫ φdV
V

Note that, if the scalar field is uniform inside the volume, the center of the scalar field and the
geometrical center are the same:
r r r
r ∫ φxdV φ ∫ xdV ∫ xdV r
x (V _ φ ) = V = V
=V = x (V )
∫ φdV
V
φ ∫ dV
V V
∫ dV
Similarly we can define the center of the scalar field into an Area:
r
r ∫ φxdA
x (A _φ) = A

∫ φdA
A

and the center of a scalar field of the curve


r
r ∫ φxdL
x (L _φ) = L

∫ φdL
L

If the scalar field φ represents the mass density ( ρ ) the center of the scalar is denoted by
Center of Mass (C.M.).
r
r ∫ ρ xdV
x (V _ ρ ) = V Center of mass (defined by a volume) (1.176)
∫ ρ dV
V

University of Castilla-La Mancha Draft By: Eduardo W. V. Chaves (2016)


Ciudad Real - Spain
154 SOLVING PROBLEMS BY MEANS OF CONTINUUM MECHANICS

Center of Vector Field delimited by a Domain


r r r
Consider that the body of volume V is subjected to the vector field b = b( x ) , and by a scalar
r
field φ ( x ) , (see Figure 1.60).

r r
b( x )

x3′′′

x2′′′
dV
x3
r
x2 x′
r
x x1′′′

r
φ ( x)
r r
x (φb )

x1

Figure 1.60

r r
By considering the systems x and x′′′ , we can obtain:
r r r r r r r r r r r
x = x (φb ) + x′′′ ⇒ φx ⋅ b = φx (φb ) ⋅ b + φx′′′ ⋅ b
r r r r r r r
⇒ φ x ⋅ bdV = φx (φb ) ⋅ bdV + φx′′′ ⋅ bdV
∫ ∫ ∫
V V V

The center of the vector field delimited by the volume V is defined by:
r r
∫ φx ′′′ ⋅ bdV = 0
V
(1.177)

Then
r r r r r r r  r  r r r r r r r r
∫ φx ⋅ bdV = ∫ φx ⋅ bdV = x ⋅  ∫ φbdV  = x ⋅ F x (φb ) ⋅ F = φx ⋅ bdV

(φb ) (φb ) (φb )

 
V V V  V

where we have denoted by


r r
F = φbdV ∫
V
(1.178)
r r r
Note that F is the resultant force and is located at the point x (φb ) .
r
If the arbitrary field b is uniform inside of volume V we can obtain:

University of Castilla-La Mancha Draft By: Eduardo W. V. Chaves (2016)


Ciudad Real - Spain
1 TENSORS 155

r r r r  r   r  r r r r 
∫φx ⋅ bdV = x ⋅  ∫φbdV  ⇒  φxdV  ⋅ b = x (φb ) ⋅ b φdV 
∫ ∫
(φb )
     
V V  V  V 
 r r  r r r  r r r r
⇒  φdV  x (V _ φ ) ⋅ b =  φdV  x (φb ) ⋅ b ⇒  φdV  x (V _ φ ) − x (φb ) ⋅ b = 0
 ∫  ∫    ∫ ( )
V  V  V 
r (V _ φ ) r (φbr ) r
⇒x −x =0
r (V _ φ ) r (φbr )
⇒x =x
and if in addition the scalar field φ is uniform we can obtain:
r r r r
x (V _ φ ) = x (φb ) = x (V )

r r
if φ is uniform ⇒ x (V _ φ ) = x (V )
r r r r
if b is uniform ⇒ x (V _ φ ) = x (φb )
r r r r r
if b and φ are uniform ⇒ x (V _ φ ) = x (φb ) = x (V )
r r
b( x )

x3′′′ x2′′′

x3′ x′2
x3 x1′′′
x1′
r r
x (φb )
x3′′ x 2′′
r
x2 x (V )

r x1′′
x (V _ φ ) r
φ ( x)

x1

Figure 1.61

r
If the scalar field φ = ρ is the mass density, and b represents the gravitational field on the
proximity of the Earth surface, the equation (1.178) becomes:

University of Castilla-La Mancha Draft By: Eduardo W. V. Chaves (2016)


Ciudad Real - Spain
156 SOLVING PROBLEMS BY MEANS OF CONTINUUM MECHANICS

 
 0   0 
   

Fi = ρb i dV =  0 = 0 
 − ρgdV  − mg 
V

V
 

 ∫
where m = ∫ ρdV stands for the total mass of the body.
V

Center of Null Rotation


Consider that:
r r r r r r r r r r r
x = x (τ) + x ′ ⇒ x ∧ b = x (τ) ∧ b + x ′ ∧ b
r r r r r r r
∫ ∫
⇒ φx ∧ bdV = φx ( τ ) ∧ bdV + φx ′ ∧ bdV
V V

V
r
The center of the vector field ( φb ) where the rotation is null is defined by:
r r r
∫ φx ′ ∧ bdV = 0
V
(1.179)

with that we can obtain:


r r r r r r r  r  r r r r
∫ φx ∧ bdV = ∫ φx ∧ bdV = x ∧  ∫ φbdV 
(τ) (τ)
⇒ x (τ) ∧ F = τ
 
V V 1V
42 43
r
F

r r
where τ is the torque that the field b produces into the body and is defined by:
r r r
τ = φx ∧ bdV ∫
V
(1.180)
r
If the scalar field represents the mass density ( φ = ρ ), and b represents the gravitational field,
r r r r r r
x (τ ) is denoted by Center of Gravity (G). Note also that x ( τ ) = x (φb ) .
r r
Next we will obtain the torque of the vector field (φb ∧ x ) :
r r r r r r r r r r r r
x = x + x′ ⇒ x ∧ (φb ∧ x ) = x ∧ (φb ∧ x ) + x ′ ∧ (φb ∧ x )
By integrating over the volume the above equation we can obtain:
r r r r r r r r r

V

x ∧ (φb ∧ x )dV = x ∧ (φb ∧ x )dV + x ′ ∧ (φb ∧ x )dV
V

V
(1.181)
r r
The center of the vector field (φb ∧ x ) of null rotation is defined by:
r r r r

V
x ′ ∧ (φb ∧ x )dV = 0 (1.182)

with that the equation in (1.181) becomes:


r r r r r r  r r  r r r

r
∫  
r r r
x ∧ (φb ∧ x )dV = x ∧ (φb ∧ x )dV = x ∧  (φb ∧ x )dV  = x ∧ x ( τ ) ∧ F = x ∧ τ
∫ ( ) (1.183)
V V V 
where we have used the equation (1.180). In Problem 1.17 we have shown that the equation
r r r r r r r r
x ∧ (b ∧ x ) = [( x ⋅ x )1 − x ⊗ x ] ⋅ b holds, so that the above equation can be rewritten as follows:

University of Castilla-La Mancha Draft By: Eduardo W. V. Chaves (2016)


Ciudad Real - Spain
1 TENSORS 157

r r r r r r r r r r r r
∫ x ∧ (φb ∧ x )dV = x ∧ τ ⇒ ∫ { [( x ⋅ x )1 − x ⊗ x ] ⋅ φb }dV = x ∧ τ
V V
r r r
⇒ { j O ⋅ φb }dV = x ∧ τ

V

where we have introduced the second-order pseudo-tensor:


r r r r
j O = ( x ⋅ x )1 − x ⊗ x ; j O ij = x k x k δ ij − xi x j

whose components are:


1 0 0  x1 x1 x1 x 2 x1 x3 
j O ij = x k x k δ ij − xi x j = ( x12 + x 22 + x32 ) 01 0 −  x1 x 2 x2 x2 x 2 x3 
0 0 1   x1 x3 x 2 x3 x3 x3 
(1.184)
( x 22 + x32 ) − x1 x 2 − x1 x3 
 2 2 
=  − x1 x 2 ( x1 + x3 ) − x 2 x3 
 −x x − x 2 x3 ( x12 + x 22 )
 1 3
r
If φ = ρ is the mass density, and b is uniform vector field, we can obtain:
r r r   r r r r r
{ j O ⋅φb }dV = x ∧ τ  ρ j O dV  ⋅ b = x ∧ τr ⇒ I O ⋅b = x ∧ τ
∫ ⇒
∫ 

V V 
where I O is the inertia tensor of mass density and is defined by:
 

 ρ ( x 2 + x 3 )dV ∫
− ρx1 x 2 dV ∫
− ρx1 x 3 dV 
2 2

V V V 

I O( ρ ) = ρ j O dV ; I O( ρ ) ij =  − ρx1 x 2 dV
∫ ∫ ρ ( x + x )dV
2
1
2
3 − ρx 2 x 3 dV 
∫ (1.185)
V  V V V 
 V ∫
 − ρx1 x 3 dV − ∫ ρx x dV
V

2 3
V
ρ ( x12 + x 22 )dV 


kg
whose SI-unit is [I O( ρ ) ] = 3
m 2 m 3 = kg m 2 .
m
Similarly, we define the inertia tensor of area:
 
∫ ∫ ∫
2 2
 ( x 2 + x3 )dA − x1 x 2 dA − x1 x3 dA 
A A A 
I O( A) ij =  − x1 x 2 dA ∫ ∫ ( x12 + x32 )dA − x 2 x3 dA 

 
 A A A 
 − x1 x3 dA
 A
∫ ∫
− x 2 x3 dA
A

A
( x12 + x 22 )dA

NOTE: A list of inertia tensor for several solids can be found in Wikipedia
http://en.wikipedia.org/wiki/List_of_moments_of_inertia

University of Castilla-La Mancha Draft By: Eduardo W. V. Chaves (2016)


Ciudad Real - Spain
158 SOLVING PROBLEMS BY MEANS OF CONTINUUM MECHANICS

r r r r r
Note that if we consider the torque (φb ∧ x ) and vector x = x + x ′ we can obtain:
r r r r r r r r
∫ x ∧ (φb ∧ x )dV = { [( x ⋅ x )1 − x ⊗ x ] ⋅φb }dV

V V
r r r r r r r r r
= { { [( x + x ′) ⋅ ( x + x ′)]1 − [( x + x ′) ⊗ ( x + x ′)] } ⋅φb }dV

V
r r r r r r r r
{ { [( x ⋅ x ) + ( x ⋅ x ′) + ( x ′ ⋅ x ) + ( x ′ ⋅ x ′)]1
= ∫
V
r r r r r r r r r
− [( x ⊗ x ) + ( x ⊗ x ′) + ( x ′ ⊗ x ) + ( x ′ ⊗ x ′) } ⋅φb }dV
r r r r r r r r r r
= { [( x ′ ⋅ x ′)1 − x ′ ⊗ x ′] ⋅φb }dV + { [( x ⋅ x )1 − x ⊗ x ] ⋅φb }dV +
∫ ∫
V V
r r r r r r r r r r
+ { [( x ′ ⋅ x )1 − x ⊗ x ′] ⋅φb }dV + { [( x ⋅ x ′)1 − x ′ ⊗ x ] ⋅φb }dV
∫ ∫
V V

(1.186)
r
Note that, if the field b is uniform then we can obtain:
r r r r r  r r r r  r
∫ { [( x ′ ⋅ x )1 − x ⊗ x ′] ⋅φb }dV =  φ[( x ′ ⋅ x )1 − x ⊗ x ′] dV  ⋅ b

 
V V 
 r r  r  r r  r
=  φ ( x ′ ⋅ x )1 dV  ⋅ b +  φx ⊗ x ′ dV  ⋅ b
∫ ∫
   
V  V 
 r  r r  r  r  r
=  φx ′ dV  ⋅ x 1 ⋅ b +  x ⊗  φx ′ dV  ⋅ b
∫ ∫
 V  
   
V


and
r r r r r  r r r r  r
{ [( x ⋅ x ′)1 − x ′ ⊗ x ] ⋅φb }dV =  φ[( x ⋅ x ′)1 − x ′ ⊗ x ] dV  ⋅ b
∫ ∫
 
V V 
 r r  r  r r  r
=  φ ( x ⋅ x ′)1 dV  ⋅ b +  φx ′ ⊗ x dV  ⋅ b
∫ ∫
   
V  V 
r  r  r  r  r r
=  x ⋅  φx ′ dV 1 ⋅ b +  φx ′ dV  ⊗ x  ⋅ b
∫ ∫
  V 
  V 
 
r r r r r
Note that we have considered that b is uniform, hence x (V _ φ ) = x (φb ) = x , and the equation
r r

V
φx ′ dV = 0 holds. With that the equation in (1.186) becomes:

r r r r r r r r
∫ x ∧ (φb ∧ x )dV = { [( x ⋅ x )1 − x ⊗ x ] ⋅φb }dV

V V
r r r r r r r r r r (1.187)
= { [( x ′ ⋅ x ′)1 − x ′ ⊗ x ′] ⋅φb }dV + { [( x ⋅ x )1 − x ⊗ x ] ⋅φb }dV
∫ ∫
V V

If we consider that:

University of Castilla-La Mancha Draft By: Eduardo W. V. Chaves (2016)


Ciudad Real - Spain
1 TENSORS 159

r r r r r r r r r
∫ x ∧ (φb ∧ x )dV = { [( x ⋅ x )1 − x ⊗ x ] ⋅ φb }dV = φ{ j O ⋅ b }dV
∫ ∫
V V V
r r r r r r r r r r
∫ x ∧ (φb ∧ x )dV = { [( x ⋅ x )1 − x ⊗ x ] ⋅ φb }dV = φ{ j O ⋅ b }dV = j O
∫ ∫ ⋅ ∫ { φb }dV (1.188)
V V V V
r r r r r r r r r
∫ x ′ ∧ (φb ∧ x ′)dV = { [( x ′ ⋅ x ′)1 − x ′ ⊗ x ′] ⋅ φb }dV = φ{ j′O ⋅ b }dV
∫ ∫
V V V

The equation in (1.187) becomes:


r r r r r r r r r r r r r r r
∫ { [( x ⋅ x )1 − x ⊗ x ] ⋅φb }dV = { [( x ′ ⋅ x ′)1 − x ′ ⊗ x ′] ⋅φb }dV + { [( x ⋅ x )1 − x ⊗ x ] ⋅φb }dV
∫ ∫
V V V
r r r
⇒ φ{ j O ⋅ b }dV = φ{ j′O ⋅ b }dV + φ{ j O ⋅ b }dV
∫ ∫ ∫
V V V
r r r r
⇒ φ{ j O ⋅ b }dV − φ{ j′O ⋅ b }dV − φ{ j O ⋅ b }dV = 0
∫ ∫ ∫
V V V

  r r
⇒  φ [ j O − j′O − j O ] dV  ⋅ b = 0

 
V 
(1.189)
r r
Note that the above equation must be true for any uniform vector field b ≠ 0 , thus:

∫φ [j
V
O − j′O − j O ] dV = 0
(1.190)
∫ ∫
⇒ φ j O dV = φ j′O dV + φ j O dV
V V

V

where the components, (see equation (1.184)), of j O , jO , and j′O are given by:
( x 22 + x32 ) − x1 x 2 − x1 x3  ( x 2′ 2 + x3′ 2 ) − x1′ x 2′ − x1′ x3′ 
   
( j O ) ij =  − x1 x 2 ( x12 + x32 ) − x 2 x3  , ( j′O ) ij =  − x1′ x 2′ ( x1′ 2 + x 3′ 2 ) − x 2′ x3′  ,
 −x x − x 2 x3 ( x12 + x 22 )   − x′ x′ − x 2′ x3′ ( x1′ 2 + x ′22 )
 1 3  1 3

( x22 + x32 ) − x1 x2 − x1 x3 
 
( jO )ij =  − x1 x2 ( x12 + x32 ) − x2 x3 
 −xx − x2 x3 ( x12 + x22 )
 1 3

Let us assume that the given systems ( x1 − x2 − x3 ) are related by the transformation law
xi* = Aij x j , where Aij is the orthogonal matrix, then it follows that xi = A ji x *j . Thus being
able to express I O ij , (see equation (1.185)), as follows:

∫ [ ] ∫ [
IO ij = ρ xk xkδ ij − xi x j dV = ρ ( xk* xk* )Aipδ
V V
pq ]
A jq − Aip x*p A jq xq* dV

 
∫ {[
= Aip ρ ( xk* xk* )δ pq ]} ∫ [
− x*p xq* A jq dV = Aip  ρ ( xk* xk* )δ pq ]
− x*p xq* dV A jq
V V 
= Aip I*O ij A jq

Note that x k x k = A ks x s* A kt x t* = x *s x t* A ks A kt = x *s x t*δ st = x *s x s* = x t* x t* = x k* x k* .

University of Castilla-La Mancha Draft By: Eduardo W. V. Chaves (2016)


Ciudad Real - Spain
160 SOLVING PROBLEMS BY MEANS OF CONTINUUM MECHANICS

Abusing a little bit of notation, we also use tensorial notation, but keep in mind that we are
working with tensor components, and we are not doing an orthogonal transformation.
r r r r r r r r
IO = ρ [( x ⋅ x ) 1 − ( x ⊗ x )] dV = ρ [( x * ⋅ x * )A T ⋅ 1 ⋅ A − (A T ⋅ x * ⊗ A T ⋅ x * )] dV
∫ ∫
V V
r r r r
= ρ [( x * ⋅ x * )A T ⋅ 1 ⋅ A − (A T ⋅ x * ⊗ x * ⋅ A )] dV

V

⋅ {ρ [( x* ⋅ x* )1 − ( x * ⊗ x* )]}⋅ A dV
r r r r
= AT∫
V

 r r r r 
= A T ⋅  ρ [( x * ⋅ x * )1 − ( x * ⊗ x * )] dV  ⋅ A = A T ⋅ I*O ⋅ A

V 

I O = A T ⋅ I *O ⋅ A Inertia tensor components after a base change


(1.191)
I O ij = A *
ip I O ij A jq (rotation)

Then, it is also true I *O = A ⋅ I O ⋅ A T , which are the inertia tensor components in the
system x1* x 2* x3* . Note that the equation (1.191) is the same component transformation law for
a second-order tensor, where A is the transformation matrix from the x1 x 2 x3 -system to
x1* x 2* x3* -system.
We can also define the relationship between the Inertia Tensor of Area in the same fashion as
the one defined in (1.190), by considering φ to be constant i.e.:

φ ∫ jO dA = φ ∫ j′O dA + φ ∫ jO dA ⇒ ∫j O dA = ∫ j′O dA + ∫j O dA
A A A A A A (1.192)
⇒ I O = I′Oxr' + IO
r
where IOxr is the inertia tensor of area for the system Ox , I′Oxr' is calculated by considering the
r
system at the Area Centroid Ox′ and IOxr is the relation between the two systems:
r r r r r r r r
IOxr = [( x ⋅ x )1 − x ⊗ x ]dA
∫ ; I′Oxr' = [( x ′ ⋅ x′)1 − x′ ⊗ x′]dA

A A
r r r r r r r r r r r r
IOxr = ∫ [( x ⋅ x )1 − x ⊗ x ]dA = [( x ⋅ x )1 − x ⊗ x ] dA = A[( x ⋅ x )1 − x ⊗ x ]

A A

Then, the equation (1.192) in indicial notation can be written as follows


r r r r
IO = I′Oxr' + IO = I′Oxr' + A[( x ⋅ x )1 − x ⊗ x ] indicial
 → IO ij = I′Oxr' ij + IO ij
⇒ IO ij = I′Oxr' ij + A[( xk xk )δ ij − xi x j ]
( x22 + x32 ) − x1 x2 − x1 x3  (1.193)
 
⇒ IO ij = I′Oxr' ij + A − x1 x2 2 2
( x1 + x3 ) − x2 x3 
 −xx − x2 x3 ( x12 + x22 )
 1 3

University of Castilla-La Mancha Draft By: Eduardo W. V. Chaves (2016)


Ciudad Real - Spain
1 TENSORS 161

x3′
r x′2
x′
x3
r x1′
x A.C.
r
x2 x

x1

Figure 1.62

r
Example: Let us calculate the Inertia Tensor of Area in the system OX (plane X 2 - X 3 ) for
the triangle described in Figure 1.63.

( X 2( k ) , X 3( k ) ) x3′
X3

G x′2
A.C. ( X 2( j ) , X 3( j ) )

r ( X 2(i ) , X 3(i ) )
X

X2
O

Figure 1.63
r
If we know the inertia tensor of area ( I′Gxr' ) for the triangle related to the system Gx ′ we can
r
calculate the inertia tensor for the system OX as follows:
( X 22 + X 32 ) − X1 X 2 − X1 X 3 
 
IOXr ij = I′Gxr' ij + A − X 1 X 2 2 2
( X1 + X 3 ) − X 2 X3 
 −X X − X2X3 ( X 12 + X 22 ) 
 1 3
(1.194)
( X 22 + X 32 ) 0 0 
 
= I′Gxr' ij + A 0 X 32 − X 2 X3
 0 − X2X3 X 22 

University of Castilla-La Mancha Draft By: Eduardo W. V. Chaves (2016)


Ciudad Real - Spain
162 SOLVING PROBLEMS BY MEANS OF CONTINUUM MECHANICS

where we have considered that X 1 = 0 .


r r
In order to calculate the inertia tensor of area I′Gxr' we will define three systems, OX ′ , ox and
r
Gx ′′ , as indicated in Figure 1.64.

r r
x3 x3′′ OX // Gx′
r r r
X 3′ OX ′ // ox // Gx′′
X3 x3′ x′2′
(k )
x2
G x′2
A.C. ( j)
X 2′
(i) o

α
X2
O
Figure 1.64
r
We define some parameters by considering the system ox as described in Figure 1.65.

x3 x3′′

(k )
x3′ a(b − x3 )
x2 =
b
c( x3 − b)
x2 =
b x′2′
b G A.C.

r
x x′2
o
(i) ( j) x2

c a

Figure 1.65

By definition the Area Centroid, (see equation (1.174)), is given by:

∫ x dA
1 ∫ x dA 2 ∫ x dA 3

x1 = A
; x2 = A
; x3 = A
Area centroid (1.195)
∫A
dA ∫ A
dA ∫
A
dA

Then we can calculate:

University of Castilla-La Mancha Draft By: Eduardo W. V. Chaves (2016)


Ciudad Real - Spain
1 TENSORS 163

Area:
 x 2 = a (b − x 3 ) 
x3 = b  b 
b
A = dA =∫ ∫ 
∫ dx2  dx3 = (a + c)
 c ( x −b)  2
(1.196)
A x3 = 0  x2 = 3 
 b 
The first moment of area:
 x2 = a (b − x3 )   x2 = a (b − x3 ) 
x3 =b  b  b
x3 =b  b  b2
∫ x dA = ∫
2

∫ x2 dx2  dx3 = (a 2 − c 2 )
 c ( x3 −b )  6
; ∫ x dA = ∫
3 x3  ∫ dx2 dx3 = (a + c)
 c ( x3 −b )  6
A x3 = 0  x2 =  A x3 = 0  x2 = 
 b   b 
(1.197)
Then;
b 2 b2
(a − c2 ) (a + c)
( a − c) b
x1 = 0 ; x2 = 6 = ; x3 = 6 =
b 3 b
(a + c) (a + c) 3
2 2
As expected, since the Area centroid for the triangle is:
x2(i ) + x2( j ) + x2( k ) (a − c) x3(i ) + x3( j ) + x3( k ) b
x2 = = ; x3 = =
3 3 3 3
r
The inertia tensor in the system ox
 

2 2
 ( x2 + x3 )dA 0 0 
IOxr 11 IOxr 12 IOxr 13   A 
  
(I oxr )ij = IOxr 12 IOxr 22 IOx 23  =
r 0 2
( x3 )dA − ( x2 x3 ) dA
∫ ∫
 
IOxr13 IOxr 23 IOxr 33   A A 


0 − ( x2 x3 )dA
A
∫A
( x22 )dA 


where
 x2 = a (b − x3 ) 
x3 =b  b  b3

Ioxr 22 = ( x32 )dA = ∫ x32  ∫
dx2 dx3 = (a + c)
 c ( x3 −b )  12
A x3 = 0  x2 = 
 b 
 x2 = a (b − x3 ) 
 x3 =b b  b

Ioxr 33 = ( x22 )dA = 
 ∫
x3 =0  x = c ( x3 −b )
 ∫
x22 dx2 dx3 = (a 3 + c3 )
12
A 
 2 b 
 x2 = a (b − x3 ) 
x3 =b  b  b2 b2

Ioxr 23 = − ( x2 x3 ) dA = − x3  ∫ ∫
x2 dx2 dx3 = − (a 2 − c 2 ) = (c 2 − a 2 )
 c ( x3 −b )  24 24
A x3 = 0  x2 = 
 b 
Then

University of Castilla-La Mancha Draft By: Eduardo W. V. Chaves (2016)


Ciudad Real - Spain
164 SOLVING PROBLEMS BY MEANS OF CONTINUUM MECHANICS

I r + I r 0 0 
I oxr13   
ox 22 ox 33
I oxr 11 I oxr 12 
 b3  b 2
( a + c)
(I oxr )ij = I oxr 12 I oxr 22 I oxr 23  =  0 (c 2 − a 2 ) 
12
I oxr13 I oxr 23  24 
I oxr 33  
b2 2 b 3 3 
(c − a 2 )  0 (a + c ) 
24  12 
r
Calculation of the inertia tensor of area in the system Gx′′ .
We can use the equation
( x22 + x32 ) − x1 x2 − x1 x3 
 
Ioxr ij = I′G′ xr ′′ij + A − x1 x2 2 2
( x1 + x3 ) − x2 x3 
 −xx − x2 x3 ( x12 + x22 ) 
 1 3

( x22 + x32 ) − x1 x2 − x1 x3 
 
⇒ I′G′ xr′′ij = Ioxr ij − A − x1 x2 2 2
( x1 + x3 ) − x2 x3 
 −x x − x2 x3 ( x12 + x22 )
 1 3

b (a − c) b
where A = (a + c) , x1 = 0 , x2 = and x3 = . Then, the above equation becomes:
2 3 3
 a − c   b 2 2

I r + I r 0 0    +  0 0 
 ox 22 ox 33   3   3  
 b3 b2 2   b
2
(a − c) b 
I′G′ xr′′ij = 0 (a + c) (c − a 2 )  − A  0   − 
 12 24   3 3 3
 b2 2 b 3 3   (a − c) b  a−c 
2
 0 (c − a 2 ) (a + c )   0 −   
 24 12  3 3  3  

After simplification we can obtain:
 b( a + c ) 2 2 2 
 36 (a + b + c + ac) 0  0
 3 2 
b b
I′Gx
′ r′′ij = 0 (a + c) (a 2 − c 2 )  (1.198)
 36 72 
 b 2 2 2 b(a + c) 2 2 
 0 (a − c ) (a + c + ac) 
 72 36 

r
Calculation of the inertia tensor of area in the system Gx′ .
r r r r r
The transformation matrix from the system OX // Gx′ to OX ′ // ox // Gx′′ , (see Figure 1.64), is
given by:
1 0 0  1 0 0
A = 0 cosα sinα  = 0
 d s 
0 − sin α cosα  0 − s d 
r r r r r
Then the transformation matrix from OX ′ // ox // Gx′′ to OX // Gx′ is A T . And we use the
transformation defined in (1.191) in order to calculate:
I′Gxr ′ = A T ⋅ I′G′ xr ′′ ⋅ A
The above equation after matrix multiplications becomes:

University of Castilla-La Mancha Draft By: Eduardo W. V. Chaves (2016)


Ciudad Real - Spain
1 TENSORS 165

I′G′ xr ′′11 0 0 
 
I′Gxr ′ij = 0 d I′G′ x ′′ 22 − 2 sd I′G′ xr ′′ 23 + s I′G′ xr ′′33
2 r 2
I′G′ xr ′′ 23 (d − s ) + sd (I′G′ xr ′′ 22 − I′G′ xr ′′33 )
2 2
(1.199)
 0 I′G′ xr ′′ 23 (d 2 − s 2 ) + sd (I′G′ xr ′′ 22 − I′G′ xr ′′33 ) s 2I′G′ xr ′′ 22 + 2 sd I′G′ xr ′′ 23 + d 2I′G′ xr ′′33 

With that we can use the equation in (1.194) in order to calculate IOXr :

IOXr ij = I′Gxr' ij + A IG′ xr' ij

I′Gxr ′11 0 0  ( X 22 + X 32 ) 0 0 
   
= 0 I′Gxr ′22 I′Gxr ′23  + A 0 X 32 − X2X3
 0 I′Gxr ′23 I′Gxr ′33   0 − X2 X3 X 22 

I′Gxr ′11 + A( X 22 + X 22 ) 0 0 
 
IOXr ij = 0 I′Gxr ′22 + AX 3
2
I′Gxr ′23 − AX 2 X 3  (1.200)
 0 I′Gxr ′23 − AX 2 X 3 I′Gxr ′33 + AX 22 

NOTE: The node connectivity i − j − k must be oriented according to the counterclockwise
direction, (see Figure 1.66).

Connectivity: i − j − k
x2 (i)
X3

( j)

(k )
( j)

x2
(i)
(k )
X2
O
Figure 1.66

Procedure
Given the node coordinates: i( X 2(i ) , X 3(i ) ) − j ( X 2( j ) , X 3( j ) ) − k ( X 2( k ) , X 3( k ) )
Calculate the transformation matrix:
L = ( X 2( j ) − X 2(i ) ) 2 + ( X 3( j ) − X 3(i ) ) 2
X 2( j ) − X 2(i )
d = cosα =
L
X − X 3(i )
( j)
s = sinα = 3
L
r
Node coordinates in the system OX ′

University of Castilla-La Mancha Draft By: Eduardo W. V. Chaves (2016)


Ciudad Real - Spain
166 SOLVING PROBLEMS BY MEANS OF CONTINUUM MECHANICS

 X ′(i )   d s   X 2(i )   X 2(i ) d + sX 3(i ) 


(Node i ) ⇒  2(i )  =    (i )  =  (i ) (i ) 
 X 3′   − s d   X 3   X 3 d − sX 2 
 X ′( j )   d s   X 2( j )   X 2( j ) d + sX 3( j ) 
(Node j ) ⇒  2( j )  =   ( j)  =  ( j) ( j) 
 X 3′   − s d   X 3   X 3 d − sX 2 
 X ′( k )   d s   X 2( k )   X 2( k ) d + sX 3( k ) 
(Node k ) ⇒  2( k )  =    (k )  =  (k ) (k ) 
 X 3′  − s d   X 3   X 3 d − sX 2 
Then
a = X 2′( j ) − X 2′( k ) , c = X 2′( k ) − X 2′(i ) , b = X 3′( k ) − X 3′(i ) .
(a + c)b
Area: A =
2
r
Calculate the area centroid related to the system ox :
(a − c) b
x2 = ; x3 =
3 3
r r
Calculation of the area centroid ( X ) related to the system OX :
r
Coordinate of o in the system OX ′ is ( X 2′( k ) ; X 3′(i ) )
r
Coordinate of G in the system OX ′ is ( X 2′( k ) + x2 ; X 3′(i ) + x3 )
r
Coordinate of G in the system OX is:
 X 2   X 2(G )  d − s   X 2′ ( k ) + x 2  ( X 2′ ( k ) + x 2 ) d − s( X 3′ (i ) + x3 ) 
  =  (G )  =   = 
X 3  X 3   s d   X 3′ (i ) + x3  ( X 2′ ( k ) + x 2 ) s + d ( X 3′ (i ) + x3 )

Calculate I′G′ xr ′′ , (see equation (1.198));


Calculate I′Gxr' , (see equation (1.199));
Calculate IOXr , (see equation (1.200)).

x3

X 3′ (k )
X3
G x2
A.C.
(k ) x3 ( j)
X 3′
X 2′
x2 ( j)
o X 2′
(i)
r (k )
(i ) ( j) X X 2′
X 3′ = X 3′ (i ) L
X 2′
α X2
O

Figure 1.67

University of Castilla-La Mancha Draft By: Eduardo W. V. Chaves (2016)


Ciudad Real - Spain
1 TENSORS 167

If we have n triangles the following is true:


n
IOXr = ∑I
e =1
( e )r
OX

and
n n


e =1
X 2( e ) A ( e ) ∑X
e =1
(e) (e)
3 A
X2 = n
; X3 = n


e =1
A (e) ∑A
e =1
(e)

Then, we can obtain the inertia tensor of area for any geometric shape in two dimensions, (see
Figure 1.68).

a) b)

Figure 1.68

r
NOTE 1: Let us suppose now that the system ox is located as shown in Figure 1.69.

ξ3
r r r
c a x=~
x +ξ

c(ξ 3 − b)
x3 ξ (2L1) = a (b − ξ 3 )
b ξ (2L 2) =
b
b
r
ξ

r o∆ ξ2
x
r
~
x
o x2

Figure 1.69

University of Castilla-La Mancha Draft By: Eduardo W. V. Chaves (2016)


Ciudad Real - Spain
168 SOLVING PROBLEMS BY MEANS OF CONTINUUM MECHANICS

According to Figure 1.69 we can state that


r r r ~ +ξ ξ 2 = x2 − ~
x2
x=x ⇒ xi = ~
xi + ξ i ⇒ ξ i = xi − ~xi ⇒  ~
ξ 3 = x3 − x3
Then
c(ξ 3 − b) c ( x3 − ~
x3 − b ) c ( x3 − ~
x3 − b ) ~
ξ (2L1) = ⇒ x2( L1) − ~
x2 = ⇒ x2( L1) = + x2
b b b
a (b − ξ 3 ) a (b − ( x3 − ~
x3 )) a(b − x3 + ~x3 ) ~
ξ (2L 2) = ⇒ x2( L 2 ) − ~
x2 = ⇒ x2( L 2 ) = + x2
b b b
Area:
~
x3 =b + ~
 x2 = a (b − x3 + x3 ) + ~x2 
x3  b  b
A = dA =∫ ∫
x3 = ~

∫ dx2 dx3 = ( a + c)
 c ( x3 − ~x3 −b ) ~  2
(1.201)
A x3  x2 = + x2 
 b 
The first moment of area:
about x2 :
~
x3 =b + ~
 x2 = a (b − x3 + x3 ) + ~x2 
x3  b  b
∫ x dA = ∫
2

∫ x2 dx2 dx3 = (a + c)(a − c + 3~
x2 ) (1.202)
x3 = ~
 c ( x3 − ~x3 −b ) ~  6
A x3  x2 = + x2 
 b 
about x3
~
x3 =b + ~
 x2 = a (b − x3 + x3 ) + ~x2 
x3  b  b
∫ x dA = ∫
3 x3  ∫ dx2 dx3 = (a + c)(b + 3~
x3 ) (1.203)
x3 = ~
 c ( x3 − ~x3 −b ) ~  6
A x3  x2 = + x2 
 b 
Then;
b b
(a + c)(a − c + 3~x2 ) ( a + c)(b + 3~
x3 )
6 (a − c) ~ 6 b
x1 = 0 ; x2 = = + x2 ; x3 = = +~
x3
b 3 b 3
(a + c) (a + c)
2 2
r
The inertia tensor in the system ox
 

2 2
 ( x2 + x3 )dA 0 0 
IOxr 11 IOxr 12 IOxr 13   A 
  
(I oxr )ij = IOxr 12 IOxr 22 IOx 23  =
r 0 2
( x3 )dA ∫ − ( x2 x3 ) dA

 
IOxr13 IOxr 23 IOxr 33   A A 


0
A

− ( x2 x3 )dA
A

( x22 )dA 

where
~
x3 =b + ~
 x2 = a (b − x3 + x3 ) + ~x2 
x3  b  b

Ioxr 22 = ( x32 )dA = ∫ x32  ∫ dx2  dx3 = (a + c)(b 2 + 4b~
x3 + 6 ~
x32 )
 c ( x3 − ~x3 −b ) ~  12
A x3 = ~
x3  x2 = + x2 
 b 

University of Castilla-La Mancha Draft By: Eduardo W. V. Chaves (2016)


Ciudad Real - Spain
1 TENSORS 169

~
x3 = b + ~
 x2 = a (b − x3 + x3 ) + ~x2 
x3  b  b

Ioxr 33 = ( x22 )dA = ∫ 
∫ x22 dx2 dx3 = (a + c)(a 2 + 4a~
x2 − ac + 6 ~
x22 − 4c~
x2 + c 2 )
x3 = ~
 c ( x3 − ~x3 −b ) ~  12
A x3  x2 = + x2 
 b 
~
x3 =b + ~
 x2 = a (b − x3 + x3 ) + ~x2 
x3  b  −b

Ioxr 23 = − ( x2 x3 )dA = − ∫ x3  ∫ x2 dx2 dx3 = ( a + c)(ab + 4a~
x3 + 12 ~
x2 ~
x3 + 4b~
x2 − cb − 4c~
x3 )
x3 = ~
 c ( x3 − ~x3 −b ) ~  24
A x3  x2 = + x2 
 b 
− (a − c) −b
Note that when the system is located at the area centroid we have ~x2 = and ~x2 = ,
3 3
and by substituting into the above equations we will obtain the same expressions as those for
I′Gx
′ r ′′ij given by the equations in (1.198).

Problem 1.133
r
Obtain the inertia tensor of mass density related to the system ox for the tetrahedron
described in Figure 1.70. The tetrahedron base plane (formed by the triangle 1 − 2 − 3 ) is lying
on the plane x2 − x3 . Consider that the mass density is constant.

x1
Connectivity: 1 − 2 − 3 − 4
Node Coordinates ( x1 , x2 , x3 ) :
4
Node 1 : (0,−c0 ,0)
Q1
Q3 Node 2 : (0, a0 ,0)
x3
1 Node 3 : (0,0, b0 )
o∆
Node 4 : (d 0 , e0 , f 0 )
3
Q2
c0
o

a0 b0
2

x2

Figure 1.70

University of Castilla-La Mancha Draft By: Eduardo W. V. Chaves (2016)


Ciudad Real - Spain
170 SOLVING PROBLEMS BY MEANS OF CONTINUUM MECHANICS

Solution:
By definition the inertia tensor of mass density is given by the equation in (1.185), i.e.:
 

 ρ ( x2 + x3 )dV ∫
− ρx1 x2 dV ∫
− ρx1 x3 dV 
2 2

Ioxr11 Ioxr12 I ox13


r  V V V 
  
I(oρxr ) ij = I oxr12 Ioxr 22 r

I ox 23  =  − ρx1 x2 dV ∫ ρ ( x + x )dV
2
1
2
3 − ρx2 x3 dV 
∫ (1.204)
I r Ioxr 33   V 
 ox13 Iox 23
r V V

 V ∫
 − ρx1 x3 dV − ∫ ρx x dV
V

2 3
V
ρ ( x12 + x22 )dV 


From now on we will adopt I(oρxr ) ≡ Ioxr .


Note that the volume integral can be expressed as follows:
 
 f ( xr )dA  dx1
∫∫ 
x1  A


where the area is defined on the plane x2 − x3 . For the tetrahedron defined in Figure 1.70 the
area on the plane x2 − x3 is defined by the arbitrary triangle formed by the points Q1 − Q2 − Q3
which is parallel to the base plane defined by the nodes 1 − 2 − 3 . In order to calculate the
integral related to the area we can use the same equations derived by considering Figure 1.69,
but now the parameters a ( x1 ) , b( x1 ) , c( x1 ) , ~x2 ( x1 ) and ~x3 ( x1 ) will depend on x1 . Now, in
order to define these parameters we have to define the points Q1 , Q2 , Q3 and o∆ , (see Figure
1.70 and Figure 1.71).

ξ3
c( x1 ) a ( x1 ) a ( x1 ) = x2(Q2 ) − x2(Q3 )
b( x1 ) = x3(Q3 ) − x3(Q1 )
Q3
c( x1 ) = x2(Q3 ) − x2(Q1 )
x3

b( x1 )
r
ξ
Q1
~
x3 ( x1 )
r o∆ Q2 ξ2
x
r
~
x
o ~
x2 ( x1 ) x2

Figure 1.71

University of Castilla-La Mancha Draft By: Eduardo W. V. Chaves (2016)


Ciudad Real - Spain
1 TENSORS 171

According to Figure 1.70 we can define the following coordinates:


 (Q1 ) (e0 + c0 )
 x2 = −c0 + d x1
 0
Point Q1 : 
 x (Q1 ) = f0 x
 3 d0
1

 ( Q2 ) (e0 − a0 )
 x2 = a0 + d x1
 0
Point Q2 : 
 x ( Q2 ) = f 0 x
 3 d0
1

 (Q3 ) e0
 x2 = d x1
 0
Point Q3 : 
 x (Q3 ) = b + ( f0 − b0 ) x
 3 0
d0
1

 ( o∆ ) ~ ( Q3 ) e0
 x2 ≡ x2 = x2 = d x1
 0
Point o∆ : 
 x ( o∆ ) ≡ ~ f
x3 = x3(Q1 ) = x3(Q2 ) = 0 x1
 3
d0
Once these points are defined we can obtain:
 (e − a )   e  a
a ( x1 ) = x2(Q2 ) − x2(Q3 ) =  a0 + 0 0 x1  −  0 x1  = a0 − 0 x1
 d0   d0  d0

 ( f −b )   f  b
b( x1 ) = x3(Q3 ) − x3(Q1 ) =  b0 + 0 0 x1  −  0 x1  = b0 − 0 x1
 d0   d0  d0

e   (e + c )  c
c( x1 ) = x2(Q3 ) − x2(Q1 ) =  0 x1  −  − c0 + 0 0 x1  = c0 − 0 x1
 d0   d0  d0

~ e
x2 ( x1 ) = 0 x1
d0

~ f
x3 ( x1 ) = 0 x1
d0
Now we can easily obtain the volume integrals:
Volume: If we take into account the equation for the area defined in equation (1.201),
b
A= (a + c) , we can obtain:
2
x1 = d 0 x1 = d 0 x1 = d 0
   b( x1 ) 
 
V= ∫ ∫

x1 = 0  A



dA dx1 = [ A( x1 )]dx1 =
x1 = 0 x1 = 0 
∫ 2
[a( x1 ) + c( x1 )] dx1

  
  b0 − b0 x1   (1.205)
  d0  
x1 = d 0
a0   c0   1
= ∫ 
x1 = 0 
2
 a0 − x1  +  c0 − x1  dx1 = b0 d 0 (a0 + c0 )
 d0   d 0   6
 
 

University of Castilla-La Mancha Draft By: Eduardo W. V. Chaves (2016)


Ciudad Real - Spain
172 SOLVING PROBLEMS BY MEANS OF CONTINUUM MECHANICS

r r
Note that the Volume Centroid ( x1(V ) ) and the Centroid of the mass density field ( x1(V _ ρ ) ) are
the same, since the mass density is a constant filed.
First Moment of Volume
About x1
  
  b0 − b0 x1  
x1 = d 0
  x1 = d 0 x1 = d 0
 
d 0   a0   c0   
∫ x1dV = ∫ x1  dA dx1 =
∫ ∫ x A( x )dx = ∫ x1   a0 − x1  +  c0 − x1    dx1
  1 1 1
2  d0   d 0   
V x1 = 0 A  x1 = 0 x1 = 0 
 
 
1
= b0 d 02 (a0 + c0 )
24
About x2 , (see equation (1.202)):
x1 = d 0 x1 = d 0
   b( x1 ) 
∫ x2 dV = 
∫ ∫ 
x dA dx =  [a ( x1 ) + c( x1 )][a ( x1 ) − c( x1 ) + 3~
∫ x2 ( x1 )] dx1
 2  1 6
V x1 = 0  A  x1 = 0  
1
= b0 d 0 (a0 + c0 )(a0 − c0 + e0 )
24
About x3 , (see equation (1.203)):
x1 = d 0 x1 = d 0
   b( x1 ) 
∫ x3dV = 
∫ ∫ 
x dA dx =  [ a( x1 ) + c( x1 )][b( x1 ) + 3~
∫ x3 ( x1 )]  dx1
 3  1 6
V x1 = 0  A  x1 = 0  
1
= b0 d 0 (a0 + c0 )(b0 + f 0 )
24
r
The Volume Centroid related to the system ox
By definition the Volume Centroid, (see equation (1.173)), is given by:

∫ x dV 1 ∫ x dV 2 ∫ x dV 3

x1(V ) = V
; x 2(V ) = V
; x3(V ) = V
Volume Centroid (1.206)

V
dV ∫
V
dV ∫
V
dV

Then

∫ x dV 1
1
24
b0 d 02 (a0 + c0 )
d
x1(V ) = V
= = 0

V

dV 1
6
b0 d 0 (a0 + c0 ) 4

∫ x dV 1
2 b0 d 0 (a0 + c0 )(a0 − c0 + e0 ) (a − c + e )
x2(V ) =V = 24 = 0 0 0 (1.207)
V

dV 1
6
b0d 0 (a0 + c0 ) 4

∫ x dV 13
24
b0 d 0 (a0 + c0 )(b0 + f 0 )
(b + f 0 )
x3(V ) = V
= = 0

V
dV∫ 1
6
b0 d 0 (a0 + c0 ) 4

University of Castilla-La Mancha Draft By: Eduardo W. V. Chaves (2016)


Ciudad Real - Spain
1 TENSORS 173

As expected, since the tetrahedron volume centroid is given by:


x1(1) + x1( 2 ) + x1(3) + x1( 4) x2(1) + x2( 2 ) + x2(3) + x2( 4 ) x3(1) + x3( 2 ) + x3(3) + x3( 4 )
x1 = ; x2 = ; x3 =
4 4 4
By considering the area integrals given in Problem 1.132-NOTE 1 and by considering
a0 b c e f
a ( x1 ) = a0 − x1 , b( x1 ) = b0 − 0 x1 , c( x1 ) = c0 − 0 x1 , ~
x2 ( x1 ) = 0 x1 and ~
x3 ( x1 ) = 0 x1 , the
d0 d0 d0 d0 d0
components of the inertia tensor of mass density, (see equation in (1.204)), can be obtained as
follows:
 x1 =d 0  x1 = d 0
  
Ioxr11 = ρ ( x22 + x32 )dV = ρ   ( x22 )dA dx1 +
∫ ∫ ∫  ( x32 )dA dx1 
∫ ∫
 x =0  A 


x1 =0  A
 
 
V  1
 x1 =d0  b 
~ ~ ~ 2   b ~ ~ 2 
= ρ ∫
2 2 2
 ( a + c )( a + 4 a x2 − ac + 6 x2 − 4 c x 2 + c )  +  ( a + c )(b + 4bx3 + 6 x3 )  dx
 1
 x1 =0  12   12  
ρ
= b0 d 0 (a0 + c0 )(b0 f 0 + b02 + a02 + c02 + a0 e0 − e0 c0 + e02 + f 02 − a0 c0 )
60
x1 = d 0 x1 = d 0
  b 
Ioxr12 ∫
= − ρx1 x2 dV = − ρ ∫ 
∫ 
x1 ( x2 )dA dx1 = − ρ x1  (a + c)(a − c + 3~
∫ x2 ) dx1
  6 
V x1 = 0 A  x1 = 0

−ρ
= b0 d 02 (a0 + c0 )(a0 + 2e0 − c0 )
120
x1 = d 0 x1 = d 0
  b 

Ioxr13 = − ρx1 x3dV = − ρ ∫ x1  ( x3 ) dA dx1 = − ρ
∫ x1  (a + c)(b + 3~
∫ x3 )  dx1
  6
V x1 =0  A  x1 =0  
−ρ
= b0 d 02 (a0 + c0 )(b0 + 2 f 0 )
120
 x1 =d 0   x1 = d 0
 2    x1 =d0 x1 = d 0
 2  

Ioxr 22 = ρ ( x12 + x32 )dV = ρ  x12  dA  dx1 +
∫  x3 dA dx1  = ρ  x12 Adx1 +
∫ ∫ ∫  x3 dA  dx1 
∫ ∫ ∫
 x =0  A        
V  1 x1 =0  A    x =0
 1 x1 =0  A  
 x1 =d0  b 
x1 = d 0
b  
= ρ ∫  (a + c)(b + 4b~
∫ x3 + 6 ~
x32 ) dx1 
2 2
x1  [a + c]  dx1 +
 x1 =0  2  x1 =0 
12  

ρ
= b0 d 0 (a0 + c0 )(b02 + b0 f 0 + f 02 + d 02 )
60
x1 = d 0
 
 (− x2 x3 )dA dx1
Ioxr 23 ∫
= − ρx2 x3 dV = ρ
V
∫ ∫ 
x1 = 0  A


x1 = d 0
 −b 
=ρ ∫  (a + c)(ab + 4a~
x3 + 12 ~
x2 ~
x3 + 4b~
x2 − cb − 4c~
x3 ) dx1
x1 = 0 
24 
−ρ
= b0 d 0 (a0 + c0 )(b0 a0 + f 0 a0 + 2 f 0e0 − b0 c0 + e0b0 − c0 f 0 )
120

University of Castilla-La Mancha Draft By: Eduardo W. V. Chaves (2016)


Ciudad Real - Spain
174 SOLVING PROBLEMS BY MEANS OF CONTINUUM MECHANICS

 x1 =d0   x1 = d 0
 2    x1 =d 0 x1 = d 0
 2  

Ioxr 33 = ρ ( x12 + x22 )dV = ρ  x12  dA dx1 +
∫ ∫  x2 dA  dx1  = ρ  x12 Adx1 +
∫ ∫ ∫ ∫ ∫  x2 dA  dx1 
 x =0  A  

 
   
A
 
 
V  1 x1 = 0 A  1x = 0 x1 = 0

 1 0 b
x = d

x1 = d 0
b  
= ρ ∫ x12  [a + c] dx1 +  (a + c)(a + 4a~
∫ x2 − ac + 6 ~
x22 − 4c~ x2 + c 2 ) dx1 
2

 x1 =0  2  x1 =0 
12  

ρ
= b0 d 0 ( a0 + c0 )(c02 − c0e0 − c0 a0 + d 02 + e02 + a0e0 + a02 )
60
If we consider that m stands for the total mass of the tetrahedron, and by the fact that the
mass density field is constant the following holds:
m m
m = ρV ⇒ ρ= =
V 1
b0 d 0 (a0 + c0 )
6
Then, the inertial tensor of mass density can be written in terms of total mass as follows:
m
Ioxr11 = (b0 f 0 + b02 + a02 + c02 + a0e0 − c0e0 + e02 + f 02 − c0 a0 )
10
−m
Ioxr12 = d 0 (a0 + 2e0 − c0 )
20
−m
Ioxr13 = d 0 (b0 + 2 f 0 )
20
m 2
Ioxr 22 = (b0 + b0 f 0 + f 02 + d 02 )
10
−m
Ioxr 23 = (b0 a0 + f 0 a0 + 2 f 0e0 − b0c0 + e0b0 − c0 f 0 )
20
m 2
Ioxr 33 = (c0 − c0 e0 − c0 a0 + d 02 + e02 + a0e0 + a02 )
10
r
Calculation of the inertia tensor of mass density in the system Gx′′ .
Now if we want to calculate the inertia tensor in the system located at the volume centroid
r
Gx′′ , (see Figure 1.72), we can use the definition:
( x22 + x32 ) − x1 x2 − x1 x3 
 
Ioxr ij = I′G′ xr ′′ij + m  − x1 x2 2 2
( x1 + x3 ) − x2 x3 
 −xx − x2 x3 ( x12 + x22 )
 1 3

( x22 + x32 ) − x1 x2 − x1 x3 
 
⇒ I′G′ xr ′′ij = Ioxr ij − m  − x1 x2 2 2
( x1 + x3 ) − x2 x3 
 −xx − x2 x3 ( x12 + x22 )
 1 3

d0 (a − c + e ) (b + f )
and by considering that x1 = , x2 = 0 0 0 and x3 = 0 0 , we can obtain:
4 4 4

University of Castilla-La Mancha Draft By: Eduardo W. V. Chaves (2016)


Ciudad Real - Spain
1 TENSORS 175

I′G′ xr ′′11 I′G′ xr ′′12 I′G′ xr ′′13 


 
I′G′ xr ′′ij = I′G′ xr ′′12 I′G′ xr ′′ 22 I′G′ xr ′′ 23  (1.208)
I′G′ xr ′′13 I′G′ xr ′′ 23 I′G′ xr ′′33 

where
m
I′G′ xr ′′11 = (−2b0 f 0 + 3b02 + 3a02 + 3c02 − 2a0e0 + 2c0 e0 + 3e02 + 3 f 02 + 2c0 a0 )
80
−m
I′G′ xr ′′12 = d 0 (−a0 + 3e0 + c0 )
80
−m
I′G′ xr ′′13 = d 0 (−b0 + 3 f 0 )
80
m
I′G′ xr ′′ 22 = (3b02 − 2b0 f 0 + 3 f 02 + 3d 02 )
80
−m
I′G′ xr ′′ 23 = (−b0 a0 − f 0 a0 + 3 f 0 e0 + b0 c0 − e0b0 + c0 f 0 )
80
m
I′G′ xr ′′ 33 = (3c02 + 2c0 e0 + 2c0 a0 + 3d 02 + 3e02 − 2a0e0 + 3a02 )
80

x1
Connectivity: 1 − 2 − 3 − 4
4 Node Coordinates ( x1 , x2 , x3 ) :
Node 1 : (0,−c0 ,0)
x1′′ x3′′
Node 2 : (0, a0 ,0)
1 x3 Node 3 : (0,0, b0 )
Node 4 : (d 0 , e0 , f 0 )
G 3
r
x
c0 Volume centroid - G
o x2′′ d0
x1 =
4
( a0 − c0 + e0 )
x2 =
4
a0 b0
2 (b0 + f 0 )
x3 =
4
x2

Figure 1.72

University of Castilla-La Mancha Draft By: Eduardo W. V. Chaves (2016)


Ciudad Real - Spain
176 SOLVING PROBLEMS BY MEANS OF CONTINUUM MECHANICS

University of Castilla-La Mancha Draft By: Eduardo W. V. Chaves (2016)


Ciudad Real - Spain
2 Continuum Kinematics
2.1 Description of Motion, Material Time Derivative,
Lagrangian and Eulerian Variables
Problem 2.1
A continuum is defined by a square with sides b , subjected to rigid body motion which is
defined by rotating the continuum counterclockwise by an angle of 30º to the origin. Find
the equations of motion. Also obtain the new position of particle D .
r r
Hint: Consider the systems x and X to be superimposed.

X 2 , x2
x2′
C′

D′ C x1′
D
30º
B′
b
30º B
A = A′ b X 1 , x1

Figure 2.1
Solution:
r r r r r r
We apply the rigid body motion equations x = c + Q ⋅ X = Q ⋅ X , to c = 0 . The
components of Q are the same as the components of the transformation matrix from the
r r
x ′ -system to the x -system, i.e.:
cos θ − sin θ 0
Qij =  sin θ cos θ 0 = A T
 0 0 1 
r r
where A is the transformation matrix form the x -system to the x ′ -system. So, the
continuum particles are governed by the equations of motion:
 x1  cos 30º − sin 30º 0  X 1 
    
 x 2  =  sin 30º cos 30º 0  X 2 
x   0 0 1  X 3 
 3 
178 SOLVING PROBLEMS BY MEANS OF CONTINUUM MECHANICS

A particle which initially was at point D ( X 1 = 0 , X 2 = b , X 3 = 0 ) moves into the following


position:
 x1D  cos 30º − sin 30º 0 0 − b sin 30º 
 D     
 x 2  =  sin 30º cos 30º 0 b  =  b cos 30º 
x D   0 0 1 0  0 
 3   

Problem 2.2
Consider the following equations of motion:
 x1 = exp t X 1 − exp − t X 2
 t −t
 x2 = exp X 1 + exp X 2 (t > 0) (2.1)
x = X
 3 3

Find velocity, acceleration in material and spatial descriptions.


Solution:
Material velocity (Lagrangian):

r r V1 = expt X 1 + exp − t X 2


r r Dx ( X , t ) components 
V ( X ,t) =   →V2 = expt X 1 − exp − t X 2 (2.2)
Dt V = 0
 3
Acceleration:
A1 = expt X 1 − exp − t X 2 ; A2 = expt X 1 + exp −t X 2 ; A3 = 0 (2.3)
To find the velocity and acceleration components in the spatial description we substitute
the equations of motion, i.e.:
Eulerian velocity (spatial description)
v1 = x2 ; v2 = x1 ; v3 = 0 (2.4)
Eulerian acceleration (spatial description)
a1 = x1 = v2 ; a2 = x2 = v1 ; a3 = 0 (2.5)

Problem 2.3
The velocity field of a fluid is given by:
r
v = x1eˆ 1 + x2 eˆ 2 + x3eˆ 3 (2.6)
r
and the temperature field is T ( x , t ) = 3 x 2 + x3 t . Find the rate of change of temperature.
Solution:
The rate of change of any property is given by the material time derivative, and when we
are dealing with Eulerian variables the material derivative is given by:
r r
DT ∂T ( x , t ) ∂T ( x , t ) ∂T  ∂T ∂T ∂T 
= + vj = +  v1 + v2 + v3  (2.7)
Dt ∂t ∂x j ∂t  ∂x1 ∂x 2 ∂x3 
DT
= x3 + (0 × x1 + 3 × x 2 + tx3 ) = x3 + (3x 2 + tx3 ) (2.8)
Dt

University of Castilla-La Mancha Draft By: Eduardo W. V. Chaves (2016)


Ciudad Real - Spain
2 CONTINUUM KINEMATICS 179

Problem 2.4
Given the following equations motion:
xi = X i + 0.2tX 2δ 1i (2.9)
and the temperature field (steady):
r
T ( x ) = 2 x1 + x 22 (2.10)
a) Find the temperature field in material description;
b) Find the rate of change of temperature for one particle that in the reference
configuration was at the position (0,1,0) .
Solution:
According to the equations of motion we have:
x1 = X 1 + 0.2tX 2 δ 11 = X 1 + 0.2tX 2
x 2 = X 2 + 0.2tX 2 δ 12 = X 2
x3 = X 3 + 0.2tX 2 δ 13 = X 3
And by means of the above equations it is possible to express the temperature in material
description:
r r r r r
T ( x ( X , t )) = 2 x1 ( X , t ) + [ x2 ( X , t )]2 = 2( X 1 + 0.2tX 2 ) + ( X 2 ) 2 = 2 X 1 + ( X 2 + 0.4t )X 2 = T ( X , t )
b) The material time derivative of temperature is given by:
r
DT ( X , t ) & r
≡ T ( X , t ) = 0 .4 X 2
Dt
For the particle ( X 1 = 0; X 2 = 1; X 3 = 0) we have:
T& (( X 1 = 0; X 2 = 1; X 3 = 0), t ) = 0.4 X 2 = 0.4
r
Note that, although the Eulerian temperature ( T ( x ) ) is independent of time, the
r
Lagrangian temperature T ( X , t ) depends on time, in other words, the temperature at a
point does not change meanwhile the particle temperature changes.
Problem 2.5
r r r r
Find the velocity field V ( X , t ) in the material description and the acceleration field A( X , t )
r r
of the particle at time t in function of the rate of change of displacement U ( X , t ) .
Solution:
r r D r r r&
V ( X , t) = U ( X , t) = U (2.11)
Dt
r r D r r r& D 2 r r &r&
A( X , t ) = V ( X , t) = V = 2 U ( X , t) = U (2.12)
Dt Dt
Problem 2.6
Consider the following equations of motion in the Lagrangian description:
r
 x1 ( X , t ) = X 2 t 2 + X 1  x1  1 t
2
0  X 1 
 r Matrix form     
 x 2 ( X , t ) = X 3 t + X 2    →  x 2  = 0 1 t  X 2  (2.13)
 r  x  0 0
 x 3 ( X , t ) = X 3  3  1   X 3 

University of Castilla-La Mancha Draft By: Eduardo W. V. Chaves (2016)


Ciudad Real - Spain
180 SOLVING PROBLEMS BY MEANS OF CONTINUUM MECHANICS

Is the motion above possible? If so, find the displacement, velocity and acceleration fields
in Lagrangian and Eulerian descriptions. Consider a particle P that at time t = 0 was at the
point defined by the triple equation X 1 = 2, X 2 = 1, X 3 = 3 . Find the velocity of P at time
t = 1s and t = 2 s .
Solution:
Motion is possible if J ≠ 0 , thus
∂x1 ∂x1 ∂x1
∂X 1
∂X 3 1 t 2 ∂X 2 0
∂xi ∂x 2 ∂x 2 ∂x 2
J= = =0 1 t =1≠ 0
∂X j ∂X 3 ∂X 1 ∂X 2
∂x3 ∂x30 0 ∂x3 1
∂X 3 ∂X 1 ∂X 2
r r r
The displacement vector field is given by the definition u = x − X . Using the equations of
motion (2.78) we obtain:
r r
u1 ( X , t ) = x1 ( X , t ) − X 1 = [ X 2t 2 + X 1 ] − X 1 = X 2t 2
 r r
u2 ( X , t ) = x2 ( X , t ) − X 2 = [ X 3t + X 2 ] − X 2 = X 3t (2.14)
 r r
u3 ( X , t ) = x3 ( X , t ) − X 3 = [ X 3 ] − X 3 = 0
which are the components of the displacement vector in the Lagrangian description. Here,
velocity and acceleration can be evaluated as follows:
r
 r
V1 ≡ v1 ( X , t ) =
du1 ( X , t ) d
dt
=
dt
(
X 2t 2 = 2 X 2t )  r dV1
 A1 ≡ a1 ( X , t ) = dt = 2 X 2
 r 
 r du 2 ( X , t ) d  r dV2
V2 ≡ v2 ( X , t ) = = ( X 3t ) = X 3 ;  A2 ≡ a2 ( X , t ) = =0 (2.15)
 dt dt  dt
r r
 r d u3 ( X , t ) d  dV3
V3 ≡ v3 ( X , t ) = = ( X 2t ) = 0  A3 ≡ a3 ( X , t ) = dt = 0
 dt dt 

The inverse form of (2.13) provides us the inverse equations of motion (Eulerian
description):
r
 X 1  1 − t 2 t 3   x1   X 1 ( x , t ) = x1 − t 2 x 2 + t 3 x 3
      r
 X 2  = 0 1 − t   x 2  ⇒  X 2 ( x , t ) = x 2 − tx 3 (2.16)
 X  0 r
 3  0 1   x 3   X 3 ( x , t ) = x 3

Then, the displacement, velocity and acceleration fields in Eulerian description can be
evaluated by substituting equation (2.16) into the equations (2.14) and (2.15), i.e.:
(Xr ( xr , t ), t ) = X ( xr , t )t = ( x − tx )t = u ( xr , t )
r
u1 2
2
2 3
2
1

u 2 (Xr ( xr , t ), t ) = X ( xr , t )t = x t = u ( xr , t )
3 3 2 (2.17)

u 3 (X ( xr , t ), t ) = u ( xr , t ) = 0
3

(Xr ( xr , t ), t ) = 2 X ( xr , t )t = 2( x − tx )t = v ( xr , t )
r
V1 2 2 3 1

V 2 (Xr ( xr , t ), t ) = X ( xr , t ) = x = v ( xr , t )
3 3 2 (2.18)

V3 (X ( xr , t ), t ) = v ( xr , t ) = 0
3

University of Castilla-La Mancha Draft By: Eduardo W. V. Chaves (2016)


Ciudad Real - Spain
2 CONTINUUM KINEMATICS 181

(Xr ( xr , t ), t ) = 2 X ( xr , t ) = 2( x
r r
 A1 2 2 − tx 3 ) = a1 ( x , t )

 A2 (Xr ( xr , t ), t ) = a ( xr , t ) = 0
2 (2.19)

 A3 (X ( xr , t ), t ) = a ( xr , t ) = 0
3

Taking into account the Lagrangian description of velocity given in (2.15), the velocity of
particle P ( X 1 = 2, X 2 = 1, X 3 = 3 ) at time t = 1s is given by:
r r r
v1 ( X , t ) = 2 X 2 t = 2 m / s ; v 2 ( X , t ) = X 3 = 3m / s ; v 3 ( X , t ) = 0
We can also observe that at time t = 1s the particle P occupies the position:
x1 = X 2 t 2 + X 1 = 3 ; x 2 = X 3t + X 2 = 4 ; x3 = X 3 = 3
So, the velocity of the particle P , (see Figure 2.2), can also be evaluated by (2.18) as:
r
v1 ( x , t ) = 2( x 2 − tx 3 )t = 2( 4 + 1 × 3) × 1 = 2m / s
 r
v 2 ( x , t ) = x 3 = 3m / s
v ( xr , t ) = 0
 3
Note that, the velocities obtained via the Lagrangian or Eulerian description are the same,
since velocity is an intrinsic property of the particle.
We can also provide the velocity of the particle P at time t = 2 s :
r
V1 ≡ v1 ( X , t ) = 2 X 2t = 2 × 2 × 1 = 4m / s
 r
V2 ≡ v2 ( X , t ) = X 3 = 3m / s
 r
V3 ≡ v3 ( X , t ) = 0

At time t = 2 s the new position of P is:


r r r
x1 ( X , t ) = X 2t 2 + X 1 = 6
x2 ( X , t ) = X 3t + X 2 = 7 ;
; x3 ( X , t ) = X 3 = 3
r r
As we can verify the Lagrangian description of motion x ( X , t ) describes the trajectory of
P.
Trajectory of particle P
r
viP ( x , t = 1s) = [2;3;0]
r
Vi P ( X P , t = 1s) = [2;3;0]
t = 1s
r
t0 Vi P ( X P , t = 2s ) = [4;3;0] P

X iP = [2;1;3]
P
xiP = [3;4;3]

t = 2s
xiP = [6;7;3]

r
viP ( x , t = 2s ) = [4;3;0]
Figure 2.2

University of Castilla-La Mancha Draft By: Eduardo W. V. Chaves (2016)


Ciudad Real - Spain
182 SOLVING PROBLEMS BY MEANS OF CONTINUUM MECHANICS

NOTE: Note that, the Eulerian velocity cannot be obtained by means of


r r
DX ( x , t ) r r r
= 0 ≠ v ( x , t ) . We can verify this by means of the proposed problem:
Dt
r r r r r
DX i ( x , t ) ∂X i ( x , t )  ∂X i ( x , t ) r ∂X i ( x , t ) r ∂X i ( x , t ) r 
= + v1 ( x , t ) + v 2 ( x, t ) + v3 ( x , t )
Dt ∂t  ∂x1 ∂x 2 ∂x 3 
thus:
r r r r r
DX 1 ( x , t ) ∂X 1 ( x , t )  ∂X 1 ( x , t ) r ∂X 1 ( x , t ) r ∂X 1 ( x , t ) r 
= + v1 ( x , t ) + v 2 ( x, t ) + v3 ( x , t )
Dt ∂t  ∂x1 ∂x 2 ∂x 3 
( ) [
= − 2tx 2 + 3t 2 x3 + 1 × 2( x 2 − tx3 )t − t 2 × x3 + t 3 × 0 = 0 ]
r r r r r
DX 2 ( x , t ) ∂X 2 ( x, t )  ∂X 2 ( x , t ) r ∂X 2 ( x , t ) r ∂X 2 ( x, t ) r 
= + v1 ( x , t ) + v 2 ( x, t ) + v 3 ( x, t )
Dt ∂t  ∂x1 ∂x 2 ∂x3 
= (− x3 ) + [0 × 2( x 2 − tx 3 )t + 1 × x3 − t × 0] = 0
r r r r r
DX 3 ( x , t ) ∂X 3 ( x , t )  ∂X 3 ( x, t ) r ∂X 3 ( x , t ) r ∂X 3 ( x , t ) r 
= + v1 ( x , t ) + v 2 ( x, t ) + v3 ( x, t )
Dt ∂t  ∂x1 ∂x 2 ∂x3 
= (0 ) + [0 × 2( x 2 − tx 3 )t + 0 × x3 + 1 × 0] = 0
r r r
Remember that u = x − X , then:
r r r r
r r
v ( X , t) =
Dx ( X , t ) D r r
Dt
=
Dt
( r r
u( X , t ) − X ( x , t ) = )Du( X , t ) r& r
Dt
≡ u( X , t )

Also, it fulfills that:


r r r r r r
r r r& r Du( x , t ) ∂u( x , t ) ∂u( x , t ) r r
v ( x , t ) = u( x , t ) ≡ = + r ⋅ v ( x, t )
Dt ∂t ∂x

Problem 2.7
The velocity field of the continuum, in Eulerian description, is given by:
x1 2 x2 3 x3
v1 = ; v2 = ; v3 = (2.20)
1+ t 1+ t 1+ t
r
a) Obtain the relationship between material and spatial coordinates xi = xi ( X , t ) ;
b) Obtain the acceleration components by means of the spatial motion description.
c) Obtain the acceleration components by means of the Lagrangian motion.
Solution:
r
dxi ( X , t )
a) Considering that vi = we can obtain:
dt
dx1 x dx dt
v1 = = 1 ⇒ 1 = (2.21)
dt 1 + t x1 1 + t
1 1
∫x 1
dx1 = ∫ 1 + t dt ⇒ Lnx 1 = Ln(1 + t ) + Ln(C1 ) ⇒
(2.22)
⇒ x1 = C1 (1 + t )

University of Castilla-La Mancha Draft By: Eduardo W. V. Chaves (2016)


Ciudad Real - Spain
2 CONTINUUM KINEMATICS 183

The initial condition is t = 0 ⇒ x1 = X 1 , with that the constant of integration is obtained:


C1 = X 1 , then
x1 = X 1 (1 + t ) (2.23)
dx 2 2 x 2 dx 2dt
v2 = = ⇒ 2 = (2.24)
dt 1 + t x2 1 + t
1 2
∫x 2
dx 2 = ∫ 1 + t dt ⇒ Lnx 2 = 2Ln(1 + t ) + LnC 2 ⇒ x 2 = C 2 (1 + t ) 2 (2.25)

for t = 0 ⇒ x 2 = X 2 ⇒ C 2 = X 2
x2 = X 2 (1 + t ) 2 (2.26)

dx3 3 x3 dx 3dt
v3 = = ⇒ 3 = (2.27)
dt 1 + t x3 1 + t
1 3
∫x 3
dx 3 = ∫ 1 + t dt ⇒ Lnx 3 = 3Ln(1 + t ) + LnC 3 ⇒ x 3 = C 3 (1 + t ) 3 (2.28)

and t = 0 ⇒ x3 = X 3 ⇒ C3 = X 3
x3 = X 3 (1 + t ) 3 (2.29)
Then, the equations of motion are:
x1 = X 1 (1 + t ) ; x2 = X 2 (1 + t ) 2 ; x3 = X 3 (1 + t )3 (2.30)
r r
b) By applying the material time derivative to the Eulerian velocity v ( x , t ) we can obtain
the acceleration as follows:
r r r r
Dv ( x , t ) r r ∂v ( x , t ) r r r r
= a ( x, t ) = + ∇ xr v ( x , t ) ⋅ v ( x , t ) (2.31)
Dt ∂t
which is indicial notation is represented by
∂v i ∂v
ai = + (vi , k )v k = i + (v i ,1v1 + v i , 2 v 2 + v i ,3 v 3 ) (2.32)
∂t ∂t
thus,
 x
x1 1 
a1 = − + 1 + 0 + 0 = 0
1 + t 1 + t
2
(1 + t ) 
2x2  2x 2  2 x2
a2 = − + 0 + 2 + 0 = (2.33)
(1 + t ) 2
 1+ t 1+ t  (1 + t )
2

3 x3  3x 3  6 x3
a3 = − + 0 + 0 + 3 =
(1 + t ) 2
 1 + t 1 + t  (1 + t ) 2
c) The Lagrangian velocity components are obtained by substituting the equations of
motion given by (2.30), i.e:
V1 = X 1 ; V2 = 2 X 2 (1 + t ) ; V3 = 3 X 3 (1 + t ) 2 (2.34)
In the same fashion we can obtain the Lagrangian acceleration components:
dV1 dV2 dV3
a1 = =0 ; a2 = = 2X2 ; a3 = = 6 X 3 (1 + t ) (2.35)
dt dt dt

University of Castilla-La Mancha Draft By: Eduardo W. V. Chaves (2016)


Ciudad Real - Spain
184 SOLVING PROBLEMS BY MEANS OF CONTINUUM MECHANICS

Problem 2.8
Consider the equations of motion:
x1 = X 1 ; x2 = X 2 + AX 3 ; x3 = X 3 + AX 2 (2.36)
where A is constant. Find the displacement vector field components in the material and
spatial descriptions.
Solution:
Lagrangian displacement vector:
u1 = x1 − X 1 = 0
r r r r components 
u = x( X ,t) − X   
→ u2 = x2 − X 2 = ( X 2 + AX 3 ) − X 2 = AX 3 (2.37)
u = x − X = ( X + AX ) − X = AX
 3 3 3 3 2 3 2

The equations of motion (2.36) in matrix form become:


 x1  1 0 0   X 1  1 0 0
 x  = 0 1 A  X  where [A ] = 0 1 A
 2    2 (2.38)
 x3  0 A 1   X 3  0 A 1 
14243
=[ A ]

The determinant and the inver of are represented, respectively, by:


1 0 0 1 − A2 0 0 
1  
det[A ] ≡ 0 1 A = 1 − A2 [A ]−1 = 2 
0 1 − A (2.39)
1− A 
0 A 1  0 − A 1 

thus the inverse of motion can be obtained as



 X 1 = x1
 X1  1 − A 2 0 0   x1  
X  = 1    1
 2  1 − A2  0 1 − A  x2  ⇒ X2 = ( x2 − Ax3 ) (2.40)
 0  1 − A2
 X 3 
 − A 1   x3 
 1
 X 3 = 1 − A2 ( x3 − Ax2 )

The Eulerian displacement vector components (spatial description) become:

u1 = x1 − X 1 = 0

 1 A( x3 − Ax 2 )
u 2 = x 2 − X 2 = x 2 − 2
( x 2 − Ax3 ) = (2.41)
 1− A 1 − A2
 1 A( x 2 − Ax3 )
u1 = x3 − X 3 = x3 − 2
( x3 − Ax 2 ) =
 1− A 1 − A2

Problem 2.9
Consider the equations of motion:
x1 = X 1 ; x2 = X 2 + X 3t ; x3 = X 3 + X 3t (2.42)
Obtain the velocity of the particles that are passing through the point (0,1,2) at time t1 = 0s
and t2 = 1s .

University of Castilla-La Mancha Draft By: Eduardo W. V. Chaves (2016)


Ciudad Real - Spain
2 CONTINUUM KINEMATICS 185

Solution:
The velocity field is given by:
r r r r
Dx ( X , t )
V ( X ,t) = (2.43)
Dt
r r
And by considering the equations of motion (2.42), x ( X , t ) , we can obtain the the
Lagrangian velocity components:
V1 = 0 ; V2 = X 3 ; V3 = X 3 (2.44)
Note that, in order to obtain the velocity of a particle we have to identify the particle, the is
identified by its material coordinate, i.e. the coordinate of the particle at time t = 0 s .
r r
At time t = 0 s we have x = X , then the particle in question is ( X 1 = 0, X 2 = 1, X 3 = 2) , thus
its velocity is:
V1 = 0 ; V2 = 2 ; V3 = 2 (2.45)
The material coordinates for the particle that is passing through the point
( x1 = 0, x 2 = 1, x3 = 2) at time t = 1s , can be obtained as follows:

x1 = 0 = X 1 

x 2 = 1 = X 2 + X 3  ⇒ ( X 1 = 0; X 2 = 0; X 3 = 1) (2.46)
x3 = 2 = X 3 + X 3 
And by means of Lagrangian velocity (2.44) we can obtain:
V1 = 0 ; V2 = 1 ; V3 = 1 (2.47)

Problem 2.10
By adopting the Cartesian system the particle motion is defined as follows:
r  ct   ct 
x1 ( X , t ) = X 1 sin  2  + X 2 cos 2
2  X +X  2

 X1 + X 2   1 2 

r  ct   ct  (2.48)
x2 ( X , t ) = − X 1 cos 2  + X 2 sin  2
2  X +X  2

 X1 + X 2   1 2 
r
x3 ( X , t ) = X 3
where c is a constant. Obtain the velocity components in spatial and material descriptions.
Solution:
The velocity components in the material (Lagrangian) description are:
r
r Dx1 ( X , t ) c   ct   ct 
V1 ( X , t ) = = 2 2  1
X cos 2  − X 2 sin  2
2   X + X 2

Dt X 1 + X 2   X1 + X 2   1 2 
r
r Dx2 ( X , t ) c   ct   ct 
V2 ( X , t ) = = 2 X sin  2  + X 2 cos 2  (2.49)
2  1 2   2 
Dt X 1 + X 2   X1 + X 2   X 1 + X 2 
r
r Dx3 ( X , t )
V3 ( X , t ) = =0
Dt
Taking into account the equation (2.48), we can note that the following relationship holds:

University of Castilla-La Mancha Draft By: Eduardo W. V. Chaves (2016)


Ciudad Real - Spain
186 SOLVING PROBLEMS BY MEANS OF CONTINUUM MECHANICS

x12 + x22 = X 12 + X 22 (2.50)


And by considering the above equation into the equation (2.48), the velocity components
in the spatial (Eulerian) description are:
r −cx r c x1 r
v1 ( x , t ) = 2 22 ; v2 ( x , t ) = ; v3 ( x , t ) = 0 (2.51)
x1 + x2 x12 + x22
r r r
The inverse equations of motion, X = X ( x , t ) , are:
  ct   ct  
 sin  2  − cos 2
2 

 x + x 2  0
  x1 + x2   1 2   x 
 X1 

 X  = cos c t  sin  c t  0  x 
1
   
 2  x2 + x2   x2 + x2   2 (2.52)
 X 3    1 2   1 2   
   x3 
 0 0 1
 

Problem 2.11
The Eulerian velocity field components are:
x2
v1 = x1 ; v2 = ; v3 = 0 (2.53)
2t + 3
Find the parametric equations of the trajectory of the particle which was at ( X 1 , X 2 , X 3 ) in
the reference configuration.
Solution:
r r r
Remember that the trajectory of a particle is given by the equation x = x ( X , t ) . Then, to
find the path line (trajectory) we must solve the system:
dx1 dx2 x dx3
= x1 ; = 2 ; =0 (2.54)
dt dt 2t + 3 dt
with the initial conditions
 x1 (t = 0) = X 1

 x2 (t = 0) = X 2 (2.55)
 x (t = 0) = X
 3 3
x1 t
dx1  x 

X1
x1 0 ∫
= dt ⇒ Ln 1  = t
 X1 
⇒ x1 = X 1 exp t

x2
(2.56)
( ) ( )
t
dx 2 dt  x  2

X2
x2
=∫0
2t + 3
⇒ Ln 2
 X2
 = Ln 2t + 3 − Ln 3

⇒ x2 = X 2
3
t +1

x3 = X 3

Then, the equations of motion are given by:


2
x1 = X 1exp t ; x2 = X 2 t + 1 ; x3 = X 3 (2.57)
3

University of Castilla-La Mancha Draft By: Eduardo W. V. Chaves (2016)


Ciudad Real - Spain
2 CONTINUUM KINEMATICS 187

Problem 2.12
Consider the following equations of motion:
x1 = X 1 ; x2 = 2 t X 3 + X 2
x3 = X 3 ; (2.58)
r
and a physical quantity represented by the scalar field q ( x , t ) in the Eulerian description:
r
q ( x , t ) = 2 x1 + x 2 − x3 + 1 (2.59)
a) Obtain the Lagrangian description of the physical quantity;
b) Obtain the Lagrangian and Eulerian velocities;
c) Obtain the rate of change of the physical quantity.
d) Obtain the local rate of change of q at the spatial point (1,3,2) .
Solution:
a) The Lagrangian description can be obtained by substituting the equations of motion
r r r r r r
x ( X , t ) into the Eulerian variable, i.e. q ( x , t ) = q ( x ( X , t ), t ) = Q( X , t ) . Then, by substituting
r
the equations of motion (2.58) into the equation of the variable q ( x , t ) given by (2.59) we
can obtain:
r
Q( X , t ) = 2 X 1 + X 2 + ( 2t − 1) X 3 + 1 (2.60)
b) The velocity vector is defined by
r r r r
Dx ( X , t )
V ( X , t) = (2.61)
Dt
And by considering the equations of motion (2.58) we can obtain the Lagrangian velocity:
V1 = 0 ; V2 = 2 X 3 ; V3 = 0 (2.62)
The inverse of the equations of motion is:
 x1 = X 1  X 1 = x1
 inverse 
 x2 = 2 t X 3 + X 2 →  X 2 = x 2 − 2 t x3
x = X X = x
 3 3  3 3

Then, the Eulerian velocity components are given by:


v1 = 0 ; v2 = 2 x3 ; v3 = 0 (2.63)
c) The rate of change of the variable is obtained by applying the material time derivative. If
we are dealing with Lagrangian variable the material time derivative is given by:
D r
Q& = Q( X , t ) = 2 X 3 (2.64)
Dt
and if we are dealing with Eulerian variables the material time derivative is given by
r
∂q ( x , t ) r
q& = + (∇ xr q ) ⋅ v
1
42 ∂t4 3 (2.65)
= 0 ( steady )

 ∂q ∂q ∂q 
q& = 0 + q, i v i = 0 +  v1 + v2 + v 3  = [(2)(0) + (1)(2 x 3 ) + ( −1)(0)] = 2 x 3 (2.66)
 ∂x1 ∂x 2 ∂x 3 

University of Castilla-La Mancha Draft By: Eduardo W. V. Chaves (2016)


Ciudad Real - Spain
188 SOLVING PROBLEMS BY MEANS OF CONTINUUM MECHANICS

We could have obtained the same result by starting from Q& = 2X 3 in which we substitute
X 3 = x3 , thus
r r r r
q& ( x , t ) = Q& ( X ( x , t ), t ) ⇒ q& ( x , t ) = 2 x3 (2.67)
r
d) Note that the physical quantity field is stationary, i.e. q = q ( x ) , then the local rate of
r
∂q ( x )
change is = 0 at any spatial point.
∂t

Problem 2.13
Given the Lagrangian displacement field:
u1 = ktX 2 ; u2 = 0 ; u3 = 0
r
and the Eulerian temperature field T ( x , t ) = ( x1 + x2 ) t .
a) Find the rate of change of temperature for a particle that at time t = 1s is passing
through the point (1,1,1) .
Solution:
r r
r ∂T ∂T ∂x r DT ( X ,t)
We can apply the definition T& ( x , t ) = + r⋅ or T& ( X , t ) = in order to
∂t ∂x ∂t Dt
obtain the material time derivative.
By means of the equation u i = xi − X i we can obtain the equations of motion:
u1 = x1 − X 1 ⇒ x1 = u1 + X 1 ⇒ x1 = X 1 + ktX 2
u2 = x2 − X 2 ⇒ x2 = u2 + X 2 ⇒ x2 = X 2
u3 = x3 − X 3 ⇒ x3 = u3 + X 3 ⇒ x3 = X 3
The Lagrangian temperature field (material description) can be obtained as follows:
r r r
T ( x ( X , t ), t ) = ( x1 + x2 ) t = (( X 1 + ktX 2 ) + ( X 2 ) ) t = X 1t + kX 2t 2 + X 2t = T ( X , t )
Then, the material time derivative becomes:
r
r DT ( X ,t) D
T& ( X , t ) = = [ X 1t + kX 2t 2 + X 2t ] = X 1 + 2kX 2t + X 2 (2.68)
Dt Dt
If we want to find the rate of change of temperature of the particle which is passing
through the point x1 = 1, x 2 = 1, x3 = 1 at t = 1s , we have two possibilities, namely: 1)
Finding the position of said particle in the reference configuration and replacing it in the
above equation. 2) The other possibility is by means of the equation of the rate of change
of temperature in the spatial (Eulerian) description. To do this, we will need to establish the
r r r
inverse of the equations of motion, i.e.: X = X ( x , t ) :
 x1 = X 1 + ktX 2  X 1 = x1 − ktx 2
 
 x2 = X 2 ⇒  X 2 = x2
x = X X = x
 3 3  3 3

And by substituting the above equations into the equation (2.68) we can obtain:
r r r
T& ( X ( x , t ), t ) = X 1 + 2kX 2 t + X 2 = ( x1 − ktx 2 ) + 2kt ( x 2 ) + ( x 2 ) = T& ( x , t )
r
by simplifying the above equation we can obtain T& ( x , t ) = x1 + ktx 2 + x 2 . Then:

University of Castilla-La Mancha Draft By: Eduardo W. V. Chaves (2016)


Ciudad Real - Spain
2 CONTINUUM KINEMATICS 189

T& ( x1 = 1, x 2 = 1, x3 = 1, t = 1) = (1 − k ) + 2k + 1 = k + 2
Alternative solution:
r
r ∂T ∂T ∂x  ∂T ∂x1 ∂T ∂x 2 ∂T ∂x 3 
T& ( x , t ) = + r⋅ = ( x1 + x 2 ) +  + + 

∂t ∂x ∂t  ∂x1 ∂t ∂x 2 ∂t ∂x 3 ∂t 
= (x1 + x 2 ) + (tkX 2 + t (0) + (0)(0) ) = x1 + x 2 + tkX 2
Note that x 2 = X 2 , then:
r
T& ( x , t ) = x1 + x 2 + tkx 2

Problem 2.14
Let us consider the following equations of motion:
t t
x1 = X 1 ; x2 = X 2 + X3 ; x3 = X 3 + X2 (2.69)
2 2
a) Is this motion possible? Justify your answer;
b) Obtain the velocity components in the Lagrangian and Eulerian descriptions;
c) Obtain the path line (trajectory equation).
Solution:
a) Obtaining the Jacobian determinant:
1 0 0
∂xi t2
J= F = = 0 1 2t = 1 − (2.70)
∂X j 4
0 2t 1

The motion is possible if J = F > 0 :

t2
J =1− >0⇒t <2 s (2.71)
4
b) The Lagrangian velocity components are obtained as follows:
r
 Dx1 ( X , t )
V1 = =0
 Dt
r
 Dx1 ( X , t ) D  t  X3
V2 = =  X2 + X3  = (2.72)
 Dt Dt  2  2
r
 Dx ( X , t ) D  t  X2
V3 = 1 =  X3 + X2  =
 Dt Dt  2  2
The inverse of the equations of motion is given by:
 x1  1 0 0   X 1   X1  J 0 0   x1 
 x  = 0 1 t   X  inverse   1
→ X 2  =  0 1

− 2t   x 2 
 2  2  2 (2.73)
J
 
 x3  0 2 1   X 3 
t  X 3  0 − 2t 1   x3 

By substituting the values of X i given by the above equations into the Lagrangian velocity
(2.72) we can obtain the velocity in the spatial description:

University of Castilla-La Mancha Draft By: Eduardo W. V. Chaves (2016)


Ciudad Real - Spain
190 SOLVING PROBLEMS BY MEANS OF CONTINUUM MECHANICS

t t
x3 −
x2 x2 −
x3
2 2 x − tx 2 = 2 x 2 − tx 3
v1 = 0 ; v 2 = 2
= 3 22 ; v3 = (2.74)
t 4−t t2 4 − t2
2− 2−
2 2
c) The trajectory can be obtained by eliminating t of the equations of motion (2.69):
 x1 = X 1

 X2 X2 (2.75)
( x3 − X 3 ) X 3 = ( x 2 − X 2 ) X 2 ⇒ x3 =
X3
x2 − 2 + X 3
X3

Problem 2.15
The Eulerian velocity field for a steady fluid is given by:
r r b2 ( x 2 − x 2 ) b2 x x
v ( x ) = U 2 1 2 22 eˆ 1 + 2U 2 1 22 2 eˆ 2 + Veˆ 3 (2.76)
( x1 + x2 ) ( x1 + x2 )
where U and V are constants.
r
Show that ∇ xr ⋅ v = 0 and find the Eulerian acceleration field.
Solution:
r ∂v1 ∂v 2 ∂v3 2 2
2 x1 ( x1 − 3 x 2 )
2 2
2 x1 ( x1 − 3 x 2 )
∇ xr ⋅ v = v i ,i = + + = −2Ub + 2Ub =0
∂x1 ∂x 2 ∂x3 ( x12 + x 22 ) 3 ( x12 + x 22 ) 3
The Eulerian acceleration field:
r
r r ∂v r r r r
a( x) = + (∇ xr v ) ⋅ v = (∇ xr v ) ⋅ v
∂t
{r
=0

The components of the spatial velocity gradient are given by:


 ∂v1 ∂v1  ∂v1
 
 ∂x1 ∂x3  ∂x2  x1 (3 x22 − x12 ) − x2 (3 x12 − x22 ) 0
∂v ∂v ∂v2  ∂v2 2Ub 2
 
(∇ xr v )ij ≡ i =  2
r
 = 2 2 3 
− x2 (3 x12 − x22 ) − x1 (3 x22 − x12 ) 0
∂x j ∂x ∂x3 ∂x2
( x1 + x2 ) 
 1
∂ ∂v3 

∂v3  0 0 0
 v3
 ∂x1∂x3  ∂x2
r r
The acceleration components are given by ai = (∇ xr v )ij (v ) j :

 b 2 ( x12 − x22 )   − 2 x1U 2b 4 


U 2 2 2   2 2 3 
 x1 (3 x22 − x12 ) − x2 (3 x12 − x22 ) 0  ( x1 + x2 )   ( x1 + x2 ) 
∂v 2Ub 2  2 2 2 2  b 2 x1 x2   
ai = i v j = 2  − x2 (3 x1 − x 2 ) − x1 (3 x2 − x1 ) 0   2U 2 2
=  − 2 x2U 2b 4 
∂x j 2 3
( x1 + x2 )  2
( x1 + x2 )   2
0 0 0  2 3 
    ( x1 + x2 ) 
 V   
   0 

University of Castilla-La Mancha Draft By: Eduardo W. V. Chaves (2016)


Ciudad Real - Spain
2 CONTINUUM KINEMATICS 191

Problem 2.16
Calculate the material time derivative for the property φ when said property is described as
follows:
r
ƒ Material description: φ ( X , t ) = X 1t 2 ;
r x1t 2
ƒ Spatial description: φ ( x , t ) = .
(1 + t )
Consider that the equations of motion by x1 = x1 ( X 1 ) , i.e. it is independent of X 2 and X 3 .
Solution:
r
a) Material time derivative of φ ( X , t ) = X 1t 2 :
D r r
φ( X , t ) ≡ φ& ( X , t ) = 2 X 1t
Dt
r x1t 2
b) Material time derivative of φ ( x , t ) = :
(1 + t )
r r r
D r ∂φ ( x , t ) r ∂φ ( x , t ) ∂φ ( x , t )
φ ( x, t ) = + (∇ xφ ) ⋅ v =
r + vi
Dt ∂t ∂t ∂xi
r r r r
∂φ ( x , t )  ∂φ ( x , t ) ∂φ ( x , t ) ∂φ ( x , t ) 
= + v1 + v2 + v3  (2.77)
∂t  ∂x1 ∂x2 ∂x3 
r
∂  x t 2   ∂φ ( x , t ) 
=  1  +  v1 + 0 + 0
∂t  (1 + t )   ∂x1 
We need to know the velocity component v1 . We start from the principle that a property is
intrinsic to the particle, then:
r r r r x1t 2 x1
φ ( X , t ) = X 1t 2 ⇒ φ ( X ( x, t ), t ) = φ ( x, t ) = ⇒ X1 =
(1 + t ) (1 + t )
The velocity becomes:
r r
v ( X ,t) =
D
Dt
( )
X 1t 2 = 2 X 1t eˆ 1 ⇒
r r x
v ( x , t ) = 2 1 t eˆ 1
(1 + t )
Then, the material time derivative (2.77) becomes:
r
D r ∂  x1t 2   ∂φ ( x , t )   2 x1t x1t 2   t 2
+

φ ( x, t ) =   +  v1 =  − 2 
X1 
Dt ∂t  (1 + t )   ∂x1   (1 + t ) (1 + t )   (1 + t ) 
 2 x1t x t2   t2 x  2 x1t
=  − 1 2  +  2 1 t =
 (1 + t ) (1 + t )   (1 + t ) (1 + t )  (1 + t )
D r r
We could also have obtained the same result by starting from φ( X , t ) ≡ φ& ( X , t ) = 2 X 1t
Dt
x1
and by substituting X 1 = , i.e.:
(1 + t )
D r r D r r r r r x
φ ( X , t ) ≡ φ& ( X , t ) = 2 X1t ⇒ φ ( X ( x, t ), t ) = φ& ( X ( x, t ), t ) = φ& ( x, t ) = 2 1 t
Dt Dt (1 + t )

University of Castilla-La Mancha Draft By: Eduardo W. V. Chaves (2016)


Ciudad Real - Spain
192 SOLVING PROBLEMS BY MEANS OF CONTINUUM MECHANICS

Problem 2.17
Consider the following equations of motion in the Lagrangian description:
 x1 = X 1t 2 + 2 X 2t + X 1  x1  t + 1 2t
2
0   X1 
 2
Matrix
form    2  
 x2 = 2 X 1t + X 2t + X 2 →  x2  =  2t t +1 0  X 2  (2.78)
x = 1 X t + X x   0 0 1
t + 1  X 3 
 3 2 3 3  3  2

Find the components of the displacement vector in Lagrangian and Eulerian descriptions.
Solution:
r r r
By definition the displacement vector is obtained by u = x − X , then by substituting the
equations of motion (2.78) we can obtain:
r r
u1 ( X , t ) = x1 ( X , t ) − X 1 = ( X 1t 2 + 2 X 2t + X 1 ) − X 1 = X 1t 2 + 2 X 2t
 r r 2 2
u2 ( X , t ) = x2 ( X , t ) − X 2 = ( 2 X 1t + X 2t + X 2 ) − X 2 = 2 X 1t + X 2t
 r r
u3 ( X , t ) = x3 ( X , t ) − X 3 = ( 2 X 3t + X 3 ) − X 3 = 2 X 3t
1 1

which are the displacement components in the Lagrangian description (material).


To obtain the Eulerian displacement we will need to obtain the inverse equations of
motion (2.78), which is:
 2tx2 − x1 (1 + t )
   X 1 = 3t 3 − 1 − t − t 2
 1
X − (1 + t ) 2t 0 x  
  1   1   2 x t 2 − x2 (1 + t 2 )
X2 = 3 2 
2t 2 − (1 + t 2 ) 0   x2  ⇒  X 2 = 1 3
 X  3t − 1 − t − t  3t 3 − 1 − t − t 2   x3   3t − 1 − t − t 2
 3  0 0   2 x3
(t + 2)   X 3 = (t + 2)
1
 2

r r r
We can also use the definition u = x − X , but now we replace the material coordinate to
obtain the displacement vector components in the Eulerian description:
 r r 2tx2 − x1 (1 + t )
u1 ( x , t ) = x1 − X 1 ( x , t ) = x1 − 3t 3 − 1 − t − t 2

 r r 2 x1t 2 − x2 (1 + t 2 )
u 2 ( x , t ) = x2 − X 2 ( x , t ) = x2 −
 3t 3 − 1 − t − t 2
 r r 2 x3
u3 ( x , t ) = x3 − X 3 ( x , t ) = x3 − (t + 2)

University of Castilla-La Mancha Draft By: Eduardo W. V. Chaves (2016)


Ciudad Real - Spain
2 CONTINUUM KINEMATICS 193

Problem 2.18
The following equations describe the
motion of a body, (see Figure 2.3): X 2 , x2 Reference configuration
 x1 = X 1 + 0.2 X 2 t t = 0s

x2 = X 2 C
1
B
x = X
 3 3 1
E
At time t = 0 , the cube of side 1 has 1
one vertex at the origin of the system
which is indicated by point O, (see O A
Figure 2.3). Obtain the configuration X 1 , x1
D
of the body at time t = 2 s . G

X 3 , x3

Figure 2.3: Reference configuration t = 0 .


Solution:
To obtain the current configuration of the body at time t = 2 s , we will analyze the particle
motion. The particle which occupies position O (origin) at t = 0 has material coordinate:
X1 = 0 ; X2 = 0 ; X3 = 0
and by substituting the above coordinates into the equations of motion we can obtain:
 x1 = 0

x i ( X 1 = 0, X 2 = 0, X 3 = 0, t ) ⇒  x 2 = 0
x = 0
 3
Then, we can conclude that the particle O does not change its position during motion.
The particles lying on the OA line, in the initial configuration, have the reference
coordinate ( X 1 , X 2 = 0, X 3 = 0) . In spatial coordinates:
 x1 = X 1 + 0.2 X 2 t = X 1

x2 = X 2 = 0
x = X = 0
 3 3

That is, all particles lying on the OA line do not move during motion. Similarly, we can
verify that the line ( X 1 , X 2 = 0, X 3 = 1) in the reference configuration ( X 1 , X 2 = 0, X 3 = 1)
does not move:
x1 = X 1 + 0.2 × 0 × 2 = X 1 ; x2 = X 2 = 0 ; x3 = X 3 = 0

The particles lying on the CB line, ( X 1 , X 2 = 1, X 3 = 0) , at time t = 2 s will move according


to:
x1 = X 1 + 0.2 × 1 × 2 = X 1 + 0.4 ; x2 = X 2 = 1 ; x3 = X 3 = 0

Then, all particles lying on the CB line will move 0.4 according to x1 -direction.
The particles belonging to line OC at t = 0 , will move to positions:

University of Castilla-La Mancha Draft By: Eduardo W. V. Chaves (2016)


Ciudad Real - Spain
194 SOLVING PROBLEMS BY MEANS OF CONTINUUM MECHANICS

 x1 = X 1 + 0.2 X 2 t = 0 + 0.2 × 2 × X 2 = 0.4 X 2



x2 = X 2
x = X = 0
 3 3

Following the same procedure for the remaining particles, we can obtain the final
configuration of the body at time t = 2 s , (see Figure 2.4).

x2

0.4 0.4

C C’ 1 B B’ Current configuration at
t = 2s
E E’ 1
A=A’
O
1 x1
D
G=G’

x3

Figure 2.4: Body configuration at time t (deformed configuration).

Problem 2.19
Consider the equations of motion:
 x1 = X 1 + t 2 X 2  x1  1 t2 0  X 1 
 2
Matrix
form    2  
 x 2 = t X 1 + X 2   →  x 2  = t 1 0  X 2 
x = X x  0 0 1   X 3 
 3 3  3 
a) Obtain the trajectory of particle Q which originally at time t 0 was at X i = (1,2,1) ;
b) By considering the current configuration at t = 0.5 s , obtain the velocity and acceleration
components of the particle P that was originally at X i = (16 ; −4 ;1) ;
15 15

c) Obtain the equations of motion in the Eulerian description;


d) Obtain the velocity and acceleration components of one particle that at time ( t = 0.5 s ) is
passing through the point xi = (1,0,1) .
Obs.: Consider the International System of Units (SI-Units).
Solution:
1)Using the equations of motion and by substituting the material coordinates of the point
X i = (1,2,1) , we can obtain:

x 1 = 1 + 2t 2 ; x2 = 2 + t 2 ; x3 = 1
The above equations represent the motion of the particle. To obtain the trajectory, we
eliminate the time of the equations of motion, i.e.:
x1 − 2 x2 = −3 ; x3 = 1

University of Castilla-La Mancha Draft By: Eduardo W. V. Chaves (2016)


Ciudad Real - Spain
2 CONTINUUM KINEMATICS 195

which indicates that the particle moves in a straight line defined by ( x1 − 2 x 2 = −3) on the
plane x3 = 1 , (see Figure 2.5).

X 3 , x3 Particle trajectory

( x1 − 2 x 2 = −3)
x3 = 1

X 2 , x2

X 1 , x1

Figure 2.5
2) The velocity and acceleration components of the particle P are given by:
r r V1 = 2tX 2
r r Dx ( X , t ) components 
V ( X , t) =   → V2 = 2tX 1
Dt V = 0
 3

r r  A1 = 2 X 2
r r Dv ( X , t ) components 
A( X , t ) =   →  A2 = 2 X 1
Dt A = 0
 3
Then, the particle which was originally located at the point X i = (16 ; −4 ;1) will achieve a
15 15
new configuration at t = 0.5 s . In this configuration, velocity and acceleration for the
particle are respectively:
V1 = 2 × 0.5 × 15
−4
= 15
−4
m/s  A1 = 2 × 15
−4 −8
= 15 m / s2
 
V 2 = 2 × 0.5 × (15 ) = 15 m / s  A2 = 2 × (15 ) = 15 m / s
16 16 16 32 2
and
V = 0 A = 0
 3  3

3) The inverse of the equations of motion can be obtained as follows:


 x1 − t 2 x 2
X1 =
 x1 = X 1 + t 2 X 2 ⇒ X 1 = x1 − t 2 X 2  1− t4
 2 2
 x 2 − t 2 x1
x2 = t X 1 + X 2 ⇒ X 2 = x2 − t X 1 ⇒ X 2 = (2.79)
x = X ⇒ X = x  1− t4
 3 3 3 3
 X 3 = x3



4) The velocity and acceleration of the particle that at time ( t = 0.5 s ) is passing through the
point xi = (1,0,1) can be obtained by means of velocity and acceleration in Eulerian
description:

University of Castilla-La Mancha Draft By: Eduardo W. V. Chaves (2016)


Ciudad Real - Spain
196 SOLVING PROBLEMS BY MEANS OF CONTINUUM MECHANICS

Velocity:
 x 2 − t 2 x1  −4
 1 v = 2t
 1− t4 v1 = 15 m / s
V1 = 2tX 2 
 substituti ng
 x1 − t 2 x 2 t = 0. 5 s  16
V
 2 = 2 X 1 t     →  2v = 2 t 4
 → v 2 = m/s
V = 0 X1 , X 2
 1 − t x (1, 01)
 15
 3 v 3 = 0 v 3 = 0
 
 

Acceleration:
 x 2 − t 2 x1  8
 1 a = 2 2

 1− t4 a1 = − 15 m / s
 A1 = 2 X 2 
 substituti ng
 x1 − t 2 x 2 t = 0. 5 s  32
A
 2 = 2 X 1      →  2a = 2 4
 → a 2 = m / s2
A = 0 X1 , X 2
 1 − t x (1, 01)
 15
 3 a 3 = 0 a 3 = 0
 
 

We can obtain the initial position X i of the particle by using the inverse of the equations
of motion which is represented by the equations in (2.79), in which we consider xi (1,0,1) :
 x1 − t 2 x 2 1 − (0.5 2 )(0) 16
 1X = = =
 1− t4 1 − (0.5) 4 15
 2
x 2 − t x1 0 − (0.5 )(1)
2
 4
X 2 = 4
= 4
=−
 1− t 1 − (0.5) 15
X = x = 1
 3 3



We can verify that it is the same particle P referred to in paragraph 2. It is logical that we
have obtained the same velocity and acceleration using either the material or spatial
description, since the velocity and acceleration are intrinsic properties of the particle.

Problem 2.20
The acceleration vector field is described by:
r r
r r D v ∂v r r
a ( x, t ) = = + (∇ xr v ) ⋅ v
Dt ∂t
Show that acceleration can also be written as:
r r r
D v ∂v  v2  r r r ∂v  v2  r r
= + ∇ x   − v ∧ (∇ x ∧ v ) ≡
r r + ∇ xr   − v ∧ rot v
Dt ∂t  2 ∂t  2
Solution:
To show the above relationship one need only demonstrate that:
r r  v2  r r r
(∇ xr v ) ⋅ v = ∇ xr   − v ∧ (∇ xr ∧ v )
 2
Expressing the terms on the right of the equation in symbolic notation we can obtain:

University of Castilla-La Mancha Draft By: Eduardo W. V. Chaves (2016)


Ciudad Real - Spain
2 CONTINUUM KINEMATICS 197

 v2  r r r 1 ∂   ∂ 
∇ xr   − v ∧ (∇ xr ∧ v ) = eˆ i (v j v j )  − (vi eˆ i ) ∧  eˆ r ∧ (vs eˆ s ) 
 2  2  ∂xi   ∂xr 
Using the definition of the permutation symbol, (see Chapter 1), we can express the vector
product as:
 v2  r r r 1 ∂  ∂v
∇ xr   − v ∧ (∇ xr ∧ v ) = eˆ i (v j v j )  − (vi eˆ i ) ∧  rst s eˆ t
 2 2  ∂xi  ∂x r
1 ∂v j  ∂v s ˆ
= eˆ i 2v j  −  rst itk vi ek
2 ∂xi  ∂xr

where we have used the equation eˆ i ∧ eˆ t =  itk eˆ k . In Chapter 1 we also proved that
 rst  itk =  rst  kit = δ rk δ si − δ ri δ sk , then:
 v2  r r r ∂v j ∂v s
∇ xr   − v ∧ (∇ xr ∧ v ) = v j
 eˆ i − (δ rk δ si − δ ri δ sk )v i eˆ k
 2  ∂x i ∂x r
∂v j  ∂v ∂v 
=vj eˆ i −  δ rk δ si v i s − δ ri δ sk v i s eˆ k
∂x i  ∂x r ∂x r 
∂v j  ∂v ∂v 
=vj eˆ i −  v s s − v i k eˆ k
∂x i  ∂x k ∂x i 

 v2  r r r ∂v j ∂v ∂v
∇ xr   − v ∧ (∇ xr ∧ v ) = v j eˆ i − vs s eˆ k + vi k eˆ k
 2 ∂xi ∂x k ∂xi
∂v j ∂ v ∂v
= δ sj vs eˆ i − vs s δ ik eˆ i + vi k eˆ k
∂xi ∂xk ∂xi
∂v s ˆ ∂v s ˆ ∂vk ˆ
= vs e i − vs e i + vi ek
∂xi ∂xi ∂xi
r r r
∂v eˆ ∂ (v ) ∂ (v ) ∂ (v ) ˆ ˆ
= k k vi = vi = δ ij v j = (e i ⋅ e j )v j
∂xi ∂xi ∂xi ∂xi
r
 ∂ (v )  r r
=  eˆ i  ⋅ eˆ j v j = (∇ xr v ) ⋅ v
 ∂xi 
NOTE: We have already discussed this problem in Chapter 1, (see Problem 1.120).

Problem 2.21
r r r
Consider the equations of motion x ( X , t ) and the temperature field T ( x , t ) given
respectively by:
 x1 = X 1 (1 + t )
 r
 x2 = X 2 (1 + t ) and T ( x ) = x12 + x22
x = X
 3 3

Find the rate of change of temperature for the particle P at time t = 1s given that particle
P was at point ( X 1 = 3, X 2 = 1, X 3 = 0) at time t = 0 .
Solution 1:
In this first solution we first obtain the material time derivative of the Lagrangian
r
temperature, so, we have to obtain the temperature in Lagrangian description T ( X , t )
(Lagrangian temperature):

University of Castilla-La Mancha Draft By: Eduardo W. V. Chaves (2016)


Ciudad Real - Spain
198 SOLVING PROBLEMS BY MEANS OF CONTINUUM MECHANICS

r
T ( x ) = x12 + x 22

By substi tuting
the equations of motion

r
T ( X , t ) = X 12 (1 + t ) 2 + X 22 (1 + t ) 2
The material time derivative of the Lagrangian temperature is given by:
r
r DT dT ( X , t)
T& ( X , t ) ≡ = = 2 X 12 (1 + t ) + 2 X 22 (1 + t )
Dt dt
By substituting t = 1s , ( X 1 = 3, X 2 = 1, X 3 = 0) , into the above equation we obtain:
r K
⇒ T& ( X , t ) = 2 X 12 (1 + t ) + 2 X 22 (1 + t ) = 2(3) 2 (1 + 1) + 2(1) 2 (1 + 1) = 40
s
Solution 2:
In this alternative solution we directly use the definition of material time derivative of the
r r
r DT ∂T ( x ) ∂T ( x ) r
Eulerian variable, i.e. T& ( x , t ) = = + vk ( x, t ) .
Dt ∂t ∂x k
From the equations of motion we obtain:
r
 x1 = X 1 (1 + t ) v1 ( X , t ) = X 1
 velocity  r
 x 2 = X 2 (1 + t )  → v 2 ( X , t ) = X 2
x = X  r
 3 3 v 3 ( X , t ) = 0
The equations of motion in Eulerian description are given by:
 x1
 X 1 = (1 + t )
 x1 = X 1 (1 + t ) 
 inverse of motion  x2
 x2 = X 2 (1 + t )    →  X 2 =
x = X  (1 + t )
 3 3  X 3 = x3


So, it is possible to obtain the Eulerian velocity as follows:
 r r r x1 r
V1 ( X ( x , t ), t ) = X 1 ( x , t ) = (1 + t ) = v1 ( x , t )

 r r r x2 r
V 2 = ( X ( x , t ), t ) = X 2 ( x , t ) = = v 2 ( x, t )
 (1 + t )
r
V3 = v 3 ( x , t ) = 0


r
Afterwards, the material time derivative of the Eulerian temperature, T ( x , t ) , is given by:
r r
DT ( x , t ) & r ∂T ( x )  ∂T ∂T ∂T 
⇒ ≡ T ( x, t ) = + v1 + v2 + v3 
Dt 12∂t3  ∂x1 ∂x 2 ∂x 3 
= 0 (Stationar y field)

r x x r 2x 2 2x 2 2
⇒ T& ( x , t ) = 2 x1 1 + 2 x 2 2 + 0 ⇒ T& ( x , t ) = 1 + 2 = ( x12 + x 22 )
1+ t 1+ t 1+ t 1+ t 1+ t
The position of particle P at time t = 1s is evaluated as follows:

University of Castilla-La Mancha Draft By: Eduardo W. V. Chaves (2016)


Ciudad Real - Spain
2 CONTINUUM KINEMATICS 199

 x1 = X 1 (1 + t ) = 3(1 + 1) = 6

 x 2 = X 2 (1 + t ) = 1(1 + 1) = 2
x = X = 0
 3 3

Then, by substituting the spatial coordinates in the expression of the material time
derivative of temperature we obtain:
r 2 2
T& ( x , t ) = T& ( x1 = 6, x 2 = 2, x 3 = 0, t = 1) = ( x12 + x 22 ) = (6 2 + 2 2 ) = 40
1+ t 1+1
r
Alternatively, the expression T& ( x , t ) could also have been obtained as:
r
T& ( X , t ) = 2 X 12 (1 + t ) + 2 X 22 (1 + t )
2 2
r r r 2 r 2  x   x 
T& ( X ( x , t ), t ) = 2[X 1 ( x , t )] (1 + t ) + 2[X 2 ( x , t )] (1 + t ) = 2  1  (1 + t ) + 2  2  (1 + t )
 (1 + t )   (1 + t ) 
2 r
= ( x12 + x 22 ) = T& ( x , t )
(1 + t )

Problem 2.22
Consider the motion:
xi = X i (1 + t ) ( t > 0)
Obtain the velocity field in the spatial description.
Solution:
The velocity is obtained by means of the material time derivative of the equations of
motion:
d
Vi = x& i = [X i (1 + t )] = X i (2.80)
dt
To find the velocity in the spatial description we will need to obtain the inverse of the
equations of motion which is
xi
xi = X i (1 + t ) ⇒ X i =
(1 + t )
and by substituting into the equation (2.80) we obtain the Eulerian velocity:
r x
vi = X i ( x , t ) = i
1+ t

Problem 2.23
r
The equations of motion and the temperature field T ( x ) are given respectively by:
r
xi = X i (1 + t ) (i = 1,2) ; T ( x ) = 2( x12 + x22 )
Find the rate of change of temperature at time t = 1s for one particle that was at position
(1,1) in the reference configuration.
r
Note that the temperature field is a steady field, i.e. T = T ( x ) .

University of Castilla-La Mancha Draft By: Eduardo W. V. Chaves (2016)


Ciudad Real - Spain
200 SOLVING PROBLEMS BY MEANS OF CONTINUUM MECHANICS

Solution 1:
We can obtain the equation for temperature in the material description:
r
 T ( x ) = 2( x12 + x22 )
 ↓
by substi tuting the equations
 of motion

 r
 [
T ( X , t ) = 2 X 2 (1 + t ) 2 + X 2 (1 + t ) 2
1 2 ]
Then, the material time derivative can be obtained as follows:
r
r
⇒ T& ( X , t ) =
DT
Dt
=
dT (
dt
X , t)
[
= 2 2 X 12 (1 + t ) + 2 X 22 (1 + t ) ]
By substituting t = 1s and the material coordinates ( X 1 = 1; X 2 = 1) into the above equation
we can obtain:
K
⇒ T& ( X 1 = 1; X 2 = 1; t = 1) = 16
s
Solution 2:
In this alternative solution we directly use the definition of the material time derivative of
Eulerian property:
r
T ( x ) = 2( x12 + x12 ) xi = (1 + t ) X i
; (i = 1,2)
r r
r DT ∂T ( x ) ∂T ( x ) ∂xk
⇒ T& ( x , t ) = = + (i = 1,2)
Dt ∂t ∂xk ∂t
r
r ∂T ( x )
Note that T ( x ) is not a function of time, so =0:
∂t
r
& r ∂T ( x ) ∂xk ∂T ∂x1 ∂T ∂x2
⇒ T ( x, t ) = = +
∂x k ∂t ∂x1 { ∂t ∂x 2 { ∂t
V1 = X 1 V2 = X 2

r x x r 4x2 4x2
⇒ T& ( x , t ) = 4 x1 1 + 4 x2 2 ⇒ T& ( x , t ) = 1 + 2
1+ t 1+ t 1+ t 1+ t
The particle that at reference configuration was at position (1,1) , at time t = 1s will be at
position xi = (1 + t ) X i = 2 X i , i.e. ( x1 = 2; x 2 = 2 ):
4( 2) 2 4( 2) 2 K
T& ( x1 = 2; x2 = 2; t = 1) = + = 16
1+1 1+1 s

Problem 2.24
Consider the equations of motion:
 x1 = X 1 exp t + X 3 (exp t − 1)
 t −t
 x 2 = X 2 + X 3 (exp − exp )
x = X
 3 3

Obtain the velocity and acceleration components in Lagrangian and Eulerian descriptions.

University of Castilla-La Mancha Draft By: Eduardo W. V. Chaves (2016)


Ciudad Real - Spain
2 CONTINUUM KINEMATICS 201

Solution:
First we obtain the inverse of the equations of motion:
 x1 = X 1exp t + X 3 (exp t − 1)  x1 − X 1exp t = x3 (exp t − 1)
 t −t
 t −t
 x2 = X 2 + X 3 (exp − exp ) ⇒  x2 − X 2 = x3 (exp − exp )
x = X ⇒ X = x x = X ⇒ X = x
 3 3 3 3  3 3 3 3

thus:
 X 1 = x1 exp − t − exp − t (exp t − 1)
 2t −t
 X 2 = x 2 − x 3 (exp − 1)exp (2.81)
X = x
 3 3

or
 x1  exp   X1   X 1  exp − exp −t (exp t − 1)   x1 
t −t
0 (exp t − 1) 0
         
 x2  =  0 1 (exp t − exp −t )   X 2  inverse
→  X 2  =  0 1 − (exp 2t − 1)exp −t   x2 
x   0 0 1  X  X   0 0 1 x 
 3   3   3   3 
a) The velocity components in the material description are given by:
V1 = X 1exp t + X 3 exp t
D r 
Vi = → V 2 = X 3 exp t + X 3 exp −t = X 3 (exp t + exp −t )
x j ( X , t)  (2.82)
Dt V = 0
 3

b) The acceleration components in the material description are given by:


r  A1 = X 1exp t + X 3 exp t
r DV i ( X , t ) 
Ai ( X , t ) = →  A2 = X 3 (exp t − exp −t )
 (2.83)
Dt A = 0
 3
To obtain the velocity and acceleration in the spatial description it is sufficient to replace
the values of X 1 , X 2 , X 3 , given by the equation (2.81), into the equations (2.82) and (2.83),
thus, we can obtain:
v1 = x1 + x 3 a1 = x1 + x 3
 t −t  t −t
v 2 = x 3 (exp + exp ) ; a 2 = x 3 (exp − exp )
v = 0 a = 0
 3  3
Velocity in the Accelerati on in the
spatial descriptio n spatial descriptio n

Problem 2.25
r r r
The motion of the continuum, x = x ( X , t ) , is given by the following equations:
 x1 = 12 ( X 1 + X 2 )exp t + 12 ( X 1 − X 2 )exp − t
 t −t
 x 2 = 2 ( X 1 + X 2 )exp − 2 ( X 1 − X 2 )exp
1 1

x = X
 3 3

0 ≤ t ≤ constant
Express the velocity components in the material and spatial descriptions.

University of Castilla-La Mancha Draft By: Eduardo W. V. Chaves (2016)


Ciudad Real - Spain
202 SOLVING PROBLEMS BY MEANS OF CONTINUUM MECHANICS

Solution:
The velocity components using material description are:
r
 Dx1 ( X , t ) 1 1
V1 = = ( X 1 + X 2 )exp t − ( X 1 − X 2 )exp −t
 Dt 2 2
r
 Dx 2 ( X , t ) 1 1
V 2 = = ( X 1 + X 2 )exp t + ( X 1 − X 2 )exp −t (2.84)
 Dt 2 2
V3 = 0


To express the velocity components in the spatial description we will need the inverse of
r r r
the equations of motion, i.e. we will need to find X = X ( x , t ) :

 x1  (exp t + exp − t ) (exp t − exp − t ) 0   X 1 


  1 t −t t −t  
 x2  = (exp − exp ) (exp + exp ) 0   X 2 
x  2  0 0 2   X 3 
 3 
 X1   (exp 2t + 1)exp − t − (exp 2t − 1)exp − t 0   x1 
  1  
→  X 2  =  − (exp 2 t − 1)exp − t (exp 2t + 1)exp − t
inverse 0   x2 
X  2  0 0 2  x3 
 3 
Then, to obtain the Eulerian velocity we substitute the above equations into the Lagrangian
velocity (2.84), with which we can obtain:
v1 = x 2 ; v 2 = x1 ; v3 = 0

Problem 2.26
Given the motion:
xi = ( X 1 + ktX 2 )δ i1 + X 2δ i 2 + X 3δ i 3 (i = 1,2,3)
r
and the temperature field T ( x ) = x1 + x2 .
Obtain the rate of change of T of a particle that in the current configuration is located at
the point (1,1,1) .
Solution:
Considering the equations of motion:
x1 = X 1 + ktX 2 ; x2 = X 2 ; x3 = X 3
r
and by substituting the values of xi into the temperature field T ( x , t ) , we can obtain the
r
temperature field in the material description T ( X , t ) :
r r
T ( x ) = x1 + x2 ⇒ T ( X , t ) = X 1 + ktX 2 + X 2
The material time derivative is given by:
r
r DT ( X , t ) D ( X 1 + ktX 2 + X 2 )
&
T ( X ,t) = = = kX 2 = k x2 ( → T& = k
1,1,1)
Dt Dt
Alternative solution:
The material time derivative for a property expressed in the spatial description is given by:

University of Castilla-La Mancha Draft By: Eduardo W. V. Chaves (2016)


Ciudad Real - Spain
2 CONTINUUM KINEMATICS 203

r r r r
& DT ( x , t ) ∂T ( x , t ) ∂T ( x , t ) ∂xk ( X , t )
T ( x1 , x2 , x3 , t ) = = +
Dt ∂t ∂x k ∂t
r
Considering T ( x ) = x1 + x2 , we can obtain:
∂T  ∂T ∂x1 ∂T ∂x 2 ∂T ∂x 3 
T& ( x1 , x 2 , x 3 , t ) = + + + 
∂t  ∂x1 ∂t
{ ∂x 2 {
∂t ∂x 3 {
∂t 
=0
{ =0 =0
=0

⇒ T& ( x1 , x 2 , x 3 , t ) = kX 2
we obtain the inverse equations of motion:
 x1 = X 1 + ktX 2  X 1 = x1 − ktx 2
 inverse 
x2 = X 2   →  X 2 = x 2
x = X X = x
 3 3  3 3
r
With that the equation T& ( X ) = kX 2 can be expressed as follows:
⇒ T& ( x1 , x 2 , x 3 , t ) = kX 2 = kx 2
For the particle in the current configuration at the position (1,1,1) we have:
T& ( x1 = 1, x 2 = 1, x 3 = 1, t ) = k

Problem 2.27
Given a steady velocity field: it asks readers to give their opinion on whether particle
velocities are constant or not. If not, in which situation is met. Justify the answer.
Solution:
r
A field φ ( x , t ) is said to be steady if the local rate of change does not vary over time, so:
r
∂φ ( x , t ) r
=0 ⇒ φ = φ( x ) Steady state (stationary) field (2.85)
∂t
For example, let us consider a stationary (steady state) velocity field as shown in Figure 2.6.
Then, as we can verify, the field representation for any time, e.g. t1 and t 2 , does not
change. However, that does not mean that the velocities of the particles do not change
over time. In light of Figure 2.6, we can now focus our attention on the fixed spatial point
r r r
x * . At time t1 the particle Q is passing through point x * with velocity v * . Let us also
consider another particle P , which is passing through another point with velocity
r r r
v P (t1 ) ≠ v * . At time t 2 the particle P is now passing through the point x * . It follows that
r
if we are dealing with a steady state velocity field, then the velocity of particle P at x *
r r r
must be v * , i.e. v P (t 2 ) = v * . We can easily contrast this with the material time derivative of
velocity, which is always associated with the same particle, i.e.:
r r r r
Dv ( x , t ) r r ∂v ( x , t ) r r r r r r r r
≡ a ( x, t ) = + (∇ xr v ) ⋅ v ( x ) = (∇ xr v ) ⋅ v ( x ) = a ( x )
Dt 1
4 ∂t4
2 3 (2.86)
r
= 0(Stationay )

The rate of change of velocity (acceleration) will be zero if the velocity field is stationary
r r
 ∂v ( x , t ) r  r
 = 0  and homogeneous ( ∇ xr v = 0 ).
 ∂t 

University of Castilla-La Mancha Draft By: Eduardo W. V. Chaves (2016)


Ciudad Real - Spain
204 SOLVING PROBLEMS BY MEANS OF CONTINUUM MECHANICS

We can also verify that, although spatial velocity is independent of time, that does not
mean material velocity is also, since:
r r r r r r r
v ( x ) = v ( x ( X , t )) = v ( X , t ) (2.87)

t1 r r
v ( x)

r r r r
v ( x * , t1 ) = v * = v Q

Particle- P Particle - Q
r r
v P ≠ v*
r
x*

t2 r r
v ( x)

r r r r
v ( x * , t2 ) = v * = v P

Particle - P

r
x*

Figure 2.6: Steady velocity field.

2.2 Deformation Gradient, Deformation/strain Tensors,


Homogeneous Deformation

Problem 2.28
A rod, which can be considered as a one-dimensional solid, undergoes a uniform stretching
which is given by λ = exp at where a = constant .
r r r
a) Obtain the equations of motion x = x ( X , t ) ;
b) Obtain the rate-of-deformation tensor components, i.e. D -components.

University of Castilla-La Mancha Draft By: Eduardo W. V. Chaves (2016)


Ciudad Real - Spain
2 CONTINUUM KINEMATICS 205

x1

λ = exp at
Figure 2.7
Solution:
Using the 1D approaching we have:
ds dx
λ= = = exp at ⇒ dx = exp at dX (2.88)
dS dX
∫ ∫
Integrating
dx = exp at dX  → x1 = exp at X 1 + C (2.89)
at t = 0 ⇒ x = X , thus
x = exp 0 X 1 + C ⇒ X = X + C ⇒ C = 0 (2.90)
with that we can obtain the equations of motion:
x1 = exp at X 1 ; x2 = X 2 ; x3 = X 3 (2.91)
The velocity field components become:
dx1
v1 = = a X 1 exp at = a x1 ; v2 = 0 ; v3 = 0 (2.92)
dt
And the rate-of-deformation tensor components can be obtained as follows:
r r a 0 0
1  ∂vi ( x , t ) ∂v j ( x , t ) 
Dij = + ⇒ Dij =  0 0 0 (2.93)
2  ∂x j ∂xi 
 0 0 0

Problem 2.29
Consider the equations of motion:
x1 = X 1 + 2 X 3 ; x2 = X 2 − 2 X 3 ; x3 = X 3 − 2 X 1 + 2 X 2
Obtain the Green-Lagrange strain tensor components, i.e. E -components.
Solution 1:
The displacement field components are given by
u1 = x1 − X 1 = 2 X 3

u 2 = x 2 − X 2 = −2 X 3
u = x − X = −2 X + 2 X
 3 3 3 1 2

The Green-Lagrange strain tensor can be expressed in function of Lagrangian displacement


as follows:

University of Castilla-La Mancha Draft By: Eduardo W. V. Chaves (2016)


Ciudad Real - Spain
206 SOLVING PROBLEMS BY MEANS OF CONTINUUM MECHANICS

1  ∂u ∂u j ∂u k ∂uk 
Eij =  i + + 

2  ∂X j ∂X i ∂X i ∂X j 

sym (2.94)
1  ∂u ∂u j  1  ∂uk ∂u k   ∂ui  1  ∂u ∂uk 
=  i + +  =  +  k 

2  ∂X j ∂X i  2  ∂X i ∂X j   ∂X j  2  ∂X i ∂X j 
     
where the material (Lagrangian) displacement gradient is given by:
 ∂u1 ∂u1 ∂u1 
 
 ∂X 1 ∂X 2 ∂X 3   0 0 2 
∂u i  ∂u 2 ∂u 2 ∂u 2  
= = 0 0 − 2
∂X j  ∂X 1 ∂X 2 ∂X 3  
   
 ∂u 3 ∂u 3 ∂u 3  − 2 2 0 
 ∂X 1 ∂X 2 ∂X 3 
Note that, for this case, the displacement gradient is an antisymmetric tensor. That is, the
symmetric part is the null tensor. Then, the equation in (2.94) becomes:
  0 0 2  T  0 0 2    2 − 2 0
1  ∂u ∂u k  1  
E ij =  k  =  0 0 − 2   0 0 − 2  =  − 2 2 0 
2  ∂X i ∂X j  2      
   − 2 2 0  − 2 2 0    0 0 4
 
Solution 2:
1
We can directly apply the definition E = ( F T ⋅ F − 1) , Eij =
2
1
2
( ) 1
( )
Fki Fkj − δ ij = Cij − δ ij ,
2
where:
 ∂x1 ∂x1 ∂x1 
 
 ∂X 1 ∂X 2 ∂X 3   1 0 2 
∂xi  ∂x2 ∂x2 ∂x2  
Fij = = = 0 1 − 2
∂X j  ∂X 1 ∂X 2 ∂X 3  
 ∂x  
 3 ∂x3 ∂x3   − 2 2 1 
 ∂X 1 ∂X 2 ∂X 3 
Thus
  1 0 2  T  1 0 2  1 0 0   2 − 2 0
1 
E ij =   0 1 − 2  0 1 − 2 − 0 1 0  = − 2 2 0
2 
  − 2 2 1  − 2 2 1  0 0 1   0 0 4
 

Problem 2.30
Consider a homogeneous transformation defined by the following equations:
x1 = X 1 + 2 X 2 + X 3 ; x2 = 2 X 2 ; x3 = X 1 + 2 X 3 (2.95)
Show that, for a homogeneous transformation, vectors whose are parallel in the reference
configuration remain parallel after deformation.
For the demonstration consider two vectors defined by the vector position of two particles
A and B in the reference configuration:

University of Castilla-La Mancha Draft By: Eduardo W. V. Chaves (2016)


Ciudad Real - Spain
2 CONTINUUM KINEMATICS 207

r r
X A = eˆ 1 + eˆ 2 ; X B = 2eˆ 1 + 2eˆ 2 + eˆ 3 (2.96)
Solution:
The vector connecting the two particles in the reference configuration is given by:
r r r
V = B − A = eˆ 1 + eˆ 2 + eˆ 3 (2.97)
and the deformation gradient is:
 ∂x1 ∂x1 ∂x1 
 
 ∂X 1 ∂X 2 ∂X 3  1 2 1 
∂xi  ∂x2 ∂x2 ∂x2  
Fij = = = 0 2 0  (2.98)
∂X j  ∂X 1 ∂X 2 ∂X 3  
 ∂x  
 3 ∂x3 ∂x3  1 0 2
 ∂X 1 ∂X 2 ∂X 3 
We can obtain the vector position of the particle in the current configuration by means of:
r r r r
dx = F ⋅ dX ⇒ Homogeneous transformation ⇒ x=F⋅X (2.99)
thus,
1 2 1  1 3 1 2 1   2 7 
xiA = 0 2 0 1 = 2 ; xiB = 0 2 0  2 =  4 (2.100)
1 0 2 0 1  1 0 2 1   4

and the vector that connect these two points is:


r r r
v = x B − x A = 4eˆ 1 + 2eˆ 2 + 3eˆ 3 (2.101)
r
then any vector parallel to V , for example the vector 2eˆ 1 + 2eˆ 2 + 2eˆ 3 , after transformation
r
becomes: 8eˆ 1 + 4eˆ 2 + 6eˆ 3 , which is parallel to v . Note that, since we are dealing with
r r
homogeneous deformation the equation v = F ⋅ V is valid, i.e.:
1 2 1  1 4
v i = Fij Vi ⇒ v i = 0 2 0  1 = 2
1 0 2 1 3

Problem 2.31
Consider a pure shear deformation represented by homogenous deformation:
r r
x = X + k t X 2ê1 (2.102)
where ê i is the Cartesian basis, and the components of the above equation are:
x1 = X 1 + k t X 2 ; x2 = X 2 ; x3 = X 3 (2.103)
Obtain the new geometry (deformed configuration) for the body (rectangle) described in
Figure 2.8.

University of Castilla-La Mancha Draft By: Eduardo W. V. Chaves (2016)


Ciudad Real - Spain
208 SOLVING PROBLEMS BY MEANS OF CONTINUUM MECHANICS

X2
Reference configuration
B C t = 0s

O A X1

Figure 2.8

Solution:
The deformation gradient components are:
 ∂x1 ∂x1 ∂x1 
 
 ∂X 1 ∂X 2 ∂X 3  1 k t 0
∂x ∂x ∂x2 ∂x2  
Fij = i =  2 = 0 1 0 (2.104)
∂X j  ∂X 1 ∂X 2 ∂X 3  
 ∂x  
 3 ∂x3 ∂x3  0 0 1 
 ∂X 1 ∂X 2 ∂X 3 
r r r r r
Note that this is a case of homogenous deformation, i.e. x = F ⋅ X + c with c = 0 .
The Jacobian determinant:
J = F =1 (2.105)
Since J = 1 there is no dilatancy (variation of volume).
The particles lying on the BC -line, coordinates ( X 1 , X 2 ,0) , in the current configuration
will become:
x1( BC ) = X 1 + k t X 2 ; x2( BC ) = X 2 ; x3( BC ) = 0 (2.106)
The particles lying on the OA -line, coordinates ( X 1 ,0,0) , in the current configuration
assume the position:
x1(OA) = X 1 ; x2(OA) = 0 ; x3(OA) = 0 (2.107)
then, the OA -line does not change its position during motion, (see Figure 2.9).
x2
Deformed configuration

B B′ C C′

O A x1

Figure 2.9

University of Castilla-La Mancha Draft By: Eduardo W. V. Chaves (2016)


Ciudad Real - Spain
2 CONTINUUM KINEMATICS 209

Problem 2.32
Consider the equations of motion:
2 2
x1 = X 1 + X2 ; x2 = X1 + X 2 ; x3 = X 3 (2.108)
2 2
a) Show that this deformation is characterized by a homogeneous transformation;
b) Obtain the displacement field components in material and spatial descriptions;
c) Consider the particles located according to the equation:
X 12 + X 22 = 2 ; X3 = 0
Obtain the new configuration of these particles in the current configuration;
d) Obtain the right Cauchy-Green deformation tensor components ( C ) and the Green-
Lagrange strain tensor ( E ).
e) Obtain the principal values of C and E .
Solution:
a) The equation of a homogeneous deformation is described by xi = Fij X j , where

 ∂x1∂x1 ∂x1   2 
   1 0
 ∂X 1
∂X 2 ∂X 3   2 
∂xi ∂x∂x2 ∂x2   2
Fij = = 2 = 1 0 (2.109)
∂X j  ∂X 2 ∂X 3   2
∂X
 1   
 ∂x3∂x3 ∂x3   0 0 1
∂X 2 ∂X 3  
 ∂X 1 
r
Note that F is independent of x , so, F is a homogeneous transformation, and the
equation xi = Fij X j is in accordance with (2.108):

 2 
 1 0
 x1   2   X1 
x  =  2
 2  1 0  X 2  ⇔ xi = Fij X j (2.110)
2
 x3     X 3 
 0 0 1
 

And the inverse of (2.110) is represented by:

 X1   2 − 2 0  x1   X 1 = 2 x1 − 2 x 2
 X  = − 2  
 2  2 0  x 2  ⇒  X 2 = − 2 x1 + 2 x 2 (2.111)
 X 3   0 0 1  x3  X = x
   3 3

b) The Lagrangian displacement field is given by:

University of Castilla-La Mancha Draft By: Eduardo W. V. Chaves (2016)


Ciudad Real - Spain
210 SOLVING PROBLEMS BY MEANS OF CONTINUUM MECHANICS

 r r 2 2
u1 ( X , t ) = x1 ( X , t ) − X 1 = X 1 + X 2 − X1 = X2
 2 2
r r r r r  r r 2 2
u( X , t ) = x ( X , t ) − X ⇒ u 2 ( X , t ) = x2 ( X , t ) − X 2 = X1 + X 2 − X 2 = X1 (2.112)
 2 2
r r
u3 ( X , t ) = x3 ( X , t ) − X 3 = 0



which, in spatial coordinates, becomes:


r r
u1 ( x , t ) = x1 − X 1 ( x , t ) = x1 − (2 x1 − 2 x2 ) = − x1 + 2 x2
r r (2.113)
u 2 ( x , t ) = x2 − X 2 ( x , t ) = x2 − (− 2 x1 + 2 x2 ) = 2 x1 − x2
r r
u3 ( x , t ) = x3 − X 3 ( x , t ) = x3 − x3 = 0

c) The particles describing the circle X 12 + X 22 = 2 in the reference configuration, in the


current configuration becomes:

(2 x1 − 2 x2 ) + (−
2
2 x1 + 2 x 2 )
2
=2 (2.114)
which is the same as:

3x12 + 3 x 22 − 4 2 x1 x 2 = 1 (an ellipse equation) (2.115)

The curve made up by the same particle during motion is called material curve. The material
curve for this example is described in Figure 2.10.

Current configuration

Reference configuration

Figure 2.10: Material curves.

d) The right Cauchy-Green deformation tensor and the Green-Lagrange strain tensor are
given, respectively, by:
1
C = FT ⋅F ; E= (C − 1) (2.116)
2
Then, the C -components are:

University of Castilla-La Mancha Draft By: Eduardo W. V. Chaves (2016)


Ciudad Real - Spain
2 CONTINUUM KINEMATICS 211

T
 2   2 
 1 0  1 0  3
2

0
2 2  2
     
2 2 3
Cij =  1 0 
 1 0 =  2 0 (2.117)
2 2  2 
     
 0 0 1  0 0 1  0 0 1
     

And the eigenvalues of (2.117) can be evaluated by means of C − λ1 = 0 , in which the


result is:
3 3
C1 = + 2 ≈ 2.914 ; C 2 = − 2 ≈ 0.086 ; C 3 = 1 (2.118)
2 2
The E -components are:
 3  
 2 0 
 1 0
 
 2 1 0 0  2 2
1   
1
2
(
Eij = Cij − δ ij ) =
2 
 2
3
2 
1
0 − 0 1 0  = 2 2
4
1 0 (2.119)

 0 0 1 0 0 1  0 0 0
   
  
The eigenvalues of E can be obtained by means of E − λ1 = 0 . Since E33 = E3 = 0 is
already an eigenvalue, then in order to obtain the remaining eigenvalues we just need to
solve:

1 2  1+ 2 2
−λ λ 1 =
4 2 =0 λ 7  4
⇒ λ2 − − =0⇒ (2.120)
2 1 2 16  1− 2 2
−λ
2 4 λ 2 = 4
Then, the three eigenvalues of E are:
1+ 2 2 1− 2 2
E1 = ≈ 0.957 ; E 2 = ≈ −0.457 ; E 3 = 0 (2.121)
4 4

Problem 2.33
Let us consider the following equations of motion:
1 1
x1 = X 1 + X2 ; x2 = X1 + X 2 ; x3 = X 3 (2.122)
2 2
r
a) Obtain the displacement field ( u ) in the Lagrangian and Eulerian descriptions;
b) Determine the material curve in the current configuration for a material circle defined in
the reference configuration as:
X 12 + X 22 = 2 ; X3 = 0
c) Obtain the components of the right Cauchy-Green deformation tensor and the Green-
Lagrange strain tensor;
d) Obtain the principal stretches.
Solution:

University of Castilla-La Mancha Draft By: Eduardo W. V. Chaves (2016)


Ciudad Real - Spain
212 SOLVING PROBLEMS BY MEANS OF CONTINUUM MECHANICS

The deformation gradient is given by:


 ∂x1
∂x1 ∂x1 
 
 ∂X 1
∂X 2 ∂X 3  2 1 0
∂xi  ∂x2
∂x2 ∂x2  1 
Fij = = = 1 2 0 ; J = F = 0.75
∂X j  ∂X 1
∂X 2 ∂X 3  2 
 ∂x 0 0 2
∂x3
 3 ∂x3 
 ∂X 1
∂X 2 ∂X 3 
r
Note that F is independent of x , so, F is a homogeneous transformation. And by
comparing this with the equations of motion in (2.122) we have:
 x1  2 1 0  X 1 
 x  = 1 1 2 0   X  ⇔ xi = Fij X j
 2 2   2 
 x3  0 0 2  X 3 

So, we can verify that the proposed example is a case of homogeneous deformation in
r r
which c = 0 . The inverse form of the above equation is given by:
 4 2
 X 1 = 3 x1 − 3 x 2
 X1   4 − 2 0  x1  
 X  = 1  − 2 4 0  x   2 4
 2 3  2  ⇒  X 2 = − x1 + x 2 (2.123)
 3 3
 X 3   0 0 3  x3 
 X 3 = x3


r r r
The displacement field is defined by u = x − X , after which the components of the
Lagrangian displacement become:
 r r 1 1
u1 ( X , t ) = x1 ( X , t ) − X 1 = X 1 + 2 X 2 − X 1 = 2 X 2

r r  r r 1 1
ui ( X , t ) = xi ( X , t ) − X i ⇒ u2 ( X , t ) = x2 ( X , t ) − X 2 = X 1 + X 2 − X 2 = X 1 (2.124)
 2 2
r r
u3 ( X , t ) = x3 ( X , t ) − X 3 = 0


The components of the Eulerian displacement can be obtained by substituting the Eulerian
description of motion (2.123) into (2.124), the result of which is:
 r r 1 r 1 2 4  r
u1 ( X ( x, t ), t ) = 2 X 2 ( x , t ) =  − x1 + x 2  = u1 ( x , t )
2 3 3 

 r r 1 r 1 2 4  r
u 2 ( X ( x , t ), t ) = X 1 ( x , t ) =  − x1 + x 2  = u 2 ( x , t ) (2.125)
 2 2 3 3 
r
u ( X ( xr , t ), t ) = u ( xr , t ) = 0
 3 3


The particles belonging to the circle X 12 + X 22 = 2 in the reference configuration will form
a new curve in the current configuration which is defined by:
2 2
4 2   2 4 
X 12 + X 22 = 2 ⇒  x1 − x 2  +  − x1 + x 2  = 2 ⇒ 20 x12 − 32 x1 x 2 + 20 x 22 = 18
3 3   3 3 

University of Castilla-La Mancha Draft By: Eduardo W. V. Chaves (2016)


Ciudad Real - Spain
2 CONTINUUM KINEMATICS 213

which is an ellipse equation (Figure 2.10 shows the material curve in different
configurations).

2.0
material curve

1.5
Reference Conf.
Current Conf.
1.0

0.5

0.0
x2

-2 -1 0 1 2
-0.5

-1.0

-1.5

-2.0
x1

Figure 2.11: Material curve.

The components of C and E can be obtained by using the definitions C = F T ⋅ F and


1
E= (C − 1) :
2
 2 1 0  2 1 0 1.25 1 0
1
C ij = Fki Fkj ⇒ C ij = 1 2 0  1 2 0 =  1 1.25 0
4
0 0 2 0 0 2  0 0 1

 1.25 1 0 1 0 0  0.125 0.5 0


1  
1
(
E ij = C ij − δ ij
2
) ⇒ E ij =   1 1.25 0 − 0 1 0  =  0.5 0.125 0
2
  0 0 1  0 0 1    0 0 0

In the principal space of C its components are given by:


λ21 0 0 λ 1 0 0
   
C ij′ =  0 λ22 0 ⇒ C ij′ =  0 λ2 0
0 0 λ23  0 0 λ 3 
 
where λ i show the principal stretches. Therefore, to calculate these we need to obtain the
C eigenvalues:
1.25 − C 1 C1 = 2.25
= 0 ⇒ C 2 − 2.5C + 0.5625 = 0 ⇒ 
1 1.25 − C C 2 = 0.25

University of Castilla-La Mancha Draft By: Eduardo W. V. Chaves (2016)


Ciudad Real - Spain
214 SOLVING PROBLEMS BY MEANS OF CONTINUUM MECHANICS

λ21 0 0 2.25 0 0 λ 1 0 0  1.5 0 0


     
C ij′ =  0 λ22 0= 0 0.25 0 ⇒ 0 λ2 0  =  0 0.5 0

0 0 λ23   0 0 1 0 0 λ 3   0 0 1 
 

Problem 2.34
Show that
r
∇ Xr ⋅ [(detF ) F −T ] = 0 (2.126)
r r r
Hint: The Nanson’s formula da = J F −T ⋅ dA , or da = da nˆ = J F −T ⋅ N
ˆ dA .

Solution:
Considering the Nanson’s formula in indicial notation da nˆ i = J Fki−1Nˆ k dA , with J = det (F )
we can apply the surface integral in order to obtain:
−1 ˆ
∫ nˆ da = ∫ J F
S
i
S0
ki N k dA (2.127)

Note that, if we consider the function f , the following is true:


∂f
∫ nˆ
S
i f da = ∫
V
f ,i dV = ∫
V
∂xi
dV

and denoting by f = 1 , we can obtain:

∫ nˆ
S
i da = 0 i

Returning to equation (2.127), and applying the divergence theorem to the integral on the
right of the equation we obtain:
∂ ( J Fki−1 )

S
nˆ i da = 0i = ∫
S0
J Fki−1Nˆ k dA = ∫
V0
( J Fki−1 ) ,k dV0 = ∫
V0
∂X k
dV0 = 0i

r (2.128)
∫∇ r
X
⋅ [(detF ) F −T
] dV0 = 0
V0

Then, if the above volume integral is valid for the whole volume we can guarantee that is
also valid locally, i.e.:
r
∇ Xr ⋅ [(detF ) F −T ] = 0 (2.129)

Problem 2.35
r r r r
Show that E& = [ F −T ⋅ ∇ Xr u& ( X , t)]sym and b) D = [∇ xr u& ( x , t)]sym , where E is the Green-
Lagrange strain tensor and D is the rate-of-deformation tensor.
Solution:

E& ≡
D
Dt
E=
D 1 T

Dt  2
(  1
) 1
[
F ⋅ F − 1  = ( F& T ⋅ F + F T ⋅ F& ) = ( F T ⋅ F& )T + ( F T ⋅ F& ) = [ F T ⋅ F& ]sym ]
 2 2
Note that:

University of Castilla-La Mancha Draft By: Eduardo W. V. Chaves (2016)


Ciudad Real - Spain
2 CONTINUUM KINEMATICS 215

F&ij =
D  ∂xi ( X , t ) 
Dt  ∂X j  ∂X j
=
∂ Dxi ( X , t )
Dt
=
∂ &
∂X j
[ ∂u& ( X , t )
u i ( X , t )] = i
∂X j
( r r
)
= ∇ Xr u& ( X , t ) ij

with that we demonstrate that:


D r r
E& ≡ E = [ F T ⋅ F& ]sym = [ F T ⋅ ∇ Xr u& ( X , t )]sym
Dt
b)
1 1 r r r r r r
D=l = [ l + l T ] = [∇ xr v + (∇ xr v )T ] = [∇ xr v ( x , t )]sym = [∇ xr u& ( x , t )]sym
sym
2 2
r r r r
where we have considered v ( x , t ) = u& ( x , t ) .

Problem 2.36
Obtain the relationship E& = F T ⋅ D ⋅ F starting from the definition
r r
(ds ) 2 − (dS ) 2 = dX ⋅ 2 E ⋅ dX . Get also the relationship between
D
Dt
(ds ) 2 and D . [ ]
If we are dealing with a rigid body motion, find the condition in order to guarantee the
rigid body motion.
Solution:
r r
Taking the material time derivative of (ds ) 2 − (dS ) 2 = dX ⋅ 2 E ⋅ dX we obtain:
D D D r r
[(ds) 2 − ( dS ) 2 ] = [(ds ) 2 ] = [ dX ⋅ 2 E ⋅ dX ]
Dt Dt Dt
D r r r& r r r r r&
= [dx ⋅ dx ] = 2d{X ⋅ E ⋅ dX + 2dX ⋅ E& ⋅ dX + 2dX ⋅ E ⋅ d{X
Dt =0 =0
r D r r r
= 2dx ⋅ [dx ] = 2dX ⋅ E& ⋅ dX
Dt
D r
The term [ dx ] can be expressed as follows:
Dt
D D  ∂xk 
D r D r  [dxk ] =  dX i 
 Dt [ d x ] = [ F ⋅ d X ]  Dt Dt  ∂X i 
Dt r   ∂  D  ∂xk 
  D x
 = F& ⋅ dX Indicial
 
→  =  k dX i =
   dX i
 r  Dt  ∂X i  DX i  ∂t 
= l ⋅ F ⋅ dX
  ∂v
  = k dX i
 ∂X i

with that we conclude that:


r r r D r
2dX ⋅ E& ⋅ dX = 2 dx ⋅ [ dx ]
r Dt r
= 2dx ⋅ l ⋅ F ⋅ dX
r r
= 2( F ⋅ dX ) ⋅ l ⋅ F ⋅ dX
r r
= 2 dX ⋅ F T ⋅ l ⋅ F ⋅ dX

We can apply the additive decomposition of the spatial velocity gradient ( l ) into a
symmetric ( D ) and an antisymmetric ( W ) part:

University of Castilla-La Mancha Draft By: Eduardo W. V. Chaves (2016)


Ciudad Real - Spain
216 SOLVING PROBLEMS BY MEANS OF CONTINUUM MECHANICS

r r r r
2dX ⋅ E& ⋅ dX
= 2 dX ⋅ F T ⋅ l ⋅ F ⋅ dX
r r
= 2dX ⋅ F T ⋅ (D + W ) ⋅ F ⋅ dX
r r r r
= 2 dX ⋅ F T ⋅ D ⋅ F ⋅ dX + 2 dX ⋅ F T ⋅ W ⋅ F ⋅ dX
r r
= 2 dX ⋅ F T ⋅ D ⋅ F ⋅ dX
r r r r r r
Note that dX ⋅ F T ⋅ W ⋅ F ⋅ dX = dx ⋅ W ⋅ dx = W : (dx ⊗ dx ) = 0 , since W is an
r r
antisymmetric tensor and (dx ⊗ dx ) is a symmetric tensor. Then, we conclude that:
E& = F T ⋅ D ⋅ F
D
With that it is possible to relate [(ds )2 ] and D as follows:
Dt
D r r r r
[(ds )2 ] = 2dX ⋅ F T ⋅ D ⋅ F ⋅ dX = 2dx ⋅ D ⋅ dx
Dt
During the rigid body motion the distances between particles do not change during
D
motion, i.e. [(ds )2 ] = 0 , and according to the above equation we can conclude that the
Dt
r r
rigid body motion is guaranteed by D = 0 ,since dx ≠ 0 .

Problem 2.37
Consider the velocity field:
v1 = −5 x2 + 2 x3 ; v2 = 5 x1 − 3 x3 ; v3 = −2 x1 + 3 x2
Show that this motion corresponds to a rigid body motion.
Solution:
At first we obtain the spatial velocity gradient ( l ) , whose components are given by:
 ∂v1 ∂v1 ∂v1 
 
r  ∂x1 ∂x2 ∂x3   0 − 5 2 
∂vi ( x , t )  ∂v2 ∂v2 ∂v2  
l ij = = = 5 0 − 3 (2.130)
∂x j ∂x ∂x2 ∂x3  
 1   0 

 v3 ∂v3 ∂v3  − 2 3
 ∂x1 ∂x2 ∂x3 

Remember that ( l ) can be decomposed into a symmetric ( D ) and antisymmetric ( W )


part, i.e. l = D + W = W . Since D = 0 , there is no strain during motion, i.e. a rigid body
motion.

Problem 2.38
Let us consider the following velocity field:
v1 = −3 x2 + 1x3 ; v2 = 3x1 − 5 x3 ; v3 = −1x1 + 5 x2
Show that this motion corresponds to rigid body motion.
Solution: First we obtain the components of the spatial velocity gradient ( l ) :

University of Castilla-La Mancha Draft By: Eduardo W. V. Chaves (2016)


Ciudad Real - Spain
2 CONTINUUM KINEMATICS 217

 ∂v1 ∂v1 ∂v1 


 
r  ∂x1 ∂x 2 ∂x3   0 − 3 1 
∂vi ( x , t )  ∂v 2 ∂v 2 ∂v 2  
l ij = =  = 3 0 − 5 = l ijskew
∂x j ∂x ∂x 2 ∂x3
 1   0 

 v3 ∂v 3 ∂v3  − 1 5
 ∂x1 ∂x 2 ∂x3 

Taking into account that l can be decomposed into a symmetric ( l sym ≡ D ) and an
antisymmetric ( l skew ≡ W ) part, i.e. l = D + W , we can thus conclude that D = 0 , which is
a characteristic of rigid body motion.

Problem 2.39
The displacement field components are given by:
u 1 = 3 X 12 + X 2 ; u 2 = 2 X 22 + X 3 ; u 3 = 4 X 32 + X 1
r
Obtain the vector dx (current configuration) correspondent to the vector in the reference
r
configuration represented by dX at the point P(1,1,1) , (see Figure 2.12).

X 3 , x3
 dX 1 
dX k =  dX 2 
Q  dX 3 
r
dX
P
X 2 , x2

X 1 , x1

Figure 2.12
Solution:
r
To determine the vector dx we need to obtain the deformation gradient F , which can be
obtained by using the relationship:
1 + 6 X 1 1 0 
∂u i 
Fij = δ ij + ⇒ Fij =  0 1+ 4X2 1 
∂X j
 1 0 1 + 8 X 3 
And the deformation gradient components evaluated at the point P(1,1,1) are:
7 1 0 
Fij = 0 5 1 
P
1 0 9 

Then, the vector components dx i are given by:


dx i = Fij dX j

University of Castilla-La Mancha Draft By: Eduardo W. V. Chaves (2016)


Ciudad Real - Spain
218 SOLVING PROBLEMS BY MEANS OF CONTINUUM MECHANICS

 dx1  7 1 0   dX 1   7 dX 1 + dX 2 
 dx  = 0 5 1   dX  = 5dX + dX 
 2   2   2 3
 dx 3  1 0 9   dX 3   dX 1 + 9 dX 3 

Problem 2.40
Consider a continuum in which the displacement field is described by the following
equations:
u1 = 2 X 12 + X 1 X 2 ; u 2 = X 22 ; u3 = 0

By definition, a material curve is always formed by the same particles. Let OP and OT be
material lines in the reference configuration, where O( X 1 = 0, X 2 = 0, X 3 = 0) ,
P ( X 1 = 1, X 2 = 1, X 3 = 0) and T ( X 1 = 1, X 2 = 0, X 3 = 0) . Find the material curves in the
current configuration. Also find the deformation gradient.
Solution:
a) The equations of motion can be obtained by means of the displacement field, i.e.:
 x1 = u1 + X 1  x1 = X 1 + 2 X 12 + X 1 X 2
 
u i = xi − X i ⇒  x2 = u 2 + X 2  substituti ng
 →  x2 = X 2 + X 22
the values of u 1 ,u 2 ,u 3
x = u + X x = X
 3 3 3  3 3

Then, to obtain the material curve, one need only substitute the material coordinates with
the particles belonging to the line OP in the equations of motion, (see Figure 2.13). Notice
that the material curve OP in the current configuration is no longer a straight line, but the
line OT is still a straight line in the current configuration, (see Figure 2.14).

2.5

2
P

1.5 material curve


x2

1 P
Current Conf.
Q Q
0.5 Reference Conf.

0
0
O 0.5 1 1.5 2 2.5 3 3.5 4 4.5
x1

Figure 2.13: Deformation of the material curve OP .

University of Castilla-La Mancha Draft By: Eduardo W. V. Chaves (2016)


Ciudad Real - Spain
2 CONTINUUM KINEMATICS 219

Reference Conf.

0.1
0.08
0.06 Reference Conf.
x2

0.04
0.02
0
T
O0 0.5 1 1.5 2 2.5 3 3.5

Current Conf.

0.1
0.08
0.06 Current Conf.
x2

0.04
0.02
T
0
O0 0.5 1 1.5 2 2.5 3 3.5
x1

Figure 2.14: Deformation of the material curve OT .

The components of the deformation gradient can be obtained as follows:


 ∂x1 ∂x1 ∂x1 
 
 ∂X 1 ∂X 2 ∂X 3  (1 + 4 X + X )
1 2 X1 0
 ∂x ∂x 2 ∂x 2  
F jk = 2 = 0 1 + 2X 2 0 
∂X ∂X 2 ∂X 3  
 1   0 0 1 
 ∂x 3 ∂x 3 ∂x 3 
 ∂X 1 ∂X 2 ∂X 3 
r
Note that we are not dealing with homogeneous deformation, since F depends on X .
Problem 2.41
D DF
Starting from the definition [det ( F )] = ij cof ( Fij ) , show that the equation
Dt Dt
r
J& = J ∇ xr ⋅ v , is valid.
∂x i
Solution: Considering that Fij = , the material time derivative of F ≡ det (F ) is given
∂X j
by:
 
D
[det ( F )] = D  ∂xi ( X , t ) cof ( Fij ) = D  ∂xi ( X , t ) cof ( Fij ) = D (vi )cof ( Fij )
Dt Dt  ∂X j  ∂X j  Dt  ∂X j
r
and considering that v i ( x ( X , t ), t ) , we can state that:

University of Castilla-La Mancha Draft By: Eduardo W. V. Chaves (2016)


Ciudad Real - Spain
220 SOLVING PROBLEMS BY MEANS OF CONTINUUM MECHANICS

D
[det ( F )] = ∂vi ∂xk cof ( Fij )
Dt ∂xk ∂X j

By referring to the definition of the cofactor: [cof ( Fij )]T = ( Fij ) −1 det ( F ) , we can also state
the following is valid:
D
[det (F )] = ∂vi ∂xk ( Fij ) −T det ( F ) = ∂vi Fkj ( Fij ) −1 det ( F ) = ∂vi δ ki det ( F ) = ∂vi det ( F )
Dt ∂xk ∂X j ∂ xk ∂ xk ∂xi
= Jvi ,i

An alternative solution is presented in Problem 1.118 in Chapter 1.


Problem 2.42
r
Let dx be a differential line element in the current configuration. Find the material time
r
derivative of dx .
Solution:
D r D r D r D r r r r r
dx = ( F ⋅ dX ) = ( F ) ⋅ dX + F ⋅ ( dX ) = l ⋅ 1⋅ d3
F2 X = l ⋅ dx ≡ (∇ xr v ) ⋅ dx
Dt Dt Dt Dt
1
424 3 r
dx
r
0

And, whose components are represented by:


r
 D r ∂v i ( x , t )
 dx  = v i , k dx k = dx k
 Dt  i ∂x k

Problem 2.43
Let us consider the equations of motion:
x1 = X 1 + 4 X 1 X 2 ; x2 = X 2 + X 22 ; x3 = X 3 + X 32
Find the Green-Lagrange strain tensor ( E ).
Solution: Referring to the E equation:
1 1
E= ( F T ⋅ F − 1) ; E ij = ( Fki Fkj − δ ij ) (2.131)
2 2
where the components of F are derived as:
 ∂x1 ∂x1 ∂x1 
 
 ∂X 1 ∂X 2 ∂X 3  (1 + 4 X )
2 4X1 0 
∂x k  ∂x 2 ∂x 2 ∂x 2  
Fkj = = = 0 1 + 2X 2 0 
∂X j  ∂X 1 ∂X 2 ∂X 3  
   0 0 1 + 2 X 3 
 ∂x 3 ∂x 3 ∂x 3 
 ∂X 1 ∂X 2 ∂X 3 
And,
(1 + 4 X 2 ) 0 0  (1 + 4 X 2 ) 4X1 0 
Fki Fkj 
=  4X1 1 + 2X 2 0    0 1 + 2X 2 0 
 0 0 1 + 2 X 3   0 0 1 + 2 X 3 
 (1 + 4 X 2 ) 2
(1 + 4 X 2 ) 4 X 1 0 
 2 2 
= (1 + 4 X 2 ) 4 X 1 ( 4 X 1 ) + (1 + 2 X 2 ) 0 
 0 0 (1 + 2 X ) 2
 3 

University of Castilla-La Mancha Draft By: Eduardo W. V. Chaves (2016)


Ciudad Real - Spain
2 CONTINUUM KINEMATICS 221

Then, by substituting the above into the equation in (2.131) we can obtain:
(1 + 4 X 2 ) 2 − 1 (1 + 4 X 2 ) 4 X 1 0 
1 2 2 
E ij =  (1 + 4 X 2 ) 4 X 1 ( 4 X 1 ) + (1 + 2 X 2 ) − 1 0 
2 2 
 0 0 (1 + 2 X 3 ) − 1
Problem 2.44
Obtain the principal invariants of E in terms of the principal invariants of C and b .
Solution:
The principal invariants of E are given by:

I E = Tr ( E ) ; II E =
1 2
2
[
I E − Tr ( E 2 ) ] ; III E = det ( E )

1
Considering E = (C − 1) , the principal invariants can also be expressed as follows:
2
The First Invariant:
1  1 1 1
I E = Tr ( E ) = Tr  (C − 1)  = Tr (C − 1) = [Tr (C ) − Tr (1) ] = (I C − 3)
2  2 2 2
The Second Invariant:

II E =
1 2
2
[
I E − Tr ( E 2 ) ]
where
2
I E2
1  1
(
=  (I C − 3) = I C2 − 6 I C + 9 )
2  4
2
1  1 1
[ 1
] (
Tr ( E ) = Tr  (C − 1)  = Tr (C − 1) 2 = Tr C 2 − 2C + 1 = Tr (C 2 ) − 2 Tr (C ) + Tr (1)
2
) [ ]
2  4 4 4
1
[
= Tr (C 2 ) − 2 I C + 3
4
]
The term Tr (C 2 ) can be obtained as follows:
C12 0 0
 
C ⋅C = C 2
⇒ Cij′ 2 =  0 C 22 0  ⇒ Tr (C 2 ) = C12 + C 22 + C32
0 0 C 32 

It is also true that:
2
1444424444
(
I C2 = (C1 + C 2 + C 3 ) = C12 + C 22 + C 32 + 2 C1 C 2 + C1 C 3 + C 2 C 3
3
)
II C

⇒ C12 + C 22 + C 32 = I C2 − 2 II C
Therefore we have:

Tr ( E 2 ) =
4
(
1 2
I C − 2 II C − 2 I C + 3 )
Whereupon, the second invariant can also be expressed as:

University of Castilla-La Mancha Draft By: Eduardo W. V. Chaves (2016)


Ciudad Real - Spain
222 SOLVING PROBLEMS BY MEANS OF CONTINUUM MECHANICS

II E = 
2 4
(
1 1 2 1
) (  1
)
I C − 6 I C + 9 − I C2 − 2 II C − 2 I C + 3  = (− 2 I C + II C + 3)
4  4
The Third Invariant:
3
1  1
III E = det ( E ) = det  (C − 1) =   det [(C − 1)]
2  2
The term det[(C − 1)] can also be expressed as:
C1 − 1 0 0
det (C − 1) = 0 C2 − 1 0 = (C1 − 1)(C 2 − 1)(C 3 − 1)
0 0 C3 − 1
= C1C 2 C 3 − C1C 2 − C1C 3 − C 2 C 3 + C1 + C 2 + C 3 − 1 = III C − II C + I C − 1
Then:
1
III E = ( III C − II C + I C − 1)
8
In short we have:
1 I C = 2I E + 3
IE = (I C − 3 )
2
1
II E = (− 2 I C + II C + 3) ; II C = 4 II E + 4 I E + 3
4
1
III E = ( III C − II C + I C − 1) III C = 8 III E + 4 II E + 2 I E + 1
8
Problem 2.45
Let Ψ = Ψ (I C , II C , III C ) be a scalar-valued tensor function, where I C , II C , III C are the
principal invariants of the right Cauchy-Green deformation tensor C . Obtain the
derivative of Ψ with respect to C and with respect to b . Check whether the following
equation is valid F ⋅ Ψ ,C ⋅ F T = Ψ ,b ⋅ b or not.
Solution:
Using the chain rule of derivative we can obtain:
∂Ψ (I C , II C , III C ) ∂Ψ ∂I C ∂Ψ ∂ II C ∂Ψ ∂ III C
Ψ ,C = = + + (2.132)
∂C ∂I C ∂C ∂ II C ∂C ∂ III C ∂C
Considering the partial derivative of the invariants:
∂I C ∂ II C ∂ III C
=1 , = IC 1 − C T = IC 1 − C , = III C C −T = III C C −1 , we can obtain:
∂C ∂C ∂C
∂Ψ ∂Ψ
Ψ ,C = 1+ (I C 1 − C ) + ∂Ψ III C C −1
∂I C ∂ II C ∂ III C
(2.133)
 ∂Ψ ∂Ψ  ∂Ψ ∂Ψ
Ψ ,C =  + I C 1 − C+ III C C −1

 C I ∂ II C  ∂ II C ∂ III C

It is also true that:

University of Castilla-La Mancha Draft By: Eduardo W. V. Chaves (2016)


Ciudad Real - Spain
2 CONTINUUM KINEMATICS 223

 ∂Ψ ∂Ψ  ∂Ψ ∂Ψ
Ψ ,b =  + I b 1 − b+ III b b −1 (2.134)
 ∂I b ∂ II b  ∂ II b ∂ III b

We apply the dot product of the above equation with F on the left and with F T on the
right, i.e.:
 ∂Ψ ∂Ψ  ∂Ψ ∂Ψ
F ⋅ Ψ ,C ⋅ F T =  + I C  F ⋅ 1 ⋅ F T − F ⋅C ⋅ F T + III C F ⋅ C −1 ⋅ F T

 CI ∂ II C  ∂ II C ∂ III C

(2.135)
And by considering the following relationships:
⇒ F ⋅1 ⋅ F T = F ⋅ F T = b
C = F T ⋅ F ⇒ F ⋅C ⋅ F T = F ⋅ F T ⋅ F ⋅ F T = b ⋅ b = b2
And considering the relationship C −1 = F −1 ⋅ b −1 ⋅ F we conclude that:
C −1 = F −1 ⋅ b −1 ⋅ F ⇒ F ⋅ C −1 ⋅ F T = F ⋅ F −1 ⋅ b −1 ⋅ F ⋅ F T = b −1 ⋅ b
Then, the equation in (2.135) can be rewritten as follows:
 ∂Ψ ∂Ψ  ∂Ψ 2 ∂Ψ
F ⋅ Ψ ,C ⋅ F T =  + I C b − b + III C b −1 ⋅ b
 ∂I C ∂ II C  ∂ II C ∂ III C

 ∂Ψ ∂Ψ  ∂Ψ ∂Ψ 
F ⋅ Ψ ,C ⋅ F T =  + I C 1 − b+ III C b −1  ⋅ b
 ∂I C ∂ II C  ∂ II C ∂ III C 
It is also valid that:
 ∂Ψ ∂Ψ  ∂Ψ ∂Ψ 
F ⋅ Ψ ,C ⋅ F T =  + I b 1 − B+ III b b −1  ⋅ b
 ∂I b ∂ II b  ∂ II b ∂ III b 

F ⋅ Ψ ,C ⋅ F T = Ψ , b ⋅ b

Taking into account the equation (2.134) we can conclude that the equation Ψ ,b ⋅ b = b ⋅ Ψ ,b
is valid, indicating that the tensors Ψ ,b and b are coaxial.

Problem 2.46
Show that the Green-Lagrange strain tensor ( E ) and the right Cauchy-Green deformation
tensor ( C ) are coaxial tensors.
Solution:
Two tensors are coaxial if they have the same principal directions. Coaxiality can also be
demonstrated if the relation C ⋅ E = E ⋅ C holds.
Starting with the definition C = 1 + 2 E , we can conclude that:
C ⋅ E = (1 + 2 E ) ⋅ E = 1 ⋅ E + 2 E ⋅ E = E ⋅ (1 + 2 E ) = E ⋅ C
Thus, we can prove that E and C are coaxial tensors.

University of Castilla-La Mancha Draft By: Eduardo W. V. Chaves (2016)


Ciudad Real - Spain
224 SOLVING PROBLEMS BY MEANS OF CONTINUUM MECHANICS

Problem 2.47
Obtain the material time derivative of the Jacobian determinant ( J& ) in terms of ( E& ), ( C& ),
( F& ).
Solution:
We starting from the relationship J& = J Tr (D ) , where D is the rate-of-deformation tensor
which is related to E& by means of the relationship D = F −T ⋅ E& ⋅ F −1 , then:
( ) (
J& = J Tr (D) = J Tr F −T ⋅ E& ⋅ F −1 = J F −T ⋅ E& ⋅ F −1 : 1 )
In indicial notation we have:
J
J& = J Fki−1 E& kp F pj−1δ ij = J Fki−1 F pi−1 E& kp = J ( F −1 ⋅ F −T ) : E& = J C −1 : E& = C −1 : C&
2
The J& can still be expressed in terms of F& . To this end let us consider the following
equation E& kp = (F&sk Fsp + Fsk F&sp ) . Then, J& can also be expressed by:
1
2
1
( J
) (
J& = J Fki−1 F pi−1 E& kp = J Fki−1 F pi−1 F&sk Fsp + Fsk F&sp = Fki−1 F pi−1 F&sk Fsp + Fki−1 F pi−1 Fsk F&sp
2 2
)
J
( ) J −1 &
( )
= δ si Fki Fsk + δ si F pi Fsp = Fks Fsk + F ps Fsp = JFts−1 F&st = JF&st Fts−1
2
−1 & −1 &
2
−1 &

−T & & −T
= JF : F = JF : F
In short, there are various different ways to express the material time derivative of the
Jacobian determinant:
J −1 &
J& = J Tr (D) = J C −1 : E& C :C = = JF& : F −T
2
J
= J Tr (C −1 ⋅ E& ) = Tr (C −1 ⋅ C& ) = J Tr ( F& ⋅ F −1 )
2
where we have used the trace property: A : B = Tr ( A ⋅ B T ) = Tr ( A T ⋅ B ) in which A and B
are arbitrary second-order tensors.
Problem 2.48
The displacement field components are given by:
u1 = 0.1 X 22 ; u2 = 0 ; u3 = 0
a) Is this motion possible? Justify;
b) Obtain the right Cauchy-Green deformation tensor;
c) Consider two vectors at the point P (1,1,0) in the reference configuration,
r r
namely: b = 0.01eˆ 1 and c = 0.015 eˆ 2 . Find the correspondent vectors in current
configuration;
r r
d) Obtain the stretches of the vectors b and c , at the point P (1,1,0) ;
r r
e) Find the angle variation defined by the two vectors b and c .
Solution:
a) A motion is possible if the Jacobian determinant is positive. The deformation gradient
components can be obtained as follows:

University of Castilla-La Mancha Draft By: Eduardo W. V. Chaves (2016)


Ciudad Real - Spain
2 CONTINUUM KINEMATICS 225

 1 0 0   0 0 .2 X 2 0  1 0 .2 X 2 0
∂u i
Fij = δ ij + = 0 1 0  + 0 0 0  = 0 1 0 
∂X j 
0 0 1  0 0 0  0 0 1 

The determinant is Fij = J = 1 > 0 . Then, the motion is possible.

b) The right Cauchy-Green deformation tensor is defined as C = F T ⋅ F , then the


components are given by:
 1 0 0 1 0.2 X 2 0  1 0 .2 X 2 0
C ij = 0.2 X 2 1 0 0
 1 0 = 0.2 X 2 0.2 X 2 + 1 0
2 2

 0 0 1 0 0 1  0 0 1


r
c) The vector b = 0.01eˆ 1 at the point P(1,1,0) deforms according to:

r r  b 1′  1 0.2 × 1 0 0.01 0.01


b′ = F ⋅b ⇒ b ′  = 0 1 0  0  =  0 
P  2 
b ′3  0 0 1   0   0 
r
and the vector c = 0.015eˆ 2 in the current configuration becomes:
 c 1′  1 0.2 × 1 0   0  0.003
c ′  = 0 1 0  0.015 = 0.015
 2 
 c ′3  0 0 1   0   0 

d) The stretch can be obtained as follows:


r
b′ 0.012
λ br = r = =1
b 0.01
r
and the stretch of c is given as:
r
c′ 0.003 2 + 0.015 2
λ cr = r = = 1.0198 ≈ 1.02
c 0.015

Alternative solution: Taking into account that λ Mˆ = Mˆ ⋅ C ⋅ Mˆ and by evaluating C at the


point P we obtain:

 1 0 .2 X 2 0 1 0 .2 0 
C ij ( X 1 = 1, X 2 = 1, X 3 = 0) = 0.2 X 2 2
0 .2 + 1 0
X 22 
= 0.2 1.04 0
 0 0 1  0 0 1
P

Then, by applying λ bˆ = bˆ ⋅ C ⋅ bˆ and λ cˆ = cˆ ⋅ C ⋅ cˆ we can obtain:

1 0 .2 0  1 
λ2bˆ = [1 0 0] 0.2 1.04 0 0  = 1
 ⇒ λ bˆ = 1
 0 0 1  0 

1 0 .2 0   0 
λ2cˆ = [0 1 0] 0.2 1.04 0 1  = 1.04
 ⇒ λ cˆ = 1.0198
 0 0 1  0 

University of Castilla-La Mancha Draft By: Eduardo W. V. Chaves (2016)


Ciudad Real - Spain
226 SOLVING PROBLEMS BY MEANS OF CONTINUUM MECHANICS

r r
e) In the current configuration the angle between the vectors b ′ and c ′ can be obtained
according to the relationship:
r r
b′ ⋅ c ′
cos θ = r r
b′ c ′

(0.01eˆ 1 + 0eˆ 2 + 0eˆ 3 ) ⋅ (0.003eˆ 1 + 0.015 eˆ 2 + 0eˆ 3 ) 0.00003


cos θ = = = 0.196116135
0.012 0.003 2 + 0.015 2 0.01 0.000234

θ = arccos(0.196116135) ≈ 78.69º
In the reference configuration the angle between these two vectors is 90 º , then the angle
variation is:
∆θ = 90 º −78 .69 º = 11 .3º
Alternative solution: Given two directions in the reference configuration represented by their
unit vectors M̂ and N̂ , the angle formed by these unit vectors in the current configuration
(after motion) is given by:
Mˆ ⋅ C ⋅ Nˆ Mˆ ⋅ C ⋅ Nˆ
cos θ = =
Mˆ ⋅ C ⋅ Mˆ Nˆ ⋅ C ⋅ Nˆ λ Mˆ λ Nˆ

Denoting by Mˆ = b̂ and Nˆ = ĉ it fulfills that:


1 0 .2 0   0 
b ⋅ C ⋅ cˆ = [1 0 0]  0.2 1.04 0  1  = 0.2
ˆ 
 0 0 1  0 

Then,
bˆ ⋅ C ⋅ cˆ bˆ ⋅ C ⋅ cˆ 0 .2
cos θ = = = = 0.196116135
bˆ ⋅ C ⋅ bˆ cˆ ⋅ C ⋅ cˆ λ bˆ λ cˆ 1 1.04

Problem 2.49
r
Let φ ( X , t ) be a scalar field in Lagrangian (material) description. Find the relationship
r r
between the material gradient of φ ( X , t ) , i.e. ∇ Xr φ( X , t ) , and the spatial gradient of
r r
φ ( x , t ) , i.e. ∇ xr φ( x , t ) .
Solution:
r
Remember that a Lagrangian variable φ ( X , t ) can be expressed in the Eulerian (current)
configuration by means of the equations of motion, i.e.:
r r r r
φ ( X , t ) = φ( X ( x , t ), t ) = φ( x , t ) .
Then, from the scalar gradient definition we obtain:
r r r r r
r ∂φ( X , t ) ∂φ ( X ( x , t ), t ) ∂x ∂φ ( x , t ) r
∇ Xr φ( X , t ) = r = r ⋅ r= r ⋅ F = ∇ xr φ ( x , t ) ⋅ F
∂X ∂x ∂X ∂x
In addition we have the inverse form:
r r r r r
r ∂φ( x , t ) ∂φ ( x ( X , t ), t ) ∂X ∂φ ( X , t ) r
∇ xr φ( x , t ) = r = r ⋅ r= r ⋅ F −1 = ∇ Xr φ( X , t ) ⋅ F −1
∂x ∂X ∂x ∂X

University of Castilla-La Mancha Draft By: Eduardo W. V. Chaves (2016)


Ciudad Real - Spain
2 CONTINUUM KINEMATICS 227

Problem 2.50
Given the following Eulerian velocity field components:
v1 = 0 ; v2 = 0 ; v3 = f ( x1 , x2 ) x3
a) Find the particle trajectory;
b) Obtain the mass density ( ρ ), knowing that at t = 0 we have ρ = f ( x1 , x 2 ) .
Solution:
dx1
= v1 = 0 ⇒ x1 (t ) = C1 at t = 0 ⇒ x1 = X 1 ⇒ x1 (t = 0) = C1 = X 1 ;
dt
dx 2
= v 2 = 0 ⇒ x 2 (t ) = C 2 at t = 0 ⇒ x 2 = X 2 ⇒ x 2 (t = 0) = C 2 = X 2
dt
dx3 dx3
dt
= v3 = f ( x1 , x 2 ) x3 = f (C1 , C 2 ) x3 ⇒
x3 ∫
= ∫ f (C , C1 2 ) dt ⇒ Ln( x 3 ) = f (C1 , C 2 )t + k

By denoting by k = Ln(C 3 ) the above equation becomes:


Ln( x3 ) − Ln(C 3) = f ( X 1 , X 2 )t
x  x
⇒ Ln 3  = f ( X 1 , X 2 )t ⇒ 3 = exp f ( X 1 , X 2 )t ⇒ x3 = C3exp f ( X 1 , X 2 )t
 C3  C3
at t = 0 ⇒ x3 = X 3 ⇒ x3 (t = 0) = C 3 = X 3
Summarizing:
x1 = X 1 ; x2 = X 2 ; x3 = X 3exp f ( X 1 , X 2 )t (2.136)
The mass density:
ρ0 ∂xi
ρ= with Fij =
F ∂X j

1 0 0
J= F =0 1 0 = exp f ( X 1 , X 2 )t
? ? exp f ( X 1 , X 2 )t
As we can see, the values marked by ( ? ) are not necessary in order to obtain the above
determinant, then:
ρ0 f (X1, X 2 )
ρ= =
F exp f ( X 1 , X 2 )t
Note that according to the problem statement, t = 0 , ρ = f ( x1 , x 2 ) , and according to the
equations in (2.136) we can conclude that ρ 0 = f ( X 1 , X 2 ) .

Problem 2.51
Obtain the equation for mass density in terms of the third invariant of the right Cauchy-
Green deformation tensor, i.e. ρ 0 = ρ 0 ( III C ) .
Solution:
Starting by the definition:

University of Castilla-La Mancha Draft By: Eduardo W. V. Chaves (2016)


Ciudad Real - Spain
228 SOLVING PROBLEMS BY MEANS OF CONTINUUM MECHANICS

r r
ρ 0 ( X ) = ρ ( x, t) J
and considering that the third invariant III C = det (C ) = det ( F T ⋅ F ) = J 2 , we obtain
J = III C , then:

ρ0 =ρ III C (2.137)

Problem 2.52
Consider the displacement field of a continuous medium by:
u1 = (a1 − 1) X 1 ; u 2 = (a 2 − 1) X 2 + a1αX 1 ; u 3 = (a 3 − 1) X 3
where α is a constant. Determine a1 , a 2 and a 3 knowing that the solid is incompressible,
that a segment parallel to the X 3 -axis does not stretch and that any element area defined in
the plane X 1 − X 3 remains unchanged.
Solution:
r r r
Based on the definition of the displacement field, i.e. u = x − X , we can obtain:
u1 = x1 − X 1 = (a1 − 1) X 1 ⇒ x1 = a1 X 1
u 2 = x 2 − X 2 = (a 2 − 1) X 2 + a1αX 1 ⇒ x 2 = a 2 X 2 + a1αX 1
u 3 = x 3 − X 3 = (a 3 − 1) X 3 ⇒ x3 = a3 X 3
Then, the equations of motion are:
 x1 = a1 X 1  x1   a1 0 0  X 1 
     
 x 2 = a 2 X 2 + a1αX 1 ⇒  x 2  = a1α a 2 0   X 2  (homogeneous deformation)
x = a X x   0 0 a 3   X 3 
 3 3 3  3 
which is possible to establish that F = a1 a 2 a 3 > 0 .
By means of the incompressibility condition dV = F dV0 ⇒ F ≡ J = 1 , the following
relationship is true:
a1 a 2 a 3 = 1
ˆ = [0 0 1] , does not stretch that
By the fact that a segment parallel to the X 3 -axis, e.g. M i
implies that the stretching according to this direction is unitary, i.e. λ Mˆ = 1 , thus

ˆ ⋅ E ⋅M
λ Mˆ = 1 + 2M ˆ = 1 + 2E = 1 ⇒ E 33 = 0
33

1
The components of the Green-Lagrange strain tensor, E = ( F T ⋅ F − 1) , can be obtained
2
as follows:
  a1 a1α 0   a1 0 0  1 0 0  a12 + a12α 2 − 1 a1 a 2α 0 
1   1 
E ij =  0 a2 0   a1α a2   
0  − 0 1 0   =  a1 a 2α 2
a2 −1 0 
2 2
 0 0 a 3   0 0 a 3  0 0 1   0 0 a 32 − 1

thus:
E 33 = a 32 − 1 = 0 ⇒ a 3 = ±1

University of Castilla-La Mancha Draft By: Eduardo W. V. Chaves (2016)


Ciudad Real - Spain
2 CONTINUUM KINEMATICS 229

Any element area on the plane X 1 − X 3 does not change


 x1   a1 0 0  X 1 
    
 x 2  =  a1α a 2 0   X 2 
x   0 0 a 3   X 3 
 3 

By defining two vectors on the plane X 1 − X 3 , i.e. Nˆ i(1) = [1 0 0] and Nˆ i(3) = [0 0 1] we


can obtain:
 a1 0 0  1  a1   a1 0 0  0  0 
        
n i(1) = a1α a 2 0  0 = a1α  ; n i(3) 
= a1α a 2 0  0 =  0 
 0 0 a 3  0  0   0 0 a 3  1 a 3 
Then, the area in the current configuration is obtained as follows:
eˆ 1 eˆ 2 eˆ 3
r (1) r (3)
n ∧ n = a1 a1α 0 = a1αeˆ 1 − a1 a 3 eˆ 2 + 0eˆ 3
0 0 a3
r r
and its module does not change N (1) ∧ N (3) = n (1) ∧ n (3) = 1 :
r r
n (1) ∧ n (3) = 1 = (a1α ) 2 + (−a1 a 3 ) 2 ⇒ a12 a 32 α 2 + a12 a 32 = 1

We have previously obtained that a 32 = 1 , with that we can obtain:


1 1
a12 a 32 α 2 + a12 a 32 = 1 ⇒ a12 α 2 + a12 = 1 ⇒ a12 = ⇒ a1 = ±
(1 + α )
2
(1 + α 2 )
with that we can conclude that:
1
a1 = ; a 2 = (1 + α 2 ) ; a3 = 1
(1 + α )2

Problem 2.53
Consider the solid shown in Figure 2.15 which is subjected to a homogenous deformation.
r r
a) Obtain the general expression for the material displacement field u( X , t ) in function of
the material displacement gradient tensor J .
r r
b) Obtain u( X , t ) knowing that also holds the following boundary conditions:
r r
u 2 ( X , t ) = u3 ( X , t ) = 0 ∀X 1 , X 2 , X 3
u1 ( X 1 = 0, X 2 , X 3 , t ) = 0
u1 ( X 1 = L, X 2 , X 3 , t ) = ∆ L
c) Justify the possible values (positive and negative) that can take ∆ L .
d) Calculate the material and spatial strain tensors and the infinitesimal strain tensor.

University of Castilla-La Mancha Draft By: Eduardo W. V. Chaves (2016)


Ciudad Real - Spain
230 SOLVING PROBLEMS BY MEANS OF CONTINUUM MECHANICS

x2

L ∆L

x1
x3

Figure 2.15
Solution:
r
A homogeneous deformation is characterized by F ( X , t ) = F (t ) . In addition, we know
that:
r r
F ( X , t ) = 1 + J ( X , t ) Homogeneou
    → F (t ) = 1 + J (t )
s deformation

where J is the material displacement gradient tensor. Note that the homogenous
deformation is independent of the vector position, with that we can obtain:
r r
∂u( X , t ) r r r r r r r
J (t ) = r ⇒ J (t ) ⋅ dX = du( X , t ) ⇒ u( X , t ) = J (t ) ⋅ X + c (t )
∫ ∫
∂X
r r r
where c(t ) is the constant of integration. And the components of u( X , t ) are:
 u1   J 11 X 1 + J 12 X 2 + J 13 X 3   c1 
     
u2  = J 21 X 1 + J 22 X 2 + J 23 X 3  + c 2 
u   J X + J X + J X  c 
 3   31 1 32 2 33 3   3 

b) From the conditions in paragraph b) we can conclude that:


r r
condition 1) u 2 ( X , t ) = u 3 ( X , t ) = 0 ∀X 1 , X 2 , X 3 :

 u1   J 11 X 1 + J 12 X 2 + J 13 X 3   c1 
      J 21 = 0; J 22 = 0; J 23 = 0, c 2 = 0
u 2 = 0 = J 21 X 1 + J 22 X 2 + J 23 X 3  + c 2  ⇒ 
u = 0   J X + J X + J X  c  J 31 = 0; J 32 = 0; J 33 = 0, c 3 = 0
 3   31 1 32 2 33 3   3 

condition 2) u1 ( X 1 = 0, X 2 , X 3 , t ) = 0 :
u1 = 0  J 11 X 1 + J 12 X 2 + J 13 X 3  c1 
     
 u2  =  0  +  0  ⇒ {J 12 = 0; J 13 = 0, c1 = 0
 u   0  0
 3     
condition 3) u1 ( X 1 = L, X 2 , X 3 , t ) = ∆ L
 u1  J 11 L  0
       ∆L
 u 2  =  0  + 0 ⇒  J 11 =
u = ∆   0  0  L
 3 L    

University of Castilla-La Mancha Draft By: Eduardo W. V. Chaves (2016)


Ciudad Real - Spain
2 CONTINUUM KINEMATICS 231

Hence, the components of the displacement gradient are:


 ∆L 
 L 0 0  c1  0
   
J ij =  0 0 0 and c 2  = 0
 
 0 0 0 c  0
 3  
 
with that the displacement field components are:
∆L 
 L X1 
r r r r r
u( X , t ) = J (t ) ⋅ X + c(t )   → ui ( X , t ) =  0 
components
 
 0 
 
c) The motion is possible and has physical meaning if F > 0 :

 ∆L 
1 + L 0 0

F (t ) = 1 + J (t )   → Fij =  0
components
1 0 ⇒ F = 1 + L > 0 ⇒ ∆ L > − L
  L
 0 0 1
 
The material strain tensor (the Green-Lagrange strain tensor):
∆ 1  ∆L 
2

 L
+   0 0
 L 2 L    m2 
E=
2
(
1 T
F ⋅ F −1 ) components
  → Eij =  0 0 0  2

0

0 0 m 

 
The spatial strain tensor (the Almansi strain tensor):
∆ 1  ∆L  
2

 L
+    1 0 0 
 L 2  L     m2 
e=
1
2
(
1 − F −T ⋅ F −1 ) components
  → eij = 2  0 0 0  2
 ∆L  0 0 0 m 
1 + 
 L 
r 1
The infinitesimal strain tensor is defined by ε = (∇ xr u) sym = ( J + J T ) , and its components
2
are:
 ∆L 
 L 0 0
m
ε ij =  0 0 0  m  (dimensionless)
   
 0 0 0
 
2
∆  ∆ 
Note that, when  L  << 1 is very small the term  L  ≈ 0 can be discarded, and in this
 L   L 
scenario we have E ≈ e ≈ ε , i.e. we are dealing with the small deformation regime.

University of Castilla-La Mancha Draft By: Eduardo W. V. Chaves (2016)


Ciudad Real - Spain
232 SOLVING PROBLEMS BY MEANS OF CONTINUUM MECHANICS

Problem 2.54
The tetrahedron shown in Figure 2.16 undergoes homogeneous deformation ( F = const. )
with the following consequences:
1. The points O , A and B do not move;
2. The solid volume becomes p times the initial volume;
p
3. The length of the segment AC becomes times the initial length;
2
4. The angle AOC becomes 45º .
a) Justify why we cannot use the infinitesimal deformation theory;
b) Obtain the deformation gradient, and the possible values for p and the displacement
field in material and spatial descriptions;
c) Draw the deformed solid.

x3

O a
B x2
a

A
x1

Figure 2.16
Solution:
a) The angle AOC = 90º becomes 45º , so we are not dealing with a small deformation,
since for the small deformation case the condition ∆φ << 1 must fulfill, and in this problem
π
we have ∆φ << ≈ 0.7854 ;
4
b) We have a case of homogeneous deformation. Then, the equations of motion are given
by:
 x1   F11 F12 F13   X 1   c1 
r r       
F23   X 2  + c 2 
r
x = F (t ) ⋅ X + c (t ) ⇒  x2  =  F21 F22
x  F F32 F33   X 3  c 3 
 3   31
The point O( X 1 = 0, X 2 = 0, X 3 = 0) does not move:
0  F11 F12 F13  0  c1   c1  0
          
0 =  F21 F22 F23  0 + c 2  ⇒ c 2  = 0
0  F F32 F33  0 c 3  c  0
   31  3  
The point A( X 1 = a, X 2 = 0, X 3 = 0) does not move:

University of Castilla-La Mancha Draft By: Eduardo W. V. Chaves (2016)


Ciudad Real - Spain
2 CONTINUUM KINEMATICS 233

a   F11 F12 F13  a  a   aF11   F11 = 1


         
0  =  F21 F22 F23  0  ⇒ 0  = aF21  ⇒  F21 = 0
0   F F32 F33  0  0  aF  F = 0
   31    31   31
The point B ( X 1 = 0, X 2 = a, X 3 = 0) does not move:
0  1 F12 F13  0  0   aF12   F12 = 0
         
a  = 0 F22 F23  a  ⇒ a  = aF22  ⇒  F22 = 1
 0  0 F F33  0  0  aF  F = 0
   32    32   32
Gathering the above information, we have:
1 0 F13 
Fij = 0 1 F23  ; F = F33 > 0
0 0 F33 

The volume of the solid becomes " p" times the initial volume. The relationship between the initial
(reference) volume and the current (final) volume is given by:
dV = F dV0 ⇒ ∫ dV = ∫ F dV 0 ⇒ V final = F Vinitial = F33Vinitial

where we have considered the homogeneous deformation case. With this, we can conclude
that F33 = p
p
(The length of segment AC becomes times the initial length). As we are dealing with a
2
homogeneous deformation, a line in the reference configuration will remain a line in the
current configuration.
The point C ( X 1 = 0, X 2 = 0, X 3 = a ) moves to:
 x1C  1 0 F13  0   x1C   aF13 
 C     C  
 x 2  = 0 1 F23  0  ⇒  x 2  = aF23 
 x C  0 0 p  a   x C   ap 
 3     3  

The length of segment AC in the reference configuration is L AC = a 2 . The vector that


connect the points A′ ≡ A( x1 = a, x 2 = 0, x3 = 0) and C ′( x1 = aF13 , x 2 = aF23 , x3 = ap) in the
current configuration is given by:
AC = (aF13 − a )eˆ 1 + (aF23 )eˆ 2 + (ap)eˆ 3
and its magnitude is:
AC = l AC = (a ( F13 − 1)) 2 + (aF23 ) 2 + (ap) 2 = a ( F13 − 1) 2 + ( F23 ) 2 + ( p) 2

p
Using the information provided by the problem l AC = L AC , we can obtain:
2
p
l AC = L AC
2
p
a ( F13 − 1) 2 + ( F23 ) 2 + ( p ) 2 = a 2
2
( F13 − 1) 2 + ( F23 ) 2 + ( p ) 2 = p

University of Castilla-La Mancha Draft By: Eduardo W. V. Chaves (2016)


Ciudad Real - Spain
234 SOLVING PROBLEMS BY MEANS OF CONTINUUM MECHANICS

thus
 F13 = 1
( F13 − 1) 2 + ( F23 ) 2 + p 2 = p 2 ⇒ ( F13 − 1) 2 + ( F23 ) 2 = 0 ⇒ 
 F23 = 0
Then, the deformation gradient components are:
1 0 1
Fij = 0 1 0 
0 0 p 
The angle AOC changes to 45º .
dx1(1)  1 0 1  1 1
 (1)      
dX i(1) = [1 0 0] ⇒ dxi(1) = Fij dX (j1) ⇒ dx 2  = 0 1 0  0 = 0
dx (1)  0 0 p  0 0
 3      

dx1( 2)  1 0 1  0  1 
 ( 2)      
dX i( 2 ) = [0 0 1] ⇒ dxi( 2 ) = Fij dX (j 2 ) ⇒ dx 2  = 0 1 0  0 =  0 
dx ( 2)  0 0 p  1   p 
 3      
r r
dx (1) ⋅ dx ( 2 ) 2
cos( AOC ′) = cos(45º ) = r (1) r ( 2 ) =
dx dx 2

r r r r
where dx (1) = 1 , dx ( 2) = 1 + p 2 , dx (1) ⋅ dx ( 2) = 1 . Then:

1 2
= ⇒ p = ±1
1+ p2 2

As the Jacobian determinant must be greater than zero F = p > 0 , this implies that p = 1 :
1 0 1
Fij = 0 1 0
0 0 1

The equations of motion become:


 x1  1 0 1  X 1   X 1 + X 3 
      
 x 2  = 0 1 0  X 2  =  X 2 
 x  0 0 1   X   X 
 3   3   3 
The material displacement field becomes:
u1   X 1 + X 3   X 1   X 3 
r r r r r components        
u( X , t ) = x ( X , t ) − X   
→ u 2  =  X 2  −  X 2  =  0 
u   X  X   0 
 3  3   3  
The spatial displacement field becomes:
u1   x3 
   
u 2  =  0 
u   0 
 3  

University of Castilla-La Mancha Draft By: Eduardo W. V. Chaves (2016)


Ciudad Real - Spain
2 CONTINUUM KINEMATICS 235

c) The deformed body is described in Figure 2.17.

x3

C′
 x1C   aF13  a 
 C     
 x2  = aF23  = 0 
xC   ap  a  a a
 3      O
a B = B′ x2

x1 A = A′

Figure 2.17

Problem 2.55
A rigid body motion is characterized by the following equation:
r r r
x = c(t ) + Q(t ) ⋅ X (2.138)
r r
Find the velocity and the acceleration fields as a function of ω , where ω is the axial vector
associated with the antisymmetric tensor ( Ω = Q & ⋅ Q T ).

Solution:
r r r
The material time derivative of x = c(t ) + Q(t ) ⋅ X is given by
r D r r& r& & r
v= x ≡ x =c + Q⋅ X
Dt
& ⋅ QT ⇒ Q
Let us consider that Ω = Q & = Ω ⋅ Q . The above equation can also be expressed as:
r r r
r r r r
v = c& + Ω ⋅ Q ⋅ X
v = c& + Ω ⋅ ( x − c )

r r r r
If Ω is an antisymmetric tensor, it holds that Ω ⋅ a = ω ∧ a , where ω (angular velocity vector)
is the axial vector associated with the antisymmetric tensor Ω . Then, the associated
velocity can be expressed as:
r r r r r r r r
v = c& + Ω ⋅ ( x − c ) = c& + ω ∧ ( x − c ) (2.139)

Note that Q(t ) is only dependent on time, hence the axial vector (angular velocity)
r r
associated with Ω is also time-dependent, i.e. ω = ω (t ) .
Then, its acceleration is given by:
r r &r& &r& && r
a = v& = x =c + Q⋅ X
&& = Ω& ⋅ Q + Ω ⋅ Q
By referring to Q & , the above equation can also be expressed as:

University of Castilla-La Mancha Draft By: Eduardo W. V. Chaves (2016)


Ciudad Real - Spain
236 SOLVING PROBLEMS BY MEANS OF CONTINUUM MECHANICS

r &r& r &r& r r
a=c + (Ω& ⋅ Q + Ω ⋅ Q
&)⋅ X =c + Ω& ⋅ Q ⋅ X + Ω ⋅ Q & ⋅X
r r r r r r r r
= &c& + Ω& ⋅ Q ⋅ X + Ω ⋅ Ω ⋅ Q ⋅ X = &c& + Ω& ⋅ ( x − c ) + Ω ⋅ Ω ⋅ ( x − c )
we can state that:
r &r& r& r r r r r r
a =c + ω ∧ ( x − c ) + ω ∧ [ω ∧ ( x − c )] (2.140)
r r
where α ≡ ω& shows the angular acceleration.
r r r r r
For a rigid body motion when c = 0 , the velocity becomes v = ω ∧ x whose components
are vi =  ipq ω p x q , and the rate-of-deformation tensor D becomes:

1  ∂vi ∂v j  1  ∂( ipq ω p x q ) ∂( jpq ω p x q )  1 


= 
∂x ∂x 
 =   ipq ω p q +  jpq ω p q 
D ij = + +
2  ∂x j ∂xi  2
  ∂x j ∂xi  2
  ∂x j ∂xi 

=
1
2
(
 ipq ω p δ qj ) 1
2
( 1
) (
+  jpq ω p δ qi =  ipj ω p +  jpi ω p =  ipj ω p −  ipj ω p = 0 ij
2
)
So, once again we have proved that D = 0 for a rigid body motion, (see Problem 2.36).

Problem 2.56
r r
Given a coordinate system x which is fixed in space, and the mobile system x *
characterized only by rotation, (see Figure 2.18). Show that the rate of change of a vector
r
b can be represented by:
r r r
 Db   Db  r r  Db  r
 
 Dt  =  
 + ϕ ∧ b =  
 +Ω T ⋅b (2.141)
  fixed  Dt  mobile  Dt  mobile
r
 Db  r r
where  
 represents the rate of change of b with respect to the fixed system x ,
 Dt  fijo
r
 Db  r
  represents the rate of change of b with respect to the mobile system which its
 Dt 
  móvil
r
angular velocity is ϕ .

r
b r
ϕ
x3
x2*
x3*
x1*

x2

x1

Figure 2.18

University of Castilla-La Mancha Draft By: Eduardo W. V. Chaves (2016)


Ciudad Real - Spain
2 CONTINUUM KINEMATICS 237

Solution:
By means of component transformation law the following relationships are true:
r r r r
b* = A ⋅b ⇔ b = A T ⋅ b * (components)
r r
where A is the matrix transformation from the system x to the system x * .
r r
The rate of change of the vector b = A T ⋅ b * can be evaluated as follows:
D r r& D
Dt
b ≡b =
Dt
[
r
] r r&
A T ⋅ b * = A& T ⋅ b * + A T ⋅ b * (2.142)

Making an analogy with rate of change of the orthogonal tensor, (see Chaves(2013) –
Chapter 2), we can state that Ω = A& ⋅ A T ⇒ A& T = A T ⋅ Ω T , where Ω T is an
r
antisymmetric tensor and represents the rate of change of rotate of the system x * with
r
respect to the fixed system x . Then, the equation in (2.142) can be rewritten as follows:
D r r& r r& r r& r r&
b ≡ b = A& T ⋅ b * + A T ⋅ b = A T ⋅ Ω T ⋅ b * + A T ⋅ b * = A T ⋅ Ω T ⋅ b * + b *  (2.143)
Dt  
r r r r
Recall the antisymmetric tensor property Ω T ⋅ b * = ϕ ∧ b * , where ϕ is the axial vector
r r
associated with the antisymmetric tensor Ω T , i.e. ϕ = ϕ (t ) is the angular velocity of the
r
mobile system x * . Then, the equation in (2.143) can also be rewritten as:
r& r r& r r r&
b = A T ⋅ Ω T ⋅ b * + b *  = A T ⋅ ϕ * ∧ b * + b *  (components) (2.144)
       
r& r& r
Note that the term A ⋅ b represents the components of b in the system x * , and also note
r& r&
that A ⋅ b ≠ b * , thus:
r * r r
A ⋅ b&  = b& * + ϕr * ∧ b * (components) (2.145)
 
which in tensorial notation becomes:
r r
 Db   Db  r r
  =  + ϕ ∧ b (tensorial notation) (2.146)
 Dt   
  fijo  Dt  móvil

Problem 2.57
r
a) A continuum is rotating as a rigid body with a constant angular velocity ω = ω 3 ê 3 , (see
Figure 2.19):
a.1) Obtain the velocity components in the spatial and material descriptions;
a.2) Obtain the acceleration in the spatial (Eulerian) description;
a.3) When ω 3 = 3rad / s , obtain the vector position, velocity and acceleration at time
t = 2.5s of the particle that in the reference configuration was at (1,1,0) .
b) Taking into account Problem 1.129 where we have obtained the body force vector (per
r GM r
unit mass) b = − r xˆ where g = b is the acceleration of gravity caused by gravitational
x
field. Now, if we consider the Earth as a sphere that rotates around its axis with angular

University of Castilla-La Mancha Draft By: Eduardo W. V. Chaves (2016)


Ciudad Real - Spain
238 SOLVING PROBLEMS BY MEANS OF CONTINUUM MECHANICS

r
velocity ω = ω 3 ê 3 , obtaining the acceleration of gravity ( g φ ) at sea level in terms of the
latitude φ .

X 3 , x3
r
ω = ω 3 ê 3
ω3 r r
r = r eˆ r = reˆ r
r ê3
r
êθ
r ê r
x

X 2 , x2

X 1 , x1

Figure 2.19
Solution:
r r r r
a.1) By means of Problem 2.55 we can conclude that v ( x , t ) = ω ∧ x , or in indicial
notation:
vi =  ijk ω j xk =  i1k ω1 xk +  i 2 k ω2 xk +  i 3k ω3 xk =  i 3k ω3 xk
{ {
=0 =0

=  i 31ω3 x1 +  i 32 ω3 x2 +  i 33 ω3 x3 =  i 31ω3 x1 +  i 32 ω3 x2
{
=0i

Then:
v1 = 132ω3 x2 = −ω3 x2 ; v2 =  231ω3 x1 = ω3 x1 ; v3 = 0 (2.147)
r r r r r
Note that the field v ( x , t ) is stationary, i.e. v = v ( x ) .
r r
For a rigid body motion when c (t ) = 0 , the equations of motion are governed by:
r r
x = Q(t ) ⋅ X
where the orthogonal matrix components are given by the transformation matrix from the
r r
system x ′ to x , thus:
 x1  cos θ(t ) − sin θ(t ) 0  X 1  cos θ(t ) X 1 − sin θ(t ) X 2 
      
 x 2  =  sin θ(t ) cos θ(t ) 0  X 2  = sin θ(t ) X 1 + cos θ(t ) X 2 
x   0 0 1  X 3   X3 
 3  
dθ(t )
Considering that ω = and by integrating we can obtain:
dt

∫ dθ(t ) = ∫ ωdt ⇒ θ(t ) = ωt

Then, we can rewrite the equations of motion as follows:

University of Castilla-La Mancha Draft By: Eduardo W. V. Chaves (2016)


Ciudad Real - Spain
2 CONTINUUM KINEMATICS 239

 x1  cos θ(t ) − sin θ(t ) 0  X 1   X 1 cos(ωt ) − X 2 sin(ωt ) 


      
 x 2  =  sin θ(t ) cos θ(t ) 0  X 2  =  X 1 sin(ωt ) + X 2 cos(ωt ) (2.148)
x   0 0 1  X 3   X3 
 3  
To obtain the expression for velocity in the material (Lagrangian) description, we replace
the equations of motion (2.148) into the equations (2.147):
r
v1 ( X , t ) = −ω 3 ( X 1 sin(ωt ) + X 2 cos(ωt ))
 r
v 2 ( X , t ) = ω 3 ( X 1 cos(ωt ) − X 2 sin(ωt )) (2.149)
 r
v3 ( X , t ) = 0

a.2) The Eulerian acceleration can be obtained by means of the definition of material time
r r
derivative of v ( x , t ) , i.e.:
r r r r r r
r r ∂v ( x , t ) ∂v ( x , t ) ∂x ( X , t ) r r r
a ( x, t ) = + r ⋅ = (∇ xr v ) ⋅ v ( x , t )
1
42 ∂t4 3 ∂x ∂t
r
0

where the spatial velocity gradient components are given by:


 ∂v1 ∂v1 ∂v1 
 
r r  ∂x1 ∂x2 ∂ x3   0 − ω3 0
 ∂v ( x , t )  r  ∂v 2 ∂v2 ∂v 2  
0  (antisymmetric)
 r  = (∇ xr v ) ij =  = ω3 0
 ∂x  ij ∂x ∂x2 ∂ x3  
 1   0 

 v3 ∂v3 ∂ v3   0 0
 ∂x1 ∂x2 ∂x3 

With that, we check that we are dealing with a rigid body motion. Then, the Eulerian
acceleration components are given by:
 0 − ω3 0  − ω3 x2   − ω3 x1 
2

   
ai ( x , t ) = [(∇ xr v ) ⋅ v ( x , t )]i = ω3 0   ω3 x1  = − ω32 x2 
r r r r
0
 0 0 0   0   0 
r r
We can express the acceleration a ( x , t ) = −ω 32 x1 eˆ 1 − ω 32 x 2 eˆ 2 in the cylindrical coordinate,
(see Figure 2.19). Note that:
x1 = r cos θ , x1 = r cos θ , eˆ 1 = eˆ r cos θ − eˆ θ sin θ , eˆ 2 = eˆ r sin θ + eˆ θ cos θ . Then, the
acceleration in the cylindrical coordinate system becomes:
r
a = −ω 32 x1 eˆ 1 − ω 32 x 2 eˆ 2
= −ω 32 ( r cos θ)(eˆ r cos θ − eˆ θ sin θ) − ω 32 ( r sin θ)(eˆ r sin θ + eˆ θ cos θ)
r
= −ω 32 r (cos 2 θ + sin 2 θ)eˆ r = −ω 32 reˆ r = −ω 32 r
The latter is known as the centripetal acceleration.
a.3) The particle at position (1,1,0) in the reference configuration describes a circular path
of radius r = 2 on the x1 − x 2 -plane, (see Figure 2.20).

University of Castilla-La Mancha Draft By: Eduardo W. V. Chaves (2016)


Ciudad Real - Spain
240 SOLVING PROBLEMS BY MEANS OF CONTINUUM MECHANICS

Particle P at t = 2.5s X 2 , x2
r r
v ( X , t = 0) Particle P
r r
v P ( x , t = 2 . 5) 1
r
r X
x

1 X 1 , x1

Trajectory of particle P

Figure 2.20
r r
In the reference configuration ( t = 0 ) it fulfills that X = x . For the particle P we have:
r
v1P ( x , t = 0) = −ω3 x 2 = −ω 3 X 2 = −(3)(1) = −3
 P r
v 2 ( x , t = 0) = ω 3 x1 = ω 3 X 1 = (3)(1) = 3
 P
v3 = 0

 − ω 32 X 1  − 9 
r    
a iP ( x , t = 0) = − ω 32 X 2  = − 9 
 0  0 
   
At time t = 2.5s the position, velocity, and acceleration of the particle P are given by:
 x1P   X 1 cos(ωt ) − X 2 sin(ωt )  cos(3 × 2.5) − sin(3 × 2.5)  − 0.59136
 P      
 x 2  =  X 1 sin(ωt ) + X 2 cos(ωt ) = sin(3 × 2.5) + cos(3 × 2.5) =  1.28464 
x P   X3   0   0 
 3      
r
v1P ( x , t = 2.5) = −ω 3 x 2 = −(3)(1.28464) = −3.85391
 P r
v 2 ( x , t = 2.5) = ω 3 x1 = (3)(−0.59136) = −1.77409
 P
v 3 = 0

 − ω 32 x1   5.322 
r    
a iP ( x , t = 2.5) = − ω 32 x 2  = − 11.562 
 0   0 
   
b) For a particle located on the surface of the Earth, due to rotation, this particle will feel as
being projected outward according to r -direction, (see Figure 2.21). Keep in mind that the
real force is the Centripetal due to the centripetal acceleration. For convenience, we adopt a
fictitious force, centrifugal force, which would be the cause of this apparent outward
v
projection. Associated with this force we have the centrifugal acceleration ( a ctfu ) which is
v
equal but opposite to the centripetal acceleration ( a ctpe ).

University of Castilla-La Mancha Draft By: Eduardo W. V. Chaves (2016)


Ciudad Real - Spain
2 CONTINUUM KINEMATICS 241

x3 , z x3 , z

ω3 ω3

r v
x3 r a ctfu
r α
R b
φ
φ
x2 , y

x1 , x

Figure 2.21

Acceleration of gravity for defined latitude φ is given by:


v r
g φ = a ctfu + b

Remember that given two vectors it holds that


v r v 2 v r r 2
a ctfu +b = a ctfu + 2 a ctfu b cos α + b , (see Problem 1.02). For this particular case,
r v v r
we have b =g, a ctpe = a ctfu = − ω32 r = ω32 r . Also check that r = R cos φ and
cos α = cos( π − φ ) = − cos φ . With that, we can obtain:

v r v 2 v r r 2
g φ = a ctfu + b = a ctpe − 2 a ctpe b cos φ + b = (ω 32 r ) 2 − 2(ω 32 r ) g cos φ + g 2

= (ω 32 R cos φ ) 2 − 2(ω 32 R cos φ ) g cos φ + g 2

thus
g φ = g 2 − 2 gω 32 R cos 2 φ + ω 34 R 2 cos 2 φ

Note that at the poles ( φ = 90 º ) we have g φPol = g , and in the line of Ecuador it holds that
g φEcu = g 2 − 2 gω 32 R + ω 34 R 2 = ( g − ω 32 R ) 2 = g − ω 32 R .

University of Castilla-La Mancha Draft By: Eduardo W. V. Chaves (2016)


Ciudad Real - Spain
242 SOLVING PROBLEMS BY MEANS OF CONTINUUM MECHANICS

Problem 2.58
Consider a rod subjected to successive displacements as shown in Figure 2.22. Show that
the engineering strain (also known as the Cauchy strain or the infinitesimal strain) is not
additive to successive increments of strain, i.e. ε (1) + ε ( 2) ≠ ε .

B0 B B

L0 L0
L(1) L( 2 ) L(f1) L ≡ L( 2)

∆L(1)
∆L
( 2)
∆L

Figure 2.22
Solution:
The engineering strain was obtained as:
∆L L − L 0
εC = = = λ −1
L0 L0

Then, the total strain experienced by the body, i.e. from the B0 -configuration to the B -
configuration is:
L( 2 ) − L0 L( 2 )
εC = = −1
L0 L0

In the B -configuration the engineering strain is:


L(1) − L0 L(1)
ε C(1) = = −1
L0 L0

In the B -configuration considering only the displacement increment u ( 2) , we obtain:

L( 2 ) − L(1) L( 2 )
ε C( 2 ) = = (1) − 1
L(1) L
thus
 L(1) L( 2 )   L( 2) 
ε C(1) + ε C( 2 ) =  − 1 + (1) − 1 ≠  − 1 = ε C
 L0 L   L0 
An essential requirement for any strain is that it can be possible to characterize the real
displacement. For this case the final length is:

University of Castilla-La Mancha Draft By: Eduardo W. V. Chaves (2016)


Ciudad Real - Spain
2 CONTINUUM KINEMATICS 243

L0 L0
 L(1)  
∫0
ε C(1) dx= 
0

L0
− 1dx = L(1) − L0 = ∆L(1) 
  (1) ( 2)
L1 L1  ⇒ ∆L + ∆L = ∆L
 L( 2 )  

0 0

ε (C2 ) dx =  (1) − 1 dx = L( 2 ) − L(1) = ∆L( 2 ) 
L  
L0 L
0
 L 
∫0
ε C dx = 
0 0

L
− 1 dx = L − L0 = ∆L

The Green-Lagrange strain tensor
Note that the Green-Lagrange strain tensor in the B -configuration is given by:

εG =
L2 − L20
=
1 2
(
λ −1 )
2 L20 2
We could have obtained the same expression by using the relationship
E=E (1)
+F (1)T
⋅ E ⋅ F , where for the uniaxial case we have
( 2) (1)
E → ε G , E (1) → ε G(1) ,
L(1)
E ( 2 ) → ε G( 2) , F (1) → λ(1) = . Then:
L0

⋅ E ( 2) ⋅ F (1)
T
E = E (1) + F (1)

1  L(1)    L(1)  1  L( 2)   L(1) 


2 2

εG = ε G(1) + λ(1) ε G( 2) λ(1) =   − 1 +    (1)  − 1  
2  L0    L0  2  L
 

   L0 
  
 L( 2 ) 2 − L2  2 2
=
0
 = ( L − L0 )
2 L20 2 L20

2.3 Polar Decomposition of the Deformation Gradient

Problem 2.59
Let us consider the Cartesian components of the deformation gradient:
 5 3 3
Fij =  2 6 3
 2 2 4

obtain the tensors U (right stretch tensor), V (left stretch tensor), and R (rotation tensor).
Solution:
Before obtaining the tensors U , V , R , we analyze the deformation gradient F .
The motion is possible if the determinant of F is greater than zero, det ( F ) = 60 > 0 . The
eigenvalues and eigenvectors of F are given by:
F11′ = 10 associated with eigenvector mˆ i(1) = [0.6396021491; 0.6396021491; 0.4264014327]

F22′ = 3 associated with mˆ i( 2 ) = [− 0.5570860145; 0.7427813527; − 0.3713906764]

University of Castilla-La Mancha Draft By: Eduardo W. V. Chaves (2016)


Ciudad Real - Spain
244 SOLVING PROBLEMS BY MEANS OF CONTINUUM MECHANICS

F33′ = 2 associated with mˆ i(3) = [− 0.4082482905; − 0.4082482905; 0.8164965809]


It is easy to check that the basis formed by these eigenvectors does not form an orthogonal
basis, i.e. mˆ i(1) mˆ i( 2) ≠ 0 , mˆ i(1) mˆ i(3) ≠ 0 , mˆ i( 2) mˆ i(3) ≠ 0 . We can also verify that if D is the
matrix containing the eigenvectors of F :
 mˆ i(1)   0.6396021491; 0.6396021491; 0.4264014327 
 ( 2)  
D = mˆ i  = − 0.5570860145; 0.7427813527; − 0.3713906764
 mˆ (3)  − 0.4082482905; − 0.4082482905; 0.8164965809 
 i   

we find that det (D ) = 0.905 ≠ 1 , and D −1 ≠ D T . However, it holds that:


10 0 0 5 2 2 5 2 2 10 0 0
D  0 3 0 D = 3 6 2 = ( F T )ij
−1   and D 3 6 2 D =  0 3 0
  −1

 0 0 2 3 3 4 3 3 4  0 0 2

The right Cauchy-Green deformation tensor components, C = F T ⋅ F , are given by:


33 31 29
C ij = Fki Fkj =  31 49 35
29 35 34 

Then the eigenvalues and eigenvectors of C are given by:


′ = 9.274739
C11 eigenvecto
  r → ˆ (1) = [0.6861511933; − 0.7023576528; 0.1894472683]
N i

′ = 3.770098
C 22 eigenvecto
  r → ˆ ( 2 ) = [0.5105143234; 0.2793856273; − 0.8132215099]
N i

′ = 102.955163
C 33 eigenvecto
  r → ˆ (3) = [− 0.518239; − 0.65470405; − 0.550264423]
N i

These eigenvectors constitute an orthogonal basis, so, it holds that AC−1 = ACT , and
det (AC ) = −1 (improper orthogonal tensor):

Nˆ (i1)   0.6861511933 − 0.7023576528 0.1894472683 


 
AC = Nˆ i( 2)  = 0.5105143234 0.2793856273 − 0.8132215099
Nˆ (3)   − 0.518239 − 0.65470405 − 0.550264423 
 i  
Furthermore, it holds that:
C11′ 0 0  33 31 29 33 31 29 C11′ 0 0 

A  0T

C 22      T 
0  AC =  31 49 35 = C ij ; AC  31 49 35 AC =  0 ′
C 22 0 
C
 0 0 ′ 
C 33 29 35 34  29 35 34   0 0 ′ 
C 33

In the C principal space we obtain the components of the right stretch tensor, U , as:
λ 1 0 0   C11 ′ 0 0  3.0454455 0 0 
  
U ′ = U′ij =  0 λ2 
0 = 0 ′
C 22 0 = 0 1.9416741 0 

 0 0 λ 3   0 0 ′  
C 33 0 0 10.1466824
 
and its inverse:

University of Castilla-La Mancha Draft By: Eduardo W. V. Chaves (2016)


Ciudad Real - Spain
2 CONTINUUM KINEMATICS 245

1 
 0 0  1
0 0

 3.0454455 
 λ1   
1 1
U ′ −1 = U′ij−1 = 0 0 = 0 0 
 λ2   1.9416741 
  1
0 1   0 0

0  
10 . 1466824
 λ 3 
We can evaluate the components of the tensor U in the original space by means of the
transformation law:
4.66496626 2.25196988 2.48328843
A U ′AC = 2.25196988 6.00314487 2.80907159 = U ij
T
C
 2.48328843 2.80907159 4.46569091

and
 0.31528844 − 0.05134777 − 0.14302659
A U ′ AC =  2.25196988
T
C
−1
0.24442627 − 0.12519889 = U ij−1
 − 0.14302659 − 0.12519889 0.38221833 

Then, the rotation tensor of the polar decomposition is given by the equation R = F ⋅ U −1 ,
which is a proper orthogonal tensor, i.e. det (R ) = 1 .
 0.9933191 0.10094326 0.05592536
R ij = Fik U kj−1 = − 0.10658955 0.98826538 0.10940847 

− 0.04422505 − 0.11463858 0.9924224 

The left Cauchy-Green deformation tensor components, b = F ⋅ F T , are given by:


 43 37 28
bij = Fik F jk = 37 49 28
28 28 24

Next, the eigenvalues and eigenvectors of b are given by:


′ = 9.274739
b11 eigenvecto
  r → nˆ i(1) = [0.6212637156 − 0.7465251613 0.238183919]

′ = 3.770098
b22 eigenvecto
  r → nˆ (i 2 ) = [0.4898263742 0.1327190337 − 0.8616587383]

′ = 102.95516
b33 eigenvecto
  r → nˆ (i 3) = [− 0.611638389 − 0.6519860747 − 0.448121233]
Note that, the tensors b and C have the same eigenvalues but different eigenvectors. If
the eigenvectors of b constitute an orthogonal basis then it holds that Ab−1 = AbT , and
det (Ab ) = −1 :

 nˆ (i1)   0.6212637156 − 0.7465251613 0.238183919 


 ˆ ( 2)  
Ab = n i  =  0.4898263742 0.1327190337 − 0.8616587383
nˆ (3)   − 0.611638389 − 0.6519860747 − 0.448121233 
 i   
and, it also holds that:

b11 0 0  43 37 28  43 37 28 ′
b11 0 0

T
A 0 ′
b22      T 
0  Ab = 37 49 28 = bij ; Ab 37 49 28 Ab =  0 ′
b22 0 
b
 0 0 ′ 
b33  28 28 24  28 28 24  0 0 ′ 
b33

University of Castilla-La Mancha Draft By: Eduardo W. V. Chaves (2016)


Ciudad Real - Spain
246 SOLVING PROBLEMS BY MEANS OF CONTINUUM MECHANICS

Since C and b have the same eigenvalues, it follows that U′ij = Vij′ , i.e. they have the same
components in their respectively principal space. Additionally, it holds that U′ij−1 = Vij′ −1 .
The components of the tensor V in the original space can be evaluated by:
 5.3720129 2.76007379 2.41222612
A V ′Ab = A U ′Ab = 2.76007379 6.04463857 2.20098553 = Vij
T
b
T
b
 2.41222612 2.20098553 3.6519622 

and
 0.28717424 − 0.07950684 − 0.14176921
A V ′ Ab = A U ′ Ab = − 0.07950684 0.23396031 − 0.08848799 = Vij−1
T
b
−1 T
b
−1

 − 0.14176921 − 0.08848799 0.42079849 

The polar decomposition rotation tensor obtained previously has to be the same as the one
obtained by R = V −1 ⋅ F .
We could also have obtained the tensors U , V , R , by means of their spectral
representation. That is, if we know the principal stretches, λ i , and the eigenvectors of C
ˆ (i ) ), and the eigenvectors of b ( nˆ (i ) ), it is easy to show that:
(N
 3 ˆ ( a )  = λ Nˆ (1)Nˆ (1) + λ Nˆ ( 2 ) Nˆ ( 2 ) + λ Nˆ (3) N

U ij =  λ a N
 a =1
ˆ (a) ⊗ N

 ij
1 i j 2 i j 3 i
ˆ (3)
j

 3 

Vij =  λ a nˆ ( a ) ⊗ nˆ ( a )  = λ 1 nˆ (i1) nˆ (j1) + λ 2 nˆ i( 2) nˆ (j2) + λ 3 nˆ i(3) nˆ (j3)
 a =1  ij

 3 (a) ˆ (a ) 
R ij =  ∑
 a =1  ij
ˆ (1) + nˆ ( 2 )Nˆ ( 2 ) + nˆ (3) Nˆ (3)
nˆ ⊗ N  = nˆ (i1) N j i j i j

 3 ˆ ( a )  = λ nˆ (1) Nˆ (1) + λ nˆ ( 2 ) Nˆ ( 2 ) + λ nˆ (3) Nˆ (3)



Fij =  λ a nˆ ( a ) ⊗ N
 a =1

 ij
1 i j 2 i j 3 i j

3 3
F= ∑
a =1
λa R ⋅N
ˆ (a) ⊗ N
ˆ (a ) =
∑λ
a =1
a nˆ ( a ) ⊗ nˆ ( a ) ⋅ R

 3
ˆ ( a )  =  λ nˆ ( a ) ⊗ nˆ ( a )  ⋅ R = R ⋅ U = V ⋅ R
3


= R ⋅  λ a N
 a =1
ˆ (a) ⊗ N
 
  a =1
a ∑ 

As we can verify, the representations of the tensors R and F are not the spectral
representations in the strict sense of the word, i.e., λ i are not eigenvalues of F , and
ˆ (i ) are eigenvectors of F .
neither nˆ (i ) nor N
Problem 2.60
The deformation gradient at one point of the body is given by:
F = 0.2eˆ 1 ⊗ eˆ 1 − 0.1eˆ 1 ⊗ eˆ 2 + 0.3eˆ 2 ⊗ eˆ 1 + 0.4eˆ 2 ⊗ eˆ 2 + 0.1eˆ 3 ⊗ eˆ 3
where eˆ i (i = 1,2,3) represents the Cartesian basis.
a) Obtain the deformation tensors b and C ;
b) Obtain the eigenvalues and eigenvectors of b and C ;

University of Castilla-La Mancha Draft By: Eduardo W. V. Chaves (2016)


Ciudad Real - Spain
2 CONTINUUM KINEMATICS 247

c) Write the “spectral representation” of F in function of the eigenvalues of C ( C a ) and


3
check if F = ∑ λ a nˆ ( a ) ⊗ N
ˆ ( a ) holds, where λ are the principal stretches, n̂ are the
a
a =1

eigenvectors of b , and N̂ are the eigenvectors of C ;


d) Obtain the spectral representation and components of: ( R ) spin tensor of the polar
decomposition; the stretch tensors U and V ;
Solution
The deformation gradient components can be represented as follows:
F = Fij eˆ i ⊗ eˆ j = 0.2eˆ 1 ⊗ eˆ 1 − 0.1eˆ 1 ⊗ eˆ 2 + 0.3eˆ 2 ⊗ eˆ 1 + 0.4eˆ 2 ⊗ eˆ 2 + 0.1eˆ 3 ⊗ eˆ 3

 F11 F12 F13  0.2 − 0.1 0 


Fij =  F21 F22 F23  = 0.3 0.4 0 
 F31 F32 F33   0 0 0.1

a) The left Cauchy-Green deformation tensor ( b = F ⋅ F T ) components are given by:


T
0.2 − 0.1 0  0.2 − 0.1 0  0.05 0.02 0 
bij = Fik F jk 
=  0 .3 0 .4  
0   0 .3 0 .4  
0  = 0.02 0.25 0  (2.150)
 0 0 0.1  0 0 0.1  0 0 0.01

The right Cauchy-Green deformation tensor ( C = F T ⋅ F ) components are given by:


T
0.2 − 0.1 0 
0.2 − 0.1 0  0.13 0.1 0 
C ij = Fki Fkj = 0.3 0.4 0 
 0.3 0.4  
0  =  0.1 0.17 0  (2.151)
 0 0 0.1
 0 0 0.1  0 0 0.01
b) The eigenvalues and eigenvectors of b and C are obtained as follows;
C ⋅N
ˆ =C N
(a)
ˆ (a) ⇒ C − C1 = 0

where the index (a ) does not indicate summation. Note that we already know one
eigenvalue of C , i.e. C (3) = 0.01 , (see C -components in (2.151)). Then, the characteristic
determinant becomes:
0.13 − C 0 .1
=0 ⇒ (0.13 − C )(0.17 − C ) − 0.01 = 0
0 .1 0.17 − C
The solution of the quadratic equation is:
C (1) = 0.25198 ; C ( 2 ) = 0.04802

Then:
0.633399  − 0.77334
Cc (1) = 0.25198 ⇒ N i =  0.77334 
ˆ (1)
; C ( 2 ) = 0.04802 ⇒ N i =  0.63399 
ˆ (2)

 0   0 
0 
C (3) = 0.01 ⇒ Nˆ i(3) = 0
1

University of Castilla-La Mancha Draft By: Eduardo W. V. Chaves (2016)


Ciudad Real - Spain
248 SOLVING PROBLEMS BY MEANS OF CONTINUUM MECHANICS

The eigenvectors of the tensor b :


b ⋅ nˆ = b( a ) nˆ ( a )

where the index (a ) does not indicate summation. Then


0.098538  − 0.995133
b(1) = 0.25198 ⇒ nˆ i(1)= 0.995133 ; b( 2 ) = 0.04802 ⇒ nˆ (i 2 ) =  0.098538 
 0   0 
0 
b(3) = 0.01 ⇒ nˆ i(3) = 0
1 

As expected, the tensors C and b have the same eigenvalues, i.e.


0.252 0 0  0.252 0 0 

C ij′ =  0 0.048 0  ; 
bij′ =  0 0.048 0 
 0 0 0.01  0 0 0.01
but they have different eigenvectors. In addition, the spectral representations of the tensors
C and b are given respectively by:
3 3
C= ∑
a =1
ˆ (a ) ⊗ N
λ2a N ˆ (a) ; b= ∑ λ nˆ
a =1
2
a
(a)
⊗ nˆ ( a )

where λ a > 0 are the principal stretches. Considering that λ2a = C a are the eigenvalues of
C and of b , the principal stretches are:
λ (1) = 0.25198 ≈ 0.501976 ; λ ( 2) = 0.04802 ≈ 0.219134 ; λ (3) = 0.01 = 0.1
3
c) To check if F= ∑ λ nˆ
a =1
a
(a) ˆ (a )
⊗N holds we calculate the components of

 3 ˆ ( a )  with the results obtained previously, i.e.:



 λ a nˆ ( a ) ⊗ N

 a =1

 ij

 3 ˆ ( a )  = λ nˆ (1) ⊗ Nˆ (1) + λ nˆ ( 2 ) ⊗ Nˆ ( 2 ) + λ nˆ (3) ⊗ Nˆ (3)



 λ a nˆ ( a ) ⊗ N

 a =1

 ij
1 i j 2 i j 3 i j

 0.76958 − 0.6309 0 0.06247 0.0762 0


= 0.50197  − 0.0762 0.06247 0 + 0.219134 0.6309 0.76958 0 +
 
 0 0 0  0 0 0
0 0 0
+ 0.10 0 0
0 0 1
0.2 − 0.1 0 
= 0.3 0.4 0  = Fij
 0 0 0.1
3
With that the equation F = ∑ λ a nˆ ( a ) ⊗ N
ˆ ( a ) holds.
a =1

University of Castilla-La Mancha Draft By: Eduardo W. V. Chaves (2016)


Ciudad Real - Spain
2 CONTINUUM KINEMATICS 249

d) The orthogonal tensor is given by

3
0.832 − 0.554 0
R= ∑ ˆ (a)
nˆ ( a ) ⊗ N components (R )ij = 0.554 0.832 0
a =1
 0 0 1
which can be verified with:
 3 ˆ ( a )  = nˆ (1) ⊗ Nˆ (1) + nˆ ( 2 ) ⊗ Nˆ ( 2 ) + nˆ (3) ⊗ Nˆ (3)
R ij =  ∑
 a =1
nˆ ( a ) ⊗ N 
ij
i j i j i j

 0.76958 − 0.6309 0 0.06247 0.0762 0 0 0 0  0.832 − 0.5547 0


R ij = − 0.0762 0.06247 0 +  0.6309 0.76958 0 + 0 0 0 = 0.5547 0.832 0
 0 0 0  0 0 0 0 0 1  0 0 1
The right stretch tensor:

3
0.333 0.139 0 
U= ∑ ˆ (a) ⊗ N
λ aN ˆ (a) components (U)ij ≈ 0.139 0.388 0 
a =1
 0 0 0.1
The left stretch tensor:

3
0.222 0.028 0 
V= ∑ λ nˆ a
(a )
⊗ nˆ (a)
components (V )ij ≈ 0.028 0.5 0 
a =1
 0 0 0.1

Problem 2.61
Consider the following equations of motion:
x1 = X 1 ; x2 = X 2 − αX 3 ; x3 = X 3 + αX 2
a) Obtain the deformation gradient, the right Cauchy-Green deformation tensor, the left
Cauchy-Green deformation tensor, the Green-Lagrange strain tensor and the Almansi
strain tensor. Check whether this case represents a homogeneous deformation.
b) Obtain the right stretch tensor, the spin tensor of polar decomposition and the principal
space of the left Cauchy-Green deformation tensor of the polar decomposition.
c) Obtain the final length of an initial length element equal to 2 which is in the X 3 -
direction, and the angular distortion of an initial angle 30º which is in the plane X 1 − X 2 .
d) Obtain the strain tensor by considering the small deformation regime.
Solution:
a) The deformation gradient ( F ) components are:
 ∂x1 ∂x1 ∂x1 
 
 ∂X 1 ∂X 2 ∂X 3  1 0 0 
∂xi  ∂x 2 ∂x 2 ∂x 2  
Fij = = = 0 1 − α 
∂X j  ∂X 1 ∂X 2 ∂X 3  
  
∂x3  0 α 1 

 ∂x3 ∂x3
 ∂X 1 ∂X 2 ∂X 3 

University of Castilla-La Mancha Draft By: Eduardo W. V. Chaves (2016)


Ciudad Real - Spain
250 SOLVING PROBLEMS BY MEANS OF CONTINUUM MECHANICS

r r
In general we have dx = F ⋅ dX , and if we are dealing with a homogeneous deformation (a
r r r
particular case of motion) the relationship x = F ⋅ X + c (t ) holds, a fact that can be
r r
checked by means of the equations of motion in matrix form with c(t ) = 0 :
 x1  1 0 0   X1 
 x  = 0 1 − α   X 
 2    2
 x3  0 α 1   X 3 

The right Cauchy-Green deformation tensor, C = F T ⋅ F , components are:


1 0 0  1 0 0  1 0 0 
    
C ij = Fki Fkj = 0 1 α  0 1 − α  = 0 1 + α 2
0 
0 − α 1  0 α 1  0 0 1 + α 2 

The left Cauchy-Green deformation tensor, b = F ⋅ F T , components are:


1 0 0  1 0 0  1 0 0 
    
bij = Fik F jk = 0 1 − α  0 1 α  = 0 1 + α 2
0 
0 α 1  0 − α 1  0 0 1 + α 2 

1
The Green-Lagrange strain tensor, E = (C − 1) , and the Almansi strain tensor,
2
1
e= (1 − b −1 ) , are defined by their components as follows:
2
 1 0 0   1 0 0  0 0 0 
1 1     1 
E ij = (C ij − δ ij ) =  0 1 + α 2
0  −  0 1 0   = 0 α 2 0 
2 2  2
 0
 0 1 + α 2   0 0 1   0 0 α 2 

    
 1 0 0 1 0 0  0 0 0 
     
1 1   1 1 α 2
eij = (δ ij − bij−1 ) =  0 1 0  −  0 0  = 0 0 
2 2    1+α 2  2  1 + α 2 
 0 0 1   1   α2 
 0 0 2  0 0 
  1 + α   1+α 2 
We can check the results by the relationship E = F T ⋅ e ⋅ F :
0 0 0 
 
1 0 0    1 0 0  0 0 0
1 α2 1
Eij = 0 1 α  0 0  0 1 
− α  = 0 α 2
0 
2  1+α 2  2
0 − α 1   α 2  0 α 1  0 0 α 2 
0 0 
 1+α 2 
b) According to the format of the Cartesian components of C , we can verify that the
original space is already the principal space of C , i.e. the principal directions are
Nˆ i(1) = [1 0 0] , Nˆ i(1) = [0 1 0] , Nˆ i(1) = [0 0 1] . By definition, the right stretch tensor is
given by U = C , and its components are:

University of Castilla-La Mancha Draft By: Eduardo W. V. Chaves (2016)


Ciudad Real - Spain
2 CONTINUUM KINEMATICS 251

 
1 0 0 
 1 0 0   
  inverse  1 
Uij =  0 1+α 2 −1
0  → Uij = 0 0 
0 0 1+α 2   1+α2 
  0 1 
0
 
 1+α 2 

By means of the right polar decomposition, F = R ⋅ U ⇒ R = F ⋅ U −1 , we can obtain:


 
1 0 0 
1 0 0    1+α 2 0 0 
 1  1  
R ij = Fik Ukj = 0 1
−1
− α  0 0 =  0 1 −α 
0 α 1   1+α 2  1+α 2  0 α 1 
0 1   
0
 
 1+α 2 
Note that by means of the format of the Cartesian components of b indicate that the
principal directions are [1 0 0] , [0 1 0] , [0 0 1] , but this is not the principal
directions of b related to the polar decomposition. Note that there are two equal
eigenvalues related to the directions [0 1 0] , [0 0 1] , then any direction in the plane
x 2 − x 3 is a principal direction, (see Figure 2.23).

X 2 , x2

Any direction on the plane


x 2 − x 3 is a principal
direction of b The principal direction n̂ (1) is
unique, associated with the
eigenvalue b1 = 1 .

X 1 , x1

X 3 , x3

Figure 2.23: Principal space of b .


Remember that the polar decomposition is unique, i.e. there is one principal base b for the
polar decomposition associated with N ˆ ( a ) . By means of the relation nˆ ( a ) = R ⋅ N
ˆ ( a ) we can
obtain the principal base of b for the polar decomposition:
 1+α 2 0 0  0 0
1  1
nˆ i( 2 ) = (R ⋅ N
ˆ ( 2) ) =
i  0 1 − α  1 = 1
 
1+α 2  0 α 1  0 1+α 2 α 
 
 1+α 2 0 0  0   0 
 1  1
nˆ i(3) = (R ⋅ N
ˆ ( 3) ) =
i  0 1 − α  0 = − α 
 
1+α 2  0 α 1  1 1+α 2  1 
 

University of Castilla-La Mancha Draft By: Eduardo W. V. Chaves (2016)


Ciudad Real - Spain
252 SOLVING PROBLEMS BY MEANS OF CONTINUUM MECHANICS

3
In addition, we can check that the relation R = ∑ nˆ ( a ) ⊗ N
ˆ ( a ) holds:
a =1

R ij = nˆ i(1)Nˆ (j1) + nˆ i( 2 )Nˆ (j2) + nˆ i(3)Nˆ (j3)


1 0  0 
1 1
= 0[1 0 0] +  1 [0 1 0] +
 
− α [0 0 1]
 
0 1+α 2 α  1+α 2  1 

1 0 0 0 0 0 0 0 0   1+α 2 0 0 
1 1 1  
R ij = 0 0 0 + 0 1
 0 + 0 0 − α  =
   0 1 −α 
0 0 0 1+α 2 0 α 0 1+α 2 0 0 1  1 +α 2  0 α 1 
 
r
dx ds
c) By means of the stretch definition according to the M̂ -direction, i.e. λ Mˆ = r = ,
dX dS
and considering that the stretch is not dependent on line integral (homogeneous
deformation), it holds that:

∫ ∫ ∫
L final = ds = λ Mˆ dS = λ Mˆ dS = λ Mˆ Linitial

The stretch according to X 3 -direction is given by:

λ X = C 33 = 1 + 2 E 33 = 1 + α 2
3

Then:
2


L final = λ Mˆ dX 2 = 1 + α 2 ( Linitial ) = 2 1 + α 2
0

As we are dealing with a homogeneous deformation, a line in the reference configuration


remains a line in the current configuration, (see Figure 2.24).

X 3 , x3
 x1A  1 0 0   X 1A 
 A    A
A′ A  x 2  = 0 1 − α   X 2 
 x3A  0 α 1   X 3A 
    
Linitial = 2 1 0 0  0  0 
L final
= 0 1 − α  0 =  − 2α 
0 α 1   2  2 
− 2α O X 2 , x2

X 1 , x1

Figure 2.24

According to Figure 2.24 we can check that:

University of Castilla-La Mancha Draft By: Eduardo W. V. Chaves (2016)


Ciudad Real - Spain
2 CONTINUUM KINEMATICS 253

L2initial = 22 + (−2α ) 2 = 4(1 + α 2 ) ⇒ Linitial = 2 1 + α 2


To obtain the angle in the current configuration formed by two unit vectors, we can use
the equation:
cos Θ + 2 M ˆ ⋅ E ⋅ Nˆ
cosθ = (2.152)
λ Mˆ λ Nˆ

where Θ is the angle between the unit vectors M̂ and N̂ in the reference configuration,
and θ is the angle between the two new unit vectors in the current configuration.
r
Considering that the Green-Lagrange strain tensor is independent of X , we adopt two unit
vectors forming an angle Θ = 30º in the plane X 1 − X 2 , e.g. Nˆ i = [1 0 0] and
Mˆ i = [cos 30º sin 30º 0] . With these data we have:

0 0 0  cos 30º 
1 
ˆ ⋅ E ⋅ Nˆ = [1 0 0] 0 α
M 
2
0   sin 30º  = 0
2
0 0 α 2   0 

The stretches:
1 0 0  1 
λ2Mˆ ˆ 
= M ⋅ C ⋅ M = [1 0 0] 0 1 + α
ˆ 2
0  0 = 1 ⇒ λ Mˆ = 1
0 0 1 + α 2  0
and
1 0 0  cos 30º 
λ2Nˆ ˆ ˆ 
= N ⋅ C ⋅ N = [cos 30º sin 30º 0] 0 1 + α 2
0   sin 30º 
0 0 1 + α 2   0 
= cos 2 30º + (1 + α 2 ) sin 2 30º = 1 + α 2 sin 2 30º

Then, λ Nˆ = 1 + α 2 sin 2 30º . Then, we can obtain:

cos Θ + 2 Mˆ ⋅ E ⋅ Nˆ cos 30º


cosθ = =
λ Mˆ λ Nˆ 1 + α 2 sin 2 30º
As we are dealing with a homogeneous deformation, we adopt two lines in the reference
configuration and we obtain the angle formed by these lines in the current configuration.
For example, adopting the lines OB = [cos 30º 0 0] and OC = [cos 30º sin 30º 0]. And
according to the equations of motion, the point O does not move. Then, we obtain the
new position of the points B and C , (see Figure 2.25):
 x1B  1 0 0   X 1B  1 0 0  cos 30º  cos 30º 
 B    B     
 x 2  = 0 1 − α   X 2  = 0 1 − α   0  =  0 
 x B  0 α 1   X B  0 α 1   0   0 
 3   3      

 x1C  1 0 0   X 1C  1 0 0  cos 30º   cos 30º 


 C    C     
 x 2  = 0 1 − α   X 2  = 0 1 − α   sin 30º  =  sin 30º 
 x C  0 α 1   X C  0 α 1   0  α sin 30º 
 3   3      

University of Castilla-La Mancha Draft By: Eduardo W. V. Chaves (2016)


Ciudad Real - Spain
254 SOLVING PROBLEMS BY MEANS OF CONTINUUM MECHANICS

X 3 , x3

A′ α sin 30º

C′

θ O sin 30º
X 2 , x2
cos 30º 30º
B = B′
C

X 1 , x1

Figure 2.25

Then, the angle formed by the new unit vectors O ′B ′ and O ′C ′ is:
O′B′ ⋅ O′C ′ = O′B′ O′C ′ cosθ
cos 30º
cos 2 30º = cos 2 30º cos 2 30º + sin 2 30º +α 2 sin 2 30º cos θ ⇒ cosθ =
1 + α 2 sin 2 30º
d)
0 0 0 
ε ij = 0 0 0
0 0 0

Problem 2.62
For a given motion (shear deformation):
x1 = X 1 + kX 2 ; x2 = X 2 ; x3 = X 3
where k is a constant. Obtain the tensors: F (deformation gradient), C (the right Cauchy-
Green deformation tensor), b (the left Cauchy-Green deformation tensor), E (the Green-
Lagrange strain tensor), U (the right stretch tensor), V (the left stretch tensor) and R (the
spin tensor of the polar decomposition).
Solution:
The deformation gradient components:
 ∂x1 ∂x1 ∂x1 
 
 ∂X 1 ∂X 2 ∂X 3  1 k 0
∂x ∂x ∂x2 ∂x2  
Fij = i =  2 = 0 1 0
∂X j ∂X 1 ∂X 2 ∂X 3  
   
 ∂x3 ∂x3 ∂x3  0 0 1 
 ∂X 1 ∂X 2 ∂X 3 

University of Castilla-La Mancha Draft By: Eduardo W. V. Chaves (2016)


Ciudad Real - Spain
2 CONTINUUM KINEMATICS 255

The right Cauchy-Green deformation tensor, C = F T ⋅ F , components are:


 1 0 0 1 k 0   1 k 0
    
C ij = Fki Fkj = k 1 0 0 1 0 = k 1 + k 2 0
 0 0 1 0 0 1  0 0 1

The left Cauchy-Green deformation tensor, b = F ⋅ F T , components are:


1 k 0  1 0 0 1 + k 0
2
k
 
bij = Fik F jk = 0 1 0  k 1 0 =  k 1 0
0 0 1  0 0 1  0 0 1

1
The Green-Lagrange strain tensor, E = (C − 1) , components are:
2
 1 k 0 1 0 0  0 k 0
1      1
E ij =   k 1 + k 2 0  − 0 1 0   =  k k 2 0
2 2
  0 0 1 0 0 1   0 0 0

Note that there is only deformation on the x1 − x 2 -plane.


Considering the polar decomposition F = R ⋅ U = V ⋅ R , we can obtain:
C = (V ⋅ R)T ⋅ (V ⋅ R) = R T ⋅ V T ⋅ V ⋅ R = R T ⋅ V ⋅ V ⋅ R = R T ⋅ V 2 ⋅ R = R T ⋅ b ⋅ R
For simplicity, we will work on the x1 − x 2 -plane, with that we represent the rotation
tensor components as follows:
cos θ − sin θ c − s 
R ij =  =  (i, j = 1,2)
 sin θ cos θ   s c 
where cos 2 θ + sin 2 θ = c 2 + s 2 = 1 holds. The relationship C = R T ⋅ b ⋅ R becomes:
1 k   c s  1 + k 2 k  c − s 
k 1 + k 2  =  − s c    
    k 1 s c 
(c 2 + c 2 k 2 + 2 sck + s 2 ) (− sck 2 − s 2 k + c 2 k ) 
= 2 2 2 
 (− sck − s k + c k ) (c 2 + s 2 k 2 − 2 sck + s 2 )

From the relationship (c 2 + c 2 k 2 + 2 sck + s 2 ) = 1 ⇒ (c 2 k 2 + 2 sck + 1) = 1 we can obtain


−k
s= c . Then, starting from the relation (− sck 2 − s 2 k + c 2 k ) = k and by considering that
2
−k
s= c , we can obtain:
2
−k
1 2 2 −k
c= = ; s= =
2 2 2
k k +4 k k2 + 4
+1 +1
4 4
thus:

University of Castilla-La Mancha Draft By: Eduardo W. V. Chaves (2016)


Ciudad Real - Spain
256 SOLVING PROBLEMS BY MEANS OF CONTINUUM MECHANICS

 2 k 
 2 0
 k +4 k2 + 4 
−k 2
R ij =  0
 2 
 k +4 k2 + 4 
 0 0 1
 
From the polar decomposition F = R ⋅ U = V ⋅ R , we obtain U = R T ⋅ F and V = F ⋅ R T ,
whose components are:
 2 −k   2 k 
0  2 0
 2 k2 + 4
 k +4 k2 + 4  1 k 0  k + 4 
k 2  k 2 + k2 
Uij = R ki Fkj =  0 0 1 0 =  0
 2 
 k +4 k2 + 4 2
 0 0 1  k + 4 k2 + 4 
 0  
0 1  0 0 1
   

  2+k 
2
 2 −k k
 2 0  0
2 2 2
1 k 0  k + 4 k +4   k +4 k +4 
k 2  k 2 
Vij = Fik R jk = 0 1 0  0 =  0
 2 
0 0 1  k + 4 k2 + 4 2
  k +4 k2 + 4 
 0  
0 1  0 0 1
   
Problem 2.63
A deformable parallelepiped of dimensions 2 × 2 × 1 is in the reference configuration as
indicated in Figure 2.26. This body is subjected to motion:
r r
x ( X , t ) = −exp X 2t eˆ 1 + tX 12 eˆ 2 + X 3 eˆ 3 (2.153)
where ( X 1 , X 2 , X 3 ) are the material coordinates, and t stands for time.
a) Obtain the deformation gradient F .
b) Obtain the right Cauchy-Green deformation tensor C , and the principal stretches.
c) Obtain the right stretch tensor U and the rotation tensor R . Check if R is a proper
orthogonal tensor.
d) Find the volume of the deformed parallelepiped at time t = 1s .

X2

X3 1 2 X1

Figure 2.26

University of Castilla-La Mancha Draft By: Eduardo W. V. Chaves (2016)


Ciudad Real - Spain
2 CONTINUUM KINEMATICS 257

Solution:
a) According to the equation (2.153), the vector position components are x1 = −exp X 2t ,
x 2 = tX 12 , x3 = X 3 , then the deformation gradient ( F ) components are given by:

 ∂x1 ∂x1 ∂x1 


 
 ∂X 1 ∂X 2 ∂X 3   0 − t exp X 2 t 0
∂x ∂x ∂x2 ∂x2   
Fij = i =  2 = 2tX 1 0 0
∂X j ∂X ∂X 2 ∂X 3  
 1 
∂x3   0 0 1
 ∂x3 ∂x3
 ∂X 1 ∂X 2 ∂X 3 

b) The right Cauchy-Green deformation tensor are defined by C = F T ⋅ F , whose


components are C ij = Fki Fkj :

 0 2tX 1 0  0 − t exp X 2t 0  4t 2 X 12 0 0
    
C ij = − t exp X 2t 0 0  2tX 1 0 0 =  0 t 2 exp 2 X 2t 0
 0 0 1  0 0 1  0 0 1

Note that this space is the principal space (principal directions) of C . Considering that λ i
are the principal stretches, the following relationship is fulfilled:
3 3
C = U2 = ∑
a =1
ˆ (a ) ⊗ N
λ2a N ˆ (a) ⇒ U= ∑λ
a =1
a
ˆ (a) ⊗ N
N ˆ (a)

As we are working in the principal space of C , we can obtain the principal stretches as
follows:
λ 1 = + 4t 2 X 12 ; λ 2 = + t 2 exp 2 X 2t ; λ3 = + 1
3
which are positive numbers, since U = ∑ λ a N
ˆ (a ) ⊗ N
ˆ ( a ) is a positive definite tensor by
a =1
definition, thus:
λ1 = 2tX 1 ; λ 2 = t exp X 2t ; λ3 = 1
c)
 1 
 2tX 0 0
2tX 1 0 0  1

 1
Uij =  0 t exp X 2 t 0 ⇒ Uij =  0
−1
0
 t exp X 2 t 
 0 0 1  
 0 0 1
 
According to the polar decomposition, F = R ⋅ U ⇒ R = F ⋅ U −1 , we can obtain the rotation
tensor ( R ) components as follows:
 1 
 0 0
 0 − t exp X 2t
0  2tX 1  0 − 1 0 
  1
R ij = 2tX 1 0 0  0 0 = 1 0 0
 t exp X 2 t 
 0 0 1   0 0 1

 0 0 1
 

University of Castilla-La Mancha Draft By: Eduardo W. V. Chaves (2016)


Ciudad Real - Spain
258 SOLVING PROBLEMS BY MEANS OF CONTINUUM MECHANICS

Note that the orthogonality condition R ⋅ R −1 = R ⋅ R T = 1 holds:


0 − 1 0  0 1 0 1 0 0
R ik R jk = 1 0 0  − 1 0 0 = 0 1 0
0 0 1  0 0 1 0 0 1

and the proper condition det (R ) = +1 .


d) To calculate the final volume we use the relationship dV = JdV0 , where J = F is the
Jacobian determinant and is given by:
0 − t exp X 2t 0
J = 2tX 1 0 0 = 2t 2 X 1exp X 2t
0 0 1

At time t = 1s we have J = 2 X 1exp X 2 . Then, the volume at time t = 1s is given by:


2 2 1

∫ ∫ ∫ ∫ ∫ (2 X exp
X2
dV = JdV0 = 1 )dX 3 dX 2 dX 1 = 4(exp 2 − 1) ≈ 25.556
V0 X 1 =0 X 2 =0 X 3 =0

NOTE: We cannot use the equation V = JV0 because we are not dealing with
homogeneous deformation case.

Problem 2.64
A body is subjected to motion:
x1 = X 1 ; x 2 = X 2 + kX 3 ; x3 = X 3 + kX 2
where k is a constant.
a) Obtain the deformation gradient ( F ); the right Cauchy-Green deformation tensor ( C );
the Green-Lagrange strain tensor ( E ).
b) Calculate the displacement field, the magnitude (dx) 2 of sides OA and OB , and
diagonal OC after deformation of Figure 2.27.
c) Consider now a square as indicated in Figure 2.28:
c.1) Obtain the stretches according to directions OC and BA ; c.2) Obtain the angle θ 23 in
the current configuration in function of k .
c.3) Apply the polar decomposition of the tensor F in order to obtain U and R .

University of Castilla-La Mancha Draft By: Eduardo W. V. Chaves (2016)


Ciudad Real - Spain
2 CONTINUUM KINEMATICS 259

X3

dX 2
B C

dX 3

O A X2
X1

Figure 2.27

x3 C′

B′
B C

θ 23
A′
x2
O A

Figure 2.28
Solution:
a) The deformation gradient components are:
 ∂x1 ∂x1 ∂x1 
 
 ∂X 1 ∂X 2 ∂X 3  1 0 0 
∂xi  ∂x2 ∂x2 ∂x2  
Fij = = = 0 1 k 
∂X j  ∂X 1 ∂X 2 ∂X 3 
   
 ∂x3 ∂x3 ∂x3  0 k 1 
 ∂X 1 ∂X 2 ∂X 3 

The right Cauchy-Green deformation tensor is given by C = F T ⋅ F , thus:


1 0 0  1 0 0  1 0 0 
    
C ij = Fki Fkj = 0 1 k  0 1 k  = 0 1 + k 2
2k 
0 k 1  0 k 1  0 2k 1 + k 2 

1
The Green-Lagrange strain tensor, E = (C − 1) , components are:
2
 1 0 0  1 0 0  0 0 0
1      1
E ij =  0 1 + k 2
2 k  − 0 1 0   = 0 k 2 2k 
2 2
 0 2k 1 + k 2  0 0 1  0 2k k 2 

University of Castilla-La Mancha Draft By: Eduardo W. V. Chaves (2016)


Ciudad Real - Spain
260 SOLVING PROBLEMS BY MEANS OF CONTINUUM MECHANICS

r r r
b.1) The displacement field, u = x − X , components are:
u1 = x1 − X 1 = 0 ; u 2 = x 2 − X 2 = kX 3 ; u 3 = x 3 − X 3 = kX 2
r 2
b.2) Calculation of dx = (ds) 2 :
r2 r r r r r r r r
(ds ) 2 = dx = dx ⋅ dx = F ⋅ dX ⋅ F ⋅ dX = dX ⋅ F T ⋅ F ⋅ dX = dX ⋅ C ⋅ dX
thus:
1 0 0   dX 1 
(ds ) = [dX 1 dX 2
2 
dX 3 ] 0 1 + k 2
2k  dX 2 
0 2k 1 + k 2   dX 3 
= ( dX 1 ) 2 + (dX 2 ) 2 (1 + k 2 ) + (dX 3 ) 2 (1 + k 2 ) + 4k (dX 2 )(dX 3 )

Then, for the diagonal OC we have [0 dX 2 dX 3 ] , with that we can obtain:


(dx )2 = (dX 2 ) 2 (1 + k 2 ) + (dX 3 ) 2 (1 + k 2 ) + 4k (dX 2 )(dX 3 )
For the side OA we have [0 dX 2 0] , with that we can obtain:
(dx )2 = (dX 2 ) 2 (1 + k 2 )
For the side OB we have [0 0 dX 3 ] , with that we can obtain:
(dx )2 = (dX 3 ) 2 (1 + k 2 )
c) The stretch according to the N̂ -direction (reference configuration) is given by the
ˆ ⋅ C ⋅N
equation (λNˆ ) 2 = N ˆ.

 1 1 
c.1) The stretch according to the OC -direction: Nˆ i = 0  , is:
 2 2

 
 0 
1 0 0  
 1 1  1 
2
(λ OC ) = 0  0 1 + k
2
2k   = (1 + k ) 2
 2 2  2 
0 2k 1 + k 2   1 
 
 2

 1 −1
The stretch according to the BA -direction: Nˆ i = 0  , with that we can obtain:
 2 2

 
 0 
1 0 0  
 1 −1   1
2
(λ BA ) = 0  0 1 + k
2
2k    = (1 − k ) 2
 2 2  2
0 2k 1 + k 2   − 1 
 
 2
c.2) The variation of the angle can be calculated by means of the equation:
ˆ ⋅C ⋅N
M ˆ ˆ ⋅ C ⋅N
M ˆ
cos θ = =
ˆ ⋅ C ⋅M
M ˆ Nˆ ⋅ C ⋅N
ˆ λ Mˆ λ Nˆ

University of Castilla-La Mancha Draft By: Eduardo W. V. Chaves (2016)


Ciudad Real - Spain
2 CONTINUUM KINEMATICS 261

ˆ = [0 0 1] , and according to the


where the unit vector according to the OB -direction is M i

OA -direction is Nˆ i = [0 1 0] . With that we can obtain:

1 0 0  0 

(λ OB ) = [0 0 1] 0 1 + k
2 2
2k  0 = 1 + k 2
0 2k 1 + k 2  1

1 0 0  0 

(λ OA ) = [0 1 0] 0 1 + k
2 2
2k  1 = 1 + k 2
0 2k 1 + k 2  0

1 0 0  0
ˆ ˆ 
Mi C ij N j = [0 0 1] 0 1 + k 2
2k  1 = 2k
0 2k 1 + k 2  0
Then:
ˆ ⋅ C ⋅N
M ˆ 2k
cos θ 23 = =
λ Mˆ λ Nˆ 1+ k2

c.3) The polar decomposition of F = R ⋅ U = V ⋅ R , where:


3 3
C = U2 = ∑
a =1
ˆ (a ) ⊗ N
λ aN ˆ (a ) ⇒ U= C = ∑
a =1
ˆ (a) ⊗ N
λaN ˆ (a)

Calculation of the eigenvalues of C . Note that according to the format of C -components,


there is only deformation according to x 2 − x 3 -plane. In addition, we know one eigenvalue
λ1 = 1 associated with the direction N i(1) = [1 0 0] . By means of the characteristic
determinant we can obtain:
(1 + k 2 ) − λ 2k
=0 ⇒ λ2 − 2(1 + k 2 ) λ + (1 − 2k 2 + k 4) = 0
2k (1 + k 2 ) − λ
⇒ λ2 − 2(1 + k 2 ) λ + (1 − k 2 ) 2 = 0

The roots are: λ 2 = 1 + k 2 + 2k = (1 + k ) 2 ; λ 3 = 1 + k 2 − 2k = (1 − k ) 2


Then, in the principal space of C we have:
1 0 0 

C ij′ = 0 (1 + k ) 2 0 

0 0 (1 − k ) 2 

 1 1   −1 1 
The principal directions are λ 2 ⇒ Ni( 2) = 0 ( 3)
 , λ 3 ⇒ N i = 0  . Then,
 2 2  2 2
the transformation matrix between the original space and the principal space is:
 
1 0 0 
 
1 1 
aij = A = 0
 2 2
 −1 1 
0 
 2 2

University of Castilla-La Mancha Draft By: Eduardo W. V. Chaves (2016)


Ciudad Real - Spain
262 SOLVING PROBLEMS BY MEANS OF CONTINUUM MECHANICS

That is, the following must be true:


C′ = A C AT
T
   
1 0 0  1 0 0 
1 0 0    1 0 0  
0 (1 + k ) 2 1 1  1 1 
 0  = 0  0 1+ k2 2 k  0
 2 2   2 2
0 0 (1 − k ) 2   −1 1  
 0 2 k 1 + k 2

  −1 1 
0  0 
 2 2  2 2
Then, in the principal space of C , we have:
1 0 0  + 1 0 0 
   
C ij′ = 0 (1 + k ) 2 0  ⇒ U ij =  0 + (1 + k ) 2 0 
0 0 (1 − k ) 2   0 0 2 
+ (1 − k ) 

1 0 0 

⇒ U ij = 0 (1 + k ) 0 
0 0 (1 − k )
The inverse tensor in the principal space can be obtained as follows:
 
1 0 0 
 
1
Uij = 0
′ −1
0 
 (1 + k ) 
 1 
0 0 
 (1 − k ) 
The components of U in the original space are given by:
U ′ −1 = A T U −1 A
T
      
1 0 0  1 0 0  1 0 0  1 0 0 
      
1 1  0 1 1 1   1 −k 
Uij = 0 0  0 = 0
 2 2  (1 + k )  2 2  (1 − k 2 ) (1 − k 2 ) 
 −1 1   1  −1 1   −k 1 
0  0 0  0  0
 2 2  (1 − k )   2 2   (1 − k 2 ) (1 − k 2 ) 

From the polar decomposition we have F = R ⋅ U ⇒ R = F ⋅ U −1 , thus


 
1 0 0 
1 0 0    1 0 0 
1 −k  
R ij = 0 1 k  0 2 
= 0 1 0
(1 − k 2 ) (1 − k )
0 k 1    
0 −k 1  0 0 1 
 (1 − k 2 ) (1 − k 2 ) 

University of Castilla-La Mancha Draft By: Eduardo W. V. Chaves (2016)


Ciudad Real - Spain
2 CONTINUUM KINEMATICS 263

Problem 2.65
Given the following equations of motion:
x1 = λ1 X 1 ; x2 = −λ 3 X 3 ; x3 = λ 2 X 2
a) Obtain the final volume to a unit cube;
b) Obtain the deformed area to a unit square defined in the X 1 − X 2 -plane, and draw the
deformed area;
c) Apply the polar decomposition and obtain the tensors U , V and R
Solution:
a)
 x1  λ 1 0 0  X 1  λ 1 0 0 
    
 x2  =  0 0 − λ 3   X 2  ⇒ Fij =  0 0 − λ 3  (homogenous deformation)
x   0 λ2 0   X 3   0 λ2 0 
 3 
The determinant of F is given by F ≡ J = λ 1λ 2 λ 3 , and the deformed volume:
integrating
dV = F dV0   → V final = F Vinitial = λ 1λ 2 λ 3

b) Applying the Nanson’s formula and by considering the particular case (homogeneous
deformation):
r r integrating r r
da = JF −T ⋅ dA   → a final = JF −T ⋅ Ainitial

where
1 
 0 0 
eˆ 1 eˆ 2 eˆ 3 λ 2 λ 3 0 0   λ1 
r 1  0 1 
Ainitial = 1 0 0 = eˆ 3 ; Fij−1 = 0 λ 1 λ 3  =  0 0
λ 1λ 2 λ 3   λ2 
0 1 0  0 − λ 1λ 2 0   
0 −1
0 
 λ3 
With that the deformed area vector is:
1 
 0 0 
 a1   λ1  0  0 
   − 1    
a 2  = λ 1 λ 2 λ 3  0 0
 0 = − λ 1 λ 2 
a  λ3   
 3   1  0 
0 1
0 
 λ2 
and its magnitude is:
r
a final = (−λ 1λ 2 ) 2 = λ 1λ 2

where the points A(1,0,0) , B(0,1,0) and C (1,1,0) , (see Figure 2.29), move according to the
equations of motion:
 x1A  λ 1 0 0  1 λ 1   x1B  λ1 0 0  0  0 
 A       B     
 x2  =  0 0 − λ 3  0 =  0  ;  x2  =  0 0 − λ 3  1 =  0 
x A   0 λ2 0  0  0  x B   0 λ2 0  0 λ 2 
 3   3 

University of Castilla-La Mancha Draft By: Eduardo W. V. Chaves (2016)


Ciudad Real - Spain
264 SOLVING PROBLEMS BY MEANS OF CONTINUUM MECHANICS

 x1C  λ 1 0 0  1  λ 1 
 C     
 x2  =  0 0 − λ 3  1 =  0 
xC   0 λ2 0  0 λ 2 
 3 

X 3 , x3
B ′(0,0, λ 2 )
r
a final = λ 1 λ 2

C ′(λ1 ,0, λ 2 )

O (0,0,0) B (0,1,0) X 2 , x2

r
Ainitial = 1
C (1,1,0)
A(1,0,0)
A′(λ 1 ,0,0)
X 1 , x1

Figure 2.29

c) According to the polar decomposition definition F = R ⋅U = V ⋅R where


U = C = F T ⋅ F and V = b = F ⋅ F T we obtain:

λ1 0 0  λ1 0 0  λ21 0 0 λ1 0 0


  
Cij =  0 0 λ 2   0 0 − λ 3  =  0 λ22 0 ⇒ Uij =  0 λ 2 0 
 0 − λ 3 0   0 λ 2 0   0 0 λ23   0 0 λ3 

λ1 0 0  λ1 0 0  λ21 0 0 λ1 0 0


 
bij =  0 0  
− λ3   0 0 λ 2  =  0 λ23 0 ⇒ Vij =  0 λ 3 0 
 0 λ 2  
0   0 − λ3 0   0 0 λ22   0 0 λ 2 

Note that the original space coincides with the principal space of C . Note also that C and
b have the same eigenvalues but different principal directions. To obtain the spin tensor of
the polar decomposition we apply R = F ⋅ U−1 = V −1 ⋅ F :
1 
 0 0
λ1 0 0   λ1  1 0 0 
1
R ij = Fik U−kj1 =  0 0 − λ3   0 0  = 0 0 − 1
λ2
 0 λ 2 0     
0 1  0 1 0 
0
 λ 3 

University of Castilla-La Mancha Draft By: Eduardo W. V. Chaves (2016)


Ciudad Real - Spain
2 CONTINUUM KINEMATICS 265

Problem 2.66
Consider the equations of motion:
x1 = 3 X 1 ; x2 = 2 X 2 ; x3 = 3 X 3 − X 2
Obtain the material ellipsoid associated with the material sphere defined in the reference
configuration by X 12 + X 22 + X 32 = 1 , (see Figure 2.30). Check that the ellipsoid in the
x1′ 2 x 2′ 2 x3′ 2
principal space of the left stretch tensor V has the format + + = 1 , where λ 1 ,
λ21 λ22 λ23
λ 2 , λ 3 are the principal stretches.

X 2 , x2
X 3 , x3

Material surface
(always constituted by the
same particles)

X 1 , x1

Figure 2.30: Material sphere.


Solution:
The equations of motion and its inverse, in matrix form, are given by:
 3 
 0 0 
 x1   3 0 0  X 1   X1   3   x1 
      inverse    1  
 x2  = 0 2 0   X 2   → X 2  =
 0 0 x2 
x  0 −1   
3  X 3  X   2  
 3   3  3 3   x3 
 0 
 6 3 
The equations of motion in the spatial description are given by:
3 x2 3 3
X1 = x1 ; X2 = ; X3 = x2 + x3
3 2 6 3
By substituting the above equations into the sphere equation we can obtain:
2 2
 3   x2  2  3 3 
X 12 + X 22 + X 32 =1 ⇒  x + + x + =1
 3 1   2   6 2 3 3 
x
   
By simplifying the above equation we can obtain:
x12 + x 22 + x32 + x 2 x3 = 3

University of Castilla-La Mancha Draft By: Eduardo W. V. Chaves (2016)


Ciudad Real - Spain
266 SOLVING PROBLEMS BY MEANS OF CONTINUUM MECHANICS

which is the equation of an ellipsoid. We now represent the ellipsoid equation in the
principal space of the left stretch tensor V . Recall that the tensor V and b are coaxial, i.e.
they have the same principal directions), and is also true that:
V = b = F ⋅FT
The components of b are
T
 3 0 0  3 0 0 3 0 0 
   
bij =  0 2 0  0 2 0  = 0 5 − 3 
0 −1  
3 0 −1 3 0 − 3 3 
  
Note that we know already one eigenvalue b1 = 3 associated with the eigenvector
nˆ i(1) = [1 0 0] . Then, the other principal directions are in the plane x 2 − x 3 , with that we
obtain
 2 − 2  − 2 − 2
b2 = 6   → nˆ i( 2) = 0
eigenvector
 ; b3 = 2    → nˆ i(3) = 0
eigenvector

 2 2   2 2 

thus:
1 0 0 

3 0 0   
− 2 2
bij′ = 0 6 0 Transforma
  tion → aij = 0
matrix
 2 2 
0 0 2  2 2
0 
 2 2 
λ 1 = 3 0 0
 
Vij′ =  0 λ2 = 6 0 
 0 0 λ3 = 2
 
Then, applying the transformation law from x1 , x 2 , x3 -system to the x1′ , x 2′ , x3′ -system we
obtain:

 
T 
  x1 = x1′
 x1  1 0 0   x1′  
   − 2 2     − 2 2
 x2  = 0  x 2′  ⇒ x2 = x 2′ + x3′
x   2 2   x′   2 2
 3  2 2  3 
0  2 2
 2 2   x3 = x ′2 + x3′
 2 2
with that, the equation of the ellipsoid in the principal space of V , (see Figure 2.31), is
represented by:
x12 + x 22 + x32 + x 2 x3 = 3
2 2
 2 2   2 2   2 2  2 2 
(x1′ ) 2
+ −
 2 2
x′ + x3′ + 
 
x 2′ + x3′ +  −
 
x 2′ + x3′

x 2′ + x 3′ = 3

 2   2 2   2 2  2 2 
Simplifying the above equation we can obtain:

University of Castilla-La Mancha Draft By: Eduardo W. V. Chaves (2016)


Ciudad Real - Spain
2 CONTINUUM KINEMATICS 267

x1′ 2 x 2′ 2 x3′ 2 x′ 2 x′2 x′ 2 x′2 x′2 x′2


+ + = 1 2 + 2 2 + 3 2 = 12 + 22 + 32 = 1
3 6 2 ( 3) ( 6) ( 2) λ1 λ2 λ3

X 3 , x3
X 2 , x2
x3′

λ3 = 2

λ2 = 6
λ1 = 3 x 2′

x1′

X 1 , x1

Figure 2.31: The material ellipsoid (deformed configuration).

x2

x1
x3
R V

X2
x2
x3′
F = V ⋅R x1
X1
x3
X3
x 2′
x1′

Figure 2.32: The left polar decomposition.

University of Castilla-La Mancha Draft By: Eduardo W. V. Chaves (2016)


Ciudad Real - Spain
268 SOLVING PROBLEMS BY MEANS OF CONTINUUM MECHANICS

Problem 2.67
A square of side b turns counterclockwise of 30º . After turning the square is deformed
such that the base maintains its initial length and the height is doubled, (see Figure 2.33).
Calculate the deformation gradient, the right Cauchy-Green deformation tensor, and the
Green-Lagrange strain tensor.

X 2 , x2
x2′ C′

D′

D C x1′
2b 30º
B′
30º B
A = A′ b X 1 , x1

Figure 2.33: Body under rotation/deformation.


Solution:
Note that we can apply the decomposition of motion: first we apply a pure deformation
and then a rotation is applied, (see Figure 2.34). The motion is governed by the right
stretch tensor of the polar decomposition:
X 2 , x2
1 0 0
U ij = 0 2 0
0 0 1 D′′ C ′′

where we have applied the definition of


stretch. Note that they are principal
values. We then apply a rotation, where 2b D C
the components of R are the same as
r
the transformation matrix from the x ′ -
r
system to the x -system:
cos θ − sin θ 0 B = B′
R ij =  sin θ cos θ 0 b X 1 , x1
A = A′
 0 0 1 
Figure 2.34

Then, by applying the left polar decomposition, F = R ⋅ U , we can obtain:


cos θ − sin θ 0 1 0 0 cos θ − 2 sin θ 0
Fij = R ik U kj =  sin θ cos θ 0 0 2 0 =  sin θ 2 cos θ 0
 0 0 1 0 0 1  0 0 1

University of Castilla-La Mancha Draft By: Eduardo W. V. Chaves (2016)


Ciudad Real - Spain
2 CONTINUUM KINEMATICS 269

For the proposed problem, we have:


cos 30º − 2 sin 30º 0
Fij =  sin 30º 2 cos 30º 0
 0 0 1
r r r
As we are dealing with a homogenous deformation the equation x = F ⋅ X + c holds, for
r r
this case with c = 0 . For example, for a particle at point D in the reference configuration
moves to the point:
 x1D  cos 30º − 2 sin 30º 0  X 1D  cos 30º − 2 sin 30º 0 0 − 2b sin 30º 
 D   D      
 x 2  =  sin 30º 2 cos 30º 0  X 2  =  sin 30º 2 cos 30º 0 b  =  2b cos 30º 
x D   0 0 1  X 3D   0 0 1 0  0 
 3   
a fact that can be easily checked by means of Figure 2.33.
By means of definition of the right Cauchy-Green deformation tensor, C = F T ⋅ F , we can
obtain the Cartesian components:
 cos θ sin θ 0 cos θ − 2 sin θ 0 1 0 0
C ij = Fki Fkj =  − 2 sin θ 2 cos θ 0  sin θ 2 cos θ 0 = 0 4 0

 0 0 1  0 0 1 0 0 1

1
The Green-Lagrange strain tensor, E = (C + 1) , and its components are:
2
 1 0 0   1 0 0    0 0 0 
1      
E ij =  0 4 0  −  0 1 0   =  0 1.5 0 
2 
 0 0 1   0 0 1    0 0 0 
 
Note that the original space coincides with the principal space. We could also have
obtained the components of C and E by means of its spectral representations:
3 3
1
C= ∑
a =1
ˆ (a) ⊗ N
λ2a N ˆ (a) , E =
∑ 2 (λ
a =1
2
a
ˆ (a ) ⊗ N
− 1)N ˆ ( a ) , where λ are the principal stretches.
a

2.4 Infinitesimal Deformation Regime

Problem 2.68
Given the equations of motion
x1 = X 1 + 4 X 1 X 2t ; x2 = X 2 + X 22t ; x3 = X 3 + X 32t (2.154)
a) Obtain the velocity field;
b) Obtain the infinitesimal strain tensor field;
c) At time t = 1 s , obtain the infinitesimal strain tensor.

University of Castilla-La Mancha Draft By: Eduardo W. V. Chaves (2016)


Ciudad Real - Spain
270 SOLVING PROBLEMS BY MEANS OF CONTINUUM MECHANICS

Solution:
a) Velocity field:
r V1 = 4 X 1 X 2
r r dx 
V ( X ,t) = ⇒ V2 = X 22 (2.155)
dt  2
V3 = X 3
b) Acceleration field:
r  A1 = 0
r r dV 
A( X , t ) = ⇒  A2 = 0 (2.156)
dt A = 0
 3
c) Displacement field:
r r
u1 ( X , t ) = x1 ( X , t ) − X 1 = X 1 + 4 X 1 X 2 − X 1 = 4 X 1 X 2
 r r 2 2
u2 ( X , t ) = x2 ( X , t ) − X 2 = X 2 + X 2 − X 2 = X 2 (2.157)
 r r 2 2
u3 ( X , t ) = x3 ( X , t ) − X 3 = X 3 + X 3 − X 3 = X 3

1 ∂u  ∂u j 
Then, the infinitesimal strain tensor components are given by εij =  i +  , and the
2  ∂x j ∂xi 
displacement gradient can be obtained as follows:
 ∂u1 ∂u1 ∂u1 
 
r  ∂X 1 ∂X 2 ∂X 3  4 X
2 4 X1 0 
∂ui ( X , t )  ∂u2 ∂u2 ∂u2  
= = 0 2X2 0  (2.158)
∂X j ∂X ∂X 2 ∂X 3  
 1   2 X 3 
 ∂u3 ∂u3 ∂u3   0 0
 ∂X 1 ∂X 2 ∂X 3 
thus:
4 X 2 X1 0 
1  ∂u ∂u j   2
εij =  i +  = 2 X1 2X2 0  (2.159)
2  ∂x j ∂xi  
  0 0 2 X 3 

which is independent of time.
Problem 2.69
Consider the infinitesimal strain tensor:
 
0 0 0 
 2 
X X X
εij = 0 µ 2 2 3 − µ 23  (2.160)
 l l 
 X2 X2X3 
0 − µ 23 −µ 2 
 l l 
and the infinitesimal spin tensor:

University of Castilla-La Mancha Draft By: Eduardo W. V. Chaves (2016)


Ciudad Real - Spain
2 CONTINUUM KINEMATICS 271

 
0 0 0 
 µ 
ωij = 0 0
2l 2
X(2
2 − X 3 
2
) (2.161)
 
µ
 2 2
0 − 2l 2 X 2 − X 3 ( ) 0



Obtain the displacement field components.


Solution:
The displacement gradient is related to the infinitesimal strain tensor and the infinitesimal
spin tensor as follows:
u i , j = ε ij + ω ij (2.162)
where
1 1
εij = (ui , j + ui , j ) ; ωij = (ui , j − ui , j ) (2.163)
2 2
thus:
0 0 0 
µ
ui , j = 2 0 2X 2 X3 X 22− 3X 3 2
(2.164)
2l
0 − ( X 22 + X 32 ) − 2 X 2 X 3 

∂u1
=0
→ u1 = 0 (2.165)
∂x1
∂u 2 µ
= 2 (2 X 2 X 3 )
∂x 2 2l
(2.166)

⇒ ∂u 2 =
µ
∫ 2l (2 X2 2X3 )∂x 2 ⇒ u 2 = µ
2l 2
[
X 22 X 3 + C1 ( X 3 ) ]
∂u 3 µ
= − 2 (2 X 2 X 3 )
∂x3 2l
(2.167)

⇒ ∂u 3 = − ∫
µ
2
(2 X 2 X 3 )∂x3 ⇒ u 3 = − µ 2 [X 32 X 2 + C 2 ( X 2 )]
2l 2l
To determine the constant C1 ( X 3 ) from the result (2.166) we take the derivative of u 3
with respect to X 3 :

∂u 2 µ
= 2
∂X 3 2l
 2 ∂C1 ( X 3 ) 
X 2 +
∂X
µ 2 2
 = 2 X 2 − 3X 3 ⇒
∂X
[
∂C1 ( X 3 )
= −3 X 32 ]
 3  2l 3 (2.168)
⇒ C1 ( X 3 ) = − X 33

In the same fashion we find the constant C2 ( X 2 ) :


∂u 3
∂X 2
µ 
= − 2  X 32 +
2l 
∂C 2 ( X 2 ) 
∂X 2 
µ
2l
2 2
 = − 2 X2 + X3 ⇒
∂C 2 ( X 2 )
∂X 2
[= X 22 ]
(2.169)
X3
⇒ C2 ( X 2 ) = 2
3

University of Castilla-La Mancha Draft By: Eduardo W. V. Chaves (2016)


Ciudad Real - Spain
272 SOLVING PROBLEMS BY MEANS OF CONTINUUM MECHANICS

Then, the displacement field is given by:

u1 = 0 ; u 2 =
µ
2l 2
[X 2
2 X3 − X 33 ] ; u3 = −
µ 
X 2
 3 2
2l 2 
X +
X 23 
3 
 (2.170)

Problem 2.70
Show that, if we are dealing with the small deformation regime, the rate of change of the
infinitesimal strain tensor ( ε& ) is equal to the rate-of-deformation tensor ( D ).
Solution:
Consider the relationship between the rate of change of the Green-Lagrange deformation
tensor ( E& ) and the rate-of-deformation tensor ( D ):
E& = F T ⋅ D ⋅ F (2.171)
For the case of small deformation F ≈ 1 holds, in addition it fulfills that E& ≈ e& ≈ ε& then:
E& = ε& = D (2.172)

Problem 2.71
Given the equations of motion
x1 = X 1 ; x2 = X 2 + X 1 (exp −2t − 1) ; x3 = X 3 + X 1 (exp −3t − 1) (2.173)
Obtain the rate-of-deformation ( D ) and compare with the rate of change of the
infinitesimal strain tensor ( ε& ).
Solution:
By definition, the rate-of-deformation tensor ( D ) is the symmetric part of the spatial
r
velocity gradient ( l = ∇ x v ):
1
D= (l + l T ) (2.174)
2
And by definition, the infinitesimal strain tensor is equal to the symmetric part of the
displacement gradient:
r r r Dε
ε ( x , t ) = ∇ symu ≡ (∇u) sym ⇒ ε& ≡ (2.175)
Dt
r r r
The displacement field is given by u = x − X . Considering the equations of motion, the
displacement field components become:
r r
u1 ( X , t ) = x1 ( X , t ) − X 1 = X 1 − X 1 = 0
 r r − 2t − 2t
u2 ( X , t ) = x2 ( X , t ) − X 2 = X 2 + X 1 (exp − 1) − X 2 = X 1 (exp − 1)
 r r − 3t − 3t
u3 ( X , t ) = x3 ( X , t ) − X 3 = X 3 + X 1 (exp − 1) − X 3 = X 1 (exp − 1)
r r
r r Du( X , t ) 
The velocity field is given by  v ( X , t ) =  . Then, the velocity field components, in
 Dt 
material coordinates, are:
r r r
v1 ( X , t ) = 0 ; v2 ( X , t ) = X 1 (−2exp −2t ) ; v3 ( X , t ) = X 1 (−3exp −3t ) (2.176)

University of Castilla-La Mancha Draft By: Eduardo W. V. Chaves (2016)


Ciudad Real - Spain
2 CONTINUUM KINEMATICS 273

Given the inverse of the equations of motion:


 x1 = X 1  X 1 = x1
 − 2t inverse  − 2t
 x2 = X 2 + X 1 (exp − 1) →  X 2 = x2 − x1 (exp − 1) (2.177)
 − 3t  − 3t
 x3 = X 3 + X 1 (exp − 1)  X 3 = x3 − x1 (exp − 1)
we can obtain the velocity field in spatial coordinates:
r r r
v1 ( x , t ) = 0 ; v2 ( x , t ) = −2 x1exp −2t ; v3 ( x , t ) = −3x1exp −3t (2.178)
The spatial velocity gradient ( l ) components are given by:
r  0 0 0
∂vi ( x , t ) 
0 0
r
( l )ij = (∇ x v )ij = = − 2exp − 2t (2.179)
∂x j
 − 3exp − 3t 0 0
and
 0 0 0  0 0 0 
T

1 1    
(D) ij = ( l ij + l ji ) = − 2exp − 2t
0 0 +  − 2exp − 2t 0 0 
2 2  
  − 3exp −3t 0 0  − 3exp −3t 0 0 
 
 3  (2.180)
 0 − exp − 2t − exp −3t 
2
 
=  − exp − 2t 0 0 
− 3 exp −3t 0 0 
 2 

We can also obtain the spin tensor W = l skew


components as follows
 3 
 0 exp − 2t exp − 3t 
2
1  
Wij = ( l ij − l ji ) =  − exp − 2t 0 0  (2.181)
2
− 3 exp − 3t 0 0 
 2 
 
The infinitesimal strain tensor (ε )
Starting from the displacement field:
r
u1 ( x , t ) = 0
 r − 2t
u2 ( x , t ) = x1 (exp − 1) (2.182)
 r − 3t
u3 ( x , t ) = x1 (exp − 1)
the displacement gradient components can be obtained as follows:
 0 0 0
r ∂ui 
(∇u)ij = = (exp − 1) 0 0
− 2t
(2.183)
∂x j 
(exp − 3t − 1) 0 0
r
We can decompose (∇u) into a symmetric and an antisymmetric part:
r r r
(∇u)ij = (∇ symu)ij + (∇ skewu)ij = (ε )ij + (ω)ij (2.184)

The symmetric part:

University of Castilla-La Mancha Draft By: Eduardo W. V. Chaves (2016)


Ciudad Real - Spain
274 SOLVING PROBLEMS BY MEANS OF CONTINUUM MECHANICS

 0 0 0  0 0 0 
T

r 1 
(∇ symu)ij =  (exp − 2t − 1) 0 0 + (exp − 2t − 1) 0 0 
2 
  (exp − 3t − 1) 0 0  (exp − 3t − 1) 0 0 
  (2.185)
 0 exp − 1 exp − 1
− 2t − 3t

1 − 2t 
= exp − 1 0 0  = ε ij
2 − 3t 
exp − 1 0 0 
We can also provide the infinitesimal spin tensor:
 0 − (exp −2t − 1) − (exp −3t − 1)
1 
(ω)ij = (exp − 2t − 1) 0 0  (2.186)
2 − 3t 
 ( exp − 1) 0 0 
Then, the rate of change of ε is:
  0 exp −2t − 1 exp −3t − 1 
D D 1 
(ε& )ij = (ε )ij =  exp − 2t − 1 0 0 
Dt Dt  2   
 exp − 1 − 3t
0 0
  
 3  (2.187)
 0 − exp − 2t − exp − 3t 
2
 
=  − exp − 2t 0 0 
 − 3 exp − 3t 0 0 
 2 
 
with that we can conclude that:
D = ε& (2.188)

Problem 2.72
Consider a material body in a small deformation regime, which is subjected to the
following displacement field:
u1 = (−2 x1 + 7 x 2 ) × 10 −3 ; u 2 = (−10 x 2 − x1 ) × 10 −3 ; u 3 = x3 × 10 −3
a) Find the infinitesimal spin and strain tensor;
b) Find the principal invariants of the infinitesimal strain tensor, as well as the
correspondent characteristic equation;
c) Draw the Mohr’s circle in strain, and obtain the maximum shear strain;
d) Find the dilatation and the deviatoric infinitesimal strain tensor.
Solution
a) For the displacement gradient we obtain:

University of Castilla-La Mancha Draft By: Eduardo W. V. Chaves (2016)


Ciudad Real - Spain
2 CONTINUUM KINEMATICS 275

 ∂u1 ∂u1 ∂u1 


 
 ∂x1 ∂x2 ∂x3  − 2 7 0
r ∂u i  ∂u 2 ∂u 2 ∂u 2   m
(∇ u)ij = = = − 1 − 10 0  × 10 − 3 m
∂x j  ∂x1 ∂x2 ∂x3    
   1 
 ∂u 3 ∂u 3 ∂u3   0 0
 ∂x1 ∂x2 ∂x3 

In the International System of Units the displacement gradient is dimensionless, i.e.


r
[∇ur ] =  ∂ur  = m .
 ∂x  m
As for the infinitesimal spin tensor we obtain:
 0 4 0
skew r 1  ∂u ∂u j  
 = − 4 0 0 × 10− 3
ωij = (∇ u)ij =  i −
2  ∂x j ∂xi   
  0 0 0
 
Then for the infinitesimal strain tensor we have:
− 2 3 0
sym r 1  ∂u ∂u j  
 = 3 − 10 0 × 10 − 3
εij = (∇ u)ij =  i +
2  ∂x j ∂xi   
 0 0 1

b) The principal invariants are defined as I ε = Tr (ε ) , II ε =


1
2
{ }
[Tr(ε)]2 − Tr (ε 2 ) ,
III ε = det (ε ) , (see Chapter 1). Then, it follows that:

I ε = Tr (ε ) = (−2 − 10 + 1) × 10 −3 = −11 × 10 −3
−2 3 0 −2 3 0 −2 3 0
 
1
{ }
II ε = [Tr (ε )] − Tr (ε ) =  3 − 10 0 + 3 − 10 0 + 3 − 10 0  × 10 −6 = −1 × 10 −6
2
2 2

 0 0 1 0 0 1 0 0 1 

III ε = det (ε ) = 11 × 10 −9

Then, the characteristic determinant is:


− 2 × 10−3 − ε 3 × 10−3 0
−3 −3
3 × 10 − 10 × 10 − ε 0 =0
−3
0 0 1 × 10 − ε

whilst the characteristic equation is:


ε 3 − I ε ε 2 + II ε ε − III ε = 0 ⇒ ε 3 + 11 × 10 −3 ε 2 + ε × 11 × 10 −6 − 11 × 10 −9 = 0
c) To draw the Mohr’s circle for strain, (see Chaves (2013) - Appendix A), we need to
evaluate the eigenvalues of ε . But, if we take a look at the components of ε we can verify
that ε = 1 is already an eigenvalue associated with the direction nˆ i = [0 0 ± 1] . So, to
obtain the remaining eigenvalues one only need solve the following system:
− 2 × 10−3 − ε 3 × 10−3 ε = −1.0 × 10−3
=0 ⇒ ε 2 + 12 × 10 − 3 ε + 11 × 10 − 6 = 0 ⇒  1
3 × 10− 3 − 10 × 10− 3 − ε ε 2 = −11.0 × 10− 3

Then by restructuring the eigenvalues such that ε I > ε II > ε III , we obtain:

University of Castilla-La Mancha Draft By: Eduardo W. V. Chaves (2016)


Ciudad Real - Spain
276 SOLVING PROBLEMS BY MEANS OF CONTINUUM MECHANICS

ε I = 1.0 × 10 −3 ; ε II = −1.0 × 10 −3 ; ε III = −11.0 × 10 −3


Then the maximum shear (tangential) strain is evaluated as follows:
ε I − ε III
ε S max = = 6 × 10 −3
2
Finally, the Mohr’s circle for strain can be depicted as:

ε S (×10 −3 )
ε S max = 12 γ max = 6

ε II = −1

ε III = −11 ε N (×10 −3 )


εI = 1

Figure 2.35: Mohr’s circle in strain.

d) The variation of volume (dilatation) - εV , for small deformation regime, is given by:
ε V = I ε = Tr (ε ) = −12 × 10 −3
The additive decomposition of ε into a spherical and a deviatoric part is denoted by
ε = ε sph + ε dev , where the spherical part is given by:
− 4 0 0 
Tr (ε )
ε ijsph = δ ij =  0 − 4 0  × 10 −3

3
 0 0 − 4
And, the deviatoric part is given by:
 − 2 3 0 − 4 0 0   2 3 0
 
ε ijdev = ε ij − ε ijsph =   3 − 10 0 −  0 − 4 0   × 10 =  3 − 6 0 × 10 −3
   −3

 0 0 0  0 0 − 4  0 0 4




Problem 2.73
At one point of the continuum, the displacement gradient is represented by its components
as follows:
4 − 1 − 4
r
(∇u) ij = 1 − 4 2  × 10 −3 (2.189)
4 0 6 
Obtain:
a) the infinitesimal strain and spin tensors;

University of Castilla-La Mancha Draft By: Eduardo W. V. Chaves (2016)


Ciudad Real - Spain
2 CONTINUUM KINEMATICS 277

b) the components of the spherical and deviatoric parts of the infinitesimal strain tensor;
c) the principal invariants of ε : I ε , II ε , III ε ;
d) the eigenvalues and eigenvectors of the rate-of-deformation tensor.
Solution:
a) The infinitesimal strain tensor ( ε ) is the symmetric part of the displacement gradient:
r 1
2
[r r
ε = ∇ sym u = (∇u) + (∇u) T ] (2.190)

Then:
  4 − 1 − 4  4 1 4  8 0 0   4 0 0
1  1

εij =  1 − 4 2  +  − 1 − 4 0  = 0 − 8 2  = 0 − 4 1
  [×10− 3 ]
2 2
 4 0 6  − 4 2 6  0 2 12 0 1 6
r
The infinitesimal spin tensor ω = ∇ skewu
 4 − 1 − 4  4 1 4  0 − 2 − 8 0 − 1 − 4
1     1
ωij =  1 − 4 2  −  − 1 − 4 0  =  2 0 2  = 1 0 1  [ ×10 − 3 ]
2 2
 4 0 6   − 4 2 6  8 − 2 0  4 − 1 0 

b) The tensor can be additively decomposed into a spherical and deviatoric part:
ε = ε sph + ε dev (2.191)
where the spherical part is given by:
 2 0 0
Tr (ε ) 6
ε sph
= 1 = 1 = 21 ⇒ ε ijsph = 0 2 0 [×10− 3 ] (2.192)
3 3
0 0 2

The deviatoric part is given by:


 4 0 0  2 0 0   2 0 0 
εijdev = 0 − 4 1 − 0 2 0 = 0 − 6 1  [×10 − 3 ] (2.193)
0 1 6 0 0 2 0 1 4

c) The principal invariants of ε are:


I ε = Tr (ε ) = 6 [ ×10 −3 ]
−4 1 4 0 4 0
II ε = + + = −17 [×10 −3 ]2 (2.194)
1 6 0 6 0 −4
III ε = 4 × (−4) × 6 − 4 = −100 [ ×10 −3 ]3
d) The infinitesimal strain tensor components:
 4 0 0
εij = 0 − 4 1 [ ×10 − 3 ] (2.195)
0 1 6

Note that ε 1 = 4 × 10 −3 is one eigenvalue associated with the eigenvector [± 1,0,0] . To


obtain the remaining eigenvalues, we need to solve the characteristic determinant:

University of Castilla-La Mancha Draft By: Eduardo W. V. Chaves (2016)


Ciudad Real - Spain
278 SOLVING PROBLEMS BY MEANS OF CONTINUUM MECHANICS

−4−λ 1
=0 ⇒ (−4 − λ)(6 − λ) − 1 = 0 ⇒ λ2 − 2λ − 25 = 0
1 6−λ
2
− b ± b 2 − 4ac 2 ± (−2) − 4 × 1 × (−25) 2 ± 4 + 4 × 25
λ= = = = 1 ± 26
2a 2 ×1 2 (2.196)

λ 1 = 6.0990
⇒
λ 2 = −4.099
thus:
ε1 = 4 × 10 −3 ; ε 2 = 6.0990 × 10 −3 ; ε 3 = −4.099 × 10 −3 (2.197)
Restructuring we obtain:
ε I = 6.0990 × 10 −3 ; ε II = 4 × 10 −3 ; ε III = −4.099 × 10 −3 (2.198)

Problem 2.74
Obtain the infinitesimal strain tensor and the infinitesimal spin tensor for the following
displacement field:
u 1 = x12 ; u 2 = x1 x 2 ; u3 = 0
Solution:
In the small deformation regime, the infinitesimal strain tensor is given by:
1  ∂u i ∂u j 

E ijL ≈ eijL ≈ ε ij = +
2  ∂x j ∂x i 

We need to obtain the displacement gradient components:
 ∂u 1 ∂u1 ∂u1 
 
 ∂x1 ∂x 2 ∂x 3   2 x
1 0 0
∂u j  ∂u 2 ∂u 2 ∂u 2  
= = x2 x1 0
∂x k  ∂x1 ∂x 2 ∂x 3  
   0
 ∂u 3 ∂u 3 ∂u 3   0 0
 ∂x1 ∂x 2 ∂x 3 
with that we can obtain:
 x2
1  ∂u ∂u j   2 x 0 0  2 x1 x2 0   2 x1 0 
E ijL ≈ eijL ≈ ε ij =  i +  1  1  2

2  ∂x j ∂x i  =   x2 x1 0 +  0 x1 0  =  x 2 
 2  x1 0 
  0 0 0  0 0 0   2 
 0 0 0 
The infinitesimal spin tensor:
 − x2 
0 0
 2 x 0 0   2 x1 x2 0    2

1  ∂u ∂u j  1  1  x
ωij =  i −  =  x2 x1 0  −  0 x1 0   =  2 0 0
2  ∂x j ∂xi  2 2 

  0 0 0   0 0 0    
 0 0 0

University of Castilla-La Mancha Draft By: Eduardo W. V. Chaves (2016)


Ciudad Real - Spain
2 CONTINUUM KINEMATICS 279

Problem 2.75
Figure 2.36 shows the transformation experienced by the square ABCD of unit side.
a) State the equations of motion;
b) Is the theory valid for small deformation? justify the answer;
c) Is the finite deformation valid? Justify.

X 2 , x2
x2′

D
C
D′
1
45º
B C′
A = A′ θ = −45º X 1 , x1
1
B′
x1′

Figure 2.36: Body subjected to rotation.


Solution:
The transformation law between systems x ⇒ x ′ is given by:
 2 2 
 − 0
 x1′   cos θ sin θ 0  x1   x1′   2 2   x1 
      θ=−45 º    2 2  
 x2′  =  − sin θ cos θ 0  x2   → x′2  =  0  x2  (2.199)
 x′   0 2 2
 3  0 1  x3   x′  
 3   x3 
 0 0 1
 
r r
NOTE: Remember that by definition of the transformation matrix a ij from x to x ′ is
given by:
 cos( x1′ , x1 ) cos( x1′ , x 2 ) cos( x1′ , x3 )   cos(315º ) cos(225º ) cos(90º )
a ij = cos( x 2′ , x1 ) cos( x 2′ , x 2 ) cos( x 2′ , x 3 ) = cos(405º ) cos(345º ) cos(90º )
 cos( x3′ , x1 ) cos( x3′ , x 2 ) cos( x3′ , x3 )   cos(90º ) cos(90º ) cos(0º ) 

Considering the spatial and material coordinates are superimposed, the equations of
motion are defined by the inverse of the equation in (2.199):
 2 2 
 0 
 x1   2 2  X1  2 2
 x1 = X1 + X2
   2 2   
0  X 2 
2 2
 x2  =  − ⇒ 
x   2 2
  X 3   2 2
 3
 0 0 1  x 2 = − 2 X 1 + 2 X 2
 

University of Castilla-La Mancha Draft By: Eduardo W. V. Chaves (2016)


Ciudad Real - Spain
280 SOLVING PROBLEMS BY MEANS OF CONTINUUM MECHANICS

For example, the point C in the reference configuration has the material coordinates
2 2
X 1C = 1 , X 2C = 1 . After the motion we have x1C = (1) + (1) = 2 ,
2 2
2 2
x 2C = − (1) + (1) = 0
2 2
Displacement field:
 2 2  2  2
u1 = x1 − X 1 = X1 − X 2 − X 1 = X 1  − 1 − X2
 2 2  2  2

 2 2 2  2 
u 2 = x 2 − X 2 = X1 + X2 − X2 = X 1 + X 2  − 1
 2 2 2  2 
Displacement material gradient:
 ∂u1 ∂u1 ∂u1   2 2 
   −1 − 0
r  ∂X 1 ∂X 2 ∂X 3   2 2 
∂ui ( X )  ∂u2 ∂u2 ∂u2   2 2
= = − 1 0
∂X j  ∂X 1 ∂X 2 ∂X 3   2 2
 ∂u  
 3 ∂u3 ∂u3   
 ∂X 1 ∂X 2 ∂X 3   0 0 0

The infinitesimal strain tensor is given by ε = ∇ symu =


r 1
2
[ r r
]
(∇u) + (∇u)T , thus:

 2 
 −1 0 0
 2 
1  ∂u ∂u j   2
ε ij =  i + = 0 − 1 0 ≠ 0 ij
2  ∂X j ∂X i   2
 
 0 0 0
 

Note that, for a rigid body motion the strain tensors must be equal to zero, i.e. ε = 0 (the
infinitesimal strain tensor), E = 0 (the Green-Lagrange strain tensor), e = 0 (the Almansi
strain tensor). Calculating the Green-Lagrange strain tensor components we have:
0 0 0 
1  ∂u ∂u j ∂u k ∂u k  
 = 0 0 0
Eij =  i + +
2  ∂X j ∂X i ∂X i ∂X j   
 0 0 0 
 

University of Castilla-La Mancha Draft By: Eduardo W. V. Chaves (2016)


Ciudad Real - Spain
3 Stress
3.1 Force, Stress Tensor, Stress vector
Problem 3.1
Ignoring the curvature of the Earth’s surface, the gravitational field can be assumed to be
uniform as shown in Figure 3.1, where g is the acceleration caused by gravity (the gravity
of the Earth). Find the resultant force acting on the body B .

g
B

x3
x2
x1

Figure 3.1: Gravitational field.


Solution:
r
All bodies immersed in a force field are subjected to the specific body force b , and in the
special case presented in Figure 3.1 this is given by:
 0 
m
b i ( x , t ) =  0 
r
s2 
 
− g 

Hence, the total force acting on the body can be evaluated as follows:
 0 
 
r  0 
Fi = ∫ ρ b i ( x , t ) dV =
 
V

 V

 − ρ g dV 

[ m3 ]
 kg   m  } kg m
V 
m  s  ∫
We can also verify the F unit: [F] =  3   2  dV = 2 = N ( Newton ) .
s
282 SOLVING PROBLEMS BY MEANS OF CONTINUUM MECHANICS

Problem 3.2
The Cauchy stress tensor components at a
point P are given by:
 8 −4 1 
σ ij =  − 4 3 0.5 Pa x3
 1 0.5 2 
r C (0,0,5)
a) Calculate the traction vector ( t (nˆ ) ) at P
which is associated with the plane ABC n̂
defined in Figure 3.2.
b) With reference to paragraph a). B (0,2,0)
O
r r x2
Obtain the normal ( σ N ) and tangential ( σ S )
traction vectors at P , Chaves(2013)-Appendix A(3,0,0)
A. x1

Figure 3.2: Plane ABC .


Solution:
First, we obtain the unit vector which is normal to the plane ABC . To do this we choose
two vectors on the plane:
→ → → → → →
BA = OA− OB = 3eˆ 1 − 2eˆ 2 + 0eˆ 3 ; BC = OC − OB = 0eˆ 1 − 2eˆ 2 + 5eˆ 3
Then, the normal vector associated with the plane ABC is obtained by means of the cross
→ →
product between BA and BC , i.e.:
eˆ 1 eˆ 2 eˆ 3
r → →
5 = 10eˆ 1 + 15eˆ 2 + 6eˆ 3
n = BC ∧ BA = 0 −2
0 3 −2
r
Additionally, the unit vector codirectional with n is given by:
v
n 10 ˆ 15 6
nˆ = v = e 1 + eˆ 2 + eˆ 3
n 19 19 19
r
Then by using the equation t ( nˆ ) = σ ⋅ nˆ , we can obtain the traction components as:
 t1   8 − 4 1  10   t1   26 
t (i nˆ ) = σ ij nˆ j ⇒  t  = 1 − 4 3 0.5 15 Pa  1 
⇒  t 2  =  8  Pa

 2  19    19
 t 3   1 0.5 2   6   t 3   29.5
r
b) The traction vector t (nˆ ) associated with the normal n̂ can be broken down into a
r r
normal ( σ N ) and a tangential ( σ S ) vector as shown in Figure 3.3. Then,
r r r r
t (nˆ ) = σ N + σ S or t ( nˆ ) = σ N nˆ + σ S sˆ
r r
where σ N and σ S are the magnitudes of σ N and σ S , respectively.

University of Castilla-La Mancha Draft By: Eduardo W. V. Chaves (2016)


Ciudad Real - Spain
3 STRESS 283

r
t (nˆ )
x3 r ˆ
r t ( n ) = σ ⋅ nˆ
σN r r r
r t (nˆ ) = σ N + σ S
σS
r
ŝ n̂ σ N = σ N ⋅ n̂
P r ˆ 2
t ( n) = σ 2N + σ 2S
ê 3

ê 2 x2
ê1
x1

Figure 3.3: Normal and tangential stress vectors.

The normal component, σ N , can be evaluated as follows:


r ˆ
σ N = t ( n ) ⋅ nˆ = (σ ⋅ nˆ ) ⋅ nˆ = nˆ ⋅ σ ⋅ nˆ = σ : (nˆ ⊗ nˆ ) = t i( n) nˆ i = (σ ij nˆ j )nˆ i = nˆ i σ ij nˆ j = σ ij (nˆ i nˆ j )
ˆ

Thus:
10 
1
σ N = t i nˆ i ⇒ σN = 2 [26 8 29.5] 15 ≈ 1.54 Pa
19
 6 

Then the tangential component, σ S , can be obtained by means of the Pythagorean


Theorem, i.e.:
r 2
t ( nˆ ) = σ 2N + σ 2S ⇒ σ 2S = t i( nˆ ) t i( nˆ ) − σ 2N

where
 26 
1
t i( nˆ ) t i( nˆ ) = 2 [26 8 29.5]  8  ≈ 4.46
19
29.5

Thus,
ˆ ˆ
σ S = t i( n ) t i( n ) − σ 2N = 4.46 − 2.3716 ≈ 2.0884 Pa

Problem 3.3
The stress state at a point in the continuum is represented by the Cartesian components of
the Cauchy stress tensor as:
2 1 0
σ ij = 1 2 0  Pa
0 0 2 

a) Obtain the components of σ in a new system x1′ , x ′2 , x3′ , where the transformation
matrix is given by Figure 3.4.

University of Castilla-La Mancha Draft By: Eduardo W. V. Chaves (2016)


Ciudad Real - Spain
284 SOLVING PROBLEMS BY MEANS OF CONTINUUM MECHANICS

b) Obtain the principal invariants of σ ;


c) Obtain the eigenvalues and eigenvectors of σ . Also verify if the eigenvectors form a
basis transformation between the original and the principal space;
d) Illustrate the Cauchy stress tensor graphically by using the Mohr’s circle in stress, (see
Appendix A in Chaves (2013));
e) Obtain the spherical ( σ sph ) and the deviatoric ( σ dev ) part of σ . Also, find the principal
invariants of σ dev ;
f) Obtain the octahedral normal ( σ oct oct
N ) and tangential ( σ S ) components of σ .

x3
x 2′
3 0 − 4
1
a ij = A = 0 5 0 
5
4 0 3  x3′ γ1
x1′
where
ê′2
a11 = cos α 1 ê 3
ê′3
ê′1 β1
a12 = cos β 1
ê 2
a13 = cos γ 1 ê1 x2
M α1

x1
Figure 3.4

Solution:
a) The transformation law for the components of a second-order tensor is given by:
σ ′ij = a ik a jl σ kl Matrix
 form
→ σ ′ = A σ AT
Thus,
 3 0 − 4   1 1 0   3 0 4   2 0 .6 0 
1 
σ ′ij = 2 0 5 0   2 2 0   0 5 0  = 0.6 2 0.8
5
4 0 3  0 0 2   − 4 0 3   0 0.8 2 

These new components σ′ij can be appreciated in Figure 3.5.


b) The principal invariants of the Cauchy stress tensor can be calculated as follows:
I σ = Tr (σ ) = σ ii = σ11 + σ 22 + σ 33

II σ =
1
2
[ 1
] (
( Trσ ) 2 − Tr (σ 2 ) = σ ii σ jj − σ ij σ ij
2
)
2 2
= σ11σ 22 + σ11σ 33 + σ 33 σ 22 − σ12 − σ13 − σ 223

III σ = det (σ ) =  ijk σ i1σ j 2 σ k 3 =


1
6
(
σ ii σ jj σ kk − 3σ ii σ jk σ jk + 2σ ij σ jk σ ki )
= σ11σ 22 σ 33 + 2σ12 σ 23 σ13 − σ11σ 223 − σ 22 σ132
− σ 33 σ122

University of Castilla-La Mancha Draft By: Eduardo W. V. Chaves (2016)


Ciudad Real - Spain
3 STRESS 285

By substituting the values of σ ij for those in the proposed problem we can obtain:
2 0 2 0 2 1
Iσ = 6 ; II σ = + + = 11 ; III σ = 6
0 2 0 2 1 2

x3
x′2
x3′
x2
P
x1 x1′

σ ′ = A σ AT
x3

σ 33
x3′
σ′33
σ 23 σ′23 σ′23
σ 13 σ 23 σ′22 x′2
σ 13 ′
σ13
σ 12 σ 22 ′
σ12
x2 ′
σ13
σ 12

σ12
σ 11

σ 11
x1

x1′
σ = AT σ ′ A

Stress state at the point P

Figure 3.5: Basis transformation.

c) The principal stresses ( σ i ) and principal directions ( nˆ (i ) ) are obtained by solving the
following set of equations:
2 − σ 1 0   n1   0 
 1 2−σ 0  n 2  = 0

 0 0 2 − σ n 3  0

To obtain the nontrivial solutions of nˆ (i ) we have to solve the characteristic determinant,


which is a cubic equation for the unknown magnitude σ :
σij − σδ ij = 0 ⇒ σ3 − I σ σ2 + II σ σ − III σ = 0

University of Castilla-La Mancha Draft By: Eduardo W. V. Chaves (2016)


Ciudad Real - Spain
286 SOLVING PROBLEMS BY MEANS OF CONTINUUM MECHANICS

However, if we look at the format of the Cauchy stress tensor components, we can notice
that we already have one solution since in the x3 -direction the tangential components are
equal to zero, then:
σ 3 = 2 Principal
  direction
→ n1(3) = n (23) = 0 , n 3(3) = 1
To obtain the other two eigenvalues, one only need solve:
2−σ 1 σ 1 = 1
= (2 − σ ) − 1 = 0
2
⇒ 
1 2−σ σ 2 = 3
Then we can express the Cauchy stress tensor components in the principal space as:
1 0 0 
σ ′ij′ = 0 3 0  Pa
0 0 2 

Additionally, the principal direction associated with σ1 = 1 is calculated as follows:

2 − 1 1 0  n1(1)  0
 n (1)  = 0 ⇒ n1 + n 2 = 0 ⇒ n (1) = −n (1)
(1) (1)
 1 2 − 1 0
 
  2    n (1) + n (1) = 0 1 2
 0 0 2 − 1 n (31)  0  1 2

2 2
with n 3(1) = 0 and by using the condition n1(1) + n (21) = 1 we can obtain:
1  1 −1 
n1(1) = −n (21) = then nˆ i(1) =  0
2  2 2 
Since σ is a symmetric tensor, the principal space is formed by an orthogonal basis, so, it
is valid that:
nˆ (1) ∧ nˆ ( 2) = nˆ ( 3) ; nˆ ( 2 ) ∧ nˆ ( 3) = nˆ (1) ; nˆ (3) ∧ nˆ (1) = nˆ ( 2 )

Thus, the second principal direction can be obtained by the cross product between n̂ ( 3)
and n̂ (1) , i.e.:

eˆ 1 eˆ 2 eˆ 3
1 ˆ 1 ˆ
nˆ (2)
= nˆ ( 3)
∧ nˆ (1)
= 0 0 1 = e1 + e2
1 −1 2 2
0
2 2
which can also be checked by the following analysis:
The Principal direction associated with σ 2 = 3 :

2 − 3 1 0  n1( 2 )  0
 n ( 2 )  = 0 ⇒ − n1 + n 2 = 0 ⇒ n ( 2 ) = n ( 2 )
( 2) ( 2)
 1 2 − 3 0
   2    n ( 2 ) − n ( 2 ) = 0 1 2
 0 0 2 − 3 n 3( 2 )  0  1 2

2 2
With n 3(3) = 0 and using the condition n1(3) + n (23) = 1 we can obtain:
1  1 1 
n1( 2 ) = n (22 ) = then nˆ i( 2 ) =  0
2  2 2 

University of Castilla-La Mancha Draft By: Eduardo W. V. Chaves (2016)


Ciudad Real - Spain
3 STRESS 287

As we have seen in Chapter 1, the eigenvectors of a symmetric tensor form the


transformation matrix D , from the original system to the principal space, i.e.
σ ′′ = D σ D T , thus:
 1 −1   1 1 
 0  0
σ 1 = 1 0 0   2 2  2 1 0  2 2 
 0 1 1 −1 1
 σ2 = 3 0  =  0  1 2 0  
 
0
 2 2   2 2 
 0 0 σ 3 = 2   0 0 2  
   
 0 0 1   0 0 1 

d) The graphical representation of a second-order tensor can be obtained from the


description in Appendix A-Chaves(2013). To do this we have to restructure the eigenvalues
of σ so that σ I > σ II > σ III , thus:
σI = 3 ; σ II = 2 ; σ III = 1
Then the three circumferences are defined by:
1 1
Circle 1 ⇒ ; (center )C1 = (σ II + σ III ) = 1.5 ; (radius ) R1 = (σ II − σ III ) = 0.5
2 2
1 1
Circle 2 ⇒ ; (center )C 2 = (σ I + σ III ) = 2.0 ; (radius ) R 2 = (σ I − σ III ) = 1.0
2 2
1 1
Circle 3 ⇒ ; (center )C 3 = (σ I + σ II ) = 2.5 ; (radius ) R3 = (σ I − σ II ) = 0.5
2 2
Then, we can illustrate the Cauchy stress tensor at P by means of Mohr’s circle in stress as
shown in Figure 3.6.

σS
1
σ S max = 1 σ S max = (σ I − σ III ) = 1.0
2

R2

R1 R3 σN
C3
σ III = 1 C1 σ II = 2 σ I = 3 = σ N max

Figure 3.6: Mohr’s circle in stress at the point P .

NOTE: Sign convention for stress when using Mohr’s circle


When we are using scientific notation, the positive value of σ ij is when it is oriented
according to the axis, (see Figure 3.5). As we know the Mohr’s circle is the representation
of the three-dimensional stress state by means of two-dimensional graph ( σ N × σ S ), due to

University of Castilla-La Mancha Draft By: Eduardo W. V. Chaves (2016)


Ciudad Real - Spain
288 SOLVING PROBLEMS BY MEANS OF CONTINUUM MECHANICS

this fact we lost some information about stress orientation (sense), so, we need to establish
a new sign convention. Here, for Solid Mechanics, we adopt that: the normal stress is
positive ( σ N > 0 ) when it is dealing with traction (tensile), otherwise, i.e. if we are dealing
with compression σ N < 0 ; the tangential (shear) stress is positive as indicated in Figure 3.7
(a). In general, materials have different behavior when they under traction or compression
loads, but in the case of tangential stress its magnitude is only what really matters, (see
Figure 3.7(c)), for this reason, sometimes the Mohr’s circle is drawn only by considering the
positive value of σ S .

(traction) σ N > 0 (compression) σ N < 0


σN > 0
σS < 0 n̂

P σS < 0 σS > 0
σS > 0 P
− n̂

σS > 0 σS < 0 σN > 0


(a) (b) (c)
Figure 3.7: Stress sign convention.

e) A second-order tensor can be broken down additively into a spherical and a deviatoric
part, i.e.:
Tensorial notation Indicial notation

σij = σijsph + σijdev


σ = σ sph + σ dev
1 (3.1)
= σm1 + σ dev = σkkδ ij + σijdev
3
= σmδ ij + σijdev
A schematic representation of these components in the Cartesian basis can be appreciated
in Figure 3.8 and the value of the scalar σ m is evaluated as follows:
σ11 + σ 22 + σ 33 σ1 + σ 2 + σ 3 1 1 I 6
σm = = = σ kk = Tr (σ ) = σ = = 2
3 3 3 3 3 3
Then the spherical part becomes:
 2 0 0
σ ijsph = σ m δ ij = 2δ ij = 0 2 0
0 0 2

And, the deviatoric part can be evaluated as follows:


 σ11 σ12 σ13  σm 0 0
σijdev = σij − σijsph = σ12 σ22 σ23  −  0 σm 0 
σ13 σ23 σ33   0 0 σm 
 13 (2σ11 − σ22 − σ33 ) σ12 σ13 
 
= σ12 1
3
(2σ22 − σ11 − σ33 ) σ23 
 σ13 σ23 1
( 2 σ − σ − σ ) 
 3 33 11 22 

Thus,

University of Castilla-La Mancha Draft By: Eduardo W. V. Chaves (2016)


Ciudad Real - Spain
3 STRESS 289

2 − 2 1 0  0 1 0 
σ ijdev 
= 1 2−2 0  = 1 0 0
 0 0 2 − 2 0 0 0

Now let us remember from Chapter 1-Chaves(2013) that σ and σ dev are coaxial tensors,
i.e., they have the same principal directions, so we can use this information to operate in
the principal space of σ to obtain the eigenvalues of σ dev = σ − σ sph . With that we can
obtain:
σ1 0 0  σ m 0 0   − 1 0 0
σ′ijdev =  0 σ2 0 − 0
  σm 0  =  0 1 0
 0 0 σ 3   0 0 σ m   0 0 0

Then the invariants of σ dev are given by:


I σ dev = Tr (σ dev ) = 0 ; II σ dev = −1 ; III σ dev = 0

Traditionally, in engineering, the invariants of the deviatoric stress tensor are represented
by:
J1 = I σ dev = 0

J 2 = − II
σ dev
1 2
=
3
(
I σ − 3 II σ )
J 3 = III σ dev =
1
27
(
2 I σ3 − 9 I σ II σ + 27 III σ )

x3
σ 33

σ 23
σ 13 σ 23
σ 13
σ 12 σ 22
σ 12
σ 11 x2

x
14414444442444444443
x3 x3
σm dev
σ 33

σ 23
σ 13 σ 23
σ 13
σm
+ σ 12
σ 12
σ dev
22

σm x2 dev
σ 11 x2

σ sph σ dev
x1 x1

Figure 3.8: The spherical and deviatoric part of σ .

University of Castilla-La Mancha Draft By: Eduardo W. V. Chaves (2016)


Ciudad Real - Spain
290 SOLVING PROBLEMS BY MEANS OF CONTINUUM MECHANICS

f) The octahedral normal and tangential components, (see Appendix A in Chaves (2013)),
can be expressed as:

σ oct
N =
1
(σ1 + σ 2 + σ 3 ) = 1 σ ii = I σ = σ m
3 3 3

1 2 (σ1dev ) 2 + (σdev 2 dev 2


2 ) + ( σ3 )
σoct
S ≡ τoct = 2 I σ2 − 6 II σ = J2 =
3 3 3

Then, by substituting the values of the proposed problem we can obtain:


2 2
σ oct
N = σm = 6 ; τ oct = J2 =
3 3

Problem 3.4
At a point P the Cauchy stress tensor Cartesian components are given by:
1 2 3 
σ ij = 2 4 6 MPa (3.2)
3 6 1

Find:
r
a) the traction vector t related to the plane which is normal to the x1 -axis;
r
b) the traction vector t associated with the plane whose normal is (1,−1,2) ;
r
c) the traction vector t associated with the plane parallel to the plane 2 x1 − 2 x 2 − x3 = 0 ;
d) the principal stress at the point P ;
e) the principal directions of σ at the point P .

Solution:
r
Recall that the traction vector is obtained by means of the equation t ( nˆ ) = σ ⋅ nˆ which in
indicial notation becomes t i( nˆ ) = σij nˆ j .
a) In this case, the unit vector is nˆ i = [1,0,0] . Then, the traction vector is given by:
 1 2 3  1   1 
= σijn j = 2 4 6 0 = 2
ˆ
t i(n) ˆ (3.3)
3 6 1 0 3

b) The unit vector associated with the direction ni = [1,−1,2] can be obtained as follows:
1
n 1  − 1
nˆ i = ri ⇒ nˆ i = (3.4)
n 6 
 2 
r r
where the module of n is n = (1) 2 + (−1)2 + (2) 2 = 6 . Thus,

University of Castilla-La Mancha Draft By: Eduardo W. V. Chaves (2016)


Ciudad Real - Spain
3 STRESS 291

 1 2 3  1  5
ˆ 1     1 10 
t i(n) = σijnˆ j =  2 4 6  − 1 = (3.5)
6 6  
3 6 1  2   − 1

c) For this case, the vector ni = [2,−2,−1] is normal to the plane and the unit vector
associated with this direction is:
 2
n ˆni = 1  − 2
r
nˆ i = ri ⇒ where n = (2) 2 + (−2)2 + (−1)2 = 3
n 3 
 − 1 

Thus,
1 2 3  2   −5 
1 1
   
= σijnˆ j =  2 4 6 − 2 = − 10
ˆ
t i(n) (3.6)
3 3
 3 6 1  − 1   − 7 

d) The principal stresses can be obtained by solving the characteristic determinant:


1− σ 2 3
2 4−σ 6 =0 ⇒ σ3 − I σ σ2 + II σ σ − III σ = 0 (3.7)
3 6 1− σ

where I σ = 6 , I σ = −40 and III σ = 0 , thus σ3 − 6σ2 − 40σ = 0 . And the solutions are:
σ1 = 10 ; σ2 = 0 ; σ 3 = −4 (3.8)
e) The principal directions are:
Associated with σ1 = 10
− 9n1 + 2n2 + 3n3 = 0 n(21) = 2n1(1)
  3
 2n1 − 6n2 + 6n3 = 0 ⇒  (1) 5 (1) with nˆ 12 + nˆ 22 + nˆ 32 = 1 ⇒ n12 = ±
 3n + 6n − 9n = 0 n3 = n1 70
 1 2 3  3
 3
1  
⇒ ni(1) =± 6
70  
5

In the same fashion we can obtain:


− 2 1
1   1  
σ2 = 0 eigenvector
 → nˆ i( 2 ) =± 1 ; σ2 = −4 eigenvector
 → nˆ i(3) =m 2
5  14  
 0  − 3

The eigenvectors are represented by the following unit vectors:


 3  − 2 1
1   1   1  
nˆ i(1) =± 6 ; nˆ i( 2 ) =± 1 ; nˆ (i 3) =m 2
70   5  14  
5  0   − 3

Note also that the eigenvectors formed an orthogonal basis, for example:

University of Castilla-La Mancha Draft By: Eduardo W. V. Chaves (2016)


Ciudad Real - Spain
292 SOLVING PROBLEMS BY MEANS OF CONTINUUM MECHANICS

eˆ 1 eˆ 2 eˆ 3
3 6 5 1 ˆ 2 ˆ 3 ˆ
nˆ (3) = nˆ (1) ∧ nˆ ( 2 ) ⇒ nˆ (3) = =− e1 − e2 + e3
70 70 70 14 14 14
−2 1
0
5 5

Problem 3.5
r r r
Show that σ S = t (n) ⋅ (1 − nˆ ⊗ nˆ ) , where t (n) is the traction vector obtained by projecting
ˆ ˆ

r
the second-order tensor σ onto the n̂ -direction, and σ S is the tangential stress vector
associated with the plane, (see Figure 3.9).
Solution 1:
r r r r r r r
By using vector addition it is true that t ( nˆ ) = σ N + σ S ⇒ σ S = t ( nˆ ) − σ N . The vector σ N can
r r r r
be represented by σ N = σ N nˆ and the magnitud σ N = σ N nˆ can be obtained by the
r r r r r r
projection of t (nˆ ) according to n̂ -direction, i.e. σ N = t (n) ⋅ nˆ , so, σ N = σ N nˆ = (t (n) ⋅ nˆ )nˆ .
ˆ ˆ

r r r
Note that is also true that σ N = ( t (n) ⋅ nˆ )nˆ = t (n) ⋅ (nˆ ⊗ nˆ ) , so
ˆ ˆ

r rˆ rˆ
[ ]
rˆ rˆ rˆ
σ S = t (n ) − t (n) ⋅ nˆ nˆ = t (n ) − t (n) ⋅ (nˆ ⊗ nˆ ) = t (n) ⋅ (1 − nˆ ⊗ nˆ )

Solution 2:
We can also solve the problem by using the components of the following equation
r ˆ
σ S = t (n) − [σ : (nˆ ⊗ nˆ )]nˆ , i.e.:
r

[ ]
σ S i = t i(n) − (nˆ k nˆ l σ kl ) nˆ i = t i(n) − nˆ inˆ k t (kn ) = t (kn )δ ik − nˆ inˆ k t (kn) = t (kn) (δ ik − nˆ inˆ k )
ˆ ˆ ˆ ˆ ˆ

which in tensorial notation becomes


r r ˆ
σ S = t (n) ⋅ (1 − nˆ ⊗ nˆ )

r
t (nˆ )

r
σN
r
σS
ŝ n̂
P

Figure 3.9: Normal and tangential stress vectors.

Problem 3.6
The stress state at one point P of the continuum is schematically represented in Figure
3.10. Obtain the value of the component σ 22 of the Cauchy stress tensor such that there is
at least one plane passing through P in which is free of stress, and obtain the direction of
this plane.

University of Castilla-La Mancha Draft By: Eduardo W. V. Chaves (2016)


Ciudad Real - Spain
3 STRESS 293

x3

1
4
1
4 σ 22

1 1
x2

x1

Figure 3.10
Solution:
r r
We seek to find a plane whose direction is n̂ (unit vector) such that t (nˆ ) = 0 . We can relate
r
the Cauchy stress tensor to the traction vector by means of the equation t (nˆ ) = σ ⋅ nˆ , thus:
 t1(nˆ )  0 1 4  nˆ 1  0
 (nˆ )    
 t 2  = 1 σ 22 1  nˆ 2  = 0
 t (nˆ )   4 1 0  nˆ 3  0
 3  
with that we can obtain the following set of equations:
 1
n2 + 4n3 = 0 ⇒ n3 = − 4 n 2

n1 + σ 22n 2 + n3 = 0 (3.9)
 1
4n1 + n2 = 0 ⇒ n1 = − n 2
 4
By combining the above equations we can obtain:
1 1  1 1
n1 + σ 22n2 + n3 = 0 ⇒ − n2 + σ 22n2 − n2 = 0 ⇒  − + σ 22 − n 2 = 0
4 4  4 4
r r
Then, for n ≠ 0 , we have:  − + σ 22 −  = 0 ⇒ σ 22 = .
1 1 1
 4 4 2
To define the direction of the plane we will start by the restriction nˆ i nˆ i = 1 , then:
2 2
 1ˆ   1 
nˆ inˆ i = 1 ⇒ nˆ 12 + nˆ 22 + nˆ 32 = 1 ⇒  − n 2  + nˆ 2 +  − nˆ 2  = 1
2

 4   4 
2 2 2
⇒ nˆ 2 = ; nˆ 1 = nˆ 3 = −
3 6
where we have used the relationships in (3.9). Thus, the normal vector to the plane in
r r
which t (nˆ ) = 0 is the unit vector:
 − 1
2 4
nˆ i =
6  
 − 1

University of Castilla-La Mancha Draft By: Eduardo W. V. Chaves (2016)


Ciudad Real - Spain
294 SOLVING PROBLEMS BY MEANS OF CONTINUUM MECHANICS

3.2 Eigenvalues and Eigenvectors of the Stress Tensor

Problem 3.7
The Cauchy stress field components are presented by:
 1 0 2 x2 
σ ij =  0 1 4 x1  (3.10)
 2 x2 4 x1 1 

where xi are the Cartesian coordinates.


a) Obtain the traction vector acting at the point ( x1 = 1, x2 = 2, x3 = 3) associated with the
plane x1 + x 2 + x3 = 6 ;
b) Obtain the projection of the traction vector according to the normal and tangential
direction to the plane x1 + x 2 + x3 = 6 ;
Solution:
a) The unit vector which is normal to the plane x1 + x 2 + x3 = 6 is:
1 1
n n 1
ni = 1 ⇒ nˆ i = ri = i = 1
(3.11)
n 3 3 
1 1
r r ˆ
is obtained by the equation t (n) = σ ⋅ nˆ and the components of σ
ˆ
The traction vector t (n)
at the point are:
1 0 4 
σ ij ( x1 = 1, x 2 = 2, x3 = 3) = 0 1 4 (3.12)
 4 4 1 

thus,
1 0 4  1 5
ˆ   1  1  
t i(n) ˆ
= σijn j = 0 1 4 1 = 5 (3.13)
3  3 
 4 4 1  1 9

b) The normal stress associated with this plane is


1
r (nˆ ) 1
σ N = t ⋅ nˆ = [5 5 9] 1 = 1 (5 + 5 + 9) = 19
1
(3.14)
3 3 3 3
1

and the tangential stress is


r ˆ r ˆ
σ 2S = −σ 2N + t (n) ⋅ t (n) (3.15)
5
r (nˆ ) r (nˆ ) 1
t ⋅t = [5 5 9] 5 1 = 131 (3.16)
3 3 3
9

thus

University of Castilla-La Mancha Draft By: Eduardo W. V. Chaves (2016)


Ciudad Real - Spain
3 STRESS 295

2
 19  131 32
σ 2S = −  + = (3.17)
 3 3 9

Problem 3.8
Given a continuum where the stress state is known at one point and is represented by the
Cauchy stress tensor Cartesian components:
1 1 0 
σ ij = 1 1 0  Pa (3.18)
0 0 2 

a) Find the principal stresses (eigenvalues) and the principal directions (eigenvectors).
Solution:
To obtain the principal stresses λ i = σ i and principal directions nˆ (i ) we must solve the
following set of equations:
1 − λ 1 0   n1  0 
(σij − λδ ij )n j = 0i  1 1− λ 0  n2  = 0 
 (3.19)
 0 0 2 − λ  n3  0 

for nontrivial solutions of nˆ (i ) the above set of equation has solution if and only if:

σij − λδ ij = 0

Note that, a direction is called principal if there is no tangential stress on the plane normal
to such direction, and according to the format of the matrix (3.18) we can note that we
have one solution (one principal direction), since the tangential components in the x3 -
direction are zero, then:
direction
λ 3 = 2   → n1(1) = n (21) = 0 , n3(1) = ±1
To obtain the remaining solutions it is sufficient to solve:
1− λ 1
= −λ (2 − λ ) = 0
1 1− λ
We can easily verify that the roots of the above equations are:
λ1 = 2 and λ 2 = 0
Then, we can express the stress tensor components in the principal space as follows:
σ1 0 0  2 0 0
σ′ij =  0 σ2 0  = 0 0 0  Pa
 0 0 σ3  0 0 2 

Note that we have a unique eigenvalue σ 2 = 0 associated with the unique direction n̂( 2 ) ,
and we have two equal eigenvalues σ1 = σ3 = 2 , so, any direction orthogonal to n̂( 2 ) is also
a principal direction.
b) To obtain the principal direction associated with the solution λ1 = 2 we substitute this
solution into the equation in (3.19), i.e.:

University of Castilla-La Mancha Draft By: Eduardo W. V. Chaves (2016)


Ciudad Real - Spain
296 SOLVING PROBLEMS BY MEANS OF CONTINUUM MECHANICS

1 − 2 1 0  n1(1)  0 
 1 1− 2   − n1(1) + n(21) = 0
 0  n(21)  = 0  ⇒  (1) ⇒ n1(1) = n(21)
n1 − n(21) = 0
 0 0 2 − 2  n3(1)  0 

2 2 1
and by using the restriction n1(1) + n(21) = 1 we can obtain: n1(1) = n(21) = , then the unit
2
 1 1 
vector is nˆ i(1) =  0 .
 2 2 
For the solution λ 2 = 0 , we can obtain:

1 − 0 1 0  n1( 2 )  0  n1( 2 ) + n(22 ) = 0


 1 1− 0    ( 2 )
 0  n(22 )  = 0  ⇒ ( 2)
n1 + n2 = 0 ⇒ n(22 ) = −n1( 2 )
 0 0 2 − 0  n3( 2 )  0   (2)
2n3 = 0

2 2 1 −1
and by using the restriction n1( 2 ) + n(22 ) = 1 , we can obtain: n1( 2 ) = , n(22 ) = , then the
2 2
 1 −1 
unit vector is nˆ i( 2 ) =  0 .
 2 2 
As we have seen, the eigenvectors form a matrix transformation ( A ) between the two
systems, i.e. σ ′ = A σ A T , thus:
T
 1 −1   1 −1 
 0  0
 σ1 = 2 0 0   2 2  1 1 0   2 2 
 0 1 1 1 1
 σ2 = 0 0  =  0  1 1 0   0
 2 2   2 2 
 0 0 σ 3 = 2   0 0 2   
 0 0 1  0 0 1
   

x3 x3 = x3′

σ3 = 2
σ 33 = 2

x2 σ1 = 2 σ2 = 0
σ 12 σ 22 x2
σ 12 x1′ x′2
σ 11
x1

x1
any direction defined on the plane x1′ − x 3′ is a
principal direction.

Figure 3.11: Stress state for the Problem 3.8.

University of Castilla-La Mancha Draft By: Eduardo W. V. Chaves (2016)


Ciudad Real - Spain
3 STRESS 297

Problem 3.9
A prismatic dam is subjected to water pressure. The dam has thickness equal to b and
height equal to h , (see Figure 3.12). Obtain the restrictions for the Cauchy stress tensor
Cartesian components on the faces BC , OB and AC .

x2
ρ a - mass density of water
g - acceleration of gravity

B C

ρa
ρ a g (h − x 2 )
h

O b A x1

Figure 3.12: Dam under water pressure.

Solution:
The face BC has normal unit vector nˆ (i BC ) = [0 1 0] . Considering that this face has no
traction vector, we can conclude that:
 σ11 σ12 σ13  0  σ12  0
t i( BC ) = 0 i = σ ij nˆ j ⇒ σ σ 22 σ 23  1 = σ 22  = 0
 21
σ 31 σ 32 σ 33  0 σ 32  0

which is the same as σ i 2 = 0 and due to the symmetry of σ we have σ 2i = 0 .


The face OB has as normal unit vector nˆ (i BC ) = [− 1 0 0] . Considering that in this face the
traction vector components are t i(OB ) = [ρ a g (h − x 2 ) 0 0] , we can conclude that:
ρ a g ( h − x 2 )   σ11 σ12 σ13  − 1  − σ11  ρ a g (h − x 2 )
t i(OB ) =  0  = σ nˆ
 ij j ⇒ σ
 21 σ 22 σ 23   0  = − σ 21  =  0 

 0 
 σ 31 σ 32 σ 33   0   − σ 31   0 

which is the same as σ i1 = ρ a g (h − x 2 )δ i1 .


The face AC has normal unit vector nˆ (i BC ) = [1 0 0] . Considering that in this face there is
no traction vector, we can conclude that:
 σ11 σ12 σ13  1  σ11  0
t i( AC ) = 0 i = σ ij nˆ j ⇒ σ σ 22 σ 23  0 = σ 21  = 0
 21
σ 31 σ 32 σ 33  0 σ 31  0

which is the same as σi1 = 0i and due to the symmetry of σ we have σ1i = 0 .

University of Castilla-La Mancha Draft By: Eduardo W. V. Chaves (2016)


Ciudad Real - Spain
298 SOLVING PROBLEMS BY MEANS OF CONTINUUM MECHANICS

3.3 Maximum shear stress, Mohr circle in stress


Problem 3.10
Obtain the maximum shear stress at a point in which the Cauchy stress Cartesian is
represented as indicated in Figure 3.13.

x2
30 MPa

20 MPa
x1

x3

Figure 3.13

Solution:
Note that the coordinate axes xi σS (MPa)
are principal directions. We can σ S max = 15
draw the Mohr’s circle by
considering the principal stresses:
σ I = 30 MPa , σ II = 20MPa and
σ III = 0 , (see Figure 3.14).
20 30 σ N (MPa)
Figure 3.14
The maximum shear stress is calculated as follows:
σ I − σ III 30 − 0
σ S max = = = 15 MPa (3.20)
2 2

Problem 3.11
Consider the Cauchy stress Cartesian x2
components at a point as indicated in
20 MPa
Figure 3.15.
a) Draw the Mohr’s circle;
b) Obtain the maximum normal stress, 5 MPa
and indicate the plane in which occurs; x1
c) Obtain the maximum shear stress. 10 MPa

x3

Figure 3.15

University of Castilla-La Mancha Draft By: Eduardo W. V. Chaves (2016)


Ciudad Real - Spain
3 STRESS 299

Solution:
The stresses represented in Figure 3.15 are in fact the eigenvalues of the stress tensor, since
there are no tangential stresses on the planes. By restructuring the eigenvalues such that
σ I > σ II > σ III we have σ I = 10 , σ II = 5 and σ III = −20 , then the Mohr’s circle in stress can
be drawn as indicated in Figure 3.16. The maximum normal stress is σ N max = σ I = 10 , and
the maximum shear stress is defined by the radius of the circumference defined by σ I and
σ I − σ III 10 − (−20)
σ III , i.e. σ S max = = = 15MPa .
2 2

σ S (MPa)

σS max = 15
σ N max = 10MPa
10 − (−20)
σS max = = 15MPa
2

σ N (MPa)
σ III = −20 σ II = 5 σ I = 10

Figure 3.16: Mohr’s circle in stress

Problem 3.12
At a point the Cauchy stress tensor components
are defined as indicated in Figure 3.17. Determine 6
for which values of σ * are possible for the
following stress cases:
Case a) (σ N = 4; σ S = 2)
Case b) (σ N = 4; σ S = 1) σ*
P
Case c) (σ N = 7; σ S = 0)
2

Figure 3.17
Solution
Recall that the pair (σ N ; σS ) is only feasible if and only if belongs to the gray zone of the
Mohr’s circle including the circumferences, (see Figure 3.18). If the pair (σ N ; σS ) is located
outside the Mohr’s circle that means that there is no plane in which the normal stress and
tangential stress are defined simultaneously by (σ N ; σS ) .
According to the problem data, (see Figure 3.17), we can draw the circumference formed
by the principal values 2 and 6 , (see Figure 3.19). In the smae figure we also draw the
three cases.

University of Castilla-La Mancha Draft By: Eduardo W. V. Chaves (2016)


Ciudad Real - Spain
300 SOLVING PROBLEMS BY MEANS OF CONTINUUM MECHANICS

σS Feasible zone for the pair ( σ N , σ S )


There is no plane in
σ SA which
( σ NA , σ SA )

σ III σ II σ NA σI σN

Figure 3.18: Mohr’s circle in stress.

σS

Case a)

2 Case b)

1 Case c)

2 6 7 σN

Figure 3.19

Case a): In this case the pair (σ N = 4; σ S = 2) belongs to the circumference formed by the
principal stresses 2 and 6 , thus σ * can assume any value, (see Figure 3.20).

σS

Case a)

2
1

σ −∞
* 2 σ* 6 σ∞
*
σN

Figure 3.20

Case b) In this case the solution is defined by:


σ *( 2) ≤ σ * ≤ σ *(1) (3.21)

University of Castilla-La Mancha Draft By: Eduardo W. V. Chaves (2016)


Ciudad Real - Spain
3 STRESS 301

where σ *( 2) and σ *(1) are identified in Figure 3.21.

σS σS Limit cases

2 2
( 4,1) ( 4,1)
1 1

2 σ* ( x ) 6 σN 2 σ * ( 2) σ * (1) 6 σN

Figure 3.21

Starting from the circumference equation:


( x − xC ) 2 + ( y − y C ) 2 = R 2 (3.22)

(σ *(1) + 2) (σ *(1) − 2)
For the case σ *(1) , we have: x = 4; x C = ; y = 1; y C = 0; R =
2 2
Substituting these values into the circumference equation we can obtain:
2 2
 (σ* + 2)   (σ* − 2) 
2 2
( x − xC ) + ( y − yC ) = R 2
⇒  4 − (1)  + (1 − 0)2 =  (1)  ⇒ σ*(1) = 4.5
 2   2 
   
(6 + σ *( 2) ) (6 − σ *( 2) )
For the case σ *( 2) , we have: x = 4; xC = ; y = 1; y C = 0; R =
2 2
substituting these values into the circumference equation we can obtain:
2 2
 (6 + σ*( 2) )   (6 − σ*( 2) ) 
2 2
( x − xC ) + ( y − yC ) = R 2
⇒ 4 −  + (1 − 0 )2 =   ⇒ σ*( 2) = 3.5
 2   2 
   
thus:

3.5 ≤ σ * ≤ 4.5 (3.23)


Case c) In this case the only possible solution is that σ N is a principal stress, then

σ* = 7 (3.24)

University of Castilla-La Mancha Draft By: Eduardo W. V. Chaves (2016)


Ciudad Real - Spain
302 SOLVING PROBLEMS BY MEANS OF CONTINUUM MECHANICS

σS

2 6 σ* = 7 σN

Figure 3.22

Problem 3.13
Obtain the maximum normal and tangential stresses and draw the corresponding Mohr’s
circle in stress for the following stress state cases:
 τ τ 0 − 2τ 0 0   0 σ12 σ13 
a) σ ij =  τ τ 0 b) σ ij =  0 τ 0  c) σij = σ12 0 0  (3.25)
0 0 0  0 0 − τ σ13 0 0 
with τ > 0 .
Solution:
Case a) If we check the format of the Cauchy stress tensor components for this case, we
can observe that the value λ (3) = 0 is already an eigenvalue. Then, to obtain the remaining
eigenvalues, it is sufficient to solve:
τ τ τ−λ τ
τ τ → τ = (τ − λ) 2 − τ 2 = 0 ⇒ τ − λ = τ ⇒ λ = 0 (3.26)
  τ − λ
 λ (1) = 0
(τ − λ ) 2 − τ 2 = 0 ⇒ τ 2 − 2λτ + λ2 − τ 2 = 0 ⇒ λ (−2τ + λ ) = 0 ⇒  (3.27)
λ ( 2) = 2τ
Then, the Mohr’s circle in stress for this case is presented in Figure 3.23.

σS
σ N max = 2τ
σS max = τ
σ S max = τ

σ II = σ III = 0 σ I = 2τ σN

Figure 3.23

University of Castilla-La Mancha Draft By: Eduardo W. V. Chaves (2016)


Ciudad Real - Spain
3 STRESS 303

b) For the case (b) we have σ I = τ , σ II = −τ and σ III = −2τ , and the Mohr’s circle is
presented in Figure 3.24.

σS
σ N max = τ
3
σ S max = τ τ − (−2τ) 3
2 σ S max = = τ
2 2

σ III = −2τ σ II = −τ σI = τ σN

Figure 3.24

c) For the case (c) the eigenvalues can be calculated as follows:


0−λ σ12 σ13
σ12 0−λ 0 =0 ⇒ − λ3 + σ13
2 2
λ + σ12 λ=0 ⇒ λ(−λ2 + σ13
2 2
+ σ12 )=0
σ13 0 0−λ
λ1 = 0

 2 2
⇒ λ 2 = + σ13 + σ12 =τ
 2 2
λ 3 = − σ13 + σ12 = −τ

By restructuring the eigenvalues such that σ I > σ II > σ III we have σ I = τ , σ II = 0 and
σ III = − τ , then the Mohr’s circle is drawn as indicated in Figure 3.25.

σS

σ S max = τ

σ III = −τ σI = τ σN

Figure 3.25

University of Castilla-La Mancha Draft By: Eduardo W. V. Chaves (2016)


Ciudad Real - Spain
304 SOLVING PROBLEMS BY MEANS OF CONTINUUM MECHANICS

Problem 3.14
Make the representation of the Mohr’s circle for the following cases:
1) One-dimensional case (traction); 2)One-dimensional case (compression); 3) Two-
dimensional case (traction); 4) Three-dimensional case.
Solution:
1) The one-dimensional case (traction) in described in Figure 3.26.

σS
σx

σ I 0 0 σI σN
σx 0 0 0

 0 0 0

Figure 3.26

2) The one-dimensional compression is described in Figure 3.27.

σS
σx

0 0 0
0 − σ 0
 II σ II σN
σx 0 0 0

Figure 3.27

3) The two-dimensional case is described in Figure 3.28.

σ II σS
σ I 0 0
0 σ II 0

 0 0 0

σI
σ II σI σN

Figure 3.28

University of Castilla-La Mancha Draft By: Eduardo W. V. Chaves (2016)


Ciudad Real - Spain
3 STRESS 305

4) The three-dimensional case is described in Figure 3.29.

σS
σ III

σ I 0 0 
σII 0 σ II 0 

 0 0 σ III  σ III σII σI
σN
σI

Figure 3.29

3.4 Feature of the stress tensor

Problem 3.15
The Cauchy stress tensor components at the point P are given by:
5 6 7 
σ ij = 6 8 9 GPa (3.28)
7 9 2

a) Obtain the mean stress; b) obtain the deviatoric and spherical part of the tensor σ .
Solution:
a) The mean stress
σ kk 5 + 8 + 2
σm = = =5 (3.29)
3 3
b) The spherical part of σ is given by
σ m 0 0  5 0 0 

σijsph = δ ij = σmδ ij =  0 σm 0  = 0 5 0 
  
3
 0 0 σm  0 0 5

and the deviatoric part becomes:


0 6 7 
σij = σijsph + σijdev ⇒ σijdev = σij − σijsph ⇒ σijdev = 6 3 9 
7 9 − 3

University of Castilla-La Mancha Draft By: Eduardo W. V. Chaves (2016)


Ciudad Real - Spain
306 SOLVING PROBLEMS BY MEANS OF CONTINUUM MECHANICS

Problem 3.16
Consider the Cauchy stress tensor components, in the Cartesian base (eˆ 1 , eˆ 2 , eˆ 3 ) :
5 3 2
σ ij =  3 1 0  (3.30)
 2 0 3

Given the transformation law between the systems x and x' :

x'1 x' 2 x '3

3 4
x1 0
5 5

x2 0 1 0

4 3
x3 − 0
5 5

where the system x' is represented by the basis (eˆ '1 , eˆ ' 2 , eˆ ' 3 ) .
r ˆ
a) Obtain the traction vector t ( e'2 ) associated with the plane whose normal is eˆ ' 2 . Express
the result in the Cartesian system (eˆ '1 , eˆ ' 2 , eˆ ' 3 ) according to the format:
r ˆ
t (e'2 ) = ( )eˆ 1′ + ( )eˆ ′2 + ( )eˆ ′3 (3.31)
b) Obtain the spherical and deviatoric parts of the Cauchy stress tensor.
Solution:
a) Recall that the first row of the transformation matrix is formed by the direction cosines
formed between the x'1 -axis and the axes x1 , x2 and x3 , thus:
3 0 − 4
1
A = 0 5 0  (3.32)
5
4 0 3 

and the transformation law for the second-order tensor components is given by:
σ' = A σ A T (3.33)
thus:
 53 0 − 45  5 3 2  53 0 4
5

9 9 2 
   3 1 0  0  1
σ′ij = 0 1 0    1 0  = 9 5 12 (3.34)
5
4 0 3   2 0 3 − 5
4
0 35  2 12 31
5 5 

 9
( eˆ ' 2 ) 1  r ˆ 9  12 
ti = 5 ⇒ t (e'2 ) =  eˆ 1′ + (1)eˆ ′2 +  eˆ ′3 (3.35)
5 5 5
12
since:

University of Castilla-La Mancha Draft By: Eduardo W. V. Chaves (2016)


Ciudad Real - Spain
3 STRESS 307

′   t 1 ( e'1 ) 
ˆ ( eˆ ' 2 ) ( eˆ '3 )

 σ11 σ12′ σ13 t1 t1
σ ′  ( eˆ ' ) ( eˆ ' ) 
σ ′23  = t 2 1
( eˆ ' )
 21 σ ′22 t2 2 t2 3  (3.36)
σ ′33   t 3 1 t3 3 
( eˆ ' ) ( eˆ ' ) ( eˆ ' )
σ′31 σ′32 t3 2
 
b) The spherical ( σijsph ) and the deviatoric ( σijdev ) parts are given by

σij = σijsph + σijdev = δ ij + σijdev (3.37)
3
where I σ = Tr (σ ) = 5 + 1 + 3 = 9 , then
3 0 0

= δ ij = 0
σijsph 3 0 (3.38)
3
0 0 3
5 − 3 3 2  2 3 2
σijdev = σij − σij =  3 1 − 3
sph
0  = 3 − 2 0 (3.39)
 2 0 3 − 3 2 0 0

Problem 3.17
Consider the Cauchy stress tensor field Cartesian components:
 0 Cx 3 0 
σ ij = Cx 3 0 − Cx1 
 0 − Cx1 0 
where C is a constant. Also consider that the body is free of body force.
a) Calculate the traction vector at the point P (4,−4,7) associated with the plane whose
r
normal vector is given by n = 2eˆ 1 + 2eˆ 2 − 1eˆ 3 .
b) Represent the Mohr’s circle in stress at the point P .
Solution:
r
a) The traction vector can be obtained by the equation t (n) = σ ⋅ nˆ or in indicial notation
ˆ

(nˆ ) r
t i = σ ij nˆ j , where n̂ is the unit vector which is normal to the plane. The vector n has
r
module equal to n = (2) 2 + (2) 2 + (−1) 2 = 3 , thus the unit vector is given by:

2
n 1
nˆ i = ri ⇒ nˆ i =  2 
n 3
 − 1

Then, by considering the stress components at the point P(4,−4,7)


 0 Cx3 0  0 7C 0 
σ ij ( x1 = 4; x 2 = −4; x3 = 7) = Cx3 0 − Cx1  = 7C 0 − 4C  (3.40)
 0 − Cx1 0   0 − 4C 0 
we can obtain:

University of Castilla-La Mancha Draft By: Eduardo W. V. Chaves (2016)


Ciudad Real - Spain
308 SOLVING PROBLEMS BY MEANS OF CONTINUUM MECHANICS

0 7C 0  2  14C 
r (nˆ ) 1  1
t i = σ ij nˆ j = 7C 0 
− 4C   2  =  18C  (3.41)
3 3
 0 − 4C 0   − 1 − 8C 

b) Let us consider that


0 7 0
σij = C 7 0 − 4 = C σij

(3.42)
0 − 4 0 

The eigenvalues of the tensor σij can be obtained by solving the determinant:

−λ 7 0 λ1 = 0
3 2 
7 −λ −4 =0 ⇒ − λ + 16λ + 49λ = 0 ⇒ − λ + 65 = 0 ⇒ λ 2 = 65
0 −4 −λ 
λ3 = − 65
With that we can obtain σ I = C 65 , σ II = 0 and σ III = −C 65 . Then, the Mohr’s circle
can be represented as indicated in Figure 3.30.

σS

σ S max = C 65

σ III = −C 65 σ I = C 65 σN

Figure 3.30

Problem 3.18
The stress state at a point of the body is
represented by the traction vectors as x3
indicated in Figure 3.31. r ˆ
t ( e 3 ) = 8eˆ 1
a) Obtain the deviatoric part of the stress
tensor;
b) Obtain the principal stresses ( σ I , σ II ,
r ˆ
σ III ) and the principal directions; t ( e 2 ) = 6eˆ 1
ê 3
c) Draw the Mohr’s circle in stress;
ê1
d) Obtain the maximum shear stress at the ê 2 x2
point;
r ˆ
t ( e1 ) = 6eˆ 2 + 8eˆ 3
x1

Figure 3.31

University of Castilla-La Mancha Draft By: Eduardo W. V. Chaves (2016)


Ciudad Real - Spain
3 STRESS 309

e) Find the traction vector associated with the plane whose normal vector is
6 ˆ
nˆ = 0.75eˆ 1 + 0.25eˆ 2 − e3 ;
4
f) Obtain the normal and tangential stress vector associated with the plane described in
paragraph (e).
Solution:
According to Figure 3.31 we can obtain the Cauchy stress tensor components as follows:
0 6 8 
σ ij = 6 0 0
8 0 0

a)
σ ij = σ ijsph + σ ijdev


The spherical part is σ ijsph = δ ij = 0 ij since I σ = 0 . Then, the deviatoric part is given by:
3
0 6 8 
σijdev = σij − σijsph = 6 0 0
8 0 0

b) The eigenvalues can be obtained by means of the characteristic determinant:


−λ 6 8
6 −λ 0 =0 ⇒ − λ3 + 100λ = 0 ⇒ ( )
λ − λ2 + 100 = 0
8 0 −λ

whose solutions are λ 1 = 0 , λ 2 = 10 , λ 3 = −10 , (principal stresses). The principal directions are:
σ1 = 0   → nˆ i(1) = [0 − 0.8 0.6]
eigenvector

σ 2 = −10   → nˆ i( 2 ) = [− 0.707 0.424 0.566]


eigenvector

σ 3 = 10   → nˆ (i 3) = [0.707 0.424 0.566]


eigenvector

σ I = 10 , σ II = 0 , σ III = −10
c) The Mohr’s circle in stress can be appreciated in Figure 3.32.

σS

σ S max = 10

σ III = −10 σ II = 0 σ I = 10 σN

Figure 3.32

University of Castilla-La Mancha Draft By: Eduardo W. V. Chaves (2016)


Ciudad Real - Spain
310 SOLVING PROBLEMS BY MEANS OF CONTINUUM MECHANICS

d) We can directly obtain the maximum shear stress by means of the Mohr’s circle, (Figure 3.32):
σ I − σ III
τ max = = 10
2
ˆ
e) Considering t i (n) = σijnˆ j , we can obtain the traction vector associated with the plane whose
6 ˆ
normal vector is nˆ = 0.75eˆ 1 + 0.25eˆ 2 − e3 :
4
 
 t1(nˆ )  0 6 8  0.75   − 3.39898
(nˆ )  (nˆ )      
ti = σijnˆ j ⇒  t 2  = 6 0 0  0.25  ≈  4.5 
 t (nˆ )  8 0 0  6  6 
 3    − 
 4 
f) Let us consider the vectors in Figure 3.33.

r ˆ
t (n)

r
σN
r
σS
ŝ n̂
P

Figure 3.33
r r r ˆ
The magnitude of σ N can be obtained by the projection σ N = t (n) ⋅ nˆ = t i (n) nˆ i , thus:
ˆ

 
 0.75 
r
= t i nˆ i ≈ [− 3.39898 4.5 6]  0.25  ≈ −5.09847
(nˆ )
σN
 
− 6 
 4 
r
The vector σ N is given by:
r r
σ N = σ N nˆ = −3.82385eˆ 1 − 1.27462eˆ 2 + 3.12216eˆ 3
r ˆ r r
In addition, the relationship t (n) = σ N + σ S holds, with that the tangent stress vector is
obtained as follows:
r r ˆ r
σ S = t (n) − σ N ≈ (− 3.39898 + 3.82385)eˆ 1 + (4.5 + 1.27462 )eˆ 2 + (6 − 3.12216 )eˆ 3
≈ (0.42487 )eˆ 1 + (5.77462 )eˆ 2 + (2.87784 )eˆ 3
and its module as:
r
σS ≈ (0.42487 )2 + (5.77462)2 + (2.87784)2 ≈ 41.808713 = 6.465966
r r ˆ r ˆ r
= t (n) ⋅ t (n) − σ N
2 2
NOTE: We could also have used the equation σ S to obtain the
r
module of σ S .

University of Castilla-La Mancha Draft By: Eduardo W. V. Chaves (2016)


Ciudad Real - Spain
3 STRESS 311

Problem 3.19
The Cauchy stress tensor field in the continuum is represented by:
 3 x1 5 x 22 0 
r  
σ ij ( x ) = σ 21 3x 2 2 x3 
σ σ 32 0 
 31
a) Obtain the body force (per unit volume) to ensure the balance of the continuum.
b) For a particular point ( x1 = 1, x 2 = 1, x3 = 0 ):
b.1) Draw the Mohr’s circle. Obtain the maximum normal and tangential stress
component.
b.2) Obtain the traction vector associated with the plane whose normal is
 1 1 1 
nˆi =  .
 3 3 3
b.2.1) Obtain the normal and tangential in this plane.
Solution:
a) Due to the symmetry of the Cauchy stress tensor ( σ = σ T ) we have:
 3 x1 5 x 22 0 
r  2 
σ ij ( x ) = 5 x 2 3x 2 2 x3 
 0 2 x3 0 

σ + σ12, 2 + σ13,3 = −ρ b1 3 + 10 x 2 + 0 = −ρ b1
r r components  11,1 
∇ xr ⋅ σ + ρ b = 0   
→σ 21,1 + σ 22, 2 + σ 23,3 = −ρ b1 ⇒ 0 + 3 + 2 = −ρ b 2
 0 + 0 + 0 = −ρ b
σ 31,1 + σ 32, 2 + σ 33,3 = −ρ b1  3

with that we can obtain:


− 10 x 2 − 3
N
ρ b i =  − 5  (Force per unit volume)  3  (3.43)
  m 
0

b) For the particular point ( x1 = 1, x 2 = 1, x3 = 0 ) we have:


3 5 0
σ ij = 5 3 0
0 0 0

where we can verify that σ 3 = 0 is one principal value. For the other eigenvalues, it is
sufficient to solve:
3−σ 5 σ1 = 8
=0 ⇒ (3 − σ) = (5) 2 ⇒ 3 − σ = ±5 ⇒ 
5 3−σ σ 2 = −2
Restructuring the eigenvalues:
σ I = 8 , σ II = 0 , σ III = −2

University of Castilla-La Mancha Draft By: Eduardo W. V. Chaves (2016)


Ciudad Real - Spain
312 SOLVING PROBLEMS BY MEANS OF CONTINUUM MECHANICS

b.1) The Mohr’s circle in stress is drawn in Figure 3.34.

σS

σ S max = 5

σ III = −2 σ II = 0 σI = 8 σN

Figure 3.34
By means of the Mohr’s circle we can obtain the maximum shear stress σ S max = 5 and the
maximum normal stress σ N max = σ I = 10 .
ˆ
e) Considering that t i (n) = σ ij nˆ j , we can obtain the traction vector associated with the plane
1 ˆ 1 ˆ 1 ˆ
whose normal vector is nˆ = e1 + e2 + e3 :
3 3 3

 t1(nˆ )  3 5 0 1 8 
 (nˆ )  1     1  
t 2  = 5 3 0 1 = 8
 t (nˆ )  3 3 
0 0 0 1 0
 3 
b.2) The normal stress component is obtained as follows:
1
1 1
ˆ
σ N = t i ni = [8 8 0] 1 = 16 ≈ 5.333
(nˆ )

3 3 3
1
r 2 r ˆ r ˆ r
To obtain the tangential component we apply directly σ S = t (n) ⋅ t (n) − σ N
2
, where
8
r ˆ r (nˆ ) r (nˆ ) 1 1
[8 8 0] 8 = 128 . Then:
2
= t ⋅ t = ti ti =
(nˆ ) (nˆ )
t (n)
3 3 3
0

rˆ rˆ 2
r r 128  16  128 128
= t ( n ) ⋅ t (n ) − σ N
2 2
σS = −  = ⇒ σS = ≈ 3.771
3  3 9 3

Problem 3.20
The stress state at one point of the body is given by means of the spherical and deviatoric
part of the Cauchy stress tensor as follows:
1 0 0 0 6 8 
σ ijsph = 0 1 0 ; σ ijdev = 6 0 0
0 0 1 8 0 0

a) Obtain the Cauchy stress tensor components;

University of Castilla-La Mancha Draft By: Eduardo W. V. Chaves (2016)


Ciudad Real - Spain
3 STRESS 313

b) Find the principal stresses ( σ I , σ II , σ III ) and principal directions;


c) Obtain the maximum shear stress;
d) Draw the Mohr’s circle in stress for the cases: d.1) the Cauchy stress tensor ( σ ij ), d.2)
the spherical part ( σ ijsph ) and; d.3) the deviatoric part ( σ ijdev );
Solution:
1 0 0 0 6 8  1 6 8
a) σ ij = σ ijsph + σ ijdev = 0 1 0 + 6 0 0 = 6 1 0
0 0 1 8 0 0 8 0 1

In Problem 3.18 we have obtained the principal values of σ ijdev whose values are the same
as for the proposed problem. As the tensor and its deviatoric part have the same principal
directions, i.e. they are coaxial, we can automatically obtain the principal stresses:
1 0 0 10 0 0  9 0 0 
σ′ij = σ′ijsph + σ ′ijdev   
= 0 1 0 +  0 0 0  = 0 1 0 
0 0 1  0 0 − 10 0 0 11

The principal directions are the same as those provided in Problem 3.18.
d) Mohr’s circle in stress can be appreciated in Figure 3.35.

σS σS
σ S max = 10

σ dev σ dev
II = 0 σ dev = 10 σ dev σN
III = −10 I N σ I = σ II = σ III = 1

Deviatoric part Spherical part


σ = σ dev + σ sph
14444444444444442444444444444444
3

σS
σ S max = 10

σ III = −9 σ II = 1 σ I = 11 σN

Figure 3.35

University of Castilla-La Mancha Draft By: Eduardo W. V. Chaves (2016)


Ciudad Real - Spain
314 SOLVING PROBLEMS BY MEANS OF CONTINUUM MECHANICS

Note that the spherical part contribution deviates (translate) the Mohr’s circle of the
deviatoric part according the σ N -axis, and does not alter the value of the maximum shear
stress.

Problem 3.21
At one point P in the continuum medium, The Cauchy stress tensor σ is represented by
its Cartesian components as follows:
1 1 0 
σ ij = 1 1 0 MPa ,
0 0 2

a) Obtain the principal stresses and principal directions at the point P ;


b) Obtain the maximum shear stress;
c) Draw the Mohr’s circle for the cases: c.1) the Cauchy stress tensor ( σ ij ), c.2) the
spherical part ( σ ijsph ) and; c.3) the deviatoric part ( σ ijdev );
d) i.) Find the traction vector associated with the plane whose normal vector is
r
n = 1.0eˆ 1 + 1.0eˆ 2 + 0eˆ 3 ;
ii.) Obtain the normal and tangential stress on the plane.
f) Obtain the eigenvalues and eigenvectors of the deviatoric part ( σ dev ).
Solution:
a) The eigenvalues are σ I = 2 , σ II = 2 , σ III = 0 , (see Problem 3.8).
b) and c)
 2 0 0 1 0 0 1 0 0 
4 2
σ ′ijdev = σ ′ij − σ ′ijsph   
= 0 2 0 − 0 1 0 = 0 1 0 
3 3
0 0 0 0 0 1 0 0 − 2

r
d) The traction vector is obtained by t (n) = σ ⋅ nˆ , we need to normalize the normal vector
ˆ
r
ˆ n 1 ˆ 1 ˆ
to the plane, i.e. n = r = e1 + e 2 + 0eˆ 3 . Thus:
n 2 2

t 1(nˆ )  1 1 0 1   2
 (nˆ )    1   1  
t 2  = 1 1 0 1  = 2  2 
t (nˆ )  0 0 2 2 0 0
 3     

University of Castilla-La Mancha Draft By: Eduardo W. V. Chaves (2016)


Ciudad Real - Spain
3 STRESS 315

σS σS

σ S max = 1
+ σN

σ III = −1.333 σ I , σ II = 0.667 σN


σ I = σ II = σ III = 1.333

Deviatoric part Spherical part


σ = σ dev + σ sph
14444444444444442444444444444444
3

σS

σ S max = 1

σ III = 0 σ I , σ II = 2 σN

Figure 3.36

Problem 3.22
The Cauchy stress tensor components at one point of the continuum are:
 29 0 0

σ ij =  0 − 26 6  Pa
 0 6 9 

Decompose the stress tensor in a spherical and a deviatoric part, and obtain the principal
stresses and principal directions of the deviatoric part.
Solution:
Consider the additive decomposition of the stress tensor into a spherical and deviatoric
part:
σ ij = σ ijdev + σ ijsph

The deviatoric part is given by


σ11 − σ m σ12 σ13 
σ ijdev =  σ12 σ 22 − σ m σ 23 

 σ13 σ 23 σ 33 − σ m 

where the mean stress is:


1 ( 29 − 26 + 9)
σm = σ ii = =4
3 3
thus:

University of Castilla-La Mancha Draft By: Eduardo W. V. Chaves (2016)


Ciudad Real - Spain
316 SOLVING PROBLEMS BY MEANS OF CONTINUUM MECHANICS

 29 − 4 0 0   25 0 0
σ ijdev 
= 0 − 26 − 4 6  =  0 − 30 6  Pa
 
 0 6 9 − 4   0 6 5 

The spherical part components are:


4 0 0
σ ijhyd ≡ σ ijsph = 0 4 0  Pa
0 0 4

To verify the above operations, the following relationship must be verified:


25 0 0  4 0 0  29 0 0
σ ij = σ ijdev + σ ijsph =  0 − 30 6  + 0 4 0  =  0 − 26 6  Pa
     ✓
 0 6 5  0 0 4   0 6 9 

To obtain the eigenvalues we solve the characteristic determinant of the deviatoric part:
σ ijdev − λδ ij = 0 
→ λ3 − λJ 2 − J 3 = 0

By solving the above cubic equation we can obtain the following principal values:
σ1dev = 25 Pa
 dev
σ 2 = 6 Pa
σ dev = −31Pa
 3

Problem 3.23
Consider the Cauchy stress tensor components:
 12 4 0 

σij = σ 21 9 − 2 MPa
σ 31 σ32 3 
a) Obtain the spherical and the deviatoric part.
b) Obtain the principal invariants of the deviatoric part.
c) Obtain the normal octahedral stress and the mean stress at this point.
Solution:
Due to the symmetry of the Cauchy stress tensor we can conclude that:
12 4 0 
σij =  4 9 − 2 MPa

 0 − 2 3 

I σ 12 + 9 + 3 24
The mean stress is given by σ m = σ oct = = = = 8.
3 3 3
The spherical and deviatoric parts are:
8 0 0 12 4 0  8 0 0  4 4 0 
σ ijsph = 0 8 0 ; σ ijdev = σ ij − σ ijsph 
= 4   
9 − 2 − 0 8 0 =  4 1 − 2

0 0 8  0 − 2 3  0 0 8 0 − 2 − 5

University of Castilla-La Mancha Draft By: Eduardo W. V. Chaves (2016)


Ciudad Real - Spain
3 STRESS 317

The principal invariants of the deviatoric part are:


I σ dev ≡ J1 = 4 + 1 − 5 = 0 , as expected, since the trace of any deviatoric tensor is zero.

1 −2 4 0 4 4
II σ dev = + + = −41 = − J 2
−2 −5 0 −5 4 1

or by using the definition: J 2 =


3
(
1 2 1
) (
I σ − 3 II σ = 24 2 − 3 × 151 = 41
3
)
III σ dev ≡ J 3 = det (σ dev ) = 44

Problem 3.24
The stress state at one point is represented by the Cauchy stress tensor components:
1 a b
σij =  a 1 c 
b c 1

where a , b and c are constants. Determine the constants a , b and c such that the
traction vector on the octahedral plane is the null vector.
Solution:
1
The octahedral plane has the following unit vector: nˆ i = [1 1 1] , and the traction
3
r (nˆ )
vector related to this plane is defined by t = σ ⋅ nˆ , whose components are:
t1(nˆ )   1 a b  1 1 + a + b  0 a + b = −1
 (nˆ )    1  1  
t 2  =  a 1 c  1 =  a + 1 + c  = 0 ⇒ a + c = −1
t (nˆ )  b c 1 3 1 3
 b + c + 1 0 b + c = −1
 3    
−1 −1 −1
by solving the above set of equations we can obtain b = , c= , a= .
2 2 2

Problem 3.25
At one point P in the continuous medium the Cauchy stress tensor σ is represented by its
Cartesian components as follows:
 57 0 24
σ ij = σ 21 50 0  MPa ,
σ 31 σ 32 43
a) Obtain the principal stresses and principal directions at the point P ;
b) Obtain the maximum tangential and normal stress at this point;
c) Draw the Mohr’s circle in stress;
r
d) Obtain the traction vector t (n) on the octahedral plane of the Haigh-Westergaard space.
Obtain the normal octahedral stress and the tangential octahedral stress.
Solution:
Considering the symmetry of the Cauchy stress tensor we can conclude that:

University of Castilla-La Mancha Draft By: Eduardo W. V. Chaves (2016)


Ciudad Real - Spain
318 SOLVING PROBLEMS BY MEANS OF CONTINUUM MECHANICS

57 0 24
σ ij =  0 50 0  MPa
24 0 43

Note that the stress σ 22 = 50 is already a principal stress and is associated with the principal
direction nˆ ( 2) = [0 ± 1 0] . To find the other principal stresses we must solve the
following system:
57 − σ 24 σ1 = 25
=0 ⇒ σ 2 − 100σ + 1875 = 0 ⇒ 
24 43 − σ σ 3 = 75
Using the definition of eigenvalue-eigenvector, we can obtain the following eigenvectors:
σ1 = 50 ⇒ nˆ (1) = [0 ± 1.0 0]

σ1 = 25 ⇒ nˆ (1) = [m 0.6 0 ± 0.8]

σ 3 = 75 ⇒ nˆ (3) = [± 0.8 0 ± 0.6]


Mohr’s circle in stress:
Restructuring the principal stresses such that σ I > σ II > σ III we have:
σ I = 75 , σ II = 50 , σ III = 25
b, c) The Mohr’s circle can be appreciated in Figure 3.37 as well as the maximum shear stress.

σS
σ I − σ III 75 − 25
σ S max = = = 25
2 2
σ S max = 25

σ III = 25 σ II = 50 σ I = 75 = σ N max σN

Figure 3.37

d) The Haigh-Westergaard space is formed by principal stress directions, and by definition


r
the traction vector is given by t (n) = σ ⋅ nˆ . The normal vector associated with the
 1 1 1 
octahedral plane is given by nˆ i =   , then
 3 3 3

t 1(n)  75 0 0  1 75


r  (n)   0 50 0  1 1 = 1 50
t (n) = σ ⋅ nˆ components
  → t 2  =   3
t ( n )  3 
 0 0 25 1 25
 3 
and its module is given by:

University of Castilla-La Mancha Draft By: Eduardo W. V. Chaves (2016)


Ciudad Real - Spain
3 STRESS 319

( )
r
1 2 8750 r
t (n)
75 2 + 50 2 + 25 2 =
= ⇒ t (n) = 54.00617
3 3
r
The normal octahedral stress is given by σ oct = t (n) ⋅ nˆ :
1
1
σoct = [75 50 25] 1 = 50MPa
3 3
1

We could have applied directly the definition of octahedral normal stress:


Iσ 75 + 50 + 25
σoct = = σm = = 50 MPa
3 3
The tangential octahedral stress can be obtained by means of the Pythagorean theorem:
r 2 8750
τ oct = t (n) 2
− σ oct = − 50 2 = 20.4124 MPa
3
We could also have applied the definition:
1 1
τ oct = 2 I σ2 − 6 II σ = 2 × 1502 − 6 × 6875 = 20.41241MPa
3 3
where I σ = 150 , II σ = 75 × 50 + 75 × 25 + 50 × 25 = 6875 .
e) The spherical part:
50 0 0 
Tr (σ )
σijsph = δ ij = σ mδ ij =  0 50 0  MPa
3
 0 0 50

and the deviatoric part:


57 0 24 50 0 0   7 0 24 
σijdev = σ ij − σijsph =  0 50 0  −  0 50 0  =  0 0 0  MPa
24 0 43  0 0 50 24 0 − 7 

f) Considering that the tensor and its deviatoric part are coaxial tensors, we can use the
principal space in order to obtain the eigenvalues of the deviatoric part:
75 0 0  50 0 0  25 0 0 
σ′ijdev = σ′ij − σ′ijsph     
=  0 50 0  −  0 50 0  =  0 0 0  MPa
 0 0 25  0 0 50  0 0 − 25

University of Castilla-La Mancha Draft By: Eduardo W. V. Chaves (2016)


Ciudad Real - Spain
320 SOLVING PROBLEMS BY MEANS OF CONTINUUM MECHANICS

3.5 Stress state in two-dimensional case (2D)

Problem 3.26
Consider at the point P we know some stresses acting on some planes as indicated in
Figure 3.38. By considering the state of plane stress, obtain the state of plane stress at the
point σ ij .

P 4

2
y

x 6

Figure 3.38

Solution:
In the state of plane stress σ ij (i, j = 1,2) we only need two planes to define the stress state
at the point:
σx τ xy 
σij =  (3.44)
τ xy σ y 

According to Figure 3.38 we can verify that σ x = 4 , τ xy = 2 and σ y = 6 , then:

 σx τ xy  4 2
σij =  = (3.45)
 τ xy σ y  2 6

τ xy = 2

P σx = 4

y
τ xy = 2

τ xy

x σy = 6

Figure 3.39

University of Castilla-La Mancha Draft By: Eduardo W. V. Chaves (2016)


Ciudad Real - Spain
3 STRESS 321

Problem 3.27
Consider a composite material, which is made up of matrix and fiber along direction of
45 º such as shows in Figure 3.40. This composite material can break if the shear stress
along the fiber exceeds the value 3.8 × 10 6 Pa ( N / m 2 ) .
For the normal stress σ x = 2.8 × 10 6 Pa , obtain the maximum value of σ y for which the
material does not break.

σy

45º σx − 45º σx


y

σy
x

Figure 3.40: Composite material (fiber-matrix).


Solution:
We need to obtain the traction vector on the plane defined by θ = −45º , and the tangential
components can directly be obtained by means of:
σx − σy
τ ′xy ≡ τ ( θ ) = − sin 2θ + τ xy cos 2θ
2
2.8 × 10 6 − σ y
τ ′xy ≡ τ ( θ = −45 º ) =− sin( −90 º ) = 3.8 × 10 6 Pa
2
⇒ σ y ≈ −4.8 × 10 6 Pa (compression)

In Problem 1.99 was the transformation law for a second-order tensor for 2D case.

Problem 3.28
The stress acting on two planes passing through the point P are shown in Figure 3.41.
Obtain the value of the shear stress τ on the plane a − a and the principal stresses at this
point.
Solution:
To obtain the stress state at a point in the two dimensional case, we need to determine σ x ,
σ y and τ xy , (see Figure 3.42).

Considering Figure 3.42, we can directly obtain σ x and τ xy by means of the projection of
the traction vector 60 Pa , (see Figure 3.42(b)), i.e.:
σ x = 60 cos( 30 º ) = 51 .962 Pa
τ xy = 60 cos( 60 º ) = 30 Pa

University of Castilla-La Mancha Draft By: Eduardo W. V. Chaves (2016)


Ciudad Real - Spain
322 SOLVING PROBLEMS BY MEANS OF CONTINUUM MECHANICS

a b

τ
80 Pa
45 º
x
60 Pa 60 º
a

b
Figure 3.41: Stress state at one point, according to the planes a and b .

In order to obtain σ y we can employ the following equations:


σx + σ y σx − σ y
σ′x ≡ σ ( θ) ≡ σ N = + cos 2θ + τ xy sin 2θ
2 2
σx − σ y
τ′xy ≡ σ S ≡ τ ( θ) = sin 2θ − τ xy cos 2θ
2
and by substituting the numerical values we can obtain:
51.962 + σ y 51 .962 − σ y
σ ( θ = 45 º ) = + cos( 90 º ) + 30 sin( 90 º ) = 80 Pa
2 2
51 .962 − σ y
τ ( θ = 45 º ) = sin( 90 º ) − 30 cos( 90 º )
2
From the first above equations we can obtain the value of σ y : σ y = 48.038 Pa .

y y

b a b
a σy σy
τ xy τ xy

80 Pa τ xy 80 Pa τ xy
σx
45º σx 45º σx
τ τ
x x
60 º τ
60 º xy

60 Pa 60 Pa
b a b a

a) b)

Figure 3.42: Stress state at a point, according to the planes a and b .

University of Castilla-La Mancha Draft By: Eduardo W. V. Chaves (2016)


Ciudad Real - Spain
3 STRESS 323

Once σ y is determined, we can obtain the component τ (θ= 45º) :


τ ( θ= 45 º ) = 1.96 Pa

The principal stresses can be obtained by means of the components σ x , σ y , τ xy , such as


indicated in the equations:
2
σx + σy  σx − σy 
σ (1, 2 ) = ±  
 + τ 2xy
2  2 

2
51.962 + 48 .038  51.962 − 48.038  σ = 80.1Pa
σ (1, 2 ) = ±   + 30 2 ⇒  1
2  2  σ 2 = 19.9 Pa
or by means of the characteristic determinant:
σx − σ τ xy 51.962 − σ 30
=0 ⇒ =0
τ xy σy − σ 30 48.038 − σ

Problem 3.29
Given a stress state σ x = 1Pa , τ xy = −4 Pa and σ y = 2 Pa . Draw a graph of angle vs.
stresses ( θ − σ x , σ y , τ xy ), where θ is the rotation angle of the coordinate system, (see
Figure 3.43).

τ xy = −4 Pa σ y = 2 Pa
τ xy = −4 Pa
σx σ x = 1Pa
P
x

τ xy

σy

Figure 3.43: Stress state at one point.


Solution:
We can calculate the values σ ′x , σ′y , τ′xy by using the equations:
σx + σy σx − σy
σ ′x = + cos 2θ + τ xy sin 2θ
2 2
σx − σy
τ′xy = − sin 2θ + τ xy cos 2θ
2
σx + σy σy − σx
σ ′y = + cos 2θ − τ xy sin 2θ
2 2

University of Castilla-La Mancha Draft By: Eduardo W. V. Chaves (2016)


Ciudad Real - Spain
324 SOLVING PROBLEMS BY MEANS OF CONTINUUM MECHANICS

We can calculate the angle corresponding to the principal direction by means of the
equation:
2τ xy 2 × ( −4 )
tan 2θ = = = 8 ⇒ (θ = 41.437 º )
σx − σy 1− 2
and the principal stresses:
2
σx + σy  σx − σy  σ1 = 5.5311P
σ1, 2 = ±  
 + τ 2xy ⇒ 
2  2  σ 2 = −2.5311Pa
Considering the transformation law, we can obtain the values of σ ′x , σ ′y , τ′xy for different
values of θ . Making θ vary from 0 to 360 º we can represent the stresses σ ′x , σ ′y , τ′xy in
function of the angle, (see Figure 3.44). We can observe that when θ = 41 .437 º we have a
principal direction, then the tangent stress is zero ( τ xy = 0 ) and the principal stresses are
σ I = 5.5311Pa and σ II = −2.5311Pa .

x′
σ1
θ = 41.437 º
σ2

x′
θ = 131.437 º
Stresses

8 σ2

6
σ1 = 5.5311 σ ′y

σy
2
τ ′xy
σx
0
0 50 100 150 200 250 300 350 θ

-2 σ ′x
45º x′
τ xy
-4 σ 2 = −2.5311
θ = 86.437º

-6

τ max = 4.0311

Figure 3.44: Stress components in function of the angle θ .

University of Castilla-La Mancha Draft By: Eduardo W. V. Chaves (2016)


Ciudad Real - Spain
3 STRESS 325

Problem 3.30
a) Consider the stress field σ ij (i, j = 1,2) in the Cartesian system x1 − x 2 − x 3 , and the
following equations:
t t t
2 2 2


m11 = σ11 x3 dx3
−t
; ∫
m12 = σ12 x 3 dx3
−t
; ∫
m22 = σ 22 x 3 dx3
−t
2 2 2

Obtain the component transformation law of mij (i, j = 1,2) in the new system x1′ − x 2′ − x 3′
which is formed by a rotation around the x3 -axis, (see Figure 3.45).
Solution:
Due to the symmetry of σ ij = σ ji , we can conclude that m12 = m21 . The transformation
matrix from x1 − x 2 − x 3 to x1′ − x 2′ − x 3′ is given as follows:
 cosθ sin θ 0
 cosθ sin θ 
aij =  − sin θ cosθ 0 2→
D
A =
− sin θ cosθ 
 0 0 1

x3 = x3′

x2′
t
x3 =
2
x2

t θ x1′
−t
x3 =
2 x1

Figure 3.45
By using the Voigt notation, we can obtain:
t t t t
2 2 2 2

∫ ∫ ∫ ∫
′ 
 m11 ′ 
 σ11 ′ 
 σ11  σ11   σ11 
     
{m′} = m22′  = σ′22  x3′ dx3′ = σ′22  x3 dx3 = [M]σ22  x3dx3 = [M]  
σ 22  x3 dx3
 m′  σ′  σ′  σ  σ 
 12   12   12   12   12 
−t −t −t −t
2 2 2 2

with that, we can conclude that:


′ 
 m11  m11 
   
{m ′} = m22
′  = [ M ]m22  = [ M ]{m} (3.46)
 m′  m 
 12   12 

University of Castilla-La Mancha Draft By: Eduardo W. V. Chaves (2016)


Ciudad Real - Spain
326 SOLVING PROBLEMS BY MEANS OF CONTINUUM MECHANICS

where [ M ] is the transformation matrix in Voigt notation for a second-order tensor, (see
Problem 1.99), and is given by:
 a112 a12
2
2a11a12   cos 2 θ sin 2 θ 2 cos θ sin θ 
   
2a21a22  =  sin 2 θ cos θ − 2 sin θ cos θ 
2 2 2
[ M ] =  a21 a22
a a a22 a12 a11a22 + a12 a21  − sin θ cos θ cos θ sin θ cos 2 θ − sin 2 θ 
 21 11 
Also considering that [ M ]−1 = [ N ]T , we can obtain {m} = [ N ]T {m′} , where
 a112 a12
2
a11a12   cos 2 θ sin 2 θ cosθ sin θ 
   
 =  sin θ cos θ − sinθ cosθ 
2 2 2 2
[ N ] =  a21 a22 a21a22
2a a 2a22 a12 a11a22 + a12 a21  − 2 sin θ cosθ 2 cosθ sin θ cos 2 θ − sin 2 θ 
 21 11 
The same result (3.46) could have been obtained by consider mij as a second-order tensor
in two dimensional case (2D), and by means of the transformation law of a second-order
tensor we can obtain:
mij′ = aik a jl mkl ; (i, j = 1,2) or m ′ = AmA T
 m′ ′   cosθ
m12 sin θ   m11 m12  cosθ − sin θ  (3.47)
⇒  11 =

m12 ′  − sin θ
m22 cosθ   m12 m22   sin θ cosθ 

3.6 Other measures of stress

Problem 3.31
Prove that the following relationship are valid:
P = J σ dev ⋅ F −T + Jσ m F −T ; S = JF −1 ⋅ σ dev ⋅ F −T + Jσ m C −1
where P and S are the first and second Piola-Kirchhoff stress tensors, respectively, C is
the right Cauchy-Green deformation tensor, F is the deformation gradient, J is the
Jacobian determinant, and the scalar σ m is the mean normal Cauchy stress. Also show that
the following relationships are true:
P : F = S : C = 3Jσ m
Solution:
Next, we will show the equation P : F = S : C :
P : F = Pij Fij = ( Fik S kj ) Fij = S kj ( Fik Fij ) = S kj ( F T ⋅ F ) kj = S kj (C )kj = S : C

where we have the relationship P = F ⋅ S . By means of the additive decomposition of σ


by σ = σ sph + σ dev = σ m 1 + σ dev , and by considering the definition P = J σ ⋅ F −T , we can
obtain:
P = J (σ dev + σ m 1) ⋅ F −T = J σ dev ⋅ F −T + Jσ m 1 ⋅ F −T = J σ dev ⋅ F −T + Jσ m F −T

Taking into account the definition S = JF −1 ⋅ σ ⋅ F −T , and by breaking down σ into


σ = σ sph + σ dev , we can obtain:

University of Castilla-La Mancha Draft By: Eduardo W. V. Chaves (2016)


Ciudad Real - Spain
3 STRESS 327

S = JF −1 ⋅ σ ⋅ F −T S ij = JFik−1 σ kp F jp−1
= JF −1 ⋅ (σ dev + σ m 1) ⋅ F −T kp + σ ( m ) δ kp ) F jp
= JFik−1 (σ dev −1

= JF −1 ⋅ σ dev ⋅ F −T + JF −1 ⋅ σ m 1 ⋅ F −T = JFik−1 σ dev −1 −1 −1


kp F jp + JFik σ ( m ) δ kp F jp

= JF −1 ⋅ σ dev ⋅ F −T + Jσ m ( F T ⋅ F ) −1 = JFik−1 σ dev −1 −1 −1


kp F jp + Jσ ( m ) Fik F jk

= JF −1 ⋅ σ dev ⋅ F −T + Jσ m C −1 = JFik−1 σ dev −1 −1


kp F jp + Jσ ( m ) C ij

Then by applying the double scalar product between S and C we can obtain:
S : C = ( JF −1 ⋅ σ dev ⋅ F −T + Jσ m C −1 ) : C = JF −1 ⋅ σ dev ⋅ F −T : C + Jσ m C −1 : C

where the term JF −1 ⋅ σ dev ⋅ F −T : C becomes:


JF −1 ⋅ σ dev ⋅ F −T : C = ( JF −1 ⋅ σ dev ⋅ F −T ) : {
C
F T ⋅F

( JF −1 ⋅ σ dev ⋅ F −T ) ij ( F T ⋅ F ) ij = ( Fip−1σ dev −1


pk F jk )( Fqi Fqj )

= J δ qp δ qk σ dev
pk

pk δ pk = J σ kk
= J σ dev dev

dev
=J σ
1
424 :1
3 =0
Tr (σ dev ) =0

Thus:
S : C = Jσ m C −1 : C = Jσ m Tr (C −1 ⋅ C ) = Jσ m Tr (1) = 3 Jσ m
Now, by taking the double scalar product between P and F we obtain:
P : F = J σ dev ⋅ F −T : F + Jσ m F −T : F

Then by analyzing the term J σ dev ⋅ F −T : F we can conclude that:


J σ dev ⋅ F −T : F = ( J σ dev ⋅ F −T ) ij ( F ) ij = J σ ikdev F jk−1 Fij = Jσ ikdev δ ik = J σ
1
dev
424 :1 = 0
3
Tr (σ dev ) =0

Thus,
P : F = Jσ m F −T : F = Jσ m Tr ( F −T ⋅ F T ) = Jσ m Tr (1) = 3 Jσ m

University of Castilla-La Mancha Draft By: Eduardo W. V. Chaves (2016)


Ciudad Real - Spain
328 SOLVING PROBLEMS BY MEANS OF CONTINUUM MECHANICS

University of Castilla-La Mancha Draft By: Eduardo W. V. Chaves (2016)


Ciudad Real - Spain
4 The Fundamental
Equations of Continuum
Mechanics

Problem 4.1
Show that Reynolds’ transport theorem is valid in the following equation:
D D
Dt V∫Φ dV =
Dt V ∫
Φ JdV 0 (4.1)
0

where V is the volume in the current configuration, V0 is the volume in the reference
configuration, J is the Jacobian determinant and Φ is a scalar field that describes the
physical quantity of a particle per unit volume at time t .
Solution:
D  DΦ DJ   DΦ r  DΦ r
∫ ∫
Φ JdV0 =  J +Φ  dV0 =  J ∫ + JΦ ∇ xr ⋅ v  dV0 =  + Φ ∇ xr ⋅ v dV
∫ (4.2)
Dt V V0 
Dt Dt  V0 
Dt  V
Dt 
0

Problem 4.2
Show that
r
D r DPijL ( x , t )
Dt V∫ V

ρ PijL ( x , t ) dV = ρ
Dt
dV (4.3)

r
where PijL ( x , t ) is a continuum property per unit mass, which can be a scalar, a vector or
higher order tensor.
Solution:
It was proven in the textbook, (Chaves (2013)), that:
D r D r r ∂v p 

Dt V ∫
Φ ( x , t )dV =  Φ ( x , t ) + Φ ( x , t )
 Dt
V 
dV
∂x p 

Then by making Φ = ρ PijL , and by considering it in the above equation we can obtain:
330 SOLVING PROBLEMS BY MEANS OF CONTINUUM MECHANICS

D D ∂v p   D Dρ ∂v 
Dt V∫  Dt
V 

ρ PijK dV =  (ρ PijK ) + ρ PijK  dV = ρ
∂x p  V 
∫ Dt
PijK + PijK
Dt
+ ρ PijK k
∂x k
dV

 D  Dρ ∂v  
=  ρ
V 
∫Dt
PijK + PijK 
 1Dt
+ρ k
∂x k


 dV

4243
=0
mass continuity equation

Thus, we can conclude that:

D  DP 
Dt V∫ V 

ρ PijK dV = ρ ijK  dV
Dt 

Problem 4.3
Prove that the following relationship is valid:
r ∂ r r r
ρa= ( ρ v ) + ∇ xr ⋅ ( ρ v ⊗ v ) (4.4)
∂t
Solution:
Based on the Reynolds’ transport theorem:
D ∂Φ r
Φ dV = dV + Φ (v ⋅ nˆ ) dS
∫ ∫ ∫
Dt V V
∂t S
r
and if we consider that Φ = ρ v we can obtain:
r
D r ∂ (ρ v ) r r
∫ρ v dV = ∫ dV + ρ v ⊗ (v ⋅ nˆ ) dS

Dt V V
∂t S

The above equation in indicial notation becomes:


D ∂ (ρ v i ) D ∂ (ρ v i )
Dt V ∫
ρ vi dV =
V
∫∂ t S

dV + ρ v i (v k nˆ k ) dS ⇒ ρ
V
Dt
12
v i dV =
3 V
∫ ∂t S

dV + (ρ v i v k )nˆ k dS ∫
= ai

Additionally, by applying the divergence theorem to the surface integral we can obtain:
∂ (ρ v i )  ∂ (ρ v i ) 
∫ρ a
V
i dV = ∫
V
∂t V

dV + (ρ vi v k ) ,k dV = 
V 
∂t ∫ + (ρ v i v k ) ,k  dV

which in tensorial notation is:
r r
r  ∂ (ρ v ) r r  r ∂ (ρ v ) r r
∫V ρ a dV = V∫  ∂t + ∇ xr ⋅ (ρ v ⊗ v ) dV ⇒ ρa=
∂t
+ ∇ xr ⋅ (ρ v ⊗ v )

Problem 4.4
Let us consider the following velocity field:
xi
vi = for t ≥ 0
1+ t
a) Find the mass density field;
b) Show that for this motion the equation ρ x1 x 2 x3 = ρ 0 X 1 X 2 X 3 is satisfied.

University of Castilla-La Mancha Draft By: Eduardo W. V. Chaves (2016)


Ciudad Real - Spain
4 THE FUNDAMENTAL EQUATIONS OF CONTINUUM MECHANICS 331

Solution:
a) By applying the mass continuity equation we can obtain:
Dρ ∂v k Dρ dρ ∂v
+ρ =0 ⇒ ≡ = −ρ k
Dt ∂x k Dt dt ∂x k
and by using the given velocity field, we can find that:
∂v i 1 ∂x i δ 3
= = ii =
∂x i 1 + t ∂x i 1 + t 1 + t
Thus,
dρ 3ρ dρ 3dt
=− ⇒ =−
dt 1+ t ρ 1+ t
Then by integrating the both sides of the above equation we can obtain:
dρ 3dt
∫ ρ
= −∫ 1+ t
⇒ Ln ρ = −3Ln(1 + t ) + C

The constant of integration C is obtained by applying the initial condition at t = 0 , in


r
which ρ ( x , t = 0) = ρ 0 holds, thus
Ln ρ 0 = −3 Ln(1 + 0) + C ⇒ C = Ln ρ 0

 1   ρ0 
Ln ρ = −3 Ln(1 + t ) + Ln ρ 0 = Ln  + Ln ρ 0 = Ln
3 

3 
 (1 + t )   (1 + t ) 
Thus, we can conclude that:
ρ0
ρ=
(1 + t )3
b) By using the velocity definition we can obtain:
dx i x dx i dt
vi = = i ⇒ =
dt 1 + t xi 1 + t
Additionally, by integrating the both sides of the above equation we can obtain:
dx i dt
∫ xi
= ∫
1+ t
⇒ Lnxi = Ln(1 + t ) + K i (4.5)

Then, by applying the initial condition, i.e. at time t = 0 ⇒ xi = X i , we can obtain:


Ln X i = Ln(1 + 0) + K i ⇒ K i = Ln X i
And by substituting the value of K i into the equation (4.5) we can obtain:
Ln xi = Ln(1 + t ) + Ln X i ⇒ Ln( xi ) = Ln[ X i (1 + t )]
Hence we can conclude that x i = X i (1 + t ) , which gives us x1 = X 1 (1 + t ) , x 2 = X 2 (1 + t ) ,
ρ0
x 3 = X 3 (1 + t ) , and if we consider that ρ = , we can obtain:
3
(1 + t )
ρ (1
12t )(1
+3 t )(1
+3
12 t)= ρ 0
+3
12 ⇒ ρ x1 x 2 x 3 = ρ 0 X 1 X 2 X 3
x x x
= 1 = 2 = 3
X1 X2 X3

University of Castilla-La Mancha Draft By: Eduardo W. V. Chaves (2016)


Ciudad Real - Spain
332 SOLVING PROBLEMS BY MEANS OF CONTINUUM MECHANICS

Problem 4.5
The equations of motion of a body are given, in Lagrangian description, by:
 x1 = X 1 + αtX 3

 x 2 = X 2 + αtX 3
 x = X − αt ( X + X )
 3 3 1 2

where α is a constant scalar. Find the mass density in the current configuration (ρ ) in
terms of the mass density of the reference configuration (ρ 0 ) , i.e. ρ = ρ (ρ 0 ) .
Solution:
We can apply the equation ρ 0 = Jρ , where J is the Jacobian determinant and is given by:
∂x1 ∂x1 ∂x1
∂X 1 ∂X 2 ∂X 3
1 0 αt
∂x i ∂x 2 ∂x 2 ∂x 2
J= F = = = 0 1 αt = 1 + 2(αt ) 2
∂X j ∂X 1 ∂X 2 ∂X 3
∂x 3 ∂x 3 ∂x 3 − αt − αt 1
∂X 1 ∂X 2 ∂X 3

ρ0 ρ0
Thus, we can obtain ρ = =
J 1 + 2(αt ) 2
Problem 4.6
Given the velocity field components:
v1 = ax1 − bx 2 ; v 2 = bx1 − ax 2 ; v3 = c x12 + x 22
where a , b and c are constants.
a) Check whether the mass continuity equation is fulfilled or not;
b) Is the motion isochoric?
Solution:
The mass continuity equation:
Dρ r
+ ρ (∇ xr ⋅ v ) = 0
Dt
where:
r
∇ xr ⋅ v = v i ,i = v1,1 + v 2, 2 + v 3,3 = a − a + 0 = 0
r Dρ
The motion is isochoric (incompressible medium), since ∇ xr ⋅ v = 0 or =0.
Dt

Problem 4.7
r
Consider a continuous medium and an Eulerian property φ ( x , t ) assigned by density, i.e.
unit of the property per unit volume. Obtain the rate of change of the property by
considering the control volume and the control surface.
Solution:
Remember that the rate of change of a property is always associated with the same
particles. By means of the material time derivative we can obtain the rate of change of a

University of Castilla-La Mancha Draft By: Eduardo W. V. Chaves (2016)


Ciudad Real - Spain
4 THE FUNDAMENTAL EQUATIONS OF CONTINUUM MECHANICS 333

property when this property is in Eulerian description. Then, the total rate of change of
r
φ ( x , t ) in the volume V which is bounded by the surface S is given by:


φ ( x, t ) + φ ( x, t ) (dV )
D r D D r r D
Dt V∫φ ( x, t )dV =
V

Dt
(φ dV ) = dV
V 
Dt ∫ Dt 

φ ( x, t ) + φ ( x, t )(∇ xr ⋅ v ) dV 
D r r r

= dV
V 
Dt 
(4.6)

D r r r 
=  φ ( x , t ) + φ ( x , t )(∇ xr ⋅ v )  dV

V 
Dt 
D r
We apply the definition of the material time derivative to φ( x , t ) :
Dt

φ ( x, t )dV =  φ ( x, t ) + φ ( x, t )(∇ xr ⋅ v )  dV
D r D r r r

Dt V V 

Dt 
r
∂ r ∂φ ( x , t ) r r r r 

=  φ ( x, t ) + r ⋅ v ( x , t ) + φ ( x , t )(∇ xr ⋅ v )  dV
V 
∂t ∂x 
r (4.7)
∂ r   ∂φ ( x , t ) r r r 

=  φ ( x , t )  dV +  ∫ r ⋅ v + φ ( x , t )(∇ xr ⋅ v )  dV
V 
∂t  V 
∂x 
∂ r  r
=  φ ( x , t )  dV + [∇ xr ⋅ (φv ) ]dV
∫ ∫
V 
∂t  V

We can apply the divergence theorem to the second integral on the right side of the
equation to obtain:
flux of φ through
suface S
6 44744 8
r
D r ∂φ( x , t ) r (4.8)
∫φ( x, t )dV = ∫ dV + ({ φv ) ⋅ nˆ dS

Dt V V 1
∂t4
42 3 S flux of φ
local
r
∂φ( x , t )
the term is local, the volume integral of the right side of the equation is a control
∂t
r
volume and the integral surface is a control surface, since the variable (φv ) is in Eulerian
r
description. The term (φv ) represents the flux of the property φ .

D r
When there is no source or sink of the property it is true that
Dt V∫φ( x, t )dV = 0 . And, note

also that when the property is the mass density ( φ = ρ ) the equation in (4.7) becomes the
mass continuity equation.
D r D r r r  ∂ r r 
ρ ( x, t )dV =  ρ ( x, t ) + ρ ( x, t )(∇ xr ⋅ v )  dV = 0 =  ρ ( x, t ) + ∇ xr ⋅ ( ρv )  dV = 0 (4.9)
∫ ∫ ∫
Dt V V 
Dt  V 
∂t 
If the above equation is valid for the entire volume then it is valid locally, so
D r r r
ρ ( x, t ) + ρ ( x, t )(∇ xr ⋅ v ) = 0 Mass continuity equation (4.10)
Dt
or

University of Castilla-La Mancha Draft By: Eduardo W. V. Chaves (2016)


Ciudad Real - Spain
334 SOLVING PROBLEMS BY MEANS OF CONTINUUM MECHANICS

∂ r r
ρ ( x, t ) + ∇ xr ⋅ (ρ v ) = 0 Mass continuity equation (4.11)
∂t

control volume r
(φv )
S
r r
V q n = [(φv ) ⋅ nˆ ] nˆ

r
r ∂φ( x , t )
x
∂t
control surface

Figure 4.1: Control volume and control surface.

material volume control volume control surface

t=0

v0

XP
X* Particle P

control surface
material volume control volume
t1

r
v( x * , t1 )

xP Particle Q
x*
control surface
material volume control volume
t2

r
v( x * , t 2 )

xP x*
Particle R

Figure 4.2: Material volume vs. control volume.

University of Castilla-La Mancha Draft By: Eduardo W. V. Chaves (2016)


Ciudad Real - Spain
4 THE FUNDAMENTAL EQUATIONS OF CONTINUUM MECHANICS 335

Problem 4.8
Show that the following equation is true:
Tensorial notation Indicial notation
∂φ r ∂ r ∂φ ∂
ρ + ρ (∇ xrφ ) ⋅ v = ( ρφ ) + ∇ xr ⋅ ( ρφv ) ρ + ρφ ,i vi = ( ρφ ) + ( ρφvi ) ,i (4.12)
∂t ∂t ∂t ∂t
r r r r
where φ ( x , t ) is a scalar field, ρ ( x , t ) is the mass density field, and v ( x , t ) is the velocity
field.
Solution:
∂ ∂φ ∂ρ ∂φ  ∂ρ 
(φρ ) + (φρv i ) ,i = ρ +φ + φ , i ( ρv i ) + φ ( ρv i ) , i = ρ + φ , i ρv i + φ  + ( ρv i ) , i 
∂t ∂t ∂t ∂t  ∂t 
∂φ
=ρ + φ , i ρv i
∂t
∂ρ
where we have considered the mass continuity equation + ( ρvi ) ,i = 0 .
∂t

4.1 Equations of Motion. Equilibrium Equations


Problem 4.9
Find the equilibrium equations by means of the differential volume element equilibrium
( dx dy dz ). For this purpose consider that the Cauchy stress tensor field in the differential
volume element varies as indicated in Figure 4.3. Adopt the engineering notation.
Solution:
To obtain the equilibrium equations we will apply the force equilibrium condition in the
volume element. First, we evaluate the equilibrium force according to the x -direction:

∑F x =0

 ∂σ x   ∂τ xy 
ρ b x dxdydz +  σ x + dx  dydz − σ x dydz +  τ xy + dy dxdz
 ∂x   ∂y 
 ∂ τ 
− τ xy dxdz +  τ xz + xz dz  dxdy − τ xz dxdy = 0
 ∂ z 

Then, by simplifying the above equation we can obtain:


∂σ x ∂τ xy ∂τ
ρ b x dxdydz + dxdydz + dxdydz + xz dxdydz = 0
∂x ∂y ∂z

∂σ x ∂τ xy ∂τ xz
⇒ ρb x + + + =0
∂x ∂y ∂z

University of Castilla-La Mancha Draft By: Eduardo W. V. Chaves (2016)


Ciudad Real - Spain
336 SOLVING PROBLEMS BY MEANS OF CONTINUUM MECHANICS

z Rear face
σx
τ xy
∂σ z
σz + dz
Rear face ∂z
∂σ yz
σ yz + dz τ xz
∂z
∂τ xz
τ xz + dz
∂z ∂τ yz dz
τ yz + dy
τ xy ρb z ∂y
σy ρb y ∂σ y
∂τ xz σy + dy
τ xz + dx ∂y
∂x ρb x ∂τ xy
τ xy + dy y
∂y
τ yz ∂τ xy
τ xy + dx dx
∂σ ∂x
σ x + x dx
∂x

τ xz

x τ yz

Rear face
σz

dy

Figure 4.3: The stress field in the differential volume element.

The equilibrium force according to the y -direction, ∑ Fy = 0 , can be expressed as follows


 ∂σ y   ∂τ yz 
ρb y dxdydz +  σ 22 + dy dxdz − σ y dxdz +  τ yz + dz  dxdy
 ∂y   ∂z 
 ∂τ xy 
− τ yz dxdy +  τ xy + dx  dydz − τ xy dydz = 0
 ∂x 
Then by simplifying the above equation we can obtain:
∂τ xy ∂σ y ∂τ yz
ρb y + + + =0
∂x ∂y ∂x z

Finally, the equilibrium according to the z -direction, ∑ Fz = 0 , is given by:

University of Castilla-La Mancha Draft By: Eduardo W. V. Chaves (2016)


Ciudad Real - Spain
4 THE FUNDAMENTAL EQUATIONS OF CONTINUUM MECHANICS 337

 ∂σ z   ∂τ 
ρ b z dxdydz +  σ z + dz  dxdy − σ z dxdy +  τ xz + xz dx  dzdy
 ∂z   ∂x 
 ∂τ yz 
− τ xz dzdy +  τ yz + dy  dxdz − τ yz dxdz = 0
 ∂y 
and after the simplification is taken place the above equation becomes:
∂τ xz ∂τ yz ∂σ z
ρb z + + + =0
∂x ∂y ∂z
Then, the equilibrium equations in engineering notation become:
 ∂σ x ∂τ xy ∂τ xz
 + + + ρb x = 0
 ∂x ∂y ∂z
 ∂τ xy ∂σ y ∂τ yz
 + + + ρb y = 0
 ∂x ∂y ∂x z
 ∂τ ∂τ ∂σ z
 xz + yz + + ρb z = 0
 ∂x ∂y ∂z

Problem 4.10
Let σ be the Cauchy stress tensor field, which is represented by its components in the
Cartesian basis as:
σ11 = x12 ; σ 22 = x 22 ; σ 33 = x12 + x 22
σ12 = σ 21 = 2 x1 x 2 ; σ 23 = σ 32 = σ 31 = σ13 = 0
Considering that the body is in equilibrium, find the body forces acting on the continuum.
Solution:
r r
By applying the equilibrium equations, ∇ xr ⋅ σ + ρ b = 0 , we can obtain:
 ∂σ 11 ∂σ12 ∂σ13
 + + + ρ b1 = 0
 ∂x1 ∂x 2 ∂x 3
2 x1 + 2 x1 + ρ b 1 = 0
 ∂σ 21 ∂σ 22 ∂σ 23 
σ ij , j + ρ b i = 0 i ⇒  + + + ρb2 = 0 ⇒ 2 x 2 + 2 x 2 + ρ b 2 = 0
 ∂x1 ∂x 2 ∂x 3 ρ b = 0
 ∂σ 31 ∂σ 32 ∂σ 33  3
 + + + ρb3 = 0
 ∂x1 ∂x 2 ∂x 3

Thus, to satisfy the equilibrium equations the following condition must be met:
4 x1 = − ρ b 1 ⇒ ρ b 1 = − 4 x 1 
 r N 
4 x 2 = − ρ b 2 ⇒ ρ b 2 = −4 x 2  ⇒ ρ b = −4( x1 eˆ 1 + x 2 eˆ 2 ) (Body force density)  m3 
  
⇒ ρb 3 = 0 

Problem 4.11
Given the velocity field components:
v1 = x1 x3 ; v 2 = x 22 t ; v3 = x 2 x 3t
and the Cauchy stress tensor field components:

University of Castilla-La Mancha Draft By: Eduardo W. V. Chaves (2016)


Ciudad Real - Spain
338 SOLVING PROBLEMS BY MEANS OF CONTINUUM MECHANICS

 x 2 x1 − x 2 x3 0 
σ ij = α − x 2 x3 x 22 − x 2 
 0 − x2 x32 
where α is a constant. Obtain the body force (per unit volume) to guarantee the principle
of conservation of the linear momentum.
Solution:
From the principle of conservation of linear momentum we obtain the equations of
motion:
r r r r r
∇ xr ⋅ σ + ρb = ρv& = ρa ⇒ ρb = ρa − ∇ xr ⋅ σ
The acceleration field:
r r r r
r ∂v ( x , t ) ∂v ( x , t ) r r ∂vi ∂vi
a= + r ⋅ v ( x, t ) ; ai = + vj
∂t ∂x ∂t ∂x j

where
r  0  r  x3 0 x1 
 ∂v  ∂vi  2   ∂v  ∂v
=  0 0 
r
  = = x2  ; (∇ xr v ) ij ≡  r  = i
2 x2 t
 ∂t  i ∂t   ∂x  ij ∂x j
 x2 x3   0 x3t x2 t 
Then
 0   x3 0 x1   x1 x3   0   x1 x32 + x1 x2 x3t 
∂vi ∂vi  
ai = + v j =  x22  +  0 2 x2t 0   x22t  =  x22  +  2 x23t 
∂t ∂x j  2
 x2 x3   0 x3t      2 2 2
x2t   x2 x3t   x2 x3   x3 x2 t + x2 x3t 
 x1 x32 + x1 x2 x3t 
 2 3 
= x2 + 2 x2t 
 x2 x3 + x3 x22t 2 + x22 x3t 2 
 
The divergence of the Cauchy stress tensor is given by:
 ∂σ 11 ∂σ 12 ∂σ 13
 + + = α ( x 2 − x3 )
 ∂ x1 ∂ x 2 ∂ x 3
 x 2 − x3 
 ∂σ ∂σ ∂σ
σ ij , j =  21 + 22 + 23 = α ( 2 x 2 ) ⇒ σ ij , j = α  2 x 2 
 ∂x1 ∂x 2 ∂x 3
 2 x 3 − 1 
 ∂σ 31 ∂σ 32 ∂σ 33
 + + = α ( 2 x 3 − 1)
 ∂x1 ∂x 2 ∂x 3
with that the body force density (per unit volume) becomes:
r r
ρ b = ρ a − ∇ xr ⋅ σ
 x1 x 32 + x1 x 2 x 3 t   x 2 − x3 
   
ρb i = ρa i − σ ij , j ⇒ ρb i = ρ  2
x2 + 2x2 t 3
 −α  2x2 
 x 2 x 3 + x 3 x 22 t 2 + x 22 x 3 t 2   2 x 3 − 1 
 

University of Castilla-La Mancha Draft By: Eduardo W. V. Chaves (2016)


Ciudad Real - Spain
4 THE FUNDAMENTAL EQUATIONS OF CONTINUUM MECHANICS 339

Problem 4.12
The Cauchy stress tensor field in the medium in equilibrium is represented by its Cartesian
components as follows:
 1 0 2 x2 
σ ij =  0 1 4 x1  (4.13)
 2 x2 4 x1 1 

where xi are the Cartesian coordinates.


a) By neglecting body forces, is the body in balance?
Solution:
The equilibrium equations:
r r
∇ xr ⋅ σ + ρ{
b=0 indicial
 → σij , j = 0i
r (4.14)
=0

and by expanding the above equation we can obtain:


σ11,1 + σ12, 2 + σ13,3 = 0

σ i1,1 + σ i 2, 2 + σ i 3,3 = 0 i ⇒ σ 21,1 + σ 22, 2 + σ 23,3 = 0 (4.15)

σ 31,1 + σ 32, 2 + σ 33,3 = 0
where we have used:
∂σ11 ∂σ ∂σ
σ11,1 = = 0; σ12, 2 = 12 = 0; σ13,3 = 13 = 0
∂x1 ∂x 2 ∂x3
∂σ 21 ∂σ 22 ∂σ 23
σ 21, 2 = = 0; σ 22, 2 = = 0; σ 23,3 = =0 (4.16)
∂x 2 ∂x 2 ∂x 3
∂σ ∂σ ∂σ
σ 31,3 = 31 = 0; σ 32, 2 = 32 = 0; σ 33,3 = 33 = 0
∂x3 ∂x 2 ∂x 3

Problem 4.13
Given a body in equilibrium in which the Cauchy stress tensor field is represented by its
components:
σ11 = 6 x13 + x 22 ; σ12 = x 32
σ 22 = 12 x13 + 60 ; σ 23 = x 2
σ 33 = 18 x 23 + 6 x33 ; σ 31 = x12
Obtain the body force density vector (per unit volume) at the point ( x1 = 2; x 2 = 4; x3 = 2 ).
Solution:
The equilibrium equations are represented by:
r r
∇ xr ⋅ σ + ρ b = 0 (4.17)
and the explicit form:

University of Castilla-La Mancha Draft By: Eduardo W. V. Chaves (2016)


Ciudad Real - Spain
340 SOLVING PROBLEMS BY MEANS OF CONTINUUM MECHANICS

 ∂σ11 ∂σ12 ∂σ13 ∂σ ∂σ ∂σ


 + + + ρ b1 = 0 ⇒ ρ b1 = − 11 − 12 − 13
 ∂x1 ∂x 2 ∂x3 ∂x1 ∂x 2 ∂x3
 ∂σ 21 ∂σ 22 ∂σ 23 ∂σ 21 ∂σ 22 ∂σ 23
 + + + ρb2 = 0 ⇒ ρb2 = − − − (4.18)
 ∂x1 ∂x 2 ∂x3 ∂x1 ∂x 2 ∂x3
 ∂σ 31 ∂σ 32 ∂σ 33 ∂σ 31 ∂σ 32 ∂σ 33
 ∂x + ∂x + ∂x + ρ b 3 = 0 ⇒ ρ b 3 = − ∂x − ∂x − ∂x
 1 2 3 1 2 3

ρ b1 = −18 x12 − 0 − 0  − 18 x12 


  
ρ b 2 = −0 − 0 − 0 ⇒ ρ bi =  0  (4.19)
ρ b = −2 x − 1 − 18 x 2  − 2 x − 1 − 18 x 2 
 3 1 2  1 2

At the point x1 = 2; x 2 = 4; x3 = 2 the body force density becomes:


 − 72
N 
ρ b i =  0  (Force per unit volume)  3  (4.20)
− 77 m 

Problem 4.14
The Cauchy stress tensor field is represented by its components as follows:
 x12 x 2 (a 2 − x 22 ) x1 0 
 1 3 
σ ij = k (a 2 − x 22 ) x1 ( x 2 − 3a 2 x 2 ) 0  (4.21)
 3 
 0 0 2ax32 
r
where k and a are constants. Obtain the specific body force field b (per unit mass) in
order to achieve equilibrium.
Solution:
 ∂σ11 ∂σ12 ∂σ13
 + + + ρ b1 = 0 ⇒ ρ b1 = −2 x1 x 2 k + 2 x1 x 2 k = 0
 ∂x1 ∂x 2 ∂x3
 ∂σ 21 ∂σ 22 ∂σ 23 k
 + + + ρ b 2 = 0 ⇒ ρ b 2 = −k (a 2 − x 22 ) − (3 x 22 − 3a 2 ) = 0 (4.22)
 ∂x1 ∂x 2 ∂x 3 3
∂σ
 31 ∂σ ∂σ
 ∂x + ∂x + ∂x + ρ b 3 = 0 ⇒ ρ b 3 = −4kax3
32 33

 1 2 3

Then:
0
4kax3   N 
bi = 0 (Force per unit mass)   (4.23)
ρ    kg 
 − 1

Problem 4.15
r
Let us assume that the specific body force is b = − gê 3 , where g is a constant and consider
the Cauchy stress tensor field components:

University of Castilla-La Mancha Draft By: Eduardo W. V. Chaves (2016)


Ciudad Real - Spain
4 THE FUNDAMENTAL EQUATIONS OF CONTINUUM MECHANICS 341

 x2 − x3 0 
σ ij = α  − x3 0 − x 2  (4.24)
 0 − x2 p 
Find p such that satisfies the equilibrium equations. Consider that α is a constant and that
r
the mass density field is homogeneous, i.e. it is independent of the vector position ( x ).
Solution:
The equilibrium equations:
r r
∇ xr ⋅ σ + ρb = 0 indicial
→ σij , j + ρbi = 0 i (4.25)
 ∂σ11 ∂σ12 ∂σ13 
 + + + ρb1 = 0
∂x ∂x ∂x 0 + 0 + 0 + ρb = 0 ⇒ b = 0
 1 2 3
 1 1
 ∂σ ∂σ ∂σ
⇒  21 + 22 + 23 + ρb 2 = 0 ⇒ 0 + 0 + 0 + ρb 2 = 0 ⇒ b 2 = 0
 ∂x1 ∂x2 ∂x3  ∂σ
 ∂σ31 ∂σ32 ∂σ 33 0 − α + 33 + ρb 3 = 0
 ∂x + ∂x + ∂x + ρb3 = 0  ∂x3
 1 2 3

thus
∂σ 33 ∂(αp ) ∂p ∂p ρg
= =α = α − ρ b3 ⇒ =1 +
∂x3 ∂x3 ∂x 3 ∂x3 α
 ρg 
⇒ dp = 1 +  dx
 α 3
 ρg   ρg 
 ∫
p = 1 + ∂x
α  3
⇒ p = 1 +

x
α  3

Problem 4.16
Show that the equilibrium equations are satisfied by considering the following Cauchy
stress field Cartesian components:
σ11 = x 22 + ν ( x12 − x 22 ) ; σ12 = −2νx1 x 2 ; σ 23 = σ13 = 0
2 2 2 2 2
σ 22 = x1 + ν ( x 2 − x1 ) ; σ 33 = ν ( x1 + x 2 )
Consider that there are no body forces.
Solution:
The equilibrium equations:
σ ij , j + ρb i = 0 i ⇒ σ ij , j = 0 i ⇒ σ i1,1 + σ i 2, 2 + σ i 3,3 = 0 i
{
=0i

 ∂σ11 ∂σ 12 ∂σ13
 + + =0
i = 1 σ 11,1 + σ 12 , 2 + σ 13,3 = 0  ∂x1 ∂x 2 ∂x 3
  ∂σ 21 ∂σ 22 ∂σ 23
i = 2 σ 21,1 + σ 22 , 2 + σ 23, 3 = 0 ⇒  + + =0
i = 3 σ 31,1 + σ 32 , 2 + σ 33,3 = 0  ∂x1 ∂x 2 ∂x 3
  ∂σ 31 ∂σ 32 ∂σ 33
 + + =0
 ∂x1 ∂x 2 ∂x 3

thus:

University of Castilla-La Mancha Draft By: Eduardo W. V. Chaves (2016)


Ciudad Real - Spain
342 SOLVING PROBLEMS BY MEANS OF CONTINUUM MECHANICS

σ 11,1 + σ 12 , 2 + σ 31,3 = 2 x1ν − 2νx1 = 0



⇒ σ 12 ,1 + σ 22 , 2 + σ 23, 3 = −2 x 2 ν + 2νx 2 = 0

σ 13,1 + σ 23, 2 + σ 33,3 = 0
with that we have shown that the body is in balance.

Problem 4.17
Consider a body in equilibrium in which the Cauchy stress field components are:
 x1 + x 2 σ12 0
r 
σ ij ( x ) =  σ12 x1 − 2 x 2 0 
 0 0 x 2 

Find σ12 , knowing that σ12 is a function of x1 and x 2 , i.e. σ12 = σ12 ( x1 , x 2 ) . It is also
known that the medium is free of body forces and the traction vector associated with the
r ˆ
plane x1 = 1 is given by t (n) = (1 + x 2 )eˆ 1 + (5 − x 2 )eˆ 2 .
Solution:
As the body is in equilibrium, it must satisfy the equilibrium equations:
σ ij , j + ρb i = 0 i ⇒ σ ij , j = 0 i ⇒ σ i1,1 + σ i 2, 2 + σ i 3,3 = 0 i
{
=0i

thus
 ∂σ11 ∂σ12 ∂σ13 ∂σ ∂σ12
 + + = 1 + 12 + 0 = 0 ⇒ = −1
 ∂x1 ∂x 2 ∂x3 ∂x 2 ∂x 2
 ∂σ 21 ∂σ 22 ∂σ 23 ∂σ12 ∂σ12
⇒ + + = −2+0=0 ⇒ =2
 ∂ x1 ∂ x 2 ∂x 3 ∂x1 ∂x1
 ∂σ 31 ∂σ 32 ∂σ 33
 + + =0+0+0=0
 ∂x1 ∂x 2 ∂x3

Now considering that for the plane x1 = 1 , the stress tensor and the traction vector, when
x1 = 1 , become respectively:

1 + x 2 σ12 0
 r
0 
ˆ
σ ij ( x1 = 1, x 2 ) =  σ12 1 − 2 x 2 and t (n) = (1 + x 2 )eˆ 1 + (5 − x 2 )eˆ 2
 0 0 x 2 
r
Remember that the traction vector can be obtained by the equation t (n) = σ ⋅ nˆ , then
ˆ

1 + x 2 σ12 0  1 1 + x 2 
t (nˆ ) 
= σ ij ( x1 = 1, x 2 )nˆ j =  σ12 1 − 2 x 2 0  0 = 5 − x 2  (4.26)
 0 0 x 2  0  0 
ˆ
t (n) = σ ij ( x1 = 1, x 2 ) nˆ j
 1 + x2 σ12 ( x1 = 1, x 2 ) 0  1   1 + x2  1 + x 2 

⇒ σ12 ( x1 = 1, x 2 ) 1 − 2x2 0  0 = σ12 ( x1 = 1, x 2 ) = 5 − x 2 
   
 0 0 x 2  0  0   0 

By means of the equilibrium equations:

University of Castilla-La Mancha Draft By: Eduardo W. V. Chaves (2016)


Ciudad Real - Spain
4 THE FUNDAMENTAL EQUATIONS OF CONTINUUM MECHANICS 343

∂σ12
∂x1
=2 ⇒ ∫ ∂σ 12 ∫
= 2∂x1 ⇒ σ12 ( x1 , x 2 ) = 2 x1 + C ( x 2 )

And by using the information given by (4.26) we can obtain the constant of integration:
σ12 ( x1 = 1, x 2 ) = 5 − x 2 = 2 + C ( x 2 ) ⇒ C ( x2 ) = 3 − x2
thus:
σ12 ( x1 , x 2 ) = 2 x1 − x 2 + 3

Problem 4.18
The stress state in an continuous medium is given by the Cauchy stress tensor Cartesian
components:
 σ11 σ12 σ13   0 Cx3 0 
σij = σ12 σ 22 σ 23  = Cx3 0 − Cx1 
σ13 σ 23 σ33   0 − Cx1 0 
where C is a constant. Consider that the body is free of body force.
a) Show whether the body is in balance or not;
Solution:
a) The continuous medium is in equilibrium if the following equations hold:
r r
∇ ⋅ σ + ρb = 0 ; σij,j + ρ bi = 0 i (the equilibrium equations) (4.27)

For the proposed problem we have ρ bi = 0i , and the vector σij,j is evaluated as follows:

 ∂σ11 ∂σ12 ∂σ13


 + +
 ∂x ∂x ∂x3
1 2
i = 1 ⇒ 0 + 0 + 0 = 0 ⇒ σ1 j,j = 0
∂σij ∂σi1 ∂σi 2 ∂σi 3  ∂σ21 ∂σ22 ∂σ 23 
σij,j = = + + ⇒ + + i = 2 ⇒ 0 + 0 + 0 = 0 ⇒ σ 2 j,j = 0
∂x j ∂x1 ∂x2 ∂x3  ∂x1 ∂x2 ∂x3 
i =3⇒ 0+0+0 = 0 ⇒ σ3 j,j = 0
 ∂σ31 ∂σ32 ∂σ33 
 + +
 ∂x1 ∂x2 ∂x3

with that we can conclude that σ ij,j = 0 i then the body is in equilibrium.

Problem 4.19
Considering the principle of conservation of angular momentum, show that:
r r r r r r r r r r
∫ ρ [( x ⊗ (a − b) − (a − b) ⊗ x] dV = ∫ [( x ⊗ t − t ⊗ x] dS
* *

V Sσ
r r r r
where x is the vector position, ρ ( x , t ) is the mass density, a ( x , t ) is the acceleration,
r r r r
b( x , t ) is the specific body force (per unit mass), and t * ( x , t ) is the prescribed traction
vector (surface force) on surface S σ .
Solution:
The principle of conservation of angular momentum states that:

University of Castilla-La Mancha Draft By: Eduardo W. V. Chaves (2016)


Ciudad Real - Spain
344 SOLVING PROBLEMS BY MEANS OF CONTINUUM MECHANICS

r r r r D r r r r

Sσ V

( x ∧ t * )dS + ( x ∧ ρ b)dV =
Dt V
( x ∧ ρ v )dV = ( x ∧ ρ a )dV
V
∫ ∫
r
Then, we apply the cross product of the above equation with an arbitrary vector z , which
r
is independent of x , and we obtain:
r r r r r r* r r r

z ∧ ( x ∧ ρ a )dV = z ∧
V Sσ
∫ (x ∧ t ∫
)dS + z ∧ ( x ∧ ρ b)dV
V
r r r r r r* r r r

⇒ z ∧ ( x ∧ ρ a )dV =
V
∫ z ∧ (x ∧ t


)dS + z ∧ ( x ∧ ρ b)dV
V

r r r
We have shown in Chapter 1 that given three vectors a , b , c , the relationship
r r r r r r r r
a ∧ (b ∧ c ) = (b ⊗ c − c ⊗ b) ⋅ a holds, (see Problem 1.17). Then, the above equation can
be rewritten as follows:
r r r r r r r* r r r r r r r r
∫ ( x ⊗ ρ a − ρ a ⊗ x) ⋅ z dV = ∫ ( x ⊗ t
V Sσ
− t * ⊗ x ) ⋅ z dS + ( x ⊗ ρ b − ρ b ⊗ x ) ⋅ z dV

V
r r r r r r r r r r r r r r r
⇒ ρ ( x ⊗ a − a ⊗ x ) ⋅ z dV − ρ ( x ⊗ b − b ⊗ x ) ⋅ z dV = ( x ⊗ t * − t * ⊗ x ) ⋅ z dS
∫ ∫ ∫
V V Sσ
r r r
[ r r r r r r r r r
⇒ ρ x ⊗ (a − b) − (a − b) ⊗ x ⋅ z dV = ( x ⊗ t * − t * ⊗ x ) ⋅ z dS
∫ ] ∫
V Sσ

 r  r r  r

V
r

r r
[ r r r

r r
⇒  ρ x ⊗ (a − b) − (a − b) ⊗ x dV  ⋅ z =  ( x ⊗ t * − t * ⊗ x ) dS  ⋅ z ] ∫
S σ 
with that we can conclude that:
r r r r r r r r r r
∫ ρ [ x ⊗ (a − b) − (a − b) ⊗ x] dV = ∫ ( x ⊗ t − t ⊗ x) dS
* *

V Sσ

Problem 4.20
1) Considering the definition of the mean stress tensor ( σ ):
V σ = σ dV ∫
V

and based on the principle that the continuum is in static equilibrium, show that:
1 r r r r 1 r r r r
σ=
2V V ∫
ρ [ x ⊗ b + b ⊗ x ] dV +
2V ∫

( x ⊗ t * + t * ⊗ x ) dS

2) Considering that the volume can be decomposed by V = V (1) − V ( 2) , (see Figure 4.4), and
by considering that the continuum is subjected to pressure p (1) on surface S (1) and to
pressure p ( 2) on surface S ( 2) , show that:
−1
σ= (1) ( 2)
( p (1)V (1) − p ( 2)V ( 2 ) )1
(V −V )
Consider that the continuum is free of body forces.

University of Castilla-La Mancha Draft By: Eduardo W. V. Chaves (2016)


Ciudad Real - Spain
4 THE FUNDAMENTAL EQUATIONS OF CONTINUUM MECHANICS 345

(1)
n̂ (1)
S

V (1) p (1)

S (2)

V ( 2)

n̂ ( 2 )
p (2)

Figure 4.4
Solution:
r r r
Taking into account the equilibrium equations ∇ xr ⋅ σ + ρ b = ρ a = 0 (the principle of
conservation of linear momentum) for the whole continuum, it must fulfill that:
r r r r r r r
∫ x ⊗ [∇ xr ⋅ σ + ρb] dV = 0 ⇒ ∫ x ⊗ (∇ xr ⋅ σ ) dV + x ⊗ ρb dV = 0 ∫ (4.28)
V V V

In Chapter 1, (see Problem 1.128), we have shown that the following holds:
r r r* r
∫ (∇ ⋅ σ ) ⊗ x dV = ∫ (σ ⋅ nˆ ) ⊗ x dS − ∫ σ dV = ∫ t
V S V S
⊗ x dS − σ dV ∫
V
(4.29)

r r r r
∫ x ⊗ (∇ ⋅ σ ) dV = ∫ x ⊗ (σ ⋅ nˆ ) dS − ∫ σ ∫ ∫
T
dV = x ⊗ t * dS − σ T dV (4.30)
V S V S V
r
where we have considered the prescribed traction vector t * = σ ⋅ nˆ . By substituting (4.30)
into the equation (4.28), we can obtain:

r r r r r r r r r
∫ x ⊗ (∇ xr ⋅ σ ) dV + x ⊗ ρb dV = 0 ⇒
∫ ∫ ∫
x ⊗ t * dS − σ T dV + x ⊗ ρb dV = 0 ∫
V V S V V
(4.31)
r r r r
∫ ∫
⇒ σ T dV = x ⊗ t * dS + x ⊗ ρb dV
V S

V

Then, the following is true:


r r r r

V

σ dV = t * ⊗ x dS + ρ b ⊗ x dV
S

V
(4.32)

r r r r
Note that the tensors x ⊗ t * and x ⊗ ρ b are not symmetric. This means that the equation
in (4.28) does not take in account the principle of conservation of angular momentum, i.e.
the symmetry of the Cauchy stress tensor. To guarantee the symmetry of σ we do:

University of Castilla-La Mancha Draft By: Eduardo W. V. Chaves (2016)


Ciudad Real - Spain
346 SOLVING PROBLEMS BY MEANS OF CONTINUUM MECHANICS

σ + σT 1 r r r r  1 r r r r 

V
2 2  S ∫ V

dV =  t * ⊗ x dS + ρb ⊗ x dV  +  x ⊗ t * dS + x ⊗ ρb dV 
 2  S V 
∫ ∫
(4.33)
1 r r r r 1 r r r r

⇒ σ sym dV =
V
2V ∫
ρ [ x ⊗ b + b ⊗ x ] dV + [ x ⊗ t * + t * ⊗ x ] dS
2S ∫
By considering the definition of the mean stress tensor, we can conclude:

σ + σT 1 r r r r  1 r r r r 

V
2 2  S ∫ V

dV =  t * ⊗ x dS + ρb ⊗ x dV  +  x ⊗ t * dS + x ⊗ ρb dV 
 2  S V 
∫ ∫
1 r r r r 1 r r r r

⇒ σ sym dV =
V
2V ∫
ρ [ x ⊗ b + b ⊗ x ] dV + [ x ⊗ t * + t * ⊗ x ] dS
2S ∫
(4.34)
1 r r r r 1 r r r r
⇒V σ =
2V ∫
ρ [ x ⊗ b + b ⊗ x ] dV + [ x ⊗ t * + t * ⊗ x ] dS
2S ∫
1 r r r r 1 r r r r
⇒σ =
2V ∫ ρ [ x ⊗ b + b ⊗ x] dV +
V
2V S ∫
[ x ⊗ t * + t * ⊗ x ] dS

In addition, if we consider that the body is free of body force, the above equation becomes:
1 r r r r
σ=
2V S∫[ x ⊗ t * + t * ⊗ x ] dS (4.35)

For the particular case described in Figure 4.4 we have V = V (1) − V ( 2) , S = S (1) + S ( 2) ,
r (1) r ( 2)
t * = − p (1) n̂ (1) and t * = − p ( 2) n̂ ( 2) . In this case, the equation (4.35) becomes:

 r r r r r* r* r 
1 r ( 2) 
∫ ∫
* * (1)
σ=  [ x ⊗ t + t ⊗ x ] dS + [ x ⊗ t + t ⊗ x ] dS 
2(V (1) − V ( 2 ) ) S (1) S (2) 

1  r r r r 
= (2) 
(1)
2(V − V ) S (1) ∫ ∫
− p (1) [ x ⊗ nˆ (1) + nˆ (1) ⊗ x ] dS (1) + − p ( 2 ) [ x ⊗ nˆ ( 2 ) + nˆ ( 2 ) ⊗ x ] dS ( 2 ) 

S (2)

−1  r r r r 
= (2) 
(1)
2(V − V )  ∫ ∫
p (1) [ x ⊗ nˆ (1) + nˆ (1) ⊗ x ] dS (1) + p ( 2 ) [ x ⊗ nˆ ( 2 ) + nˆ ( 2 ) ⊗ x ] dS ( 2) 

S (1 ) S (2)

r r
We have shown in Chapter 1 that is true ∫ ( x ⊗ nˆ + nˆ ⊗ x) dS = 2V 1 ,
S
where n̂ is the

outward unit normal to surface S , (see Problem 1.128). For this example, n̂ ( 2) is the
r r
inward unit normal to surface S ( 2) , then, we have ∫ [ x ⊗ nˆ + nˆ ( 2 ) ⊗ x ] dS ( 2 ) = −2V ( 2) 1 ,
( 2)

S (2)
with that we can obtain:

−1  (1) r 
ˆ (1) + nˆ (1) ⊗ xr ] dS (1) + p ( 2 ) [ xr ⊗ nˆ ( 2) + nˆ ( 2 ) ⊗ xr ] dS ( 2 ) 
σ=
2(V (1) − V ( 2 ) ) 
 p ∫[ x ⊗ n ∫ 
S ( 1) S (2)

=
−1
(1)
( 2)
2(V − V )
{
p (1) 2V (1) 1 − p ( 2 ) 2V ( 2 ) 1 = (1)
−1
}
(V − V ( 2 ) )
{
p (1)V (1) − p ( 2 )V ( 2 ) 1 }

University of Castilla-La Mancha Draft By: Eduardo W. V. Chaves (2016)


Ciudad Real - Spain
4 THE FUNDAMENTAL EQUATIONS OF CONTINUUM MECHANICS 347

Problem 4.21
r
Starting from ρ u& = σ : D − ∇ xr ⋅ q + ρr , show that the energy equation can also be written
as follows:
D  1 2 r r r r
ρ  u + v  = ∇ xr ⋅ (v ⋅ σ ) + ρb ⋅ v − ∇ xr ⋅ q + ρr
Dt  2 
(4.36)
D  1 2
ρ  u + v  = (v j σ ji ) ,i + ρ b i v i − q i ,i + ρr
Dt  2 
or
∂ 1 2   1 2  r r r r r
ρ  u + v  + ρ ∇ xr  u + v  ⋅ v = ∇ xr ⋅ (v ⋅ σ ) + ρb ⋅ v − ∇ xr ⋅ q + ρr
∂t  2    2 
(4.37)

ρ  u + v 2  + ρ  u + v 2  vi = (v j σ ji ) ,i + ρ b i vi − qi ,i + ρr
1 1
∂t  2   2  ,i
or
∂  1 2   1 2  r r r r r
ρ  u + v  + ∇ xr ⋅  ρ  u + v v  = ∇ xr ⋅ (v ⋅ σ ) + ρb ⋅ v − ∇ xr ⋅ q + ρr
∂t  2    2  
(4.38)
∂   1 2    1 2 
∂t  ρ  u + 2 v   +  ρ  u + 2 v vi  = (v j σ ji ) ,i + ρ b i vi − qi ,i + ρr
       ,i
where ρ is the mass density, u is specific internal energy, v is magnitude of the velocity
r r r r
( v 2 = v = v ⋅ v ), σ is the Cauchy stress tensor, b is the specific body force (per unit
2

r
mass), q is the flux vector, r is the radiant heat constant (also called the heat source).
Solution:
Taking into account the energy equation:
r
ρ u& = σ : D − ∇ xr ⋅ q + ρr ρ u& = σ ij D ij − qi ,i + ρr
where D is the rate-of-deformation tensor which is the symmetric part of the spatial
r
velocity gradient ( D = (∇ xr v ) sym ≡ l sym ). Note also that σ : D = σ : l sym = σ : l since the
double scalar between symmetric ( σ = σ T ) and antisymmetric tensor ( l skew ) is zero, i.e.
σ : l skew = 0 , thus
r
σ ij D ij = σ ij ( l ) ij = σ ij (∇ xr v ) ij = σ ij vi , j

Note also that (σ ij vi ) , j = σ ij , j vi + σ ij vi , j ⇒ σ ij vi , j = (σ ij vi ) , j − σ ij , j vi = σ ijD ij , thus the energy


equation becomes:
r
ρ u& = σ : D − ∇ xr ⋅ q + ρr ρ u& = σ ijD ij − qi ,i + ρr
r r r
⇒ ρ u& = ∇ xr ⋅ (v ⋅ σ ) − (∇ xr ⋅ σ ) ⋅ v − ∇ xr ⋅ q + ρr ⇒ ρ u& = (σ ij vi ) , j − σ ij , j vi − qi ,i + ρr

Taking into account the equations of motion we can obtain that:


r
∇ xr ⋅ σ + ρb = ρu &r& = ρvr& σ ij , j + ρ b i = ρ u
&&i = ρv&i
r r ⇒ σ ij , j = ρv&i − ρ b i
⇒ ∇ xr ⋅ σ = ρv& − ρb
r r r r r ⇒ σ ij , j vi = ρv&i vi − ρ b i vi
⇒ (∇ xr ⋅ σ ) ⋅ v = ρv& ⋅ v − ρb ⋅ v
With that the energy equation can also be written as follows:

University of Castilla-La Mancha Draft By: Eduardo W. V. Chaves (2016)


Ciudad Real - Spain
348 SOLVING PROBLEMS BY MEANS OF CONTINUUM MECHANICS

r r r
ρ u& = ∇ xr ⋅ (v ⋅ σ ) − (∇ xr ⋅ σ ) ⋅ v − ∇ xr ⋅ q + ρr ρ u& = σ ijD ij − qi ,i + ρr
r r r r r r
ρ u& = ∇ xr ⋅ (v ⋅ σ ) − ( ρv& ⋅ v − ρb ⋅ v ) − ∇ xr ⋅ q + ρr ρ u& = (σ ij vi ) , j − ( ρv&i vi − ρ b i vi ) − qi ,i + ρr
r r r r r r
⇒ ρ u& + ρv& ⋅ v = ∇ r ⋅ (v ⋅ σ ) + ρb ⋅ v − ∇ r ⋅ q + ρr ⇒ ρ u& + ρv&i vi = (σ ij vi ) , j + ρ b i vi − qi ,i + ρr
x x

D r r r r r r r r r r 1 D r r 1 D 2
Note that (v ⋅ v ) = (v& ⋅ v ) + (v ⋅ v& ) = 2(v& ⋅ v ) ⇒ (v& ⋅ v ) = (v ⋅ v ) = (v ) . Thus, the
Dt 2 Dt 2 Dt
energy equation becomes
r r r r r r
⇒ ρ u& + ρv& ⋅ v = ∇ xr ⋅ (v ⋅ σ ) + ρb ⋅ v − ∇ xr ⋅ q + ρr
 1 D 2  r r r r
⇒ ρ  u& + (v )  = ∇ xr ⋅ (v ⋅ σ ) + ρb ⋅ v − ∇ xr ⋅ q + ρr
 2 Dt  (4.39)
D  1 2 r r r r
⇒ρ  u + v  = ∇ xr ⋅ (v ⋅ σ ) + ρb ⋅ v − ∇ xr ⋅ q + ρr
Dt  2 
which in indicial notation becomes
⇒ ρ u& + ρv&i vi = (σ ij vi ) , j + ρ b i vi − qi ,i + ρr
 Du 1 D 2 
⇒ρ + (v )  = (σ ij vi ) , j + ρ b i vi − qi ,i + ρr
 Dt 2 Dt 
D  1 2
⇒ρ  u + v  = (σ ij vi ) , j + ρ b i vi − qi ,i + ρr
Dt  2 
with which we show the equation in (4.36). The equation in (4.37) can be easily obtained if
D• ∂• r
we apply the material time derivative ≡ •& = + (∇ xr •) ⋅ v to the equation in (4.39), i.e.:
Dt ∂t
r r
ρ  u + v 2  = ∇ xr ⋅ (v ⋅ σ ) + ρb ⋅ v − ∇ xr ⋅ q + ρr
D 1 r r
Dt  2 
∂ 1    1  r r r r r
⇒ ρ  u + v 2  + ρ ∇ xr  u + v 2   ⋅ v = ∇ xr ⋅ (v ⋅ σ ) + ρb ⋅ v − ∇ xr ⋅ q + ρr
∂t  2    2 

∂φ r ∂ r
In Problem 4.8 we have shown that ρ + ρ (∇ xrφ ) ⋅ v = ( ρφ ) + ∇ xr ⋅ ( ρφv ) holds, and if
∂t ∂t
 1 
we consider that φ =  u + v 2  we show the equation in (4.38).
 2 

4.2 Some Useful Concepts for the Classical “Mechanics of


Materials”
Problem 4.22
r
Consider a prismatic bar, (see Figure 4.5), in which the Cauchy stress field ( σ = σ ( x ) )
Cartesian components are:
 σ11 σ12 σ13 
σij = σ 21 σ 22 σ 23  (4.40)
σ31 σ32 σ33 
Obtain the resultant force and momentum acting on the cross section A . The cross section
is characterized by the unit vector nˆ i = [1 0 0] , in which it is also subjected to the stress
r
state described in (4.40). Adopt the Cartesian system Ox described in Figure 4.5.

University of Castilla-La Mancha Draft By: Eduardo W. V. Chaves (2016)


Ciudad Real - Spain
4 THE FUNDAMENTAL EQUATIONS OF CONTINUUM MECHANICS 349

σ 33 rˆ
Prismatic bar rˆ σ 23 t ( e1 )
t ( e1 ) x2
σ13
x3

σ31 σ 21
A r x2 σ11
x x1
Rear face
O ê1
x1 Face on the
cross section A

Figure 4.5: Stress field on the cross section A .

Solution:
r r
The traction vector t (n) is related to the Cauchy stress tensor as follows t (n) = σ ⋅ nˆ . For
ˆ ˆ

this problem we have nˆ = eˆ 1 , thus:

σ13  1   σ11  t1( e1 ) 


ˆ
 σ11 σ12
 ˆ 
= σ ij nˆ j = σ 21 σ 22 σ 23  0 = σ 21  = t (2e1 ) 
ˆ
t i(e1 ) (4.41)
σ 33  0 σ 31  t (3e1 ) 
ˆ
σ 31 σ 32
 
The Resultant Forces
r r ˆ
∑ ∫ ∑F ∫
ˆ
F = t ( e1 ) dA ; xi = t i( e1 ) dA
A A

Thus,
 
 ( eˆ )   σ11dA 

 t1 1 dA  A 



∑ Fx1   F11   A

  
∑ ∫ ∑ Fx2  ≡  F21  =  t (2e1 ) dA =  σ 21dA
∫ ∫
ˆ ˆ
Fxi = t i(e1 ) dA ⇒  (4.42)
 
A 
 ∑ Fx3   F31   A ( eˆ )   A 
 A

 t 3 1 dA 
  σ 31dA


A 
The Resultant Moments
r ˆ
Resultant moment due to t ( e1 ) (related to the system O − x1 − x 2 − x3 ):
r r r ˆ
∑M O ∫
= x ∧ t (e1 ) dA ; ∑M = ∫ i
( eˆ 1 )
ijk x j t k dA (4.43)
A A

ˆ
The explicit form of the term  ijk x j t (ke1 ) is given by:
ˆ ˆ ˆ ˆ
 ijk x j t (ke1 ) =  ij1 x j t1(e1 ) +  ij 2 x j t (2e1 ) +  ij 3 x j t 3( e1 )
ˆ ˆ ˆ ˆ ˆ ˆ
=  i 21 x2 t1( e1 ) +  i 31 x3 t1( e1 ) +  i12 x1t (2e1 ) +  i 32 x3 t (2e1 ) +  i13 x1t (3e1 ) +  i 23 x2 t 3(e1 )

University of Castilla-La Mancha Draft By: Eduardo W. V. Chaves (2016)


Ciudad Real - Spain
350 SOLVING PROBLEMS BY MEANS OF CONTINUUM MECHANICS

(i = 1) ⇒
ˆ ˆ ˆ ˆ ˆ
1 jk x j t (ke1 ) = 132 x3 t (2e1 ) + 123 x2 t 3(e1 ) = − x3 t (2e1 ) + x2 t 3(e1 )
ˆ  ˆ ˆ ˆ ˆ ˆ
 ijk x j t (ke1 ) ⇒ (i = 2) ⇒ 1 jk x j t (ke1 ) =  231 x3 t1( e1 ) +  213 x1t (3e1 ) = x3 t1( e1 ) − x1t 3( e1 )
(i = 3) ⇒
ˆ ˆ ˆ ˆ ˆ
1 jk x j t (ke1 ) =  321 x2 t1( e1 ) +  312 x1t (2e1 ) = − x2 t1( e1 ) + x1t (2e1 )

Then, the explicit components of (4.43) are, (see Figure 4.6):
 

( eˆ )
 1 jk x j t k 1 dA 


∑ M 1   M x1   A
   

∑M = ∫ ( eˆ 1 )
ijk x j t k dA ⇒  ∑ M 2  ≡  M x 2  =  2 jk x j t k dA

( eˆ 1 )
i  
A 
 ∑ M 3   M x3   A 

ˆ
 3 jk x j t (ke1 ) dA
 A 

   


( eˆ 1 )
 1 jk x j t k dA   A ∫  A

  ( x2 t (3eˆ 1 ) − x3 t (2eˆ 1 ) )dA  ( x2 σ31 − x3σ 21 )dA

M x   A     
 1

⇒  M x 2  =   2 jk x j t (ke1 ) dA =  ( x3 t1( e1 ) − x1t 3(e1 ) )dA  =  ( x3σ11 − x1σ31 )dA 
∫ ∫ ∫
ˆ ˆ ˆ
   (4.44)
  
M x   A   A
  A

 3 

ˆ
 3 jk x j t (ke1 ) dA    
∫ ∫
( eˆ 1 ) ( eˆ 1 )
 A   ( x1t 2 − x2 t1 )dA   ( x1σ 21 − x2 σ11 )dA 
A  A 
Note that the equations (4.42) and (4.44) are valid if the section is defined by a plane
otherwise these equations are no longer valid.

x3
σ31
σ 21 x3

σ11
r M x3
x
x2
x2
ê1
O
x1
A F31
M x2
F21
O
    F11
A

 σ11dA
 

 ( x2σ31 − x3σ 21 )dA
 M x1
 F11     M x1   A 
 F  =  σ dA     x1

 21   21  ; M
 x2  = ∫
( x σ
3 11 − x σ
1 31 ) dA

 F31   A  M x   A 
 3
   
A

 σ31dA
 A

 ( x1σ 21 − x2σ11 ) dA 

Figure 4.6: Resultant force and moment on cross section A .

University of Castilla-La Mancha Draft By: Eduardo W. V. Chaves (2016)


Ciudad Real - Spain
4 THE FUNDAMENTAL EQUATIONS OF CONTINUUM MECHANICS 351

If we are using Engineering notation the equations for forces and moments are represented
by:
     
A

 σ11dA   σ x dA 
 A 
∫ 

 ( yτ xz − zτ xy )dA

 F11   N      M x  M T   A 
 F  ≡  F  =  σ dA ≡  τ dA  1
    

 21   y   21   xy  ; ∫ ∫
 M x2  ≡  M y  =  ( zσ x − xτ xz )dA 
 F31   Fz   A  A M x   M z   A 
 3  
     
A

 σ 31dA  τ xz dA
 A 
∫ A

 ( xτ xy − yσ x )dA

r r r r
NOTE 1: Note that, if the body is in equilibrium we have F = 0 and ∑ M O = 0 , so, ∑
in the cross section at the point O we have to apply forces and moments with the same
magnitudes and directions but with opposite senses as those indicated in Figure 4.6, (see
Figure 4.7).

x3
Equilibrium
r r
M x3 ∑F = 0
r r
x2 ∑ M = 0 O

F31
M x2
F21 x3
(− )
O
F11
A M x1 M x1
x2
F11
O (+ )

F21 F31
x1

M x2
M x3

Figure 4.7: Cross section in equilibrium.

r
NOTE 2: Let us suppose now we have another system defined by Ox ′ , (see Figure 4.8),
r r r
where it holds x = x + x ′ . Then, the resultant force and resultant moment at the new
system are defined by:

University of Castilla-La Mancha Draft By: Eduardo W. V. Chaves (2016)


Ciudad Real - Spain
352 SOLVING PROBLEMS BY MEANS OF CONTINUUM MECHANICS

The resultant forces


 

 σ11dA 
A 


∑ Fx′1   F11′  
   
  F11 
∑ ∫ ∑ Fx′2  ≡  F21′  = σ 21dA =  F21 

ˆ
Fx′i = t i(e1 ) dA ⇒   
A 
 ∑ Fx′3   F31′   A

  F31 


 σ 31dA
A 
The resultant moments
   

 M x′1   A  A

 ( x2′ σ 31 − x3′ σ 21 )dA  (( x2 − x2 )σ 31 − ( x3 − x3 )σ 21 )dA

     
∫ ∫
 M x′ 2  =  ( x3′ σ11 − x1′σ 31 )dA  =  (( x3 − x3 )σ11 − ( x1 − x1 )σ 31 )dA 
 M x′   A  A 
 3 
∫ A   A

( x1′σ 21 − x′2 σ11 )dA   (( x1 − x1 )σ 21 − ( x2 − x2 )σ11 )dA 

(4.45)
 
 M ′x   A ∫ A

A   1

 ( x2 σ 31 − x3σ 21 )dA − x2 σ 31dA + x3 σ 21dA  M x − x2 F31 + x3 F21 

 1  
⇒  M x2  = ( x3 σ11 − x1σ 31 )dA − x3 σ11dA + x1 σ 31dA = M x2 − x3 F11 + x1 F31 
′ ∫ ∫


   
 M x′   A A A   
 3 
 A
∫ A

A 

( x1σ 21 − x2 σ11 )dA − x1 σ 21dA + x2 σ11dA   M x3 − x1 F21 + x2 F11 

x3′

M ′x3

x′2
r r r F31′
x = x + x′
M ′x2
x3
F21′
r O′
x′ F11′
M x3
r M ′x1
x x1′

r x2
F31 x
M x2
F21
O
F11
A
M x1
x1

Figure 4.8

University of Castilla-La Mancha Draft By: Eduardo W. V. Chaves (2016)


Ciudad Real - Spain
4 THE FUNDAMENTAL EQUATIONS OF CONTINUUM MECHANICS 353

Note that, the result given by equation in (4.45) could also have been obtained by:
r r r r r r r
M O′ xr ′ = M Oxr + (− x ) ∧ FOxr = M Oxr − x ∧ FOxr
r r
where the above vectors are defined in Figure 4.9. The term x ∧ FOxr can be evaluated as
follows:

r r eˆ 1 eˆ 2 eˆ 3
x ∧ FOxr = x1 x2 x3 = ( x2 F31 − x3 F21 )eˆ 1 + ( x3 F11 − x1 F31 )eˆ 2 + ( x1 F21 − x2 F11 )eˆ 3
F11 F31 F21
r
Then, the components of M O′ xr ′ are:

r r  M x1   x2 F31 − x3 F21 
(
r
) r    
M O′ xr ′ i =  M Oxr − x ∧ FOxr  =  M x2  −  x3 F11 − x1 F31 
 i
 M x   x1 F21 − x2 F11 
 3  

r r r r
M O′ xr ′ = M Oxr − x ∧ FOxr
x3′
r
M O′ xr ′ x′2
r r r
x = x + x′
x3
r r
FO′ xr ′ = FOxr
r O′
x′
r x1′
x

r x2
x
r
FOxr r
r FOxr = F11eˆ 1 + F21eˆ 2 + F31eˆ 3
M Oxr r
O M Oxr = M x1 eˆ 1 + M x2 eˆ 2 + M x3 eˆ 3
A x1

Figure 4.9

NOTE 3: In sight of Figure 4.9 there is a point O ′ in which M x′1 = 0 , this point is called
Shear Center (S.C.) and fulfills that M ′x1 = M x1 − x2 F31 + x3 F21 = 0 . Note also that when
M x1
F21 = 0 the center can be obtained by M x1 − x2 F31 = 0 ⇒ x2 = x2( S .C .) = and when
F31
− M x1
F31 = 0 we can obtain x3 = x3( S .C .) = and the point where these two lines intercept is
F21
the Shear Center, (see Figure 4.10). Note also that when F21 and F31 are applied at the
shear center there is no torsion moment, i.e. the beam will only be subjected to flexural
moments ( M ′x 2 , M ′x3 ), and due to this reason the shear center is also called Flexural Center.

University of Castilla-La Mancha Draft By: Eduardo W. V. Chaves (2016)


Ciudad Real - Spain
354 SOLVING PROBLEMS BY MEANS OF CONTINUUM MECHANICS

x3 M x1
x2 = x2( S .C .) =
M x1 = x2 F31 − x3 F21 F31

x2
O x2
− M x1 F31
x3 = = x3( S .C .)
F21 F21
x3 S.C.
M ′x1(′ S .C .) = 0

Figure 4.10: Shear center.

NOTE 4: Now instead of considering the stress in the infinitesimal element described in
Figure 4.5, let us consider a differential element according to x1 -direction, (see Figure 4.3).
Then, on the face x1 + dx1 we can calculate the resultant forces, (see equation (4.42)), and
moments, (see equation (4.44)), by considering the stress distribution given in Figure 4.11.

∂σ31
σ31 + dx1 ∂σ 21
∂x1 σ 21 + dx1
∂x1

∂σ11
σ11 + dx1
x3 ∂x1
r
x
x2
ê1
O′′ x1
dx1 A

Figure 4.11: Stress distribution in the differential x1 + dx1 .

And according to the Figure 4.11 we can obtain:


The resultant forces by means of the equation (4.42):
  ∂σ   ∂σ  ∂  
∫ ∫ ∫
 F11+ dx1 =  σ11 + 11 dx1 dA = σ11dA +  11 dx1  dA = F11 + σ11dA  dx1 ∫
 A
∂x1  A A
∂x1  ∂x1  A 


  ∂σ 21   ∂σ  ∂  

 F21+ dx1 =  σ 21 + ∫ ∫
dx1  dA = σ 21dA +  21 dx1  dA = F21 + σ 21dA dx1 ∫
 A
∂x1  A A
∂x1  ∂x1  A 


  ∂σ 31   ∂σ 31  ∂  
 dx1
 F31+ dx1
= ∫



σ 31 +
∂x
dx1 


dA = ∫σ 31 dA + 
 ∂x

∫ dx1 


dA = F31 +
∂x 
A
σ 31 dA



 A 1 A A 1 1

with which we can conclude that

University of Castilla-La Mancha Draft By: Eduardo W. V. Chaves (2016)


Ciudad Real - Spain
4 THE FUNDAMENTAL EQUATIONS OF CONTINUUM MECHANICS 355

∂F11 ∂F21 ∂F31


F11+ dx1 = F11 + dx1 ; F21+ dx1 = F21 + dx1 ; F31+ dx1 = F31 + dx1
∂x1 ∂x1 ∂x1
By considering the definition (4.44), the resultant moments at the point O′′ , (see Figure
4.11), are:
  ∂σ   ∂σ 

A

M x1 + dx1 =  x2  σ 31 + 31 dx1  − x3  σ 21 + 21 dx1  dA
 ∂x1   ∂x1 

∂   ∂M x1

= ( x2 σ31 − x3σ 21 )dA + ∫( x2 σ31 − x3σ 21 )dA dx1 = M x1 + dx1
A
∂x1  A 
 ∂x1
  ∂σ   ∂σ 

A

M x2 + dx1 =  x3  σ11 + 11 dx1  − x1  σ 31 + 31 dx1  dA
 ∂x1   ∂x1 

∂   ∂M x2

= ( x3σ11 − x1σ31 )dA + ∫ ( x3σ11 − x1σ 31 ) dA dx1 = M x2 + dx1
A
∂x1  A 
 ∂x1

  ∂σ   ∂σ 

A 

M x3 + dx1 =  x1  σ 21 + 21 dx1  − x2  σ11 + 11 dx1   dA
∂x1   ∂x1 

∂   ∂M x3

= ( x1σ 21 − x2 σ11 )dA + ∫( x1σ 21 − x2 σ11 )dA  dx1 = M x3 + dx1
A
∂x1  A 
 ∂x1

If we take into account the differential element dx1 , (see Figure 4.12), and by means of
r r
∑M O′′ = 0 we can obtain that:

According to x1 -direction:
∂M x1
∑M O ′′x1 =0 ⇒ M x1 + dx1 − M x1 = M x1 +
∂x1
dx1 − M x1 = 0
(4.46)
∂M x1 Engineering Notation ∂M T
⇒ =0     
→ =0
∂x1 ∂x

According to x2 -direction:
∂M x2
∑M O ′′x2 =0 ⇒ M x2 + dx1 − M x2 − F31dx1 = M x2 +
∂x1
dx1 − M x2 − F31dx1 = 0
(4.47)
∂M x2 Engineering Notation ∂M y
⇒ = F31     
→ = Fz
∂x1 ∂x

According to x3 -direction:
∂M x3
∑M O′′x3 =0 ⇒ M x3 + dx1 − M x3 + F21dx1 = M x3 +
∂x1
dx1 − M x3 + F21dx1 = 0
(4.48)
∂M x3 Engineering Notation ∂M z
⇒ = − F21     
→ = − Fy
∂x1 ∂x

That is, which cause the shearing force F31 is the variation of the moment M x2 along x1 ,
and which cause the shearing force F21 is the variation of the moment M x3 along x1 .

University of Castilla-La Mancha Draft By: Eduardo W. V. Chaves (2016)


Ciudad Real - Spain
356 SOLVING PROBLEMS BY MEANS OF CONTINUUM MECHANICS

∂F11
x3 F11+ dx1 = F11 + dx1
∂x1
Rear face
∂F21
M x3 + dx1 F21+ dx1 = F21 + dx1
∂x1
M x1 x2 ∂F31
F31+ dx1 = F31 + dx1
∂x1
F11 F31+ dx1
O(+ ) ∂M x 2
M x2 + dx1 = F31
∂x1
F21 F31 F21+ dx1
∂M x3
O′′ = − F21
M x2 ∂x1
A F11+ dx1
M x1 + dx1
M x3 dx1 x1

Figure 4.12: Resultant forces and moments in the differential dx1 .

Problem 4.23
Consider the problem established in Problem 4.22, (see Figure 4.5). Now let us suppose
that the stress state on the cross section A is given by:
σ11 0 0
σ ij =  0 0 0 (4.49)
 0 0 0

Express the normal stress field component σ11 ( x 2 , x3 ) in terms of resultant force and
moment, (see equations (4.42) and (4.44)).
Hypothesis (approximation): Consider that the normal stress field on the cross section
varies according to the plane equation.
Solution:
For this particular case, and according to the results established in Problem 4.22, we can
conclude that:
 
 σ11dA  
 F11   ∫   M x1   0

 F  =  A 0  and  M  =  ( x σ )dA 
 21     x2   3 11  ∫ (4.50)
 F31   0  M x   A 
 3 
  − ( x2 σ11 )dA
 A 

Since the stress σ11 ( x 2 , x3 ) varies according to the plane equation, we can adopt:
σ11 ( x 2 , x3 ) = c1 + c 2 x 2 + c3 x3
where c1 , c 2 and c3 are constant to be determined. According to equations in (4.50) we
can obtain:


A

A

F11 = σ11dA = (c1 + c2 x2 + c3 x3 )dA = c1 dA + c2 x2 dA + c3 x3 dA
A

A

A (4.51)
= c1 A + c2 x2 A + c3 x3 A

University of Castilla-La Mancha Draft By: Eduardo W. V. Chaves (2016)


Ciudad Real - Spain
4 THE FUNDAMENTAL EQUATIONS OF CONTINUUM MECHANICS 357

where we have applied the definition of area centroid, (see Figure 4.13 and the
Complementary NOTE 2 at the end of Chapter 1).

x3′
Area centroid:
x3 ∫ x dA 1

x1 = A
Area Centroid – A.C.
∫ dA
A

x3
∫ x dA 2

x2 = A
G x′2 ∫ dA
A

Area: A = ∫ dA
O x2 x2 ∫ x dA 3

x3 = A
A
∫ dA
A

Figure 4.13: Area centroid of cross section A .

From the moment equations (4.50) we can conclude that:


 M x = ( x3 σ11 )dA = [ x3 (c1 + c 2 x 2 + c3 x3 )]dA = c1 x3 dA + c 2 x3 x 2 dA + c3 x32 dA
 2
A
∫ A
∫ A A
∫ A
∫ ∫

∫ ∫ ∫ ∫ ∫
2
 M x3 = − ( x 2 σ11 )dA = − [ x 2 (c1 + c 2 x 2 + c3 x3 )]dA = −c1 x 2 dA − c 2 x 2 dA − c 3 x 2 x3 dA
 A A A A A

Recall that in the Complementary Note at the end of Chapter 1 we have defined some area
geometrical properties such as the inertia tensor of area (second-order pseudo-tensor):
 
∫ ∫ ∫
2 2
 ( x2 + x3 )dA − x1 x2 dA − x1 x3dA 
I11 I12 I13   A A A 
IO( A)ij = I12 I 22 I23  =  − x1 x2 dA
∫ ∫ ( x12 + x32 )dA − x2 x3dA 

 
I13 I 23 I33   A A A 
 A

 − x1 x3dA ∫
− x2 x3dA
A

A
( x12 + x22 )dA

which for our particular reference system we can obtain:
 2 

2
 ( x2 + x3 )dA 0 0 
A   I11 − I12 − I13 
IO( A)ij =  0 2
x3 dA − x2 x3dA =  − I12
∫ ∫ I22 − I23  (4.52)
 
 A A   − I13 − I23 I33 


0 − x2 x3dA
A A
x22 dA 

∫ ∫
Then, the moment equations can be rewritten as follows:
M x = c1 x3 dA + c 2 x3 x 2 dA + c 3 x 32 dA = c1 x3 A − c 2 I 23 + c3 I 22
 2
A A
∫ A
∫ ∫
 (4.53)
∫ ∫ ∫
2
M x3 = −c1 x 2 dA − c 2 x 2 dA − c3 x 2 x 3 dA = −c1 x 2 A − c 2 I 33 + c3 I 23
 A A A

University of Castilla-La Mancha Draft By: Eduardo W. V. Chaves (2016)


Ciudad Real - Spain
358 SOLVING PROBLEMS BY MEANS OF CONTINUUM MECHANICS

Taking into account the equations (4.51) and (4.53) we can obtain the following set of
equations:
 1 x2 x3 
 F11 ≡ N = c1 A + c2 x2 A + c3 x3 A N   c 
 1   I22   
1
I 23
M x 2 ≡ M y = c1 x3 A − c2I23 + c3I 22 ⇒ M y  =  x3 −   c2  (4.54)
 A   A A
M x3 ≡ M z = −c1 x2 A − c2I33 + c3I23 M z   I33 I23   c3 
−x −
 2 A A 
N   c1   c1  N 
1  c  = [ B ]−1 1 M 
⇒ M y  = [ B ]c2  ⇒  2  y (4.55)
A  A 
M z   c3   c3  M z 
1 −1
and [ B]−1 = [adj( B )] , where B ≡ det ( B) = 2 (I223 − I22I33 + 2 Ax2 x3I23 + Ax32I33 + Ax22I22 ) ,
B A

 − (I223 − I22I33 ) − ( x2I23 + x3I33 ) ( x2I22 + x3I23 ) 


 
 A2 A A 
2
− ( x2I22 + x3I23 ) (I23 + x2 x3 A) (−I22 + x3 A) 
[adj( B )] = 
 A A A 
 − (x I + x I ) − (−I33 + x22 A) − (I23 + x2 x3 A) 
 2 23 3 33 
 A A A 
With that the coefficients c1 , c 2 and c3 can be determined:
− (− N I 223 + N I 22 I 33 − M y Ax 2 I 23 − M y Ax3 I 33 + M z Ax 2 I 22 + M z Ax3 I 23 )
c1 =
A(I 223 − I 22 I 33 + 2 Ax 2 x3 I 23 + Ax32 I 33 + Ax 22 I 22 )
− (− N x 2 I 22 − N x3 I 23 + M y I 23 + M y Ax 2 x 3 − M z I 22 + M z Ax 32 )
c2 = (4.56)
(I 223 − I 22 I 33 + 2 Ax 2 x3 I 23 + Ax 32 I 33 + Ax 22 I 22 )
( N x 2 I 23 + N x3 I 33 − M y I 33 + M y Ax 22 + M z I 23 + M z Ax 3 x 2 )
c3 =
(I 223 − I 22 I 33 + 2 Ax 2 x3 I 23 + Ax32 I 33 + Ax 22 I 22 )
Note that the set of equations (4.54) can also be written as follows
N   A Ax2 Ax3   c1 
  
M y  =  Ax3 − I 23 I 22  c2  (4.57)
 M   − Ax − I33 I 23  c3 
 z  2

And the solution can be obtained by means of Cramer’s rule, (see Problem 1.16):
N Ax2 Ax3 A N Ax3 A Ax2 N
My − I 23 I 22 Ax3 My I 22 Ax3 − I 23 My
Mz − I33 I 23 − Ax2 Mz I 23 − Ax2 − I33 Mz
c1 = ; c2 = ; c3 = (4.58)
A Ax2 Ax3 A Ax2 Ax3 A Ax2 Ax3
Ax3 − I 23 I 22 Ax3 − I 23 I 22 Ax3 − I 23 I 22
− Ax2 − I33 I 23 − Ax2 − I33 I 23 − Ax2 − I33 I 23

which must match the equations in (4.56). Once the coefficients ci = ci ( N , M y , M z , Iij , xi )
are obtained, the normal stress component can be defined in terms of resultant forces and
moments:

University of Castilla-La Mancha Draft By: Eduardo W. V. Chaves (2016)


Ciudad Real - Spain
4 THE FUNDAMENTAL EQUATIONS OF CONTINUUM MECHANICS 359

(For any system in which the plane x2 − x3 is


σ11 ( x2 , x3 ) = c1 + c2 x2 + c3 x3 (4.59)
lying on the plane defined by the cross section)
Note that all variables ( N , M y , M z , Iij , xi , xi ) must be expressed in the adopted system.
N
Note that when x2 = x2 and x3 = x3 the equation (4.59) reduces to σ11 ( x2 , x3 ) = .
A
If the adopted system is located at the area centroid, we have x2 = 0 , x3 = 0 . In this
situation the coefficients (4.56) become:
N − ( M y I 23 − M z I 22 ) (− M y I33 + M z I 23 )
c1 = ; c2 = ; c3 = (4.60)
A (I 223 − I 22I33 ) (I 223 − I 22I33 )
and the normal stress component can be evaluated as follows:

N ( M y I23 − M z I 22 ) (− M y I33 + M z I 23 ) (The system is located


σ11 ( x2 , x3 ) = − x2 + x3 (4.61)
A 2
(I 23 − I 22I33 ) (I 223 − I 22I33 ) at the Area Centroid)

or
N ( M z I22 + M y I23 ) ( M y I33 + M z I23 )
σ11 ( x2 , x3 ) = − 2
x2 + x3 (4.62)
A (I22I33 − I23 ) (I22I33 − I223 )
The above two equations can also be written as follows:
N (I 23 x2 + I 33 x3 ) (I x + I x )
σ11 ( x2 , x3 ) = − 2 M y + 222 2 23 3 M z
A (I 23 − I 22I 33 ) (I 23 − I 22I 33 )
or (4.63)
N (I33 x3 − I23 x2 ) (I x − I 23 x3 )
σ11 ( x2 , x3 ) = + 2
M y − 22 2 Mz
A (I 22I33 − I 23 ) (I22I33 − I223 )
In view of the equation (4.61) note that if the adopted system is at the area centroid, we
have x2 = 0 , x3 = 0 , and if the system is also the principal axes of inertia the product of
area inertia is zero, i.e. I 23 = 0 . In this situation the coefficients (4.56) become:
N − Mz My
c1 = ; c2 = ; c3 = (4.64)
A I 33 I 22
and the normal stress σ11 can be obtained as follows:

N Mz My (The system is located at the Area Centroid


σ11 ( x2 , x3 ) = − x2 + x3 (4.65)
A I 33 I 22 and is the principal axes of inertia)

NOTE 1: The Neutral Axis (N.A.)


The neutral axis is defined by the absence of normal stress σ11 ( x2 , x3 ) = 0 and the neutral
axis equation can be obtained as follows:
By considering an arbitrary system, (see equation (4.59)), i.e.:
σ11 ( x2 , x3 ) = c1 + c2 x2 + c3 x3 = 0 (Neutral Axis -for any system in which the
plane x2 − x3 is lying on the plane defined by (4.66)
⇒ c2 x2 + c3 x3 = −c1
the cross section)
where the coefficients ci are given by the equations in (4.58).

University of Castilla-La Mancha Draft By: Eduardo W. V. Chaves (2016)


Ciudad Real - Spain
360 SOLVING PROBLEMS BY MEANS OF CONTINUUM MECHANICS

If the adopted system is located at the area centroid, the neutral axis can be defined by
means of the equation in (4.61), i.e.:
N ( M y I 23 − M z I 22 ) (− M y I33 + M z I 23 ) (The system is
σ11 ( x2 , x3 ) = − 2
x2 + x3 = 0
A (I 23 − I 22I33 ) (I 223 − I 22I33 ) located at the Area (4.67)
⇒ − A( M y I 23 − M z I 22 ) x2 + A(− M y I 33 + M z I 23 ) x3 = − N (I 23 − I 22I 33 ) Centroid)
2

which represents a straight line defined on the plane ( x2 − x3 ) . When N ≠ 0 , the canonic
form of the above equation is represented by:
x2 x3 N (I 223 − I 22I33 ) N (I 223 − I 22I33 )
+ =1 with a = ; b= (4.68)
a b A( M y I 23 − M z I 22 ) A( M y I33 − M z I 23 )

where a and b are the points in which the Neutral Axis intercepts the axis x2 and x3
respectively.
Note also that, when N = 0 the neutral axis pass through the area centroid. And by means
of the equation in (4.61) the neutral line can be established as follows
( M y I 23 − M z I 22 ) (− M y I33 + M z I 23 )
σ11 ( x2 , x3 ) = − x2 + x3 = 0
(I 223 − I 22I33 ) (I 223 − I 22I33 )
(4.69)
( M y I 23 − M z I 22 )
⇒ ( − M y I33 + M z I 23 ) x3 − ( M y I 23 − M z I 22 ) x2 = 0 ⇒ x3 = x2
(− M y I33 + M z I 23 )

And taking into account that I 23 = − ∫ x2 x3 dA = − I 23 ∴ I 23 = ∫ x2 x3 dA , the equation in


A A
(4.69) can be rewritten as follows:
( M y I33 + M z I 23 ) x3 − ( M y I 23 + M z I 22 ) x2 = 0 (4.70)

NOTE 2: An interesting relationship is the derivative of σ11 with respect to x1 . If we take


into account the equation (4.59) we can obtain:
∂σ11 ∂c1 ∂c2 ∂c (For any system in which the plane x2 − x3 is
= + x2 + 3 x3 (4.71)
∂x1 ∂x1 ∂x1 ∂x1 lying on the plane defined by the cross section)

If we consider that the cross section is constant along x1 -direction and N is also constant
along x1 -direction or absent we can obtain
 N Ax2 Ax3  N Ax2 Ax3
  ∂
 My − I 23 I 22  M y − I 23 I 22
  ∂x1 A Ax2 Ax3
∂c1 ∂  M z − I33 I 23  M z − I33 I 23
= = ∴ X = Ax3 − I 23 I 22
∂x1 ∂x1  A Ax2 Ax3  X
  − Ax2 − I33 I 23
 Ax3 − I 23 I 22 
 − Ax2 − I33 I 23 

University of Castilla-La Mancha Draft By: Eduardo W. V. Chaves (2016)


Ciudad Real - Spain
4 THE FUNDAMENTAL EQUATIONS OF CONTINUUM MECHANICS 361

∂N
Ax2 Ax3
∂x1 0 Ax2 Ax3
∂c ∂
1 My 1
⇒ 1= − I 23 I 22 = Fz − I 23 I 22
∂x1 X ∂x1 X
∂M z − Fy − I33 I 23
− I33 I 23
∂x1
where we have applied the equations (4.47) and (4.48). In the same fashion we can obtain
A N Ax3 ∂N
∂ A Ax3
Ax3 My I 22 ∂x1
∂x1 A 0 Ax3
∂c2 − Ax2 Mz I 23 1 ∂M y 1
= = Ax3 I 22 = Ax3 Fz I 22
∂x1 X X ∂x1 X
∂M z − Ax2 − Fy I23
− Ax2 I 23
∂x1
and
A Ax2 N ∂N
∂ A Ax2
Ax3 − I 23 My ∂x1
∂x1 A Ax2 0
∂c3 − Ax2 − I33 Mz 1 ∂M y 1
= = Ax3 − I 23 = Ax3 − I 23 Fz
∂x1 X X ∂x1 X
∂M z − Ax2 − I33 − Fy
− Ax2 − I33
∂x1

Then, the equation (4.71) can be written as follows

 0 Ax2 Ax3 A 0 Ax3 A Ax2 0 


∂σ11 1  
=  Fz − I 23 I 22 + Ax3 Fz I 22 x2 + Ax3 − I 23 Fz x3 
∂x1 X   (4.72)
 − Fy − I 33 I 23 − Ax2 − Fy I 23 − Ax2 − I 33 − Fy 
 
(For any system in which the plane x2 − x3 is lying on the plane defined by the cross section)
If we are adopting the system at the Area Centroid and if we take the derivative of the
equation (4.63) with respect to x1 we can obtain:

∂σ11 (I x + I x ) ∂M y (I 22 x2 + I 23 x3 ) ∂M z (The system is located at


= − 232 2 33 3 + 2 (4.73)
∂x1 (I 23 − I 22I33 ) ∂x1 (I 23 − I 22I33 ) ∂x1 the Area Centroid)
or
∂σ11 (I33 x3 − IO 23 x2 ) ∂M y (I22 x2 − IO 23 x3 ) ∂M z
= −
∂x1 (I22I33 − I223 ) ∂x1 (I22I33 − I223 ) ∂x1
and by considering the equations (4.47) and (4.48), the equation (4.73) becomes:
∂σ11 (I x + I x ) (I x + I x ) (The system is located at
= − 232 2 33 3 Fz − 222 2 23 3 Fy (4.74)
∂x1 (I23 − I22I33 ) (I23 − I22I33 ) the Area Centroid)
or
∂σ11 (I33 x3 − IO 23 x2 ) (I x − IO 23 x3 )
= Fz + 22 2 Fy
∂x1 2
(I22I33 − I23 ) (I22I33 − I223 )

University of Castilla-La Mancha Draft By: Eduardo W. V. Chaves (2016)


Ciudad Real - Spain
362 SOLVING PROBLEMS BY MEANS OF CONTINUUM MECHANICS

Example: Let us consider the cross section described in Figure 4.14, (Ugural&Fenster
(1984)). And the geometrical characteristics for the quadrangular cross section are
described in Figure 4.15. The cross section described in Figure 4.14 can be constructed by
the two rectangles as described in Figure 4.16.

X 3, Z

0.02

0.15

0.02
O X 2 ,Y
0.15

Figure 4.14: Cross section – Dimensions in meter ( m ).

z′ The inertia tensor of area


 
 Ig 22 + I g 33 0 0 
 ab 3 
y′ I gij = 0 0 
b  12 
g
 ba 3 
 0 0
 12 
Area: A = ab
a g - area centroid

Figure 4.15: Rectangular cross section.

University of Castilla-La Mancha Draft By: Eduardo W. V. Chaves (2016)


Ciudad Real - Spain
4 THE FUNDAMENTAL EQUATIONS OF CONTINUUM MECHANICS 363

Area Centroid (A.C.) Calculation:


Geometrical decomposition, (see Figure 4.16).

Position vectors Areas


X 3, Z r
X (1) = 0.01Jˆ + 0.075Kˆ A(1) = 0.02 × 0.15 = 0.003m 2
0.02 r
X ( 2) = 0.085Jˆ + 0.01K
ˆ A( 2) = 0.13 × 0.02 = 0.0026m 2

(1) A = A(1) + A( 2) = 0.0056m 2

0.13
0.075 r
X (1)
r
X ( 2) ( 2) 0.02
O B X 2 ,Y
0.085

Figure 4.16: Geometric decomposition of the cross section.


By using equations described in Figure 4.13 we can obtain the Area Centroid related to the
system X , Y , Z :
2

∫ X 2 dA ∑A
a =1
(a )
X 2( a )
A(1) X 2(1) + A( 2 ) X 2( 2) 0.003 × 0.01 + 0.0026 × 0.085
X2 ≡ Y = A
= = =
∫ dA
A
A ( A(1) + A( 2) ) 0.0056

= 0.04482143m ≈ 0.045m
2

∫ X 3dA ∑A
a =1
(a)
X 3( a )
A(1) X 3(1) + A( 2) X 3( 2 ) 0.003 × 0.075 + 0.0026 × 0.01
X3 ≡ Z = A
= = =
∫ dA
A
A ( A(1) + A( 2 ) ) 0.0056

= 0.04482143m ≈ 0.045m
Then, the area centroid of the cross section is given by Y = 0.045m; Z = 0.045m , (see Figure
4.17). Note that the cross section has a symmetric axis, so, as expected the point A.C. lies
on the symmetric axis. At the point A.C. we define a new system x, y, z , (see Figure 4.17).
r r r
The position vector x ( a ) = X ( a ) − X , (see Figure 4.17), can be obtained as follows:
r r r
x (1) = X (1) − X = ( X 2(1) − Y )ˆj + ( X 3(1) − Z )kˆ = (0.01 − 0.045)ˆj + (0.075 − 0.045)kˆ
= −0.035ˆj + 0.03kˆ
r r r
x ( 2) = X ( 2) − X = ( X 2( 2) − Y )ˆj + ( X 3( 2) − Z )kˆ = (0.085 − 0.045)ˆj + (0.01 − 0.045)kˆ
= 0.04ˆj − 0.035kˆ

University of Castilla-La Mancha Draft By: Eduardo W. V. Chaves (2016)


Ciudad Real - Spain
364 SOLVING PROBLEMS BY MEANS OF CONTINUUM MECHANICS

r r r r r r
X 3, Z Y = 0.045 X + x (a) = X (a) ⇒ x (a) = X (a) − X

0.02 r
X = 0.045Jˆ + 0.045K ˆ
r
X (1) = 0.01Jˆ + 0.075Kˆ
(1) r ( 2)
z X = 0.085Jˆ + 0.01K ˆ

r
0.15 k̂ x (1) = −0.035ˆj + 0.03kˆ
r r
g x (1) y x ( 2 ) = 0.04ˆj − 0.035kˆ
A.C.
r ĵ
G r
X x ( 2)
r Z = 0.045
X (1)
K̂ r
X ( 2) ( 2) 0.02
g
O Ĵ X 2 ,Y

Figure 4.17: Area Centroid - Cross section.

Find the inertia tensor of area for the system located at the Area Centroid (A.C.):
We will use the definition of the parallel theorem, (see Chapter 1 COMPLEMENTARY
NOTES at the end of the Chapter 1). We use IG( sys
r ) (1)r ( 2r)
x = I Gx + I Gx , (see Problem 4.32 -NOTA
r r r r
1), where IGxr = I g − A [( x ⊗ x ) − ( x ⋅ x ) 1] (the Steiner’s theorem), which in indicial
notation becomes:
IGxr ij = Igij − A[ xi x j − ( x12 + x22 + x32 )δ ij ]

or in matrix notation:
 I11 I12 I13   x22 + x32 − x1 x2 − x1 x3 
   
IGxr ij =  I12 I22 I23  + A − x1 x2 x12 + x32 − x2 x3 
I I23 I33   − x1 x3 − x2 x3 x12 + x22 
 13 
(4.75)
 I11 0 0  x22 + x32 0 0 
   
=0 I22 I23  + A 0 x1 + x32
2
− x2 x3 
0 I23 I33   0 − x2 x3 x12 + x22 
 
Note that this problem can be treated as two-dimensional case on the plane defined by
( x 2 − x3 ) .

Rectangle 1 - IG(1x)r
 ab 3   0.02 × 0.153 
 I22 0
I23   12   0  562 .5 0 
(1)
( I g )ij =  12 × 10 −8 m 4
= 3  =  3 =  
 I23 I33   ba   0.15 × 0.02   0 10.0
0 0
 
12    12 

r (1)
Area centroid vector position: x = −0.035 ˆj + 0.03kˆ , ( x1(1) = 0, x 2( 2) = −0.035, x3( 2) = 0.03) :

University of Castilla-La Mancha Draft By: Eduardo W. V. Chaves (2016)


Ciudad Real - Spain
4 THE FUNDAMENTAL EQUATIONS OF CONTINUUM MECHANICS 365

I I23   2
(1) x1 + x3
2
− x2 x3 
(I G(1x)r )ij =  22 + A  
 I23 I33   − x2 x3 x12 + x22 
562 .5
=
0 
× 10 −8
+ 0 . 003
 (0.03)2 − (− 0.035 )(0.03)
  
 0 10.0 − (− 0.035 )(0.03) (− 0.035 )2 
832.5 315  −8
=  × 10
 315 377 . 5 

Rectangle 2 - IG( 2xr)


 ab 3   0.13 × 0.023 
I I23   12 0   0 
( I g( 2 ) )ij =  22 12
= = 3
 I23 I33   ba 3   0.02 × 0.13 
0 0
 12   12 
8.667 0  −8 4
=  × 10 m
 0 366 . 167 
r
Area Centroid vector position: x ( 2 ) = 0.04 ˆj − 0.035 kˆ , ( x1(1) = 0, x 2( 2) = 0.04, x3( 2) = −0.035) :
By applying the equation (4.75) we can obtain:
I I23   2
( 2 ) x1 + x3
2
− x2 x3 
(I G( 2xr) )ij =  22 + A  
 I23 I33   − x2 x3 x12 + x22 
8.667 0  −8
 (− 0.035 )2 − (0.04 )(− 0.035 )
=  × 10 + 0 . 0026  
 0 366.167  − (0.04 )(− 0.035 ) (0.04 )2 
327.16 364  −8
=  × 10
 364 782 . 167 
Then, we can calculate the inertia tensor of the cross section related to the system located
at the Area Centroid:
832.5 315  327 .16 364 
(I G( sys
r ) (1)r ( 2r)
x ) ij = (I Gx )ij + (I Gx ) ij =   × 10 −8 +   × 10 −8
 315 377.5  364 782.167 
(4.76)
1159 .66 679  −8 4
=  × 10 m
 679 1159 . 66 

Calculation of the normal stress at any point of the cross section:


Let us suppose that on the cross section is acting the moment M z = 11000.0 Nm ,
M y = 0 and N = 0 . To obtain the normal stress σ11 ( x 2 , x3 ) we can apply the equation
(4.61), i.e.:
( M y I23 − M z I22 ) (− M y I33 + M z I23 )
σ11 ( x2 , x3 ) = − x2 + x3
(I 223 − I 22I33 ) (I223 − I 22I33 )
− 11000 × 1159.66 × 10−8 11000 × 679 × 10−8
=− −8 2 −8 2
x2 + x3
((679 × 10 ) − (1159.66 × 10 ) ) ((679 × 10−8 ) 2 − (1159.66 × 10−8 )2 )
= (−1443.39 × 106 x2 − 845.129 × 106 x3 ) Pa = (−1443.39 x2 − 845.129 x3 ) MPa

University of Castilla-La Mancha Draft By: Eduardo W. V. Chaves (2016)


Ciudad Real - Spain
366 SOLVING PROBLEMS BY MEANS OF CONTINUUM MECHANICS

For the pointsr O , B , C , D , E and F , (see Figure 4.19), and by using the definition
r r
x ( P ) = X ( P ) − X , the normal stresses are given by:

Point coordinates (meter-m) Normal stress

X 2 [ m] X 3 [ m] x2 = X 2 − Y [m] x3 = X 3 − Z [m] σ11[ MPa]

O 0 .0 0 .0 − 0.045 − 0.045 102.98


B 0.15 0 .0 0.105 − 0.045 − 113.52
C 0.15 0.02 0.105 − 0.025 − 130.42
D 0.02 0.02 − 0.025 − 0.025 57.21
E 0 .0 0.15 − 0.045 0.105 − 23.78
F 0.02 0.15 − 0.025 0.105 − 52.65

Y = 0.045
A (1) = 0.02 × 0.15 = 0.003m 2
Z A ( 2 ) = 0.13 × 0.02 = 0.0026m 2
r
0.02 axis of symmetry x (1) = −0.035ˆj + 0.03kˆ
r
x ( 2 ) = 0.04ˆj − 0.035kˆ

Mz

z
0.15 r
x (1)
A.C. y
r ( 2)
(1) G x
Z = 0.045
( 2) 0.02
O B Y
0.13
2
= 0.065
0.13

Cross section
0.15 (Dimensions in meter-m)

Figure 4.18: Cross section.


The Neutral Axis can be obtained by means of the equation (4.69) with σ11 ( x 2 , x3 ) = 0 , i.e.:
( M y I 23 − M z I 22 ) (− M y I 33 + M z I 23 )
σ11 ( x2 , x3 ) = − x2 + x3 = 0
(I 223 − I 22 I 33 ) (I 223 − I 22 I 33 )
⇒ (− M y I33 + M z I 23 ) x3 − ( M y I 23 − M z I 22 ) x2 = 0

University of Castilla-La Mancha Draft By: Eduardo W. V. Chaves (2016)


Ciudad Real - Spain
4 THE FUNDAMENTAL EQUATIONS OF CONTINUUM MECHANICS 367

⇒ (0 + 11000 × 679 ×10 −8 ) x3 − (0 − 11000 ×1159.667 × 10 −8 ) x2 = 0


⇒ 7.469 ×10 −2 x3 + 12.756337 × 10 −2 x2 = 0
− 12.756337 ×10 −2
⇒ x3 = x2 = −1.708 x2 or z = −1.708 y
7.469 × 10 − 2
⇒ z = tan(α ) y ∴ α = arctan(−1.708) = −59.65º
Calculation of principal inertia tensor of area:
The inertia tensor of area is a second-order pseudo-tensor and this tensor has same
transformation described for a second-order tensor. For example, for component
transformation law for a second-order tensor:
I′O = A ⋅ I O ⋅ A T Inertia tensor components after a base
(4.77)
I′O ij = Aip I O pj Aqj change (rotation)

where A is the transformation matrix from the x1 x 2 x3 -system to x1′x′2 x3′ -system.
For two-dimensional problem, (see Problem 1.99), we have shown that:
 T11′   cos θ sin 2 θ 2 cos θ sin θ   T11 
2

T′  =  
 22   sin θ
2
cos 2 θ − 2 sin θ cos θ   T22  (4.78)
 T12′   − sin θ cos θ cos θ sin θ cos 2 θ − sin 2 θ   T12 

and the principal direction is characterized by:

1  2 T12 
θ = arctan  (4.79)
2  T11 − T22 

where Tij (i, j = 1,2) are the second-order tensor components for 2D problems.
For the problem proposed here
1159.66 679  −8 4
(I G( sys
r )
x )ij =   × 10 m
 679 1159 . 66 
Then, we can obtain:
1  2 T12  1  2(679)  1
θ = arctan  = arctan  = arctan(∞ ) = 45º (4.80)
2  T11 − T22  2  (1159.66) − (1159.66)  2
And by applying the equation in (4.78) when θ = 45º we can obtain:
 T11′   cos θ sin 2 θ 2 cosθ sin θ   T11 
2

T′  =  
 22   sin θ
2
cos θ
2
− 2 sin θ cosθ   T22 
 T12′  − sin θ cosθ cosθ sin θ cos 2 θ − sin 2 θ   T12 
 
(4.81)
 0.5 0.5 1  1159.66 18.39 
=  0.5 0.5 − 1 1159.66 × 10 =  4.807 ×10 −6 m 4
    −8

− 0.5 0.5 0   679   0 

Then, for the principal system of the inertia tensor y ′ − z ′ we have:


18.39 0  −6 4
(I′G( xPr _ sys ) )ij =   × 10 m
 0 4. 807 

University of Castilla-La Mancha Draft By: Eduardo W. V. Chaves (2016)


Ciudad Real - Spain
368 SOLVING PROBLEMS BY MEANS OF CONTINUUM MECHANICS

Y = 0.045
Neutral axis related to the system -
O −Y − Z :
Z z = −1.708 y
0.02
axis of symmetry ⇒ ( Z − 0.045) = −1.708(Y − 0.045)
⇒ Z = −1.708Y + 0.12186
E
y′
F Mz
z′

z
0.15 θ = 45º
A.C. y
C
D Z = 0.045
0.02
O B Y

0.15
z = −1.708 y (Neutral Axis - σ11 = 0 )

Figure 4.19: Cross section (dimensions in m ).

X 3, Z

F
E
σ11 σ11
E
F

C
σ11

D
σ11 B
σ11

O B X 2 ,Y

A
σ11

Figure 4.20: Normal stress distribution on the cross section.

University of Castilla-La Mancha Draft By: Eduardo W. V. Chaves (2016)


Ciudad Real - Spain
4 THE FUNDAMENTAL EQUATIONS OF CONTINUUM MECHANICS 369

NOTE 3: The normal stress components could also have been obtained by adopting the
system X 2 − X 3 , in this case we have to apply the equation in (4.59),
σ11 = c1 + c 2 X 2 + c3 X 3 , and all variables must be expressed in the system X 2 − X 3 . The
inertia tensor for this system can be obtained by considering Figure 4.16 and equation
(4.75) in which x2 and x3 are now related to the system X 2 − X 3 :
 ab 3 
I I23   X 32 − X 2 X 3   12 0   X 32 − X2X3
I (OqXr) ij =  22 + A =  + ab  
 I23 I33  − X 2 X 3 X 22   ba 3  − X 2 X 3 X 22 
0 (4.82)
 12 
ab b 2 + 12 X 32 − 12 X 2 X 3 
⇒ I (OqXr) ij =  
12  − 12 X 2 X 3 a 2 + 12 X 22 
where
Rectangle q = 1 : a = 0.02 , b = 0.15 , X 2 = 0.01 , X 3 = 0.075 :

ab b 2 + 12 X 32 − 12 X 2 X 3  (0.02)(0.15) (0.15) 2 + 12(0.075) 2 − 12(0.01)(0.075) 


I(O1X)r ij =  =  
12  − 12 X 2 X 3 a 2 + 12 X 22  12  − 12(0.01)(0.075) (0.02) 2 + 12(0.01) 2 
 22.5 − 2.25
= × 10 − 6 m 4
 − 2.25 0.4 

Rectangle q = 2 : a = 0.13 , b = 0.02 , X 2 = 0.085 , X 3 = 0.01 :

ab b 2 + 12 X 32 − 12 X 2 X 3  (0.13)(0.02) (0.02) 2 + 12(0.01) 2 − 12(0.085)(0.01) 


I(O2Xr) ij =  =  
12  − 12 X 2 X 3 a 2 + 12 X 22  12  − 12(0.085)(0.01) (0.13) 2 + 12(0.085) 2 
0.3466667 − 2.21 
= × 10 − 6 m 4
 − 2.21 22.4466667 
Then
  22.5 − 2.25 0.3466667 − 2.21
I O( Sys
r ) = I (1)r + I ( 2 r) =   +  × 10 −6
X ij OX ij OX ij
  − 2.25 0.4   − 2.21 22.4466667  
I 22 I 23   22.8466667 − 4.46 
⇒ I O( Sys
r ) =
I = × 10 −6 m 4
X ij
 23 I 33   − 4.46 22.8466667 

And by considering the parameters, x2 ← X 2 = 0.04482143m , x3 ← X 3 = 0.04482143m ,


A = 0.0056 m 2 , N = 0 , M y = 0 and M z = 11000 Nm , the coefficients ci can be obtained as
follows:
N Ax2 Ax3 A N Ax3
My − I 23 I 22 Ax3 My I 22
Mz − I33 I 23 − Ax2 Mz I 23 Pa
c1 = = 102.581×106 Pa ; c2 = = −1443.46 × 106
A Ax2 Ax3 A Ax2 Ax3 m
Ax3 − I 23 I 22 Ax3 − I 23 I 22
− Ax2 − I33 I 23 − Ax2 − I33 I 23

University of Castilla-La Mancha Draft By: Eduardo W. V. Chaves (2016)


Ciudad Real - Spain
370 SOLVING PROBLEMS BY MEANS OF CONTINUUM MECHANICS

A Ax2 N
Ax3 − I 23 My
− Ax2 − I 33 Mz Pa
c3 = = −845.2 × 10 6
A Ax2 Ax3 m
Ax3 − I 23 I 22
− Ax2 − I 33 I 23
Thus
σ11 ( X 2 , X 3 ) = c1 + c2 X 2 + c3 X 3 = 102.581 − 1443.46 X 2 − 845.2 X 3 ( MPa)
For example, for the point O we have
σ11 ( X 2 = 0, X 3 = 0) = 102.581 − 1443.46 X 2 − 845.2 X 3 = 102.581( MPa)
Point B : σ11 ( X 2 = 0.15, X 3 = 0) = 102.581 − 1443.46 X 2 − 845.2 X 3 = −113.938( MPa)
Point E : σ11 ( X 2 = 0, X 3 = 0.15) = 102.581 − 1443.46 X 2 − 845.2 X 3 = −24.199( MPa)
The Neutral axis
If we are adopting the system X 2 − X 3 , the neutral axis can be obtained by means of the
equation (4.66):
− c1 c2
c2 X 2 + c3 X 3 = −c1 ⇒ c3 X 3 = −c2 X 2 − c1 ⇒ X3 = − X2
c3 c3
− (102.581 × 10 6 ) (−1443.46 × 10 6 )
X3 = − X2 ⇒ X 3 = 0.12137 − 1.7078 X 2
(−845.2 × 10 6 ) (−845.2 × 10 6 )
which matches the equation presented in Figure 4.19.
The inertia tensor at the Area Centroid, (see equation in (4.75)), can be obtained by means
of the Steiner’s theorem:
r r r r r r r r
I O( Sys r ) − A [( X ⊗ X ) − ( X ⋅ X ) 1]
r ) = I ( Sys ⇒ IG( Sys r ) + A [( X ⊗ X ) − ( X ⋅ X ) 1]
r ) = I ( Sys
X Gx x OX

whose components are:


 IG 22 IG 23   X 32 − X2X3
( IG( Sys
r )
x ) ij = 
( Sys )
 = I OXr ij − A  (4.83)
 IG 23 IG 33  − X 2 X 3 X 22 
r
where X is the vector position of the Area Centroid of the cross section related to the
r
r ) , A = 0.0056 m 2 , X = 0.04482143m ,
(Sys
system OX . By substituting the variable values ( I OX 2

X 3 = 0.04482143m ) we can obtain:

22.8466667 − 4.46  −6  (0.0448) 2 − (0.0448) 2 


( IG( Sys
r )
x ) ij =  × 10 − ( 0 . 0056 )  
 − 4.46 22.8466667  − (0.0448)
2
(0.0448) 2 
1159 .6487 679.01793 −8 4
=  × 10 m
 679 . 01793 1159 . 6487 
which matches the equation in (4.76).

University of Castilla-La Mancha Draft By: Eduardo W. V. Chaves (2016)


Ciudad Real - Spain
4 THE FUNDAMENTAL EQUATIONS OF CONTINUUM MECHANICS 371

NOTE 4: Let us consider the rectangle as the one indicated in Figure 4.21. Next we will
r
obtain the Inertia Tensor of Area related to the system OX .

Node Coordinates
x3′ x3
X3 Node: i ( X 2(i ) , X 3(i ) )
x′2
a j Node: j ( X 2( j ) , X 3( j ) )
α
g x2
i
t r r
Transformation matrix from gx to gx ′
r  cos α sin α   l m
X (g) A = =
 − sin α cos α   − m l 
a = ( X 2( j ) − X 2(i ) ) 2 + ( X 3( j ) − X 3(i ) ) 2

X 2( j ) − X 2(i )
l = cosα =
O X2 a
X − X 3(i )
( j)
m = sinα = 3
a

Figure 4.21
r
The inertia tensor of area related to the system gx ′ is given by

1  at 3 0
′ r′ =
I gx  
ij 12  0 ta 3 
Taking into account the component transformation law for a second-order tensor, (see
r
equation (4.77)), we can obtain the inertia tensor of area in the system gx , i.e.:
1 l − m   at 3 0  l m
I gxr ij = Aip I g′ xr ′ ij Aqj = 
12  m l   0 
ta 3   − m l 
at (t 2 l 2 + a 2 m 2 ) lm (t 2 − a 2 ) 
⇒ I gxr ij =  
12  lm (t 2 − a 2 ) (t 2 m 2 + a 2 l 2 )
Then, by means of the equation in (4.75) we can obtain:
I I 23   ( X 3( g ) ) 2 − X 2( g ) X 3( g ) 
I OXr ij =  22  = Igxr ij + A 
I 23 I 33  (g) (g)
− X 2 X 3 ( X 2( g ) ) 2 

X 2(i ) + X 2( j ) X (i ) + X 3( j )
where A = at , X 2( g ) = and X 3( g ) = 3 . The above equation can also be
2 2
written as follows

at  t 2 l 2 + a 2 m 2 + 12( X 3( g ) ) 2 lm (t 2 − a 2 ) − 12 X 2( g ) X 3( g ) 
I OXr ij =   (4.84)
12 lm (t 2 − a 2 ) − 12 X 2( g ) X 3( g ) t 2 m 2 + a 2 l 2 + 12( X 2( g ) ) 2 

University of Castilla-La Mancha Draft By: Eduardo W. V. Chaves (2016)


Ciudad Real - Spain
372 SOLVING PROBLEMS BY MEANS OF CONTINUUM MECHANICS

Problem 4.24
Consider a cross section described in Figure 4.22, (Buchanan (1988)), in which acts only
the normal stress σ11 . Knowing that the moments at the Area Centroid are
M y = −67.5kNm and M z = −28.13kNm , obtain the normal stress σ11 at the points O , B ,
C and D .

X 3, Z 0.02

0.02
D

0 .3

0 .2

0.04

O B X 2 ,Y
0 .3

Figure 4.22: Cross section – Dimensions in meter ( m ).


Solution:
Geometry decomposition:

X 3, Z r
0.02 X (1) = 0.01Jˆ + 0.1K
ˆ
r ( 2)
X = 0.15Jˆ + 0.02K ˆ
(3) r ( 3)
0.02 X = 0.29Jˆ + 0.15K ˆ

A(1) = 0.02 × 0.2 = 0.004m 2


(1) A( 2) = 0.26 × 0.04 = 0.0104m 2
0 .3
r A(3) = 0.02 × 0.3 = 0.006m 2
X ( 3)
0 .2
r A = A(1) + A( 2 ) + A(3) = 0.0204m 2
X (1) r
X ( 2) (2) 0.04

O X 2 ,Y

0 .3

Figure 4.23: Geometric decomposition of the cross section.

University of Castilla-La Mancha Draft By: Eduardo W. V. Chaves (2016)


Ciudad Real - Spain
4 THE FUNDAMENTAL EQUATIONS OF CONTINUUM MECHANICS 373

Calculation of the Area Centroid- G :


3

∫ X 2 dA ∑A
a =1
(a)
X 2( a )
A(1) X 2(1) + A( 2 ) X 2( 2 ) + A(3) X 2(3)
X2 ≡ Y = A
= =
∫ dA
A
A ( A(1) + A( 2 ) + A(3) )

0.004 × 0.01 + 0.0104 × 0.15 + 0.006 × 0.29


= = 0.163723m
0.0204
3

∫ X dA ∑ A
3
a =1
(a )
X 3( a )
A(1) X 3(1) + A( 2 ) X 3( 2 ) + A(3) X 3(3)
X3 ≡ Z = A
= =
∫ dA
A
A ( A(1) + A( 2 ) + A(3) )

0.004 × 0.1 + 0.0104 × 0.02 + 0.006 × 0.15


= = 0.073922m
0.0204
r
Calculation of the position vector - x ( a )
r r r
The position vector x ( a ) = X ( a ) − X can be obtained as follows:
r r r
x (1) = X (1) − X = ( X 2(1) − Y )ˆj + ( X 3(1) − Z )kˆ = (0.01 − 0.163723)ˆj + (0.1 − 0.073922)kˆ
= −0.153723ˆj + 0.026078kˆ
r ( 2) r ( 2) r
x = X − X = ( X 2( 2 ) − Y )ˆj + ( X 3( 2 ) − Z )kˆ = (0.15 − 0.163723)ˆj + (0.02 − 0.073922)kˆ
= −0.013723ˆj − 0.053922kˆ
r ( 3) r ( 3) r
x = X − X = ( X 2(3) − Y )ˆj + ( X 3(3) − Z )kˆ = (0.29 − 0.163723)ˆj + (0.15 − 0.073922)kˆ
= 0.126277ˆj + 0.076078kˆ
Calculation of the inertia tensor at the Area Centroid:
Rectangle 1 - IG(1xr) : a = 0.02; b = 0.2
  0.02 × 0.23
 ab 3 
 I 0
I23   12
  0  1.333 0 
Ig(1) ij =  22 12 −5 4
=
3 =  3 =   × 10 m
 I23 I33  
ba   0 . 2 × 0 . 02   0 0 . 01333 
0 0
12  
 12 
r
Area centroid vector position: x (1) = −0.153723 ˆj + 0.026078kˆ , A(1) = 0.004m 2 :
I I23   2
(1) x1 + x3
2
− x2 x3 
(I G(1x)r )ij =  22 + A  
 I23 I33   − x2 x3 x12 + x22 
1.333
=
0 
× 10 −5
+ 0 . 004
 (0.026078 )2 − (− 0.153723 )(0.026078 )
  
 0 0.01333  − (− 0.153723 )(0.026078 ) (− 0.153723 )2 
1.605 1.604  −5 4
=  × 10 m
1 . 604 9 . 466 
( 2r)
Rectangle 2 - I Gx : a = 0.26; b = 0.04
 ab 3   0.26 × 0.043 
I I23   0   0  0.13867 0 
Ig( 2 ) ij =  22 12 12 × 10 − 5 m 4
= = 3 =  
 I23 I33   ba 3   0.04 × 0.26   0 5.859 
0 0
 12   12 

University of Castilla-La Mancha Draft By: Eduardo W. V. Chaves (2016)


Ciudad Real - Spain
374 SOLVING PROBLEMS BY MEANS OF CONTINUUM MECHANICS

r
Area centroid vector position: x ( 2 ) = −0.013723ˆj − 0.053922 kˆ , A( 2) = 0.0104m 2 :
I I23   2
( 2 ) x1 + x3
2
− x2 x3 
(I G( 2xr) )ij =  22  + A  
 I23 I33   − x2 x3 x12 + x22 
0.13867
=
0  −5

× 10 + 0.0104 
(0.026078 )
2
− (0.013723 )(0.053922 )
 
 0 5.859  − (0.013723 )(0.053922 ) (− 0.013723 )2 
 3.163 − 0.76957 
=  × 10 −5 m 4
 − 0.76957 6.0545 

Rectangle 3 - IG(3xr) : a = 0.02; b = 0.3


  0.02 × 0.33
 ab 3 
 I 0 
I23   12
 0   4 .5 0 
Ig( 3) ij =  22 12 −5 4
=
3 =  3 =   × 10 m
 I23 I33  
ba   0 . 3 × 0 . 02   0 0 . 02 
0 0
12  
 12 
r
Area centroid vector position: x (3) = 0.126277 ˆj + 0.076078 kˆ , A(3) = 0.006m 2 :
I I23   2
( 3) x1 + x3
2
− x2 x3 
(I G( 3xr) )ij =  22  + A  
 I23 I33   − x2 x3 x12 + x22 
 4 .5
=
0 
× 10 −5
+ 0 . 006
 (0.076078 )2 − (0.126277 )(0.076078 )
  
 0 0.02   − (0.126277 )(0.076078 ) (0.126277 )2 
 7.9727 − 5.764 
=  × 10 −5 m 4
 − 5.764 9.5875 
Then, we can calculate the inertia tensor of the cross section related to the system located
at the Area Centroid - G :
Nb
(I G( sys
r )
x ) ij = ∑ (I
a =1
( ar)
Gx ) ij = (I G(1x)r )ij + (I G( 2xr) )ij + (IG( 3xr) )ij

 1.605 1.604   3.163 − 0.76957   7.9727 − 5.764  


=   +  +   × 10 − 5 (4.85)
 1.604 9.466   − 0.76957 6.0545   − 5.764 9.5875  
 1.2741 − 0.49302  I 22 I 23 
= × 10 − 4 m 4 = 
 − 0.49302 2.5108  
I23 I33 
Calculation of the normal stress:
(− M y I 33 + M z I 23 ) x 3 − ( M y I 23 − M z I 22 ) x 2
σ11 ( x 2 , x 3 ) =
(I 223 − I 22 I 33 )
(4.86)
 M y I 23 − M z I 22   − M y I 33 + M z I 23 
⇒ σ11 ( x 2 , x 3 ) = − x +  x
 I 223 − I 22 I 33  2
 I 223 − I 22 I 33  3
   

x ) ij , (see equation (4.85)), and M y = −67.5kNm and


Taking into account the values for (I (Gsys
r )

M z = −28.13kNm , the above equation becomes:


σ11 ( x 2 , x3 ) = 233.83859 x 2 − 620.2893 x3 [MPa] (4.87)

University of Castilla-La Mancha Draft By: Eduardo W. V. Chaves (2016)


Ciudad Real - Spain
4 THE FUNDAMENTAL EQUATIONS OF CONTINUUM MECHANICS 375

Point coordinates (meter-m) Normal stress

X 2 [ m] X 3 [ m] x2 = X 2 − Y [m] x3 = X 3 − Z [m] σ11[ MPa]

O 0 .0 0 .0 − 0.16372 − 0.07392 7.568242


B 0 .3 0 .0 0.13628 − 0.07392 77.71982
C 0 .3 0 .3 0.13628 0.22608 − 108.366858
D 0 .0 0 .2 − 0.16372 0.12608 − 116.489543
E 0.02 0 .2 − 0.14372 0.12608 − 111.81
F 0.02 0.04 − 0.14372 − 0.03392 − 12.57
Q 0.28 0.04 0.116277 − 0.03392 48.23
H 0.28 0 .3 0.116277 0.22608 − 113.04

θ = 19.28287º 0.02
C
M y = −67.5kNm H

X3,Z M z = −28.13kNm

0.02
D
E
Mz
0 .3
x3′
x3 , z
0 .2 Neutral Axis
x2′
G θ

A.C. x2 , y My
F
Q
0.04
X 2 ,Y
O 0 .3 B

Figure 4.24: Cross section – Dimensions in meter ( m ).

The Neutral Axis (N.A.) can be obtained as follows:


233.83859
σ11 ( x 2 , x3 ) = 233.83859 x 2 − 620.2893x3 = 0 ⇒ x3 = x2
620.2893
then, the neutral axis is given by:
x3 = 0.3769833x 2
(4.88)
⇒ x3 = tan(α ) x 2 ∴ α = arctan(0.3769833) = 20.656º

University of Castilla-La Mancha Draft By: Eduardo W. V. Chaves (2016)


Ciudad Real - Spain
376 SOLVING PROBLEMS BY MEANS OF CONTINUUM MECHANICS

Calculation of the Inertia Tensor Principal Space:


1  2I23  1  2(−0.49302)  1
θ = arctan  = arctan
  = arctan(0.797311)
2  I22 − I33  2  (1.2741) − (2.5108)  2
= 0.3365496rad (4.89)
180
⇒ θ = 0.3365496 = 19.28287º
π
Then the principal values for the inertia tensor are:
I′22   cos θ sin 2 θ 2 cos θ sin θ  I 22 
2

I′  =  sin 2 θ cos 2 θ



− 2 sin θ cos θ  I 33 
 33  
I′23   − sin θ cos θ cos θ sin θ cos 2 θ − sin 2 θ  I 23 
 
(4.90)
 0.89095 0.10905 0.62341   1.2741  1.10158 
=  0.10905 0.89095 − 0.62341  2.5108  × 10 = 2.68326 × 10 − 4 m 4
    −4

 − 0.62341 0.31171 0.78189  − 0.49302  0 

H
σ11
C
σ11

D
σ11 E
σ11

F
σ11

Q
σ11
O
σ11

B
σ11

Figure 4.25: Normal stress distribution on the cross section.

University of Castilla-La Mancha Draft By: Eduardo W. V. Chaves (2016)


Ciudad Real - Spain
4 THE FUNDAMENTAL EQUATIONS OF CONTINUUM MECHANICS 377

Problem 4.25
Consider a cross section described in Figure 4.26, (Cervera&Blanco (2001)), in which acts
only the normal stress σ11 . Knowing that at the point p( X 2 = 9.5cm; X 3 = 19.5cm) there is
r
a compression force P = −150kNÎ , obtain the normal stress σ11 at the points B , C , D , E
and F .

X3,Z

E
1
D

18

F 1
O C X 2 ,Y
9 1 14

Figure 4.26: Cross section – Dimensions in centimeter ( cm ).


Solution:
Geometry decomposition:

r
X (1) = 4.5Jˆ + 19.5Kˆ
X 3,Z r (2)
X = 9.5Jˆ + 10K ˆ
r ( 3)
(1) (2) X = 17 Jˆ + 0.5K ˆ

A(1) = 9 × 1 = 9cm 2
r
X (1) A( 2) = 1× 20 = 20cm 2
A(3) = 14 ×1 = 14cm 2
r
X ( 2) A = A(1) + A( 2) + A(3) = 43cm 2

r
X ( 3) (3) X 2 ,Y
O

Figure 4.27: Geometric decomposition of the cross section.

University of Castilla-La Mancha Draft By: Eduardo W. V. Chaves (2016)


Ciudad Real - Spain
378 SOLVING PROBLEMS BY MEANS OF CONTINUUM MECHANICS

Calculation of the Area Centroid - G :


3

∫ X 2 dA ∑A
a =1
(a)
X 2( a )
A (1) X 2(1) + A ( 2 ) X 2( 2 ) + A(3) X 2(3)
X2 ≡ Y = A
= =
∫ dA
A
A ( A(1) + A( 2 ) + A(3) )

9 × 4.5 + 20 × 9.5 + 14 ×17


= = 10.895cm
43
3

∫ X 3 dA ∑A
a =1
(a)
X 3( a )
A(1) X 3(1) + A( 2 ) X 3( 2 ) + A(3) X 3(3)
X3 ≡ Z = A
= =
∫ dA
A
A ( A (1) + A ( 2 ) + A (3) )

9 × 19.5 + 20 × 10 + 14 × 0.5
= = 8.895cm
43
r
Vector position of the Area Centroid: X = 10.895Jˆ + 8.895Kˆ , (see Figure 4.28).
Calculation of the position vector
r r r
The position vector x ( a ) = X ( a ) − X can be obtained as follows:
r r r
x (1) = X (1) − X = ( X 2(1) − Y )ˆj + ( X 3(1) − Z )kˆ = (4.5 − 10.895)ˆj + (19.5 − 8.895)kˆ
= −6.395ˆj + 10.605kˆ
r r r
x ( 2 ) = X ( 2 ) − X = ( X 2( 2 ) − Y )ˆj + ( X 3( 2 ) − Z )kˆ = (9.5 − 10.895)ˆj + (10 − 8.895)kˆ
= −1.395ˆj + 1.105kˆ
r r r
x (3) = X (3) − X = ( X (3) − Y )ˆj + ( X (3) − Z )kˆ = (17 − 10.895)ˆj + (0.5 − 8.895)kˆ
2 3

= 6.105ˆj − 8.395kˆ

Calculation of the inertia tensor at the Area Centroid:


Rectangle 1 - I G(1x)r : a = 9cm; b = 1cm
 ab 3
  9 × 13 
 I 0
I23   12
  0  0.75 0  4
Ig(1) ij =  22 =
3
=  12 3
= cm
 I23 I33  
ba   1× 9   0 60.75
0 0

12   12 
r
Area centroid vector position: x (1) = −6.395ˆj + 10.605kˆ , A (1) = 9cm 2 :
I I23   2
(1) x1 + x3
2
− x2 x3 
(I G(1x)r ) ij =  22 +A  
 I23 I33   − x2 x3 x12 + x22 
0.75
=
0  
+ 9
(10.605 )2 − (− 6.395 )(10.605 )
  
 0 60.75  − (− 6.395 )(10.605 ) (− 6.395 )2 
1.01294 × 10 3 610 .37077  4
=  cm
 610 .37077 428.81422 

University of Castilla-La Mancha Draft By: Eduardo W. V. Chaves (2016)


Ciudad Real - Spain
4 THE FUNDAMENTAL EQUATIONS OF CONTINUUM MECHANICS 379

Rectangle 2 - I G( 2xr) : a = 1cm; b = 20cm


 ab 3
 1 × 20 3 
 I 0
I23   12
  0  666.6667 0  4
Ig( 2 ) ij =  22 =
3
=  12 3
= cm
 I23 I33  
ba   20 × 1   0 666.6667 
0 0

12   12 
r ( 2)
Area centroid vector position: x = −1.395ˆj + 1.105kˆ , A ( 2) = 20cm 2 :
I I23   2
( 2 ) x1 + x3
2
− x2 x3 
(I G( 2xr) ) ij =  22  + A  
 I23 I33   − x2 x3 x12 + x22 
666.6667
=
0 
+ 20
 (1.105 )2 − (− 1.395 )(1.105 )
  
 0 666 .6667   − (− 1.395 )(1.105 ) (− 1.395 )2 
691.08717 30.8295  4
= cm
 30.8295 40.58717 

Rectangle 3 - I G(3xr) : a = 14cm; b = 1cm


 ab 3
 14 × 13 
 I 0
I23   12
  0  1.166667 0  4
Ig( 3) ij =  22 =
3
=  12 3
= cm
 I23 I33  
ba   1× 14   0 228.66667 
0 0

12   12 
r
Area centroid vector position: x (3) = 6.105ˆj − 8.395kˆ , A (3) = 14cm 2 :
I I23   2
( 3) x1 + x3
2
− x2 x3 
(I G( 3xr) ) ij =  22 + A  
 I23 I33   − x2 x3 x12 + x22 
1.166667
=
0 
+ 14
 (− 8.395 )2 − (6.105 )(− 8.395 )
  
 0 228.66667   − (6.105 )(− 8.395 ) (6.105 )2 
987 .83102 717 .52065  4
=  cm
717 .52065 750 .46102 
Then, we can calculate the inertia tensor of the cross section related to the system located
at the Area Centroid - G :
(I G( sys
r ) (1)r ( 2r) ( 3r)
x ) ij = (I Gx ) ij + (I Gx ) ij + (I Gx ) ij

1.01294 × 103 610.37077  691.08717 30.8295  987.83102 717.52065 


= + +
 610.37077 428.81422   30.8295 40.58717  717.52065 750.46102 
 2.69186 1.35872  I I 
=  × 103 cm 4 =  22 23 
1.35872 1.21986  I 23 I33 
(4.91)
Calculation of the normal stress:
Note that the cross section is under a compression force, so that we must use the equation
in (4.61), i.e.:
N ( M y I 23 − M z I 22 ) (− M y I 33 + M z I 23 )
σ11 ( x 2 , x3 ) = − 2
x2 + x3 (4.92)
A (I 23 − I 22 I 33 ) (I 223 − I 22 I 33 )

University of Castilla-La Mancha Draft By: Eduardo W. V. Chaves (2016)


Ciudad Real - Spain
380 SOLVING PROBLEMS BY MEANS OF CONTINUUM MECHANICS

To use the above equations, all the variables must be expressed in the system
G − x1 − x2 − x3 .
r
p -point Vector position in the system X 1 − X 2 − X 3 ⇒ X ( P ) = 9.5Jˆ + 19.5K ˆ
r r r r r r r
Knowing that X + x = X ⇒ x = X − X , where X = 10.895Jˆ + 8.895Kˆ , we can calculate the
p -point Vector position in the system G − x1 − x2 − x3 as follows:
r r r r
x (P) = X (P) − X ⇒ x ( P ) = (9.5 − 10.895)ˆj + (19.5 − 8.895)kˆ = −1.395ˆj + 10.605kˆ
r
Then, the moment due to the force P = −150kNIˆ = −150kNˆi , at the point G , can be
obtained as follows:
r r r
M Gxr = x ( P ) ∧ P = (−1.395ˆj + 10.605kˆ ) ∧ (−150ˆi )
= (−1.395) × (−150) + ˆ{ j ∧ ˆi (10.605) × (−150)k{ˆ ∧ ˆi
= −kˆ = ˆj

= −209.25kˆ − 1590.75ˆj (kNcm)


= M kˆ + M ˆj
z y

X 3, Z
x3 , z
X 2 = 10.895

D 1

M z = −209.25

a A.C. x2 , y
18
G
M y = −1590.75
Neutral Axis b
X 3 = 8.895
B

F 1
O
C X 2 ,Y
9 1 14

Figure 4.28: Cross section – Dimensions in centimeter ( cm ).

x ) ij , (see equation (4.91)), and N = −150 kN ,


Taking into account the values for (I (Gsys
r )

A = 43cm 2 , M y = −1590.75kNcm and M z = −209.25kNcm , the equation in (4.92) becomes:

σ11 ( x2 , x3 ) = −34.88372 − 11.520644 x3 − 11.1167 x2 [MPa] (4.93)


By means of the above equation and by considering the coordinates of the points we can
obtain:

University of Castilla-La Mancha Draft By: Eduardo W. V. Chaves (2016)


Ciudad Real - Spain
4 THE FUNDAMENTAL EQUATIONS OF CONTINUUM MECHANICS 381

Point coordinates ( × 10 −2 m ) Normal stress

X2 X3 x2 = X 2 − X 2 x3 = X 3 − X 3 σ11[ MPa]

B 24 1 13.105 − 7.895 − 89.6125

C 24 0 13.105 − 8.895 − 78.092

D 0 19 − 0.895 11.105 − 30.1834

E 10 20 0.13628 0.22608 − 152.8710

F 9 0 − 1.895 − 8.895 88.6585

The Neutral Axis (N.A.), (see Figure 4.28), can be obtained as follows:
σ11 ( x 2 , x3 ) = −34.88372 − 11.520644 x3 − 11.1167 x 2 = 0
⇒ −11.520644 x3 − 11.1167 x 2 = 34.88372
x3 x2
⇒ + = 1 .0
(−3.02793) (−3.13796)
then, the neutral axis is given by its canonic form as follows:
x3 x2 x3 x 2
+ = 1 .0 ⇔ + = 1.0 (4.94)
(−3.02793) (−3.13796) b a

Calculation of the Inertia Tensor Principal Space:


1  2I 23  1  2(1.35872)  1
θ = arctan  = arctan  = arctan(1.8460882)
2  I 22 − I33  2  (2.69186) − (1.21986)  2
= 0.5371793rad (4.95)
180
⇒ θ = 0.5371793 = 30.778º
π
Then the principal values for the inertia tensor are:
I′22   cos θ sin 2 θ 2 cosθ sin θ  I 22 
2

I′  =  
 33   sin θ
2
cos 2 θ − 2 sin θ cosθ  I33 
I′23  − sin θ cosθ cosθ sin θ cos 2 θ − sin 2 θ  I 23 
 
(4.96)
 0.73815 0.26185 0.87928   2.69186  3501.12 
=  0.26185 0.73815 − 0.87928 1.21986  = 410.6056 cm 4
− 0.43964 0.43964 0.4763  1.35872   0 

University of Castilla-La Mancha Draft By: Eduardo W. V. Chaves (2016)


Ciudad Real - Spain
382 SOLVING PROBLEMS BY MEANS OF CONTINUUM MECHANICS

Problem 4.26
A foundation in Engineering is a structural element which serves to transmit the load from
the structure to the soil. Consider a Mat-Slab Foundation described in Figure 4.29 in which
we have six columns. Knowing that soils cannot resist to traction stress, verify whether the
design (dimensions) of the foundation, from a structural stability point of view, is
appropriated or not.
Hypothesis (approximation): Consider that the mat foundation is infinitely rigid, so that
the normal stress distribution in the soil will be a planar distribution.

Column loads
F11C1 = −300kN ; F11C2 = −450kN ; F11C3 = −600kN
F11C4 = −450kN ; F11C5 = −800kN ; F11C6 = −1100kN
dimensions in meter (m)

0.2 C1 C2 C3

A A

2.5 2.9

0.2 C6
C4 C5

0.2 3.7 2.5 0.2

6.6

C1 C2 C3

Section - AA

Figure 4.29: Mat foundation.

Solution:
The structural stability will be acceptable if on the ground (mat foundation base) there is
only normal stress of compression. And based on the fact that the mat foundation is
infinitely rigid we can apply the equation (4.65) if the reference system is located at the area
centroid- G :
N Mz My (The system is located at the Area
σ11 ( x2 , x3 ) = − x2 + x3 Centroid and is the principal axes (4.97)
A I33 I 22
of inertia)
Let us adopt the system located at the area centroid as indicated in Figure 4.30.

University of Castilla-La Mancha Draft By: Eduardo W. V. Chaves (2016)


Ciudad Real - Spain
4 THE FUNDAMENTAL EQUATIONS OF CONTINUUM MECHANICS 383

x3
p1 : (−3.3;1.45) p2 : (3.3;1.45)

F11C1 F11C2 F11C3


r
1.25
r
x C1 x C2 r
x C3
2.9
r G r x2
x C4 x C6
1.25 r
x C5
F11C4 F11C5 F11C6

p3 : (−3.3;−1.45) 3.1 0.6 p4 : (3.3;−1.45)


3.1
dimensions in
6.6
meter ( m )

Figure 4.30
Calculation of the total force ( F11Total ):
r 6 rC r r r r r r
F11Total = ∑F
a =1
11
a
= F11C1 + F11C2 + F11C3 + F11C4 + F11C5 + F11C6
r
F11Total = −300ˆi − 450ˆi − 600ˆi − 450ˆi − 800ˆi − 1100ˆi = −3700ˆi kN
r
Calculation of the total moment at the area centroid ( M Total ):
r 6
r Ca r
M Total = ∑( x
a =1
∧ F11Ca )
r r r r r r r r r r r r r
M Total = ( x C1 ∧ F11C1 ) + ( x C2 ∧ F11C2 ) + ( x C3 ∧ F11C3 ) + ( x C4 ∧ F11C4 ) + ( x C5 ∧ F11C5 ) + ( x C6 ∧ F11C6 )
where
r r
x C1 ∧ F11C1 = (−3.1ˆj + 1.25kˆ ) ∧ (−300ˆi ) = (−3.1)(−300)ˆ{
j ∧ ˆi + (1.25)(−300)k{
ˆ ∧ ˆi = −930kˆ − 375ˆj
ˆj
−kˆ
r r
x C2 ∧ F11C2 = (0.6ˆj + 1.25kˆ ) ∧ (−450ˆi ) = (0.6)(−450)ˆ{
j ∧ ˆi + (1.25)(−450)k{
ˆ ∧ ˆi = 270kˆ − 562.5ˆj
ˆj
−kˆ
r r
x C3 ∧ F11C3 = (3.1ˆj + 1.25kˆ ) ∧ (−600ˆi ) = (3.1)(−600)ˆ{
j ∧ ˆi + (1.25)(−600)k
ˆ ∧ ˆi = 1860kˆ − 750ˆj
{
ˆj
−kˆ
r r
x C4 ∧ F11C4 = (−3.1ˆj − 1.25kˆ ) ∧ (−450ˆi ) = (−3.1)(−450)ˆ{
j ∧ ˆi + (−1.25)(−450)k
ˆ ∧ ˆi = −1395kˆ + 562.5ˆj
{
ˆj
−kˆ
r r
x C5 ∧ F11C5 = (0.6ˆj − 1.25kˆ ) ∧ (−800ˆi ) = (0.6)(−800)ˆ{
j ∧ ˆi + (−1.25)(−800)k
ˆ ∧ ˆi = 480kˆ + 1000ˆj
{
ˆj
−kˆ
r r
x C6 ∧ F11C6 = (3.1ˆj − 1.25kˆ ) ∧ (−1100ˆi ) = (0.6)(−1100)ˆ{
j ∧ ˆi + (−1.25)(−1100)k
ˆ ∧ ˆi = 3410kˆ + 1375ˆj
{
ˆj
−kˆ
Then,

University of Castilla-La Mancha Draft By: Eduardo W. V. Chaves (2016)


Ciudad Real - Spain
384 SOLVING PROBLEMS BY MEANS OF CONTINUUM MECHANICS

r 6
r Ca r M y = 1250kN m
M Total = ∑( x ∧ F11Ca ) = 3695kˆ + 1250ˆj = M zkˆ + M y ˆj ⇒ 
a =1 M z = 3695kN m
Calculation of the normal stress field
r
For the rectangular cross section, (see Figure 4.15), by considering the system ox we have:
(6.6)(2.9)3 (6.6)3 (2.9)
I 22 = = 13.414m 4 ; I33 = = 69.478m 4 ; I 23 = 0
12 12
A = (6.6)( 2.9) = 19.14m 2
Then,
N Mz My − 3700 (3695) (1250)
σ11 ( x2 , x3 ) = − x2 + x3 = − x2 + x3
A I 33 I 22 19.14 69.478 13.414

 kN 
⇒ σ11 ( x2 , x3 ) = −193.3124 − 53.182 x2 + 93.187 x3  m 2 = kPa 
 
Point p1 : σ11p1 ( x2 = −3.3; x3 = 1.45) = −193.3124 − 53.182 x2 + 93.187 x3 = 117.309
Point p2 : σ11p 2 ( x2 = 3.3; x3 = 1.45) = −193.3124 − 53.182 x2 + 93.187 x3 = −233.693
Point p3 : σ11p3 ( x2 = −3.3; x3 = −1.45) = −193.3124 − 53.182 x2 + 93.187 x3 = −152.932
Point p4 : σ11p 4 ( x2 = 3.3; x3 = −1.45) = −193.3124 − 53.182 x2 + 93.187 x3 = −503.934
So, as we can see, the design established for the mat foundation is not the appropriated
one, since traction stress appears in the soil, (see Figure 4.31).

σ11p1 Traction
p2
p1
x1 x3
Neutral axis
σ11p 2
G x2

p4
p3

σ11p 3 Compression

σ11p 4

Figure 4.31: Normal stress distribution on the ground.

University of Castilla-La Mancha Draft By: Eduardo W. V. Chaves (2016)


Ciudad Real - Spain
4 THE FUNDAMENTAL EQUATIONS OF CONTINUUM MECHANICS 385

r r r r
NOTE: The centroid of the concentrated forces ( ~x ) is defined by ~x ∧ F11Total = M Total , (see
r r
Figure 4.32). Then, by considering that M Total = 1250ˆj + 3695kˆ and F11Total = −3700î kN we
can obtain:
r r
~ ∧ Fr Total = M
x
r Total
⇒ ~ ∧ (−3700ˆi ) = 1250ˆj + 3695kˆ
x
11

where
(~
x1ˆi + ~
x2 ˆj + ~
x3kˆ ) ∧ (−3700ˆi ) = −3700~
x1ˆi ∧ ˆi − 3700~
x2ˆj ∧ ˆi − 3700~
x3kˆ ∧ ˆi = 3700~
x2kˆ − 3700~
x3ˆj
r r r
so, by applying ~x ∧ F11Total = M Total we can obtain:
 ~ ~ 1250
− 3700 x3 = 1250 → x3 =
− 3700
= −0.338
3700~
x2kˆ − 3700~
x3ˆj = 1250ˆj + 3695kˆ ⇒ 
3700~ 3695
x2 = 3695 → ~ x2 = = 0.999
 3700
r r
This position ( ~x ) is called eccentricity of F11Total which represents the centroid of the forces.
The centroid of the forces can also be obtained in the same fashion as the volume/area
centroid definition, i.e.:
6

~
∑F
a =1
Ca
11 x2Ca
F11C1 x2C1 + F11C2 x2C2 + F11C3 x2C3 + F11C4 x2C4 + F11C5 x2C5 + F11C6 x2C6 3695
x2 = 6
= 6
= = 0.999
3700
∑F
a =1
Ca
11 ∑F
a =1
Ca
11

~
− ∑F Ca Ca
11 x3
− ( F11C1 x3C1 + F11C2 x3C2 + F11C3 x3C3 + F11C4 x3C4 + F11C5 x3C5 + F11C6 x3C6 ) − 1250
x3 = a =1
6
= 6
= = −0.338
3700
∑F
a =1
Ca
11 ∑F a =1
Ca
11

r
M G = M y ˆj + M z kˆ r r
r ~ ∧ Fr
MG = x
F = F11î

x1 x3 x1 x3
F11
Mz ~
x3
F11
My = r
~
x
G x2 G ~
x2 x2

r
~
x - eccentricity

Figure 4.32

University of Castilla-La Mancha Draft By: Eduardo W. V. Chaves (2016)


Ciudad Real - Spain
386 SOLVING PROBLEMS BY MEANS OF CONTINUUM MECHANICS

Problem 4.27
Consider a prismatic bar with a walled cross section as indicated in Figure 4.33(a). By
considering the differential element given by Figure 4.33(b), obtain the equilibrium
equations by considering the system ( x1 , s ).

x3
a) Prismatic bar (walled cross section)
rˆ ∧ sˆ = eˆ 1

x2
r
r

t

ê1 t - thickness
O
b) The differential element

rear faces
ds x1

dx1
σ11 σ s1 σs
σ1s

dt
s
∂σ s dt ∂σ11 x1
σs + ds σ11 + dx1
∂s ∂x1
∂σ s1
∂σ σ s1 + dx1
σ1s + 1s ds ∂x1
∂s

Figure 4.33: Thin-walled member.


Solution:
Equilibrium according to x1 -direction:
 ∂σ   ∂σ 
∑F x1 =0  σ11 + 11 dx1 ds dt − σ11ds dt +  σ1s + 1s ds dx1 dt − σ1s dx1 dt = 0


∂x1   ∂s 
∂σ ∂σ ∂σ11 ∂σ1s
⇒ 11 dx1ds dt + 1s dsdx1 dt = 0 ⇒ + =0
∂x1 ∂s ∂x1 ∂s
Equilibrium according to s -direction:
 ∂σ   ∂σ 
∑F s =0  σ s + s ds dx1 dt − σs dx1 dt +  σ s1 + s1 dx1 ds dt − σ s1ds dt = 0


∂s   ∂x1 
∂σ ∂σ ∂σ s ∂σ s1
⇒ s dsdx1 dt + s1 dx1ds dt = 0 ⇒ + =0
∂s ∂x1 ∂s ∂x1

University of Castilla-La Mancha Draft By: Eduardo W. V. Chaves (2016)


Ciudad Real - Spain
4 THE FUNDAMENTAL EQUATIONS OF CONTINUUM MECHANICS 387

The equilibrium equations at a material point on the cross section are:


 ∂σ11 ∂σ1s
 ∂x + ∂s = 0
 1

 ∂σ s + ∂σ s1 = 0
 ∂s ∂x1
NOTE 1: Thin-Walled Members
Let us consider that the thickness t is very small and the stress distributions in the
thickness is given as indicated in Figure 4.34(b), then, we can adopt that q = σ s1t and σ11
are constant along the thickness, where q is called the shear flow. By changing the
nomenclature σ s1 = τ we rewrite q = σ s1t = τ t .

a) b)

r̂ r̂
t
σ (sS1 )
t
very small
→ t
σ s1 = σ s1 (t )

( x2 , x3 ) σ11 ( x2 , x3 ) σ11

σ s1 x1
σ s1 = τ x1
s s
σ11 = σ11 (t )

σ (s1I )
q = σ s1t = τ t (shear flow)

Figure 4.34
Then, the equilibrium equations can be rewritten as follows:
 q
 ∂σ ∂ 
 ∂σ11 ∂σ1s  11 +  t  = 0  ∂σ11 ∂q
 ∂x + ∂s = 0  ∂x1 ∂s t ∂x + ∂s = 0
 1  1
 ⇒  ⇒  (4.98)
 ∂σ s + ∂σ s1 = 0  q
∂  t ∂σ s + ∂q = 0
 ∂s ∂x1  ∂σ s t  ∂s ∂x1
 +   =0
 ∂s ∂x1
If we integrate the first equilibrium equation over s -coordinate we can obtain:
s s
∂σ11 ∂q ∂q  ∂σ 
∫ ∫
ds = −  t 11  ds
ating over s
t + =0 by
integr
  →
∂x1 ∂s 0
∂s 0
∂x1 
s
(4.99)
 ∂σ 
0

⇒ q ( s) − q (0) = −  t 11 ds
∂x1 

University of Castilla-La Mancha Draft By: Eduardo W. V. Chaves (2016)


Ciudad Real - Spain
388 SOLVING PROBLEMS BY MEANS OF CONTINUUM MECHANICS

The s -coordinate is measured along the cross-section perimeter, (see Figure 4.35). By
considering that N is independent of x1 or ( N = 0) we can rewrite the equation (4.74) as
follows:
∂σ11 (I x + I x ) (I x + I x )
= − 232 2 33 3 Fz − 222 2 23 3 Fy
∂x1 (I 23 − I 22I 33 ) (I 23 − I 22I 33 )
(4.100)
∂σ (I 23 Fz + I 22 Fy ) (I 23 Fy + I 33 Fz )
⇒ 11 = − 2 x2 − 2 x3
∂x1 (I 23 − I 22I33 ) (I 23 − I 22I 33 )
And by substituting the above equation into the equation in (4.99) we can obtain:
s
 ∂σ 
0

q ( s ) = q (0) −  t 11 ds
∂x1 
(4.101)

 I 23 Fz + I 22 Fy s I F +I F s The system is
q ( s ) = q ( 0) +  2  (t x2 )ds +  23 y 33 z
∫  (t x3 )ds located at the Area
∫ (4.102)
 I 23 − I 22I 33   I 223 − I 22I 33 
 0  0 Centroid
The above shear flow is indeterminate because we do not know q (0) when we are dealing
with closed cross section. The solution strategy will be discussed in Chapter 6.
If we have an open cross section we can assume that at s = 0 ⇒ q(0) = 0 , (see Figure 4.35).

t ŝ
s
s
q (s ) s = 0 ⇒ q ( 0) = 0
r
r r0
r

G
ê1

x1

Free edge ⇒ q = 0

Figure 4.35
NOTE 1.1: The equation in (4.72) can be rewritten in a compact form as follows:
∂σ11 Y0 Y2 Y (For any system in which the plane x2 − x3 is lying
= + x2 + 3 x3 (4.103)
∂x1 X X X on the plane defined by the cross section)
where
0 Ax 2 Ax 3 A 0 Ax3
Y0 = Fz − I 23 I 22 [ Nm 7 ] ; Y2 = Ax3 Fz I 22 [ Nm 6 ] (4.104)
− Fy − I 33 I 23 − Ax 2 − Fy I 23

University of Castilla-La Mancha Draft By: Eduardo W. V. Chaves (2016)


Ciudad Real - Spain
4 THE FUNDAMENTAL EQUATIONS OF CONTINUUM MECHANICS 389

A Ax2 0 A Ax2 Ax3


6
Y3 = Ax3 − I 23 Fz [ Nm ] ; X = Ax3 − I 23 I 22 [m10 ] (4.105)
− Ax2 − I 33 − Fy − Ax2 − I 33 I 23

Then, the equation in (4.101) becomes


s s s s
 ∂σ  Y Y Y N 

0
x1  X 0
X 0

q ( s ) = q (0) −  t 11  ds = q(0) − 0 t ds − 2 (t x2 )ds − 3 (t x3 )ds
∂ X 0 ∫ ∫ m
  (4.106)
(For any system in which the plane x2 − x3 is lying on the plane defined by the cross section)

NOTE 2: Closed Thin-Walled Cross Section under Torsion only.


If the thickness is very small we can adopt the stress distribution as the one shown in
Figure 4.34(b) and if we also consider that σ11 = 0 we can conclude, according to the
∂σ11 ∂q ∂q
equilibrium equation (4.98), ( t + = 0 ), that = 0 , i.e. the shear flow does not vary
∂x1 ∂s ∂s
with s -coordinate, i.e. q is constant. For this scenario we can calculate the moment
according to x1 -coordinate as follows:
r r r  
M Ox1 = r ∧ qds = qrrˆ ∧ sˆds = q reˆ 1ds = q rds eˆ 1 = q(2 A)eˆ 1 = 2qAeˆ 1
∫ ∫ ∫ ∫ (4.107)
 
s s s s 
r
where we have applied the equation rds = r ⋅ rˆds = 2 A , (see NOTE in Problem 1.128).
∫ ∫
s s

Then, given the moment of torsion M Ox1 ≡ M T we can calculate the shear flow ( q ) and the
tangential stress ( τ ) as follows:
MT MT
M T = 2qA ⇒ q= ⇒ τ= (4.108)
2A 2 At
where q = σ s1t = τ t . The above equation could be a good approximation if the thickness
( t ) is very small. In Chapter 6 we will discuss another approximation to tackle this
problem.

x3
Only torsion - F21 = F31 = 0

MT
ŝ q =τ t =
2A
r
q = qsˆ r q (constant)
r

t
O x2
Maximum stress
MT tmin
τ max = A
2 Atmin

Figure 4.36

University of Castilla-La Mancha Draft By: Eduardo W. V. Chaves (2016)


Ciudad Real - Spain
390 SOLVING PROBLEMS BY MEANS OF CONTINUUM MECHANICS

Example: Let us consider a circular cross section, (see Figure 4.37), and the applied
torsion is M T = 5.0 × 103 Nmm . Then, the shear stress can be calculated as follows:
MT
q 2A MT 5.0 ×103 N
τ= = = = ≈ 4.2
t t 2 At 2 × 498.76 × 1.2 mm 2

Data: x3 Parameters:
3
M OT = 5 × 10 Nmm r R1 + R2
q Rm = = 12.6mm
R1 = 12.0mm 2
R2 = 13.2mm R2 t = R2 − R1 = 1.2mm
A = πRm2 ≈ 498.76mm 2
Rm
R1
O x2
t

(Dimensions in millimeter- mm )

Figure 4.37

Problem 4.28
Consider an arbitrary cross section in which is acting a compression force F11 = − P , where
r
P is a positive real number, and the adopted system ( ox ) is located at the Area Centroid.
Consider that when F11 = − P is located at the Area Centroid there is no moment, i.e.
My = Mz = 0 .

Obtain the loci for the all possible position for F11 = − P such as there is no traction on the
cross-section, i.e. σ11 ( x2 , x3 ) ≥ 0 .
NOTE 1: The region in which the compression force does not produce traction on the
cross section is called the core or “kernel” of a section.

Solution:
We can adopt the equation in (4.61), since the adopted system is located at the Area
Centroid of the cross section:
N ( M y I23 − M z I 22 ) (− M y I33 + M z I 23 )
σ11 ( x2 , x3 ) = − 2
x2 + x3 (4.109)
A (I 23 − I 22I33 ) (I 223 − I 22I33 )
Then, for this problem we have N = F11 = − P , M y = − P~x3 , and M z = P~x2 , (see Figure
4.38). Note that we are looking for ( ~x2 , ~x3 ) in such a way that in the cross section there is
no traction, i.e. σ11 ( x2 , x3 ) ≥ 0 , so we can establish that in the boundary of the section the
normal stress is zero σ11 ( x2 , x3 ) = 0 , i.e.:

University of Castilla-La Mancha Draft By: Eduardo W. V. Chaves (2016)


Ciudad Real - Spain
4 THE FUNDAMENTAL EQUATIONS OF CONTINUUM MECHANICS 391

N ( M y I 23 − M z I 22 ) (− M y I33 + M z I 23 )
σ11 ( x2 , x3 ) =− 2
x2 + x3 = 0
A (I 23 − I 22I33 ) (I 223 − I 22I33 )
(− P) (− P~ x3I 23 − ( P~ x2 )I 22 ) (−(− P~ x3 )I33 + ( P~ x2 )I 23 )
⇒ − 2
x 2 + 2
x3 = 0
A (I 23 − I 22I33 ) (I 23 − I 22I33 )
− 1 (~x I +~ x2I 22 ) (~x I +~ x2I 23 )
⇒ + 32 23 x2 + 32 33 x3 = 0
A (I 23 − I 22I33 ) (I 23 − I 22I33 )
(I 2 − I I )
⇒ (~
x3I 23 + ~
x2I 22 ) x2 + ( ~
x3I33 + ~
x2I 23 ) x3 = 23 22 33
A
or

(I 2 − I I ) The system is located at


(I 22 x2 + I 23 x3 ) ~
x2 + (I 23 x2 + I 33 x3 ) ~
x3 = 23 22 33 (4.110)
A the Area Centroid

Then, for the position ( x2 , x3 ) in which we assume that σ11 ( x2 , x3 ) = 0 we can find the
geometric position ( ~x2 , ~x3 ) for F11 = − P .

M y = 0 M y = − P~
x3
x1  x1  ~
M z = 0 M z = Px2

z, x3
z, x3
−P Mz −P
~
x3
A.C. A.C.
G y , x2 My ~ y , x2
x2

Figure 4.38

For example, if the cross section is a circle of radius r , (see Figure 4.39), and as the system
πr 4
is located at the area centroid the following is true A = πr 2 , I22 = I33 = = I , I 23 = 0 . The
4
coordinates ( x2 , x3 ) for the boundary can be represented in terms of the radius: x2 = r cos θ
and x3 = r sin θ , therefore
(I 2 − I I )
(I 22 x2 + I 23 x3 ) ~
x2 + (I 23 x2 + I33 x3 ) ~
x3 = 23 22 33
A
2
−I −I −I
⇒ Ix2 ~ x2 + Ix3 ~ x3 = ⇒ r cos θ~
x2 + r sin θ~
x3 = ⇒ cos θ~
x2 + sin θ~
x3 =
A A Ar
 πr 4
−  
~ ~ −I ~ ~  4  − r
cos θx2 + sin θx3 = ⇒ cos θx2 + sin θx3 = =
Ar πr 2 r 4

University of Castilla-La Mancha Draft By: Eduardo W. V. Chaves (2016)


Ciudad Real - Spain
392 SOLVING PROBLEMS BY MEANS OF CONTINUUM MECHANICS

−r −r −r
⇒ cos θ(r~ cos θ) + sin θ(~
r sin θ) = ~
r (cos 2 θ + sin 2 θ) =
⇒ ⇒ ~ r=
4 4 4
~
Then, if F11 = − P assume any position inside the circle of radius r it will not produce
traction on the cross section.

z, x3 A = πr 2
r − rr
~
r= σ11 ( x2 , x3 ) = 0 πr 4
4 I22 = I33 =
4
r
r I23 = 0

r θ
~ y , x2
r
P x2 = r cos θ
Core of the section x3 = r sin θ

Figure 4.39: Core of the circular cross section.

Let us consider a rectangular cross section, (see Figure 4.40). In this case we have A = ah ,
ah 3 a 3h
I 22 = , I33 = and I 23 = 0 . Then, the equation (4.110) becomes:
12 12
(I 2 − I I ) −I I
(I 22 x2 + I 23 x3 ) ~
x2 + (I 23 x2 + I 33 x3 ) ~
x3 = 23 22 33 ⇒ I 22 x2 ~
x2 + I 33 x3 ~
x3 = 22 33
A A
 ah  a 3 h 
3
−   
 ah 3  ~  a 3 h  ~  12  12  − h2a 2
⇒   x2 x2 +      h 2 x2 ~
x 2 + a 2 x3 ~
12   12  x3 x3 = bh
⇒ x3 =
12
   
 a h
For the point  x2 = , x3 =  we have:
 2 2

− h2a2 a h − h2a2
h 2 x2 ~
x2 + a 2 x3 ~
x3 = ⇒ h2 ~ x2 + a 2 ~x3 =
12 2 2 12
a h 2 2
−h a ~
x2 ~
x3
⇒ h2 ~x2 + a 2 ~x3 = ⇒ + =1
2 2 12 −a −h
   
 6   6 
−a −h
which is a line that intercepts x2 in   and intercepts x3 in   . If we make the
 6   6 
same procedure for other points we can obtain the geometric shape of the core of the
rectangular section as the one indicated in Figure 4.40.

University of Castilla-La Mancha Draft By: Eduardo W. V. Chaves (2016)


Ciudad Real - Spain
4 THE FUNDAMENTAL EQUATIONS OF CONTINUUM MECHANICS 393

z, x3
σ11 ( x2 , x3 ) = 0 A = ah
ah 3
I 22 =
a a 12
6 6
a 3h
I33 =
12
I 23 = 0
h
6 y , x2
h
h
6

Core of the
section
x2 x3
−a
+ −h
=1
6 6

Figure 4.40: Core of the rectangular cross section.

Problem 4.29
Consider the cantilever (beam fixed at one end) under static equilibrium, (see Figure
4.41(a)), in which we have the concentrated force ( F31 = P ) applied at the end x1 = L .
Obtain the equation for the tangential stress field σ 31 on the cross section, (see Figure
4.41(b)).
Hypotheses (approximations):
a) Consider that on the cross section, σ31 does not vary with x2 , (see Figure 4.41(b)), i.e.
σ31 = σ31 ( x3 ) , and σ 2i = 0 i .
b) Consider the problem without body forces.

University of Castilla-La Mancha Draft By: Eduardo W. V. Chaves (2016)


Ciudad Real - Spain
394 SOLVING PROBLEMS BY MEANS OF CONTINUUM MECHANICS

Approximations:
σ31 = σ31 ( x3 ) ; σ 2i = 0i
x3
σ13 = 0 x3


P = σ31dA
A
σ31
b
2
x2 x3
x2
G
x1
b
L 2 A = ab

a) Cantilever b) Cross-section

Figure 4.41: Beam fixed at one end (cantilever).


Solution:
The equilibrium equations state that
 ∂σ11 ∂σ12 ∂σ13
 + + =0  ∂σ11 ∂σ13
 ∂x1 ∂x2 ∂x3  ∂x + ∂x = 0
 ∂σ 21 ∂σ 22 ∂σ 23  1 3

σij , j + ρb i = ρ&u&i ⇒  + + =0 ⇒ 0 = 0
{ {
=0 i =0 i  ∂x1 ∂x2 ∂x3  ∂σ
 ∂σ31 ∂σ32 ∂σ33  31 = 0

 ∂x1
+ + =0  ∂x1
∂x 2 ∂x3

∂σ11
If the system is located at the Area Centroid, the term can be expressed by the
∂x1
equation in (4.74):
∂σ11 (I x + I x ) (I x + I x ) − (I 23 x2 + I 33 x3 )
= − 232 2 33 3 Fz − 222 2 23 3 Fy = P (4.111)
∂x1 (I 23 − I 22I 33 ) (I 23 − I 22I 33 ) (I 223 − I 22I33 )
where we have considered Fz = F31 = P , Fy = 0 . Then, the first equilibrium equation
becomes
∂σ11 ∂σ13 − (I 23 x2 + I33 x3 ) ∂σ ∂σ13 (I 23 x2 + I33 x3 )
+ =0 ⇒ P + 13 = 0 ⇒ = 2 P (4.112)
∂x1 ∂x3 (I 223 − I 22I33 ) ∂x3 ∂x3 (I 23 − I 22I33 )
by integrating over x3 we can obtain
I 33 P x32 I P
σ13 = 2
+ 2 23 x2 x3 + K
(I 23 − I 22I 33 ) 2 (I 23 − I 22 I33 )
The constant of integration can be obtained by the condition

University of Castilla-La Mancha Draft By: Eduardo W. V. Chaves (2016)


Ciudad Real - Spain
4 THE FUNDAMENTAL EQUATIONS OF CONTINUUM MECHANICS 395

2
b I33 P  b I P  b
x3 = ± ⇒ σ13 = σ 31 = 0 ⇒ σ13 = 2  ±  + 2 23 ±  x2 + K = 0
2 2(I 23 − I 22 I33 )  2  (I 23 − I 22 I33 )  2
2
I 33 P b I P b
⇒K =−   m 2 23   x2
2(I 223 − I 22I 33 )  2  (I 23 − I 22I 33 )  2 
Then, the tangential stress can be expressed as follows
I 33 P I P
σ13 = x32 + 2 23 x2 x3 + K
2(I 223− I 22I 33 ) (I 23 − I 22 I33 )
I33 P I 23 P  I 33 P b
2
I 23 P  b  
⇒ σ13 = x 2
+ x x −    ±   x2
2(I 223 − I 22 I33 )
3
(I 223 − I 22I 33 )
2 3
 2(I 223 − I 22 I33 )  2  (I 223 − I 22I 33 )  2  

I33 P  2  b 2  I Px   b 
⇒ σ13 = 2  x3 −    + 2 23 2  x3 −  ±  
2(I 23 − I 22 I33 )   2   (I 23 − I 22I 33 )   2 

The adopted system is at the centroid area and is the axis of symmetry, then
ab 3 ba 3
I 22 = ; I33 = ; I 23 = 0 ; A = ab
12 12
Thus, the equation for σ13 = σ31 , (see Figure 4.42), is given by

I 33 P  2  b 2  I 23 Px2   b  − P  2  b  
2
σ13 =  x3 −    + x
 3 −  ±   =  x3 −   
2(I 223 − I 22I 33 )   2   2I 22 
2
 2   (I 23 − I 22 I33 )   2  

The maximum value of σ13 occurs at x3 = 0 , which value is


2 2
P b P b 3P
σ13 max = σ13 ( x3 = 0) =   =   =
2I 22  2   ab   2 
3
2A
2 

 12 

σ13 = 0 x3
P  b  2 2

σ13 =   − x3 
2I 22  2  
b
2
3P
σ13 max =
x2 2A
b
2


P = σ13dA
σ13 = 0 a
A

Figure 4.42: Tangential stress distribution on the cross section.

University of Castilla-La Mancha Draft By: Eduardo W. V. Chaves (2016)


Ciudad Real - Spain
396 SOLVING PROBLEMS BY MEANS OF CONTINUUM MECHANICS

NOTE: We can generalize the previous equations. Let us consider the equation (4.112), i.e.
∂σ13 (I 23 x2 + I33 x3 ) I P I P
= 2 P = 2 33 x3 + 2 23 x2 (4.113)
∂x3 (I 23 − I 22I 33 ) (I 23 − I 22 I33 ) (I 23 − I 22I 33 )
and consider the approximation made in Figure 4.43.

σ13 (c ) = 0 x3
∫σ
x2
13 dx2

σ13 ( x3 ) =
σ13 ( x2 , x3 ) A a ( x3 )

a ( x3 )
c P = σ31dA ∫
A
x3

G x2

Figure 4.43
And by integrating the equation (4.113) over x3 from x3 to c we can obtain
c c c
I33 P I P

x3
∂σ13 dx3 = 2
(I 23 − I 22I 33 ) x ∫
x3∂x3 + 2 23
(I 23 − I 22 I33 ) x
x2 ∂x3 ∫
3 3

c c
I P I P
⇒ σ13 (c) − σ13 ( x3 ) = 2 33
123 (I 23 − I 22 I33 ) x
x3∂x3 + 2 23 ∫
(I 23 − I 22I 33 ) x
x2 ∂x3 ∫
=0 3 3

c c
− I33 P I P
⇒ σ13 ( x3 ) =
(I 223 − I 22 I33 ) x ∫
x3∂x3 − 2 23
(I 23 − I 22 I33 ) x
x2 ∂x3 ∫
3 3

Now by integrating over x2 we can obtain


c c
− I 33 P I 23 P

x2
σ13 ( x3 ) dx2 = 2
(I 23 − I 22I 33 ) x ∫ ∫ x3
x3 dx3 dx2 − 2
(I 23 − I 22 I33 ) x ∫ ∫ x dx dx
x3
2 3 2
2 2

− I33 P I P
⇒ σ13 ( x3 ) a ( x3 ) =
(I 223− I 22 I33 ) A ∫
x3 dA − 2 23
(I 23 − I 22I 33 ) A
x2 dA ∫
thus

− I33 P I 23 P The system is located


σ13 ( x3 ) = 2
a ( x3 )(I 23 − I 22I33 ) A∫x3 dA −
a ( x3 )(I 223 − I 22I33 ) A
x2 dA ∫ at the Area Centroid
(4.114)

where ∫ x dA
A
3 is the first moment of area A about the x2 -axis, and ∫ x dA
A
2 is the first

moment of area about the x3 -axis. Note that, if the x3 -axis is an axis of symmetry, then
∫ x dA = 0 holds, and the above equation reduces to:
A
2

University of Castilla-La Mancha Draft By: Eduardo W. V. Chaves (2016)


Ciudad Real - Spain
4 THE FUNDAMENTAL EQUATIONS OF CONTINUUM MECHANICS 397

P
c
P The system is located at the
σ13 ( x3 ) =
a ( x3 )I 22 ∫ ∫ x3dx3dx2 ≡
a ( x3 )I 22 ∫ x3dA Area Centroid and is the (4.115)
x2 x3 A principal axes of inertia

Example: Let us consider a circular cross section, (see Figure 4.44), and the adopted
system is at the area centroid and is the principal axis of inertia, then we can apply the
equation in (4.115):
c= R c= R
P P P
σ13 ( x3 ) =
a ( x3 )I 22 ∫A
x3 dA =
a ( x3 )I 22 ∫
x3
x3 a ( x3 ) dx3 =
2
2 (R −
∫ 2x
x32 )I 22 x3
3 ( R 2 − x32 )

P 2 P ( R 2 − x32 )
= ( R 2 − x32 ) 3 =
2I 22 ( R 2 − x32 ) 3 3I 22

∂σ11 − (I 23 x2 + I33 x3 ) Px ∂ σ13 ( x3 ) ∂  P ( R 2 − x32 )  − 2 Px3


Note that = P = 3 and =  = , and
∂x1 2
(I 23 − I 22I33 ) I 22 ∂x3 ∂x3  3I 22 
 3I 22
by replacing it into the equilibrium equation we can obtain:
∂σ11 ∂σ13 Px3 − 2 Px3 Px3
+ =0 ⇒ + = ≠0
∂x1 ∂x3 I 22 3I 22 3I 22
which does not satisfy the equilibrium equations.

x3 P ( R 2 − x32 )
dA = a ( x3 ) dx3 σ13 ( x3 ) =
3I 22
πR 4
I 22 = I33 =
c=R 4
A
σ13 ( x3 ) 2 2
x2 + x3 = R 2

a ( x3 ) x3
x2
2
a ( x3 ) = 2 ( R − x32 )

Figure 4.44: Circular cross section.

Problem 4.30
Consider the cantilever (beam fixed at one end) with an I-shaped cross section, (see Figure
4.45). Obtain the tangential stress distribution on the cross section due to the shearing
force F31 = Fz = 11000 N .

University of Castilla-La Mancha Draft By: Eduardo W. V. Chaves (2016)


Ciudad Real - Spain
398 SOLVING PROBLEMS BY MEANS OF CONTINUUM MECHANICS

Fz = 11kN
x3
a1 = a3 = 140mm; a2 = 45mm
b1 = b3 = 45mm; b2 = 140mm

X3
a3
x2
Fz b3
x1

G
⊗ b2
a2
a) Cantilever
b1
O
X2
a1

b) I-shaped cross section

Figure 4.45: I-Shaped cross section.

Solution:
Area Centroid
Due to the geometrical symmetry the Area Centroid- G is located at
a1 b2
X2 = = 0.07 m ; X 3 = b1 + = 0.115m
2 2
And the total area is given by A = A1 + A2 + A3 = a1b1 + a2b2 + a3b3 = 2a1b1 + a2b2 = 0.0189 m 2 .
The inertia tensor related to the system X 2 − X 3 :
For each rectangle we will apply the equation in (4.82) in which x2 and x3 are now related
r
to the system OX ( X 2 − X 3 ) :

ab b 2 + 12 X 32 − 12 X 2 X 3 
I(OX
q r)
=   (4.116)
ij 12  − 12 X 2 X 3 a 2 + 12 X 22 

a1 b
Rectangle q = 1 : a = a1 = 0.140 , b = b1 = 0.045 , X 2 = = 0.070 , X 3 = 1 = 0.0225 :
2 2
ab b 2 + 12 X 32 − 12 X 2 X 3   4.2525 − 9.9225
IO(1X)r ij =  = × 10 −6 m 4
12  − 12 X 2 X 3 a 2 + 12 X 22   − 9.9225 41.16 

a1 b
Rectangle q = 2 : a = a2 = 0.045 , b = b1 = 0.140 , X 2 = = 0.070 , X 3 = b1 + 2 = 0.115 :
2 2
ab b 2 + 12 X 32 − 12 X 2 X 3   93.6075 − 50.715  −6 4
IO( 2Xr) ij =  =  × 10 m
12  − 12 X 2 X 3 a 2 + 12 X 22   − 50.715 31.933125 

University of Castilla-La Mancha Draft By: Eduardo W. V. Chaves (2016)


Ciudad Real - Spain
4 THE FUNDAMENTAL EQUATIONS OF CONTINUUM MECHANICS 399

a1 b
Rectangle q = 3 : a = a1 = 0.140 , b = b1 = 0.045 , X 2 = = 0.070 , X 3 = 0.230 − 3 = 0.2075 :
2 2
ab b 2 + 12 X 32 − 12 X 2 X 3   272.3175 − 91.5075 
IO( 3X)r ij =  = × 10 −6 m 4
12  − 12 X 2 X 3 a 2 + 12 X 22   − 91.5075 41.16 

Then
 3.701775 − 1.52145  −4 4
IO( Sys
r ) = I (1)r + I ( 2r) + I ( 3)r =
X ij OX ij OX ij OX ij  − 1.52145 1.142531  × 10 m
 
The inertia tensor related to the system located at the Area Centroid:
The inertia tensor at the Area Centroid, (see equation in (4.75)), can be obtained by means
of the Steiner’s theorem:
r r r r r r r r
r ) = I r − A [( X ⊗ X ) − ( X ⋅ X ) 1]
IO( Sys ⇒ IGxr = I OXr + A [( X ⊗ X ) − ( X ⋅ X ) 1]
X Gx

whose components are:


I IG 23  ( Sys )  X 32 − X2X3
( IGxr ) ij =  G 22 =I r − A  (4.117)
 IG 23 IG 33  OX ij − X 2 X 3 X 22 

By substituting the variable values we can obtain:


 IG 22 IG 23  1.20225 0 
 =  × 10 −4 m 4
 IG 23 IG 33   0 0.2164312 

Tangential stress on the cross section


Since the axes are axes of symmetry we can apply the equation in (4.115) in order to obtain
the tangential stress (shear stress):
c
P P
σ13 ( x3 ) =
a ( x3 )IG 22 ∫ ∫ x dx dx
x2 x3
3 3 2 ≡
a ( x3 )I G 22 ∫ x dA
A
3 (4.118)

(f)
σ13 ( x3 = 0.115) = 0

P P 11000
∫ x dA = a I
(+ g )
σ13 ( x3 = 0.070 ) = 3 [ A3 x3( A3 ) ] = [(0.0063)(0.0925)]
a3IG 22 A 3 G 22 (0.140)(1.20225)
(+ g )
⇒ σ13 ( x3 = 0.070) = 3.808484 × 10 5 Pa

P P 11000
∫ x dA = a I
(− g )
σ13 ( x3 = 0.070 ) = 3 [ A3 x3( A3 ) ] = [(0.0063)(0.0925)]
a2I G 22 A 2 G 22 (0.045)(1.20225 )
(− g )
⇒ σ13 ( x3 = 0.070) = 11.848462 × 10 5 Pa
At the neutral axis:
P P  A2 b2  11000

(h) ( A3 )
σ13 ( x3 = 0.0) = x3 dA =  A3 x3 + 2 4  = (0.045)(1.20225 ) [(0.0063)(0.0925)]
a2IG 22 A a2IG 22  
(h)
⇒ σ13 ( x3 = 0.0) = 14.09025 × 10 5 Pa

University of Castilla-La Mancha Draft By: Eduardo W. V. Chaves (2016)


Ciudad Real - Spain
400 SOLVING PROBLEMS BY MEANS OF CONTINUUM MECHANICS

x3 [ σ13 ] = MPa
(f)
f f σ13 =0
(+ g )
σ13 = 0.3808484
g g (− g )
σ13 = 1.1848462

(h)
x2 σ13 = 1.409025
h h

(+ p)
p p σ13 = 1.1848462
(− p)
σ13 = 0.3808484
q q
(q)
σ13 =0

Figure 4.46: Tangential stress distribution on the I-Shaped cross section.

Problem 4.31
Consider the cross section described in Figure 4.47 in which is acting the shearing forces
F21 ≡ Fy and F31 ≡ Fz . a) Obtain the shear flux on the flanges. b) Locate the Shear Center
r
(S.C.) of the cross section by adopting the system OX .
Hypothesis (approximation): Consider that the thickness ( t ) is very small when
compared with a , (see Figure 4.47).

X3
a3
a1 = 2a − t ; b1 = t
3
b3 a2 = 2t ; b2 = 2a + t
2
a3 = a − t ; b3 = 2t
t - thickness

b2

b1
O
X2
a2 a1

Figure 4.47: Cross section.

University of Castilla-La Mancha Draft By: Eduardo W. V. Chaves (2016)


Ciudad Real - Spain
4 THE FUNDAMENTAL EQUATIONS OF CONTINUUM MECHANICS 401

Solution:
Since the thickness is very small we can apply the equation in (4.106) in order to obtain the
shear flux.
Geometric Properties
For each rectangle we will apply the equation in (4.82) in which x2 and x3 are now related
r
to the system OX ( X 2 − X 3 ) :

( qr) aq bq bq2 + 12 X 32 − 12 X 2 X 3 
IOX =  
ij
12  − 12 X 2 X 3 aq2 + 12 X 22 

t
Rectangle q = 1 : a1 = 2a − t ≈ 2a , b1 = t , X 2(1) = a + ≈ a , X 3(1) = 0 :
2
Area: A1 = 2at
a1b1 b12 + 12 X 32 − 12 X 2 X 3  2at t 2 0  1 t 3a 0 
IO(1X)r ij =  =   =  
12  − 12 X 2 X 3 a12 + 12 X 22  12 0 4a 2 + 12a 2  6  0 16 a 3t 

3
Rectangle q = 2 : a2 = 2t , b2 = 2a + t ≈ 2a , X 2( 2 ) = 0 , X 3( 2 ) = a :
2
Area: A2 = 4at
a2b2 b22 + 12 X 32 − 12 X 2 X 3  1 32a 3t 0 
IO( 2Xr) ij =  =  
12  − 12 X 2 X 3 a22 + 12 X 22  6  0 8t 3a 

a+t a
Rectangle q = 3 : a3 = a − t ≈ a , b3 = 2t , X 2(3) = ≈ , X 3( 3) = 2a :
2 2
Area: A3 = 2at
a3b3 b32 + 12 X 32 − 12 X 2 X 3  1  48a 3t − 12 a 3t 
IO( 3X)r ij =  =  
12  − 12 X 2 X 3 a32 + 12 X 22  6  − 12 a 3t 4 a 3t 

Then
Total Area: A = A1 + A2 + A3 = 8at
The Inertia Tensor or Area:
1 80 a 3t + 48t 3 a − 12 a 3t  1  80a 3t − 12a 3t 
IO( Sys
r ) = I (1)r + I ( 2r) + I ( 3)r =
X ij OX ij OX ij OX ij  ≈  
6  − 12 a 3t 20 a 3t + 8t 3 a  6  − 12 a 3t 20 a 3t 
Calculation of the Area Centroid - G :
3

∑A (q)
X 2( q )
A (1)
X 2(1)
+A (2)
X 2( 2)
+A ( 3)
X 2(3)
a
(4at )(a ) + (2at )(0) + (2at ) 
X2 =
q =1
= ≈ 2 = 3a
A ( A(1) + A ( 2)
+ A( 3) ) 8at 8
3

∑A
q =1
(q)
X 3( q )
A(1) X 3(1) + A( 2 ) X 3( 2 ) + A(3) X 3(3) (4at )(0) + (2at )(a ) + (2at )(2a )
X3 = = ≈ =a
A ( A(1) + A( 2) + A(3) ) 8at

University of Castilla-La Mancha Draft By: Eduardo W. V. Chaves (2016)


Ciudad Real - Spain
402 SOLVING PROBLEMS BY MEANS OF CONTINUUM MECHANICS

The Shear Flux


Since we are adopting a system which is not at the Area Centroid we have to consider the
equation (4.106) in order to obtain the shear flux:
s s s s
 ∂σ  Y Y Y

0
∂x1  X 0 X 0 ∫ X 0 ∫
q ( s ) = q (0) −  t 11  ds = q (0) − 0 t ds − 2 (t X 2 )ds − 3 (t X 3 )ds ∫ (4.119)

where the coefficients Y0 , Y2 , Y3 and X are given by the equations (4.107) and (4.108).

22 23 I I  1  80 a 3t − 12 a 3t  3
For this problem we have:, I(OSys r ) =
I ≈  3 
, x2 ← X 2 ≈ a ,
 23 I33  6  − 12 a t
X ij 3
20a t  8
x3 ← X 3 ≈ a , A = 8at . With that the coefficients become:

0 Ax2 Ax3
Fz − I 23 I 22 Ax2 Ax3 Ax2 Ax3
Y0 − Fy − I33 I 23 − I33 I 23 − I 23 I 22 − 27 93
= = − Fz − Fy = 2
Fy − Fz ; (4.120)
X A Ax2 Ax3 X X 97a t 388a 2t
Ax3 − I 23 I 22
− Ax2 − I33 I 23
A 0 Ax3
Ax3 Fz I 22 A Ax3 A Ax3
Y2 − Ax2 − Fy I 23 − Ax2 I 23 Ax3 I 22 48 9
= = Fz + Fy = 3
Fy + Fz ; (4.121)
X A Ax2 Ax3 X X 97a t 97 a 3t
Ax3 − I 23 I 22
− Ax2 − I33 I 23
A Ax2 0
Ax3 − I 23 Fz A Ax2 A Ax2
Y3 − Ax2 − I33 − Fy − Ax2 − I33 Ax3 − I23 9 159
= = − Fz − Fy = Fy + Fz (4.122)
X A Ax2 Ax3 X X 97 a 3t 776a 3t
Ax3 − I23 I22
− Ax2 − I33 I23

Then, the equation in (4.119) becomes


s s
 − 27 93   48 9 
q ( s ) = q ( 0) −  2
 97a t
Fy − 2
Fz  t ds − 
388a t  0 ∫ 3
 97 a t
Fy + 3
Fz  (t x2 )ds
97 a t  0 ∫
s
(4.123)
 9 159 
−
 97 a 3
t
Fy +
776 a 3
t
Fz  (t x3 )ds
0 ∫
We will discretize the path s such as s = s ( 43) + s (32) + s ( 21) , (see Figure 4.48), where the
origin of s = 0 is located at point 4 .

University of Castilla-La Mancha Draft By: Eduardo W. V. Chaves (2016)


Ciudad Real - Spain
4 THE FUNDAMENTAL EQUATIONS OF CONTINUUM MECHANICS 403

X3
a
q ( s = 0) ≡ q ( 4 ) = 0 (free edge)

3
s ( 43) 4
( 32 )
s s = s ( 43) + s ( 32 ) + s ( 21)

X 2 = 83 a

2a ⊗
G

X3 = a

s ( 21) 1
2
O X2
2a

Figure 4.48: Path of s .

Path 4 → 3 : X 2 = a − s ( 43) , X 3 = 2a , t = 2t , 0 ≤ s ( 43) ≤ a


Then, the equation (4.123) becomes:
s s
 − 27 93   48 9 
∫ ∫
( 43) ( 4)
q = q{ −  2
Fy − 2
Fz  ( 2t )ds ( 43) −  3
Fy + 3
Fz  (2t ) (a − s ( 43) )ds ( 43)
=0  97a t 388a t  0  97a t 97 a t  0
s
 9 159 
− 3
 97 a t
Fy + 3
776a t  0 ∫
Fz  (2t )(2a )ds ( 43)

After the integrals are solved we can obtain:


−3 3
q ( 43) ( s ) = 3
s(26a − 16s ) Fy − s (17a − 3s) Fz
97 a 97 a 3
And at the point 3 , ( s = a) , we can obtain:
−3 3 − 30 42
q ( 43) ( s = a ) = q (3) = 3
a(26a − 16a ) Fy − 3
a (17a − 3a ) Fz = Fy − Fz
97a 97 a 97a 97a
Path 3 → 2 : X 2 = 0 , X 3 = 2a − s (32 ) , t = 2t , 0 ≤ s (32) ≤ 2a
Then, the equation (4.123) becomes:
s s
 − 27 93   48 9 
∫ ∫
( 32 ) ( 3)
q =q − 2
Fy − 2
Fz  (2t )ds (32) −  3
Fy + 3
Fz  (2t ) (0)ds (32)
 97 a t 388a t  0  97 a t 97 a t  0
s
 9 159 
−
 97 a 3
t
Fy +
776 a 3
t 0 ∫
Fz  (2t )(2a − s (32) )ds (32)

After the integrals are solved we can obtain:

University of Castilla-La Mancha Draft By: Eduardo W. V. Chaves (2016)


Ciudad Real - Spain
404 SOLVING PROBLEMS BY MEANS OF CONTINUUM MECHANICS

3 3
q (32) ( s ) = q (3) + 3
s (48a + 24s ) Fy − s (88a − 53s ) Fz
776a 776a 3
 − 30 42  3 3
⇒ q (32) ( s ) =  Fy − Fz  + 3
s (48a + 24s ) Fy − s (88a − 53s ) Fz
 97a 97 a  776a 776a 3
And at the point 2 , ( s = 2a) , we can obtain:
 − 30 42  3 3
q ( 2) =  Fy − Fz  + 3
2a (48a + 24(2a )) Fy − 2a (88a − 53(2a )) Fz
 97a 97 a  776a 776a 3
42 57
⇒ q ( 2) = Fy − Fz
97 a 194a
Path 2 → 1 : X 2 = s ( 21) , X 3 = 0 , t = t , 0 ≤ s ( 21) ≤ 2a
Then, the equation (4.123) becomes:
s s
 − 27 93   48 9 
q ( 21) = q ( 2 ) − 
 97 a 2
t
Fy −
388 a 2
t 0 ∫
Fz  (t )ds ( 21) − 
 97 a 3
t
Fy +
97 a 3
t 0 ∫
Fz  (t ) ( s ( 21) ) ds ( 21)

s
 9 159 
− 3
 97 a t
Fy + 3
Fz  (t )(0)ds ( 21)
776a t  0 ∫
After the integrals are solved we can obtain:
3 3
q ( 21) ( s ) = q ( 2 ) + 3
s(36a − 32s ) Fy + s(31a − 6 s) Fz
388a 388a 3
 42 57  3 3
⇒ q ( 21) ( s) =  Fy − Fz  + 3
s (36a − 32 s) Fy + s (31a − 6s ) Fz
 97 a 194a  388a 388a 3
And at the point 1 , ( s = 2a) , we can obtain:
 42 57  3 3
q (1) =  Fy − Fz  + 3
2a(36a − 32(2a )) Fy + 2a (31a − 6(2a)) Fz = 0
 97 a 194a  388a 388a 3

as expected, q (1) = 0 is zero, since we are dealing with a free edge. The Shear Flux can be
appreciated in Figure 4.49.
We have defined the Shear Center (S.C.) in Problem 4.22-NOTE 3. We can calculate the
torsion moment at any point, but by simplicity we will adopt the point 2 ≡ O , since the
shear fluxes q (32) ( s ) and q ( 21) ( s ) will not contribute to the torsion moment, and the
torsion moment produced by the shear flux q ( 43) ( s ) is given by:
s s s a
r r r
∫ ∫ ∫ ∫
M O = X ∧ q ds = ( X 3q ( 43) )eˆ 1ds = ((2a)q ( 43) )eˆ 1ds = 2a q ( 43) ds eˆ 1 = 2af ( 43) eˆ 1
0 0 0 0424
1 3
( 43 )
=f

where we have considered


eˆ 1 eˆ 2 eˆ 3 eˆ 1 eˆ 2 eˆ 3
r r
X ∧ q = X1 X2 X3 = 0 X2 X 3 = ( X 3 q2( 43) )eˆ 1 = (2aq2( 43) )eˆ 1
q1 q2 q3 0 − q ( 43) 0

and F ( 43) is the total force on the flange 4 → 3 . Then, in order to obtain the torsion
moment we have to solve the integral:

University of Castilla-La Mancha Draft By: Eduardo W. V. Chaves (2016)


Ciudad Real - Spain
4 THE FUNDAMENTAL EQUATIONS OF CONTINUUM MECHANICS 405

a a
r  −3 3 
0

M O = 2a q ( 43) dseˆ 1 = 2a 
0
97 a 3 ∫
s(26a − 16 s) Fy −
97a 3
s (17 a − 3s ) Fz  dseˆ 1

r  − 45a 46a  ˆ − 45a 46a
⇒ MO =  Fz − Fy e1 ∴ M X1 = F31 − F21
 97 97  97 97
And if we compare with the equation in Figure 4.10 we can conclude that
− 45a 46a
X 2( S .C .) = ; X 3( S .C .) =
97 97

X3
Shear Center (S.C.)
− 30 F y q ( 43)
42 Fz
− = q ( 3) − 45a
97a 97a q (4) = 0 X 2( S .C .) =
97
q ( 43)
46a
3 X 3( S .C .) =
s ( 43) 4 97
s (32 )

−3
q ( 43) ( s ) = s[(26a − 16 s ) Fy + (17 a − 3s ) Fz ]
97 a 3
q ( 32 ) q ( 32 ) 3
q ( 32) = q ( 3) + s[(48a + 24 s) Fy − (88a − 53s ) Fz ]
776a 3
3
q ( 21) = q ( 2 ) + s[(36a − 32 s ) Fy + (31a − 6s ) Fz ]
(S.C.) 388a 3

r ( S .C .)
X s ( 21) 1
O 2 X2
q ( 21) q (1) = 0
42 Fy 57 Fz
− = q( 2)
97 a 194a
q ( 21)

Figure 4.49: Shear Flux on the flanges.

NOTE 1: Let us consider a generic flange element with length a and thickness t which is
r
constant along the flange element, (see Figure 4.50). By considering the systems OX and
r r r r r r
ix we can obtain that X = X (i ) + x and note that the systems ix and ix ′ are related to
r r r r
each other by the transformation matrix A as follows x′ = A x and x = A T x′ , (see
Figure 4.21), where the transformation matrix is given by
 cosα sin α   l m
A = =
 − sin α cosα   − m l 
r r r r r
Then X = X (i ) + x = X (i ) + A T x′ whose components are:
r r r  X 2   X (i )   l − m   x′2 = s   X 2(i ) + l s 
X = X (i ) + A T x ′ components
  →   =  2(i )  +  =
 X 3   X 3  m l   x3′ = 0  X 3(i ) + m s 

University of Castilla-La Mancha Draft By: Eduardo W. V. Chaves (2016)


Ciudad Real - Spain
406 SOLVING PROBLEMS BY MEANS OF CONTINUUM MECHANICS

0≤s≤a
X3 Node Coordinates
x3
x3′ x′2 , s Node: i ( X 2(i ) , X 3(i ) )
a j
Node: j ( X 2( j ) , X 3( j ) )
r ŝ
r x i α
X
x2

r q (i ) -shear flux at i t - thickness (constant)


X (i )

O X2

Figure 4.50

By means of the previous considerations we can solve the following integrals:


s s s s
s 2l s 2m

0

X 2 ds = ( X 2(i ) + l s )ds = X 2(i ) s +
0
2
; ∫
0

X 3ds = ( X 3(i ) + m s )ds = X 3(i ) s +
0
2

And the equation in (4.119) can be written as follows


s s s
Y0 t Yt Yt
q ( s ) = q (i ) −
X 0 ∫ ds − 2 ∫ X 2 ds − 3 ∫ X 3ds
X 0 X 0
Y0 t Y t s 2l  Y3 t  (i ) s 2m 
⇒ q( s) = q (i ) − s − 2  X 2(i ) s + −
 X 
 X3 s + 

X X  2   2 
 Y Y X ( i ) Y X (i )  Y l Y m 
⇒ q ( s ) = q (i ) − t  0 + 2 2 + 3 3  s − t  2 + 3  s 2
X X X   2X 2X 
⇒ q ( s ) = q (i ) + t d1s + t d 2 s 2
where
 Y Y X (i ) Y X (i )  Y l Y m 
d1 = − 0 + 2 2 + 3 3  ; d 2 = − 2 + 3 
X X X   2X 2X 

The coefficients Y0 , Y2 and Y3 , (see equations (4.120)-(4.122)), can be rewritten as follows


0 AX 2 AX 3
AX 2 AX 3 AX 2 AX 3
Y0 = Fz − I 23 I 22 = − Fy − Fz = p0(1) Fy + p0( 2) Fz (4.124)
− I 23 I 22 − I33 I 23
− Fy − I33 I 23
A 0 AX 3
A AX 3 A AX 3
Y2 = AX 3 Fz I 22 = Fy + Fz = p2(1) Fy + p2( 2) Fz (4.125)
AX 3 I22 − AX 2 I23
− AX 2 − Fy I 23
A AX 2 0
A AX 2 A AX 2
Y3 = AX 3 − I23 Fz = − Fy − Fz = p3(1) Fy + p3( 2 ) Fz (4.126)
AX 3 − I 23 − AX 2 − I33
− AX 2 − I33 − Fy

University of Castilla-La Mancha Draft By: Eduardo W. V. Chaves (2016)


Ciudad Real - Spain
4 THE FUNDAMENTAL EQUATIONS OF CONTINUUM MECHANICS 407

And the coefficients d1 and d 2 can be rewritten as follows:


 Y Y X (i ) Y X (i ) 
d1 = − 0 + 2 2 + 3 3 
X X X 

⇒ d1 =
− 1 (1)
X
[
p0 Fy + p0( 2) Fz + ( p2(1) Fy + p2( 2 ) Fz ) X 2(i ) + ( p3(1) Fy + p3( 2) Fz ) X 3(i ) ]
 − 1 (1)
⇒ d1 =  [   − 1 (2)
p0 + p2(1) X 2(i ) + p3(1) X 3(i ) Fy +  ] 
[ F
p0 + p2( 2 ) X 2(i ) + p3( 2 ) X 3(i )  Fz = d1 y Fy + d1Fz Fz ]
X  X 
 Y l Y m  −1
d 2 = − 2 + 3  = [
( p2(1) Fy + p2( 2 ) Fz ) l + ( p3(1) Fy + p3( 2 ) Fz ) m ]
 2X 2X  2X

⇒ d2 = 
 − 1 (1)
[ 
p2 l + p3(1) m  Fy + 
 − 1 ( 2)
]  F
[
p2 l + p3( 2 ) m  Fz = d 2 y Fy + d 2Fz Fz ]
 2X   2X 
Then, we can represent the shear flux ( q ( s ) = q (i ) + t d1s + t d 2 s 2 ) as follows

F F
q ( s) = (qF(iy) + t d1 y s + t d 2 y s 2 ) Fy + (qF(iz) + t d1Fz s + t d 2Fz s 2 ) Fz (Shear Flux)

where
F
d1 = d1 y Fy + d1Fz Fz
F
d1 y =
−1
X
[p (1)
0 + p2(1) X 2(i ) + p3(1) X 3(i ) ] ; d1Fz =
−1
X
[p ( 2)
0 + p2( 2 ) X 2(i ) + p3( 2 ) X 3(i ) ]
F
d 2 = d 2 y Fy + d 2Fz Fz
F
d2 y =
− 1 (1)
2X
p2 [ l + p3(1) m ] ; d 2Fz =
2X
p2[
− 1 (2)
l + p3( 2 ) m]
X 2( j ) − X 2(i ) ( j)
− X 3(i )
with l = , m = X3 (4.127)
a a
AX 2 AX 3 AX 2 AX 3
p0(1) = − ; p0( 2 ) = −
− I 23 I22 − I33 I 23

A AX 3 A AX 3
p2(1) = ; p2( 2 ) =
AX 3 I22 − AX 2 I23

A AX 2 AX 3
A AX 2 A AX 2
p3(1) =− ; p3( 2 ) =− , X = AX 3 − I 23 I 22
AX 3 − I 23 − AX 2 − I33
− AX 2 − I33 I 23

The shear flux at the end of the flange (node j ) is given by


F F
q F( yj ) ( s = a ) = q F( yj ) = q F(iy) + t d1 y a + t d 2 y a 2 ; q F( zj ) ( s = a ) = q F( zj ) = q F(iz) + t d1Fz a + t d 2Fz a 2

The torsion moment at the point O


r
The torsion moment due to the shear flux q = qsˆ can be calculated as follows:

University of Castilla-La Mancha Draft By: Eduardo W. V. Chaves (2016)


Ciudad Real - Spain
408 SOLVING PROBLEMS BY MEANS OF CONTINUUM MECHANICS

a
r r r

M O = X ∧ q ds
0

where
eˆ ′2 = sˆ   l m  eˆ 2   l eˆ 2 + m eˆ 3 
 ˆ = =
 e′3   − m l  eˆ 3  − m eˆ 2 + l eˆ 3 
r r
X ∧ q = ( X 2eˆ 2 + X 3eˆ 3 ) ∧ (qsˆ) = q( X 2eˆ 2 + X 3eˆ 3 ) ∧ (l eˆ 2 + m eˆ 3 )
= l X 2 q eˆ 2 ∧ eˆ 2 + m X 2 q eˆ 2 ∧ eˆ 3 + l X 3q eˆ 3 ∧ eˆ 2 + m X 3q eˆ 3 ∧ eˆ 3
142r4 3 1
424 3 1424 3 12r 3
=0 = eˆ 1 = − eˆ 1 =0

= (m X 2 q − l X 3q )eˆ 1
Then, the magnitude of the torsion moment is given by:
a a
MO = ∫ m X 2 qds − ∫ l
0 0
X 3 qds

a a

∫ m (X 2 + l ∫l ( X 3(i ) + m s)(q (i ) + t d1 s + t d 2 s 2 )ds


(i )
⇒ MO = s )(q (i ) + t d1 s + t d 2 s 2 )ds −
0 0

After the integrals are solved we can obtain:

[m X 2(i ) − l X 3(i ) ]a
MO = (3atd1 + 2a 2td 2 + 6q (i ) ) (4.128)
6
Taking into account the equations in (4.127), the torsion moment can be split additively
into
F
M O = M O y Fy + M OFz Fz = −( X 3( S .C .) ) Fy + ( X 2( S .C .) ) Fz (4.129)
where
F [m X 2(i ) − l X 3(i ) ]a F F
MOy = (3atd1 y + 2a 2td 2 y + 6qF(iy) ) (4.130)
6
and
[m X 2(i ) − l X 3(i ) ]a
M OFz = (3atd1Fz + 2a 2td 2Fz + 6qF(iz) ) (4.131)
6
If we compare the equation (4.129) with the equation in Figure 4.10 we can conclude that
F
X 2( S .C .) = M OFz ; X 3( S .C .) = − M O y (Shear Center)
The total force in the flange
The total force in the flange can be obtained as follows:
a a a

∫ ∫ ∫
F F
f (e)
= q ( s )ds = Fy ( q F(iy) +t d1 y s +t d 2 y s 2 )ds + Fz (q F(iz) + t d1Fz s + t d 2Fz s 2 )ds
0 0 0

 F a F a 
3
 a a3 
⇒ f ( e ) =  qF(iy) a + t d1 y + t d 2 y  Fy + qF(iz) a + t d1Fz + t d 2Fz (e) (e)
 Fz = f Fy Fy + f Fz Fz
 2 3   2 3 

University of Castilla-La Mancha Draft By: Eduardo W. V. Chaves (2016)


Ciudad Real - Spain
4 THE FUNDAMENTAL EQUATIONS OF CONTINUUM MECHANICS 409

Problem 4.32
Consider the cross section described in Figure 4.51 in which is acting the shearing forces
F21 ≡ Fy and F31 ≡ Fz . a) Obtain the shear flux on the flanges. b) Locate the Shear Center
r
(S.C.) of the cross section by adopting the system OX .
Hypothesis (approximation): Consider that the thickness ( t ) is very small when
compared with length flange, (see Figure 4.51).

X3
Node Coordinates i
Node X 2( i ) X 3( i )
1 i =1 15 42.32
1 i=2 0 33.66
i=3 0 8.66
2 i=4 15 0

Flanges e
Path
2 Flange Thickness (t )
i→ j
e =1 1→ 2 2
e=2 2→3 2
e=3 3→ 4 2

3
3

4
O
X2

Figure 4.51: Cross section – Dimensions in centimeter ( cm ).


Solution:
We will use the information provided in Problem 4.23-NOTE 4, in which we have defined

at  t 2 l 2 + a 2 m 2 + 12( X 3( g ) ) 2 lm (t 2 − a 2 ) − 12 X 2( g ) X 3( g ) 
I O( eX)r ij =   (4.132)
12 lm (t 2 − a 2 ) − 12 X 2( g ) X 3( g ) t 2 m 2 + a 2 l 2 + 12( X 2( g ) ) 2 
( j)
X 2( j ) − X 2(i ) − X 3(i )
where a = ( X 2( j ) − X 2(i ) ) 2 + ( X 3( j ) − X 3(i ) ) 2 , l = , m = X3 ,
a a
X 2(i ) + X 2( j ) X (i ) + X 3( j )
X 2( g ) = , X 3( g ) = 3
2 2
And the inertia tensor of area for the compound if given by:
3
I O( Sys
r ) =
X ∑I
e =1
( e )r
OX

University of Castilla-La Mancha Draft By: Eduardo W. V. Chaves (2016)


Ciudad Real - Spain
410 SOLVING PROBLEMS BY MEANS OF CONTINUUM MECHANICS

The flange geometric characteristics are described in Table 4.1.


Table 4.1
Area
Flange ( X 2(i ) ; X 3(i ) ) ( X 2( j ) ; X 3( j ) ) t a l m X 2( g ) X 3( g )
(e )
A
e =1 ( 15;42.32 ) ( 0;33.662 ) 2 17.32 34.64 − 0.866 − 0.5 7 .5 37.99
e=2 ( 0;33.66 ) ( 0;8.66 ) 2 25 50 0 −1 0 21.16
e=3 ( 0;8.66 ) ( 15;0 ) 2 17.32 34.64 0.866 − 0 .5 7 .5 4.33

Area Centroid of the Compound and Total Area


The total area is A = A(1) + A( 2) + A(3) = 34.64 + 50 + 34.64 = 119.28cm2
And the area centroid is given by
A(1) X 2( g1) + A( 2 ) X 2( g 2 ) + A(3) X 2( g 3)
X2 = = 4.35614cm
A(1) + A( 2 ) + A(3)
A(1) X 3( g1) + A( 2 ) X 3( g 2 ) + A(3) X 3( g 3)
X3 = = 21.16cm
A(1) + A( 2 ) + A(3)
The Inertia Tensor of Area
As we are adopting that the thickness is very small the terms related to t 2 in the equation
(4.132) can be discarded, so the equation (4.132) becomes

at  a 2 m 2 + 12( X 3( g ) ) 2 − lma 2 − 12 X 2( g ) X 3( g ) 
IO( eX)r ij ≈   (4.133)
12  − lma 2 − 12 X 2( g ) X 3( g ) a 2 l 2 + 12( X 2( g ) ) 2 

Flange 1: By substituting the data related to the flange e = 1 , (see Table 4.1), we can obtain
 5.02103 − 1.02448 
I (O1X)r ij ≈  4
 × 10 cm
4

 − 1 . 02448 0 . 259792 
Flange 2: By substituting the data related to the flange e = 2 , (see Table 4.1), we can obtain
 2.49914 0
I O( 2Xr) ij ≈  4
 × 10 cm
4

 0 0 
Flange 3: By substituting the data related to the flange e = 3 , (see Table 4.1), we can obtain
0.086594919 − 0.0749978 
IO(3X)r ij ≈   × 10 4 cm 4
 − 0.0749978 0.259792 
Then, the inertia tensor for the compound is given by
I I 23  − 1.09947 
(I )  7.60677
3
( Sys
r )
OX ij
=  22
I 23 I33 
= ∑ (I
e =1
( e )r
OX ij
) = IO(1X)r ij + IO( 2Xr) ij + IO( 3X)r ij = 
 − 1.09947 5.19585 
× 10 4 cm 4

Shear Flux
By means of the equations in (4.127) we can calculate the shear flux in the flanges. Taking
into account the geometric properties calculated previously we can calculate the
coefficients related to the cross section:

University of Castilla-La Mancha Draft By: Eduardo W. V. Chaves (2016)


Ciudad Real - Spain
4 THE FUNDAMENTAL EQUATIONS OF CONTINUUM MECHANICS 411

AX 2 AX 3 AX 2 AX 3
p0(1) = − = −1.17732 × 107 ; p0( 2 ) = − = −7.40145 × 106
− I 23 I 22 − I33 I23

A AX 3 A AX 3
p2(1) = = 2.70296 × 106 ; p2( 2 ) = = −41.19111
AX 3 I22 − AX 2 I 23

A AX 2 A AX 2
p3(1) = − = −41.19111 ; p3( 2 ) = − = 3.49793 × 105 ,
AX 3 − I 23 − AX 2 − I 33

A AX 2 AX 3
X = AX 3 − I 23 I 22 = 7.92654 × 109
− AX 2 − I33 I 23

Flange 1: (Flange data ( X 2(i ) , X 3(i ) , t , l , m ) in Table 4.1)


q F(iy) = 0 , q F(iz) = 0 (free edge)

F
d1 y =
−1
X
[p (1)
0 ]
+ p2(1) X 2(i ) + p3(1) X 3(i ) = −3.62951 × 10 −3

d1Fz =
−1
X
[p ( 2)
0 ]
+ p2( 2) X 2(i ) + p3( 2 ) X 3(i ) = −9.33723 × 10 −4

F
d2 y =
− 1 (1)
2X
[
p2 l + p3(1) m = 1.47652 × 10 −4 ]
d 2Fz =
− 1 (2)
2X
[
p2 l + p3( 2 ) m = 1.10301 × 10 −5 ]
Shear Flux in the Flange 1
F F
q ( s ) = (q F(iy) + t d1 y s + t d 2 y s 2 ) Fy + (q F(iz) + t d1Fz s + t d 2Fz s 2 ) Fz = q F( ey=1) Fy + q F( ez =1) Fz

The terms q F( ey=1) and q F( ez =1) can be evaluated, respectively, as follows:


F F
q F( ey=1) ( s) = q F(iy) + t d1 y s + t d 2 y s 2 = t[(−3.62951 × 10 −3 ) s + (1.47652 × 10 −4 ) s 2 ]
{
=0

q F( ez =1) ( s ) = q F(iz) + t d1Fz s + t d 2Fz s 2 = t[(−9.33723 × 10 −4 ) s + (1.10301 × 10 −5 ) s 2 ]


{
=0

at the end s = 17.32 , (node 2 )


q F( ey=1) ( s = 17.32) = t[(−3.62951 × 10 −3 ) s + (1.47652 × 10 −4 ) s 2 ] = −0.01857t = q F( yj )

q F( ez =1) ( s = 17.32) = t[(−9.33723 × 10 −4 ) s + (1.10301 × 10 −5 ) s 2 ] = −0.01286t = q F( zj )

Torsion Moment at O due to Flange 1, (see equations (4.130) and (4.131)):

(M ) F y ( e =1)
O =
[m X 2(i ) − l X 3(i ) ]a
6
F F
(3atd1 y + 2a 2td 2 y + 6qF(iy) ) = −16.82927

and

(M ) Fz ( e =1)
O =
[m X 2(i ) − l X 3(i ) ]a
6
(3atd1Fz + 2a 2td 2Fz + 6qF(iz) ) = −7.05101

University of Castilla-La Mancha Draft By: Eduardo W. V. Chaves (2016)


Ciudad Real - Spain
412 SOLVING PROBLEMS BY MEANS OF CONTINUUM MECHANICS

Update Variables
q F(iy) ← q F( yj ) = −0.01857t ; q F(iz) ← q F( zj ) = −0.01286t

Flange 2: (Flange data ( X 2(i ) , X 3(i ) , t , l , m ) in Table 4.1)


q F(iy) = −0.01857t , q F(iz) = −0.01286t

F
d1 y =
−1
X
[p (1)
0 ]
+ p2(1) X 2(i ) + p3(1) X 3(i ) = 1.48547 × 10 −3

d1Fz =
−1
X
[p (2)
0 + p2( 2 ) X 2(i ) + p3( 2 ) X 3(i ) ] = −5.5164 ×10 −4

F
d2 y =
− 1 (1)
2X
[
p2 l + p3(1) m = −2.5983 × 10 −9 ]
d 2Fz =
− 1 (2)
2X
[
p2 l + p3( 2 ) m = 2.20647 × 10 −5 ]
Shear Flux in the Flange 2
F F
q ( s ) = (q F(iy) + t d1 y s + t d 2 y s 2 ) Fy + (q F(iz) + t d1Fz s + t d 2Fz s 2 ) Fz = q F( ey=2 ) Fy + q F(ez =2) Fz

The terms q F( ey=2) and q F( ez =2) can be evaluated, respectively, as follows:


F F
q F( ey=2 ) ( s ) = q F(iy) + t d1 y s + t d 2 y s 2 = t[(−0.01857) + (1.48547 × 10 −3 ) s + (−2.5983 × 10 −9 ) s 2 ]

q F( ez =2 ) ( s ) = q F(iz) + t d1Fz s + t d 2Fz s 2 = t[(−0.01286) + (−5.5164 × 10 −4 ) s + (2.20647 × 10 −5 ) s 2 ]

at the end s = 25 , (node 3 )


q F( ey=2 ) ( s = 25) = t[(−0.01857) + (1.48547 × 10 −3 ) s + (−2.5983 × 10 −9 ) s 2 ] = 0.01857t = q F( yj )

q F( ez =2 ) ( s = 25) = t[(−0.01286) + (−5.5164 × 10 −4 ) s + (2.20647 × 10 −5 ) s 2 ] = −0.01286t = q F( zj )

Torsion Moment at O due to Flange 2, (see equations (4.130) and (4.131)):

(M ) Fy (e = 2)
O =
[m X 2(i ) − l X 3(i ) ]a
6
F F
(3atd1 y + 2a 2td 2 y + 6qF(iy) ) = 0

and

(M ) Fz ( e = 2 )
O =
[m X 2(i ) − l X 3(i ) ]a
6
(3atd1Fz + 2a 2td 2Fz + 6qF(iz) ) = 0

Update Variables
q F(iy) ← q F( yj ) = 0.01857t ; qF(iz) ← qF( zj ) = −0.01286t

Flange 3: (Flange data ( X 2(i ) , X 3(i ) , t , l , m ) in Table 4.1)


q F(iy) = 0.01857t , q F(iz) = −0.01286t

F
d1 y =
−1
X
[p (1)
0 ]
+ p2(1) X 2(i ) + p3(1) X 3(i ) = 1.48534 × 10 −3

d1Fz =
−1
X
[p ( 2)
0 ]
+ p2( 2 ) X 2(i ) + p3( 2 ) X 3(i ) = 5.51595 × 10 −4

University of Castilla-La Mancha Draft By: Eduardo W. V. Chaves (2016)


Ciudad Real - Spain
4 THE FUNDAMENTAL EQUATIONS OF CONTINUUM MECHANICS 413

F
d2 y =
− 1 (1)
2X
[
p2 l + p3(1) m = −1.47655 × 10 −4 ]
d 2Fz =
− 1 (2)
2X
[
p2 l + p3( 2 ) m = 1.10346 × 10 −5 ]
Shear Flux in the Flange 3
F F
q ( s ) = (q F(iy) + t d1 y s + t d 2 y s 2 ) Fy + ( q F(iz) + t d1Fz s + t d 2Fz s 2 ) Fz = q F( ey=3) Fy + q F( ez =3) Fz

The terms q F( ey=3) and q F( ez=3) can be evaluated, respectively, as follows:


F F
q F( ey=3) ( s ) = q F(iy) + t d1 y s + t d 2 y s 2 = t[(0.01857) + (1.48534 × 10 −3 ) s + (−1.47655 × 10 −4 ) s 2 ]

q F( ez =3) ( s ) = q F(iz) + t d1Fz s + t d 2Fz s 2 = t[(−0.01286) + (5.51595 × 10 −4 ) s + (1.10346 × 10 −5 ) s 2 ]

at the end s = 17.32 , (node 4 )


q F( ey= 3) = t[(0.01857) + (1.48534 × 10 −3 ) s + (−1.47655 × 10 −4 ) s 2 ] = −2.78 × 10−6 t = qF( jy) ≈ 0

q F( ez = 3) = t[(−0.01286) + (5.51595 × 10−4 ) s + (1.10346 × 10 −5 ) s 2 ] = 4.24 × 10 −11 t = qF( zj ) ≈ 0

Torsion Moment at O due to Flange 3, (see equations (4.130) and (4.131)):

(M ) F y ( e = 3)
O =
[m X 2(i ) − l X 3(i ) ]a
6
F F
(3atd1 y + 2a 2td 2 y + 6qF(iy) ) = −4.32891

and

(M ) F z ( e = 3)
O =
[m X 2(i ) − l X 3(i ) ]a
6
(3atd1Fz + 2a 2td 2Fz + 6qF(iz) ) = 1.81423

Update Variables
q F(iy) ← q F( yj ) = −2.78 × 10 −6 t ≈ 0 ; q F(iz) ← q F( zj ) = 4.24 × 10 −11 t ≈ 0

The Total Torsion Moment at O


(M ) Fy ( Sys )
O ( )
= M Oy
F ( e =1)
( )
+ M Oy
F ( e=2)
( )
+ M Oy
F ( e =3 )
= (−16.82927) + (0) + (−4.32891) = −21.1582

(M ) Fz ( Sys )
O (
= M OFz )
( e=1)
(
+ M OFz )
( e =2 )
(
+ M OFz )
( e =3 )
= (−7.05101) + (0) + (1.81423) = −5.2367
Then
F
M O = M O y Fy + M OFz Fz = −(21.1582) Fy + (−5.2367) Fz = −( X 3( S .C .) ) Fy + ( X 2( S .C .) ) Fz

The Shear Center


If we compare the above equation with the equation in Figure 4.10 we can conclude that
X 2( S .C .) = −5.2367cm ; X 3( S .C .) = 21.1582cm

Note that the cross section has one axis of symmetry at X 3( A.C .) = X 3( S .C .) = 21.1582cm .

University of Castilla-La Mancha Draft By: Eduardo W. V. Chaves (2016)


Ciudad Real - Spain
414 SOLVING PROBLEMS BY MEANS OF CONTINUUM MECHANICS

Problem 4.33
Obtain the shear flux in each flange and locate the shear center for the cross section
described in Figure 4.52. Note that the nodes 2, 7 and 8 have the same coordinates, and the
same for the nodes 4, 9 and 10.

X3 Coordinates
Nodes
X2 X3
3 1 60 60
1
s ( 2−8) 2 30 30
s (1−7 ) 3 0 60
2 4 30 0
1
5 15 0
6 45 0
8 7 7 30 30
8 30 30
9 30 0
s ( 2−4) 2
10 30 0

Element Connectivity
3 i→ j Thickness
e
1 1→ 7 t1 = 1
4 2 3→8 t2 = 1
s ( 9 −5 ) s (10−6)
3 2→4 t3 = 1
O 5 4 9 10 5 6 X2 4 9→5 t4 = 2
5 10 → 6 t5 = 2

Figure 4.52: Cross section – dimensions in centimeter ( cm ).


Solution:
The element length can be calculated by means of a = ( X 2( j ) − X 2(i ) ) 2 + ( X 3( j ) − X 3(i ) ) 2 . The
flange geometric characteristics are described in Table 4.2.
Table 4.2
Area
Flange ( X 2(i ) ; X 3(i ) ) ( X 2( j ) ; X 3( j ) ) t a
(e )
l m X 2( ge ) X 3( ge )
A
−1 −1
e =1 ( 60;60 ) ( 30;30 ) 1 30 2 30 2 45 45
2 2
1 −1
e=2 ( 0;60 ) ( 30;30 ) 1 30 2 30 2 15 45
2 2
e=3 ( 30;30 ) ( 30;0 ) 1 30 30 0 −1 30 15
e=4 ( 30;0 ) ( 15;0 ) 2 15 30 −1 0 22.5 0
e=5 ( 30;0 ) ( 45;0 ) 2 15 30 1 0 37.5 0

Area Centroid of the Compound and Total Area


The total area is A = A(1) + A( 2) + A(3) + A( 4) + A(5) = 174.8528

University of Castilla-La Mancha Draft By: Eduardo W. V. Chaves (2016)


Ciudad Real - Spain
4 THE FUNDAMENTAL EQUATIONS OF CONTINUUM MECHANICS 415

And the area centroid is given by


A(1) X 2( g1) + A( 2 ) X 2( g 2) + A(3) X 2( g 3) + A( 4 ) X 2( g 4 ) + A(5) X 2( g 5)
X2 = = 30
A(1) + A( 2 ) + A(3) + A( 4 ) + A(5)
A(1) X 3( g1) + A( 2 ) X 3( g 2) + A(3) X 3( g 3) + A( 4 ) X 3( g 4 ) + A(5) X 3( g 5)
X3 = = 24.411255
A(1) + A( 2 ) + A(3) + A( 4 ) + A(5)
The Inertia Tensor of Area (for the flange e )

at  a 2 m 2 + 12( X 3( g ) ) 2
− lma 2 − 12 X 2( g ) X 3( g ) 
IO( eX)r ij ≈   (4.134)
12  − lma 2 − 12 X 2( g ) X 3( g )
a 2 l 2 + 12( X 2( g ) ) 2 
r
The inertia tensors for the flanges related to the system OX are
 8.90955 − 8.90955   8.90955 − 2.54558
I(OX
1)r
≈  × 10 4 ; I(OX
2 r)
≈ 4
 × 10 ;
ij
 − 8 . 90955 8 . 90955 
ij
 − 2 . 54558 1 . 27279 

( 3 )r  0 .9 − 1.35 0 0  0 0 
IOX ≈  × 10 4 ; I(OX
4 r)
≈  × 10 4 and I(OX
4 r)
≈  × 10
4
ij
 − 1 . 35 2 . 7 
ij
 0 1 . 575 
ij
 0 4 . 275 
Then, the inertia tensor for the compound is given by
I 22 I 23  5 ( e )
I(OSys
r ) =
X ij I
 23
= ∑
(I r ) ij = I(O1X)r ij + IO( 2Xr) ij + IO( 3X)r ij + I(O4Xr) ij + IO( 5X)r ij
I33  e =1 OX
I 22 I 23   1.87191 − 1.28051
⇒ IO( Sys
r ) =  =  − 1.28051 1.87323  × 10
5
X ij I I
 23 33   
Just as exercise, let us calculate the inertia tensor at the Area Centroid, which can be
obtained by means of the Steiner’s theorem:
r r r r r r r r
I O( Sys r ) − A [( X ⊗ X ) − ( X ⋅ X ) 1]
r ) = I ( Sys ⇒ IG( Sys r ) + A [( X ⊗ X ) − ( X ⋅ X ) 1]
r ) = I ( Sys
X G x x OX

whose components are:


 IG 22 IG 23  ( Sys )  X 32 − X2X3
( IG( Sys
r )
x ) ij =   = I r − A  
 IG 23 IG 33  OX ij − X 2 X 3 X 22 

 IG 22 IG 23  8.2994479 0 
( IG( Sys
r )
x ) ij =  =  × 10 4
 IG 23 IG 33   0 2.9955844 

Shear Flux
By means of the equations in (4.127) we can calculate the shear flux in the flanges. Taking
into account the geometric properties calculated previously we can calculate the
coefficients related to the cross section:
AX 2 AX 3 AX 2 AX 3
p0(1) = − = −4.35355 ×108 ; p0( 2 ) = − = −1.27863 × 108
− I 23 I 22 − I33 I 23

A AX 3 A AX 3
p2(1) = = 1.45118 × 107 ; p2( 2 ) = = 9.85892 × 10−10
AX 3 I 22 − AX 2 I 23

A AX 2 A AX 2
p3(1) = − = 9.85892 × 10 −10 ; p3( 2 ) = − = 5.23786 ×106 ,
AX 3 − I 23 − AX 2 − I33

University of Castilla-La Mancha Draft By: Eduardo W. V. Chaves (2016)


Ciudad Real - Spain
416 SOLVING PROBLEMS BY MEANS OF CONTINUUM MECHANICS

A AX 2 AX 3
X = AX 3 − I23 I22 = 4.34714 × 1011
− AX 2 − I33 I 23

Flange 1: (Flange data ( X 2(i ) , X 3(i ) , t , l , m ) in Table 4.2)


q F(iy) = 0 , q F(iz) = 0 (free edge)

F
d1 y =
X
[p
−1 (1)
0 ]
+ p2(1) X 2(i ) + p3(1) X 3(i ) = −1.00147 × 10− 3 ⇒
F
td1 y = −1.00147 ×10 − 3

d1Fz =
−1
X
[p ( 2)
0 + p2( 2) X 2(i ) + p3( 2 ) X 3(i ) ] = −4.28809 ×10 −4
⇒ td1Fz = −4.28809 × 10 − 4

F
d2 y =
− 1 (1)
2X
[
p2 l + p3(1) m = 1.18025 ×10 − 5 ] ⇒
F
td 2 y = 1.18025 ×10 − 5

d 2Fz =
− 1 (2)
2X
[
p2 l + p3( 2 ) m = 4.25996 × 10 − 6 ] ⇒ td 2Fz = 4.25996 × 10 − 6

Shear Flux in the Flange 1


q F(iy) = qF(1y) = 0 , q F(iz) = qF(1z) = 0
F F
q ( s ) = (q F(iy) + t d1 y s + t d 2 y s 2 ) Fy + (q F(iz) + t d1Fz s + t d 2Fz s 2 ) Fz = q F( ey=1) Fy + q F( ez =1) Fz

The terms q F( ey=1) and q F( ez =1) can be evaluated, respectively, as follows:


F F
q F( ey=1) ( s ) = qF(1y) + t d1 y s + t d 2 y s 2 = −1.00147 × 10−3 s + 1.18025 × 10 −5 s 2
{
=0

q F( ez =1) ( s ) = qF(iz) + t d1Fz s + t d 2Fz s 2 = −4.28809 × 10 −4 s + 4.25996 × 10 −6 s 2


{
=0

at the end s = 30 2 , (node 7 )


q F( ey=1) ( s = 30 2 ) = −1.00147 × 10 −3 s + 1.18025 × 10 −5 s 2 = −0.0212445 = q F( 7y)

q F( ez =1) ( s = 30 2 ) = −4.28809 × 10 −4 s + 4.25996 × 10 −6 s 2 = −0.0105249 = qF( 7z )

Torsion Moment at O due to Flange 1, (see equations (4.130) and (4.131)):

(M ) F y ( e =1)
O =
[m X 2(i ) − l X 3(i ) ]a
6
F F
(3atd1 y + 2a 2td 2 y + 6qF(iy) ) = 0

and

(M ) Fz ( e =1)
O =
[m X 2(i ) − l X 3(i ) ]a
6
(3atd1Fz + 2a 2td 2Fz + 6qF(iz) ) = 0

Flange 2: (Flange data ( X 2(i ) , X 3(i ) , t , l , m ) in Table 4.2)


q F(iy) = q F(3y) = 0 , q F(iz) = qF(3z) = 0

F
d1 y =
X
[p
−1 (1)
0 ]
+ p2(1) X 2(i ) + p3(1) X 3(i ) = 1.001474 × 10 − 3 ⇒
F
td1 y = 1.001474 × 10 − 3

d1Fz =
−1
X
[p ( 2)
0 + p2( 2 ) X 2(i ) + p3( 2 ) X 3(i ) ] = −4.2880858 ×10 −4
⇒ td1Fz = −4.2880858 × 10 − 4

University of Castilla-La Mancha Draft By: Eduardo W. V. Chaves (2016)


Ciudad Real - Spain
4 THE FUNDAMENTAL EQUATIONS OF CONTINUUM MECHANICS 417

F
d2 y =
− 1 (1)
2X
[ ]
p2 l + p3(1) m = −1.1802485 × 10 − 5 ⇒
F
td 2 y = −1.1802485 × 10 − 5

d 2Fz =
− 1 ( 2)
2X
[
p2 l + p3( 2) m = 4.2599628 × 10− 6 ] ⇒ td 2Fz = 4.2599628 × 10 − 6

Shear Flux in the Flange 2


F F
q ( s ) = (q F(iy) + t d1 y s + t d 2 y s 2 ) Fy + (q F(iz) + t d1Fz s + t d 2Fz s 2 ) Fz = q F( ey=2 ) Fy + q F(ez =2) Fz

The terms q F( ey=2) and q F( ez =2) can be evaluated, respectively, as follows:


F F
q F( ey= 2) ( s) = qF(iy) + t d1 y s + t d 2 y s 2 = 1.001474 × 10 −3 s − 1.1802485 × 10 −5 s 2

q F( ez = 2 ) ( s ) = qF(iz) + t d1Fz s + t d 2Fz s 2 = −4.2880858 × 10 −4 s + 4.2599628 × 10 −6 s 2

at the end s = 30 2 , (node 8 )


q F( ey= 2 ) ( s = 30 2 ) = 1.001474 × 10 −3 s − 1.1802485 × 10 −5 s 2 = 0.0212445 = qF(8y)

q F( ez = 2 ) ( s = 30 2 ) = −4.2880858 × 10 −4 s + 4.2599628 × 10 −6 s 2 = −0.01052487 = qF(8z)

Torsion Moment at O due to Flange 2, (see equations (4.130) and (4.131)):

(M ) Fy ( e = 2)
O =
[m X 2(i ) − l X 3(i ) ]a
6
F F
(3atd1 y + 2a 2td 2 y + 6qF(iy) ) = −25.49336686

and

(M ) Fz ( e = 2 )
O =
[m X 2(i ) − l X 3(i ) ]a
6
(3atd1Fz + 2a 2td 2Fz + 6qF(iz) ) = 11.772767

Shear flux compatibility at node 2

8 7
q ( 7 ) + q (8) = q ( 2 )
qF( 2 ) = −0.0212445 + 0.0212445 = 0
 y
 ( 2)
qFz = −0.0105249 − 0.0105249 = −0.0210497

Flange 3: (Flange data ( X 2(i ) , X 3(i ) , t , l , m ) in Table 4.2)


q F( 2y) = 0 , q F( 2z ) = −0.0210497

F
d1 y =
X
[p
−1 (1)
0 + p2(1) X 2(i ) + p3(1) X 3(i ) = 0] ⇒ td 1
Fy
=0

d1Fz =
−1
X
[p ( 2)
0 + p2( 2 ) X 2(i ) + p3( 2 ) X 3(i ) ] = −6.73388 ×10 −5
⇒ td1Fz = −6.73388 × 10 − 5

F
d2 y =
− 1 (1)
2X
[
p2 l + p3(1) m = 0 ⇒ td 2 y = 0 ]
F

d 2Fz =
− 1 (2)
2X
[
p2 l + p3( 2 ) m = 6.0245 × 10 − 6 ]⇒ td 2Fz = 6.0245 × 10− 6

University of Castilla-La Mancha Draft By: Eduardo W. V. Chaves (2016)


Ciudad Real - Spain
418 SOLVING PROBLEMS BY MEANS OF CONTINUUM MECHANICS

Shear Flux in the Flange 3


F F
q ( s ) = (q F(iy) + t d1 y s + t d 2 y s 2 ) Fy + ( q F(iz) + t d1Fz s + t d 2Fz s 2 ) Fz = q F( ey=3) Fy + q F( ez =3) Fz

The terms q F( ey=3) and q F( ez=3) can be evaluated, respectively, as follows:


F F
q F( ey= 3) ( s ) = qF(iy) + t d1 y s + t d 2 y s 2 = 0

q F( ez = 3) ( s ) = qF(iz) + t d1Fz s + t d 2Fz s 2 = qF( 2z ) − 6.73388 ×10 −5 s + 6.0245 ×10 −6 s 2


= −0.0210497 − 6.73388 ×10 − 5 s + 6.0245 ×10 − 6 s 2
at the end s = 30 , (node 4 )
q F( ey= 3) = 0 = qF( 4y)

q F( ez = 3) = −0.0210497 − 6.73388 × 10 −5 s + 6.0245 × 10−6 s 2 = −0.0176479 = qF( 4z )

Torsion Moment at O due to Flange 3, (see equations (4.130) and (4.131)):

(M ) F y ( e = 3)
O =
[m X 2(i ) − l X 3(i ) ]a
6
F F
(3atd1 y + 2a 2td 2 y + 6qF(iy) ) = 0

and

(M ) F z ( e = 3)
O =
[m X 2(i ) − l X 3(i ) ]a
6
(3atd1Fz + 2a 2td 2Fz + 6qF(iz) ) = −0.717541 − 900qF( 2z ) = 18.227233

Flange 4: (Flange data ( X 2(i ) , X 3(i ) , t , l , m ) in Table 4.2)


q F(iy) = qF(9y) , q F( jy) = q F(5y) = 0

F
d1 y =
X
[p
−1 (1)
0 ]
+ p2(1) X 2(i ) + p3(1) X 3(i ) = 0 ⇒ td 1
Fy
=0

d1Fz =
−1
X
[p ( 2)
0 + p2( 2) X 2(i ) + p3( 2 ) X 3(i ) ] = 2.94131×10 −4
⇒ td1Fz = 5.8826214 × 10 − 4

F
d2 y =
− 1 (1)
2X
[
p2 l + p3(1) m = 1.66912 × 10− 5 ⇒ ] F
td 2 y = 3.3382468 × 10 − 5

d 2Fz =
− 1 ( 2)
2X
[
p2 l + p3( 2 ) m = 0 ⇒ td 2Fz = 0 ]
Shear Flux in the Flange 4
F F
q ( s ) = (q F(iy) + t d1 y s + t d 2 y s 2 ) Fy + ( q F(iz) + t d1Fz s + t d 2Fz s 2 ) Fz = q F( ey=3) Fy + q F( ez =3) Fz

The terms q F( ey= 4) and q F( ez = 4) can be evaluated, respectively, as follows:


F F
q F( ey= 4 ) ( s ) = qF(iy) + t d1 y s + t d 2 y s 2 = qF(9y) + 3.3382468 × 10 −5 s 2

q F( ez = 4) ( s) = qF(iz) + t d1Fz s + t d 2Fz s 2 = q F(9z ) + 5.8826214 × 10 −4 s

at the end s = 15 , (node 5 )


q F( ey= 4 ) ( s = 15) = qF(9y) + 3.3382468 ×10 −5 s 2 = qF(9y) + 7.5110552 × 10 −3 = qF(5y) = 0
⇒ q F(9y) = −7.5110552 × 10− 3

University of Castilla-La Mancha Draft By: Eduardo W. V. Chaves (2016)


Ciudad Real - Spain
4 THE FUNDAMENTAL EQUATIONS OF CONTINUUM MECHANICS 419

q F( ez = 4) ( s = 15) = q F(9z ) + 5.8826214 × 10−4 s = qF(9z ) + 0.0088239 = qF(5z ) = 0


⇒ q F(9z ) = −0.0088239

Torsion Moment at O due to Flange 4, (see equations (4.130) and (4.131)):

(M ) Fy (e = 4)
O =
[m X 2(i ) − l X 3(i ) ]a
6
F F
(3atd1 y + 2a 2td 2 y + 6qF(iy) ) = 0

and

(M ) Fz ( e = 4 )
O =
[m X 2(i ) − l X 3(i ) ]a
6
(3atd1Fz + 2a 2td 2Fz + 6q F(iz) ) = 0

Flange 5: (Flange data ( X 2(i ) , X 3(i ) , t , l , m ) in Table 4.2)


q F(iy) = qF(10y ) , q F( jy) = q F( 6y) = 0

F
d1 y =
X
[p
−1 (1)
0 ]
+ p2(1) X 2(i ) + p3(1) X 3(i ) = 0 ⇒ td 1
Fy
=0

d1Fz =
−1
X
[p ( 2)
0 + p2( 2) X 2(i ) + p3( 2 ) X 3(i ) ] = 2.94131×10 −4
⇒ td1Fz = 5.8826214 ×10 − 4

F
d2 y =
− 1 (1)
2X
[
p2 l + p3(1) m = −1.66912 × 10 − 5 ]

F
td 2 y = −3.3382468 ×10 − 5

d 2Fz =
− 1 (2)
2X
[
p2 l + p3( 2 ) m = 0 ⇒ td 2Fz = 0 ]
Shear Flux in the Flange 5
F F
q ( s ) = (q F(iy) + t d1 y s + t d 2 y s 2 ) Fy + ( q F(iz) + t d1Fz s + t d 2Fz s 2 ) Fz = q F( ey=3) Fy + q F( ez =3) Fz

The terms q F( ey= 5) and q F( ez = 5) can be evaluated, respectively, as follows:


F F
q F( ey= 5) ( s ) = qF(iy) + t d1 y s + t d 2 y s 2 = qF(10y ) − 3.3382468 × 10 −5 s 2

q F( ez = 5) ( s ) = q F(iz) + t d1Fz s + t d 2Fz s 2 = qF(10z ) + 5.8826214 × 10 −4 s

at the end s = 15 , (node 6 )


q F( ey= 5) ( s = 15) = qF(10y ) − 3.3382468 × 10 −5 s 2 = qF(10y ) − 7.5110552 × 10−3 = qF( 6y) = 0
⇒ q F(10y ) = 7.5110552 × 10− 3

q F( ez = 5) ( s = 15) = qF(10z ) + 5.8826214 ×10 −4 s = qF(10z ) + 0.0088239 = qF( 6z ) = 0


⇒ q F(10z ) = −0.0088239

Torsion Moment at O due to Flange 5, (see equations (4.130) and (4.131)):

(M ) Fy ( e = 5)
O =
[m X 2(i ) − l X 3(i ) ]a
6
F F
(3atd1 y + 2a 2td 2 y + 6qF(iy) ) = 0

and

(M ) Fz ( e = 5 )
O =
[m X 2(i ) − l X 3(i ) ]a
6
(3atd1Fz + 2a 2td 2Fz + 6qF(iz) ) = 0

Checking compatibility at the node 4

University of Castilla-La Mancha Draft By: Eduardo W. V. Chaves (2016)


Ciudad Real - Spain
420 SOLVING PROBLEMS BY MEANS OF CONTINUUM MECHANICS

q ( 4 ) = q (9) + q (10)
4
q F( 4 ) = −7.5110552 ×10 − 3 + 7.5110552 ×10 − 3 = 0
 y
9 10  ( 4)
q Fz = −0.008823 − 0.0088239 = −0.017646

The Total Torsion Moment at O

(M ) F y ( Sys )
O ( )
= MOy
F ( e =1)
( )
+ MOy
F (e = 2)
( )
+ MOy
F ( e = 3)
( )
+ MOy
F ( e = 4)
( )F
+ M Oy
( e = 5)

= (0) + (−25.49336686) + (0) + (0) + (0) = −25.49336686

(M ) Fz ( Sys )
O (
= M OFz )
( e =1)
(
+ M OFz )
(e = 2)
(
+ M OFz )
( e = 3)
(
+ M OFz )
(e = 4)
(
+ M OFz )
( e = 5)

= (0) + (11.772767) + (18.227233) + (0) + (0) = 30


Then
F
M O = M O y Fy + M OFz Fz = −(25.49336686) Fy + (30) Fz = −( X 3( S .C .) ) Fy + ( X 2( S .C .) ) Fz

The Shear Center


If we compare the above equation with the equation in Figure 4.10 we can conclude that
X 2( S .C .) = 30cm ; X 3( S .C .) = 25.49336686

Note that the cross section has one axis of symmetry at X 2( A.C .) = X 2( S .C .) = 30cm .

References related to Mechanics of Materials


TIMOSHENKO, S. (1940). Strength of Materials Part I and II (second edition). D. Van Nostrand
Company, Inc. New York, USA, (1st Edition: 1930).
SOKOLNIKOFF, I.S. (1956). Mathematic theory of elasticity. New York, McGraw-Hill, (1st
Edition: 1946).
SECHLER, E. (1952). Elasticity in Engineering. John Willey & Sons, Inc. New York.
POPOV, E. P. (1999). Engineering Mechanics of Solids. Prentice Hall, (1st Edition: 1968).
UGURAL, A.C. & FENSTER, S.K. (1984). Advanced strength and applied elasticity. Edward
Arnolds (Publishers), London, England, (1st Edition: 1981).
BUCHANAN, G.R. (1988). Mechanics of Materials. Holt, Rinehart and Winston, Inc., New
York, USA.
CERVERA RUIZ, M. & BLANCO DÍAZ, E. (2001). Mecánica de Estructuras Libro 1 – Resistencia de
materiales. Edicions UPC, Barcelona. Eapaña.

University of Castilla-La Mancha Draft By: Eduardo W. V. Chaves (2016)


Ciudad Real - Spain
4 THE FUNDAMENTAL EQUATIONS OF CONTINUUM MECHANICS 421

4.3 Introduction to Flux Problems

Problem 4.34
1) Consider a continuum motion in which the stress power is equal to zero. Also, consider
r
that the heat flux is given by q = −K (T ) ⋅ ∇ xr T , which is known as Fourier’s law of thermal
conduction, where K (T ) is a second-order tensor called the thermal conductivity tensor (the
∂u (T )
thermal property of the material), and c = , where c is the specific heat capacity at a
∂T
constant deformation (the thermal property of the material) and is expressed in units of
J
joule per kelvin, i.e. [c] = . Taking into account all previous considerations, find the
K
energy equation for this process. Then also provide the unit of K (T ) in the International
System of Units (SI).
2) Consider the stress power is equal to zero, and that there is a continuous medium with
no internal heat source. Also consider that there is a heterogeneous material where
r
K = K ( x ) is an arbitrary second-order tensor (not necessarily symmetrical). a) Show that
the thermal conductivity tensor is semi-definite positive, b) Check in which scenario the
r
skew part of K ( x ) does not affect the outcome of the heat conduction problem. c) Taking
into account that the material is isotropic, in what format is K ?
Solution: For this problem we know that the stress power is equal to zero, σ : D = 0 . It then
follows that, the energy equation becomes:
∂u ∂T r r
ρ u& = ρ =σ
12 D − ∇ xr ⋅ q + ρr = −∇ xr ⋅ q + ρr
:3
∂T ∂t =0

∂T r ∂T
⇒ ρc = −∇ xr ⋅ q + ρ r ⇒ ρc = −∇ xr ⋅ [− K (T ) ⋅ ∇ xr T ] + ρ r
∂t ∂t
or
∂T
∇ xr ⋅ [K (T ) ⋅ ∇ xr T ] + ρ r = ρ c
∂t
The above equation is called the heat flux equation which is applied to the thermal
conduction problem.
DT ∂T
Obs.: If there is no mass transport it fulfills T& ≡ = .
Dt ∂t
∂T  K
Then if we take into account the following units: [q] =
r J W 
= 2 , ∇ xr T ≡ r  = , we
2
m s m  ∂x  m
can ensure that the units are consistent if the following is met:
[qr ] = [K ] ⋅ [∇ xr T ]
 J W   J W  K 
 m 2 s = m 2  =  s m K = m K  m 
    
 J W 
thus, we can draw the conclusion that [K ] =  = .
s m K m K 

University of Castilla-La Mancha Draft By: Eduardo W. V. Chaves (2016)


Ciudad Real - Spain
422 SOLVING PROBLEMS BY MEANS OF CONTINUUM MECHANICS

NOTE: As we will see later, when the stress power is equal to zero, we can decouple the
thermal and mechanical problem. That is, we can study these problems separately. ■
2) a) We start from the heat conductivity inequality:
r r
− q ⋅ ∇ xr T = −(−K ( x ) ⋅ ∇ xr T ) ⋅ ∇ xr T ≥ 0 − q i T,i = −(− K ij T, j )T,i ≥ 0
r or
∇ xr T ⋅ K ( x ) ⋅ ∇ xr T ≥ 0 T,i K ij T, j ≥ 0
r r
Remember that the arbitrary tensor A is semi-definite positive if it holds that x ⋅ A ⋅ x ≥ 0
r r r
for all x ≠ 0 thereby demonstrating that K ( x ) is a semi-definite positive tensor. Then, as a
r
result the eigenvalues of K ( x ) are all real values greater than or equal to zero, i.e. K 1 ≥ 0 ,
r
K 2 ≥ 0 , K 3 ≥ 0 . Also remember that since K ( x ) is not symmetric, the principal space of
r
K ( x ) does not define an orthonormal basis. Moreover, it is noteworthy that: the
r
antisymmetric part of K ( x ) does not affect the heat conduction inequality since:
r
∇ xrT ⋅ K ( x ) ⋅ ∇ xrT = ∇ xrT ⋅ [K sym + K skew ] ⋅ ∇ xrT = ∇ xrT ⋅ K sym ⋅ ∇ xrT + ∇ xrT ⋅ K skew ⋅ ∇ xrT ≥ 0
∇ xrT ⋅ K sym ⋅ ∇ xrT + K skew : (∇ xrT ⊗ ∇ xrT ) ≥ 0

Notice that K skew : (∇ xr T ⊗ ∇ xr T ) = 0 , since the double scalar product between an


antisymmetric tensor ( K skew ) and a symmetric one (∇ xr T ⊗ ∇ xr T ) is equal to zero, then:
r
0 ≤ ∇ xr T ⋅ K ( x ) ⋅ ∇ xr T = ∇ xr T ⋅ K sym ⋅ ∇ xr T ≥ 0
r
That is, the above inequality is always true whether K ( x ) is symmetric or not.
b) For the proposed problem the only remaining governing equation is the energy
Du r r
equation: ρ ≡ ρu& = σ : D − ∇ xr ⋅ q + ρr = −∇ xr ⋅ q , where u is the specific internal
Dt
energy, σ : D is the stress power, and ρ r is the internal heat source per unit volume. Then:
ρu& = −qi ,i = −( −K ijT, j ),i = K ij ,iT, j + K ijT, ji = (∇ xr ⋅ K T ) ⋅ (∇ xrT ) + K : ∇ xr (∇ xrT )
= (∇ xr ⋅ K T ) ⋅ (∇ xrT ) + [K sym + K skew ] : ∇ xr (∇ xrT )
= (∇ xr ⋅ K T ) ⋅ (∇ xrT ) + K sym : ∇ xr (∇ xrT ) + K skew : ∇ xr (∇ xrT )
= (∇ xr ⋅ K T ) ⋅ (∇ xrT ) + K sym : ∇ xr (∇ xrT )
where we have considered the symmetry of [∇ xr (∇ xr T )]ij = T,ij = T, ji . If the material is
r
homogeneous the implication is that the K field does not depend on ( x ) , so K ij ,i = 0 j . In
this scenario the heat equation reduces to:
ρu& = K sym : ∇ xr (∇ xr T )
Therefore, when the material is homogeneous, the antisymmetric part of K does not affect
the outcome.
c) The feature of isotropic materials is that their properties (at one material point) do not
change if the coordinate system is changed. It follows then that K must be an isotropic
tensor. An isotropic second-order tensor has the format of a spherical tensor, (see Chapter
1), then the tensor K must be of the type: K = K1 , where K is a scalar:
1 0 0
K ij = K 0 1 0
0 0 1

University of Castilla-La Mancha Draft By: Eduardo W. V. Chaves (2016)


Ciudad Real - Spain
4 THE FUNDAMENTAL EQUATIONS OF CONTINUUM MECHANICS 423

Problem 4.35
Consider a thermal conduction problem, (see Problem 4.34), in a wall with thickness equal
to h in which the temperature at the outer face ( x1 = 0 ) is equal to 38º C and the
temperature in the interior face ( x1 = h ) is equal to 21º C , (see Figure 4.53). Obtain the
heat flow for case defined by: stationary problem, the temperature field according to x 2
and x3 -directions is homogeneous, there is no heat source, and the material is isotropic
and homogeneous.

x2
h
T ( A) = 38º C Data:
h = 0.04m
T ( B ) = 21º C W
K = 0.19
mK
(Interior)
(Exterior)
x1
r
q

Figure 4.53
Solution:
As we saw in Problem 4.34 the governing equation for this problem is the equation
∂T ∂T
∇ xr ⋅ [K ⋅ ∇ xr T ] + ρ r = ρ c
. If we consider the stationary problem we have = 0 . If
∂t ∂t
there is no heat source this implies that r = 0 . With these simplifications the governing
equation becomes ∇ xr ⋅ [K ⋅ ∇ xr T ] = 0 , in addition, if the material is homogenous, the tensor
r
with the thermal properties K do not vary with x , then ∇ xr ⋅ [K ⋅ ∇ xr T ] = K : ∇ xr [∇ xr T ] = 0 ,
which in indicial notation is [K ijT , j ],i = K ij ,iT , j + K ij T , ji = K ijT , ji = 0 . By expanding this
123
=0
equation we obtain:
∂ 2T ∂ 2T ∂ 2T ∂ 2T ∂ 2T ∂ 2T
K 11 + K 12 + K 13 + K 21 + K 22 + K 23 +
∂x12 ∂x 2 ∂x1 ∂x3 ∂x1 ∂x1∂x 2 ∂x 22 ∂x3 ∂x 2
(4.135)
∂ 2T ∂ 2T ∂ 2T
+ K 31 + K 32 + K 33 2 = 0
∂x1∂x3 ∂x 2 ∂x3 ∂x3

If the temperature field according to x 2 and x3 -directions is homogenous, this implies that
the temperature gradient components according to these directions are equal to zero, i.e.
∂T ∂T
= = 0 . For an isotropic material, the thermal conductivity tensor components, (see
∂x 2 ∂x3
Chapter 5 of the textbook), are given by:

University of Castilla-La Mancha Draft By: Eduardo W. V. Chaves (2016)


Ciudad Real - Spain
424 SOLVING PROBLEMS BY MEANS OF CONTINUUM MECHANICS

K 0 0 
K ij =  0 K 0 
 0 0 K 

With these considerations the equation (4.135) becomes:


∂ 2T 11 =K
∂ 2T
K 11 =0 K → ⇒K =0 (4.136)
∂x12 ∂x12

∂ 2T
By integrating the equation K = 0 we can obtain:
∂x12

∂ 2T integrating ∂T dT
K =0   → K + q1 = 0 ⇒ q1 = −K
∂x12 ∂x1 dx1

which is the Fourier’s law of thermal conduction. Note that for this case q1 is a constant, i.e. it is
independent of x1 . By integrating once more we can obtain:
− q1 − q1
∫ dT = ∫ K
dx1 ⇒ T ( x1 ) =
K
x1 + C

Applying the boundary condition, x1 = 0 ⇒ T = T ( A) , we can obtain the constant of


− q1
integration C = T ( A) , and in turn the equation T ( x1 ) = x1 + T ( A) . In addition, for x1 = h
K
we have
− q1 (T ( B ) − T ( A) )
T ( x1 = h) = T ( B ) = h + T ( A) ⇒ q1 = −K
K h
In this case (one-dimensional case), the temperature gradient can be represented by the
slope of the line defined by the temperature, which varies linearly in the wall, (see Figure
4.53).
By replacing the problem data, (see Figure 4.53), we can obtain the heat flux:
(T ( B ) − T ( A) )  W  (21 − 38)( K ) W J
q1 = −K = −0.19  = 80.75 2 = 80.75 2
h  mK  0.04(m) m m s
Note that the temperature conversion from degrees Celsius to Kelvin is given by
K = º C + 273.15 , then the temperature variation ( ∆T ) either in degrees Celsius or in Kelvin
is the same. Note also that the heat flux flows from the higher temperature to the lower
temperature region.

University of Castilla-La Mancha Draft By: Eduardo W. V. Chaves (2016)


Ciudad Real - Spain
4 THE FUNDAMENTAL EQUATIONS OF CONTINUUM MECHANICS 425

NOTE: Let us suppose now that we have two walls with different properties, (see Figure
4.54).

T ( A)

T (B )

2
T (C )
1 r
q

K (1) K ( 2)

x1
(1) (2)
h h

Figure 4.54
(T ( B ) − T ( A) )
Note that the equation q1 = −K (1) is still valid. This also applies to the material
h (1)
(T (C ) − T ( B ) )
2 : q1 = −K ( 2) . To obtain the heat flux we must apply the compatibility in
h (2)
temperature on the face B , i.e.:
(T ( B _ 1) − T ( A) ) q1 h (1)
q1 = −K (1) ⇒ T ( B _ 1) = T ( A) −
h (1) K (1)
(T (C ) − T ( B _ 2 ) ) q1 h ( 2)
q1 = −K ( 2) ⇒ T ( B _ 2) = T (C ) +
h ( 2) K ( 2)
T ( B _ 1) = T ( B _ 2 )
q1 h (1) q1 h ( 2 )
T ( A) − = T (C ) +
K (1) K ( 2)
thus:
− (T (C ) − T ( A) )
q1 =
 h (1) h ( 2 ) 
 
 K (1) + K ( 2 ) 
 

Problem 4.36
Next, we assume that at a material point there are two types of material that are
represented by a physical quantity per unit volume in such a way that c = c f + c s , and the
r r r
following holds v = v f + v s , (see Figure 4.55). Considering an isothermal process, an
incompressible medium, and that the property c s does not affect the velocity of the
material f and that the c f -field is homogeneous, and there is no source of the material
f . Show that:

r ∂c s Convection-diffusion
Q s − ∇ xr ⋅ (v f c s ) + ∇ xr ⋅ (D ⋅ ∇ xr c s ) = (4.137)
∂t equation

University of Castilla-La Mancha Draft By: Eduardo W. V. Chaves (2016)


Ciudad Real - Spain
426 SOLVING PROBLEMS BY MEANS OF CONTINUUM MECHANICS

r
where the flux of the property s is given by q ( D ) = −D ⋅ ∇ xr c s .
r
vf
Control volume c f

cs
r
v r
dV vs

Figure 4.55: Heterogeneous medium.

Solution:
Starting from the continuity equation for this physical quantity, we can obtain:

Q=
∂Φ
∂t
+ ∇ xr
r
⋅ (Φv ) ⇒ Q=
∂ (c f + c s ) ∂
∂t
[ r r
+ r (c f + c s )(v f + v s )
∂x
] (4.138)

with Q = Q s + Q f . Thus:

Qs + Q f =
∂ (c f + c s ) ∂
∂t
[ r
+ r (c f + c s )(v f + v s )
∂x
r
]
[ ]
f s
∂ (c + c ) ∂ f r f r r r
⇒ Qs + Q f = + r c v + c f v s + csv f + csv s
∂t ∂x
(4.139)
[ ]
f s
∂c ∂c r r r r
⇒ Qs + Q f = + + ∇ xr ⋅ c f v f + c f v s + c s v f + c s v s
∂t ∂t
 ∂c r  ∂c s
[ ]
f
r r r
⇒ Qs + Q f =  + ∇ xr ⋅ (c f v f ) + + ∇ xr ⋅ (c s v f ) + ∇ xr ⋅ c f v s + c s v s
 ∂t  ∂t

∂c f r
If we are assuming that there is no ( f )-material source, then + ∇ xr ⋅ (c f v f ) = 0 and
∂t
f
Q = 0 hold, which is the continuity equation of the physical quantity c f with which the
equation in (4.139) becomes:

Qs =
∂c s
∂t
r
[ r r
+ ∇ xr ⋅ (c s v f ) + ∇ xr ⋅ c f v s + c s v s ] (4.140)

∂c s r r r
⇒ Qs = + ∇ xr ⋅ (c s v f ) + ∇ xr ⋅ (c s v s ) + ∇ xr ⋅ (c f v s ) (4.141)
∂t
∂c s r r r r
⇒ Qs = + ∇ xr ⋅ (c s v f ) + ∇ xr ⋅ (c s v s ) + (∇ xr c f ) ⋅ v s + c f (∇ xr ⋅ v s ) (4.142)
∂t
r
If the physical quantity c f does not change with x , then the gradient of c f becomes
r
∇ xr c f = 0 . In addition if we consider the medium ( s ) to be incompressible we can
r
consider ∇ xr ⋅ v s = 0 . These simplifications indicate that the material ( s ) does not affect
the velocity field of the material ( f ). So, if the amount of the material ( s ) is significant,
this approach is no longer valid. Then, with these approximations we can obtain:

University of Castilla-La Mancha Draft By: Eduardo W. V. Chaves (2016)


Ciudad Real - Spain
4 THE FUNDAMENTAL EQUATIONS OF CONTINUUM MECHANICS 427

∂c s r r ∂c s r r
Qs = + ∇ xr ⋅ (c s v f ) + ∇ xr ⋅ (c s v s ) = + ∇ xr ⋅ (c s v f ) + ∇ xr ⋅ q ( D ) (4.143)
∂t ∂t
r r
Notice that the term (c s v s ) ≡ q ( D ) represents the flux caused by the ( s )-material
r r
concentration, the diffusive term. The term (c s v f ) ≡ q (C ) is related to mass transport, the
r
convective term. Considering that q ( D ) = −D ⋅ ∇ xr c s the equation (4.143) becomes:
∂c s r r
Qs = + ∇ xr ⋅ (c s v f ) + ∇ xr ⋅ q ( D )
∂t
∂c s r
⇒ Qs = + ∇ xr ⋅ (c s v f ) + ∇ xr ⋅ (−D ⋅ ∇ xr c s ) (4.144)
∂t
r ∂c s
⇒ Q s − ∇ xr ⋅ (c s v f ) + ∇ xr ⋅ (D ⋅ ∇ xr c s ) =
∂t
with that we have shown the equation in (4.137).

Problem 4.37

Consider a water reservoir with


sediment concentration, (see
Figure 4.56). The sediment
concentration (concentration
density) is given by
( − kx3t ) h
c( x3 , t ) = C t exp , per unit
volume, where C and k are
positive constants. a) Obtain the
total mass of sediment in the
reservoir; b) Obtain the sediment
flux knowing that the flux is only
a function of x3 and time t , i.e. x3 x2
r r b
q = q( x3 , t ) .

x1
a

Figure 4.56: Reservoir with sediments.

Solution:
To obtain the total mass we have to solve the integral:
h b a h
( − kx3t ) ( − kx3t )

M = c s dV =
V
∫∫∫
0 0 0
C t exp ∫
dx1 dx 2 dx3 = ab C t exp
0
dx3

h
− C
= ab 
 k
exp
( − kx3t ) 


− C
= ab 
 k
C  − abC
exp ( − kht ) +  =
k  k
exp ( − kht ) − 1[ ]
0

To obtain the flux, we can apply the continuity equation of the concentration:

University of Castilla-La Mancha Draft By: Eduardo W. V. Chaves (2016)


Ciudad Real - Spain
428 SOLVING PROBLEMS BY MEANS OF CONTINUUM MECHANICS

∂c s r r ∂c s
Q= + ∇ xr ⋅ q ⇒ ∇ xr ⋅ q = q i ,i = − (4.145)
∂t ∂t
where we have considered that there is no source of the sediment, i.e. Q = 0 . For this
problem, the flux is not dependent on x 2 and x1 . With this condition we have
q1,1 = q 2, 2 = 0 . Then:

∂q1 ∂q2 ∂q3 ∂c s ∂q3 ∂c s


qi ,i = q1,1 + q2, 2 + q3,3 = + + =− ⇒ =− (4.146)
∂x1 ∂x2 ∂x3 ∂t ∂x3 ∂t

∂c s ∂ ( − kx t ) ( − kx t ) ( − kx t )
where = [C t exp 3 ] = C exp 3 − C t k x3 exp 3 and by substituting into the
∂t ∂t
equation (4.146) we can obtain:
dq3 ∂c s ( − kx t ) ( − kx t )
=− = −C exp 3 + C t k x3 exp 3
dx3 ∂t
( − kx3t ) ( − kx3t )
∫ ∫
⇒ dq3 = [−C exp + C t k x3 exp ]dx3
(4.147)
C ( − kx t ) C ( − kx t ) C k x3 t ( − kx t )
⇒ q3 = exp 3 − exp 3 − exp 3 + K 3
kt kt kt
( − kx3t )
⇒ q3 = −C x3 exp + K3
{
=0

r ( − kx3t )
The flux vector in the Cartesian basis is given by q = −C x3 exp eˆ 3 .

University of Castilla-La Mancha Draft By: Eduardo W. V. Chaves (2016)


Ciudad Real - Spain
4 THE FUNDAMENTAL EQUATIONS OF CONTINUUM MECHANICS 429

4.4 Introduction to Rigid Body Motion

Problem 4.38
Starting from the Fundamental Equations of Continuum Mechanics, obtain the governing
equations for a rigid solid problem.
Solution:
The fundamental equations of Continuum Mechanics are:
The Fundamental Equations of Continuum Mechanics
(Current configuration)
The Mass Continuity Equation Dρ r
+ ρ (∇ xr ⋅ v ) = 0 (4.148)
(The principle of conservation of mass) Dt
The Equations of Motion r r
(The principle of conservation of linear ∇ xr ⋅ σ + ρ b = ρ v& (4.149)
momentum)
Cauchy Stress Tensor symmetry
(The principle of conservation of angular σ = σT (4.150)
momentum)
The Energy Equation r
ρ u& = σ : D − ∇ xr ⋅ q + ρr (4.151)
(The principle of conservation of energy)
The Entropy Inequality r 1 1 1 r
ρη& ( x, t ) + σ : D − ρ u& − 2 q ⋅ ∇ xr T ≥ 0 (4.152)
(The principle of irreversibility) T T T
For a rigid body motion there is no mass transportation, so, the principle of conservation
of mass plays no rule. The rigid body can be treated as the whole mass is concentrated at
one point, so, at the time of establishing the governing equations for rigid body we do not
use the local form of the equations (4.149)-(4.150). We will adopt the global formulation of
the Principles.
We can start from the definition of the principle of conservation of linear momentum
which states that:
r D r r&
∑ F = Dt ∫ ρ v dV = L
V
r r
Then we can use the equation of linear momentum L = m v , (see Problem 4.39), to
obtain:
r D r r& r& r
∑ F = Dt ∫ ρ v dV = L = m v = m a
V

Then we have:
r r
∑F = m a
Now let us consider the principle of conservation of angular momentum which states:
r D r r D r r&
∑M O =
Dt V ∫
( x ∧ ρ v )dV =
Dt
HO ≡ HO

By which we can obtain:

University of Castilla-La Mancha Draft By: Eduardo W. V. Chaves (2016)


Ciudad Real - Spain
430 SOLVING PROBLEMS BY MEANS OF CONTINUUM MECHANICS

r r& r r&
∑M O = HO or ∑M G = HG
r
where the equation of angular momentum H O was obtained in Problem 4.39. The set of
r r r r&
equations ∑ F = m a and ∑M G = H G inform us that the two systems, described in
Figure 4.57, are equivalent.

System I System II r&


r r HG
F(n ) F( 2 )

G G
= r
ma

r
F(1) G - center of mass

Figure 4.57

Example: Consider the beam with the load and boundary conditions as described in
Figure 4.58. Obtain the support reactions VA , VB and H A in order to achieve equilibrium.

P y P sin α P

α P cosα
= A
B
A B
HA x

L L VA L L VB
2 2 2 2

Figure 4.58: Isostatic beam.

Solution:
Although in the beam there is deformation (small deformation regime) and stress, for
purposes of support reaction calculation of an isostatic beam we can consider as a rigid
body case and the necessary equations, (see equations in (4.153)), are:
 ∑F =0
∑F
x
r r r   = H A + P cosα = 0 ⇒ H A = − P cosα
∑ ∑F
x
F =ma =0 ⇒ =0 ⇒ 
∑F
y
  = VA + VB + P sin α = 0 ⇒ V A = −VB − sin α
∑F
y
 z =0

University of Castilla-La Mancha Draft By: Eduardo W. V. Chaves (2016)


Ciudad Real - Spain
4 THE FUNDAMENTAL EQUATIONS OF CONTINUUM MECHANICS 431

r


∑M x =0
r r&  L − P sin α
∑ MA = HA = 0 ⇒ ∑M y =0 ⇒ 

∑M z = VB L + P sin α
2
= 0 ⇒ VB =
2

∑M
 z =0
− P sin α
with which we can obtain VA = −VB − P sin α = . Note that we have 3 equations and
2
3 unknowns (a statically determinate system or isostatic). If we have a system in which
there are more unknowns than equations (a statically indeterminate system or hyperstatic),
this procedure is no longer valid since the reactions will depend on the beam deformation
and this depends on the beam stiffness.

NOTE: If we are dealing with rigid body motion, the governing equations are:
r r r r& Governing equations for rigid body
∑F =ma and ∑M O = HO
motion
(4.153)

The set of equations in (4.153) governs several problems such as: machine components
which are in rotation, satellite motion, navigation, etc. In navigation, motion is governed by
a device called gyroscope, which are governed by the set of equations in (4.153).
The set of equations in (4.153), in general, are non-linear and the analytical solution is very
complex to be obtained. Next, we will try to express the set of equations (4.153) more
friendly.

University of Castilla-La Mancha Draft By: Eduardo W. V. Chaves (2016)


Ciudad Real - Spain
432 SOLVING PROBLEMS BY MEANS OF CONTINUUM MECHANICS

Problem 4.39
Find the linear and angular momentum for the solid described in Figure 4.59 and subjected
to rigid body motion.

r x3′
F(n ) r
F( 2 )
Rigid body Bt
G
x2′
r
r v
x3 x
x1′

O
r G - mass center
x2 F(1)
x1

Figure 4.59: Rigid body motion.


Solution:
According to Problem 2.55 in Chapter 2, we have obtained the velocity for rigid body
motion as:
r r r r r
v = c& + ω ∧ ( x − c )
r
where ω is the axial vector (angular velocity) associated with the antisymmetric tensor W
(the spin tensor).
Linear momentum:
r r r r r r

r
( r r
)
r r
L = ρ v dV = ρ c& + ω ∧ ( x − c ) dV = ρ c& dV + ρ ω ∧ x dV − ρ ω ∧ c dV
∫ ∫ ∫ ∫
V V V V V
r r r r r
= c& ρ dV + ω ∧ ρ x dV − ω ∧ c ρ dV
∫ ∫ ∫
V V V

r r
By definition ∫ ρ x dV = mx is the first moment of inertia, where m is the total mass, and
V
r
x k is the vector position of the center of mass G . The first moment of inertia is equal to
r r r
zero if the Cartesian system originates at the center of mass, so, ρ x ′ dV = mx ′ = 0 . ∫
V

r r r r r
L = m[c& + ω ∧ ( x − c )]
r (Linear momentum for rigid body motion) (4.154)
=mv

r r r r r
where v = c& + ω ∧ ( x − c ) is the velocity of the center of mass.
Angular momentum:
r r r

H O = ( x ∧ ρ v ) dV = [∫ xr ∧ ρ (cr& + ωr ∧ ( xr − cr ))] dV
V V

Thus

University of Castilla-La Mancha Draft By: Eduardo W. V. Chaves (2016)


Ciudad Real - Spain
4 THE FUNDAMENTAL EQUATIONS OF CONTINUUM MECHANICS 433

r r r r r r r r r
H O = ρ x ∧ c& dV + ρ x ∧ (ω ∧ x ) dV − ρ x ∧ (ω ∧ c ) dV
∫ ∫ ∫
V V V

  r   (4.155)
r r r r r r r
=  ρ x dV  ∧ c& + ρ x ∧ (ω ∧ x ) dV −  ρ x dV  ∧ (ω ∧ c )
∫ ∫ ∫
V  V V 
Next, we discuss the second integral of the previous equation.
r r r
It was shown in Chapter 1 that given three vectors a , b , c , the relationship
r r r r r r r r r r r
a ∧ (b ∧ c ) = (a ⋅ c )b − (a ⋅ b)c holds, thus when a=c it holds that
r r r r r r r r r r r r r r r r r r
a ∧ (b ∧ a) = (a ⋅ a)b − (a ⋅ b)a , so, ∫ ρ x ∧ (ω ∧ x) dV = ∫ ρ [( x ⋅ x)ω − ( x ⋅ ω) x ] dV ,
V V
with

which we can obtain:

∫ ρ [x
V
k ] ∫ [
x k ω i − x p ω p x i dV = ρ x k x k ω p δ pi − x p ω p x i dV = ρ x k x k δ pi − x p x i ω p dV
V
] ∫ [
V
]
= ∫ ρ [x k ]
x k δ pi − x p x i dV ω p = I O ip ω p
V

or in tensorial notation:
r r r  r r r r  r r
∫ ρ x ∧ (ω ∧ x ) dV =  ∫ ρ [( x ⋅ x ) 1 − ( x ⊗ x )] dV  ⋅ ω = I O ⋅ ω
V V 
r r r r
where I O = ∫ ρ [( x ⋅ x ) 1 − ( x ⊗ x )] dV is the inertia tensor with respect to the origin O . As
V
we can observe, I O is a second-order pseudo-tensor, since it depends on the reference
system, and the components IO ij = ∫ ρ [ xk xkδ ij − xi x j ] dV can be explicitly expressed as:
V

I O 11 = ρ [( x1 x1 + x 2 x 2 + x 3 x 3 )δ 11 − x1 x1 ] dV = ρ x 22 + x 32 dV

V V
∫ [ ]
∫ [
I O 22 = ρ x12 + x 32 dV
V
] ; ∫ [
I O 33 = ρ x12 + x 22 dV
V
]
I O 12 = ρ [( x1 x1 + x 2 x 2 + x 3 x 3 )δ 12 − x1 x 2 ] dV = − ρ [x1 x 2 ] dV = − I O 12
∫ ∫
V V

I O 13 = − ρ [x1 x 3 ] dV = − I O 13
∫ ; I O 23 = − ρ [x 2 x 3 ] dV = − I O 23

V V

where I O 11 , I O 22 , I O 33 , are moments of inertia of the body relative to the reference point O ,
and I O 12 , I O 13 , I O 23 , are the products of inertia of the body relative to the reference point
O . Note also that the SI-unit for the inertia tensor is:
 
[IO ] =  ∫ ρ [( xr ⋅ xr ) 1 − ( xr ⊗ xr )] dV  = [ ρ ][ xr ][ xr ][dV ] = kg3 m m m3 = kg m2
V  m

The inertia tensor components in matrix form are represented as follows

University of Castilla-La Mancha Draft By: Eduardo W. V. Chaves (2016)


Ciudad Real - Spain
434 SOLVING PROBLEMS BY MEANS OF CONTINUUM MECHANICS

 

 ρ [ x2 + x3 ] dV ∫
− ρ [ x1 x2 ] dV ∫
− ρ [ x1 x3 ] dV 
2 2

V V V   IO11 − IO12 − IO13 


 
IOij =  − ρ [ x1 x2 ] dV
∫ ∫ρ + [ x12 x32 ] dV − ρ [ x2 x3 ] dV  =  − IO12
∫ IO 22 − IO 23 
 V V V   − IO13 − IO 23 IO 33 
 V∫
 − ρ [ x1 x3 ] dV − ∫ ρ [ x x ] dV ∫
V
2 3
V
ρ [ x12 + x22 ] dV 

(4.156)
Returning to the equation in (4.155) we can state that:
r  r  r r r r  r  r r
H O =  ρ x dV  ∧ c& + ρ x ∧ (ω ∧ x ) dV −  ρ x dV  ∧ (ω ∧ c )
∫ ∫ ∫
V  V V 
r r& r r r r r r& r r r
= m x ∧ c + I O ⋅ ω − m x ∧ (ω ∧ c ) = m x ∧ [c − (ω ∧ c )] + I O ⋅ ω
r r r
Then by adding and subtracting the term m x ∧ (ω ∧ x ) in the above equation we can
obtain:
r r r r r r r r r r r r r r r
H O = m x ∧ (c& − ω ∧ c ) + I O ⋅ ω = m x ∧ [c& + ω ∧ ( x − c )] − m x ∧ (ω ∧ x ) + I O ⋅ ω
r r r r
[ r r r r r r
]
r r r r r
= m x ∧ v − m ( x ⋅ x ) 1 − ( x ⊗ x ) ⋅ ω + I O ⋅ ω = m x ∧ v + m ( x ⊗ x ) − ( x ⋅ x ) 1 + IO ⋅ ω
r r
{[ ] }
r
= m x ∧ v + I ⋅ω
r r r
= m x ∧ v + HG
(4.157)
r r r r
where I = I O + m[( x ⊗ x ) − ( x ⋅ x ) 1] is the inertia pseudo-tensor, which is related to the
reference system at the center of mass. By means of this equation we can calculate the
inertia tensor in any reference system if we know the inertia tensor at the center of mass:
IO ij = Iij − m[ xi x j − ( x12 + x22 + x32 )δ ij ] . Explicitly, these components can be expressed as
follows:
IO11 = I11 + m( x22 + x32 ) ; IO12 = I12 − m( x1 x2 )
IO 22 = I22 + m( x12 + x32 ) ; IO 23 = I23 − m( x2 x3 ) Steiner’s theorem (4.158)
IO 33 = I33 + m( x12 + x22 ) ; IO13 = I13 − m( x1 x3 )

Note that, the above equations represent the parallel axis theorem (Steiner’s theorem) from
Classical Mechanics, which in matrix notation is given by:
 I11 I12 I13   x 22 + x 32 − x1 x 2 − x1 x 3 
   
I O ij =  I12 I 22 I 23  + m  − x1 x 2 x12 + x 32 − x 2 x 3  Steiner’s theorem (4.159)
I I 23 I33   −x x − x 2 x3 x12 + x 22 
 13  1 3

NOTE 1: If we have two bodies B (1) and B ( 2) we can conclude that


r r r r ( 2) r
∫ (x ∧ ρ ∫ (x ∧ ρ
(1)
HO = v ) dV (1) + v ) dV ( 2 )
V (1 ) V (2)
r r r r r r
= m (1) x (1) ∧ v (1) + I (1) ⋅ ω (1) + m ( 2 ) x ( 2 ) ∧ v ( 2 ) + I ( 2 ) ⋅ ω ( 2 )
r r r r r r
= m (1) x (1) ∧ v (1) + m ( 2 ) x ( 2 ) ∧ v ( 2 ) + I (1) ⋅ ω (1) + I ( 2 ) ⋅ ω ( 2 )
r r r
= m ( sys ) x ( sys ) ∧ v ( sys ) + I ( sys ) ⋅ ω ( sys )

University of Castilla-La Mancha Draft By: Eduardo W. V. Chaves (2016)


Ciudad Real - Spain
4 THE FUNDAMENTAL EQUATIONS OF CONTINUUM MECHANICS 435

where • (1) and • ( 2 ) stand for properties of the bodies B (1) and B ( 2) respectively. If the
r r r r
two bodies are attached they have the same angular velocity ω (1) = ω ( 2 ) = ω ( sys ) = ω , so, we
can conclude that:
r r r r
x ( sys ) ∧ v ( sys ) + [ I (1) + I ( 2 ) ] ⋅ ω
HO = m ( sys )

r r r
and if the system Ox is at the center of mass of the system ( v (sys ) = 0 ) we can obtain:
r r r
H O = I ( sys ) ⋅ ω = [ I (1) + I ( 2 ) ] ⋅ ω

Problem 4.40
Consider a parallelepiped whose dimensions are a × b × c , (see Figure 4.60), in which the
r
mass density field, ρ ( x ) , is homogeneous. Obtain the inertia tensor with respect to system
in the center of gravity.
Solution:
We will use the equation in (4.156):
 

 ρ [ x2 + x3 ] dV ∫
− ρ [ x1 x2 ] dV
− ρ [ x1 x3 ] dV  ∫
2 2

V V V   IO11 − IO12 − IO13 


IOij =  − ρ [ x1 x2 ] dV
∫ ρ [ x1 + x3 ] dV − ρ [ x2 x3 ] dV  = − IO12 IO 22 − IO 23 

2 2  

 V V V   − IO13 − IO 23 IO 33 

 V

 − ρ [ x1 x3 ] dV
V

− ρ [ x2 x3 ] dV
V

ρ [ x12 + x22 ] dV 

r
Note that, for this problem, the mass density is independent of x (homogeneous material),
with which it fulfills that:


m = ρ dV = ρ dV = ρV = ρabc
V

V

Then, the moment of inertia I O11 becomes:


c b a
2 2 2
abc 2 m

I O11 = ρ [ x22 + x32 ] dV = ρ ∫ ∫ ∫[x + x32 ]dx1dx 2 dx3 = ρ
2
2 (b + c 2 ) = (b 2 + c 2 )
V −c −b − a
12 12
2 2 2

m 2 m
In the same fashion we can obtain I O 22 = ( a + c 2 ) and I O 33 = ( a 2 + b 2 ) .
12 12
We leave to the reader show that I O12 = I O 13 = I O 23 = 0 . Recall that the inertia tensor give
us information about how the mass is distributed according to the adopted system, and
note that the mass is equally distributed according to the plane x1 x2 , thus ∫ ρ [ x1 x2 ] dV = 0 .
V
Note also that the adopted axes are principal axes of inertia:
m 2 2 
12 (b + c ) 0 0 
 m 2 
I ′Oij = 0 (a + c 2 ) 0  [ kg m 2 ]
 12 
 m 2 2 
 0 0 (a + b )
12 

University of Castilla-La Mancha Draft By: Eduardo W. V. Chaves (2016)


Ciudad Real - Spain
436 SOLVING PROBLEMS BY MEANS OF CONTINUUM MECHANICS

The inertia tensor of mass density


x3′
 (b 2 + c 2 ) 0 0 
m 
I′Oij =  0 2 2
(a + c ) 0 
12  2 2 
a  0 0 ( a + b ) 

O
c x ′2

x1′

Figure 4.60: Parallelepiped.

Problem 4.41
Consider three thin rods of length a and mass m , (see Figure 4.61). Obtain the inertia
r
tensor of the compound related to the system Ox .

x2
r r x′2
x (1) = 0
rod 2
r a a
x ( 2 ) = eˆ 1 + eˆ 2
2 2
r ( 3) − a a
x = eˆ 1 + eˆ 3 a a
a
2 2 2 2
2

x1′

x′2 r
x ( 2) a
2

O
r x1
a x ( 3)
2 rod 1

a
x1′
2
x3

rod 3
x3′

Figure 4.61: System compounded by three rods.

University of Castilla-La Mancha Draft By: Eduardo W. V. Chaves (2016)


Ciudad Real - Spain
4 THE FUNDAMENTAL EQUATIONS OF CONTINUUM MECHANICS 437

Data: The inertia tensor for the thin rod, in which the all mass is distributed along its axis,
is given by Figure 4.62.

x′2
a a
2 2
0 0 0 
ma 2 
g I gij =  0 1 0
12
0 0 1 
x1′

x3′
Figure 4.62: Inertia tensor of the rod related to the principal system.

Solution:
To calculate the inertia tensor of the system we will use the equation
I O( sys
r
x
)
= I O(1x)r + I O( 2xr) + I O(3xr) where IO ij = Iij − m[ xi x j − ( x12 + x22 + x32 )δ ij ] (the Steiner’s theorem),
(see Problem 4.39 (NOTA 1)):
 I11 I12 I13   x 22 + x 32 − x1 x 2 − x1 x 3 
   
I O ij =  I12 I 22 I 23  + m  − x1 x 2 x12 + x 32 − x 2 x3  (4.160)
I I 23 I33   −x x − x 2 x3 x12 + x 22 
 13  1 3

Rod 1 - I O(1X)r

Mass center vector position: ( x1(1) = 0, x2(1) = 0, x3(1) = 0)


0 0 0 
ma 2 
(1)
(I Oxr)
ij =  0 1 0
12
0 0 1

Rod 2 - I O( 2Xr)
a a ( 2)
Mass center vector position: ( x1( 2 ) = , x 2( 2 ) = , x 3 = 0)
2 2
By applying the equation (4.160) we can obtain:
 I11 I12 I13   x 22 + x 32 − x1 x 2 − x1 x 3 
( 2)    
(I Oxr ) ij =  I12 I 22 I 23  + m  − x1 x 2 x12 + x 32 − x 2 x3 
I I 23 I 33   − x1 x 3 − x2 x3 x12 + x 22 
 13 
 a  2 2  a  a  
  + 0 −    0
2
   2  2  
1 0 0   4 − 3 0
ma 2    a  a  a
2
 ma 2  
=  0 0 0 + m  −      +0
2
0 = − 3 3 0
12   2  2  2
   12
0 0 1  0 0 7 
 a a 
2 2
 0 0   +  
  2   2  

University of Castilla-La Mancha Draft By: Eduardo W. V. Chaves (2016)


Ciudad Real - Spain
438 SOLVING PROBLEMS BY MEANS OF CONTINUUM MECHANICS

Rod 3 - I O(3X)r
− a ( 2) a
Mass center vector position: ( x1( 2 ) = , x 2 = 0, x 3( 2 ) = )
2 2
 I11 I12 I13   x 22 + x 32 − x1 x 2 − x1 x 3 
( 2)    
(I Oxr ) ij =  I12 I 22 I 23  + m  − x1 x 2 x12 + x 32 − x 2 x3 
I I 23 I 33   − x1 x 3 − x2 x3 x12 + x 22 
 13 
 2  a 2  − a  a  
(0 ) +   0 −   
  2  2  2  
1 0 0   4 0 3
 + m  ma 2 
2 2
ma 2  −a a 
= 0 1 0  0   +   0 = 0 7 0
12  
  2  2  12
0 0 0  3 0 3
  − a  a  −a
2
2

−    0   +0 
  2  2   2  
Then, we can calculate
(I O( sys
r ) (1)r ( 2r) ( 3r)
x ) ij = (I Ox ) ij + (I Ox ) ij + (I Ox ) ij

0 0 0   4 − 3 0  4 0 3  8 −3 3
ma 2   ma 2   ma 2   ma 2 
=  0 1 0 +  − 3 3 0 +  0 7 0 =  − 3 11 0 
12 12 12 12
0 0 1  0 0 7  3 0 3  3 0 11

Problem 4.42
Find the kinetic energy related to rigid body motion in terms of the inertia tensor, (see
Problem 4.39 and Problem 4.38).
r r r r r
Solution: The rigid body motion velocity can be expressed as v = c& + ω ∧ ( x − c ) . Then, the
kinetic energy becomes:

K (t ) =
1

2V
r r r& r
ρ (v ⋅ v )dV =
1 r r r r
∫ ρ  [ r r 
c + ω ∧ ( x − c ) ⋅ c& + ω ∧ ( x − c )  dV
2V 
][ ]
r r r r
Using the following vector sum x = x + x ′ , where x is the mass center vector position,
r
and x ′ is the particle vector position with respect to the system that has its origin in the
center of mass, the energy equation becomes:
1
K(t ) =
2V 
r r
∫ [
r r r r r r r
][ r
ρ  c& + ω ∧ (( x + x ′) − c ) ⋅ c& + ω ∧ (( x + x ′) − c)  dV

]
=
1
2V 
r r
∫ [(
r r r r
)
r r r r
] [(r r
ρ  c& + ω ∧ ( x − c ) + (ω ∧ x ′) ⋅ c& + ω ∧ ( x − c) + (ω ∧ x ′)  dV

) ]
r r r r r
Note that v = c& + ω ∧ ( x − c) is the center of mass velocity, thus:

K (t ) =
1
2V ∫
ρ { [vr + (ωr ∧ xr ′)]⋅ [vr + (ωr ∧ xr ′)] }dV
or:
1 r r 1 r r r 1 r r r 1 r r r r
K (t ) = ρv ⋅ v dV +
∫ ρv ⋅ (ω ∧ x ′) dV +
∫ ρ (ω ∧ x ′) ⋅ v dV + ∫ ρ (ω ∧ x ′) ⋅ (ω ∧ x ′) dV

2V 2V 2V 2V

University of Castilla-La Mancha Draft By: Eduardo W. V. Chaves (2016)


Ciudad Real - Spain
4 THE FUNDAMENTAL EQUATIONS OF CONTINUUM MECHANICS 439

Then by simplifying the above equation we can obtain:


1 r r r r r 1 r r r r
K (t ) = ρ v ⋅ v dV + ρ v ⋅ (ω ∧ x ′) dV + ρ (ω ∧ x ′) ⋅ (ω ∧ x ′) dV
∫ ∫ ∫
2V V
2V

Next, we will discuss separately the terms of the previous equation:


1 r r 1 r 2 1
1) ρ v ⋅ v dV = v
∫ ∫ ρ dV = 2 mv
2
2V 2 V

r r r r r r  r r r
2) ∫ ρ v ⋅ (ω ∧ x ′) dV = v ⋅ ω ∧ ∫ ρ x ′ dV  = v ⋅ (ω ∧ m {
x ′) = 0
r
V  V  =0
r
Note that, the system x ′ is located at the center of mass ( G ), hence the center of mass
r
vector position related to the system x ′ is zero.
r r r r
3) ρ [(ω ∧ x ′) ⋅ (ω ∧ x ′)] dV = ρ  ijk ω j xk′  ipq ω p xq′ dV = ρ (δ jpδ kq − δ jqδ kp )ω j x′k ω p x′q dV
∫ ∫ ∫
V V V


= ρ ω j (δ jpδ kq xk′ xq′ − δ jqδ kp xk′ x′q )ω p dV
V

 

= ρ ω j (δ jp xk′ x k′ − x′p x′j )ω p dV = ω j  ρ (δ
∫ jp x ′k x k′ − x′j x′p ) dV ω p
 
V V 
= ω j I jp ω p

or in tensorial notation as:


r r r r r  r r r r  r r r
∫ ρ [(ω ∧ x′) ⋅ (ω ∧ x′)] dV = ω ⋅ ∫ ρ [( x′ ⋅ x′) 1 − ( x′ ⊗ x′)] dV  ⋅ ω = ω ⋅ I ⋅ ω
V V 
where I is the inertia pseudo-tensor related to the system located at the center of mass,
(see Problem 4.39).
Then if we bear in mind all the above considerations, the kinetic energy equation for rigid
body motion becomes:
1 r r 1 r r r 1 r r r r
K (t ) = ∫ρ v ⋅ v dV + 2ρ v ⋅ (ω ∧ x ′) dV + ρ (ω ∧ x ′) ⋅ (ω ∧ x ′) dV
∫ ∫
2V 2 2V
1V44424443
=0

1 1r r Kinetic energy for rigid


K(t ) = mv 2 + ω ⋅ I ⋅ ω (4.161)
2 2 body motion

Additionally, if we take into account that:


 

 ρ [ x2′ + x3′ ] dV ∫
− ρ [ x1′ x2′ ] dV ∫
− ρ [ x1′ x3′ ] dV 
2 2

V V V   I11 − I12 − I13 


 
Iij =  − ρ [ x1′ x2′ ] dV
∫ ∫ ρ [ x′ +1
2
x3′2 ] dV − ρ [ x2′ x3′ ] dV  =  − I12
∫ I 22 − I 23 
 V V V   − I13 − I 23 I33 

 − ρ [ x1′ x3′ ] dV − ∫ ρ [ x′ x′ ] dV ∫
2 3 ρ [ x1 + x2 ] dV  
′ 2
′ 2

 V V V 

we can obtain an explicit equation for the kinetic energy as:

University of Castilla-La Mancha Draft By: Eduardo W. V. Chaves (2016)


Ciudad Real - Spain
440 SOLVING PROBLEMS BY MEANS OF CONTINUUM MECHANICS

 I11 − I12 − I13   ω1 


1 1 1 1  
K(t ) = mv 2 + ωk Ikj ω j = mv 2 + [ω1 ω2 ω3 ] − I12 I22 − I23  ω2 
2 2 2 2 − I
 13 − I23 I33  ω3 

=
1 2 1
2
[
mv + I11ω12 + I22ω22 + I33ω32 − 2 I12ω1ω2 − 2 I13ω1ω3 − 2 I23ω2ω3
2
]
1
K(t ) = mv 2 +
2
1
2
[
I11ω12 + I 22 ω22 + I 33 ω32 − 2 I12 ω1ω 2 − 2 I13 ω1ω3 − 2 I 23 ω 2 ω3 ] (4.162)

Problem 4.43
Consider the inertia pseudo-tensor, I O , with respect to the system x1 x 2 x3 , (see Figure
4.63). a) Make the physical interpretation of the inertia tensor. b) Given another
orthonormal system, represented by x1* x 2* x3* . Obtain the inertia tensor components in this
new system. c) Show that the inertia tensor is positive definite tensor. For a solid in
DI O &
motion, find in which situation the term ≡ I O is equal to zero.
Dt

x3
x3* x2*

x1*

O x2

x1

Figure 4.63
Solution:
The inertia pseudo-tensor depends on the adopted coordinate system, and by definition is
given by:
r r r r
IO = ρ [( x ⋅ x ) 1 − ( x ⊗ x )] dV
∫ ; ∫
IO ij = ρ [ xk xkδ ij − xi x j ] dV [ kg m 2 ]
V V

or in components
 

 ρ [ x2 + x3 ] dV ∫
− ρ [ x1 x2 ] dV ∫
− ρ [ x1 x3 ] dV 
2 2

V V V 
Iij =  − ρ [ x1 x2 ] dV
∫ ∫ ρ [ x + x ] dV
2
1
2
3 − ρ [ x2 x3 ] dV 

 V V V 
 V ∫
 − ρ [ x1 x3 ] dV − ∫ ρ [ x x ] dV ∫
V
2 3
V
ρ [ x12 + x22 ] dV 


a) The inertia tensor gives us the information as the body mass is distributed according to
the adopted system.

University of Castilla-La Mancha Draft By: Eduardo W. V. Chaves (2016)


Ciudad Real - Spain
4 THE FUNDAMENTAL EQUATIONS OF CONTINUUM MECHANICS 441

The term ∫ ρ [ x x ] dV
V
1 2 indicates how the mass is distributed along the plane x1 − x 2 . Then,
r
if the material is homogeneous, i.e. the mass density field is independent of x , and x1 − x 2
is a plane of symmetry, i.e. the mass is distributed equally with respect to plane x1 − x 2 , the
term ∫ ρ [ x x ] dV
V
1 2 is equal to zero. With this, we can conclude that: if the planes x1 − x 2 ,

x1 − x 3 , x 2 − x 3 , are planes of symmetry, the inertia matrix is a diagonal matrix.


Let us consider a student attached to a disc with outstretched arms, each hand holding a
r
weight, (see Figure 4.64(a) – initial system). The disk rotates with angular velocity ω (i ) and
r
the inertia tensor according to the system x is given by I O(i ) . If we consider a system
without energy dissipation, what will it happen when the student moves the arms inwardly
as shown in Figure 4.64(b) – final system? As we are dealing with a conservative system,
the angular momentum is conserved too, i.e.:
r r
H O( i ) = H O( f )
r r
I (Oi ) ⋅ ω ( i ) = I O( f ) ⋅ ω ( f )

Since for the final system the mass is more concentrated according to the rotation axis than
r r
to the initial system the inequality I O( f ) < I (Oi ) holds and as consequence ω ( f ) > ω (i ) .
Conservative system
x3 r r x3
a) Initial system I O(i ) ⋅ ω (i ) = I O( f ) ⋅ ω ( f ) b) Final system

r
ω (i )
r
I O( f ) ⇒ ω( f )
I O(i )

Figure 4.64

b) Let us assume that the given systems, (see Figure 4.63), are related by the transformation
law xi* = Aij x j , where Aij is the orthogonal matrix, then it follows that xi = A ji x *j . Thus,
it is possible to express I O ij as follows:

University of Castilla-La Mancha Draft By: Eduardo W. V. Chaves (2016)


Ciudad Real - Spain
442 SOLVING PROBLEMS BY MEANS OF CONTINUUM MECHANICS

∫ [
V
] ∫ [
I O ij = ρ x k x k δ ij − xi x j dV = ρ ( x k* x k* )Aip δ pq A jq − Aip x *p A jq x q* dV
V
]
 
∫ {[ ]} ∫ [
= Aip ρ ( x k* x k* )δ pq − x *p x q* A jq dV = Aip  ρ ( x k* x k* )δ pq − x *p x q* dV A jq ]
V V 
= Aip I *O ij A jq

Note that x k x k = A ks x s* A kt x t* = x *s x t* A ks A kt = x *s x t*δ st = x *s x s* = x t* x t* = x k* x k* .


Abusing a little bit of notation, we also use tensorial notation, but bear in mind that we are
working with tensor components, and we are not doing an orthogonal transformation.

∫ [ ]
r r r r r r r r
IO = ρ [( x ⋅ x ) 1 − ( x ⊗ x )] dV = ρ ( x * ⋅ x * )A T ⋅ 1 ⋅ A − (A T ⋅ x * ⊗ A T ⋅ x * ) dV

V V

∫ [ ]
r r r r
= ρ ( x * ⋅ x * )A T ⋅ 1 ⋅ A − (A T ⋅ x * ⊗ x * ⋅ A ) dV
V
r r
{[ r r
= A T ⋅ ρ ( x * ⋅ x * )1 − ( x * ⊗ x * )
∫ ]}⋅ A dV
V

 
∫ [ ]
r r r r
= A T ⋅  ρ ( x * ⋅ x * )1 − ( x * ⊗ x * ) dV  ⋅ A = A T ⋅ I*O ⋅ A
V 

I O = A T ⋅ I *O ⋅ A Inertia tensor components after a base


(4.163)
I O ij = A *
ip I O ij A jq change (rotation)

Then, it is also true I *O = A ⋅ I O ⋅ A T , which are the inertia tensor components in the
system x1* x 2* x3* . Note that the equation (4.163) is the same component transformation law
for a second-order tensor, where A is the transformation matrix from the x1 x 2 x3 -system
to x1* x 2* x3* -system.
c) For a positive definite tensor, by definition, its eigenvalues are greater than zero.
We will start from the kinetic energy obtained in Problem 4.42, i.e.:

K(t ) = mv 2 +
1
2
1
2
[
I11ω12 + I 22 ω22 + I 33 ω32 − 2 I12 ω1ω 2 − 2 I13 ω1ω3 − 2 I 23 ω 2 ω3 ]
The kinetic energy is a scalar and is always a positive number, and only in two situations the
kinetic energy is zero, namely: when there is no mass or when the body is at rest. We adopt
a system such that the origin is at the center of mass and the adopted axes are axes of
symmetry (inertia principal system) and that the body is rotating around the origin (center
of mass). In this situation the kinetic energy becomes:
 I1 0 0   ω1 
1
K(t ) = [ω1 ω2
2
1
[
ω3 ]  0 I2 0  ω2  = I1ω12 + I2ω22 + I3ω32 > 0

2
]
 0 0 I3   ω3 
1442443
Eigenvalues of the
Inertia tensor

University of Castilla-La Mancha Draft By: Eduardo W. V. Chaves (2016)


Ciudad Real - Spain
4 THE FUNDAMENTAL EQUATIONS OF CONTINUUM MECHANICS 443

1
In addition, if we have a motion such that ω 2 = ω 3 = 0 , we have K(t ) = I1ω12 , then, the
2
only way that the kinetic energy is always positive is when I1 > 0 holds. Similarly, we can
conclude that I 2 > 0 and I 3 > 0 . Hence, the inertia tensor is a positive definite tensor.
d) As the inertia pseudo-tensor is dependent on the adopted system, for the following
situations the inertia tensor related to a solid in motion does not change with time:
1) If the adopted system is attached to the solid.
2) If the solid is rotating along the axis of symmetry, for example, if a cylinder is rotating
along the prismatic axis, then during motion the mass distribution is not changing with
respect to the adopted system, (see Figure 4.65).

r
ω
reference system fixed in space

Figure 4.65

Problem 4.44
Consider a homogeneous cylinder of radius r and height h = 3r with total mass equal to
m , (see Figure 4.66). Find the inertia tensor for the cylinder related to the system Ox1′ x ′2 x3′ .
The system Ox1′ x 2′ x3′ is given by the rotation of the system Ox1′′x 2′′ x3′′ of 45º along the axis
x1′′ . The systems Gx1 x 2 x3 and Ox1′′x 2′′ x3′′ have the same orientation.
Hint: For the reference system Gx1 x 2 x3 we know the inertia tensor components and are
given by:
1 2 2 
12 m(3r + h ) 0 0 
 mr 2 
2 0 0
 1
I G ij = 0 m(3r 2 + h 2 ) 0 =  0 2 0
 12  2 
 1 2 0 0 1
0 0 mr
 2 

University of Castilla-La Mancha Draft By: Eduardo W. V. Chaves (2016)


Ciudad Real - Spain
444 SOLVING PROBLEMS BY MEANS OF CONTINUUM MECHANICS

x3

x3′′
x3′ G x2
x1 h = 3r

r x2′
rG

45º
O x2′′

x1′′, x1′
Figure 4.66
Solution:
We can obtain the inertia tensor related to the system Ox1′′x ′2′ x3′′ by means of the Steiner
theorem, (see equation (4.158) in Problem 4.39). After that, we can obtain the components
due to a rotation by means of the equation (4.163), (see Problem 4.43).
By means of the equations in (4.159):
 I11 I12 I13   x 22 + x32 − x1 x 2 − x1 x3 
   
I ′O′ ij =  I12 I 22 I 23  + m  − x1 x 2 x12 + x32 − x2 x3  (4.164)
I I 23 I33   −x x − x2 x3 x12 + x 22 
 13  1 3

where ( x1 , x 2 , x 3 ) are the coordinates of the center of mass with respect to the system
r 3
Ox1′′x ′2′ x3′′ , and by consider the vector rG = x1eˆ 1′′ + x2eˆ ′2′ + x3eˆ ′3′ = 0eˆ 1′′ + r eˆ ′2′ + r eˆ ′3′ , we can
2
obtain:
 2 3 2 
 r + ( 2 r )  0 0 
2 0 0    34 0 0 
mr 2   2 3 2  3   mr 2 
I′O′ ij = 0 2 0 + m  0 0 + ( 2 r )  ( r )( 2 r )   = 4  0 13 − 6

2      
0 0 1     0 − 6 6 


0
 3 
( r )( 2 r )  [02 + r 2  ] 
 
Considering the transformation matrix from the Ox1′′x ′2′ x3′′ -system to the Ox1′ x ′2 x3′ -system:
1 0 0 
A = 0 cos 45º sin 45º 

0 − sin 45º cos 45º 

and by applying the equation (4.163) we can obtain:


34 0 0 
mr 2 
I O′ ij = A I ′O′ A = Aip I ′O′ ij A jq
T
=  0 7 − 7 
8
 0 − 7 31 

University of Castilla-La Mancha Draft By: Eduardo W. V. Chaves (2016)


Ciudad Real - Spain
4 THE FUNDAMENTAL EQUATIONS OF CONTINUUM MECHANICS 445

Problem 4.45
r r r r r r r
Taking into account the angular momentum H O = m x ∧ v + I ⋅ ω = m x ∧ v + H G , (see
Problem 4.39),find the rate of change of the angular momentum in such a way that we do
not need to calculate at each instant of time the inertia tensor.
r
r r ϕ
ω - angular velocity of the body HG r
r r ω
ϕ - angular velocity of the system x *
x3′
x2*
r x3*
HO x1*
x3
G x 2′
r
x

O x2 x1′
G - center of mass

x1

Figure 4.67

Solution: Applying the material time derivative we can obtain:


r r r
D ( H O ) r&
Dt
≡ HO =
D
Dt
[
r r r
m x ∧ v + HG =
D
Dt
]
r r D r
m x∧v +
Dt
[
HG = m
Dx r
Dt
]
r Dv r&
∧v +m x∧
Dt
+ HG [ ]
r r r r r&
= m v12
∧3 v + m x ∧ a + HG
r
=0

Thus,
r
D ( H O ) r& r r r&
≡ HO = m x ∧ a + HG (4.165)
Dt
r r
where a is the acceleration of the center of mass. Next, we will discuss the term H& G . We
adopt the mobile system x1′ x ′2 x3′ but with fixed orientation in space which is parallel to the
r
fixed system x1 x 2 x3 , (see Figure 4.67). By expressing the components of I and ω in the
system x1′ x ′2 x3′ , we can obtain:
r
r r D ( H G′ ) r& & r r&
H G′ = I ′ ⋅ ω′
  → rate of change
≡ H G′ = I ′ ⋅ ω′ + I ′ ⋅ ω ′
Dt
r
Note that, as the solid is rotating with respect to the system x ′ the inertia tensor changes,
r
since the mass distribution is changing with respect to the system x ′ . Then, at each time
step we have to calculate the inertia tensor. This procedure is very laborious. To solve this
r
problem, we adopt a new system x * , which has origin at the center of mass, (see Figure
4.67). By means of the component transformation law, the following is true:

University of Castilla-La Mancha Draft By: Eduardo W. V. Chaves (2016)


Ciudad Real - Spain
446 SOLVING PROBLEMS BY MEANS OF CONTINUUM MECHANICS

r r r r
 H G* = A ⋅ H G′ ; H G′ = A T ⋅ H G*
 r r r r
(components) ω * = A ⋅ ω ′ ; ω ′ = A T ⋅ ω *
 *
I O = A ⋅ I O′ ⋅ A I O′ = A T ⋅ I O* ⋅ A
T
;
r r
where A is the transformation matrix from the x ′ -system to x * -system.
r r
The rate of change of H G′ = A T ⋅ H G* becomes:
D r
Dt
r&
H G′ ≡ H G′ =
D
Dt
[ r
] r r&
A T ⋅ H G* = A& T ⋅ H G* + A T ⋅ H G* (4.166)

By analogy with the rate of change of the orthogonal tensor, (see Chapter of the textbook),
we can conclude that Ω = A& ⋅ A T ⇒ A& T = A T ⋅ Ω T , where Ω T is the antisymmetric
r
tensor and represents the rate of change of rotation of the system x * with respect to the
r
system x ′ . Then, we can express (4.166) as follows:
r& r r& r r&
H G′ = A T ⋅ Ω T ⋅ H G* + A T ⋅ H G* = A T ⋅ Ω T ⋅ H G* + H G*  (components) (4.167)
 
r r r r
Resorting to the antisymmetric tensor property such that Ω T ⋅ H G* = ϕ ∧ H G* , where ϕ is
r r
the axial vector associated with the antisymmetric tensor Ω T , i.e. ϕ = ϕ (t ) is the angular
r
velocity of the rotating system x * . Proving that (4.167) can still be written as follows:
r& r r& r r r&
H G′ = A T ⋅ Ω T ⋅ H G* + H G*  = A T ⋅ ϕ * ∧ H G* + H G*  (components) (4.168)
   

where
r*
r&
H G* =
Dt
[
I ⋅ω =
Dt
]
D * r * DI * r *
⋅ ω + I * ⋅ Dω
Dt
DI *
The term is equal to zero when one of the two possibilities holds:
Dt
DI * r r r
1) = 0 if the system x * is attached to the solid. In this case, the equation ϕ = ω
Dt
holds, i.e. the mobile system velocity is equal to the angular velocity of the solid.
DI *
2) = 0 if the solid rotates around a prismatic axis, (see Figure 4.65 in Problem 4.43).
Dt
NOTE 1: The equation in (4.168) can be rewritten as follows:
r& r r r&
H G′ = A T ⋅ ϕ * ∧ H G* + H G* 
 
r& r* r& * (components) (4.169)
T r*   r * r * r& * 
⇒ A ⋅ H G = A ⋅ A ⋅ ϕ ∧ H G + H G  = ϕ ∧ H G + H G 

   
r r r
Note that the term A ⋅ H& G′ are the components of H& G′ in the system x * , and note also
r r
that A ⋅ H& G′ ≠ H& G* , then:
r * r r
A ⋅ H& ′  = H& * + ϕr * ∧ H * (components) (4.170)
 G  G G

we can also express the above equation in tensorial notation:

University of Castilla-La Mancha Draft By: Eduardo W. V. Chaves (2016)


Ciudad Real - Spain
4 THE FUNDAMENTAL EQUATIONS OF CONTINUUM MECHANICS 447

r r
 DH G   DH G  r r
  =  + ϕ ∧ HG (tensorial notation) (4.171)
 Dt   
  f  Dt  r
r
 DH G  r
where   represents the rate of change of H G with respect to the fixed system,

 Dt  f
r
 DH G  r
 
 Dt  represents the rate of change of H G with respect to the rotating system with an
 r
r
angular velocity ϕ .
NOTE 2: The equation in (4.171) is valid for any vector, (see Figure 4.68), i.e. the rate of
r r
change of the vector b respect to the fixed system x ′ is equal to the rate of change of the
r r
vector b respect to the rotating system x * plus the vector product between angular
r
velocity of the system ( ϕ which is associated to the antisimetric tensor Ω T ) and the vector
r
b:
r r r
 Db   Db  r  Db  r r
 
 Dt  = 


 + Ω T
⋅ b =  
 Dt  + ϕ ∧b (4.172)
  fixed  Dt  rotating   rotating
r r r
 Dϕ   Dϕ  r r  Dϕ 
Note also that   =  +ϕ1
∧ϕ =
2r3  Dt 
.
 Dt  f  Dt  r r
=0

r r
b ϕ
x3′
x2*
x3*
x1*

x 2′

x1′

Figure 4.68

NOTE 3: Note that the equation (4.172) is the convective rate, (see Chapter on The
C
r r r
Objectivity of Tensors in the textbook), which is defined by a = a& + l T
⋅ a , where
C
r r r r r
l = D + W , then a = a& + l T ⋅ a = a& + (D + W) T ⋅ a . Recall from Chapter 2 (Chaves (2013))
1
[ ]
that W = R ⋅ U& ⋅ U −1 − U −1 ⋅ U& ⋅ R T + R& ⋅ R T holds. And if we are considering rigid solid
2
Cr r r
motion we have D = 0 , U& = 0 , and W = Ω = R& ⋅ R T , with that we obtain a = a& + Ω T ⋅ a .

University of Castilla-La Mancha Draft By: Eduardo W. V. Chaves (2016)


Ciudad Real - Spain
448 SOLVING PROBLEMS BY MEANS OF CONTINUUM MECHANICS

NOTE 4: Let us expose a simple example to obtain Ω T . Let us assume that the êi -system
is rotating according to the êi -system, (see Figure 4.69), and to obtain Ω T we procedure
as follows. The transformation matrix from êi to êi is given by:

 cos θ sin θ 0
A =  − sin θ cos θ 0 (4.173)
 0 0 1 

 d (cosθ ) d (sin θ ) 
 0
 & θ& cosθ 0  − sin θ
  − θ sin θ cos θ
dt dt 0
d (A )  d (sin θ ) d (cosθ ) 
&
≡ A = − 0 = − θ& cos θ − θ& sin θ 0 = θ& − cos θ − sin θ 0
dt  dt dt  
 0 0 0  0 0 0  0 0 0

 
 

 − sin θ cosθ 0 cosθ − sin θ 0  0 1 0  0 θ& 0


 
Ω = A ⋅ A = θ − cosθ
& T & − sin θ 0  sin θ cosθ 0 = θ& − 1 0 0 = − θ& 0 0
 0 0 0  0 0 1  0 0 0  0 0 0
 
 0 − θ& 0  0 −ϕ 3 ϕ2  0
 
Ω T = θ& 0 0 =  ϕ 3 0 − ϕ 1  ⇒ ϕ i =  0 
0 0 0 − ϕ 2 ϕ3 0  θ& 
 

r
where ϕ is the axial vector associated with the antisymmetric tensor Ω T .

ϕ 3 = θ&

ê3
ê2 ê3
ê1
θ ê 2
ê1

Figure 4.69

University of Castilla-La Mancha Draft By: Eduardo W. V. Chaves (2016)


Ciudad Real - Spain
4 THE FUNDAMENTAL EQUATIONS OF CONTINUUM MECHANICS 449

NOTE 5: Inertial forces


Let us consider the system OX 1 X 2 X 3 , (see Figure 4.70), which is fixed in space. This
system is denoted by inertial reference frame. To this system the Newton’s law is applied, and if
there is a falling body it is true that:
r r
F = mA
Let us consider also that an observer (attached to the system ox1 x 2 x3 ) is moving (for
simplicity’s sake we will just consider translation). Since the system ox1 x 2 x3 is moving we
denote it by non-inertial reference frame. By means of vector summation, (see Figure 4.70), we
can obtain:
r r r
X =c+ x
The material time derivative of the above equation becomes:
D
r& r r &r& &r& &r& r &r& r
X = c& + x& →
Dt
X =c +x ⇒ A=c + &x&
and if we multiply by mass ( m ) we can obtain:
r &r& + m&xr& r r &r& = Fr − mc
&r& r r &r&
mA = mc ⇒ m&x& = mA − mc ⇒ ma = F − mc
r
Note that, for the observer it appears the additional force (−m&c&) to the “Newton’s law”.
This additional force is a fictitious force or pseudo force which is denoted by inertial force. In
addition, inertial forces appear if the observer’s system is rotating, e.g. centripetal force.

r r
mA x
X2 r
X x2

x1
r
c

O
X1

X3

Figure 4.70

University of Castilla-La Mancha Draft By: Eduardo W. V. Chaves (2016)


Ciudad Real - Spain
450 SOLVING PROBLEMS BY MEANS OF CONTINUUM MECHANICS

Problem 4.46
r
Show that the acceleration at a fixed system a f can be expressed as:
r r r r r r r
a f = ar + 2(ω ∧ v r ) + ω ∧ (ω ∧ x ) (4.174)
r r
where a r and v r are, respectively, the acceleration and the velocity of a particle with
r
respect to an observer that is rotating with the system x * , (see Figure 4.68). Consider also
r r r
that ϕ = ω is the angular velocity of the system x * , which is constant with time.
Solution:
We use directly the equation in (4.172) to obtain the velocity:
r r
 Dx   Dx  r r r r r r
  =   +ω∧ x ⇒ v f = vr + ω ∧ x
 Dt  f  Dt  r
We apply the same definition to the above equation in order to obtain the acceleration, i.e.:
r r r r r r r
 Dv f   D[v r + ω ∧ x ]   D[v r + ω ∧ x ] r r r r
  =  =  + ω ∧ [v r + ω ∧ x ]
 Dt  f  Dt f  Dt r
r r r
r  Dv   D[ω ∧ x ]  r r r r r
af =  r  +  + ω ∧ v r + ω ∧ (ω ∧ x )
 Dt  r  Dt  r
r r r
r  Dv   Dω  r r  Dx  r r r r r
af =  r  +  ∧ x + ω ∧   + ω ∧ v r + ω ∧ (ω ∧ x )
 Dt  r  Dt  r  Dt  r
r r r& r r r r r r r r
a f = a r + ω ∧ x + ω ∧ v r + ω ∧ v r + ω ∧ (ω ∧ x )
r r r& r r r r r r
a f = ar + ω ∧ x + 2(ω ∧ v r ) + ω ∧ (ω ∧ x )
r r
As we are assuming angular velocity constant ω& = 0 , i.e. the angular acceleration is zero,
with that we obtain the equation in (4.174). Then, we can conclude:
r r r& r r r r r r
a f = ar + ω ∧ x + 2(ω ∧ vr ) + ω ∧ (ω ∧ x ) (4.175)

Note that to obtain the above equation we have not used any principle of conservation.
The above equation is just relating the acceleration in a fixed system in function of
parameters defined in the rotating system.
r r r r r r r r r
NOTE 1: Using the identity a ∧ (b ∧ c ) = (a ⋅ c )b − (a ⋅ b )c , (see Problem 1.17), we can
r r r r r r r r r r r r r r r
conclude that ω ∧ (ω ∧ x ) = (ω ⋅ x )ω − (ω ⋅ ω ) x = (ω ⋅ x )ω − ω x . Note that, if ω = ω3ê 3 ,
2

(see Figure 4.71), and also if we adopt the system ( ê r , êθ , ê3 ) and taking into account that
r r r r r r r r r 2r r 2r
ω ⋅ r = 0 we can obtain the following equation ω ∧ (ω ∧ r ) = (ω ⋅ r )ω − ω r = − ω r ,
which is the centripetal acceleration, (see Problem 2.57). Earth rotates at a rate
rad 2π rad rad r r r
ω3 = 2π = ≈ 0.727 × 10− 4 . Note that the term ω ∧ (ω ∧ x ) is very small
day 86400 s s
r r
compared with the term 2(ω ∧ vr ) .

University of Castilla-La Mancha Draft By: Eduardo W. V. Chaves (2016)


Ciudad Real - Spain
4 THE FUNDAMENTAL EQUATIONS OF CONTINUUM MECHANICS 451

x3 , z
r
ω = ω3ê 3
ω3

ê3
r
x3 r êθ
r
x ê r
φ
x2 , y

x1 , x

Figure 4.71

r r
NOTE 2: The term 2(ω ∧ v r ) was established by Gustave-Gaspard Coriolis in 1835, and
r r
is associated with the fictitious force called Coriolis force. Next, we will represent 2(ω ∧ v r )
in the system eˆ ′i , (see Figure 4.71).

ω3

N
ê′2
ê′3
Latitude
r ê1′
x
eˆ 3 , eˆ ′2′
φ
eˆ 2 , eˆ 1′′
Equator

eˆ 1 , eˆ ′3′

Figure 4.72
The transformation law from ê i to eˆ ′i is given by:

 eˆ 1′   0 1 0   eˆ 1   0 1 0 
ˆ    
e′2  = − sin φ 0 cosφ  eˆ 2  ⇒ B = − sin φ 0 cosφ 

(4.176)
eˆ ′   cosφ 0 sin φ  eˆ   cosφ 0 sin φ 
 3   3 

University of Castilla-La Mancha Draft By: Eduardo W. V. Chaves (2016)


Ciudad Real - Spain
452 SOLVING PROBLEMS BY MEANS OF CONTINUUM MECHANICS

r
ω = ω3 cos(φ )eˆ ′2 + ω3 sin(φ )eˆ ′3
ω3 ω3ê3

N
ê′2
ê′3

S
Figure 4.73

r r
The term 2(ω ∧ v r ) can be obtained as follows:
eˆ 1′ eˆ ′2 eˆ ′3
r r
2(ω ∧ vr ) = 0 ω3 cos(φ ) ω3 sin(φ )
vr1 vr 2 vr 3 (4.177)
= 2eˆ 1′ [ω3 cos(φ )vr 3 − ω3 sin(φ )vr 2 ] − 2eˆ ′2 [− ω3 sin(φ )vr1 ] + 2eˆ ′3 [− ω3 cos(φ )vr1 ]
= 2[ω3 cos(φ )vr 3 − ω3 sin(φ )vr 2 ]eˆ 1′ + 2[ω3 sin(φ )vr1 ]eˆ ′2 − 2[ω3 cos(φ )vr1 ]eˆ ′3
The term f = 2ω3 sin(φ ) is known as Coriolis parameter. To small value of vr 3 the above
equation reduce to:
r
 Dv r  r r
  = −2(ω ∧ v r ) = [2ω3 sin(φ )vr 2 ]eˆ 1′ + [− 2ω3 sin(φ )vr1 ]eˆ ′2 = [ f vr 2 ]eˆ 1′ + [− f vr1 ]eˆ ′2
 Dt  r
 Dv r 1
 Dt = f vr 2
⇒
 Dv r 2 = − f v
 Dt r1

Figure 4.74: Coriolis effect (Ref.: Wikipedia “Coriolis effect”).

University of Castilla-La Mancha Draft By: Eduardo W. V. Chaves (2016)


Ciudad Real - Spain
4 THE FUNDAMENTAL EQUATIONS OF CONTINUUM MECHANICS 453

NOTE 3: Deflection of vertically falling body


A very simple application of the Coriolis effect is presented next. Let us consider an
observer on the surface of the Earth. Let us consider also that a body of mass m is free-
falling from rest with the following initial conditions: at t = 0 . ( x3′ = h) , ( x1′ = 0) ,
d
( x3′ = v3 = 0) , (v1 = 0) , (v2 = 0) . As the body is falling we will calculate the deflection of
dt
the body, i.e. we will obtain x1′ related to the observer which is attached to a system which
is rotating with the Earth. We will adopt the system used in Figure 4.72.
r r
The Newton’s Second Law ( F = ma f ) (apply to an inertial reference frame), then
r r r r r r r r r r r r r
F = m[ar + 2(ω ∧ v r ) + ω ∧ (ω ∧ x )] ⇒ ma r = F − 2m(ω ∧ v r ) = − mgeˆ ′3 − 2m(ω ∧ v r )

− 2[ω3 cos(φ )vr 3 − ω3 sin(φ )vr 2 ] − 2ω3 cos(φ )vr 3 


r r r r    
⇒ ar = − geˆ ′ − 2(ω ∧ vr ) ⇒ (a r ) i =  − 2[ω3 sin(φ )vr1 ] = 0 

 2 [ω3 cos(φ ) vr1 ] − g  
  − g 

r
where the acceleration a f is given by (4.175), and we are considering that the term
r r r r r
ω ∧ (ω ∧ x ) is very small when compared with the term 2(ω ∧ vr ) whose components are
given by (4.177). Then
 d 2 x1′ 
 2 
ar1   dt2  − 2ω3 cos(φ )vr 3 
r    d x′   
(ar )i = ar1  =  22  =  0  (4.178)
a   dt2   − g 
 r1   d x3′   
 dt 2
 
Note that
d 2 x3′ dx′
2
= − g integratin
 g → 3 = − gt + C1 ⇒ vr 3 = − gt
dt dt
dx′ t2 t2
 g → x3′ = − g + C2
⇒ 3 = − gt integratin ⇒ x3′ = − g + h
dt 2 2
where we have considered the initial conditions, i.e. at t = 0 ⇒ (vr 3 = 0) ⇒ C1 = 0 , and
2 2
t gt
t = 0 ⇒ ( x3′ = h) ⇒ C2 = h . Note that x3′ = − g +h=0⇒h= .
2 2
Considering the equation vr 3 = − gt into the first component of (4.178) we can obtain:
d 2 x'1 dx '1 t2
= − 2 ω3 cos(φ ) vr3 = 2 ω3 gt cos(φ ) integratin g
   → = 2 ω3 g cos(φ ) + C1 = vr1
dt 2 dt 2
where the constant of integration is obtained with the initial condition
at (t = 0) ⇒ {v′r1 = 0 ⇒ C1 = 0

dx '1 t3
= vr1 = ω3 g cos(φ )t 2 integratin
 g → x'1 = ω3 g cos(φ ) + C2
dt 3
1
Note also that C2 = 0 , so, the above equation becomes: x'1 = ω3 g cos(φ )t 3 .
3

University of Castilla-La Mancha Draft By: Eduardo W. V. Chaves (2016)


Ciudad Real - Spain
454 SOLVING PROBLEMS BY MEANS OF CONTINUUM MECHANICS

1 2 2h
As the body is falling from height h we can state that h = gt ⇒ t = , with that the
2 g
above equation becomes:
3
1 ω g  2h  2
x'1 = ω3 g cos(φ )t 3 = 3   cos(φ )
3 3  g 
NOTE 4: Acceleration due to sphericity
Local system ê1′ (east)- ê′2 (north)- ê′3 (radially upward)

N – North
N S – South
N
ê′2 E – East
r ê′3
r W – West
Latitude
E
W ê1′
r S
x r r
ê3 r = x cosφ
φ ê3
ê1 ê 2
θ
Equator
ê1

S Pole

ê3 r dθ r dθ
v r1 = r = x cosφ ( ê1′ )
vr 2 dt dt
N
NP
r ê′2
r ê′3
r
x
φ S ê2
NP
r v r1
r E
θ
ê1′
SP
W
r dφ
vr 2 = x ( ê′2 ) ê1
dt

Figure 4.75

University of Castilla-La Mancha Draft By: Eduardo W. V. Chaves (2016)


Ciudad Real - Spain
4 THE FUNDAMENTAL EQUATIONS OF CONTINUUM MECHANICS 455

Previously we have obtained the transformation matrix from êi to êi , (see equation
(4.173)):
 cos θ sin θ 0
A =  − sin θ cosθ 0 (4.179)
 0 0 1

and the transformation matrix from ê i to eˆ ′i , (see equation (4.176)), is given by:
 0 1 0 
B = − sin φ 0 cosφ 

(4.180)
 cosφ 0 sin φ 

Then the transformation matrix from êi to eˆ ′i is given by:


 0 1 0   cosθ sin θ 0  − sin θ cosθ 0 
C = BA = − sin φ 0 cosφ  − sin θ
 cosθ 0 =  − sin φ cosθ − sin φ sin θ cosφ 
 cosφ 0 sin φ   0 0 1  cosφ cosθ cosφ sin θ sin φ 

The rate of change of C is given by:


 − θ& cos θ − θ& sin θ 0 
d (C ) &  & 
≡ C = ( −φ cos φ cos θ + θ& sin φ sin θ ) ( −φ& cos φ sin θ − θ& sin φ cos θ ) − φ& sin φ 
dt ( −φ& sin φ cos θ − θ& cos φ sin θ ) ( −φ& sin φ sin θ + θ& cos φ cos θ ) φ& cos φ 
 
After the algebraic operation Ω = C&C T is taken place we can obtain:
 v r1 v 
 0 r sin φ − r r1 cos φ 
 0 θ& sin φ − θ& cos φ   x cos φ x cos φ 
   v vr 2 
Ω = C&C T = − θ& sin φ 0 − φ&  = − r r1 sin φ 0 − r 
 θ& cos φ φ& 0   x cos φ x 
   v r1 vr 2 
 xr cos φ cos φ r
x
0 
 
 0 vr1 tan φ − vr1 
1 
= r − vr1 tan φ 0 − vr 2 
x
 vr1 vr 2 0 

which is an antisymmetric matrix, as expected. Notice that according to Figure 4.75 the
dθ v dφ v r 2
following relationships θ& ≡ = r r1 and φ& ≡ = r hold.
dt x cosφ dt x
We apply the definition, (see equation (4.171)),
r r
 Dv 
 = 
Dv  r r

 Dt   + ϕ ∧ vr
  f  Dt  r
r r r
Note also that Ω T ⋅ v r = ϕ ∧ v r holds, so:

University of Castilla-La Mancha Draft By: Eduardo W. V. Chaves (2016)


Ciudad Real - Spain
456 SOLVING PROBLEMS BY MEANS OF CONTINUUM MECHANICS

 0 − vr1 tan φ v r1   v r1  − vr1vr 2 tan φ + vr1vr 3 


r 1     1  2 
Ω ⋅ v r = r vr1 tan φ
T
0 vr 2  vr 2  = r  vr1 tan φ + vr 2 vr 3 
x x 
 − vr1 − vr 2 0  vr 3   − vr21 − vr22 

(4.181)
− vr1vr 2 tan φ + vr1vr 3  − vr1vr 2 tan φ + vr1vr 3 
r r 1  2  r r 1  2 
⇒ a f = a r + r  vr1 tan φ + vr 2 vr 3  ⇒ a r = a f − r  vr1 tan φ + vr 2 vr 3 
x   x  
 − vr21 − vr22   − vr21 − vr22 

NOTE 5: Coriolis + Curvature acceleration


The acceleration related to the Coriolis terms, (see equations (4.177) and (4.175)), and
curvature is given by:
r r r r r r r r
a f = a r + 2(ω ∧ v r ) + Ω T ⋅ vr + ω ∧ (ω ∧ x ) (4.182)
where

2[ω3 vr 3 cos(φ ) − ω3 vr 2 sin(φ )] − vr1vr 2 tan(φ ) + vr1vr 3 


r r T r   1  2 
2(ω ∧ v r ) + Ω ⋅ v r =  2[ω3 vr1 sin(φ )]  + r  vr1 tan(φ ) + vr 2 vr 3  (4.183)
 − 2[ω3 vr1 cos(φ ) ]  x  − vr21 − vr22 
   

Problem 4.47
Consider the rigid body in motion in which there are no forces acting on the body and also
consider a torque-free motion. a) Show the Euler’s equations of motion:

I1ω & 1 = ω2 ω3 (I 2 − I3 )

I 2 ω
& 2 = ω1ω3 (I3 − I1 ) Euler’s equations of motion (4.184)
I ω
 3 & 3 = ω1ω2 (I1 − I 2 )

where Ii are the principal moment of inertia related to the system G xyz whose origin is at
r
the center of mass G , ωi are the components of the body angular velocity ( ω ), and
Dωi
ω
&i ≡ denotes the time derivative of the angular velocity.
Dt
b) Show that the kinetic energy is constant.
Solution:
The governing equations for a rigid body motion, (see Problem 4.38), are:
r r r r&
∑ F = ma and ∑ MG = HG

If the body is free of forces and torque we have that:


r r r r r&
∑F =0 and ∑M G = 0 = HG
r
Next we will evaluate the term H& G .
We will consider a mobile system G xyz attached to the body, (see Figure 4.76), so, in this
r r
situation we have that ϕ = ω .

University of Castilla-La Mancha Draft By: Eduardo W. V. Chaves (2016)


Ciudad Real - Spain
4 THE FUNDAMENTAL EQUATIONS OF CONTINUUM MECHANICS 457

r r
ω - angular velocity of the body HG r r
r r ϕ =ω
ϕ - angular velocity of the system x
x3′
x1 , x2 , x3 - principal axes of inertia x2
x3
x1

G x 2′

x1′
G - center of mass

Figure 4.76

r
In Problem 4.45 we have obtained an efficient equation in order to calculate H& G , (see
r r
equation (4.171)), and by considering ϕ = ω we can obtain:
r r r
 DH G   DH G  r r  DH G  r r
  =  +ϕ ∧ H G =   +ω ∧ H Gxyz
 Dt   Dt   Dt 
 f  r  r
For this problem we have:
I 1 0 0  ω1 
 
(I Gxyz ) ij =  0 0 
r
I2 ; (ω ) i = ω 2 
 0 0 I 3  ω 
 3
Note that we are already considering that the system Gxyz is the principal inertia axis.
The angular momentum:
r r r r
H Gxyz = I Gxyz ⋅ ω components
  → ( H Gxyz ) i = (I Gxyz ) ij (ω ) j
r
 ( H Gxyz ) 1  I 1 0 0   ω1   I 1 ω1 
 r      
( Hr Gxyz ) 2  =  0 I2 0  ω 2  = I 2 ω 2 
( H   I 3  ω 3  I 3 ω 3 
 Gxyz ) 3   0 0

The rate of change of the angular momentum:


Note that, since the system G xyz is attached to the body the mass distribution respect to
this system does not change during motion, so, I Gxyz does not change as well, i.e.
I& Gxyz = 0 . With that we can obtain:
r
 ( H& )  & & 1   I1ω
 r Gxyz 1   I1ω1 + I1ω &1 
r r
 DH G   DH Gxyz  r&  &   
  =  ≡ H Gxyz components
  → &
( H Gxyz ) 2  = I 2 ω2 + I 2 ω & 2 
& 2  = I 2 ω
 Dt   Dt 
 r   Gxyz  r&   I& ω + I ω &  I3ω & 3 
(H )
 Gxyz 3  
3 3 3 3

University of Castilla-La Mancha Draft By: Eduardo W. V. Chaves (2016)


Ciudad Real - Spain
458 SOLVING PROBLEMS BY MEANS OF CONTINUUM MECHANICS

eˆ 1 eˆ 2 eˆ 3
r r
ω ∧ H Gxyz = ω1 ω2 ω3
I 1 ω1 I 2 ω2 I 3 ω3
= (ω 2 I 3 ω 3 − ω 3 I 2 ω 2 )eˆ 1 + (ω 3 I 1 ω1 − ω1 I 3 ω 3 )eˆ 2 + (ω1 I 2 ω 2 − ω 2 I 1 ω1 )eˆ 3
= ω 2 ω 3 (I 3 − I 2 )eˆ 1 + ω1 ω 3 (I 1 − I 3 )eˆ 2 + ω1 ω 2 (I 2 − I 1 )eˆ 3
Components:
ω2 ω3 (I3 − I 2 ) 
r r  
{ω ∧ H Gxyz }i =  ω1ω3 (I1 − I3 ) 
 ω ω (I − I ) 
 1 2 2 1 
With that we can calculate
r
 DH G  r r r r
  = H& Gxyz + ω ∧ H Gxyz = 0
 Dt 
 f
whose components are:
 I1ω& 1  ω2ω3 (I3 − I 2 ) 0
r& r r r      
{H Gxyz }i + {ω ∧ H Gxyz }i = {0 }i ⇒ I 2 ω
& 2  +  ω1ω3 (I1 − I3 )  = 0
I ω     
 3 & 3   ω1ω2 (I 2 − I1 )  0
I1ω& 1 = ω2ω3 (I 2 − I3 )

⇒ I 2ω
& 2 = ω1ω3 (I3 − I1 )
I ω
 3 & 3 = ω1ω2 (I1 − I 2 )
b) The kinetic energy for rigid body motion, (see equation (4.161) in Problem 4.42), is
given by:
1 1r r
K(t ) = mv 2 + ω ⋅ I ⋅ ω
2 2
Since the origin of the adopted system is at G (mass center) we have v = 0 , with that we
can obtain:
I 1 0 0   ω1 
1 1
K(t ) = ω k I kj ω j = [ω1
2 2
ω2 ω 3 ]  0 I2
1
[
0  ω 2  = I1 ω12 + I 2 ω 22 + I 3 ω 32
2
]
 0 0 I 3   ω 3 
And the rate of change of the kinetic energy becomes:
D
Dt
K(t ) = K& (t ) =
1 D
2 Dt
[ 1
] [
I1ω12 + I 2 ω22 + I 3ω32 = 2ω1 I1ω
2
& 1 + 2ω 2 I 2 ω
& 2 + 2ω3I 3ω
&3 ]
= ω1 I1ω & 1 + ω2I 2 ω
& 2 + ω3I 3ω
&3
If we consider the Euler’s equation (4.184) the above equation becomes:
K& (t ) = ω1 I1ω& 1 + ω2I 2 ω& 2 + ω3I3ω& 3 = ω1 ω2ω3 (I 2 − I3 ) + ω2ω1ω3 (I3 − I1 ) + ω3ω1ω2 (I1 − I 2 )
= ω1 ω2 ω3 (I 2 − I3 + I3 − I1 + I1 − I 2 )
=0
with that we have shown that the kinetic energy is constant for any problem which is
governed by Euler’s equations of motion.

University of Castilla-La Mancha Draft By: Eduardo W. V. Chaves (2016)


Ciudad Real - Spain
4 THE FUNDAMENTAL EQUATIONS OF CONTINUUM MECHANICS 459

Problem 4.48
Obtain a simplified form of the rigid body governing equations for the particular case:
a) Rigid body rotation around a fixed axis without forces.
Solution:
We will consider the fixed system OX 1 X 2 X 3 and we will adopt the rotation axis by the X 3 -
axis, (see Figure 4.77), and the mobile system Ox1 x2 x3 is attached to the body.

X 3 , x3 r r
X1 ϕ = ω = ω 3 eˆ 3 = ω 3 Eˆ 3
x1
ω3

r
ω - angular velocity of the body
x2 r r
ϕ - angular velocity of the system x
O

X2

system OX 1 X 2 X 3 => orthonormal base (Eˆ 1 , Eˆ 2 , Eˆ 3 ) or (Iˆ , Jˆ , Kˆ )


system Ox1 x2 x3 => orthonormal base (eˆ 1 , eˆ 2 , eˆ 3 ) or (ˆi , ˆj, kˆ )

Figure 4.77

If the body is free of forces the governing equations becomes:


r r r r&
∑F =0 and ∑M O = HO
r
where H& O can be calculated by means of
r r r
r&  DH O   DH Oxr  r r  DH Oxr  r r
HO ≡   =  + ϕ ∧ H Oxr =   +ω ∧ H Oxr
 Dt  r  Dt  r  Dt  r
  OX   Ox   Ox
The angular momentum:
r r r r
H Oxr = I Oxr ⋅ ω  components
 → ( H Oxr ) i = (I Oxr ) ij (ω ) j
r
 ( H Oxr ) 1   I O 11 − I O 12 − I O 13   0   − I O 13 ω 3 
 r r      
( Hr Ox ) 2  = − I O 12 I O 22 − I O 23   0  = − I O 23 ω 3 
( H r )   − I − I O 23 I O 33  ω 3   I O 33 ω 3 
 Ox 3   O 13

And its rate of change:


r
 ( H& r )   − I ω
O13 3 
&
 r& Ox 1   
( H Oxr ) 2  = − I O 23ω
& 3
r
( H& r )   I ω & 
 Ox 3   O 33 3 

University of Castilla-La Mancha Draft By: Eduardo W. V. Chaves (2016)


Ciudad Real - Spain
460 SOLVING PROBLEMS BY MEANS OF CONTINUUM MECHANICS

r r
And we need to calculate the vector ω ∧ H Oxr :

eˆ 1 eˆ 2 eˆ 3
r r
ω ∧ H Oxr = 0 0 ω 3 = I O 23 ω 32 eˆ 1 − I O 13 ω 32 eˆ 2
− I O 13 ω 3 − I O 23 ω 3 I O 33 ω 3

thus
& 3   I O 23ω32   I O 23ω32 − IO13ω
 − I O13ω &3 
r&      
& 3  + − I O13ω32  = − I O13ω32 − I O 23ω
( H O ) i = − I O 23ω & 3
 I ω   0   IO 33ω 
 O 33 & 3    
&3

r r&
By applying ∑ M O = H O we can obtain the following set of equations:



∑M O1 ≡ ∑M X = I O 23ω32 − IO13ω
&3
 ∑M O2 ≡ ∑M Y
2
= − IO13ω3 − I O 23ω&3

 ∑M O3 ≡ ∑M Z = I O 33ω
&3

where ω& = α stands for angular acceleration.

NOTE: If the body is prismatic and if we adopt the prismatic axis the same as the rotating
axis the above equations reduce to:


∑M O1 ≡ ∑M X =0
 ∑M O2 ≡ ∑M Y =0

 ∑M O3 ≡ ∑M Z = I O 3ω
&3

since the system Ox1 x2 x3 is principal axes of inertia, (see Figure 4.78).

z
I O 1 0 0 
x  
ω3 I Gxyz =  0 IO 2 0 
 0 0 I O 3 
IO1 = I O 2
G

Figure 4.78

Problem 4.49
A rigid body consists of two masses m at each extremity of the weightless rod of length
2l . The rod is inclined about θ respect to the vertical line and rotates with angular
velocity ω as indicated in Figure 4.79.
a) Find the angular momentum of the body;
r
b) Find the torque ( ∑ M ) in order to maintain the rotation.

University of Castilla-La Mancha Draft By: Eduardo W. V. Chaves (2016)


Ciudad Real - Spain
4 THE FUNDAMENTAL EQUATIONS OF CONTINUUM MECHANICS 461

θ
l

Figure 4.79

Solution:
We apply the governing equations for a rigid solid motion, (see Problem 4.38). We will
adopt the fixed system in space OXYZ and a mobile system Oxyz which is attached to the
body, (see Figure 4.80).

Y
r
ω ω = ωJˆ
r
ω= ω
m
y
r
θ ω
l

X
Z≡z
l
m

Figure 4.80

University of Castilla-La Mancha Draft By: Eduardo W. V. Chaves (2016)


Ciudad Real - Spain
462 SOLVING PROBLEMS BY MEANS OF CONTINUUM MECHANICS

The inertia tensor I (system Oxyz ) is given by:


0 0 0 
IOxyz 
= 0 2ml 2 0 
0 0 2ml 2 
r
The angular velocity ω (system Oxyz ):
r
ω = −ω cos(θ )ˆi + ω sin(θ )ˆj + 0kˆ
r
where ω is the module of ω .
r
The angular momentum H O :
r r
HO = I ⋅ ω
 H Ox  0 0 0  − ω cos(θ )  0 
  
⇒ H Oy  = 0 2ml 2    
0   ω sin(θ )  =  2ml ω sin(θ )
2

 H Oz  0 0 2ml 2   0   0 
r
⇒ H O = 0ˆi + [2ml 2 ω sin(θ )] ˆj + 0kˆ
r
The torque ∑ M can be evaluated as follows:
r r& r& r r
∑ M = H O = ( H O ) Oxyz + ϕ ∧ H O
r& r r r
We can observe that (H O ) Oxyz = 0 and ϕ = ω hold, then:

 ˆi ˆj kˆ 
r r& r r  
∑ M = H O = ω ∧ H O =  − ω cos(θ ) ω sin(θ ) 0
 0 2ml 2 ω sin(θ ) 0 
 
r r&
⇒ ∑ M = H O = −ω cos(θ )2ml ω sin(θ )kˆ = −ω ml sin(2θ )kˆ
2 2 2

Y
r
ω = −ω cos(θ )ˆi + ω sin(θ )ˆj + 0kˆ
ω
r
H = 0ˆi + [ 2ml 2ω sin(θ )] ˆj + 0kˆ
O

m
y
r
θ ω r
l HO
X
Z≡z
l
m

Figure 4.81

University of Castilla-La Mancha Draft By: Eduardo W. V. Chaves (2016)


Ciudad Real - Spain
4 THE FUNDAMENTAL EQUATIONS OF CONTINUUM MECHANICS 463

Solution using the system OXYZ


The transformation matrix from OXYZ to Oxyz is given by:
T
 π  π  
 cos 2 − θ  sin  2 − θ  0
       sin (θ ) − cos(θ ) 0
 π   π  
A = − sin  − θ  cos − θ  0 = cos(θ ) sin (θ ) 0

 2  2  
 0 0 1  0 0 1
 
 
The inertia tensor for the system OXYZ is:
IOXYZ = A T IOxyz A
 sin (θ ) cos(θ ) 0 0 0 0   sin (θ ) − cos(θ ) 0
IOXYZ   
=  − cos(θ ) sin (θ ) 0 0 2ml 2 0  cos(θ ) sin (θ ) 0
 0 0 1 0 0 2ml 2   0 0 1
 2ml 2 cos 2 (θ ) 2ml 2 sin (θ ) cos(θ ) 0 
 
=  2ml 2 sin (θ ) cos(θ ) 2ml 2 sin 2 (θ ) 0 
 0 0 2ml 2 

The angular momentum becomes:
r r
H OXYZ = A T H Oxyz

 H Ox   0 
H  =  2ml 2ω sin(θ ) 
 Oy   
 H Oz   0 

H OX   sin (θ ) cos(θ ) 0    2ml ω cos(θ ) sin(θ )


2
0
 
⇒  H OY  =  − cos(θ ) sin (θ ) 0 2ml 2 ω sin(θ ) =  2ml 2 ω sin 2 (θ ) 
 H OZ   0 0 1  0   0 
 
The torque:
r
∑M = −ω2 ml 2 sin(2θ )K̂

Problem 4.50
A gyroscope consists of an outer gimbal, inner gimbal and a rotor with mass m , (see
Figure 4.82). The outer gimbal can rotate about the Z -axis defining the angle φ
(precession angle), the inner gimbal can rotate about the y -axis defining the angle θ
(nutation angle), the rotor can rotate about the z -axis defining the angle ψ (rotation
angle). The angles ( φ , θ , ψ ) are called Euler angles.
Obtain the governing equations for the gyroscope.

University of Castilla-La Mancha Draft By: Eduardo W. V. Chaves (2016)


Ciudad Real - Spain
464 SOLVING PROBLEMS BY MEANS OF CONTINUUM MECHANICS

φ - precession angle
Precession axis Z θ - nutation angle
ψ - rotation angle
θ
φ&

z - spin axis

outer gimbal
inner gimbal

ψ&
O θ&
φ ψ Y

rotor
x
X

Figure 4.82

Consider the inertia tensor components of the rotor related to the system Oxyz as follows:
I′ 0 0
(I Oxr )ij =  0 I′ 0
 0 0 I 

Solution:
We will adopt the orthonormal basis of the fixed system OXYZ by ( Î , Ĵ , K̂ ), and for the
mobile system Oxyz we will adopt the orthonormal basis ( î , ĵ , k̂ ).
Angular velocity of the rotor is given by, (see Figure 4.83):
r
ω = φ& K
ˆ + θ& ˆj + ψ& kˆ

= [−φ& sin(θ ) ˆi + +φ& cos(θ )] kˆ ] + θ& ˆj + ψ& kˆ


= −φ& sin(θ ) ˆi + θ& ˆj + [ψ& + φ& cos(θ )] kˆ

University of Castilla-La Mancha Draft By: Eduardo W. V. Chaves (2016)


Ciudad Real - Spain
4 THE FUNDAMENTAL EQUATIONS OF CONTINUUM MECHANICS 465

Z
z

φ& K̂ φ& cos(θ )

− φ& sin(θ )

Figure 4.83

The governing equations for a rigid solid motion are given by:
r r r r&
∑F =ma and ∑M O = HO
r
where H& O can be calculated by means of
r r
r&  DH O   DH Oxr  r r
H O ≡   =
 
 + ϕ ∧ H Oxr

 Dt  OXr  Dt  Oxr
Angular momentum:
r r r r
H Oxr = I Oxr ⋅ ω components
 → ( H Oxr )i = (IOxr )ij (ω) j
r
 ( H Oxr )1  I′ 0 0  − φ& sin(θ )   − I′φ& sin(θ ) 
 r r    θ&
 
I′θ&

( Hr Ox )2  =  0 I′ 0  = 
( H r )   0 0 I  [ψ& + φ& cos(θ )] I [ψ& + φ& cos(θ )]
 Ox 3      
and its rate of change can be obtained as follows:
r
 ( H& r )   − I′φ& sin(θ )  − I′[φ
&& sin(θ ) + φ& cos(θ )θ& ]
 r& Ox 1  D    
( H Oxr ) 2  =  I′θ& = I′θ&& 
( Hr& r )  Dt I [ψ& + φ& cos(θ )]  I D [ψ& + φ& cos(θ )] 
 Ox 3     Dt 
Note that due to the symmetry of the rotor, the inertia tensor does not change over time
respected to the system Oxyz .
r r r
We need to calculate the vector ϕ ∧ H Oxr , where ϕ is the angular velocity of the rotating
system Oxyz . Note that the mobile system can rotate about the K̂ -axis and about the ĵ -
axis, and cannot rotate about the k̂ -axis, (see Figure 4.83), then, the angular velocity of the
mobile system can be given by:

University of Castilla-La Mancha Draft By: Eduardo W. V. Chaves (2016)


Ciudad Real - Spain
466 SOLVING PROBLEMS BY MEANS OF CONTINUUM MECHANICS

r
ϕ = φ& Kˆ + θ& ˆj
= [−φ& sin(θ ) ˆi + φ& cos(θ )] kˆ ] + θ& ˆj
= −φ& sin(θ ) ˆi + θ& ˆj + φ& cos(θ ) kˆ
ˆi ˆj kˆ
r r
ϕ ∧ H Oxr = − φ& sin(θ ) θ& φ& cos(θ )
− I′φ& sin(θ ) I′θ& I [ψ& + φ& cos(θ )]

= {θ& I [ψ& + φ& cos(θ )] − φ& I′θ& cos(θ )}ˆi − {−Iφ& sin(θ ) [ψ& + φ& cos(θ )] + φ& I′φ& sin(θ ) cos(θ )}ˆj
+ {−I′θ& φ& sin(θ ) + I′θ& φ& sin(θ )}kˆ

 θ& I [ψ& + φ& cos(θ )] − φ& I′θ& cos(θ ) 


r r  & 
(ϕ ∧ H Oxr ) i = Iφ sin(θ ) [ψ& + φ& cos(θ )] − φ& 2 I′ sin(θ ) cos(θ ) 
 0 
 
thus
r
r&  DH r   r r
( H O ) i =  Ox   + (ϕ ∧ H Oxr ) i
 r
 Dt  Ox i
 ′ && & &  
− I [φ sin(θ ) + φ θ cos(θ )]  θ& I [ψ& + φ& cos(θ )] − φ& I′θ& cos(θ ) 

= I′θ&&  + Iφ& sin(θ ) [ψ& + φ& cos(θ )] − φ& I′ sin(θ ) cos(θ ) 
2

 I D [ψ& + φ& cos(θ )]   0 


 Dt   
 && & & & & & 
 − I′[φ sin(θ ) + 2φ θ cos(θ )] + θ I [ψ + φ cos(θ )] 
 
= Iφ& sin(θ ) [ψ& + φ& cos(θ )] − φ& 2 I′ sin(θ ) cos(θ ) + I′θ&&
 D & & 
 I [ψ + φ cos(θ )] 
 Dt 
r r&
Applying ∑ M O = H O we can obtain the following set of equations:




∑M O1 ≡ ∑M x
&& sin(θ ) + 2φ& θ& cos(θ )] + I θ& [ψ& + φ& cos(θ )]
= −I′[φ
 ∑M O2 ≡ ∑M y = I′ [θ&& − φ& 2 sin(θ ) cos(θ )] + Iφ& sin(θ ) [ψ& + φ& cos(θ )] (4.185)
 D & &


∑M O3 ≡ ∑M z =I
Dt
[ψ + φ cos(θ )]

NOTE: Particular case: Steady precession.


In this case the variables θ , φ& and ψ& are constant, the following equations are true:
θ& = 0 , φ&& = 0 and ψ&& = 0 , so
r r
 ( H Oxr ) 1   − I ′φ& sin(θ )   (ϕ )1  − φ& sin(θ ) 
 r    r & ˆ components  r   
( Hr Oxr ) 2  =  0  ; ϕ = φ K   → (ϕ ) 2  =  0 ;
( H r )  I [ψ& + φ& cos(θ )] (ϕr )   φ& cos(θ ) 
 Ox 3     3  

University of Castilla-La Mancha Draft By: Eduardo W. V. Chaves (2016)


Ciudad Real - Spain
4 THE FUNDAMENTAL EQUATIONS OF CONTINUUM MECHANICS 467

r
 (ω) 1  ω x   − φ& sin(θ ) 
 r     
(ω ) 2  = ω y  =  0 
(ωr     & & 
 ) 3  ω z  [ψ + φ cos(θ )]
The equations in (4.185) become:



∑M O1 ≡∑M x =0
 ∑M O2 ≡ ∑M y = −I′ φ& 2 sin(θ ) cos(θ ) + Iφ& sin(θ ) [ψ& + φ& cos(θ )]
14 4244 3
 =ω z

 ∑ M O3 ≡∑ Mz = 0

∑ M O1 ≡ ∑M x =0

⇒ ∑ M O2 ≡ ∑M y = [I ω z − I′ φ& cos(θ )] φ& sin(θ )
 M
∑ O3 ≡ ∑M z =0

Steady precession
Z
θ θ

φ&  = constant z

ψ& 

r
ϕ = φ& K̂ r
ωz k̂
ω

ψ& k̂
ωx = −φ& sin(θ ) y

r
∑M O

Figure 4.84: Steady precession.


Rigid Solid Motion References
BEER, F.P. & JOHNSTON, E.R. (1987). Vector Mechanics for Engineers: Dynamics. Seventh
Edition. 2 Volumes. McGraw-Hill Science/Engineering/Math; 4 edition.
BEER, F.P.; JOHNSTON, E.R. & CLAUSEN, W.E. (2004). Instructor's and Solutions Manual to
Accompany Vector Mechanics for Engineers - Dynamics. Seventh Edition. 2 Volumes. McGraw
Hill Higher Education; Seventh edition (2004).

University of Castilla-La Mancha Draft By: Eduardo W. V. Chaves (2016)


Ciudad Real - Spain
468 SOLVING PROBLEMS BY MEANS OF CONTINUUM MECHANICS

University of Castilla-La Mancha Draft By: Eduardo W. V. Chaves (2016)


Ciudad Real - Spain
5 Introduction to:
Constitutive Equations,
IBVP Statement, and
IBVP Solution Strategies
Problem 5.1
Describe the constitutive equations and the free variables for simple thermoelastic
materials when we are considering the specific Helmholtz free energy ψ .
Solution:
The constitutive equations for a simple material are in function of the following free
variables:
Constitutive equation for energy ψ = ψ ( F , T )
∂ψ ( F , T )
Constitutive equation for stress P( F , T ) = ρ 0
∂F
∂ψ ( F , T )
Constitutive equation for entropy η ( F , T ) = −
∂T
r r
q
Constitutive equation for heat conduction 0 = q 0 ( F , T , ∇ rT )
X

The free variables are F -deformation gradient, T -temperature, ∇ Xr T -temperature


gradient, (see Chaves 2013 – Chapter 6). The constitutive equations can also be expressed
as follows
ψ = ψ (F , T )
ψˆ = ψ ( E , T )
∂ψ ( F , T ) T
∂ψ ( E , T ) σ=ρ ⋅F
S = ρ0 ∂F
∂E ∂ψ ( F , T )
; η (F , T ) = −
∂ψ ( E , T )
η (E ,T ) = − ∂T
∂T r −1 r
r r q = J q0 ( F , T , ∇ Xr T ) ⋅ F T
qˆ 0 = q0 ( E , T , ∇ Xr T ) r
= J −1F ⋅ q0 ( F , T , ∇ Xr T )

Problem 5.2
Consider an elastic material in which the energy density (energy per unit volume) is known
and is given by:
408 SOLVING PROBLEMS BY MEANS OF CONTINUUM MECHANICS

1
Ψ ( I E , II E ) = (λ + 2 µ )I E2 − 2 µ II E
2
where λ and µ are material constants, I E = I E (E ) and II E = II E (E ) are, respectively, the
first and second principal invariants of the Green-Lagrange strain tensor. Obtain the
constitutive equations for this problem. Also obtain the explicit expression for the
constitutive equations in terms of λ , µ , I E and II E .
Formulary

I E = I E ( E ) = Tr ( E ) ; II E = II E ( E ) =
1
2
[ ] ∂I
( TrE ) 2 − Tr ( E 2 ) ; E = 1 ;
∂E
∂ II E
∂E
= Tr ( E )1 − E T .

Solution:
According to the problem, the energy density is only a function of the Green-Lagrange
strain tensor. We know that the general expressions for the constitutive equations for a
simple thermoelastic material are:
ψˆ = ψ ( E , T )
∂ψ ( E , T )
S = ρ0
∂E
∂ψ ( E , T )
η (E ,T ) = −
∂T
r r
qˆ 0 = q0 ( E , T , ∇ Xr T )

Considering the equation for the given energy density, we can conclude that the problem is
independent of temperature, since the energy density equation is not a function of
temperature. Then, the remaining constitutive equation is the one related to stress, i.e.:
∂ψ ( E ) ∂Ψ ( I E , II E ) ∂Ψ ( I E , II E ) ∂I E ∂Ψ ( I E , II E ) ∂ II E
S = ρ0 = = +
∂E ∂E ∂I E ∂E ∂ II E ∂E
2 
(
=  (λ + 2 µ )I E (1) + (−2 µ ) Tr ( E )1 − E T )
2 
By simplifying the above equation, and taking into account that E T = E , I E = Tr (E ) , we
can obtain:
S = λI E 1 + 2 µE
Problem 5.3
1
Consider the specific Gibbs free energy G(S, T ) = ψ ( E , T ) − S : E as constitutive
ρ0
equation for energy for thermoelastic materials. Obtain the remaining constitutive
equations for thermoelastic materials, which is based on the principle that G(S, T ) does
not depend on the temperature gradient.
Solution:
We start from the Clausius-Duhem inequality in terms of specific Helmholtz free energy in
the reference configuration:
1r
S : E& − ρ 0 [ψ& + T&η ] − q0 ⋅ ∇ Xr T ≥ 0 (5.1)
T
Taking into account the specific Gibbs free energy we can obtain the rate of change:

University of Castilla-La Mancha Draft By: Eduardo W. V. Chaves (2016)


Ciudad Real – Spain
5 INTRODUCTION TO: CONSTITUTIVE EQUATIONS, IBVP, AND IBVP SOLUTION STRATEGIES 409

& (S, T ) = ψ& ( E , T ) − 1 S& : E − 1 S : E&


G
ρ0 ρ0
& (S, T ) + 1 S& : E + 1 S : E&
⇒ ψ& ( E , T ) = G
ρ0 ρ0
and by replacing the above equation into the inequality (5.1) we can obtain:
& 1 & 1  1r
S : E& − ρ 0 G (S , T ) + S:E+ S : E& + T&η  − q0 ⋅ ∇ Xr T ≥ 0
 ρ0 ρ0  T (5.2)
& (S , T ) − S& : E − ρ T&η − 1 qr
⇒ − ρ 0G 0 ⋅∇ XT ≥ 0
r
0
T

Note that S& : E = E : S& holds. The above inequality suggests that for a variation of Gibbs
free energy we must have the following relationships: Strain for “variation” of stress,
Entropy for a variation of temperature, and heat conduction in terms of temperature
gradient.
& (S , T ) can also be expressed as follows:
The term G
DG(S, T ) & ∂G(S, T ) & ∂G(S , T ) &
≡ G(S, T ) = :S + T
Dt ∂S ∂T
and by replacing the above equation into the equation in (5.2) we can obtain:
& (S, T ) − E : S& − ρ T&η − 1 q r
− ρ 0G 0 ⋅∇ XT ≥ 0
r
0
T
∂G(S, T ) & ∂G(S, T ) & 1r
⇒ −ρ 0 :S − ρ0 T − E : S& − ρ 0T&η − q0 ⋅ ∇ Xr T ≥ 0 (5.3)
∂S ∂T T
 ∂G(S, T )   ∂G(S , T )  1 r
⇒ − ρ 0 + E  : S& − ρ 0  + η T& − q0 ⋅ ∇ Xr T ≥ 0
 ∂S   ∂T  T

The above inequality must be satisfied for any admissible thermodynamic process. Let us
r r
now consider the process such that T& = 0 (isothermal process), and q 0 = 0 (adiabatic
process), then the above entropy inequality becomes:
 ∂G(S, T ) 
−ρ0 + E  : S& ≥ 0 (5.4)
 ∂S 
Note that the above inequality must also be met for any thermodynamic process. Then if in
the current process the condition in (5.4) is met, we can apply another process such that
S& = −S& , in which the entropy inequality (5.4) is violated. Thus, the only way in which the
inequality in (5.4) is satisfied is when:
∂G(S, T ) ∂G(S, T )
ρ0 +E =0 ⇒ E = −ρ 0
∂S ∂S
Then if we take into account the above equation into the inequality (5.3), we can obtain:
 ∂G(S, T )   ∂G(S, T )  1r
−ρ0 + E  : S& − ρ 0  + η T& − q0 ⋅ ∇ Xr T ≥ 0
 ∂S   ∂T  T
(5.5)
 ∂G(S, T )  1r
⇒ −ρ 0 + η T& − q0 ⋅ ∇ Xr T ≥ 0
 ∂T  T

University of Castilla- La Mancha Draft By: Eduardo W. V. Chaves (2016)


Ciudad Real – Spain
410 SOLVING PROBLEMS BY MEANS OF CONTINUUM MECHANICS

r
Now let us consider a process where ∇ Xr T = 0 (uniform temperature field), then the
inequality becomes:
 ∂G(S , T ) 
− ρ 0 + η T& ≥ 0
 ∂T 
Starting from this point, we could apply another process where T& = −T& , in which the
entropy inequality is violated. Thus, the only way in which the above inequality is satisfied
is when:
∂G(S, T ) ∂G(S, T )
+η=0 ⇒ η=−
∂T ∂T
Then, the constitutive equations are:
Constitutive equation for energy G = G(S, T )
∂G(S , T ) ∂g(S , T )
Constitutive equation for strain E = − ρ 0 =
∂S ∂S
(5.6)
∂G(S, T )
Constitutive equation for entropy η = −
∂T
r r
Constitutive equation for heat conduction q0 = q0 (∇ Xr T )
where g(S, T ) = −ρ 0 G(S, T ) . Note that the free variables are (S , T ) .

Problem 5.4
Show that for an isothermal adiabatic process and with no rate of change of stress the
specific Gibbs free energy cannot increase.
Solution:
We start directly from the inequality in (5.3):
r
& (S, T ) − E : S& − ρ T&η − 1 q
− ρ 0G 0 ⋅∇ XT ≥ 0 (5.7)
r
0
T
r r
Taking into account the isothermal adiabatic process we have T& = 0 , q0 = 0 , and with no
rate of change of stress the equation S& = 0 holds. With that the inequality in (5.7)
becomes:
& (S, T ) ≥ 0
− ρ 0G (5.8)
Note that ρ 0 > 0 is always positive, then to satisfy the above inequality the condition
& (S, T ) ≤ 0 must hold.
G

Problem 5.5
Find the governing equations for a continuum solid which has the following features:
Isothermal and adiabatic processes; an infinitesimal strain regime and a linear elastic
relationship between stress and strain.
b) Once the stress-strain linear relationship has been established, find the equation in which
σ (ε ) is a tensor-valued isotropic tensor function.
Solution:

University of Castilla-La Mancha Draft By: Eduardo W. V. Chaves (2016)


Ciudad Real – Spain
5 INTRODUCTION TO: CONSTITUTIVE EQUATIONS, IBVP, AND IBVP SOLUTION STRATEGIES 411

When we are dealing with isothermal and adiabatic processes, temperature and entropy play
no role.
In an infinitesimal strain regime, the following is satisfied:
r
Strain tensors: E ≈ e ≈ ε = ∇ sym u
Stress tensors: P ≈ S ≈ σ
F ≈1 ; ∇ Xr ≈ ∇ xr ≈ ∇ ; ρ ≈ ρ 0 . If we take this approach, mass density is no
longer unknown ( ρ& = 0 ).
Then, taking into account the fundamental equations:
The Fundamental Equations of Continuum Mechanics
(Current configuration)
The Mass Continuity Equation Dρ r
+ ρ (∇ xr ⋅ v ) = 0 (5.9)
(The principle of conservation of mass) Dt
The Equations of Motion r r
(The principle of conservation of linear ∇ xr ⋅ σ + ρ b = ρ v& (5.10)
momentum)
Cauchy Stress Tensor symmetry
(The principle of conservation of angular σ = σT (5.11)
momentum)
The Energy Equation r
ρ u& = σ : D − ∇ xr ⋅ q + ρr (5.12)
(The principle of conservation of energy)
The Entropy Inequality r 1 1 1 r
ρη& ( x, t ) + σ : D − ρ u& − 2 q ⋅ ∇ xr T ≥ 0 (5.13)
(The principle of irreversibility) T T T
the remaining equations for the proposed problem are:
1) The equations of motion
r r
∇ ⋅ σ + ρ b = ρ v&
2) The energy equation (reference configuration):
r r r
ρ 0 u&( X , t ) = S : E& − ∇ Xr ⋅ q 0 + ρ 0 r ( X , t ) ⇒ ρu& = σ : ε&
Du D
where u& is the specific internal energy, and the relationship = [ψ + Tη ] = ψ& holds,
Dt Dt
where ψ is the specific Helmholtz free energy. Note also that
ρψ& = Ψ& e = σ : ε&
where Ψ e is the strain energy density, in which Ψ& e = ρ& ψ + ρψ& = ρψ& . Then if we bear in
mind the entropy inequality, we can observe that the proposed problem is characterized by
a process without any energy dissipation (an elastic process), i.e. all stored energy caused by ε
will recover when ε = 0 .
3) For this problem, the constitutive equations described in Problem 5.1 become:
ψ = ψ (ε )
∂ψ (ε ) ∂Ψ e (ε )
S≈σ=ρ = = σ (ε )
∂ε ∂ε

University of Castilla- La Mancha Draft By: Eduardo W. V. Chaves (2016)


Ciudad Real – Spain
412 SOLVING PROBLEMS BY MEANS OF CONTINUUM MECHANICS

Energy ( ψ ) and stress are only functions of strain. Then, if we calculate the rate of change
∂ψ (ε ) &
of the Helmholtz free energy, i.e. ψ& (ε ) = : ε , and by substituting it into the equation
∂ε
D ( ρψ ) D (Ψ ) & e e
ρψ& = = = Ψ = σ : ε& , we can obtain:
Dt Dt
∂ψ (ε ) & ∂Ψ& e (ε ) & ∂Ψ e (ε )
ρ :ε = : ε = σ : ε& ⇒ σ=
∂ε ∂ε ∂ε
Thus, we can conclude that the energy equation is a redundant one, i.e. if the stress is
known the energy can be evaluated and vice-versa. So, we can summarize the governing
equations for the problem proposed as follows:
The equations of motion:
r r &r& (3 equations)
∇ ⋅ σ + ρb = ρv& = ρu
The constitutive equations for stress:
∂Ψ e (ε ) (5.14)
σ (ε ) = (6 equations)
∂ε
Kinematic equations:
r
ε = ∇ symu (6 equations)
r
The unknowns of the proposed problem are: σ (6), u (3) and ε (6), making a total of 15
unknowns and 15 equations, so the problem is well-posed. Then, to achieve the unique
solution of the set of partial differential equations given by (5.14) one must introduce the
initial and boundary conditions, hence defining the Initial Boundary Value Problem (IBVP) for
the linear elasticity problem. The initial and boundary conditions for this problem are:
The displacement boundary condition, on S u :
r r r r r r
u( x , t ) =u* ( x , t ) ui ( x , t ) = u*i ( x , t ) (5.15)
The stress boundary condition, on S σ :
r r r r
σ ( x , t ) ⋅ nˆ = t * ( x , nˆ , t ) σ jk nˆ k = t *j ( x, t ) (5.16)
The initial conditions ( t = 0 ):
r r r
u( x , t = 0) = u0 r r
r r u i ( x , t = 0) = u 0 i ( x )
∂u0 ( x , t ) r r r r r (5.17)
= u& 0 ( x , t ) = v0 ( x ) u& 0 i ( x ) = v 0 i
∂t t =0

In the particular case when we are dealing with a static or quasi-static problem, the
r r
equations of motion become the equilibrium equations ( ∇ ⋅ σ + ρ b = 0 ), and the initial
conditions become redundant.

B Sσ
Su
r r
dV t * ( x)
r r r
u*
ρ b( x )

Figure 5.1: Solid under external actions.

University of Castilla-La Mancha Draft By: Eduardo W. V. Chaves (2016)


Ciudad Real – Spain
5 INTRODUCTION TO: CONSTITUTIVE EQUATIONS, IBVP, AND IBVP SOLUTION STRATEGIES 413

In subsection 1.6.1 The Tensor Series (Chapter 1-textbook, Chaves (2013)), we have seen
that we can approach a tensor-valued tensor function by means of the following series:
1 1 ∂σ (ε 0 ) 1 ∂ 2 σ (ε 0 )
σ (ε ) ≈ σ (ε 0 ) + : (ε − ε 0 ) + (ε − ε 0 ) : : (ε − ε 0 ) + L
0! 1! ∂ε 2! ∂ε ⊗ ∂ε
∂σ (ε 0 ) 1 ∂ 2σ (ε 0 )
≈ σ0 + : ( ε − ε 0 ) + (ε − ε 0 ) : : (ε − ε 0 ) + L
∂ε 2 ∂ε ⊗ ∂ε
Then, by considering the application point ε 0 = 0 and σ (ε 0 ) = σ 0 = 0 , and also taking
into account that the relationship σ - ε is linear, higher order terms can be discarded, thus:
∂σ (ε 0 ) ∂ 2Ψ e (ε 0 ) ∂σij ∂ 2Ψ e (ε 0 )
σ (ε ) = :ε = : ε = Ce : ε σij = ε kl = e
ε kl = Cijkl ε kl
∂ε ∂ε ⊗ ∂ε ∂ε kl ∂εij ∂ε kl

∂ 2Ψ e (ε )
where C e = is a symmetric fourth-order tensor which is known as the elasticity
∂ε ⊗ ∂ε
tensor, which contains the material mechanical properties.
Note that, the energy equation has to be quadratic with which we can guarantee that the
∂Ψ e (ε )
relationship σ - ε is linear, since σ (ε ) = . We can also use series expansion to
∂ε
represent the strain energy density as follows:
1 1 ∂Ψ e (ε 0 ) 1 ∂ 2Ψ e (ε 0 )
Ψ e ( ε ) = Ψ e (ε 0 ) + : ( ε − ε 0 ) + (ε − ε 0 ) : : (ε − ε 0 ) + L
0! 1! ∂ε 2! ∂ε ⊗ ∂ε
1 ∂ 2Ψ e (ε 0 )
= Ψ e0 + σ 0 : (ε − ε 0 ) + (ε − ε 0 ) : : (ε − ε 0 ) + L
2 ∂ε ⊗ ∂ε
1 ∂ 2Ψ e (ε 0 ) 1
= ε: : ε = ε : Ce : ε
2 ∂ε ⊗ ∂ε 2
where we have also considered that ε 0 = 0 ⇒ Ψ e0 = 0 , σ 0 = 0 .
NOTE 1: Although the energy equation is a redundant one, at the time of establishing an
analytical or numerical method to solve the problem, we will always start from energy
principles, hence the importance of studying the energy equation in a system.
NOTE 2: Analyzing C e :
e
Note that, according to the equation σ ij = C ijkl ε kl and due to the symmetry of σ ij = σ ji
e
and ε kl = ε lk , the tensor C e has minor symmetry, i.e. C ijkl = C ejikl = C ijlk
e
= C ejilk . Note also
that:
∂ 2Ψ e (ε ) ∂ 2Ψ e (ε )
Ceijkl = = = C eklij (major symmetry)
∂ε ij ∂ε kl ∂ε kl ∂εij
NOTE 3: To better illustration of the problem established here, let us consider a particular
case (a one-dimensional case) in which the stress and strain components are given by:
σ 0 0   ε 0 0
σ ij =  0 0 0 ; ε ij = 0 0 0 ⇒ σ11 = C1111
e
ε11 ⇒ σ = Eε
 0 0 0 0 0 0

University of Castilla- La Mancha Draft By: Eduardo W. V. Chaves (2016)


Ciudad Real – Spain
414 SOLVING PROBLEMS BY MEANS OF CONTINUUM MECHANICS

In this case, the stress-strain linear relationship becomes σ = Eε (Hooke’s law) and the
1 1 ∂ 2Ψ e ∂σ
strain energy density is given by Ψ e = σε = εEε , and = = E , (see Figure 5.2).
2 2 ∂ε∂ε ∂ε

Current state
Ψ e (ε) σ(ε)
1 1
Ψ e = εEε = σε σ
2 2
Ψe Stored energy
1
Ψ e = σε
E 2
1
σ0 = 0
Ψ e0 = 0 ε ε ε0 = 0 ε ε

Figure 5.2: Stress-strain relationship (one-dimensional case).

NOTE 4: Here it should be pointed out that in the case of elastic processes the
constitutive equation σ (ε ) is only dependent on the current value of ε , i.e. it is
independent of the deformation history. ■
b) The tensor-valued tensor function σ (ε ) is isotropic if the following is satisfied:
Ψ e (ε′kl ) = Ψ e (ε kl ) ⇒ σ′ij (ε′kl ) = σij (ε′kl )

Then, taking into account that the relationship σ - ε is given by σij (ε ) = Ceijkl ε kl (indicial
notation), we can conclude that:
σ′ij (ε′kl ) = σij (ε′kl ) ⇒ C′ijkl
e
ε′kl = Cijkl
e
ε′kl ⇒ C′ijkl
e
= Ceijkl

That is, the fourth-order tensor C e is isotropic. An isotropic symmetric fourth-order


e
tensor has the form C ijkl = λδ ij δ kl + µ (δ ik δ jl + δ il δ jk ) or C e = λ1 ⊗ 1 + 2µ I , (see Chapter
1), and here the parameters λ and µ are known as Lamé constants. As we have seen in
Chapter 1, a symmetric isotropic fourth-order tensor is a function of two variables ( λ , µ ).
We will see that it is possible to express C e in terms of other parameters, e.g. ( E , ν ),
( κ , G ), where E is the Young’s modulus (or longitudinal elastic modulus), ν is the Poisson’s ratio,
κ is the bulk modulus, and G = µ is the shear modulus (or transversal elastic modulus).

NOTE 5: Figure 5.3 shows the stress-strain relationship for an isotropic material. Note
that, for an isotropic linear elastic material in an infinitesimal strain regime the constitutive
equation for stress becomes σ (ε) = (λ1 ⊗ 1 + 2µ I) : ε = λTr(ε)1 + 2µ ε :
∂Ψ e (ε )
σ (ε ) = linear
→ σ (ε ) = C e : ε isotropic
 → σ (ε ) = λTr (ε )1 + 2 µε
14424 ∂4
ε3
Elastic

University of Castilla-La Mancha Draft By: Eduardo W. V. Chaves (2016)


Ciudad Real – Spain
5 INTRODUCTION TO: CONSTITUTIVE EQUATIONS, IBVP, AND IBVP SOLUTION STRATEGIES 415

It should be emphasized here that due to the fact that the C e -components are independent
of the coordinate system, the tensors σ and ε share the same principal space
(eigenvectors), (see Figure 5.3).

σ ′22 ′
σ11

σ12 x1′

σ′ij = C ′ijkl
e
ε ′kl P
σ′ij = a ip a jq σ pq
ε ′22 ′
ε12 ′
ε11 ε ′ij = a ip a jq ε pq

e
ε 22 σ ij = C ijkl ε kl σ 22
ε12 σ12
ε11 σ11
P P

P
x1
ε ′22

σ′ij′ = C ′ijkl
′ e ε ′kl′
P
′′
ε11
σ ′22

Principal space

P
Isotropic material
′′
σ11
Ceijkl = C′ijkl
e
= C′ijkl
′e

Ψ e (ε′kl ) = Ψ e (ε kl ) x1′′

Figure 5.3: Stress-strain relationship (isotropic material).

University of Castilla- La Mancha Draft By: Eduardo W. V. Chaves (2016)


Ciudad Real – Spain
416 SOLVING PROBLEMS BY MEANS OF CONTINUUM MECHANICS

NOTE 6: We denote the complementary strain energy density by Ψ e (σ ) which is a function of


σ , (see Figure 5.4), and is given by:
1 ∂Ψ (σ 0 ) ∂ 2Ψ e (σ 0 )
e
1 1
Ψ e (σ ) = Ψ e (σ 0 ) + : (σ − σ 0 ) + (σ − σ 0 ) : : (σ − σ 0 ) + L
0! 1! ∂σ 2! ∂σ ⊗ ∂σ
1 ∂ 2Ψ e (σ 0 )
= Ψ 0 + σ 0 : ( σ − σ 0 ) + (σ − σ 0 ) :
e
: (σ − σ 0 ) + L
2 ∂σ ⊗ ∂σ
1 ∂ 2Ψ e (σ 0 ) 1 1 −1
= σ: : σ = σ :De : σ = σ : Ce : σ
2 ∂σ ⊗ ∂σ 2 2
Note that if we are dealing with linear elastic material Ψ e (σ ) = Ψ e (ε) holds, and
∂Ψ e (σ )
ε= .
∂σ

a) Linear elastic material. 1


Complementary strain energy density - Ψ e (σ) = σE −1σ
σ(ε) 2
σ
Ψ ( ε ) = Ψ (σ )
e e
Stored energy
1
Strain energy density - Ψ e (ε) = εEε
E 2
1
σ0 = 0
ε0 = 0 ε ε

Ψ e + Ψ e = σε
b) Non-linear elastic material.
Complementary strain energy density - Ψ e (σ)

σ(ε)

Strain energy density - Ψ e (ε)


Ψ e (ε) ≠ Ψ e (σ)

σ0 = 0 ε
ε0 = 0 ε

Figure 5.4: Complementary strain energy density (one dimensional case).

NOTE 7: Note that Ψ e (σ) = σε − Ψ e (ε) tensorial


→Ψ e (σ ) = g = σ : ε − Ψ e (ε ) = − ρ 0 G(σ ) ,
where g(σ ) = −ρ 0 G(σ ) is the Gibbs free energy density (per unit volume) with reversed
sign, (see equations in (5.6) in Problem 5.3).

University of Castilla-La Mancha Draft By: Eduardo W. V. Chaves (2016)


Ciudad Real – Spain
5 INTRODUCTION TO: CONSTITUTIVE EQUATIONS, IBVP, AND IBVP SOLUTION STRATEGIES 417

NOTE 8: Taking into account the constitutive equation for stress for an isotropic linear
elastic material σ (ε ) = λTr (ε )1 + 2 µε and considering the additive decomposition of the
Tr (ε )
tensor into a spherical and deviatoric parts, i.e. ε = ε sph + ε dev = 1 + ε dev , we can
3
obtain:
 Tr (ε ) 
σ (ε ) = λTr (ε )1 + 2 µε = λTr (ε )1 + 2 µ (ε sph + ε dev ) = λTr (ε )1 + 2 µ  1 + ε dev 
 3 
 2µ 
= λ +  Tr (ε )1 + 2 µε dev = κ Tr (ε )1 + 2 µε dev = σ sph + σ dev
 3 

σ33 σm dev
σ 33

σ 23 σ 23
σ13 σ 23 = + σ13 σ 23

σ13 σ13
σ12 σ 22 σm σ12 σ dev
22
σ12 σ12
σ11 σm dev
σ11

σ ij = λTr (ε )δ ij + 2µ ε ij Tr (σ )δ ij = 3κ Tr (ε )δ ij σ ijdev = 2µ ε ijdev

ε 33 εm dev
ε 33

ε13
ε 23
ε 23
= + ε 13
ε 23
ε 23

ε13 ε13
ε12 ε 22 εm ε 12 ε dev
22
ε12 ε 12
ε11 εm dev
ε 11

Figure 5.5: Additive decomposition of the constitutive equation.

Recall that, if we are dealing with small deformation regime, the volume ratio (dilatation) is
given by:
dV − dV 0 ∆V
εv = = = ε 11 + ε 22 + ε 33 = Tr (ε ) = I ε
dV0 dV0

 2µ 
And if we take the trace of σ (ε ) =  λ +  Tr (ε )1 + 2 µε we can obtain:
dev

 3 

 2µ 
σ : 1 = λ +  Tr (ε )1 : 1 + 2 µε dev : 1
 3 
 2µ  Tr (σ )  2µ 
⇒ Tr (σ ) = 3 λ +  Tr (ε ) ⇒ = σ m = λ +  εv
 3  3  3 

University of Castilla- La Mancha Draft By: Eduardo W. V. Chaves (2016)


Ciudad Real – Spain
418 SOLVING PROBLEMS BY MEANS OF CONTINUUM MECHANICS

where we have considered that 1 : 1 = 3 and Tr (ε dev ) = 0 . If we are dealing with a


compression stress state ( p > 0 ) we have:
− p 0 0 
 2µ 
σ ij =  0 − p 0 
 ∴ 3σ m = Tr (σ ) = −3 p < 0 ⇒ − p = λ +  εv = κ εv
 3 
 0 0 − p 
For these reason, the parameter κ is called bulk modulus (or modulus of compression), (see
Figure 5.6), and is given by:

κ=λ+ (5.18)
3
Just as the spherical part of the tensor ( σ sph = κ Tr (ε )1 ) is associated with the volume
change, the deviatoric part ( σ dev = 2 µε dev ) is associated with the shape change, and the
parameter µ = G defines the stiffness to the shape change, where G is known as shear
modulus or transversal elastic modulus, (see NOTE 9).

y y τ xy y

p
σx σx

τ xy p

x x x

E -Young’s modulus G -Shear modulus κ -Bulk modulus

σx τ xy p ε v -volumetric
strain

E µ =G κ
1 1 1

εx γ xy εv

Figure 5.6: Some material mechanical properties.

NOTE 9: In the laboratory the parameters (λ, µ ) are not the more appropriated to be
obtained. Next we try to rewrite the constitutive equation in terms of other parameters.
Recall that the reverse form of the constitutive equation σ (ε ) = λTr (ε )1 + 2 µε was
obtained in Problem 1.98 which is:
1 λ 1 λ
ε= σ− Tr (σ )1 indicial
 → ε ij = σ ij − (σ11 + σ 22 + σ 33 )δ ij
2µ 2 µ ( 2 µ + 3λ ) 2µ 2 µ ( 2 µ + 3λ )

 ε11 ε12 ε13   σ11 σ12 σ13  1 0 0


ε  1   λ(σ11 + σ 22 + σ 33 ) 
 21 ε 22 ε 23  = σ 21 σ 22 σ 23  − 0 1 0 (5.19)
2µ  2 µ (2 µ + 3λ ) 
 ε13 ε 23 ε 33   σ13 σ 23 σ 33  0 0 1

University of Castilla-La Mancha Draft By: Eduardo W. V. Chaves (2016)


Ciudad Real – Spain
5 INTRODUCTION TO: CONSTITUTIVE EQUATIONS, IBVP, AND IBVP SOLUTION STRATEGIES 419

Notice also that the normal stress components σ11 , σ 22 , and σ33 only produce normal
strain components. Let us consider a particular case in which we only have the normal
stress σ11 , σ 22 = 0 , σ 33 = 0 , then:
 ε 11 ε 12 ε 13  σ11 0 0 1 0 0
ε  1   λ(σ11 ) 
 21 ε 22 ε 23  = 0 0 0 − 0 1 0
2µ  2 µ (2 µ + 3λ ) 
ε 13 ε 23 ε 33   0 0 0 0 0 1

with that the normal strain components are:


1 λ  1 λ   (µ + λ ) 
ε11 = σ11 − (σ11 )δ 11 =  − σ11 =  σ11 (5.20)
2µ 2 µ (2 µ + 3λ )  2 µ 2 µ (2 µ + 3λ )   µ (2 µ + 3λ ) 
−λ −λ
ε 22 = (σ11 )δ 22 = σ11 (5.21)
2 µ (2 µ + 3λ ) 2 µ (2 µ + 3λ )
−λ −λ
ε 33 = (σ11 )δ 33 = σ11 (5.22)
2 µ (2 µ + 3λ ) 2 µ (2 µ + 3λ )
From the equation for ε11 given by the equation in (5.20) we can obtain:
 (µ + λ )  µ (3λ + 2 µ )
ε11 =  σ11 ⇒ σ11 = ε11 ⇒ σ11 = Eε11
 µ (2 µ + 3λ )  (λ + µ )

µ (3λ + 2 µ )
where we have denoted by E = , which is known as Young’s modulus, or
(λ + µ )
longitudinal elastic modulus.
As expected, due to the material isotropy, the influence of σ11 upon ε 22 and ε 33 is the
same, and we can also obtain:
−λ −λ  µ (3λ + 2 µ )  −λ
ε 22 = σ11 =  ε11  = ε11 = −ν ε11
2 µ (2 µ + 3λ ) 2 µ (2 µ + 3λ )  (λ + µ )  2(λ + µ )
−λ −λ  µ (3λ + 2 µ )  −λ
ε 33 = σ11 =  ε11  = ε11 = −ν ε11
2 µ (2 µ + 3λ ) 2 µ (2 µ + 3λ )  (λ + µ )  2(λ + µ )
λ
where we have denoted by ν = , which is known as Poisson’s ratio. And the
2(λ + µ )
Poisson’s ratio can assume − 1.0 < ν < 0.5 , (see Problem 1.92). Note that
λ 2νµ
ν= ⇒λ = and if we replace it into the equation of E we can obtain:
2(λ + µ ) (1 − 2ν )

  2νµ     2ν    6ν 
3  + 2 µ  3  + 2 µ  + 2
µ (3λ + 2 µ )   (1 − 2ν )     (1 − 2ν )    (1 − 2ν ) 
E= =µ =µ =µ
(λ + µ )  2νµ    2ν    2ν 
  + µ    + 1 µ  + 1
 (1 − 2ν )    (1 − 2ν )    (1 − 2ν ) 
 6ν + 2(1 − 2ν ) 
 
 (1 − 2ν ) 
=µ = 2 µ (1 + ν )
 2ν + (1 − 2ν ) 
 
 (1 − 2ν ) 
thus:

University of Castilla- La Mancha Draft By: Eduardo W. V. Chaves (2016)


Ciudad Real – Spain
420 SOLVING PROBLEMS BY MEANS OF CONTINUUM MECHANICS

E 2νµ νE
G=µ= and λ= =
2(1 + ν ) (1 − 2ν ) (1 + ν )(1 − 2ν )
The physical interpretation of µ comes next, (see Figure 5.6). Let us suppose a stress state
in which is acting just the component σ12 , with that and according to the equation in (5.19)
we can obtain the only strain not equal to zero:
1
ε12 = ε 21 = σ12 ⇒ σ12 = µ 2ε12 ⇒ τ xy = µ γ xy = G γ xy ∴ G=µ
2µ {
τ xy
{
γ xy

We can also express the bulk modulus in function of ( E ,ν ) :


2µ νE 2 E 3νE + E (1 − 2ν ) E (1 + ν ) E
κ =λ + = + = = =
3 (1 + ν )(1 − 2ν ) 3 [2(1 + ν )] 3(1 + ν )(1 − 2ν ) 3(1 + ν )(1 − 2ν ) 3(1 − 2ν )
So, we can obtain the relationships between these mechanical properties:
G=µ= E= κ= λ= ν=
f (G; E ) GE G (E − 2G ) E − 2G
G E
9G − 3E 3G − E 2G
f (G; κ) 9Gκ 2G 3κ − 2G
G κ κ−
3κ + G 3 2(3κ + G )
f (G; λ) G (3λ + 2G ) 2 λ
G λ+ G λ
λ+G 3 2(λ + G )
f (G;ν ) 2G (1 + ν ) 2Gν
G 2G (1 + ν ) ν
3(1 − 2ν ) 1 − 2ν
f ( E; κ) 3κE κ(9 κ − 3E ) 3κ − E
E κ
9κ − E 9κ − E 6κ
f ( E;ν ) E E Eν
E ν
2(1 + ν ) 3(1 − 2ν ) (1 + ν )(1 − 2ν )
f ( κ; λ ) 3(κ − λ ) 9 κ(κ − λ ) λ
κ λ
2 3κ − λ 3κ − λ
f ( κ;ν ) 3κ(1 − 2ν ) 3κν
3κ(1 − 2ν ) κ ν
2(1 + ν ) 1+ν
We leave the reader to show:

Tensorial notation Indicial notation

σ = λTr (ε )1 + 2 µε σ ij = λε kkδ ij + 2 µε ij (5.23)

νE E νE E
σ= Tr (ε )1 + ε σij = ε kkδ ij + ε ij (5.24)
(1 + ν )(1 − 2ν ) (1 + ν ) (1 + ν )(1 − 2ν ) (1 + ν )

 2µ   2µ 
σ = κ −  Tr (ε )1 + 2 µε σ ij =  κ − ε kkδ ij + 2 µε ij (5.25)
 3   3 

and

University of Castilla-La Mancha Draft By: Eduardo W. V. Chaves (2016)


Ciudad Real – Spain
5 INTRODUCTION TO: CONSTITUTIVE EQUATIONS, IBVP, AND IBVP SOLUTION STRATEGIES 421

Tensorial notation Indicial notation

−λ 1 −λ 1
ε= Tr (σ )1 + σ εij = σ kkδ ij + σij (5.26)
2 µ (3λ + 2 µ ) 2µ 2 µ (3λ + 2 µ ) 2µ

−ν 1+ν −ν 1+ν
ε= Tr (σ )1 + σ ε ij = σ kkδ ij + σij (5.27)
E E E E
 2 µ − 3κ  1  2 µ − 3κ  1
ε =   Tr (σ )1 − σ ε ij =  σ kk δ ij − σ ij (5.28)
 18κµ  2µ  18κµ  2µ

and that the elasticity tensor for isotropic material can be written as follows:
C e = λ1 ⊗ 1 + 2 µI
νE E
Ce = 1 ⊗1 + I
(1 + ν )(1 − 2ν ) (1 + ν ) Elasticity tensor (5.29)
 1 
C e = κ1 ⊗ 1 + 2 µ I − 1 ⊗ 1
 3 

and:
−1 −λ 1
Ce ≡ De = 1 ⊗1 + I
2 µ (3λ + 2 µ ) 2µ
−1 −ν (1 + ν )
Ce ≡ De = 1 ⊗1 + I Elastic compliance tensor (5.30)
E E
−1 1 1  1 
Ce ≡ De = 1 ⊗1 + I − 1 ⊗ 1
9κ 2µ  3 

where I ≡ I sym is the symmetric unit fourth-order tensor. In the International System of
Units we have [G ] = [ µ ] = [λ] = [ κ] = [ E ] = Pa , and ν is a dimensionless quantity.

Problem 5.6
In tensile testing we have evaluated the following points:
Point σ( Pa ) ε(×10 −3 )
1 6.67 0.667
2 13.3 1.33
3 20 2
4 24 3
5 22 3 .6
Calculate Young’s modulus ( E ) and define the stress-strain curve limit points.
Solution: First, we verify that the first three points maintain the same proportionalities:
σ (1) σ ( 2 ) σ ( 3) 20
E= = ( 2 ) = ( 3) = = 10 000 Pa = 10 kPa
ε (1)
ε ε 2 × 10 −3
The stress-strain curve can be appreciated in Figure 5.7, in which we define the following
points: σ e - the proportionality point; σ Y - the yield point; σ u - the ultimate strength
point; and σ r - the rupture strength point.

University of Castilla- La Mancha Draft By: Eduardo W. V. Chaves (2016)


Ciudad Real – Spain
422 SOLVING PROBLEMS BY MEANS OF CONTINUUM MECHANICS

σ(Pa ) 30
σu
25 σY σr
3; 24
σe 3.6; 22
20 2; 20

15
1.33; 13.3
10
0.667; 6.67
5
E E
0 0; 0
0 0.5 1 1.5 2 2.5 3 3.5 4
0 . 2% −3
ε(×10 )

Figure 5.7: Stress-strain curve.

Problem 5.7
Show that the strain energy density, for an isotropic linear elastic material, can be written as
follows:
1
a) Ψ e (ε ) = (λ + 2 µ ) I ε2 − 2 µ II ε (5.31)
2
or
(λ + µ ) 1
b) Ψ e (σ ) = I σ2 − II σ (5.32)
2 µ (3λ + 2 µ ) 2µ

or
κ
Ψ e (ε ) = [Tr (ε )]2 + 1
µ 42
ε dev : ε dev
4 43 4
c) 2
14243 (5.33)
purely distortional
purely volumetric energy
energy

or
1 1
Ψ e (σ ) = I σ2 + J2
d) 6(3λ + 2 µ ) 2µ (5.34)
144244 3 123
purely volumetric purely distortional
energy energy

where I ε = Tr (ε ) is the trace of ε (infinitesimal strain tensor), I σ = Tr (σ ) is the first


invariant of the Cauchy stress tensor σ , and II σ dev = − J 2 is the second invariant of the
deviatoric Cauchy stress tensor. Note that, for linear elastic material the relationship
Ψ e (σ ) = Ψ e (ε) holds, (see Figure 5.4).

University of Castilla-La Mancha Draft By: Eduardo W. V. Chaves (2016)


Ciudad Real – Spain
5 INTRODUCTION TO: CONSTITUTIVE EQUATIONS, IBVP, AND IBVP SOLUTION STRATEGIES 423

Solution:
1
a) Taking into account the strain energy Ψ e = ε : σ and σ (ε ) = λTr (ε )1 + 2 µε , (see
2
equation (5.23)), we can obtain:
1 1
Ψ e = ε : σ = ε : [λTr (ε )1 + 2 µε ]
2 2
1 1 1 (5.35)
= λTr (ε ) 1 1 + µ ε : ε = λ[Tr (ε )] + µ ε : ε = λI ε2 + µ ε : ε
2
ε2:3
2 Tr (ε )
2 2

Taking into account the definition of the second invariant and the symmetry of ε we can
obtain:
1
II ε =
2
[ ] [
1
2
1
2
] [ 1
] [
[ Tr (ε )] 2 − Tr (ε 2 ) = I ε2 − Tr (ε ⋅ ε ) = I ε2 − Tr (ε ⋅ ε T ) = I ε2 − ε : ε
2
] (5.36)
2
⇒ ε : ε = I ε − 2 II ε
Then, the equation in (5.35) can be rewritten as follows:
1 1 1
Ψ e = λI ε2 + µ ε : ε = λI ε2 + µ ( I ε2 − 2 II ε ) = (λ + 2 µ ) I ε2 − 2 µ II ε
2 2 2

1
b) Taking into account the strain energy density Ψ e = ε : σ and
2
−λ 1
ε= Tr (σ )1 + σ , (see equation (5.26)), we can obtain:
2 µ (3λ + 2 µ ) 2µ

1 1 −λ 1 
Ψ e = ε :σ =  Tr (σ )1 + σ : σ
2 2  2 µ (3λ + 2 µ ) 2µ 
−λ 1 −λ
= Tr (σ ) 1
σ2 :31+ σ:σ = [Tr (σ )]2 + 1 σ : σ (5.37)
4 µ (3λ + 2 µ ) Tr ( σ )
4µ 4 µ (3λ + 2 µ ) 4µ
−λ 1
= I σ2 + σ:σ
4 µ (3λ + 2 µ ) 4µ

According to the equation (5.36) we can conclude that σ : σ = I σ2 − 2 II σ , with that the
above equation becomes:
−λ 1 −λ 1
Ψe= I σ2 + σ:σ = I σ2 + ( I σ2 − 2 II σ )
4 µ (3λ + 2 µ ) 4µ 4 µ (3λ + 2 µ ) 4µ
(λ + µ ) 1
= I σ2 − II σ
2 µ (3λ + 2 µ ) 2µ
1  2µ 
c) Taking into account the strain energy Ψ e = ε : σ and σ =  κ −  Tr (ε )1 + 2 µε , (see
2  3 
equation (5.25)), we can obtain:
1 1  2µ  
Ψ e = ε : σ = ε :  κ −  Tr (ε )1 + 2 µε 
2 2  3  
(5.38)
1 2µ  κ µ
1 + µ ε : ε =  − [Tr (ε )] + µ ε : ε
2
= κ−  Tr (ε ) 1
ε2
:3
2 3  Tr (ε ) 2 3

University of Castilla- La Mancha Draft By: Eduardo W. V. Chaves (2016)


Ciudad Real – Spain
424 SOLVING PROBLEMS BY MEANS OF CONTINUUM MECHANICS

If we consider that a second-order tensor can be split additively into a spherical and
Tr (ε )
deviatoric parts, i.e. ε = ε sph + ε dev = 1 + ε dev , the expression ε : ε can be written as:
3
 Tr (ε )   Tr (ε ) 
ε :ε =  1 + ε dev  :  1 + ε dev 
 3   3 
2
 Tr (ε )  Tr (ε ) Tr (ε ) dev
=  1 :1 + 1 : ε dev + ε : 1 + ε dev : ε dev (5.39)
 3  3 3

=
[Tr(ε)]2 + ε dev : ε dev
3
where we have applied that 1 : 1 = 3 , 1 : ε dev = ε dev : 1 = Tr (ε dev ) = 0 (the trace of any
deviatoric tensor is zero). With that the equation in (5.38) becomes:
κ µ κ µ  [Tr (ε )]2 
Ψ e =  − [Tr (ε )]2 + µ ε : ε =  − [Tr (ε )]2 + µ  + ε dev : ε dev 

2 3 2 3  3 
κ
= [Tr(ε)]2 + µ ε dev : ε dev
2
d) To show the equation (5.34) we will use the strain tensor defined in (5.26),
−λ 1
ε= Tr (σ )1 + σ . Then, the strain energy density can be written as:
2 µ (3λ + 2 µ ) 2µ

1 1 −λ 1 
Ψ e = ε :σ =  Tr (σ )1 + σ : σ
2 2  2 µ (3λ + 2 µ ) 2µ 
(5.40)
−λ 1 −λ
= Tr (σ ) 1
12:3 σ + σ :σ = [Tr (σ )]2 + 1 σ : σ
4 µ (3λ + 2 µ ) Tr ( σ )
4µ 4 µ (3λ + 2 µ ) 4µ

Note that σ : σ =
[Tr(σ )]2 + σ dev : σ dev holds, (see equation (5.39)). Taking into account
3
the equation of the second invariant of a second-order tensor we can obtain:
1 −1
[ Tr (σ dev )] 2 − Tr (σ dev ) =
2 2
II σ dev = Tr (σ dev )
2 
 2
−1 −1 − 1 dev
Tr (σ dev ⋅ σ dev ) = Tr (σ dev ⋅ σ dev ) =
T
= σ : σ dev
2 2 2
where we have used that: the trace of the deviatoric tensor is zero Tr (σ dev ) = 0 , the
symmetry of the tensor σ dev = σ dev , and trace property Tr ( A ⋅ B T ) = A : B . Then, we can
T

obtain:

σ :σ =
[Tr(σ )]2 + σ dev : σ dev = [Tr(σ )]2 − 2 II =
[Tr(σ )]2 + 2J
σ dev 2
3 3 3
By substituting the above equation into the equation in (5.40), we can obtain:

University of Castilla-La Mancha Draft By: Eduardo W. V. Chaves (2016)


Ciudad Real – Spain
5 INTRODUCTION TO: CONSTITUTIVE EQUATIONS, IBVP, AND IBVP SOLUTION STRATEGIES 425

−λ
Ψe= [Tr(σ )]2 + 1 σ : σ
4 µ (3λ + 2 µ ) 4µ
−λ 1  [Tr (σ )] 
2
= [ ]2
Tr (σ ) + + 2J 2 
4 µ (3λ + 2 µ ) 4 µ  3 

 −λ 1  1
=  + [Tr (σ )]2 + J2
 4 µ (3λ + 2 µ ) 12 µ  2µ
1
= [Tr(σ )]2 + 1 J 2
6(3λ + 2 µ ) 2µ

Problem 5.8
Write in Voigt notation: a.1) the strain energy density and, a.2) the constitutive equations in
stress for an isotropic linear elastic material: a.2.1) in terms of ( λ , µ ) and, a.2.2) in terms of
Eν E
( E , ν ) where λ = and µ = . b) Write the infinitesimal strain tensor
(1 + ν )(1 − 2ν ) 2(1 + ν )
ε in Voigt notation such as {ε } = [ L(1) ]{u } where {u } is the displacement field, obtain the
matrix [ L(1) ] .
c) Write the equations of motion in Voigt notation.
Solution:
a.1) The strain energy density (Ψ e (ε) -scalar) can be expressed as follows:
1 1 1 1
Ψ e (ε ) = ε : C e : ε = ε : σ = σ : ε = σ ij ε ij
2 2 2 2
where we have used σ = C e : ε . Note that
σ ij ε ij = σ1 j ε1 j + σ 2 j ε 2 j + σ 3 j ε 3 j
123 123 123
σ11ε11 σ 21ε 21 σ31ε31
+ + +
σ12ε12 σ22ε 22 σ32ε32
+ + +
σ13ε13 σ23ε 23 σ33ε33

thus
1 1
Ψ e (ε) = σij εij = (σ11ε11 + σ 22 ε 22 + σ 33ε 33 + 2σ12 ε12 + 2σ 23ε 23 + 2σ13ε13 )
2 2
and
 ε11 
ε 
 22 
1 1  ε 33  1
Ψ e (ε) = σij εij = [σ11 σ22 σ33 σ12 σ 23 σ13 ]   = {σ } {ε }
T
2 2  2 ε12  2
2ε 23 
 
 2ε13 
Then, the tensors σ and ε in Voigt notation are stored as follows:

University of Castilla- La Mancha Draft By: Eduardo W. V. Chaves (2016)


Ciudad Real – Spain
426 SOLVING PROBLEMS BY MEANS OF CONTINUUM MECHANICS

 σ11   ε11 
σ  ε 
 22   22 
 σ33   ε33 
{σ } =   ; {ε } =  
 σ12   2ε12 
σ23  2ε 23 
   
 σ13   2ε13 
a.2.1) The constitutive equation for stress in Voigt notation is:
 σ11  λ + 2 µ λ λ 0 0 0   ε11 
σ   λ λ + 2µ λ 0 0 0   ε 22 
 22  
 σ33   λ λ λ + 2µ 0 0 0   ε33 
σ = C e : ε Voigt
 →   =    ⇒ {σ } = [C ] {ε }
 σ12   0 0 0 µ 0 0   2ε12 
σ23   0 0 0 0 µ 0  2ε 23 
    
 σ13   0 0 0 0 0 µ   2ε13 
(5.41)
More detail about this formulation is provided in Problem 1.98 in Chapter 1 where we
have also obtained
1 λ
ε= σ− Tr (σ )1
2µ 2µ ( 2µ + 3λ )
and
 µ +λ −λ −λ 
 µ ( 2 µ + 3λ ) 0 0 0
2 µ ( 2 µ + 3λ ) 2 µ ( 2 µ + 3λ )
 
 −λ µ +λ −λ
ε11   0 0 0  σ11 

   2 µ ( 2 µ + 3λ ) µ ( 2 µ + 3λ ) 2 µ ( 2 µ + 3λ )
 σ 22 
ε 22   −λ −λ µ +λ
0 0 0   
ε 33   2 µ ( 2 µ + 3λ ) 2 µ ( 2 µ + 3λ ) µ ( 2 µ + 3λ )  σ 33 
 = 1  
2ε12   0 0 0 0 0  σ12  (5.42)
2ε 23   µ  σ 23 
  1  
2ε13   0 0 0 0 0  σ13 
 µ
 
 1
0 0 0 0 0
 µ 
{ε } = [ C ]−1 {σ }
a.2.2) Note that
Eν E E
λ + 2µ = +2 = (1 − ν )
(1 + ν )(1 − 2ν ) 2(1 + ν ) (1 + ν )(1 − 2ν )
E
λ= ν
(1 + ν )(1 − 2ν )
E E (1 − 2ν )
µ= =
2(1 + ν ) (1 + ν )(1 − 2ν ) 2
then, the equation (5.41) can be rewritten as follows:

University of Castilla-La Mancha Draft By: Eduardo W. V. Chaves (2016)


Ciudad Real – Spain
5 INTRODUCTION TO: CONSTITUTIVE EQUATIONS, IBVP, AND IBVP SOLUTION STRATEGIES 427

(1 − ν ) ν ν 0 0 0 
σ
 11   0   ε11 
σ   ν (1 − ν ) ν 0 0
 
 22   ν ν (1 − ν ) 0 0 0   ε 22 
σ 33  E  (1 − 2ν ) ε 
 =  0 0 0 0 0   33  (5.43)
 σ (1 + ν )(1 − 2ν )  2   2ε12 
(1 − 2ν )
12
σ 23   0 0 0 0 0  2ε 23 
   2  
 σ13   (1 − 2ν )   2ε13 
 0 0 0 0 0 
 2 
Note that
Eν E E
λ+µ = + =
(1 + ν )(1 − 2ν ) 2(1 + ν ) 2(1 + ν )(1 − 2ν )
E  E Eν  E2
µ (2 µ + 3λ) =  2 +3  =
2(1 + ν )  2(1 + ν ) (1 + ν )(1 − 2ν )  2(1 + ν )(1 − 2ν )
λ+µ E 2(1 + ν )(1 − 2ν ) 1
= =
µ (2 µ + 3λ) 2(1 + ν )(1 − 2ν ) E2 E
λ Eν (1 + ν )(1 − 2ν ) ν
= =
2 µ (2 µ + 3λ) (1 + ν )(1 − 2ν ) E2 E
1 2(1 + ν ) 1
= = 2(1 + ν )
µ E E
Then, the equation (5.42) becomes:
ε11   1 −ν −ν 0 0  σ11 
0
  − ν  
ε 22   1 −ν 0 0 0  σ 22 
ε 33  1  − ν −ν 1 0 0 0  σ 33 
 =    (5.44)
2ε12  E  0 0 0 2(1 + ν ) 0 0  σ12 
2ε 23   0 0 0 0 2(1 + ν ) 0  σ 23 
    
2ε13   0 0 0 0 0 2(1 + ν )  σ13 

b) According to the definition ε ij = 12 (u i , j + u j ,i ) we can obtain:

 ∂u1 1  ∂u1 ∂u 2  1  ∂u1 ∂u 3  


  +   + 
∂x1 2  ∂x 2 ∂x1  2  ∂x3 ∂x1  
 ε 11 ε12 ε 13  
 1  ∂u ∂u  ∂u 2 1  ∂u 2 ∂u 3  
ε ij = ε 21 ε 22 ε 23  =   1 + 2   + 
2 ∂x ∂x1 ∂x 2 2  ∂x3 ∂x 2  
ε 31 ε 32 ε 33    2 

1  ∂u ∂u  1  ∂u 2 ∂u 3  ∂u 3
  1 + 3 

 +  
 2  ∂x3 ∂x1  2  ∂x3 ∂x 2  ∂x3 

University of Castilla- La Mancha Draft By: Eduardo W. V. Chaves (2016)


Ciudad Real – Spain
428 SOLVING PROBLEMS BY MEANS OF CONTINUUM MECHANICS

 ∂u1   ∂ 
 ∂x   ∂x 0 0 
 1   1 
 ∂u 2   ∂
 ε11   0 0 
  
 ε   ∂x2  
∂x2
 22   ∂u3   0 ∂  u
0  1 
ε 
{ε } =  33  =  ∂u ∂x3∂u  =  ∂ ∂
∂x3   
u
 2 

r r
{ε ( x )} = [ L(1) ]{u ( x )}
 2ε12   1 + 2   0  u 
2ε 23   ∂x2 ∂x1   ∂x2 ∂x1  3

   ∂u ∂u  ∂ ∂ 
 2ε13   2 + 3   0
 ∂x2 
 ∂x3 ∂x2   ∂x3
 ∂u1 + ∂u3   ∂ ∂ 
0 
 ∂x 
 3 ∂x1   ∂x3 ∂x1 

NOTE: If we adopt the engineering notation, i.e. x1 = x , x 2 = y , x3 = z , u1 = u , u 2 = v ,


u 3 = w , ε11 = ε x , ε 22 = ε y , ε 33 = ε z , 2ε 12 = γ xy , 2ε 23 = γ yz , 2ε 13 = γ xz , the above equation
becomes:

 ∂u   ∂ 
 ∂x   ∂x 0 0
 ∂v   ∂ 
ε11   ε x    0 0
ε     ∂y   ∂y 
 22   ε y   ∂w   ∂ u 
ε   ε    0 0 
{ε } =  33  =  z  =  ∂u ∂z ∂v  =  ∂ ∂
∂z   v 
  ⇒
r r
{ε ( x )} = [ L(1) ]{u ( x )} (5.45)
2ε12  γ xy   +   0  w
2ε 23  γ yz   ∂y ∂x   ∂y ∂x  
     ∂v ∂w   ∂ ∂
2ε13   γ xz   + 0 
∂z ∂y   ∂z ∂y 
 
 ∂u + ∂w   ∂ 0
∂
 ∂z ∂x   ∂z ∂x 
r r r
c) Let us consider the equations of motion, ∇ ⋅ σ + ρ b = ρ v& = ρ u
&& , (see equation (5.14)), in
indicial notation σ ij , j + ρ b i = ρ u
&& i and its explicit form:

σ ij , j + ρb i = σ i1,1 + σ i 2, 2 + σ i 3,3 + ρb i = ρu
&& i
 ∂σ11 ∂σ12 ∂σ13
 + + + ρb1 = ρu&&1
σ11,1 + σ12, 2 + σ13,3 + ρb1 = ρu &&1  ∂x1 ∂x 2 ∂x 3
  ∂σ 21 ∂σ 22 ∂σ 23
⇒ σ 21,1 + σ 22, 2 + σ 23,3 + ρb 2 = ρu&& 2 ⇒  + + + ρb 2 = ρu&& 2
  ∂x1 ∂x 2 ∂x 3
σ 31,1 + σ 32, 2 + σ 33,3 + ρb 3 = ρu 3
&&
 ∂σ 31 ∂σ 32 ∂σ 33
 + + + ρb 3 = ρu&& 3
 ∂x1 ∂x 2 ∂x 3
Then, if we consider the stress tensor in Voigt notation, the above set of equations
becomes:

University of Castilla-La Mancha Draft By: Eduardo W. V. Chaves (2016)


Ciudad Real – Spain
5 INTRODUCTION TO: CONSTITUTIVE EQUATIONS, IBVP, AND IBVP SOLUTION STRATEGIES 429

σ
 ∂ ∂ ∂   11 
 0 0 0 
 ∂x1 ∂x2 ∂x3  σ22   ρb   ρu && 
 0 ∂ ∂ ∂  σ33   1   1 
 0 0    +  ρb 2  =  ρu
&&2 
∂x2 ∂x1 ∂x3 σ
      && 
∂  σ   ρb3   ρu3 
12
∂ ∂ (5.46)
 0 0 0 23
 ∂x3 ∂x2 ∂x1  σ 
 13 

⇒ [ L(1) ]T {σ } + {ρb } = {ρu&&}

Problem 5.9
Consider an isotropic homogeneous linear elastic material described in Problem 5.5.
Obtain the governing equation so as to result in a system of three equations and three
unknowns, namely: u1 , u 2 , u 3 , (Displacement Formulation – established by Navier (1827)).
Solution:
As we have seen in Problem 5.5, the governing equations, for an isotropic linear elastic
material in small deformation regime, are:
Tensorial notation Indicial notation
The equations of motion: The equations of motion:
r r &r& (3 equations)
∇ ⋅ σ + ρ b = ρ v& = ρ u σ ij , j + ρ b i = ρ u
&& i (3 equations)

The constitutive equations for stress: The constitutive equations for stress:
(5.47)
σ = λTr (ε )1 + 2 µε (6 equations) σ ij = λε kk δ ij + 2µ ε ij (6 equations)

The kinematic equations: The kinematic equations:


r
ε = ∇ sym u (6 equations) 1  ∂u i ∂u j 
 (6 equations)
ε ij = +
2  ∂x j ∂x i 

which results in a system with 15 equations and 15 unknowns (u i , σ ij , ε ij ) .
The divergence of the Cauchy stress tensor ( ∇ ⋅ σ ) can be obtained by means of the
constitutive equations for stress, i.e.:
σij = λε kkδ ij + 2 µ εij ⇒ σij , j = (λε kkδ ij + 2 µ εij ), j
⇒ σij , j = λ, j ε kkδ ij + λε kk , jδ ij + λε kk δ ij , j + 2 µ , j εij + 2 µ εij , j
{ { { (5.48)
=0 j = 0i =0 j

⇒ σij , j = λε kk , jδ ij + 2 µ εij , j ⇒ σij , j = λε kk ,i + 2 µ εij , j

Note that, if the mechanical properties λ and µ are constants throughout the medium, i.e.
r ∂λ
if they do not vary with x (homogeneous material) we can obtain λ , j ≡ = 0 j and
∂x j
∂µ
µ,j ≡ = 0 j . We can also express the terms ε kk ,i and ε ij, j in function of displacements.
∂x j
For this, we use the kinematic equations:

University of Castilla- La Mancha Draft By: Eduardo W. V. Chaves (2016)


Ciudad Real – Spain
430 SOLVING PROBLEMS BY MEANS OF CONTINUUM MECHANICS

1  ∂u i ∂u j  1
ε ij =
2  ∂x j
+
∂xi  2
( )
 ≡ u i , j + u j ,i divergence 1
 → ε ij , j = u i , jj + u j ,ij
2
( )

Note that
∂ 2ui ∂  ∂ui  r r r
= ≡ ui , jj ≡ [∇ ⋅ (∇u)]i ≡ [∇ 2u]i (Laplacian of the vector u )
∂x j ∂x j ∂x j  ∂x j 

∂ 2u j ∂ 2u j ∂  ∂u j  r
 ≡ u j , ji ≡ [∇ (∇ ⋅ u)]i
u j ,ij ≡ = =
∂x j ∂xi ∂xi ∂x j ∂xi  ∂x j 

1  ∂u k ∂u k  ∂u k
ε kk =  +  = gradient
≡ u k ,k   → ε kk ,i = u k ,ki = u j , ji
2  ∂x k ∂x k  ∂x k

With that the equation in (5.48) can be rewritten as:

σ ij , j = λε kk ,i + 2µ ε ij , j = λu j , ji + 2µ
1
2
( )
u i , jj + u j , ji = (λ + µ )u j , ji + µ u i , jj

By replacing the above equation into σ ij , j + ρ b i = ρ u


&& i (equations of motion), we obtain:

σ ij , j + ρ b i = ρ &u& i
⇒ (λ + µ )u j , ji + µ u i , jj + ρ b i = ρ u
&& i

Thus resulting in 3 equations and 3 unknowns ( u1 , u 2 , u 3 ):

(λ + µ )u j , ji + µu i , jj + ρb i = ρu
&&i
Navier’s equations (5.49)
r r r
&r&
(λ + µ )[∇ (∇ ⋅ u)] + µ [∇ ⋅ (∇u)] + ρb = ρu

where ∇ 2  ≡ ∇ ⋅ (∇) stands for the Laplacian of  .


NOTE 1: The above equations are known as the Navier’s equations also known as Navier-
Lamé equations. The explicit form of the equation (5.49) is presented as follows:
(λ + µ )u j , ji + µ u i , jj + ρ b i = (λ + µ )(u1,1i + u 2, 2i + u 3,3i ) + µ (u i ,11 + u i , 22 + u i ,33 ) + ρ b i = ρ u
&& i

(λ + µ )(u1,11 + u 2, 21 + u 3,31 ) + µ (u1,11 + u1, 22 + u1,33 ) + ρ b1 = ρ u &&1



(λ + µ )(u1,12 + u 2, 22 + u 3,32 ) + µ (u 2,11 + u 2, 22 + u 2,33 ) + ρ b 2 = ρ u&& 2

(λ + µ )(u1,13 + u 2, 23 + u 3,33 ) + µ (u 3,11 + u 3, 22 + u 3,33 ) + ρ b 3 = ρ u 3
&&

or:
 ∂  ∂u1 ∂u 2 ∂u 3   ∂ 2u ∂ 2 u1 ∂ 2 u1 
(λ + µ )  + +  + µ  21 + +  + ρ b1 = ρ u
&&1
∂x1  ∂x1 ∂x 2 ∂x3   ∂x ∂x 22 ∂x32 
  1

 ∂  ∂u1 ∂u 2 ∂u 3   ∂ 2u 2 ∂ 2u 2 ∂ 2u 2 
λ µ µ  
 ∂x 2 + ∂x 2 + ∂x 2  + ρ b 2 = ρ u 2
( + ) 
 ∂x + + 
 + &&
 ∂x 2  1 ∂ x 2 ∂ x 3   1 2 3 

(λ + µ ) ∂  ∂u1 + ∂u 2 + ∂u 3  + µ  ∂ u 3 + ∂ u 3 + ∂ u 3  + ρ b = ρ u
2 2 2
&& 3
 ∂x  ∂x ∂x ∂x   ∂x 2 ∂ x 2
∂x 2  3
 3  1 2 3   1 2 3 

University of Castilla-La Mancha Draft By: Eduardo W. V. Chaves (2016)


Ciudad Real – Spain
5 INTRODUCTION TO: CONSTITUTIVE EQUATIONS, IBVP, AND IBVP SOLUTION STRATEGIES 431

NOTE 2: The above set of equations in matrix form becomes [ A]{u} = {p} , where:
 ∂2 D2 ∂2 ∂2 
(λ + µ ) 2 + µ∇ − ρ 2 (λ + µ ) (λ + µ )
2

 ∂x1 Dt ∂x1∂x2 ∂x1∂x3 
 ∂ 2
∂2 D2 ∂2 
[A] =  (λ + µ ) (λ + µ ) + µ∇ 2
− ρ (λ + µ ) ,
 ∂x 2 ∂x1 ∂x22 Dt 2 ∂x2 ∂x3 
 ∂2 ∂2 ∂2 D2 
 (λ + µ ) (λ + µ ) (λ + µ ) 2 + µ∇ − ρ 2 
2

 ∂x3 ∂x1 ∂x3 ∂x2 ∂x3 Dt 

u1   ρb1 
   
{u} = u 2  , and {p} =  ρb 2  .
u   ρb 
 3  3
r r ∂ ∂ ∂2 ∂2 ∂2 ∂2 ∂2 ∂2
Note that ∇ 2 = (∇ ) ⋅ (∇ ) = = + + = 2 + 2 + 2 . The matrix
∂xk ∂xk ∂x1∂x1 ∂x2 ∂x2 ∂x3 ∂x3 ∂x1 ∂x2 ∂x3
[A] can also be written as follows:

 ∂2 D2 ∂2 ∂2 
(λ + µ ) 2 + µ∇ − ρ 2 (λ + µ ) (λ + µ )
2

 ∂x1 Dt ∂x1∂x2 ∂x1∂x3 
 ∂ 2
∂2 D2 ∂2 
[A] =  (λ + µ ) (λ + µ ) + µ∇ 2 − ρ 2 (λ + µ ) 
 ∂x2 ∂x1 2
∂x2 Dt ∂x2 ∂x3 
 ∂2 ∂2 ∂2 D2 
 (λ + µ ) (λ + µ ) (λ + µ ) 2 + µ∇ − ρ 2 
2

 ∂x3∂x1 ∂x3∂x2 ∂x3 Dt 


 ∂2 ∂2 ∂2 
 
 ∂x1
2
∂x1∂x2 ∂x1∂x3 
1 0 0
 ∂2 ∂2 ∂2   ∂2 D 2 
= (λ + µ )   
+µ − ρ 2  0 1 0

∂x ∂x ∂x22 ∂x2 ∂x3  ∂xk ∂xk Dt 
 22 1  0 0 1
 ∂ ∂2 2
∂ 
 ∂x ∂x ∂x3∂x2 ∂x32 
 3 1
Using the indicial and tensorial notations the above equation can be written as follows:
∂ ∂  ∂2 D2  r r  D2 
A ij = (λ + µ ) +  µ − ρ 2 δ ij and A = (λ + µ )[(∇ ) ⊗ (∇ )] +  µ∇ 2 − ρ 2  1
∂xi ∂x j  ∂xk ∂xk Dt   Dt 
Then, we can also express the Navier’s equation as follows:
 r r  D2   r r
 (λ + µ )[(∇ ) ⊗ ( ∇ )] +  µ∇ 2
− ρ  1  ⋅ u = − ρb
 Dt  
2
 
or
 ∂ ∂  ∂2 D2  
(λ + µ ) +  µ − ρ 2 δ ij  u j = − ρb i
 ∂xi ∂x j  ∂xk ∂xk Dt  

The above equation could have been easily obtained by means of the equation in (5.49),
i.e.:

University of Castilla- La Mancha Draft By: Eduardo W. V. Chaves (2016)


Ciudad Real – Spain
432 SOLVING PROBLEMS BY MEANS OF CONTINUUM MECHANICS

(λ + µ )u j , ji + µu i , jj + ρb i = ρ&u&i
⇒ (λ + µ )u k ,ki + µu k , jjδ ik + ρb i = ρu
&& kδ ik
⇒ (λ + µ )u k ,ki + µu k , jjδ ik − ρu
&& kδ ik = − ρb i
⇒ (λ + µ )u k ,ki + ( µu k , jj − ρu
&& k )δ ik = − ρb i

∂u k  ∂ 2u k  D 2u k
⇒ (λ + µ ) +  µ δ ik = − ρb i
−ρ

∂xk ∂xi  ∂xk ∂xk  Dt 2
 ∂  ∂ 2
D  
2
⇒ (λ + µ ) +  µ − ρ 2 δ ik u k = − ρb i
 ∂xk ∂xi  ∂xk ∂xk Dt  

NOTE 3: We have shown in Problem 1.106 (Chapter 1) that the following is true:
r r r r r
∇ ∧ (∇ ∧ a) = ∇ (∇ ⋅ a) − ∇ 2 a indicial
 →  ilq  qjk a k , jl = a j , ji − a i , jj
Then, we can obtain
r r r r r r
∇ ⋅ (∇u) ≡ ∇ 2 u = ∇ (∇ ⋅ u) − ∇ ∧ (∇ ∧ u) indicial
 → u i , jj = u j , ji −  ilq  qjk u k , jl
with which the equation (5.49) can also be written as follows:
(λ + µ )u j , ji + µui , jj + ρbi = ρu
&&i
⇒ (λ + µ )u j , ji + µ (u j , ji −  ilq  qjk u k , jl ) + ρbi = ρu
&&i
⇒ (λ + 2 µ )u j , ji − µilq  qjk uk , jl + ρbi = ρu
&&i
and the equivalent in tensorial notation:
r r r
&r&
(λ + µ )[∇ (∇ ⋅ u)] + µ [∇ ⋅ (∇u)] + ρb = ρu
[ ]
r r r r r r
&r&
⇒ (λ + µ )[∇ (∇ ⋅ u)] + µ ∇ (∇ ⋅ u) − ∇ ∧ (∇ ∧ u) + ρb = ρu
[ ]
r r r r r
⇒ (λ + 2 µ )[∇ (∇ ⋅ u)] − µ ∇ ∧ (∇ ∧ u) + ρb = ρu &r& (5.50)
[ ]
r r r r r
⇒ (λ + 2 µ )[∇ (∇ ⋅ u)] − µ ∇ ∧ (∇ ∧ u) + ρb = ρu &r&
⇒ (λ + 2 µ )u j , ji − µ ilq  qjk u k , jl + ρb i = ρu
&& i
In the Cartesian System we have:
r
u = u i eˆ i = u1 eˆ 1 + u 2 eˆ 2 + u 3 eˆ 3
r r r r  ∂u ∂u   ∂u ∂u   ∂u ∂u 
(∇ ∧ u) ≡ rot (u) = (rot (u) )i eˆ i =  3 − 2 eˆ 1 +  1 − 3 eˆ 2 +  2 − 1 eˆ 3
x 2 ∂x3 
1∂442r443
x3 ∂x1 
1∂442r443
x1 ∂x 2 
1∂442r443
= (rot (u) )1 = (rot (u) )2 = (rot (u) )3
r r r r r r
r r r  ∂ (rot (u) )3 ∂ (rot (u) )2   ∂ (rot (u) )1 ∂ (rot (u) )3   ∂ (rot (u) )2 ∂ (rot (u) )1 
∇ ∧ (∇ ∧ u) =  − eˆ 1 + 
  ∂x − eˆ 2 + 
 − eˆ 3
 ∂x 2 ∂x 3   3 ∂x1   ∂x1 ∂x 2 

r r
 ∂ (rot (u) )3 ∂ (rot (u) )2   ∂  ∂u 2 − ∂u1  − ∂  ∂u1 − ∂u 3 
 −   ∂x  ∂x   
 ∂x 2r ∂x3   2  1 ∂x 2  ∂x3  ∂x3 ∂x1 
r  
 ∂ (rot (u) ) ∂ (rot (u) )3   ∂  ∂u 3 ∂u 2  ∂  ∂u 2 ∂u1 
[r r r
∇ ∧ (∇ ∧ u) i = ] 1
− = 
 − 
− 
 − 

 ∂x 3 ∂x1   ∂x 3  ∂x 2 ∂x 3  ∂x1  ∂x1 ∂x 2 
r
 ∂ (rot (u r
) )2 ∂ (rot (u) )1   ∂  ∂u1 ∂u 3  ∂  ∂u 3 ∂u 2 

 −    −  −  − 
 ∂x1 ∂x 2   ∂x1  ∂x3 ∂x1  ∂x 2  ∂x 2 ∂x3 
 

University of Castilla-La Mancha Draft By: Eduardo W. V. Chaves (2016)


Ciudad Real – Spain
5 INTRODUCTION TO: CONSTITUTIVE EQUATIONS, IBVP, AND IBVP SOLUTION STRATEGIES 433

NOTE 4: If we are dealing with heterogeneous material, the equations in (5.48) must be
treated as follows:
σ ij = λε kk δ ij + 2µ ε ij
⇒ σ ij , j = (λε kk δ ij + 2µ ε ij ) , j
⇒ σ ij , j = (λε kk ) , j δ ij + (2µ ε ij ) , j = (λε kk ) ,i + ( 2µ ε ij ) , j
Taking into account that 2ε ij = u i , j + u j ,i and ε kk = u k ,k , the above equation becomes:
σ ij , j = (λε kk ) ,i + (2µ ε ij ) , j
[
⇒ σ ij , j = (λu k , k ) ,i + µ (u i , j + u j ,i ) , j ]
whereby
σ ij , j + ρ b i = ρ &u& i ⇒ [ ]
(λu k ,k ) ,i + µ (u i , j + u j ,i ) , j + ρ b i = ρ &u& i (5.51)
Note that
r r
u k , k = Tr (∇u) = (∇ ⋅ u) , and

&&i ≡ Dui = ∂ui + ∂ui v j = ∂ui + ∂ui v1 + ∂ui v2 + ∂ui v3 , and its components
& & & & & & &
u
Dt ∂t ∂x j ∂t ∂x1 ∂x2 ∂x3
 ∂u& 1 ∂u& 1 ∂u& ∂u& 
 + v1 + 1 v 2 + 1 v3 
 ∂t ∂x1 ∂x 2 ∂x3 
 ∂u& 2 ∂u& 2 ∂u 2
& ∂u& 2 
&& i = 
u + v1 + v2 + v3 
 ∂t ∂x1 ∂x 2 ∂x 3 
 ∂u& ∂u& ∂u& ∂u& 
 3 + 3 v1 + 3 v 2 + 3 v3 
 ∂t ∂x1 ∂x 2 ∂x3 

[ ]
µ (u i , j + u j ,i ) , j =

∂x j
[µ (u i , j + u j ,i ) ]
∂ ∂ ∂
=
∂x1
[
µ (u i ,1 + u1,i ) + ]
∂x 2
[
µ (u i , 2 + u 2,i ) +
∂x3
]
µ (u i ,3 + u 3,i ) [ ]
 ∂ ∂ ∂ 
 [
2µ (u1,1 ) + ] [
µ (u1, 2 + u 2,1 ) + ]
µ (u1,3 + u 3,1 )  [ ]
 ∂x1 ∂x 2 ∂x3 
 ∂ 
[ ]
µ (u i , j + u j ,i ) , j =  [
µ (u 2,1 + u1, 2 ) +

]2µ (u 2, 2 ) + [ ∂
]
µ (u 2,3 + u 3, 2 )  [ ]
 ∂x1 ∂x 2 ∂x3 
 ∂ ∂ ∂ 
 [
µ (u 3,1 + u1,3 ) + ] [
µ (u 3, 2 + u 2,3 ) + 2µ (u 3,3 )  ] [ ]
 ∂x1 ∂x 2 ∂x3 
The three equations in (5.51), ( i = 1,2,3 ), are explicitly given by:
 ∂ ∂ ∂ ∂
∂x
r
 [λ(∇ ⋅ u)] +
∂x
[
2 µ (u1,1 ) +
∂x
] [
µ (u1, 2 + u2,1 ) +
∂x
]
µ (u1,3 + u3,1 ) + ρb1 = ρu&&1 [ ]
 1 1 2 3
 ∂
 [λ(∇ ⋅ ur )]+ ∂ µ (u2,1 + u1,2 ) + ∂ 2 µ (u2,2 ) + ∂ µ (u2,3 + u3,2 ) + ρb 2 = ρu&&2
[ ] [ ] [ ]
 ∂x2 ∂x1 ∂x2 ∂x3
 ∂
 [λ(∇ ⋅ ur )] + ∂ µ (u3,1 + u1,3 ) + ∂ µ (u3,2 + u2,3 ) + ∂ 2µ (u3,3 ) + ρb3 = ρu&&3
[ ] [ ] [ ]
 ∂x3 ∂x1 ∂x2 ∂x3
or

University of Castilla- La Mancha Draft By: Eduardo W. V. Chaves (2016)


Ciudad Real – Spain
434 SOLVING PROBLEMS BY MEANS OF CONTINUUM MECHANICS

 ∂ ∂ ∂

∂x
[ r
]
λ(∇ ⋅ u) + 2 µ (u1,1 ) +
∂ x
[
µ (u1, 2 + u2,1 ) +
∂x
] [
µ (u1,3 + u3,1 ) + ρb1 = ρu&&1 ]
 1 2 3
 ∂ ∂ ∂

∂x
[ r
λ(∇ ⋅ u) + 2 µ (u2, 2 ) +

]x
[
µ (u2,1 + u1, 2 ) +
∂ x
] [
µ (u2,3 + u3, 2 ) + ρb 2 = ρu&&2 ] (5.52)
 2 1 3
 ∂ ∂ ∂

∂x
[ r
λ(∇ ⋅ u) + 2 µ (u3,3 ) +] ∂ x
[
µ (u3,1 + u1,3 ) +
∂ x
] [
µ (u3, 2 + u2,3 ) + ρb3 = ρu&&3]
 3 1 2

NOTE 5: Wave equations


If we apply the divergence to the equation (5.50) we can obtain:
r r r r r
&r&
(λ + 2 µ )∇ ⋅ [∇ (∇ ⋅ u)] − µ ∇ ⋅ [∇ ∧ (∇ ∧ u)] + ρ∇ ⋅ b = ρ∇ ⋅ u
1442443
=0
which in indicial notation becomes
(λ + 2 µ )u j , jii − µ ilq  qjk u k , jli + ρbi ,i = ρu
&&i ,i
then, we can obtain

Tensorial notation Indicial notation


r r
&r&
⇒ (λ + 2 µ )∇ ⋅ [∇ (∇ ⋅ u)] + ρ∇ ⋅ b = ρ∇ ⋅ u (λ + 2 µ )u j , jii − µ ilq  qjk u k , jli + ρb i ,i = ρu
&&i ,i
r r
&r&
⇒ (λ + 2 µ )∇ 2 (∇ ⋅ u) + ρ∇ ⋅ b = ρ∇ ⋅ u ⇒ (λ + 2 µ )u j , jii + ρb i ,i = ρu
&&i ,i

&r& = (λ + 2 µ ) ∇ 2 (∇ ⋅ u
r r (λ + 2 µ )
⇒ ∇ ⋅u ) + ∇ ⋅b ⇒u
&&i ,i = u j , jii + b i ,i
ρ ρ
D2 r (λ + 2 µ ) 2 r r D 2  ∂ui  (λ + 2 µ ) ∂ 2  ∂u j  ∂b i
⇒ (∇ ⋅ u) = ∇ (∇ ⋅ u ) + ∇ ⋅ b ⇒  = +
Dt 2 ρ Dt 2  ∂xi  ρ ∂xi ∂xi  ∂x j  ∂xi
D 2θ (λ + 2 µ ) 2 r D 2θ 2 ∂ θ ∂b
2
⇒ = ∇ θ + ∇ ⋅ b ⇒ = α + i
Dt 2
ρ Dt 2
∂xi ∂xi ∂xi
Dθ2 r
⇒ 2
= α 2∇ 2θ + ∇ ⋅ b
Dt
r
⇒ θ&& = α 2∇ 2θ + ∇ ⋅ b
(5.53)
r r r
where we have considered that θ = ∇ ⋅ u and ∇ ⋅ (∇ ∧ v ) = 0 , (see Problem 1.108), and
(λ + 2 µ )
α= P-wave velocity (5.54)
ρ
r
If the body forces do not change in space we have that ∇ ⋅ b = 0 , thus the equation in
(5.53) becomes:
D 2θ
= α 2∇ 2θ P- wave equation (5.55)
Dt 2
P-waves have no rotation.
r
Now if we apply the curl ( ∇ ∧ ) to the equation (5.50) we obtain:

University of Castilla-La Mancha Draft By: Eduardo W. V. Chaves (2016)


Ciudad Real – Spain
5 INTRODUCTION TO: CONSTITUTIVE EQUATIONS, IBVP, AND IBVP SOLUTION STRATEGIES 435

r r r r r r
[ r r
]
r &r&
(λ + 2 µ )∇ ∧ [∇ (∇ ⋅ u)] − µ∇ ∧ ∇ ∧ (∇ ∧ u) + ρ (∇ ∧ b) = ρ (∇ ∧ u )
r r
[ r r
] r r
⇒ − µ∇ ∧ ∇ ∧ (∇ ∧ u) + ρ (∇ ∧ b) = ρ (∇ ∧ u)
r &r&

r r
[ r r
] r r D2 r r
⇒ − µ∇ ∧ ∇ ∧ (∇ ∧ u) + ρ (∇ ∧ b) = ρ 2 (∇ ∧ u)
Dt
r r r r r 2 r

⇒ − µ∇ ∧ (∇ ∧ ϕ ) + ρ (∇ ∧ b) = ρ
Dt 2 (5.56)
r r r r r r r r
D 2ϕ D 2ϕ
⇒ − µ∇ ∧ (∇ ∧ ϕ ) = ρ ⇒ ρ 2 = − µ∇ ∧ (∇ ∧ ϕ )
Dt 2 Dt
2r
Dϕ µr r r
⇒ = − ∇ ∧ (∇ ∧ ϕ )
Dt 2
ρ
r r r r
D 2ϕ
⇒ = − β 2
∇ ∧ (∇ ∧ϕ)
Dt 2
r r r r r r r
where we have considered ϕ = ∇ ∧ u , and that the b -field is conservative thus ∇ ∧ b = 0 .
r r r r
Note that ∇ ∧ [∇ (∇ ⋅ u)] = ∇ ∧ [∇φ ] = 0 , (see Problem 1.108), and
µ
β= Shear wave velocity (5.57)
ρ
r r r r r r r r r r r r
Note that ∇ 2ϕ = ∇ (∇ ⋅ ϕ ) − ∇ ∧ (∇ ∧ ϕ ) ⇒ ∇ 2ϕ = −∇ ∧ (∇ ∧ ϕ ) , since ∇ ⋅ ϕ = ∇ ⋅ (∇ ∧ u) = 0
r r r r
holds for any vector, so ∇ (∇ ⋅ ϕ ) = ∇ (∇ ⋅ (∇ ∧ u)) = 0 , (see Problem 1.108). With that the
equation in (5.56) becomes:
r
D 2ϕ r Shear wave equation
= β 2∇ 2ϕ (5.58)
Dt 2 (S-wave equation)
Shear waves have no change in volume.
In the case when µ = 0 the equation in (5.55) becomes the acoustic wave equations:
D 2θ
2
= c 2∇ 2θ Acoustic wave equation (5.59)
Dt
with
λ
c= Speed of propagation (5.60)
ρ
r r r r
Note that the displacement field was split up into: u = ∇θ + ∇ ∧ ϕ where ∇ ⋅ϕ = 0 . We can
r r r r r
prove this by means of the identity ∇ ∧ (∇ ∧ a) = ∇ (∇ ⋅ a) − ∇ 2a . If we consider the vectors
r r r r r r r r r
u = ∇ 2a and ϕ = −∇ ∧ a , and the scalar θ = ∇ ⋅ a , we obtain u = ∇θ + ∇ ∧ ϕ , with that we
can obtain:
r r r r r r r r r r r r
∇ ⋅ u = ∇ ⋅ (∇θ ) + ∇ ⋅ (∇ ∧ ϕ ) = ∇ ⋅ (∇θ ) and ∇ ∧ u = ∇ ∧ (∇θ ) + ∇ ∧ (∇ ∧ ϕ ) = ∇ ∧ (∇ ∧ ϕ )

University of Castilla- La Mancha Draft By: Eduardo W. V. Chaves (2016)


Ciudad Real – Spain
436 SOLVING PROBLEMS BY MEANS OF CONTINUUM MECHANICS

a)

b)

Figure 5.8: Displacement occurring from a harmonic plane P-wave (a) and S-wave (b). P-
wave has no rotation and S-wave no volume change.
Eν E
NOTE 5.1: If we consider λ = and µ = we can obtain:
(1 + ν )(1 − 2ν ) 2(1 + ν )
(λ + 2 µ ) Eν E
+2
α ρ (λ + 2 µ ) (1 + ν )(1 − 2ν ) 2(1 + ν ) (2 − 2ν )
= = = =
β µ µ E (1 − 2ν )
ρ 2(1 + ν )

(2 − 2ν )
⇒α = β
(1 − 2ν )
14243
>1
With that we can conclude that the ratio of P- to S-wave velocities depends only on
Poisson’s ratio. Note that P-wave travels faster than S-wave, since for isotropic material
(2 − 2ν )
− 1 < ν < 0.5 , then > 1 , (see Figure 5.9).
(1 − 2ν )

(2 − 2ν )
(1 − 2ν )
zone not feasible

− 1 < ν < 0.5

Figure 5.9

University of Castilla-La Mancha Draft By: Eduardo W. V. Chaves (2016)


Ciudad Real – Spain
5 INTRODUCTION TO: CONSTITUTIVE EQUATIONS, IBVP, AND IBVP SOLUTION STRATEGIES 437

r
NOTE 6: In the previous note (NOTE 5) it was shown that the displacement field u can
r r r
be split up into u = ∇θ + ∇ ∧ ϕ , which is applied to any vector field, i.e. given a vector field
r
F , the following is true:
r r r Helmholtz theorem or Helmholtz
F = ∇θ + ∇ ∧ ϕ (5.61)
decomposition
r
which is known as Helmholtz theorem, where θ is a scalar potential field and ϕ is a vector
r r r
potential field, in which the relationships ∇ ⋅ϕ ≡ div (ϕ ) = 0 and ∇ ∧ (∇θ ) ≡ rot (∇θ ) = 0
r r
hold. Note also that the SI units of [θ ] = [ϕ ] = m 2 , since [u] = m(meter) .
r r r
Then, by substituting u = ∇θ + ∇ ∧ ϕ , ( ui = θ ,i + ipqϕ q, p ), into the Navier’s equations given
by (5.49) we can obtain:
r r r
&r&
(λ + µ )[∇ (∇ ⋅ u)] + µ [∇ ⋅ (∇u)] + ρb = ρu
[ ] [ ]
r r r r r
&r&
⇒ (λ + µ ) ∇ (∇ ⋅ (∇θ + ∇ ∧ ϕ )) + µ ∇ ⋅ (∇ (∇θ + ∇ ∧ ϕ )) + ρb = ρu
 r r   r r  r
&r&
⇒ (λ + µ ) ∇ (∇ ⋅ (∇θ )) + ∇ (∇ ⋅ (∇ ∧ ϕ )) + µ ∇ ⋅ (∇ (∇θ )) + ∇ ⋅ (∇ (∇ ∧ ϕ )) + ρb = ρu
 14243   14243 144 244 3
=0   = ∇ (∇ ⋅(∇θ ))
r r
∇ ∧ ( ∇ 2ϕ )  (5.62)
 
[ ]
r r r r r
⇒ (λ + 2 µ ) ∇ (∇ ⋅ (∇θ )) + µ ∇ ∧ (∇ 2ϕ ) + ρb = ρ∇θ&& + ρ∇ ∧ ϕ&&
 1 424 3 
 = ∇ 2θ 
[ ] [ ]
r r r r r
⇒ (λ + 2 µ ) ∇ (∇ θ ) + µ ∇ ∧ (∇ 2ϕ ) + ρb = ρ (∇θ&&) + ρ (∇ ∧ ϕ&& )
2

The above algebraic manipulations in indicial notation are given by:


(λ + µ )u j , ji + µui , jj + ρbi = ρu
&&i
⇒ (λ + µ )(θ , j +  jpqϕ q , p ), ji + µ (θ ,i + ipqϕ q , p ), jj + ρbi = ρu
&&i (5.63)
⇒ (λ + µ )(θ , jji +  jpqϕ q , pji ) + µ (θ ,ijj + ipqϕ q , pjj ) + ρbi = ρu
&&i
Note that  jpqϕ q , pji =  jpqϕ q ,ijp =  jpqϕ q , ijp = − pjqϕ q , ijp = 0i , since  jpq = − pjq is
antisymmetric in jp and ϕ q,ijp is symmetric in jp . Note also that
r r r r
ipqϕ q , pjj = ipqϕ q , jjp = ipq (ϕ q, jj ), p =  ipq ([∇ 2ϕ ]q ), p =  ipq ([∇ 2ϕ ]q ), p = [∇ ∧ (∇ 2ϕ )]i
and θ , jji = θ ,ijj . With the above considerations the equation (5.63) becomes:
(λ + µ )(θ , jji +  jpqϕ q , pji ) + µ (θ ,ijj + ipqϕ q , pjj ) + ρbi = ρθ&&, i + ρipqϕ
&& q , p
⇒ (λ + 2 µ )(θ , jji ) + µ (ipqϕ q , pjj ) + ρbi = ρθ&&, i + ρipqϕ
&& q , p
NOTE 7: Galerkin Vector
The displacement field can be expressed as follows:
r r r
u = c(∇ 2g) − ∇ (∇ ⋅ g) ui = cgi , pp − g p , pi (5.64)
r r
where g is the Galerkin vector with the SI unit [g] = m3 , and c is a constant to be
determined which is dimensionless. If the Galerkin vector for a problem is known the
problem is solved.
Let us consider a static linear elastic problem, then the Navier’s equations (5.49) can be
expressed as follows (λ + µ )u j , ji + µui , jj + ρbi = ρu
&&i = 0 i , and taking into account that
displacement field (5.64) we can obtain:

University of Castilla- La Mancha Draft By: Eduardo W. V. Chaves (2016)


Ciudad Real – Spain
438 SOLVING PROBLEMS BY MEANS OF CONTINUUM MECHANICS

(λ + µ )u j , ji + µui , jj + ρbi = 0 i
⇒ (λ + µ )(cg j , pp − g p , pj ), ji + µ (cgi , pp − g p , pi ), jj + ρbi = 0 i (5.65)
⇒ (λ + µ )(cg j , ppji − g p , pjji ) + µ (cgi , ppjj − g p , pijj ) + ρbi = 0 i

Note that g j , ppji = g p , pjji = g p, pijj , then the above equation becomes:

(λ + µ )(cg j , ppji − g p , pjji ) + µ (cgi , ppjj − g p , pijj ) + ρbi = 0 i


(5.66)
⇒ [(λ + µ )(c − 1) − µ ]g p , pjji + cµgi , ppjj + ρbi = 0 i

The constant c can be obtained by taking the term between brackets equal to zero, i.e.:
λ + 2µ
[(λ + µ )(c − 1) − µ ] = 0 ⇒ c= = 2(1 − ν ) (5.67)
λ+µ
Then, the displacement field (5.64) becomes:
r λ + 2µ 2 r r λ + 2µ
u= (∇ g) − ∇ (∇ ⋅ g) ui = g −g
λ+ µ λ + µ i , pp p , pi
 λ + 2µ r 1 r  λ + 2 µ ∂ 2 (gi ) ∂ (g p , p )
= 2 µ  (∇ 2 g) − ∇ (∇ ⋅ g)  = −
 2 µ (λ + µ ) 2µ  λ + µ ∂x p ∂x p ∂xi
r (5.68)
λ + 2µ 2 ∂(∇ ⋅ g)
= ∇ (gi ) −
λ+µ ∂xi
r
 λ + 2µ 1 ∂ (∇ ⋅ g) 
= 2 µ  ∇ 2 (gi ) − 
 2 µ (λ + µ ) 2 µ ∂xi 
And the Navier’s equation (5.66) in terms of Galerkin vector becomes
−1
[(λ + µ )(c − 1) − µ ]g p , pjji + cµgi , ppjj + ρbi = 0 i ⇒ gi , ppjj = ρb i
144 42444 3 cµ
=0

− (λ + µ )
⇒ gi , ppjj = ρb
µ (λ + 2 µ ) i
 ∂ 2 (gi ) 

∂2   = ∇ 2∇ 2 (gi ) = ∇ 4 (gi ) = − (λ + µ ) ρbi
∂x p ∂x p  ∂x j ∂x j  µ (λ + 2 µ )
 
Thus
r − (λ + µ ) r −1 r − (λ + µ ) −1
∇ 4 (g) = ρb = ρb ∇ 4 (gi ) = ρb i = ρbi (5.69)
µ (λ + 2 µ ) 2 µ (1 − ν ) µ (λ + 2 µ ) 2 µ (1 − ν )

Note that in the absence of body force, each component of the Galerkin vector ( g i ) is
biharmonic function, i.e. ∇ 4 (g i ) ≡ g i ,kkjj = 0 i .
r
The infinitesimal strain tensor ( ε = (∇u) sym ) in terms of Galerkin vector becomes:
r  λ + 2µ 2 r r   λ + 2µ r r 
∇u = ∇  (∇ g) − ∇ (∇ ⋅ g)  =  ∇ (∇ 2g) − ∇[∇ (∇ ⋅ g)] 
 λ+µ   λ+µ 
 λ + 2µ 2 r r 
=  ∇ (∇g) − ∇[∇ (∇ ⋅ g)] 
 λ+µ 
r r r r
Note that [∇ 2 (∇g)]sym = ∇ 2 [(∇g) sym ] , {∇[∇ (∇ ⋅ g)]}sym = ∇[∇ (∇ ⋅ g)] . Then,

University of Castilla-La Mancha Draft By: Eduardo W. V. Chaves (2016)


Ciudad Real – Spain
5 INTRODUCTION TO: CONSTITUTIVE EQUATIONS, IBVP, AND IBVP SOLUTION STRATEGIES 439

sym
r  λ + 2µ 2 r r  λ + 2 µ 2 r sym r
ε = (∇u) sym =  ∇ (∇g) − ∇[∇ (∇ ⋅ g)]  = ∇ [(∇g) ] − ∇[∇ (∇ ⋅ g)]
 λ+µ  λ+µ

 λ + 2µ r 1 r 
ε = 2 µ  ∇ 2 [(∇g) sym ] − ∇[∇ (∇ ⋅ g)] 
 2 µ (λ + µ ) 2µ 
or (5.70)
 (1 − ν ) 2 r 1 r 
ε = 2 µ  ∇ [(∇g) sym ] − ∇[∇ (∇ ⋅ g)] 
 µ 2µ 
In indicial notation we have:

1 1  λ + 2 µ   λ + 2µ  
ε ij = (ui , j + u j , i ) =  gi , pp − g p , pi  +  g j , pp − g p , pj  
2 2  λ + µ , j  λ + µ ,i 

1  λ + 2 µ   λ + 2µ 
=  gi , ppj − g p , pij  +  g j , ppi − g p , pji  
2  λ + µ   λ+µ 
λ + 2µ λ + 2µ
=
2(λ + µ )
[(gi , jpp + g j , ipp )] − (g p , pji + g p , pij ) =
1
2 2(λ + µ )
[(gi, j + g j,i )], pp − g p, pij
λ + 2µ
=
2(λ + µ )
[gi, j + g j ,i ], pp − (g p, p ),ij
The stress tensor field for isotropic linear elastic material ( σ = λTr(ε)1 + 2 µε ) in terms of
Galerkin vector becomes:
λ + 2µ
Tr (ε ) = ε kk = [(gk , k + gk , k )], pp − (g p, p ), kk = λ + 2µ (gk , kpp + gk , kpp ) − g p, pkk
2(λ + µ ) 2(λ + µ )
 λ + 2µ  µ
=  − 1gk , kpp = g
 (λ + µ )  λ + µ k , kpp
Note that gk , kpp = g p , pkk . Then,

 µ   λ + 2µ 
σij = λTr(ε )δ ij + 2 µεij = λ gk , kpp δ ij + 2µ  gi , j + g j ,i [ ] − (g p, p ),ij 
λ + µ  2(λ + µ )
, pp
 
 λ λ + 2µ 
= 2µ  gk , kppδ ij + [gi , j + gi , j ], pp − (g p, p ),ij  (5.71)
 2(λ + µ ) 2(λ + µ ) 

{ [
= 2µ ν gk , kppδ ij + (1 − ν ) gi , j + g j , i ], pp
− (g p , p ), ij }
The above equation in tensorial notation becomes:
 λ
σ = 2µ 
r
[∇ 2 (∇ ⋅ g)] 1 +
λ + 2 µ 2 r sym
[
r 
∇ (∇g) − ∇ (∇ (∇ ⋅ g)) ]
 2(λ + µ ) (λ + µ ) 
(5.72)
or
{ r r
[ r
σ = 2 µ ν [∇ 2 (∇ ⋅ g)] 1 + 2(1 − ν )∇ 2 (∇g) sym − ∇ (∇ (∇ ⋅ g)) ] }
where we have considered that

University of Castilla- La Mancha Draft By: Eduardo W. V. Chaves (2016)


Ciudad Real – Spain
440 SOLVING PROBLEMS BY MEANS OF CONTINUUM MECHANICS

λ Eν 2(1 + ν )(1 − 2ν )
= =ν
2(λ + µ ) 2(1 + ν )(1 − 2ν ) E
λ + 2 µ 1 2(1 + ν )(1 − 2ν ) E (1 − ν )
= = (1 − ν )
2(λ + µ ) 2 E (1 + ν )(1 − 2ν )

NOTE 8: Love’s Strain Function


The Love’s strain function is a particular case of Galerkin vector in which
g1 = 0 ; g2 = 0 ; g3 = L
where L is the Love’s strain function. With that, the equation in (5.69) becomes
− (λ + µ ) −1
∇ 4 ( L) = ρb 3 = ρb 3 (5.73)
µ (λ + 2 µ ) 2 µ (1 − ν )

where we have considered that b1 = b 2 = 0 . The Love’s strain function can be applied to
axially symmetric problem.
Taking into account that
 ∇ 2 (g1 )   0 
r ∂L    
∇ ⋅ g = gi , i = g1,1 + g2, 2 + g3,3 = L,3 ≡ , ∇ 2 (gi ) = ∇ 2 (g2 ) =  0  , the displacement field
∂x3 ∇ 2 (g )  ∇ 2 L 
 3   
(5.68) becomes:
 − ∂2L 
 
 ∂x1∂x3 
r
 λ + 2µ 1 ∂ (∇ ⋅ g)   − ∂2L 
ui = 2 µ  2
∇ (gi ) −  ⇒ ui =   (5.74)
 2 µ (λ + µ ) 2 µ ∂xi   ∂x ∂x
2 3 
λ + 2 µ 2 ∂2L 
 ∇ L− 
 λ+µ ∂x3∂x3 

Problem 5.10
a) Obtain the stress field correspondent to the Galerkin vector:
r
g = 2 x14 eˆ 1 + x24 eˆ 2 + (−8 x13 x3 − 4 x23 x3 )eˆ 3
{ { 1442443
= g1 =g2 = g3

b) Obtain the body force density.


Solution:
The stress in terms of Galerkin vector is given by (5.71), i.e.:
σij = 2 µ{ ν gk , kppδ ij + (1 − ν )[gi , j + g j ,i ], pp − (g p , p ),ij }
r r r (5.75)
= 2 µ{ ν [∇ 2 (∇ ⋅ g)] 1 + 2(1 − ν )∇ 2 [(∇g) sym ] − ∇ (∇ (∇ ⋅ g))}ij

The gradient of the Galerkin vector is given by:

University of Castilla-La Mancha Draft By: Eduardo W. V. Chaves (2016)


Ciudad Real – Spain
5 INTRODUCTION TO: CONSTITUTIVE EQUATIONS, IBVP, AND IBVP SOLUTION STRATEGIES 441

 ∂g1 ∂g1 ∂g1 


 
 ∂x1 ∂x2 ∂x3   8 x1 
3
0 0
r  ∂g2 ∂g2 ∂g2   3 
(∇g)ij = gi , j = = 0 4 x2 0
 ∂x1 ∂x2 ∂x3   
 ∂g  3
 − 24 x1 x3 − 12 x2 x3 − 8 x1 − 4 x2 
2 2 3

 3 ∂g3 ∂g3  
 ∂x1 ∂x2 ∂x3 
r r
Note that (∇ ⋅ g) = Tr(∇g) = 8x13 + 4 x23 + (−8 x13 − 4 x23 ) = 0
r r
By applying the Laplacian to (∇g) we can obtain ∇ 2 (∇g) , which in indicial notation
becomes:
r ∂ 2 (g i , j )
[∇ 2 (∇g)]ij = (g i , j ) ,kk = = g i , jkk
∂x k ∂x k
 48x1 0 0 
∂ 2 (g i , j ) ∂ 2 (g i , j ) ∂ 2 (g i , j )  
= + + = 0 24 x 2 0 
∂x1∂x1 ∂x 2 ∂x 2 ∂x 3 ∂x 3 − 48x
 3 − 24 x 3 − 48 x1 − 24 x 2 

  48 x1 0 0  48 x1 0 − 48 x3 
r sym 1    
{∇ [(∇g) ]}ij =   0
2
24 x2 0 + 0 24 x2 − 24 x3 
2 
 − 48 x3 − 24 x3 − 48 x1 − 24 x2   0 0 − 48 x1 − 24 x2  
 
 48 x1 0 − 24 x3 
 
= 0 24 x2 − 12 x3 
 − 24 x − 12 x − 48 x − 24 x 
 3 3 1 2

r r
Note that [∇ 2 (∇g)]sym = ∇ 2 [(∇g) sym ] . Then, the equation for stress (5.75) becomes:
r r r r
σij = 2µ{ ν [∇ 2 (∇ ⋅ g)] 1 + 2(1 − ν )∇ 2 [(∇g) sym ] − ∇ (∇ (∇ ⋅ g))}ij = 2 µ{ 2(1 − ν )∇ 2 [(∇g) sym ]}ij

  48 x1 0 − 24 x3 
r sym   
σij = 2 µ{ 2(1 − ν )∇ [(∇g) ]}ij = 2 µ 2(1 − ν )  0
2
24 x2 − 12 x3 
 − 24 x3 − 12 x3 − 48 x1 − 24 x2  
  
 96 x1 0 − 48x3 
 
= 2 µ (1 − ν )  0 48x2 − 24 x3 
− 48 x3 − 24 x3 − 48(2 x1 + x2 )

r
b) According to the equation in (5.69) the body force density ( ρb ) and the Galerkin vector
r
( g ) are related to each other by

 ∂ 2 (gi )  − (λ + µ )
∂2
∇ 4 (gi ) =  = ρb
 ∂x j ∂x j  µ (λ + 2 µ ) i
∂xk ∂xk
 
(5.76)
− µ (λ + 2 µ ) 4  (λ + 2 µ )  4
⇒ ρb i = ∇ (gi ) = −2 µ  ∇ (gi ) = −2 µ (1−ν )∇ 4 (gi )
(λ + µ )  2(λ + µ ) 

University of Castilla- La Mancha Draft By: Eduardo W. V. Chaves (2016)


Ciudad Real – Spain
442 SOLVING PROBLEMS BY MEANS OF CONTINUUM MECHANICS

in which
g1 = 2 x14 ; g2 = x24 ; g3 = −8 x13 x3 − 4 x23 x3
∂ 2 (gi ) ∂ 2 (gi ) ∂ 2 (gi ) ∂ 2 (gi )
= + +
∂x j ∂x j ∂x1∂x1 ∂x2∂x2 ∂x3∂x3
 ∂ 2 ( 2 x14 ) ∂ 2 (2 x14 ) ∂ 2 ( 2 x14 )
i = 1 ⇒ + + = 24 x12 = a1
 ∂x1∂x1 ∂x2 ∂x2 ∂x3∂x3
∂ 2 (gi )  ∂ 2 ( x24 ) ∂ 2 ( x24 ) ∂ 2 ( x24 )
= ai = i = 2 ⇒ + + = 12 x22 = a2
∂x j ∂x j  ∂x1∂x1 ∂x2 ∂x2 ∂x3∂x3
 ∂ 2 (g3 ) ∂ 2 (g3 ) ∂ 2 (g3 )
i = 3 ⇒ + + = −48 x1 x3 − 24 x2 x3 = a3
 ∂x1∂x1 ∂x2∂x2 ∂x3∂x3
 ∂ 2 (24 x12 ) ∂ 2 (24 x12 ) ∂ 2 ( 24 x12 )
i = 1 ⇒ + + = 48 x1
 ∂x1∂x1 ∂x2 ∂x2 ∂x3∂x3
∂ 2  ∂ 2
( g )  ∂ 2
( a )  ∂ 2 (12 x22 ) ∂ 2 (12 x22 ) ∂ 2 (12 x22 )
∇ 4 (gi ) =  i 
= i
= i = 2 ⇒ + + = 24 x2
 
∂xk ∂xk  ∂x j ∂x j  ∂xk ∂xk  ∂x1∂x1 ∂x2∂x2 ∂x3∂x3
 ∂ 2 (a3 ) ∂ 2 (a3 ) ∂ 2 (a3 )
i = 3 ⇒ + + =0
 ∂x1∂x1 ∂x2∂x2 ∂x3∂x3
Then, the equation in (5.76) becomes:
48 x1 
 
ρbi = −2 µ (1−ν )∇ (gi ) = −2 µ (1−ν )24 x2 
4

 0 
 

Problem 5.11
a) Show that:
r r
∇ xr ∧ (∇ xr ∧ ε )T = 0 Indicial
 →  qjk til εij , kl = 0 qt (5.77)

where  ijk is the permutation symbol, and ε is the infinitesimal strain tensor.
b) Show also that:
ε ij , kl + ε kl ,ij − ε il , jk − ε jk ,il = O ijkl (5.78)
c) Express the explicit form of the equations in (5.77).
Solution:
1  ∂u j ∂u  1
The infinitesimal strain tensor is given by ε ij =  + i = (u + u i , j ) , and if we
 2 j ,i
2  ∂xi ∂x j 
r
take the derivative with respect to ( x ) we can obtain:
∂ε ij 1
≡ ε ij ,k = (u j ,ik + u i , jk )
∂x k 2

University of Castilla-La Mancha Draft By: Eduardo W. V. Chaves (2016)


Ciudad Real – Spain
5 INTRODUCTION TO: CONSTITUTIVE EQUATIONS, IBVP, AND IBVP SOLUTION STRATEGIES 443

Note that u i , jk = u i ,kj is symmetric in jk , and if we multiply by the antisymmetric tensor in


jk , i.e.  qjk = − qkj , we can obtain: u i , jk  qjk = 0 iq , thus

1 1 1 1
 qjk ε ij ,k = (u j ,ik + u i , jk ) qjk = u j ,ik  qjk + u i , jk  qjk = u j ,ik  qjk
2 2 21424
3 2
= 0 iq

r
once again we take the derivative with respect to ( x ) and we can obtain:
∂ ( qjk ε ij ,k ) 1
=  qjk ε ij ,kl = u j ,ikl  qjk
∂xl 2
Note that u j ,ikl = u j ,kil = u j ,kli is symmetric in il and  til = − tli is antisymmetic in il . With
that, if we multiply both sides of the equation by  til we can obtain the equation in (5.77),
i.e.:
1
 til  qjk ε ij ,kl = u j ,ikl  til  qjk = 0 jkt  qjk = 0 qt
2 Q.E.D.
b) Now, if we multiply both sides of the above equation by  tab  qmn , we can obtain:
 tab  qmn  til  qjk ε ij ,kl = 0 qt  tab  qmn = O abmn
Remember that the relationships  tab  til = δ ai δ bl − δ al δ bi and  qmn  qjk = δ mj δ nk − δ mk δ nj
hold, thus:
tab  qmn til  qjk εij , kl = O abmn
⇒ (δ aiδ bl − δ alδ bi )(δ mjδ nk − δ mkδ nj )ε ij , kl = O abmn
⇒ (δ aiδ blδ mjδ nk − δ aiδ blδ mkδ nj − δ alδ biδ mjδ nk + δ alδ biδ mkδ nj )εij , kl = O abmn

Then we can obtain ε am, nb − ε an , mb − εbm, na + εbn , ma = O abmn , which is the same as:
ε am,bn + ε bn,am − ε an,mb − ε mb,an = O ambn
Q.E.D.

Note that, if we multiply the above equation by δ bn we can also express (5.78) as follows:
ε am,bnδ bn + εbn,amδ bn − ε an ,mbδ bn − ε mb,anδ bn = O ambnδ bn
⇒ ε am,bb + εbb,am − ε ab,bm − ε mb,ba = 0ambb
⇒ [∇ xr ⋅ (∇ xr ε )]am + [∇ xr [∇ xr [Tr (ε )] ] ]am − [∇ xr (∇ xr ⋅ ε )]am − [∇ xr (∇ xr ⋅ ε )]ma = 0ambb
⇒ [∇ xr ⋅ (∇ xr ε )]am + [∇ xr [∇ xr [Tr (ε )] ] ]am = [∇ xr (∇ xr ⋅ ε )]am + [∇ xr (∇ xr ⋅ ε )]ma
[ ]
⇒ ∇ 2xr ε am + [∇ xr [∇ xr [Tr (ε )] ] ]am = [∇ xr (∇ xr ⋅ ε )]am + [∇ xr (∇ xr ⋅ ε )]ma
which in tensorial notation becomes:

∇ xr ⋅ (∇ xr ε ) + ∇ xr [∇ xr [Tr (ε )] ] = ∇ xr (∇ xr ⋅ ε ) + [∇ xr (∇ xr ⋅ ε )] = 2[∇ xr (∇ xr ⋅ ε )]
T sym
(5.79)

c) Note that the equation in (5.77) is symmetric, and has 6 independent equations.
For the case when q = 1, t = 1 we can obtain 1 jk 1il ε ij ,kl and by expanding the index l we
can obtain:
1 jk 1il ε ij ,kl = 1 jk 1i1ε ij , k1 + 1 jk 1i 2 ε ij ,k 2 + 1 jk 1i 3 ε ij ,k 3 = 1 jk 1i 2 ε ij ,k 2 + 1 jk 1i 3 ε ij ,k 3
Expanding the index i the above equation becomes:

University of Castilla- La Mancha Draft By: Eduardo W. V. Chaves (2016)


Ciudad Real – Spain
444 SOLVING PROBLEMS BY MEANS OF CONTINUUM MECHANICS

1 jk 1il ε ij ,kl = 1 jk 1i 2 ε ij ,k 2 + 1 jk 1i 3 ε ij ,k 3 = 1 jk 132 ε 3 j , k 2 + 1 jk 123 ε 2 j ,k 3 = −1 jk ε 3 j , k 2 + 1 jk ε 2 j ,k 3


and by expanding the remaining indices we can obtain:
1 jk 1il ε ij ,kl = −1 jk ε 3 j ,k 2 + 1 jk ε 2 j , k 3 = −123 ε 32,32 − 132 ε 33, 22 + 123 ε 22,33 + 132 ε 23, 23
= −ε 32,32 + ε 33, 22 + ε 22,33 − ε 23, 23 = ε 33, 22 + ε 22,33 − 2ε 23, 23 = 0
∂ 2 ε 33 ∂ 2 ε 22 ∂ 2 ε 23
= + −2 =0
∂x 22 ∂x32 ∂x 2 ∂x3

note that ε 23, 23 = ε 32,32 .


We leave the reader with the following demonstrations:
when q = 2, t = 2
 2 jk  2il ε ij ,kl = −ε 31,31 + ε 33,11 + ε11,33 − ε13,13 = ε 33,11 + ε11,33 − 2ε13,13 = 0
∂ 2 ε 33 ∂ 2 ε11 ∂ 2 ε 13
= + −2 =0
∂x12 ∂x32 ∂x1∂x3
when q = 3, t = 3
 3 jk  3il ε ij ,kl = ε11, 22 − ε12,12 − ε 21, 21 + ε 22,11 = ε11, 22 + ε 22,11 − 2ε12,12 = 0
∂ 2 ε11 ∂ 2 ε 22 ∂ 2 ε 12
= + −2 =0
∂x 22 ∂x12 ∂x1 ∂x 2
when q = 1, t = 2
1 jk  2il εij , kl = −ε12,33 + ε13, 23 + ε32,31 − ε33, 21 = ε13, 23 + ε 23,13 − ε33,12 − ε12,33 = 0
∂ 2 ε13 ∂ 2 ε 23 ∂ 2 ε 33 ∂ 2 ε12 ∂  ∂ε 23 ∂ε13 ∂ε12  ∂ 2 ε 33
= + − − =  + − − =0
∂x2 ∂x3 ∂x1∂x3 ∂x1∂x2 ∂x3∂x3 ∂x3  ∂x1 ∂x2 ∂x3  ∂x1∂x2

when q = 2, t = 3
 2 jk  3il ε ij ,kl = −ε11,32 + ε13,12 + ε 21,31 − ε 23,11 = ε13,12 + ε12,13 − ε 23,11 − ε11, 23 = 0
∂ 2 ε13 ∂ 2 ε 12 ∂ 2 ε 23 ∂ 2 ε11 ∂  ∂ε13 ∂ε12 ∂ε 23  ∂ 2 ε11
= + − − =  + −  − =0
∂x1 ∂x 2 ∂x1 ∂x3 ∂x1∂x1 ∂x 2 ∂x 3 ∂x1  ∂x 2 ∂x3 ∂x1  ∂x 2 ∂x 3
and when q = 1, t = 3
1 jk  3il ε ij ,kl = ε12,32 − ε13, 22 − ε 22,31 + ε 23, 21 = ε12, 23 − ε13, 22 − ε 22,13 + ε 23,12 = 0
∂ 2 ε 12 ∂ 2 ε 13 ∂ 2 ε 22 ∂ 2 ε 23 ∂  ∂ε12 ∂ε 13 ∂ε 23  ∂ 2 ε 22
= − − + =  − +  − =0
∂x 2 ∂x3 ∂x 2 ∂x 2 ∂x1∂x 3 ∂x1∂x 2 ∂x 2  ∂x3 ∂x 2 ∂x1  ∂x1∂x3
By regrouping the previous 6 equations we can obtain:

University of Castilla-La Mancha Draft By: Eduardo W. V. Chaves (2016)


Ciudad Real – Spain
5 INTRODUCTION TO: CONSTITUTIVE EQUATIONS, IBVP, AND IBVP SOLUTION STRATEGIES 445

 ∂ 2 ε 33 ∂ 2 ε 22 ∂ 2 ε 23
S11 = + − 2 =0
 ∂x 22 ∂x32 ∂x 2 ∂x3
 2 2 2
S 22 = ∂ ε 33 + ∂ ε11 − 2 ∂ ε13 = 0
 ∂x12 ∂x32 ∂x1 ∂x3

 ∂ 2 ε 11 ∂ 2 ε 22 ∂ 2 ε 12
 S 33 = + − 2 =0
 ∂x 22 ∂x12 ∂x1 ∂x 2

S = ∂  ∂ε 23 + ∂ε 13 − ∂ε12  − ∂ ε 33 = 0
2

 12 ∂x  ∂x ∂x 2 ∂x3  ∂x1∂x 2
Compatibility equations for 3D (5.80)
3 
 1

 ∂  ∂ε 23 ∂ε 13 ∂ε 12  ∂ 2 ε 11
S 23 = − + + − =0
 ∂x1  ∂x1 ∂x 2 ∂x 3  ∂x 2 ∂x3

S = ∂  ∂ε 23 − ∂ε13 + ∂ε12  − ∂ ε 22 = 0
2

 13 ∂x 2  ∂x1 ∂x 2 ∂x3  ∂x1 ∂x3



or in tensorial notation
r r
∇ xr ∧ (∇ xr ∧ ε )T = 0

The above equations in Voigt notation become:


 ∂2 ∂2 − ∂2 
 0 0 0 
 ∂x32 ∂x 22 ∂x 2 ∂x3 
 ∂2 ∂2 − ∂2 
0 0 0
 S11   ∂x 2 ∂x12

∂x1∂x3   ε11  0
S   3
   
  22  ∂ 2
∂2 − ∂2   ε 22  0
0 0 0
 S 33   ∂x 2 ∂x12 ∂x1 ∂x 2
  ε  0
 =
2   33  =  
 S  − ∂2 ∂2 ∂2 ∂ 2
  2ε12  0 (5.81)
12
0 0 − 12 2 1 1
 S 23   ∂x1 ∂x 2 ∂x3
2
∂x1∂x3 2 
∂x 2 ∂x 3  2ε 23  0
      
 S13   − ∂ ∂ 2   2ε 13  0
2
∂2 ∂2
 0 0 1
2
− 12 2 1
2 
 ∂x 2 ∂x 3 ∂x1 ∂x3 ∂x1 ∂x1 ∂x 2 
 − ∂2 ∂2 ∂ 2
∂2 
 0 0 1
2
1
2
− 12 2 
 ∂x1∂x3 ∂x 2 ∂x3 ∂x1∂x 2 ∂x 2 
{S} = [ L( 2 ) ] {ε } = {0}

NOTE 1: The equations in (5.80) are known as the compatibility equations. The
compatibility equations guarantee that the displacement field is unique and continuous, (see
Figure 5.10). In other words, the 6 components of the strain tensor are not independent
and cannot be arbitrary.

University of Castilla- La Mancha Draft By: Eduardo W. V. Chaves (2016)


Ciudad Real – Spain
446 SOLVING PROBLEMS BY MEANS OF CONTINUUM MECHANICS

(Current configuration?)

2 2
1 3 3
1
4 5 6 5
4 6
7 8 9
7 8 9
(Reference configuration)
The compatibility equations are
not satisfied

The compatibility 1 2 3
4 5 6
equations are satisfied (Current configuration)
7 8 9

Figure 5.10
NOTE 2: When using numerical method for obtaining the solution, e.g. finite element
method, the way to ensure the compatibility equations is by means of the continuity of the
displacement field. With regards the finite element method, when we assembly the
elements (tie nodes), in general, we are ensuring that the compatibility equations are
satisfied.
r r
NOTE 3: When the displacement field is independent of one direction, e.g. u = u( x1 , x 2 ) ,
the compatibility equations reduce to:
∂ 2 ε11 ∂ 2 ε 22 ∂ 2 ε12 Compatibility equation
S 33 = + −2 =0 (5.82)
∂x 22 ∂x12 ∂x1∂x 2 for 2D

since ε i 3 = ε 3i = 0 . The above equation in Engineering notation becomes:

∂ 2ε x ∂ 2ε y ∂ 2 γ xy Compatibility equation
Sz = + − =0 for 2D (Engineering (5.83)
∂y 2 ∂x 2 ∂x∂y notation)
NOTE 4: To understand the compatibility condition let us consider an example in two
dimensional case (2D), where we have the scalar field φ = φ ( x1 , x2 ) and we know the
∂φ ∂φ
following derivatives: = x1 + 3x2 and = x12 , we can see clearly that the scalar field φ
∂x1 ∂x2
is incompatible since
∂φ ∂  ∂φ  ∂ 2φ ∂( x1 + 3 x 2 ) 
= x1 + 3x 2 = F1 ⇒   = = = 3
∂x1 ∂x 2 ∂x1  ∂x 2 ∂x1 ∂x 2  ∂ 2φ ∂ 2φ
 ⇒ ≠
∂φ ∂  ∂φ  ∂ 2φ ∂ ( x12 )  1 ∂x 2 ∂x1 ∂x1∂x 2
= x12 = F2 ⇒   = = = 2 x1 442443
∂x 2 ∂x1  ∂x 2  ∂x1∂x 2 ∂x 2  incompatible

The scalar field φ = φ ( x1 , x2 ) will be compatible if and only if:
∂φ 
= F1 ( x1 , x2 ) 
∂x1  ∂F1 ∂F2
 φis 
compatible field iff
   → = (5.84)
∂φ ∂x2 ∂x1
= F2 ( x1 , x2 )
∂x2 

University of Castilla-La Mancha Draft By: Eduardo W. V. Chaves (2016)


Ciudad Real – Spain
5 INTRODUCTION TO: CONSTITUTIVE EQUATIONS, IBVP, AND IBVP SOLUTION STRATEGIES 447

If we consider the Green’s theorem, (Chaves(2013)-Chapter), which states:


r r r r  ∂F2 ∂F1 
∫ F ⋅ dΓ = Ω∫ (∇
∧ F) ⋅ e 3 dS   → F1dx1 + F2 dx2 = 
ˆ dS3 ∫ ∫
components
r
x −
Γ Γ Ω  ∂x1 ∂x2 
r
and also considering the equation in (5.84), we conclude that: if F = ∇ xr φ , φ is compatible
r r r r r r r
if and only if F ⋅ dΓ = (∇ xr ∧ F) ⋅ eˆ 3dS = 0 ⇒ ∇ xr ∧ F = 0 .
∫ ∫
Γ Ω

r r
x2 dS = dSê 3 dΓ = dΓ p̂

x3 Γ
ê3

x1

Figure 5.11: Green’s theorem.


r r r r r
NOTE 5: Let us consider that F = (∇ xr ∧ ε ) ⋅ a = a ⋅ (∇ xr ∧ ε )T , where ε is a second-order
r r
tensor field and a is an arbitrary vector independent of x (constant). Note also that the
following relations are true:
r r r r r r
r
[ r r r
] r
[
(a) ∫ F ⋅ dΓ = ∫ (∇ xr ∧ ε ) ⋅ a ⋅ dΓ = ∫ a ⋅ (∇ xr ∧ ε)T ⋅ dΓ = a ⋅ ∫ (∇ xr ∧ ε )T ⋅ dΓ ]
Γ Γ Γ Γ
and

∫ {∇ ∧ [(∇ ]} ∫ {∇ ∧ [a ⋅ (∇ ]}⋅ dSr


r r r r r r r r r r
∫ (∇ xr ∧ F) ⋅ dS = r
x
r
x ∧ ε ) ⋅ a ⋅ dS = r
x
r
x ∧ ε )T
Ω Ω Ω
(b)
∫ { } { } ⋅ dS
r r r r r r r r
= a ⋅ ∇ xr ∧ (∇ xr ∧ ε )T ⋅ dS = a ⋅
T T




∇ r
x ∧ (∇ xr ∧ ε )T

In indicial notation becomes


r r r r r r r
(a) ∫ Fi (dΓ )i = ∫ (∇ xr ∧ ε )ij a j (dΓ )i = ∫ a j (∇ xr ∧ ε )ij (dΓ )i = a j ∫ (∇ xr ∧ ε )ij (dΓ )i
Γ Γ Γ Γ
(b)

∫ [ [ ] ]
r r r r r r



(∇ xr ∧ F) i (dS ) i =  ijk Fk , j (dS ) i =  ijk a p (∇ xr ∧ ε )T
Ω Ω
kp , j (dS ) i

  r
∫ {
r
[ r
]
=  ijk a p , j (∇ xr ∧ ε )T kp + a p (∇ xr ∧ ε )T kp , j  (dS ) i

[ ]
Ω  =0 pj 


r
[ r
] r r
=  ijk a p (∇ xr ∧ ε )T kp , j (dS ) i = a p  ijk (∇ xr ∧ ε )T kp , j (dS ) i ∫Ω [ ]

∫ [ ]
r r
= a p  ijk  psq ε qk ,s

,j ∫
(dS ) i = a p  ijk  psq ε qk ,sj (dS ) i

∫ [∇ ] ∫ {∇ } ⋅ dS
r r r r r r r
( dS ) i = a ⋅
T
= ap r
x ∧ (∇ xr ∧ ε )T ip
r
x ∧ (∇ xr ∧ ε )T
Ω Ω

University of Castilla- La Mancha Draft By: Eduardo W. V. Chaves (2016)


Ciudad Real – Spain
448 SOLVING PROBLEMS BY MEANS OF CONTINUUM MECHANICS

It would be worth reviewing Problem 1.110, in which we have shown that the following
r r
relationship (∇ xr ∧ ε) =  ksq ε qp , s eˆ k ⊗ eˆ p holds, thus (∇ xr ∧ ε)T =  psq ε qk , s eˆ k ⊗ eˆ p . Also in
r r
Problem 1.110 we have shown that ∇ xr ∧ (∇ xr ∧ ε )T =  ipq tsj εqj , ps eˆ t ⊗ eˆ i , which is
r r
equivalent to ∇ xr ∧ (∇ xr ∧ ε )T =  psq  ijk εqk ,sj eˆ i ⊗ eˆ p .
And by consider the Stokes’ Theorem, (see Chaves (2013)-Chapter 1), we conclude:
r r r r r
∫ F ⋅ dΓ = Ω∫ (∇
Γ
r
x ∧ F ) ⋅ dS
14444244443


r
∫{ } ⋅ dS
r r r r r r
a⋅ (∇ xr ∧ ε )T ⋅ dΓ = a ⋅ ∇ xr ∧ (∇ xr ∧ ε )T
T

Γ Ω

r
∫{ } ⋅ dS
r r r r
(∇ xr ∧ ε )T ⋅ dΓ =
T

Γ Ω
∇ xr ∧ (∇ xr ∧ ε )T

Then, for a compatible field it must fulfill:


{∇ }
r r T r r
r
x ∧ (∇ xr ∧ ε )T =0 ⇒ ∇ xr ∧ (∇ xr ∧ ε )T = 0
r r
r ∂x r
Now let us consider A = F ⋅ a where F is the gradient deformation, F = r , and a is an
∂X
arbitrary vector. By apply the Stokes’ theorem we can obtain:
r r r r r r r r r r
∫ A ⋅ dΓ = (∇ xr ∧ A ) ⋅ dS
∫ ⇒ ∫ ( F ⋅ a) ⋅ dΓ = (∇ xr ∧ ( F ⋅ a)) ⋅ dS

Γ Ω Γ Ω
1444444444444442444444444444443
(5.85)

r r r
∫Ω {∇ } ⋅ dS ∫Ω {∇ } ⋅ dS
r r r r r
a ⋅ F T ⋅ dΓ = a ⋅ ⋅ dΓ
T T
∫ ∫Γ F
T
r
x ∧F ⇒ = r
x ∧F
Γ
Then, for a compatible field it must fulfill:
{∇ }
r T r
r
x ∧F =0 ⇒ ∇ xr ∧ F = 0
More detail about these algebraic manipulations is provided in Problem 1.110.
NOTE 6: Note that if:
r
∫Γ T ⋅ dΓ = Ω∫ {∇ } ⋅ dS
r T r
r
x ∧TT (5.86)

Then, for a compatible field it must fulfill:


{∇ }
r T r
r
x ∧TT =0 ⇒ ∇ xr ∧ T T = 0
And if we use indicial notation we can obtain:
r
∇ xr ∧ T T =  ipqT jq , p eˆ i ⊗ eˆ j = 0ij eˆ i ⊗ eˆ j

By multiply by  ikt we can obtain:


ipq  iktT jq , p =  ikt 0ij = 0 ktj

University of Castilla-La Mancha Draft By: Eduardo W. V. Chaves (2016)


Ciudad Real – Spain
5 INTRODUCTION TO: CONSTITUTIVE EQUATIONS, IBVP, AND IBVP SOLUTION STRATEGIES 449

Considering  ipq  ikt = (δ pkδ qt − δ ptδ qk ) the above equation can be written as follows:
ipq iktT jq , p = 0 ktj ⇒ T jq , p (δ pkδ qt − δ ptδ qk ) = 0 ktj ⇒ ⇒ T jt , k − T jk ,t = 0 ktj (5.87)
In other words, the above equation is a necessary and sufficient condition that the
r
integrands of ∫ T ⋅ dΓ be exact differentials.
Γ
NOTE 7: In this note we will demonstrate the compatibility equations, for small
deformation regime, using the demonstration described by E. Cesàro, (see Sokolnikoff
(1956), Love(1944)).
r r
Let us consider the material point P 0 ( x 0 ) in which the displacement ui0 ( x 0 ) and the
r
infinitesimal spin tensor ω ij0 ( x 0 ) are known. Next we will determine the displacement at
r
any other material point P ′( x ′) in terms of ε , (see Figure 5.12).

at point P 0 :
r r r P0
u( x 0 ) ≡ u0 - displacement
r P′
ω( x 0 ) ≡ ω 0 - spin tensor
r r
x0 x′
x2
at point P′ :
r r r
u( x ′) ≡ u′ - displacement

x1
x3

Figure 5.12.

Consider the displacement differential element


r r r r r r r r r
du = (∇ xr u) ⋅ dx = ((∇ xr u) sym + (∇ xr u) skew ) ⋅ dx = (ε + ω) ⋅ dx = ε ⋅ dx + ω ⋅ dx ,
which in indicial notation becomes dui = ε ij dx j + ω ij dx j , and by integrating it along the
path ( P 0 → P ′ ) we can obtain:
P′ P′ P′ P′ P′
P′
∫ ∫ ∫
du i = ε ij dx j + ω ij dx j ⇒ ui P0 ∫ ∫
= ε ij dx j + ω ij dx j
P0 P0 P0 P0 P0
P′ P′
r r

⇒ u′i ( x ′) − u i0 ( x 0 ) = ε ij dx j + ω ij dx j ∫ (5.88)
P0 P0
P′ P′
r r

⇒ u′i ( x ′) = u i0 ( x 0 ) + ε ij dx j + ω ij dx j ∫
P0 P0

Note that
P′ P′ P′ P′
∂ ( x j − x ′j ) P′ ∂ω ij
∫ 0

ω ij dx j = ω ijδ jk dxk = ω ij
0
∫ 0 ∂xk
dxk = [ω ij ( x j − x′j )]
P 0
− ∫ ∂x0 k
( x j − x′j )dxk
P P P P

University of Castilla- La Mancha Draft By: Eduardo W. V. Chaves (2016)


Ciudad Real – Spain
450 SOLVING PROBLEMS BY MEANS OF CONTINUUM MECHANICS

where we have applied the integration by parts. The above equation can also be expressed
as follows:
P′ P′
P′ ∂ω ij
∫ 0
ω ij dx j = [ω ij ( x j − x′j )]
P 0
− ∫ ∂x0 k
( x j − x′j )dxk
P P
P′
r r

= [ω′ij ( x ′)( x′j − x′j )] − [ω ij0 ( x 0 )( x 0j − x′j )] − ω ij ,k ( x j − x ′j )dxk
P0
P′


= −[ω ij0 ( x 0j − x ′j )] − ω ij ,k ( x j − x′j )dxk
P0

Then, the equation (5.88) can be rewritten as follows:


P′ P′
r r

u′i ( x ′) = u i0 ( x 0 ) + ε ij dx j + ω ij dx j ∫
P0 P0
P′ P′
(5.89)
r r
∫ ∫
⇒ u′i ( x ′) = u i0 ( x 0 ) + ε ij dx j − [ω ij0 ( x 0j − x′j )] − ω ij ,k ( x j − x′j ) dxk
P0 P0
r
Considering that ϕ is the axial vector associated with the antisymmetric tensor ω the
following is true ω ij = −ϕ q  qij = ϕ q  iqj , then ω ij ,k = −ϕ q ,k  qij = ϕ q ,k  iqj .
∂ϕ q r
We can prove that = [∇ xr ∧ ε ]qk , (see Problem 5.12, Eq. (5.100)). Then, we can say
∂xk
that
r
ω ij ,k ( x j − x′j )dxk = ϕ q ,k  iqj ( x j − x′j )dxk = [∇ xr ∧ ε ]qk  iqj ( x j − x′j )dxk
r r r
= {[∇ xr ∧ ε ]T ∧ ( x − x ′)}ki dxk (5.90)
r r r r
= {{[∇ xr ∧ ε ]T ∧ ( x − x ′)}T ⋅ dx}i
Taking into account the above equation, the equation in (5.89) becomes
P′ P′
r r
∫ ∫
u′i ( x ′) = u i0 ( x 0 ) + ε ij dx j − [ω ij0 ( x 0j − x′j )] − ω ij ,k ( x j − x ′j )dxk
P0 P0
P′ P′
(5.91)
r r r r r r
⇒ u′i ( x ′) = u i0 ( x 0 ) + ε ij dx j − [ω ij0 ( x 0j − x′j )] − {{[∇ xr ∧ ε ]T ∧ ( x − x ′)}T ⋅ dx}i
∫ ∫
P0 P0

or in tensorial notation:
P′ P′
r r r r r r r r r
u′ = u0 + ε ⋅ dx − [ω 0 ⋅ ( x 0 − x ′)] − {[∇ xr ∧ ε ]T ∧ ( x − x ′)}T ⋅ dx , thus
∫ ∫
P0 P0

P′
r r0 r0 r r r r r
u′ = u − [ω ⋅ ( x − x ′)] + [ε − {[∇ xr ∧ ε ]T ∧ ( x − x ′)}T ] ⋅ dx
0
∫ (5.92)
P0

Note that the line integral (from P 0 to P′ ) must be path-independent, hence the line
integral vanish to a closed path (conservative system), i.e.:

University of Castilla-La Mancha Draft By: Eduardo W. V. Chaves (2016)


Ciudad Real – Spain
5 INTRODUCTION TO: CONSTITUTIVE EQUATIONS, IBVP, AND IBVP SOLUTION STRATEGIES 451

r r r r r
∫ [ε − {[∇ xr ∧ ε ]T ∧ ( x − x ′)}T ] ⋅ dΓ = 0
Γ
r r r r r (5.93)
⇒ [ε T − {[∇ xr ∧ ε ]T ∧ ( x − x ′)}]T ⋅ dΓ = 0

Γ
And by applying the Stokes theorem, (see equation (5.85)), we can conclude that:
r r T r
∫ {∇ } ⋅ dS = 0
r r r r r r r
⋅ dΓ
T
∫ [ε − {[∇ xr ∧ ε ] ∧ ( x − x ′)}] ∧ [ε − {[∇ xr ∧ ε ] ∧ ( x − x ′)}]
T T T T
= r
x (5.94)
Γ Ω
In Problem 1.115 we have shown that
r r r r r r r
∇ xr ∧ {[∇ xr ∧ ε ]T ∧ x} = {∇ xr ∧ [∇ xr ∧ ε ]T } ∧ x + [∇ xr ∧ ε ]

Then, taking into account the above equation and ε T = ε , the equation (5.94) becomes:
{ }
r r r r r r
∧ [ε T − {[∇ xr ∧ ε ]T ∧ ( x − x ′)}] ⋅ dS = 0
T


∫ ∇ r
x

{ }
r r r r r r r
⋅ dS = 0
T


∫ [∇ r
x ∧ ε T ] − [∇ xr ∧ {[∇ xr ∧ ε ]T ∧ ( x − x ′)}]
(5.95)
∫{ }
r r r r r r r r
⋅ dS = 0
T
⇒ [∇ xr ∧ ε ] − [{∇ xr ∧ [∇ xr ∧ ε ]T } ∧ ( x − x ′) + [∇ xr ∧ ε ]]

{ }
r r r r r r
⋅ dS = 0
T


∫ −{∇ r
x ∧ [∇ xr ∧ ε ]T } ∧ ( x − x ′)

with that we can conclude that

{ −{∇r r r r T
∧ [∇ xr ∧ ε ]T } ∧ ( x − x ′)
r
x =0 }
r r r r
⇒ {∇ xr ∧ [∇ xr ∧ ε ]T } ∧ ( x − x ′) = 0
r r r r
Since the vector ( x − x ′) must be arbitrary we can conclude that ∇ xr ∧ [∇ xr ∧ ε]T = 0 .
∂ϕ q −1  ∂ε ∂ε 
Note that, if we take into account that =  qst  sk − tk  , (see equation in
∂xk 2  ∂xt ∂xs 
(5.100)), the equation in (5.90) can be rewritten as follows:
−1  ∂ε ∂ε 
ω ij ,k ( x j − x′j )dxk = ϕ q ,k  iqj ( x j − x′j )dxk =  qst  sk − tk  iqj ( x j − x′j )dxk
2  ∂xt ∂xs 
1
=  qst  qij (ε sk ,t − ε tk ,s )( x j − x′j )dxk
2
1
= (δ siδ tj − δ sjδ ti )(ε sk ,t − εtk ,s )( x j − x′j )dxk
2 (5.96)
1
= (δ siδ tj ε sk ,t − δ siδ tj ε tk ,s − δ sjδ ti ε sk ,t + δ sjδ ti ε tk ,s )( x j − x′j )dxk
2
1
= (ε ik , j − ε jk ,i − ε jk ,i + ε ik , j )( x j − x′j )dxk
2
= (εik , j − ε jk ,i )( x j − x′j )dxk

Then, the equation in (5.89) can also be written as follows:

University of Castilla- La Mancha Draft By: Eduardo W. V. Chaves (2016)


Ciudad Real – Spain
452 SOLVING PROBLEMS BY MEANS OF CONTINUUM MECHANICS

P′ P′
r r
∫ ∫
u′i ( x ′) = ui0 ( x 0 ) + ε ij dx j − [ω ij0 ( x 0j − x′j )] − ω ij ,k ( x j − x′j )dxk
P0 P0
P′ P′
r r
∫ ∫
⇒ u′i ( x ′) = ui0 ( x 0 ) − [ω ij0 ( x 0j − x′j )] + ε ik dxk − (ε ik , j − ε jk ,i )( x j − x′j )dxk
P0 P0
P′
(5.97)
r r

⇒ u′i ( x ′) = ui0 ( x 0 ) − [ω ij0 ( x 0j − x′j )] + [ε ik − (ε ik , j − ε jk ,i )( x j − x′j )]dxk
P0
P′
r r
⇒ u′i ( x ′) = ui0 ( x 0 ) − [ω ij0 ( x 0j − x′j )] + Tik dxk∫
P0

where Tik = [εik − (ε ik , j − ε jk ,i )( x j − x′j )] . Since the displacement must be independent of the
path of integration, the integrands Tik dxk must be exact differentials, (see equation (5.87)).
Hence, applying a necessary and sufficient condition that the integrands be exact
differentials we can obtain T jt ,k − T jk ,t = 0 ktj , in which:
T jt = [ε jt − (ε jt , p − ε pt , j )( x p − x′p )] and T jk = [ε jk − (ε jk , p − ε pk , j )( x p − x′p )] ,

thus
T jt ,k − T jk ,t = 0 ktj
⇒ [ε jt − (ε jt , p − ε pt , j )( x p − x′p )],k − [ε jk − (ε jk , p − ε pk , j )( x p − x′p )],t = 0 ktj
⇒ ε jt ,k − (ε jt , p − ε pt , j ) ,k ( x p − x′p ) − (ε jt , p − ε pt , j )( x p − x′p ) ,k
− ε jk ,t + (ε jk , p − ε pk , j ) ,t ( x p − x′p ) + (ε jk , p − ε pk , j )( x p − x′p ) ,t = 0 ktj
⇒ ε jt ,k − (ε jt , pk − ε pt , jk )( x p − x′p ) − (ε jt , p − ε pt , j )δ pk

− ε jk ,t + (ε jk , pt − ε pk , jt )( x p − x′p ) + (ε jk , p − ε pk , j )δ pt = 0 ktj
⇒ ε jt ,k − (ε jt , p − ε pt , j )δ pk − ε jk ,t + (ε jk , p − ε pk , j )δ pt

+ (ε jk , pt − ε pk , jt − ε jt , pk + ε pt , jk )( x p − x′p ) = 0 ktj
⇒ ε jt ,k − ε jt ,k + ε kt , j − ε jk ,t + ε jk ,t − ε tk , j
+ (ε jk , pt − ε pk , jt − ε jt , pk + ε pt , jk )( x p − x′p ) = 0 ktj

with that we can obtain:


(ε jk , pt − ε pk , jt − ε jt , pk + ε pt , jk )( x p − x′p ) = 0 ktj (5.98)
Since the vector ( x p − x′p ) is arbitrary we can conclude that:
ε jk , pt − ε pk , jt − ε jt , pk + ε pt , jk = 0 jkpt (5.99)
which matches the equation in (5.78).

Problem 5.12
r
Given the infinitesimal strain tensor ε , and the displacement field u , (a) show that:

 ε 11 ε 12 − ϕ 3 ε 13 + ϕ 2 
r ∂u i 
( J ) ij ≡ (∇ xr u) ij = = ε 12 + ϕ 3 ε 22 ε 23 − ϕ 1 
∂x j 
ε 13 − ϕ 2 ε 23 + ϕ 1 ε 33 

where ϕ i are the rotation vector components.

University of Castilla-La Mancha Draft By: Eduardo W. V. Chaves (2016)


Ciudad Real – Spain
5 INTRODUCTION TO: CONSTITUTIVE EQUATIONS, IBVP, AND IBVP SOLUTION STRATEGIES 453

b) Show also that:

− 1  ∂ε  −1
∂ϕ k
∂x p
=
−1 ∂ε
 kij ωij , p =  kij  ip − jp =
 2
r
( r
− [∇ xr ∧ ε ]kp − [∇ xr ) r
∧ ε ]kp = [∇ xr ∧ ε ]kp
2 2  ∂x j ∂xi 
  ∂ε 33 ∂ε 23  
 ∂ω 23 ∂ω 23 ∂ω 23   ∂ε13 − ∂ε12   ∂ε 23 − ∂ε 22   − 
   ∂x2 ∂x3   ∂x2 ∂x3   ∂x2 ∂x3  
 ∂x1 ∂x2 ∂x3  
∂ω ∂ω31 ∂ω31   ∂ε11 ∂ε13   ∂ε12 ∂ε 23   ∂ε13 ∂ε33  
= −  31 =  −   −   −  
 ∂x1 ∂x2 ∂x3   ∂x3 ∂x1   ∂x3 ∂x1   ∂x3 ∂x1 
 ∂ω ∂ω12 ∂ω12   ∂ε
 12 ∂ε11   ∂ε 22 ∂ε12   ∂ε 23 ∂ε13  
 12      
 ∂x1 ∂x2 ∂x3   ∂x − ∂x   ∂x − ∂x  −
 1 2   1 2   ∂x1 ∂x2  
(5.100)
r ∂ϕ k
where (∇ xr ϕ ) kp = , and ω is the infinitesimal spin tensor. And the relationship
∂x p
r r
∇ xϕ = ∇ xr ∧ ε holds, (see Problem 1.110).
r

Solution:
r
a) The displacement gradient J ≡ ∇ xr u can be split additively into a symmetric and an
antisymmetric part:

J ≡ ∇ xr u =
r 1
[ r r 1
] [
r r r
] r
(∇ xr u) + (∇ xr u) T + (∇ xr u) − (∇ xr u) T = (∇ xr u) sym + (∇ xr u) skew = ε + ω
2 44424443 1 2 44424443 14 2 43 1 4 2 43
1 r r =ε =ω
= ( ∇ xr u) sym = ( ∇ xr u) skew
r
where the symmetric part ε = (∇ xr u) sym represents the infinitesimal strain tensor, and the
r
antisymmetric part ω = (∇ xr u) skew represents the infinitesimal spin tensor (rotation tensor).
r
If we consider that ϕ is the axial vector associated with the antisymmetric tensor ω we
can conclude that:
 0 ω12 ω13   0 ω12 ω13   0 −ϕ 3 ϕ2 
ωij = ω 21 0 ω 23  = − ω12 0 ω 23  =  ϕ 3 0 − ϕ 1 
ω31 ω 32 0   − ω13 − ω 23 0   − ϕ 2 ϕ1 0 
with that
 ε 11 ε 12 ε 13   0 −ϕ 3 ϕ 2   ε11 ε 12 − ϕ 3 ε 13 + ϕ 2 
r ∂u i 
(∇ x u) ij
r = = ε 12 ε 22 ε 23  +  ϕ 3 0  
− ϕ 1  = ε 12 + ϕ 3 ε 22 ε 23 − ϕ 1 
∂x j 
ε 13 ε 23 ε 33   − ϕ 2 ϕ1 0  ε13 − ϕ 2 ε 23 + ϕ 1 ε 33 
b) Recall from chapter on Tensors that an antisymmetric tensor ( ω ) and its axial vector
r
( ϕ ) are related to each other, in indicial notation, by means of ωij = −ϕ k  kij or
1 r
ϕ k = −  kij ωij . And the gradient of ϕ can be obtained as follows:
2
∂ϕ 1  1
≡ ϕ k , p = −  kij ωij  = −  kij ωij , p
∂x p 2 , p 2
By expanding the dummy indices i, j , we can obtain the following terms:
1 1
ϕ k , p = −  kij ωij , p = − ( k12ω12, p +  k13ω13, p +  k 21ω 21, p +  k 23ω 23, p +  k 31ω31, p +  k 32ω32, p )
2 2

University of Castilla- La Mancha Draft By: Eduardo W. V. Chaves (2016)


Ciudad Real – Spain
454 SOLVING PROBLEMS BY MEANS OF CONTINUUM MECHANICS

Note that the rows of ϕ k , p ( k = 1,2,3 ) are given by:


−1
ϕ k, p =  kij ωij , p
2
 −1 −1
(k = 1) ⇒ ϕ 1, p = 2 (123ω 23, p + 132ω32, p ) = 2 (ω 23, p − ω32, p ) = −ω 23, p

 −1 −1
⇒ (k = 2) ⇒ ϕ 2, p = ( 213ω13, p +  231ω31, p ) = (−ω13, p + ω31, p ) = −ω31, p
 2 2
 −1 −1
(k = 3) ⇒ ϕ 3, p = 2 (312 ω12, p + 321ω 21, p ) = 2 (ω12, p − ω 21, p ) = −ω12, p

where we have used the antisymmetric tensor definition ωij = −ω ji . Taking into account
the above equation we can conclude that:
 ∂ω 23 ∂ω 23 ∂ω 23 
 
ω 23,1 ω 23, 2 ω 23,3   ∂x1 ∂x2 ∂x3 
−1   ∂ω ∂ω31 ∂ω31 
ϕ k, p =  kij ωij , p = −  ω31,1 ω31, 2 ω31,3  = −  31 (5.101)
2  ∂x1 ∂x2 ∂x3 
 ω12,1 ω12, 2 ω12,3   ∂ω

 12 ∂ω12 ∂ω12 
 ∂x1 ∂x2 ∂x3 

To obtain the derivative of ω ij with respect to x p we will start from the definition
1  ∂ui ∂u j  1
[ r
ωij = (∇ xr u) skew ] ij = −
2  ∂x j ∂xi
 = (ui , j − u j ,i ) , thus:
 2

∂ωij 1 1
≡ ωij , p = (ui , j − u j ,i ), p = (ui , jp − u j ,ip )
∂x p 2 2

1
The value of the above equation is not altered if we add and subtract the term u p,ij , thus:
2
1 1 1
ωij , p = (ui , jp − u j ,ip + u p ,ij − u p , ij ) = (ui , jp + u p , ij ) − (u j , ip + u p ,ij )
2 2 2
1 1  1  1 
= (ui , pj + u p ,ij ) − (u j , pi + u p , ji ) =  (ui , p + u p ,i )  −  (u j , p + u p , j ) 
2 2  2 , j  2 , i
∂ε ip ∂ε jp
= ε ip , j − ε jp ,i = −
∂x j ∂xi
Substituting the above equation into the equation (5.101) and by expanding the dummy
indices i, j we can obtain:
−1 −1
ϕ k, p =  kij (ε ip , j − ε jp ,i ) = ( kij ε ip , j −  kij ε jp ,i )
2 2
−1
= ( k12 ε1 p ,1 +  k13 ε 1 p ,3 +  k 21ε 2 p ,1 +  k 23 ε 2 p ,3 +  k 31ε 3 p ,1 +  k 32 ε 3 p , 2
2
−  k 12 ε 2 p ,1 −  k13 ε 3 p ,1 −  k 21ε 1 p , 2 −  k 23 ε 3 p , 2 −  k 31 ε 1 p ,3 −  k 32 ε 2 p ,3 )

Note that the rows of ϕ k , p ( k = 1,2,3 ) can be obtained as follows:

University of Castilla-La Mancha Draft By: Eduardo W. V. Chaves (2016)


Ciudad Real – Spain
5 INTRODUCTION TO: CONSTITUTIVE EQUATIONS, IBVP, AND IBVP SOLUTION STRATEGIES 455

 −1
(k = 1) ⇒ ϕ 1, p = 2 ( 123 ε 2 p ,3 + 132 ε 3 p , 2 − 123 ε 3 p , 2 − 132 ε 2 p ,3 ) = ε 3 p , 2 − ε 2 p ,3

 −1
ϕ k, p = (k = 2) ⇒ ϕ 2, p = ( k13 ε 1 p ,3 +  k 31ε 3 p ,1 +  k13 ε 3 p ,1 −  k 31ε 1 p ,3 ) = ε 1 p ,3 − ε 3 p ,1
 2
 −1
(k = 3) ⇒ ϕ 3, p = 2 ( 312 ε 1 p ,1 +  321ε 2 p ,1 −  312 ε 2 p ,1 −  321 ε 1 p , 2 ) = ε 2 p ,1 − ε 1 p , 2

Then:
(ε 31, 2 − ε 21,3 ) (ε 32, 2 − ε 22,3 ) (ε 33, 2 − ε 23,3 ) 
  r
ϕ k, p =  (ε11,3 − ε 31,1 ) (ε12,3 − ε 32,1 ) (ε13,3 − ε 33,1 )  = [∇ xr ∧ ε ]kp (5.102)
 (ε 21,1 − ε11, 2 ) (ε 22,1 − ε12, 2 ) (ε 23,1 − ε13, 2 ) 
 
Then, taking into account the equation (5.101) and (5.102) we can conclude that:
−1 − 1  ∂ε ∂ε 
ϕ k,p =  kij ω ij , p =  kij  ip − jp 

2 2  ∂x j ∂xi 
 ∂ω 23 ∂ω 23 ∂ω 23   ∂ε13 − ∂ε12   ∂ε 23 − ∂ε 22   ∂ε 33 − ∂ε 23 
     ∂x3   ∂x2 ∂x3 
 ∂x1 ∂x2 ∂x3   ∂x2 ∂x3   ∂x2
∂ω ∂ω 31 ∂ω 31   ∂ε11 ∂ε13   ∂ε12 ∂ε 23   ∂ε13 ∂ε 33  
= −  31 =  −   −   − 
 ∂x ∂x2 ∂x3   ∂x3 ∂x1   ∂x3 ∂x1   ∂x3 ∂x1  
 1 
 ∂ω12 ∂ω12 ∂ω12   ∂ε ∂ε   ∂ε ∂ε   ∂ε
 12 − 11   22 − 12   23 − 13  
∂ε  
 ∂x1 ∂x2 ∂x3  
 ∂x1 ∂x2   ∂x1 ∂x2   ∂x1 ∂x2  

where we have used the symmetry of ε , i.e. ε ij = ε ji .


Example: Let us suppose that we know the infinitesimal strain tensor which is given by
 − x2 3 2 
 8 x1 x1 x 3 
 ε 11 ε 12 ε 13   2 2 
− x2
ε ij = ε 12 ε 22 ε 23  =  x1 0  (5.103)
 2 
ε 13 ε 23 ε 33   3 2 3 
 2 x1 x 3 0 x1 
 
with the following boundary conditions:
3t 
r r r r r r r r
u i ( x = 0, t ) =  0  and ω ( x = 0, t ) = 0 ⇒ ϕ ( x = 0, t ) = 0
 0 

To this example, we can obtain:


 ∂ε 13 ∂ε 12   ∂ε 23 ∂ε 22   ∂ε 33 ∂ε 23 
 −   −   −   
 ∂x 2 ∂x3   ∂x 2 ∂x 3   ∂x 2 ∂x 3   0 0 0

 ∂ε ∂ε   ∂ε ∂ε   ∂ε ∂ε    −3 2
ϕ k, p =  11 − 13   12 − 23   13 − 33   = − x1 x 3 0 x1 
 ∂x 3 ∂x1   ∂x 3 ∂x1   ∂x 3 ∂x1    2 
−3
 ∂ε ∂ε   ∂ε ∂ε   ∂ε ∂ε    0 0 
 12 − 11   22 − 12   23 − 13    2 
 ∂x1 ∂x 2   ∂x1 ∂x 2   ∂x1 ∂x 2  

Note also that the following holds:

University of Castilla- La Mancha Draft By: Eduardo W. V. Chaves (2016)


Ciudad Real – Spain
456 SOLVING PROBLEMS BY MEANS OF CONTINUUM MECHANICS

 ∂ϕ 1 ∂ϕ 1 ∂ϕ 1 
   
 ∂x1 ∂x 2 ∂x 3   0 0 0

 ∂ϕ ∂ϕ 2 ∂ϕ 2   −3 2
ϕ k, p = 2 =  − x1 x 3 0 x1 
∂x ∂x 2 ∂x 3   2 
 1  3
 ϕ3
∂ ∂ϕ 3 ∂ϕ 3   0 0 
 
 ∂x1 ∂x 2 ∂x 3   2

with that and by means of the integration we can obtain the components ϕ i :
∂ϕ 1  ∂ϕ 2 
= 0 = − x1 x3 
∂x1  ∂x1 
∂ϕ 1  ∂ϕ 2  −3 2
= 0 ⇒ ϕ 1 = C1 (t ) ; =0  ⇒ϕ 2 = x1 x 3 + C 2 (t )
∂x 2  ∂x 2  2
∂ϕ 1  ∂ϕ 2 − 3 2 
= 0 = x1 
∂x3  ∂x 3 2 
∂ϕ 3 
=0
∂x1 
∂ϕ 3 3  3
=  ⇒ ϕ 3 = x 2 + C 3 (t )
∂x 2 2 2
∂ϕ 3 
=0
∂x 3 
r r r r
By applying the boundary condition ϕ ( x = 0, t ) = 0 , we can conclude that C i (t ) = 0 . Then:
 0 
ϕ 1   
 − 3 
ϕ i = ϕ 2  =  x12 x3 
2
ϕ 3   3 
 x2 
 2 
And the infinitesimal spin tensor becomes:
 −3 −3 2 
0 x2 x1 x3 
 0 ω12 ω13   0 −ϕ 3 ϕ 2   2 2

3
ωij = ω 21 0 ω 23  =  ϕ 3 0 − ϕ 1  =  x2 0 0 
 2 
ω31 ω32 0  − ϕ 2 ϕ1 0   3 2
x x 0 0 
 2 1 3 

The displacement field can be obtained by means of


 ε11 ε12 ε13   0 ω12 ω13 
∂ui 
= ε12 ε 22 ε 23  + ω 21 0 ω 23 
∂x j 
ε13 ε 23 ε 33  ω31 ω32 0 
 − x2 3 2   −3 −3 2 
 8 x1 x1 x3   0 x2 x1 x3 
− 2 x2
  1
2 2 2 2 8x 0
 −x   3
= 2
x1 0  +  x2 0 0  =  x2 x1 0 
 2   2   2
 3 x2 x 3   3 x2 x 3x1 x3 0 x13 
0 x1 0 0 
 2 1 3   2 1 3 

University of Castilla-La Mancha Draft By: Eduardo W. V. Chaves (2016)


Ciudad Real – Spain
5 INTRODUCTION TO: CONSTITUTIVE EQUATIONS, IBVP, AND IBVP SOLUTION STRATEGIES 457

or
 ∂u1 ∂u1 ∂u1 
 
 ∂x1 ∂x 2 ∂x 3   8 x
1 − 2x2 0
∂u i  ∂u 2 ∂u 2 ∂u 2  
= = x x1 0 
∂x j  ∂x1 ∂x 3   2
2
∂x 2
  3x1 x 3 0 x13 
 ∂u 3 ∂u 3 ∂u 3 
 ∂x1 ∂x 2 ∂x 3 
with that we can obtain
∂u1  ∂u 2 
= 8 x1  = x2 
∂x1  ∂x1 
∂u1  ∂u 2 
= −2 x 2  ⇒ u1 = 4 x12 − x 22 + K 1 (t ) ; = x1  ⇒ u 2 = x1 x 2 + K 2 (t )
∂x 2  ∂x 2 
∂u1  ∂u 2 
=0  =0 
∂x 3  ∂x 3 
∂u 3 
= 3 x12 x 3 
∂x1 
∂u 3  3
=0  ⇒ u 3 = x1 x3 + K 3 (t )
∂x 2 
∂u1 
= x13 
∂x 3 

The constants of integration can be obtained by means of the boundary condition:


4 x12 − x 22 + K 1 (t )  K 1 (t )  3t 
  r r r r
u i ( x = 0, t ) =  K 2 (t ) =  0 
r x =0
u i ( x , t ) =  x1 x 2 + K 2 (t )  
→
 x 3 x + K (t )   K 3 (t )   0 
 1 3 3 
Then, the displacement field becomes:
 4 x12 − x 22 + 3t 
r  
u i ( x, t ) =  x1 x 2 
 x 3
x 
 1 3 
It is interesting to verify that the displacement field is compatible, since the infinitesimal
strain tensor field, (see equation (5.103)), fulfills the compatible equations (see equations in
(5.80)). We leave the reader to verify this fact.
Problem 5.13
Consider a cantilever beam in which the infinitesimal strain tensor is given by
 κ x2 x 3 0 0
ε ik ( x1 , x3 ) =  0 0 0 ; ε = κ x 2 x 3 eˆ 1 ⊗ eˆ 1 (5.104)
 0 0 0

where κ x2 = κ x2 ( x1 ) is the curvature of the beam which is constant on cross section, (see
Figure 5.13). a) Check whether the compatibility equations are fulfilled or not. b) Obtain
the displacement field.

University of Castilla- La Mancha Draft By: Eduardo W. V. Chaves (2016)


Ciudad Real – Spain
458 SOLVING PROBLEMS BY MEANS OF CONTINUUM MECHANICS

x3
x2 ε11 = κ x2 x3

r r cross section
x0 = 0
x1
∂u3 ( x1 = 0)
=0
∂x1 M x2

x1′

Figure 5.13: Cantilever beam.


Solution:
a) We can use the compatibility equations in (5.80) to verify that:
 ∂ 2 ε33 ∂ 2 ε 22 ∂ 2 ε 23
 2 + − 2 =0 X
 ∂x2 ∂x32 ∂x2 ∂x3
 2 2 ∂ 2 ( κ x2 x3 ) ∂ ( κ x2 )
 ∂ ε33 + ∂ ε11 − 2 ∂ ε13 =
2
=− =0 X
 ∂x1 2
∂x32
∂x1∂x3 ∂x3 2
∂x3

 ∂ 2 ε11 ∂ 2 ε 22 ∂ 2 ε12 ∂ 2 ( κ x2 x3 )
 2 + − 2 = =0 X
 ∂x2 ∂x12 ∂x1∂x2 ∂x22

 ∂  ∂ε 23 + ∂ε13 − ∂ε12  − ∂ ε 33 = 0 X
2

 ∂x  ∂x ∂x2 ∂x3  ∂x1∂x2


 3 1
 ∂  ∂ε ∂ε ∂ε  ∂ 2 ε11 ∂ 2 ( κ x2 x3 ) ∂ ( κ x2 )
  − 23 + 13 + 12  − =− =− =0 X
 ∂x1  ∂x1 ∂x2 ∂x3  ∂x2 ∂x3 ∂x2 ∂x3 ∂x2

 ∂  ∂ε 23 − ∂ε13 + ∂ε12  − ∂ ε 22 = 0 X
2

 ∂x2  ∂x1 ∂x2 ∂x3  ∂x1∂x3



Note that, the compatibility equations are satisfied if κ x2 is neither a function of x 2 nor
x3 , i.e. κ x2 must be constant on the cross-section.
r r
We can also verify the compatibility equations by means of ∇ xr ∧ (∇ xr ∧ ε) T = 0 , i.e.:
r ∂ ˆ  ∂ ∂ ˆ ∂ ˆ 
(∇ xr ∧ ε ) = e i ∧ (ε11eˆ 1 ⊗ eˆ 1 ) =  eˆ 1 + e2 + e 3  ∧ [ κ x 2 x3eˆ 1 ⊗ eˆ 1 ]
∂xi  ∂x1 ∂x2 ∂x3 
∂ ( κ x 2 x3 ) ∂ ( κ x 2 x3 ) ∂ ( κ x 2 x3 )
= eˆ 1 ∧ eˆ 1 ⊗ eˆ 1 + eˆ 2 ∧ eˆ 1 ⊗ eˆ 1 + eˆ 3 ∧ eˆ 1 ⊗ eˆ 1
∂x1 12r 3 ∂x 123 ∂x3 123
=0 142243 = − eˆ 3 = eˆ 2
=0

= κ x 2 eˆ 2 ⊗ eˆ 1
r
(∇ xr ∧ ε )T = [ κ x2 eˆ 2 ⊗ eˆ 1 ]T = κ x2 eˆ 1 ⊗ eˆ 2

University of Castilla-La Mancha Draft By: Eduardo W. V. Chaves (2016)


Ciudad Real – Spain
5 INTRODUCTION TO: CONSTITUTIVE EQUATIONS, IBVP, AND IBVP SOLUTION STRATEGIES 459

r r  ∂ ∂ ˆ ∂ ˆ 
∇ xr ∧ (∇ xr ∧ ε )T =  eˆ 1 + e2 + e3  ∧ [ κ x 2 eˆ 1 ⊗ eˆ 2 ]
 ∂x1 ∂x2 ∂x3 
∂(κ x2 ) ∂(κ x2 ) ∂(κ x2 )
= eˆ 1 ∧ eˆ 1 ⊗ eˆ 2 + eˆ 2 ∧ eˆ 1 ⊗ eˆ 2 + eˆ 3 ∧ eˆ 1 ⊗ eˆ 2
∂x1 1 2 r
3 ∂x2 1 2 3 ∂x3 123
=0 123 = − eˆ 3 123 = eˆ 2
=0 =0

=0 X
b) The displacement field can be obtained by means of the equation in (5.97), i.e.:
P′
r r

u′i ( x ′) = ui0 ( x 0 ) − [ωij0 ( x 0j − x′j )] + [ε ik − (ε ik , j − ε jk ,i )( x j − x′j )]dxk
0
(5.105)
P

and by applying the boundary conditions, (see Figure 5.13), we can obtain:
r r
x′ x′
r

r
0

u′i ( x ′) = Tik dxk = [ε ik − (ε ik , j − ε jk ,i )( x j − x′j )]dxk
r
0
r
x′
r
x′
(5.106)

= [ε ik − ε ik , j ( x j − x′j ) + ε jk ,i ( x j − x′j )]dxk =
r
0

r
0
[Tik(1) − Tik( 2 ) + Tik(3) ]dxk

where Tik(1) = ε ik ,
Tik( 2 ) = ε ik , j ( x j − x′j ) = ε ik ,1 ( x1 − x1′ ) + ε ik , 2 ( x2 − x2′ ) + ε ik ,3 ( x3 − x3′ ) = ε ik ,1 ( x1 − x1′ ) + ε ik ,3 ( x3 − x3′ )
 κ x2 ,1 x3 ( x1 − x1′ ) + κ x2 ( x3 − x3′ ) 0 0
=  0 0 0
 0 0 0
∂κ
where κ x2 ,1 ≡ x2 and
∂x1

Tik(3) = ε jk ,i ( x j − x′j ) = ε1k ,i ( x1 − x1′ ) + ε 2 k ,i ( x2 − x2′ ) + ε 3k ,i ( x3 − x3′ ) = ε1k ,i ( x1 − x1′ )


 κ x2 ,1 x3 ( x1 − x1′ ) 0 0
 
= 0 0 0
 κ x ( x1 − x1′ ) 0 0
 2

Then,
 κ x2 x3 0 0  κ x2 ,1 x3 ( x1 − x1′ ) + κ x2 ( x3 − x3′ ) 0 0  κ x2 ,1 x3 ( x1 − x1′ ) 0 0
 
Tik =  0 0 0 −  0 0 0 +  0 0 0
 0 0 0  0 0 0  κ x2 ( x1 − x1′ ) 0 0

 κ x2 x3′ 0 0
 
Tik =  0 0 0
 κ x ( x1 − x1′ ) 0 0
 2 
 κ x2 x3′ 0 0  dx1   κ x2 x3′ dx1 
    
and Tik dxk =  0 0 0 dx2  =  0 
 κ x ( x1 − x1′ ) 0 0 dx3  κ x ( x1 − x1′ )dx1 
 2    2 
Then, by substituting the above equation into the equation (5.106) we can obtain:

University of Castilla- La Mancha Draft By: Eduardo W. V. Chaves (2016)


Ciudad Real – Spain
460 SOLVING PROBLEMS BY MEANS OF CONTINUUM MECHANICS

r r
 x′   x′

r
x′

u1′   r ∫
κ x2 x3′ dx1   x3′ κ x2 dx1 
  r ∫

r    0   0


u′i ( x ′) = Tik dxk
r
⇒ u′2  =  xr′
u′  
0  =  xr′
 
0 

∫ ∫
0
 3   κ x ( x1 − x1′ )dx1   κ x ( x1 − x1′ )dx1 
2 2
 0r   0r 

Note that, if we consider that κ is constant, we can obtain:


r
x′
   
u1′  
 κ ′
x2 3 dx1
x
r


 


κ x2 x3′ ( x1 ) 0   κ x2 x3′ x1′ 
x1′ 
   0
    
u′2  =  xr ′ 0 = 0
′1
= 0 
u′     x 2 x
  x1′ 
2

2
 0r

 3  κ x ( x1 − x1′ )dx1  κ ( 1 − x′ x )  − κ x2
 
x2
2
1 1
  2 
0 

Then
∂u1′ ∂ 2u′3
= x3′ κ x2 ; = − κ x2
∂x1 ∂x1∂x1
Note that for the neutral line (line at x3 ) there is no u1 -displacement, there is only
deflection ( u 3 -displacement).
The stress field (with ν = 0 ), (see Problem 5.5 – NOTE 9), can be obtained
1 0 0  ε11 0 0
νE E νE E 
σ ij = ε kk δ ij + ε ij = ε 11 0 1 0 + 0 0 0
(1 + ν )(1 − 2ν ) (1 + ν ) (1 + ν )(1 − 2ν ) (1 + ν ) 
0 0 1  0 0 0
ε11 (1 − ν ) 0 0   Eε11 0 0
E 
= 0 νε11 0  =  0 0 0
(1 + ν )(1 − 2ν ) 
 0 0 νε11   0 0 0
The resultant force on a cross-section can be obtained as follows:

∫ ∫ ∫
F = σ11dA = Eε11dA = Eκ x2 x3 dA = Eκ x2 x3 dA = 0
A A A A 23

1
=0

Note that the first moment of the area about the x 2 -axis ( ∫ x3 dAx 2 ) is equal to zero, since
A
the system is located at the geometrical center.
The bending moment on the cross-section can be obtained as follows:

∫ ∫ ∫
M x2 = σ11 x3 dA = Eε11 x3 dA = Eκ x2 x32 dA = Eκ x2 x32 dA = Eκ x2 I x2
A A A A 23

1
=Ix 2

where I x2 = ∫ x32 dA is the second moment of the area about the x 2 -axis. With that we can
A
conclude that on the cross-section the following is true:
M x2 M x2
κ x2 = ; σ11 ( x3 ) = Eε11 = Eκ x2 x3 = x3
EI x2 I x2

University of Castilla-La Mancha Draft By: Eduardo W. V. Chaves (2016)


Ciudad Real – Spain
5 INTRODUCTION TO: CONSTITUTIVE EQUATIONS, IBVP, AND IBVP SOLUTION STRATEGIES 461

Problem 5.14
Consider the infinitesimal strain tensor field
 κ x2 x3 0  0
 
ε ik ( x1 , x3 ) =  0 − νκ x2 x3 0 
 0 − νκ x2 x3 
(5.107)
 0
ε = κ x2 x3eˆ 1 ⊗ eˆ 1 − νκ x2 x3eˆ 2 ⊗ eˆ 2 − νκ x2 x3eˆ 3 ⊗ eˆ 3

where κ x2 is a function of x1 , i.e. κ x2 = κ x2 ( x1 ) and ν (Poisson’s ratio) is a constant. a)


Check whether the compatibility equations are fulfilled or not. b) In the case that the
compatibility equations are not satisfied, what should be met to ensure the continuity of
the displacement field?
Solution:
r r
We can verify the compatibility equations by means of [∇ xr ∧ (∇ xr ∧ ε)T ] = 0 , i.e.:
r ∂ ˆ
(∇ xr ∧ ε ) = ei ∧ ( κ x2 x3eˆ 1 ⊗ eˆ 1 − ν κ x2 x3eˆ 2 ⊗ eˆ 2 − ν κ x2 x3eˆ 3 ⊗ eˆ 3 )
∂xi
 ∂ ∂ ˆ ∂ ˆ 
=  eˆ 1 + e2 + e3  ∧ [ κ x2 x3eˆ 1 ⊗ eˆ 1 − ν κ x2 x3eˆ 2 ⊗ eˆ 2 − ν κ x2 x3eˆ 3 ⊗ eˆ 3 ]
 ∂x1 ∂x2 ∂x3 
∂ (ν κ x 2 x3 ) ∂ (ν κ x2 x3 )
=− eˆ 1 ∧ eˆ 2 ⊗ eˆ 2 − eˆ 1 ∧ eˆ 3 ⊗ eˆ 3
∂x1 1 2 3 ∂x1 123
= eˆ 3 = − eˆ 2

∂ ( κ x 2 x3 ) ∂ (ν κ x2 x3 )
+ eˆ 2 ∧ eˆ 1 ⊗ eˆ 1 − eˆ 2 ∧ eˆ 3 ⊗ eˆ 3
∂x 123 ∂x2 1424 3
4224
1 3 = − eˆ 3 14243 = eˆ 1
=0 =0

∂ ( κ x 2 x3 ) ∂ (ν κ x2 x3 )
+ eˆ 3 ∧ eˆ 1 ⊗ eˆ 1 − eˆ 3 ∧ eˆ 2 ⊗ eˆ 2
∂x3 123 ∂x3 1424 3
= eˆ 2 = − eˆ 1

∂κ x2 ∂κ x2
= −ν x3 eˆ 3 ⊗ eˆ 2 + ν x3 eˆ 2 ⊗ eˆ 3 + κ x2 eˆ 2 ⊗ eˆ 1 + ν κ x2 eˆ 1 ⊗ eˆ 2
∂x1 ∂x1
thus
r ∂κ x2 ∂κ x2
(∇ xr ∧ ε )T = −ν x3 eˆ 2 ⊗ eˆ 3 + ν x3 eˆ 3 ⊗ eˆ 2 + κ x 2 eˆ 1 ⊗ eˆ 2 + ν κ x 2 eˆ 2 ⊗ eˆ 1
∂x1 ∂x1
r r  ∂ ∂ ˆ ∂ ˆ  r
∇ xr ∧ (∇ xr ∧ ε )T =  eˆ 1 + e2 + e3  ∧ [(∇ xr ∧ ε )T ]
 ∂x1 ∂x2 ∂x3 
 ∂ ∂ ˆ ∂ ˆ  ∂κ x 2 ∂κ x2
=  eˆ 1 + e2 + e3  ∧ [−ν x3 eˆ 2 ⊗ eˆ 3 + ν x3 eˆ 3 ⊗ eˆ 2
 ∂x1 ∂x2 ∂x3  ∂x1 ∂x1
+ κ x2 eˆ 1 ⊗ eˆ 2 + νκ x2 eˆ 2 ⊗ eˆ 1 ]
∂ 2κ x2 ∂ 2 κ x2 ∂κ x2 ∂κ x2
= −ν x3 eˆ 3 ⊗ eˆ 3 − ν x3 eˆ 2 ⊗ eˆ 2 + ν eˆ 3 ⊗ eˆ 1 + ν eˆ 1 ⊗ eˆ 3
∂x12 ∂x12 ∂x1 ∂x1

University of Castilla- La Mancha Draft By: Eduardo W. V. Chaves (2016)


Ciudad Real – Spain
462 SOLVING PROBLEMS BY MEANS OF CONTINUUM MECHANICS

 ∂κ x 2 
 0 0 ν 
 ∂x1 
r r  ∂ 2 κ x2 
T
Sij = [∇ xr ∧ (∇ xr ∧ ε ) ]ij =  0 − ν x3 0  ≠ 0ij
 ∂x12 
 ∂κ x2 ∂ 2 κ x2 
ν 0 − ν x3 
 ∂x1 ∂x12 
Note that the compatibility equations are not satisfied. One possibility to guarantee the
r r
continuity of the displacement field ( [∇ xr ∧ (∇ xr ∧ ε ) T ] = 0 ), related to the strain field
(5.107), is when κ x2 is a constant, another possibility is when ν = 0 , (see Problem 5.13).
Note also that the above equation could have been obtained by means of the equation in
(5.80), i.e:
 ∂ 2 ε 33 ∂ 2 ε 22 ∂ 2 ε 23
S
 11 = + − 2 =0 X
 ∂x22 ∂x32 ∂x2 ∂x3
 2 2 ∂ 2 κ x2
S 22 = ∂ ε 33 + ∂ ε11 − 2 ∂ ε13 = −νx3
2

 ∂x12 ∂x32 ∂x1∂x3 ∂x12


≠0 x it fails

 ∂ 2 ε11 ∂ 2 ε 22 ∂ 2 ε12 ∂ 2 κ x2
S =
 33 ∂x 2 +
∂x12
− 2
∂x1∂x2
= −ν x3
∂x12
≠0 x it fails
 2

S = ∂  ∂ε 23 + ∂ε13 − ∂ε12  − ∂ ε 33 = 0 X
2

 ∂x3  ∂x1 ∂x2 ∂x3  ∂x1∂x2


12

 ∂  ∂ε 23 ∂ε13 ∂ε12  ∂ 2 ε11
S 23 =  − + +  − =0 X
 ∂ x 1  ∂x 1 ∂x 2 ∂ x 3  ∂x2 ∂x3

S = ∂  ∂ε 23 − ∂ε13 + ∂ε12  − ∂ ε 22 = ν ∂κ x2 ≠ 0
2

 13
∂x2  ∂x1 ∂x2 ∂x3  ∂x1∂x3 ∂x1
x it fails

Problem 5.15
By considering a homogeneous isotropic linear elastic material, and a static problem
without body forces in which the stress field is given by:
 C

2 (λ + µ )
[ (λ + 2 µ ) x22 + λx12 ] − Cλ
(λ + µ )
x1 x2 0


 
r
[σ ( x )]ij = 

− Cλ
(λ + µ )
x1 x2
C
2(λ + µ )
[
(λ + 2 µ ) x12 + λx22 ] 0 

 Cλ 
 0 0 ( x12 + x22 )
 2(λ + µ ) 
where C ≠ 0
a) Check if the equations of motion are satisfied;
b) Check if the stress field is appropriated to represent any continuum.
Solution:

University of Castilla-La Mancha Draft By: Eduardo W. V. Chaves (2016)


Ciudad Real – Spain
5 INTRODUCTION TO: CONSTITUTIVE EQUATIONS, IBVP, AND IBVP SOLUTION STRATEGIES 463

r r
a) For the static problem, the equations of motions ( ∇ xr ⋅ σ + ρb = ρa ) become the
r r
equilibrium equations ( ∇ xr ⋅ σ + ρb = 0 ). And the equilibrium equations without body
r r
forces ( ρb = 0 ) is represented by:
r r
∇ xr ⋅ σ + {
ρb = 0 indicial
 → σij , j = 0i
r (5.108)
=0

thus
 ∂σ11 ∂σ12 ∂σ13
σ11,1 + σ12, 2 + σ13,3 = + + =0
 ∂x1 ∂x2 ∂x3
 ∂σ 21 ∂σ 22 ∂σ 23
σ i1,1 + σ i 2, 2 + σ i 3,3 = 0 i ⇒ σ 21,1 + σ 22, 2 + σ 23,3 = + + =0
 ∂x1 ∂x2 ∂x3
 ∂σ ∂σ ∂σ
σ31,1 + σ32, 2 + σ33,3 = 31 + 32 + 33 = 0
 ∂x1 ∂x2 ∂x3
Then, by substituting the given stress field components we can obtain:
 C ∂[(λ + 2 µ ) x22 + λx12 ] − Cλ ∂ ( x1 x2 ) C − Cλ
 + +0= [2λx1 ] + ( x1 ) = 0 X
 2(λ + µ ) ∂x1 (λ + µ ) ∂x2 2(λ + µ ) (λ + µ )
 − Cλ ∂( x x ) C ∂[(λ + 2 µ ) x12 + λx22 ] − Cλ C

1 2
+ +0= ( x2 ) + [2λx2 ] = 0 X
 (λ + µ ) ∂x1 2(λ + µ ) ∂x2 (λ + µ ) 2(λ + µ )
 Cλ ∂ ( x12 + x22 )
0 + 0 + =0=0 X
 2(λ + µ ) ∂x3
Then, the three equations are satisfied.
b) Any continuum must satisfy the compatibility equations, so, for a given stress field if the
correspondent strain field satisfies the compatibility equations, the stress field is acceptable
to represent the continuum stress state. In Problem 5.5, (see equation (5.26)), we have
shown that:
−λ 1 −λ 1
ε= Tr (σ )1 + σ εij = σ kkδ ij + σij
2 µ (3λ + 2 µ ) 2µ 2 µ (3λ + 2 µ ) 2µ

For this problem we have:


C
Tr (σ ) = σ11 + σ 22 + σ33 = {[(λ + 2 µ ) x22 + λx12 ] + [(λ + 2 µ ) x12 + λx22 ] + [λ( x12 + x22 )]
2(λ + µ )
C (3λ + 2 µ ) 2
= ( x1 + x22 )
2(λ + µ )
then
−λ 1 −λ C (3λ + 2 µ ) 2 1
ε= Tr (σ )1 + σ= ( x1 + x22 )1 + σ
2 µ (3λ + 2 µ ) 2µ 2 µ (3λ + 2 µ ) 2(λ + µ ) 2µ
− λC 1
= ( x12 + x22 )1 + σ
4 µ (λ + µ ) 2µ

University of Castilla- La Mancha Draft By: Eduardo W. V. Chaves (2016)


Ciudad Real – Spain
464 SOLVING PROBLEMS BY MEANS OF CONTINUUM MECHANICS

whose components are:


 − λC 2 2 1 Cx22
ε
 11 = ( x + x ) + σ =
4 µ (λ + µ ) 2µ 2(λ + µ )
1 2 11

 − λC 1 Cx12
ε 22 = ( x12 + x22 ) + σ 22 =
 4 µ (λ + µ ) 2µ 2(λ + µ )
 − λC 1
ε 33 = ( x12 + x22 ) + σ33 = 0
− λC 1  4 µ (λ + µ ) 2µ
ε ij = ( x1 + x2 )δ ij +
2 2
σij ⇒ 
4 µ (λ + µ ) 2µ ε = 1 σ = − Cλ x x
 12 2 µ 12 2 µ (λ + µ ) 1 2

ε = 1 σ = 0
 23 2 µ 23

 1
ε13 = 2 µ σ13 = 0

Now, we check the compatibility equations:
 ∂ 2 ε 33 ∂ 2 ε 22 ∂ 2 ε 23
S
 11 = + − 2 =0+0−0=0 X
 ∂x22 ∂x32 ∂x2 ∂x3
 ∂ 2 ε 33 ∂ 2 ε11 ∂ 2 ε13
S 22 = + − 2 =0+0−0=0 X
 ∂x12 ∂x32 ∂x1∂x3

S = ∂ ε11 + ∂ ε 22 − 2 ∂ ε12 = C + C − 2 − Cλ  = C (λ + 2 µ ) ≠ 0
2 2 2

 33 ∂x22 ∂x12 ∂x1∂x2 (λ + µ ) (λ + µ )  2 µ (λ + µ )  µ (λ + µ ) x


  

S = ∂  ∂ε 23 + ∂ε13 − ∂ε12  − ∂ ε 33 = 0 + 0 − 0 − 0 = 0 X
2

 12 ∂x3  ∂x1 ∂x2 ∂x3  ∂x1∂x2



 ∂  ∂ε 23 ∂ε13 ∂ε12  ∂ 2 ε11
S
 23 = − + + − = −0 + 0 + 0 − 0 = 0 X
 ∂x1  ∂x1 ∂x2 ∂x3  ∂x2 ∂x3
  
S13 = ∂  ∂ε 23 − ∂ε13 + ∂ε12  − ∂ ε 22 0 − 0 + 0 − 0 = X
2

 ∂x2  ∂x1 ∂x2 ∂x3  ∂x1∂x3

As we can see the equation S33 ≠ 0 fails. So, the given stress field is not appropriated to
represent any continuum.
In Problem 5.16 we will derive a set of equations which is taking into account the equations
of motion, constitutive equations and the compatibility equations simultaneously. This formulation is
called Stress Formulation. And for the particular case in which the problem is static and
without body force the stress formulation is called Beltrami’s equation.

University of Castilla-La Mancha Draft By: Eduardo W. V. Chaves (2016)


Ciudad Real – Spain
5 INTRODUCTION TO: CONSTITUTIVE EQUATIONS, IBVP, AND IBVP SOLUTION STRATEGIES 465

Problem 5.16
a) Show that the governing equations for a homogeneous isotropic linear elastic material,
(see equations in (5.47)), can be replaced by six equations and six unknowns ( σ ij ), (Stress
Formulation), i.e.:
Indicial notation
2(λ + µ ) λ
σij ,kk + σ kk ,ij − σll ,kkδ ij = 2[( ρu
&&i ), j ]sym − 2[( ρbi ), j ]sym
(2 µ + 3λ ) (2 µ + 3λ )
Tensorial notation
2(λ + µ ) r r λ r
∇ 2xr σ + ∇ x [∇ x [Tr (σ )]] − &r&)][
∇ 2xr [Tr (σ )]1 = 2[∇ xr ( ρu ]sym − 2[∇ xr ( ρb)]sym
(2 µ + 3λ) (2 µ + 3λ )
(5.109)
where ∇ 2xr σ ≡ ∇ xr ⋅ (∇ xr σ ) and ∇ 2xr [Tr (σ )] ≡ ∇ xr ⋅ [∇ xr [Tr (σ )]] .
b) or by:
Indicial notation
2(λ + µ ) −λ
σij ,kk + σ kk ,ij = [( ρbk ),k − ( ρu
&&k ),k ]δ ij + 2[( ρu
&&i ), j ]sym − 2[( ρbi ), j ]sym
(2 µ + 3λ) ( 2 µ + λ)
Tensorial notation
2(λ + µ ) r r −λ r r
∇ 2xr σ + ∇ x [∇ x [Tr (σ )]] = &r&)] 1 + 2[∇ r ( ρu
∇ xr ⋅[( ρb) − ( ρu &r&)]sym − 2[∇ r ( ρb)]sym
(2 µ + 3λ) (2 µ + λ)
x x

(5.110)
Eν E
c) Considering that λ = and µ = , express the equations (5.109) and
(1 + ν )(1 − 2ν ) 2(1 + ν )
(5.110) in function of ( E ,ν ) .
r
Hint: The kinematic equations ε = ∇ sym u can be considered by using the equation, (see
Problem 5.11):
ε ij , kl + ε kl ,ij − ε il , jk − ε jk ,il = O ijkl (5.111)
Solution: a) We can obtain the inverse of the constitutive equation in stress ( σ = C e : ε ):
−1 −1 −1
Ce : σ = Ce : C e : ε = I sym : ε = ε sym = ε ⇒ ε = Ce :σ
For isotropic materials, (see equation (5.26)), the strain tensor can be obtained as follows:
1 λ 1 λ
ε= σ− Tr (σ )1 indicial
 → ε ij = σ ij − σ qqδ ij .
2µ 2 µ (2 µ + 3λ ) 2µ 2 µ (2 µ + 3λ )
As we are considering a homogeneous material, the mechanical properties do not vary with
r ∂λ ∂µ
x , i.e. λ ,i ≡ = 0 i and µ ,i ≡ = 0 i , then:
∂x i ∂xi

∂ 2 ε ij  1 λ  1 λ
≡ ε ij , kl =  σij − σ qqδ ij  = σ ij ,kl − σ qq , klδ ij (5.112)
∂xk ∂xl  2µ 2 µ (2 µ + 3λ)  ,kl 2 µ 2 µ ( 2 µ + 3λ)

Moreover, if we multiply the equation in (5.111) (“kinematic equations”) by δ jk we can


obtain:

University of Castilla- La Mancha Draft By: Eduardo W. V. Chaves (2016)


Ciudad Real – Spain
466 SOLVING PROBLEMS BY MEANS OF CONTINUUM MECHANICS

ε ij , kl δ jk + ε kl ,ij δ jk − ε il , jk δ jk − ε jk ,il δ jk = O ijkl δ jk


(5.113)
⇒ ε ik ,kl + ε kl ,ik − ε il , kk − ε kk ,il = 0 il
Note that, according to equation (5.112) the following is true:
1 λ 1 λ
ε ik , kl = σ ik , kl − σ qq , klδ ik = σ ik ,kl − σ qq ,il
2µ 2 µ (2 µ + 3λ ) 2µ 2 µ (2 µ + 3λ)
1 λ 1 λ
ε kl ,ik = σ kl ,ik − σ qq ,ikδ kl = σ lk ,ki − σ qq ,il
2µ 2 µ (2 µ + 3λ ) 2µ 2 µ (2 µ + 3λ)
1 λ
ε il , kk = σ il ,kk − σ qq , kkδ il
2µ 2 µ (2 µ + 3λ )
1 λ 1 3λ
ε kk ,il = σ kk ,il − σ qq ,il δ kk = σ kk ,il − σ qq ,il
2µ 2 µ (2 µ + 3λ) {
=3
2 µ 2 µ ( 2 µ + 3λ )
1 3λ  1 3λ  2µ
= σ qq ,il − σ qq ,il =  − σ qq ,il = σ qq ,il
2µ 2 µ ( 2 µ + 3λ)  2 µ 2 µ (2 µ + 3λ)  2 µ (2 µ + 3λ)

∂ 2 ε ij 1 λ
≡ ε ij , kl = σ ij , kl − σ qq , klδ ij
∂xk ∂xl 2µ 2 µ (2 µ + 3λ)
With that the equation in (5.113) becomes:
ε ik , kl + ε kl ,ik − ε il , kk − ε kk ,il = 0 il
1  2λ λ 2µ 
 σik , kl − σ qq ,il + σ lk , ki − σ il , kk + σ qq ,kkδ il − σ qq ,il  = 0 il
2µ  (2 µ + 3λ) (2 µ + 3λ) (2 µ + 3λ) 
 2λ 2µ  λ
⇒ σ ik ,kl −  + σ qq ,il + σ lk , ki − σ il , kk + σ qq , kkδ il = 0 il
 ( 2 µ + 3λ) (2 µ + 3λ)  (2 µ + 3λ)
2( µ + λ) λ
⇒ σ ik ,kl − σ qq ,il + σ lk , ki − σil , kk + σ qq , kkδ il = 0 il
(2 µ + 3λ) ( 2 µ + 3λ)

− 2( µ + λ ) λ
⇒ σ qq ,il − σ il , kk + σ qq ,kkδ il = −σ ik , kl − σ lk ,ki (5.114)
(2 µ + 3λ ) (2 µ + 3λ)

From the equations of motion σ ij , j + ρ b i = ρ u


&& i we can obtain:

σ ij , jk + (ρ b i ) , k = (ρ u
&& i ) ,k

with that the following is true:


σ ik ,kl + (ρ b i ) ,l = (ρ u
&& i ) ,l ⇒ − σ ik ,kl = (ρ b i ) ,l − (ρ u
&& i ) ,l

σ lk , ki + (ρ b l ) ,i = (ρ u
&& l ) ,i ⇒ − σ lk ,ki = (ρ b l ) ,i − (ρ u
&& l ) ,i .

And note that − σik , kl − σlk , ki = ( ρbi ),l − ( ρu&&i ),l + ( ρbl ),i − ( ρu&&l ),i = 2[( ρbi ),l ]sym − 2[( ρu&&i ),l ]sym
By replacing the above equation into the equation (5.114) we can obtain:
− 2( µ + λ ) λ
σ qq ,il − σ il , kk + σ qq , kkδ il = 2[( ρb i ) ,l ]sym − 2[( ρu
&&i ) ,l ]sym
(2 µ + 3λ) (2 µ + 3λ)
Restructuring the above and considering that ( l = j ) we can obtain:

University of Castilla-La Mancha Draft By: Eduardo W. V. Chaves (2016)


Ciudad Real – Spain
5 INTRODUCTION TO: CONSTITUTIVE EQUATIONS, IBVP, AND IBVP SOLUTION STRATEGIES 467

2( µ + λ ) λ
σ ij ,kk + σ kk ,ij − σ ll ,kkδ ij = 2[( ρu
&&i ) , j ]sym − 2[( ρb i ), j ]sym (5.115)
(2 µ + 3λ ) (2 µ + 3λ )

Q.E.D.
which matches the equation in (5.109). The above equation could have been obtained by
means of equation in (5.79):

∇ xr ⋅ (∇ xr ε ) + ∇ xr [∇ xr [Tr (ε )]] = ∇ xr (∇ xr ⋅ ε ) + [∇ xr (∇ xr ⋅ ε )]
T
(5.116)

1 λ
where ε = σ− Tr (σ )1 , and is also true that
2µ 2 µ (2 µ + 3λ)

  1 λ 
∇ xr ⋅ (∇ xr ε ) = ∇ xr ⋅ ∇ xr  σ− Tr (σ )1  
  2µ 2 µ (2 µ + 3λ)  ;
1 r λ
= ∇ x ⋅ (∇ xr σ ) − ∇ xr ⋅ ∇ xr [ Tr (σ )]1
2µ 2 µ (2 µ + 3λ)
1 1
Tr (ε ) = Tr (σ ) ⇒ ∇ xr [∇ xr [Tr (ε )]] = ∇ xr [∇ xr [Tr (σ )]] ;
(2 µ + 3λ ) (2 µ + 3λ )
1 r λ 1 r λ
∇ xr ⋅ ε = ∇ x ⋅σ − ∇ xr ⋅ ( Tr (σ )1) = ∇ x ⋅σ − ∇ xr ( Tr (σ ))
2µ 2 µ (2 µ + 3λ) 2µ 2 µ (2 µ + 3λ)
Note that [∇ xr ⋅ ( Tr (σ )1)]i = (σ kkδ ij ) , j = σ kk , jδ ij + σ kkδ ij , j = σ kk , jδ ij = σ kk ,i = [∇ xr ( Tr (σ ))]i ,
r r
and if we consider ∇ xr ⋅ σ + ρb = ρu
&& we can obtain:

1 r λ 1 &r& r λ
∇ xr ⋅ ε = ∇x ⋅σ − ∇ xr ( Tr (σ )) = ( ρu − ρb ) − ∇ xr ( Tr (σ ))
2µ 2 µ (2 µ + 3λ) 2µ 2 µ (2 µ + 3λ)
1 r &r& r λ
⇒ ∇ xr (∇ xr ⋅ ε ) = ∇ x [( ρu − ρb)] − ∇ xr [∇ xr ( Tr (σ ))]
2µ 2 µ (2 µ + 3λ)
with that we can obtain:
∇ xr (∇ xr ⋅ ε ) + [∇ xr (∇ xr ⋅ ε )] = 2[∇ xr (∇ xr ⋅ ε )]
T sym

r 2λ
=
2 &r& − ρb)]}sym −
{∇ xr [( ρu {∇ xr [∇ xr ( Tr (σ ))]}sym
2µ 2 µ (2 µ + 3λ)
r 2λ
=
2 &r& − ρb)]}sym −
{∇ xr [( ρu ∇ xr [∇ xr ( Tr (σ ))]
2µ 2 µ (2 µ + 3λ)
{∇ xr [∇ xr ( Tr (σ ))]}ij = σ kk ,ij = σ kk , ji = {∇ xr [∇ xr ( Tr (σ ))]} ji (symmetric). Taking into account the
above equations into the equation (5.116) we can obtain:
∇ xr ⋅ (∇ xr ε ) + ∇ xr [∇ xr [Tr (ε )]] = ∇ xr (∇ xr ⋅ ε ) + [∇ xr (∇ xr ⋅ ε )]
T

1 r λ 1
⇒ ∇ x ⋅ (∇ xr σ ) − ∇ xr ⋅ ∇ xr [ Tr (σ )]1 + ∇ xr [∇ xr [Tr (σ )]] =
2µ 2 µ (2 µ + 3λ) (2 µ + 3λ)
r 2λ
2 &r& − ρb)]}sym −
{∇ xr [( ρu ∇ xr [∇ xr ( Tr (σ ))]
2µ 2 µ (2 µ + 3λ)

University of Castilla- La Mancha Draft By: Eduardo W. V. Chaves (2016)


Ciudad Real – Spain
468 SOLVING PROBLEMS BY MEANS OF CONTINUUM MECHANICS

λ 2µ
⇒ ∇ xr ⋅ (∇ xr σ ) − (∇ xr ⋅ ∇ xr [ Tr (σ )])1 + ∇ xr [∇ xr [Tr (σ )]] =
(2 µ + 3λ) (2 µ + 3λ)
r 2λ
&r& − ρb)]}sym −
2{∇ xr [( ρu ∇ xr [∇ xr ( Tr (σ ))]
(2 µ + 3λ)
λ 2( µ + λ) r r r
⇒ ∇ 2xr σ − ∇ 2xr [ Tr (σ )] 1 + &r& − ρb)]}sym
∇ x [∇ x [Tr (σ )]] = 2{∇ xr [( ρu
(2 µ + 3λ) (2 µ + 3λ)
which matches the equation in (5.115) or (5.109).
b) Starting from the equation (5.115):
2( µ + λ ) λ
σ ij ,kk + σ kk ,ij = σ ll ,kkδ ij + 2[( ρu
&&i ) , j ]sym − 2[( ρb i ) , j ] sym (5.117)
(2 µ + 3λ) (2 µ + 3λ)
Our goal now is to obtain an expression for σ ll, kk . If we multiply equation (5.111) by
δ jk δ li we can obtain:
ε ij ,klδ jkδ li + ε kl ,ijδ jkδ li − ε il , jkδ jkδ li − ε jk ,ilδ jkδ li = O ijklδ jkδ li
⇒ ε ij , ji + ε ji ,ij − ε ii , jj − ε jj ,ii = 2ε ij ,ij − 2ε ii , jj = 0 (5.118)
⇒ ε ij ,ij − ε ii , jj = 0
If we use the inverse of the constitutive equation, (see equation (5.112)), we can obtain:
1 λ 1 λ
ε ij ,ij = σ ij ,ij − σ qq ,ijδ ij = σ ij ,ij − σ qq ,ii
2µ 2 µ (2 µ + 3λ) 2µ 2 µ (2 µ + 3λ)
(5.119)
1 λ  2µ 
ε ii ,kk = σ ii , kk − σ qq , kkδ ii =  σ ii , kk
2µ 2 µ ( 2 µ + 3λ)  2 µ (2 µ + 3λ) 
With that the equation in (5.118) becomes:
⇒ ε ij ,ij − ε ii , jj = 0
1 λ  2µ 
⇒ σ ij ,ij − σ qq ,ii −  σ ii ,kk = 0
2µ 2 µ (2 µ + 3λ)  2 µ (2 µ + 3λ) 
 λ 2µ  (5.120)
⇒ σ ij ,ij −  + σ ii , kk = 0
 (2 µ + 3λ) (2 µ + 3λ) 
 2µ + λ 
⇒ σ ij ,ij =  σ ii , kk
 (2 µ + 3λ) 
The above equation can also be written in terms of ν
2µ + λ 1−ν
σij , ij = σii , kk ; σij , ij = σii , kk
2 µ + 3λ 1+ν
(5.121)
2µ + λ r 1−ν r
∇ xr ⋅ (∇ xr ⋅ σ ) = ∇ x ⋅ [∇ xr [Tr (σ )]] ; ∇ xr ⋅ (∇ xr ⋅ σ ) = ∇ x ⋅ [∇ xr [Tr (σ )]]
2 µ + 3λ 1+ν

Now, by means of the equations of motion σ ij , j + ρ b i = ρ u


&& i we can obtain:

σ ij , ji + (ρ b i ) ,i = (ρ u
&& i ) ,i ⇒ σ ij , ji = (ρ u
&& i ) ,i − (ρ b i ) ,i

With that the equation in (5.120) becomes:

University of Castilla-La Mancha Draft By: Eduardo W. V. Chaves (2016)


Ciudad Real – Spain
5 INTRODUCTION TO: CONSTITUTIVE EQUATIONS, IBVP, AND IBVP SOLUTION STRATEGIES 469

 2µ + λ 
σ ij ,ij =  σ ii ,kk
 (2µ + 3λ ) 
 2µ + λ 
⇒ (ρ u && i ) ,i − (ρ b i ) ,i = 
 (2µ + 3λ ) σ ii ,kk (5.122)
 
(2µ + 3λ ) (2µ + 3λ )
⇒ σ ii ,kk = σ ll ,kk =
2µ + λ
(ρ u [
&& k ) ,k − (ρ b k ) ,k = − ]2µ + λ
[
( ρ b k ) , k − (ρ u
&& k ) , k ]
Replacing equation (5.122) into (5.117), we can obtain:
2( µ + λ) λ
σij ,kk + σkk ,ij = σll ,kkδ ij + 2[( ρu
&&i ), j ]sym − 2[( ρbi ), j ]sym
(2 µ + 3λ) (2 µ + 3λ)
2( µ + λ) −λ (2 µ + 3λ)
σij ,kk + σkk ,ij = [( ρb k ),k − ( ρu
&&k ),k ]δ ij + 2[( ρ&u&i ), j ]sym − 2[( ρbi ), j ]sym
(2 µ + 3λ) (2 µ + 3λ) 2 µ + λ
2( µ + λ) −λ
⇒ σij ,kk + σkk ,ij = [( ρb k ),k − ( ρu
&&k ),k ]δ ij + 2[( ρu
&&i ), j ]sym − 2[( ρbi ), j ]sym
(2 µ + 3λ) (2 µ + λ)

Q.E.D. (5.123)
Thus obtaining the equation in (5.110)
c) After some algebraic manipulations we can obtain:
1 (1 − 2ν ) λ (1 − 2ν ) Eν ν
= ; = = ;
(2 µ + 3λ) E (2 µ + 3λ) E (1 + ν )(1 − 2ν ) (1 + ν )
µ (1 − 2ν ) E (1 − 2ν ) 2( µ + λ ) ν (1 − 2ν ) 1
= = ; =2 +2 = ,
(2 µ + 3λ ) E 2(1 + ν ) 2(1 + ν ) (2 µ + 3λ) (1 + ν ) 2(1 + ν ) (1 + ν )
E Eν E (1 − ν )
( 2 µ + λ) = 2 + = ;
2(1 + ν ) (1 + ν )(1 − 2ν ) (1 + ν )(1 − 2ν )
λ Eν (1 + ν )(1 − 2ν ) ν 2µ + λ 1 − ν
= = ; = ,
(2 µ + λ) (1 + ν )(1 − 2ν ) E (1 − ν ) (1 − ν ) 2 µ + 3λ 1 + ν
whereby the equation (5.109) becomes:
1 ν
σij , kk + σ kk ,ij − σll , kkδ ij = 2[( ρu
&&i ), j ]sym − 2[( ρbi ), j ]sym
(1 + ν ) (1 + ν )
Tensorial notation
ν r
∇ 2xr σ +
1
∇ xr [∇ xr [Tr (σ )]] − &r&)]sym − 2[∇ r ( ρb)]sym
∇ 2xr [Tr (σ )]1 = 2[∇ xr ( ρu x
(1 + ν ) (1 + ν )
(5.124)
and the equation (5.110) becomes:
1 −ν
σij , kk + σ kk ,ij = [( ρb k ), k − ( ρ&u&k ), k ]δ ij + 2[( ρu
&&i ), j ]sym − 2[( ρbi ), j ]sym
(1 + ν ) (1 − ν )
Tensorial notation
−ν r r
∇ 2xr σ +
1
∇ xr [∇ xr [Tr (σ )]] = &r&)] 1 + 2[∇ r ( ρu
∇ xr ⋅ [( ρb) − ( ρu &r&)]sym − 2[∇ r ( ρb)]sym
x x
(1 + ν ) (1 − ν )
(5.125)

University of Castilla- La Mancha Draft By: Eduardo W. V. Chaves (2016)


Ciudad Real – Spain
470 SOLVING PROBLEMS BY MEANS OF CONTINUUM MECHANICS

NOTE 1: For a static problem the above equation becomes:


1 −ν
σij , kk + σ kk ,ij = [( ρb k ), k ]δ ij − 2[( ρbi ), j ]sym
(1 + ν ) (1 − ν ) Michell’s
(5.126)
1 −ν r r equations
∇ 2xr σ + ∇ xr [∇ xr [Tr (σ )]] = [∇ xr ⋅ ( ρb)] 1 − 2[∇ xr ( ρb)]sym
(1 + ν ) (1 − ν )

which are known as Michell’s equations, which were obtained by Michell in 1900.
r
If the body forces do not vary with x , the Michell’s equations (5.126) reduce to:
1
σij , kk + σ kk ,ij = 0ij
(1 + ν )
Beltrami’s equations (5.127)
1
∇ 2xr σ + ∇ xr [∇ xr [Tr (σ )]] = 0
(1 + ν )

which are known as Beltrami’s equations, which were obtained by Beltrami in 1892, (see
Sokolnikoff (1956) first edition in (1946)).
∂ 2σij 1  ∂( ρbi ) ∂( ρb j ) 
If we take into account that σij , kk = = ∇ 2xr σij , [( ρbi ), j ]sym = + , and
∂xk ∂xk 2  ∂x j ∂xi 
r
[( ρb k ), k ]δ ij = [][∇ xr ⋅ ( ρb)]δ ij , the Michell’s equations can be rewritten explicitly as follows:

1 −ν
σij , kk + σkk ,ij = [( ρb k ), k ]δ ij − 2[( ρbi ), j ]sym
(1 + ν ) (1 − ν )
1 ∂ 2 [ Tr (σ )] −ν r  ∂ ( ρbi ) ∂ ( ρb j ) 
⇒ ∇ 2xr σij + = [∇ xr ⋅ ( ρb)]δ ij −  + 
(1 + ν ) ∂xi ∂x j (1 − ν )  ∂x j ∂xi 

Then, the above six equations are:
 2 1 ∂ 2 [ Tr (σ )] −ν r ∂( ρb1 )
∇ rσ +
 x 11 = [∇ xr ⋅ ( ρb)] − 2
 (1 + ν ) ∂x1 2
(1 − ν ) ∂x1
 2
1 ∂ [ Tr (σ )] −ν r ∂ ( ρb 2 )
∇ 2xr σ22 + = [∇ xr ⋅ ( ρb)] − 2
 (1 + ν ) ∂x2 2
(1 − ν ) ∂x2
 r
∇ 2r σ + 1 ∂ [ Tr (σ )] = − ν [∇ r ⋅ ( ρb)] − 2 ∂ ( ρb3 )
2

 x 33
(1 + ν ) ∂x32 (1 − ν )
x
∂x3

 (5.128)
∇ 2r σ + 1 ∂ [ Tr (σ )] = − ∂ ( ρb1 ) + ∂ ( ρb 2 ) 
2

 x 12 (1 + ν ) ∂x1∂x2  ∂x ∂x1 
 2

 2 1 ∂ 2 [ Tr (σ )]  ∂ ( ρb 2 ) ∂ ( ρb3 ) 
∇ rσ +
 x 23 = − + 

 (1 + ν ) ∂x 2 ∂ x3  ∂ x3 ∂ x2 

∇ 2xr σ13 + 1 ∂ [ Tr (σ )] = − ∂ ( ρb1 ) + ∂( ρb3 ) 
2
 
 (1 + ν ) ∂x1∂x3  ∂x1 
 ∂x3

University of Castilla-La Mancha Draft By: Eduardo W. V. Chaves (2016)


Ciudad Real – Spain
5 INTRODUCTION TO: CONSTITUTIVE EQUATIONS, IBVP, AND IBVP SOLUTION STRATEGIES 471

NOTE 2: For a static problem (u&& k = 0 k ) , the equation in (5.122) becomes:


(2 µ + 3λ) (1 + ν ) − (1 + ν ) r r
σll , kk = − ( ρb k ) , k = − ( ρb k ) , k ∇ xr ⋅ {∇ xr [Tr (σ )]} = ∇ x ⋅ ( ρ b)
2µ + λ (1 − ν ) (1 − ν )
r (5.129)
− (1 + ν ) r
∇ 2xr [Tr (σ )] = ∇ x ⋅ ( ρb)
(1 − ν )
The above equation can also be obtained by means of equation (5.126) with ( i = j ), i.e.: we
are obtaining the trace of (5.126):
1 −ν
σii , kk + σkk ,ii = [( ρb k ), k ]δ ii − 2[( ρbi ),i ]
(1 + ν ) (1 − ν ) {
=3

 1   − 3ν 
⇒ 1 + σii , kk =  − 2 [( ρb k ), k ] (5.130)
 (1 + ν )   (1 − ν ) 
 (2 + ν )   (2 + ν )  (1 + ν )
⇒  σii , kk = − [( ρb k ), k ] ⇒ σii , kk = − [( ρb k ), k ]
 (1 + ν )   (1 − ν )  (1 − ν )
Note that σ ii ,kk = σ kk ,ii and (ρ b k ) ,k = (ρ b i ) ,i . The above equation in tensorial notation
becomes:
r r
(∇ 2xr σ ) : 1 +
1
{∇ xr [∇ xr [Tr(σ )]]} : 1 = − ν [∇ xr ⋅ ( ρb)] 1 : 1 − 2[∇ xr ( ρb)]sym : 1 (5.131)
(1 + ν ) (1 − ν )

Note that
(∇ 2xr σ ) : 1 ≡ {∇ xr ⋅ (∇ xr σ )}: 1 = ∇ xr ⋅ [∇ xr [Tr(σ )]] ≡ ∇ 2xr [Tr(σ )]
{∇ xr [∇ xr [Tr(σ )]]}: 1 = ∇ xr ⋅ [∇ xr [Tr(σ )]] ≡ ∇ 2xr [Tr(σ )]
1 :1 = 3
[ r sym
∇ xr ( ρb) ] r
: 1 = ∇ xr ⋅ ( ρb)
with that the equation (5.131) becomes:
1 − 3ν r r
∇ 2xr [Tr (σ )] + ∇ 2xr [Tr (σ )] = [∇ xr ⋅ ( ρb)] − 2[∇ xr ⋅ ( ρb)]
(1 + ν ) (1 − ν )
− (1 + ν ) r r
⇒ ∇ 2xr [Tr (σ )] = [∇ x ⋅ ( ρb)]
(1 − ν )
NOTE 3: For the two-dimensional elasticity case, the stress formulation is provided in
Problem 6.34.

Problem 5.17
Consider a static linear elastic problem, and also that the mass density ( ρ ) and the
r
mechanical properties ( λ, µ ) are homogeneous fields, and that the specific body force b is
a conservative and homogeneous field. Show that the Cauchy stress tensor, the
infinitesimal strain tensor, and the displacement components are biharmonic functions.
Solution:

University of Castilla- La Mancha Draft By: Eduardo W. V. Chaves (2016)


Ciudad Real – Spain
472 SOLVING PROBLEMS BY MEANS OF CONTINUUM MECHANICS

r r
Taking into account the static problem, the equations of motion ∇ xr ⋅ σ + ρb = ρu
&& becomes
r r
the equilibrium equations ∇ xr ⋅ σ + ρb = 0 , and by applying the divergence to it we can
obtain:
Tensorial notation Indicial notation
r r
∇ xr ⋅ σ + ρb = 0 σij , j + ρbi = 0 i
r
⇒ ∇ xr ⋅ (∇ xr ⋅ σ ) + ∇ xr ⋅ ( ρb) = 0 ⇒ σij , ji + ( ρbi ),i = 0 (5.132)
1424 3 123
=0
=0

⇒ ∇ xr ⋅ (∇ xr ⋅ σ ) = 0 ⇒ σij ,ij = 0
r r
where we have considered that the body force density ( ρb) does not change with x
(homogeneous field).
If we take into account the equation in (5.121) we can conclude that:
Tensorial notation Indicial notation
1−ν r 1−ν
∇ xr ⋅ (∇ xr ⋅ σ ) = ∇ x ⋅ [∇ xr [Tr (σ )]] = 0 σ ij ,ij = σ ii ,kk = 0
1+ν 1+ν (5.133)
⇒ ∇ xr ⋅ [∇ xr [Tr (σ )]] ≡ ∇ 2xr [Tr (σ )] = 0 ∂ 2 σ ii
⇒ σ ii ,kk = = ∇ 2xr (σ ii ) = 0
∂xk ∂xk
with that we show that [ Tr (σ )] is harmonic function. Then it is easy to show that [ Tr (ε )]
 2µ 
is also harmonic function, since [ Tr (σ )] = 3 λ +  [ Tr (ε )] , (see Problem 5.5 NOTE 8):
 3 

  2µ  
∇ 2xr [Tr (σ )] = ∇ 2xr 3 λ +  [ Tr (ε )] = 0 ⇒ ∇ 2xr [ Tr (ε )] = 0 (5.134)
  3  
If we apply the Laplacian to the Beltrami’s equations (5.127) we can obtain:
Tensorial notation Indicial notation
1 1
∇ 2xr σ
+ ∇ xr [∇ xr [Tr (σ )]] = 0 σij , kk + σkk ,ij = 0ij
(1 + ν ) (1 + ν )
1 1
⇒ ∇ 2xr ∇ 2xr σ + ∇ 2xr {∇ xr [∇ xr [Tr (σ )]]} = 0 ⇒ σij , kkpp + σkk ,ijpp = 0ij
(1 + ν ) (1 + ν )
  1 (5.135)
1 ⇒ σij , kkpp + (σkk , pp ),ij = 0ij
⇒ ∇ 2xr ∇ 2xr σ + ∇ xr ∇ xr [ ∇ 2xr [ Tr (σ )] ] = 0 (1 + ν ) 123
(1 + ν )  14243
 =0
 =0
∂ 2  ∂ 2 (σij ) 
⇒ ∇ 2xr ∇ 2xr σ ≡ ∇ 4xr σ = 0 ⇒ σij , kkpp =  =0
∂xk ∂xk  ∂x p ∂x p 
 
⇒ ∇ 2xr ∇ 2xr (σij ) ≡ ∇ 4xr (σij ) = 0ij
With that we show that the Cauchy stress tensor is biharmonic function, where the
operator ∇ 4xr ≡ ∇ 2xr ∇ 2xr is known as the bilaplacian. We can show that the infinitesimal strain
tensor is also biharmonic function, i.e.: ∇ 4xr ε = 0 . Taking into account the above equation
and the constitutive equation in stress for isotropic linear elastic material
σ = λTr (ε )1 + 2 µε we obtain:

University of Castilla-La Mancha Draft By: Eduardo W. V. Chaves (2016)


Ciudad Real – Spain
5 INTRODUCTION TO: CONSTITUTIVE EQUATIONS, IBVP, AND IBVP SOLUTION STRATEGIES 473

∇ 4xr σ = 0
⇒ ∇ 4xr (λTr (ε )1 + 2 µε ) = ∇ 4xr (λTr (ε )1) + ∇ 4xr (2 µε ) = ∇ 4xr ( Tr (ε ))λ1 + 2 µ∇ 4xr (ε ) = 0
14243
=0

⇒ ∇ 4xr ε =0
To show that the displacement components are biharmonic function, we will start with the
Navier’s equations (5.49) for a static case, (λ + µ )u j , ji + µui , jj + ρbi = ρu
&&i = 0 i , and if we
apply the Laplacian to it we can obtain:
(λ + µ )u j , jikk + µui , jjkk + ( ρbi ), kk = 0i
1424 3 (5.136)
= 0i
r
where we have considered that λ , µ and ( ρbi ) do not change with x . Note also that
r
ui , jjk = ui , kjj = ∇ 2xr (ui , k ) = {∇ 2xr [∇ xr u]}ik

and if we consider the infinitesimal strain tensor:


1 1
εij = (ui , j + u j ,i ) trace
→ ε kk = (uk , k + uk , k ) = uk , k
2 2
trace r
⇒ ε kk ,ij = uk , kij  → ε kk , pp = uk , kpp = 0 tensorial
 → ∇ 2xr [ Tr (ε )] = ∇ 2xr [ Tr (∇ xr u) ] = 0

where we have used the equation (5.134). Then, the equation in (5.136) can be written as
follows:
(λ + µ )u j , jikk + µui , jjkk = 0i ⇒ (λ + µ )(u j , jkk ),i + µui , jjkk = 0i
12 3
=0

∇ 4xr u1 =0 (5.137)
 4
⇒ ui , jjkk = ∇ 4xr ui = 0i ⇒ ∇ xr u 2 = 0
 4r
∇ x u3 = 0
with that we show that the displacement components are biharmonic functions.

Problem 5.18
a) Given a scalar field Φ such as:
∂ 2Φ ∂ 2Φ − ∂ 2Φ
σ11 = ; σ 22 = ; σ12 = σ 21 = (5.138)
∂x 22 ∂x12 ∂x1 ∂x 2
Show that
Indicial notation Tensorial notation
Φ , iijj = 0 (i, j = 1,2) ∇ ⋅ {∇ [∇ ⋅ (∇Φ )]} = 0
⇒ ∇ 2∇ 2 Φ = 0
(5.139)
∂ 4Φ ∂ 4Φ ∂ 4Φ
⇒ 4
+2 2 2 + 4 =0
∂x1 ∂x1 x2 ∂x2 ⇒ ∇ 4Φ = 0
Consider a linear elastic material, a static problem, and with no body forces. Consider also
that the Cauchy stress tensor is only dependent of x1 and x 2 , i.e. σ = σ ( x1 , x 2 ) .
b) Show whether the equilibrium equations are satisfied or not.
Solution:
a) In Problem 5.16, (see equation (5.130)), we have shown that:

University of Castilla- La Mancha Draft By: Eduardo W. V. Chaves (2016)


Ciudad Real – Spain
474 SOLVING PROBLEMS BY MEANS OF CONTINUUM MECHANICS

− (1 + ν )
σii , kk =
(1 − ν )
[ ]
( ρb k ) , k = 0

where we have considered that (ρ b k ) ,k = 0 . For the proposed problem we have i, k = 1,2 ,
with which:
σ ii ,kk = 0 ⇒ σ ii ,11 + σ ii , 22 = 0 ⇒ σ11,11 + σ 22,11 + σ11, 22 + σ 22, 22 = 0
∂ 2 σ11 ∂ 2 σ 22 ∂ 2 σ11 ∂ 2 σ 22
⇒ + + + =0
∂x12 ∂x12 ∂x 22 ∂x 22
Using the definition given by (5.138), we can conclude that:
∂ 2 σ11 ∂ 2 σ 22 ∂ 2 σ11 ∂ 2 σ 22
+ + + =0
∂x12 ∂x12 ∂x 22 ∂x 22
∂ 2 ∂ 2Φ ∂ 2 ∂ 2Φ ∂ 2 ∂ 2Φ ∂ 2 ∂ 2Φ
⇒ + + + =0
∂x12 ∂x 22 ∂x12 ∂x12 ∂x 22 ∂x 22 ∂x 22 ∂x12
∂ 4Φ ∂ 4Φ ∂ 4Φ
⇒ + 2 + =0
∂x14 ∂x12 ∂x 22 ∂x 24
Q.E.D.

b) For the bidimensional case (2D), the equilibrium equations (without body forces) reduce
to:
 ∂σ11 ∂σ12
 ∂x + ∂x = 0
i , j =1, 2 )  1 2
σ ij , j = 0 i (  → σ i1,1 + σ i 2, 2 = 0 i ⇒ 
 ∂σ 21 + ∂σ 22 = 0
 ∂x1 ∂x 2
Using the definition (5.138), we can obtain:
 ∂σ11 ∂σ12  ∂ ∂ 2Φ ∂ ∂ 2Φ ∂ 3Φ ∂ 3Φ
 ∂x + ∂x = 0  − = − =0 X
 ∂x1 ∂x2 ∂x2 ∂x1 ∂x2 ∂x1∂x2 ∂x1 ∂x2
2 2 2
 1 2
 ⇒ 
 ∂σ21 + ∂σ22 = 0
2 2 3 3
− ∂ ∂ Φ + ∂ ∂ Φ = − ∂ Φ + ∂ Φ = 0 X
 ∂x1 ∂x2  ∂x ∂x ∂x ∂x2 ∂x12 ∂x12 ∂x2 ∂x2∂x12
 1 1 2

With this, we show that the expressions for stresses given by (5.138) satisfy the equilibrium
equations.
NOTE: In the literature, Φ is known as the Airy stress function, (see Problem 6.34), and the
SI unit of Φ is [Φ] = N ( Newton) .

Problem 5.19
Let us consider that the Cauchy stress tensor field can be obtained as follows:
r r N
σ = ∇ xr ∧ (∇ xr ∧ P )T [σ ] = = Pa( Pascal ) (5.140)
m2
where the second-order tensor P has the following Cartesian components:

University of Castilla-La Mancha Draft By: Eduardo W. V. Chaves (2016)


Ciudad Real – Spain
5 INTRODUCTION TO: CONSTITUTIVE EQUATIONS, IBVP, AND IBVP SOLUTION STRATEGIES 475

χ 1 0 0
Pij =  0 χ 2 0  ∴ [ P ] = N ( Newton)
 0 0 χ 3 
a) Obtain the explicit components of the stress tensor in function of χ i . b) Check whether
the body is in equilibrium by considering the static state and without body force.
Solution:
a) In Problem 1.110 we have shown that the following is true:
r r
σ qt = [∇ xr ∧ (∇ xr ∧ P ) T ] qt =  qjk  til Pij , kl

Note also that the explicit equations for σ qt , (given by (5.140)), have the same structure as
the one used to obtain the components Sij in equation (5.80), (see Problem 5.11), so

 ∂ 2 P33 ∂ 2 P22 ∂ 2 P23


σ
 11 = + − 2
 ∂x22 ∂x32 ∂x2 ∂x3
 2
∂ P33 ∂ P11 2
∂ 2 P13
σ22 = + − 2
 ∂x12 ∂x32 ∂x1∂x3
 2 2 2
σ = ∂ P11 + ∂ P22 − 2 ∂ P12
 33
∂x22 ∂x12 ∂x1∂x2

 (5.141)
σ = ∂  ∂P23 + ∂P13 − ∂P12  − ∂ P33
2

 ∂x3  ∂x1 ∂x2 ∂x3  ∂x1∂x2


12


 ∂  ∂P23 ∂P13 ∂P12  ∂ 2 P11
σ23 = − 
∂ x  ∂x + ∂x + ∂x  − ∂x ∂x
 1  1 2 3  2 3

σ13 = ∂  ∂P23 − ∂P13 + ∂P12  − ∂ P22
2

 ∂x2  ∂x1 ∂x2 ∂x3  ∂x1∂x3

Taking into account that P11 = χ 1 , P22 = χ 2 , P33 = χ 3 and P12 = P23 = P13 = 0 , the stress
components become:
 ∂ 2χ 3 ∂ 2χ 2  ∂ 2χ 3 ∂ 2χ 2 
 +  − − 
 ∂x2
2
∂x32  ∂x1∂x2 ∂x1∂x3 
 ∂ 2χ 3  ∂ χ 3 ∂ χ1 
2 2
∂ χ1
2 
σ qt =  −  +  −  (5.142)
 ∂x1∂x2  ∂x 2 ∂x32  ∂x2 ∂x3 
 1
 ∂ 2χ 2 ∂ 2χ 1  ∂ 2χ 1 ∂ 2χ 2 
 − −  2 + 
 ∂x1∂x3 ∂x2 ∂x3  ∂x 2 
 2 ∂ x 1  
r
b) We start from the equations of motion ∇ xr ⋅ σ + ρb = ρv& = ρu
r &r& , (see equation (5.14)), in
indicial notation σ ij , j + ρ b i = ρ u
&& i and by considering the static state ( u &&i = 0i ) and without
body force ( bi = 0i ) the equations of motion become the equilibrium equations, namely:

University of Castilla- La Mancha Draft By: Eduardo W. V. Chaves (2016)


Ciudad Real – Spain
476 SOLVING PROBLEMS BY MEANS OF CONTINUUM MECHANICS

σij , j = σi1,1 + σi 2, 2 + σi 3,3 = 0i


 ∂σ11 ∂σ12 ∂σ13
 + + =0
σ11,1 + σ12, 2 + σ13,3 = 0  ∂x1 ∂x2 ∂x3
  ∂σ 21 ∂σ22 ∂σ23
⇒ σ 21,1 + σ 22, 2 + σ 23,3 = 0 ⇒  + + =0
  ∂x1 ∂x2 ∂x3
σ31,1 + σ32, 2 + σ33,33 = 0  ∂σ31 ∂σ32 ∂σ33
 + + =0
 ∂x1 ∂x2 ∂x3
And by substituting the stress components given by Eq. (5.142) we can conclude that
 ∂σ11 ∂σ12 ∂σ13 ∂  ∂ 2χ 3 ∂ 2χ 2  ∂  ∂ 2χ 3  ∂  ∂ 2χ 2 
 + + =  + −  −  =0 X
 ∂x1 ∂x2 ∂x3 ∂x1  ∂x22 ∂x32  ∂x2  ∂x1∂x2  ∂x3  ∂x1∂x3 

 ∂σ21 ∂σ 22 ∂σ 23 ∂  ∂ 2χ 3  ∂  ∂ 2χ 3 ∂ 2χ1  ∂  ∂ 2χ1 
     

∂x
+
∂x
+
∂x
=−
∂ x  ∂x ∂x  + ∂x  ∂x 2 + ∂x 2  − ∂x  ∂x ∂x  = 0 X
 1 2 3 1  1 2  2  1 3  3  2 3 

 ∂σ31 + ∂σ32 + ∂σ33 = − ∂  ∂ χ 2  − ∂  ∂ χ1  + ∂  ∂ χ1 + ∂ χ 2  = 0 X
2 2 2 2

 ∂x1
 ∂x2 ∂x3 ∂x1  ∂x1∂x3  ∂x2  ∂x2 ∂x3  ∂x3  ∂x22 ∂x12 

NOTE 1: In the literature χ i are known as stress functions. In the particular case when
χ 3 = Φ and χ1 = χ 2 = 0 we fall back into the two-dimensional problem discussed in
Problem 5.18, where Φ = Φ ( x1, x2 ) is the Airy stress function. In this case the Eq. (5.142)
becomes:
 ∂ 2Φ ∂ 2Φ 
 − 0
 ∂x2
2
∂x1∂x2 
 ∂ 2Φ ∂ 2Φ 
σqt = − 0
 ∂x1∂x2 ∂x12 
 0 0 0
 
 
NOTE 2: Note that the stress field can also be expressed by other stress function ξ , (see
Love (1944)). In this case the P -components are:
 0 ξ3 ξ 2
−1 
Pij = ξ 3 0 ξ 1  ∴ [ P ] = N ( Newton)
2 
ξ 2 ξ 1 0 

By substituting these components into the equation in (5.141) we can obtain:


 ∂ 2ξ 1 − 1 ∂  ∂ξ 1 ∂ξ 2 ∂ξ 3  − 1 ∂  ∂ξ 1 ∂ξ 2 ∂ξ 3  
  + −   − + 
 ∂x2∂x3 2 ∂x3  ∂x1 ∂x2 ∂x3  2 ∂x2  ∂x1 ∂x2 ∂x3  
 − 1 ∂  ∂ξ ∂ξ ∂ξ  ∂ 2ξ 2 − 1 ∂  ∂ξ 1 ∂ξ 2 ∂ξ 3 
σ qt =   1 + 2 − 3 − + + 
  2 ∂x1  ∂x1 ∂x2 ∂x3 
 2 ∂x3  ∂x1 ∂x2 ∂x3  ∂x1∂x3
 − 1 ∂  ∂ξ ∂ξ ∂ξ  − 1 ∂  ∂ξ 1 ∂ξ 2 ∂ξ 3  ∂ 2ξ 3 
  1 − 2 + 3 − + +  

 2 ∂x2  ∂x1 ∂x2 ∂x3  2 ∂x1  ∂x1 ∂x2 ∂x3  ∂x1∂x2 
(5.143)
We leave the reader to check whether the equilibrium equations are satisfied or not.
Let us suppose that ξ 3 = ξ 3 ( x2 , x3 ) and ξ 1 = ξ 2 = 0 , with that the stress field becomes:

University of Castilla-La Mancha Draft By: Eduardo W. V. Chaves (2016)


Ciudad Real – Spain
5 INTRODUCTION TO: CONSTITUTIVE EQUATIONS, IBVP, AND IBVP SOLUTION STRATEGIES 477

 − 1 ∂  ∂ξ 3  − 1 ∂  ∂ξ 3  
 0 −    ∂φ ∂φ 
2 ∂x3  ∂x3  2 ∂x2  ∂x3   0 − 
  ∂x3 ∂x2 
 − 1 ∂  ∂ξ    ∂φ
σ qt =  − 3 
  0 0 = 0 0 
 2 ∂x3  ∂x3    ∂x3 
 − 1 ∂  ∂ξ    ∂φ
  3 0 0  − 0 0 

 2 ∂x2  ∂x3    ∂x2
1 ∂ξ 3
where we have considered that φ = φ ( x2 , x3 ) = . In the literature φ is known as the
2 ∂x3
Prandtl’s stress function, (see 6.4 Introduction to Torsion – Problem 6.44 – NOTE 2).

r r
Additional NOTE: The components of ∇ xr ∧ (∇ xr ∧ P ) (not symmetric) were obtained in
Problem 1.110 and are given by:
r r
[∇ xr ∧ (∇ xr ∧ P )]tj = Psj ,ts − Ptj , ss

then
Psj ,ts − Ptj , ss = P1 j ,t1 + P2 j ,t 2 + P3 j ,t 3 − ( Ptj ,11 + Ptj , 22 + Ptj ,33 )
(t = 1, j = 1) ⇒ P11,11 + P21,12 + P31,13 − ( P11,11 + P11, 22 + P11,33 ) = −(χ1, 22 + χ1,33 )
(t = 2, s = 2) ⇒ P12, 21 + P22, 22 + P32, 23 − ( P22,11 + P22, 22 + P22,33 ) = −(χ 2,11 + χ 2,33 )
(t = 3, j = 3) ⇒ P13,31 + P23,32 + P33,33 − ( P33,11 + P33, 22 + P33,33 ) = −(χ 3,11 + χ 3, 22 )
(t = 1, j = 2) ⇒ P12,11 + P22,12 + P32,13 − ( P12,11 + P12, 22 + P12,33 ) = χ 2,12
(t = 2, j = 3) ⇒ P13, 21 + P23, 22 + P33, 23 − ( P23,11 + P23, 22 + P23,33 ) = χ 3, 23
(t = 1, j = 3) ⇒ P13,11 + P23,12 + P33,13 − ( P13,11 + χ13, 22 + P13,33 ) = χ 3,13
M
Thus
  ∂ 2χ1 ∂ 2χ1  ∂ 2χ 2 ∂ 2χ 3 
 −  2 +  
  ∂x2 ∂x32  ∂x1∂x2 ∂x1∂x3 
r r  ∂ 2
χ  ∂ 2
χ ∂ 2
χ  ∂ 2
χ 
[∇ xr ∧ (∇ xr ∧ P )]tj =  1
−  2
+ 2  3

 ∂x1∂x2  ∂x1
2
∂x32  ∂x2∂x3 
 ∂ χ1
2
∂ χ2
2
∂ χ
2
∂ χ 3 
2
 −  2 3 + 
 ∂x1∂x3 ∂x2 ∂x3  ∂x1 ∂x22 

Note that
r r r r
[∇ xr ∧ (∇ xr ∧ P )T ] ≠ [∇ xr ∧ (∇ xr ∧ P )]

In the case when P is a spherical tensor, e.g. P = α 1 , in which α = α ( x1 , x2 , x3 ) , we can


conclude that
r r r r
P = α1 ⇒ [∇ xr ∧ (∇ xr ∧ P )T ] = −[∇ xr ∧ (∇ xr ∧ P )]

University of Castilla- La Mancha Draft By: Eduardo W. V. Chaves (2016)


Ciudad Real – Spain
478 SOLVING PROBLEMS BY MEANS OF CONTINUUM MECHANICS

Problem 5.20
Consider the governing equation for the linear elastic problem described in Problem 5.5.
r
Obtain an equivalent formulation such as the unknowns are displacement u and stress σ
(Mixed Formulation). Use Voigt notation.
Solution:
Taking into account the governing equations for the elastic linear problem:
Tensorial notation Voigt notation
The equations of motion: The equations of motion:
r
r
r
∇ x ⋅ σ + ρb = ρv& = ρu &r& (3 equations) [ L ] {σ } + {ρb } = {ρu&&} (3 equations)
(1) T

The constitutive equations for stress: The constitutive equations for stress:
(5.144)
σ (ε ) = C e : ε (6 equations) {σ } = [C ] {ε } (6 equations)
The kinematic equations: The kinematic equations:
r
ε = ∇ sym
x u (6 equations)
r {ε } = [ L(1) ] {u } (6 equations)

where the equations in Voigt notation were obtained in Problem 5.8, where
 ∂ ∂ ∂ 
 0 0 0 
 ∂x1 ∂x2 ∂x3 
∂ ∂ ∂
[ L(1) ]T =  0 0 0 
∂x2 ∂x1 ∂x3
 
 0 ∂ ∂ ∂ 
0 0
 ∂x3 ∂x2 ∂x1 
To eliminate the strain from the governing equations, we replace the kinematic equation
into the constitutive equations for stress, i.e.:
{σ } = [C ] {ε } ⇒ {σ } = [C ] [ L(1) ]{u }
⇒ [C ]−1{σ } = [C ]−1[C ] [ L(1) ]{u }
1424 3
=[1 ]
⇒ [C ]−1{σ } − [ L(1) ]{u } = {0 }
whereby the system (5.144) becomes:
 [ L(1) ]T {σ } + {ρb } = { ρu&&}
 −1
 [C ] {σ } − [ L ]{u } = {0 }
(1)

which is also possible to write the above set of equations as follows:


 [0 ] [ L(1) ]T  {u } − {ρb } + {ρu&&}
  = 
− [ L(1) ] [C ]−1  {σ }  {0 } 

NOTE 1: The above formulation is known as Mixed Formulation. It is interesting to note


that in the formulations either in displacement or in stress, (see Problem 5.9 and Problem
5.16), we have second derivative of the unknowns, meanwhile in the mixed formulation we
deal only with the first derivative of the unknowns, and moreover this formulation does
not deal with the derivative of the mechanical properties.

University of Castilla-La Mancha Draft By: Eduardo W. V. Chaves (2016)


Ciudad Real – Spain
5 INTRODUCTION TO: CONSTITUTIVE EQUATIONS, IBVP, AND IBVP SOLUTION STRATEGIES 479

NOTE 2: We can summarize that the linear elastic problem, considering an isotropic
homogenous linear elastic material, is governed by the set of partial differential equations:
Tensorial notation Voigt notation
The equations of motion: The equations of motion:
r r &r& (3 equations)
∇ ⋅ σ + ρ b = ρ v& = ρ u [ L ] {σ } + {ρb } = {ρu&&} (3 equations)
(1) T

The constitutive equations for stress: The constitutive equations for stress:
(5.145)
σ (ε ) = C e : ε (6 equations) {σ } = [C ] {ε } (6 equations)
The kinematic equations: The kinematic equations:
r
ε = ∇ sym u (6 equations) {ε } = [ L(1) ] {u } (6 equations)

making a total of 15 equations and 15 unknowns, namely (u i , σ ij , ε ij ) .


This set of equations can also be represented by:
1) Displacement Formulation, (see Problem 5.9):

(λ + µ )u j , ji + µ u i , jj + ρ b i = ρ u
&& i
Navier’s equations (5.146)
r r r
&r&
(λ + µ )[∇ (∇ ⋅ u)] + µ [∇ ⋅ (∇u)] + ρ b = ρ u

in which we have 3 equations and 3 unknowns (u i ) .


2) Stress Formulation, (see Problem 5.16):
Indicial notation
2(λ + µ ) λ
σij , kk +
(2 µ + 3λ )
σ kk ,ij −
(2 µ + 3λ )
[
σll , kkδ ij = 2 ( ρu
&&i ), j ]
sym
[
− 2 ( ρbi ), j ]
sym

Tensorial notation

∇ 2xr σ +
2(λ + µ ) r r
(2 µ + 3λ)
∇ x [∇ x [Tr (σ )]] −
λ
(2 µ + 3λ)
[ ]r
[
&r&) sym − 2 ∇ r ( ρb) sym
∇ 2xr [Tr (σ )]1 = 2 ∇ xr ( ρu x ]
(5.147)
in which we have 6 equations and 6 unknowns (σ ij ) .
3) Mixed Formulation, (see Problem 5.20):
 [ L(1) ]T {σ } + {ρb } = {ρu&&} (3 equations)
 −1
(5.148)
 [C ] {σ } − [ L ]{u } = {0 }
(1)
(6 equations)

in which we have 9 equations and 9 unknowns ( u i , σ ij ).

University of Castilla- La Mancha Draft By: Eduardo W. V. Chaves (2016)


Ciudad Real – Spain
480 SOLVING PROBLEMS BY MEANS OF CONTINUUM MECHANICS

Problem 5.21
Let us consider two systems made up by the same linear elastic material but with different
load conditions as indicated in Figure 5.14.
r
System I Surface force - t *
r
Specific body force - b
Su B Sσ
r r
r dV t * ( x) Stress field - σ
u* r r
ρ b( x ) Strain field - ε
r
n̂ Displacement field - u

r
System II Surface force - t *
r
Specific body force - b
Su B Sσ
r r r
u* dV t * ( x) Stress field - σ
r
ρb Strain field - ε
r
n̂ Displacement field - u

Figure 5.14: Two systems under external actions.

Show the Betti’s theorem also known as Betti’s reciprocal theorem:

∫ σ : ε dV = ∫ σ : εdV
V V
Betti’s theorem (5.149)

Solution:
Taking into account the constitutive equation for stress, σ = C e : ε , in indicial notation:
e
σ ij = C ijkl ε kl

And by multiplying both sides of the equation by the field ε we can obtain:
σ ij ε ij = ε ij C eijkl ε kl Major
 Simmetry
 of C
→ e
σ ij ε ij = ε ij C ijkl ε kl = ε kl C eklij ε ij
e
where we have applied the major symmetry of the elasticity tensor ( C ijkl = C eklij ). Since the
both systems are made up by the same material the relationship σ = C e : ε holds. With that
the above equation becomes:
σ ij ε ij = ε ij C eijkl ε kl = ε kl C eklij ε ij = ε kl σ kl Tensorial
  notation
→ σ :ε = σ :ε

If now we integrate over the whole volume we can obtain the Betti’s theorem:

∫ σ : ε dV = ∫ σ : ε dV
V V
(5.150)

University of Castilla-La Mancha Draft By: Eduardo W. V. Chaves (2016)


Ciudad Real – Spain
5 INTRODUCTION TO: CONSTITUTIVE EQUATIONS, IBVP, AND IBVP SOLUTION STRATEGIES 481

e
NOTE 1: The above equation is only valid if C ijkl = C eklij holds, i.e. if C e has major
e
symmetry. In other words, the condition C ijkl = C eklij enforces the existence of the stored-
energy function (Ψ e ), such as:
e ∂ 2Ψ e (ε ) ∂ 2Ψ e (ε )
C ijkl = = = C eklij
∂ε ij ∂ε kl ∂ε kl ∂ε ij
NOTE 2: The Betti’s theorem is the start point to obtain the formulation of the Boundary
Element Method.
NOTE 3: The Betti’s theorem can also be expressed in another form which we show
below.
1 ∂u  ∂u j  1
Recall that ε ij =  i +  = (u i , j + u j ,i ) , which is also valid for the system II, i.e.
2  ∂x j ∂xi  2
1
ε ij = ( ui , j + u j ,i ) . Then:
2

∫ σ ε dV = ∫ σ ε dV
V
ij ij
V
ij ij

1 1
2V ∫
σij (ui , j + u j ,i )dV =
2V ∫
σij ( ui , j + u j ,i )dV (5.151)

∫σ u
V
ij i , j dV ∫
= σij ui , j dV
V

where σij u i , j = σ ij u j ,i and σ ij ui , j = σ ij u j ,i hold due to the symmetry of σ and σ ,


respectively. Also note that:
( σ ij u i ), j = σij , j u i + σij u i , j ⇒ σ ij u i , j = ( σ ij u i ), j − σij , j u i
(σ ij ui ), j = σ ij , j ui + σ ij ui , j ⇒ σ ij ui , j = (σ ij ui ), j −σ ij , j ui

With that the equation in (5.151) becomes:

∫σ u
V
ij i , j dV ∫
= σ ij ui , j dV
V

∫ (σ u ),
V
ij i j ∫
− σ ij , j u i dV = (σ ij ui ), j −σ ij , j ui dV
V
(5.152)

∫ (σ u ),
V
ij i j ∫
V

dV − σ ij , j u i dV = (σ ij ui ), j dV − σ ij , j ui dV
V

V

Applying the divergence theorem to the first one integral on both sides of the equation, we
can obtain:

∫ σ u nˆ dS − ∫ σ
S
ij i j
V
ij , j u i dV ∫S

= σ ij ui nˆ j dS − σ ij , j ui dV
V
(5.153)
∫ ∫
⇒ t i u i dS − σ ij , j u i dV = t i ui dS − σ ij , j ui dV
S V

S

V
r r
where we have applied the definition σ ⋅ nˆ = t and σ ⋅ nˆ = t . The above equation in
tensorial notation becomes:
r r r r r r
∫ t ⋅ udS − ∫ (∇ ⋅ σ ) ⋅ udV = ∫ t ⋅ u dS − ∫ (∇ ⋅ σ ) ⋅ u dV (5.154)
S V S V

University of Castilla- La Mancha Draft By: Eduardo W. V. Chaves (2016)


Ciudad Real – Spain
482 SOLVING PROBLEMS BY MEANS OF CONTINUUM MECHANICS

If we resort to the equations of motion, it is satisfied that:


r r r
&r& &&
∇ ⋅σ + ρb = ρu ⇒ − ∇ ⋅ σ = ρ (b − u )
r r r
&r&
∇ ⋅ σ + ρb = ρu ⇒ − ∇ ⋅ σ = ρ (b − u
&&)

with that the equation in (5.154) becomes:


r r r r r r r r r r
&&
∫ t ⋅ udS + ρ (b − u ) ⋅ udV = t ⋅ u dS + ρ (b − u

&&) ⋅ u dV
∫ ∫ Betti’s theorem (5.155)
S V S V

Note that, if we consider S = S u + S σ we have:


r r r r r r
∫ t ⋅ udS = ∫ t * ⋅ udS + t ⋅ u* dS

S Sσ Su
r r r r r r (5.156)
∫ t ⋅ u dS = ∫ t * ⋅ u dS + t ⋅ u * dS

S Sσ Su

For the particular case when the system is in equilibrium and in the absence of body force,
the equation (5.155) becomes:
r r r r
∫ t ⋅ udS = t ⋅ u dS
∫ (5.157)
S S

In addition, if we have concentrated forces instead of surface force, the above equation
becomes:
r r r r
Fi loc u loc
i = Filoc uiloc F loc ⋅ u loc = F loc ⋅ u loc (5.158)

Problem 5.22
Let us consider two systems as described in Figure 5.14. Show the Principle of Virtual
Work which states that:
r* r r r r
∫t ⋅ u dS + ρ (b − u&&) ⋅ u dV = σ : ε dV
∫ ∫

144444244444
3 V V
14243 Principle of Virtual Work (5.159)
Total external virtual work Total internal
virtual work

r r
where u = u* on S u is known (prescribed).
Solution:
We can prove the Principle of Virtual Work by starting directly from the relationship:
1
V
∫σ ij ε ij dV =
2V ∫ ∫
σ ij ( ui , j + u j ,i )dV = σ ij ui , j dV
V
(5.160)

Note that (σ ij ui ), j = σ ij , j ui + σ ij ui , j ⇒ σ ij ui , j = (σ ij ui ), j −σ ij , j ui , thus:

V
∫σ ij ε ij dV ∫ ∫
= σ ij ui , j dV = (σ ij ui ), j −σ ij , j ui dV
V V
(5.161)
∫ ∫
⇒ σ ij ε ij dV = (σ ij ui ), j dV − σ ij , j ui dV
V V

V

University of Castilla-La Mancha Draft By: Eduardo W. V. Chaves (2016)


Ciudad Real – Spain
5 INTRODUCTION TO: CONSTITUTIVE EQUATIONS, IBVP, AND IBVP SOLUTION STRATEGIES 483

by applying the divergence theorem to the first volume integral on the right side of the
equation, we can obtain:

∫ σ ε dV = ∫ (σ u ),
V
ij ij
V
ij i j
V
∫ ∫


dV − σij , j ui dV = σij uinˆ j dS σ − σij , j ui dV
V
(5.162)
= ∫


t *i ui dS − σij , j ui dV
V
r
where we have applied the definition σ ⋅ nˆ = t * . The above equation in tensorial notation
becomes:
r r r
∫ σ : ε dV = ∫ t * ⋅ u dS − (∇ ⋅ σ ) ⋅ u dV
∫ (5.163)
V Sσ V
r r r r
If we use the equations of motion we can obtain ∇ ⋅ σ + ρb = ρu
&& ⇒ −∇ ⋅ σ = ρ (b − u
&&) , with
that the equation in (5.163) becomes:
r r r r r
∫ t * ⋅ u dS + ρ (b − u

&&) ⋅ u dV = σ : ε dV

Sσ V V
14243
144444244444
3
Total external virtual work Total internal
virtual work

which is known as the Principle of Virtual Work. Note that, for the demonstration, we
have not used the major symmetry of C e .
For the particular case when the system is in equilibrium and in the absence of body force,
the above equation becomes:
r r r
∫ t * ⋅ u ( x )dS = σ : ε dV
∫ (5.164)
Sσ V

In addition, if we have concentrated forces instead of surface force, the above equation
becomes:
Tensorial notation Voigt notation
r r (5.165)
F loc ⋅ u loc = σ : ε dV
∫ ∫
{F loc }T {u loc } = {σ }T {ε } dV
V V

where the direction of uiloc -component is the same as the Filoc -component direction,
where {F loc } = {F1 , F2 ,..., Fn }T , {u loc } = {U1 , U2 ,..., Un }T .

F1 F2
F3
u
u

σ, ε σ, ε

REAL VIRTUAL

Figure 5.15

University of Castilla- La Mancha Draft By: Eduardo W. V. Chaves (2016)


Ciudad Real – Spain
484 SOLVING PROBLEMS BY MEANS OF CONTINUUM MECHANICS

NOTE 1: The Principle of Virtual Work states: “A structure is in equilibrium, under a


system of external forces, if and only if the total external virtual work equals the total internal
r
virtual work for every virtual displacement field ( u )”.
NOTE 2: The Principle of Virtual Work is used to discretization techniques of the
problem such as the Finite Element Technique, in which the fundamental unknown is the
displacement.
NOTE 3: It is easy to show that the equation in (5.159) is also valid for rate of change of
& r
virtual fields u , ε& , i.e.:
r r& r r r&
∫ t * ⋅ u dS + ρ (b − u
&&) ⋅ u dV = σ : ε& dV
∫ ∫

1 V
44444244444
3 V
14243 Principle of Virtual Work (5.166)
Total external virtual work Total internal
virtual work

r
Also it is valid for a variation of the virtual field δu ⇒ δε , i.e.:
r r r r r
∫ t * ⋅ δ u dS + ρ (b − u
&&) ⋅ δ u dV = σ : δε dV
∫ ∫

1444442444443 V V
14243 Principle of Virtual Work (5.167)
Total external virtual work Total internal
virtual work

NOTE 4: We can also define the Principle of complementary virtual work as follows:
r r r r r
&&
∫ t ⋅ u*dS + ρ (b − u ) ⋅ udV =
∫ ∫ σ : ε dV Principle of Complementary
S ur
14444
4244444
3 V V
14243 (5.168)
Total internal
Virtual Work
Total external complementary virtual work
complementary virtual work

r
with σ ⋅ nˆ = t * on S σ . Considering a static case without body forces and that the external
action is characterized by concentrated forces, the principle of complementary virtual work
becomes:
r
loc r loc Principle of Complementary
F
142 ⋅ u43 = ∫ σ : ε dV Virtual Work (static case
Total external complementary virtual work V
14243 (5.169)
(due to concentrated forces)
Total internal
without body forces and with
complementary virtual work concentrated forces)
NOTE 5: Note that, if we are using the Principle of Virtual Work the fundamental
unknowns are displacements (strains), if we are using the Principles of Complementary
Virtual Work the fundamental unknowns are forces (stresses), and if we are using the
Betti’s reciprocal theorem the fundamental unknowns are displacements and forces
simultaneously, (see equation (5.154)).

Problem 5.23
Consider a sub-domain ( Ω ) made up by a homogeneous, isotropic linear elastic material.
Consider also that at some points of the sub-domain boundary there are concentrated
forces {F ( e ) } ≡ {F loc } , and that the displacement field into the sub-domain is
r r
approximated by {u( x )} = [ N ( x )]{u(e ) } where {u( e ) } ≡ {uloc } are the displacements at the
points where concentrated forces are applied. Prove that the governing equations for a
linear elastic problem in static equilibrium can be replaced by:

University of Castilla-La Mancha Draft By: Eduardo W. V. Chaves (2016)


Ciudad Real – Spain
5 INTRODUCTION TO: CONSTITUTIVE EQUATIONS, IBVP, AND IBVP SOLUTION STRATEGIES 485

{F (e ) } = [ K ( e ) ]{u(e ) } with ∫
[ K ( e ) ] = [B]T [C ] [B] dV
V
(5.170)

r
where [C ] is the elasticity tensor in Voigt notation, and obtain an expression for [B( x )] .
r
Hint: Use the Principle of Virtual Work and use the same approximate used by {u( x )} to
r
approach the virtual field {u ( x )} .
Solution:
We can start directly from the equation in (5.165), which is equivalent to:
r r r r r r
F ⋅ u = σ : ε dV = σ : (∇ sym u ) dV
∫ ∫ ⇒ u ⋅ F = (∇ sym u ) : σ dV
∫ (5.171)
V V V

The above equation in Voigt notation becomes:


r r r
u ⋅ F = (∇ sym u ) : σ dV Voigt
∫ → {u (e ) }T {F ( e ) } = {ε }T {σ } dV∫ (5.172)
V V

Note that, the above equation is already considering the equilibrium equations, (see equations
(5.163)-(5.165)). The constitutive equations in stress, in Voigt notation is given by
r r
{σ ( x )} = [C ]{ε ( x )} , where the strain tensor field is given by the kinematic equations
r r
ε ( x ) = ∇ sym u . In Problem 5.8 we have obtained the symmetric part of the displacement
field gradient, ε ij = 12 (u i , j + u j ,i ) , in Voigt notation, i.e.:

 ∂u1   ∂ 
 ∂x   ∂x 0 0 
 1   1 
 ∂u2   ∂
 ε11   0 0 
∂ x   ∂ x 
ε   2
 
2

 22   ∂ u ∂
3
  0 0  u1 
r  ε 33   ∂x3   ∂x3    r r
{ε ( x )} =   =  ∂u ∂u  =  ∂ u 2  ⇒ {ε ( x )} = [ L ]{u( x )}
(1)
∂ 
2ε12   1 + 2   0  u 
2ε 23   ∂x2 ∂x1   ∂x2 ∂x1  3

   ∂u ∂u  ∂ ∂ 
 2ε13   2 + 3   0
 ∂x3 ∂x2 
 ∂x3 ∂x2  
∂u
 1+ 3  ∂u ∂ ∂ 
0 
 ∂x 
 3 ∂x1   ∂x3 ∂x1 
r r
Then, taking into account that {u( x )} = [ N ( x )] {u(e ) } the above equation becomes:
r r r r
{ε ( x )} = [ L(1) ] {u( x )} = [ L(1) ] [ N ( x )]{u(e ) } = [B( x )] {u( e) }
where
r r
[B( x )] = [ L(1) ] [ N ( x )] (5.173)
The stress field can be expressed as follows:
r r r
{σ ( x )} = [C ] {ε ( x )} = [C ] [B( x )] {u(e ) }
We can adopt the same displacement field approach to approximate the virtual
displacement field, with which we can obtain:
r r r r
{u ( x )} = [ N ( x )]{u ( e) } ⇒ {ε ( x )} = [B( x )] {u ( e ) }

University of Castilla- La Mancha Draft By: Eduardo W. V. Chaves (2016)


Ciudad Real – Spain
486 SOLVING PROBLEMS BY MEANS OF CONTINUUM MECHANICS

Then, the equation in (5.172) becomes:

∫ {[B( x)]{u } } [C ][B( x)]{u


r r T

{u ( e) }T {F ( e ) } = {ε }T {σ} dV = (e) (e)
} dV
V V

or:
r r

{u ( e ) }T {F ( e ) } = {u ( e ) }T [B( x )]T [C ][B( x )]{u( e ) } dV
V
(5.174)
r
Note that neither {u (e ) } nor {u (e) } depend on x , then:
 r r 
{u ( e) }T {F ( e ) } = {u ( e ) }T  [B( x )]T [C ][B( x )] dV {u(e ) }

 
V 
Since the vector {u (e ) } is arbitrary, we can conclude that
 r r 
⇒ {F (e ) } =  [B( x )]T [C ][B( x )] dV {u( e ) }
∫ ⇒ {F (e ) } = [ K (e ) ]{u(e ) } (5.175)
 
V 
NOTE: [ K (e ) ] is known as the stiffness matrix of the sub-domain (finite element), and the
r r r
matrix [ N ( x )] from the relationship {u( x )} = [ N ( x )]{u(e ) } is known as the shape function
matrix. The shape functions are functions defined into the domain that allows us to obtain
r
{u( x )} at any point of the domain through the nodal values of the function {u (e ) } . A
special emphasis about shape functions is taken place in Problem 6.40 in Chapter 6.

Problem 5.24
a) Consider a sub-domain ( Ω ) made up by a homogeneous, isotropic linear elastic material.
Consider also that at some points of the sub-domain boundary there are concentrated
forces {F ( e ) } ≡ {F loc } (nodal forces), and that the displacement field is approximated by
r r
{u( x )} = [ N ( x )]{u(e ) } where the nodal displacements {u ( e ) } ≡ {u loc } are the displacements
at the points where concentrated forces are applied. Prove that the governing equations for
a linear elastic problem can be replaced by:

[ K ( e ) ]{u( e ) } + [ M ( e ) ]{u
&& ( e ) } = {F ( e ) } (5.176)

where [ K ( e ) ] = ∫ [B]T [C ] [B] dV (stiffness matrix), [ M ( e ) ] = ∫ ρ [ N ]T [ N ] dV (mass matrix),


V V
[C ] is the elasticity tensor in Voigt notation.
b) Show that the equation in (5.176) represents a conservative system.
Hint: Use the Principle of Virtual Work and consider the following approximations for the
fields:

University of Castilla-La Mancha Draft By: Eduardo W. V. Chaves (2016)


Ciudad Real – Spain
5 INTRODUCTION TO: CONSTITUTIVE EQUATIONS, IBVP, AND IBVP SOLUTION STRATEGIES 487

r r
{u( x )} = [ N ( x )]{u(e ) } (displacement field)

{u& ( xr )} = [ N ( xr )]{u& (e ) } (velocity field)

 && r r
{u( x )} = [ N ( x )]{u } (acceleration field)
&& (e )
 r (1) r (e) r (e)
{ε ( x )} = [ L ] [ N ( x )]{u } = [B( x )]{u } (strain field) (5.177)
 r r (e)
{σ ( x )} = [C ]{ε ( x )} = [C ][B]{u } (stress field)
 r r
r
{t * ( x )} = [Nt ( x )]{ f r(e ) } (vector traction field)
t

 r r
b r r( e )
{b( x )} = [N ( x )]{ f b } (body force field)
Use the same approximations for the respective virtual fields.
Solution:
The Principle of Virtual Work states that:
r r r r r
∫ σ : ε dV = t * ⋅ u dSσ + ρ (b − u

&&) ⋅ u dV

V Sσ
14243 144444V2444443
Total internal Total external virtual work
virtual work

We rewrite the above equation using Voigt notation:

∫ {σ} {ε} dV = ∫{t} {u} dS + ∫ {ρb} {u} dV − ∫ {ρu&&} {u} dV


T T T T

V Sσ V V
(5.178)

= {u} {t} dS + {u} {ρb} dV − {u}T {ρu ∫ ∫
T &&} dV T

Sσ V V

Using the adopted approximations, (see equations in (5.177)), we can obtain the following
terms:

∫{σ} {ε} dV = ∫{ε} {σ} dV = ∫ {[B]{u }} {[C ][B]{u }}dV =


T T (e) T (e)

V V V

 
= {u (e ) }T [B]T [C ][B]{u( e ) } dV = {u ( e ) }T  [B]T [C ][B] dV {u( e ) }
∫ ∫
 
V V 
= {u ( e ) }T [ K (e ) ]{u( e ) }

∫ {[ N ]{u }} [N ]{ f
r r
T
∫ ∫
t
{u}T {t} dS = (e) r( e ) } dS
t
= {u (e ) }T [ N ]T [Nt ]{ f tr( e ) } dS
Sσ Sσ Sσ

 r  r
= {u ( e ) }T  [ N ]T [Nt ] dS { f tr(e ) } = {u ( e ) }T [Gt ]{ f tr( e ) } = {u ( e ) }T {Frt( e ) }

 
 Sσ 
 
∫ {[ N ]{u }} ρ[N ]{ f
r r
= {u (e ) }T  ρ [ N ]T [Nb ] dV { fbr( e ) }
T
∫ {u}T {ρb} dV = ∫
(e) b r( e ) } dV
b  
V V V 
r r
= {u (e ) }T [Gb ]{ fbr(e ) } = {u ( e ) }T {Fbr(e ) }

University of Castilla- La Mancha Draft By: Eduardo W. V. Chaves (2016)


Ciudad Real – Spain
488 SOLVING PROBLEMS BY MEANS OF CONTINUUM MECHANICS

∫ {u} {ρu&&} dV = ∫ {[ N ]{u }} ρ[ N ]{u&&


T

} dV = ρ {u ( e ) }T [ N ]T [ N ]{u
T (e) (e) && ( e ) } dV
V V V

  (e)
= {u ( e ) }T  ρ [ N ]T [ N ] dV {u
∫ && } = {u ( e ) }T [ M ( e ) ]{u
&& (e ) }
 
V 
Taking into account the above relationships into the equation (5.178) we can obtain:
{u ( e ) }T [ K ( e ) ]{u( e ) } = {u (e ) }T {Ftr( e ) } + {u ( e ) }T {Fbr( e ) } − {u (e ) }T [ M ( e ) ]{u
&& (e ) }

( ) (
⇒ {u ( e ) }T [ K ( e ) ]{u(e ) } = {u ( e ) }T {Ftr(e ) } + {Fbr(e ) } − [ M ( e ) ]{u
&& ( e ) } )
Since the virtual displacement {u (e ) } is arbitrary we can conclude that:
[ K (e ) ]{u( e ) } = {Ftr(e ) } + {Fbr( e ) } − [ M ( e ) ]{u
&& ( e ) }

⇒ [ K (e ) ]{u( e ) } + [ M ( e ) ]{u
&& (e ) } = {Fr( e ) } + {Fr( e ) }
t b

⇒ [ K (e ) ]{u( e ) } + [ M ( e ) ]{u
&& (e ) } = {F ( e ) }
Q.E.D.
b) To show that the above system is conservative we will consider the discretization of
time where the current time we denote by t and the next time by t + ∆t , where ∆t is the
time increment. In any time the above equation must be true, so:
[ K (e ) ]t {u( e ) }t + [ M (e ) ]t {u
&& ( e ) }t = {F ( e ) }t
 (e) (e) (e)
[ K ]t + ∆t {u }t + ∆t + [ M ]t + ∆t {u && ( e ) }t + ∆t = {F (e ) }t + ∆t = {F ( e ) }t

⇒ [ K (e ) ]t + ∆t {u( e ) }t + ∆t + [ M ( e ) ]t + ∆t {u
&& ( e ) }t + ∆t = [ K ( e ) ]t {u( e ) }t + [ M ( e ) ]t {u
&& ( e ) }t

where the force {F (e ) } is constant over time. Notice that, if the vector {F (e ) } is constant
over time we can obtain:
D
Dt
( )
&& ( e ) } = D {F (e ) } = {0}
[ K (e ) ]{u (e ) } + [ M ( e ) ]{u
Dt
( (e) (e) ( e ) && ( e )
⇒ [ K ]{u } + [ M ]{u } = constant over time )
NOTE 1: The equation in (5.176) is a forced harmonic motion. Let us consider the one-
dimensional case where [ K (e ) ] represents the spring constant k , [ M (e) ] represents the
mass m , and the displacement and acceleration are represented by u and u&& respectively,
(see Figure 5.16). With that the equation in (5.176), without applied force, becomes:
ku + mu&& = 0 ⇒ ku = − mu&&

Note that the energy is conserved. Considering the internal energy for the spring ( 12 uku )
and the kinetic energy ( 12 u&mu& = 12 mv 2 ) for the particle of mass m , and by apply the energy
equation we can obtain:
DK DU DW DQ D 1  D 1 
+ = + =0 ⇒  u&mu&  +  uku  = 0
Dt Dt 1Dt Dt Dt  2  Dt  2  J 
4243  s = W  (5.179)
=0  
⇒ mu&&u& + kuu& = 0 ⇒ ⇒ (mu&& + ku )u& = 0 ⇒ mu&& + ku = 0
where K is the kinetic energy, U is the internal energy. The equation mu&& + ku = 0 is
denoted by the simple harmonic motion, (see Figure 5.16).

University of Castilla-La Mancha Draft By: Eduardo W. V. Chaves (2016)


Ciudad Real – Spain
5 INTRODUCTION TO: CONSTITUTIVE EQUATIONS, IBVP, AND IBVP SOLUTION STRATEGIES 489

k
u=0
t

m u

b) Mechanical model (mass+spring) b) Displacement vs. time

Figure 5.16: Mechanical model for simple harmonic motion.

NOTE 2: If we do the experiment using the mechanical model described in Figure 5.16
we will observe that the motion in reality is not conservative, i.e. there is dissipation of
energy. In other word, there is damping of the system until the rest is achieved, (see Figure
5.18). This phenomenon occurs due to the internal mechanisms of the structures.
Traditionally, this damping intrinsic of the structures can be dealt by means of the
parameter d (damping) multiply by velocity, (see Figure 5.17).

k u (displacement)
FS = ku

FI = mu&& F (t )
m

d FD = du&

Figure 5.17: Mechanical model (mass+dashpot+spring).

Taking into account the discrete mechanical model described in Figure 5.17 and by
applying the force equilibrium we can obtain:
FI + FD + FS = F (t ) ⇒ mu&& + du& + ku = F (t )
And the equation in (5.176) can be rewritten in order to take the damping effect as follows:

[ K ( e ) ]{u(e ) } + [ D (e ) ]{u& ( e ) } + [ M ( e ) ]{u


&& (e ) } = {F ( e ) } (5.180)

where [ D (e ) ] is the damping matrix. Note that to solve the equation (5.180) we need to
integrate over time. To solve (5.180) we must transform the equation in (5.180) into an
equivalent system as follows:
[ K eff ]{u( e ) }t + ∆t = {F eff } (5.181)

where [ K eff ] is the effective stiffness matrix, and {F eff } is the effective nodal force vector.
For more details about this the reader is referred to Annex A at the end of the Book.

University of Castilla- La Mancha Draft By: Eduardo W. V. Chaves (2016)


Ciudad Real – Spain
490 SOLVING PROBLEMS BY MEANS OF CONTINUUM MECHANICS

t
u=0

Figure 5.18: Displacement vs. time – Mechanical model with damping.

NOTE 3: Although the matrices [ K (e) ] and [ M (e ) ] are obtained by means of material and
geometrical properties, the “viscous damping” matrix [ D(e ) ] there is no universal formula,
and estimating a suitable damping matrix is still a challenging task. Whereas the equation
[ K ( e ) ]{u( e ) } + [ M ( e ) ]{u
&& ( e) } = {F (e ) } (forced harmonic motion) can be obtained from the
fundamental principles of Continuum Mechanics, the equation (5.180) cannot, in other
words, from a Continuum Mechanics point of view there is no matrix [ D(e ) ] . Some
authors adopt [ D(e ) ] as a function of [ K (e) ] , or a function of [ M (e ) ] , or Rayleigh damping
which is a linear combination between the two, i.e.: [ D(e ) ] = α [ M ( e) ] + ξ [ K (e ) ] , so as to
guarantee definite positiveness of the matrix [ D(e ) ] . Moreover, identification of the valid
damping coefficients α and ξ , for large systems, is highly complicated. Nowadays,
characterization of damping force has been an active area of research in structural
dynamics, (see Clough&Penzien (1975), Chaves (2015)).

Structural Dynamics References


CLOUGH, R.W. & PENZIEN, J. (1975). Dynamic of Structures. McGraw-Hill Companies.

TEDESCO, J.M.; MCDOUGAL, W.G. & ROSS, C.A.(1998). Structural dynamics: theory and
applications. Addison Wesley Longman, Inc.
CHAVES, E.W.V. (2015). Dynamic analysis: a new point of view. Continuum Mechanics and
Thermodynamics, Springer, DOI 10.1007/s00161-015-0419-4.

University of Castilla-La Mancha Draft By: Eduardo W. V. Chaves (2016)


Ciudad Real – Spain
5 INTRODUCTION TO: CONSTITUTIVE EQUATIONS, IBVP, AND IBVP SOLUTION STRATEGIES 491

Problem 5.25
For an equilibrium system let us consider the total potential energy Π defined as follows:
r r r r r
Π (u) = Ψ e (ε ) dV − t * ⋅ udS − ( ρb) ⋅ udV
∫ ∫ ∫ The total potential energy (5.182)
V Sσ V

where

1

U int = Ψ e (ε ) dV =
V
∫ 2 σ : ε dV
V
The internal potential energy (5.183)

and
r r r r
U ext = t * ⋅ udS + ( ρb) ⋅ udV
∫ ∫ The external potential energy (5.184)
Sσ V

Also let us consider that the first variation of Π , denoted by δΠ , equals zero for a
stationary value of Π . Show that, if δΠ = 0 is equivalent to a stationary value of Π , so
r
Π (u) assume a minimum value.
r r
Obs.: Consider that during the deformation process, the external actions ( t * , b ) do not
vary, and also consider a linear elastic material.
Solution:
The first variation ( δΠ ) can be obtained as follows:
 1 r r r r 
δΠ = δ σ : ε dV − t * ⋅ udS − ( ρb) ⋅ udV 
∫ ∫ ∫
 2 
V Sσ V 
1 r* r r r
= δ σ : ε dV − δ t ⋅ udS − δ ( ρb) ⋅ udV
∫ ∫ ∫ (5.185)
V
2 S V σ

1 r r r r
= ∫ δ(σ : ε ) dV − t * ⋅ δudS − ( ρb) ⋅ δudV
∫ ∫
V
2 S V
σ

Note that:
1
2
1
2
1
[
δΨ e (ε ) = δ(σ : ε ) = (δσ : ε + σ : δε ) = δ(C e : ε ) : ε + σ : δε
2
]
1
2
[ 1
2
] [ 1
= (C e : δε ) : ε + σ : δε = ε : C e : δε + σ : δε = [σ : δε + σ : δε ]
2
] (5.186)
= σ : δε
∂Ψ e
= : δε
∂ε
∂Ψ e
where we have considered σ = , (see Problem 5.5). For small deformation regime we
∂ε
can also write the above equation as follows:
∂Ψ e r r r
δΨ e (ε ) = : δε = σ : δε = σ : δ(∇ symu) = σ : (∇ sym δu) = σ : (∇δu) (5.187)
∂ε

University of Castilla- La Mancha Draft By: Eduardo W. V. Chaves (2016)


Ciudad Real – Spain
492 SOLVING PROBLEMS BY MEANS OF CONTINUUM MECHANICS

where we have used the property A sym : B = A sym : (B sym + B skew ) = A sym : B sym . Then, the
equation in (5.185) becomes:
1 r r r r
δΠ = ∫ δ(σ : ε ) dV − t * ⋅ δudS − ( ρb) ⋅ δudV
∫ ∫
V
2 Sσ V
r* r r r
= σ : δε dV − t ⋅ δudS − ( ρb) ⋅ δudV
∫ ∫ ∫
V Sσ V
r r r r
= δΨ e dV − t * ⋅ δudS − ( ρb) ⋅ δudV
∫ ∫ ∫
V Sσ V
r r
The expression Π (u + δu) can be obtained as follows, (see equation (5.182)):
r r r r r r r r
Π (u + δu) = Ψ e (ε + δε ) dV − t * ⋅ (u + δu)dS − ( ρb) ⋅ (u + δu)dV
∫ ∫ ∫ (5.188)
V Sσ V

By using the Taylor series to approach Ψ e (ε + δε) we can obtain:


∂Ψ e (ε ) 1 ∂ 2Ψ e (ε )
Ψ e (ε + δε ) = Ψ e (ε) + : δε + δε : : δε + ... (5.189)
∂ε 2 ∂ε ⊗ ∂ε
∂Ψ e (ε ) ∂ 2Ψ e (ε )
Note that : δε = δΨ e , (see equation (5.186)), and C e = , (see Problem
∂ε ∂ε ⊗ ∂ε
5.5), with which the equation in (5.189) becomes:
∂Ψ e (ε ) 1 ∂ 2Ψ e (ε )
Ψ e (ε + δε) = Ψ e (ε) + : δε + δε : : δε + ...
∂ε 2 ∂ε ⊗ ∂ε
1
≈ Ψ e (ε ) + δΨ e + δε : C e : δε
2
and by replace the above equation into the equation (5.188) we can obtain:
r r r r r r r r
Π (u + δu) = Ψ e (ε + δε ) dV − t * ⋅ (u + δu)dS − ( ρb) ⋅ (u + δu)dV
∫ ∫ ∫
V Sσ V

1

= Ψ e (ε ) dV + δΨ e dV + ∫ ∫ 2 δε : C
e
: δε dV
V V V
r r r r r r
− t * ⋅ (u + δu)dS − ( ρb) ⋅ (u + δu)dV
∫ ∫
Sσ V
r r r r
= Ψ e (ε ) dV − t * ⋅ udS − ( ρb) ⋅ udV +
∫ ∫ ∫
V Sσ V
r r r r 1
+ δΨ e dV − t * ⋅ δudSσ − ( ρb) ⋅ δudV +
∫ ∫ ∫
δε : C e : δε dV ∫
V S V V
2
σ

(5.190)
Note that:
r r r r r
Π (u) = Ψ e (ε ) dV − t * ⋅ udS − ( ρb) ⋅ udV
∫ ∫ ∫
V Sσ V

and
r r r r
δΠ = δΨ e dV − t * ⋅ δudS − ( ρb) ⋅ δudV = 0
∫ ∫ ∫
V Sσ V

University of Castilla-La Mancha Draft By: Eduardo W. V. Chaves (2016)


Ciudad Real – Spain
5 INTRODUCTION TO: CONSTITUTIVE EQUATIONS, IBVP, AND IBVP SOLUTION STRATEGIES 493

Taking into account the previous equations into the equation in (5.190) we can conclude
that:
r r r 1
Π (u + δu) = Π (u) + δΠ +
V
2 ∫
δε : C e : δε dV

r r r 1
⇒ Π (u + δu) − Π (u) = δΠ +
V
2 ∫
δε : C e : δε dV

r r r 1
⇒ Π (u + δu) − Π (u) =
V
2 ∫
δε : C e : δε dV

where we have considered δΠ = 0 . Note that the term δε : C e : δε > 0 is always positive for
any valor of δε since C e is a positive definite tensor, (see Chapter 1). Then, we can
guarantee that:
r r r 1 r r r
∆Π = Π (u + δu) − Π (u) =
V
2 ∫
δε : C e : δε dV > 0 ⇒ Π (u + δu) > Π (u)
r
So, δΠ = 0 ⇒ Π (u) is a minimum
NOTE 1: For a system characterized by a linear elastic problem, the equilibrium point
corresponds to the minimum value of Π , (see Figure 5.19). This is known as the principle of
minimum potential energy.

NOTE 2: When the external action is characterized by concentrated forces and in the
absence of body forces, the equation (5.182) becomes:
r 1
Π (u) = U int + U ext = ∫
V
2
σ : ε dV − {F loc }T {uloc } The total potential energy (5.191)

F Π (u )

u (2) u u ( 3)
Π (u ( 2 ) )
Π
Π (u )

Deformation corresponding to Π (u ( 3 ) )
the equilibrium ∂Π
=0
∂u

Figure 5.19

University of Castilla- La Mancha Draft By: Eduardo W. V. Chaves (2016)


Ciudad Real – Spain
494 SOLVING PROBLEMS BY MEANS OF CONTINUUM MECHANICS

NOTE 3: By means of equation (5.191), it is easy to show the Castigliano’s Theorem – Part I:
r
∂Π (u)
=
∂U int
+
∂U ext
=
∂U int

[
∂ {F loc }T {u loc }
=0
]
∂{u loc } ∂{u loc } ∂{u loc } ∂{u loc } ∂{u loc }
∂U int
⇒ {F loc } =
∂{u loc }

where {F loc } = {F1 , F2 ,..., Fn }T , {u loc } = {U1, U2 ,..., Un }T . Note that the term U int must be in
function of {uloc } .
Note also that the Potential can also be expressed in terms of forces Π ({F loc }) , so, we can
define the Castigliano’s Theorem – Part II
∂Π ({F loc })
=
∂U int
+
∂U ext
=
∂U int

[
∂ {F loc }T {uloc }
=0
]
∂{F loc } ∂{F loc } ∂{F loc } ∂{F loc } ∂{F loc }
∂U int
⇒ {uloc } =
∂{F loc }

in this U int must be expressed in terms of {F loc } .


NOTE 4: For better illustration of the proposed problem, we will consider a rod of length
L and cross-sectional area A . Consider also that the stress and strain fields are
homogeneous and given by:
σ 0 0  ε 0 0
σ ij =  0 0 0 ; ε ij = 0 0 0 ⇒ σ11 = C1111
e
ε11 ⇒ σ = Eε
 0 0 0 0 0 0

Consider also that the displacement field is approached by a linear function


( u ( x) = a1 + a2 x ), and that on the extremities of the rod, we have the forces F (1) , F ( 2) , and
the nodal displacements U (1) , U ( 2) , (see Figure 5.20).

A
σ
(3D ) V = AL (volume)
σ


F = σdA = σA
A
(1) (1) ( 2) ( 2)
F ,U F ,U
(1D )
1 2 x

Figure 5.20: Rod under axial force.

The goal now is to express the total potential energy in terms of U(1) ,U( 2) . Note that, due to
the concentrated forces we have:

University of Castilla-La Mancha Draft By: Eduardo W. V. Chaves (2016)


Ciudad Real – Spain
5 INTRODUCTION TO: CONSTITUTIVE EQUATIONS, IBVP, AND IBVP SOLUTION STRATEGIES 495

ext loc T loc (1)


U(1) 
U = {F } {u } = {F F } ( 2 )  = F (1)U(1) + F ( 2 )U( 2 )
(2)
(5.192)
U 
For this case, (see Problem 5.5 - NOTE 3), the linear stress-strain relationship is given by
1 1
σ = Eε , and the strain energy density by Ψ e = σε = εEε . Then, the total internal energy
2 2
is given by:
1 1 1 1
∫ 2 σ : ε dV ∫ ∫ ∫
1D
U int = → U int = σε dV = Eεε dV = Eε 2 dV
V
2V 2V 2V

∂u1 ∂u ( x)
where ε11 = = = ε , thus
∂x1 ∂x
2
1 1  ∂u ( x) 
U int =
2V ∫
Eε 2 dV = E
2 V  ∂x  ∫  dV (5.193)

Now we will express the displacement field in terms of their nodal values U(1) ,U( 2) . To do
this we will use the approach adopted u ( x) = a1 + a2 x , where:
u ( x = 0) = U(1) = a1  U(1)  1 0  a1 
 ⇒  ( 2)  =   
u ( x = L ) = U( 2 ) = a1 + a2 L  U  1 L  a2 

Next we evaluate the coefficients a1 and a 2 . To do this, we obtain the reverse form of the
above relationship, i.e.:
a1 = U(1)
U(1)  1 0  a1  reverse a1  1  L 0 U(1)  
 (2)  =  →  =  ⇒
 
U  1 L  a2 
  ( 2) 
a2  L − 1 1 U 

(
1 (2)
a 2 = U − U
 L
(1)
)
with which we can obtain the displacement field in terms of its nodal values:
1 (2)
u ( x) = a1 + a2 x = U(1) +
L
( x x
U − U(1) x = 1 − U(1) + U( 2 ) )
 L L
(5.194)
x   x  U 
(1)
 r
⇒ u ( x) = 1 −     ( 2 )  = [ N ( x )]{u(e ) }
 L   L  U 
and the equation in (5.193) becomes:
2 2

∫ ( )
1  ∂u ( x)  E  1 (2) (1)  E
 U − U  dV = 2  U − 2U(1)U( 2 ) + U(1)  dV
( 2) 2 2
∫ ∫
int
U = E  dV =
2 V  ∂x  2VL  2L V  

Note that U(1) and U( 2) are independent of x , then:


E  (2) 2 E
− 2U(1)U( 2 ) + U(1)  dV = 2  U( 2 ) − 2U(1)U( 2 ) + U(1) V
2 2 2
U int =
2L
U
2  
V
2L  ∫ 
(5.195)
EAL  ( 2) 2 EA  ( 2 ) 2
− 2U(1)U( 2 ) + U(1)  = − 2U(1)U( 2 ) + U(1) 
2 2
= 2 
U U
2L   2L  
Then, the total potential energy, (see equation (5.191)), is given by equations (5.192) and
(5.195), i.e.:
r
Π (u) = U int − U ext =
EA  ( 2 ) 2
U
2L 
2


( )
− 2U(1)U( 2 ) + U(1)  − F (1)U(1) + F ( 2 )U( 2 ) = Π (U(1) , U( 2 ) )

University of Castilla- La Mancha Draft By: Eduardo W. V. Chaves (2016)


Ciudad Real – Spain
496 SOLVING PROBLEMS BY MEANS OF CONTINUUM MECHANICS

As we are looking for a stationary state, the following must be true:


 ∂Π (U (1) , U ( 2 ) )
 =
∂  EA  ( 2 ) 2
(1) 
U
2
(
− 2U (1) U ( 2) + U (1)  − F (1) U (1) + F ( 2 ) U ( 2) ) = 0
 ∂U (1) ∂U  2 L   

 =
EA
2L
( )
− 2U ( 2 ) + 2U (1) − F (1) = 0


 =
EA (1)
L
( )
U − U ( 2 ) − F (1) = 0

( ) = 0
(1) ( 2)
 ∂Π (U , U ) =
∂  EA  ( 2 ) 2
U − 2U (1) U ( 2) + U (1)  − F (1) U (1) + F ( 2 ) U ( 2)
2
 ( 2) 
∂U ( 2 ) ∂U  2 L   


 =
EA
2L
( )
2U ( 2 ) − 2U (1) − F ( 2 ) = 0



=
L
(
EA ( 2 )
)
U − U (1) − F ( 2 ) = 0

Rearranging the above equations in matrix form we can obtain:


EA  1 − 1 U  F 
(1) (1)

  =   ⇔ [ K ( e ) ]{u( e ) } = {F ( e ) } (5.196)
L  − 1 1  U( 2 )  F ( 2 ) 

Note that [ K (e ) ] has no inverse, since det[ K (e ) ] = 0 , which indicates that the problem has
infinity solution since we have not imposed any restriction to motion. To solve the
problem we have to introduce the boundary conditions in order to guarantee the unique
solution.
Note that the matrix [ K (e ) ] of the above equation could have been obtained by means of
the equation (5.175), (see Problem 5.23), and for this particular case we have [C ] = E ,
then, the equation (5.194) becomes:
r r ∂  x   x    − 1   1  
[B( x )] = [ L(1) ][ N ( x )] = 1 −     =     
∂x  L   L    L   L  
thus
 − 1 
 L 
r T r    − 1   1   1  1 − 1

[ K ] = [B( x )] [C ][B( x )] dV =  ∫ ∫
(e)
E      dV = E 2   dV
 1    L   L  
V 
L − 1 1 
V
   V
 L  
E  1 − 1 E  1 − 1 EA  1 − 1
=
L2  V

− 1 1  dV = 2  − 1 1 V =
L  

L − 1 1 

NOTE 5: It is interesting to note that, initially we had a continuum problem (infinity of


material points) represented by its governing equations and we have transformed this
continuum problem into a set of discrete equations (5.196). In other words, we have
applied a numerical technique to solve the problem. The main goal of any numerical
technique is to transform the continuum governing equations into a set of discrete
equations. Among these techniques we can quote: Finite Differences, Finite Element,
Boundary Element, Finite Volume, etc.

University of Castilla-La Mancha Draft By: Eduardo W. V. Chaves (2016)


Ciudad Real – Spain
5 INTRODUCTION TO: CONSTITUTIVE EQUATIONS, IBVP, AND IBVP SOLUTION STRATEGIES 497

r
NOTE 6: Analyzing [ N ( x )]
T
r r r  x   x 
Note that the shape functions are [ N ( x )] = [N1 ( x ) N 2 ( x )]T = 1 −    . And these
 L   L 
functions are drawn as indicated in Figure 5.21.

N1 x  N1 ( x = 0) = 1
N2 N1 ( x ) = 1 −⇒
L  N1 ( x = L ) = 0
x  N 2 ( x = 0) = 0
N 2 ( x) = ⇒ 
1 L  N 2 ( x = L) = 1
1

x N1 ( x ) + N 2 ( x ) = 1
1 2
L

Figure 5.21: Shape function (linear approach)


r
The adopted approximation for [ N ( x )] will depend on the problem. For the previous
problem we have that the strain is constant into the domain, so, it is sufficient to adopt a
∂u ( x)
linear approximation for displacement since by definition ε = . As consequence we
∂x
r
need only two points on the boundary to define [ N ( x )] . If a problem requires a quadratic
r
function for displacement approximation, so, we will need three points to define [ N ( x )] ,
and so on.

NOTE 7:
Principle of the Stationarity of Potential Energy
In this problem we have establish the principle of the stationarity of Potential Energy, (see
equation in (5.182)):
r r r r r
Π (u) = Ψ e (ε ) dV − t * ⋅ udS − ( ρb) ⋅ udV
∫ ∫ ∫ (5.197)
V Sσ V

1
where we have considered Ψ e (ε) = σ : ε . The functional is stationary if and only if
2
r
δ ur Π (u) = 0 .

Hellinger-Reissner’s Variational Principle


In Problem 5.5, (see NOTE 7) have established that
Ψ e (σ) = σε − Ψ e (ε)  →Ψ e (σ ) = σ : ε − Ψ e (ε ) = − ρ 0 G(σ ) = g(σ )
tensorial

(5.198)
⇒ Ψ e (ε ) = σ : ε − g(σ )

where g (σ ) is the Gibbs free energy density with reversed sign.

University of Castilla- La Mancha Draft By: Eduardo W. V. Chaves (2016)


Ciudad Real – Spain
498 SOLVING PROBLEMS BY MEANS OF CONTINUUM MECHANICS

σ g(σ) - Complementary strain energy

Ψ e (ε) - Strain energy

Figure 5.22: Strain energies.

By replacing Ψ e (ε ) = σ : ε − g(σ ) into the functional (5.197) we can obtain:


r r r r r
Π (u) = Ψ e (ε ) dV − t * ⋅ udS − ( ρb) ⋅ udV
∫ ∫ ∫
V Sσ V
r r r r r (5.199)
⇒ Π HR (u, σ ) = σ : ε − g(σ ) dV − t * ⋅ udS − ( ρb) ⋅ udV
∫ ∫ ∫
V Sσ V
r r
Note that σ : ε = σ : (∇ sym u) = σ : (∇u) . Then, we can obtain:
r r r r r r
Π HR (u, σ ) = σ : (∇u) − g(σ ) dV − t * ⋅ udS − ( ρb) ⋅ udV
∫ ∫ ∫ (5.200)
V Sσ V

r
The functional (5.200) is stationary for variation of u vanishing on S ur if and only if σ
satisfies the equilibrium equations, and is stationary for variation of σ if and only if they
satisfy the constitutive equation (strain-stress).
r r r r r r
δur Π HR (u, σ ) = σ : (∇δu) dV − t * ⋅ δudS − ( ρb) ⋅ δudV = 0
∫ ∫ ∫
V Sσ V

∫ ∫
= σij (δu)i , j dV − t*i (δu)i dS − ( ρb)i (δu)i dV = 0
V Sσ V

(5.201)
∫ ∫ ∫
= σij (δu)i nˆ j dS − σij , j (δu)i dV − t *i (δu)i dS σ − ( ρb)i (δu)i dV = 0
Sσ V Sσ V

∫[
V
]
= − σij , j + ( ρb)i (δu)i dV + ∫ [σ nˆ

ij j ]
− t *i (δu)i dS = 0

In the volume we can obtain the equilibrium equations: σ ij , j + (ρ b) i = 0 i .

On surface S σ we can obtain the boundary condition in stress: σ ij nˆ j − t *i = 0 i


r r

δ σ Π HR (u, σ ) = δσ : (∇ sym u) − δg(σ ) dV = 0
V
r ∂g(σ )
V

= (∇ sym u) : δσ −
∂σ
: δσ dV = 0 (5.202)

 r ∂g(σ ) 
V 

= (∇ sym u) −
∂σ 
: δσ dV = 0

University of Castilla-La Mancha Draft By: Eduardo W. V. Chaves (2016)


Ciudad Real – Spain
5 INTRODUCTION TO: CONSTITUTIVE EQUATIONS, IBVP, AND IBVP SOLUTION STRATEGIES 499

r ∂g(σ )
In the volume we can obtain the constitutive equation for strain: (∇ sym u) − =0.
∂σ
Hu-Washizu’s Variational Principle
The Hu-Washizu’s principle is a generalization of the Hellinger-Reissner’s principle, in
r
which the functional, in addition of the independent fields (u, σ ) , also depends on ε -field:
r r r r r r r r
Π HW (u, σ , ε ) = [Ψ e (ε ) − σ : (ε − ∇ symu) − ( ρb) ⋅ u] dV − (σ ⋅ nˆ ) ⋅ (u* − u)dS − t * ⋅ udS
∫ ∫ ∫
V S ur Sσ

(5.203)
and is stationary if and only if:
r
δur Π HW (u, σ , ε ) = 0 ⇒ Equilibrium equations
 r r
δu = 0 ⇒ on Sur

 r Kinematic Equations
δσ Π HW (u, σ , ε ) = 0 ⇒ 
 Boundary condition on Sur

 r
δε Π HW (u, σ , ε ) = 0 ⇒ Constitutive equations for stress


That is:
r
ƒ δ ur Π HW (u, σ , ε ) = 0
r
δur Π HW (u, σ , ε ) = [∫ σ : (∇ δur ) − ( ρbr ) ⋅ δur ] dV − ∫ tr ⋅ δur dS
sym *

V Sσ

(u, σ , ε ) = ∫ [(∇ ⋅ σ ) ⋅ (δu) − ( ρb) ⋅ δu] dV − ∫ t ⋅ δudS


r r r r r r *
⇒ δur Π HW
V Sσ

(u, σ , ε ) = ∫ [(∇ ⋅ σ ) − ( ρb)]⋅ (δu) dV − ∫ t ⋅ δudS


r r r r r *
⇒ δur Π HW
V Sσ

r
ƒ δ σ Π HW (u, σ , ε ) = 0

∫[ ]
r r r r
δσ Π HW (u, σ , ε ) = − δσ : (ε − ∇ symu) dV − (δσ ⋅ nˆ ) ⋅ (u* − u)dS = 0

V S ur

∫ [ ] ∫ [nˆ ⊗ (u ]
r r r* r
⇒ δσ Π HW (u, σ , ε ) = − (ε − ∇ symu) : δσ dV − − u) : δσdS = 0
V S ur

r
ƒ δ ε Π HW (u, σ , ε ) = 0

∫[ ]
r
δε Π HW (u, σ , ε ) = δεΨ e (ε ) − σ : (δε ε ) dV = 0
V

r  ∂Ψ e (ε )   ∂Ψ e (ε ) 
⇒ δε Π HW (u, σ , ε ) = 
V 

∂ε
: δε ε − σ : (δε ε ) dV = 
 V 
∂ε 

− σ  : δε ε dV = 0

University of Castilla- La Mancha Draft By: Eduardo W. V. Chaves (2016)


Ciudad Real – Spain
500 SOLVING PROBLEMS BY MEANS OF CONTINUUM MECHANICS

NOTE 8: Discretization of the Fields

The variation of the Hu-Washizu’s principle can be written as follows

∫[ ]
r r r r r
δΠ HW = δ Ψ e (ε ) − σ : (ε − ∇ symu) − ( ρb) ⋅ u dV − δ t * ⋅ udS = 0 ∫
V Sσ

∫[ ]
r r r r r
= Ψ e (ε ) : δε dV − δ σ : (ε − ∇ symu) dV − ( ρb) ⋅ δu dV − t * ⋅ δudS = 0
∫ ∫ ∫
V V V Sσ

∫[ ]
r r r r r
= δε :Ψ e (ε ) dV + δ σ : (∇ symu − ε ) dV − δu ⋅ ( ρb)dV − δu ⋅ t *dS = 0
∫ ∫ ∫
V V V Sσ

∫[ ]
r r r r r
= δε : σ dV + δ σ : (∇ symu − ε ) dV − δu ⋅ ( ρb)dV − δu ⋅ t *dS = 0
∫ ∫ ∫
V V V Sσ

In the implementation of finite element methods we often use Voigt notation when we are
dealing with symmetric matrix. Using Voigt notation the above equation becomes:

∫ ∫
δΠ HW = {δε}T {σ} dV + δ {σ}T {∇ symu − ε}dV − {δu}T {ρb} dV − {δu}T {t *} dS = 0
V V

V


(5.204)
∫ ∫
⇒ {δε}T {σ} dV + δ {σ}T {∇ symu − ε}dV = {δu}T {ρb} dV + {δu}T {t *} dS
V V V
∫ ∫

Consider as approximation for displacement, strain, and stress fields, respectively, (see
Jirásek (1998)), as follows:
{u} ≈ [N ]{d} + [N c ]{d c } {δu} ≈ [N ]{δd} + [N c ]{δd c }
 
 
{ε} ≈ [B]{d} + [G]{e} and {δε} ≈ [B]{δd} + [G]{δe} (5.205)
 
{σ} ≈ [S]{s} {δσ} ≈ [S]{δs}
 
where the matrices [N ] and [B] contain the displacement interpolation functions and their
derivatives (strain interpolation matrix), respectively. [N c ] and [G] are matrices containing
some enrichment terms for displacement and strain respectively. [S] is a stress
interpolation matrix. {d} , {d c } , {e} and {s} collect the degrees of freedom corresponding
to nodal displacement, enhanced displacement modes, enhanced strain modes, and stress
parameters, respectively. If we consider the variation of the Hu-Washizu’s principle:

∫{δε} {σ} dV + δ∫{σ} {∇ } ∫ ∫


u − ε dV = {δu}T { ρb} dV + {δu}T {t *} dS
T T sym

V
144244
3 V 44424443
1 V Sσ (5.206)
14444442444444
3
1 2 3

we can obtain:

∫ ∫ {[B]{δd} + [G]{δe}} {σ} dV = ∫ {[B]{δd}} {σ} dV + ∫ {[G]{δe}} {σ} dV


T T T
(1) ⇒ {δε}T {σ} dV =
V V V V


= {δd}T [B]T {σ} dV + {δe}T [G]T {σ} dV
V

V

University of Castilla-La Mancha Draft By: Eduardo W. V. Chaves (2016)


Ciudad Real – Spain
5 INTRODUCTION TO: CONSTITUTIVE EQUATIONS, IBVP, AND IBVP SOLUTION STRATEGIES 501

∫ { } ∫
(2) ⇒ δ {σ}T ∇ symu − ε dV = {δσ}T ∇ symu − ε dV + {σ}T ∇ symδu − δε dV { } ∫ { }
V V
144424443 V
1444
424444
3
2 .1 2.2


r
{
(2.1) ⇒ {δσ}T ∇ symu − ε dV = } ∫ {[S]{δs}} {∇ ([N ]{d} + [N ]{d }) − ([B]{d} + [G]{e})}dV
T sym
c c
V V

{
= {δs}T [S]T ∇ sym ([N ]{d} + [N c ]{d c }) dV − {δs}T [S]T {([B]{d} + [G]{e})}dV
∫ } ∫
V V

∫ [S] {∇ ([N]{d})}+ [S] {∇ ([N ]{d })}dV − {δs} ∫ [S] {([B]{d} + [G]{e})}dV
T T
= {δs}T sym sym
c c
T T

V V

= {δs}T [S]T [B]{d} + [S]T [B c ]{d c }dV − {δs}T [S]T [B]{d} + [S]T [G]{e}dV
∫ ∫
V V


= {δs}T [S]T [B c ]{d c }dV − {δs}T [S]T [G]{e}dV
V

V

∫ {
= {δs}T [S]T [B c ]{d c } − [G]{e} dV }
V

where we have considered {∇ sym ([N ]{d})}= [B]{d} and {∇ sym ([N c ]{dc })}= [B c ]{dc } .


r
{
(2.2) ⇒ {σ}T ∇ sym δu − δε dV = } ∫ {∇
sym
}
δu − δε {σ}dV
T

V V

∫ { ∇ ([N]{δd} + [N ) − ([B]{δd} + [G]{δe}) } {[S]{s}}dV


sym T
= c ]{δd c }
V

∫{ ∇ ([N ]{δd}) + ∇ sym ([N c ]{δd c }) − ([B]{δd} + [G]{δe}) } {[S]{s}}dV


sym T
=
V

∫ { [B]{δd} + [B ]{δd } − ([B]{δd} + [G]{δe}) } {[S]{s}}dV


T
= c c
V

= ∫ { [B c ]{δd c } − [G]{δe} } {[S]{s}}dV = ∫ { {δd } [B ]


T
c
T
c
T
− {δe}T [G]T }{[S]{s}}dV
V V

= {δd c }T [B c ]T {[S]{s}}dV − {δe}T [G]T {[S]{s}}dV


∫ ∫
V V

∫ ∫
(3) ⇒ {δu}T {ρb} dV + {δu}T {t *} dS = {δd}T { f ext } + {δd c }T { f c }
V Sσ

Taking into account the previous terms, the equation in (5.206) becomes:

University of Castilla- La Mancha Draft By: Eduardo W. V. Chaves (2016)


Ciudad Real – Spain
502 SOLVING PROBLEMS BY MEANS OF CONTINUUM MECHANICS

∫ ∫ ∫ {
{δd}T [B]T {σ} dV + {δe}T [G]T {{σ} − {[S]{s}}} dV + {δs}T [S]T [B c ]{d c } − [G]{e} dV + }
V V V

+ {δd c }T [B c ]T {[S]{s}}dV = {δd}T { f ext } + {δd c }T { f c }



V
(5.207)
r
Since {u} , {ε} and {σ} are variables of the independent fields, so, we can say that:

 V

{δd}T [B]T {σ} dV = {δd}T { f ext }


{δe} [G] {{σ} − {[S]{s}}} dV = {0}

T T

 V
 (5.208)
 T

T
{
{δs} [S] [B c ]{d c } − [G]{e} dV = {0} }
 V

{δd }T [B ]T {[S]{s}}dV = {δd }T { f } = {0}
 c
 V
c ∫ c c

If we consider { f c } = {0} , the above equations can be rewritten as follows:




 [B]T {σ} dV = { f ext }
V


 [G] {{σ} − {[S]{s}}} dV = {0}

T

V
 (5.209)


T
{
 [S] [B c ]{d c } − [G]{e} dV = {0} }
V

 [B ]T {[S]{s}}dV = {0}
∫ c
V
Taking into account that the stress-strain relationship is given by the following expression:
{σ} = [C ] {ε} = [C ] {[B]{d} + [G]{e}} (5.210)
and by substituting into the equation in (5.209) we can obtain:


V V

 [B]T [C ][B] dV {d} + [B]T [C ][G] dV {e} = { f ext } (a)


∫ ∫ ∫
T T T
 [G] [C ][B] dV {d} + [G] [C ][G] dV {e} − [G] [S] dV {s} = {0} (b)
V V V
 (5.211)

∫ ∫
T T
 [S] [B c ] dV {d c } − [S] [G] dV {e} = {0} (c)
V V

 [B ]T [S] dV {s} = {0}
∫
V
c (d)

Rewriting the above equation in matrix form we can obtain:

University of Castilla-La Mancha Draft By: Eduardo W. V. Chaves (2016)


Ciudad Real – Spain
5 INTRODUCTION TO: CONSTITUTIVE EQUATIONS, IBVP, AND IBVP SOLUTION STRATEGIES 503

[B]T [C ][B] [B]T [C ][G] 0 0   {d}  { f ext }


 T   {e}   {0} 
[G] [C ][B] [G] [C ][G] − [G] [S]
T T
0  dV    
  = 
0 − [S]T [G] 0 [S]T [B c ]  {s}   {0}  (5.212)
 
 0 0 [B c ]T [S] 0  {d c }  {0} 

Let us suppose that we do not introduce any displacement enhancement terms, thus
{d c } = {0} → [B c ] = [0] , with that the equation in (5.211)(c) becomes:

∫ [S] [G] dV {e} = {0}


T
(5.213)
V

Thus, piecewise constant stress functions {σ} will require [S] = [1] (unit matrix). The
compatibility conditions (5.213) now read:

V
∫ [G] dV = [ 0] (5.214)

Discontinuity on displacement and strain fields – Applying the Principle of Virtual


Work
As we have seen before, virtual work is the work done by real force acting through virtual
displacements. A virtual displacement is any displacement consistent with the constraints
of the structure, i.e. which satisfies the boundary conditions.
The principle states that the virtual work of the internal forces must be equal to the virtual
work of the external forces:
r r r r r
∫ σ : ε dV = ∫ t * ⋅ u dS + ρ (b − u

&&) ⋅ u dV
V
14243 Sσ V
Total internal
144444244444
3 (5.215)
Total external virtual work
virtual work
Wint Wext

r
for all the admissible virtual displacements u .
Let us consider a discretized system where we can say that all the forces are applied in the
nodes of the finite element (CST-Constant Strain Triangle), (see Figure 5.23).

University of Castilla- La Mancha Draft By: Eduardo W. V. Chaves (2016)


Ciudad Real – Spain
504 SOLVING PROBLEMS BY MEANS OF CONTINUUM MECHANICS

Fy3 u 3y
Fx3 ŝ u 3x
3 3
Fy2
n̂ u 2y
0 α 4y

0 α 4x
4 2 Fx2 4 2 u x2
Fy1 u 1y

1 Fx1 1 u 1x
(a) nodal forces (b) nodal displacements

Figure 5.23: Discontinuous CST finite element.

The nodal forces and nodal displacements can be expressed as follows:


 Fx1   u1x 
   1
 Fy1  u y 
 Fx2  u 2 
   x2 
 {F}   Fy2   {a e }   u y 
 =  ;  = 3 (5.216)
 {F}   Fx3   {α e }   u x 
 Fy3   u 3y 
   4
 0  α x 
 0  α 4 
  y
Hence, the external virtual work becomes:

Wext = {a*e }T {F} + {α *e }T {F} = {a*e }T { }


{F}
{α *e }T   (5.217)
{F}
We consider the strain field and the virtual strain field are compound by two parts:
~ ~
{ε} = {ε} + {ε} ; {ε * } = {ε * } + {ε * } (5.218)
thus the internal virtual work becomes:


~
∫ { ~ T
Wint = {σ}T {ε} d V = {σ}T {ε * } + {ε * } d V = {ε *} + {ε *} {σ} d V } ∫{ } (5.219)
V V V

Symmetric formulation
The discretization for the first approximation is:
{a e } {a* }
{ε} = [B] {a} + [Ge ]{α e } = {[B] [Ge ]} ; {ε * } = [B]{a*e } + [Ge ]{α*e } = {[B] [Ge ]} e* 
123 1 424 3 123 1 424 3
{α e } {α e }
{ε } {~ε } {ε* } {~ε* }
(5.220)

Notice that we have used the same approximation function [B] , [Ge ] for virtual and real
strains. Then, the stress field can be written as follows:
{a e }
{σ} = [C ]{ε} = [C ]{[B] [Ge ]}  (5.221)
{α e }

University of Castilla-La Mancha Draft By: Eduardo W. V. Chaves (2016)


Ciudad Real – Spain
5 INTRODUCTION TO: CONSTITUTIVE EQUATIONS, IBVP, AND IBVP SOLUTION STRATEGIES 505

By replace the approximations (5.220) and (5.221) into the equation (5.219), the internal
virtual work becomes:
T
 {a* }
∫{ }
~ T {a e }
Wint = {ε } + {ε *} {σ} d V = {[B] [Ge ]} e*  [C ]{[B] [Ge ]}
*
∫  dV
V V {α e } 
 {α e }
T
{a* } {a e }
=  e*  {[B] [Ge ]} [C ]{[B] [Ge ]}
∫ (5.222)
T
 dV
V
{α e } {α e }

[ ]∫ [[GB]] [C ]{[B] {a e }


T
= {a*e }T {α *e }T T
[Ge ]} d V  
V e  {α e }
By apply Wext = Wint , (see Eq. (5.217) and (5.222)), we can obtain

{F}  [B]T [C ][B] [B]T [C ][Ge ]  {a e }


 =
{F} V
∫ 

 dV
[Ge ]T [C ][B] [Ge ]T [C ][Ge ]
 
{α e }
(5.223)
e

and considering the traction vector continuity, i.e. {F} = {0} , we obtain:

{F}  [B]T [C ][B] [B]T [C ][Ge ]  {a e }


 =  T ∫ T  dV
{0} Ve [Ge ] [C ][B] [Ge ] [C ][Ge ]
 
{α e } (5.224)
144444424444443
= [K e ]

Anti-symmetric formulation
Now consider the real and virtual strain approximation by:

{a e }
{ε} = [B] {a e } + [Ge ]{α e } = [[B] [Ge ]] 
1424 3 1 424 3 {α e }
{ε } {~ε }
(5.225)
{ε * } = [B]{a*e } + [G*e ]{α *e } = [B] [ ]
{a* }
[G*e ]  e* 
{α e }

where we are considering different approximation functions for virtual and real strains i.e.
[Ge ] ≠ [G*e ] .
Using equation (5.219), and discretization (5.225) we can obtain:

{ }∫ [[GB]]  {a e }
T
Wint = {a*e }T {α *e }T * T [C ]{[B] [Ge ]} dV   (5.226)

Ve e  {α e }

Considering Wext = Wint and considering the traction vector continuity, we can obtain:

{F}  [B]T [C ][B] [B]T [C ][Ge ]  {a e }


  =  *T ∫ * T  dV
{0} Be [Ge ] [C ][B] [Ge ] [C ][Ge ]
 
{α e } (5.227)
144444424444443
= [K e ]

University of Castilla- La Mancha Draft By: Eduardo W. V. Chaves (2016)


Ciudad Real – Spain
506 SOLVING PROBLEMS BY MEANS OF CONTINUUM MECHANICS

According to Jirásek(1998) there are three major classes of these models:


• SOS (Statically Optimal Symmetric) formulation cannot properly reflect the
kinematics of a completely open crack but it gives a natural stress continuity
condition;
• KOS (Kinematically Optimal Symmetric) formulation describes the kinematic
aspects satisfactorily but leads to an awkward relationship between the stress in the
continuous part of the element and the tractions across the discontinuity line.
These findings motivate the development of the nonsymmetric;
• SKON (Statically and Kinematically Optimal Nonsymmetric) formulation, which
combines the strong points of each of the symmetric formulations.

Reference
JIRÁSEK, M. (1998). Finite elements with embedded cracks. LSC Internal Report 98/01,
April.

University of Castilla-La Mancha Draft By: Eduardo W. V. Chaves (2016)


Ciudad Real – Spain
5 INTRODUCTION TO: CONSTITUTIVE EQUATIONS, IBVP, AND IBVP SOLUTION STRATEGIES 507

Problem 5.26
Consider a rod of length L and cross-sectional area A which undergoes deformation
because of its own weight, (see Figure 5.24 (a)). The rod is fixed at the top and is in static
equilibrium. Use the total potential energy to obtain an analogous equation as the one
obtained in Problem 5.25 in NOTE 4, i.e. obtain an equivalent equation
[ K (e ) ]{u( e ) } = {F (e ) } associated with this problem. Obtain also the displacement field.
Hypothesis: Homogeneous isotropic linear elastic material, small deformation regime.

u=0

y , x2 U(1) = 0

z, x3 L
dV EA 2
r
ρb
U( 2 )
A
L
g 2
L g 
  U(3)
bi =  0 
0 
 
x, x1 x

a) 3D b) 1D

Figure 5.24

Solution:
To find out which displacement approach we must adopt, we will analyze the equilibrium
r r
equations ( ∇ ⋅ σ + ρb = 0 ):
σ ij , j + ρb i = σ i1,1 + σ i 2, 2 + σ i 3,3 + ρb i = ρ&u&i = 0 i
 ∂σ11 ∂σ12 ∂σ13
 + + = − ρ b1
 ∂x ∂x ∂x
σ11,1 + σ12, 2 + σ13,3 + ρb1 = 0 1 2 3
  ∂σ 21 ∂σ 22 ∂σ 23
⇒ σ 21,1 + σ 22, 2 + σ 23,3 + ρb 2 = 0 ⇒  + + = − ρb 2
  ∂ x1 ∂ x 2 ∂ x 3
σ 31,1 + σ 32, 2 + σ 33,3 + ρb 3 = 0  ∂σ 31 ∂σ 32 ∂σ 33
 + + = − ρb 3
 ∂x1 ∂x2 ∂x3
and for this problem we have:
σ 0 0  ε 0 0
σ ij =  0 0 0 ; ε ij = 0 0 0 ⇒ σ11 = C1111
e
ε11 ⇒ σ = Eε
 0 0 0 0 0 0

Then, the equilibrium equations reduce to:

University of Castilla- La Mancha Draft By: Eduardo W. V. Chaves (2016)


Ciudad Real – Spain
508 SOLVING PROBLEMS BY MEANS OF CONTINUUM MECHANICS

 ∂σ11 ∂σ
 ∂x = − ρg Engineerin
  g  = − ρg
notation
 →
∂x
 1
⇒ 0 = 0 (5.228)
0 = 0


Note that the term ρg is constant in the rod, and according to the above equilibrium
equation, the stress σ must be a linear function in x . And if we consider that σ = Eε , ε
also requires a linear function in x , and as a consequence the displacement u must be a
∂u
quadratic function in x since ε = .
∂x
Then, the displacement field will be approached by the quadratic function
( u ( x) = a1 + a 2 x + a 3 x 2 ), hence we will need three points to be able to define this function.
L
We will adopted the points: x = 0 , x = and x = L . With that we can obtain:
2
u ( x = 0) = U(1) = a1 
 U(1)  1 0 0   a1 
L (2) L L  2
 (2)   L L2   
u ( x = ) = U = a1 + a2 + a3  ⇒ U  = 1  a 2 
2 2 4 U(3)   2 4  
u ( x = L) = U = a1 + a2 L + a3 L 
( 3) 2   1 L L2  a3 

Taking the reverse form of the above equation we can obtain:

a1 = U (1)
 a1   L 2
0 0  U  (1)

  1    (2)   − 3 (1) 4 ( 2 ) 1 (3)
a 2  = 2  − 3L 4 L − L  U  ⇒ a 2 = U + U − U
a  L  2 4 L L
 3  − 4 2  U (3)  
 2 (1) 4 ( 2 ) 2 (3)
a 3 = 2 U − 2 U + 2 U
 L L L
With that the displacement field in terms of U (1) , U( 2) , and U (3) is given by:
 − 3 (1) 4 ( 2 ) 1 (3)   2 (1) 4 ( 2) 2 (3)  2
u = a1 + a2 x + a3 x 2 = U(1) +  U + U − U x +  2 U − 2 U + 2 U x
 4 L L  L L L 
by simplifying the above equation we can obtain:
 3x 2 x 2  (1)  4 x 4 x 2  ( 2 )  − x 2 x 2  (3)
u ( x) = 1 − + 2 U +  − 2 U +  + 2 U
 L L   L L   L L 
= N1U(1) + N 2U( 2 ) + N 3U(3)
U(1)  (5.229)
 
= [N1 ( x) N 2 ( x) N 3 ( x)]U( 2 ) 
U(3) 
 
= [ N ( x)]{u(e ) }
where N 1 ( x) , N 2 ( x) and N 3 ( x) are the shape functions.
The goal now is to express the total potential energy in terms of U (1) , U ( 2) and U (3) .

University of Castilla-La Mancha Draft By: Eduardo W. V. Chaves (2016)


Ciudad Real – Spain
5 INTRODUCTION TO: CONSTITUTIVE EQUATIONS, IBVP, AND IBVP SOLUTION STRATEGIES 509

The term U ext , (see equation (5.184)), becomes:


r r r r r r
U ext = ∫ t * ⋅ udS σ + ( ρb) ⋅ udV = ( ρb) ⋅ udV = ρgu ( x) Adx = ρgA u ( x)dx
∫ ∫ ∫ ∫
Sσ V V x x
L
 3x 2 x 2  (1)  4 x 4 x 2  ( 2)  − x 2 x 2  ( 3) 

0 
L ∫
= ρgA 1 − + 2
L
U +  −


 L
 L2
U + 


 L + L2

U  dx

 
After the integration is taken place we can obtain:
L 2 L ( 2 ) L ( 3) 
U ext (U (1) , U ( 2 ) , U (3) ) = ρgA U (1) + U + U  (5.230)
6 3 6 
The term U int for this problem is the same as the one given by the equation in (5.193), i.e.
2 L 2
1 1  ∂u ( x)  1  ∂u ( x) 
∫ ∫ ∫
int
U = Eε 2 dV = E  dV = AE   dx (5.231)
2V 2 V  ∂x  20  ∂x 
where
∂u ∂  3 x 2 x 2  (1)  4 x 4 x 2  ( 2)  − x 2 x 2  ( 3) 
= 1 − + 2 U +  − 2 U +  + 2 U 
∂x ∂x  L L   L L   L L 
 
 − 3 4 x  (1)  4 8 x  ( 2)  − 1 4 x  (3)
= + 2 U +  − 2 U +  + 2 U
 L L  L L   L L 
thus
L 2
EA  − 3 4 x  (1)  4 8 x  ( 2 )  − 1 4 x  (3) 

int
U =  + 2 U +  − 2 U +  + U  dx
2 0  L L  L L   L L2  
By solving the above integral we can obtain:
EA  (1) 2
+ 16U ( 2 ) + 7U (3) + 2U (1) U (3) − 16U (1) U ( 2 ) − 16U ( 2 ) U (3) 
2 2
U int (U (1) , U ( 2 ) , U (3) ) = 7U
6 L  
(5.232)
The total potential energy is given by:
Π (U (1) , U ( 2 ) , U (3) ) = U int − U ext
EA  (1) 2
+ 16U ( 2 ) + 7U (3) + 2U (1) U (3) − 16U (1) U ( 2 ) − 16U ( 2 ) U (3) 
2 2
= 7U
6 L  
L 2 L ( 2 ) L ( 3) 
− ρgA U (1) + U + U 
6 3 6 
As we are looking for the stationary state, the following must be fulfilled:
 ∂Π (U (1) , U ( 2 ) , U (3) ) ρgAL
 =0 ⇒
EA
(
7U (1) − 8U ( 2 ) + U (3) −) =0
 ∂U (1) 3L 6
 ∂Π (U (1) , U ( 2 ) , U (3) ) 2 ρgAL
 =0 ⇒
EA
(
− 8U (1) + 16U ( 2 ) − 8U (3) − ) =0
 ∂U ( 2 ) 3L 3
 ∂Π (U (1) , U ( 2 ) , U (3) ) ρgAL


=0 ⇒
EA (1)
(
U − 8U ( 2) + 7U (3) − )=0
∂U (3) 3L 6

Rearranging the above set of equations in matrix form we can obtain:

University of Castilla- La Mancha Draft By: Eduardo W. V. Chaves (2016)


Ciudad Real – Spain
510 SOLVING PROBLEMS BY MEANS OF CONTINUUM MECHANICS

 7 − 8 1  U 
(1)
1 
EA    ( 2 )  ρgAL  
− 8 16 − 8 U  = 4  ⇔ [ K (e ) ]{u(e ) } = {F (e ) } (5.233)
3L   ( 3)  6  
 1 − 8 7  U 1 
 
Note that [ K (e ) ] has no inverse, since det[ K (e ) ] = 0 . To solve the problem we have to
introduce the boundary conditions. According to the problem statement, the displacement
at x = 0 is equal to zero, i.e. U (1) = 0 . We apply this boundary condition by eliminate the
first line and column of the system (5.233), in other words we eliminate the terms
associated with the degree-of-freedom U (1) , i.e.:
 7 − 8 1  U  0  U(1) 
(1)
1 1 0 0
EA    ( 2)  ρgAL   EA    ( 2 )  ρgAL  
− 8 16 − 8 U  = 4  or 0 16 − 8 U  = 4 
3L   ( 3)  6   3L   ( 3)  6  
 1 − 8 7  U 1 0 − 8 7  U 1 
   

EA  16 − 8 U ( 2 )  ρgAL 4
⇒   =  
3L  − 8 7  U (3)  6 1 
−1 −1
 EA  16 − 8   EA  16 − 8  U ( 2 )  ρgAL  EA  16 − 8  4
⇒        
 3L  − 8 7   U (3)  = 6  3L  − 8 7   1 
 3L  − 8 7            
−1
U ( 2 )  ρgAL  EA  16 − 8  4
⇒  ( 3)  =  
U 6  3L  − 8 7   1 
      
U  ρgAL  L 7 8 4
( 2)
⇒  ( 3)  =     
U  6  16 EA 8 1 1 
U ( 2 )  ρgL2 3
⇒  ( 3)  =  
U  8E 4
Now, if we substitute the values of U (1) , U( 2) , and U (3) into the displacement field, (see
equation (5.229)), we can obtain:
 3x 2 x 2  (1)  4 x 4 x 2  ( 2 )  − x 2 x 2  (3)
u = 1 − + 2 U +  −
  L
U +  
 L + L2 U
 L L   L2   
 3 x 2 x   4 x 4 x  3 ρgL   − x 2 x 2  4 ρgL2
2 2  2

= 1 − + 2 0 +  − 2  + + 2  
 L L   L L  8E   L L  8 E 

By simplifying the above equation we can obtain:
ρg
u (Q ) = (2 Lx − x 2 ) (5.234)
2E
which is also the exact solution for the proposed problem. Then, the strain and stress field
can be obtained as follows:
∂u ( x) ∂  ρg  ρg
ε (Q ) = = (2 Lx − x 2 ) = ( L − x) (5.235)
∂x ∂x  2 E  E
and
σ ( Q ) = ε ( Q ) E = ρg ( L − x ) (5.236)
If we replace the nodal displacement into the total potential energy we can obtain:

University of Castilla-La Mancha Draft By: Eduardo W. V. Chaves (2016)


Ciudad Real – Spain
5 INTRODUCTION TO: CONSTITUTIVE EQUATIONS, IBVP, AND IBVP SOLUTION STRATEGIES 511

1 ( ρg ) 2 AL3
Π (Q ) = −
6 E
r
NOTE 1: The shape functions [N ( x )]
Note that the shape functions obtained (5.229) are:
 N 1 ( x = 0) = 1
3x 2 x 2 
N 1 ( x) = 1 − + 2 ⇒ N1 ( x = 2 ) = 0
L
L L  N ( x = L) = 0
 1
 N 2 ( x = 0) = 0
4x 4x 2 
N 2 ( x) = − 2 ⇒ N 2 ( x = 2 ) = 1
L
L L  N ( x = L) = 0
 2
 N 3 ( x = 0) = 0
− x 2x 2 
N 3 ( x) = + 2 ⇒ N 3 ( x = 2 ) = 0
L
L L  N ( x = L) = 1
 3
Note also that N 1 ( x) + N 2 ( x) + N 3 ( x) = 1 holds. And these functions are drawn into the
domain as indicated in Figure 5.25.

N2  3x 2 x 2
N1 N3  N1 = 1 − + 2
 L L
 4x 4x 2
1 1 1 N 2 = − 2
 L L
2 3 x  − x 2x2
1 N
 3 = + 2
 L L
x=0 L x=L
x=
2 N1 ( x ) + N 2 ( x) + N 3 ( x ) = 1

Figure 5.25: Shape functions (1D) – quadratic function.

NOTE 2: Analytical solution (the exact one) – by using the direct integration
∂σ
We start from the equilibrium equation = − ρg , (see equation (5.228)), and by
∂x
integrating the equation over x we can obtain:
∂σ
∂x
= − ρg integratin
 g → ∂σ = − ρg∂x ∫ ∫ ⇒ σ = − ρgx + C1

The constant of integration can be obtained at x = 0 . In this situation the total force at
x = 0 is given by F = ρgV = ρgAL , and the stress can be obtained by
F ρgAL
σ( x = 0) = = = ρgL . Then, the constant of integration becomes
A A
σ( x = 0) = C1 = ρgL . Hence, the stress field becomes:
σ = − ρgx + ρgL = ρg ( L − x)

University of Castilla- La Mancha Draft By: Eduardo W. V. Chaves (2016)


Ciudad Real – Spain
512 SOLVING PROBLEMS BY MEANS OF CONTINUUM MECHANICS

Using the constitutive equation, the strain field can be obtained as follows:
σ ρg ( L − x)
σ = Eε ⇒ ε= =
E E
∂u
Taking into account the relationship ε = (kinematic equation), and by integrating the
∂x
equation over x we can obtain:
∂u  ρg ( L − x)  ρg  x2 
∫ ∫ ∫  + C2
integrating
ε=  → ∂u = ε∂x =  ∂x ⇒ u ( x) = Lx −
∂x  E  E  2 

At x = 0 there is no displacement, so, u ( x = 0) = 0 ⇒ C2 = 0 , then, the displacement field,


(see Figure 5.26), becomes:
ρg  x 2  ρg
= =L ρgL2
u ( x) = Lx − (2 Lx − x 2 ) x
 → u ( x = L) = (5.237)
E  2  2 E 2E

σ( x = 0) = ρgL
ρgL
ε ( x = 0) =
E

ρg
u= (2 Lx − x 2 )
2E
L + ∆L ρg
ε= ( L − x) σ = ρg ( L − x )
E

x x x

a) Displacement (x-direction) b) Strain c) Stress

Figure 5.26

NOTE 3: Note that for simple problems the analytical solution is very easy to be obtained,
and this solution serves to indicate how good is the technique employed. Note that the
analytical solution (equation (5.237)), is the same as the numerical solution (5.234) in which
we have used the quadratic function to approach the displacement field, and if we consider
a cubic function to approach the displacement field the solution must be the same. Let us
check this fact:
Displacement field (cubic function): u ( x) = a1 + a2 x + a3 x 2 + a4 x 3 = a2 x + a3 x 2 + a4 x 3 .
Note the, at x = 0 there is no displacement, so, u ( x = 0) = a1 = 0 .
∂u
Then, u ( x) = a2 x + a3 x 2 + a4 x 3 ⇒ = a2 + 2a3 x + 3a4 x 2
∂x

University of Castilla-La Mancha Draft By: Eduardo W. V. Chaves (2016)


Ciudad Real – Spain
5 INTRODUCTION TO: CONSTITUTIVE EQUATIONS, IBVP, AND IBVP SOLUTION STRATEGIES 513

Internal Potential in function of (a 2 , a 3 , a 4 ) :


L 2 L
AE  ∂u ( x)  AE
∫ ∫
int
U =   dx = (a2 + 2a3 x + 3a4 x 2 ) 2 dx
2 0  ∂x  2 0
AEL
= (27 L4 a42 + 45 L3 a4 a3 + 30 L2 a2 a4 + 20 L2 a32 + 30 La2 a3 + 15a22 )
30
External Potential in function of (a 2 , a 3 , a 4 ) :
L
1 1 1 
x
∫ 0

U ext = ρgA u ( x)dx = ρgA (a 2 x + a 3 x 2 + a 4 x 3 )dx = ρgA L2 a 2 + L3 a 3 + L4 a 4 
2 3 4 
The total potential energy in function of (a 2 , a 3 , a 4 ) :
Π (a2 , a3 , a4 ) = U int − U ext
AEL
= (27 L4 a42 + 45 L3 a4 a3 + 30 L2 a2 a4 + 20 L2 a32 + 30 La2 a3 + 15a22 )
30
1 1 1 
− ρgA L2 a2 + L3 a3 + L4 a4 
 2 3 4 
As we are looking for the stationary state, the following must be fulfilled:
 ∂Π (a2 , a3 , a4 ) EAL ρgAL2
 =0 ⇒ (30 L2 a4 + 30 La3 + 30a2 ) − =0
 ∂a2 30 2
 ∂Π (a , a , a ) EAL ρgAL3

2 3 4
=0 ⇒ (45 L3 a4 + 40 L2 a3 + 30 La2 ) − =0
 ∂ a 3 30 3
 ∂Π (a , a , a ) EAL ρgAL4
 2 3 4
=0 ⇒ (54 L4 a4 + 45 L3 a3 + 30 L2 a2 ) − =0
 ∂a4 30 4
Simplifying and rearranging the above set of equations in matrix form we can obtain:
 30 30 L 30 L2   a2   6L  a2  2 ρgL 
E  3   ρg  2  Solve   1 
30 L 40 L2 45L   a3  = 4 L  
→  a3  = − ρg 
30  2  2E 
30 L
2
45L3 54 L4   a4   3L3  a4   0 

Then, the displacement field ( u ( x) = a2 x + a3 x 2 + a4 x 3 ) becomes:


1 ρg
u ( x) = a2 x + a3 x 2 + a4 x 3 = (2 ρgLx − ρgx 2 ) = (2 Lx − x 2 )
2E 2E
which is the same solution as the one provided by the quadratic function used to approach
the displacement field.
Next, let us adopt a linear function to approach the displacement field u ( x) = a2 x , then
Internal Potential:
L 2 L
AE  ∂u ( x)  AE AEL 2
∫ ∫
int
U =   dx = (a2 ) 2 dx = a2
2 0  ∂x  2 0 2

External Potential:

University of Castilla- La Mancha Draft By: Eduardo W. V. Chaves (2016)


Ciudad Real – Spain
514 SOLVING PROBLEMS BY MEANS OF CONTINUUM MECHANICS

L
1

x

U ext = ρgA u ( x)dx = ρgA (a 2 x)dx =
0
2
ρgAL2 a 2

The total potential energy:


AEL 2 1
Π (a 2 ) = U int − U ext = a 2 − ρgAL2 a 2
2 2
The equilibrium point:
∂Π (a 2 ) ρgAL2 ρgL
=0 ⇒ EALa 2 − =0 ⇒ a2 =
∂a 2 2 2E
Then
ρgL ∂u ( L ) ρgL 1
u ( L ) = a2 x = x ; ε ( L) = = ; σ ( L ) = Eε = ρgL
2E ∂x 2E 2
Note that, for this case a linear function is not a good approach.
AEL 2 1 1 ( ρg ) 2 AL3
The total Π ( L ) = a2 − ρgAL2 a2 = −
2 2 8 E
NOTE 4:
Next, we will establish the stiffness matrix [ K (e ) ] of the rod element by considering the
linear approximation for the displacement field. By adopting the linear function
u ( x) = a1 + a 2 x we can obtain:

u ( x = 0) = U(1) = a1  U(1)  1 0   a1  Reverse  a1  1  L 0 U(1) 


 ⇒  ( 2)  =      →a  = L  − 1 1  ( 2) 
u ( x = L ) = U( 2 ) = a1 + a2 L  U  1 L  a2   2   U 
Then, the displacement field becomes:
 −1 1   x x
u ( x) = a1 + a2 x = U(1) +  U(1) + U( 2 )  x = 1 − U(1) + U( 2)
 L L   L L
 x   x   U(1) 
= 1 −      ( 2 ) 
 L   L   U 
= [ N ]{u( e ) }
whose equation has already been obtained in Problem 5.25 (NOTE 4). Then, the stiffness
matrix will be the same, but the nodal forces will be not the same.
Internal Potential:
L 2 L 2
AE  ∂u ( x)  AE  − 1 (1) 1 ( 2 )  AE  (1) 2
 U − 2U (1)U ( 2 ) + U ( 2) 
2
U int = ∫

2 0  ∂x 
 dx =  U + U  dx =
2 0 L L ∫ 2L  

External Potential:
L
 x  x  1
x
∫ 0 

U ext = ρgA u ( x)dx = ρgA 1 − U (1) + U ( 2 )  dx = ρgAL(U (1) + U ( 2 ) )
L L  2

The total potential energy:


AE  (1) 2 1
 U − 2U (1)U ( 2 ) + U ( 2 )  − ρgAL(U (1) + U ( 2 ) )
2
Π (U (1) , U ( 2 ) ) = U int − U ext =
2L   2

University of Castilla-La Mancha Draft By: Eduardo W. V. Chaves (2016)


Ciudad Real – Spain
5 INTRODUCTION TO: CONSTITUTIVE EQUATIONS, IBVP, AND IBVP SOLUTION STRATEGIES 515

The equilibrium point:


∂Π (U (1) , U ( 2 ) ) AE 1 AE (1) 1
(1)
=0 ⇒ ( 2U (1) − 2U ( 2 ) ) − ρgAL = 0 ⇒ (U − U ( 2 ) ) = ρgAL
∂U 2L 2 L 2
∂Π (U (1) , U ( 2 ) ) AE 1 AE ( 2 ) 1
(2)
=0 ⇒ ( 2U ( 2 ) − 2U (1) ) − ρgAL = 0 ⇒ (U − U (1) ) = ρgAL
∂U 2L 2 L 2
Simplifying and rearranging the above set of equations in matrix form we can obtain:
EA  1 − 1 U(1)  1 1
− 1 1   ( 2 )  = ρgAL 1 ⇔ [ K (e ) ]{u(e ) } = {F ( e ) }
L   U  2 
As we have seen in the previous NOTE, there is an error when we use the linear
approximation for the displacement field. Next, we will divide the domain in sub-domain.
To establish the displacement field we will adopt a generic element, where the initial point
is x (i ) and the final point is x ( f ) , (see Figure 5.27 (a)).

U(1) = 0

x (i ) L
(1) 1 L(1) =
(EA) 2
U(i )
e
x( f ) U( 2 )
U( f ) L
(EA) ( 2) 2 L( 2 ) =
2

U(3)

x x

a) Generic element b) Discretization by using 2 elements

Figure 5.27

Taking into account the linear approximation u ( x) = a1 + a 2 x we can obtain:


u ( x = x (i ) ) = U(i ) = a1 + a2 x (i )   U(i )  1 x (i )   a1 
 ⇒  ( f ) =  ( f )  
u ( x = x ( f ) ) = U( f ) = a1 + a2 x ( f )  U  1 x  a2 
a  1  x ( f ) − x (i )   U(i ) 
→ 1  = ( f )
Inverse (i )   
a 2  ( x − x )  − 1 1  U( f ) 
Then, the displacement field becomes:
1 1
u ( x) = a1 + a 2 x ⇒ u ( x) = (f) (i )
( x ( f ) − x)U (i ) + ( f ) ( x − x (i ) )U ( f )
(x −x ) ( x − x (i ) )
and its derivative:

University of Castilla- La Mancha Draft By: Eduardo W. V. Chaves (2016)


Ciudad Real – Spain
516 SOLVING PROBLEMS BY MEANS OF CONTINUUM MECHANICS

∂u ( x) −1 1
= (f) U (i ) + ( f ) U( f )
∂x (i )
(x − x ) (x − x ) (i )

Let us divide the domain into 2 sub-domains (2 finite elements), (see Figure 5.27 (b)),
where:
Element e = 1 :
x (i ) = 0 
  (1)  2 x  (1)  2 x  ( 2 )
x( f ) =
L  u = 1 − U + 1 − U
   L  L 
2  ⇒  (1)
(i )
U = U (1)   ∂u =  − 2 U (1) +  2 U ( 2 )
  ∂x  L  L
U( f ) = U ( 2) 
Element e = 2 :
L 
x (i ) =   (2)  2 x  ( 2 )  2 x  ( 3)
2 u =  2 − U +  − 1U
 L 
x( f ) =L     L 
 ⇒  (2)
(i )
U =U ( 2)   ∂u =  − 2 U ( 2 ) +  2 U (3)
  ∂x  L  L
U ( f ) = U (3) 
Internal Potential:
L
L 2 2 L 2
1  ∂u ( x)  1  ∂u (1)
2
 1 ( 2 )  ∂u
( 2)

∫ ( AE ) (1)  ∫  dx +
∫   dx
int
U = AE   dx =  ( AE )  ∂x 
20  ∂x  20  ∂x  2L  
2
(1) ( 2)
( AE )  U (1) 2 − 2U (1) U ( 2 ) + U ( 2 ) 2  + ( AE )  U ( 2) 2 − 2U ( 2 ) U (3) + U (3) 2 
=
L   L  
External Potential:
L
2 L

∫ ∫
U ext = ρg Au ( x)dx = ρg A(1)u (1) dx + ρg A( 2)u ( 2 ) dx
x 0

L
2
1 1 1
= ρgA(1) LU(1) + ρgL( A(1) + A( 2) )U( 2) + ρgA( 2) LU(3)
4 4 4
The total potential energy
Π (U (1) , U ( 2 ) , U (3) ) = U int − U ext
( AE ) (1)  (1) 2 ( AE ) ( 2 )  ( 2) 2
− 2U (1) U ( 2 ) + U ( 2 )  + − 2U ( 2 ) U (3) + U (3) 
2 2
= U U
L   L  
1 1 1 
−  ρgA (1) LU (1) + ρgL( A (1) + A ( 2 ) )U ( 2 ) + ρgA ( 2) LU (3) 
4 4 4 

University of Castilla-La Mancha Draft By: Eduardo W. V. Chaves (2016)


Ciudad Real – Spain
5 INTRODUCTION TO: CONSTITUTIVE EQUATIONS, IBVP, AND IBVP SOLUTION STRATEGIES 517

The equilibrium point


∂Π (U ( a ) ) 2( AE ) (1) (1) 1
=0 ⇒ (U − U ( 2) ) − ρgLA (1) = 0
∂U (1) L 4
∂Π (U ( a ) ) 2( AE ) ( 2 ) ( 2 ) 2( AE ) ( 2) ( 2 ) 1
=0 ⇒ (U − U (1) ) + (U − U (3) ) − ρgL( A (1) + A ( 2 ) ) = 0
∂U ( 2 ) L L 4
∂Π (U ( a ) ) 2( AE ) ( 2 ) (3) 1
=0 ⇒ (U − U ( 2 ) ) − ρgLA ( 2 ) = 0
∂U (3) L 4
Rearranging the above set of equations in matrix form we can obtain:
   (1)   [ A(1) ] 
[( AE )(1) ] [−( AE )(1) ]  U  1
2   
[−( AE )(1) ] [( AE )(1) + ( AE )( 2 ) ] [−( AE )( 2 ) ]  U( 2 )  = ρgL [ A(1) + A( 2 ) ] 
L  4
 [−( AE )( 2 ) ] [( AE )( 2 ) ]  U(3)  
 [ A( 2 ) ]
 

[ K ]{u} = {F }
Note that the above matrix could have been obtained directly if we consider the stiffness
matrix of the element and the nodal force vector of the element e :
( EA)( e )  1 − 1 1 1
[k ( e) ] =   ; { f (e)} = ρg ( LA)( e)  
L(e )  − 1 1  2 1
L
Element e = 1 , ( L(1) = ):
2
2( EA)(1)  1 − 1 1 1 U(1) 
[k (1) ] = − 1 1  ; { f (1) } = ρgLA(1)   ; {u(1) } =  ( 2) 
L   4 1 U 
L
Element e = 2 , ( L( 2) = ):
2
2( EA)( 2 )  1 − 1 1 1 U( 2 ) 
[k ( 2) ] = − 1 1  ; { f ( 2) } = ρgLA( 2)   ; {u( 2 ) } =  (3) 
L   4 1 U 
Then the global stiffness matrix [K ] can be obtained by adding the contribution of each
stiffness matrix of the element into [K ] , and the same to the global nodal vector. This
process is called the assemble process.
Considering the ( EA) (1) = ( EA) ( 2) = EA the set of discrete equations becomes:

 1 − 1 0  U 
(1)
1 
2 AE    (2)  1  
 − 1 2 − 1 U  = ρgLA2
L 4
 0 − 1 1  U (3)  1 
 
 
Applying the boundary condition and solving the system the nodal displacement can be
obtained:
1 0 0   U  U(1) 
(1)
0  0 
2 AE    ( 2)  1    ( 2 )  ρgL2  
 0 2 − 1 U  = ρgLA2 Solve
 → U  = 3 
L 4 U(3)  8 E 4
0 − 1 1  U(3)  1 
   
   

University of Castilla- La Mancha Draft By: Eduardo W. V. Chaves (2016)


Ciudad Real – Spain
518 SOLVING PROBLEMS BY MEANS OF CONTINUUM MECHANICS

which matches the exact solution.


The procedure we have developed is the basis of the Finite Element Method which
basically consists of:
• Adopt an approach to the unknown field;
• Split (discretize) the domain into sub-domain (finite element);
• Set the stiffness matrix of each sub-domain and the nodal force vector;
• Assemble the global stiffness matrix of the structure;
• Apply the boundary condition;
• Solve the system.

For more detail about Finite Element Method the reader is referred to Zienkiewicz &
Taylor (1994), Bathe(1996).

University of Castilla-La Mancha Draft By: Eduardo W. V. Chaves (2016)


Ciudad Real – Spain
5 INTRODUCTION TO: CONSTITUTIVE EQUATIONS, IBVP, AND IBVP SOLUTION STRATEGIES 519

Problem 5.27
Show that:

∫t
r* r r& r r
[
( X , t ) ⋅ u dS 0σ + ρ 0 b( X , t ) − u

r
0 ]
X
r
&r&( Xr , t ) ⋅ u& dV = P : ∇ r u& dV
0 ∫ (5.238)
Sσ V0 V0

r
where u is the virtual displacement field, and P is the first Piola-Kirchhoff stress tensor.

Current
Reference F configuration
configuration S0σ

V0
V
B0 dV0 r r r
t * ( X , t ) = t *0 B r r
r r dV t* ( x, t )
u( X , t )
r r
r r r u( x , t )
ρ 0b( X , t ) = ρ 0b0
r r
ρb( x , t )

Solution:
r r r r
Although the variables t * ( X , t ) and b( X , t ) are not intrinsic variables of the reference
r r r
configuration like the variables ρ 0 , S 0 , V0 , for simplicity, we denote t * ( X , t ) = t *0 and
r r r
b( X , t ) = b 0 .
Remember also, (see Chapter 2 of the textbook), that:
r r r
D ∂ ∂x ( X , t ) ∂ ∂x ( X , t ) ∂u& i ( X , t) r
Fij ≡ F&ij = i
= i
= = u& i , J ( X , t )
Dt ∂t ∂X j ∂X j 142 ∂t 43 ∂X j
x&i

r& r
r& r ∂ u ( X , t)
or F = l ⋅ F = ∇ Xr u( X , t ) =
& r
∂X
r r
and l = F& ⋅ F −1 = ∇ Xr u& ( X , t ) ⋅ F −1
r r r r
F& −1 = − F −1 ⋅ l = − F −1 ⋅ ∇ Xr u& ( X , t ) ⋅ F −1 = − F −1 ⋅ ∇ xr u& ( x , t )

Taking into account the above relations, it is also valid that:


r& r r& r r& r
F& = ∇ Xr u ( X , t ) y F& −1 = − F −1 ⋅ ∇ Xr u ( X , t ) ⋅ F −1 = − F −1 ⋅ ∇ xr u ( x , t )

With that we can obtain:


& r
∫ P : F dV = ∫ P 0 iJ F&iJ dV0 = PiJ u& i , J ( X , t ) dV0

V0 V0 V0

(PiJ u& i ) , J = PiJ , J u& i + PiJ u& i , J ⇒ PiJ u& i , J = (PiJ u& i ) , J − PiJ , J u& i
thus:

University of Castilla- La Mancha Draft By: Eduardo W. V. Chaves (2016)


Ciudad Real – Spain
520 SOLVING PROBLEMS BY MEANS OF CONTINUUM MECHANICS

r r r
∫ P : F& dV0 = PiJ u& i , J ( X , t ) dV0 = (PiJ u& i ( X , t )) , J − PiJ , J u& i ( X , t ) dV0
∫ ∫
V0 V0 V0

& r r

V0
∫ P : F dV = ∫ (P 0
V0
iJ ∫
u& i ( X , t )) , J dV0 − PiJ , J u& i ( X , t ) dV0
V0

& r r

V0
∫ P : F dV = ∫ P 0
S0
iJ
V0

u& i ( X , t )nˆ J dS 0 − PiJ , J u& i ( X , t ) dV0

where we have applied the divergence theorem. The above in tensorial notation becomes:
r r
∫ P : F dV = ∫ (P ⋅ nˆ ) ⋅ u& ( X , t ) dS − ∫ (∇ ⋅ P) ⋅ u& i ( X , t ) dV0
& r
0 i 0 X
V0 S0 V0

Remember that the equations of motion in the reference configuration are given by:
r r
&r&( Xr , t )
∇ Xr ⋅ P + ρ 0b0 = ρ 0u ⇒ &r&( Xr , t )]
− ∇ Xr ⋅ P = ρ 0 [b0 − u
r& r r
and taking into account that F& = ∇ Xr u ( X , t ) and t *0 = P ⋅ nˆ we can obtain:
& r r
∫ P : F dV0 = (P ⋅ nˆ ) ⋅ u& i ( X , t ) dS0 − (∇ Xr ⋅ P ) ⋅ u& i ( X , t ) dV0
∫ ∫
V0 S0 V0
r& r r &r& r r
P : ∇ Xr u ( X , t ) dV0 = t *0 ⋅ u &r&( Xr , t )] ⋅ u& ( Xr , t ) dV

V0

( X , t ) dS0 + ρ 0 [b0 − u
S0
i ∫
V0
0

Reminder: Recall from Chapter 5 of the textbook that the stress power can be expressed
in different ways, namely:
1 1

w int (t ) = P : F& dV0 = S : E& dV0 = P : F& dV0 =
V0

V0 V0
∫ 2V ∫
S : C& dV0 =
V
J ∫
P : F& dV
0

ρ
= ∫ρ
V 0
P : F& dV = σ : D dV = { ∫
Jσ : D dV0 = τ : D dV0
V

V0 τ

V0

NOTE 1: Remember that neither P nor F& are in any configuration, but the scalar P : F&
is in the reference configuration.
NOTE 2: Taking into account the above. The total external virtual work can also be
expressed as follows:
r& r sym r& r r& r
∫σ : D dV = σ : ∇ xr u ( x , t ) dV = σ : ∇ xr u ( x , t ) dV = P : F& dV0 = P : ∇ Xr u ( X , t ) dV0
∫ ∫ ∫ ∫
 
V V V V V 0 0

r r
where we have used that D = l sym = [∇ xr u& ( x, t)]sym , (see Problem 2.35). Note that, due to
r& r r& r
the symmetry of σ the relationship σ : [∇ xr u ( x , t )]sym = σ : ∇ xr u ( x , t ) holds.
NOTE 3: From a Variational Principle point of view, (see Holzapfel (2000)), the equation
in (5.238) is also valid for a variation of the virtual field:
r r r r r r r
&r&( Xr , t )] ⋅ δ u dV = P : ∇ r δ u dV
t * ( X , t ) ⋅ δ u dS0 σ + ρ 0 [b( X , t ) − u

∫ ∫
V0
0 X 0 ∫
V0
(5.239)

University of Castilla-La Mancha Draft By: Eduardo W. V. Chaves (2016)


Ciudad Real – Spain
5 INTRODUCTION TO: CONSTITUTIVE EQUATIONS, IBVP, AND IBVP SOLUTION STRATEGIES 521

Problem 5.28
a) Show that the symmetric second-order tensor A = A sym can be split up into
A = A P + A S where A P = P P : A , A S = P S : A , with P P = (bˆ ⊗ bˆ ) ⊗ (bˆ ⊗ bˆ ) and
P S = I sym − (bˆ ⊗ bˆ ) ⊗ (bˆ ⊗ bˆ ) , where b̂ is a unit vector according to certain direction, and
I sym is the symmetric part of the fourth-order unit tensor. b) Show that the constitutive
equation for stress σ = C e : ε can be written as follows:
σ P  C PP C PS  ε  σijP  Cijkl
PP PS
Cijkl  ε kl 
 S  =  SP :   S  =  SP SS    (5.240)
σ   C C SS  ε  σij  Cijkl Cijkl  ε kl 

where
C PP = P P : C e : P P PP
Cijkl P
= Pijpq Cepqst Pstkl
P

C PS = P P : C e : P S PS
Cijkl P
= Pijpq Cepqst Pstkl
S

(5.241)
C SP = P S : C e : P P SP
Cijkl S
= Pijpq Cepqst Pstkl
P

C SS = P S : C e : P S SS S
Cijkl = Pijpq Cepqst Pstkl
S

Solution:
a) Bu using the Cartesian system the tensor A can be represented as follows:
A = A ij (eˆ i ⊗ eˆ j ) = A i1 (eˆ i ⊗ eˆ 1 ) + A i 2 (eˆ i ⊗ eˆ 2 ) + A i 3 (eˆ i ⊗ eˆ 3 )
= A 11 (eˆ 1 ⊗ eˆ 1 ) + A 21 (eˆ 2 ⊗ eˆ 1 ) + A 31 (eˆ 3 ⊗ eˆ 1 ) + A 12 (eˆ 1 ⊗ eˆ 2 ) + A 22 (eˆ 2 ⊗ eˆ 2 ) + A 32 (eˆ 3 ⊗ eˆ 2 )
+ A 13 (eˆ 1 ⊗ eˆ 3 ) + A 23 (eˆ 2 ⊗ eˆ 3 ) + A 33 (eˆ 3 ⊗ eˆ 3 )
and its components in matrix form are given by:
 A 11 A12 A13   0 A 12 A13   A11 0 0
A ij =  A 21 A 22 A 23  =  A 21 A 22 A 23  +  0 0 0 = A ij + A ij
 A 31 A 32 A 33   A 31 A 32 A 33   0 0 0
ˆ
Note also that the normal component A 11 = A (Ne1 ) (according to ê1 -direction) can also be
obtained by A11 = A : (eˆ 1 ⊗ eˆ 1 ) = (eˆ 1 ⊗ eˆ 1 ) : A , so the tensor A = A11 (eˆ 1 ⊗ eˆ 1 ) can be
written as follows:
A = (eˆ 1 ⊗ eˆ 1 ) A 11 ≡ (eˆ 1 ⊗ eˆ 1 ) ⊗ A 11 = (eˆ 1 ⊗ eˆ 1 ) ⊗ (eˆ 1 ⊗ eˆ 1 ) : A
thus
A = A − A = A − (eˆ 1 ⊗ eˆ 1 ) ⊗ (eˆ 1 ⊗ eˆ 1 ) : A = I sym : A − (eˆ 1 ⊗ eˆ 1 ) ⊗ (eˆ 1 ⊗ eˆ 1 ) : A
[ ]
= I sym − (eˆ 1 ⊗ eˆ 1 ) ⊗ (eˆ 1 ⊗ eˆ 1 ) : A

Although we have shown the equation by considering the unit vector ê1 , the above
equation is also valid for any unit vector, i.e.:
A P = (bˆ ⊗ bˆ ) ⊗ (bˆ ⊗ bˆ ) : A = P P : A
[ ]
A S = I sym − (bˆ ⊗ bˆ ) ⊗ (bˆ ⊗ bˆ ) : A = P S : A

Note that A (Nb ) = (bˆ ⊗ bˆ ) : A = bˆ ⋅ A ⋅ bˆ is the normal component according to b̂ -direction,


ˆ

i.e. it is parallel to b̂ . It is interesting to review Problem 1.119.

University of Castilla- La Mancha Draft By: Eduardo W. V. Chaves (2016)


Ciudad Real – Spain
522 SOLVING PROBLEMS BY MEANS OF CONTINUUM MECHANICS

b) We can apply the above definition in order to obtain:


σ = σ P + σS = PP : σ + PS : σ
ε = εP + εS = PP : ε + PS : ε
with that, and by considering that σ = C e : ε , we can obtain:
σ P = P P : σ = P P : C e : ε = P P : C e : (ε P + ε S ) = P P : C e : ε P + P P : C e : ε S
= P P : Ce : P P : ε + P P : Ce : P S : ε

σ S = P S : σ = P S : C e : ε = P S : C e : (ε P + ε S ) = P S : C e : ε P + P S : C e : ε S
= P S : Ce : P P : ε + P S : Ce : P S : ε
thus
σ P  P P : C e P P : C e  ε P 
 S= S e : 
σ   P : C P S : C e  ε S 
or
σ P   P P : C e : P P P P : C e : P S  ε 
 S= S : 
σ   P : C : P
e P
P S : C e : P S  ε 

University of Castilla-La Mancha Draft By: Eduardo W. V. Chaves (2016)


Ciudad Real – Spain
5 INTRODUCTION TO: CONSTITUTIVE EQUATIONS, IBVP, AND IBVP SOLUTION STRATEGIES 523

Appendix 5A
E
λ= ν
(1 + ν )(1 − 2ν )
E E (1 − 2ν )
µ= =
2(1 + ν ) (1 + ν )(1 − 2ν ) 2
Eν E E
λ + 2µ = +2 = (1 − ν )
(1 + ν )(1 − 2ν ) 2(1 + ν ) (1 + ν )(1 − 2ν )
Eν E E
λ+µ= + =
(1 + ν )(1 − 2ν ) 2(1 + ν ) 2(1 + ν )(1 − 2ν )
E  E Eν  E2
µ (2 µ + 3λ) = 2 +3 =
2(1 + ν )  2(1 + ν ) (1 + ν )(1 − 2ν )  2(1 + ν )(1 − 2ν )
λ+µ E 2(1 + ν )(1 − 2ν ) 1
= =
µ (2 µ + 3λ) 2(1 + ν )(1 − 2ν ) E2 E
λ Eν (1 + ν )(1 − 2ν ) ν
= =
2 µ (2 µ + 3λ) (1 + ν )(1 − 2ν ) E2 E
1 2(1 + ν ) 1
= = 2(1 + ν )
µ E E
1 (1 − 2ν )
= ,
(2 µ + 3λ) E
λ (1 − 2ν ) Eν ν
= = ,
(2 µ + 3λ) E (1 + ν )(1 − 2ν ) (1 + ν )
µ (1 − 2ν ) E (1 − 2ν )
= = ,
(2 µ + 3λ ) E 2(1 + ν ) 2(1 + ν )
2( µ + λ ) ν (1 − 2ν ) 1
=2 +2 = ,
(2 µ + 3λ) (1 + ν ) 2(1 + ν ) (1 + ν )
E Eν E (1 − ν )
( 2 µ + λ) = 2 + = ,
2(1 + ν ) (1 + ν )(1 − 2ν ) (1 + ν )(1 − 2ν )
λ Eν (1 + ν )(1 − 2ν ) ν
= =
(2 µ + λ) (1 + ν )(1 − 2ν ) E (1 − ν ) (1 − ν )
,
λ Eν 2(1 + ν )(1 − 2ν )
= = 2ν
(λ + µ ) (1 + ν )(1 − 2ν ) E
2µ + λ 1 − ν
= ,
2 µ + 3λ 1 + ν

University of Castilla- La Mancha Draft By: Eduardo W. V. Chaves (2016)


Ciudad Real – Spain
524 SOLVING PROBLEMS BY MEANS OF CONTINUUM MECHANICS

University of Castilla-La Mancha Draft By: Eduardo W. V. Chaves (2016)


Ciudad Real – Spain
6 Linear Elasticity
6.1 Three-Dimensional Elasticity (3D)
Problem 6.1
The cylinder described in Figure 6.1 is made up of an isotropic linear elastic material, and is
subjected to a strain state (in cylindrical coordinates) as follows:
err = eθθ = a sin θ
a cos θ
e rθ = (6.1)
2
e zz = eθz = erz = 0

where eij are the Almansi strain tensor components.


r
Calculate the traction vector t on the boundary by using the cylindrical coordinates.
Hypothesis: Assumptions: Small deformation regime and consider the Lamé constants
λ, µ .

x3

Π
t

Π ê z
ê θ
t
ê r

r x2

x1

Figure 6.1
526 SOLVING PROBLEMS BY MEANS OF CONTINUUM MECHANICS

Solution:
Small deformation regime: e ≈ E ≈ ε
 a cos θ 
a sin θ 0
 ε rr ε rθ ε rz   2
 a cos θ 
ε( r , θ, z ) = ε rθ ε θθ ε θz  =  a sin θ 0 (6.2)
 2 
 ε rz ε θz ε zz   0 0 0
 

σ = λTr (ε )1 + 2µ ε (6.3)
Tr (ε ) = 2a sin θ (6.4)
thus,
 a cos θ 
 a sin θ 0
 1 0 0  2
 a cos θ 
σ = λ 2a sin θ 0 1 0 + 2µ 
 a sin θ 0 (6.5)
 2 
0 0 1  0 0 0
 
λ 2a sin θ + 2µ a sin θ µ a cos θ 0 

σ (r,θr,θ =  µ a cos θ λ 2a sin θ + 2µ a sin θ 0 
 (6.6)
 0 0 λ 2a sin θ
r
The traction vector t :
r ˆ
t (n) = σ ⋅ nˆ (6.7)
nˆ = (1,0,0)

t 1(nˆ )   2λa sin θ + 2µ a sin θ


 (nˆ )   
t 2  =  µ a cos θ  (6.8)
t (nˆ )   0 
 3  

Problem 6.2
The parallelepiped described in Figure 6.2 is deformed as indicated by the dashed lines. The
displacement components are given as follows:
u = C1 xyz ; v = C 2 xyz ; w = C3 xyz (6.9)
a) Obtain the strain state at the point E . In the current reference the point is represented
by E ′ whose coordinates are E ′(1.503; 1.001; 1.997) ;
b) Obtain the normal strain at the point E in the direction of the line EA ;
c) Calculate the angular distortion at the point E that undergoes the angle formed by the
lines EA and EF .
d) Find the volume variation and the average volumetric deformation.

University of Castilla-La Mancha Draft By: Eduardo W. V. Chaves (2016)


Ciudad Real – Spain
6 LINEAR ELASTICITY 527

G = G′ F = F′

E
D = D′ 2m

E′

C = C′
O = O′
y

1 .5 m
A = A′
B = B′
1m
x

Figure 6.2
Solution:
a) The strain state in function of the displacements is given by:
1  ∂u i ∂u j 

ε ij = + (6.10)
2  ∂x j ∂x i 

which in engineering notation becomes:
 ∂u 1  ∂v ∂u  1  ∂w ∂u  
 1 1    +   + 
 εx γ xy γ xz 
 ∂x 2  ∂x ∂y  2  ∂x ∂z  
2 2
1 1   1  ∂v ∂u  ∂v 1  ∂w ∂v  
ε ij =  γ xy εy γ yz  =   +   +  (6.11)
2 2   2  ∂x ∂y  ∂y 2  ∂y ∂z  
1 1 
 2 γ xz γ yz ε z   1  ∂w ∂u  1  ∂w ∂v 

∂w 
  2  ∂x + ∂z  +  
2  ∂y ∂z 
2
   ∂z 
To calculate the strain state at any point we need a priori to calculate the displacement field.
Calculation of the constants:
By substituting the values given by the point E (1.5; 1.0; 2.0) , we can obtain:
u ( E ) = X 1( E ) − x1( E ) = 1.503 − 1.5 = C1 (1.5)(1.0)(2.0) ⇒ C1 = 0.001
0.001
v ( E ) = X 2( E ) − x2( E ) = 1.001 − 1.0 = C2 (1.5)(1.0)(2.0) ⇒ C2 = (6.12)
3
w( E ) = X 3( E ) − x3( E ) = 1.997 − 2.0 = C3 (1.5)(1.0)(2.0) ⇒ C3 = −0.001

University of Castilla-La Mancha Draft By: Eduardo W. V. Chaves (2016)


Ciudad Real - Spain
528 SOLVING PROBLEMS BY MEANS OF CONTINUUM MECHANICS

Then, the displacement field becomes:


Engineering notation Scientific notation
u = 0.001 xyz u1 = 0.001 X 1 X 2 X 3
0.001 0.001 (6.13)
v= xyz u2 = X1X 2 X 3
3 3
w = −0.001 xyz u 3 = −0.001 X 1 X 2 X 3
∂u
εx = = 0.001 yz = 0.002 = ε 11
∂x
∂v 0.001
εy = = xz = 0.001 = ε 22
∂y 3
∂w
εz = = −0.001xy = −0.0015 = ε 33
∂z
 ∂v ∂u  0.001 0.011
γ xy =  +  = yz + 0.001xz = = 2ε 12
 ∂x ∂y  3 3
 ∂w ∂u 
γ xz =  +  = −0.001 yz + 0.001xy = −0.0005 = 2ε 13
 ∂x ∂z 
 ∂w ∂v  0.001
γ yz =  +  = −0.001xz + xy = −0.0025 = 2ε 23
 ∂y ∂z  3
The strain field becomes:

 yz
1  yz


+ xz 
1
(xy − yz ) 
 2 3  2 
1  yz  xz 1  xy 
ε ij = 0.001  + xz   − xz  
2 3  3 2 3 
 
1
 ( xy − yz ) 1  xy  
 − xz  − xy
 2 2 3  

The strain state at the point E ( x = 1.5; y = 1.0; z = 2.0) is:

 1 1    0.011 


 − 0.00025
 εx γ xy γ xz   0.002
2 2  6 
1 1    0.011  
ε ij =  γ xy εy 
γ yz  =  − 0.00125
E   0.001
 (6.14)
2 2   6 
1 1   
 2 γ xz γ yz ε z  − 0.00025 − 0.00125 − 0.0015 
2   

b) The normal strain component associated with the direction M̂ is obtained as follows:
ˆ ⋅ε ⋅ M
ε Mˆ = M ˆ indicial
 → ε Mˆ = ε ij Mˆ i Mˆ j (6.15)
By expanding the above equation and by considering the symmetry of the strain tensor we
can obtain:
ε Mˆ = ε11 Mˆ 12 + ε 22 Mˆ 22 + ε 33 Mˆ 32 + 2ε12 Mˆ 1Mˆ 2 + 2ε13 Mˆ 1Mˆ 3 + 2ε 23 Mˆ 2 Mˆ 3 (6.16)
or by using the engineering notation:
ε Mˆ = ε x Mˆ 12 + ε y Mˆ 22 + ε z Mˆ 32 + γ xy Mˆ 1Mˆ 2 + γ xz Mˆ 1Mˆ 3 + γ yz Mˆ 2 Mˆ 3 (6.17)

University of Castilla-La Mancha Draft By: Eduardo W. V. Chaves (2016)


Ciudad Real – Spain
6 LINEAR ELASTICITY 529

The unit vector components M̂ i is given by the direction cosines of the direction of the
line EA :
−1 −2
Mˆ 1 = 0 ; Mˆ 2 = ; Mˆ 3 = (6.18)
5 5
By substituting the corresponding values into the equation (6.17), we can obtain:
ε Mˆ = ε y Mˆ 22 + ε z Mˆ 32 + γ yz Mˆ 2 Mˆ 3
1 4 2 (6.19)
ε Mˆ = 0.001 + (−0.0015) + (−0.0025) = −2 × 10 −3
5 5 5
c) For small deformation, the distortion of the angle at the point E formed by the lines
EA and EF , with Θ = 90º , becomes:

−1 1  − 2Mˆ ⋅ ε ⋅ Nˆ  ˆ
ε Mˆ Nˆ = ∆θ Mˆ Nˆ = −   = M ⋅ ε ⋅ Nˆ components
  → ε Mˆ Nˆ = εij Mˆ i Nˆ j (6.20)
2 2 sin Θ 

More details about the above equation are provided in the textbook in Chapter 2 –
Continuum Kinematics (in the sub-section small deformation regime). Expanding the
above expression and by considering the symmetry of the strain tensor we can obtain:
(
ε Mˆ Nˆ = ε11 Mˆ 1 Nˆ 1 + ε 22 Mˆ 2 Nˆ 2 + ε 33 Mˆ 3 Nˆ 3 + ε12 Mˆ 1 Nˆ 2 + Mˆ 2 Nˆ 1 + )
(6.21)
(
ε13 Mˆ 1 Nˆ 3 + Mˆ 3 Nˆ ) + ε (Mˆ
1 23 2 Nˆ 3 + Mˆ 3 Nˆ 2 )
or in engineering notation:
γ Mˆ Nˆ
2
= ε x Mˆ 1 Nˆ 1 + ε y Mˆ 2 Nˆ 2 + ε z Mˆ 3 Nˆ 3 +
γ xy ˆ ˆ
2
(
M 1 N 2 + Mˆ 2 Nˆ 1 + )
(6.22)
γ xz ˆ ˆ
2
(
M 1 N 3 + Mˆ 3 Nˆ 1 +
γ yz
2
) (
Mˆ 2 Nˆ 3 + Mˆ 3 Nˆ 2 )
and by considering the following unit vectors according to EA and EF direcctions
respectively:
 −1 − 2
Mˆ i = 0  ; Nˆ i = [− 1 0 0] (6.23)
 5 5
we can obtain:
γ Mˆ Nˆ  0.011   −1 −2
= ε12 Mˆ 2 Nˆ 1 + ε13 Mˆ 3 Nˆ 1 =  (−1)  + (−0.00025)(−1)  
2  6   5  5 (6.24)
γ Mˆ Nˆ
= 5.96284793998 × 10 − 4 ⇒ γ Mˆ Nˆ = 1.1925696 × 10 −3
2
Alternative Solution
We can construct an orthogonal basis associated with the unit vectors M̂ and N̂ by means
of the cross product Pˆ = Mˆ ∧ Nˆ . Then, we can obtain the components of the unit vector
P̂ :
eˆ 1 eˆ 2 eˆ 3
−1 −2 2 ˆ 1 ˆ  2 −1
Pˆ = M
ˆ ∧ Nˆ = 0 = e2 − e3 ⇒ Pˆi = 0  (6.25)
5 5 5 5  5 5
−1 0 0

University of Castilla-La Mancha Draft By: Eduardo W. V. Chaves (2016)


Ciudad Real - Spain
530 SOLVING PROBLEMS BY MEANS OF CONTINUUM MECHANICS

Then, the transformation matrix from the system X 1 X 2 X 3 to the system made up by the
unit vectors M̂ , N̂ and P̂ are given by:
 −1 − 2
 Mˆ 1 Mˆ 2 Mˆ 3   0 5 5

  
A = a ij =  Nˆ 1 Nˆ 2 Nˆ 3  =  − 1 0 0  (6.26)
 Pˆ 2 −1
 1 Pˆ
2 Pˆ3   0
 5 5 

By applying the component transformation law for a second-order tensor components, i.e.
ε ij = a ik a jl ε kl or in matrix form ε ′ = A ε A T , we can obtain:

  0.011   T
 −1 − 2   0.002   − 0.00025  −1 − 2
0   6  
0 
 5 5    0.011   5 5
ε ′ = − 1 0 0    0.001 − 0.00125 − 1 0 0  (6.27)
2 
−1  6   2 −1 
0  − 0.00025 − 0.00125 − 0.0015   0
 5 5     5 5 

 
Thus:
γ Mˆ Nˆ
ε M̂ ε Mˆ Nˆ =
2

 
 − 2 × 10 −3 5.96284794 × 10 − 4 − 2.5 × 10 − 4
 
 −3 
ε ′ij = 5.96284794 × 10 − 4 2 × 10 −3 − 1.75158658 × 10  (6.28)
 −4 
 − 2.5 × 10 − 1.75158658 × 10 −3 1.5 × 10 −3 
 

NOTE: Note that this example is not a case of homogeneous deformation, i.e. a straight
edge in the reference configuration is no longer straight line in the current configuration.
To obtain the deformed unit vector we must apply the linear transformation mˆ = F ⋅ Mˆ
and nˆ = F ⋅ Nˆ , where F is the deformation gradient.
∆( dV )
d) The volume ratio (dilatation) by definition is εV = where dV is the differential
dV
volume.
For small deformation regime we have:
∆ (dV )
εV = = εx + ε y + εz ⇒ ∆ (dV ) = (ε x + ε y + ε z )dV (6.29)
dV
by integrating the above equation over the volume we can obtain the volume variation:
2.0 1 1.5

∫ (ε )  xz 
∆V =
V
x + ε y + ε z dV = 0.001 ∫ ∫ ∫  yz +
z = 0 y =0 x = 0
3
− xy  dxdydz

(6.30)

thus:

∆V = 1.125 × 10 −3 m 3 (6.31)

University of Castilla-La Mancha Draft By: Eduardo W. V. Chaves (2016)


Ciudad Real – Spain
6 LINEAR ELASTICITY 531

Then:
∆ (dV ) 1.125 × 10 −3
εV = = = 0.375 × 10 −3 (6.32)
dV 1 .5 × 1 .0 × 2 .0

Problem 6.3
The stress state at one point of the structure, which is made up of an isotropic linear elastic
material, is given by:
 6 2 0
σ ij =  2 − 3 0 MPa
0 0 0

a) Obtain the engineering strain tensor components. Consider the Young’s modulus
( E = 207GPa ) and the shear modulus ( G = 80GPa ).
b) Consider that a cube of side 5cm is subjected to this stress state. Obtain the volume
variation in the cube.
Solution:
The strain components can be obtained by means of the equations:

εx =
1
E
[ ( )]
σ x − ν σ y + σ z = 3.333 ×10 − 5 ; γ xy =
1
G
τ xy = 2.5 × 10 − 5
1
[ ]
ε y = σ y − ν (σ x + σ z ) = −2.318 ×10 − 5 ;
E
1
γ yz = τ yz = 0
G
(6.33)
1
[ ( )]
ε z = σ z − ν σ x + σ y = −4.348 × 10 − 6 ;
E
1
γ xz = τ xz = 0
G
where the Poisson’s ratio can be obtained by:
E E 207
G= ⇒ ν= −1 = − 1 ≈ 0.29375
2(1 + ν ) 2G 160
Thus:
33.24 12.5 0 

ε ij =  12.5 − 23.01 0  × 10 −6
 0 0 − 4.257 

Alternative solution:
−1 (1 + ν ) ν
In the textbook (Chaves(2013)) we have shown that C e = I − 1 ⊗ 1 , with that we
E E
can obtain:
−1  (1 + ν ) ν  (1 + ν ) ν (1 + ν ) ν
ε = Ce : σ =  I − 1 ⊗ 1 : σ = I : σ − 1 ⊗1 : σ = σ − Tr (σ )1
 E E  E E E E
And its components are:
 σ11 σ12 σ13  1 0 0
(1 + ν )  ν
ε ij = σ12 σ 22 
σ 23  − Tr (σ ) 0 1 0
E  E
σ13 σ 23 σ33  0 0 1 

University of Castilla-La Mancha Draft By: Eduardo W. V. Chaves (2016)


Ciudad Real - Spain
532 SOLVING PROBLEMS BY MEANS OF CONTINUUM MECHANICS

6 2 0 1 0 0 33.24 12.5 0 
−6   −6   
ε ij = 6.251×10 2 − 3 0 − 4.2609 × 10 0 1 0 =  12.5 − 23.01 0  × 10− 6
0 0 0 0 0 1  0 0 − 4.257 

(1 + ν )  1  ν
where we have considered = 6.25 × 10 − 6   and Tr (σ ) = 4.25725 ×10 −6 .
E  MPa  E
In the small deformation regime the volumetric deformation (linear) is equal to the trace of
the strain tensor:
DVL ≡ ε V = I ε = (33.24 − 23.01 − 4.257 ) × 10 −6 = 5.973 × 10 −6
Then, the volume variation can be evaluated as follows:
∆V = ε V V0 = 5.973 × 10 −6 (5 × 5 × 5) = 7.466 × 10 −4 cm 3
Problem 6.4
A parallelepiped (elastic body) of dimensions a = 3cm , b = 3cm , c = 4cm , is made up of an
isotropic homogeneous linear elastic material, which is accommodated into a cavity of the
same shape and dimensions as the parallelepiped, (see Figure 6.3). The cavity walls are
made up of a very rigid material (undeformable), (Ortiz Berrocal (1985)).
Via a rigid plate (dimensions a × b ) of negligible weight and negligible friction we apply a
perpendicular compression force equal to F = 200 N which compresses the elastic block.
Consider that the elastic body properties are: E = 2 × 10 4 N / cm 2 (Young’s modulus);
ν = 0.3 (Poisson’s ratio).
a) Calculate the lateral force exerted by the cavity wall on the parallelepiped;
b) Calculate the height variation of the elastic body, i.e. find ∆c .

F
z

y
c

Figure 6.3
Solution:
At any point of the elastic body the stress state is only characterized by normal components
σ x , σ y and σ z . The stress σ z is given by:

University of Castilla-La Mancha Draft By: Eduardo W. V. Chaves (2016)


Ciudad Real – Spain
6 LINEAR ELASTICITY 533

200 200 200 N


σz = − =− =− (6.34)
ab 3× 3 9 cm 2
Note that, due to the problem symmetry the stresses σ x and σ y are equal, then:

εx = ε y =
1
E
[ (
σx − ν σ y + σz = 0 )] ⇒
1
E
[σ x − ν (σ x + σ z )] = 0
⇒ σ x − ν (σ x + σ z ) = 0 (6.35)
ν σz
⇒ σx =
(1 − ν )
thus:
ν σz 0.3  − 200  200 N
σx = =  =− (6.36)
(1 − ν ) (1 − 0.3)  9  21 cm2
The force applied by the wall on the elastic body is given by:
− 200
Fy = σ y a c = × 3 × 4 = −114.28 N
21
− 200
Fx = σ x b c = × 3 × 4 = −114.28 N
21
The strain ε z can be obtained as follows:

εz =
1
E
[ ( 1
E
)]
σ z − ν σ x + σ y = [σ z − 2ν σ x ] =
1  200
4 
2 × 10  9
− + 2 × 0.3 ×
200 
21 
= −8.25 × 10− 4

Then, the height variation is given by:


∆c = ε z c = −8.25 × 10 −4 × 4 = −0.0033cm (6.37)

Problem 6.5
Under the approximation of small deformation theory, the displacement field is given by:
r
u = ( x1 − x3 ) 2 × 10 −3 eˆ 1 + ( x 2 + x3 ) 2 × 10 −3 eˆ 2 − x1 x 2 × 10 −3 eˆ 3
Obtain the infinitesimal strain tensor, the infinitesimal spin tensor at the point P(0,2,−1) .
Solution:
Displacement gradient field components:
 ∂u1 ∂u1 ∂u1 
 
 ∂x1 ∂x2 ∂x3   2( x − x )
1 3 0 − 2( x1 − x3 )
∂ui  ∂u 2 ∂u2 ∂u2  
= = 0 2( x2 + x3 ) 2( x2 + x3 )  × 10− 3
∂x j  ∂x1 ∂x2 ∂x3 
 ∂u  
 3 ∂u3 ∂u3   − x2 − x1 0
 ∂x1 ∂x2 ∂x3 
and at the particular point P(0,2 − 1) the above equation becomes
 2( x1 − x 3 ) 0 − 2( x1 − x 3 )  2 0 − 2
∂u i 
= 0 2( x 2 + x3 ) 2( x 2 + x 3 )  =  0 2 2  × 10 −3
∂x j
P  − x 2 − x1 0   − 2 0 0 

University of Castilla-La Mancha Draft By: Eduardo W. V. Chaves (2016)


Ciudad Real - Spain
534 SOLVING PROBLEMS BY MEANS OF CONTINUUM MECHANICS

r
The second-order tensor ( ∇ xr u ) can be split additively into a symmetric ( ε ) and an
antisymmetric part ( ω ):
r ∂u
(∇ xr u)ij = i = ε ij + ωij
∂x j
where
Infinitesimal strain tensor Infinitesimal spin tensor
 2 0 − 2 0 0 0 
1  ∂u ∂u j  
 = 0 2 1  × 10 −3 1  ∂ui ∂u j   (6.38)
ε ij =  i + ωij = − = 0 0 1 × 10− 3
2  ∂x j ∂xi    2  ∂x j ∂xi  
 − 2 1 0  0 − 1 0
 

Problem 6.6
Under the restriction of small deformation theory, the displacement field is given by:
r
u = a ( x12 − 5 x 22 ) eˆ 1 + (2 a x1 x 2 )eˆ 2 − (0) eˆ 3
a) Obtain the linear strain tensor and the linear spin tensor;
b) Obtain the principal strains and principal stresses;
c) Given the shear modulus G , obtain the value of the Young’s modulus E to guarantee
the balance at any point of the continuum.
Obs.: The body forces can be discarded.
Solution:
a) Considering that u1 = a ( x12 − 5 x 22 ) , u 2 = 2 a x1 x 2 , u 3 = 0 , the displacement gradient
components are given by:
 ∂u1 ∂u1 ∂u1 
 
 ∂x1 ∂x2 ∂x3   2 x a
1 − 10ax2 0
r ∂u ∂u ∂u 2 ∂u 2  
(∇ xr u)ij = i =  2 = 2ax2 2ax1 0
∂x j  ∂x1 ∂x2 ∂x3 
 ∂u  0
 3 ∂u3 ∂u3   0 0
 ∂x1 ∂x2 ∂x3 
Decomposing additively the displacement gradient in a symmetric part (the linear strain
tensor - ε ij ) and an antisymmetric part (the infinitesimal spin tensor- ωij ) we can obtain:
∂ui
= εij + ωij
∂x j
where
  2 x1a − 10ax2 0  2 x1a 2ax2 0   2 x1a − 4ax2 0
1  ∂u ∂u j  1 
εij =  i + =  2ax2 2ax1 0 + − 10ax2 2ax1 0  =  − 4ax2 2ax1 0
2  ∂x j ∂xi  2
  0 0 0  0 0 0   0 0 0

and

University of Castilla-La Mancha Draft By: Eduardo W. V. Chaves (2016)


Ciudad Real – Spain
6 LINEAR ELASTICITY 535

  2 x1a − 10ax2 0  2 x1a 2ax2 0   0 − 6ax2 0


1  ∂u ∂u j  1  
ωij =  i − =  2ax2 2ax1 0 − − 10ax2 2ax1 0  = 6ax2 0 0
2  ∂x j ∂xi  2
  0 0 0  0 0 0   0 0 0

b) Note that since εi 3 = ε3i = 0 , ε33 = 0 is already a principal value which is associated with
the unit vector nˆ i = 0[0,01] . The remaining principal values can be obtained as follows:
2 x1 a − λ − 4ax 2
=0 ⇒ (2 x a − λ ) − (4ax )
1
2
2
2
=0
− 4ax 2 2ax1 − λ
λ1 = 2 x1 a − 4ax 2
(
⇒ 2 x1 a − λ ) = (4ax )
2
2
2
⇒ 2 x1 a − λ = ±4ax 2 ⇒ 
λ 2 = 2 x1 a + 4ax 2
 2 x1a + 4ax2 0 0
ε′ij =  0 2 x1a − 4ax2 0 (principal strains)
 0 0 0
Since the strain and stress share the same principal space we can use the equation
σij = λ 4 x1aδ ij + 2 µε ij to obtain the principal stresses:
1 0 0  2 x1a + 4ax2 0 0
 
σ′ij = λ 4 x1aδ ij + 2 µε′ij = λ 4 x1a 0 1 0 + 2 µ   0 2 x1a − 4ax2 0
0 0 1   0 0 0
λ 4 x1a + 2 µ (2 x1a + 4ax2 ) 0 0 
= 0 λ 4 x1a + 2 µ ( 2 x1a − 4ax2 ) 0 
 0 0 λ 4 x1a 
r r
c) The equilibrium equations without body forces ( ρb = 0 ) become:
r r
∇ ⋅ σ + ρ{ b = 0 Indicial
 → σ ij , j = 0 i
r
=0

and by expanding the above equation, we can obtain:


 ∂σ11 ∂σ12 ∂σ13
 + + =0
 ∂x ∂x ∂x
σ11,1 + σ12, 2 + σ13,3 = 0 1 2 3
  ∂σ21 ∂σ22 ∂σ23
σij , j = 0i ⇒ σ21,1 + σ22, 2 + σ23,3 = 0 ⇒  + + =0
  ∂ x1 ∂ x2 ∂ x 3
σ31,1 + σ32, 2 + σ33,3 = 0  ∂σ31 ∂σ32 ∂σ33
 + + =0
 ∂x1 ∂x2 ∂x3

and by considering that ε kk = 4 x1 a , the stress tensor components σ ij = λε kk δ ij + 2µ ε ij


become σ ij = λ 4 x1 aδ ij + 2µ ε ij , thus
σ11 = λ 4 x1 aδ 11 + 2µ ε11 = λ 4 x1 a + 2µ (2 x1 a ) = 4 x1 a (λ + µ )
σ12 = λ 4 x1 aδ 12 + 2µ ε12 = 2µ ( −4ax 2 ) = −8µ ax 2
σ13 = 0
W can also use:

University of Castilla-La Mancha Draft By: Eduardo W. V. Chaves (2016)


Ciudad Real - Spain
536 SOLVING PROBLEMS BY MEANS OF CONTINUUM MECHANICS

1 0 0  ε 11 ε 12 ε 13 
 
σ ij = λ 4 x1 aδ ij + 2 µε ij = λ 4 x1 a 0 1 0 + 2 µ ε 12 ε 22 ε 23 
0 0 1 ε 13 ε 23 ε 33 
1 0 0  2 x1 a − 4ax 2 0  4 x1 a (λ + µ ) − 8 µax 2 0 
  
= λ 4 x1 a 0 1 0 + 2 µ − 4ax 2  
2ax1 0 =  − 8 µax 2 4 x1 a (λ + µ ) 0 
0 0 1   0 0 0  0 0 λ 4 x1 a 
Then, the first equilibrium equation becomes:
∂σ11 ∂σ12 ∂σ13
+ + =0 ⇒ 4a (λ + µ ) − 8 µa = 0 ⇒ λ + µ = 2µ ⇒ λ = µ = G
∂x1 ∂x2 ∂x3
G (3λ + 2G )
In addition, note that E = , which was obtained by means of the relationships
λ+G
Eν E
λ= and µ = G = . Then, we can conclude that:
(1 + ν )(1 − 2ν ) 2(1 + ν )
G (3λ + 2G ) G (3G + 2G )
E= = = 2.5G
λ+G G+G

Problem 6.7
The stress state at a point of the continuum is represented by the Cauchy stress tensor
components:
 − 26 6 0 
σ ij =  6 9 0  kPa
 0 0 29
By considering an isotropic linear elastic material: a) Obtain the principal invariants of σ ;
b) Obtain the spherical and deviatoric parts of σ ; ) Obtain the eigenvalues and
eigenvectors of σ ; d) Draw the Mohr’s circle in stress. Obtain the maximum normal and
tangential stress; e) Considering a small deformation regime and taking into account that
the elastic mechanical properties are λ = 20000kPa and µ = 20000kPa ( λ, µ are the Lamé
constants), obtain the infinitesimal strain tensor; f) Obtain the eigenvalues and eigenvectors
of ε .
Solution:
a) The principal invariants of σ
I σ = 12 ×103 ( Pa)

9 0 − 26 0 − 26 6
II σ = × 106 + × 106 + × 106 = −763 × 106 ( Pa) 2
0 29 0 29 6 9

III σ = det (σ ) = −7830 × 109 ( Pa ) 3

The spherical and deviatoric parts are related to tensor by σ ij = σ ijdev + σ ijsph :
1 1 ( 29 − 26 + 9)
The mean stress: σ m = σ ii = I σ = = 4 × 10 3 Pa
3 3 3

University of Castilla-La Mancha Draft By: Eduardo W. V. Chaves (2016)


Ciudad Real – Spain
6 LINEAR ELASTICITY 537

σ m 0 0  4 0 0 
σijhyd ≡ σijsph =  0 σm 0  = 0 4 0  kPa
 0 0 σ m  0 0 4 

The deviatoric part σ ijdev = σ ij − σijsph becomes:


 − 26 − 4 6 0   − 30 6 0 
σ ijdev 
= 6 9−4 0  =  6 5 0  kPa
 0 0 29 − 4   0 0 25

By solving the characteristic equation we can obtain the eigenvalues:


σ I = 29kPa ; σ II = 10kPa ; σ III = −27 kPa :
The eigenvectors:
σ I = 29kPa principal direction
    → nˆ i(1) = [0 0 1]
σ II = 10kPa principal direction
    → nˆ i( 2 ) = [0.1644 0.98639 0]
σ III = −27kPa principal direction
    → nˆ (i 3) = [0.98639 − 0.1644 0]

σ S (kPa)
29 − (−27)
σ S max = = 28
2

σ N (kPa)

σ III = −27
σ I = σ N max = 29

σ II = 10

Figure 6.4

−λ 1
σ ij = λTr (ε )δ ij + 2 µε ij reverse
 form 
→ ε ij = Tr (σ )δ ij + σ ij
2 µ (3λ + 2 µ ) 2µ
−λ
where = −5 × 10 −9 ( Pa ) −1 , Tr (σ ) = 1.2 × 10 4 ( Pa)
2µ (3λ + 2µ )

1 0 0 − 26 6 0 
−9   4 −8 
ε ij = (−5 × 10 )(1.2 × 10 ) 0 1 0 + 2.5 × 10  6 9 0  × 10 3
0 0 1  0 0 29
1 0 0  − 26 6 0  − 7.1 1.5 0 
−5   −5   
= −6 × 10 0 1 0 + 2.5 × 10  6 9 0  =  1.5 1.65 0  × 10 − 4
0 0 1   0 0 29  0 0 6.65

University of Castilla-La Mancha Draft By: Eduardo W. V. Chaves (2016)


Ciudad Real - Spain
538 SOLVING PROBLEMS BY MEANS OF CONTINUUM MECHANICS

As the material is isotropic, the stress and strain have the same principal directions, so, we
can work in the principal space in order to obtain the principal strains:
−λ 1
ε ′ij = Tr (σ )δ ij + σ′ij
2µ (3λ + 2µ ) 2µ
1 0 0  29 0 0  66.5 0 0 
−5   −5   
= −6 × 10 0 1 0 + 2.5 × 10  0 10 0  =  0 19 0  × 10 −5
0 0 1   0 0 − 27   0 0 73.5

Problem 6.8
Show that the constitutive equations in stress, for an isotropic linear elastic material, can be
represented by the set of equations:
σ dev = 2µ ε dev

 Tr (σ ) = 3κTr (ε )
where µ = G is the shear modulus, and κ is the bulk modulus.
Solution:
σ = C e : ε = [λ1 ⊗ 1 + 2 µI] : ε = λTr (ε )1 + 2 µε

σ = σ dev + σ sph = λTr (ε )1 + 2 µ (ε dev + ε sph )


Tr (σ )
⇒ σ dev + 1 = λTr (ε )1 + 2 µ (ε dev + ε sph )
3
Tr (ε ) Tr (σ )
⇒ σ dev = λTr (ε )1 + 2 µε dev + 2 µ 1− 1
3 3
 2µ  Tr (σ )
⇒ σ dev = λ +  Tr (ε )1 + 2 µε −
dev
1
 3  3
The trace of the stress tensor:
Tr (σ ) = σ : 1 = [λTr (ε )1 + 2µ ε ] : 1 = λTr (ε )3 + 2µ Tr (ε ) = (3λ + 2µ )Tr (ε )
with that we can obtain:
 2µ  Tr (σ )
⇒ σ dev =  λ +  Tr (ε )1 + 2µ ε −
dev
1
 3  3

⇒ σ dev =  λ +
2µ  (3λ + 2µ )Tr(ε) 1
 Tr (ε )1 + 2µ ε −
dev

 3  3

⇒ σ dev =  λ +
2µ  (3λ + 2µ )Tr(ε) 1 + 2µ ε dev
 Tr (ε )1 −
14434444 424444444
3
3
=0

To the equations σ dev = 2µ ε dev we must add the constraint:


 2µ  (3λ + 2 µ )Tr(ε ) 1 = 0 ⇒  λ + 2 µ  Tr(ε )1 − Tr (σ ) 1 = 0
λ +  Tr (ε )1 −  
 3  3  3  3
 2µ 
⇒ Tr (σ )1 = 3 λ +  Tr (ε )1 ⇒ ⇒ Tr (σ )1 = 3κ Tr (ε )1
 3 
or Tr (σ ) = 3κ Tr (ε ) .

University of Castilla-La Mancha Draft By: Eduardo W. V. Chaves (2016)


Ciudad Real – Spain
6 LINEAR ELASTICITY 539

σ33 σm dev
σ 33

σ 23 σ 23
σ13 σ 23 = + σ13 σ 23

σ13 σ13
σ12 σ 22 σm σ12 σ dev
22
σ12 σ12
σ11 σm dev
σ11

σ ij = λTr (ε )δ ij + 2µ ε ij
= Tr (σ )δ ij = 3κ Tr (ε )δ ij
+ σ ijdev = 2µ ε ijdev

ε 33 εm dev
ε 33

ε 23 ε 23
ε13 ε 23 = + ε13 ε 23

ε13 ε13
ε12 ε 22 εm ε12 ε dev
22
ε12 ε12
ε11 εm dev
ε11

Figure 6.5: Constitutive equations for isotropic material.

Alternative solution:
Starting from the constitutive equation in stress for an isotropic linear elastic material
σ = σ (ε ) = λTr (ε )1 + 2µ ε , and by considering the linear regime the relationship
σ = σ (ε ) = σ (ε sph + ε dev ) = σ (ε sph ) + σ (ε dev ) holds, where:

σ (ε sph ) = λTr (ε sph )1 + 2 µε sph


 Tr (ε )  Tr (ε ) Tr (ε )  2µ 
σ sph = λTr  11 + 2 µ 1 = λTr (ε )1 + 2 µ 1 = λ +  Tr (ε )1 = κ Tr (ε )1
 3  3 3  3 
Tr (σ )
1 = κ Tr (ε )1
3
Tr (σ )1 = 3κ Tr (ε )1

σ (ε dev ) = λ Tr (ε dev )1 + 2µ ε dev = 2µ ε dev


1424 3
=0

Note that Tr[σ (ε sph )] = Tr[σ sph ] = Tr[σ ] holds.

University of Castilla-La Mancha Draft By: Eduardo W. V. Chaves (2016)


Ciudad Real - Spain
540 SOLVING PROBLEMS BY MEANS OF CONTINUUM MECHANICS

NOTE: Note that for an isotropic material if we have a purely spherical state of
compression:
p
p>0

− p 0 0 
p 
σij =  0 − p 0  ∴ Tr (σ ) = −3 p < 0
p  0 0 − p 

E
we have that Tr (σ ) = 3κ Tr (ε ) < 0 , and considering that κ = , we can conclude
3(1 − 2ν )
that: if ν > 0.5 this implies that κ < 0 and as consequence Tr (ε ) > 0 , i.e. an expansion,
which has no physical meaning for a compression state in isotropic materials. With that we
can conclude that ν < 0.5 .

Problem 6.9
A parallelepiped of dimensions a = 0.10m , b = 0.20m , c = 0.30m , (see Figure 6.6), is made
up of an elastic material whose mechanical properties are: Poisson’s ration ν = 0.3 and
Young’s modulus E = 2 × 10 6 N / m 2 . Said parallelepiped is introduced into a cavity of width
b whose walls are very rigid, so that two opposite faces of the parallelepiped are in contact
with the cavity walls. Once the parallelepiped is this position the temperature is raised by
the increment ∆T = 30º C .
a) Calculate the values of the principal stresses at any point of the parallelepiped.
b) Find the strain components.
Consider that the thermal expansion coefficient of the material is 1.25 × 10 −5 º C −1 .
Solution:
For an isotropic material the temperature variation ( ∆T ) only affects the normal strain
1+ν ν 
components  ε = σ − Tr (σ )1 + α∆T 1  , so, the solid will be only subjected by
 E E 
normal stresses. Note also that the solid can deform freely according to the directions x
and z , hence the normal stresses are σ x = σ z = 0 . The solid is restricted to move according
to the y -direction, hence ε y = 0 :

εy =
1
E
[ ] 1
σ y − ν (σ x + σ z ) + α ∆T = σ y + α ∆T = 0
E
⇒ σ y = − Eα ∆T

By means of the problem data , (Figure 6.6), we can obtain:


N
σ y = − Eα ∆T = −2 ×10 6 ×1.25 ×10 −5 (30) = −750
m2
The Cauchy stress tensor filed is constant, so, the stress components at any point of the
body are:
0 0 0
σ ij = 0 − 750 0 Pa

0 0 0

University of Castilla-La Mancha Draft By: Eduardo W. V. Chaves (2016)


Ciudad Real – Spain
6 LINEAR ELASTICITY 541

b)
−ν σy
εx = εz = + α ∆T = 1.125 × 10 − 4 + 3.75 × 10 − 4 = 4.875 × 10 − 4
E
The strain tensor components:
4.875 0 0 

ε ij =  0 0 0  × 10 − 4
 0 0 4.875

z
Data:
a
a = 0.10m
b = 0.20m
c = 0.30m
E = 2 ×10 6 N / m 2
ν = 0 .3
∆T = 30º C
α = 1.25 ×10 −5 º C −1
c
y

b
x

Figure 6.6

Problem 6.10
Consider a container whose squared cross section has dimensions 0.10m × 0.10m , consider
also that the container walls are very rigid. In the interior of said container is placed a
synthetic rubber block whose dimensions are 0.10m × 0.10m × 0.5m , (see Figure 6.7(a)). The
rubber block fits perfectly into the rigid container.
Consider that the mechanical properties of the rubber are E = 2.94 × 10 6 N / m 2 (Young’s
modulus) and ν = 0.1 (Poisson’s ratio).
Above the rubber is poured 0.004m 3 of mercury, whose mass density is 13580kg / m 3 .
a) Obtain the height H that reach the mercury, (see Figure 6.7(b));
b) Obtain the stress state at any point of the rubber block.
Hypothesis: 1) the weight of the rubber is negligible. 2) Consider the acceleration of gravity
equal to g = 10m / s 2 , and also consider that between the rubber block faces and the
container walls there are no friction.

University of Castilla-La Mancha Draft By: Eduardo W. V. Chaves (2016)


Ciudad Real - Spain
542 SOLVING PROBLEMS BY MEANS OF CONTINUUM MECHANICS

z Rigid walls
Rigid walls

Mercury

Rubber
L0 = 0.5m

y
a) b)
x

Figure 6.7
Solution:
The total force acting in the rubber, due to the weight of mercury, can be calculated as
follows:
 kg  m  kgm 
F = ma = Vmer ρ mer g = 0.004(m 3 ) × 13580 3  × 10 2  = 543.20 2 ≡ N 
m  s   s 
Then, the normal stress according to the z -direction is:
F 543.20 N
σz = − =− = −54.320 × 10 3 2
A (0.1 × 0.1) m
According to the directions x and y the rubber does not deform, hence ε x = ε y = 0 , and
by using these restrictions we can conclude that:

εx =
1
E
[
σ x − ν (σ y + σ z ) = 0 ] ⇒ σ x = ν (σ y + σ z )

1
[
ε y = σ y − ν (σ x + σ z ) = 0
E
] ⇒ σ y = ν (σ x + σ z )

σ y = ν (σ x + σ z ) = ν {[ν (σ y + σ z )] + σ z } = ν 2 σ y + ν 2 σ z + νσ z = ν 2 σ y + (ν 2 + ν )σ z
(ν 2 + ν ) ν
⇒ σy = σz = σ z = −6035.55 Pa = σ x
(1 − ν ) 2
(1 − ν )
The normal strain according to the z -direction is given by:

University of Castilla-La Mancha Draft By: Eduardo W. V. Chaves (2016)


Ciudad Real – Spain
6 LINEAR ELASTICITY 543

εz =
1
[
σ z − ν (σ x + σ y ) = ] 1
{− 54320 − 0.1[2(−6035.55)]} = −0.0180656
E 2.94 × 10 6
b) The height H reached by the mercury is given by:
H = hmer + ( L0 − ∆L)
where the length variation of the rubber block is:
∆L = L0 ε z = 0.5 × (− 0.018656) = −0.00903m
By considering the mercury incompressible, the parameter hmer can be calculated as:
0.004
Vmer = b 2 × hmer = 0.004 ⇒ hmer = = 0.4m
0.1 × 0.1
thus,
H = hmer + ( L0 − ∆L) = 0.4 + (0.5 − 0.00903) = 0.891m

Problem 6.11
By means of a material test in the laboratory, it was obtained the following relationships:
 1   − ν 21   − ν 31 
ε x =  σ x +  σ y +  σ z
 E1   E2   E3 
 − ν 12   1   − ν 32 
ε y =  σ x +  σ y +  σ z
 (6.39)
 E1   E2   E3 
 − ν 13   − ν 23   1 
ε z =  σ x +  σ y +  σ z
 E1   E2   E3 
where ν 12 = 0.2 , ν 13 = 0.3 , ν 23 = 0.25 , E1 = 1000 MPa , E 2 = 2000MPa , E3 = 1500MPa .
Knowing that the analyzed material is orthotropic, obtain the values of ν 21 , ν 31 and ν 32 .
Solution:
The elasticity matrix for orthotropic materials has the following format:

 C11 C12 C13 0 0 0


C C22 C23 0 0 0 
 12
C C23 C33 0 0 0 Orthotropic symmetry
[C ] =  13  (6.40)
0 0 0 C44 0 0 9 independent constants
0 0 0 0 C55 0
 
 0 0 0 0 0 C66 

By restructuring the relationships given by (6.39) in matrix form we can obtain:

University of Castilla-La Mancha Draft By: Eduardo W. V. Chaves (2016)


Ciudad Real - Spain
544 SOLVING PROBLEMS BY MEANS OF CONTINUUM MECHANICS

  1   − ν 21   − ν 31  
      
E E  E  0 0 0
 ε xx    1   2   3   σ xx 

 ε   − ν 12   1   − ν 32   
 yy   E      0 0 0  σ yy 
 ε zz   1   E2   E3   σ 
  =  − ν 13   − ν 23   1    zz  (6.41)
2ε xy        0 0 0  σ xy 
2ε yz   E1   E2   E3   σ 
   0 0 0 C44 0 0   yz 
 2ε xz    σ 
 0 0 0 0 C55 0   xz 
 0 0 0 0 0 C66 

Then, for orthotropic material it must fulfill that:
 − ν 21   − ν 12   − ν 31   − ν 13   − ν 32   − ν 23 
  =   ;   =   ;   =  
 E 2   E1   E3   E1   E3   E2 
with that we can obtain
ν 21 ν 12 E 2ν 12 2000 × 0.2
= ⇒ ν 21 = = = 0.4
E2 E1 E1 1000
ν 31 ν 13 E3ν 13 1500 × 0.3
= ⇒ ν 31 = = = 0.45
E3 E1 E1 1000
ν 32 ν 23 E3ν 23 1500 × 0.25
= ⇒ ν 32 = = = 0.1875
E3 E2 E2 2000

Problem 6.12
Given an isotropic linear elastic material whose mechanical properties are E = 71 GPa
(Young’s modulus), G = 26.6 GPa (shear modulus), find the strain tensor components and
the strain energy density at the point in which the stress state, in Cartesian basis, is
represented by:
 20 − 4 5 
σ ij =  − 4 0 10  MPa
 5 10 15

Solution: Poisson’s ratio can be obtained by means of the equation:


E E (1 + ν ) 1 1
µ =G= ⇒ν = − 1 ≈ 0.335 and = = (GPa ) −1
2(1 + ν ) 2G E 2G 53.2
1
ε11 = [σ11 − ν (σ 22 + σ 33 )] = 1 9 [20 − 0.335 (0 + 15)]10 6 = 211 × 10 −6
E 71 × 10
1
ε 22 = [σ 22 − ν (σ11 + σ 33 )] = 1 9 [0 − 0.335 (20 + 15 )]10 6 = −165 × 10 −6
E 71 × 10
1 1
ε 33 = [σ 33 − ν (σ11 + σ 22 )] = [15 − 0.335 (20 + 0 )]10 6 = 117 × 10 −6
E 71 × 10 9
1+ν 1 + 0.335
ε12 = σ12 = 9
( −4 × 10 6 ) = 75 × 10 − 6
E 71 × 10
1+ν 1 + 0.335
ε 13 = σ13 = (5 × 10 6 ) = 94 × 10 −6
E 71 × 10 9

University of Castilla-La Mancha Draft By: Eduardo W. V. Chaves (2016)


Ciudad Real – Spain
6 LINEAR ELASTICITY 545

1+ν 1 + 0.335
ε 23 = σ 23 = 9
(10 × 10 6 ) = 188 × 10 − 6
E 71 × 10
thus:
 211 − 75 94 
ε ij =  − 75 − 165 188  × 10 − 6
 94 188 117 

We can also use the equation


1 λ (1 + ν ) ν
ε ij = σij − Tr (σ )δ ij = σ ij − Tr (σ )δ ij
2µ 2 µ (3λ + 2 µ ) E E
(1 + ν ) 1 ν
where = ( MPa) −1 , Tr (σ ) = 35( MPa) , = 4.71831× 10 − 6 ( MPa) −1 , then
E 53200 E
 20 − 4 5  1 0 0
1 
ε ij =  − 4 0 10 MPa − (4.71831× 10 )( MPa) (35MPa) 0 1 0
 −6 −1
53200MPa
 5 10 15 0 0 1
 211 − 75 94 
=  − 75 − 165 188 × 10 − 6
 94 188 117 
Then, the strain energy density for a linear elastic material is obtained by the equation:
1 1 1
Ψ e (ε ) = ε : C e : ε = ε : σ indicial
 → Ψ e = ε ij σ ij
2 2 2
Next, by considering the symmetry of the tensors σ and ε , the strain energy density can
be calculated as follows:
1
Ψe= [ε11σ11 + ε 22 σ 22 + ε33σ 33 + 2ε12 σ12 + 2ε 23σ 23 + 2ε13σ13 ]
2
1
= [( 211)( 20) + ( −165)(0) + (117 )(15) + 2( −75)( −4) + 2(188)(10) + 2(94)(5) ] = 5637 .5 J / m 3
2
We can also obtain the strain energy density by using the equation:
1 1 1 1
Ψ e (σ ) = I σ2 − − II σ dev = I σ2 + − J 2
6(3λ + 2 µ ) 2µ 6(3λ + 2 µ ) 2µ

and if we consider that I σ = 3.5 × 10 7 ; II σ = −2.4933 × 1014 ; λ ≈ 5.3804 × 10 10 Pa ; µ = G , we


can obtain Ψ e ≈ 5638 .03 J / m 3 . Note that Ψ e (σ ) = Ψ e (ε ) since we are dealing with linear
elastic material, (see Problem 5.5). And any discrepancies in the numerical results of Ψ e
are due to numerical approximations.

University of Castilla-La Mancha Draft By: Eduardo W. V. Chaves (2016)


Ciudad Real - Spain
546 SOLVING PROBLEMS BY MEANS OF CONTINUUM MECHANICS

Problem 6.13
Find the strain energy density in terms of the principal invariants of ε .
Solution:
1 1 λTr (ε ) λ[Tr (ε )]2
Ψ e = ε : σ = ε : [λTr (ε )1 + 2 µε ] = ε2
1 1 + µ ε:ε =
:3 +µ ε:ε
2 2 2 Tr ( ε )
2

λ[Tr (ε )] 2
λ[Tr (ε )] 2
= + µ Tr (ε ⋅ ε T ) = + µ Tr (ε ⋅ ε )
2 2
λ[Tr (ε )]2
= + µ Tr (ε 2 )
2
We can add and subtract the term µ [Tr (ε )]2 without altering the above outcome:
λ[Tr (ε )]2
Ψe=
2
+ µ [Tr (ε )] + µ Tr (ε 2 ) − µ [Tr (ε )] =
2 2 1
2
{
(λ + 2 µ )[Tr (ε )]2 − µ [Tr (ε )]2 − Tr (ε 2 ) }
Finally, if we consider that the principal invariants of the strain tensor ε are I ε = Tr (ε ) ,
II ε =
2
{
1 2
}
I ε − Tr (ε 2 ) , we can obtain:

1
Ψ e = (λ + 2 µ )I ε2 − 2 µ II ε = Ψ e ( I ε , II ε )
2

Problem 6.14
The responses of a liner thermoelastic solid due to two actions are known, namely:
r r (I ) r (I ) r r ( II ) r ( II )
I (b ( I ) , t * on S σ ; u* on Sur ; ∆T ( I ) )
and II (b ( II ) , t * on S σ ; u* on Sur ; ∆T ( II ) ) .
Obtain the response of the system formed by I + II and justify, (see Oliver (2000)).
Solution:
As we are dealing with a linear regime the following is satisfied:
r r r r r ( I ) r ( II ) r r ( I ) r ( II )
b = b ( I ) + b ( II ) ; ∆T = ∆T ( I ) + ∆T ( II ) ; t* = t* + t* ; u* = u* + u*
The same is true for the fields:
r r r
u = u ( I ) + u ( II ) ; ε = ε ( I ) + ε ( II ) ; σ = σ ( I ) + σ ( II )
Starting from the governing equations of linear thermoelastic equilibrium we have:
ƒ The equilibrium equations:
r r r r r r
∇ xr ⋅ σ + ρb = ∇ xr ⋅ (σ ( I ) + σ ( II ) ) + ρ (b ( I ) + b ( II ) ) = [∇ xr ⋅ σ ( I ) + ρb ( I ) ] + [∇ xr ⋅ σ ( II ) + ρb ( II ) ] = 0

ƒ The kinematic equations:

ε = ε ( I ) + ε ( II ) =
2
[
1 r r (I ) r 1
] [
r r
∇ x u + (∇ xr u ( I ) ) T + ∇ xr u ( II ) + (∇ xr u ( II ) ) T
2
]
=
1
2
{[ r
∇ xr u ( I )
r ( II )
+ ∇ xr u ] [ r (I ) r ( II ) T
+ ∇ xr u + ∇ xr u ]}
2
{ [r r
] [
r r T 1
2
]} {
r rT
= ∇ xr u ( I ) + u ( II ) + ∇ xr (u ( I ) + u ( II ) ) = ∇ xr u + [∇ xr u] = ε
1
}

University of Castilla-La Mancha Draft By: Eduardo W. V. Chaves (2016)


Ciudad Real – Spain
6 LINEAR ELASTICITY 547

ƒ The constitutive equations in stress:


σ = C e : ε + M∆T
where M is the thermal stress tensor
σ = C e : ε + M∆T = C e : (ε ( I ) + ε ( II ) ) + M (∆T ( I ) + ∆T ( II ) )
= (C e : ε ( I ) + M∆T ( I ) ) + (C e : ε ( II ) + M∆T ( II ) )
= σ ( I ) + σ ( II )
Then, we can conclude that all the conditions are met. Then, we can apply the principle of
superposition to the linear thermoelastic problem, since we are dealing with linear regime.

Problem 6.15
Let us consider the rod of length L = 7.5m , whose cross sectional diameter is equal to
0.1m . The rod is made up of a material whose thermo-mechanical properties are:
1
E = 2.0 × 10 11 Pa (Young’s modulus) and α = 20 × 10 −6 (coefficient of thermal
ºC
expansion). Initially the rod has a temperature equal to 15º C which later rises to 50 º C .
a) Considering that the rod can expand freely, calculate the total elongation of the rod, ∆L ;
b) Now assume that the rod cannot expand freely because concrete blocks have been
placed at its ends, (see Figure 6.8(b)). Find the stress in the rod.
Hint: Consider the problem in one dimension.

x
∆L = ∆L(1) + ∆L( 2 )
∆L(1)
∆T
∆T
L
L

∆L( 2)
a) b)

Figure 6.8: Rod under thermal effect.

Solution: a) To obtain the elongation, we pre-calculate the thermal strain according to the
rod axis direction ε ij = α ∆T δ ij . Since this is a one-dimensional case, we need only
consider the normal strain component according to the x -direction, ε 11 = ε x , then:
ε 11 = ε x = 20 × 10 −6 (50 − 15) = 7 × 10 −4

Then, the total elongation, ∆L = ∆L(1) + ∆L( 2) , is obtained by solving the integral:
L


∆L = ε x dx = ε x L = 7 × 10 − 4 × 7.5 = 5.25 × 10 −3 m
0

Note that as the rod can expand freely, it is stress-free.

University of Castilla-La Mancha Draft By: Eduardo W. V. Chaves (2016)


Ciudad Real - Spain
548 SOLVING PROBLEMS BY MEANS OF CONTINUUM MECHANICS

b) If the ends cannot move, there will be a homogeneous stress field equal to:
σ x = − Eα ∆T = − E " ε x " = −2.0 ×1011 × 7 × 10 −4 = −1.4 × 108 Pa
Note that in the case 2) there is no strain, since ∆L = 0 . Moreover, it is the same as when
the initial length is equal to L + ∆L in which we apply compression stress in order to
obtain a final length equal to L .

Problem 6.16
Consider an isotropic linear elastic material with the following thermo mechanical
properties E = 10 6 Pa (Young’s modulus), ν = 0.25 (Poisson’s ratio), α = 20 × 10 −6 º C −1
(Coefficient of thermal expansion).
Consider that that at one point of the solid the stress tensor components are given by:
12 0 4
σ ij =  0 0 0 Pa
 4 0 6

a) Obtain the principal stresses and principal directions of the stress tensor; Obtain the
maximum shear stress.
b) Obtain the strain tensor components. And find the principal strains and directions.
c) Obtain the strain energy density.
d) If the solid undergoes a change in temperature ∆T = 50º C , obtain the final strain state at
this point.
e) We can say that we are dealing with a state of plane stress?
Solution:
a) We obtain the eigenvalues by solving the characteristic determinant. Note that we
already know an eigenvalue σ 2 = 0 which is associated with the direction nˆ i( 2) = [0 ± 1 0] .
Then, to obtain the remaining eigenvalues, it is sufficient to solve:
12 − σ 4
=0 ⇒ σ 2 − 18σ + 56 = 0
4 6−σ
Solving the quadratic equation we can obtain:
18 ± 324 − 224 σ1 = 14
σ (1,3) = ⇒ 
2 σ 3 = 4
14 0 0
σ'ij =  0 0 0 Pa
 0 0 4
And the eigenvectors (unit vectors) are given by:
 2 1 
σ1 = 14 eigenvecto
  r → nˆ i(1) =  0  = [0.8944 0 0.4472]
 5 5
σ2 = 0 eigenvecto
  r → nˆ (i 2 ) = [0 1 0]
 1 − 2
σ3 = 4 eigenvecto
  r → nˆ i(3) =  0  = [0.4472 0 − 0.8944]
 5 5

University of Castilla-La Mancha Draft By: Eduardo W. V. Chaves (2016)


Ciudad Real – Spain
6 LINEAR ELASTICITY 549

Making the change of nomenclature such that σ I > σ II > σ III , we have σ I = 14 , σ II = 4 ,
σ III = 0 , and the Mohr’s circle can be represented as shown in Figure 6.9.

σ S (Pa )

σ S max = 7

σ N (Pa)

σ III = 0 σ II = 4
σ I = 14

Figure 6.9

We can obtain the maximum shear stress, (see Figure 6.9), as follows:
σ I − σ III (14) − (0)
σ S max = = = 7 Pa
2 2
b) The Cauchy stress tensor components are given by:
−λ 1
σ ij = λTr (ε )δ ij + 2µ ε ij inverse
→ ε ij = Tr (σ )δ ij + σ ij
2µ (3λ + 2µ ) 2µ
−1
Remember that σ = C e : ε , and the reciprocal form ε = C e : σ .
Eν E 1
where λ= = 4 × 10 5 Pa , µ =G = = 4 × 10 5 Pa , = 1.25 × 10 − 6 ,
(1 + ν )(1 − 2ν ) 2(1 + ν ) 2µ
−λ − λTr (σ )
Tr (σ ) = 18 , = −2.5 × 10 −7 Pa , = −4.5 × 10 − 6 Pa
2µ (3λ + 2µ ) 2 µ (3λ + 2 µ )

1 0 0 12 0 4 10.5 0 5
−6   −6   
ε ij = −4.5 × 10 0 1 0 + 1.25 × 10  0 0 0  =  0 − 4.5 0 × 10 − 6
0 0 1  4 0 6   5 0 3
For an isotropic linear material the principal directions of the stress and strain match. The
−λ 1
principal strains can be obtained by means of ε ′ij = Tr (σ )δ ij + σ ′ij in the
2 µ (3λ + 2 µ ) 2µ
principal space, i.e.:
1 0 0 14 0 0 13 0 0
ε′ij = −4.5 × 10 0 1 0 + 1.25 × 10  0 0 0 =  0 − 4.5 0  ×10 − 6
−6   −6   
0 0 1  0 0 4  0 0 0.5

1 1
The strain energy density is given by Ψ e = σ : ε = σ ij ε ij . We can use the principal
2 2
space to obtain the strain energy density, i.e.:

University of Castilla-La Mancha Draft By: Eduardo W. V. Chaves (2016)


Ciudad Real - Spain
550 SOLVING PROBLEMS BY MEANS OF CONTINUUM MECHANICS

13 0 0 14 0 0
ε′ij =  0 − 4.5 0  ×10 −6 (dimensionless)
 ; σ′ij =  0 0 0 Pa
 0 0 0.5  0 0 4

c) With that we can obtain:


1 1 1
Ψ e = σij ε ij = σ′ij ε′ij = [σ11′ ε11′ + σ′33ε′33 ] = 92 × 10 − 6 J3
2 2 2 m
d) Using the principle of superposition:
ε ij = ε ij (σ ) + ε ij (∆T ) = ε ij (σ ) + α ∆T δ ij

and by substituting the variables values we can obtain:


10.5 0 5 1 0 0 1010.5 0 5 

ε ij =  0  −6 −6   
− 4.5 0 × 10 + 20 ×10 (50) 0 1 0 =  0 995.5 0  × 10 − 6
 5 0 3 0 0 1  5 0 1003
For isotropic materials, the principal directions of the infinitesimal strain tensor are the
same as the stress tensor.
e) We cannot say that we are dealing with a state of plane stress, since we do not know any
information about how stresses vary in the continuum, i.e. we do not know the stress field.
Remember that the state of plane stress is considered when the stress tensor field is
independent of one direction.
Problem 6.17
Let us consider a bar to which at one end we apply the force 6000 N , (see Figure 6.10).
Find ε x , ε y , ε z , and the length change of the bar. Consider also that the bar is made up of a
material whose mechanical properties are: Young’s modulus: E = 10 7 Pa ; Poisson’s ratio:
ν = 0 .3 .

1m
100 m
1m =

y, v
6000
σy =
1×1

x, u F = 6000 N

z, w

Figure 6.10

University of Castilla-La Mancha Draft By: Eduardo W. V. Chaves (2016)


Ciudad Real – Spain
6 LINEAR ELASTICITY 551

Solution: Using the normal strain expressions we can obtain:


ν
εx =
1
[ ( )]
σx −ν σ y + σz = − σ y = −
(0.3)(6000 )
= −0.00018
E E 10 7
σ y 6000
1
[
ε y = σ y − ν (σ x + σ z ) = =] = 0.0006
E E 10 7
ν
εz =
1
E
[ ( )]
σ z − ν σ x + σ y = − σ y = −0.00018
E
The total change in cross-sectional dimensions is u = w = −0.00018 × 1 = −1.8 × 10 −4 m , and
the total change in length is v = 0.0006 × 100 = 6.0 × 10 −2 m .

Problem 6.18
Let us consider a prism (rectangular parallelepiped) whose mechanical properties are:
E P = 27.44 × 105 N / cm 2 (Young’s modulus) and ν = 0.1 (Poisson’s ratio). The side length
of the squared cross section is a = 20cm . In both bases of the prism are placed two plates
perfectly smooth and rigid, such plates are connected together by four identical cables
whose cross section areas are AC = 1cm 2 and they have as mechanical property: Young’s
modulus ( E1 = 19.6 × 10 6 N / cm 2 ). Initially the length of the prism is equal to l = 1m , (see
Figure 6.11). Later, on two opposite sides of the prism we apply a compressive pressure
p = 7350 N / cm 2 as indicated in Figure 6.11.
a) Obtain the stress on the cables σ C ;
b) Obtain the principal stresses in the prism;
c) Obtain the volume variation of the prism.

z Plate

z
cable

∆l p
p p

Prism l = 1m

a
y
a

a
a) Reference configuration b) Current configuration
x
Figure 6.11

University of Castilla-La Mancha Draft By: Eduardo W. V. Chaves (2016)


Ciudad Real - Spain
552 SOLVING PROBLEMS BY MEANS OF CONTINUUM MECHANICS

Solution:
In Figure 6.12 we show the behavior of the prism with and without the cables.

z
Prism deformation Prism deformation
without cables F with cables
p
p p p

Prism without
pressure

Figure 6.12
Verify that the cable and the prism deform in the same way according to the z -direction,
thus:
ε Pz = ε Cz
On the cable it fulfills that:
σCz
σC = EC ε Cz ⇒ ε Cz =
EC
Since the prism has only normal length variation, we will have only normal strain, and in
turn normal stress only. The stress field in the prism is given by:
 
0 0 0 
P  
σ ij = 0 − p 0 
 − 4σ Cz AC 
0 0 
 a2 
The strain in the prism according to the direction z :
1  − 4σ Cz AC 
ε Pz =
1
EP
[ (
σz −ν σx + σy = 
E P  2
)] + ν p
a 
By applying the condition ε Pz = ε Cz we can obtain the equation:

1  − 4σ z AC  σC
C
ε Pz = ε Cz ⇒  +ν p = z
E P  a 2  E C
After some algebraic manipulations we can obtain the stress on the cable:
νEC pa 2 0.1 × 19.6 × 106 × 7350 × 20 2 N
σCz = = = 4900 2
( E P a 2 + 4 EC AC ) 5 2 6
(27.44 × 10 × 20 + 4 × 19.6 × 10 × 1) cm
The normal stress in the prism according to the z -direction becomes:

University of Castilla-La Mancha Draft By: Eduardo W. V. Chaves (2016)


Ciudad Real – Spain
6 LINEAR ELASTICITY 553

4σCz AC 4 × 4900 × 1 N
σ Pz = − 2
=− 2
= −49 2
a 20 cm
0 0 0 
 N
σ ijP = 0 − 7350 0 
cm 2
0 0 − 49
The volume variation of the prism is obtained as follows:
∆V = ε V V0
where εV = I ε is the linear volumetric deformation (small deformation regime), in which
2µ E
the relation Tr (σ ) = 3κTr (ε ) holds, where κ = λ + = is the bulk modulus (see
3 3(1 − 2ν )
Problem 6.8), then:
σx + σy + σz
εV = I ε = ε x + ε y + ε z = (1 − 2ν ) = −2.12857 × 10 −3
EP

and V0 = 4 × 10 4 cm 3 is the initial prism volume, thus:


∆V = ε V V0 = (−2.12857 × 10 −3 )(4 × 10 4 ) = −85.1428cm 3
Problem 6.19
Two rectangular parallelepipeds made up of same material and same shape a × b × c are
placed on either side of a rigid flat plate attached thereto by their sides a × c . Both
parallelepipeds, together with the plate, are introduced into a cavity such as indicated in
Figure 6.13. The walls of the cavity are flat, rigid and perfectly smooth. We apply the
pressures (force per unit surface area) p1 and p 2 on the upper faces of the prisms as
indicated in Figure 6.13. Consider the Young’s modulus E and the Poisson’s ratio ν .
a) Obtain the principal stresses in both prisms;
b) Obtain the block edge length variations.

z plate
a
p1 p2

cavity

1 2

c
y

b b
x

Figure 6.13

University of Castilla-La Mancha Draft By: Eduardo W. V. Chaves (2016)


Ciudad Real - Spain
554 SOLVING PROBLEMS BY MEANS OF CONTINUUM MECHANICS

Solution:
Prism 1: σ (x1) = 0 ; σ (y1) ; σ (z1) = − p1

Prism 2: σ (x2) = 0 ; σ (y2) ; σ (z2) = − p 2

For compatibility of stress:


σ (y1) = σ (y2) = σ y

ε (y1) + ε (y2 ) = 0 ⇒
1 (1)
E
[ (1
)] [
σ y − ν σ (x1) + σ (z1) + σ (y2 ) − ν σ (x2) + σ (z2) = 0
E
( )]
[
⇒ σ y − ν σ (z1) ] [ + σy −ν σz = 0 ⇒
( 2)
] [
σ y + ν p1 + σ y + ν p 2 = 0 ] [ ]
thus
− ν ( p1 + p2 )
σy =
2
ν ( p1 + p2 )
Prism 1: σ (x1) = 0 ; σ (y1) = − ; σ (z1) = − p1
2
ν ( p1 + p2 )
Prism 2: σ (x2 ) = 0 ; σ (y2) = − ; σ (z2 ) = − p 2
2
The strains in each prism are given by:
Prism 1:

ε (x1) =
E
[
1 (1)
(
σ x − ν σ (y1) + σ (z1) =
ν
2E
)]
[ν ( p1 + p2 ) + 2 p1 ]
ε (y1)
1
[ (
= σ (y1) − ν σ (x1) + σ (z1) =
E
ν
2E
)]
( p1 − p2 )

ε (z1)
1
[ (
= σ (z1) − ν σ (x1) + σ (y1) =
E
1 2
2E
)] [
ν ( p1 + p2 ) − 2 p1 ]
Prism 2:

ε (x2 ) =
1 ( 2)
E
[ (
σ x − ν σ (y2 ) + σ (z2) =
ν
2E
)]
[ν ( p1 + p2 ) + 2 p2 ]
ε (y2 )
1
[ (
= σ (y2 ) − ν σ (x2 ) + σ (z2) =
E
ν
2E
)]
( p 2 − p1 )

ε (z2 )
1
[ (
= σ (z2 ) − ν σ (x2 ) + σ (y2) =
E
1 2
2E
)]
ν ( p1 + p2 ) − 2 p2 [ ]
The edge variations:
Prism 1 Prism 2

aν νa
∆a (1) = ε (x1) a = [ν ( p1 + p2 ) + 2 p1 ] ∆a ( 2 ) = ε (x2 ) a = [ν ( p1 + p2 ) + 2 p2 ]
2E 2E
νb νb (6.42)
∆b (1) = ε (y1) b = ( p1 − p2 ) ∆b ( 2 ) = ε (y2 ) b = ( p2 − p1 )
2E 2E
∆c (1) = ε (z1) c =
c 2
2E
[
ν ( p1 + p2 ) − 2 p1 ] ∆c ( 2) = ε (z2 ) c =
c 2
2E
[
ν ( p1 + p2 ) − 2 p2 ]

University of Castilla-La Mancha Draft By: Eduardo W. V. Chaves (2016)


Ciudad Real – Spain
6 LINEAR ELASTICITY 555

Problem 6.20
A metallic cube with sides a = 0.20m is immersed in the sea at the depth z = 400m .
Knowing the Young’s modulus of the metal E = 21× 1010 Pa , and the Poisson’s ratio
ν = 0.3 , calculate the volume variation of the cube. Consider the acceleration of gravity
equals to g = 10m / s 2 .
Hypothesis: Although the mass density varies with temperature, salinity, and pressure
(depth), consider that the mass density of seawater equal to ρ = 1027 kg / m 3 and constant.
Because of the depth and cube dimensions we can take as a good approximation that the
whole cube is subjected to the same pressure as indicated in Figure 6.14.

h = 400m p

≈p
≈p

Figure 6.14
Solution:
F
The pressure can be obtained by p = , where A is the area and F can be obtained by
A
means of the Newton’s second law F = ma = Vρ g (weight of water column). Then:
F Vρ g Ahρ g kg m kg m
p= = = = ρ gh = 1027 3 10 2 400m = 4.108 × 10 6 2 2 = 4.108 × 10 6 Pa
A A A m s m s
The stress tensor components in the cube are given by:
− p 0 0   − 4.108 0 0 
  
σij ≈  0 − p 0  =  0 − 4.108 0  MPa
 0 0 − p   0 0 − 4.108
As we have only normal stress components and the material is isotropic, only normal
strains appear:

εz = ε y = εx =
1
[ (
σx −ν σ y + σz =
1
)] [− 4.108 − 0.3 (− 4.108 − 4.108)]× 10 6
E 21 × 1010
thus,
 − 7.82 0 0 
ε z = ε y = ε x = −7.82 × 10 −6
⇒ 
ε ij =  0 − 7.82 0  × 10 − 6
 0 0 − 7.82

University of Castilla-La Mancha Draft By: Eduardo W. V. Chaves (2016)


Ciudad Real - Spain
556 SOLVING PROBLEMS BY MEANS OF CONTINUUM MECHANICS

For small deformation regime the linear volumetric deformation is equal to the trace of the
infinitesimal strain tensor ( DVL ≡ εV = Tr (ε ) ), and the volume variation of the cube is
∆V
= DVL ≡ ε V = Tr (ε ) ⇒ ∆V = V0 Tr (ε ) = 0.2 3 × (−2.346 × 10 −5 ) = −1.8768 × 10 − 7 m 3
V0

where we have considered that Tr (ε ) = −2.346 × 10 −5 .

Problem 6.21
A solid cylinder of radius R = 0.05m and height 0.25m is made up of a material whose
mechanical properties are: E = 3 × 10 4 MPa (Young’s modulus) and ν = 0.2 (Poisson’s
ratio). Said cylinder is placed between two pistons, which can be considered infinitely rigid,
and all of this is enclosed in a hermetically sealed container as shown in Figure 6.15.
The container is filled with oil, and by suitable mechanism, the fluid pressure is raised to
the value p = 15MPa . By operating the mechanical press, we apply a total axial force of
F = 2.35619 × 10 5 N (piston force+pressure) on the bases of the cylinder.
At a generic point of the body:
a) Obtain the stress tensor components;
b) Obtain the strain tensor components;
c) Obtain the displacement field components ( u , v , w ).

0.25m R
x x
A A′

Cross section AA′


F
0.1m

Figure 6.15: Triaxial compression test.

Solution:
a) The stress tensor components
F 2.35619 × 10 5
σz = − =− = −30MPa ; σ x = σ y = − p = −15MPa
A π(0.05) 2

University of Castilla-La Mancha Draft By: Eduardo W. V. Chaves (2016)


Ciudad Real – Spain
6 LINEAR ELASTICITY 557

thus,
 − 15 0 0 

σ ij =  0 − 15 0  MPa
 0 0 − 30
b) For an isotropic linear elastic material, the normal stresses only produce normal strains,
then:
 1
[ (
ε x = E σ x − ν σ y + σ z )]

 1
[
ε y = σ y − ν (σ x + σ z )
E
]

 1
[ (
ε z = E σ z − ν σ x + σ y )]

By substituting the variable values we can obtain the following strain tensor components:
− 2 0 0
ε ij =  0 − 2 0  × 10 − 4

 0 0 − 8
c) The displacement field
As we are considering the small deformation regime, the following is fulfilled:
∂u ∂v ∂w
εx = ; εy = ; εz =
∂x ∂y ∂z
Integrating and obtaining the constants of integration we finally obtain the displacement
field:
u = −2 × 10 −4 x ; v = −2 × 10 −4 y ; w = −8 × 10 −4 z

Problem 6.22
The cube of sides 0.1m is made up of a material whose mechanical properties are
represented by the Lamé constants: λ = 8333.33MPa , µ = 12500 MPa .
A deformation is imposed to the material as shown in Figure 6.16, in which all faces
remains plane, the faces AEFB and DHGC become parallelograms and the remaining
faces continue squares:
a) Obtain the displacement field;
b) Obtain the strain field;
c) Obtain the stress field;
d) Obtain the actions performed by the testing machine on the faces ABFE and BCGF .

University of Castilla-La Mancha Draft By: Eduardo W. V. Chaves (2016)


Ciudad Real - Spain
558 SOLVING PROBLEMS BY MEANS OF CONTINUUM MECHANICS

z, w
H H′ G G′

tan(α ) ≈ α = 0.001
E
E′ F F′

α C = C′
D = D′ y, v

A = A′
B = B′
x, u

Figure 6.16: The deformed hexahedron.


Solution:
a) According to Figure 6.16 we can verify that there are only shear strain components.
Moreover we can also verify that there are no displacements according to the directions x
and z , then u = 0 , w = 0 . By means of triangle analogy we can obtain the displacement v :
v
small rotation ⇒ tan(α ) ≈ α = 0.001 = ⇒ v( z ) = 0.001z
z
The displacement field can be appreciated in Figure 6.17.

z, w

u = 0
E E′ 
v( z ) = 0.001z
w = 0

v( z )
z

y, v

Figure 6.17

University of Castilla-La Mancha Draft By: Eduardo W. V. Chaves (2016)


Ciudad Real – Spain
6 LINEAR ELASTICITY 559

b) By considering the strain tensor components:


 ∂u 1  ∂v ∂u  1  ∂w ∂u  
 1 1    +   + 
 εx γ xy γ xz 
 ∂x 2  ∂x ∂y  2  ∂x ∂z  
2 2
1 1   1  ∂v ∂u  ∂v 1  ∂w ∂v  
ε ij =  γ xy εy γ yz  =   +   + 
2 2   2  ∂x ∂y  ∂y 2  ∂y ∂z  
1 1 
 2 γ xz γ yz ε z   1  ∂w ∂u  1  ∂w ∂v 

∂w 
  2  ∂x + ∂z  +  
2  ∂y ∂z 
2
   ∂z 
we can conclude that ε x = ε y = ε z = γ xy = γ xz = 0 and the component γ yz is given by:
∂v ∂w
γ yz = + = 0.001
∂z ∂y

 1 1 
 εx 2
γ xy
2
γ xz 
1 1  0 0 0 
ε ij =  γ xy εy 
γ yz  = 0 0 0.0005
2 2 
1 γ 1 0 0.0005 0 
 2 xz γ yz ε z 
2 

c) The stress field σ = λTr (ε )1 + 2µ ε


Considering Tr (ε ) = 0 , λ = 8333.33MPa , µ = 12500MPa , we can obtain:
0 0 0  0 0 0 

σ ij = 2 × (12500) 0 0 
0.0005 MPa = 0 0 12.5 MPa

0 0.0005 0  0 12.5 0 

d) The principal strains:


−ε 0.0005 ε 2 = +0.0005
=0 ⇒ ε 2 = 0.0005 2 ⇒ ε = ±0.0005 ⇒ 
0.0005 −ε ε 3 = −0.0005
Remember that in the small deformation regime, the stress and strain share the same
principal directions, then we can work in the principal space in order to obtain the principal
stresses by using σ = λTr (ε )1 + 2µ ε , i.e.:
0 0 0  0 0 0 

σ′ij = 2 × (12500) 0 0.0005   0  MPa
0  MPa = 0 12.5
0 0 − 0.0005 0 0 − 12.5
e) To obtain the total force acting on one surface, we multiply the surface force by the area
of the corresponding face. The force per surface unit is obtained by means of the traction
vector t (n) = σ ⋅ nˆ . For the face ABFE the unit vector is given by nˆ i = [1,0,0] , thus:
ˆ

 t 1 ( ABFE )  0 0 0  1  0 
 ( ABFE )      
t 2  = 0 0 12.5 0 = 0
 t ( ABFE )  0 12.5 0  0 0
 3      

For the face BCGF , the unit vector is given by nˆ i = [0,1,0] , thus

University of Castilla-La Mancha Draft By: Eduardo W. V. Chaves (2016)


Ciudad Real - Spain
560 SOLVING PROBLEMS BY MEANS OF CONTINUUM MECHANICS

 t 1 ( BCGF )  0 0 0  0   0 
 ( BCGF )      
t 2  = 0 0 12.5 1 =  0  MPa
 t ( BCGF )  0 12.5 0  0 12.5
 3      
If we do the same procedure for the other faces we obtain the representation of the surface
forces on the faces as indicated in Figure 6.18:

z
H H′ G
G′

E E′ F F′

α C = C′
D = D′ y

A = A′ B = B′

Figure 6.18: The surface forces in the hexahedron.

Problem 6.23
Consider the prism (rectangular parallelepiped) as indicated in Figure 6.19, we apply the
forces F1 = 10 N and F2 = 2 N as indicated in Figure 6.19. The prism edge lengths are:
10
AB = 4cm , AD = cm , AA′ = 2cm . Consider the following material properties:
3
N 1
E = 2.5 × 10 6 (Young’s modulus), ν = 0.25 (Poisson’s ratio), and α = 5 × 10 −8
cm 2 ºC
(coefficient of thermal expansion).
a) Obtain the principal stresses; b) Obtain the traction vector on the plane Π . Is it on that
plane Π where the maximum shear acts? Justify your answer. c) Obtain the values of the
forces F1 and F2 to be applied to guarantee that in the solid there is no displacement
according to the directions x1 and x2 , when the prism is subjected to a temperature
variation of ∆T = 20º C .

University of Castilla-La Mancha Draft By: Eduardo W. V. Chaves (2016)


Ciudad Real – Spain
6 LINEAR ELASTICITY 561

x2 A2

A1
F2
B

Π
D
A
F1
F1
x1

60º
A′
F2

x3

Figure 6.19

a) The stress field


 F1 
A 0 0
 1  1.25 0 0
10 F2  N
A1 = 8.0 , A2 = 4 × ⇒ σ ij =  0 − 
0 = 0 − 0.15 0 2
3  A2  cm
   0 0 0
0 0 0
 
whose values are also the principal stresses, since there is no shear stresses.
b)
x2
r ˆ
t (n)

Π n̂
D
A

x1

60º
A′

x3
Figure 6.20

University of Castilla-La Mancha Draft By: Eduardo W. V. Chaves (2016)


Ciudad Real - Spain
562 SOLVING PROBLEMS BY MEANS OF CONTINUUM MECHANICS

 3 1  r ˆ
The unit vector components are: nˆ i =  ; ; 0 . Then, the traction vector t (n) is
 2 2 
given by:
 3
 
1.25 0 0  2   1.0825 
r ˆ 1  
t (n) = σ ⋅ nˆ =  0 − 0.15 0  = − 0.075
ˆ ˆ
; t i(n) = σ ij nˆ j ⇒ t (i n)
 2  
 0 0 0  0   0 
 
 
The normal stress component is:
 3
 
 2 
r ˆ 1 
σ N = t (n) ⋅ nˆ = t i(n) nˆ i σ N = [1.0825 − 0.075 0] 
ˆ
⇒ = 0.9
 2 
 0 
 
 
The tangential stress component can be obtained by means of the Pythagorean Theorem:
r ˆ 2 r ˆ 2
t (n) = σ 2N + σ 2S ⇒ σS = t (n) − σ 2N

where
 1.0825 
r ˆ r (nˆ ) r (nˆ )
= t ⋅ t = t i t i = [1.0825 − 0.075 0] − 0.075 = 1.1775
2
(nˆ ) (nˆ )
t (n)
 0 

Thus:
r ˆ 2
σS = t (n) − σ 2N = 1.1775 − 0.9 2 = 0.60621778

The Mohr’s circle in stress is drawn as described in Figure 6.21.

σ S ( N / cm2 )

σ III = −0.15 0 σ I = 1.25 σ N ( N / cm 2 )

Figure 6.21

University of Castilla-La Mancha Draft By: Eduardo W. V. Chaves (2016)


Ciudad Real – Spain
6 LINEAR ELASTICITY 563

We can verify that for any point in the solid, the maximum tangential stress is on the plane
 2 2 
defined by the unit vector nˆ i =  ; ; 0 and the maximum tangential stress is:
 2 2 
σ I − σ III
σ S max = = 0 .7 > σ S
2
c) We consider the following strain field:
1+ν ν 1+ν ν
ε= σ − Tr (σ )1 + α ∆T 1 ; ε ij = σ ij − Tr (σ )δ ij + α ∆T δ ij
E E E E
For the particular case Tr (σ ) = σ11 + σ 22 we have:
0 0 0  σ11 0 0 1 0 0 
0 0 0  = 1 + ν  0 σ  ν 
0 + α ∆T − Tr (σ ) 0 1 0

  E  22
 E 
0 0 ε 33   0 0 0 0 0 1

Then, we can construct the following set of equations:


 1+ν  ν  1+ν  ν 
ε11 = 0 = E σ11 + α ∆T − E Tr (σ ) = E σ11 + α ∆T − E (σ11 + σ 22 )
    

ε = 0 = 1 + ν σ + α ∆T − ν Tr (σ )  = 1 + ν σ + α ∆T − ν (σ + σ )
 22 E
22  E  E
22  E
11 22 
   
By solving the above set of equations we can obtain:
Eα ∆T N
σ11 = σ 22 = − = −3.33333 2
(1 − ν ) cm
Then, the forces are given by:
 F1 = σ11 A1 = −26.66666 N

 F2 = σ 22 A2 = −44.44444 N

University of Castilla-La Mancha Draft By: Eduardo W. V. Chaves (2016)


Ciudad Real - Spain
564 SOLVING PROBLEMS BY MEANS OF CONTINUUM MECHANICS

6.2 Two-Dimensional Linear Elasticity (2D)

Problem 6.24
a) Define the state of plane stress and the state of plane strain. b) Obtain the relationships
for σ (ε ) and for ε(σ ) by considering both plane states. c) Give practical examples in
which we can apply these states.
Solution:
a.1) In the case of plane stress one of the dimensions of the structural elements is very
small when compared to the other two, and the load is perpendicular to the direction of
smallest dimension. As a result of this the stress tensor field components related to this
r r
direction are equal to zero, e.g. σi 3 ( x ) = σ3i ( x ) = 0 .
a.2) In the case of plane strain, the structural element has a prismatic axis, in which the
dimension that corresponds to the direction of the prismatic axis is much larger than the
other two dimensions. Additionally, the loads applied are normal to the prismatic axis.
Under these conditions the strain tensor field components: ε13 , ε 23 and ε 33 are zero, i.e.
r r
ε i 3 ( x ) = ε 3i ( x ) = 0 .
b.1 – State of plane stress
In this case, the stress tensor field components have the format:
 σ11 σ12 0  σ x τ xy 0
r 
σ ij ( x ) = σ12 σ 22 0 = τ xy σy 0 (6.43)
 0 0 0  0 0 0
Let us start from the strain equation:
−λ 1 −λ 1
ε= Tr (σ )1 + σ ; ε ij = Tr (σ )δ ij + σ ij
2 µ (3λ + 2 µ ) 2µ 2 µ (3λ + 2 µ ) 2µ
and its trace can be obtained as follows:
−λ 1 − 3λ 1 1
ε :1 = Tr (σ )1 : 1 + σ :1 = Tr (σ ) + Tr (σ ) = Tr (σ )
2 µ (3λ + 2 µ ) 2µ 2 µ (3λ + 2 µ ) 2µ (3λ + 2 µ )
1
⇒ Tr (ε ) = Tr (σ ) ⇔ Tr (σ ) = (3λ + 2 µ ) Tr (ε )
(3λ + 2 µ )
The component ε 33 is no longer an unknown since:
−λ 1 −λ
ε 33 = Tr (σ )δ 33 + σ 33 = Tr (σ )
2 µ (3λ + 2 µ ) {
=1
2 µ = 0 2 µ (3λ + 2 µ )
{

−λ −λ −λ
⇒ ε 33 = Tr (σ ) = (3λ + 2 µ ) Tr (ε ) = Tr (ε )
2 µ (3λ + 2 µ ) 2 µ (3λ + 2 µ ) 2µ
(6.44)
−λ
⇒ ε 33 = (ε 11 + ε 22 + ε 33 )

−λ
⇒ ε 33 = (ε 11 + ε 22 )
(λ + 2 µ )
The stress components σ ij = λTr (ε )δ ij + 2 µε ij become:

University of Castilla-La Mancha Draft By: Eduardo W. V. Chaves (2016)


Ciudad Real – Spain
6 LINEAR ELASTICITY 565

1 0 0  ε11 ε 12 0 
σ ij = λ(ε11 + ε 22 + ε 33 ) 0 1 0 + 2 µ ε12 ε 22 0 
  
0 0 1  0 0 ε 33 
1 0 0  ε11 ε 12 0 
− λ2
= [λ(ε11 + ε 22 ) + (ε 11 + ε 22 )]0 1 0 + 2 µ ε 12
  ε 22 0  (6.45)
(λ + 2 µ )
0 0 1   0 0 ε 33 
1 0 0  ε11 ε 12 0 
2λµ
= (ε 11 + ε 22 ) 0 1 0 + 2 µ ε12 ε 22 0 
  
(λ + 2 µ )
0 0 1  0 0 ε 33 
In indicial notation the above equation becomes:
 λµ
σ ij = (λ + 2 µ ) Tr (ε )δ ij + 2 µε ij ; (i, j = 1,2) with Tr (ε ) = ε11 + ε 22

 (6.46)
ε = −λ 1
Tr (σ )δ ij + σ ij (i, j = 1,2,3) (the same as 3D )
 ij 2 µ (3λ + 2 µ ) 2µ
or
 νE E
σ ij = (1 − ν 2 ) Tr (ε )δ ij + (1 + ν ) ε ij ; (i, j = 1,2) with Tr (ε ) = ε11 + ε 22
 (6.47)
ε = − ν Tr (σ )δ + (1 + ν ) σ (i, j = 1,2,3) (the same as 3D )
 ij E
ij
E
ij

The equation in (6.45) can also be written as follows:


 4 µ (λ + µ ) 2λµ  
 ε11 + ε 22  2 µ ε12 0
 (λ + 2 µ ) (λ + 2 µ )  
  4 µ (λ + µ ) 2λµ  
σ ij =  2 µ ε12  ε 22 + ε11  0
  (λ + 2 µ ) (λ + 2 µ )  
 
 0 0 0
 
Taking into account the relationships between the mechanical parameters we can obtain:
4 µ (λ + µ ) E 2λµ Eν E E (1 − ν ) E (1 − ν )
= , = , 2µ = = = , thus:
(λ + 2 µ ) (1 − ν ) (λ + 2 µ ) (1 − ν )
2 2
(1 + ν ) (1 + ν )(1 − ν ) (1 − ν 2 )

(ε + ν ε ) (1 − ν )ε 0
E  11 22 12
 E (ε11 + ε 22 )
σ ij =  (1 − ν )ε12 (ε 22 + ν ε11 ) 0 ; Tr (σ ) =
(1 − ν ) 
2
 (1 − ν )
 0 0 0
Alternative solution: Voigt notation and engineering notation
Taking into account the conditions σi 3 = σ3i = 0 , and the relationship for strain ε(σ ) in
Voigt notation:

University of Castilla-La Mancha Draft By: Eduardo W. V. Chaves (2016)


Ciudad Real - Spain
566 SOLVING PROBLEMS BY MEANS OF CONTINUUM MECHANICS

   1 −ν −ν  
 εx   E 0 0 0 σx 
   E E
−ν 1 −ν  
 εy   0 0 0 σ y  0
   E E E 
   −ν −ν 1  
 εz   0 0 0 σz 
 = E E E
1   (6.48)
 γ xy   0 0 0 0 0 τ xy 
   G   0
 γ yz   0 1  τ yz 
   0 0 0 0    0
  G
 γ zx   1 
τ zx 
 0 0 0 0 0 
   G   

Then, if we remove the columns and rows associated with the zero stresses, the strain-
stress relationship ( ε(σ ) ) for the plane stress case is given by:
 1 −ν 
0
 εx   E    εx  −ν  σx 
 σ x  G= E
E  1 0
   −ν 1 2 (1+ν )   1  σ 
εy  =  E 0   σ y   →  ε y  = − ν 1 0  y (6.49)
 γ xy   
E E
   0 1  τ xy   γ xy 
   0 0 2(1 + ν )  τ xy 
 0
G 
The reciprocal of the above equation results the Hooke’s law ( σ (ε ) ) for the state of plane
stress:
 
1 ν
σx  0   εx 
  E  
σ y  = 1 − ν 2 ν 1 0  εy  ⇔ {σ } = [C ( 2 D _ 1) ]{ε } (6.50)
τ xy   1−ν   
  0 0  γ xy
 2  
Note that the normal strain ε z is not equal to zero, since ε z is not just dependant on the
normal stress σ z :
−ν − ν (ε11 + ε 22 )
εz =
1
E
[ ( 1
)]
σz −ν σx + σ y = −ν σx + σ y =
E
[ (
E
Tr (σ ) = )]
(1 − ν )
(6.51)

Then, the strain tensor components are represented as follows:


 εx 1
2
γ xy 0
r 1 
ε ij ( x ) =  2 γ xy εy 0 (6.52)
 0 0 ε z 

NOTE 1: If we want to be extremely rigorous there is no state of plane stress, in other


words, there is no real structure such as the strain field has the format presented in Eq.
(6.52). Moreover, as we will see later, the compatibility equations, (see Problem 5.11), are
not satisfied, in general, if the strain field has the format presented by the equation in
(6.52). But, when we are dealing with small deformation regime and the thickness ( t ) is
very small compared to the other dimensions ( L, h ), (see Figure 6.22), the error committed
by using the state of plane stress is small. Keep in mind that the state of plane stress is
always an approximation, and the only way in which the compatibility equations are
satisfied is by discarding completely the third dimension.

University of Castilla-La Mancha Draft By: Eduardo W. V. Chaves (2016)


Ciudad Real – Spain
6 LINEAR ELASTICITY 567

b.2 – State of Plane Strain


In this case the strain tensor field components have the format:
 ε11 ε12 0  ε x 1
2 γ xy 0
r   
ε ij ( x ) = ε12 ε 22 0 =  12 γ xy εy 0 (6.53)
 0 0 0  0 0 0
Let us start from the stress equation:
σ = λTr (ε )1 + 2 µε ; σ ij = λTr (ε )δ ij + 2 µε ij

and its trace can be obtained as follows:


σ : 1 = λTr (ε )1 : 1 + 2 µε : 1 ⇒ Tr (σ ) = 3λTr (ε ) + 2 µTr (ε ) = [3λ + 2 µ ]Tr (ε )
Tr (σ ) σ + σ22 + σ33
⇒ Tr (ε ) = = 11
3λ + 2 µ 3λ + 2 µ
The component σ 33 is no longer an unknown since:

σ ij = λTr (ε )δ ij + 2µ ε ij ⇒ σ 33 = λTr (ε )δ 33 + 2µ ε 33 ⇒ σ 33 = λTr (ε ) (6.54)

Then, the component σ 33 is defined as follows:


λ
σ 33 = λTr (ε ) = (σ11 + σ 22 + σ 33 )
3λ + 2 µ
λ λ
⇒ σ 33 − σ 33 = (σ11 + σ 22 )
3λ + 2 µ 3λ + 2 µ
 λ  λ
⇒ σ 33 1 −  = (σ11 + σ 22 )
 3λ + 2 µ  3λ + 2µ
λ
⇒ σ 33 = (σ11 + σ 22 )
2(λ + µ )
−λ 1
And the strain components ε ij = Tr (σ )δ ij + σ ij become:
2 µ (3λ + 2 µ ) 2µ

1 0 0  σ11 σ12 0 
−λ   1 
ε ij = (σ11 + σ 22 + σ 33 ) 0 1 0 +  σ12 σ 22 0 
2 µ (3λ + 2 µ ) 2µ
0 0 1  0 0 σ 33 
1 0 0  σ11 σ12 0 
−λ  λ   1 
=  σ11 + σ 22 + (σ11 + σ 22 ) 0 1 0 + σ12 σ 22 0  (6.55)
2 µ (3λ + 2 µ )  2(λ + µ )  0 0 1  2 µ  0
   0 σ 33 
1 0 0  σ11 σ12 0 
−λ   1 
= (σ11 + σ 22 ) 0 1 0 +  σ12 σ 22 0 
4 µ (λ + µ ) 2µ
0 0 1  0 0 σ 33 
In indicial notation the above equation becomes:
 −λ 1
ε ij = 4 µ (λ + µ ) Tr (σ )δ ij + 2 µ σ ij ; (i, j = 1,2) with Tr (σ ) = σ11 + σ 22
 (6.56)
σ = λTr (ε )δ + 2 µε (i, j = 1,2,3) (the same as 3D )
 ij ij ij

University of Castilla-La Mancha Draft By: Eduardo W. V. Chaves (2016)


Ciudad Real - Spain
568 SOLVING PROBLEMS BY MEANS OF CONTINUUM MECHANICS

or
 − ν (1 + ν ) (1 + ν )
ε ij = Tr (σ )δ ij + σ ij ; (i, j = 1,2) with Tr (σ ) = σ11 + σ 22
E E
 (6.57)
σ ij = Eν E
Tr (ε )δ ij + ε ij (i, j = 1,2,3) (the same as 3D)
 (1 + ν )(1 − 2ν ) (1 + ν )
The equation in (6.55) can also be written as follows:
 (2λ + µ ) −λ  1 
 σ11 + σ 22  σ12 0
 4 µ (λ + µ ) 4 µ (λ + µ )  2µ 
 1  (2λ + µ ) −λ  
ε ij =  σ12  σ 22 + σ11  0
 2µ  4 µ (λ + µ ) 4 µ (λ + µ )  
 
 0 0 0
 
Taking into account the relationships between the mechanical parameters we can obtain:
(λ + 2 µ ) (1 + ν )(1 − ν ) −λ − ν (1 + ν ) 1 (1 + ν )
= , = , = , thus:
4 µ (λ + µ ) E 4 µ (λ + µ ) E 2µ E

(1 − ν )σ − ν σ σ12 0
(1 + ν )  
11 22
ε ij =  σ12 (1 − ν )σ 22 − ν σ11 0
E  
 0 0 0
Alternative solution: Voigt notation and engineering notation
If we start from the generalized Hooke’s law and by deleting the columns and rows
associated with the zero strains, i.e.:

 σx  1 − ν ν ν 0 0 0   εx 
σ   ν 1−ν ν 0 0 0   ε y  0
 y    
 σz   ν ν 1 −ν 0 0 0   εz 
  E  1 − 2ν  
 τ xy =  0 0 0 0 0   γ xy  0 (6.58)
(1 + ν )(1 − 2ν )  2 
  1 − 2ν 
 τ yz   0 0 0 0 0   γ yz 
   2   0
 τ zx   1 − 2ν   γ 
   0 0 0 0 0
2  
zx


we can obtain:
  
1 − ν ν
σx 
  E
0   εx 
 σ y  = (1 + ν )(1 − 2ν )  ν 1−ν 0  εy  ⇔ {σ } = [C ( 2 D _ 2 ) ]{ε } (6.59)
τ xy   1 − 2ν   
   0 0  γ xy
 2  
Then, the stress according to the direction z is given by:

σz =
(1 + ν )(1 − 2ν )
(
εx + ε y ) (6.60)

Additionally, the reciprocal of (6.59) is:

University of Castilla-La Mancha Draft By: Eduardo W. V. Chaves (2016)


Ciudad Real – Spain
6 LINEAR ELASTICITY 569

 εx  1 − ν −ν 0  σ x 
  1+ν  −ν  
εy  = E  1−ν 0   σ y  (6.61)
 γ xy   0 0 2 τ xy 
 
c.1 – We can apply the plane stress approximation for the deep beam problems, (see Figure
6.22).

2D – Plane stress
q

h
y

x
L

Figure 6.22: Deep beam.

c.2 – We can apply the plane strain approximation for cylinder under pressure, (Figure
6.23), Tunnels, (see Figure 6.24), dams, (see Figure 6.25).

y
2D – Plane strain p - pressure

x
p
x

Cross section

per unit length


z

prismatic axis

Figure 6.23: Cylinder under pressure.

University of Castilla-La Mancha Draft By: Eduardo W. V. Chaves (2016)


Ciudad Real - Spain
570 SOLVING PROBLEMS BY MEANS OF CONTINUUM MECHANICS

2D – Plane strain

Figure 6.24: Tunnel.

2D – Plane strain
1

Cross section of the dam

Figure 6.25: Dam.

Note that to adopt the state of plane strain, the cross section properties, e.g. dimensions,
mechanical properties, cannot vary along the prismatic axis, otherwise we will have an error
associated with it.

University of Castilla-La Mancha Draft By: Eduardo W. V. Chaves (2016)


Ciudad Real – Spain
6 LINEAR ELASTICITY 571

Problem 6.25
Consider the stress-strain relationship:
 
1 ν
σx  0   εx 
  E  
σ y  = 1 − ν 2 ν 1 0  εy  (6.62)
τ xy   1−ν   
  0 0  γ xy
 2  
Find the values for E and ν in order to achieve the stress-strain relationships for the
states of plane stress and plane strain.
Solution:
If we compare the equations (6.62) and (6.50) we can conclude that for state of plane stress
we have E = E and ν = ν . Now let us consider the strain equations for both states:
Strain for state of plane stress, (see equation (6.47)):
−ν (1 + ν )
ε ij = Tr (σ )δ ij + σij (i, j = 1,2,3) (the same as 3D )
E E
(6.63)
−ν (1 + ν )
= Tr (σ )δ ij + σij
E E
Strain for state of plane strain, (see equation (6.57)):
− ν (1 + ν ) (1 + ν )
ε ij = Tr (σ )δ ij + σij ; (i, j = 1,2) with Tr (σ ) = σ11 + σ 22 (6.64)
E E
Then, by means of the equations (6.63) and (6.64) we can obtain the following equations:
− ν − ν (1 + ν ) νE (1 + ν ) 
= ⇒ ν= 
E E E  ν (1 + ν ) E (1 + ν ) ν (1 + ν )
 ⇒ ν =E =
(1 + ν ) (1 + ν ) E (1 + ν )  E (1 + ν ) E
= ⇒ E=
E E (1 + ν ) 

E (1 + ν ) ν (1 + ν ) ν
thus, ν = = (1 + ν )ν ⇒ ν=
(1 + ν ) E (1 − ν )
and
 ν 
E 1 + 
E (1 + ν )  (1 − ν )  E E
E= = = =
(1 + ν ) (1 + ν ) (1 − ν )(1 + ν ) (1 − ν 2 )

Then,
 E = E
for state of plane stress 
   ν = ν
1 ν
σx  0   εx 
  E     E
σ y  = 1 − ν 2 ν 1 0   εy  ∴   E = (1 − ν 2 ) (6.65)
τ xy   1−ν    for state of plane strain 
  0 0  γ xy 
 2   
ν = ν

  (1 − ν )

University of Castilla-La Mancha Draft By: Eduardo W. V. Chaves (2016)


Ciudad Real - Spain
572 SOLVING PROBLEMS BY MEANS OF CONTINUUM MECHANICS

Problem 6.26
Figure 6.26 (a) shows a support device for a machine. Said support apparatus is made up of
a neoprene block of dimensions ( 50 × 20cm ) which is characterized by the element ABCD
described in Figure 6.26(b).

a) b)
1 .1
1 .2
D
D’ C
C’ 1 20
A A’ B B’
1 1 .1 x
50

Dimensions in centimeters - cm

Figure 6.26
Under the action of vertical and horizontal loads the neoprene deforms as shown in Figure
6.26 (b) (A’B’C’D’) in which the displacement field ( u, v) is represented as follows:
u = a1 x + b1 y + c1
v = a2 x + b2 y + c2
where a1 , b1 , c1 , a 2 , b2 , c 2 are constants to be determined.
a) Calculate the strain tensor components and the volumetric deformation at any point;
b) Calculate the stresses at any point;
c) The maximum normal stress;
d) Obtain the unit extension according to the direction of the diagonal AC .
Hypothesis:
1 – Isotropic linear elastic material with Young’s modulus equals to 1000 N / cm 2 and the
1
shear modulus equals to N / cm 2 .
0.0028
2 – It is assumed a state of plane strain.
Solution:
 u = a1 x + b1 y + c1
 (6.66)
v = a 2 x + b2 y + c 2
According to Figure 6.26 we can obtain:
u (0;0) = 1 = c1
u (50;0) = 1.1 = 50a1 + 1 ⇒ a1 = 0.002 (6.67)
u (0;20) = 1.1 = 20b1 + 1 ⇒ b1 = 0.005
thus
u = 0.002 x + 0.005 y + 1 (6.68)

University of Castilla-La Mancha Draft By: Eduardo W. V. Chaves (2016)


Ciudad Real – Spain
6 LINEAR ELASTICITY 573

For the vertical displacement:


v(0;0) = 0 = c 2
u (50;0) = 0 = 50a 2 ⇒ a 2 = 0 (6.69)
u (0;20) = −1 = 20b2 ⇒ b2 = −0.05
v = −0.05 y (6.70)
Then:
u = 0.002 x + 0.005 y + 1
 (6.71)
v = −0.05 y
a) Strains
∂u ∂v ∂u ∂v
εx = = 0.002 ; εy = = −0.05 ; γ xy = + = 0.005 (6.72)
∂x ∂y ∂y ∂x
The linear volumetric deformation (small deformation regime):
DVL = ε V = ε x + ε y + ε z = I ε = −0.048 (6.73)
b) Stress components
E E
G= ⇒ν = − 1 = 0 .4
2(1 + ν ) 2G

σx =
E
(1 + ν )(1 − 2ν )
[
(1 − ν )ε x + ν ε y ]
= 3571.4286 × [(0.6) × 0.002 − 0.4 × 0.05] = −67.1428 ( N / cm 2 )

σy =
E
(1 + ν )(1 − 2ν )
[
(1 − ν )ε y + ν ε x ] (6.74)

= 3571.4286 × [(0.6) × (−0.05) + 0.4 × 0.002] = −104.2857 ( N / cm 2 )


1
τ xy = Gγ xy = × 0.005 = 1.785714 ( N / cm 2 )
0.0028
ν E Tr (ε ) E
As an alternative solution we can use the equation σ ij = δ ij + ε ij ,
(1 + ν )(1 − 2ν ) (1 + ν )
where:
 εx 1
2
γ xy 1
2
γ xz   0.002 1
2
(0.005) 0
   
ε ij =  12 γ xy εy 1
2
γ yz  =  12 (0.005) − 0.05 0
 1 γ xz 1
γ yz ε z   0 0 0
2 2

ν E Tr (ε) N E N
= −68.571429 2 , = 714.285714 2
(1 + ν )(1 − 2ν ) cm (1 + ν ) cm

1 0 0  0.002 1
2
(0.005) 0
   
σ ij = −68.5714290 1 0 + 714.285714 12 (0.005) − 0.05 0
0 0 1  0 0 0

− 67.1428 1.785714 0 
 N
≈  1.785714 − 104.2857 0 
cm 2
 0 0 − 68.571

University of Castilla-La Mancha Draft By: Eduardo W. V. Chaves (2016)


Ciudad Real - Spain
574 SOLVING PROBLEMS BY MEANS OF CONTINUUM MECHANICS

c) The principal stresses


2
σx + σ y  σx − σy 
σ (1,2) = ±   + τ 2xy

(6.75)
2  2 
2
− 67.1428 − 104.2857  − 67.1428 + 104.2857  2
σ (1,2) = ±   + 5.35714
2  2  (6.76)
= −171.4285 ± 19.328675
σ1 = −152.099824 N cm 2
 2 (6.77)
σ 2 = −190.757175 N cm
d) The unit extension
The diagonal ( AC ) in the reference configuration measures:

L0 = AC = 50 2 + 20 2 = 53.852cm (6.78)
and the deformed diagonal

A′C ′ = 50.2 2 + 19 2 = 53.675cm ⇒ ∆L = A′C ′ − AC = −0.177cm (6.79)


The unit extension is:
∆L − 0.177
ε= = = −0.0033 (6.80)
L0 53.852

Problem 6.27
Consider a soil made up of a linear elastic material. At a point in the soil the volumetric
deformation is εV = −2 × 10 −3 , the shear deformation is ε12 = − 3 × 10 −3 and the normal
strain is ε11 = 0 . The soil is subjected to a state of plane strain according to the plane
x1 − x 2 .
a) Obtain the Cartesian components of the infinitesimal strain tensor. Obtain the principal
strains, and the directions where they occur.
1
b) Assuming that the mechanical properties are E = 50MPa (Young’s modulus) and ν =
4
(Poisson’s ratio), obtain the stress tensor components and the principal stresses. Obtain the
maximum normal and shear stresses.
c) Obtain the strain energy density, i.e. the energy per unit volume.
Solution:
a) By means of the problem data, the infinitesimal strain tensor components are given by:
 0 − 3 × 10 −3 0
 
ε ij =  − 3 × 10 −3 ε 22 0
 0 0 0
 
where we have considered the plane strain hypothesis ε 3i = ε i 3 = 0 . By means of the
volumetric deformation: DVL ≈ ε V = I ε = ε11 + ε 22 + ε 33 = −2 × 10 −3 ⇒ ε 22 = −2 × 10 −3 . Then,
the strain components are:

University of Castilla-La Mancha Draft By: Eduardo W. V. Chaves (2016)


Ciudad Real – Spain
6 LINEAR ELASTICITY 575

 0 − 3 0
   0 − 3
ε ij =  − 3 − 2 0 × 10 −3 
plane strain
 → ε ij =   × 10
−3

 0 0 0 − 3 −2 

The principal strains
0−λ − 3 λ1 = 1
=0 ⇒ λ2 + 2 λ − 3 = 0 ⇒ 
− 3 −2−λ λ 2 = −3
thus
ε1 = 1 × 10 −3 1 × 10 −3 0 
 ⇒ ε′ij =  −3 
ε 2 = −3 × 10 −3  0 − 3 × 10 

b)

y
ε1 x′
y′ ε xy
ε yy

ε2 θ

Figure 6.27
The Mohr’s circle in strain is drawn in Figure 6.28.
γ
εS = (×10 −3 )
2

(ε N = 0; ε S = 3 )

ε III = −3 εI =1 ε N × 10 −3

(ε N = 0; ε S = −2)

(ε N = −2; ε S = − 3 )

Figure 6.28

University of Castilla-La Mancha Draft By: Eduardo W. V. Chaves (2016)


Ciudad Real - Spain
576 SOLVING PROBLEMS BY MEANS OF CONTINUUM MECHANICS

Note that the radius is R = (1 − (−3)) / 2 = 2 . Then:


3
tan(2θ) = ⇒ 2θ = arctan( 3 ) ⇒ θ = 30º
1

b) Applying σ ij = λTr (ε )δ ij + 2µ ε ij , where λ = = 20MPa ,
(1 + ν )(1 − 2ν )
E
µ= = 20MPa , Tr (ε ) = −2 × 10 −3 . Then:
2(1 + ν )

1 0 0  0 − 3 0
   
σ ij = λTr (ε ) 0 1 0 + 2µ − 3 − 2 0 × 10 3
0 0 1  0 0 0
 
 − 40 0 0   0 − 3 0 
  

=  0 − 40 
0  + 40 − 3 − 2 0  × 10 −3
MPa
  14 2 43
  0 0 − 40  0 0 
0  =10 3 Pa
  
Thus:
 − 40 − 40 3 0 
 
σ ij =  − 40 3 − 120 0  kPa
 0 0 − 40
 
As the material is isotropic, the stress and strain share the same principal space. In addition,
the eigenvalues of σ and ε can be related to each other as follows.
By substituting the value of σ = λTr (ε )1 + 2µ ε into the definition of the eigenvalue-
eigenvector, we can obtain:
σ ⋅ nˆ = γ σ nˆ
(λTr(ε)1 + 2 µε ) ⋅ nˆ = γ σ nˆ ⇒ λTr (ε )1 ⋅ nˆ + 2 µε ⋅ nˆ = γ σ nˆ
⇒ λTr (ε )nˆ + 2 µε ⋅ nˆ = γ σ nˆ ⇒ 2 µε ⋅ nˆ = γ σ nˆ − λTr (ε )nˆ
 γ − λTr (ε ) 
⇒ 2 µε ⋅ nˆ = (γ σ − λTr (ε ) )nˆ ⇒ ε ⋅ nˆ =  σ nˆ
 2µ 
⇒ ε ⋅ nˆ = γ ε nˆ
Then:
γ σ − λTr (ε )
γε = ⇒ γ σ = 2µ γ ε + λTr (ε )

And the eigenvalues of σ can be obtained as follows:
γ σ(1) ≡ σ I = 2µ γ ε(1) + λTr (ε ) = (40 × 10 6 ) × (1 × 10 −3 ) + (20 × 10 6 ) × (−2 × 10 −3 ) = 0
γ σ( 2 ) ≡ σ II = 2µ γ ε( 2 ) + λTr (ε ) = (40 × 10 6 ) × (0) + (20 × 10 6 ) × (−2 × 10 −3 ) = −40 × 10 3 Pa
γ σ(3) ≡ σ III = 2µ γ ε(3) + λTr (ε ) = (40 × 10 6 ) × (−3 × 10 −3 ) + (20 × 10 6 ) × (−2 × 10 −3 ) = −160 × 10 3 Pa

We can also use the equation σ = λTr (ε )1 + 2µ ε in the principal space:

University of Castilla-La Mancha Draft By: Eduardo W. V. Chaves (2016)


Ciudad Real – Spain
6 LINEAR ELASTICITY 577

 − 40 0 0  1 0 0  0 0 0 
     
σ′ij =   0 − 40 0  + 40 0 − 3 0  × 10
142
−3
3 = 0 − 160
MPa
4 0  kPa
 0 0 − 40 0 0 0  =103 Pa 0 0 − 40

The Mohr’s circle in stress is described in Figure 6.29.

σ S (kPa )
0 − (−160)
σ S max = = 80
σ S max 2

− 160 − 40 0 σ N (kPa)

Figure 6.29
1
c) The strain energy density is Ψ e = σ : ε . We can use the principal space in order to
2
obtain the strain energy density, i.e.:
1 1
Ψ e = σ ij ε ij = (σ1ε1 + σ 2 ε 2 + σ 3ε 3 )
2 2
1
[ ]
m
m
N m
m m m
J
= 0 + (−160 ×10 3 )(−3 ×10 −3 ) + 0 = 240 Pa = 240 2 = 240 3
2
where
σ1 0 0  0 0 0  ε 1 0 0  1 0 0
0  = 0 − 160 m
 σ2 0   0  × 10 3 Pa ; 0
 ε2 0  = 0 − 3 0 × 10 −3 m
 
 0 0 σ 3  0 0 − 40  0 0 ε 3  0 0 0

Problem 6.28
A solid, which can be approximated by the state of plane strain, has one point in which the
infinitesimal strain tensor components are given by:
− 2 3 0
ε ij =  3 − 10 0 × 10 −3

 0 0 0
Consider that the material has an isotropic linear elastic behavior defined by the Young’s
modulus E = 10MPa and Poisson’s ratio ν = 0.25 .
a) Obtain the volumetric deformation and the deviatoric part of the strain tensor;
b) Obtain the principal strains and the principal directions;
c) Obtain the Cauchy stress tensor components;
d) Obtain the maximum and minimum normal stress;
e) It is known that the material fails when the tangential stress exceeds the value 40 kPa .
Check whether the material fails or not.

University of Castilla-La Mancha Draft By: Eduardo W. V. Chaves (2016)


Ciudad Real - Spain
578 SOLVING PROBLEMS BY MEANS OF CONTINUUM MECHANICS

Solution:
a) Volumetric deformation ( εV ):
εV = I ε = Tr (ε ) = ( −2 − 10) × 10 −3 = −12 × 10 −3
Additive decomposition of the strain tensor into a spherical and deviatoric parts
ε = ε sph + ε dev , where the spherical part is given by:
− 4 0 0 
Tr (ε )
ε ijsph = δ ij =  0 − 4 0  × 10 −3

3
 0 0 − 4
and the deviatoric part is:
 − 2 3 0 − 4 0 0  2 3 0 
 
ε ijdev = ε ij − ε ijsph =   3 − 10 0 −  0 − 4 0   × 10 = 3 − 6 0  × 10 −3
   −3

 0 0 0  0 0 − 4  0 0 4



b) The principal strains are obtained by means of the characteristic determinant:
−2−λ 3
=0 ⇒ λ2 + 12λ + 11 = 0
3 − 10 − λ
By solving the above quadratic equation we can obtain:
− (12) ± (12) 2 − 4(1)(11) − 12 ± 10 λ (1) = −1.0
λ (1, 2 ) = = ⇒ 
2(1) 2 λ ( 2 ) = −11

Then, the principal strains are:


ε1 = −1.0 × 10 −3 ; ε 2 = −11.0 × 10 −3

The principal directions can be obtained by solving (ε ij − λ δ ij )n (jλ ) = 0 i (i, j = 1,2)

The principal direction associated with the eigenvalue λ (1) = −1.0 :

 n1(1)  0 − n1 + 3n 2 = 0 ⇒ n1 = 3n 2


(1) (1) (1) (1)
− 2 − (−1) 3
  = ⇒
 3 − 10 − (−1) n (21)  0 3n1(1) − 9n (21) = 0

2 2 2 1
restriction n1(1) + n (21) = 1 , with that we can obtain (3n (21) ) 2 + n (21) = 1 ⇒ n (21) = , and
10
3
n1(1) =
10
The principal direction associated with the eigenvalue λ (1) = −11.0 :

 n1( 2)  0 9n1 + 3n 2 = 0


( 2) ( 2)
− 2 − (−11) 3
   = ⇒ 
 3 − 10 − (−11) n (22)  0 3n1( 2 ) + n (22 ) = 0 ⇒ n (22) = −3n1( 2 )

2 2 1 −3
with the restriction n1( 2) + n (22) = 1 , we can obtain n1( 2) = , and n (22) =
10 10
We summarize the eigenvalues and eigenvectors as follows:

University of Castilla-La Mancha Draft By: Eduardo W. V. Chaves (2016)


Ciudad Real – Spain
6 LINEAR ELASTICITY 579

 3 1 
ε1 = −1 × 10 −3 principal
 direction
→ nˆ i(1) =  0
 10 10 
 1 −3 
ε 2 = −11 × 10 −3 principal
 direction
→ nˆ (i 2 ) =  0
 10 10 
ε1 = 0 principal direction
   → nˆ i(3) = [0 0 1]
c) The Cauchy stress tensor components are given by:
σ ij = λTr (ε )δ ij + 2µ ε ij

Eν E
where λ = = 4 MPa , µ = G = = 4 MPa , Tr (ε ) = −12 × 10 −3 :
(1 + ν )(1 − 2ν ) 2(1 + ν )

 1 0 0 − 2 3 0   − 64 24 0 
      
−3
σ ij =  4 × (−12) 0 1 0 + 2 × (4)  3 − 10 0  × 10 MPa =  24 − 128 0  kPa
 0 0 1  0 0 0   0 0 − 48

As the material is isotropic the principal directions for the stress and strain are the same.
The principal stresses can be obtained by working in the principal space
σ′ij = λTr (ε )δ ij + 2µ ε ′ij :

 1 0 0 − 1 0 0   − 56 0 0 
      
−3
σ′ij =  4 × (−12) 0 1 0 + 2 × (4)  0 − 11 0  × 10 MPa =  0 − 136 0  kPa
 0 0 1  0 0 0   0 0 − 48

d) By considering that σ I = −48kPa , σ II = −56kPa , σ III = −136kPa , the Mohr’s circle in
stress is described in Figure 6.30.

σ S (kPa)

σ S max = 44

σ II = −56

σ III = −136 σ N (kPa)


σ I = −48

Figure 6.30: Mohr’s circle in stress


The maximum shear stress, (see Figure 6.30), can be obtained as follows:
σ I − σ III (−48) − (−136)
σ S max = = = 44kPa
2 2
Then, the material fails.

University of Castilla-La Mancha Draft By: Eduardo W. V. Chaves (2016)


Ciudad Real - Spain
580 SOLVING PROBLEMS BY MEANS OF CONTINUUM MECHANICS

Problem 6.29
A strain gauge (or strain gage) is a device used to calculate the strain according to one
direction. Consider a strain rosette that contains three strain gauges arranged as indicated in
Figure 6.31. At one point we have calculated the following strain values:
ε x = 0.33 × 10 −3 ; ε ′x = 0.22 × 10 −3 ; ε y = −0.05 × 10 −3

Consider an isotropic linear elastic material with the following mechanical properties:
E = 29000 Pa (Young’s modulus); ν = 0.3 (Poisson’s ratio).
a) Find the maximum shear stress at the point in question.
b) Obtain the eigenvalues (principal strains) and eigenvectors (principal directions) of the
strain tensor;
c) Obtain the eigenvalues (principal stresses) and eigenvectors (principal directions) of the
stress tensor.
Hypothesis: Consider the state of plane strain.

strain gauge x′

45º
45º
x

Figure 6.31: Strain rosette.


Solution:
We have to obtain the strain tensor components in the system x, y, z and to do so we will
use the coordinate transformation law in order to obtain the component γ xy = 2ε12 .
Remember that in two-dimensional cases the normal component in a new system, (see
Problem 1.99 in Chapter 1), is given by:
ε 11 + ε 22 ε 11 − ε 22
′ =
ε 11 + cos( 2θ) + ε 12 sin( 2θ)
2 2
The above equation was obtained by means of the transformation law, (see Chapter 1 of
the textbook), which in engineering notation becomes:
εx + εy εx − εy γ xy
ε ′x = + cos( 2θ) + sin( 2θ)
2 2 2
Then, γ xy can be obtained as follows:

2  (ε + ε y ) (ε x − ε y ) 
γ xy =  ε ′x − x − cos( 2θ)  = 0.16 × 10 − 3

sin( 2θ)  2 2 
thus

University of Castilla-La Mancha Draft By: Eduardo W. V. Chaves (2016)


Ciudad Real – Spain
6 LINEAR ELASTICITY 581

0.33 0.08 0 
ε ij = 0.08 − 0.05 0  × 10 −3
 0 0 0 

Then, the stress components can be evaluated as follows:

σx =
E
[
(1 + ν )(1 − 2ν )
]
(1 − 2ν )ε x + ν ε y = 12 .0462 Pa

σy =
E
(1 + ν )(1 − 2ν )
[ ]
(1 − 2ν )ε y + ν ε x = 3.5692 Pa


τ xy =
E
2(1 + ν )
γ xy = 1.7846 Pa ; σ z =
(1 + ν )(1 − 2ν )
[ ]
ε x + ε y = 4.684 Pa

Additionally, the maximum shear stress is given by:


2
 σx + σ y 
σ S max =   + τ 2xy = 4.5988 Pa
 2 
a) The characteristic equation for the strain tensor (2D) is:
ε 2 − 0.28 ε − 2.29 × 10 −2 = 0 (× 10 −3 )
Then, by solving the above equation we can find the eigenvalues (principal strains) given
by:
ε 1 = 0.346155 × 10 −3 ; ε 2 = −0.06615528 × 10 −3
Then, the eigenvectors of the infinitesimal strain tensor are:
associated with ε1
Eigenvecto
  r   →  0.9802 0.1979 0 
→ − 0.1979 0.9802 0 
Eigenvecto r associated with ε 2
       
associated with ε 3
Eigenvecto
  r   →  0 0 1 

b) By considering the stress tensor components:


12.0462 1.7846 0 

σ ij =  1.7846 3.5692 0  Pa
 0 0 4.684 

we can obtain the characteristic determinant and in turn the eigenvalues (principal stresses)
σ 1 = 12.40654 , σ 2 = 3.208843 . Additionally, the eigenvectors of the stress tensor are:
associated with σ1
Eigenvecto
  r   →  0.9802 0.1979 0 
→  − 0.1979 0.9802 0 
Eigenvecto r associated with σ 2
       
associated with σ3
Eigenvecto
  r   →  0 0 1 

As expected, the eigenvectors of stress and strain are the same; since we are working with
isotropic linear elastic material.
b) Alternative solution for the stress tensor components:
Knowing the strain tensor components:
0.33 0.08 0 
ε ij = 0.08 − 0.05 0  × 10 −3
 0 0 0 

University of Castilla-La Mancha Draft By: Eduardo W. V. Chaves (2016)


Ciudad Real - Spain
582 SOLVING PROBLEMS BY MEANS OF CONTINUUM MECHANICS

We can apply the constitutive equation: σ ij = λTr (ε )δ ij + 2µ ε ij , where the Lamé constants
are given by:
Eν E
λ= = 16.7307692 × 103 Pa , µ = = 11.15384615 × 103 Pa
(1 + ν )(1 − 2ν ) 2(1 + ν )

and Tr (ε ) = 0.27999972 × 10 −3 ≈ 0.28 × 10 −3 , with that σ ij = λTr (ε )δ ij + 2µ ε ij becomes:

1 0 0  ε11 ε12 ε13  1 0 0 0.33 0.08 0


 
σij = λTr (ε ) 0 1 0 + 2 µ ε12 ε 22  
ε 23  = λTr (ε ) 0 1 0 + 2 µ 0.08 − 0.05 0 × 10 − 3

0 0 1 ε13 ε 23 ε 33  0 0 1  0 0 0


 12.0461 1.784615 0 

= 1.784615 3.5692 0  Pa
 0 0 4.6846
As the material is isotropic, the tensors σ and ε share the same principal directions, then
we can use the same equation σ′ij = λTr (ε )δ ij + 2µ ε ′ij in the principal space, i.e.:

1 0 0 ε1 0 0 
 
σ′ij = λTr (ε ) 0 1 0 + 2 µ  0 ε 2 0 
0 0 1  0 0 ε3 
1 0 0 0.346155 0 0 12.40752 0 0 
 
= λTr (ε ) 0 1 0 + 2 µ   0  −3 
− 0.0662 0 ×10 =  0 3.20783 0  Pa
0 0 1  0 0 0  0 0 4.6846

Problem 6.30
A strain gauge (or strain gage) is a device used to obtain the strain in only one direction.
Consider a strain rosette that contains three strain gauges arranged according to a equilateral
triangle, (see Figure 6.32), and records the strain values according to the directions x1 , x1′
and x1′′ .

x2
x1′′ x1′

30 º
60º Strain gauge
30 º

60º 60º

x1

Figure 6.32

The strain calculated according to the directions x1 , x1′ and x1′′ are respectively:

University of Castilla-La Mancha Draft By: Eduardo W. V. Chaves (2016)


Ciudad Real – Spain
6 LINEAR ELASTICITY 583

ε 11 = −4 × 10 −4 ; ′ = 1 × 10 −4
ε 11 ; ′′ = 4 × 10 −4
ε 11
Obtain ε 22 = ε y , 2ε12 = γ xy , ε ′22 ≡ ε ′y . Show that ε11 + ε 22 = ε11
′ + ε ′22 .

Hypothesis: Consider a state of plane strain.


Solution:
Using the component transformation law for the second-order tensor and considering the
plane state, we can obtain that:
 ε + ε 22 ε11 − ε 22
′ = 11
ε11 + cos( 2θ1 ) + ε12 sin( 2θ1 ) (6.81)
2 2

ε′′ = ε11 + ε 22 + ε11 − ε 22 cos( 2θ ) + ε sin( 2θ )
 11 2 2
2 12 2 (6.82)

−1 3
where θ1 = 60º and θ 2 = 120º , thus cos 2θ1 = cos 2θ 2 = and sin 2θ1 = − sin 2θ 2 = .
2 2
Then, by combining the two above equations it is possible to eliminate ε 12 , i.e.:
ε11 − ε 22 2 ε 
′ + ε11
ε11 ′′ = ε11 + ε 22 − ⇒ ε 22 =  ε11 ′′ − 11  = 4.66667 × 10 − 4
′ + ε11
2 3 2 

Once the value ε 22 = 4.66667 × 10 −4 is obtained, we can replace it into the equation (6.81)
with which we can obtain:
1
γ xy = 2ε 12 = (4ε11′ − ε11 − 3ε 22 ) = −3.46410 × 10 − 4 ⇒ ε 12 = −1.73205 × 10 − 4
3
To obtain ε ′22 , we must obtain the angle formed by x1 and x′2 , which is
θ 3 = 60º +90º = 150º , thus:
ε11 + ε 22 ε 11 − ε 22
ε ′22 = + cos( 2θ 3 ) + ε 12 sin( 2θ 3 ) = −0.33333 × 10 − 4
2 2
Checking that:
′ + ε′22 = 0.66667 × 10 −4 = Tr (ε )
ε11 + ε 22 = ε11
As expected, since the trace is an invariant.

Problem 6.31
Consider the dam cross section,
(Figure 6.33), in which the y, v
displacement is known and given
by:
u ( x, y ) = −4 x 2 − y 2 + 2 xy + 2 x, u

v ( x, y ) = −4 y 2 − x 2 + 2 xy + 5

Figure 6.33

This structure is made up of a material with the following mechanical properties:


E = 100 MPa (Young’s modulus), G = 35 .7 MPa (shear modulus), ν = 0.4 (Poisson’ratio).
Assuming that the structure is under a small deformation regime: a) Find the stress field; b)

University of Castilla-La Mancha Draft By: Eduardo W. V. Chaves (2016)


Ciudad Real - Spain
584 SOLVING PROBLEMS BY MEANS OF CONTINUUM MECHANICS

For the given displacement field, show whether the equilibrium equations are satisfied or
not.
Solution:
a) We can calculate the strain tensor components as follows:
∂u ∂v ∂u ∂v
εx = = −8 x + 2 y ; εy = = −8 y + 2 x γ xy = + =0
∂x ∂y ∂y ∂x
which in matrix form is:
− 8 x + 2 y 0 0
ε ij =  0 − 8 y + 2 x 0 
 0 0 0 

b) For the dam, as we have seen, we can adopt the approximation of state of plane strain,
so,
  
σx 
  E 1 − ν ν 0   εx   0 .6 0 . 4 0   − 8 x + 2 y 
 σ y  = (1 + ν )(1 − 2ν )  ν 1−ν 0   ε y  = 357 .1428 0.4 0.6 0   − 8 y + 2 x  MPa
1−ν     
 τ xy   0 0   γ xy   0 0 0.3  0 
   2 
σ x  − 4 x − 2 y 
 
⇒  σ y  = 357.1428  − 2 x − 4 y  MPa
 τ xy   0 
 

σz = (ε x + ε y ) = 357.1428 × [( −8 x + 2 y ) + ( −8 y + 2 x )]
(1 + ν )(1 − 2ν )
Then, the equilibrium equations become:
 ∂σ x ∂τ xy ∂τ xz
 + + + ρb x = 0 − 4 + 0 + 0 + 0 ≠ 0 (it fails)
 ∂x ∂y ∂z
 ✘
 ∂τ xy ∂σ y ∂τ yz 
+ + + ρb y = 0 ⇒ 0 − 4 + 0 + 0 ≠ 0

 ∂x ∂y ∂z 
(it fails)

∂σ
 ∂τ ∂τ yz ∂σ z 0 + 0 + z + 0 = 0
 xz + + + ρb z = 0  ∂z
 ∂x ∂y ∂z

So, the given displacement field does not satisfy the equilibrium equations.

Problem 6.32
A gravity dam of triangular cross section is made up of concrete with “specific weight”
5
equal to γ , where γ is the specific weight of water. The shape and dimensions of the
2
cross section are indicated in Figure 6.34, and the stress field in the dam (state of plane
strain) is given by:
γ
σ11 = −γx2 ; σ22 = ( x1 − 3x2 ) ; σ12 = −γ x1
2
1
Consider: Poisson’s ratio: ν = ; Young’s modulus E .
4

University of Castilla-La Mancha Draft By: Eduardo W. V. Chaves (2016)


Ciudad Real – Spain
6 LINEAR ELASTICITY 585

a) Obtain the graphical representation of the surface force (traction vector) acting on the
face AB due to the ground reaction;
b) Obtain the principal stresses at the points A and B . Starting from the Mohr’s circle in
stress, obtain the extreme values of the stresses at the respective points.
c) Obtain the strain field in the dam.

NOTE: Although in the literature γ is known as the specific weight, also known as the
unit weight, in reality γ is the module of the body force per unit volume, i.e.
r r r r N m
γ = p = ρb = ρg , where b is the body force per unit mass [b] = = . Recall that in
kg s 2
the International System of Units (SI) the term “specific” is related to “per unit mass”, which is
not the case of γ , the correct term would be the weight density, since the term “density” is
related to “per unit volume”.

O
x1
45º γ = ρg
g -acceleration of gravity
h ρ - mass density
kg m N
5 [γ ] = = 3
γ γ 3 2
m s m
2

A B
x2

Figure 6.34
Solution:
a) The stress and strain fields in the dam cross section are respectively:
 − γ x2 − γ x1 0   ε11 ε12 0
 γ 
σij =  − γ x1 ( x1 − 3 x2 ) 0  ; εij = ε12 ε 22 0
 2 
 0 0 σ33   0 0 0

We can obtain the surface force by means of the traction vector t (n) = σ ⋅ nˆ . For the side
ˆ

AB whose normal unit vector is nˆ i = [0,1,0] , we can obtain:

 t1( AB )   − γx2 − γx1 0  0  − γx1 


 ( AB )   γ    γ 
t 2  =  − γx1 ( x1 − 3 x2 ) 0  1 =  ( x1 − 3 x2 )
t   2  2 
σ 33  0 
( AB )
 3   0 0 0 
The surface force on the base can be appreciated in Figure 6.35.
b) Note that σ 33 is already a principal stress. Starting from σ = λTr (ε )1 + 2µ ε we can
obtain σ 33 , i.e.:

University of Castilla-La Mancha Draft By: Eduardo W. V. Chaves (2016)


Ciudad Real - Spain
586 SOLVING PROBLEMS BY MEANS OF CONTINUUM MECHANICS

σ ij = λTr (ε )δ ij + 2µ ε ij ⇒ σ 33 = λTr (ε )δ 33 + 2µ ε 33 ⇒ σ 33 = λTr (ε )

The term Tr (ε ) can be obtained by means of the double scalar product between
σ = λTr (ε )1 + 2µ ε and the second-order unit tensor, thus:
σ : 1 = λTr (ε )1 : 1 + 2 µε : 1 ⇒ Tr (σ ) = 3λTr (ε ) + 2 µTr (ε ) = [3λ + 2 µ ]Tr (ε )
Tr (σ ) σ + σ 22 + σ 33
⇒ Tr (ε ) = = 11
3λ + 2 µ 3λ + 2 µ
Then, the component σ 33 is defined as follows:
λ λ λ
σ 33 = λTr (ε ) = (σ11 + σ 22 + σ 33 ) ⇒ σ 33 − σ 33 = (σ11 + σ 22 )
3λ + 2 µ 3λ + 2 µ 3λ + 2 µ
 λ  λ λ
⇒ σ 33 1 −  = (σ11 + σ 22 ) ⇒ σ 33 = (σ11 + σ 22 ) = ν (σ11 + σ 22 )
 3λ + 2 µ  3λ + 2 µ 2(λ + µ )

h h
A B A B
− 3hγ
− hγ 2 − hγ

( AB ) ( AB )
t1 (according to x1 -direction) t2 (according to x 2 -direction)

O x1

45º

5
γ
2
A B
(nˆ )
t
x2

Figure 6.35

By substituting the values of σ11 and σ 22 , we can obtain:


λ
σ 33 = (σ11 + σ 22 ) = ν − γx2 + γ ( x1 − 3x2 ) = γ ν [x1 − 5 x2 ] = γ [x1 − 5 x2 ]
2(λ + µ )  2  2 8

University of Castilla-La Mancha Draft By: Eduardo W. V. Chaves (2016)


Ciudad Real – Spain
6 LINEAR ELASTICITY 587

λ
where we have considered ν = .
2(λ + µ )
The stress state at the point A( x1 = 0; x 2 = h) is given by:
− γ x2 − γ x1 0  − γ h 0 0  − 1 0 0 
    
 γ   − 3hγ   −3 
σij( A) =  − γ x1 ( x1 − 3 x2 ) 0 = 0 0 =0 0  hγ
 2   2   2 
 0 γ − 5hγ   − 5
0 ( x1 − 5 x2 )  0 0 0 0
 8   8   8 
Note that this space is already the principal space. Mohr’s circle in stress at the point A is
drawn in Figure 6.36.

σ S (×hγ )

σ S max = 0.4375

− 1.5 −1 − 0.625 σ N (×hγ )


Figure 6.36

The stress state at the point B( x1 = h; x 2 = h) is given by:


   
− γ x2 − γ x1 0  − γ h −γ h 0  − 1 − 1 0 
 γ   γ   
σij( B ) =  − γ x1 ( x1 − 3 x2 ) 0  = − γ h (h − 3h) 0  = − 1 − 1 0 γh
 2   2   − 1
 0 γ   γ  0 0 
0 ( x1 − 5 x2 ) 0 0 (h − 5h)  2
 8   8 
The principal stresses at the point B( x1 = h; x 2 = h) are given by:
−1− σ −1
=0 ⇒ (−1 − σ) 2 − 1 = 0 ⇒ (−1 − σ) 2 = 1 ⇒ ( −1 − σ) = ±1
−1 −1− σ
σ1 = −2
⇒
σ 2 = 0
The Mohr’s circle in stress for the point B is drawn in Figure 6.37.

University of Castilla-La Mancha Draft By: Eduardo W. V. Chaves (2016)


Ciudad Real - Spain
588 SOLVING PROBLEMS BY MEANS OF CONTINUUM MECHANICS

σ S (×hγ )

σ S max = 1

−2 − 0.5 0 σ N (×hγ )

Figure 6.37

c) We can obtain the expression for the strain field by starting from the equation:
σ = λTr (ε )1 + 2 µε
1 λ
σ = λTr (ε )1 + 2 µε ⇒ 2 µε = σ − λTr (ε )1 ⇒ ε= σ− Tr (ε )1
2µ 2µ
Tr (σ )
Remember that we have obtained that Tr (ε ) = , then the above equation becomes:
3λ + 2µ
1 λ 1 λ
ε= σ− Tr (ε )1 = σ− Tr (σ )1
2µ 2µ 2µ 2µ (3λ + 2µ )
We can also express the above equation in terms of E and ν :
E 1 (1 + ν ) µ (3λ + 2 µ ) 1 1
µ =G = ⇒ = , E= ⇒ =
2(1 + ν ) 2µ E λ+µ µ (3λ + 2 µ ) E (λ + µ )
λ λ 1 ν
= =
2 µ (3λ + 2 µ ) 2 E (λ + µ ) E
Then:
1 λ (1 + ν ) ν
ε= σ− Tr (σ )1 or ε= σ − Tr (σ )1
2µ 2 µ (3λ + 2 µ ) E E
The trace of σ is given by:
γ  γ  5
Tr (σ ) = σ11 + σ 22 + σ 33 = (−γx2 ) +  ( x1 − 3x2 ) +  ( x1 − 5 x2 ) = γ ( x1 − 5 x2 )
2  8  8
1
With that we can obtain the strain tensor components, when ν = , as follows:
4
5 5
ε ij = σ ij − γ ( x1 − 5 x2 )δ ij
4E 32 E

University of Castilla-La Mancha Draft By: Eduardo W. V. Chaves (2016)


Ciudad Real – Spain
6 LINEAR ELASTICITY 589

 
 − γx2 − γx1 0  1 0 0
5  γ  5
ε ij =  − γx1 ( x1 − 3 x2 ) 0 − γ ( x1 − 5 x2 ) 0 1 0
4E  2  32 E
γ 0 0 1 
 0 0 [x1 − 5 x2 ]
 8 
 1 
 − 8 ( x1 + 3x2 ) − x1 0
5γ  1 
=  − x1 − (−3 x1 + 7 x2 ) 0
4E  8 
 0 0 0
 

University of Castilla-La Mancha Draft By: Eduardo W. V. Chaves (2016)


Ciudad Real - Spain
590 SOLVING PROBLEMS BY MEANS OF CONTINUUM MECHANICS

Problem 6.33
Consider the infinitesimal strain tensor field (2D):
 − κ x3 x 2 ε 12 
ε ij =  ( i, j = 1,2 ) (6.83)
 ε12 νκ x3 x 2 
where κ x3 = κ x3 ( x1 ) , i.e. κ x3 is a function of x1 and ν is a constant (Poisson’s ratio).
Consider the state of plane stress, (see Problem 6.24)), with no body force.
a) Obtain ε12 in order to achieve the equilibrium and obtain the stress field.
a
As boundary condition, consider that ε12 ( x 2 = ± ) = 0 .
2
b) Express the infinitesimal strain tensor and the stress tensor in terms of ( P , E , I x3 ),
where P is the concentrated force at x1 = L , E is the Young’s modulus, I x3 is the
moment of inertia of the cross-sectional area about the x3 -axis, which for a rectangular
b a
x3 = x2 =
2 2
ba 3
section is I x3 = ∫ x22 dA = ∫ ∫ x22 dx2 dx3 = . For boundary condition, consider that at
A −b −a
12
x3 = x2 =
2 2
PL

x1 : P = σ12 dA and that at x1 = 0 ⇒ κ x3 =
A
EI x3
, where EI is called modulus of flexural

rigidity.

σy
τ xy
σx
x2 , y
x2 , y

x3 , z
a
x1 , x M x3

L b

Cross section

Figure 6.38: Fixed-free beam (boundary conditions).

University of Castilla-La Mancha Draft By: Eduardo W. V. Chaves (2016)


Ciudad Real – Spain
6 LINEAR ELASTICITY 591

Solution:
Before applying the equilibrium equations we will need to obtain the stress field. In
Problem 6.24 the stress field for the state of plane stress was obtained and by considering
the strain components (6.83) we can obtain:
 E 
− Eκ x3 x 2 ε 12 
E (ε 11 + ν ε 22 ) (1 − ν )ε 12   (1 + ν )
σ ij =  =  (6.84)
(1 − ν 2 )  (1 − ν )ε12 (ε 22 + ν ε11 )  E ε 0 
 (1 + ν ) 12 

Considering the equilibrium equations without body forces, σ ij , j = 0 i (i, j = 1,2) , we can
obtain:
 ∂σ11 ∂σ12
 ∂x + ∂x = 0
 1 2
σ ij , j = 0 i ⇒ σ i1,1 + σ i 2, 2 = 0 i ⇒  (6.85)
∂σ ∂σ
 21 + 22 = 0
 ∂x1 ∂x 2
By substituting the stress components given by equation (6.84) we can obtain:
 ∂σ11 ∂σ12  − ∂( Eκ x3 x 2 ) E ∂(ε12 )
 ∂x + ∂x = 0  + =0
 1 2  ∂x1 (1 + ν ) ∂x 2
 ⇒  (6.86)
∂σ ∂σ
 21 + 22 = 0  E ∂ (ε 12 ) = 0 ⇒ ∂ (ε12 ) = 0
 ∂x1 ∂x 2  (1 + ν ) ∂x1 ∂x1

Note that ε12 does not depend on x1 . From the first equilibrium equation we can obtain:
− ∂ ( Eκ x3 x 2 ) E ∂ (ε 12 ) ∂ε 12 ∂κ x3
+ =0 ⇒ = (1 + ν ) x 2 ≡ (1 + ν ) x 2 κ x3 ,1
∂x1 (1 + ν ) ∂x 2 ∂x 2 ∂x1
By integrating in x 2 the above equation we can obtain:
∂ε 12 x 22
= (1 + ν ) x 2 κ x3 ,1 integrating
 → ε 12 = (1 + ν ) κ x3 ,1 +C (6.87)
∂x 2 2
The constant of integration can be obtained by means of the boundary condition
a
ε12 ( x 2 = ± ) = 0 :
2
a 2
1 x2 = ± 1 a
ε12 =(1 + ν ) κ x3 ,1 x 22 + C 
2→ ε12 = (1 + ν ) κ x3 ,1   + C = 0
2 2 2
− (1 + ν ) κ x3 ,1 a 2
⇒C =
2 4
Then, the strain ε12 , (see equation (6.87)), becomes:
x 22 x 22 (1 + ν ) κ x3 ,1 a 2 (1 + ν ) κ x3 ,1  2 a2 
ε12 = (1 + ν ) κ x3 ,1 + C = (1 + ν ) κ x3 ,1 − =  x2 −  (6.88)
2 2 2 4 2  4 
 
With that the infinitesimal strain field becomes:

University of Castilla-La Mancha Draft By: Eduardo W. V. Chaves (2016)


Ciudad Real - Spain
592 SOLVING PROBLEMS BY MEANS OF CONTINUUM MECHANICS

 (1 + ν ) κ x3 ,1  2 a 2  
 − κ x3 x 2  x2 − 
2  4 
ε ij =   
 (1 + ν ) κ x ,1  a2  
 3
 x 22 −  νκ x3 x 2 
2  4 
  
And the stress field:

 E   − Eκ x3 x2
Eκ x3 ,1  2 a 2 
 x2 −  
 − Eκ x3 x2 ε12  
(1 + ν )  2  4 
σij =  =
 E ε   Eκ x3 ,1  2 a  
2
0  x − 0 
 (1 + ν )   2 
12
4 
2


b) By applying the condition P = ∫ σ12 dA we can obtain:


A

Eκ x3 ,1  2 a 2  Eκ x3 ,1  2 a2 

P = σ12 dA =
A

A
2 
 x2 −  dA =
4  2 ∫ x2 −

A
4
dA


Eκ x3 ,1  2 2  Eκ x3 ,1  a 2  Eκ x3 ,1  a2 
 x 2 dA − a dA  =
=
2  A ∫ 4 A  ∫ Ix −
2  3
A =
4  2
Ix −
 3
 4
ba 

(6.89)

Eκ x3 ,1  3ba 3  Eκ x3 ,1
=
2  3
Ix −
12 
=
2
( )
I x3 − 3I x3 = − Eκ x3 ,1 I x3

with that we can obtain that:


−P
P = − Eκ x3 ,1 I x3 ⇒ κ x3 ,1 = (6.90)
EI x3

By integrating we can obtain:


∂κ x3 −P −P P
κ x3 ,1 ≡ = integratin
 g → κ x3 = x1 + C = ( L − x1 ) (6.91)
∂x1 EI x3 EI x3 EI x3

PL PL
where we have applied the boundary condition ( x1 = 0 ⇒ κ x3 = ) to obtain C = .
EI x3 EI x3

With that the stress components can also be expressed as follows:


Px 2 P  a2 2
σ11 = − Eκ x3 x 2 = ( x1 − L) σ 22 = 0 σ12 =  
I x3
; ;
2 I x3  4 − x2  (6.92)
 
or by using Engineering notation:
Py P  a2 2
σ x = − Eκ z y = ( x − L) σy = 0 τ xy =  
Iz
; ;
2I z  4 −y 
 
Note that, the equation for σ12 , given by (6.92), is independent of x1 and σ12 has
parabolic distribution on the cross section area.

University of Castilla-La Mancha Draft By: Eduardo W. V. Chaves (2016)


Ciudad Real – Spain
6 LINEAR ELASTICITY 593

b x2
 a
 x2 = ± 2 ⇒ σ12 = 0
σ12 ( x2 ) ⇒ 
σ12 x = 0 ⇒ σ = σ 3P
 2 12 12 max =
2A

a
x1

P = σ12 dA
A

Figure 6.39: Tangential stress distribution on the cross-section area.

The maximum tangential stress is acting at x 2 = 0 :

P  a2 2 x =0 Pa 2 3P 3P
σ12 =  
 4 − x2  → σ12 max = = =
2

2 I x3   8 I x3 2ab 2 A

where A is cross section area.


NOTE 1: The displacement field can be obtained by starting from the strain definition:
∂u1 P − Px 2  x2 
ε11 = =− ( L − x1 ) x 2 integratin
 g in x1
→ u1 ( x 2 , x1 ) =  Lx1 − 1  + f 1 ( x 2 ) + C1
∂x1 EI x3 EI x3  2 

∂u 2 P P x 22
ε 22 = =ν ( L − x1 ) x 2     → u 2 ( x 2 , x1 ) = ν
integrating in x2
( L − x1 ) + f 2 ( x1 ) + C 2
∂x 2 EI x3 EI x3 2

Applying the boundary condition u 2 ( x 2 = 0, x1 = 0) = u1 ( x 2 , x1 ) = 0 , we can conclude that


C1 = C 2 = 0 , thus

 − Px 2  x2 
u1 ( x 2 , x1 ) =  Lx1 − 1  + f 1 ( x 2 )
 EI x3  2 

 (6.93)
u ( x , x ) = Pν x 22 ( L − x1 ) + f 2 ( x1 )
 2 2 1 2 EI x3

The tangential strain component can be obtained as follows:
 x 2  ∂f ( x )
 Lx1 − 1  + 1 2 − Pν x 22 + 2 1
∂u1 ∂u 2 −P ∂f ( x )
2ε 12 = + =  (6.94)
∂x 2 ∂x1 EI x3  2  ∂x 2 2 EI x3 ∂x1

Note that we have obtained previously that ε12 is independent of x1 , so, the following
must hold:
−P  x 2  ∂f ( x ) ∂f 2 ( x1 ) P  x2 
 Lx1 − 1  + 2 1 = 0 ⇒ =  Lx1 − 1 
EI x3  2  ∂x1 ∂x1 EI x3  2 
 
P  Lx12 x13 
integratin
 g in x1
→ f 2 ( x1 ) =  
EI x3  2 − 6 
 

University of Castilla-La Mancha Draft By: Eduardo W. V. Chaves (2016)


Ciudad Real - Spain
594 SOLVING PROBLEMS BY MEANS OF CONTINUUM MECHANICS

Then, the equation in (6.94) becomes


∂u1 ∂u 2 ∂f 1 ( x 2 ) Pν 2  a 2  − P (1 + ν )  2 a 2 
2ε 12 = + = − x 2 = (1 + ν ) κ x3 ,1  x 22 − =  x2 − 
∂x 2 ∂x1 ∂x 2 2 EI x3  4  EI x3 
 4 
∂f 1 ( x 2 ) Pν 2 − P (1 + ν )  2 a 2 
⇒ − x2 =  x2 − 
∂x 2 2 EI x3 EI x3  4 

∂f 1 ( x 2 ) Pν 2 P(1 + ν )  2 a 2 
⇒ = x2 −  x2 − 
∂x 2 2 EI x3 EI x3  4 

By integrating the above equation in x 2 we can obtain

Pν x 23 P (1 + ν )  x 23 a 2 
f1 ( x2 ) = − − x2 
2 EI x3 3 EI x3  3 4 
Then, the displacement field (6.93) becomes:
  x2  P (1 + ν )  x 23 a 2 
 Lx1 − 1  + Pν x 23 −
− Px 2
u1 ( x 2 , x1 ) =  
EI x3  2  6 EI EI  3 − 4 x2 
   x3 x3  
 (6.95)
 Pν P  Lx12 x13 
x 22 ( L  
u 2 ( x 2 , x1 ) = 2 EI − x1 ) +
EI x3  2 − 6 
 x3  
NOTE 2: We will check the compatibility equation for two-dimensional problem, (see
Problem 5.11 – NOTE 3),
∂ 2 ε 11 ∂ 2 ε 22 ∂ 2 ε12 ∂ 2 κ x3 ∂ 2 κ x3
+ −2 = 0 + νx 2 − 2(1 + ν ) x 2 =0 X
∂x 22 ∂x12 ∂x1 ∂x 2 ∂x12 ∂x12

∂ 2 κ x3
Note that ≡ κ x3 ,11 = 0 , a fact already verified by the equilibrium equations.
∂x12
The problem presented previously is only valid if we discard completely the dimension x3 .
The reason follows.
As we are treating the problem by the state of plane stress we do not have stress σ i 3 but
we have the strain ε 33 ≠ 0 , (see Problem 6.24). Then, the strain field becomes:
 (1 + ν ) κ x3 ,1  2 a 2  
 − κ x3 x 2  x2 −  0 
2  4 
  
 (1 + ν ) κ x ,1  2 a 2  
ε ij =  3
 x2 −  νκ x x2 0 
2  4 
 
3

 
 0 0 νκ x3 x 2 
 

For the above strain field, the compatibility equations, (see Problem 5.11), are not satisfied,
i.e.:

University of Castilla-La Mancha Draft By: Eduardo W. V. Chaves (2016)


Ciudad Real – Spain
6 LINEAR ELASTICITY 595

 ∂ 2 ε33 ∂ 2 ε 22 ∂ 2 ε 23
 2 + −2 =0 X
 ∂x2 ∂x3 2
∂x2 ∂x3
 ∂ 2ε ∂ 2 ε11 ∂ 2 ε13
 233 + − 2 =0 X
 ∂x1 ∂x32 ∂x1∂x3
 2 2 2
 ∂ ε11 + ∂ ε 22 − 2 ∂ ε12 = 0 X
 ∂x22 ∂x12 ∂x1∂x2


 ∂  ∂ε 23 + ∂ε13 − ∂ε12  − ∂ ε 33 = − ∂ (νκ x3 x2 ) = −νκ ≠ 0
2 2

 ∂x3  ∂x1 ∂x2 ∂x3  ∂x1∂x2 ∂x1∂x2


x3 ,1 ✘ (it fails)

 ∂  ∂ε 23 ∂ε13 ∂ε12  ∂ 2 ε11
  − + +  − =0 X
 ∂x1  ∂x1 ∂x2 ∂x3  ∂x2 ∂x3

 ∂  ∂ε 23 − ∂ε13 + ∂ε12  − ∂ ε 22 = 0 X
2

 ∂x2  ∂x1 ∂x2 ∂x3  ∂x1∂x3

∂ 2 κ x3
where we have used κ x3 ,11 ≡ = 0 , since ε12 is independent of x1 , (see equation
∂x12
(6.88)), this implies that κ x3 ,1 is a constant.

University of Castilla-La Mancha Draft By: Eduardo W. V. Chaves (2016)


Ciudad Real - Spain
596 SOLVING PROBLEMS BY MEANS OF CONTINUUM MECHANICS

NOTE 3: Let us consider the case in which L = 5m , a = 3m , b = 0.1m , P = 1.0 × 108 N ,


E = 210 × 109 Pa , ν = 0.4 , (see Figure 6.38). With these data the beam deformation by using
equations (6.95) is presented in Figure 6.40. Note that a plane cross section before
deformation does not remain plane after deformation. Note that the fiber at x2 = 0 there is
no displacement according to the x1 -direction (displacement u1 ). As academic example, let
us change the parameter a = 3m by a = 1m . In this case we can appreciate that the cross
section remains plane after deformation, (see Figure 6.41).
The assumption in which the plane cross section remains plane and perpendicular to the
neutral line ( σ11 = 0 ) after deformation is known as Euler-Bernoulli’s hypothesis, and by means
of this assumption it is possible to treat beams as one dimensional case (Classical Beam
Theory), (see section 6.5 Introduction to one-dimensional elements).

cross section (not plane)


x2 ,u2 (after deformation)

u1 = 0.0

u 2 = 8.818mm

plane cross section


L = 5m (before deformation)

Figure 6.40: Beam deformation (amplified deformation) – for the case a = 3m .

neutral line- σ11 = 0


x2 ,u2

u1 = 0.0

u 2 ≈ 23.8cm

L = 5m

Figure 6.41: Beam deformation for the case a = 1m (amplified deformation).

University of Castilla-La Mancha Draft By: Eduardo W. V. Chaves (2016)


Ciudad Real – Spain
6 LINEAR ELASTICITY 597

6.2.1 Using Stress Function to solve 2D Linear Elasticity


Problems
Problem 6.34
In Problem 5.16 we have shown the Stress Formulation for three-dimensional elasticity.
Obtain the equivalent formulation for two-dimensional elasticity, i.e. considering the state
of plane stress and strain.
Solution:
As seen in Problem 5.5, the governing equations, for an isotropic linear elastic material in
small deformation regime, are:
Tensorial notation Indicial notation
The equations of motion: The equations of motion:
r
&r& = ρar (2 equations)
∇ xr ⋅ σ + ρb = ρu σij , j + ρbi = ρu
&&i = ρai (2 equations)

The constitutive equations for stress: The constitutive equations for stress:
(6.96)
σ (ε ) = λTr (ε )1 + 2µ ε (3 equations) σij = λε kkδ ij + 2 µεij (3 equations)

The kinematic equations: The kinematic equations:


r
ε = ∇ sym
x u (3 equations)
r
1  ∂u i ∂u j 
 (3 equations)
ε ij = +
2  ∂x j ∂x i 

where we have also considered two-dimensional problem ( i, j = 1,2 ). Note that for two-
dimensional problem we have 8 equations and 8 unknowns namely, ui (2 unknowns), εij
(3 unknowns) and σij (3 unknowns). The kinematic equations can be replaced by the
compatibility equations, (see Problem 5.11):
ε ij , kl + ε kl ,ij − ε il , jk − ε jk ,il = O ijkl

In the two-dimensional case the compatibility equations, (see Problem 5.11 – NOTE 3),
reduce to:
2 2
∂ 2ε11 ∂ 2ε 22 ∂ 2ε12 Engineering Notation ∂ 2ε x ∂ ε y ∂ γ xy
S33 = + −2 = 0     → S z = 2 + 2 − =0 (6.97)
∂x22 ∂x12 ∂x1∂x2 ∂y ∂x ∂x∂y
And the equations of motion for two-dimensional case become:
σij , j + ρb i = σ i1,1 + σ i 2, 2 + σ i 3,3 + ρbi = ρu
&&i 2D
→ σi1,1 + σi 2, 2 + ρb i = ρu
&&i (i = 1,2)
 ∂σ11 ∂σ12
 ∂x + ∂x + ρb1 = ρu1 = ρa1
&&
σ11,1 + σ12, 2 + ρb1 = ρu &&1  1 2
⇒ ⇒ 
σ 21,1 + σ 22, 2 + ρb 2 = ρu
&&2 ∂σ ∂σ
 21 + 22 + ρb = ρ&u& = ρa
 ∂x1 ∂x2
2 2 2

or in engineering notation:
 ∂σ11 ∂σ12  ∂σ x ∂τ xy
 ∂x + ∂x + ρb1 = ρu1 + + ρb x = ρa x
&& 
 1 2 Engineering Notation  ∂x ∂y
     →
 ∂σ 21 + ∂σ 22 + ρb = ρu
&&  ∂τ xy + ∂σ y + ρb = ρa
 ∂x1 ∂x2
2 2
 ∂x ∂y
y y

University of Castilla-La Mancha Draft By: Eduardo W. V. Chaves (2016)


Ciudad Real - Spain
598 SOLVING PROBLEMS BY MEANS OF CONTINUUM MECHANICS

We take the derivative of the first equation with respect to x and the second one with
respect to y , i.e.:
 ∂σ ∂τ xy ∂
 ∂ 2σ x ∂ 2 τ xy ∂
 x+ + ρb x = ρax →
∂x
 2 + = ( ρax − ρb x )
 ∂x ∂y  ∂x ∂x∂y ∂x
 ∂  2 2
 ∂τ xy ∂σ y  ∂ τ xy ∂ σ y ∂
 + + ρb y = ρa y → ∂y
 ∂x∂y + = ( ρa y − ρb y )
 ∂x ∂y  ∂y 2 ∂y
 ∂ 2 τ xy ∂ 2σ ∂
 = − 2x + ( ρax − ρb x ) (1)
 ∂x∂y ∂x ∂x
⇒ 2 2
 ∂ τ xy ∂ σy ∂
 ∂x∂y = − ∂y 2 + ∂y ( ρa y − ρb y ) (2)

By adding the both equations, (1)+(2), we can obtain
∂ 2 τ xy∂ 2σ x ∂ ∂ 2σ y ∂
2 = − 2 + ( ρa x − ρb x ) − + ( ρa y − ρb y ) (6.98)
∂x∂y ∂x ∂x ∂y 2 ∂y

a) The state of plane stress


The constitutive equations for the state of plane stress were obtained in Problem 6.24 and
the strain field, (see equation (6.49)), is given by:
 1 ν
ε x = E σ x − E σ y
 εx   1 −ν 0  σx  
  1    −ν 1
 ε y  = E − ν 1 0   σ y  ⇒ ε y = σx + σ y
 E E
 γ xy   0 0 2(1 + ν )  τ xy 
   2(1 + ν )
γ xy = τ xy
 E
By substituting the above strain components into the compatibility equation (“kinematic
equations”), and by considering the homogeneous material, we can obtain:

2 2
∂ 2 ε x ∂ ε y ∂ γ xy
+ − =0
∂y 2 ∂x 2 ∂x∂y
∂2 1 ν  ∂2  −ν 1  ∂  2(1 + ν )
2

⇒  σx − σ y  + 2  σx + σy  −  τ xy  = 0
∂y 2 E E  ∂x  E E  ∂x∂ y  E 
2 2 2
(6.99)
1 ∂ 2 σ x ν ∂ σ y ν ∂ 2 σ x 1 ∂ σ y 2(1 + ν ) ∂ τ xy
⇒ − − + − =0
E ∂y 2 E ∂y 2 E ∂x 2 E ∂x 2 E ∂x∂y
∂ 2σ x ∂ 2σ y 2
∂ 2σ x ∂ σ y ∂ 2 τ xy
⇒ − ν − ν + − 2 (1 + ν ) =0
∂y 2 ∂y 2 ∂x 2 ∂x 2 ∂x∂y
To consider simultaneously the two equations of motion we can use the equation in (6.98):
∂ 2 τ xy  ∂ 2σ x ∂   ∂ 2σ y ∂ 
⇒ 2(1 + ν ) = (1 + ν ) − + ( ρ a x − ρ b )
x  + (1 + ν ) − + ( ρa y − ρb y ) 
∂x∂y  ∂x
2
∂x   ∂y
2
∂y 
and by substituting the above equation into the equation (6.99) we can obtain:

University of Castilla-La Mancha Draft By: Eduardo W. V. Chaves (2016)


Ciudad Real – Spain
6 LINEAR ELASTICITY 599

∂ 2σ x ∂ 2σ y 2
∂ 2σ x ∂ σ y ∂ 2 τ xy
− ν − ν + − 2 (1 + ν ) =0
∂y 2 ∂y 2 ∂x 2 ∂x 2 ∂x∂y
∂ 2σ x ∂ 2σ y 2
∂ 2σ x ∂ σ y  ∂ 2σ x ∂ 
⇒ − ν − ν + − (1 + ν )  − + ( ρ a x − ρ b x ) 
∂y 2 ∂y 2 ∂x 2 ∂x 2  ∂x
2
∂x 
 ∂ 2σ y ∂ 
− (1 + ν )  − + ( ρa y − ρb y ) = 0
 ∂y
2
∂y 
By simplifying the above equation we can obtain:
Stress formulation 2D – The state of plane stress
2 2
∂ 2σ x ∂ 2σ x ∂ σ y ∂ σ y ∂ ∂  (6.100)
+ + + = (1 + ν )  ( ρa x − ρb x ) + ( ρa y − ρb y ) 
 ∂x ∂y
2 2 2 2
∂x ∂y ∂y ∂x 

For the static or quasi-static case the above equation reduces to:
Stress formulation 2D – The state of plane stress (static case)
2 2
∂ 2σ x ∂ 2σ x ∂ σ y ∂ σ y ∂ ∂  (6.101)
+ + + = −(1 + ν )  ( ρb x ) + ( ρb y )
 ∂x ∂y
2 2 2 2
∂x ∂y ∂y ∂x 

b) The state of plane strain


The constitutive equations for the state of plane strain were obtained in Problem 6.24 and
the strain field, (see equation (6.61)), is given by:
 (1 + ν )(1 − ν ) ν (1 + ν )
ε x = E
σx −
E
σy
 εx  1 − ν − ν 0  σ x  
  1+ν  −ν 1−ν    ν (1 + ν ) (1 + ν )(1 − ν )
εy  = E  0  σ y  ⇒ ε y = − σx + σy
 E E
 γ xy   0 0 2 τ xy 
   2(1 + ν )
γ xy = τ xy
 E
By substituting the above strain components into the compatibility equation (“kinematic
equations”), and by considering the homogeneous material, we can obtain:
2 2
∂ 2 ε x ∂ ε y ∂ γ xy
+ − =0
∂y 2 ∂x 2 ∂x∂y
∂ 2  (1 + ν )(1 − ν ) ν (1 + ν )  ∂ 2  ν (1 + ν ) (1 + ν )(1 − ν ) 
⇒  σ x − σy  + 2 − σx + σy 
∂y 2  E E  ∂x  E E 
(6.102)
∂ 2  2(1 + ν ) 
−  τ xy  = 0
∂x∂y  E 
∂ 2σ x ∂ 2σ y ∂ 2σ x ∂ 2σ y ∂ 2 τ xy
⇒ (1 − ν ) − ν − ν + (1 − ν ) − 2 =0
∂y 2 ∂y 2 ∂x 2 ∂x 2 ∂x∂y
To consider simultaneously the two equations of motion we can use the equation in (6.98):
∂ 2 τ xy
 ∂ 2σ x ∂   ∂ 2σ y ∂ 
⇒2 = − + ( ρ a x − ρb x )  +  − + ( ρ a y − ρ b y ) 
∂x∂y  ∂x 2 ∂x   ∂y
2
∂y 

University of Castilla-La Mancha Draft By: Eduardo W. V. Chaves (2016)


Ciudad Real - Spain
600 SOLVING PROBLEMS BY MEANS OF CONTINUUM MECHANICS

and by substituting the above equation into the equation (6.102) we can obtain:
∂ 2σ x ∂ 2σ y ∂ 2σ x ∂ 2σ y ∂ 2 τ xy
(1 − ν ) − ν − ν + (1 − ν ) − 2 =0
∂y 2 ∂y 2 ∂x 2 ∂x 2 ∂x∂y
∂ 2σ x ∂ 2σ y ∂ 2σ x ∂ 2σ y  ∂ 2σ x ∂ 
⇒ (1 − ν ) − ν − ν + (1 − ν ) − − + ( ρa x − ρb x ) 
∂y 2
∂y 2
∂x 2
∂x 2
 ∂x
2
∂x 
 ∂ 2σ y ∂ 
− − + ( ρa y − ρ b y ) =0
 ∂y
2
∂y 
By simplifying the above equation we can obtain:
Stress formulation 2D – The state of plane strain
2 2
∂ 2σ x ∂ 2σ x ∂ σ y ∂ σ y 1 ∂ ∂  (6.103)
+ + + =  ( ρa x − ρb x ) + ( ρa y − ρb y ) 
∂x 2
∂y 2
∂y 2
∂x 2
(1 − ν )  ∂x ∂y 

For the static or quasi-static case the above equation reduces to:
Stress formulation 2D – The state of plane strain (static case)
2 2
∂ 2σ x ∂ 2σ x ∂ σ y ∂ σ y −1  ∂ ∂  (6.104)
+ + + =  ( ρb x ) + ( ρb y ) 
∂x 2
∂y 2
∂y 2
∂x 2
(1 − ν )  ∂x ∂y 

NOTE 1: Recall that the body forces can be represented by means of the potential φ , i.e.
r r − ∂φ − ∂φ
b = −∇ xrφ , since b is a conservative field. Then, we can write b x = and b y = .
∂x ∂y
Recall also that in Problem 5.18 we have defined the Airy stress function Φ . If we take
into account the body forces we can write:
∂ 2Φ ∂ 2Φ ∂ 2Φ
σ x − ρφ = ; σ y − ρφ = ; τ xy = τ yx = − (6.105)
∂y 2 ∂x 2 ∂x∂y
thus
∂ 2Φ ∂ 2Φ
σ x = ρφ + ; σ y = ρφ + (6.106)
∂y 2 ∂x 2
Substituting the above stress components into the equation (6.101) and by considering the
mass density field homogeneous we can obtain:
2 2
∂ 2σ x ∂ 2σ x ∂ σ y ∂ σ y ∂ ∂   ∂b ∂b y 
+ + + = −(1 + ν )  ( ρb x ) + ( ρb y ) = − ρ (1 + ν )  x + 
 ∂x ∂y  ∂x ∂y 
2 2 2 2
∂x ∂y ∂y ∂x 
∂2  ∂ 2Φ  ∂ 2  ∂ 2Φ  ∂ 2  ∂ 2Φ  ∂ 2  ∂ 2Φ   ∂ 2φ ∂ 2φ 
 ρφ + 2  + 2  ρφ + 2  + 2  ρφ + 2  + 2  ρφ + 2  = ρ (1 + ν )  2 + 2 
∂x 2  ∂y  ∂y  ∂y  ∂y  ∂x  ∂x  ∂x 
     ∂x ∂y 
∂ 4Φ ∂ 4Φ ∂ 4Φ  ∂ 2φ ∂ 2φ   ∂ 2φ ∂ 2φ 
⇒ + 2 + + 2 ρ  +  = ρ (1 + ν )  2 + 2
∂x 4 ∂x 2 ∂y 2 ∂y 4  ∂x 2 ∂y 2 
   ∂x ∂y 
∂ 4Φ ∂ 4Φ ∂ 4Φ  ∂ 2φ ∂ 2φ 
⇒ + 2 + = ρ [(1 + ν ) − 2]  2 + 2
∂x 4 ∂x 2 ∂y 2 ∂y 4  ∂x ∂y 

University of Castilla-La Mancha Draft By: Eduardo W. V. Chaves (2016)


Ciudad Real – Spain
6 LINEAR ELASTICITY 601

thus:
Stress formulation 2D – The state of plane stress (static case)

∂ 4Φ ∂ 4Φ ∂ 4Φ  ∂ 2φ ∂ 2φ  (6.107)
+ 2 + = − ρ (1 − ν )  2 + 2
∂x 4 ∂x 2 ∂y 2 ∂y 4  ∂x ∂y 

Now, if we substitute the stress components (6.106) into the equation (6.104) we can
obtain:
∂2  ∂ 2Φ  ∂ 2  ∂ 2Φ  ∂ 2  ∂ 2Φ  ∂ 2  ∂ 2Φ  − ρ  ∂b x ∂b y 
 ρφ + 2  + 2  ρφ + 2  + 2  ρφ + 2  + 2  ρφ + 2  =  + 
∂x 2  ∂y  ∂y  ∂y  ∂y  ∂x  ∂x  
∂x  (1 − ν )  ∂x ∂y 
   
∂ 4Φ ∂ 4Φ ∂ 4Φ  ∂ 2φ ∂ 2φ  ρ  ∂ 2φ ∂ 2φ 
⇒ + + + ρ  
∂x 4
2
∂x 2 ∂y 2 ∂y 4
2 +
 ∂x 2 ∂y 2  (1 − ν )  ∂x 2 + ∂y 2 
=
   
∂ 4Φ ∂ 4Φ ∂ 4Φ  1   ∂ 2φ ∂ 2φ 
⇒ + 2 + = ρ 
 (1 − ν ) − 2   2 + 2 
∂x 4 ∂x 2 ∂y 2 ∂y 4    ∂x ∂y 
thus
Stress formulation 2D – The state of plane strain (static case)

∂ 4Φ ∂ 4Φ ∂ 4Φ (1 − 2ν )  ∂ 2φ ∂ 2φ  (6.108)
+ 2 + = − ρ  + 
∂x 4 ∂x 2 ∂y 2 ∂y 4 (1 − ν )  ∂x 2 ∂y 2 

Near to the Earth surface the body forces can be considered uniform (homogenous field),
∂ 2φ ∂ 2φ
hence ≈ ≈ 0 . With that the governing equation for two-dimensional cases, (see
∂x ∂y
equations (6.107) and (6.108)), becomes:
Stress formulation 2D – (static case and homogenous body forces field)

∂ 4Φ ∂ 4Φ ∂ 4Φ
+ 2 + =0 ; ∇ 4xr Φ = 0 ; Φ ,iijj = 0 (i, j = 1,2) (6.109)
∂x 4 ∂x 2∂y 2 ∂y 4

NOTE 2: Note that we have reduced the original problem, 8 equations and 8 unknowns,
(see equation (6.96)), to 1 equation (6.109) and 1 unknown ( Φ ).
Recall that the analytical solution (the exact one) in most practical cases is quite complex
and even impossible to be obtained. So we resort to numerical technique, which consists
in: given a problem we find the solution. During the era of G.B. Airy (1862) the only
possible solution was the analytical one, since the numerical techniques were scarce. Then,
they used to address the elastic problem through inverse method (Laier&Barreiro (1983)), i.e.
for a given solution of the equation (6.109) they seek which problem represents such
solution.
The stress function can be adopted, for example, by a polynomial function, (see Figure
6.42):
Φ = K 1 + K 2 x + K 3 y + K 4 x 2 + K 5 xy + K 6 y 2 + K 7 x 3 + K 8 x 2 y + K 9 xy 2
+ K 10 y 3 + K 11 x 4 + K 12 x 3 y + K 13 x 2 y 2 + K 14 xy 3 + K 15 xy 4 + K 16 x 5 (6.110)
4 3 2 2 3 4 5 6
+ K 17 x y + K 18 x y + K 19 x y + K 20 xy + K 21 y + K 22 x + L

University of Castilla-La Mancha Draft By: Eduardo W. V. Chaves (2016)


Ciudad Real - Spain
602 SOLVING PROBLEMS BY MEANS OF CONTINUUM MECHANICS

1 constant term
x y linear terms

x2 xy y2 quadratic terms

x3 x2 y xy 2 y3 cubic terms

x4 x3 y x2 y2 xy 3 y4 quartic terms

x5 x4 y x3 y 2 x2 y3 xy 4 y5 quintic terms

Figure 6.42: Pascal’s polynomial for 2D.

Example: Let us assume that the Airy stress function is given by the polynomial:
Φ = K 4 x 2 + K 5 xy + K 6 y 2 (6.111)
where K1 , K 2 , and K3 are constants. If we are not considering the body forces the stress
field (6.105) becomes:
∂ 2Φ ∂ 2Φ ∂ 2Φ
σx = = 2K 6 ; σy = = 2K 4 = −K 5
; τ xy = −
(6.112)
∂y 2 ∂x 2 ∂x∂y
r
Note that the stress field is homogenous, i.e. it is independent of x . For the particular case
when K 4 = K 5 = 0 we can obtain the problem represented by the bar subjected to axial
force F at its ends, (see Figure 6.43):
Stress field (Bar subjected to axial force):
 F F
σ x = A = 2 K 6 ⇒ K6 =
2A

σ y = 0

τ xy = 0

Strain field (Bar subjected to axial force):
 σ x 2K 6 F
ε x = E = E = EA

 σx 2K 6 − ν F
ε y = −ν = −ν =
 E E EA

γ xy = 0

University of Castilla-La Mancha Draft By: Eduardo W. V. Chaves (2016)


Ciudad Real – Spain
6 LINEAR ELASTICITY 603

y, v 2 D ( x, y ) A (Area)

x, u

σx F
σx =
A
Figure 6.43: Bar subjected to axial force.

Displacement field (Bar subjected to axial force):


 ∂u F integrating in x F
 = εx =   → u ( x, y ) = x + f 1 ( y ) + C1
 ∂x EA EA
 ∂v ν F integrating in y −ν F
 = εy = −     → v( x, y ) = y + f 2 ( x) + C 2
 ∂y EA EA
 ∂v ∂u
 + = γ xy = 0
 ∂x ∂y
where f1 ( y ) is a function of y , f 2 ( x) is a function of x , C1 and C 2 are constants of
integration. By means of the above third equation we can obtain:
∂v ∂u
+ = γ xy = 0
∂x ∂y
∂  −ν F  ∂  F 
⇒  y + f 2 ( x) + C 2  +  x + f1 ( y ) + C1  = 0
∂x  EA  ∂y  EA 
∂f ( x) ∂f1 ( y )
⇒ 2 + =0
∂x ∂y
∂f ( x) ∂f ( y )
⇒ 2 =− 1
∂x ∂y
The only possible solution is when:
∂f 2 ( x) ∂f1 ( y )
= −C3 and = C3
∂x ∂y
where C3 is a constant. Then, the displacement field becomes:
 F
u ( x, y ) = EA x + C 3 y + C1

v( x, y ) = − ν F y − C x + C

3 2
EA
The constants C1 , C2 and C3 can be obtained by means of the problem boundary
condition. Let us assume that the bar has the boundary condition as indicated in Figure
6.44.

University of Castilla-La Mancha Draft By: Eduardo W. V. Chaves (2016)


Ciudad Real - Spain
604 SOLVING PROBLEMS BY MEANS OF CONTINUUM MECHANICS

According to the boundary conditions, (see Figure 6.44), we can obtain:


u ( x = 0, y = 0) = 0 ⇒ u ( x = 0, y = 0) = C1 = 0

v( x = 0, y = 0) = 0 ⇒ u ( x = 0, y = 0) = C 2 = 0
u ( x = 0, y ) = 0 ⇒ u ( x = 0, y = 0) = C = 0
 3

in which the displacement field becomes:


 F
u = EA x
 (6.113)
v = − ν F y
 EA

y, v
2 D ( x, y ) A (cross section area)

x, u

F
σx =
A

Figure 6.44: Bar subjected to axial force.

Problem 6.35
Obtain the displacement field for a problem (without body force) described in Figure 6.45.
As boundary condition consider that at ( x = 0, y = 0, z = 0) ⇒ (u = 0, v = 0, w = 0) .

y rigid y
p p

x z

L L

Figure 6.45
Solution:
Let us assume the following Airy stress function:
Φ = K 4 x 2 + K 5 xy + K 6 y 2 (6.114)

University of Castilla-La Mancha Draft By: Eduardo W. V. Chaves (2016)


Ciudad Real – Spain
6 LINEAR ELASTICITY 605

with that we can obtain the stresses:


∂ 2Φ ∂ 2Φ ∂ 2Φ
σx = = 2 K6 ; σy = = 2K4 ; τ xy = − = − K5 = 0 (6.115)
∂y 2 ∂x 2 ∂x∂y
Note that K 5 = 0 since normal stress only produce normal strain, so, γ xy = 0 ⇒ τ xy = 0 .
Note also that according to Figure 6.45 we can conclude that
∂ 2Φ −p
σx = = 2K6 = − p ⇒ K6 =
∂y 2 2
For this problem we have that ε y = 0 and by considering the stress by considering the state
of plane stress, (see Problem 6.24), we can obtain the stress field:
   
σx   1 ν 0   εx   1 ν 0  ε x   εx 
  E   E   E 
 σ y  = (1 − ν 2 ) ν 1 0  εy  = ν 1 0 0= 2 
ν ε x 
1 − ν    (1 − ν )  1 − ν    (1 − ν ) 
2
τ xy  
  0 0   γ xy  0 0 0  0 
 2   2 
 E 
 ε x (1 − ν 2 )  ε x E 
σx     (1 − ν 2 )   − p 
   E   
⇒ σ y  = ν ε x =  ν σ x  = − νp 
 2 
(1 − ν )
 τ xy       0 
 
 0    0   

 
− (1 − ν 2 )
By means of the above equation we can obtain the normal strain ε x p.
E
Let us check the strain field:
 εx   1 −ν 0  σx  σ x − ν σ y  (− p ) − ν (−ν p) (1 − ν 2 )
  1   1  1  −p  
 ε y  = E − ν 1  
0   σ y  = σ y − ν σ x  = (−ν p ) − ν (− p) =  0 
E E E  0 
 γ xy   0 0 2(1 + ν )  τ xy   0   0 
   
The normal strain ε z can be obtained as follows:
−ν − ν (ε x + ε y ) − ν (1 − ν 2 )  − p  ν (1 − ν 2 ) ν (1 + ν )
εz = Tr (σ ) = =  = p= p
E (1 − ν ) (1 − ν )  E  E (1 − ν ) E
Taking into account the definition of the normal strain we can obtain:
∂u − p −p
εx = = (1 − ν 2 ) integratin
 g in x
→ u= (1 − ν 2 ) x + C1
∂x E E
∂w ν (1 + ν ) ν (1 + ν ) p
εz = = p integratin
  g in z
→ w= z + C2
∂z E E
and by applying the boundary condition at ( x = 0, y = 0, z = 0) we can conclude that Ci = 0 ,
thus
 −p
u = E (1 − ν ) x
2


v = 0
 ν (1 + ν ) p
w = z
 E

University of Castilla-La Mancha Draft By: Eduardo W. V. Chaves (2016)


Ciudad Real - Spain
606 SOLVING PROBLEMS BY MEANS OF CONTINUUM MECHANICS

Problem 6.36
Consider the Airy stress function:
Φ = K 10 y 3 (6.116)
where K10 is a constant. What is the problem governed by the Airy stress function (6.116)?
Obtain the stress, strain and displacement fields. Consider the state of plane stress.
Solution:
If we are not considering the body forces the stress field (6.105) becomes:
∂ 2Φ ∂ 2Φ ∂ 2Φ
σx = = 6 K 10 y ; σy = =0 ; τ xy = − =0 (6.117)
∂y 2 ∂x 2 ∂x∂y
Note that the stress field σ x = σ x ( y ) depends only on y . For a given cross section of the
bar we have:
Resultant force on the cross-section:
b a
z= y=
2 2


F = σ x dA =
A

−b −a
∫ 6K 10 ydydz =0
z= y=
2 2

Bending moment on the cross-section:

∫ ∫
M = yσ x dA = 6 K 10 y 2 dA = 6 K10 y 2 dA = 6 K 10 I z
A A

A

where I z = ∫ y 2 dA is the moment of inertia of the cross-sectional area about the z axis.
A
Note that, this is the case of pure bending, (see Figure 6.46). We can also obtain that
M
M = 6 K10 I z ⇒ K10 = .
6I z
Let us analyze the sign of M . According to our sign convention the moment is positive if
the moment vector has the same sense as the axis, e.g. the vector M z is positive if it has
the same sense as the z -axis, (see Figure 6.46). Note also that according to this sign
convention we have σ x < 0 for values of y > 0 , so, for a positive value of M z we have
K 10 < 0 :
−Mz
K 10 =
6I z
Stress field (Pure bending):
− Mz
σ x = 6 K10 y = y ; σy = 0 ; τ xy = 0
Iz
Strain field (pure bending):

University of Castilla-La Mancha Draft By: Eduardo W. V. Chaves (2016)


Ciudad Real – Spain
6 LINEAR ELASTICITY 607

 σ x 6 K10 y − M z
ε x = E = E = EI y
 z
 σx 6 K10 y ν M z
ε y = −ν = −ν = y
 E E EI z

γ xy = 0

εy
εx
y y

σx σx
Mz

z
a
x
Mz

Cross section

Figure 6.46: Beam under pure bending.

Displacement field (Pure bending):


 ∂u −Mz integrating in x −Mz
 ∂x = ε x = EI y     → u ( x, y ) =
EI z
yx + f 1 ( y ) + C1
 z
 ∂v ν Mz ν Mz 2
 = εy = y integratin
  g in y
→ v( x, y ) = y + f 2 ( x) + C 2
 ∂y EI z 2 EI z
 ∂v ∂u
 + = γ xy = 0
 ∂x ∂y
Taking into account that:
∂v ∂u
+ = γ xy = 0
∂x ∂y
∂ ν M z 2  ∂ − Mz 
⇒  y + f 2 ( x) + C 2  +  yx + f 1 ( y ) + C1  = 0
∂x  2 EI z  ∂y  EI z 
∂f 2 ( x) − M z ∂f 1 ( y )
⇒ + x+ =0
∂x EI z ∂y
∂f 2 ( x) − M z ∂f ( y )
⇒ + x=− 1
∂x EI z ∂y
Similarly to the previous example we can conclude that:

University of Castilla-La Mancha Draft By: Eduardo W. V. Chaves (2016)


Ciudad Real - Spain
608 SOLVING PROBLEMS BY MEANS OF CONTINUUM MECHANICS

∂f 2 ( x) M z ∂f 1 ( y )
− x = C3 and = −C 3
∂x EI z ∂y
By integrate the above equations we can obtain:
∂f 2 ( x) M z Mz 2
− x = C3 ⇒ f 2 ( x) = C 3 x + x
∂x EI z 2 EI z
∂f 1 ( y )
= −C 3 ⇒ f1 ( y ) = −C 3 y
∂y
with that the displacement field becomes:
 −Mz −Mz
u = EI yx + f 1 ( y ) + C1 = EI yx − C 3 y + C1
 z z

v = ν M ν M Mz 2 Mz
z
y 2 + f 2 ( x) + C 2 = z
y 2 + C3 x + x + C2 = (ν y 2 + x 2 ) + C 3 x + C 2
 2 EI z 2 EI z 2 EI z 2 EI z

where the constants C1 , C2 and C3 can be obtained by means of the problem boundary
conditions. Let us assume that the bar has the boundary condition as indicated in Figure
6.47, in which one end of the beam has a fixed support (clamped or cantileved) and the
other end is free.

y, v

Mz
u = 0
( x = 0, y = 0)
v = 0
x, u

Deformation
u = 0  ∂v
( x = 0, y ≠ 0) and ( x = 0, y ) = 0
v ≠ 0  ∂x

Figure 6.47: Fixed-free beam (boundary conditions).

With these conditions we can obtain:


u ( x = 0, y = 0) = 0 ⇒ u ( x = 0, y = 0) = C1 = 0

v( x = 0, y = 0) = 0 ⇒ u ( x = 0, y = 0) = C 2 = 0
Mz ∂v( x, y ) M z
v ( x, y ) = (ν y 2 + x 2 ) + C 3 x + C 2 ⇒ = x + C3
2 EI z ∂x EI z
∂v( x = 0, y = 0)
= 0 = C3
∂x
∂ 2 v ( x, y ) M z
Note also that = , the second derivative of the deflection v( x, y ) is positive.
∂x 2 EI z
Then, the displacement field becomes:

University of Castilla-La Mancha Draft By: Eduardo W. V. Chaves (2016)


Ciudad Real – Spain
6 LINEAR ELASTICITY 609

 − Mz
u ( x, y ) = EI xy
 z

v( x, y ) = M z
(ν y 2 + x 2 )
 2 EI z

− Mz
The neutral line corresponds to the line in which σ x = y = 0 . And the defection of the
Iz
Mz 2
neutral line ( y = 0 ) is given by v = x .
2 EI z

Problem 6.37
Obtain the stress field for the problem, without body force, which is represented by the
Airy stress function:
Φ = K 5 xy + K 10 y 3 + K 14 xy 3 (6.118)
As boundary condition (B.C.) consider that
a
at ( y = ± ) ⇒ τ xy = 0
2
at x ⇒ P = ∫ τ xy dA , where A = ab is the area of the rectangular cross section
A

at x = 0 ⇒ M = − PL (Bending moment)
Solution:
For this problem we have
∂Φ ∂Φ
= K 5 y + K14 y 3 ; = K 5 x + 3K 10 y 2 + 3K14 xy 2
∂x ∂y
Then,
∂ 2Φ ∂ 2Φ ∂ 2Φ
σx = = 6 K 10 y + 6 K 14 xy ; σy = =0 ; τ xy = − = −( K 5 + 3K 14 y 2 )
∂y 2 ∂x 2 ∂x∂y
(6.119)
a
Applying the boundary condition ( y = ± ) ⇒ τ xy = 0 , we can conclude that
2
a a2 a2
τ xy ( y = ± ) = − K 5 − 3K 14 =0 ⇒ K 5 = −3K 14
2 4 4
With that the tangential stress becomes
 a2   a2 2
τ xy = − − 3K 14  − 3K 14 y 2 = 3K 14  − y 
 4   4


By applying the boundary condition at x ⇒ P = ∫ τ xy dA , we can obtain:


A

University of Castilla-La Mancha Draft By: Eduardo W. V. Chaves (2016)


Ciudad Real - Spain
610 SOLVING PROBLEMS BY MEANS OF CONTINUUM MECHANICS

 a2   a2   a2 
∫ ∫
P = τ xy dA = 3K 14  − y 2  dA = 3K 14  ∫ dA − y 2 dA  = 3K 14 
∫ A − I z 
 4 
A A  4   A A   4 
 a2   3ba 3 
⇒ P = 3K 14  ab − I z  = 3K 14  − I z  = 3K 14 (3I z − I z ) = 6 K 14 I z
 4   12 
P
⇒ K 14 =
6I z
Then, the stresses given by (6.119) become
P P  a2 2
σ x = 6 K 10 y + xy ; σy = 0 ; τ xy =  − y  (6.120)
Iz 2I z  4 
 
The bending moment M acting at the cross-section can be obtained as follows:
 P  P
A
∫ A

M ( x) = σ x ydA =  6 K 10 y +
I z
xy  ydA = 6 K 10 y 2 dA +
 A 23
I z A 23

x y 2 dA ∫
1 1
=I z =I z

P
⇒ M ( x) = 6 K 10 I z + xI z
Iz
⇒ M ( x) = 6 K 10 I z + Px
The constant K10 can be obtained by means of the B.C.: x = 0 ⇒ M = − PL :
− PL
M ( x = 0) = 6 K 10 I z = − PL ⇒ K 10 =
6I z
Then, the stresses given by (6.119) become
Py P  a2 
σx = ( x − L) ; σy = 0 ; τ xy =  − y2  (6.121)
Iz 2 I z  4 

The strain-stress relationship ( ε(σ ) ) for the state of plane stress is given by:

 ε11   ε x   1 −ν 0  σx 
 ε  =  ε  = 1 − ν 1
 
0   σ y 
 22   y  E 
2ε12   γ xy   0 0 2(1 + ν ) τ xy 
 Py 
 Py   ( x − L)  (6.122)
 1 −ν 0   I ( x − L)   EI z 
1   z   Py 
= − ν 1 0  0  =  −ν ( x − L) 
E EI z
2(1 + ν )  P  a − y 2   
2
 0 0
     
P (1 + ν ) a 2
 
 2I z  4    − y 2 
 
 EI z  4 
By considering that
−P ∂κ z −P
κz = ( x − L) ; ≡ κz,x =
EI z ∂x EI z
the equation in (6.122) can be rewritten as follows

University of Castilla-La Mancha Draft By: Eduardo W. V. Chaves (2016)


Ciudad Real – Spain
6 LINEAR ELASTICITY 611

 Py 
 ( x − L)   
 
 ε11   ε x   EI z 
 − κz y 

 ε  =  ε  = − ν Py ( x − L) 
 22   y    =  ν κ z y 
 EI z   
2ε12   γ xy   2
 − (1 + ν ) κ z , x  a − y 2 
 P(1 + ν )  a 2  
  − y 2     4 
   
 EI z  4  
 (1 + ν ) κ z , x  2 a 2 
 − κz y  y − 
 ε11 ε12   2  4 
⇒ = 
  
ε12 ε 22   (1 + ν ) κ z , x  y 2 − a 
2

 ν κ y 
4 
z
 2  
Note that this problem was already established in Problem 6.33.

Problem 6.38
Obtain the stress field for a problem (without body force) for the problem represented in
Figure 6.48.

y y A = ab

b
x z 2

Qy Qy b
2

qy = p
a

L L ab 3
2 2 ∫
I z = y 2 dA =
A
12

Figure 6.48

Consider the following Airy stress function, (Sechler (1952)):


Φ = K 4 x 2 + K 8 x 2 y + K10 y 3 + K11 x 4 + K17 x 4 y + K19 x 2 y 3 + K 21 y 5 (6.123)
and consider that σ x = σ x ( y ) is not a function of x .

As boundary condition (B.C.) consider that


b
at ( y = ) ⇒ σ y = τ xy = 0
2

University of Castilla-La Mancha Draft By: Eduardo W. V. Chaves (2016)


Ciudad Real - Spain
612 SOLVING PROBLEMS BY MEANS OF CONTINUUM MECHANICS

−b
at ( y = ) ⇒ σy = − p ; τ xy = 0
2

L pL
at ( x = ± ) ⇒
2 ∫τ
A
xy dA = Qy = m
2
; ∫ σ dA = F
A
x x =0 ; ∫σ
A
x ydA = Mz = 0

Solution:
The following must hold at any point of the beam:
∂ 4Φ ∂ 4Φ ∂ 4Φ
+ 2 + =0 ; ∇ 4xr Φ = 0 ; Φ ,iijj = 0 (i, j = 1,2) (6.124)
∂x 4 ∂x 2∂y 2 ∂y 4
Then, let us take the derivatives:
Φ = K 4 x 2 + K8 x 2 y + K10 y 3 + K11 x 4 + K17 x 4 y + K19 x 2 y 3 + K 21 y 5
∂Φ
= 2 K 4 x + 2 K 8 xy + 4 K11 x 3 + 4 K17 x 3 y + 2 K19 xy 3
∂x
∂ 2Φ
2
= 2 K 4 + 2 K 8 y + 12 K11 x 2 + 12 K17 x 2 y + 2 K19 y 3
∂x
∂ 3Φ
= 24 K11 x + 24 K17 xy
∂x 3
∂ 4Φ
= 24 K11 + 24 K17 y
∂x 4
Φ = K 4 x 2 + K 8 x 2 y + K10 y 3 + K11 x 4 + K17 x 4 y + K19 x 2 y 3 + K 21 y 5
∂Φ
= K 8 x 2 + 3K10 y 2 + K17 x 4 + 3K19 x 2 y 2 + 5 K 21 y 4
∂y
∂ 2Φ
= 6 K10 y + 6 K19 x 2 y + 20 K 21 y 3
∂y 2
∂ 3Φ
3
= 6 K10 + 6 K19 x 2 + 60 K 21 y 2
∂y
∂ 4Φ
= 120 K 21 y
∂y 4

∂  ∂ 2Φ  ∂ 4Φ ∂

2 

2 
= 2 2
= 2 (2 K 4 + 2 K8 y + 12 K11 x 2 + 12 K17 x 2 y + 2 K19 y 3 ) = 12 K19 y
∂y  ∂x  ∂y ∂x ∂y
∂  ∂ 2Φ  ∂ 4Φ ∂

2 

2 
= 2 2
= 2 (6 K10 y + 6 K19 x 2 y + 20 K 21 y 3 ) = 12 K19 y = 12 K19 y
∂x  ∂y  ∂x ∂y ∂x

University of Castilla-La Mancha Draft By: Eduardo W. V. Chaves (2016)


Ciudad Real – Spain
6 LINEAR ELASTICITY 613

Then, the equation in (6.124) becomes:


∂ 4Φ ∂ 4Φ ∂ 4Φ
+ 2 + =0
∂x 4 ∂x 2 ∂y 2 ∂y 4 (6.125)
⇒ 24 K11 + 24 K17 y + 2(12 K19 y ) + 120 K 21 y = 0
Note that the above equation is only fulfilled if and only if K11 = 0 , then
Φ = K 4 x 2 + K 8 x 2 y + K10 y 3 + K11 x 4 + K17 x 4 y + K19 x 2 y 3 + K 21 y 5
(6.126)
⇒ Φ = K 4 x 2 + K 8 x 2 y + K10 y 3 + K17 x 4 y + K19 x 2 y 3 + K 21 y 5
From equation (6.125) we can obtain the following equation:
24 K17 y + 2(12 K19 y ) + 120 K 21 y = 0
⇒ y (24 K17 + 24 K19 + 120 K 21 ) = 0 (6.127)
⇒ 24 K17 + 24 K19 + 120 K 21 = 0 ⇒ K17 + K19 + 5K 21 = 0
The stress field can be expressed as follows:
∂ 2Φ
σx = = 6 K10 y + 6 K19 x 2 y + 20 K 21 y 3
∂y 2
∂ 2Φ 2 2 3 2 3
σy = 2
= 2 K 4 + 2 K 8 y + 12 K
{11 x + 12 K17 x y + 2 K19 y = 2 K 4 + 2 K 8 y + 12 K17 x y + 2 K19 y
∂x =0

∂ 2Φ − ∂
τ xy = − = ( K 8 x 2 + 3K10 y 2 + K17 x 4 + 3K19 x 2 y 2 + 5 K 21 y 4 ) = −2 K 8 x − 4 K17 x 3 − 6 K19 xy 2
∂x∂y ∂x
(6.128)
Note that σ y is not a function of x , thus K17 = 0 , then:

 ∂ 2Φ 2 3
σ x = 2 = 6 K10 y + 6 K19 x y + 20 K 21 y
 ∂y
 ∂ 2Φ 3
σ y = 2 = 2 K 4 + 2 K 8 y + 2 K19 y (6.129)
 ∂ x
 ∂ 2Φ
τ xy = − = −2 K 8 x − 6 K19 xy 2
∂x ∂y

Taking into account that K17 = 0 , the equation (6.127) becomes:
K17 + K19 + 5 K 21 = 0 ⇒ K19 + 5 K 21 = 0 ⇒ K19 = −5 K 21 (6.130)
With that the stress field (6.129) can be rewritten as follows:
 ∂ 2Φ 2 3 3 2
σ x = 2 = 6 K10 y + 6(−5 K 21 ) x y + 20 K 21 y = 6 K10 y + (20 y − 30 x y ) K 21
 ∂y
 ∂ 2Φ
σ
 y = 2
= 2 K 4 + 2 K 8 y + 2(−5K 21 ) y 3 = 2 K 4 + 2 K 8 y − 10 K 21 y 3 (6.131)
 ∂ x
 ∂ 2Φ
τ xy = − = −2 K 8 x − 6(−5 K 21 ) xy 2 = −2 K 8 x + 30 K 21 xy 2
∂x ∂y


University of Castilla-La Mancha Draft By: Eduardo W. V. Chaves (2016)


Ciudad Real - Spain
614 SOLVING PROBLEMS BY MEANS OF CONTINUUM MECHANICS

Applying the Boundary Conditions


b
1) at ( y = ) ⇒ σ y = τ xy = 0
2
2
b b b2
τ xy ( ) = −2 K 8 x + 30 K 21 xy 2 = −2 K 8 x + 30 K 21 x  = 0 ⇒ − 2 K 8 = −30 K 21
2 2 4
(6.132)
4 15b 2
⇒ K 21 = K8 ⇔ K8 = K 21
15b 2 4
and
15b 2
σ y ( y ) = 2 K 4 + 2 K 8 y − 10 K 21 y 3 = 2 K 4 + 2 K 21 y − 10 K 21 y 3
4
(6.133)
 15b 2 
⇒ σ y ( y ) = 2 K 4 +  y − 10 y 3  K 21
 2 
b
then, when ( y = )
2

b  15b 2  b   b  
3
σ y ( ) = 2K 4 +  − 10  K 21 = 0
2  2  2   2  

 15b 3 10b 3   10b 3  (6.134)
⇒ 2 K 4 +  −  K 21 = 0
 ⇒ K 4 +   K 21 = 0

 4 8   8 
− 5b3 −4
⇒ K4 = K 21 ⇒ K 21 = K4
4 5b 3
Then, the equation in (6.133):
 15b 2  − 5b 3  15b 2 
σ y ( y ) = 2 K 4 +  y − 10 y 3  K 21 ⇒ σ y ( y) = 2 K 21 +  y − 10 y 3  K 21
 2  4  2 
(6.135)
 − 5b3 15b 2 
⇒ σ y ( y ) =  + y − 10 y 3  K 21
 2 2 
−b
2) at ( y = ) ⇒ σ y = − p . Then:
2

− b  − 5b3 15b 2   − 5b 3 15b 2  − b   − b  


3
σy ( ) =  + y − 10 y 3  K 21 =  +   − 10   K 21 = − p
2  2
 2 2   2  2   2  
(6.136)
 − 10b 3 15b 3 5b3  p
⇒  − +  K 21 = − p ⇒ K 21 = 3
 4 4 4  5b
Then
p
K 21 =
5b 3
15b 2 15b 2 p 3p
K8 = K 21 = 3
= (6.137)
4 4 5b 4b
3 3
− 5b − 5b p −p
K4 = K 21 = 3
=
4 4 5b 4

University of Castilla-La Mancha Draft By: Eduardo W. V. Chaves (2016)


Ciudad Real – Spain
6 LINEAR ELASTICITY 615

The stress σ y ( y ) , (see equation (6.135)), becomes

 − 5b 3 15b 2   − 5b 3 15b 2  p
σ y ( y ) =  + y − 10 y 3  K 21 ⇒ σ y ( y ) =  + y − 10 y 3  3
 2 2   2 2  5b (6.138)
− p 3p 2p
⇒ σ y ( y) = + y − 3 y3
2 2b b
and
 3 2 3 2 p 3 2 p
σ x = 6 K10 y + (20 y − 30 x y ) K 21 = 6 K10 y + (20 y − 30 x y ) 3 = 6 K10 y + (4 y − 6 x y ) 3
 5b b
 − p 3p 2p
σ y = + y − 3 y3
 2 2b b
 2  3p   p  2 − 3p 6p
τ xy = −2 K8 x + 30 K 21 xy = −2  x + 30 3  xy = x + 3 xy 2
 4
 b  5b  2b b
(6.139)
To determine the coefficient K10 we must apply the boundary condition:

L
at ( x = ± ) ⇒
2 ∫σ
A
x ydA = Mz = 0

   L   p 
2

∫ A

M z = σ x ydA = 6 K10 y +  4 y 3 − 6  y  3  ydA = 0
  2   b 
A 
 a b

4p 4 6p L 2  2 2
ab 5 


⇒ 6 K10 y 2 dA + ∫ ∫ y dA = 0 ∫
 y 4 dA =
∫∫ 
2
3
y dA − 3 ∴ y 4 dydz =
b A b 4 A 80 
A A  − a −b 
 2 2 
4 p  ab 5  3 pL2 12 p  ab 3  3 pL2
⇒ 6 K10 I z +  − Iz = 0 ⇒ 6 K10 I z +  − Iz = 0
b 3  80  2b 3 20b  12  2b 3
p pL2  L2 1 
⇒ K10 + − 3 =0 ⇒ K10 = p 3 − 

10b 4b  4b 10b 
Then, the stress field becomes:
 p  L2 1  p
σ x = 6 K10 y + (4 y − 6 x y ) 3 = 3 p 3 −  y + (4 y − 6 x y ) 3
3 2 3 2

 b  2b 5b  b
 − p 3p 2p
σ y = + y − 3 y3 (6.140)
 2 2b b
 −3p 6p 2
τ xy = 2b x + b 3 xy


Note that the following equations is true


ab 3 12 I z 12 I z 12 I z
Iz = ⇒ b3 = ⇒ b2 = ⇒ b=
12 a ab ab 2
Then we can also express the stress field as follows

University of Castilla-La Mancha Draft By: Eduardo W. V. Chaves (2016)


Ciudad Real - Spain
616 SOLVING PROBLEMS BY MEANS OF CONTINUUM MECHANICS

 
 L2 1 
σ x = 3 p −  y + (4 y 3 − 6 x 2 y ) p
 2 12 I z 5 12 I z  12 I z
 2 
 a ab  a
 aL 2
ab 
2
ap
⇒ σ x = 3 p −  y + (4 y 3 − 6 x 2 y )

 24 I z 60 I z  12 I z
pa  L2 b 2  ap
⇒ σx =  − y + (4 y 3 − 6 x 2 y )
 
I z  8 20  12 I z

⇒ σx =
pa
160 I z
20 L2 − 8b 2 y +( ap
12 I z
)
(4 y 3 − 6 x 2 y )

We can restructure the above equation to obtain:

σx =
pa
160 I z
(
20 L2 y −
pa
160 I z
)
8b 2 y +
ap
12 I z
(
(4 y 3 ) − )
ap
12 I z
(6 x 2 y )

⇒ σx =
pa 2
8I z
(
L − 4x2 y +
pa
60 I z
)
20 y 3 − 3b 2 y ( )

− p 3p 2p −p 3p 2p 3
σy = + y − 3 y3 = + y− y
2 2b b 2 12 I 12 Iz
2 2z
ab a
⇒ σy =
pa
24 I z
(
− b 3 + 3b 2 y − 4 y 3 )

− 3p 6p − 3p 6p
τ xy = x + 3 xy 2 = x+ xy 2
2b b 12 I z 12 I z
2 2
ab a
⇒ τ xy =
pa
8I z
(
4 y2 − b2 x )
Then we can also express the stress field as follows, (Sechler (1952)):

σ x =
pa 2
8I z
(L − 4x2 y +
pa
60 I z
)
20 y 3 − 3b 2 y ( )


σ y =
pa
24 I z
(
− b3 + 3b 2 y − 4 y 3 ) (6.141)

 pa
τ xy = (4 y 2 − b 2 ) x
 8I z
Stress Function References
LAIER, J.E. & BARREIRO, J.C. (1983). Complementos de Resistências dos Materiais. Publicação
073/92, São Carlos, Universidade de São Paulo, Escola de Engenharia de São Carlos.
SECHLER, E. (1952). Elasticity in Engineering. John Willey & Sons, Inc. New York.
UGURAL, A.C. & FENSTER, S.K. (1981). Advanced strength and applied elasticity. Edward Arnold,
London - U.K.

University of Castilla-La Mancha Draft By: Eduardo W. V. Chaves (2016)


Ciudad Real – Spain
6 LINEAR ELASTICITY 617

6.3 Introduction to Finite Element for Linear Elasticity


Problems
Problem 6.39
Let us consider a two-dimensional problem such as the infinitesimal strain field, {ε ( x, y )} ,
into the domain ( Ω ) is given by:
 (1) 
 ∂N1 ∂N 2 ∂N 3  u 
 0 0 0   v (1) 
ε x   ∂x ∂x ∂x 
  ∂N1 ∂N 2 ∂N 3  u ( 2 ) 
{ε ( x, y)} = ε y  =  0 0 0   (6.142)
   ∂y ∂y ∂y  v ( 2 ) 
γ xy   ∂N1 ∂N1 ∂N 2 ∂N 2 ∂N 3 ∂N 3  u (3) 

 ∂y ∂x ∂y ∂x ∂y ∂x   (3) 
v 
where u (i ) , v (i ) are the displacements at the nodes ( i = 1,2,3 ) associated with the directions
x and y respectively, and N 1 = N 1 ( x, y ) , N 2 = N 2 ( x, y ) and N 3 = N 3 ( x, y ) are continuous
functions. Check whether the compatibility equation is satisfied or not. Express the
displacement fields ( u ( x, y ) , v( x, y ) ) in terms of nodal displacements {u ( e ) } . Obtain also
the stress field for plane stress and plane strain states.

v (3)
u ( 3)
3
v ( 2)

Ω u (2)
2
y, v
v (1)

x, u 1 u (1)

Figure 6.49: Domain Ω .


Solution:
For two-dimensional problem the compatibility equations, (see Problem 5.11 – NOTE 3),
reduce to
2 2
∂ 2 ε11 ∂ 2 ε 22 ∂ 2 ε12 ∂ 2 ε x ∂ ε y ∂ γ xy
+ − 2 =0 Engineerin
  g  notation
 → + − =0 (6.143)
∂x22 ∂x12 ∂x1∂x2 ∂y 2 ∂x 2 ∂x∂y
By means of equation (6.142) we can obtain:
∂ 2ε x 3
∂ 2  ∂N 1 (1) ∂N 2 ( 2) ∂N 3 (3)  ∂ N 1 (1) ∂ N 2 ( 2) ∂ N 3 (3)
3 3
= 
 u + u + u 
 = u + u + u
∂y 2 ∂y 2  ∂x ∂x ∂x 2
 ∂y ∂x ∂y 2 ∂x ∂y 2 ∂x
∂ 2ε y 3
∂ 2  ∂N 1 (1) ∂N 2 ( 2 ) ∂N 3 (3)  ∂ N 1 (1) ∂ N 2 ( 2 ) ∂ N 3 (3)
3 3
=  v + v + v  = v + v + v
∂x 2 ∂x 2  ∂y ∂y ∂y  ∂y∂x 2
 ∂y∂x 2 ∂y∂x 2

University of Castilla-La Mancha Draft By: Eduardo W. V. Chaves (2016)


Ciudad Real - Spain
618 SOLVING PROBLEMS BY MEANS OF CONTINUUM MECHANICS

∂ 2 γ xy ∂ 2  ∂N 1 (1) ∂N 1 (1) ∂N 2 ( 2 ) ∂N 2 ( 2) ∂N 3 (3) ∂N 3 (3) 


=  u + v + u + v + u + v 
∂x∂y ∂x∂y  ∂y ∂x ∂y ∂x ∂y ∂x 
∂ 3 N1 (1) ∂3N2 ( 2) ∂3 N3 ( 3) ∂ 3 N1 (1) ∂3N2 ( 2) ∂3N3
= 2
u + 2
u + 2
u + 2
v + 2
v + 2
v ( 3)
∂y ∂x ∂y ∂x ∂y ∂x ∂x ∂y ∂x ∂y ∂x ∂y
Then, by substituting the above derivatives into the equation in (6.143) we can conclude
that the compatibility equation is satisfied.
The displacement field can be obtained by means of the normal strain definition, i.e.:

εx =
∂u ∂N1 (1) ∂N 2 ( 2 ) ∂N 3 (3) ∂
=
∂x ∂x
u +
∂x
u +
∂x
u =
∂x
N1u (1) + N 2u ( 2 ) + N 3u (3) =
∂u
∂x
( )
εy =
∂v ∂N1 (1) ∂N 2 ( 2) ∂N 3 (3) ∂
=
∂y ∂y
v +
∂y
v +
∂y
v =
∂y
N1v (1) + N 2v ( 2 ) + N 3v (3) =
∂v
∂y
( )
Thus,
 u (1) 
 (1) 
v 
u ( x, y )   N1 0 N2 0 N3 0  u ( 2 ) 
 =   ⇒ {u( x, y )} = [ N ( x, y )]{u( e ) }
v( x, y )   0 N1 0 N2 0 N 3  v ( 2 ) 
u (3) 
 ( 3) 
 v 
Note also that
 ∂u    u (1) 
 ∂ 
  ∂ 0  0  (1) 
ε x   ∂x   ∂x v 
  ∂x 
   ∂v   ∂  u ( x, y )  ∂   N1 0 N2 0 N3 0  u ( 2 ) 
ε y  =  = 0  = 0   
   ∂y   ∂y  v( x, y )   ∂y   0 N1 0 N2 0 N 3  v ( 2) 
γ xy   ∂u ∂v   ∂ ∂ ∂ ∂ 
u (3) 
  
 +   ∂y ∂x   ∂y ∂x   (3) 
 ∂y ∂x    v 
or in compact form:
{ε ( x, y)} = [ L(1) ]{u( x, y)} = [ L(1) ][ N ( x, y)]{u (e) } = [ B( x, y)]{u (e) } (6.144)
The stress-strain relationship for two-dimensional problem, (see Problem 6.25)), can be
expressed as follows:
   E = E
1 ν
σx  0   εx  if state of plane stress 
  E
ν 1
 
0   εy   ν = ν
σ y  = 1 − ν 2  
τ xy   1−ν    E
  0 0  γ xy ∴   E = (1 − ν 2 ) (6.145)
 2   if state of plane strain 
 
 ν = ν
{σ ( x, y )} = [C ( 2 D ) ]{ε ( x, y )}   (1 − ν )

Then, if we consider the relationship between the strain field {ε ( x, y )} and the nodal
displacement {u(e) } , (see equation (6.144)), we can express the stress in terms of nodal
displacement:
{σ ( x, y )} = [C ( 2 D ) ]{ε ( x, y )} ⇒ {σ ( x, y )} = [C (2 D ) ][ B( x, y )]{u(e) }

University of Castilla-La Mancha Draft By: Eduardo W. V. Chaves (2016)


Ciudad Real – Spain
6 LINEAR ELASTICITY 619

Problem 6.40
Consider a linear elastic problem and let us adopt the linear approximation for the
displacement field {u( x, y )} for two-dimensional problem, (see Figure 6.50):
u ( x, y ) = a1 + a2 x + a3 y
{u( x, y )} =  (6.146)
v( x, y ) = a4 + a5 x + a6 y
where ak ( k = 1,2,3,4,5,6) are constants to be determined.

{u( x, y)}
v (3) 
{ε ( x, y )}
u ( 3) {σ ( x, y )}
( x ( 3) , y ( 3) ) 
3 Nodal displacement
( 2)
v
 u (1) 
 (1) 
u (2) v 
2
u 
( x, y ) ( 2)
( x ( 2) , y (2) ) {u ( e ) } =  ( 2 ) 
Ω v 
v (1)
u (3) 
y 1 t - thickness  ( 3) 
(1)
(x , y ) (1)  v 
A - triangle area
u (1)
V = At - volume
x
Figure 6.50: Domain Ω .

a) Obtain an explicit expression between displacement field ( {u( x, y )} ) and nodal


displacement ( {u(e ) } ), (see Figure 6.50).
b) In Problem 5.23 it was obtained the equation { f ( e ) } = [k ( e ) ]{u (e ) } which is equivalent to
the governing equation for static linear elastic problem, where { f (e ) } stands for nodal
forces whose directions are coincident with the nodal displacement directions. Obtain the
stiffness matrix [k (e ) ] for the problem established here.
Solution:
The displacement field (6.146) can also be expressed as follows:
u ( x, y ) = a1 + a2 x + a3 y
{u( x, y )} = 
v( x, y ) = a4 + a5 x + a6 y
 a1 
a 
 2 (6.147)
1 x y 0 0 0  a3 
⇒ {u( x, y )} =    ⇒ {u( x, y )} = [ X ]{α }
0 0 0 1 x y   a4 
 a5 
 
a6 
The next step is determine the coefficients ak ( k = 1,2,3,4,5,6) , i.e. the vector {α } .
The displacement field given by (6.147) is also valid for the nodes 1, 2 and 3, so:

University of Castilla-La Mancha Draft By: Eduardo W. V. Chaves (2016)


Ciudad Real - Spain
620 SOLVING PROBLEMS BY MEANS OF CONTINUUM MECHANICS

u ( x (1) , y (1) ) ≡ u (1) = a1 + a2 x (1) + a3 y (1)


 (1) (1) (1) (1) (1)
v( x , y ) ≡ v = a4 + a5 x + a6 y
u ( x ( 2 ) , y ( 2 ) ) ≡ u ( 2 ) = a + a x ( 2 ) + a y ( 2 )
1 2 3
 ( 2) ( 2) ( 2) ( 2) ( 2)
 v ( x , y ) ≡ v = a 4 + a5 x + a 6y
u ( x (3) , y (3) ) ≡ u (3) = a + a x (3) + a y (3)
 ( 3) ( 3) 1 2 3
v( x , y ) ≡ v (3) = a4 + a5 x (3) + a6 y (3)

or in matrix form:
 u (1)  1 x (1) y (1) 0 0 0   a1 
 (1)    
 v  0 0 0 1 x (1) y (1)  a2 
u ( 2 )  1 x ( 2 ) y ( 2) 0 0 0  a3 
 ( 2)  =    ⇔ {u (e ) }6×1 = [ A ]6×6 {α }6×1
 v  0 0 0 1 x ( 2) y ( 2 )  a 4 
u (3)  1 x (3) y ( 3) 0 0 0  a5 
 ( 3)    
 v  0 0 0 1 x ( 3) y (3)  a6 
Then, if the inverse of [A ] is known the vector {α } can be determined, i.e.:
{u ( e ) } = [ A ]{α } ⇒ [ A ]−1{u ( e ) } = [ A ]−1[ A ]{α } ⇒ [ A ]−1{u ( e ) } = [1]{α } = {α }
⇒ {α } = [ A ]−1{u ( e ) }
And by substituting the above equation into the displacement field (6.147) we can obtain:
{u( x, y )} = [ X ]{α } ⇒ {u( x, y )} = [ X ][ A ] −1 {u (e ) } = [ N ]{u ( e ) } (6.148)
Note that by definition the shape function relates the function field to the nodal value of the
function, so, we can conclude that the shape functions to approach the displacement field
are:
[ N ]2×6 = [ X ]2×6 [ A ]6−×16 (6.149)
The matrix [ A ]−1 is given by:
 x ( 2 ) y ( 3)   x (3) y (1)   x (1) y ( 2)  
 ( 2 ) ( 3)  0   0   0 
 − y x   − y (3) x (1) 
 
 − y (1) x ( 2 ) 
  
( y ( 2 ) − y ( 3) ) 0 ( y (3) − y (1) ) 0 ( y (1) − y ( 2 ) ) 0 
 
−1 1  ( x ( 3) − x ( 2 ) ) 0 ( x (1) − x (3) ) 0 ( x ( 2 ) − x (1) ) 0 
[A ] =
2A   x ( 2 ) y ( 3)   x (3) y (1)  x y(1) ( 2 )

 0   0   0  
  − y ( 2 ) x ( 3)   − y (3) x (1)   − y (1) x ( 2 )  
      
 0 ( y ( 2 ) − y ( 3) ) 0 ( y (3) − y (1) ) 0 ( y (1) − y ( 2 ) )
 0 (x ( 3)
−x ) ( 2)
0 (x (1)
−x )( 3)
0 ( x ( 2 ) − x (1) ) 

where A is the triangle area. Then, after the matrix multiplication in (6.149) is taken place
we can obtain:
 N1 ( x , y ) 0 N 2 ( x, y ) 0 N 3 ( x, y ) 0 
[N ] = 
 0 N1 ( x, y ) 0 N 2 ( x, y ) 0 N 3 ( x, y )

where

University of Castilla-La Mancha Draft By: Eduardo W. V. Chaves (2016)


Ciudad Real – Spain
6 LINEAR ELASTICITY 621

 1 ( 2) ( 3) ( 3) (2) ( 2 ) ( 3) ( 2 ) ( 3)
 N1 ( x, y ) = 2 A [ x( y − y ) + y ( x − x ) + ( x y − y x )]

 1
 N 2 ( x, y ) = [ x( y (3) − y (1) ) + y ( x (1) − x (3) ) + ( x (3) y (1) − y (3) x (1) )] (6.150)
 2 A
 1 (1) (2) ( 2) (1) (1) ( 2 ) (1) ( 2 )
 N 3 ( x, y ) = 2 A [ x( y − y ) + y ( x − x ) + ( x y − y x )]

In Problem 5.23 we have shown that
{ f ( e ) } = [k ( e ) ]{u (e ) }

where the stiffness matrix [k (e ) ] can be obtained as follows:


r r

[k ( e ) ] = [B( x )]T [C ] [B( x )] dV
V
(6.151)

where the matrix [B] relates strain field to nodal displacements, (see Problem 5.23), i.e.:

 ∂u    u (1) 
 ∂ 
  ∂ 0  0  (1) 
ε x   ∂x   ∂x v 
  ∂x 
   ∂v   ∂  u ( x, y )  ∂   N1 0 N 2 0 N 3 0  u ( 2 ) 
ε y  =  = 0   = 0   
   ∂y   ∂y  v( x, y )   ∂y   0 N1 0 N 2 0 N 3  v ( 2) 
  
γ xy   ∂u ∂v   ∂ ∂
 
∂ ∂
 u (3) 
 +   ∂y ∂x   ∂y ∂x   ( 3) 
 ∂y ∂x   1 444444442 r 444r 444443  v 
(1 )
=[ L ] [ N ( x )]=[B ( x )]
r r r r
or {ε ( x )} = [ L(1) ] {u( x )} = [ L(1) ] [ N ( x )]{u( e ) } = [B( x )] {u( e ) } .
By considering the shape functions (6.150), the matrix [B] becomes:
∂   ∂N1 ∂N 2 ∂N 3 
 0  0 0 0 
 ∂x   ∂x ∂x ∂x 
r ∂   N1 0 N2 0 N3 0  ∂N1 ∂N 2 ∂N 3 
[B( x )] =  0  = 0 0 0
 ∂y   0 N1 0 N2 0 N 3   ∂y ∂y ∂y 
∂ ∂   ∂N ∂N1 ∂N 2 ∂N 2 ∂N 3 ∂N 3 
   1 
 ∂y ∂x   ∂y ∂x ∂y ∂x ∂y ∂x 
where
∂N 1 1 ∂ 1
= [ x( y ( 2 ) − y (3) ) + y ( x (3) − x ( 2 ) ) + ( x (1) y (3) − y ( 2) x (3) )] = ( y ( 2) − y (3) )
∂x 2 A ∂x 2A
∂N 1 1 ∂ 1
= [ x( y ( 2 ) − y (3) ) + y ( x (3) − x ( 2 ) ) + ( x (1) y (3) − y ( 2 ) x (3) )] = ( x ( 3) − x ( 2 ) )
∂y 2 A ∂y 2A
∂N 2 1 ∂ 1
= [ x( y (3) − y (1) ) + y ( x (1) − x (3) ) + ( x (3) y (1) − y (3) x (1) )] = ( y (3) − y (1) )
∂x 2 A ∂x 2A
∂N 2 1 ∂ 1
= [ x( y (3) − y (1) ) + y ( x (1) − x (3) ) + ( x (3) y (1) − y (3) x (1) )] = ( x (1) − x (3) )
∂y 2 A ∂y 2A
∂N 3 1 ∂ 1
= [ x( y (1) − y ( 2 ) ) + y ( x ( 2 ) − x (1) ) + ( x (1) y ( 2 ) − y (1) x ( 2 ) )] = ( y (1) − y ( 2 ) )
∂x 2 A ∂x 2A
∂N 3 1 ∂ 1
= [ x( y (1) − y ( 2 ) ) + y ( x ( 2 ) − x (1) ) + ( x (1) y ( 2 ) − y (1) x ( 2 ) )] = ( x ( 2) − x (1) )
∂y 2 A ∂y 2A

University of Castilla-La Mancha Draft By: Eduardo W. V. Chaves (2016)


Ciudad Real - Spain
622 SOLVING PROBLEMS BY MEANS OF CONTINUUM MECHANICS

Then, the matrix [B] becomes:


 ∂N1 ∂N 2 ∂N 3 
 0 0 0 
 ∂x ∂x ∂x 
∂N1 ∂N 2 ∂N 3 
[B] =  0 0 0
 ∂y ∂y ∂y 
 ∂N ∂N1 ∂N 2 ∂N 2 ∂N 3 ∂N 3 
 1 
 ∂y ∂x ∂y ∂x ∂y ∂x 
 y ( 2 ) − y ( 3) 0 y (3) − y (1) 0 y (1) − y ( 2) 0  (6.152)
1  (1) 
= 0 x − x( 2)
( 3)
0 x − x ( 3)
(1)
0 ( 2)
x −x 
2 A  (3)
x − x
(2)
y ( 2 ) − y ( 3) x (1) − x (3) y (3) − y (1) x ( 2 ) − x (1) y (1) − y ( 2 ) 
 a1 0 a2 0 a3 0
1 
= 0 b1 0 b2 0 b3 
2A 
 b1 a1 b2 a2 b3 a3 
⇒ [B] = [[B1 ]3×2 [B 2 ]3×2 [B3 ]3×2 ] (6.153)
Note that, since the displacement field is linear the matrix [B] is constant into the sub-
domain, and as consequence the strain and stress fields are also constant fields into the
sub-domain. For this reason this triangular sub-domain is called Constant Strain Triangle –
CST.
Then, the stiffness matrix can be obtained as follows:


V

[k ( e ) ]6×6 = [B]T [C ( 2 D ) ] [B] dV = [B]T [C ( 2 D ) ] [B] dV = [B]T6×3 [C ( 2 D ) ]3×3 [B]3×6 At
V (6.154)
123
=V = At

where the matrix [C ( 2 D ) ] for 2D case was obtained in Problem 6.25, i.e.:

   E = E
1 ν
σx  0   εx  if state of plane stress 
  E
ν 1
 
0   εy   ν = ν
σ y  = 1 − ν 2  
τ xy   1−ν    E
  0 0  γ xy ∴   E = (1 − ν 2 ) (6.155)
 2   if state of plane strain 
 
 ν = ν
{σ ( x, y )} = [C ( 2 D ) ]{ε ( x, y )}   (1 − ν )

The stress field can also be expressed in terms of nodal displacements as follows:
{σ ( x, y)} = [C ( 2 D ) ]{ε ( x, y )} = [C (2 D ) ][B( xr )] {u (e) } (6.156)
NOTE 1: Note that we can obtain the explicit form of the stiffness matrix by means of the
matrix multiplications given by equation in (6.154), but in some cases the explicit form of
the stiffness matrix is not so easy to be obtained, then we resort to Numerical Integration (also
called Quadrature) in order to solve numerically the integral (6.151).
The explicit form of (6.154) follows. The matrix [Bi ]3×2 from the equation (6.153) can be
rewritten as follows:

University of Castilla-La Mancha Draft By: Eduardo W. V. Chaves (2016)


Ciudad Real – Spain
6 LINEAR ELASTICITY 623

 ∂N i 
 0 
 ∂x  a i 0
∂N i  1 
[B i ]3×2 = 0 = 0 bi 
 ∂y  2 A 
 ∂N  bi a i 
∂N i 
 i 
 ∂y ∂x 
Then, the equation in (6.154) can be rewritten as follows
[k ( e ) ] = At[B]T6×3 [C ( 2 D ) ]3×3 [B]3×6
 [B1 ]T  
 
T
⇒ [k ] = At  [B2 ] [C
(e) (2 D)
] [[B1 ] [B2 ] [B3 ]] 
 
 [B3 ]T  
  
 [B1 ]T [C ( 2 D ) ]  
  
⇒ [k ( e ) ] = At  [B2 ]T [C ( 2 D ) ] [[B1 ] [B2 ] [B3 ]] 
 
 [B3 ]T [C ( 2 D ) ] 
   
[B1 ] [C
T (2 D) T
][B1 ] [B1 ] [C (2 D)
][B2 ] [B1 ]T [C ( 2 D ) ][B3 ] 
 
⇒ [k ( e ) ] = At [B2 ]T [C ( 2 D ) ][B1 ] [B2 ]T [C ( 2 D ) ][B2 ] [B2 ]T [C ( 2 D ) ][B3 ]
[B ]T [C ( 2 D ) ][B ] [B ]T [C ( 2 D ) ][B ] [B ]T [C ( 2 D ) ][B ]
 3 1 3 2 3 3 

⇒ [k ( e ) ] = At[[Bi ]T [C ( 2 D ) ][B j ]] (i, j = 1,2,3)

Let us consider the elasticity tensor components for 2D case:


 C11 C12 0 
[C (2D)
] =  C12 C22 0 
 0 0 C33 
Then, we can obtain:
 C11 C12 0  a j 0
1  ai 0 bi     1
T
[Bi ] [C ][B j ] =  C12 C22 0   0 bj  = [k ij ]
4 A2  0 bi ai   4 A2
 0 0 C33  b j aj

where
 ai C11a j + bi C33b j ai C12b j + bi C33a j 
[k ij ]2×2 = 
bi C12 a j + ai C33b j bi C22b j + ai C33a j 

Then, the stiffness matrix becomes


k11( e ) k12( e ) k13(e ) k14( e ) k15( e ) k16(e ) 
 
 k 22( e ) k 23(e ) k 24( e ) k 25( e ) k 26(e ) 
[k 11 ] [k 12 ] [k 13 ]
t   t  k33(e ) k 34( e ) k 35( e ) k 36(e ) 
[k ( e ) ]6×6 = [ k
 21 ] [ k 22 ] [ k ]
23  =   (6.157)
4A   4A  k 44( e ) k 45( e ) k 46(e ) 
[ k ] [ k ] [ k 33 ]  symmetric
 31 k 56(e ) 
32
k 55( e )
 
 k 66(e ) 

where

University of Castilla-La Mancha Draft By: Eduardo W. V. Chaves (2016)


Ciudad Real - Spain
624 SOLVING PROBLEMS BY MEANS OF CONTINUUM MECHANICS

k11(e ) = a12 C11 + b12 C33 ; k12( e ) = a1C12b1 + b1C33 a1 ; k13( e ) = a1C11a2 + b1C33b2 ;
k14(e ) = a1C12b2 + b1C33 a2 ; k15( e ) = a1C11a3 + b1C33b3 ; k16( e ) = a1C12b3 + b1C33 a3 ;
k 22(e ) = b12 C22 + a12 C33 ; k 23( e ) = b1C12 a2 + a1C33b2 ; k 24( e ) = b1C22b2 + a1C33 a2 ;
k 25(e ) = b1C12 a3 + a1C33b3 ; k 26( e ) = b1C22b3 + a1C33 a3 ; k33( e ) = a22 C11 + b22 C33 ; (6.158)
k34(e ) = a2 C12b2 + b2 C33 a2 ; k35( e ) = a2 C11a3 + b2 C33b3 ; k36( e ) = a2 C12b3 + b2 C33a3 ;
k 44(e ) = b22 C22 + a22 C33 ; k 45( e ) = b2 C12 a3 + a2 C33b3 ; k 46( e ) = b2 C22b3 + a2 C33 a3 ;
k55(e ) = a32 C11 + b32 C33 ; k56( e ) = a3 C12b3 + b3 C33 a3 ; k 66( e ) = b32 C22 + a32 C33 ;
with
a1 = y ( 2 ) − y (3) ; a2 = y (3) − y (1) ; a3 = y (1) − y ( 2 ) ;
(6.159)
b1 = x (3) − x ( 2) ; b2 = x (1) − x (3) ; b3 = x ( 2 ) − x (1) .

NOTE 2: The shape functions for Strain Constant Triangle can be appreciated in Figure
6.51. Note that the shape function N1 at node 1 has the value equal to 1 and assumes zero
for the remaining nodes:
1
N1 ( x, y ) = [ x ( y ( 2 ) − y (3) ) + y ( x (3) − x ( 2 ) ) + ( x ( 2 ) y (3) − y ( 2 ) x (3) )]
2A
1 (1) ( 2 )
( x = x (1) , y = y (1) ) ⇒ N1 ( x (1) , y (1) ) = [ x ( y − y (3) ) + y (1) ( x (3) − x ( 2 ) ) + ( x ( 2) y (3) − y ( 2 ) x (3) )] = 1
2A
And the summation of the shape functions must be equal to 1, i.e. N1 + N 2 + N 3 = 1 .

3
2

1


N1 1 

 N1 + N 2 + N 3 = 1
+ 

1 

N2 
 1


=
+ 



N3 





Figure 6.51: Shape functions for triangle.

University of Castilla-La Mancha Draft By: Eduardo W. V. Chaves (2016)


Ciudad Real – Spain
6 LINEAR ELASTICITY 625

NOTE 3: Another way to obtain the shape function follows. Let us consider only the
displacement according to x -direction:
 a1 
 
u ( x, y ) = a1 + xa2 + ya3 = [1 x y ]a2  (6.160)
a 
 3
and its nodal values
u (1) = a1 + x (1) a2 + y (1) a3   u (1)  1 x (1) y (1)   a1 
  (2)    
u ( 2) = a1 + x ( 2 ) a2 + y ( 2 ) a3  ⇒ u  = 1 x
( 2)
y ( 2 )   a2 
 u (3)  1 x (3) y (3)  a3 
u (3) = a1 + x (3) a2 + y (3) a3    
In Problem 1.16 we have used the Cramer’s rule to obtain the solution for the above set of
equations, i.e.:
u (1) x (1) y (1) 1 u (1) y (1) 1 x (1) u (1)
u ( 2) x( 2) y ( 2) 1 u ( 2) y (2) 1 x ( 2) u (2)
u ( 3) x ( 3) y ( 3) 1 u (3) y ( 3) 1 x (3) u ( 3)
a1 = ; a2 = ; a3 =
1 x (1) y (1) 1 x (1) y (1) 1 x (1) y (1)
1 x (2) y (2) 1 x (2) y (2) 1 x( 2) y ( 2)
1 x ( 3) y ( 3) 1 x ( 3) y ( 3) 1 x ( 3) y ( 3)

where • ≡ det (•) stands for the determinant of • . Note that

1 x (1) y (1)  1 x (1) y (1)  


 
1 x( 2) y ( 2 ) ≡ det 1 x ( 2 ) y ( 2)   = 2! A = 2 A
 
1 x ( 3) y ( 3)  1 x ( 3) y (3)  
 
where A is the triangle area. Note also that
u (1) x (1) y (1)
x( 2) y ( 2) ( 2) x
(1)
y (1) ( 3) x
(1)
y (1)
u ( 2) x ( 2) y ( 2 ) = u (1) (3) − u + u
x y ( 3) x ( 3) y ( 3) x( 2) y (2)
u ( 3) x ( 3) y ( 3)

1 u (1) y (1)
1 y (2) ( 2) 1 y (1) ( 3) 1 y (1)
1 u ( 2) y ( 2 ) = −u (1) + u − u
1 y ( 3) 1 y ( 3) 1 y (2)
1 u ( 3) y ( 3)

1 x (1) u (1)
1 x( 2) ( 2) 1 x
(1)
( 3) 1 x (1)
1 x (2) u ( 2 ) = u (1) − u + u
1 x ( 3) 1 x ( 3) 1 x (2)
1 x ( 3) u ( 3)

Then, the displacement field given by the equation in (6.160) can be expressed as follows:

University of Castilla-La Mancha Draft By: Eduardo W. V. Chaves (2016)


Ciudad Real - Spain
626 SOLVING PROBLEMS BY MEANS OF CONTINUUM MECHANICS

u ( x, y ) = a1 + xa2 + ya3
1 x (1) y (1)
x ( 2) y ( 2) x (1) y (1) x (1) y (1)
⇒ 1 x( 2) y ( 2 ) u ( x, y ) = u (1) (3) − u ( 2) + u ( 3) +
x y ( 3) x ( 3) y ( 3) x (2) y ( 2)
1 x ( 3) y ( 3)
 1 y ( 2) (2) 1 y (1) ( 3) 1 y (1)   (1) 1 x ( 2 ) ( 2) 1 x
(1)
( 3) 1 x (1) 
x − u (1) + u − u  + y u − u + u 
 1 y ( 3) 1 y ( 3) 1 y (2)   1 x ( 3) 1 x ( 3) 1 x( 2) 
   

1 x (1) y (1)
 x( 2) y (2) 1 y ( 2) 1 x ( 2) 
⇒ 1 x( 2) y ( 2 ) u ( x, y ) = u (1)  (3) −x +y +
x y ( 3) 1 y ( 3) 1 x (3) 
1 x ( 3) y ( 3) 
(6.161)
 x (1) y (1) 1 y (1) 1 x (1)  (3)  x (1) y (1) 1 y (1) 1 x (1) 
u ( 2 )  − ( 3) + x − y +u  − x + y 
 x y ( 3) 1 y ( 3) 1 x ( 3)   x( 2) y ( 2) 1 y (2) 1 x (2) 
   
Note also that
1 x y
 x( 2)
(1)  y ( 2) 1 y ( 2) 1 x( 2) 
u 1 ( 3) − x + y  = u 1 x ( 2)
(1)
y ( 2)
 x y ( 3) 1 y ( 3) 1 x ( 3) 
  1 x ( 3) y ( 3)

1 x (1) y (1)
 x (1) y (1) 1 y (1) 1 x (1) 
u ( 2)  − 1 (3) + x − y  = u (2) 1 x y
 x y ( 3) 1 y ( 3) 1 x ( 3) 
  1 x ( 3) y ( 3)

1 x (1) y (1)
 x (1) y (1) 1 y (1) 1 x (1) 
u ( 3)  ( 2 ) − x + y  = u ( 3) 1 x ( 2 ) y ( 2)
x y ( 2) 1 y ( 2) 1 x( 2) 
  1 x y

Then, the displacement field (6.161) can also be expressed as follows:


1 x y 1 x (1) y (1) 1 x (1) y (1)
1 x (2) y (2) 1 x y 1 x( 2) y ( 2)
1 x ( 3) y ( 3) 1 x ( 3) y ( 3) 1 x y
⇒ u ( x, y ) = u (1) (1) (1)
+ u (2) (1) (1)
+ u ( 3)
1 x y 1 x y 1 x (1) y (1)
1 x (2) y (2) 1 x (2) y (2) 1 x( 2) y ( 2)
1 x ( 3) y ( 3) 1 x ( 3) y ( 3) 1 x ( 3) y ( 3)

with that we can define the shape functions as follows:

1 x y 1 x (1) y (1) 1 x (1) y (1)


1 x ( 2) y ( 2) 1 x y 1 x( 2) y ( 2)
r 1 x ( 3) y ( 3) r 1 x ( 3) y ( 3) r 1 x y
N1 ( x ) = , N2 ( x ) = , N3 ( x ) = (6.162)
1 x(1) y (1) 1 x (1) y (1) 1 x (1) y (1)
1 x ( 2) y ( 2) 1 x(2) y (2) 1 x( 2) y ( 2)
1 x ( 3) y ( 3) 1 x ( 3) y ( 3) 1 x ( 3) y ( 3)

University of Castilla-La Mancha Draft By: Eduardo W. V. Chaves (2016)


Ciudad Real – Spain
6 LINEAR ELASTICITY 627

Note that the following is also true:


1 x y 0 1 0

1 x (2) y ( 2)
1 x ( 2)
y (2)
∂x
∂N1 1 x ( 3) y ( 3) 1 x ( 3) y ( 3) y ( 2 ) − y ( 3)
= = =
∂x 1 x (1) y (1) 1 x (1) y (1) 2A
1 x( 2) y (2) 1 x (2) y ( 2)
1 x ( 3) y ( 3) 1 x ( 3) y ( 3)

1 x y 0 0 1

1 x ( 2) y ( 2)
1 x (2)
y ( 2)
∂y
∂N1 1 x ( 3) y ( 3) 1 x ( 3) y (3) x ( 3) − x ( 2 )
= = =
∂y 1 x (1) y (1) 1 x (1) y (1) 2A
1 x ( 2) y ( 2) 1 x ( 2) y ( 2)
1 x ( 3) y ( 3) 1 x (3) y ( 3)

and so on. Note that it is also true that

1 x y ∫ dA ∫ xdA ∫ ydA
1 x( 2) y (2)
∫ 1 x ( 2) y ( 2 ) dA
1 x ( 3) y ( 3)
1 x (3) y ( 3)
∫ N dA =
1
1 x (1) y (1)
=
1 x (1) y (1)
1 x (2) y ( 2) 1 x (2) y (2)
1 x ( 3) y ( 3) 1 x ( 3) y ( 3)

NOTE 3.1: The procedure used to obtain the shape functions given by equations in
(6.162) can be extrapolated in order to obtain the shape functions for other elements. For
example, let us consider the tetrahedron element with 4 nodes, (see Figure 6.52), in which
r
u ( x ) ≡ u ( x, y, z ) = a1 + xa2 + ya3 + za4 (linear function).

w( 4 )  u (1)  1 x (1) y (1) z (1)   a1 


v ( 4)
 (2)    
u  1 x (2) y (2) z ( 2 )  a 2 
4 (l ) ( 4)  ( 3)  =   
z ( 3 )   a3 
u
u  1 x ( 3) y ( 3)

u ( 4 )  1 x (4) y (4) z ( 4 )  a4 
w( 3) ( 3)
  
v
w(1)
(1)
v
u (3)
1 (i) u (1) 3 (k )
z, w w( 2 )
v ( 2)
y, v
u ( 2)
2 ( j)
x, u

Figure 6.52: Tetrahedron with 4 nodes.

University of Castilla-La Mancha Draft By: Eduardo W. V. Chaves (2016)


Ciudad Real - Spain
628 SOLVING PROBLEMS BY MEANS OF CONTINUUM MECHANICS

By analogy with the shape functions (6.162) and by considering the relationship between
the nodal displacements and the coefficients ai , (see Figure 6.52), the shape functions can
be obtained as follows:

1 x y z 1 x (1) y (1) z (1) 1 x (1) y (1) z (1)


1 x( 2) y ( 2) z ( 2) 1 x y z 1 x( 2) y ( 2) z ( 2)
1 x ( 3) y ( 3) z ( 3) 1 x ( 3) y ( 3) z ( 3) 1 x y z
( 4) ( 4) ( 4) ( 4) ( 4) ( 4)
1 x y z 1 x( 4) y ( 4) z ( 4) 1 x y z
N1 = (1) (1) (1)
, N2 = (1) (1) (1)
, N3 = (1) (1)
,
1 x y z 1 x y z 1 x y z (1)
1 x( 2) y ( 2) z ( 2) 1 x( 2) y ( 2) z ( 2) 1 x( 2) y ( 2) z ( 2)
1 x ( 3) y ( 3) z ( 3) 1 x ( 3) y ( 3) z ( 3) 1 x ( 3) y ( 3) z ( 3)
1 x( 4) y ( 4) z ( 4) 1 x( 4) y ( 4) z ( 4) 1 x( 4) y ( 4) z ( 4)
(6.163)
1 x (1) y (1) z (1)
1 x ( 2) y ( 2) z (2)
1 x (3) y (3) z ( 3) 1 x (1) y (1) z (1)
1 x y z 1 x( 2) y ( 2) z ( 2)
N4 = where = 3!V = 6V
1 x (1)
y (1)
z (1) 1 x ( 3) y ( 3) z (3)
1 x ( 2) y ( 2) z (2) 1 x( 4) y ( 4) z ( 4)
1 x (3) y (3) z ( 3)
1 x ( 4) y ( 4) z (4)

where V stands for the tetrahedron volume. Then, the displacement field becomes:
u ( x, y, z ) = N1u (1) + N 2u ( 2 ) + N 3u (3) + N 4u ( 4)
r r
By considering the same approximation for the fields v( x ) and w( x ) , we can obtain:
 u (1) 
 (1) 
v 
 w(1) 
 (2) 
u 
 v ( 2) 
 u ( x, y, z )   N1 0 0 N2 0 0 N3 0 0 N4 0 0   (2) 
   w 
 v ( x, y , z )  =  0 N1 0 0 N2 0 0 N3 0 0 N4 0   (3)  (6.164)
w( x, y, z )  0 u
   0 N1 0 0 N2 0 0 N3 0 0 N 4   (3) 
v 
 ( 3) 
w 
 u (4) 
 
 v ( 4) 
 (4) 
w 
r r
{u( x )} = [ N ( x )]{u(e ) } (6.165)
In order to construct the polynomial we can resort to the Pascal’s polynomial in 3D, (see
Figure 6.53).

University of Castilla-La Mancha Draft By: Eduardo W. V. Chaves (2016)


Ciudad Real – Spain
6 LINEAR ELASTICITY 629

1 constant term

x z linear terms

y
x2 quadratic terms
z2
xz
xy yz
2
y
x3 z3 cubic terms
2
x z xz 2
x2 y yz 2

xy 2 y2z
y3

Figure 6.53: Pascal’s polynomial for 3D.


r
NOTE 3.2: In NOTE 3 we have obtained the shape functions in the Cartesian system ( x )
for the triangle using linear function. We can also obtain the shape functions in another
system. For example, let us obtain the shape functions for the triangle described in Figure
6.54. Then, by using the definition in equation (6.162) we can obtain:
1 ξ η 1 ξ η 1 0 0 1 0 0
1 ξ ( 2) η ( 2)
1 1 0 1 ξ η 1 1 0
r 1 ξ ( 3) η ( 3)
1 0 1 r 1 0 1 r 1 ξ η
N1 (ξ ) = = = 1 − ξ − η ; N 2 (ξ )= = ξ ; N 3 (ξ ) = =η
1 ξ (1) η (1) 1 0 0 1 0 0 1 0 0
1 ξ ( 2) η ( 2) 1 1 0 1 1 0 1 1 0
1 ξ ( 3) η ( 3) 1 0 1 1 0 1 1 0 1

Let us adopt another nomenclature such as L1 = 1 − ξ − η , L2 = ξ , L3 = η , which is known


as Area Coordinates, (see NOTE 5 – Area Coordinates).

η = ξ2
L1 = 1 − ξ − η
(ξ ( 3)= 0,η ( 3)= 1) L2 = ξ

3 L3 = η

1 2 ξ = ξ1
(ξ = 0,η = 0)
(1) (1)
(ξ ( 2)= 1,η ( 2)= 0)

Figure 6.54: Triangle element (3 nodes) – Normalized space.

University of Castilla-La Mancha Draft By: Eduardo W. V. Chaves (2016)


Ciudad Real - Spain
630 SOLVING PROBLEMS BY MEANS OF CONTINUUM MECHANICS

r
Now, we will consider the displacement field u ( x ) which is approached by a quadratic
function, (see Figure 6.55):
r
u ( x ) ≡ u ( x, y, z ) = a1 + xa2 + ya3 + x 2 a4 + xya5 + y 2 a6
The polynomial can be easily obtained by means of Pascal’s polynomial, (see Figure 6.42).
Note also that to define the coefficients ai (i = 1,2,...,6) we will need to define 6 nodes. The
nodal values by using the above polynomial become:
 u (1)  1 y (1)   a1 
2 2
x (1) y (1) x (1) x (1) y (1)
 ( 2)   2 2  
u  1 x( 2) y ( 2) x ( 2) x ( 2) y ( 2 ) y ( 2 )   a2 
u (3)  1 x ( 3) y ( 3) x ( 3) 2
x ( 3) y ( 3)
2
y (3)  a3 
 ( 4)  =  ( 4) 2 2   ⇔ {u} = [ H ]{a}
u  1 x( 4) y ( 4) x x ( 4) y ( 4 ) y ( 4 )   a4 
u (5)  1 x ( 5) y ( 5) x ( 5)
2
x ( 5) y ( 5 )
2
y (5)  a5 
 (6)    
u  1 x ( 6) y ( 6) x(6)
2
x ( 6) y ( 6 ) y (6 )  a6 
2

Nodal displacement
v (3)  u (1) 
 (1) 
u ( 3) v 
( x ( 3) , y ( 3) ) u ( 2 ) 
3
v ( 5)  (2) 
v 
u (5) u (3) 
v ( 6) 5  
v ( 2) (e)  v ( 3) 
( x ( 5) , y ( 5) ) {u } =  ( 4 ) 
u (6) u 
( x (6) , y (6) )
6 u (2) v( 4) 
v ( 4) 2
 ( 5) 
(4)
( x ( 2) , y (2) ) u 
4 u  v ( 5) 
v (1)  
y 1 ( x (4) , y (4) ) u ( 6) 
 (6) 
( x (1) , y (1) ) v 
u (1)
x
Figure 6.55: Triangle domain – quadratic function.

r
Then, the shape function N1 ( x ) can be obtained as follows:
1 x y x2 xy y2
2 2
1 x( 2) y ( 2) x (2) x ( 2 ) y ( 2) y ( 2)
2 2
1 x ( 3) y ( 3) x ( 3) x ( 3) y ( 3) y (3)
2 2
1 x( 4) y ( 4) x (4) x ( 4 ) y ( 4) y ( 4)
2 2
1 x ( 5) y ( 5) x ( 5) x ( 5) y ( 5) y ( 5)
2 2
r 1 x ( 6) y (6) x(6) x ( 6 ) y ( 6) y (6)
N1 ( x ) =
H

University of Castilla-La Mancha Draft By: Eduardo W. V. Chaves (2016)


Ciudad Real – Spain
6 LINEAR ELASTICITY 631

Now, let us consider the normalized space, (see Figure 6.54).

η = ξ2
N1 = (2 L1 − 1) L1
(ξ ( 3)= 0,η ( 3)= 1)
N 2 = (2 L2 − 1) L2
3 N 3 = (2 L3 − 1) L3
N 4 = 4 L1 L2
(ξ (5)= 12 ,η ( 5)= 12 ) N 5 = 4 L2 L3
(ξ ( 6)= 0,η ( 6)= 12 )
6 5 N 6 = 4 L1 L3

∑N a =1
1 4 2 ξ = ξ1 a =1

(ξ (1)= 0,η (1)= 0) (ξ ( 4)= 12 ,η ( 4)= 0) (ξ ( 2)= 1,η ( 2)= 0)

Figure 6.56: Triangle element (6 nodes) – Normalized space.


For this case the matrix [H ] becomes
1 ξ (1) η (1) ξ (1)
2
ξ (1)η (1) η (1)  1
2
0 0 0 0 0 
 2 
1 ξ ( 2) η (2) ξ (2)
2
ξ ( 2)η ( 2) η ( 2)  1 1 0 1 0 0 
 2
r η (3)  = 1 1 
2
1 ξ ( 3) η ( 3) ξ ( 3) ξ (3)η (3) 0 1 0 0
[ H (ξ )] =  
1 ξ ( 4) η (4) ξ (4)
2
ξ ( 4)η ( 4) η ( 4)  1
2 1
2
0 (12 )2 0 0 

1 ξ ( 5) η ( 5) ξ ( 5)
2
ξ (5)η (5)
2
η (5)  1
 1
2
1
2
(12 )2 1 1
2 2
(12 )2 

1 ξ ( 6) η (6) ξ (6)
2
ξ ( 6)η ( 6) η (6)  1
2
0 1
2
0 0 (12 )2 
1
⇒ det ([ H ]) ≡ H =
64
r
then, the shape function N1 (ξ ) can be obtained as follows:
1 ξ η ξ2 ξη η2
1 1 0 1 0 0
r 1 0 1 0 0 1
N1 (ξ ) = 64 = (2ξ + 2η − 1)(ξ + η − 1) = (2 L1 − 1) L1
1 1
2
0 (12 )2 0 0 1424 43 4 1424
= − ( 2 L1 −1)
3
= − L1
1 1
2
1
2
(12 )2 1 1
2 2
(12 )2
1 0 1
2
0 0 (12 )2
And is easy to show that N 2 = (2 L2 − 1) L 2 and N 3 = (2 L3 − 1) L3 .

University of Castilla-La Mancha Draft By: Eduardo W. V. Chaves (2016)


Ciudad Real - Spain
632 SOLVING PROBLEMS BY MEANS OF CONTINUUM MECHANICS

The shape function N 4 becomes:


1 0 0 0 0 0
1 1 0 1 0 0
1 0 1 0 0 1
N 4 = 64 ξ (1
= 4{ 1 − ξ − η ) = 4 L1 L2
1 ξ η ξ 2
ξη η2 424 3
= L2
(12 )2 (12 )2
= L1
1 1 1 1
1 2 2 2 2
1 0 1
2
0 0 (12 )2
And is easy to show that N 5 = 4 L2 L3 and N 6 = 4 L1 L3 .
Summarizing
N1 = (2 L1 − 1) L1 ; N 2 = (2 L2 − 1) L2 ; N 3 = (2 L3 − 1) L3
(6.166)
N 4 = 4 L1L2 ; N 5 = 4 L2 L3 ; N 6 = 4 L1L3

NOTE 3.3: Another example: let us consider the quadrangular element, (see Figure 6.57),
in which the displacement field can be approached by the function:
u ( x, y ) = a1 + a 2 x + a 3 y + a 4 xy

v( x, y ) = a 5 + a 6 x + a 7 y + a 8 xy

v (3)
Nodal displacement
v ( 4) ( x ( 3) , y ( 3) )
u ( 3) u (1) 
3  (1) 
( x ( 4) , y ( 4) )
u (4) v 
4 u ( 2 ) 
v ( 2)  ( 2) 
Ω (e ) v 
{u } =  (3) 
2 u (2) u 
v (1) ( x, y )  v ( 3) 
( x ( 2) , y (2) )  (4) 
1 u 
y v ( 4) 
( x (1) , y (1) )  
u (1)

Figure 6.57: Quadrilateral element.

For the displacement field u ( x, y ) = a1 + a2 x + a3 y + a4 xy we can obtain the nodal


displacements:
 u (1)  1 x (1) y (1) x (1) y (1)   a1 
 (2)    
u  1 x (2) y ( 2) x ( 2 ) y ( 2 )  a 2 
 ( 3)  =   
u  1 x ( 3) y ( 3) x ( 3 ) y ( 3 )   a3 

u ( 4 )  1 x (4) y ( 4) x ( 4 ) y ( 4 )  a4 
  
Then, the shape functions can be obtained as follows:

University of Castilla-La Mancha Draft By: Eduardo W. V. Chaves (2016)


Ciudad Real – Spain
6 LINEAR ELASTICITY 633

1 x y xy 1 x (1) y (1) x (1) y (1)


1 x( 2) y (2) x ( 2 ) y ( 2) 1 x y xy
1 x ( 3) y ( 3) x ( 3) y ( 3) 1 x ( 3) y ( 3) x ( 3) y ( 3)
1 x( 4) y (4) x ( 4 ) y ( 4) 1 x ( 4) y ( 4) x (4) y ( 4)
N1 = ; N2 = ;
1 x (1) y (1) x (1) y (1) 1 x (1) y (1) x (1) y (1)
1 x( 2) y (2) x ( 2 ) y ( 2) 1 x ( 2) y ( 2) x (2) y ( 2)
1 x ( 3) y ( 3) x ( 3) y ( 3) 1 x ( 3) y ( 3) x ( 3) y ( 3)
1 x( 4) y (4) x ( 4 ) y ( 4) 1 x ( 4) y ( 4) x (4) y ( 4)
(6.167)
1 x (1) y (1) x (1) y (1) 1 x (1) y (1) x (1) y (1)
1 x ( 2) y ( 2) x( 2) y (2) 1 x (2) y (2) x ( 2) y ( 2 )
1 x y xy 1 x ( 3) y ( 3) x (3) y (3)
1 x ( 4) y ( 4) x( 4) y (4) 1 x y xy
N3 = (1) (1) (1) (1)
; N4 = (1) (1)
1 x y x y 1 x y x y (1)
(1)

1 x ( 2) y ( 2) x( 2) y (2) 1 x (2) y (2) x ( 2) y ( 2 )


1 x ( 3) y ( 3) x (3) y (3) 1 x ( 3) y ( 3) x (3) y (3)
1 x ( 4) y ( 4) x( 4) y (4) 1 x (4) y (4) x ( 4) y ( 4 )
Then
 u (1) 
 (1) 
v 
u ( 2 ) 
 
u ( x, y )  N 1 0 N2 0 N3 0 N4 0  v ( 2) 
 =   ⇔ {u( x, y )} = [ N ]{u ( e ) }
 v ( x, y )   0 N1 0 N2 0 N3 0 N 4  u (3) 
 v ( 3) 
 (4) 
u 
v ( 4) 
 
Let us consider a particular case, the regular quadrilateral (rectangle), (see Figure 6.58).
In this case the shape functions become:
1 x y xy 1 0 0 0
1 a 0 0 1 x y xy
1 a b ab 1 a b ab
1 0 b 0 1 1 0 b 0 1
N1 = = (ab − bx − ay + xy ) ; N2 = = (bx − xy ) ;
1 0 0 0 ab 1 0 0 0 ab
1 a 0 0 1 a 0 0
1 a b ab 1 a b ab
1 0 b 0 1 0 b 0

University of Castilla-La Mancha Draft By: Eduardo W. V. Chaves (2016)


Ciudad Real - Spain
634 SOLVING PROBLEMS BY MEANS OF CONTINUUM MECHANICS

1 0 0 0 1 0 0 0
1 a 0 0 1 a 0 0
1 x y xy 1 a b ab
1 0 b 0 1 1 x y xy 1
N3 = = xy ; N4 = = (ay − xy)
1 0 0 0 ab 1 0 0 0 ab
1 a 0 0 1 a 0 0
1 a b ab 1 a b ab
1 0 b 0 1 0 b 0
where
1 0 0 0
1 a 0 0
= −(ab)2
1 a b ab
1 0 b 0

y Shape functions
v (3) 1
v ( 4) N1 = (ab − bx − ay + xy)
ab
( x ( 4) = 0, y ( 4) = b) u (4) ( x (3) = a, y ( 3) = b) u ( 3) 1
N2 = (bx − xy )
4 3 ab
1
N3 = xy
ab
b 1
Ω N4 = (ay − xy)
v (1) ab
( x, y ) v ( 2)
1 2 x 4
u (2)
∑N a =1
( x(1) = 0, y (1) = 0) u (1) ( x ( 2) = a, y ( 2) = 0) a =1

Figure 6.58: Rectangle element – shape functions.

We can apply the same procedure to obtain the shape functions in the normalized space,
(see Figure 6.59). Then, if we replace ( x, y ) by (ξ ,η ) in the shape functions (6.167) we can
obtain:

University of Castilla-La Mancha Draft By: Eduardo W. V. Chaves (2016)


Ciudad Real – Spain
6 LINEAR ELASTICITY 635

1 ξ η ξη 1 ξ η ξη
1 ξ ( 2)
η ( 2) ξ η
( 2) (2)
1 1 −1 −1
1 ξ η
( 3) ( 3)
ξ η
( 3) ( 3)
1 1 1 1
1 ξ ( 4)
η ( 4) x η
( 4) ( 4 )
1 −1 1 −1 1
N1 = = = (1 − ξ )(1 − η ) ;
1 ξ (1) η (1) ξ (1)η (1) 1 −1 −1 1 4
1 ξ ( 2) η ( 2) ξ η
( 2) (2) 1 1 −1 −1
1 ξ ( 3) η ( 3) ξ η
( 3) ( 3) 1 1 1 1
1 ξ ( 4) η ( 4) ξ η
( 4) (4) 1 −1 1 −1
1 −1 −1 1 1 −1 −1 1
1 ξ η ξη 1 1 −1 −1
1 1 1 1 1 ξ η ξη
1 −1 1 −1 1 1 −1 1 −1 1
N2 = = (1 + ξ )(1 − η ) ; N 3 = = (1 + ξ )(1 + η ) ;
1 −1 −1 1 4 1 −1 −1 1 4
1 1 −1 −1 1 1 −1 −1
1 1 1 1 1 1 1 1
1 −1 1 −1 1 −1 1 −1

1 −1 −1 1
1 1 −1 −1
1 1 1 1 1 −1 −1 1
1 ξ η ξη 1 1 1 −1 −1
N4 = = (1 − ξ )(1 + η ) where = −16 (6.168)
1 −1 −1 1 4 1 1 1 1
1 1 −1 −1 1 −1 1 −1
1 1 1 1
1 −1 1 −1

v ( 4) ξ 2 ≡η v (3) 1
N1 = (1 − ξ )(1 − η )
4
(ξ ( 4)= −1,η ( 4)= 1) u (4) (ξ ( 3)= 1,η ( 3)= 1) u ( 3) 1
N 2 = (1 + ξ )(1 − η )
4 3 4
1
ξ1 ≡ ξ N 3 = (1 + ξ )(1 + η )
4
v ( 2) 1
N 4 = (1 − ξ )(1 + η )
v (1)
1 2 4
u ( 2)
4
(ξ (1)= −1,η (1)= −1) u (1)
(ξ = 1,η = −1)
( 2) (2) ∑N
a =1
a =1

Figure 6.59: Rectangle element – normalized space.

University of Castilla-La Mancha Draft By: Eduardo W. V. Chaves (2016)


Ciudad Real - Spain
636 SOLVING PROBLEMS BY MEANS OF CONTINUUM MECHANICS

NOTE 3.4: Another example: let us consider a one-dimensional case in which the
displacement field is approach by a quadratic function ( u ( x) = a1 + a2 x + a3 x 2 ), so we will
need three points in order to define the quadratic function, (see Figure 6.60). Next, we will
obtain the shape functions Ni (x) in order to express the displacement field:
u ( x) = N1u (1) + N 2u ( 2 ) + N 3u (3)

Nodal displacement
x u (1) 
 
{u ( e ) } = u ( 2) 
1 u (1) 2 u (2) 3 u ( 3) u (3) 
 
( x (1) ) ( x( 2) ) ( x( 3) )

Figure 6.60: 1D element – quadratic function (generic system).

 u (1)  1 x (1) x (1)   a1 


2

 ( 2)   2  
u ( x) = a1 + a2 x + a3 x 2 Nodal
 val
ues
 → u  = 1 x
( 2)
x ( 2 )  a2 
u (3)  1 x (3) x (3)  a3 
2
   
And the shape functions become:
2 2
1 x x2 1 x (1) x (1) 1 x (1) x (1)
2 2
1 x ( 2) x (2) 1 x x2 1 x ( 2) x (2)
2 2
1 x ( 3) x ( 3) 1 x ( 3) x ( 3) 1 x x2
N1 ( x) = 2
; N 2 ( x) = 2
; N 3 ( x) = 2
(6.169)
1 x (1) x (1) 1 x (1) x (1) 1 x (1) x (1)
2 2 2
1 x ( 2) x (2) 1 x (2) x ( 2) 1 x ( 2) x (2)
2 2 2
1 x ( 3) x ( 3) 1 x ( 3) x ( 3) 1 x ( 3) x ( 3)

Let us consider a particular case which is described in Figure 6.61. For this particular case
the shape functions N i (x) are:
1 x x2 1 0 02 1 0 02
1 L2 ( L2 ) 2 1 x x2 1 L2 ( L2 ) 2
2
1 L L2 3x 2 x 2 ; N = 1 L L = 4x − 4 x2 ; N = 1 x x2 − x 2 x2
N1 = 2
=1− + 2 2 2 2 3 2
= + 2
1 0 0 L L 1 0 0 L L 1 0 0 L L
1 L
2
( L2 ) 2 1 L
2
( L2 ) 2 1 L2 ( L2 ) 2
2 2
1 L L 1 L L 1 L L2

Nodal displacement
x u (1) 
 
{u ( e ) } = u ( 2) 
1 u (1) 2 u (2) 3 u ( 3) u (3) 
 
( x (1) = 0) L ( x ( 3) = L)
( x( 2) = )
2

Figure 6.61: 1D element – quadratic function.

University of Castilla-La Mancha Draft By: Eduardo W. V. Chaves (2016)


Ciudad Real – Spain
6 LINEAR ELASTICITY 637

Note that the above shape functions are the same as the one obtained in Problem 5.26
NOTE 1. We can also apply the equations in (6.169) for another system which is described
in Figure 6.62.
Then, by considering the nodal values given by Figure 6.62, the equation (6.169) becomes:
1 ξ ξ2 1 − 1 (−1) 2 1 − 1 (−1) 2
1 0 02 1 ξ ξ2 1 0 02
1 1 12 ξ 1 1 12 1 ξ ξ2 ξ
N1 = 2
= (ξ − 1) ; N 2 = = 1− ξ 2 ; N3 = = (1 + ξ )
1 − 1 (−1) 2 1 − 1 (−1) 2
1 − 1 (−1) 2 2
1 0 02 1 0 02 1 0 02
1 1 12 1 1 12 1 1 12
(6.170)

ξ
ξ1 ≡ ξ N1 = (ξ − 1)
2
N2 = 1− ξ 2
1 u (1) 2 u (2) 3 u ( 3) ξ
N3 = (1 + ξ )
(ξ (1)= −1) (ξ ( 2)= 0) (ξ (3)= 1) 2

Figure 6.62: Quadratic element (normalized space).

NOTE 3.5: Another example: let us consider a one dimensional case in which the
displacement field (according to z -direction) is approached by a cubic function
( w( x) = a1x 3 + a2 x 2 + a3 x + a4 ), (see Figure 6.63). The nodal “displacement” vector is
represented by:
 w(1) 
 (1) 
(e)  w′ 
{u } =  ( 2 ) 
w 
w′( 2 ) 
 
where w′ is the derivative of w( x) with respect to x . Next, we will obtain the shape
functions in order to obtain the displacement field:
w( x) = N1w(1) + N 2 w′(1) + N 3 w( 2 ) + N 4 w′( 2 )

z y
Nodal displacement
 w(1) 
 (1) 
w( 2) , w′( 2)  w′ 
w(1) , w′(1) (e)
{u } =  ( 2 ) 
w 
x w′( 2 ) 
 
( x (1) = 0 ) 1 L 2
( x ( 2) = L )

Figure 6.63: 1D element.

University of Castilla-La Mancha Draft By: Eduardo W. V. Chaves (2016)


Ciudad Real - Spain
638 SOLVING PROBLEMS BY MEANS OF CONTINUUM MECHANICS

The derivative of w( x) = a1x 3 + a2 x 2 + a3 x + a4 with respect to x is given by:


∂w( x)
w( x) = a1 x 3 + a 2 x 2 + a3 x + a 4 derivative
 → ≡ w′( x) = 3a1 x 2 + 2a2 x + a3
∂x
Then, the relationship between the nodal values and the coefficients ai is given by:

w( x = 0) = w(1) = a4   w(1)   0 0 0 1  a1 
  (1)  
w′( x = 0) = w′(1) = a3   w′   0 0 1 0 a2 
 Matricial
 →  ( 2)  = 3  
w   L L2 L 1 a3 
( 2) 3 2
w( x = L) = w = a1 L + a2 L + a3 L + a4 
 w′( 2 )  3L2 
2 L 1 0 a4 
w′( x = L) = w′( 2 ) = 3a1L2 + 2a2 L + a3   
Then, the shape functions can be obtained as follows
x3 x2 x 1 0 0 0 1
3 2
0 0 1 0 x x x 1
3 2
L L L 1 L3 L2 L 1
2L 1 0   x  
3 2
3L2  x 3L2 2L 1 0  x3 2 x 2 
N1 = =  2  − 3  + 1 ; N2 = = 2 − + x ;
0 0 0 1  L L  0 0 0 1 L L 

0 0 1 0 0 0 1 0
L3 L2 L 1 L3 L2 L 1
3L2 2L 1 0 3L2 2L 1 0

0 0 0 1 0 0 0 1
0 0 1 0 0 0 1 0
x3 x2 x 1 L3 L2 L 1
2L 1 0   x   x 
3 2
3L2 x3 x2 x 1  x3 x 2 
N3 = =  − 2  + 3   ; N4 = = 2 − 
0 0 0 1  L  L   0 0 0 1 L L

0 0 1 0 0 0 1 0
L3 L2 L 1 L3 L2 L 1
3L2 2L 1 0 3L2 2L 1 0
Then,
w = N1 w (1) + N 2 w′(1) + N 3 w ( 2 ) + N 4 w′( 2 )
  x 3  x  2   x3 2x 2    x 3  x  2   x3 x 2 
w =  2  − 3  + 1 w (1) +  2 − + x  w′(1) +  − 2  + 3   w ( 2) +  2 −  w′( 2 )
  L  L  L L    L   L   L L

If we want to obtain the function w′ = N1w(1) + N 2 w′(1) + N 3 w ( 2) + N 4 w′( 2) , we can obtain


similarly, i.e., now instead of replacing the terms ( x 3 , x 2 , x,1 ) we will replace the terms
related to the derivative function w′( x) = 3a1 x 2 + 2a 2 x + a3 , ( 3x 2 ,2 x,1,0 ), i.e.:

University of Castilla-La Mancha Draft By: Eduardo W. V. Chaves (2016)


Ciudad Real – Spain
6 LINEAR ELASTICITY 639

3x 2 2x 1 0 0 0 0 1
2
0 0 1 0 3x 2x 1 0
L3 L2 L 1 L3 L2 L 1
3L2 2 L 1 0 6 x 2 6 x ∂N1 3L2 2 L 1 0 3x 2 4 x ∂N 2
N1 = = 3 − 2 = ; N2 = = 2 − +1 =
0 0 0 1 L L ∂x 0 0 0 1 L L ∂x
0 0 1 0 0 0 1 0
3 2
L L L 1 L3 L2 L 1
3L2 2L 1 0 3L2 2L 1 0

0 0 0 1 0 0 0 1
0 0 1 0 0 0 1 0
3x 2 2x 1 0 L3 L2 L 1
3L2 2L 1 0 6 x 2 6 x ∂N 3 3x 2 2 x 1 0 3x 2 2 x ∂N 4
N3 = =− 3 + 2 = ; N4 = = 2 − =
0 0 0 1 L L ∂x 0 0 0 1 L L ∂x
0 0 1 0 0 0 1 0
L3 L2 L 1 L3 L2 L 1
3L2 2L 1 0 3L2 2L 1 0
then
∂N1 (1) ∂N 2 (1) ∂N 3 ( 2 ) ∂N 4 ( 2 )
w′ = w + w′ + w + w′
∂x ∂x ∂x ∂x
w′ = N1 w (1) + N 2 w′ (1) + N 3 w ( 2 ) + N 4 w′ ( 2 )
 6x2 6x   3 x 2 4 x  (1)  6 x 2 6 x  ( 2 )  3 x 2 2 x  ( 2 )
w′ =  3 − 2  w (1) +  2 − + 1 w′ + − 3 + 2  w +  2 −  w′
 L L  L L   L L  L L

Next, let us consider a very simple problem which was already discussed in Problem 5.25-
NOTE 4. In this case the function is linear, (see Figure 6.64), so, u ( x) = a1 + a2 x , and
 u (1)  1 x (1)   a1 
 (2)  =  (2)   
u  1 x  a2 
Then, the shape functions for this problem can be obtained as follows:
1 x 1 x (1)
1 x (2) ( x ( 2) − x) 1 x ( x − x (1) )
N1 ( x) = = ; N 2 ( x) = = (6.171)
1 x (1) ( x ( 2 ) − x (1) ) 1 x (1) ( x ( 2 ) − x (1) )
1 x (2) 1 x ( 2)

x Nodal displacement
 u (1) 
1 u (1) 2 u (2) {u ( e ) } =  ( 2 ) 
u 
( x (1) ) ( x( 2) )

Figure 6.64: 1D element – linear function.

University of Castilla-La Mancha Draft By: Eduardo W. V. Chaves (2016)


Ciudad Real - Spain
640 SOLVING PROBLEMS BY MEANS OF CONTINUUM MECHANICS

Now if we consider the normalized space, (see Figure 6.65), the shape functions become:
ξ
1 1 ξ 1 −1
1 ξ ( 2)
1 1 (1 − ξ ) 1 ξ (1 + ξ )
N1 (ξ ) = = = ; N 2 (ξ ) = =
1 ξ (1) 1 −1 2 1 −1 2
1 ξ ( 2) 1 1 1 1

ξ
1
N1 = (1 − ξ )
2
1 u (1) 2 u (2) 1
N 2 = (1 + ξ )
2
(ξ (1)= −1) (ξ ( 2)= 1)

Figure 6.65: 1D element – linear function (normalized space).


Note that the shape functions can be used to approach any function even the geometry, so,
if we want to represent the geometry x by using the normalized space, (see Figure 6.66), it
is enough to do:
(1 − ξ ) (i ) (1 + ξ ) ( f ) ( x (i ) + x ( f ) ) ( x ( f ) − x (i ) )
x = N1 (ξ ) x (i ) + N 2 (ξ ) x ( f ) = x + x = + ξ (6.172)
2 2 2 2
and the differential dx can be obtained as follows:
 ( x ( i ) + x ( f ) ) ( x ( f ) − x ( i ) )  ( x ( f ) − x (i ) )
dx = d  + ξ  = dξ (6.173)
 2 2  2

x( f )
ξ
x (i )
2
1 (ξ ( 2)= 1)
(ξ (1)= −1)

Figure 6.66: 1D element – linear function (normalized space).

Note that the equation in (6.172) is the transformation between the normalized system
(Figure 6.65) and the Cartesian system (Figure 6.64).
When the displacement and the geometry are approached by the same shape functions the
element is called Isoparametric Element.

University of Castilla-La Mancha Draft By: Eduardo W. V. Chaves (2016)


Ciudad Real – Spain
6 LINEAR ELASTICITY 641

NOTE 4: Shape Functions in the Normalized Space.


NOTE 4.1: Shape Functions for 1D in the Normalized Space.
We can generalize the shape functions for one-dimensional (1D) in the Normalized space
by using the Lagrange’s Polynomial of degree (n − 1) , namely:
(ξ − ξ (1) )(ξ − ξ ( 2 ) )L(ξ − ξ ( a −1) )(ξ − ξ ( a +1) )L(ξ − ξ ( n ) )
N a( n −1) (ξ ) =
(ξ ( a ) − ξ (1) )(ξ ( a ) − ξ ( 2 ) )L(ξ ( a ) − ξ ( a −1) )(ξ ( a ) − ξ ( a +1) )L(ξ ( a ) − ξ ( n ) )
(6.174)
n
 (ξ − ξ ( a ) ) 
= ∏  (a ) 
 (ξ − ξ ( j ) ) 
a =1( a ≠ j )  
(n − number of nodes)

For example, for the element with 3-nodes, described in Figure 6.62, we have:
(ξ − ξ (1) )(ξ − ξ ( 2 ) )(ξ − ξ (3) )
N a( 2 ) (ξ ) =
(ξ ( a ) − ξ (1) )(ξ ( a ) − ξ ( 2 ) )(ξ ( a ) − ξ (3) )

 ( 2) (ξ − ξ ( 2 ) )(ξ − ξ (3) ) (ξ − 0)(ξ − 1) 1


 1 N (ξ ) = = = ξ (ξ − 1)
 (ξ − ξ )(ξ − ξ )
(1) ( 2) (1) ( 3)
(( − 1) − 0 )(( −1) − 1) 2
 (ξ − ξ (1) )(ξ − ξ (3) ) (ξ − (−1))(ξ − 1)
⇒  N 2( 2 ) (ξ ) = ( 2) = = (1 − ξ 2 )
 (ξ − ξ (1)
)(ξ ( 2)
− ξ ( 3)
) ( 0 − ( −1 ))( 0 − 1)
 (ξ − ξ (1) )(ξ − ξ ( 2 ) ) (ξ − (−1))(ξ − 0) 1
 N 3( 2 ) (ξ ) = (3) = = ξ (ξ + 1)
 (ξ − ξ (1) )(ξ (3) − ξ ( 2 ) ) (1 − (−1))(1 − 0) 2
which results match with the one in equation (6.170).
The shape functions for the 1D element with 4-nodes, described in Figure 6.67, can be
obtained as follows:
(ξ − ξ (1) )(ξ − ξ ( 2 ) )(ξ − ξ (3) )(ξ − ξ ( 4) )
N a(3) (ξ ) =
(ξ ( a ) − ξ (1) )(ξ ( a ) − ξ ( 2 ) )(ξ ( a ) − ξ (3) )(ξ ( a ) − ξ ( 4 ) )
Then
 ( 3) (ξ − ξ ( 2) )(ξ − ξ (3) )(ξ − ξ ( 4 ) ) [ξ − ( −31 )][ξ − ( 13 )](ξ − 1) 1
 N1 = = = (9ξ 2 − 1)(1 − ξ )
 (ξ − ξ )(ξ − ξ )(ξ − ξ ) [(−1) − ( 3 )][(−1) − ( 3 )][(−1) − 1] 16
(1) ( 2) (1) ( 3) (1) ( 4) −1 1

 (ξ − ξ (1) )(ξ − ξ (3) )(ξ − ξ ( 4 ) ) [ξ − (−1)][ξ − ( 13 )](ξ − 1) 9


 N 2(3) = = = (1 − ξ 2 )(1 − 3ξ )
 (ξ − ξ )(ξ − ξ )(ξ − ξ ) [( 3 ) − (−1)][( 3 ) − ( 3 )][( 3 ) − 1] 16
(2) (1) (2) ( 3) ( 2) ( 4) −1 −1 1 −1


 N ( 3) (ξ − ξ (1) )(ξ − ξ ( 2 ) )(ξ − ξ ( 4 ) ) [ξ − ( −1)][ξ − ( −31 )](ξ − 1) 9
= = = (1 − ξ 2 )(1 + 3ξ )
 3 (ξ − ξ )(ξ − ξ )(ξ − ξ ) [( 3 ) − (−1)][( 3 ) − ( 3 )][( 3 ) − 1] 16
( 3) (1) ( 3) (2) ( 3) ( 4) 1 1 −1 1

 ( 3) (ξ − ξ (1) )(ξ − ξ ( 2 ) )(ξ − ξ (3) ) [ξ − (−1)][ξ − ( −31 )][ξ − ( 13 )] 1
N4 = = = (9ξ 2 − 1)(1 + ξ )
 (ξ − ξ )(ξ − ξ )(ξ − ξ ) [(1) − (−1)][(1) − ( 3 )][(1) − ( 3 )] 16
(4) (1) (4) ( 2) (4) ( 3) − 1 1

1 2 3 4

(ξ (1)= −1) (ξ ( 2)= −1


3
) (ξ (3)= 13 ) (ξ ( 4)= 1)

Figure 6.67: Cubic element (normalized space).

University of Castilla-La Mancha Draft By: Eduardo W. V. Chaves (2016)


Ciudad Real - Spain
642 SOLVING PROBLEMS BY MEANS OF CONTINUUM MECHANICS

The shape functions for the element with 5-nodes described in Figure 6.68 can be obtained
as follows:
(ξ − ξ (1) )(ξ − ξ ( 2 ) )(ξ − ξ (3) )(ξ − ξ ( 4 ) )(ξ ( a ) − ξ (5) )
N a( 4 ) (ξ ) =
(ξ ( a ) − ξ (1) )(ξ ( a ) − ξ ( 2 ) )(ξ ( a ) − ξ (3) )(ξ ( a ) − ξ ( 4) )(ξ ( a ) − ξ (5) )
Then
 ( 4) (ξ − ξ ( 2 ) )(ξ − ξ (3) )(ξ − ξ ( 4) )(ξ ( a ) − ξ (5) ) 1
 N1 (ξ ) = = ξ (4ξ 2 − 1)(ξ − 1)
 (ξ − ξ )(ξ − ξ )(ξ − ξ )(ξ − ξ ) 6
(1) (2) (1) ( 3) (1) ( 4) (1) (5)

 (ξ − ξ (1) )(ξ − ξ (3) )(ξ − ξ ( 4) )(ξ ( a ) − ξ (5) ) 4


 N 2( 4) (ξ ) = = ξ (ξ 2 − 1)(1 − 2ξ )
 (ξ − ξ )(ξ − ξ )(ξ − ξ )(ξ − ξ ) 3
( 2) (1) ( 2) ( 3) ( 2) ( 4) ( 2) ( 5)


 ( 4) (ξ − ξ (1) )(ξ − ξ ( 2 ) )(ξ − ξ ( 4 ) )(ξ ( a ) − ξ (5) )
 N 3 (ξ ) = = ξ (1 − ξ 2 )(1 − 4ξ 2 )
 (ξ − ξ )(ξ − ξ )(ξ − ξ )(ξ − ξ )
( 3) (1) ( 3) (2) ( 3) ( 4) ( 3) ( 5)

 (ξ − ξ (1) )(ξ − ξ ( 2) )(ξ − ξ (3) )(ξ ( a ) − ξ (5) ) 4


 N 4( 4) (ξ ) = = ξ (1 − ξ 2 )(1 + 2ξ )
 (ξ − ξ )(ξ − ξ )(ξ − ξ )(ξ − ξ ) 3
( 4) (1) ( 4) ( 2) (4) ( 3) ( 4) ( 5)


 N ( 4) (ξ ) = (ξ − ξ (1) )(ξ − ξ ( 2 ) )(ξ − ξ (3) )(ξ − ξ ( 4 ) ) 1
= ξ (4ξ 2 − 1)(1 + ξ )
 5 (ξ − ξ )(ξ − ξ )(ξ − ξ )(ξ − ξ ) 6
( 5) (1) ( 5) (2) ( 5) ( 3) ( 5) ( 4)

1 2 3 4 5

(ξ (1)= −1) (ξ ( 2)= −1


2
) (ξ ( 3)= 0) (ξ ( 4)= 12 ) (ξ (5)= 1)

Figure 6.68: Quartic element (normalized space).

University of Castilla-La Mancha Draft By: Eduardo W. V. Chaves (2016)


Ciudad Real – Spain
6 LINEAR ELASTICITY 643

Table 6.1: Shape functions for 1D in the Normalized Space [−1,1] .

Linear Element  (1) 1


 N1 (ξ ) = 2
(1 − ξ )

 N (1) (ξ ) = 1
ξ (1) = −1 ξ ( 2) = 1 (1 + ξ )
 2 2

Quadratic Element  ( 2) 1
 N1 (ξ ) = 2 ξ (ξ − 1)
 ( 2 )
 N 2 (ξ ) = (1 − ξ )
2

ξ (1) = −1 ξ ( 2) = 0 ξ ( 3) = 1  1
 N 3( 2 ) (ξ ) = ξ (ξ + 1)
 2

Cubic Element  ( 3) 1
 N1 (ξ ) = 16 (9ξ − 1)(1 − ξ )
2


 N (3) (ξ ) = 9 (1 − ξ 2 )(1 − 3ξ )
 2 16

ξ (1) = −1 ξ ( 2) = −31 ξ (3) = 13 ξ ( 4) = 1  N (3) (ξ ) = 9 (1 − ξ 2 )(1 + 3ξ )
 3 16
 1
 N 4(3) (ξ ) = (9ξ 2 − 1)(1 + ξ )
 16

Quartic Element  ( 4) 1
 N1 (ξ ) = 6 ξ (4ξ − 1)(ξ − 1)
2


 N ( 4 ) (ξ ) = 4 ξ (ξ 2 − 1)(1 − 2ξ )
 2 3
 ( 4)
 N 3 (ξ ) = ξ (1 − ξ )(1 − 4ξ )
2 2
ξ (1) = −1 ξ ( 2) = − 12 ξ (3) = 0 ξ ( 4) = 12 ξ ( 5) = 1
 4
 N 4( 4 ) (ξ ) = ξ (1 − ξ 2 )(1 + 2ξ )
 3
 ( 4) 1
 N 5 (ξ ) = ξ (4ξ − 1)(1 + ξ )
2

 6

NOTE 4.2: Shape Functions for 2D in the Normalized Space.


The shape functions for two-dimensional (2D) elements in the normalized space can be
obtained by combining the shape functions for 1D according to the directions ξ and η .
For example, for the case presented in Figure 6.59 and by taking into account the Table 6.1
we can conclude that:
 1 1 1
 N1 (ξ ,η ) = N1 (ξ ) N1 (η ) = 2 (1 − ξ ) 2 (1 − η ) = 4 (1 − ξ )(1 − η )
(1) (1)


 N (ξ ,η ) = N (1) (ξ ) N (1) (η ) = 1 (1 + ξ ) 1 (1 − η ) = 1 (1 + ξ )(1 − η )
 2 2 1
2 2 4
 (6.175)
1 1 1
 N (ξ ,η ) = N (1) (ξ ) N (1) (η ) = (1 + ξ ) (1 + η ) = (1 + ξ )(1 + η )
 3 2 2
2 2 4
 1 1 1
 N 4 (ξ ,η ) = N1(1) (ξ ) N 2(1) (η ) = (1 − ξ ) (1 + η ) = (1 − ξ )(1 + η )
 2 2 4

University of Castilla-La Mancha Draft By: Eduardo W. V. Chaves (2016)


Ciudad Real - Spain
644 SOLVING PROBLEMS BY MEANS OF CONTINUUM MECHANICS

which results match the shape functions given in Figure 6.59.


For the rectangle with 9 nodes (see Figure 6.69), the shape functions can be obtained by
combining the shape functions given by 1D element with 3 nodes (see Table 6.1), i.e.:
 1 1 1
 N1 (ξ ,η ) = N1 (ξ ) N1 (η ) = 2 ξ (ξ − 1) 2 η (η − 1) = 4 ξη (ξ − 1)(η − 1)
( 2) (2)


 N (ξ ,η ) = N ( 2) (ξ ) N ( 2 ) (η ) = (1 − ξ 2 ) 1 η (η − 1) = 1 η (1 − ξ 2 )(η − 1)
 2 1 1
2 2

 N 3 (ξ ,η ) = N1( 2 ) (ξ ) N1( 2) (η ) = ξ (ξ + 1) η (η − 1) = 1 ξη (ξ + 1)(η − 1)
1 1
 2 2 4
 1 1
 N 4 (ξ ,η ) = N 3( 2) (ξ ) N 2( 2 ) (η ) = ξ (ξ + 1)(1 − η 2 ) = ξ (ξ + 1)(1 − η 2 )
 2 2
 1 1 1
 N 5 (ξ ,η ) = N 3 (ξ ) N 3 (η ) = ξ (ξ + 1) η (η + 1) = ξη (ξ + 1)(η + 1)
( 2) ( 2)
(6.176)
 2 2 4
 1 1
 N 6 (ξ ,η ) = N 2 (ξ ) N 3 (η ) = (1 − ξ ) 2 η (η + 1) = 2 η (1 − ξ )(η + 1)
( 2) ( 2) 2 2


 N (ξ ,η ) = N ( 2) (ξ ) N ( 2 ) (η ) = 1 ξ (ξ − 1) 1 η (η + 1) = 1 ξη (ξ − 1)(η + 1)
 7 1 3
2 2 4

1 1
 N 8 (ξ ,η ) = N1( 2 ) (ξ ) N 2( 2) (η ) = ξ (ξ − 1)(1 − η 2 ) = ξ (ξ − 1)(1 − η 2 )
 2 2

 N 9 (ξ ,η ) = N 2 (ξ ) N 2 (η ) = (1 − ξ )(1 − η ) = (1 − ξ )(1 − η )
( 2) (2) 2 2 2 2

η ( 3) = 1
7 6 5
η ) = η (η − 1) 
1
N1( 2) (
2
 4 ξ
N 2( 2) (η ) = (1 − η 2 )  η (2) = 0 8 9
1 
N 3( 2) (η ) = η (η + 1)  ( 2) 1
2  1 2 3  N1 (ξ ) = 2 ξ (ξ − 1)
η (1) = −1  ( 2)
 N 2 (ξ ) = (1 − ξ )
2

 1
 N 3( 2) (ξ ) = ξ (ξ + 1)
ξ (1) = −1 ξ (2) = 0 ξ ( 3) = 1  2

Figure 6.69: Quadrilateral element – quadratic function – normalized space.

For the rectangle with 16 nodes, (see Figure 6.70), the shape functions are obtained by
combining the 1D shape functions for 1D element with 4 nodes Ni(3) , i.e.:
N1 (ξ ,η ) = N1(3) (ξ ) N1(3) (η ); N 2 (ξ ,η ) = N 2(3) (ξ ) N1(3) (η ); N 3 (ξ ,η ) = N 3(3) (ξ ) N1(3) (η );
N 4 (ξ ,η ) = N 4(3) (ξ ) N1(3) (η ); N 5 (ξ ,η ) = N 4(3) (ξ ) N 2(3) (η ); N 6 (ξ ,η ) = N 4(3) (ξ ) N 3(3) (η );
N 7 (ξ ,η ) = N 4(3) (ξ ) N 4(3) (η ); N 8 (ξ ,η ) = N 3(3) (ξ ) N 4(3) (η ); N 9 (ξ ,η ) = N 2(3) (ξ ) N 4(3) (η );
(6.177)
N10 (ξ ,η ) = N1(3) (ξ ) N 4(3) (η ); N11 (ξ ,η ) = N 4(3) (ξ ) N 3(3) (η ); N12 (ξ ,η ) = N 4(3) (ξ ) N 2(3) (η );
N13 (ξ ,η ) = N 2(3) (ξ ) N 2(3) (η ); N14 (ξ ,η ) = N 3(3) (ξ ) N 2(3) (η ); N15 (ξ ,η ) = N 3(3) (ξ ) N3(3) (η );
N16 (ξ ,η ) = N 2(3) (ξ ) N 3(3) (η )

University of Castilla-La Mancha Draft By: Eduardo W. V. Chaves (2016)


Ciudad Real – Spain
6 LINEAR ELASTICITY 645

η η

η (4) = 1
10 9 8 7
16 15
η (3) = 13 11 6
ξ

η ( 2) = −31 12 13 14 5

1 2 3 4
η (1) = −1
ξ

ξ (1) = −1 ξ ( 2) = −31 ξ (3) = 13 ξ ( 4) = 1


Figure 6.70: Quadrilateral element – cubic function – normalized space.

NOTE 4.3: Shape Functions for 3D in the Normalized Space.


The shape functions for the hexahedron element with 8 nodes in the normalized space,
(see Figure 6.71), can be obtained by combining the shape functions for 1D according to
the directions ξ , η and ζ , i.e.:
 r 1 1 1 1
 N1 ( ξ ) = N1(1) (ξ ) N1(1) (η ) N1(1) (ζ ) = (1 − ξ ) (1 − η ) (1 − ζ ) = (1 − ξ )(1 − η )(1 − ζ )
2 2 2 8
 r
 N (ξ ) = N (1) (ξ ) N (1) (η ) N (1) (ζ ) = (1 + ξ ) (1 − η ) (1 − ζ ) = 1 (1 + ξ )(1 − η )(1 − ζ )
1 1 1
 2 2 1 1
2 2 2 8
 r
 N (ξ ) = N (1) (ξ ) N (1) (η ) N (1) (ζ ) = 1 (1 + ξ ) 1 (1 + η ) 1 (1 − ζ ) = 1 (1 + ξ )(1 + η )(1 − ζ )
 3 2 1 1
2 2 2 8
 r
 N 4 (ξ ) = N1(1) (ξ ) N 2(1) (η ) N1(1) (ζ ) = (1 − ξ ) (1 + η ) (1 − ζ ) = 1 (1 − ξ )(1 + η )(1 − ζ )
1 1 1
 2 2 2 8
 r (6.178)
 N (ξ ) = N (1) (ξ ) N (1) (η ) N (1) (ζ ) = 1 (1 − ξ ) 1 (1 − η ) 1 (1 + ζ ) = 1 (1 − ξ )(1 − η )(1 + ζ )
 5 1 1 2
2 2 2 8
 r 1 1 1 1
 N 6 (ξ ) = N 2(1) (ξ ) N1(1) (η ) N 2(1) (ζ ) = (1 + ξ ) (1 − η ) (1 + ζ ) = (1 + ξ )(1 − η )(1 + ζ )
 2 2 2 8
 r 1 1 1 1
 N 7 (ξ ) = N 2(1) (ξ ) N 2(1) (η ) N 2(1) (ζ ) = (1 + ξ ) (1 + η ) (1 + ζ ) = (1 + ξ )(1 + η )(1 + ζ )
 2 2 2 8
 r 1 1 1 1
 N8 (ξ ) = N1 (ξ ) N 2 (η ) N 2 (ζ ) = (1 − ξ ) (1 + η ) (1 + ζ ) = (1 − ξ )(1 + η )(1 + ζ )
(1) (1) (1)
 2 2 2 8

University of Castilla-La Mancha Draft By: Eduardo W. V. Chaves (2016)


Ciudad Real - Spain
646 SOLVING PROBLEMS BY MEANS OF CONTINUUM MECHANICS

ζ
ζ ζ =1 8
7
ζ ( 2)
=1 5
η
6
η = −1
4
3 ξ
η
ζ (1)
= −1 1
ξ =1
2
η ( 2) = 1
ξ (1) = −1
ξ ( 2) = 1 η (1) = −1
ξ
Figure 6.71: Hexahedron element with 8 nodes – linear function – normalized space.

The shape functions for the hexahedron element with 27 nodes in the normalized space,
(Oñate (1992)), can be obtained by combining the shape functions for 1D according to the
directions ξ , η and ζ , (see Figure 6.72).

For the nodes i = 1,2,3,4,5,6,7,8 :


r 1 2
N i (ξ ) = (ξ + ξξ (i ) )(η 2 + ηη (i ) )(ζ 2 + ζζ (i ) ) (6.179)
8
For the nodes i = 9,10,11,12,13,14,15,16,17,18,19,20 :
r 1 (i ) 2 2 (i ) 2 1 2
N i (ξ ) = η (η − ηη )ζ
(i ) 2 (i )
(ζ − ζζ )(1 − ξ ) + ζ (i ) (ζ 2 − ζζ (i ) ) +
2
4 4
(6.180)
2 1 2 2
ξ (i ) (ξ 2 − ξξ (i ) )(1 − η 2 ) + ξ (i ) (ξ 2 − ξξ (i ) )η (i ) (η 2 − ηη (i ) )(1 − ζ 2 )
4
For the nodes i = 21,22,23,24,25,26 :
r 1 1 2
N i (ξ ) = (1 − ξ )(1 − η )(ζ − ζ 2ζ (i ) ) + (1 − η 2 )(1 − ζ 2 )(ξ 2 − ξ 2ξ (i ) ) +
2 2 2
2 2
(6.181)
1
(1 − ξ 2 )(1 − ζ 2 )(η 2 − η 2η (i ) )
2
For the node i = 27 :
r
N 27 (ξ ) = N 2( 2 ) (ξ ) N 2( 2 ) (η ) N 2( 2 ) (ζ ) = (1 − ξ 2 )(1 − η 2 )(1 − ζ 2 ) (6.182)

University of Castilla-La Mancha Draft By: Eduardo W. V. Chaves (2016)


Ciudad Real – Spain
6 LINEAR ELASTICITY 647

ζ η

8 19 7
ζ 20 25 18
17 6
ζ ( 3) = 1 5
16 24 15
27 ξ
21
26
23 η
ζ ( 2)
=0 13
11 14
4
22 3 η ( 3) = 1
12
9 10 η (2) = 0
1
ζ (1)
= −1
2
η (1) = −1

ξ (1) = −1 ξ (2) = 0 ξ ( 3) = 1 ξ

Figure 6.72: Hexahedron element with 27 nodes – quadratic function – normalized space.

NOTE 5: Area Coordinates


Let us consider the triangle and the plane defined in Figure 6.73(a), and by using triangle
properties the following is true:
1 L1 h1
= ⇒ L1 = (6.183)
h h1 h
Note that the areas A and A1 , (see Figure 6.73(b)), can be expressed as follows:
bh 2A
A= ⇒ h= (6.184)
2 b
and
bh1 2 A1
A1 = ⇒ h1 = (6.185)
2 b
Then
2 A1
h1 A
L1 = = b = 1 (6.186)
h 2A A
b
In Figure 6.73(b) we can appreciate how L1 changes into the triangle.
A2 A
In the same fashion we can define that L2 = and L3 = 3 , then defining the Area
A A
Coordinates:
A1 A2 A3
L1 = ; L2 = ; L3 = (6.187)
A A A

University of Castilla-La Mancha Draft By: Eduardo W. V. Chaves (2016)


Ciudad Real - Spain
648 SOLVING PROBLEMS BY MEANS OF CONTINUUM MECHANICS

L1=1
L1=0.8
L1=0.6
L1=0.4
L1=0.2
L1=0.0

3 3
1

1 b L1 A1
L1 1

h1
2 2
h

(a) (b)

L1 = 0

3
A1 2 ( L1 , L2 , L3 )
( L1 , L2 , L3 )
A3 (0,1,0)
(0,0,1) A2 P ( x, y )
P( L1 , L2 , L3 )
L3 = 0
L2 = 0
y

1 ( L1 , L2 , L3 )
x (c)
(1,0,0)

Figure 6.73: Area coordinates.

Note that since A = A1 + A2 + A3 the following is true:


A1 A2 A3
L1 + L2 + L3 = + + =1 (6.188)
A A A
The geometry can be approached by using area coordinates as follows:
1   1 1 1   L1 
 x = L1 x (1) + L2 x ( 2 ) + L3 x (3) Matrix form    (1)  
    → x  =  x x( 2) x (3)   L2  (6.189)
 y = L1 y (1) + L2 y ( 2) + L3 y (3)  y   y (1)
   y ( 2)
y (3)   L3 

If the function is in terms of area coordinates f = f ( L1, L2 , L3 ) , the derivative with respect
to x and y can be obtained as follows:
3 3
∂f ∂f ∂La ∂f ∂f ∂La
∂x
= ∑ a =1 ∂La ∂x
;
∂y
= ∑ ∂L
a =1 a ∂y
(6.190)

University of Castilla-La Mancha Draft By: Eduardo W. V. Chaves (2016)


Ciudad Real – Spain
6 LINEAR ELASTICITY 649

The integral can easily be obtained:


a!b!
∫ L L dL = (a + b + 1)! L
a b
1 2
L
a!b!c!
∫ L L L dA = (a + b + c + 2)! 2 A
a b c
1 2 3 (6.191)
A
a!b!c!d !
∫ L L L L dV = (a + b + c + d + 3)! 3!V
a b c d
1 2 3 4
V

where n! stands for the factorial of n , i.e. n!= 1 × 2 × 3L × (n − 1) × n


Triangle element with 3-nodes.
We can use the Lagrange’s Polynomial, (see equation (6.174)), in order to derive the shape
functions for triangles and tetrahedrons. In this case the coordinates is constituted by the
coordinates Li . For example, for the triangle with 3-nodes, (see Figure 6.74) we have:
( L1 − L1( 2 − 3) ) ( L1 − 0)
N1 ( Li ) = = = L1
( L1(1) − L1( 2 − 3) ) (1 − 0)
( L2 − L(21− 3) ) ( L2 − 0)
N 2 ( Li ) = = = L2 (6.192)
( L(22 ) − L(21− 3) ) (1 − 0)
( L3 − L(31− 2 ) ) ( L3 − 0)
N 3 ( Li ) = = = L3
( L(33) − L(31− 2 ) ) (1 − 0)

L3 = 1 L3 = 0

L2 = 0 L1 = 0
3 2

L2 = 1
L1 = 0

L1 = 1
y
L3 = 0
1
L2 = 0

Figure 6.74: Area coordinates – Triangle with 3-nodes.

University of Castilla-La Mancha Draft By: Eduardo W. V. Chaves (2016)


Ciudad Real - Spain
650 SOLVING PROBLEMS BY MEANS OF CONTINUUM MECHANICS

Triangle with 6-nodes, (see Figure 6.75). For this case we have:
( L1 − L1( 4 − 6 ) )( L1 − L1( 2 −3) ) ( L1 − 12 )( L1 − 0)
N1 = = = L1 (2 L1 − 1)
( L1(1) − L1( 4 − 6 ) )( L1(1) − L1( 2 − 3) ) (1 − 12 )(1 − 0)
( L2 − L(24 −5) )( L2 − L(21− 3) ) ( L2 − 12 )( L2 − 0)
N2 = = = L2 (2 L2 − 1)
( L(22) − L(24 −5) )( L(22 ) − L(21− 3) ) ( L(22 ) − 12 )( L(22 ) − 0)
( L3 − L(36 − 5) )( L3 − L(31− 2 ) ) ( L − 1 )( L − 0)
N3 = ( 3) ( 6 − 5) ( 3) (1− 2 )
= (33) 12 (33) = L3 (2 L3 − 1)
( L3 − L3 )( L3 − L3 ) ( L3 − 2 )( L3 − 0)

N 4 = N 4 ( L1 ) N 4 ( L2 ) = 4 L1L2
N 5 = N 5 ( L2 ) N 5 ( L3 ) = 4 L2 L3
N 6 = N 6 ( L1 ) N 6 ( L3 ) = 4 L1L3

L3 = 1 L3 = 1
2
L3 = 0

L1 = 0
3 5 2

L2 = 1

L1 = 1
2

6 4
L2 = 1
2

L1 = 1
y
1
L2 = 0

x
Figure 6.75: Area coordinates – Triangle with 6-nodes.

Triangle with 10-nodes, (see Figure 6.76)


For this case the shape function N1 can be obtained as follows:
( L1 − L1( 4 − 9 ) )( L1 − L1(5 −8) )( L1 − L1( 2 − 3) ) ( L1 − 23 )( L1 − 13 )( L1 − 0) 1
N1 = = = L1 (3L1 − 2)(3L1 − 1)
( L1(1) − L1( 4 − 9 ) )( L1(1) − L1(5 −8) )( L1(1) − L1( 2 −3) ) (1 − 23 )(1 − 13 )(1 − 0) 2
( L2 − L(25 − 6 ) )( L2 − L(24 − 7 ) )( L2 − L(21− 3) ) ( L2 − 23 )( L2 − 13 )( L2 − 0) 1
N2 = = = L2 (3L2 − 2)(3L2 − 1)
( L(22 ) − L(25 − 6 ) )( L(22 ) − L(24 − 7 ) )( L(22 ) − L(21−3) ) (1 − 23 )(1 − 13 )(1 − 0) 2
( L3 − L(37 −8) )( L3 − L(36 − 9) )( L3 − L(31− 2 ) ) ( L3 − 23 )( L3 − 13 )( L3 − 0) 1
N3 = = = L3 (3L3 − 2)(3L3 − 1)
( L(33) − L(37 −8) )( L(33) − L(36 − 9) )( L(33) − L(31− 2) ) (1 − 23 )(1 − 13 )(1 − 0) 2

Note that the node 4, (see Figure 6.76), depends on coordinates L1 and L2 :

University of Castilla-La Mancha Draft By: Eduardo W. V. Chaves (2016)


Ciudad Real – Spain
6 LINEAR ELASTICITY 651

( L1 − L1(5 −8) )( L1 − L1( 2 − 3) ) ( L1 − 23 )( L1 − 0) 3


N 4 ( L1 ) = = = L1 (3L1 − 1)
( L1( 4 ) − L1(5 −8) )( L1( 4) − L1( 2 −3) ) ( 13 − 23 )( 13 − 0) 2
( L2 − L(21− 3) ) ( L2 − 0)
N 4 ( L2 ) = = = 3L2
( L(24 ) − L(21−3) ) ( 13 − 0)
3 9
⇒ N 4 ( L1 , L2 ) = N 4 ( L1 ) N 4 ( L2 ) = L1 (3L1 − 1)3L2 = L1L2 (3L1 − 1)
2 2
Similarly we can obtain:
9
N 5 ( L1 , L2 ) = N 5 ( L1 ) N 5 ( L2 ) = L1L2 (3L2 − 1)
2
9
N 6 ( L2 , L3 ) = N 5 ( L2 ) N 5 ( L3 ) = L2 L3 (3L2 − 1)
2
9
N 7 ( L2 , L3 ) = N 5 ( L2 ) N 5 ( L3 ) = L2 L3 (3L3 − 1)
2
9
N8 ( L1 , L3 ) = N 5 ( L1 ) N 5 ( L3 ) = L1L3 (3L3 − 1)
2
9
N 9 ( L1 , L3 ) = N 5 ( L1 ) N 5 ( L3 ) = L1L3 (3L1 − 1)
2
And the node 10:
N10 ( L1 , L2 , L3 ) = 27 L1L2 L3

L3 = 2
3
L3 = 13 L3 = 0

(L1,L2,L3) (L1,L2,L3)
3 7 6 2
(0,0,1) (0,1,0)

L1 = 0

8 5
10
L2 = 2
3
L1 = 1
3

9 4
L2 = 13

y L1 = 2
3

1
L2 = 0
(L1,L2,L3)
x
(1,0,0)

Figure 6.76: Area coordinates – Triangle with 10-nodes.

University of Castilla-La Mancha Draft By: Eduardo W. V. Chaves (2016)


Ciudad Real - Spain
652 SOLVING PROBLEMS BY MEANS OF CONTINUUM MECHANICS

NOTE 6: Transformation between the Original System and the Normalized Space.
To obtain the transformation between the original system and the normalized space we will
make an analogy with the Continuum Kinematics discussed in Chapter 2 in Chaves (2013).
Let us suppose that the Reference Configuration represents the Normalized space and the
Current Configuration represents the Original system, (see Figure 6.77). In Figure 6.77, F
is the Jacobian (“Deformation Gradient”), J = det ( F ) ≡ F represents the Jacobian
r r
determinant, σ ( x ) is the Cauchy Stress tensor, S ( X ) is the Second Piola-Kirchhoff stress
r
tensor, E ( X ) is the Green-Lagrange strain tensor.

r r r
dx = F ( X ) ⋅ dX
X2 = ξ 2 ≡η
dV = J dV0
dV
r r σ
dV0 dX = dξ x2 = y r
dx
E, S X1 = ξ 1 ≡ ξ

X3 = ξ 3 ≡ ζ x1 = x

Normalized space F −1 Original system


(“Reference configuration”) x3 = z (“Current configuration”)

Figure 6.77: Normalized space and the original system.

Here the Jacobian F (“Deformation Gradient”) is given by:


 ∂x1 ∂x1 ∂x1   ∂x ∂x ∂x 
   
 F11 F12 F13   ∂X 1 ∂X 2 ∂X 3   ∂ξ ∂η ∂ζ 
∂xi  ∂x ∂x2 ∂x2   ∂y ∂y ∂y 
Fij = = F21 F22 F23  =  2 = (6.193)
∂X j  ∂X ∂X 2 ∂X 3   ∂ξ ∂η ∂ζ 
 F31 F33   ∂x 1  
F32
 3 ∂x3 ∂x3   ∂z ∂z ∂z 
 ∂X 1 ∂X 2 ∂X 3   ∂ξ ∂η ∂ζ 

1
The inverse can easily be obtained by using the definition F −1 = [cof ( F )]T , i.e.:
J
T
 F22 F23 F21 F23 F21 F22 
 − 
 F32 F33 F31 F33 F31 F32 
∂X i ∂ξ i 1  F12 F13 F11 F13 F F12 
Fij−1 = = = − − 11  (6.194)
∂x j ∂x j J  F32 F33 F31 F33 F31 F32 
 F12 F13 F11 F13 F11 F12 
 − 
 F22 F23 F21 F23 F21 F22 

Note that for the two-dimensional case the above equations become:

University of Castilla-La Mancha Draft By: Eduardo W. V. Chaves (2016)


Ciudad Real – Spain
6 LINEAR ELASTICITY 653

 ∂x ∂x 
∂xi  F11 F12   ∂ξ ∂η 
Fij2 D = = =  (6.195)
∂X j  F21 F22   ∂y ∂y 
 ∂ξ ∂η 
−1 1
and F ( 2 D ) = (2D)
[cof ( F ( 2 D ) )]T :
J
T
 ∂y ∂y 

1  ∂η ∂ξ 
T
∂X i ∂ξ i 1  F22 − F21 
( Fij2 D ) −1 = = =   =   (6.196)
∂x j ∂x j J − F12 F11  J − ∂x ∂x 
 ∂η ∂ξ 
r
The stiffness matrix is defined in the original space (current configuration - x ) and by using
the transformation dV = J dV0 we can obtain:
r r r r

V

[k ( e ) ] = [B( x )]T [C ] [B( x )] dV = [B( x )]T [C ] [B( x )] J dV0
V0
(6.197)

Note that the integrand is defined in the original space. Let us adopt that the geometry can
be approached by the same shape functions as those used to approach the displacement
(Isoparametric Element), i.e.:
n r n r n r
x1 = x = ∑ N a (ξ ) x ( a ) ; x2 = y = ∑ N a (ξ ) y ( a ) ; x3 = z = ∑ N a (ξ ) z ( a ) (6.198)
a =1 a =1 a =1

where n is the number of nodes. With that the Jacobian can be obtained as follows:
  n r (a)   n r   n r 
  a =1

 ∂ N a (ξ ) x 


∂ N a (ξ ) x ( a ) 
 a =1 

∂ N a (ξ ) x ( a )  
 a =1 
 ∂ξ ∂η ∂ζ 
 ∂x ∂x ∂x  
  r r r ( a )  
∂ζ   ∂ N (ξ ) y ( a )   n   n
n
 ∂ξ
∂y
∂η
∂y ∂y    a =1

a 


∂ N a (ξ ) y ( a ) 
 a =1 

∂ N a (ξ ) y  
 a =1 
Fij =  = (6.199)
 ∂ξ ∂η ∂ζ   ∂ξ ∂η ∂ζ 
 ∂z ∂z ∂z    n r r r (a )  
   n   n
 ∂ξ ∂η ∂ζ  
 
 a =1

 ∂ N a (ξ ) z ( a ) 



∂ N a (ξ ) z ( a ) 
 a =1 

∂ N a (ξ ) z  
 a =1 
 
 ∂ξ ∂η ∂ζ 
 
 
which can also be expressed as follows:
r r r
 ∂N a (ξ ) ( a ) ∂N a (ξ ) ( a ) ∂N a (ξ ) ( a ) 
 x x x 
 ∂ξ r ∂η r ∂ζ r 
n 
r ∂N (ξ ) ( a ) ∂N a (ξ ) ( a ) ∂N a (ξ ) ( a ) 
Fij (ξ ) = ∑  a
a =1  ∂ξ r
y
∂η r
y
∂ζ r
y 

(6.200)
 ∂N (ξ ) ∂N a (ξ ) ( a ) ∂N a (ξ ) ( a ) 
 a z (a ) z z 
 ∂ξ ∂η ∂ζ 
r
To obtain the matrix [B( x )] we have to calculate the derivative of the shape functions
r r
N ( x ) with respect to ( x ) . Recall that given a scalar field in the Reference configuration

University of Castilla-La Mancha Draft By: Eduardo W. V. Chaves (2016)


Ciudad Real - Spain
654 SOLVING PROBLEMS BY MEANS OF CONTINUUM MECHANICS

r r
φ ( X ) and the same scalar field defined in the Current configuration φ ( x ) the follow is
true:
r r r r r
r ∂φ ( X ) ∂φ ( X ( x )) ∂x ∂φ ( x ) r
∇ Xr φ ( X ) ≡ r = r ⋅ r = r ⋅ F = ∇ xrφ ( x ) ⋅ F
∂X ∂x ∂X ∂x
and
r r r r r
∂φ ( x ) ∂φ ( x ( X )) ∂ ∂φ r r
⋅ r = r ) ⋅ F −1 = {∇ Xr φ ( X )} ⋅ F −1 = F −T ⋅ {∇ Xr φ ( X )}
X ( X
r
∇ xrφ ( x ) ≡ r = r
∂x ∂X ∂x ∂X
r r r r
So, if we consider that φ ( x ) = N a ( x ) and X = ξ we can conclude that:
r r r r r r
r ∂ ( N ( x )) ∂ ( N ( x (ξ ))) ∂ξ ∂ ( N (ξ )) ∂ ( N (ξ ))
∇ xr ( N a ( x )) = a
r = a
r ⋅ r= a
r ⋅F = F ⋅
−1 − T a
r (6.201)
∂x ∂ξ ∂x ∂ξ ∂ξ
which in indicial notation becomes:
r r r r r
r ∂ ( N a ( x )) ∂ ( N a ( x (ξ ))) ∂ξ k ∂( N a (ξ )) −1 −1 ∂ ( N a (ξ ))
( N a ( x )),i = = = Fki = Fki (6.202)
∂xi ∂ξ k ∂xi ∂ξ k ∂ξ k
More explicitly we have:
r r r r
r ∂( N a (ξ )) −1 ∂ ( N a (ξ )) −1 ∂( N a (ξ )) −1 ∂ ( N a (ξ )) −1
( N a ( x )),i = Fki = F1i + F2i + F3i
∂ξ k ∂ξ 1 ∂ξ 2 ∂ξ 3
r r r (6.203)
∂( N a (ξ )) −1 ∂ ( N a (ξ )) −1 ∂( N a (ξ )) −1
= F1i + F2i + F3i
∂ξ ∂η ∂ζ
The equation (6.202) can also be expressed in matrix form as follows:
r
r
 ∂ ( N a ( x ))   ∂ ( N a (ξ )) 
   
 ∂ ξ r 
 ∂x1 r  r r r

1

 ∂ ( N a ( x ))   ∂ ( N a (ξ )) ∂ ( N a (ξ )) ∂ ( N a (ξ ))  −1 −1 T  ∂ ( N (ξ )) 
= [ Fki ] = [ Fki ] 
a
  (6.204)
 ∂x2 r   ∂ξ 1 ∂ξ 2 ∂ξ 3   ∂ξ 2r 
 ∂ ( N a ( x ))   ∂ ( N a (ξ )) 
 ∂x   
 3   ∂ξ 3 
For two-dimensional case we have:
r r
 ∂ ( N a ( x ))   ∂ ( N a (ξ )) 
 ∂x   
−1 T  ∂ξ 1r 
 ∂ ( N ( xr ))  = [ Fki ]
1
 
 a   ∂ ( N a (ξ )) 
 ∂x2   ∂ξ 
 2 
r (6.205)
r
 ∂ ( N a ( x ))   ∂y ∂y   ∂( N a (ξ )) 
−  
  1  ∂η ∂ξ  ∂ξ r 
⇒  ∂ ( N∂x( xr ))  =  
∂x ∂x   ∂( N (ξ )) 
 a
 J −  a 
 ∂y   ∂η ∂ξ   ∂η 

where we have used the equation in (6.196) in order to express [ Fki−1 ]T for 2D case.

University of Castilla-La Mancha Draft By: Eduardo W. V. Chaves (2016)


Ciudad Real – Spain
6 LINEAR ELASTICITY 655

NOTE 7: Introduction to Numerical Integration (or Quadrature)


x( f )
Let us consider the function f (x) and we want to obtain the integral ∫ f ( x)dx . If the
x (i )
function varies linearly into the interval we can use the shape functions, given by (6.171), to
2
approach the function, i.e. f ( x) = f1N1 ( x) + f 2 N 2 ( x) = ∑ f a N a ( x) , so, the integral can be
a =1
represented as follows:
x( f ) x( f ) 2  x( f )
2  2

∫ f ( x)dx = ∫∑ f a N a ( x)dx =  f a N a ( x)dx  =


∑ ∫ ∑( f P )

a =1 
 a a
x (i )
x (i ) a =1 x (i )  a =1

x( f )
where we have considered that Pa = ∫ N ( x)dx . Note that we can use the normalized space
a
x (i )
to solve Pa . Then, by considering the equations (6.172) and (6.173) we can obtain that:
x( f ) 1 1
( x ( f ) − x (i ) ) ( x ( f ) − x (i ) ) ( x ( f ) − x (i ) )
Pa = ∫ N a ( x)dx = N a (ξ ) ∫
−1
2
dξ =
2 −1

N a (ξ )dξ =
2
Wa
x(i )

Note also that


1 1 1 1
(1 − ξ ) (1 + ξ )

W1 = N 1 (ξ )dξ =
−1

−1
2
dξ = 1 ; ∫
W2 = N 2 (ξ )dξ =
−1

−1
2
dξ = 1

Then, we can conclude that:


x( f ) 2 2
 ( x( f ) − x(i ) )  ( x( f ) − x(i ) ) 2
∫ f ( x)dx = ∑ ( f a Pa ) = ∑  fa

a =1  2
Wa  =
2
( f aWa ) ∑
x (i ) a =1  a =1

If we are using a quadratic function, (see equations in (6.170)), to approach the function
f (x) , the parameters Wa ( a = 1,2,3 ) can be obtained as follows:
1 1 1 1
ξ  1 4
−1

W1 = N1 (ξ )dξ =  (ξ − 1)  dξ =
−1
2  3 ∫ ; ∫
W2 = N 2 (ξ )dξ = (1 − ξ 2 )dξ =
−1

−1
3
1 1
ξ  1
−1

W3 = N 3 (ξ )dξ =  (1 + ξ ) dξ =
−1
2  3 ∫
3 3
For example, let us obtain numerically the integral ∫
2

f ( x)dx = (4 x 2 + 5 x)dx = 37.833 . Let
2
us consider that
x( f ) 3
( x ( f ) − x (i ) ) ( x ( f ) − x (i ) )
∫ f ( x)dx =
2
∑a =1
( f aWa ) =
2
( f1W1 + f 2W2 + f 3W3 )
x( i )

where
f1 = f ( x = 2) = 4 × (2)2 + 5 × (2) = 26 ; f 2 = f ( x = 2.5) = 4 × (2.5) 2 + 5 × (2.5) = 37.5

f 3 = f ( x = 3) = 4 × (3) 2 + 5 × (3) = 51
Then

University of Castilla-La Mancha Draft By: Eduardo W. V. Chaves (2016)


Ciudad Real - Spain
656 SOLVING PROBLEMS BY MEANS OF CONTINUUM MECHANICS

x( f )
( x ( f ) − x (i ) ) (3 − 2)  1 4 1
∫ f ( x)dx =
2
( f1W1 + f 2W2 + f3W3 ) =
2  3
 26 + 37.5 + 51  = 37.833
3 3
x(i )

which matches the exact solution. This particular numerical integration is known as Newton-
Cotes formula. There are other quadratures more suitable, (Bathe (1996)), e.g. Gauss-Legendre
Quadrature which is widely used in the Finite Element Technique in order to obtain
r r
numerically the integrals, for example, the stiffness matrix [k ( e ) ] = ∫ [B( x )]T [C ] [B( x )] dV .
V

In general, the numerical integration for 1D, 2D and 3D, in the Normalized Space, are
represented respectively by:
np

∫ f (ξ )dξ = ∑W
L0 p =1
p f (ξ p )

nq np

∫ f (ξ ,η ,ζ )dξdη = ∑∑W W
A0 q =1 p =1
q p f (ξ p ,η q ) (6.206)

nr nq np

∫ f (ξ ,η ,ζ )dξdηdζ = ∑∑∑W W W
V0 r =1 q =1 p =1
r q p f (ξ p ,η q ,ζ r )

where n p , nq and nr are the number of integration points according to the directions ξ ,
η and ζ , respectively. The weight Wi and the point (ξ p ,η q ,ζ r ) in which the function is
evaluated f (ξ p ,η q ,ζ r ) will depend on the numerical technique employed. For example, by
using the Gauss-Legendre interpolation the integration points and the weights are given by
Table 6.2, (Bathe (1996), Oñate (1992), Chaves&Mínguez(2009)).

University of Castilla-La Mancha Draft By: Eduardo W. V. Chaves (2016)


Ciudad Real – Spain
6 LINEAR ELASTICITY 657

Table 6.2: Integration points and weights by using Gauss-Legendre Quadrature.


1 np

∫ f (ξ )dξ = ∑W
p =1
p f (ξ p )
−1

Integration points - ξ p Weights - W p


np = 1
0.00000 00000 00000 2.00000 00000 00000
np = 2
± 0.57735 02691 89626 1.00000 00000 00000
np = 3
0.00000 00000 00000 0.88888 88888 88888
± 0.77459 66692 41483 0.55555 55555 55555
np = 4
± 0.33998 10435 84856 0.65214 51548 61630
± 0.86113 63115 94053 0.34785 48451 37448
np = 5
0.00000 00000 00000 0.56888 88888 88889
± 0.53846 93101 05683 0.47862 86704 86297
± 0.90617 98459 38664 0.23692 68850 56182
np = 6
± 0.23861 91860 83197 0.46791 39345 72689
± 0.66120 93864 66264 0.36076 15730 13980
± 0.93246 95142 03152 0.17132 44923 79162
np = 7
0.00000 00000 00000 0.41795 91836 73469
± 0.40584 51513 77397 0.38183 00505 05069
± 0.74153 11855 99394 0.27970 53914 37510
± 0.94910 79123 42758 0.12948 49661 68862
np = 8
± 0.18343 46424 95650 0.36268 37833 78362
± 0.52553 24099 16329 0.31370 66458 77676
± 0.79666 64774 13627 0.22238 10344 53374
± 0.96028 98564 97536 0.10122 85362 90370

University of Castilla-La Mancha Draft By: Eduardo W. V. Chaves (2016)


Ciudad Real - Spain
658 SOLVING PROBLEMS BY MEANS OF CONTINUUM MECHANICS

Problem 6.41
Returning to three-dimensional problem, obtain the explicit form for the stiffness matrix,
from the relationship { f ( e ) } = [k ( e ) ]{u (e ) } , by considering the tetrahedron as sub-domain,
(see Figure 6.52). Consider that the displacement fields are approached by considering a
r
linear function {u( x )} = [ N ]{u(e ) } , (see equation (6.164)). Consider also the Orthotropic
Symmetry for the elasticity tensor.
Hint: Use the information given by NOTE 3.1 in Problem 6.40.
Solution:
r r
We have to obtain the explicit form of [k ( e ) ] = ∫ [B( x )]T [C ] [B( x )] dV .
V
r
The relationship between the infinitesimal strain field {ε ( x )} and the nodal displacement
r r r
{u (e ) } is given by {ε ( x )} = [ L(1) ] {u( x )} = [ L(1) ][ N ]{u( e ) } = [B( x )] {u( e ) } , (see Problem 5.8):

 ∂u   ∂ 
 ∂x   ∂x 0 0
 ∂v   ∂ 
 εx    0 0
 ε   ∂y   ∂y 
  y  ∂w   ∂   u ( xr ) 
ε    0 0 
{ε } =  z  =  ∂u ∂z ∂v  =  ∂ ∂ ∂z   v( xr ) 
γ xy   +   0  w( xr ) 
γ yz   ∂y ∂x   ∂y ∂x  
   ∂v ∂w   ∂ ∂ 
 γ xz   +   0 ∂z ∂y 
 ∂z ∂y   
 ∂u + ∂w   ∂ 0
∂
 ∂z ∂x   ∂z ∂x 
r r
and by considering the displacement field {u( x )} = [ N ( x )]{u(e ) } , (see equation (6.164)), we
can obtain:
 u (1) 
 (1) 
∂  v 
 ∂x 0 0
 w(1) 
 ∂   (2) 
0 0 u 
 ∂y   v( 2) 
 ∂ N 0 0 N2 0 0 N3 0 0 N4 0 0   ( 2) 
0 0  1 w 
{ε } =  ∂ ∂z   0 N1 0 0 N2 0 0 N3 0 0 N4 0   (3) 
 ∂ 
0  0 0 N1 0 0 N2 0 0 N3 0 0
u
N 4   ( 3) 
 ∂y ∂x  v 
 ∂ ∂  ( 3) 
0  w 
 ∂z ∂y   u (4) 
∂ 0
∂  
 ∂z ∂x   v( 4) 
 ( 4) 
w 
r
Then, the [B( x )] matrix becomes

University of Castilla-La Mancha Draft By: Eduardo W. V. Chaves (2016)


Ciudad Real – Spain
6 LINEAR ELASTICITY 659

∂ 
 ∂x 0 0
 ∂ 
0 0
 ∂y 
 ∂ N 0 0 N2 0 0 N3 0 0 N4 0 0 
0 0  1
r
[B( x )] =  ∂ ∂z   0 N1 0 0 N2 0 0 N3 0 0 N4 0 
 ∂ 
0  0 0 N1 0 0 N2 0 0 N3 0 0 N 4 
 ∂y ∂x 
 ∂ ∂
0 
 ∂z ∂y 
∂ ∂
 ∂z 0
∂x 
 ∂N 1 ∂N 2 ∂N 3 ∂N 4 
 ∂x 0 0 0 0 0 0 0 0 
∂x ∂x ∂x
 ∂N 1 ∂N 2 ∂N 3 ∂N 4 
 0 0 0 0 0 0 0 0 
 ∂y ∂y ∂y ∂y 
 ∂N 1 ∂N 2 ∂N 3 ∂N 4 
 0 0 0 0 0 0 0 0 
[B] =  ∂N ∂z ∂z ∂z ∂z 
 1 ∂N 1 ∂N 2 ∂N 2 ∂N 3 ∂N 3 ∂N 4 ∂N 4
0 0 0 0 
 ∂y ∂x ∂y ∂x ∂y ∂x ∂y ∂x 
 ∂N 1 ∂N 1 ∂N 2 ∂N 2 ∂N 3 ∂N 3 ∂N 4 ∂N 4 
 0 0 0 0
 ∂z ∂y ∂z ∂y ∂z ∂y ∂z ∂y 
 ∂N 1 ∂N 1 ∂N 2 ∂N 2 ∂N 3 ∂N 3 ∂N 4 ∂N 4 
 0 0 0 0 
 ∂z ∂x ∂z ∂x ∂z ∂x ∂z ∂x 

⇒ [B] = [[B1 ]6×3 [B2 ]6×3 [B3 ]6×3 [B4 ]6×3 ] (6.207)

The matrix [Bi ]6×3 from the equation (6.207) can be rewritten as follows:
 ∂N i 
 ∂x 0 0 
 ∂N i 
 0 0   ai 0 0
 ∂y  0
 ∂N i   bi 0 
 0 0 
[Bi ]6×3 =  ∂N ∂z  = 1  0 0 ci 
∂N i  
 i 0  6V  bi ai 0
 ∂y ∂x  0 ci bi 
 ∂N i ∂N i   
 0   ci 0 ai 
 ∂z ∂y 
 ∂N i ∂N i 
 ∂z 0
 ∂x 
r
As we will see later, the matrices [Bi ] are constant, i.e. it is independent of x , then

University of Castilla-La Mancha Draft By: Eduardo W. V. Chaves (2016)


Ciudad Real - Spain
660 SOLVING PROBLEMS BY MEANS OF CONTINUUM MECHANICS


[k ( e ) ] = [B]T [C ] [B] dV
V

 [B1 ]T  
  
 [B2 ]T  

(e)
⇒ [k ] =  [ C ] [[ B ] [ B ] [ B ] [ B ]]  dV
[B ]T  1 2 3 4
V 3 
 
 [B ]T  
 4  
 [B1 ]T [C ]  
  
 [B2 ]T [C ] 

(e)
⇒ [k ] =  [[B ] [B2 ] [B3 ] [B4 ]]  dV
[B ]T [C ] 1
V 
 3 
 [B ]T [C ] 
 4  
 [B1 ]T [C ][B1 ] [B1 ]T [C ][B2 ] [B1 ]T [C ][B3 ] [B1 ]T [C ][B4 ]  
  
 [B ]T [C ][B1 ] [B2 ]T [C ][B2 ] [B2 ]T [C ][B3 ] [B2 ]T [C ][B4 ] 
⇒ [k ] =   2 T
(e)
∫  dV
[B ] [C ][B ] [B3 ]T [C ][B2 ] [B3 ]T [C ][B3 ] [B3 ]T [C ][B4 ]
V 
 3 1

 [B ]T [C ][B ] [B4 ]T [C ][B2 ] [B4 ]T [C ][B3 ] [B4 ]T [C ][B4 ] 
 4 1 


⇒ [k ( e ) ] = [[Bi ]T [C ][B j ]] dV
V
(i, j = 1,2,3,4)

V

⇒ [k ( e ) ] = [[Bi ]T [C ][B j ]] dV = V [[Bi ]T [C ][B j ]] (i, j = 1,2,3,4)

By considering the Orthotropic Symmetry for the elasticity tensor:


 C11 C12 C13 0 0 0 
C C22 C23 0 0 0 
 12
 C13 C23 C33 0 0 0 
[C ] =  
0 0 0 C44 0 0 
0 0 0 0 C55 0 
 
 0 0 0 0 0 C55 
we can obtain:
 C11 C12 C13 0 0 0  a j 0 0
C  0 
C22 C23 0 0 0   0 bj
ci  
12
 ai 0 0 bi 0
1   C C23 C33 0 0 0  0 0 cj 
T
[Bi ] [C ][B j ] = 0 bi 0 ai ci 0   13  
36V 2  0 0 0 C44 0 0  b j aj 0
 0 0 ci 0 bi ai  
 0 0 0 0 C55 0  0 cj bj 
  
 0 0 0 0 0 C55   c j 0 a j 
1
[Bi ]T [C ][B j ] = [k ij ] (i, j = 1,2,3,4)
36V 2
where
 ai C11a j + bi C44b j + ci C66 c j ai C12b j + bi C44 a j ai C13c j + ci C66 a j 
 
[k ij ]3×3 = bi C12 a j + ai C44b j bi C22b j + ai C44 a j + ci C55 c j bi C23c j + ci C55b j 
 ci C13 a j + ai C66 c j ci C23b j + bi C55c j ci C33c j + bi C55b j + ai C66 a j 

University of Castilla-La Mancha Draft By: Eduardo W. V. Chaves (2016)


Ciudad Real – Spain
6 LINEAR ELASTICITY 661

Then, the stiffness matrix becomes


[k11 ] [k12 ] [k13 ] [k14 ]
 
V 1 [k 21 ] [k 22 ] [k 23 ] [k 24 ]
[k ( e ) ]12×12 = V [[Bi ]T [C ][B j ]] = [ k ] =
36V [k 31 ] [k 32 ] [k 33 ] [k 34 ]
ij
36V 2
 
[k 41 ] [k 42 ] [k 43 ] [k 44 ]

Now we have to obtain the derivatives of [N ] in order to derive the coefficients a i , bi and
ci . The shape functions for the linear tetrahedron were obtained in equation (6.163). Note
that the shape function N1 can be rewritten as follows:
1 x y z 1 x y z
1 x (2) y (2)
z ( 2)
1 x (2) y (2)
z ( 2)

( 3) ( 3) ( 3) ( 3) ( 3)
1 x y z 1 x y z ( 3) 1 x y z
1 x (4) y (4) z ( 4) 1 x (4) y (4) z ( 4) 1 x (2) y (2)
z ( 2)
N1 = = ⇒ 6VN1 = ( 3) ( 3)
1 x (1)
y (1)
z (1) 6V 1 x y z ( 3)
1 x (2) y (2) z ( 2) 1 x (4) y (4) z ( 4)
1 x ( 3) y ( 3) z (3)
1 x (4) y (4) z ( 4)
x (2) y (2) z ( 2) 1 y ( 2) z ( 2) 1 x ( 2) z ( 2) 1 x ( 2) y ( 2)
⇒ 6VN1 = x (3) y ( 3) z ( 3) − x 1 y ( 3) z ( 3) + y 1 x ( 3) z ( 3) − z 1 x ( 3) y ( 3)
x (4) y (4) z ( 4) 1 y ( 4) z ( 4) 1 x ( 4) z ( 4) 1 x ( 4) y ( 4)

Then, the derivatives are given by:


1 y ( 2) z ( 2) 1 x (2) z ( 2) 1 x ( 2) y ( 2)
∂N1 ∂N1 ∂N1
6V = − 1 y ( 3) z (3) = a1 ; 6V = 1 x ( 3) z (3) = b1 ; 6V = − 1 x ( 3) y (3) = c1
∂x ∂y ∂z
1 y ( 4) z ( 4) 1 x (4) z ( 4) 1 x ( 4) y ( 4)
The shape function N 2 , (see equation (6.163)), can be rewritten as follows:
x (1) y (1) z (1) 1 y (1) z (1) 1 x (1) z (1) 1 x (1) y (1)
6VN 2 = − x (3) y ( 3) z ( 3) + x 1 y ( 3) z ( 3) − y 1 x ( 3) z ( 3) + z 1 x ( 3) y ( 3)
x (4) y (4) z ( 4) 1 y (4) z ( 4) 1 x ( 4) z (4) 1 x (4) y (4)

and the derivatives:


1 y (1) z (1) 1 x (1) z (1) 1 x (1) y (1)
∂N 2 ∂N 2 ∂N 2
6V = 1 y (3) z ( 3) = a2 ; 6V = − 1 x ( 3) z ( 3) = b2 ; 6V = 1 x ( 3) y ( 3) = c 2
∂x ∂y ∂z
1 y ( 4) z ( 4) 1 x ( 4) z (4) 1 x ( 4) y ( 4)

The shape function N3 , (see equation (6.163)), can be rewritten as follows:


x (1) y (1) z (1) 1 y (1) z (1) 1 x (1) z (1) 1 x (1) y (1)
6VN 3 = x (3) y ( 3) z ( 3) − x 1 y ( 2 ) z ( 2 ) + y 1 x ( 2) z ( 2 ) − z 1 x ( 2) y (2)
x (4) y (4) z ( 4) 1 y ( 4) z ( 4) 1 x ( 4) z ( 4) 1 x ( 4) y (4)

and the derivatives:

University of Castilla-La Mancha Draft By: Eduardo W. V. Chaves (2016)


Ciudad Real - Spain
662 SOLVING PROBLEMS BY MEANS OF CONTINUUM MECHANICS

1 y (1) z (1) 1 x (1) z (1) 1 x (1) y (1)


∂N 3 ∂N 3 ∂N 3
6V = − 1 y ( 2) z ( 2 ) = a3 ; 6V = 1 x ( 2) z ( 2 ) = b3 ; 6V = − 1 x (2) y ( 2 ) = c3
∂x ∂y ∂z
1 y ( 4) z (4) 1 x ( 4) z ( 4) 1 x (4) y (4)

The shape function N 4 , (see equation (6.163)), can be rewritten as follows:


x (1) y (1) z (1) 1 y (1) z (1) 1 x (1) z (1) 1 x (1) y (1)
6VN 4 = − x (3) y ( 3) z ( 3) + x 1 y ( 2 ) z ( 2) − y 1 x ( 2) z (2) + z 1 x (2) y (2)
x (4) y (4) z ( 4) 1 y ( 3) z ( 3) 1 x ( 3) z ( 3) 1 x ( 3) y ( 3)

and the derivatives:


1 y (1) z (1) 1 x (1) z (1) 1 x (1) y (1)
∂N 4 ∂N 4 ∂N 4
6V = 1 y ( 2) z ( 2 ) = a4 ; 6V = − 1 x ( 2) z ( 2 ) = b4 ; 6V = 1 x ( 2) y ( 2 ) = c4
∂x ∂y ∂z
1 y ( 3) z ( 3) 1 x ( 3) z ( 3) 1 x ( 3) y ( 3)

Then, we summarize the coefficients as follows:


1 y ( 2) z ( 2) 1 x ( 2) z ( 2) 1 x ( 2) y ( 2)
a1 = − 1 y (3) z ( 3) ; b1 = 1 x (3) z ( 3) ; c1 = − 1 x (3) y ( 3)
1 y ( 4) z ( 4) 1 x ( 4) z ( 4) 1 x ( 4) y ( 4)

1 y (1) z (1) 1 x (1) z (1) 1 x (1) y (1)


a 2 = 1 y ( 3) z ( 3) ; b2 = − 1 x (3) z ( 3) ; c 2 = 1 x ( 3) y (3)
1 y ( 4) z (4) 1 x ( 4) z ( 4) 1 x ( 4) y ( 4)

1 y (1) z (1) 1 x (1) z (1) 1 x (1) y (1)


a3 = − 1 y ( 2 ) z ( 2) ; b3 = 1 x ( 2) z (2) ; c3 = − 1 x ( 2) y ( 2)
1 y (4) z ( 4) 1 x ( 4) z (4) 1 x ( 4) y ( 4)

1 y (1) z (1) 1 x (1) z (1) 1 x (1) y (1)


a4 = 1 y ( 2 ) z ( 2) ; b4 = − 1 x ( 2) z (2) ; c4 = 1 x ( 2 ) y (2)
1 y ( 3) z ( 3) 1 x ( 3) z ( 3) 1 x ( 3) y ( 3)

Problem 6.42
Obtain the displacements for the structure described in Figure 6.78 which can be
approached by the state of plane stress. As mechanical properties consider that E = 10
(Young’s modulus) and ν = 0.0 (Poisson’s ratio). Consider that the domain is discretized
by two triangles with 3-nodes (CST), (see Figure 6.78).
NOTE: Note that this an academic example, since due to the fact that the thickness is not
very small so that in a real practical case we cannot adopt the state of plane stress.
Solution:
For the problem presented here we have 2 degrees-of-freedom per node, and a total of 8
degrees-of-freedom, (see Figure 6.79), so, the discrete system which represents this
problem is given by:
{F }8×1 = [ K ]8×8 {U }8×1 (6.208)

University of Castilla-La Mancha Draft By: Eduardo W. V. Chaves (2016)


Ciudad Real – Spain
6 LINEAR ELASTICITY 663

y
t = 2 (thickness)

Element A t connectivity
2 4 F = 10 1 0 .5 2 1− 4 − 2
2 0 .5 2 1− 3 − 4
1

1 Element connectivity: counterclockwise


2
F = 10

1 3 x
1

Figure 6.78: Dicretization by using two-CST elements.

y, v n – nodes

e – elements Displacement vector


v ( 2) v ( 4)
4 8
3 7 U 1   v (1) 
U   (1) 
2 u ( 2) 4 u (4)  2  v 
U 3  u ( 2 ) 
   ( 2) 
1 U  v 
{U } =  4  =  (3) 
2 U 5  u 
v (1) U 6   v (3) 
2 v ( 3) 6    ( 4) 
1 5 U 7  u 
U  v ( 4 ) 
1 u (1)
3 u (3) x, u  8  

Figure 6.79: Degrees-of-freedom – Global system.

The global stiffness matrix and the global force vector are obtained by the contribution of
each finite element and they are represented, respectively, by:
N elem N elem

A [k ( e) ] A[ f
(e)
[K ] = ; {F } = ] (6.209)
e =1 e =1

where A stands for assemble operator. For this problem, we only have nodal forces, so,
the global force vector is given by:

University of Castilla-La Mancha Draft By: Eduardo W. V. Chaves (2016)


Ciudad Real - Spain
664 SOLVING PROBLEMS BY MEANS OF CONTINUUM MECHANICS

global degrees - of - freedom


14444244443

0 1
0 2
 
0 3
 
0 4 (6.210)
{F } =  
10 5
0 6
 
10 7
0 8
 
The elasticity matrix for the state of plane stress is given by:
 
 C11 C12 0 
E 1 ν 0  10 0 0
[C] = C12 C22 0  = ν 1 0  =  0 10 0 (6.211)
1−ν 2  1−ν  
 0 0 C33  0 0   0 0 5
 2 
The stiffness matrix for CST-element can be obtained by means of equation (6.157), i.e.:
 k11( e ) k12( e ) k13( e ) k14( e ) k15( e ) k16( e ) 
 
 k22( e ) k23( e ) k24( e ) k25( e ) k26( e ) 
[k11 ] [k12 ] [k13 ]
t (e)   t (e)  k33( e ) k34( e ) k35( e ) k36( e ) 
[k ( e ) ]6×6 = [k 21 ] [k 22 ] [k 23 ] =   (6.212)
4 A( e ) [k ] [k ] [k ] 4 A
(e)
 k44( e ) k45( e ) k46( e ) 
 31 32 33   symmetric k55( e ) k56( e ) 
 
 k66( e ) 
where
k11(e ) = a12 C11 + b12 C33 ; k12( e ) = a1C12b1 + b1C33 a1 ; k13( e ) = a1C11a2 + b1C33b2 ;
k14(e ) = a1C12b2 + b1C33 a2 ; k15( e ) = a1C11a3 + b1C33b3 ; k16( e ) = a1C12b3 + b1C33 a3 ;
k 22(e ) = b12 C22 + a12 C33 ; k 23( e ) = b1C12 a2 + a1C33b2 ; k 24( e ) = b1C22b2 + a1C33 a2 ;
k 25(e ) = b1C12 a3 + a1C33b3 ; k 26( e ) = b1C22b3 + a1C33 a3 ; k33( e ) = a22 C11 + b22 C33 ; (6.213)
k34(e ) = a2 C12b2 + b2 C33 a2 ; k35( e ) = a2 C11a3 + b2 C33b3 ; k36( e ) = a2 C12b3 + b2 C33a3 ;
k 44(e ) = b22 C22 + a22 C33 ; k 45( e ) = b2 C12 a3 + a2 C33b3 ; k 46( e ) = b2 C22b3 + a2 C33 a3 ;
k55(e ) = a32 C11 + b32 C33 ; k56( e ) = a3 C12b3 + b3 C33 a3 ; k 66( e ) = b32 C22 + a32 C33 ;
with
a1 = y ( 2 ) − y (3) ; a2 = y (3) − y (1) ; a3 = y (1) − y ( 2 ) ;
(6.214)
b1 = x (3) − x ( 2) ; b2 = x (1) − x (3) ; b3 = x ( 2 ) − x (1) .
Element – 1
t (e=1) 2
( e =1)
= =1
4A 4 × 0 .5
x (1) = 0; y (1) = 0;
 a1 = 0; a2 = 1; a3 = −1;
x ( 2 ) = 1; y ( 2 ) = 1;  ⇒
b1 = −1; b2 = 0; b3 = 1.
x (3) = 0; y (3) = 1; 

University of Castilla-La Mancha Draft By: Eduardo W. V. Chaves (2016)


Ciudad Real – Spain
6 LINEAR ELASTICITY 665

1 2 7 8 3 4 Global
k11(1) k12(1) k13(1) k14(1) k15(1) k16(1)  1
 (1) (1) (1) (1) (1) 
 k 22 k 23 k 24 k 25 k 26  2
 (1)
k33 (1)
k34 (1)
k35 (1) 
k36 7
[k (1) ]6×6 = (1) (1) (1)

 k 44 k 45 k 46  8
 symmetric (1)
k55 (1) 
k56 3
 (1)

 k66  4
(6.215)
5 0 0 − 5 − 5 5 
 10 0 0 0 − 10

 10 0 − 10 0 
= 
 5 5 −5 
 symmetric 15 −5 
 
 15 
Element – 2
t (e = 2) 2
(e = 2)
= =1
4A 4 × 0 .5
x (1) = 0; y (1) = 0; 
 a1 = −1; a2 = 1; a3 = 0;
x ( 2 ) = 1; y ( 2 ) = 0; ⇒
b1 = 0; b2 = −1; b3 = 1.
x (3) = 1; y (3) = 1; 

1 2 5 6 7 8 Global
k11( 2 ) k12( 2) k13( 2 ) k14( 2 ) k15( 2 ) k16( 2 )  1
 ( 2) ( 2) ( 2) ( 2) (2) 
 k 22 k 23 k 24 k 25 k 26  2
 ( 2)
k33 ( 2)
k34 ( 2)
k35 (2) 
k36 5
[k ( 2) ]6×6 = ( 2) ( 2) (2)

 k 44 k 45 k 46  6
 symmetric ( 2)
k55 (2) 
k56 7
 (2)

 k 66  8
(6.216)
10 0 − 10 0 0 0 
 5 5 −5 5 0 

 15 − 5 − 5 0 
= 
 15 5 − 10
 symmetric 5 0 
 
 10 

University of Castilla-La Mancha Draft By: Eduardo W. V. Chaves (2016)


Ciudad Real - Spain
666 SOLVING PROBLEMS BY MEANS OF CONTINUUM MECHANICS

Global Stiffness matrix


1 2 3 4 5 6 7 8 Global
 k11(1) + k11( 2 ) k12(1) + k12( 2 ) k15(1) k16(1) k13( 2 ) k14( 2 ) k13(1)
+ k15( 2 ) k14(1)+ k16( 2 )  1
 (1) ( 2) (1) (1) ( 2) (2) (1) ( 2) (1) (2) 
 k 22 + k 22 k 25 k 26 k 23 k 24 k 23 + k 25 k 24 + k 26  2
 (1)
k55 (1)
k56 0 0 (1)
k35 (1)
k 45 + k36(1) 
3
 (1) (1) (1)

 k66 0 0 k36 k 46  4
[K ] =  ( 2) (2) (2) ( 2) 
symmetric k33 k34 k35 k36 5
 
(2) (2) ( 2)
 k 44 k 45 k 46  6
 (1) ( 2) (1) (2) 
 k33 + k55 k34 + k56  7
 (1)
k 44 + k 66(2) 
 8

(6.217)

15 0 − 5 5 − 10 0 0 − 5  U 1   0 

 15 0 − 10 5 −5 5 0  U 2   0 
 15 − 5 0 0 − 10 5  U 3   0 
    
 15 0 0 0 − 5  U 4   0 
{F }8×1 = [ K ]8×8 {U }8×1 ⇒
  = 
15 − 5 − 5 0  U 5  10
 
 15 5 − 10 U 6   0 
    
symmetric 15 0  U 7  10
 
 15  U 8   0 
By applying the boundary conditions we have
1 0 0 0 0 0  U 1   0 
0 U 1   u (1)  0
0
0 1 0 0 0    
0  U 2   0 
0  U   (1)  0
0
  2  v   
0 0 1 0 0 0 0 0  U 3   0  U 3  u ( 2 )  0
          
0 0 0 15 0 0 0 − 5  U 4   0  Solve U 4  v ( 2 )  0
0   =   →  =  (3)  =  
0 0 0 15 − 5 − 5 0  U 5  10 U 5  u  1
 
0 0 0 0 − 5 15 5 − 10 U 6   0  U 6   v (3)  0
0        ( 4)   
0 0 0 −5 5 15 0  U 7  10 U 7  u  1
 
− 5 0 − 10 0 15  U 8   0  U  v ( 4 )  0
0 0 0  8    
It is interesting to compare with the result presented in equation (6.113) - Problem 6.34:
 ( 3) ( 4) F 20
u ( x = 1) = u = u = EA x = 10 × 2 ×1 = 1

v( x = 1) = v (3) = v ( 4 ) = − ν F y = 0
 EA
here A = t × 1 = 2 is cross section area and F = 2 × 10 = 20 is the total force.

University of Castilla-La Mancha Draft By: Eduardo W. V. Chaves (2016)


Ciudad Real – Spain
6 LINEAR ELASTICITY 667

Problem 6.43
Obtain the explicit formation for the stiffness matrix for the element presented in Figure
6.58.
Solution:
The displacement field is given by:
 u (1) 
 (1) 
v 
u ( 2 ) 
 
u ( x, y )  N1 0 N2 0 N3 0 N4 0  v ( 2) 
 =   ⇔ {u( x, y )} = [ N ]{u ( e ) }
 v ( x, y )   0 N1 0 N2 0 N3 0 N 4  u (3) 
 v ( 3) 
 ( 4) 
u 
v ( 4) 
 
And the shape functions are:
1 1
N1 = (ab − bx − ay + xy ); N 2 = (bx − xy);
ab ab
1 1
N3 = xy; N4 = (ay − xy ).
ab ab
r
The [B( x )] can be obtained as follows:
∂ 
 0
 ∂x 
r ∂   N1 0 N2 0 N3 0 N4 0
[B( x )] =  0 
 ∂y   0 N1 0 N2 0 N3 0 N 4 
∂ ∂
 
 ∂y ∂x 
 ∂N1 ∂N 2 ∂N 3 ∂N 4 
 0 0 0 0 
 ∂x ∂x ∂x ∂x 
∂N1 ∂N 2 ∂N 3 ∂N 4 
= 0 0 0 0
 ∂y ∂y ∂y ∂y 
 ∂N ∂N1 ∂N 2 ∂N 2 ∂N 3 ∂N 3 ∂N 4 ∂N 4 
 1 
 ∂y ∂x ∂y ∂x ∂y ∂x ∂y ∂x 
y −b 0 b− y 0 y 0 −y 0 
1 
= 0 x−a 0 −x 0 x 0 a − x 
ab 
 x − a y − b − x b − y x y a − x − y 
 g1 0 g 2 0 g 3 0 g 4 0 
1 
= 0 h1 0 h2 0 h3 0 h4 
ab 
 h1 g1 h2 g 2 h3 g 3 h4 g 4 

⇒ [B] = [[B1 ]3×2 [B2 ]3×2 [B3 ]3×2 [B4 ]3×2 ] (6.218)

The matrix [Bi ]3×2 can be rewritten as follows:

University of Castilla-La Mancha Draft By: Eduardo W. V. Chaves (2016)


Ciudad Real - Spain
668 SOLVING PROBLEMS BY MEANS OF CONTINUUM MECHANICS

 ∂N 
 1 0 
 ∂x   gi 0
∂N i  1 
[Bi ]3×2 = 0 = 0 hi 
 ∂y  ab 
 ∂N  hi g i 
∂N i 
 i 
 ∂y ∂x 
The stiffness matrix can be obtained as follows


[k (e ) ] = [B]T [C ] [B] dV
V

 [B1 ]T  
 
T
 [ B ] 
⇒ [k ( e ) ] =   2 T [C ] [[B1 ] [B2 ] [B3 ] [B4 ]]  dV
∫ [B ] 
V 
 3 
 [B ]T  
 4  
 [B1 ]T [C ]  
  
 [B2 ]T [C ] 

(e)
⇒ [k ] =  [[B ] [B2 ] [B3 ] [B4 ]]  dV
[B ]T [C ] 1
V 3 
 
 [B ]T [C ] 
  4  
 [B1 ]T [C ][B1 ] [B1 ]T [C ][B2 ] [B1 ]T [C ][B3 ] [B1 ]T [C ][B4 ]  
  
 [B ]T [C ][B1 ] [B2 ]T [C ][B2 ] [B2 ]T [C ][B3 ] [B2 ]T [C ][B4 ] 
⇒ [k ( e) ] =   2 T
∫  dV
[B ] [C ][B ] [B ]T [C ][B ] [B3 ]T [C ][B3 ] [B3 ]T [C ][B4 ]
V 
 3 1 3 2

 [B ]T [C ][B ] [B ]T [C ][B ] [B4 ]T [C ][B3 ] [B4 ]T [C ][B4 ] 
 4 1 4 2 


⇒ [k ( e ) ] = [[Bi ]T [C ][B j ]] dV
V
(i, j = 1,2,3,4)


⇒ [k ( e ) ] = t [[Bi ]T [C ][B j ]] dA
A

The elasticity matrix is given by:


 C11 C12 0 
[C ] = C12 C22 0 
 0 0 C33 
Then, we can obtain:
 C11 C12 0 g j 0
1  gi 0 hi     1
T
[Bi ] [C ][B j ] =  C12 C22 0   0 hj  = [k ij ]
(ab) 2  0 hi g i   ( ab) 2
 0 0 C33   h j gj

where
 g i C11 g j + hi C33 h j g i C12 h j + hi C33 g j 
[k ij ]2×2 = 
hi C12 g j + g i C33 h j hi C22 h j + g i C33 g j 
y =b x = a
t t
[k (e) ] = 2
(ab) A ∫
[k ij ]2×2 dA =
(ab) 2 ∫ ∫ [k
y =0 x =0
ij ]2× 2 dxdy (i, j = 1,2,3,4)

Then, the stiffness matrix becomes

University of Castilla-La Mancha Draft By: Eduardo W. V. Chaves (2016)


Ciudad Real – Spain
6 LINEAR ELASTICITY 669

[k11 ] [k12 ] [k13 ] [k14 ]


 
t [k 21 ] [k 22 ] [k 23 ] [k 24 ]
[k (e ) ]8×8 =
(ab) 2 [k 31 ] [k 32 ] [k 33 ] [k 34 ]
 
[k 41 ] [k 42 ] [k 43 ] [k 44 ]
After the integration is taken place we can obtain:
k11( e ) k12( e) k13(e ) k14( e ) k15( e ) k16( e) k17(e ) k18( e ) 
 ( e) (e) (e) (e) ( e) (e) (e) 
 k 22 k 23 k 24 k 25 k 26 k 27 k 28 
 (e)
k33 (e)
k34 (e)
k35 ( e)
k36 (e)
k37 (e) 
k38
 (e) (e) ( e) (e) (e)

 k 44 k 45 k 46 k 47 k 48 
[k (e) ] = t  (e) ( e) (e) (e) 
(6.219)
k55 k56 k57 k58
 
 symmetric ( e)
k66 (e)
k67 (e)
k68 
 (e) (e) 
 k77 k78 
 (e) 
k88 
where
bC11 aC33 C12 + C33 − bC11 aC33
k11(e ) = + ; k12( e ) = ; k13( e ) = + ;
3a 3b 4 3a 6b
C − C33 − bC11 aC33
k14(e ) = 12 ; k15( e ) = − ; k16( e ) = − k12( e ) ;
4 6a 6b
bC aC aC bC
k17(e ) = 11 − 33 ; k18( e ) = −k14 ; (e) (e)
k 22 = 11 + 33 ;
6a 3b 3b 3a
(e ) aC11 bC33
k 23 = −k14(e ) ; (e)
k 24 = − ; (e)
k 25 = − k12 ; (e)
6b 3a
(e ) − aC11 bC33 (e) − aC11 bC33
k 26 = − ; k 27 = −k14( e ) ; (e)
k 28 = + ; (6.220)
6b 6a 3b 6a
(e )
k33 = k11(e ) ; (e)
k34 = −k12( e ) ; (e)
k35 = k17(e ) ;
(e )
k36 = k14(e ) ; (e)
k37 = k15( e ) ; (e)
k38 = k12(e ) ;
(e ) (e) (e)
k 44 = k 22 ; k 45 = −k14( e ) ; (e)
k 46 (e )
= k 28 ;
(e ) (e) (e) (e) (e) (e )
k 47 = k12 ; k 48 = k 26 ; k55 = k11 ;
(e )
k56 = k12(e ) ; (e)
k57 = k13( e ) ; (e)
k58 = k14(e ) ;
(e ) (e) (e)
k 66 = k 22 ; k67 = −k14( e ) ; (e)
k68 (e )
= k 24 ;
(e )
k 77 = k11(e ) ; (e)
k78 = −k12( e ) ; (e)
k88 (e )
= k 22 ;

University of Castilla-La Mancha Draft By: Eduardo W. V. Chaves (2016)


Ciudad Real - Spain
670 SOLVING PROBLEMS BY MEANS OF CONTINUUM MECHANICS

Example: If we consider the structure and data given in Problem 6.42, and if we only
consider one rectangle element, (see Figure 6.80), the stiffness matrix becomes:
 k11(1) k12(1) k13(1) k14(1) k15(1) k16(1) k17(1) k18(1) 
 (1) (1) (1) (1) (1) (1) (1) 
 k 22 k 23 k 24 k 25 k 26 k 27 k 28 
 (1)
k 33 (1)
k 34 (1)
k 35 (1)
k 36 (1)
k 37 (1) 
k 38
 (1) (1) (1) (1) (1)

 k 44 k 45 k 46 k 47 k 48 
[k (1) ] = t  (1) (1) (1) (1) 
k 55 k 56 k 57 k 58
 (1) (1) (1)

 k 66 k 67 k 68 
 symmetric (1) (1) 
 k 77 k 78 
 (1) 
k 88 
  20 5 − 5 − 5 −5 −5 5  0
  
  5 20 5 0 − 5 − 5 5 − 5 
 − 5 5 20 − 5 0 −5 0 5  
  
 1 − 5 0 − 5 20 5 − 5 5 − 5 
= 2  
 20 5 − 5 − 5 
 4 − 5 − 5 0 5

 − 5 − 5 − 5 − 5 5 20 5 0 
  
 0 5 0 5 − 5 5 20 − 5 

  5 −5 5 −5 − 5 0 − 5 20  
 
The system to be solved is
 20 5 − 5 − 5 −5 −5 5  U 1   0 
0
 5 20 5
 0 −5 −5 − 5 U 2   0 
5
 − 5 5 20 − 5 0 −5 0 5  U 3  10
    
1  − 5 0 − 5 20 5 − 5 5 − 5 U 4   0 
{F }8×1 = [ K ]8×8 {U }8×1 ⇒  = 
2 − 5 − 5 0 5 20 5 − 5 − 5 U 5  10
 
− 5 − 5 − 5 − 5 5 20 5 0  U 6   0 
0    
5 0 5 − 5 5 20 − 5 U 7   0 
 
 5 − 5 5 − 5 − 5 0 − 5 20  U 8   0 
By applying the boundary conditions we have
1 0 0 0 0 0 0 0  U 1   0  U 1   u (1)  0
0 1 0 0 0 0 0    
0  U 2   0  U   (1)  0
  2  v   
0 0 20 − 5 0 −5 0 5  U 3  10 U 3  u ( 2 )  1
          
1 0 0 − 5 20 5 −5 0 − 5 U 4   0  Solve U 4  v ( 2 )  0
  =   →  =  (3)  =  
2 0 0 0 5 20 5 0 − 5 U 5  10 U 5  u  1
 
0 0 −5 −5 5 20 0 0  U 6   0  U 6   v (3)  0
0        ( 4)   
0 0 0 0 0 1 0  U 7   0  U 7  u  0
 
−5 −5     U  v ( 4 )  0
0 0 5 0 0 20  U 8   0   8    
Note that the node numeration is different from the one presented in Problem 6.42.

University of Castilla-La Mancha Draft By: Eduardo W. V. Chaves (2016)


Ciudad Real – Spain
6 LINEAR ELASTICITY 671

y
Nodal displacement
v ( 4) v (3) u (1) 
( x ( 4) = 0, y ( 4) = b)  (1) 
u (4) ( x (3) = a, y ( 3) = b) v 
u ( 3) F = 10 u ( 2 ) 
4 3  ( 2) 
(e ) v 
{u } =  (3) 
u 
b  v ( 3) 

(1)
 (4) 
v v ( 2) u 
v ( 4) 
1 2 F = 10  
u (2)
u (1) ( x ( 2) = a, y ( 2) = 0) x
(1) (1)
( x = 0, y = 0)
a

Figure 6.80: Rectangle element.

Finite Element Technique References


BATHE, K-J (1996). Finite Element Procedures. Prentice Hall, New Jersey, USA.
ZIENKIEWICZ, O.C. & TAYLOR, R.L. (1994a). El método de los elementos finitos. Volumen 1:
Formulación básica y problemas lineales. CIMNE, Barcelona, 4ª edición.
ZIENKIEWICZ, O.C. & TAYLOR, R.L. (1994b). El método de los elementos finitos. Volumen 2:
Mecánica de sólidos y fluidos. Dinámica y no linealidad. CIMNE, Barcelona, 4ª edición.
OÑATE, E. (1992). Cálculo de estructuras por el método de elementos finitos análisis estático lineal.
Centro Internacional de Métodos Numéricos en Ingeniería. Barcelona- España.
CHAVES, E.W.V. & MÍNGUEZ, R. (2009). Mecánica Computacional en la Ingeniería con Aplicaciones
en MATLAB. Editorial UCLM, ISBN:978-84-692-8273-1. (in Spanish).

University of Castilla-La Mancha Draft By: Eduardo W. V. Chaves (2016)


Ciudad Real - Spain
672 SOLVING PROBLEMS BY MEANS OF CONTINUUM MECHANICS

6.4 Introduction to Torsion


Problem 6.44
Consider the hypothesis (approximations) for the Saint-Venant torsion problem, (see
Figure 6.81):
ƒ The body is prismatic (consider as prismatic axis the x1 -axis);
ƒ The unit torsion angle θ (angle of twist per unit length) is constant according to the x1 -
axis;
ƒ The projection of transversal cross section on ( x2 − x3 )-plane has rigid body motion
(rotation about the x1 -axis).
Show that this problem can be governed by the equation:

∇ 2 u1 = 0 with u1 = u1 ( x 2 , x3 ) (6.221)
 ∂ 2u ∂ 2u 
or G 21 + 21  = 0 , where G is the shear modulus (6.222)
 ∂x2 ∂x3 
∂  ∂u1  ∂  ∂u1 
or G  − x 3 θ  + G  + x 2 θ  = 0 (6.223)
∂x 2  ∂x 2  ∂x 3  ∂x3 

x1 = x
x = x1 MT
z = x3 z = x3

P2′
θx1
S2 P2
α

y = x2 y = x2

θ
x1

S1 r
r P1

Figure 6.81: Torque applied to a prismatic body.

University of Castilla-La Mancha Draft By: Eduardo W. V. Chaves (2016)


Ciudad Real – Spain
6 LINEAR ELASTICITY 673

Obs.:
With these hypotheses, in general, the transversal cross section does not remain plane after
deformation.
Considering the prismatic body and by applying the torque at the free end, the body
displaces as indicated in Figure 6.81.
Solution:
r
Let us consider the point P1 located on the fixed section S1 whose position vector is r ,
(see Figure 6.81). Also consider another cross section S 2 (free to rotate and warping)
which distance from the section S1 is x1 and by projecting the point P1 on the cross
section S 2 we obtain the point P2 . After the torque is applied the point P2 moves to P2′ as
indicated in Figure 6.82.

x3 ,u3
x2 = r cosα
u2 x3 = r sinα
P2′ r 2 = x 22 + x32

r rθx1 u3

θx1 P2

α
x2 ,u 2

Figure 6.82: Motion of the prismatic body cross section.

Geometrically, (see Figure 6.82), we can obtain the displacements:


u2 = −r θ x1 sin α = − x3 θ x1
 (6.224)
u3 = r θ x1 cosα = x2 θ x1
where x2 = r cosα , x3 = r sinα , and u2 stands for the displacement according x2 -direction,
and u3 is the displacement according to the x3 -direction. The displacement of the point P2
according to the x1 -direction can be any, thus we summarize:
u1 = u1 ( x 2 , x 3 ) ; u 2 = − x 3 θ x1 ; u 3 = x 2 θ x1 (6.225)
The displacement u1 (warping function) is the warping of the cross section, which is
independent of x1 .

University of Castilla-La Mancha Draft By: Eduardo W. V. Chaves (2016)


Ciudad Real - Spain
674 SOLVING PROBLEMS BY MEANS OF CONTINUUM MECHANICS

(Kinematic equations)
The strain-displacement relationships become:
∂u1 ∂u 2 ∂u 3
ε11 = =0; ε 22 = = 0; ε 33 = = 0;
∂x1 ∂x 2 ∂x3
1  ∂u1 ∂u2  1  ∂u2 ∂u3  1  ∂u1 ∂u3 
ε12 =  +    ε13 =  + 
2  ∂x2 ∂x1  ε 23 = +
2  ∂x3 ∂x2  2  ∂x3 ∂x1 
1  ∂u1  1 1  ∂u 
=  − x3 θ  ; = (− θ x1 + θ x1 ) ; =  1 + x2 θ 
2  ∂x2  2 2  ∂x3 
=0

Thus
  ∂u1   ∂u1 
 − x3 θ   
 0  ∂x + x2 θ  
 ε11 ε12 ε13    ∂x2   3 
1  ∂u  
εij = ε12 ε 22 ε 23  =  1 − x3 θ  0 0  (6.226)
2  ∂x2  
ε13 ε 23 ε33   ∂u  
 1 + x2 θ  0 0 
 ∂x3  
Note that the compatibility equations, (see Problem 5.11), are automatically satisfied, since
the displacement field is continuous, a fact verified by the fulfillment of the compatibility
equations, i.e.:
 ∂ 2ε33 ∂ 2ε 22 ∂ 2ε 23
S
 11 = + − 2 =0 (a)
 ∂ x2
2
∂x 2
3 ∂ x2 ∂x 3
 ∂ 2ε33 ∂ 2ε11 ∂ 2ε13
S22 = + − 2 =0 (b)
 ∂x12 ∂x32 ∂x1∂x3
 2 2 2
S = ∂ ε11 + ∂ ε 22 − 2 ∂ ε12 = 0 (c)
 33
∂x22 ∂x12 ∂x1∂x2

 (6.227)
S = ∂  ∂ε 23 + ∂ε13 − ∂ε12  − ∂ ε33 = 0
2
(d)
 12 ∂x3  ∂x1 ∂x2 ∂x3  ∂x1∂x2

 ∂  ∂ε 23 ∂ε13 ∂ε12  ∂ 2ε11
= − 
S
 23
∂ x  ∂x + ∂x + ∂x  − ∂x ∂x = 0 (e)
 1  1 2 3  2 3
   2
S13 = ∂  ∂ε 23 − ∂ε13 + ∂ε12  − ∂ ε 22 = 0 (f)
 ∂x2  ∂x1 ∂x2 ∂x3  ∂x1∂x3

From equations (6.227) (d) and (f) we can conclude that:


∂  ∂ε13 ∂ε12  ∂  ∂ε13 ∂ε12   ∂ε12 ∂ε13 
 − =0 − + =0 ⇒  
∂x3  ∂x2 ∂x3 
;
∂x2  ∂x2 ∂x3   ∂x − ∂x  = constant
 3 2 

In fact, and is equal to:


∂ε12 ∂ε13 1 ∂  ∂u1  1 ∂  ∂u1  1
− =  − x3 θ  −  + x2 θ  = (−θ − θ) = −θ
∂x3 ∂x2 2 ∂x3  ∂x2 
 2 ∂x2  ∂x3  2
In other words, the compatibility equations are satisfied if the following is true:

University of Castilla-La Mancha Draft By: Eduardo W. V. Chaves (2016)


Ciudad Real – Spain
6 LINEAR ELASTICITY 675

∂ε12 ∂ε13 ∂γ xy ∂γ xz
− = −θ Engineerin
  g  notation
→ − = −2θ
∂x3 ∂x2 ∂x3 ∂x2

(Constitutive equations)
The constitutive equations for stress are given by:
νE E E
σ ij = ε kkδ ij + ε ij = ε ij (6.228)
(1 + ν )(1 − 2ν ) (1 + ν ) (1 + ν )
Note that ε kk = 0 (trace of ε is zero). Then, by substituting the strain field into the above
equation we can obtain:
  ∂u1   ∂u1 
 0  − x3 θ   + x θ 
 ∂x 2 
  ∂x2   3 
E 1 E  ∂u1  
σij = εij =  − x3 θ  0 0 
(1 + ν ) 2 (1 + ν )  ∂x2  
 ∂u  
 1 + x2 θ  0 0 
 ∂x3  
(6.229)
  ∂u1   ∂u1 
 − x3 θ   
 0  ∂x + x2 θ  
 0 σ12 σ13    ∂x2   3 
 ∂u  
= σ12 0 0  = G  1 − x3 θ  0 0 
σ13 0 0   ∂x2  
 ∂u  
 1 + x2 θ  0 0 
 ∂x3  

Let us suppose that we need to obtain the stress components in a new system x1′ − x ′2 − x 3′ ,
which is formed by a rotation around the x1 -axis, (see Figure 6.83). Then the
transformation matrix from x1 − x 2 − x 3 to x1′ − x ′2 − x 3′ is given as follows:
 a11 a12 a13  1 0 0
a ij = a 21 a 22 a 23  = 0 nˆ 2 nˆ 3  (6.230)
 a 31 a 32 a 33  0 sˆ 2 sˆ 3 
Recall that the transformation law, (see Problem 1.99), for a second-order tensor
components is given by
σ′ij = aik a jl σkl Matrix
 form
→ σ ′ = AσA T Voigt
→ {σ ′} = [M]{σ }

where
 a11 2 2
a12 2
a13 2a11a12 2a12 a13 2a11a13 
 2 22 2 
 a21 a22 a23 2a21a22 2a22 a23 2a21a23 
 2 2 2 
[M] =  a31 a32 a33 2a31a32 2a32 a33 2a31a33

 a21a11 a22 a12 a13a23 (a11a22 + a12a21 ) (a13a22 + a12a23 ) (a13a21 + a11a23 )
a a a a a33a23 (a31a22 + a32a21 ) (a33a22 + a32a23 ) (a33a21 + a31a23 )
 31 21 32 22
 a31a11 a32 a12 a33a13 (a31a12 + a32a11 ) (a33a12 + a32a13 ) (a33a11 + a31a13 )
Then, for this particular transformation, (see equation (6.230)), we can obtain:

University of Castilla-La Mancha Draft By: Eduardo W. V. Chaves (2016)


Ciudad Real - Spain
676 SOLVING PROBLEMS BY MEANS OF CONTINUUM MECHANICS

′  1
 σ11 0 0 0 0 0   σ11 = 0   0 
σ ′  0 nˆ 2 nˆ 32 0 2nˆ 2 nˆ 3 0  σ 22 = 0  0 
 22   2 
σ ′33  0 sˆ 22 sˆ 32 0 2sˆ 2 sˆ 3 0  σ 33 = 0   0 
 =  =  (6.231)
′  0
 σ12 0 0 nˆ 2 0 nˆ 3   σ12  nˆ 2 σ12 + nˆ 3 σ13 
σ ′23  0 sˆ 2 nˆ 2
  
ˆs 3 nˆ 3 0 (sˆ 3nˆ 2 + sˆ 2 nˆ 3 ) 0  σ 23 = 0 
  
0 

′  0
 σ13 0 0 ˆs 2 0 sˆ 3   σ13   sˆ 2 σ12 + sˆ 3 σ13 

x3
x3′ x2′
σ13
x3

σ12
x2

σ ′ = AσA T
x2
x3′ x2′
cross section ′
σ13 ′
σ12

Figure 6.83
(Equilibrium equations)
Using the equilibrium equations without considering the body forces we can obtain:

 ∂σ11 ∂σ12 ∂σ13  ∂σ12 ∂σ13


+ + =0  + =0

 ∂x1 ∂x2 ∂x3  ∂x2 ∂x3
 ∂σ12 ∂σ22 ∂σ23  ∂σ12
 + + =0 ⇒  = 0 ⇒ σ12 = σ12 ( x2 , x3 ) (6.232)
 ∂x1 ∂x2 ∂x3  ∂x1
 ∂σ13 ∂σ 23 ∂σ33  ∂σ13
 ∂x + ∂x + ∂x = 0  = 0 ⇒ σ13 = σ13 ( x2 , x3 )
 1 2 3  ∂x1

Note that the stresses σ12 and σ13 are not function of x1 , i.e. they do not vary with x1 . By
substituting the values of σ12 and σ13 into the first equation of the equilibrium equations
we can obtain:
∂σ12 ∂σ13  ∂u1
∂  ∂  ∂u1 
+ =0 ⇒ G  − x3 θ  + G  + x2 θ  = 0 (6.233)
∂x2 ∂x3 ∂x2
 ∂x2  ∂x3  ∂x3 
∂ u
2
∂ u1 
2
G  21 + =0
2  (6.234)
∂x
 2 ∂ x 3 

∇ 2u1 = 0 where u1 = u1 ( x2 , x3 ) (6.235)

University of Castilla-La Mancha Draft By: Eduardo W. V. Chaves (2016)


Ciudad Real – Spain
6 LINEAR ELASTICITY 677

which is the differential equation of Saint-Venant torsion. The displacement according to x1 –


direction is known as warping, as we will see later for the circular cross section there is no
warping phenomenon, i.e. u1 = 0 , (see Figure 6.84).

no warping

warping

Figure 6.84: Torsion.

External Equilibrium
According to the problem statement, (see Figure 6.81), and by considering the external
force equilibrium the following must be true:
External moment (Torque), (see Problem 4.22):
M x ≡ MT = ∫ (σ
A
13 x2 − σ12 x3 )dA (6.236)

By using the stress components σ12 and σ13 given by the equations in (6.229), the above
equation can also be written as follows:
 ∂u   ∂u  
MT = ∫ (σ
A
13 x2 − σ12 x3 )dA = G  1 + x2 θ  x2 −  1 − x3 θ  x3  dA

 ∂x3
A   ∂x2  
 ∂u ∂u   ∂u x ∂u x  (6.237)
A
∫ ∂x2  A
∂x3 θ ∂x2 θ ∫
⇒ M T = G  1 x2 + x22 θ − 1 x3 + x32 θ dA = Gθ  1 2 − 1 3 + x22 + x32
∂x3
 dA

⇒ M T = GθJT
where we have introduced the polar moment of inertia:
 ∂u x ∂u x 
A

JT =  1 2 − 1 3 + x22 + x32
∂x3 θ ∂x2 θ
 dA

(6.238)

where JT is the Saint-Venant torsional stiffness of the section. The shearing forces according
to the x2 -direction and x3 -direction are equal to zero, so:

University of Castilla-La Mancha Draft By: Eduardo W. V. Chaves (2016)


Ciudad Real - Spain
678 SOLVING PROBLEMS BY MEANS OF CONTINUUM MECHANICS

 ∂u   ∂u 
A
∫ A 
∫  A A 
∂x3 

Qx2 = σ12 dA = G 1 − x3 θ dA = 0 ; Qx3 = σ13dA = G 1 + x2 θ dA = 0
∂x2 ∫ (6.239)

where we have used the stress components σ12 and σ13 given by the equations in (6.229).
To complete the boundary conditions for the torsion problem. The boundary condition is
defined by the absence of normal stress component on the external surface of the prismatic
′ = 0 on the boundary surface, (see Figure 6.85). By means of equation (6.231) we
body σ13
can conclude that:
′  
 σ11 0  0 
σ ′   0   
 22    0 
σ ′33   0  0 
 = =  (6.240)
′  nˆ 2 σ12
 σ12 + nˆ 3 σ13  0
σ ′23   0  0 
     
′   sˆ 2 σ12
 σ13 + sˆ 3 σ 13   τ 

x3 x2′
x3′

n̂ On boundary surface - Γ :
ŝ ′ =0
σ12
′ ≡τ
σ13

x2

cross section

Figure 6.85

NOTE 1: Prismatic circular cross-sectional rod


Note that, when the cross section is circular there is no warping, since u1 = 0 .

[ϕ ] = rad
A
θ rad
[θ] =
M x ≡ MT m
A′ x

θx1 = ϕ

θdx1 = dϕ

x1 ⇒θ=
dx1

Figure 6.86: The rod (circular cross section) subjected to torque.

University of Castilla-La Mancha Draft By: Eduardo W. V. Chaves (2016)


Ciudad Real – Spain
6 LINEAR ELASTICITY 679

Then, in this case the strain field on the cross section is given by:
 ε11 ε12 ε13   0 − x3θ x2 θ  0 − x3 x2 
1 θ
ε ij = ε12 ε 22 
ε 23  = − x3θ 0 
0  = − x3 0 0 
2 2
ε13 ε 23 ε 33   x2 θ 0 0   x2 0 0 

The stress field on the cross section is given by:


 σ11 σ12 σ13   0 − Gx3 θ Gx 2 θ  0 − x3 x2 
σ ij = σ12 σ 22  
σ 23  = − Gx3 θ 0 0  = Gθ− x 3
 0 0 
σ13 σ 23 σ 33   Gx 2 θ 0 0   x 2 0 0 

τ
z, x 3 MT
σ13 τ max = R
JT
− σ12
τ max
σ13 τ(r )

r
r σ12

y, x 2

Figure 6.87: Tangential stresses – Moment of torsion.


The moment of torsion is given by:
MT = ∫ (σ
A
13 x 2 − σ12 x3 )dA = ∫ ((Gθx
A
2 ) x2 ∫(
− (−Gθx3 ) x3 )dA = Gθ x 22 + x32 dA
A
)
(6.241)

2
= Gθ r dA = GθJ T
A

πR 4
where JT = ∫ r 2dA = is the polar inertia moment.
A
2

According to Figure 6.87 we can conclude that:


(−σ12 ) 2 + (σ13 ) 2 = τ 2 ⇒ (Gθ) 2 ( x22 + x32 ) = τ 2
⇒ (Gθ) 2 r 2 = τ 2 ⇒ τ(r ) = Gθr
MT
And according to the equation in (6.241) the relationship Gθ = holds, with which the
JT
above equation can be rewritten as follows:
MT
τ(r ) = Gθr ⇒ τ( r ) = r (6.242)
JT

and the unit torsion angle (angle of twist per unit length) can be obtained as follows:

University of Castilla-La Mancha Draft By: Eduardo W. V. Chaves (2016)


Ciudad Real - Spain
680 SOLVING PROBLEMS BY MEANS OF CONTINUUM MECHANICS

MT τ(r ) M T τ(r )
θ= ; θ= ; = = Gθ (6.243)
GJT Gr JT r
where the term GJT stands for the torsional rigidity (or torsional stiffness).
The maximum value of τ(r ) occurs in r = R , (see Figure 6.87):
MT
τ(r = R ) = τ max = GθR ⇒ τ(r = R ) = τ max = R
JT
For the hollow circular section the expressions are the same in which the polar inertia
π
moment is given by JT = ∫ r 2 dA = ( R24 − R14 ) , (see Figure 6.88).
A
2

z, x 3
π 4
JT = ( R2 − R14 )
2 τ max
τ(r )

r
r R1

y, x 2

R2

Figure 6.88: Tangential stresses – hollow circular cross section.


NOTE 2: Prandtl’s Stress Function
Let us adopt the Prandtl’s stress function φ such as:
∂φ ∂φ
σ12 = ; σ13 = − (6.244)
∂x3 ∂x2
Notice that this function satisfies the equilibrium equation (6.232):
∂σ12 ∂σ13 ∂  ∂φ  ∂  ∂φ 
+ =0 ⇒  −  =0
∂x2 ∂x3 ∂x2  ∂x3  ∂x3  ∂x2 
Recall that the governing equation for the linear elastic problem can be represented by the
6 stress components, which is known as stress formulation, (see Problem 5.16 and
Problem 5.19-NOTE 2). In addition, if we are considering a static problem in which the
r
body force does not vary with x we fall back in Beltrami’s equations, (see Problem 5.16-
NOTE 1):
1
σij , kk + σ kk ,ij = 0ij
(1 + ν )
Beltrami’s equations (6.245)
1
∇ 2xr σ + ∇ xr [∇ xr [Tr (σ )]] = 0
(1 + ν )

University of Castilla-La Mancha Draft By: Eduardo W. V. Chaves (2016)


Ciudad Real – Spain
6 LINEAR ELASTICITY 681

Taking into account the stress components, (see equation (6.229)):


  ∂u1   ∂u1 
 0  − x3 θ   + x2 θ 
  ∂x2   ∂x3   0 σ12 σ13 
 ∂u   
σij = G  1 − x3 θ  0 0  = σ12 0 0 
 ∂x2   σ 0 0 
 ∂u    13
 1 + x2 θ  0 0 
 ∂x3  
where the trace of stress tensor is zero σ kk = 0 , the Beltrami’s equations become:
σij ,kk = σij ,11 + σij , 22 + σij , 22 = 0ij
 σ11,11 σ12,11 σ13,11   σ11, 22 σ12, 22 σ13, 22   σ11,33 σ12,33 σ13,33 
     
⇒ σij ,kk = σ12,11 σ 22,11 σ 23,11  + σ12, 22 σ 22, 22 σ 23, 22  + σ12,33 σ 22,33 σ 23,33  = 0ij
σ13,11 σ 23,11 σ33,11  σ13, 22 σ 23, 22 σ33, 22  σ13,33 σ 23,33 σ33,33 
  

 0 (σ12,11 + σ12, 22 + σ12,33 ) (σ13,11 + σ13, 22 + σ13,33 )


 
⇒ σij ,kk = (σ12,11 + σ12, 22 + σ12,33 ) 0 0  = 0ij
 (σ13,11 + σ13, 22 + σ13,33 ) 0 0 
 
With that we can obtain:
σ12,11 + σ12, 22 + σ12,33 = 0

σ13,11 + σ13, 22 + σ13,33 = 0

Note that σ12 = σ12 ( x2 , x3 ) and σ13 = σ13 ( x2 , x3 ) , (see equation (6.232)), so, the above
equations become:
 ∂ 2 σ12 ∂ 2 σ12
 2 + =0
∇ 2xr σ12 = 0
σ12, 22 + σ12,33 = 0  ∂x2 ∂x32
 ⇒  2 ⇒  2 (6.246)
σ13, 22 + σ13,33 = 0 2
 ∂ σ13 + ∂ σ13 = 0 ∇ xr σ13 = 0
 ∂x 2 ∂x32
 2
If we consider the stresses defined in (6.244) into the equations (6.246) we can obtain:
 2  ∂φ  ∂ ∂  ∂ 2φ ∂ 2φ 
∇ 2xr σ12 = 0
∇ xr 

 =

( )
∇ 2xrφ = 0 ⇒

 2 + 2 =0
 ∂x 
  x 3  x 3 x 3  2 ∂x3 
 2 ⇒  (6.247)
∇ xr σ13 = 0  
 2r  − ∂φ  − ∂ 2r
( ) ( φ
) φ
2 2
∂ ∂ ∂ ∂
 
∇ x  ∂x  = ∂x ∇ xφ = 0 ⇒ ∂x ∇ xφ = ∂x  ∂x 2 + ∂x 2  = 0
2r

  2  2 2 2  2 3 

As ∇ 2xrφ does not vary with x2 and x3 we can conclude that:

∂ 2φ ∂ 2φ
∇ 2xrφ = + = C = constant (6.248)
∂x22 ∂x32

Then, any function φ which satisfies the above equation will fulfill the equilibrium and
compatibility equations.
Starting from the stress components, (see equation (6.229)):

University of Castilla-La Mancha Draft By: Eduardo W. V. Chaves (2016)


Ciudad Real - Spain
682 SOLVING PROBLEMS BY MEANS OF CONTINUUM MECHANICS

 ∂u  ∂u1 σ12
σ12 = G 1 − x3 θ  ⇒ = + x3 θ
 ∂x 2  ∂x 2 G
(6.249)
 ∂u  ∂u1 σ13
σ13 = G 1 + x 2 θ  ⇒ = − x2 θ
 ∂x3  ∂x3 G

and taking the derivative σ12 with respect to x3 , and taking the derivative of σ13 with
respect to x2 we can obtain
∂ ∂u1 ∂  σ12  1 ∂σ12
=  + x3 θ  = +θ
∂x3 ∂x2 ∂x3  G  G ∂x3
∂ ∂u1 ∂  σ13  1 ∂σ13
=  − x2 θ  = −θ
∂x2 ∂x3 ∂x2  G  G ∂x2
and by subtracting the two expressions we can obtain that
∂ ∂u1 ∂ ∂u1 1 ∂σ12 1 ∂σ13
− = +θ− +θ
∂x3 ∂x2 ∂x2 ∂x3 G ∂x3 G ∂x2
1 ∂σ12 1 ∂σ13
⇒0= + 2θ −
G ∂x3 G ∂x2
∂σ12 ∂σ13
⇒ − = −2Gθ
∂x3 ∂x2
∂φ ∂φ
and by substituting the stress values σ12 = and σ13 = − into the above equation we
∂x3 ∂x2
can obtain:
∂σ12 ∂σ13 ∂ ∂φ ∂ ∂φ  ∂ 2φ ∂ 2φ 
− = −2Gθ ⇒ + = +  = −2Gθ
∂x3 ∂x2 ∂x3 ∂x3 ∂x2 ∂x2  ∂x22 ∂x32 
and if we take into account the equation in (6.248) we can conclude that:
∂ 2φ ∂ 2φ
+ = −2Gθ = constant = C ∇ xr ⋅ (∇ xrφ ) ≡ ∇ 2xrφ = −2Gθ = constant (6.250)
∂x22 ∂x32

where ∇ 2xr stands for the Laplacian.


To complete the problem statement we have to define the boundary conditions. To the
torsion problem the boundary condition is defined by the absence of normal traction
vector on the external surface of the prismatic body, (see Figure 6.89).
The traction vector in terms of the Cauchy stress tensor becomes:
t1(nˆ )   σ11 σ12 σ13  nˆ 1  0
 (nˆ )      
σ23  nˆ 2  = 0
ˆ
t i(n) = σijnˆ j ⇒ t 2  = σ12 σ22
t (nˆ )  σ σ23 σ33  nˆ 3  0
 3   13
t1(nˆ )   0 σ12 σ13  nˆ 1 = 0 σ12nˆ 2 + σ13nˆ 3  0
 ˆ       
⇒ t (2n)  = σ12 0 0   nˆ 2  =  0  = 0
t (nˆ )  σ 0  
0   nˆ 3    0  0
 3   13   
with that the boundary condition is:
σ12nˆ 2 + σ13nˆ 3 = 0

University of Castilla-La Mancha Draft By: Eduardo W. V. Chaves (2016)


Ciudad Real – Spain
6 LINEAR ELASTICITY 683

x3 dx j
nˆ i = 1ij
ds
σ13
n̂  0 
n1  
ˆ 
   dx 
dx3 σ12 nˆ i = nˆ 2  =  3 
dx2 nˆ   ds 
ds  3  − dx2 
 ds 
x2  kij - permutation symbol
ds - differential arc-length element

Figure 6.89

Taking into account the stresses given by (6.244), the above condition can be rewritten as
follows:
∂φ ˆ ∂φ ˆ ∂φ dx3 ∂φ  dx2 
σ12nˆ 2 + σ13nˆ 3 = 0 ⇒ n2 − n3 = 0 ⇒ − − =0
∂x3 ∂x2 ∂x3 ds ∂x2  ds 
(6.251)
∂φ dx3 ∂φ dx2 dφ
⇒ + =0 ⇒ =0
∂x3 ds ∂x2 ds ds
With that we can conclude that φ is constant on the boundary and can be assume any
value, with which we adopt zero.
Let us consider that:
∂φ ∂φ
F3 ( x2 , x3 ) = σ12 = ; F2 ( x2 , x3 ) = −σ13 = (6.252)
∂x3 ∂x2
The function φ is a compatible field if and only if:
∂φ 
= F2 ( x2 , x3 )
∂x2  compatible iff ∂F2 ∂F3
   → = (6.253)
∂φ  ∂x3 ∂x2
= F3 ( x2 , x3 )
∂x3 

If we consider the Green’s theorem, (see Chapter 1 of the textbook), we can establish that:
r r r r  ∂F ∂F 
∫Γ F ⋅ dΓ = Ω∫ (∇ ∧ F) ⋅ eˆ 1dA   → F2 dx2 + F3 dx3 =  2 − 3 dA1
∫ ∫
r components
x
Γ Ω  ∂x3 ∂x2 
With which we can conclude:
 ∂F ∂F  ∂φ ∂φ  ∂ ∂φ ∂ ∂φ 
∫ F dx
Γ
2 2 ∫
+ F3 dx3 =  2 − 3 dA1
Ω  ∂x3 ∂x2 

Γ ∂x2
dx2 +
∂x3 ∫
dx3 =  −
Ω  ∂x3 ∂x2 ∂x2 ∂x3 

dA1 = 0

∂φ ∂φ r
⇒ ∫ dx2 + dx3 = ∇ xrφ ⋅ dx = dφ = 0
∫ ∫
∂x
Γ 2 ∂ x3 Γ Γ

University of Castilla-La Mancha Draft By: Eduardo W. V. Chaves (2016)


Ciudad Real - Spain
684 SOLVING PROBLEMS BY MEANS OF CONTINUUM MECHANICS

r
x3 dA = dAê1
x1 Γ
ê1

x2

Figure 6.90: Green’s theorem.


Now let us obtain the expression for the moment of torsion. Recalling the moment of
torsion equation we can obtain:
MT = ∫ (σ13 x2 − σ12 x3 )dA = − (φ , 2 x2 + φ ,3 x3 )dA = − (φ ,i xi )dA
∫ ∫ (i = 2,3)
A A A

note that (φxi ) ,i = φ ,i xi + φxi ,i = φ ,i x i + φδ ii( 2 D ) = φ ,i x i + 2φ ⇒ φ ,i x i = (φxi ) ,i − 2φ , where we


have applied the trace of the Kronecker delta for 2D, i.e. xi ,i = δ ii( 2 D ) = δ 22 + δ 33 = 2 , thus

M T = − (φ ,i xi )dA = − ((φxi ) ,i − 2φ )dA = − (φxi ) ,i dA +


∫ ∫ ∫ ∫ (2φ )dA
A A A A


= − (φxi )nˆ i dΓ +
Γ
∫ (2φ )dA
A

= −2 Aφ Γ + ∫ (2φ )dA
A

If we consider that on the boundary the value of φ is zero, i.e. φ Γ = 0 , we can obtain that:

M T = 2 φ dA ∫
A
(6.254)

The same can be shown by means of tensorial notation (in 2D):


M T = − (φ ,i xi )dA = − [(∇ xr φ ) ⋅ x ]dA = − [∇ xr ⋅ (φx )]dA + [φ (∇ xr ⋅ x )]dA
r r r

A

A

A

A
r
= − φx ⋅ nˆ dΓ + 2φ dA = −2 Aφ Γ + 2φ dA
∫ ∫ ∫ (6.255)
Γ A A


= 2 φ dA
A
r
where we have used the property 2 A = ∫ x ⋅ n̂dΓ , (see Problem 1.128).
Γ

It is easy to show that there exist iso-curves of φ , (see Figure 6.91). Note that for the
system ( x′2 − x3′ ) on iso-curve of φ , we have:
∂φ ∂φ
′ =
σ12 =0 ; ′ =τ=
− σ13 (6.256)
∂x3′ ∂x′2
That is, τ is equal to the slope of φ according to the x′2 -direction, which has the same
direction as ∇φ , (see Figure 6.91).

University of Castilla-La Mancha Draft By: Eduardo W. V. Chaves (2016)


Ciudad Real – Spain
6 LINEAR ELASTICITY 685

x3′ x 2′

∇φ
Γ
φ3
τ
ŝ φ ( 3) φ (Γ) = 0
φ (1)

φ ( 2) τφ1
x3 τ(Γ ) τφ 2

x2 cross section

Figure 6.91: Iso-curves of φ .


As additional note, next we will show in a different way that the shearing force on the cross
section is zero, and that the net moment is equal twice the volume formed by φ ( x2 , x3 ) . We
start from the shearing force ( Qx2 ) according to the x2 -direction:
x 2( 2 ) x3( 2 ) x 2( 2 )  x3( 2 ) 
∂φ ∂φ  ∂φ × 1dx3  dx2

Qx2 = σ12 dA =
∂x3
dA = ∫ ∫ ∫ (1) ∂x3
dx3dx2 = ∫ ∫  ∂x3 
A A x 2(1) x3 x 2(1)  x3(1) 
 (2)  (6.257)
x 2( 2 ) x 2( 2 )
 x3( 2 ) x3 ∂(1) 

⇒ Qx2 = φ x (1) − φ ×
∂ x ∫
dx3 dx2 = φ ( x3( 2 ) ) − φ ( x3(1) ) dx2 = 0 ∫[ ]
x 2(1)  
3
x3(1) 1
424 33 x 2(1)
 =0 

where we have applied the integration by part. Note that φ ( x3( 2) ) and φ ( x3(1) ) are values of
the membrane on the bar surface, as we have shown previously, φ is constant on the
boundary so, φ ( x3( 2) ) − φ ( x3(1) ) = 0 . The same procedure can be used to show that Qx3 = 0 .
As we have seen previously, the net moment is obtained as follows:
∂φ  ∂φ 
MT = ∫ (σ13 x2 − σ12 x3 )dA = − ∫ 
A A
∂x2
x2 + x3  dA
∂x3 
(6.258)

Also note that:


x2( 2 )  x3( 2 )  x2( 2 )  x3( 2 )  x2( 2 )  x3( 2 ) 
∂φ ∂φ x3( 2 ) ∂( x3 )
∫ ∂x
x3 dA = ∫ ∫ 
(1)  (1) ∂x3
x3 dx3  dx2 = φx3
  ∫ x3(1)
− φ× ∫ ∂x3
dx3  dx2 = −  φdx3  dx2
   ∫ ∫
A 3 x2  x3  x2(1)  x3(1)  x2(1)  x3(1) 


= − φdA

where we have used that φ at any point of the boundary is zero, i.e. φ ( Γ ) = 0 , so the term
∂φ
φ1(2
x3( 2 ) ) x3( 2 ) − φ ( x3(1) ) x3(1) = 0 . Similarly, we can obtain that
3 123 ∫ ∂x
A 2

x 2 dA = − φdA . With that the
=0 =0

equation in (6.258) becomes M T = 2 ∫ φdA , (see equation (6.254)).


A

University of Castilla-La Mancha Draft By: Eduardo W. V. Chaves (2016)


Ciudad Real - Spain
686 SOLVING PROBLEMS BY MEANS OF CONTINUUM MECHANICS

∂φ − ∂φ
NOTE 3: If we consider that τ 1 = 0 , τ 2 = σ12 = , and τ 3 = σ13 = , the following is
∂x3 ∂x2
r
true (∇ xr ∧ τ ) ⋅ eˆ 1 = −∇ 2xrφ , (see Problem 1.107). And by taking into account the equation
r
(6.250) we can conclude that (∇ xr ∧ τ ) ⋅ eˆ 1 = −∇ 2xrφ = 2Gθ , then if we take the integral over
the cross section area we can obtain:
r
A
∫ (∇ xr ∧ τ ) ⋅ eˆ 1dA = ∫ (2Gθ)dA = 2θ∫ GdA = 21G2θ3A
A A G −constant
(6.259)
r r
By using the Stoke’s theorem we can say that ∫ (∇ r
x ∧ τ ) ⋅ eˆ 1dA = τ ⋅ sˆ dΓ = τ( Γ ) dΓ , then
∫ ∫
A Γ Γ
the above equation can be rewritten as follows:
r
∫ (∇ r
x ∧ τ ) ⋅ eˆ 1dA = τ( Γ ) dΓ = 2GθA
∫ (6.260)
A Γ

where τ(Γ ) is the tangential stress on the boundary, (see Figure 6.91).

University of Castilla-La Mancha Draft By: Eduardo W. V. Chaves (2016)


Ciudad Real – Spain
6 LINEAR ELASTICITY 687

NOTE 4: Prandtl’s membrane analogy (Soap-film analogy)


Consider a homogenous membrane fixed at its extremities, which is subjected to a uniform
N
lateral pressure p , [ p ] = , (see Figure 6.92). Due to the pressure in the membrane a
m2
stress state S appears. Next, we will define which equation governs such problem.

x1
see Figure 6.94

Sdx3 h( x2 , x3 )
Sdx3
p x2

x3

x3

Sdx2

dx3

dx2
Sdx2
p

x1

x2

Figure 6.92: Membrane under pressure.

Note that the slope of the membrane at the point ( x2 , x3 ) is given by the derivative of the
∂h ∂h
function h( x2 , x3 ) , i.e. and , which are tangents to the curve at the point. We
∂x2 ∂x3
∂h ∂h
denoted by tan(α 2 ) = and tan(α 3 ) = , and if we are considering small angles the
∂x2 ∂x3
relationships tan(α ) ≈ sin(α ) ≈ α hold. In the differential element dx 2 − dx3 the variation of
the tangents are indicated in Figure 6.93.

University of Castilla-La Mancha Draft By: Eduardo W. V. Chaves (2016)


Ciudad Real - Spain
688 SOLVING PROBLEMS BY MEANS OF CONTINUUM MECHANICS

∂α 3 ∂α 3 ∂α 3
α3 + dx3 α3 + dx3 + dx2
∂x3 ∂x3 ∂x2

( x 2 , x 3 + dx3 ) ( x 2 + dx 2 , x 3 + dx3 )

∂α 2 ∂α 2 ∂α 2
α2 + dx3 α2 + dx2 + dx3
∂x3 ∂x2 ∂x3
r

dx 3

∂α 2
α2 + dx 2
(α 3 ) ∂x 2

( x 2 , x3 ) ( x 2 + dx 2 , x3 )
x3
(α 2 ) ∂α 3
α3 + dx2
∂x2
dx 2
x2

Figure 6.93: Variation of the tangents in the differential element.

∂α 3
If we consider that there is no distortional of the tangents the terms dx 2 = 0 and
∂x 2
∂α 2
dx 3 = 0 hold, (see Figure 6.94).
∂x 3

University of Castilla-La Mancha Draft By: Eduardo W. V. Chaves (2016)


Ciudad Real – Spain
6 LINEAR ELASTICITY 689

Sdx 2
∂α 3
α3 + dx3
∂x3

Sdx3

α2
dx3
∂α 2
α2 + dx 2
∂x 2
dx 2
Sdx3
α3

Sdx 2

 ∂α 2   ∂α 2 
Sdx3 sin α 2 + dx2  ≈ Sdx3 α 2 + dx2  Sdx3
 ∂x2   ∂x2 

∂α 2
α2 + dx2
∂x2
α2

Sdx3 − Sdx 3 sin(α 2 ) ≈ − Sdx 3α 2

Figure 6.94

By apply the force equilibrium condition according to x1 -direction, we can obtain:

∑F x1 =0
 ∂h ∂ 2h   ∂h   ∂h ∂ 2h   ∂h 
pdx2 dx3 + Sdx3 sin  + 2 dx2  − Sdx3 sin   + Sdx2 sin  + 2 dx3  − Sdx2 sin   = 0
 ∂x2 ∂x2   ∂x2   ∂x3 ∂x3   ∂x3 
 ∂h ∂ 2h   ∂h   ∂h ∂ 2h   ∂h 
⇒ pdx2 dx3 + Sdx3  + 2 dx2  − Sdx3   + Sdx2  + 2 dx3  − Sdx2   = 0
 ∂x2 ∂x2   ∂x2   ∂x3 ∂x3   ∂x3 
∂h ∂ 2h ∂h ∂h ∂ 2h ∂h
⇒ pdx2 dx3 + Sdx3 + 2 Sdx3dx2 − Sdx3 + Sdx2 + 2 Sdx2 dx3 − Sdx2 =0
∂x2 ∂x2 ∂x2 ∂x3 ∂x3 ∂x3
∂ 2h ∂ 2h
⇒ pdx2 dx3 + Sdx3 dx 2 + Sdx2 dx3 = 0
∂x22 ∂x32
 ∂ 2h ∂ 2h 
⇒ pdx2 dx3 + Sdx2 dx3  2 + 2  = 0
 ∂x2 ∂x3 

University of Castilla-La Mancha Draft By: Eduardo W. V. Chaves (2016)


Ciudad Real - Spain
690 SOLVING PROBLEMS BY MEANS OF CONTINUUM MECHANICS

with which we can conclude that the governing equation for the membrane under pressure
is given by:

∂ 2h ∂ 2h p p
2
+ 2 =− ∇ xr ⋅ (∇ xr h) ≡ ∇ 2xr h = − (6.261)
∂x2 ∂x3 S S

Making an analogy between the above equation and the torsion problem equation (6.250),
we can conclude that
p
h =φ and 2Gθ = = constant (6.262)
S
with which we can say that the moment of torsion, (see equation (6.254)), is equal to two
times the volume defined by the membrane:


A

M T = 2 φ dA = 2 h( x2 , x3 ) dA = 2Vmemb
A
(6.263)

Problem 6.45
Consider a prismatic bar in which there is a longitudinal cavity, and the cross section of the
bar is shown in Figure 6.95. Obtain the equation for M T in terms of the areas A0 and Ai .

x3 1 r ˆ
A= ∫x ⋅ ndΓ

0

− n̂i −1 r ˆ
Ai = x ⋅ n i dΓ

n̂i 2 Γ
i

A0 Γi
Ai x2

n̂0 A0 = A − Ai
Γ0

Figure 6.95: Cross section of the bar with longitudinal cavity.


Solution:
The equation in (6.255) states that
r
M T = − φx ⋅ n̂dΓ + 2φ dA
∫ ∫ (6.264)
Γ A

In order to apply the above equation we will decompose the area as shown in Figure 6.96,
then, the torque can be expressed as follows:
   
 r ˆ   r ˆ 
MT = M TA0 + M TA0 =  − φx ⋅ ndΓ + 2φ dA  +  − φx ⋅ ndΓ + 2φ dA 
∫ ∫ ∫ ∫ (6.265)
 Γ   Γ 
 A0 A0
  A0 A0

University of Castilla-La Mancha Draft By: Eduardo W. V. Chaves (2016)


Ciudad Real – Spain
6 LINEAR ELASTICITY 691

Γ 0 = Γ 1a + Γ a2
d
Γ 2
n̂1d Γ i = Γ 1c + Γ c2
n̂ d2
Γ 1d Γ c2 A0 = A0 + A0
n̂c2 Γ A0 = Γ 1a + Γ 1b + Γ 1c + Γ 1d
n̂1a
Γ A = Γ a2 + Γ b2 + Γ c2 + Γ d2
Γ a n̂1c Γi 0

1
Γ 1c
A0
A0 Γ a2
n̂b2 Γ b2
Γ0
Γ 1b n̂ a2
n̂1b

Figure 6.96: Decomposition of the cross.


By considering that:
1) The boundary of the area A0 is Γ A0 = Γ 1a + Γ 1b + Γ 1c + Γ 1d ;

2) The boundary of the area A0 is Γ A = Γ a2 + Γ b2 + Γ c2 + Γ d2 ;


0

3) There is continuity of the function φ on the boundaries Γ b and Γ d , then we can


b b d d
conclude that φ (Γ 1 ) = φ (Γ 2 ) and φ (Γ 1 ) = φ (Γ 2 ) . Note also that nˆ b2 = −nˆ 1b , nˆ d2 = −nˆ 1d .
Then, the equation (6.265) can be rewritten as follows
   
 r   r 
M T =  − φx ⋅ nˆ dΓ + 2φ dA  +  − φx ⋅ nˆ dΓ + 2φ dA 
∫ ∫ ∫ ∫
 Γ   Γ 
 A0 A0
  A0 A0

r r r r
= − φx ⋅ nˆ dΓ − φx ⋅ nˆ dΓ − φx ⋅ nˆ dΓ − φx ⋅ nˆ dΓ
∫ ∫ ∫ ∫
Γ 1a Γ 1b Γ 1c Γ 1d

 
r r r r  
− φx ⋅ nˆ dΓ − φx ⋅ nˆ dΓ − φx ⋅ nˆ dΓ − φx ⋅ nˆ dΓ +  2φ dA + 2φ dA 
∫ ∫ ∫ ∫ ∫ ∫
A 
Γ 2a Γ b2 Γ c2 Γ 2d  0 A0 
 r r   r r   r r 
M T = − φx ⋅ nˆ dΓ + φx ⋅ nˆ dΓ  −  φx ⋅ nˆ dΓ + φx ⋅ nˆ dΓ  −  φx ⋅ nˆ dΓ + φx ⋅ nˆ dΓ 
∫ ∫ ∫ ∫ ∫ ∫
 a   b   d 
Γ1 Γ 2a  1 4444244443 14444244443
Γ1 Γ b2   Γ1 Γ 2d
=0 =0

 r r   
−  φx ⋅ nˆ dΓ + φx ⋅ nˆ dΓ  +  2φ dA 
∫ ∫ ∫
 c   
Γ 2 Γ 1c   A0 

r r
M T = − φx ⋅ nˆ dΓ + φx ⋅ nˆ dΓ + 2φ dA
∫ ∫ ∫ (6.266)
Γ0 Γi A0

And by means of the equation in (6.255) we can conclude that

University of Castilla-La Mancha Draft By: Eduardo W. V. Chaves (2016)


Ciudad Real - Spain
692 SOLVING PROBLEMS BY MEANS OF CONTINUUM MECHANICS

r r
M T = − φx ⋅ nˆ dΓ + φx ⋅ nˆ dΓ + 2φ dA
∫ ∫ ∫
Γ0 Γi A0
(6.267)

= −2 Aφ Γ 0 + 2 Aiφ Γ i + 2φ dA = 2 Aiφ Γ i + 2φ dA
A0

A0

where A = A0 + Ai , and we have considered that φ on the boundary Γ 0 is zero, i.e.


φΓ 0 = 0 .
The equation (6.267) could also have obtained by considering the equation in (6.255), in
which M T = M TA − M TAi , A = A0 + Ai , φ Γ 0 = φ Γ , then

   
M T = M TA − M TAi =  − 2 Aφ Γ + 2φ dA  −  − 2 Aiφ Γ i + 2φ dA 
∫ ∫
   
 A = A0 + Ai   Ai 


A0
{∫
Ai

= −2 Aφ Γ + 2 Aiφ Γ i + 2φ dA + 2φ dA − 2φ dA = −2 Aφ Γ 0 + 2 Aiφ Γ i + 2φ dA
Ai

A0
=0


= 2 Aiφ Γ i + 2φ dA
A0

NOTE 1: Note that the above equation is also true if the bar contains n longitudinal
cavities, so in this case the torque is given by
n
MT = ∑ (2 A φ Γ ) + ∫ 2φ dA
i =1
i i (6.268)
A0

NOTE 2: Note that if we are dealing with open cross section the torque is smaller than the
closed cross section, (see Figure 6.97).

M T = 2 Aiφ Γ i + 2φ dA∫
A0
M T = 2φ dA ∫
A0

φΓ = 0
0

A0 Ai
φΓ i
A0
φΓ = 0
0

φΓ
φ
i

φΓ = 0
0
φΓ = 0
0

a) Closed cross section b) Open cross section

Figure 6.97

University of Castilla-La Mancha Draft By: Eduardo W. V. Chaves (2016)


Ciudad Real – Spain
6 LINEAR ELASTICITY 693

Problem 6.46
Using the Prandtl’s stress function, a) show that for an elliptical cross section which is
subjected to the torque M T the tangential stresses are:
2M T 2M T
σ12 = − x3 ; σ13 = x2 (6.269)
πab 3 πa 3b
b) Draw the tangential stress distribution on the cross section; c) Obtain the function
u1 ( x2 , x3 ) .

x3

σ13
b
σ12
x2
b

a a

Figure 6.98: Elliptical cross section.


Solution:
a) The ellipse equation is given by:
x22 x32
+ −1 = 0
a2 b2
Since the value of the stress function φ on the boundary is constant, we can assume that:
 x22 x32 
φ = m 2
+ 2
− 1 (6.270)
a b 
where m is a constant to be determined. From the above equation we can obtain:
∂φ  2x  ∂ 2φ 2m
= m 22  ⇒ = 2
∂x 2  a  ∂x 22 a
∂φ  2x  ∂ 2φ 2m
= m 23  ⇒ = 2
∂x3  b  ∂x32 b
and by substituting the above equations into the equation (6.250) we obtain that:
∂ 2φ ∂ 2φ 2m 2 m
+ = −2Gθ = constant = C ⇒ + 2 =C
∂x22 ∂x32 a2 b
 2 2   2( a 2 + b 2 ) 
⇒ m 2 + 2  = C ⇒ m 2 2
=C
 (6.271)
a b   a b 
a 2b 2
⇒m= C
2( a 2 + b 2 )

University of Castilla-La Mancha Draft By: Eduardo W. V. Chaves (2016)


Ciudad Real - Spain
694 SOLVING PROBLEMS BY MEANS OF CONTINUUM MECHANICS

Substituting the value of m into the equation (6.270) we can obtain:


 x22 x32  a 2 b 2  x22 x32 
φ = m 2
+ 2
− 1 ⇒ φ= 2

2  2
+ 2 − 1C (6.272)
a b  2( a + b )  a b 
Next step: Determine C
By substituting the value of φ into the moment of torsion given by (6.254) we can obtain:
 a 2b 2  x22 x32   a 2b 2  x22 x32 
A

M T = 2 φ dA = 2 
A

 2 (∫a 2
+ b )

2  2
 a
+
b 2
− 1 

C
 

dA =
( a 2
+ b 2
)
C a
A

 2 + 2 − 1 dA
b 

a 2b 2  1 1  a 2b 2  1 1 
∫ ∫ ∫
2 2
= 2 2  2
x 2 dA + 2
x3 dA − dA  C = 2 2  2 x3
I + 2 I x 2 − AC
(a + b )  a A b A A  (a + b )  a b 

where:


I x 3 = x 22 dA - moment of inertia of the cross-sectional area about the x3 -axis;
A


I x 2 = x32 dA - moment of inertia cross-sectional area about x2 -axis;
A

A= ∫ dA - cross-sectional area.
A

πba 3 πb 3 a
For an elliptical cross section it fulfils that I x 3 = , I x2 = and A = πab . Then, the
4 4
expression for the moment of torsion becomes:
a 2b 2  1 1  a 2 b 2  1 πba 3 1 πb 3 a 
MT = 2
I
2  2 x3
+ I
2 x2
− A C = 2 2  2
+ 2 − πab C
(a + b )  a b  (a + b )  a 4 b 4 
πa 3b 3
=− C
2( a 2 + b 2 )
And the value of C can be determined by:
− 2M T (a 2 + b 2 )
C=
πa 3b 3
Finally, the stress function (6.272) becomes:
a 2 b 2  x22 x32  − a 2 b 2  x22 x32  2M T (a 2 + b 2 )
φ= 
2  2
+ 2 − 1C ⇒ φ=  + − 1 
2
2( a + b )  a b  2( a 2 + b 2 )  a 2 b 2 
 πa 3b 3
− M T  x 2 x3
2 2

⇒φ =  2 + 2 − 1
πab  a b 

The stresses defined in (6.244) can be expressed by
∂φ ∂  − MT  x22 x32  − 2 M T
σ12 = =   2 + 2 − 1  = x3
∂x3 ∂x3  πab a b  πab3
 
(6.273)
∂φ ∂  − MT  x22 x32  2M T
σ13 = − =−   2 + 2 − 1 = 3 x2
∂x2 ∂x2  πab a 
 b  πa b
b) By means of the above equations we can obtain that:

University of Castilla-La Mancha Draft By: Eduardo W. V. Chaves (2016)


Ciudad Real – Spain
6 LINEAR ELASTICITY 695

σ12 = 0

x3 = 0 ⇒  2M T ( x =a) 2M
σ13 = πa 3b x2 2 → σ13 max = 2T
πa b

 − 2M T ( x =b) − 2M T
σ12 = x3 3 → σ12 max =
x2 = 0 ⇒  πab 3
πab 2
σ = 0
 13
whose components can be appreciated in Figure 6.99. By means of the Pythagorean
Theorem the resultant tangential stress can be obtained:
2 2
 − 2M T   2M T  2M T x22 x32
τ2 = (σ12 ) 2 + (σ13 ) 2 =  x3  +  3 x2  ⇒ τ= +
 πab
3
  πa b  πab b4 a 4

2M T x3
σ12 max = −
πab 2

b 2M T
σ13 max =
πa 2 b

x2
b

a a

Figure 6.99: Tangential stress distribution in the elliptical cross section.

c) We can obtain the angle of twist per unit length by means of equation (6.271):
 − 2M T (a 2 + b 2 ) 
−  
 M (a 2 + b 2 )
−C  πa 3b 3 = T
C = −2Gθ ⇒ θ= =
2G 2G πa 3b 3G
Taking into account the displacement field given by (6.224), we can obtain:
 − M T (a 2 + b2 )
u2 = − x3 θ x1 = x3 x1
 πa 3b3G
 2 2
(6.274)
u = x θ x = M T ( a + b ) x x
 3 2 1
πa 3b3G
2 1

By considering the above equations and the one in (6.249) we can obtain:
∂u1 σ12 ∂u1 σ12 M T (a 2 + b 2 )
= + x3 θ ⇒ = + x3
∂x2 G ∂x2 G πa 3b 3G
(6.275)
∂u1 σ13 ∂u1 σ13 M T (a 2 + b 2 )
= − x2 θ ⇒ = − x2
∂x3 G ∂x3 G πa 3b 3G
Integrating the above equations we can obtain that:

University of Castilla-La Mancha Draft By: Eduardo W. V. Chaves (2016)


Ciudad Real - Spain
696 SOLVING PROBLEMS BY MEANS OF CONTINUUM MECHANICS

σ M (a 2 + b 2 )  σ12 M (a 2 + b 2 )
∫ ∫
∂u1 =  12 + T 3 3
 G πa b G
x3 ∂x2


⇒ u1 =
G
x2 + T 3 3
πa b G
x3 x2 + f ( x3 )

σ M (a 2 + b 2 )  σ13 M (a 2 + b 2 )
∫ ∫
∂u1 =  13 − T 3 3
 G πa b G
x2 ∂x3

⇒ u1 =
G
x3 − T 3 3
πa b G
x3 x2 + f ( x2 )

By substituting the values of σ12 and σ13 , (see equations (6.273)), into the above equations
we can obtain:
 2M T  1 M T (a 2 + b 2 ) M x x
u1 =  − x
3 3  x 2 + 3 3
x3 x2 + f ( x3 ) = T 33 32 (b 2 − a 2 ) + f ( x3 )
 πab G πa b G Gπa b
 2M 1 M (a 2 + b 2 ) M x x
u1 =  3T x2  x3 − T 3 3 x3 x2 + f ( x2 ) = T 33 32 (b 2 − a 2 ) + f ( x2 )
 πa b  G πa b G Gπa b
Note that the two above equations must be the same at the same point ( x2 , x3 ) , hence
f ( x2 ) = f ( x3 ) = 0 , thus the warping function is given by:

M T (b 2 − a 2 )
u1 ( x2 , x3 ) = x2 x3
G πa 3b 3
The above function in the cross-section can be appreciated in Figure 6.100.
For the particular case when a = b , we recover the expressions for the circular cross
section, (see Problem 6.44 - NOTE 1), and in this case there is no warping since
u1 ( x2 , x3 ) = 0 .

x3

u1 positive (up)
u1 negative (down)

x2

a a

Figure 6.100: Function u1 ( x2 , x3 ) .

By means of stress components given by (6.273) we can show that, and as expected, the
shearing forces are equal to zero:
2M T 2M T

Qx2 = σ12 dA = −
A
∫ πab
A
3
x3 dA = −
πab 3 A ∫
x3 dA = 0
123
=0
(6.276)
2M T 2M
A

Qx3 = σ13dA =
A
3
πa b ∫
x2 dA = 3T x2 dA = 0
πa b A ∫
123
=0

University of Castilla-La Mancha Draft By: Eduardo W. V. Chaves (2016)


Ciudad Real – Spain
6 LINEAR ELASTICITY 697

where ∫ x dA
A
3 is the first moment of area about x3 -axis, and is equal to zero, since the

reference system is located at the geometrical center, (see Complementary Note 2 at the
end of Chapter 1).

Problem 6.47
Consider a circular cross section with radius R , and that the Prandtl’s stress function is
given by:
φ = K ( x22 + x32 − R 2 ) (6.277)
Obtain the stress and displacement fields.
Solution:
By means of equation (6.250) we can obtain:
∂ 2φ ∂ 2φ Gθ
+ = −2Gθ ⇒ 4 K = −2Gθ ⇒ K =− (6.278)
∂x22 ∂x32 2
thus,
Gθ 2
φ =− ( x2 + x32 − R 2 ) (6.279)
2
And by applying the net moment defined in (6.254) we can obtain:
 

A
∫ A
∫  A

M T = 2 φ dA = −Gθ ( x22 + x32 − R 2 ) dA = −Gθ  ( x22 + x32 ) dA − R 2 dA
A 

 
∫ [
= −Gθ (r 2 ) dA − R 2 A = −Gθ JT − R 2 A = −Gθ]
 πR 4 
− R 2 πR 2  (6.280)
 A   2 
πR 4
= Gθ
2
πR 4
where J T = ∫ r 2 dA = is the polar inertia moment, and the circle area is A = πR 2 , then
A
2
2M T
θ= .
πGR 4
The stress field, (see equations in (6.244)), becomes:
 ∂φ Gθ ∂ 2 2 2 2M T
σ12 = ∂x = − 2 ∂x ( x2 + x3 − R ) = −Gθx3 = − πR 4 x3
 3 3
 (6.281)
σ = − ∂φ = Gθ ∂ ( x 2 + x 2 − R 2 ) = Gθx = 2 M T x
 13 ∂x2 2 ∂x2
2 3 2
πR 4
2

By using the stress definition (6.229) we can obtain:


 ∂u  ∂u1 σ12 − Gθx3 
σ12 =  1 − x3 θ  ⇒ = + x3 θ = + x3 θ = 0
 ∂x2  ∂x2 G G 
 ⇒ u1 = 0
 ∂u  ∂u1 σ13 Gθx2
σ13 =  1 + x2 θ  ⇒ = − x2 θ = − x2 θ = 0 
 ∂x3  ∂x3 G G 

University of Castilla-La Mancha Draft By: Eduardo W. V. Chaves (2016)


Ciudad Real - Spain
698 SOLVING PROBLEMS BY MEANS OF CONTINUUM MECHANICS

As expected u1 = 0 , since for circular cross section there is no warping. This problem was
already discussed in Problem 6.44 - NOTE 1.

6.4.1 Torsion of Thin-Walled Cross Section


6.4.1.1 Open Thin-Walled Section

Problem 6.48
Apply the torsion theory to obtain the maximum shearing stress ( τ max ) in a thin rectangular
section described in Figure 6.101. Express the result in terms of ( M T , t , b) . Consider that
the Prandtl’s stress function, (Ugural&Fenster (1984)), is given by:
 2  t 2 
φ = K  x3 −    (6.282)
  2  

x3

t
2 x2
t
2

Figure 6.101: Thin rectangular cross section.


Solution:
By means of equation (6.250) we can obtain:
∂ 2φ ∂ 2φ
+ = −2Gθ ⇒ 2 K = −2Gθ ⇒ K = −Gθ (6.283)
∂x 22 ∂x32
thus,
 2  t 2 
φ = −Gθ x3 −    (6.284)
  2  

And by applying the net moment defined in equation (6.254) we can obtain:
 t 
2
 t 
2

A
∫ A


M T = 2 φ dA = −2Gθ  x32 −    dA = −2Gθ  x32 −    dA
 2   A
  2  

b
t  b
 t

2
 2  2  t 2   2  3
 x3  t  2
 2

∫ ∫
−b  −t 
 −b 

= −2Gθ   x3 −    dx3  dx2 = −2Gθ   −   x3   dx2
 2     3  2   − t 
2 2  2  2 
b b
2
t3  t3  2

= −2Gθ   dx2 = −2Gθ x2 
6
−b    6  −b
2 2

Then, we can obtain

University of Castilla-La Mancha Draft By: Eduardo W. V. Chaves (2016)


Ciudad Real – Spain
6 LINEAR ELASTICITY 699

t 3b
M T = Gθ (6.285)
3
t 3b
By considering that M T = GθJT , (see equation (6.237)), we can conclude that JT = ,
3
3M T
then, Gθ = .
t 3b
The tangential stress field becomes:
 ∂φ ∂  2 t 
2
6M
σ12 = = −Gθ  x3 −    = −2Gθx3 = − 3 T x3
 ∂x3 ∂x3   2   tb

∂  2 t 
2
 ∂φ
σ13 = − = Gθ  x3 −    = 0
∂x2 ∂x2   2  

t
The maximum shearing stress occurs at x3 = ± , thus:
2
t 6M t 3M
σ12 ( x3 = ± ) = τmax = m 3 T = m 2 T
2 tb 2 t b

x3 − 3M T
τ max =
t 2b

t
σ12 ( x3 ) 2 x2
t
2

b 3M T
τ max =
t 2b

Figure 6.102: Stress distribution.

NOTE 1: Let us obtain the same result by means of Prandtl’s membrane analogy, (see
Problem 6.44 – NOTE 4). The membrane deflection ( h ) for thin rectangular cross
section, (see Figure 6.101), can be appreciated in Figure 6.103.
∂h
Note that the membrane deflection does not depend on x2 , then = 0 . With that the
∂x2
equation in (6.261) becomes:
∂ 2h ∂ 2h p ∂ 2h p
2
+ 2 =− ⇒ 2
=− (6.286)
∂x2 ∂x3 S ∂x3 S
Then by integrate the above equation over x3 we can obtain:
∂ 2h ∂ 2h p d 2h p integrating over x dh p
2
+ 2 =− ⇒ 2
=−     
3 → = − x3 + C1 (6.287)
∂x2 ∂x3 S dx3 S dx3 S

University of Castilla-La Mancha Draft By: Eduardo W. V. Chaves (2016)


Ciudad Real - Spain
700 SOLVING PROBLEMS BY MEANS OF CONTINUUM MECHANICS

x3 x3
h
(paraboloid)

t
2 x2 p x1
t
2

b ∂h
=0
∂x3

Figure 6.103: Membrane deflection for thin rectangular cross section.


∂h
Note that at x3 = 0 the membrane slope is equal to zero, i.e. = 0 , (see Figure 6.103),
∂x3
then the constant of integration equals zero, C1 = 0 . Then,
dh p integratin g over x p x32
= − x3     
3 → h=− + C2 (6.288)
dx3 S S 2
t
At x3 = ± the membrane deflection is equal to zero, thus
2
2
t
 
p 2 pt 2
h( x3 = ± 2 ) = −
t
+ C2 = 0 ⇒ C2 =
S 2 8S
With that the membrane deflection equation becomes:
p  t  
2
p x32 pt 2 2
h=− + =   − x3  (6.289)
S 2 8S 2S  2  
2
The maximum deflection of the membrane occurs at x3 = 0 , and is equal to hmax = pt . The
8S
volume of the paraboloid defined by the membrane is equal to:
2 2 pt 2 bt 3 p bt 3 Gθbt 3
Vmemb = bhmaxt = b t= = 2Gθ = (6.290)
3 3 8S 12S 12 6
where we have used the definition p = 2Gθ , (see equation (6.262)). And by using equation
S
(6.263) the moment of torsion can be expressed as follows:
Gθbt 3

M T = 2 h( x2 , x3 ) dA = 2Vmemb =
A
3
(6.291)

By considering that M T = GθJT , (see equation (6.237)), we can conclude that


M T bt 3
JTeff = = (6.292)
Gθ 3
The maximum tangential stress, (see Figure 6.102), can be expressed in terms of JTeff as
follows:

University of Castilla-La Mancha Draft By: Eduardo W. V. Chaves (2016)


Ciudad Real – Spain
6 LINEAR ELASTICITY 701

3M T M t M t
τ max = = 3T = T
2
t b  t b  JTeff (6.293)
 
 3 
 
This solution is the base to solve other cases in which the cross section is made up by
several elongated rectangles (thin open wall cross section), (see Figure 6.104), with the
bi
condition > 10 .
ti

b3

t3
bi
> 10
t4 b4 ti

t2 ti - thickness
b2
bi - base

t1
b1

Figure 6.104: Thin open wall cross section.


In this case we have:
n
bi t i3
J Teff = ∑
i =1 3
(6.294)

and
MT MT
θ= = n
GJTeff bi ti3 (6.295)
G ∑
i =1 3

And the tangential stress can be evaluated as follows


MT t
τ= or τ = tGθ (6.296)
JTeff

For example, let us consider Figure 6.104 in which t1 = t2 = t3 = t 4 = t , b1 = b3 = a and


a
b2 = b4 = . Then, the equation (6.294) becomes:
2
a 3 a
4 at 3 + t + at 3 + t 3
bi ti3 b1t13 + b2t 23 + b3t33 + b4t43
JTeff = ∑
i =1 3
=
3
= 2
3
2 = at 3 (6.297)

And the angle of twist per unit length, (see equation (6.295)), becomes:
MT MT
θ= = (6.298)
GJTeff Gat 3

University of Castilla-La Mancha Draft By: Eduardo W. V. Chaves (2016)


Ciudad Real - Spain
702 SOLVING PROBLEMS BY MEANS OF CONTINUUM MECHANICS

The maximum tangential stress, (see equation (6.293)), becomes


MT t MT t MT
τ max = = 3 = 2 (6.299)
JTeff at at

6.4.1.2 Closed Thin-Walled Section

Problem 6.49
Obtain the equations for torsion problem by means of membrane analogy and by
considering a thin-walled closed section.
Hypothesis: Consider the membrane deflection as the one described in Figure 6.105.

Average circumference - s Thin-walled closed section

x3 r

A- Area formed by s x2

membrane deflection

h
α α

t t

Figure 6.105: Thin-walled closed section.


Solution:
According to the membrane analogy, (see equation (6.262)), it is true that:
p
h =φ and 2Gθ = (6.300)
S
And according to Figure 6.105 the membrane slope is constant, and then the tangential
stress is also constant, (see equation (6.256)). The tangential stress ( τ ) can be obtained by
means of membrane slope:
h
tan α = τ = (6.301)
t
The moment of torsion is related to the membrane volume by M T = 2Vmemb , and the
membrane volume can be obtained as follows Vmemb = Ah , where A is the area formed by
the average circumference s , (see Figure 6.105). Then M T = 2 Ah , and the stress (6.301)
becomes:
h MT
τ= = (6.302)
t 2 At
which is the same as the one presented in NOTE 2-Problem 4.27.

University of Castilla-La Mancha Draft By: Eduardo W. V. Chaves (2016)


Ciudad Real – Spain
6 LINEAR ELASTICITY 703

We have obtained that ∫ τ dΓ = 2GθA , (see equation (6.260)), thus:


Γ

 MT 
∫ τ dΓ Γ∫  2 At  dΓ (6.303)
θ= Γ
=
2GA 2GA
The above equation is known as Bredt’s formula (Ugural&Fenster (1984)). When the
thickness is constant we can conclude that:
 MT 
∫  2 At  dΓ
Γ MT s (6.304)
θ= =
2GA 4GA2t
where s is the perimeter of the cross section. For the case presented in Figure 6.105 we
have
MT MT MT
τ= = = (6.305)
2 At 2(πr )t 2πr 2t
2

MT s M T (2πr ) MT
and θ = = = .
4GA t 4G (πr ) t 2Gπr 3t
2 2 2

Once the angle of twist per unit length is obtained we can obtain the JT by means of the
equation M T = GJT θ , (see equation (6.237)):
GJT M T
M T = GJT θ = ⇒ JTeff = 2πr 3t (6.306)
2Gπr 3t

NOTE 1: Multiply Connected Thin-Walled Section, (Sechler (1952))


We can also use the previous study to solve problem with multiply connected thin-walled
section, (see Figure 6.106).

s2 - perimeter
τ1 1
s1 - perimeter
t2 - thickness
t1 - thickness s3 - perimeter
τ2
1 t3 - thickness 3
Cell 2 2
h
τ1 = 1 τ3
t1 Cell 1
h2 A1 - Cell 1 area
τ2 =
t2 A2 - Cell 2 area
h − h1 τ1 2
τ3 = 2
t3
h3
h2
h1 Vmemb = A1h1 + A2h2

Figure 6.106: Multiply connected thin-walled section.

University of Castilla-La Mancha Draft By: Eduardo W. V. Chaves (2016)


Ciudad Real - Spain
704 SOLVING PROBLEMS BY MEANS OF CONTINUUM MECHANICS

In this case we have two cells. For each cell we apply the definition ∫ τ dΓ = 2GθA , (see
Γ
equation (6.260)), thus
Cell 1: ∫ τ dΓ = 2GθA
Γ
1 ⇒ τ1s1 + τ3 s3 = 2GA1θ ⇒ τ1s1 + τ3 s3 − 2GA1θ = 0
(6.307)

Cell 2: ∫ τ dΓ = 2GθA2 ⇒ τ 2 s2 − τ3 s3 = 2GA2 θ ⇒ τ2 s2 − τ3 s3 − 2GA2θ = 0 (6.308)


Γ

We apply the equation M T = 2Vmemb , where the membrane volume is Vmemb = A1h1 + A2h2 ,
thus:
MT
M T = 2Vmemb = 2( A1t1τ1 + A2t2 τ2 ) ⇒ A1t1τ1 + A2t2 τ2 = (6.309)
2
Next we apply the flux continuity at each node. In this case we have two nodes:
Node 1: t1τ1 = t2 τ2 + t3τ3 ⇒ − t1τ1 + t2 τ2 + t3τ3 = 0 (6.310)
and
Node 2: t2 τ2 + t3τ3 = t1τ1 ⇒ t1τ1 − t2 τ2 − t3τ3 = 0 (6.311)
Note that the last equation is redundant one.
Then, by considering the equations (6.307), (6.308), (6.309) and (6.310), we can construct
the following set of equations:
Cell 1  s1 0 s3 − 2GA1   τ1   0 
Cell 2  0  
 s2 − s3 − 2GA2  τ 2   0 
     
   =  M  ⇔ [ F ]{τ } = { g} (6.312)
moment 0   τ3   T
 A1t1 A2t 2 0 
    2 
flux at node 1     
 − t1 t2 t3 0   θ   0 

There is solver that does not accept that the terms of principal diagonal are zero, if it is the
case we can overcome this drawback by sum the third and fourth rows by the first row.
Recall that if we sum rows the result is not affected. Then:
 s1 0 s3 − 2GA1   τ1   0 
 0     0 
 s2 − s3 − 2GA2  τ 2   
      
   =  M T  ⇔ [ F ]{τ } = { g} (6.313)
 A1t1 + s1 A2t 2 s3 − 2GA1  τ3   + ( g1 = 0)
    2 
    
 − t1 + s1 t2 t 3 + s3 − 2GA1   θ   0 + ( g = 0) 
 1 
The solution for the above set of equations is:

 τ1  − GM T ( s2t3 A1 + s3t2 A2 + s3t2 A1 )


τ   − GM ( s t A + s t A + s t A ) 
 2 1  T 1 3 2 31 2 31 1 
{τ } =   =  GM T ( s1t 2 A2 − s2t1 A1 )  (6.314)
τ3  det[ F ]  − 1 
 θ   M T ( s1s2t3 + s1s3t 2 + s2 s3t1 ) 
 2 

University of Castilla-La Mancha Draft By: Eduardo W. V. Chaves (2016)


Ciudad Real – Spain
6 LINEAR ELASTICITY 705

where det[ F ] = −2G[t1t 2 s3 ( A1 + A2 ) 2 + t3 ( s1t2 A22 + s2t1 A12 )] . Note that, when t = t1 = t2 ,
s = s1 = s2 , A = A1 = A2 , the solution becomes:

 MT 
 4 At 
 τ1   
τ   MT 
 2  
{τ } =   =  4 At  (6.315)
τ3   
 θ   0 
 sM 
 T

 8GA2t 
Once the angle of twist per unit length is obtained we can obtain the JT by means of the
equation M T = GJT θ , (see equation (6.237)):
GJT sM T 8 A3t
M T = GJT θ = ⇒ JTeff = (6.316)
8GA2t s

Problem 6.50
Obtain the tangential stress in each segment and the angle of twist θ for the multiply
connected thin-walled section described in Figure 6.107, Cervera&Blanco (2004), in which
the moment of torsion is M T = 5× 10 3 Nm and the shear modulus is G = 1× 1011 Pa .

τ2 τ3
2
. 2
.3 3
.4 Data (Dimensions in mm )
a = 50
τ4
Segment Lengths:
9 τ9 4 a
Cell 2 s1 = 100 , s2 = s3 = s4 = s5 = 50
s6 = s7 = s8 = s9 = s10 = 50
τ1 1
Cell 1 .
8 8 5
. Segment Thicknesses:
τ8 t1 = t2 = t3 = t4 = t5 = t6 = t7 = 5
5
10 t8 = t9 = t10 = 4
τ 10 Cell 3 τ5
a
Cell areas:

τ7 τ6 A1 = 2a × a = 5000mm2
1 . 7
. 7 6 6
. A2 = A3 = a × a = 2500mm2

a a

Figure 6.107: Multiply connected thin-walled section.


Solution:
For the three cells we apply the equation (6.260), i.e.:
Cell 1: ∫ τ dΓ = 2GθA
Γ C1
1 ⇒ τ1 s1 + τ 2 s2 + τ 7 s7 + τ 9 s9 + τ10 s10 = 2GA1θ

University of Castilla-La Mancha Draft By: Eduardo W. V. Chaves (2016)


Ciudad Real - Spain
706 SOLVING PROBLEMS BY MEANS OF CONTINUUM MECHANICS

Cell 2:
Γ C2
∫ τ dΓ = 2GθA 2 ⇒ τ3 s3 + τ 4 s4 + τ8 s8 − τ9 s9 = 2GA2θ

Cell 3: ∫ τ dΓ = 2GθA
Γ C3
3 ⇒ τ 5 s5 + τ 6 s6 − τ8 s8 − τ10 s10 = 2GA3 θ

Next the following must fulfill:


M T = 2Vmemb = 2( A1t1τ1 + A2t4 τ 4 + A3t6 τ6 ) (moment equation)
We have 11 unknowns and 4 equations, the 7 missing equations can be obtained by
tangential stress flux compatibility at each node. Note that we have 8 nodes, but we only
need 7 of them:
Node 1: − t1τ1 + t7 τ7 = 0
Node 2: − t1τ1 + t2τ2 = 0
Node 3: − t2 τ2 + t3τ3 + t9 τ9 = 0
Node 4: − t3τ3 + t4τ4 = 0
Node 5: − t4 τ4 + t5τ5 + t8τ8 = 0
Node 6: − t5τ5 + t6τ6 = 0
Node 7: − t6 τ6 + t7 τ7 − t10 τ10 = 0

Then, by restructuring the above 11 equations in matrix form we can obtain:


Cell 1  s1 s2 0 0 0 0 s7 0 s9 s10 − 2GA1   τ 1   0 
 0  
Cell 2
 0 s3 s4 0 0 0 s8 − s9 0 − 2GA2  τ 2   0 
Cell 3  0 0 0 0 s5 s6 0 − s8 0 − s10 − 2GA3  τ 3   0 
    M 
moment  A1t1 0 0 A2t4 0 A3t6 0 0 0 0 0  τ 4   T 
2
node 1  t1 0 0 0 0 0 − t7 0 0 0 0  τ 5   0 
     
node 2  − t1 t2 0 0 0 0 0 0 0 0 0  τ 6  =  0 
node 3  0 −t t3 0 0 0 0 0 t9 0 0  τ 7   0 
 2
   
node 4  0 0 − t3 t4 0 0 0 0 0 0 0  τ 8   0 
    
node 5
 0 0 0 − t4 t5 0 0 t8 0 0 0  τ 9   0 
node 6  0 0 0 0 − t5 t6 0 0 0 0 0  τ 10   0 
    
node 7  0 0 0 0 0 − t6 t7 0 0 − t10 0   θ   0 
[ F ]{τ } = { g}
(6.317)
The above matrix [F ] has zeros in the principal diagonal, we will sum rows to overcome
this drawback, and then after summation is taken place we can obtain:

University of Castilla-La Mancha Draft By: Eduardo W. V. Chaves (2016)


Ciudad Real – Spain
6 LINEAR ELASTICITY 707

 s1 s2 0 0 0 0 s7 0 s9 s10 − 2GA1   τ 1   0 
s  
 1 s2 s3 s4 0 0 s7 s8 0 s10 − 2G ( A2 + A1 ) τ 2   0 
 0 0 − t3 t4 s5 s6 0 − s8 0 − s10 − 2GA3  τ 3   0 
    M T 
 A1t1 0 0 A2t4 0 A3t6 0 0 0 0 0  τ 4   2 
 t1 0 0 0 − t5 t6 t7 0 0 0 0  τ 5   0 
     
 − t1 t2 0 0 − t5 t6 0 0 0 0 0  τ 6  =  0 
 0 −t t3 0 0 − t6 t7 0 t9 − t10 0  τ   
 2
 7   0 
 0 0 − t3 0 t5 0 0 t8 0 0 0  τ 8   0 
    
 0 − t2 t3 − t4 t5 − t6 t7 t8 t9 − t10 0  τ 9   0 
 0 0 0 0 − t5 0 t7 0 0 − t10 0  τ 10   0 
    
 s1 s2 0 0 0 − t6 t7 + s7 0 s9 s10 − t10 − 2GA1   θ   0 

After substituting the data and solving the above system we can obtain:
rad
τ 1 = τ 2 = τ 3 = τ 4 = τ 5 = τ 6 = τ 7 = 50 × 106 Pa ; τ 8 = τ 9 = τ 10 = 0 ; θ = 0.01
m
Note that, this problem is the same as the result presented in equation (6.315), in which:
MT 5 × 10 3
τ= = −6 −3
= 50 × 10 6 Pa ,
4 At 4 × (5000 × 10 ) × (5 × 10 )

sM T sτ (200 × 10 −3 ) × (50 × 10 6 ) rad


θ= 2
= = 11 −6
= 0.01 ,
8GA t 2GA 2(10 )(5000 × 10 ) m
where s = s1 + s2 + s7 + s3 + s4 + s5 + s6 is the total perimeter.
NOTE 1: We can also use this methodology for a cross section formed by only one cell,
(see Figure 6.108).

4 3
t3

τ3 3
τ4 t2
2
Cell 1
b
4 τ2
t4
τ1 1

1 t1
2
a

Figure 6.108: Thin-walled section.

Solution for the problem described in Figure 6.108:


Cell 1: ∫ τ dΓ = 2GθA
Γ C1
⇒ τ1s1 + τ 2 s2 + τ3 s3 + τ 4 s4 = 2GAθ

The moment equation is:


M T = 2Vmemb = 2 At2 τ 2

University of Castilla-La Mancha Draft By: Eduardo W. V. Chaves (2016)


Ciudad Real - Spain
708 SOLVING PROBLEMS BY MEANS OF CONTINUUM MECHANICS

We have 5 unknowns and 2 equations, the 3 missing equations can be obtained by


tangential stress flux compatibility at each node. Note that we have 4 nodes, but we only
need 3 of them:
Node 1: − t4 τ4 + t1τ1 = 0
Node 2: − t1τ1 + t2 τ2 = 0
Node 3: − t2 τ2 + t3τ3 = 0
Then, the set of equations to be solved is:
Cell 1  s1 s2 s3 s4 − 2GA τ 1   0 
 0 At M 
moment  2 0 0 0  τ 2   T 
 t1    2 
node 1 0 0 − t4 0  τ 3  =  0 
 
node 2 − t1 t2 0 0 0  τ 4   0 
   
node 3  0 − t2 t3 0 0   θ   0 

And the solution is:


 1 
 2 At1 
 
 1 
τ 1 
τ   2 At2 
 2  1 
 
τ
 3 = M T 2 At3  (6.318)
τ   
 4  1 
 θ   2 At4 
 
 1  s1 s2 s3 s4 
+ + +  
 4 A2G  t1 t2 t3 t4 

For the particular case when t1 = t 2 = t3 = t 4 = t , s1 = s3 = a , s2 = s4 = b , A = ab and the


total perimeter sT = s1 + s2 + s3 + s4 = 2(a + b) , we can obtain
MT M
τ1 =τ 2 =τ 3 =τ 4 =τ = = T
2 At 2abt
M T  s1 s2 s3 s4  M T sT M (2(a + b)) M T (a + b)
θ= 2 
 + + + = = T =
 2
4 A G  t1 t2 t3 t4  4 A Gt 4(ab) 2 Gt 2a 2b 2Gt

By considering that M T = GθJT , (see equation (6.237)), we can conclude that


M T (a + b) M T 2 a 2b 2t
θ= = ⇒ JT =
2a 2b 2Gt GJT (a + b)

a a2
For the particular case when t1 = t 2 = t3 = t 4 = t , s1 = s3 = a , s2 = s4 = b = , A= and the
2 2
total perimeter sT = s1 + s2 + s3 + s4 = 3a , the above equations become
MT MT
τ1 =τ 2 =τ 3 =τ 4 =τ = =
2 At a 2t

M T  s1 s2 s3 s4  M T (a + b) 3M T
θ=  + + + = = 3
4 A2G  t1 t2 t3 t4  2a 2b 2Gt a Gt

University of Castilla-La Mancha Draft By: Eduardo W. V. Chaves (2016)


Ciudad Real – Spain
6 LINEAR ELASTICITY 709

By considering that M T = GθJT , (see equation (6.237)), we can conclude that


M T sT 3M M 4 A2t a 3t
θ= = 3 T = T ⇒ JT = =
4 A Gt a Gt GJT
2
sT 3
Note that this particular case has the same geometry as the one given by the results in
equations (6.297)-(6.299), in which we were dealing with an open cross section. As we can
see, since t << a , the close cross section is more rigid than the open close, a fact that can
be checked by the angle of twist per unit length, (see equation (6.298)):
3M T   t   M T   t  2  ( open)
2
θ(close ) = =    
3 = 3   θ ⇒ θ( close ) < θ( open)
a 3Gt   a   Gat 3   a  
1424 3
<< 1

And according to equation (6.297) we can also conclude


a 3t
JT(Open ) = at 3 << = JT(Close )
3
We can also apply the same methodology to cross section formed by curved segment, (see
Figure 6.109), in which the curved segment is discretized by linear segments.

2
3

2
4
3
1
4 1
5 Cell 1
5
8
x3 6
6 7

7
8
x2

Figure 6.109: Thin-walled section formed by curved segment.


If we consider the e -element connectivity as Node i( x2(i ) , x3(i ) ) → Node j ( x2( j ) , x3( j ) ) , the
area of the cell can be obtained as follows:
N elem
 (i ) ( j ) ( x 2 − x2 ) 
(i ) ( j)
ACell = ∑ ( x3 + x3 )
e =1  2

University of Castilla-La Mancha Draft By: Eduardo W. V. Chaves (2016)


Ciudad Real - Spain
710 SOLVING PROBLEMS BY MEANS OF CONTINUUM MECHANICS

Problem 6.51
Obtain the tangential stress in each segment and the angle of twist per unit length θ for the
multiply connected thin-walled section described in Figure 6.110, in which the moment of
torsion is M T = 360 and the mechanical properties are E = 2.1 × 10 7 (Young’s modulus),
ν = 0.3 (Poisson’s ratio).

X3

7 3 6 9 5

τ4 4 τ3 τ9 8 τ8
1 4
Cell 1 Cell 3
2
τ1 Cell 2
6 7
τ2 τ6
1 τ7
5

2 τ5 3 X2

Element Connectivity
Coordinates i→ j Thickness
Nodes e
X2 X3 1 1→ 2 t1 = 0.03
1 0 1 .5 2 2→6 t 2 = 0.01
2 3 0
3 6→7 t3 = 0.03
3 9 0
4 7 →1 t 4 = 0.03
4 12 1 .5
5 12 2 5 2→3 t5 = 0.03
6 6 2 6 3→6 t6 = 0.01
7 0 2 7 3→ 4 t7 = 0.03
8 4→5 t8 = 0.03
9 5→6 t9 = 0.03

Figure 6.110: Multiply connected thin-walled section.


Solution:
Loop in Elements in order to calculate the element lengths:
se = ( X 2( j ) − X 2(i ) ) 2 + ( X 3( j ) − X 3(i ) )2

e = 1(i = 1; j = 2) ⇒ s1 = (3 − 0) 2 + (0 − 1.5) 2 = 11.25

s2 = 13 , s3 = 6 , s4 = 0.5 , s5 = 6 , s6 = 13 , s7 = 11.25 , s8 = 0.5 , s9 = 6


Loop in Cells in order to calculate the cell areas:
 (i )
N elem
( j) ( X 2 − X 2 ) 
(i ) ( j) 4
 (i ) ( j) ( X 2 − X 2 ) 
(i ) ( j)

Cell 1: Elements 1,2,3,4 :


A1 = ∑ ( X 3 + X 3 )
e =1  2
= ∑
( X 3 + X 3 )
2

 e=1  
(0 − 3) (3 − 6) ( 6 − 0) (0 − 0)
A1 = (0 + 1.5) + (0 + 2) + (2 + 2) + (2 + 1.5) = 6.75
2 2 2 2
Cell 2: Elements 5,6,2 : Area: A2 = 6

University of Castilla-La Mancha Draft By: Eduardo W. V. Chaves (2016)


Ciudad Real – Spain
6 LINEAR ELASTICITY 711

Cell 3: Elements 7,8,9,6 : Area: A3 = 6.75


For the three cells we apply the equation (6.260), i.e.:
Cell 1: ∫Γ τ dΓ = 2GθA 1 ⇒ τ1s1 + τ 2 s2 + τ3 s3 + τ 4 s4 − 2GA1θ = 0
C1

Cell 2: ∫ τ dΓ = 2GθA
Γ C2
2 ⇒ τ 5 s5 + τ 6 s6 − τ 2 s 2 − 2GA2 θ = 0

Cell 3: ∫ τ dΓ = 2GθA
Γ C3
3 ⇒ τ 7 s7 + τ8 s8 + τ9 s9 − τ 6 s6 − 2GA3θ = 0

The moment equation:


M T = 2Vmemb = 2( A1t1τ1 + A2 t5 τ5 + A3t7 τ 7 )
We have 10 unknowns and 4 equations, the 6 missing equations can be obtained by
tangential stress flux compatibility at each node. Note that we have 7 nodes, but we only
need 6 of them:
Node 1: t 4 τ 4 − t1τ1 = 0
Node 2: t1τ1 − t 2 τ 2 − t5 τ 5 = 0
Node 3: t5 τ 5 − t6 τ 6 − t 7 τ 7 = 0
Node 4: t7 τ 7 − t8 τ8 = 0
Node 5: t8 τ8 − t9 τ 9 = 0
Node 6: t9 τ 9 + t 6 τ 6 + t 2 τ 2 − t3 τ3 = 0
Then, by restructuring the above 10 equations in matrix form we can obtain:
Cell 1  s1 s2 s3 s4 0 0 0 0 0 − 2GA1  τ 1   0 
 0  
Cell 2
 − s2 0 0 s5 s6 0 0 0 − 2GA2  τ 2   0 
Cell 3  0 0 0 0 0 − s6 s7 s8 s9 − 2GA3  τ 3   0 
    M 
moment  A1t1 0 0 0 A2 t5 0 A3t 7 0 0 0  τ 4   T 
2
node 1  − t1 0 0 t4 0 0 0 0 0 0  τ 5   0 

   =  
node 2  t1 − t2 0 0 − t5 0 0 0 0 0  τ 6   0 
node 3  0 0 0 0 t5 − t6 − t7 0 0 0  τ 7   0 
    
node 4  0 0 0 0 0 0 t7 − t8 0 0  τ 8   0 
 
node 5  0 0 0 0 0 0 0 t8 − t9 0  τ 9   0 
   
node 6  0 − t3 0   θ   0
t2 0 0 t6 0 0 t9 
(6.319)
E
where G = (shear modulus). After substituting the data and solving the above system
2(1 + ν )
we can obtain:
τ 1 = τ 3 = τ 4 = τ 7 = τ 8 = τ 9 = 300.8354544 , τ 2 = −τ 6 = −66.85432 , τ 5 = 323.120227 ,
θ = 2.497666 × 10 −5 .

University of Castilla-La Mancha Draft By: Eduardo W. V. Chaves (2016)


Ciudad Real - Spain
712 SOLVING PROBLEMS BY MEANS OF CONTINUUM MECHANICS

Problem 6.52
Obtain the shear flux in each segment and the angle of twist θ for the multiply connected
thin-walled section described in Figure 6.111, in which the moment of torsion is M T and
consider the mechanical property G (shear modulus).

X3

4 3 3 7
6
τ3 τ7
4 Cell 1
τ4 τ2 6
Cell 2 τ6

2
τ1 τ5
1 1 2 5 5 X2

Element Connectivity
Coordinates i→ j Thickness
Nodes e
X2 X3 1 1→ 2 t1 = t
1 0 0 2 2→3 t2 = t
2 2a 0
3 3→ 4 t3 = t
3 2a a
4 4 →1 t4 = t
4 0 a
5 3a 0 5 2→5 t5 = t
6 3a a 6 5→6 t6 = t
7 6→3 t7 = t

Figure 6.111: Multiply connected thin-walled section.


Solution:
Loop in Elements in order to calculate the element lengths:
se = ( X 2( j ) − X 2(i ) ) 2 + ( X 3( j ) − X 3(i ) )2

e = 1(i = 1; j = 2) ⇒ s1 = ( 2a − 0) 2 + (0 − 0) 2 = 2a

s2 = a , s3 = 2a , s4 = a , s5 = a , s6 = a , s7 = a
Loop in Cells in order to calculate the cell areas:
Cell 1: Elements 1,2,3,4 :
N elem
 ( X 2(i ) − X 2( j ) )  4
 ( X 2(i ) − X 2( j ) ) 
A1 = ∑ ( X
e =1
(i )
3 + X 3( j ) )
2
=

∑ ( X
e =1
(i )
3 + X 3( j ) )
2


2
⇒ A1 = 2a
Cell 2: Elements 5,6,7,2 : Area: A2 = 2a 2
For the three cells we apply the equation (6.260), i.e.:
Cell 1: ∫ τ dΓ = 2GθA
Γ C1
1 ⇒ τ1s1 + τ 2 s2 + τ3 s3 + τ 4 s4 − 2GA1θ = 0

University of Castilla-La Mancha Draft By: Eduardo W. V. Chaves (2016)


Ciudad Real – Spain
6 LINEAR ELASTICITY 713

Cell 2: ∫ τ dΓ = 2GθA
Γ C2
2 ⇒ τ5 s5 + τ 6 s6 + τ 7 s7 − τ 2 s2 − 2GA2 θ = 0

The moment equation:


M T = 2Vmemb = 2( A1t1τ1 + A2t5 τ5 )
We have 8 unknowns and 3 equations, the 5 missing equations can be obtained by
tangential stress flux compatibility at each node. Note that we have 6 nodes, but we only
need 5 of them:
Node 1: t 4 τ 4 − t1τ1 = 0
Node 2: t1τ1 − t 2 τ 2 − t5 τ5 = 0
Node 3: t2 τ 2 − t3τ3 + t7 τ7 = 0
Node 4: t3τ3 − t4 τ 4 = 0
Node 5: t5 τ5 − t6 τ6 = 0
Then, by restructuring the above 8 equations in matrix form we can obtain:
Cell 1  s1 s2 s3 s4 0 0 0 − 2GA1  τ 1   0 
 0  
Cell 2
 − s2 0 0 s5 s6 s7 − 2GA2  τ 2   0 
M
moment  A1t1 0 0 0 A2t5 0 0 0  τ 3   T 
    2 
node 1  − t1 0 0 t4 0 0 0 0  τ 4   0 
 t1  = 
node 2 − t2 0 0 − t5 0 0 0  τ 5   0 
 
node 3  0 t2 − t3 0 0 0 t7 0  τ 6   0 
 0    
node 4 0 t3 − t4 0 0 0 0  τ 7   0 
 
node 5  0 0 0 0 t5 − t6 0 0   θ   0 

(6.320)
After substituting the data and solving the above system we can obtain:
9M T M 2M T 23M T
τ 1 =τ 3 =τ 4 = , τ 2 = T2 , τ 5 = τ 6 = τ 7 = 2
, θ= .
2
52a t 52a t 13a t 104a 3tG
And the shear flux ( q = tτ ) can be obtained as follows
9M T M 2M T
q (1) = t1,3, 4τ 1,3, 4 ⇒ q (1,3, 4 ) = 2
, q ( 2 ) = T 2 , q ( 5, 6 , 7 ) =
52a 52a 13a 2

University of Castilla-La Mancha Draft By: Eduardo W. V. Chaves (2016)


Ciudad Real - Spain
714 SOLVING PROBLEMS BY MEANS OF CONTINUUM MECHANICS

6.4.1.3 Combined Open and Closed Thin-Walled Section

Problem 6.53

Consider a hybrid cross section as the tf


one indicated in Figure 6.112 which is
made up by a hollow tube with radial
fins. Obtain the equations for the bf
tangential stress in the tube and in the tt
fins when the cross section is subjected
r
to a torsion moment M T .
MT

Figure 6.112: Hybrid thin wall cross section.


Solution:
Since we are dealing with a monolithic bar, the angle of twist per unit length is the same for
the tube and for the fins, i.e. θ = θ(t ) = θ( f ) . Moreover, we can apply the decomposition of
the cross section into the tube and the fins, (see Figure 6.113).

tt JT(t )
tf
r
bf M T(t )
tt

r tube
= +
tf JT( f )

4x M T( f )
bf

fin

Figure 6.113: Decomposition of the hybrid cross section.

Then, the following is true


M T = M T(t ) + 4M T( f ) ; JT = JT(t ) + 4 JT( f )
By means of the equation M T = GθJT we can also conclude that

University of Castilla-La Mancha Draft By: Eduardo W. V. Chaves (2016)


Ciudad Real – Spain
6 LINEAR ELASTICITY 715

MT M (t ) M ( f ) M T M T(t ) M T( f )
θ = θ(t ) = θ( f ) ⇒ = T(t ) = T( f ) ⇒ = (t ) = ( f )
GJT GJT GJT JT JT JT
M T JT(t ) M J (t ) M T JT( f ) M J( f )
⇒ M T(t ) = = (t ) T T ( f ) ; M T( f ) = = (t ) T T ( f )
JT ( JT + 4 JT ) JT ( JT + 4 JT )

For the tube we have JT(t ) = 2πr 3t , (see equation (6.306)), and for the fin we have
b f t 3f
JT( f ) = , (see equation (6.294)), then
3
4b f t 3f 2 MT 3M T
JT = JT(t ) + 4 JT( f ) 3
= 2πr tt + = (3πr 3tt + 2b f t 3f ) ; θ= =
3 3 GJT 2G (3πr 3tt + 2b f t 3f )

The tangential stress in the tube can be obtained by means of the equation in (6.305):
M T(t ) M T(t ) M T(t ) 3M T r
t)
τ (max = = = =
2 Att 2(πr )tt 2πr tt 2(3πr 3tt + 2b f t 3f )
2 2

And the tangential stress in the fin can be obtained by means of the equation in (6.296):
M T( f )t f 3M T( f )t f 3M T t f
τ (max
f)
= = =
JT( f ) b f t 3f 2(3πr 3tt + 2b f t 3f )

NOTE 1: Next we will automatize the above procedure in order to obtain the solution of
Hybrid Cross Section. Let us consider the example described in Figure 6.112 which was
discretize as indicated in Figure 6.114.

5
τ5 Perimeters
2πr
s1 = s 2 = s3 = s 4 = s =
1 1 4 4
τ1 τ4 s5 = s 6 = s 7 = s8 = b f
τ6 τ8 Thicknesses
2 4
tt = t1 = t 2 = t3 = t 4
6 8
t f = t 5 = t 6 = t 7 = t8
τ2 τ3
2 3 Area of the closed cell
3
A = πr 2
7 τ7

Figure 6.114: Hybrid cross section.


Here we will consider that the fin is an Open Cell in which is valid τ = tGθ , (see equation
(6.296)). For the closed cell we can state that
Closed Cell 1:

∫Γ τ dΓ = 2GθA ⇒ τ1s1 + τ 2 s2 + τ3 s3 + τ 4 s4 = 2GAθ ⇒ τ1s + τ2 s + τ3 s + τ 4 s = 2GAθ


C1

And for the fins (open cell):

University of Castilla-La Mancha Draft By: Eduardo W. V. Chaves (2016)


Ciudad Real - Spain
716 SOLVING PROBLEMS BY MEANS OF CONTINUUM MECHANICS

Open Cell 2 ( e = 5 ): τ5 = t5Gθ ⇒ τ5 − t f Gθ = 0

Open Cell 3 ( e = 6 ): τ6 = t6Gθ ⇒ τ6 − t f Gθ = 0

Open Cell 4 ( e = 7 ): τ7 = t7Gθ ⇒ τ7 − t f Gθ = 0

Open Cell 5 ( e = 8 ): τ8 = t8Gθ ⇒ τ8 − t f Gθ = 0

The moment equation is now given by:


 b f t 3f 
M T = M T(Closed ) + M T(Open) = 2Vmemb
( Closed )
+ 4 M T( f ) = 2 At1τ1 + 4( JT( f )Gθ) = 2 Att τ1 + 4 Gθ 
 3 
 
MT  b f t 3f 
⇒ = Att τ1 + 4 G θ
2  6 
 
We have 9 unknowns and 6 equations, the 3 missing equations can be obtained by
tangential stress flux compatibility at each node. Note that we have 4 nodes, but we only
need 3 of them, namely:
Node 1: t4 τ4 − t1τ1 = 0 ⇒ tt τ 4 − tt τ1 = 0
Node 2: t1τ1 − t2 τ2 = 0 ⇒ tt τ1 − tt τ 2 = 0
Node 3: t2 τ2 − t3τ3 = 0 ⇒ tt τ 2 − t t τ 3 = 0
Then, the set of equations to be solved is:
Cell 1  s s s s 0 0 0 0 − 2GA 
 0 τ   0 
0 0 0 1 0 0 0 − t f G   1  
0 
Cell 2  τ
Cell 3  0 0 0 0 0 1 0 0 − t f G  2  
  τ   0 
Cell 4  0 0 0 0 0 0 1 0 − t f G  3   
 0  τ 4  0 
Cell 5 0 0 0 0 0 0 1 −tfG    
  τ 5  =  0 
  b f t f     M T 
3
moment  Att 0 0 0 0 0 0 0 4 G  τ 6
 6     2 
   τ 7   0 
node 1  − tt 0 0 tt 0 0 0 0 0    
  τ 8   0 
node 2  tt − tt 0 0 0 0 0 0 0  θ   0 
node 3  0 tt − tt 0 0 0 0 0 0     

And the solution is:


3 AM T 3M T r
τ 1 =τ 2 =τ 3 =τ 4 = 2
=
2(3 A t t + 4 sb f t 3f ) 2(3πr 3 t t + 2b f t 3f )
3st f M T 3M T t f
τ 5 =τ 6 =τ 7 =τ 8 = =
(3 A 2 t t + 4 sb f t 3f ) 2(3πr 3 t t + 2b f t 3f )
3sM T 3M T
θ= 2
=
G (3 A t t + 4 sb f t 3f ) 2G (3πr 3 t t + 2b f t 3f )

and
MT 3 A 2 tt + 4 sb f t 3f 2
M T = GθJT ⇒ JT = = = (3πr 3tt + 2b f t 3f )
 3sM T  3s 3
G 2 3 
 G (3 A tt + 4 sb f t f ) 

University of Castilla-La Mancha Draft By: Eduardo W. V. Chaves (2016)


Ciudad Real – Spain
6 LINEAR ELASTICITY 717

6.4.2 Torsion and Bending of Thin-Walled Cross Section


6.4.2.1 Introduction

The shear flux problem for bending was established in Chapter 4 - “4.2 Some useful concepts
for the classical “mechanics of materials”, and we summarize as indicated in Figure 6.115.

x3′ x3 j ( X 2( j ) , X 3( j ) ) Transformation matrix


X3 r r
from gx to gx ′
i ( X 2( i ) , X 3( i ) ) x′2
a j  ml
A = 
α − m l 
g x2
s
t i
a = ( X 2( j ) − X 2(i ) ) 2 + ( X 3( j ) − X 3(i ) ) 2
r
X 2( j ) − X 2(i ) ( j)
− X 3(i )
X (g) l = cosα = , m = sinα = X 3
a a
X 2(i ) + X 2( j ) X (i ) + X 3( j )
O X2 A = at ; X 2( g ) = ; X 3( g ) = 3
2 2

at  a 2 m 2 + 12( X 3( g ) ) 2 − lma 2 − 12 X 2( g ) X 3( g )  Inertia Tensor of Area related


I OXr ij ≈   r
12  − lma 2 − 12 X 2( g ) X 3( g ) a 2 l 2 + 12( X 2( g ) ) 2  to the system OX , ( t << a )

F F
q ( s) = (qF(iy) + t d1 y s + t d 2 y s 2 ) Fy + (qF(iz) + t d1Fz s + t d 2Fz s 2 ) Fz (Shear Flux)

F
[
d1 y = −X −1 p0(1) + p2(1) X 2(i ) + p3(1) X 3(i ) ] ; [
d1Fz = −X −1 p0( 2 ) + p2( 2 ) X 2(i ) + p3( 2) X 3(i ) ]
F
d 2 y = −(2X ) −1 p2(1) [ l + p3(1) m] ; d 2Fz = −(2X ) −1 p2( 2) [ l + p3( 2) m]
AX 2 AX 3 AX 2 AX 3
p0(1) = − ; p0( 2 ) = −
− I23 I22 − I33 I 23
A AX 3 A AX 3
p2(1) = ; p2( 2 ) =
AX 3 I22 − AX 2 I23
A AX 2 AX 3
A AX 2 A AX 2
p3(1) =− ; p3( 2 ) =− , X = AX 3 − I 23 I 22
AX 3 − I 23 − AX 2 − I33
− AX 2 − I33 I 23
F
M O = M O y Fy + M OFz Fz = −( X 3( S .C .) ) Fy + ( X 2( S .C .) ) Fz (Torsion moment)
F F F
M O y = (6) −1 [m X 2(i ) − l X 3(i ) ]a(3atd 1 y + 2a 2 td 2 y + 6q F(iy) )
M OFz = (6) −1 [m X 2(i ) − l X 3(i ) ]a (3atd 1Fz + 2a 2 td 2Fz + 6q F(iz) )

 F a F a 
3
 (i ) Fz a a3 
f = q F(iy) a + t d 1 y + t d 2 y  F y + q Fz a + t d 1 + t d 2Fz  Fz (Force)
 2 3   2 3

Figure 6.115

University of Castilla-La Mancha Draft By: Eduardo W. V. Chaves (2016)


Ciudad Real - Spain
718 SOLVING PROBLEMS BY MEANS OF CONTINUUM MECHANICS

6.4.2.2 Open Cross Section

The problem characterized by bending of open thin-walled section was already discussed in
Chapter 4 - “4.2 Some useful concepts for the classical “mechanics of materials”. And, since we are
dealing with linear elasticity, we can use the superposition principle in order to obtain the
response with an additional effect due to torsion problem only, (see Figure 6.116).

S.C.-Shear center

x3′′ x3′′

+ F31 = F31
MT
F21 x′2′ MT F21 x′2′
F21 = F31 = 0 o′′ o′′
S.C. S.C.
M o′′x1′′ = 0 M o′′x1′′ = M T

a) Torsion only b) Bending only c) Torsion+Bending

Figure 6.116: Torsion and bending of open section.

The tangential stress distributions, for “torsion only” and for “bending only”, can be
appreciated in Figure 6.117. In Problem 4.31 we have obtained the shear flux for the
problem described in Figure 6.117(b).

τ (max
1)

τ(max
1)
q ( e =1)

τ(max
2)
q
τ=
τ(max
2)
q (e = 2)
t
t max = t1 ⇒ τ max = τ (max
1)

τ(max
3)

τ(max
3)
q ( e = 3)

a) Torsion only b) Bending only


Figure 6.117: Tangential stress distribution for torsion and bending of open section.

University of Castilla-La Mancha Draft By: Eduardo W. V. Chaves (2016)


Ciudad Real – Spain
6 LINEAR ELASTICITY 719

6.4.2.3 Closed Cross Section

As example, let us consider the closed cross section as the one indicated in Figure 6.118,
where the shearing forces are F21 = Fy = 0 and F31 = Fz = 1000 , and are applied at node 1.

X3 Node X 2( i ) X 3( i )
4 i =1 1 1
Node Coordinates i = 2 13 1
1 .0 i =3 14 5
3 i=4 1 10
4
3
Path
F31 = Fz Flange Thickness (t )
2 i→ j
1 e =1 1→ 2 0 .1
2 e=2 2→3 0 .1
1 F21
1 .0 e=3 3→ 4 0 .1
e=4 4 →1 0 .1
O X2

Figure 6.118: Closed thin-walled cross section.


The flange geometric characteristics are described in Table 6.1.
Table 6.1
Area
Flange ( X 2(i ) ; X 3(i ) ) ( X 2( j ) ; X 3( j ) ) t a
(e )
l m X 2( ge ) X 3( ge )
A
e =1 ( 1;1 ) ( 13;1 ) 0 .1 12 1 .2 1 0 7 1
e=2 ( 13;1 ) ( 13;5 ) 0 .1 4 0 .4 0 1 13 3
− 12 5
e=3 ( 13;5 ) ( 1;10 ) 0 .1 13 1 .3 7 7 .5
13 13
e=4 ( 1;10 ) ( 1;1 ) 0 .1 9 0 .9 0 −1 1 5.5

Area Centroid of the Compound and Total Area


The total area is A = A(1) + A( 2) + A(3) + A( 4) = 1.2 + 0.4 + 1.3 + 0.9 = 3.8
And the area centroid is given by
A(1) X 2( g1) + A( 2 ) X 2( g 2 ) + A(3) X 2( g 3) + A( 4 ) X 2( g 4)
X2 = = 6.2105
A(1) + A( 2) + A(3) + A( 4)
A(1) X 3( g1) + A( 2 ) X 3( g 2 ) + A(3) X 3( g 3) + A( 4 ) X 3( g 4)
X3 = = 4.5
A(1) + A( 2) + A(3) + A( 4)
The Inertia Tensor of Area (for the flange e )

at  a 2 m 2 + 12( X 3( g ) ) 2
− lma 2 − 12 X 2( g ) X 3( g ) 
IO( eX)r ij ≈   (6.321)
12  − lma 2 − 12 X 2( g ) X 3( g )
a 2 l 2 + 12( X 2( g ) ) 2 
r
The inertia tensors for the flanges related to the system OX are

University of Castilla-La Mancha Draft By: Eduardo W. V. Chaves (2016)


Ciudad Real - Spain
720 SOLVING PROBLEMS BY MEANS OF CONTINUUM MECHANICS

 1 .2 − 8 .4  ( 2 )  4.1333 − 15.6 (3) 75.8333 − 61.75


I (OX
1)r
≈  ; I OXr ij ≈   ; I OXr ij ≈  and
ij
 − 8.4 73.2   − 15.6 67.6   − 61.75 79.3 

 33.3 − 4.95
I (OX
4 r)
≈
ij
 − 4.95 0.9 
Then, the inertia tensor for the compound is given by
I I 23  − 90.7 
(I ) 114.46667
3
( Sys
r )
OX ij
=  22
I 23 I 33 
= ∑ (I
e =1
( e )r
OX ij
) = I O(1X)r ij + I O( 2Xr) ij + I O( 3X)r ij + I O( 4Xr) ij = 
 − 90.7 221 

Just to exercise let us calculate the inertia tensor at the Area Centroid, (see Chapter 4),
which can be obtained by means of the Steiner’s theorem:
r r r r r r r r
I O( Sys r ) − A [( X ⊗ X ) − ( X ⋅ X ) 1]
r ) = I ( Sys ⇒ IG( Sys r ) + A [( X ⊗ X ) − ( X ⋅ X ) 1]
r ) = I ( Sys
X Gx x OX

whose components are:


 IG 22 IG 23   X 32 − X2X3
( IG( Sys
r )
x ) ij = 
( Sys )
 = I OXr ij − A 
 IG 23 IG 33  − X 2 X 3 X 22 

( Sys 114.46667 − 90.7   ( 4.5) 2 − (6.21)(6.21)  37.5167 15.5 


r )) = −
( IGx  − 90.7  (3 . 8)  =
74.432 
ij
 221   − (6.21)(6.21) (6.21) 2   15.5
Shear Flux
By means of Figure 6.115 we can calculate the shear flux in the flanges. Taking into
account the geometric properties calculated previously we can calculate the coefficients
related to the cross section:
p0(1) = −1.15044 × 103 ; p0( 2 ) = −1.63858 × 103 ; p2(1) = 142.56333 ; p2( 2 ) = 58.9 ; p3(1) = 58.9 ;
p3( 2 ) = 282.84 , X = 9.69826 × 103

Flange 1: (Flange data ( X 2(i ) , X 3(i ) , t , l , m ) in Table 6.1)


qF(iy) = qF(1y) , qF(iz) = qF(1z)
F
[
d1 y = −X −1 p0(1) + p2(1) X 2(i ) + p3(1) X 3(i ) = 0.09785 ] ⇒
F
d1 y t = 0.009785
d1Fz = −X −1 [p ( 2)
0 + p2( 2) X 2(i ) + p3( 2 ) X 3(i ) ] = 0.13372 ⇒ d1Fz t = 0.013372
F
d 2 y = −(2X ) −1 p2(1) [ l + p3(1) m ] = −7.34994 × 10−3 ⇒
F
d 2 y t = −7.34994 × 10 −4
d 2Fz = −( 2X ) −1 [p 2 l
( 2)
+ p3( 2 ) m ] = −3.03663 × 10 −3 ⇒ d 2Fz t = −3.03663 × 10 −4
Shear Flux in the Flange 1
F F
q ( s ) = (q F(iy) + t d1 y s + t d 2 y s 2 ) Fy + (q F(iz) + t d1Fz s + t d 2Fz s 2 ) Fz = q F( ey=1) Fy + q F( ez =1) Fz

The terms q F( ey=1) and q F( ez =1) can be evaluated as follows:


F F
q F( ey=1) ( s) = q F(1y) + t d1 y s + t d 2 y s 2 = q F(1y) + (0.009785) s + (−7.34994 × 10 −4 ) s 2

q F( ez =1) ( s) = q F(1z) + t d1Fz s + t d 2Fz s 2 = q F(1z) + (0.013372) s + (−3.03663 × 10 −4 ) s 2

at the end s = 12 , (node 2 )


q F( ey=1) ( s = 12) = q F(1y) + (0.009785) s + (−7.34994 × 10 −4 ) s 2 = q F(1y) + 0.0115815 = q F( 2y )

University of Castilla-La Mancha Draft By: Eduardo W. V. Chaves (2016)


Ciudad Real – Spain
6 LINEAR ELASTICITY 721

qF( ez =1) ( s = 12) = qF(1z) + (0.013372) s + ( −3.03663 × 10−4 ) s 2 = qF(1z) + 0.1167351 = qF( 2z )

The total force in the flange 1 (see Problem 4.31-NOTE 1)

( e =1)  F a F a 
3
 (i ) Fz a Fz a 
3
f =  q F(iy) a + t d 1 y + t d 2 y F + q
 y  Fz a + t d 1 + t d 2  Fz
 2 3  2 3
( e =1)
⇒ f = (12q F(1y) + 0.281167) F y + (12q F(1z) + 0.787866) Fz

Torsion Moment at O due to Flange 1, (see Problem 4.31-NOTE 1):

(M ) Fy ( e =1)
O =
[m X 2(i ) − l X 3(i ) ]a
6
F F
(3atd1 y + 2a 2td 2 y + 6q F(iy) ) = −0.281167 − 12q F(1y)

[m X 2(i ) − l X 3(i ) ]a
(M ) Fz ( e=1)
O =
6
(3atd1Fz + 2a 2td 2Fz + 6q F(iz) ) = −0.7878655 − 12q F(1z)

Update Variables
q F(iy) ← q F( yj ) = q F(1y) + 0.0115815 ; q F(iz) ← q F( zj ) = q F(1z) + 0.1167351

Flange 2: (Flange data ( X 2(i ) , X 3(i ) , t , l , m ) in Table 6.1)


q F(iy) = q F(1y) + 0.0115815 , q F(iz) = q F(1z) + 0.1167351
F
[
d1 y = −X −1 p0(1) + p2(1) X 2(i ) + p3(1) X 3(i ) = −0.078548 ] ⇒
F
d1 y t = −0.0078548
d1Fz = −X −1 [p( 2)
0 + p2( 2) X 2(i ) + p3( 2 ) X 3(i )] = 0.0608398 ⇒ d1Fz t = 0.00608398
F
d 2 y = −(2X ) −1 p2(1) [ l + p3(1) m ] = −3.0366259 × 10 −3 ⇒
F
d 2 y t = −3.0366259 × 10 −4
d 2Fz = −( 2X ) −1 [p 2 l
( 2)
+ p3( 2 ) m ] = −0.014582 ⇒ d 2Fz t = −0.0014582
Shear Flux in the Flange 2
F F
q ( s ) = (q F(iy) + t d1 y s + t d 2 y s 2 ) Fy + (q F(iz) + t d1Fz s + t d 2Fz s 2 ) Fz = q F( ey=2 ) Fy + q F(ez =2) Fz

The terms q F( ey=2) and q F( ez =2) can be evaluated as follows:


F F
q F( ey=2 ) ( s) = q F(iy) + t d1 y s + t d 2 y s 2 = q F( 2y ) + (−0.0078548) s + (−3.0366259 × 10 −4 ) s 2
= ( q F(1y) + 0.0115815) + (−0.0078548) s + (−3.0366259 × 10 − 4 ) s 2

q F( ez =2 ) ( s) = q F(iz) + t d1Fz s + t d 2Fz s 2 = q F( 2z ) + (0.00608398) s + (−0.0014582) s 2


= (q F(1z) + 0.1167351) + (0.00608398) s + ( −0.0014582) s 2

at the end s = 4 , (node 3 )


qF( ey= 2 ) ( s = 4) = qF( 2y) − 0.03627783 = qF(3y)
= ( qF(1y) + 0.0115815) − 0.03627783 = qF(1y) − 0.0246964 = qF(3y)

q F( ez =2 ) ( s = 4) = q F( 2z ) + 0.00100472 = q F(3z )
= (q F(1z) + 0.1167351) + 0.00100472 = q F(1z) + 0.1177398 = q F(3z )

University of Castilla-La Mancha Draft By: Eduardo W. V. Chaves (2016)


Ciudad Real - Spain
722 SOLVING PROBLEMS BY MEANS OF CONTINUUM MECHANICS

The total force in the flange 2 (see Problem 4.31-NOTE 1)


 F a F a 
3
 a a3 
f ( e = 2 ) = qF( iy) a + t d1 y + t d 2 y  Fy + qF( iz) a + t d1Fz + t d 2Fz  Fz
 2 3  2 3
= ( 4qF( 2y) − 0.069317) Fy + ( 4qF( 2z ) + 0.017564) Fz

by considering qF( 2y) = qF(1y) + 0.0115815 , qF( 2z ) = qF(1z) + 0.1167351 we can obtain

⇒ f ( e=2) = ( 4qF(1y) − 0.02299078) Fy + (4q F(1z) + 0.484504) Fz

Torsion Moment at O due to Flange 2, (see Problem 4.31-NOTE 1):

(M ) Fy ( e = 2 )
O =
[m X 2(i ) − l X 3(i ) ]a
6
F F
(3atd1 y + 2a 2td 2 y + 6qF(iy) ) = −0.90111577 + 52qF( 2y)

= −0.2988802 + 52qF(1y)

[m X 2(i ) − l X 3(i ) ]a
(M ) Fz ( e = 2 )
O =
6
(3atd1Fz + 2a 2td 2Fz + 6qF(iz) ) = 0.228326 + 52qF( 2z )

= 6.2985524 + 52qF(1z)

Update Variables
qF(iy) ← qF( jy) = qF(1y) − 0.0246964 ; qF(iz) ← qF( zj ) = qF(1z) + 0.1177398

Flange 3: (Flange data ( X 2(i ) , X 3(i ) , t , l , m ) in Table 6.1)


qF(iy) = qF(3y) = qF(1y) − 0.0246964 , qF(iz) = qF(3z ) = qF(1z) + 0.1177398
F
[
d1 y = −X −1 p0(1) + p2(1) X 2(i ) + p3(1) X 3(i ) = −0.10284 ] ⇒
F
d1 y t = −0.010284
d1Fz = −X −1 [p ( 2)
0 + p2( 2) X 2(i ) + p3( 2 ) X 3(i )] = −0.05582 ⇒ d1Fz t = −0.005582
F
d 2 y = −(2X ) −1 p2(1) [ l + p3(1) m ] = 5.61663 × 10 −3 ⇒
F
d 2 y t = 5.61663 × 10 −4
d 2Fz = −( 2X ) −1 [p 2 l
( 2)
+ p3( 2 ) m ] = −2.80542 × 10 −3 ⇒ d 2Fz t = −2.80542 × 10 −4
Shear Flux in the Flange 3
F F
q ( s ) = (q F(iy) + t d1 y s + t d 2 y s 2 ) Fy + ( q F(iz) + t d1Fz s + t d 2Fz s 2 ) Fz = q F( ey=3) Fy + q F( ez =3) Fz

The terms q F( ey=3) and q F( ez=3) can be evaluated as follows:


F F
qF(ey= 3) ( s) = qF(iy) + t d1 y s + t d 2 y s 2 = qF(3y) + ( −0.010284) s + (5.61663 × 10−4 ) s 2
= (qF(1y) − 0.0246964) + (−0.010284) s + (5.61663 × 10− 4 ) s 2

qF(ez = 3) ( s) = qF(iz) + t d1Fz s + t d 2Fz s 2 = qF(3z) + (−0.005582) s + (−2.80542 × 10 −4 ) s 2


= (qF(1z) + 0.1177398) + (−0.005582) s + (−2.80542 × 10− 4 ) s 2

at the end s = 13 , (node 4 )


qF( ey= 3) ( s = 13) = ( qF(1y) − 0.0246964) − 0.0387724 = qF(3y) − 0.0387724 = qF( 4y)
= qF(1y) − 0.0634688 = qF( 4y)

University of Castilla-La Mancha Draft By: Eduardo W. V. Chaves (2016)


Ciudad Real – Spain
6 LINEAR ELASTICITY 723

qF( ez = 3) ( s = 13) = ( qF(1z) + 0.1177398) − 0.1199726 = qF(3z ) − 0.1199726 = qF( 4z )


= qF(1z) − 0.002232771 = qF( 4z )

The total force in the flange 3 (see Problem 4.31-NOTE 1)


 F a F a 
3
 a a3 
f (e =3) = qF(iy) a + t d1 y + t d 2 y  Fy +  qF(iz) a + t d1Fz + t d 2Fz  Fz
 2 3  2 3
= (13qF(3y) − 0.4576828) Fy + (13qF(3z ) − 0.6770968) Fz

by considering qF(iy) = qF(1y) − 0.0246964 , qF(iz) = qF(1z) + 0.1177398 we can obtain

⇒ f ( e=3) = (13q F(1y) − 0.7787357) Fy + (13q F(1z) + 0.853521) Fz

Torsion Moment at O due to Flange 3, (see Problem 4.31-NOTE 1):

(M ) F y ( e = 3)
O =
[m X 2(i ) − l X 3(i ) ]a
6
F F
(3atd1 y + 2a 2td 2 y + 6qF(iy) ) = −4.4007962 + 125qF(3y)

= −7.48784 + 125qF(1y)

[m X 2(i ) − l X 3(i ) ]a
(M ) Fz ( e = 3 )
O =
6
(3atd1Fz + 2a 2td 2Fz + 6qF(iz) ) = −6.510547 + 125qF(3z )

= 8.206993 + 125qF(1z)

Update Variables
qF(iy) ← qF( yj ) = qF(1y) − 0.0634688 ; qF(iz) ← qF( zj ) = qF(1z) − 0.002232771

Flange 4: (Flange data ( X 2(i ) , X 3(i ) , t , l , m ) in Table 6.1)


qF(iy) = qF( 4y) = qF(1y) − 0.0634688 , qF(iz) = qF( 4z ) = qF(1z) − 0.002232771
F
[
d1 y = −X −1 p0(1) + p2(1) X 2(i ) + p3(1) X 3(i ) = 0.04319 ] ⇒
F
d1 y t = 0.004319
d1Fz = −X −1 [p ( 2)
0 + p2( 2) X 2(i ) + p3( 2 ) X 3(i )] = −0.12876 ⇒ d1Fz t = −0.012876
F
d 2 y = −(2X ) −1 p2(1) [ l + p3(1) m ] = 3.03663 × 10 −3 ⇒
F
d 2 y t = 3.03663 × 10 −4
d 2Fz = −(2X ) −1 [p 2 l
(2)
+ p3( 2 ) m ] = 0.01458 ⇒ d 2Fz t = 0.001458
Shear Flux in the Flange 4
F F
q ( s ) = (qF(iy) + t d1 y s + t d 2 y s 2 ) Fy + (qF(iz) + t d1Fz s + t d 2Fz s 2 ) Fz = qF( ey= 4 ) Fy + qF( ez = 4) Fz

The terms qF( ey= 4) and qF( ez = 4) can be evaluated as follows:


F F
qF(ey= 4 ) ( s ) = qF(iy) + t d1 y s + t d 2 y s 2 = qF( 4y) + (0.004319) s + (3.03663 × 10−4 ) s 2
= (qF(1y) − 0.0634688) + (0.004319) s + (3.03663 × 10− 4 ) s 2

qF(ez = 4 ) ( s ) = qF(iz) + t d1Fz s + t d 2Fz s 2 = qF( 4z ) + (−0.012876) s + (0.001458) s 2


= (qF(1z) − 0.002232771) + (−0.012876) s + (0.001458) s 2

at the end s = 9 , (node 1 )


qF( ey= 4 ) ( s = 9) = (qF(1y) − 0.0634688) + 0.0634688 = qF(1y)

University of Castilla-La Mancha Draft By: Eduardo W. V. Chaves (2016)


Ciudad Real - Spain
724 SOLVING PROBLEMS BY MEANS OF CONTINUUM MECHANICS

q F( ez =4 ) ( s = 9) = (q F(1z) − 0.002232771) + 0.002232771 = q F(1z)

The total force in the flange 4 (see Problem 4.31-NOTE 1)


 F a F a 
3
 a a3 
f (e = 4 ) =  qF(iy) a + t d1 y + t d 2 y  Fy +  qF(iz) a + t d1Fz + t d 2Fz  Fz
 2 3  2 3
= (9qF( 4y) + 0.248715) Fy + (9qF( 4z ) − 0.167124) Fz

by considering qF(iy) = qF( 4y) = qF(1y) − 0.0634688 , qF(iz) = qF( 4z ) = qF(1z) − 0.002232771 we get

⇒ f ( e=4 ) = (9q F(1y) − 0.3225045) Fy + (9q F(1z) − 0.1872187) Fz

Torsion Moment at O due to Flange 4, (see Problem 4.31-NOTE 1):

(M ) Fy ( e = 4 )
O =
[m X 2(i ) − l X 3(i ) ]a
6
F F
(3atd1 y + 2a 2td 2 y + 6qF(iy) ) = −0.24871451 − 9qF( 4y) = 0.3225 − 9qF(1y)

[m X 2(i ) − l X 3(i ) ]a
(M ) Fz ( e = 4 )
O =
6
(3atd1Fz + 2a 2td 2Fz + 6qF(iz) ) = 0.167124 − 9qF( 4z ) = 0.18722 − 9qF(1z)

The Total Torsion Moment at O


(M ) Fy ( Sys )
O ( )
= M Oy
F ( e=1)
( )
+ M Oy
F ( e=2)
( )
+ M Oy
F ( e =3 )
( )
+ M Oy
F ( e=4)
= −7.7453862 + 156q F(1y)

(M ) Fz ( Sys )
O (
= M OFz )
( e=1)
(
+ M OFz )
( e= 2 )
(
+ M OFz )
( e =3 )
(
+ M OFz )
( e=4)
= 13.9048382 + 156q F(1z)
F
M O = M O y Fy + M OFz Fz = −( X 3( S .C .) ) Fy + ( X 2( S .C .) ) Fz (6.322)
⇒ M O = ( −7.7453862 + 156q F(1y) ) Fy + (13.9048382 + 156q F(1z) ) Fz = −( X 3( S .C .) ) Fy + ( X 2( S .C .) ) Fz
(6.323)
The Total Force due to shear flux
( FT ) ( Sys ) = f ( e=3) + f ( e=3) + f ( e=3) + f ( e=3)
= ( −0.843064 + 38q F(1y) ) Fy + (1.9386719 + 38q F(1z) ) Fz

The Shear Center


Note that when the total force is at the shear center the torsion moment is zero, so
(−0.843064 + 38q F(1y) ) Fy × (− X 3( S .C .) ) = 0 ⇒ q F(1y) = 0.02218589
(1.9386719 + 38q F(1z) ) Fz × ( X 2( S .C .) ) = 0 ⇒ q F(1z) = −0.05101768

then, the total moment becomes


M O = (−7.7453862 + 156q F(1y) ) Fy + (13.9048382 + 156q F(1z) ) Fz = −( X 3( S .C .) ) Fy + ( X 2( S .C .) ) Fz
⇒ M O = (−7.7453862 + 156 × (0.02218589)) Fy + (13.9048382 + 156 × (−0.05101768)) Fz
⇒ M O = −(4.2843866) Fy + (5.9460799) Fz = −( X 3( S .C .) ) Fy + ( X 2( S .C .) ) Fz

with that we can conclude that the shear center is located at:
X 2( S .C .) = 5.9460799 ; X 3( S .C .) = 4.2843866

University of Castilla-La Mancha Draft By: Eduardo W. V. Chaves (2016)


Ciudad Real – Spain
6 LINEAR ELASTICITY 725

Shear Flux due to Bending Only ( q F(1y) = 0.02218589; q F(1z) = −0.05101768 )

Flange 1
q F(ey=1) = q F(1y) + 0.009785s − 7.34994 × 10 −4 s 2 = 0.02218589 + 0.009785s − 7.34994 × 10 −4 s 2
q F(ez =1) = q F(1z) + 0.013372 s − 3.03663 × 10 − 4 s 2 = −0.05101768 + 0.013372 s − 3.03663 × 10 − 4 s 2

q (e =1) ( s ) = q F(ey=1) Fy + q F( ez =1) Fz = −51.01768 + 13.372 s − 0.303663s 2


{
=0

at the end s = 12 , (node 2 )


q F(ey=1) = q F(1y) + 0.0115815 = 0.02218589 + 0.0115815 = 0.0337673 = q F( 2y )

q F(ez =1) = q F(1z) + 0.1167351 = −0.05101768 + 0.1167351 = 0.0657174 = q F( 2z )

The total force at the flange 1


f ( e=1) = (12q F(1y) + 0.281167) Fy + (12q F(1z) + 0.787866) Fz = (0.5473978) Fy + (0.1756534) Fz

Flange 2
q F( ey= 2) ( s ) = q F(1y) + 0.0115815 − 0.0078548s − 3.0366259 × 10 −4 s 2
= 0.0337673 − 0.0078548s − 3.0366259 × 10 − 4 s 2
q F(ez =2 ) ( s ) = q F(1z) + 0.1167351 + 0.00608398s − 0.0014582 s 2
= 0.0657174 + 0.00608398s − 0.0014582 s 2
q (e =2 ) ( s ) = q F( ey= 2) Fy + q F( ez = 2 ) Fz = 65.7174 + 6.08398s − 1.4582s 2
{
=0

at the end s = 4 , (node 3 )


q F(ey=2 ) ( s = 4) = q F(1y) − 0.0246964 = 0.02218589 − 0.0246964 = −2.5104 × 10 −3 = q F(3y)

q F(ez =2 ) ( s = 4) = q F(1z) + 0.1177398 = (−0.05101768) + 0.1177398 = 0.0667222 = q F(3z )

The total force at the flange 2


f ( e= 2 ) = (4q F(1y) − 0.02299078) Fy + (4q F(1z) + 0.484504) Fz = (0.0657528) Fy + (0.2804333) Fz

Flange 3
q F(ey=3) ( s) = q F(1y) − 0.0246964 − 0.010284 s + 5.61663 × 10 −4 s 2
= −2.51048 × 10 −3 − 0.010284 s + 5.61663 × 10 −4 s 2
q F(ez =3) ( s) = q F(1z) + 0.1177398 − 0.005582 s − 2.80542 × 10 −4 s 2
= 0.0667222 − 0.005582 s − 2.80542 × 10 −4 s 2
at the end s = 13 , (node 4 )
q F(ey=3) ( s = 13) = q F(1y) − 0.0634688 = −0.0412829 = q F( 4y )

q F(ez =3) ( s = 13) = q F(1z) − 0.002232771 = −0.0532505 = q F( 4z )

University of Castilla-La Mancha Draft By: Eduardo W. V. Chaves (2016)


Ciudad Real - Spain
726 SOLVING PROBLEMS BY MEANS OF CONTINUUM MECHANICS

The total force in the flange 3 (see Problem 4.31-NOTE 1)


f ( e=3) = (13q F(1y) − 0.7787357) Fy + (13q F(1z) + 0.853521) Fz = −0.4903191Fy + 0.1902911Fz

Flange 4
q F(ey=4 ) ( s ) = q F(1y) − 0.0634688 + 0.004319 s + 3.03663 × 10 −4 s 2
= −0.0412829 + 0.004319 s + 3.03663 × 10 − 4 s 2
q F( ez =4 ) ( s) = q F(1z) − 0.002232771 − 0.012876 s + 0.001458s 2
= −0.0532505 − 0.012876 s + 0.001458s 2
at the end s = 9 , (node 1 )
q F(ey=4 ) ( s = 9) = (q F(1y) − 0.0634688) + 0.0634688 = 0.0221859 = q F(1y)

q F(ez =4 ) ( s = 9) = (q F(1z) − 0.002232771) + 0.002232771 = −0.0510177 = q F(1z)

The total force in the flange 4 (see Problem 4.31-NOTE 1)


f ( e= 4 ) = (9q F(1y) − 0.3225045) Fy + (9q F(1z) − 0.1872187) Fz = −0.1228315 Fy − 0.6463778 Fz

S.C.- Shear Center Shear Flux


 X 2( S .C .) = 5.9460799
 ( S .C .) q ( e =1) = −51.01768 + 13.372s − 3.03663 × 10 −1 s 2
 X 3 = 4.2843866
q ( e = 2 ) = 65.7174 + 6.08398s − 1.4582s 2
q ( e =3) = 66.7222 − 5.582 s − 2.80542 × 10 −1 s 2
X3
q ( e=4 ) = −53.2505 − 12.876 s + 1.458s 2
4 Fz = 1000
s (e= 4)
q ( e = 3)
Nodal Shear Flux
3 s ( e = 3) q (1) = −51.01768
4 ⊗
q (e = 4) S.C. 3 q ( 2) = 65.7174
2 q (e =2) q (3) = 66.7222
1
s (e= 2) q ( 4) = −53.2505
1 q ( e =1) 2
s ( e =1)

O X2

Figure 6.119: Shear flux due to Fz - Bending only.

University of Castilla-La Mancha Draft By: Eduardo W. V. Chaves (2016)


Ciudad Real – Spain
6 LINEAR ELASTICITY 727

Torsion only
The solution for this problem was already done, (see equation (6.318)):
 1 
 2 At1 
 
 1 
τ 1 
τ   2 At 2   MT
 2  
 1 
 τ 1 = τ 2 = τ 3 = τ 4 = τ = 2 At
τ
 3 = M T  2 At 3  ⇒ 
τ    θ = s T
 4  1   4 A 2 Gt
 θ   2 At 4 
 
 1  s1 s s s
+ 2 + 3 + 4


 4 A 2 G  t1 t2 t3 t4 

 12 × 5 
where M T = −4946.08 , ACell = 12 × 9 −  = 78 , t = 0.1
 2 
MT MT
τ= = −317.05641 ⇒ τ t=q= = −31.705641
2 ACell t 2 ACell

X3
Fz = 1000
Torsion at the shear center
1 .0 4.94608
M T = M o′′ = −1000 × 4.94608 = −4946.08

4
Fz = 1000

q ( e = 3)
MT
3
4 ⊗

S.C. 3
q (e = 4)
2 q (e =2)
1
1 q ( e =1) 2

O X2

Figure 6.120

NOTE 1: Note that the total moment at the point O obtained in (6.323), M OFz Fz , must be
equal to the moment produced by the force Fz = 1000 at the same point, so
M OFz Fz = Fz ×1 ⇒ (13.9048382 + 156q F(1z) ) Fz = Fz × 1
(6.324)
⇒ 13.9048382 + 156q F(1z) − 1 = 0 ⇒ q F(1z) = −0.08272332

University of Castilla-La Mancha Draft By: Eduardo W. V. Chaves (2016)


Ciudad Real - Spain
728 SOLVING PROBLEMS BY MEANS OF CONTINUUM MECHANICS

Note that we have found for bending only problem that q F(1z) _ bending = −0.05101768
q − 31.705641
(bending), and for torsion only q F(1z) _ torsion = = = −0.031705641 (torsion),
Fz 1000
then
q F(1z) = q F(1z) _ bending + q F(1z) _ torsion = −0.05101768 − 0.031705641 = −0.08272332

which matches the result in (6.324).


NOTE 2:
Just by considering the effect due to Fz we have obtained:
Flange 1
(
qF(1z) + 0.1167351 = qF( 2z ) , f ( e =1) = (12qF(1z) + 0.787866) Fz , M OFz )
( e =1)
= −0.7878655 − 12qF(1z)

Flange 2
(
qF( 2z ) + 0.00100472 = qF(3z ) , f ( e=2 ) = ( 4qF( 2z ) + 0.017564) Fz , M OFz )
(e = 2)
= 0.228326 + 52qF( 2z )

Flange 3
qF(3z) − 0.1199726 = qF( 4z ) , f (e=3) = (13q F(3z ) − 0.6770968) Fz , M OFz ( ) ( e = 3)
= −6.510547 + 125qF(3z )

Flange 4
qF(1z) = qF(1z) (redundant equation), f (e=4 ) = (9q F( 4z ) − 0.167124) Fz , M OFz ( )
(e = 4)
= 0.167124 − 9qF( 4z )

The total moment can also be represented as follows


M OFz Fz = [(−0.7878655 − 12qF(1z) ) + (0.228326 + 52qF( 2z ) ) + (−6.510547 + 125qF(3z) )
+ (0.167124 − 9qF( 4z ) )]Fz
= [−12qF(1z) + 52qF( 2z ) + 125qF(3z) − 9qF( 4z ) − 6.90296222]Fz

which must be equal to 1 × Fz , (see Figure 6.118).


[−12q F(1z) + 52q F( 2z ) + 125q F(3z ) − 9q F( 4z ) − 6.90296222]Fz = 1 × Fz

Then, we can construct the following system:


0   qFz   − 0.1167351   qF(1)  − 0.082723322
(1)
 1 −1 0
 0  ( 2)    ( 2z)  
 1 − 1 0   Fz   − 0.00100472  Solve qFz   0.034011797 
 q 
 =  → (3)  =  
 0 0 1 − 1 qF(3z )   0.1199726  qFz   0.035016513 
  ( 4)
− 12 52 125 − 9 qFz  6.90296222 + 1 qF( 4 )  − 0.084956093
 z 
Bending Only
If the force Fz is applied at the shear center the total moment is zero, so
M OFz Fz − X 2( S .C .) Fz = 0
⇒ −12q F(1z) + 52q F( 2z ) + 125q F(3z ) − 9q F( 4z ) − X 2( S .C .) = 6.90296222

The total force can be represented as follows

University of Castilla-La Mancha Draft By: Eduardo W. V. Chaves (2016)


Ciudad Real – Spain
6 LINEAR ELASTICITY 729

FTFz = [(12q F(1z) + 0.787866) + (4q F( 2z ) + 0.017564) + (13q F(3z ) − 0.6770968) + (9q F( 4z ) − 0.167124)]Fz
= [12q F(1z) + 4q F( 2z ) + 13q F(3z ) + 9q F( 4z ) − 0.03879148]Fz

And, it is also true that

FTFz × X 2( S .C .) = 0 ⇒ 12q F(1z) + 4q F( 2z ) + 13q F(3z ) + 9q F( 4z ) = 0.03879148

Note that condition is the same as the equation in (6.260) in which:


q
∫Γ τ ∫Γ t dΓ = 2GθA ∫Γ q dΓ = 2GθAt
(Γ)
dΓ = 2GθA ⇒ t
- constant
→

And if the forces are applied at the shear center the angle of twist per unit length (or the
torsion) is equal to zero, so
MT s
∫ q dΓ = 2GθAt = { ∫ q dΓ = 0
Center θ = 0
Shear
  →
Γ
2A Γ
=0
(see equation (6.304))

And the system to be solved is:


 1 −1 0 0 0   qF(1z)   − 0.1167351   qF(1)  − 0.051017682
 0    − 0.00100472  ( 2z )   
 1 − 1 0 0 q (2)
  Fz     q Fz   0.065717437 
   
 0 0 1 − 1 0   q F(3)  =  0.1199726  Solve → q F(3)  =  0.066722153 
   (4)  
z
  ( 4 )  − 0.053250453
z

− 12 52 125 − 9 − 1  q Fz   6.90296222   q Fz   
 12 4 13 9 0   X 2S C   0.03879148 
( . .)  X 2( S .C .)   5.946079856 

University of Castilla-La Mancha Draft By: Eduardo W. V. Chaves (2016)


Ciudad Real - Spain
730 SOLVING PROBLEMS BY MEANS OF CONTINUUM MECHANICS

Problem 6.54
Consider the cross section described in Figure 6.111 in which a = 2 and t = 0.1 . Obtain the
shear flux in each segment and the shear center of the cross section.
Solution:
In order to automatize the procedure we will create additional nodes at the point in which
bifurcation occurs, (see Figure 6.121).

X3

4 3 9 10 7
6
τ3 3 τ7

4 Cell 1
τ4 τ2 Cell 2 6
2 τ6

τ1 7 τ5
1 1 2 8 5 5 X2

Element Connectivity
Coordinates i→ j Thickness
Nodes e
X2 X3 1 1→ 2 t1 = t
1 0 0 2 7→3 t2 = t
2,7,8 2a 0
3 9→4 t3 = t
3,9,10 2a a
4 4 4 →1 t4 = t
0 a
5 3a 0 5 8→5 t5 = t
6 3a a 6 5→6 t6 = t
7 6 → 10 t7 = t

Figure 6.121
The flange geometric characteristics are described in Table 6.2.
Table 6.2
Area
Flange ( X 2(i ) ; X 3(i ) ) ( X 2( j ) ; X 3( j ) ) t a
(e )
l m X 2( ge ) X 3( ge )
A
e =1 ( 0;0 ) ( 4;0 ) 0 .1 4 0 .4 1 0 2 0
e=2 ( 4;0 ) ( 4;2 ) 0 .1 2 0 .2 1 0 4 1
e=3 ( 4;2 ) ( 2;2 ) 0 .1 4 0 .4 −1 0 2 2
e=4 ( 0;2 ) ( 0;0 ) 0 .1 2 0 .2 0 −1 0 1
e=5 ( 4;0 ) ( 6;0 ) 0 .1 2 0 .2 1 0 5 0
e=6 ( 6;0 ) ( 6;2 ) 0 .1 2 0 .2 0 1 6 1
e=7 ( 6;2 ) ( 4;2 ) 0 .1 2 0 .2 −1 0 5 2

University of Castilla-La Mancha Draft By: Eduardo W. V. Chaves (2016)


Ciudad Real – Spain
6 LINEAR ELASTICITY 731

Total Area and Area Centroid of the Compound


The total area is A = A(1) + A( 2) + A(3) + A( 4) + A(5) + A( 6) + A( 7 ) = 1.8
And the area centroid is given by
7 7


e =1
A(e ) X 2( ge ) ∑A
e=1
(e)
X 3( ge )
X2 = = 3.1111111 ; X3 = = 1 .0
A A
The Inertia Tensor of Area (for the flange e )

at  a 2 m 2 + 12( X 3( g ) ) 2 − lma 2 − 12 X 2( g ) X 3( g ) 
IO( eX)r ij ≈   (6.325)
12  − lma 2 − 12 X 2( g ) X 3( g ) a 2 l 2 + 12( X 2( g ) ) 2 
r
The inertia tensors for the flanges related to the system OX are
0 0  ( 2) 0.2666667 − 0. 8 ( 3 )  1 .6 − 1 .6 
I(OX
1)r
≈  ; IOXr ij ≈   ; IOXr ij ≈  ;
0 2.1333333   − 0 .8 3 .2   − 1.6 2.133333
ij

( 4r) 0.2666667 0 (5) 0 0  (6) 0.2666667 − 1 .2 


IOX ≈  ; IOXr ij ≈   ; IOXr ij ≈  ;
ij
 0 0 0 5.0666667   − 1 .2 7.2 

 0 .8 −2 
I(OX
7 r)
≈ 
 − 2 5.0666667 
ij

Then, the inertia tensor for the compound is given by


 3.2 − 5.6 I 22 I 23 
(I ) = ∑ (I
7
( Sys
r ) ( e )r
) = =
OX ij
e =1
OX ij
 − 5.6 24.8  I 23 I33 

Just as exercise, let us calculate the inertia tensor at the Area Centroid, (see Chapter 4),
which can be obtained by means of the Steiner’s theorem:
r r r r r r r r
I O( Sys r ) − A [( X ⊗ X ) − ( X ⋅ X ) 1]
r ) = I ( Sys ⇒ IG( Sys r ) + A [( X ⊗ X ) − ( X ⋅ X ) 1]
r ) = I ( Sys
X G x x OX

whose components are:


 X 32 − X2X3  IG 22 IG 23  1.4 0 
( IG( Sys
r ) ) = I ( Sys
x ij
r ) − A
  ⇒ ( IG( Sys
r )
x ) ij =  = 
X 22   IG 23 IG 33  
OX ij
− X 2 X 3 0 7 . 3777778 
Shear Flux
By means of Figure 6.115 we can calculate the shear flux in the flanges. Taking into
account the geometric properties calculated previously we can calculate the coefficients
related to the cross section:
p0(1) = −7.84 ; p0( 2) = −13.28 ; p2(1) = 2.52 ; p2( 2) = 2.487 × 10 −15 ; p3(1) = 2.487 ×10 −15 ;
p3( 2) = 13.28 , X = 18.592

Flange 1: (Flange data ( X 2(i ) , X 3(i ) , t , l , m ) in Table 6.2)


qF(iy) = qF(1y) , qF(iz) = qF(1z)
F
[
d1 y = −X −1 p0(1) + p2(1) X 2(i ) + p3(1) X 3(i ) = 0.4216867 ] ⇒
F
d1 y t = 0.04216867
d1Fz = −X −1 [p ( 2)
0 + p2( 2 ) X 2(i ) + p3( 2 ) X 3(i ) ] = 0.7142857 ⇒ d1Fz t = 0.07142857

University of Castilla-La Mancha Draft By: Eduardo W. V. Chaves (2016)


Ciudad Real - Spain
732 SOLVING PROBLEMS BY MEANS OF CONTINUUM MECHANICS

F
d 2 y = −(2X ) −1 p2(1) [ l + p3(1) m ] = −0.0677711 ⇒ d 2F t = −0.00677711 y

d 2Fz = −(2X ) −1 [p 2 l
(2)
+ p3( 2 ) m ] = −0 ⇒ d 2F t = 0 z

Shear Flux in the Flange 1


F F
q ( s ) = (q F(iy) + t d1 y s + t d 2 y s 2 ) Fy + (q F(iz) + t d1Fz s + t d 2Fz s 2 ) Fz = q F( ey=1) Fy + q F( ez =1) Fz

The terms q F( ey=1) and q F( ez =1) can be evaluated as follows:


F F
q F(ey=1) ( s ) = qF(1y) + t d1 y s + t d 2 y s 2 = qF(1y) + 0.04216867 s − 0.00677711s 2

q F(ez =1) ( s ) = q F(1z) + t d1Fz s + t d 2Fz s 2 = qF(1z) + 0.07142857 s

at the end s = 4 , (node 2 )


q F(ey=1) ( s = 4) = qF(1y) + 0.04216867 s − 0.00677711s 2 = q F(1y) + 0.060241 = q F( 2y )

q F(ez =1) ( s = 4) = q F(1z) + 0.07142857 s = q F(1z) + 0.2857143 = q F( 2z )

The total force in the flange 1, (see Problem 4.31-NOTE 1)


 F a F a 
3
 a a3 
f ( e=1) = q F(iy) a + t d1 y + t d 2 y  Fy + q F(iz) a + t d1Fz + t d 2Fz  Fz
 2 3  2 3
⇒ f ( e=1) = (4q F(1y) + 0.1927711) Fy + (4q F(1z) + 0.5714286) Fz

Torsion Moment at O due to Flange 1, (see Problem 4.31-NOTE 1):

(M ) Fy ( e=1)
O =
[m X 2(i ) − l X 3(i ) ]a
6
F F
(3atd1 y + 2a 2td 2 y + 6q F(iy) ) = 0

(M ) Fz ( e =1)
O =
[m X 2(i ) − l X 3(i ) ]a
6
(3atd1Fz + 2a 2td 2Fz + 6q F(iz) ) = 0

Flange 2: (Flange data ( X 2(i ) , X 3(i ) , t , l , m ) in Table 6.2)


q F(iy) = q F( 7y ) , q F(iz) = q F( 7z )
F
[
d1 y = −X −1 p0(1) + p2(1) X 2(i ) + p3(1) X 3(i ) = −0.1204819 ] ⇒
F
d1 y t = −0.01204819
d1Fz = −X −1 [p (2)
0 + p2( 2 ) X 2(i ) + p3( 2 ) X 3(i )] = 0.7142857 ⇒ d1Fz t = 0.07142857
F
[
d 2 y = −(2X ) −1 p2(1) l + p3(1) m = −0 ] ⇒ d
Fy
2 t =0
d 2Fz = −( 2X ) −1 [p (2)
2 l + p3( 2 ) m ] = −0.3571429 ⇒ d 2Fz t = −0.03571429
Shear Flux in the Flange 2
F F
q ( s ) = (q F(iy) + t d1 y s + t d 2 y s 2 ) Fy + (q F(iz) + t d1Fz s + t d 2Fz s 2 ) Fz = q F( ey=2 ) Fy + q F(ez =2) Fz

The terms q F( ey=2) and q F( ez =2) can be evaluated as follows:


F F
q F(ey=2 ) ( s ) = q F(iy) + t d1 y s + t d 2 y s 2 = q F(7y ) − 0.01204819 s

q F(ez =2 ) ( s ) = qF(iz) + t d1Fz s + t d 2Fz s 2 = q F( 7z ) + 0.07142857 s − 0.03571429 s 2

at the end s = 2 , (node 3 )

University of Castilla-La Mancha Draft By: Eduardo W. V. Chaves (2016)


Ciudad Real – Spain
6 LINEAR ELASTICITY 733

q F(ey=2 ) ( s = 2) = qF( 7y ) − 0.0240964 = qF(3y)

q F(ez =2 ) ( s = 2) = qF(7z ) + 0 = q F(3z )

The total force in the flange 2, (see Problem 4.31-NOTE 1)


 F a F a 
3
 a a3 
f ( e=2) = qF(iy) a + t d1 y + t d 2 y  Fy + qF( iz) a + t d1Fz + t d 2Fz  Fz
 2 3  2 3
= ( 2q F( 7y ) − 0.02409639) Fy + (2qF( 7z ) + 0.047619) Fz

Torsion Moment at O due to Flange 2, (see Problem 4.31-NOTE 1):

(M ) Fy ( e= 2 )
O =
[m X 2(i ) − l X 3(i ) ]a
6
F F
(3atd1 y + 2a 2td 2 y + 6qF(iy) ) = 8q F(7y ) − 0.0963855

(M ) Fz ( e =2 )
O =
[m X 2(i ) − l X 3(i ) ]a
6
(3atd1Fz + 2a 2td 2Fz + 6qF(iz) ) = 8qF( 7z ) + 0.1904762

Flange 3: (Flange data ( X 2(i ) , X 3(i ) , t , l , m ) in Table 6.1)


q F(iy) = q F(9y ) , q F(iz) = q F(9z )
F
[
d1 y = −X −1 p0(1) + p2(1) X 2(i ) + p3(1) X 3(i ) = −0.10284 ] ⇒
F
d1 y t = −0.010284
d1Fz = −X −1 [p ( 2)
0 + p2( 2 ) X 2(i ) + p3( 2 ) X 3(i )] = −0.71429 ⇒ d1Fz t = −0.071429
F
d 2 y = −(2X ) −1 p2(1) [ l + p3(1) m ] = 0.06777 ⇒
F
d 2 y t = 0.006777
d 2Fz = −(2X ) −1 [p 2 l
( 2)
+ p3( 2) m ] = 0 ⇒ d 2Fz t = 0
Shear Flux in the Flange 3
F F
q ( s ) = (q F(iy) + t d1 y s + t d 2 y s 2 ) Fy + ( q F(iz) + t d1Fz s + t d 2Fz s 2 ) Fz = q F( ey=3) Fy + q F( ez =3) Fz

The terms q F( ey=3) and q F( ez=3) can be evaluated as follows:


F F
q F( ey=3) ( s ) = q F(iy) + t d1 y s + t d 2 y s 2 = q F(9y) − 0.010284 s + 0.006777 s 2

q F(ez =3) ( s ) = qF(iz) + t d1Fz s + t d 2Fz s 2 = q F(9z ) − 0.071429 s

at the end s = 4 , (node 4 )


q F(ey=3) ( s = 4) = q F(9y ) − 0.010284 s + 0.006777 s 2 = q F(9y ) + 0.060241 = q F( 4y )

q F(ez =3) ( s = 4) = qF(9z ) − 0.071429 s = qF(9z ) − 0.2857143 = qF( 4z )

The total force in the flange 3, (see Problem 4.31-NOTE 1)


 F a F a 
3
 a a3 
f ( e=3) =  qF(iy) a + t d1 y + t d 2 y  Fy + q F(iz) a + t d1Fz + t d 2Fz  Fz
 2 3  2 3
= (4qF(9y ) + 0.0481928) Fy + (4q F(9z ) − 0.5714286) Fz

Torsion Moment at O due to Flange 3, (see Problem 4.31-NOTE 1):

(M ) Fy ( e=3)
O =
[m X 2(i ) − l X 3(i ) ]a
6
F F
(3atd1 y + 2a 2td 2 y + 6qF(iy) ) = 8qF(9y) + 0.09639

University of Castilla-La Mancha Draft By: Eduardo W. V. Chaves (2016)


Ciudad Real - Spain
734 SOLVING PROBLEMS BY MEANS OF CONTINUUM MECHANICS

(M ) Fz ( e=3)
O =
[m X 2(i ) − l X 3(i ) ]a
6
(3atd1Fz + 2a 2td 2Fz + 6q F(iz) ) = 8q F(9z ) − 1.14286

Flange 4: (Flange data ( X 2(i ) , X 3(i ) , t , l , m ) in Table 6.2)


q F(iy) = q F( 4y ) , q F(iz) = q F( 4z )
F
[
d1 y = −X −1 p0(1) + p2(1) X 2(i ) + p3(1) X 3(i ) = 0.42169 ] ⇒
F
d1 y t = 0.042169
d1Fz = −X −1 [p ( 2)
0 + p2( 2 ) X 2(i ) + p3( 2 ) X 3(i )] = −0.71429 ⇒ d1Fz t = −0.071429
F
d 2 y = −(2X ) −1 p2(1) [ l + p3(1) m]= 0 ⇒ F
d2 y t = 0
d 2Fz = −(2X ) −1 [p 2 l
( 2)
+ p3 m ] = 0.35714
( 2)
⇒ d 2Fz t = 0.035714
Shear Flux in the Flange 4
F F
q ( s ) = (qF(iy) + t d1 y s + t d 2 y s 2 ) Fy + (qF(iz) + t d1Fz s + t d 2Fz s 2 ) Fz = qF( ey= 4 ) Fy + qF( ez = 4) Fz

The terms qF( ey= 4) and qF( ez = 4) can be evaluated as follows:


F F
q F(ey=4 ) ( s ) = qF(iy) + t d1 y s + t d 2 y s 2 = q F( 4y ) + 0.042169 s

q F(ez =4 ) ( s ) = q F(iz) + t d1Fz s + t d 2Fz s 2 = qF( 4z ) − 0.071429 s + 0.035714 s 2

at the end s = 2 , (node 1 )


q F(ey=4 ) ( s = 2) = q F( 4y ) + 0.042169 s = qF( 4y ) + 0.0843373 = q F(1y)

q F( ez = 4) ( s = 2) = qF( 4z ) − 0.071429 s + 0.035714s 2 = qF( 4z ) + 0 = qF(1z)

The total force in the flange 4, (see Problem 4.31-NOTE 1)


 F a F a 
3
 a a3 
f ( e=4 ) = q F(iy) a + t d1 y + t d 2 y  Fy +  q F(iz) a + t d1Fz + t d 2Fz  Fz
 2 3  2 3
= (2q F( 4y ) + 0.0843373) Fy + (2q F( 4z ) − 0.047619) Fz

Torsion Moment at O due to Flange 4, (see Problem 4.31-NOTE 1):

(M ) Fy ( e= 4 )
O =
[m X 2(i ) − l X 3(i ) ]a
6
F F
(3atd1 y + 2a 2td 2 y + 6q F(iy) ) = 0

(M ) Fz ( e=4 )
O =
[m X 2(i ) − l X 3(i ) ]a
6
(3atd1Fz + 2a 2td 2Fz + 6q F(iz) ) = 0

Flange 5: (Flange data ( X 2(i ) , X 3(i ) , t , l , m ) in Table 6.2)


q F(iy) = q F(8y) , q F(iz) = q F(8z )
F
[
d1 y = −X −1 p0(1) + p2(1) X 2(i ) + p3(1) X 3(i ) = −0.12048 ] ⇒
F
d1 y t = −0.012048
d1Fz = −X −1 [p ( 2)
0 + p2( 2 ) X 2(i ) + p3( 2 ) X 3(i )] = 0.71429 ⇒ d1Fz t = 0.071429
F
d 2 y = −(2X ) −1 p2(1) [ l + p3(1) m ] = −0.0677711 ⇒ F
d 2 y t = −0.00677711
d 2Fz = −(2X ) −1 [p 2 l
(2)
+ p3( 2 ) m ] = 0 ⇒ d 2F t = 0 z

Shear Flux in the Flange 5

University of Castilla-La Mancha Draft By: Eduardo W. V. Chaves (2016)


Ciudad Real – Spain
6 LINEAR ELASTICITY 735

F F
q( s ) = (q F(iy) + t d1 y s + t d 2 y s 2 ) Fy + (q F(iz) + t d1Fz s + t d 2Fz s 2 ) Fz = qF( ey=5) Fy + q F( ez =5) Fz

The terms q F( ey= 5) and q F( ez = 5) can be evaluated as follows:


F F
q F(ey=5) ( s ) = q F(iy) + t d1 y s + t d 2 y s 2 = q F(8y) − 0.012048s − 0.00677711s 2

q F(ez =5) ( s ) = q F(iz) + t d1Fz s + t d 2Fz s 2 = q F(8z ) + 0.071429 s

at the end s = 2 , (node 5 )


q F(ey=5) ( s = 2) = q F(8y) − 0.012048s − 0.00677711s 2 = qF(8y) − 0.0512048 = qF(5y)

q F(ez =5) ( s = 2) = q F(8z ) + 0.071429 s = q F(8z ) + 0.1428571 = q F(5z )

The total force in the flange 5, (see Problem 4.31-NOTE 1)


 F a F a 
3
 a a3 
f ( e=5) =  qF(iy) a + t d1 y + t d 2 y  Fy + q F(iz) a + t d1Fz + t d 2Fz  Fz
 2 3  2 3
= (2qF(8y) − 0.0421687) Fy + (2q F(8z ) + 0.14285771) Fz

Torsion Moment at O due to Flange 5, (see Problem 4.31-NOTE 1):

(M ) Fy ( e=5 )
O =
[m X 2(i ) − l X 3(i ) ]a
6
F F
(3atd1 y + 2a 2td 2 y + 6q F(iy) ) = 0

(M ) Fz ( e=5 )
O =
[m X 2(i ) − l X 3(i ) ]a
6
(3atd1Fz + 2a 2td 2Fz + 6q F(iz) ) = 0

Flange 6: (Flange data ( X 2(i ) , X 3(i ) , t , l , m ) in Table 6.2)


q F(iy) = qF(5y) , q F(iz) = qF(5z )
F
[
d1 y = −X −1 p0(1) + p2(1) X 2(i ) + p3(1) X 3(i ) = −0.3915663 ] ⇒
F
d1 y t = −0.03915663
d1Fz = −X −1 [p ( 2)
0 + p2( 2 ) X 2(i ) + p3( 2 ) X 3(i )] = 0.7142857 ⇒ d1Fz t = 0.07142857
F
d 2 y = −(2X ) −1 p2(1) [ l + p3(1) m ] = 0 ⇒ d 2F t = 0 y

d 2Fz = −(2X ) −1 [p 2 l
(2)
+ p3( 2 ) m ] = −0.3571429 ⇒ d 2Fz t = −0.03571429
Shear Flux in the Flange 6
F F
q( s ) = (qF(iy) + t d1 y s + t d 2 y s 2 ) Fy + (qF(iz) + t d1Fz s + t d 2Fz s 2 ) Fz = qF(ey=6 ) Fy + q F(ez =6 ) Fz

The terms q F(ey=6) and q F(ez =6) can be evaluated as follows:


F F
q F(ey=6 ) ( s ) = q F(iy) + t d1 y s + t d 2 y s 2 = qF(5y) − 0.03915663s

q F(ez =6 ) ( s ) = q F(iz) + t d1Fz s + t d 2Fz s 2 = qF(5z ) + 0.07142857 s − 0.3571429s 2

at the end s = 2 , (node 6 )


q F(ey=6 ) ( s = 2) = q F(5y) − 0.03915663s = q F(5y ) − 0.0783133 = qF( 6y )

q F(ez =6 ) ( s = 2) = q F(5z ) + 0.07142857 s − 0.3571429s 2 = qF(5z ) + 0 = qF( 6z )

University of Castilla-La Mancha Draft By: Eduardo W. V. Chaves (2016)


Ciudad Real - Spain
736 SOLVING PROBLEMS BY MEANS OF CONTINUUM MECHANICS

The total force in the flange 6, (see Problem 4.31-NOTE 1)


 F a F a 
3
 a a3 
f ( e=6 ) = q F(iy) a + t d1 y + t d 2 y  Fy +  qF(iz) a + t d1Fz + t d 2Fz  Fz
 2 3  2 3
= (2q F(5y) − 0.0783133) Fy + (2q F(5z ) + 0.047619) Fz

Torsion Moment at O due to Flange 6, (see Problem 4.31-NOTE 1):

(M ) Fy ( e=6 )
O =
[m X 2(i ) − l X 3(i ) ]a
6
F F
(3atd1 y + 2a 2td 2 y + 6q F(iy) ) = 12qF(5y) − 0.4698795

(M ) Fz ( e=6 )
O =
[m X 2(i ) − l X 3(i ) ]a
6
(3atd1Fz + 2a 2td 2Fz + 6q F(iz) ) = 12q F(5z ) + 0.2857143

Flange 7: (Flange data ( X 2(i ) , X 3(i ) , t , l , m ) in Table 6.2)


q F(iy) = qF( 6y) , q F(iz) = q F( 6z )
F
[
d1 y = −X −1 p0(1) + p2(1) X 2(i ) + p3(1) X 3(i ) = −0.3915663 ] ⇒
F
d1 y t = −0.03915663
d1Fz = −X −1 [p ( 2)
0 + p2( 2 ) X 2(i ) + p3( 2 ) X 3(i )] = −0.7142857 ⇒ d1Fz t = −0.07142857
F
d 2 y = −(2X ) −1 p2(1) [ l + p3(1) m ] = 0.0677711 ⇒ d 2F t = 0.00677711
y

d 2Fz = −(2X ) −1 [p 2 l
(2)
+ p3( 2 ) m ] = 0 ⇒ d 2F t = 0 z

Shear Flux in the Flange 7


F F
q( s ) = (qF(iy) + t d1 y s + t d 2 y s 2 ) Fy + (q F(iz) + t d1Fz s + t d 2Fz s 2 ) Fz = q F( ey=7 ) Fy + q F( ez =7 ) Fz

The terms q F(ey=7 ) and q F(ez =7 ) can be evaluated as follows:


F F
q F(ey=7 ) ( s ) = qF(iy) + t d1 y s + t d 2 y s 2 = q F( 6y ) − 0.03915663s + 0.00677711s 2

q F(ez =7 ) ( s ) = q F(iz) + t d1Fz s + t d 2Fz s 2 = q F( 6z ) − 0.07142857 s

at the end s = 2 , (node 10 )


q F(ey=7 ) ( s = 2) = qF(6y ) − 0.03915663s + 0.00677711s 2 = qF( 6y ) − 0.0512048 = q F(10y )

q F( ez = 7 ) ( s = 2) = q F( 6z ) − 0.07142857 s = q F( 6z ) − 0.1428571 = q F(10z )

The total force in the flange 7, (see Problem 4.31-NOTE 1)


 F a F a 
3
 a a3 
f ( e=7 ) = q F(iy) a + t d1 y + t d 2 y  Fy +  q F(iz) a + t d1Fz + t d 2Fz  Fz
 2 3  2 3
= (2q F(6y ) − 0.060241) Fy + (2qF( 6z ) − 0.1428571) Fz

Torsion Moment at O due to Flange 7, (see Problem 4.31-NOTE 1):

(M ) Fy ( e=7 )
O =
[m X 2(i ) − l X 3(i ) ]a
6
F F
(3atd1 y + 2a 2td 2 y + 6q F(iy) ) = 4q F( 6y ) − 0.1204819

(M ) Fz ( e =7 )
O =
[m X 2(i ) − l X 3(i ) ]a
6
(3atd1Fz + 2a 2td 2Fz + 6qF(iz) ) = 4q F(6z ) − 0.2857143

University of Castilla-La Mancha Draft By: Eduardo W. V. Chaves (2016)


Ciudad Real – Spain
6 LINEAR ELASTICITY 737

The Total Torsion Moment at O

(M ) ∑ (M )
7
Fy ( Sys ) Fy ( e )
O = O
e=1

= (0) + (8q F( 7y ) − 0.0963855) + (8q F(9y ) + 0.09639) + (0) + (0) + (12q F(5y ) − 0.4698795)
+ (4q F( 6y ) − 0.1204819)

(M ) ∑ (M )
7
Fz ( Sys ) Fz ( e )
O = O
e=1

= (0) + (8q F(7z ) + 0.1904762) + (8q F(9z ) − 1.14286) + (0) + (0) + (12q F(5z ) + 0.2857143)
+ ( 4q F( 6z ) − 0.2857143)

By restructuring the above two equations we can obtain

(M ) Fy ( Sys )
O = 12qF(5y) + 4q F(6y ) + 8q F(9y ) + 8q F(7y ) − 0.5903614

(M ) Fz ( Sys )
O = 12q F(5z ) + 4q F( 6z ) + 8qF( 7z ) + 8qF(9z ) − 0.952381
F
M O = M O y Fy + M OFz Fz = −( X 3( S .C .) ) Fy + ( X 2( S .C .) ) Fz (6.326)
Solution due to Fy
For each flange we have obtained the following relationships between nodal shear fluxes:
q F(1y) + 0.060241 = qF( 2y) ⇒ − qF(1y) + qF( 2y) = 0.060241 (flange 1) (6.327)
q F( 7y) − 0.0240964 = q F(3y) ⇒ qF(3y) − qF(7y) = −0.0240964 (flange 2) (6.328)
q F(9y) + 0.060241 = q F( 4y) ⇒ q F( 4y) − q F(9y) = 0.060241 (flange 3) (6.329)
q F(8y) − 0.0512048 = qF(5y) ⇒ qF(5y) − qF(8y) = −0.0512048 (flange 5) (6.330)
q F(5y) − 0.0783133 = qF(6y) ⇒ qF(6y) − qF(5y) = −0.0783133 (flange 6) (6.331)
q F( 6y) − 0.0512048 = qF(10y ) ⇒ qF(10y ) − q F( 6y) = −0.0512048 (flange 7) (6.332)
For the two bifurcation points the following must be true
Shear flux compatibility at nodes 2,7,8 Shear flux compatibility at nodes 3,9,10

7
9 10
q (7 )
q (9 ) q (10)
q (2)
q ( 3)
8
2 q (8 ) 3

q ( 7 ) + q (8) = q ( 2 ) q (3) + q (10) = q (9 ) (6.333)


Another equation we can add to the system is the total torsion moment obtained
previously, and since we are searching for the shear center, the following must be true
F
M O y Fy − (− X 3( S .C .) ) Fy = 0 ⇒ 12q F(5y) + 4qF( 6y) + 8qF(9y) + 8qF( 7y) − (− X 3( S .C .) ) = 0.5903614 (6.334)

Up to now we have 9 equations and 11 unknowns (10 for shear fluxes and 1 for X 3( S .C .) ).

University of Castilla-La Mancha Draft By: Eduardo W. V. Chaves (2016)


Ciudad Real - Spain
738 SOLVING PROBLEMS BY MEANS OF CONTINUUM MECHANICS

The two missing equations can be added by considering the following equation for each
cell:
MT s
∫ q dΓ = 2GθAt = { ∫ q dΓ = 0
Center θ=0
Shear
  →
Γ
2A Γ
=0

Note that we have obtained previously the forces for each flange, (see Figure 6.122).

X3 f F(ye =1) = 4q F(1y) + 0.1927711

4 3 9 10 7 6 f F(ye = 2 ) = 2qF( 7y) − 0.02409639

f F(ye=3) f F(ye = 3) = 4qF(9y) + 0.0481928


3 f F(ye=7 )
4 f F(ye = 4 ) = 2qF( 4y) + 0.0843373
6
Cell 1 Cell 2 f F(ye = 5) = 2qF(8y) − 0.0421687
f F(ye=4) f F(ye=2) 2
f F(ye=6) f F(ye = 6 ) = 2qF(5y) − 0.0783133

f F(ye=1) f F(ye = 7 ) = 2qF( 6y) − 0.060241


7 f F(ye=5)

1 1 2 8 5 5 X2

Figure 6.122: Forces on the flanges due to Fy .

Cell 1 ( f F(ye =1) + f F(ye = 2) + f F(ye = 3) + f F(ye = 4) = 0 )

(4qF(1y) + 0.1927711) + (2qF(7y) − 0.02409639) + (4qF(9y) + 0.0481928) + (2qF( 4y) + 0.0843373) = 0

⇒ 4qF(1y) + 2qF( 4y) + 2qF( 7y) + 4qF(9y) = −0.30120482 (6.335)

Cell 2 ( f F(ye = 5) + f F(ye = 6) + f F(ye = 7) − f F(ye = 2) = 0 )

(2qF(8y) − 0.0421687) + (2qF(5y) − 0.0783133) + (2qF(6y) − 0.060241) − (2qF(7y) − 0.02409639) = 0

⇒ 2qF(5y) + 2qF( 6y) − 2q F( 7y) + 2qF(8y) = 0.15662651 (6.336)


Then, by restructuring the equations (6.327)-(6.336) in matrix form we can obtain
 qF(1) 
− 1 1 0 0 0 0 0 0 0 0 0  y
  0.06024096 
0 0 1 0 0 0 −1 0 0 0 0   q ( 2)
F  − 0.02409639
 
y
( 3)   
0 0 0 1 0 0 0 0 −1 0 0   q Fy   0.06024096 
   
0   qFy  − 0.05120482
( 4)
0 0 0 0 1 0 0 −1 0 0
 
0 0 0   qFy  − 0.07831325
( 5)
0 0 −1 1 0 0 0 0
    
0 0 0 0 0 −1 0 0 0 1 0   qF( 6)  = − 0.05120482 Solve
→
   
y
 0 −1 0 0 0 0 1 1 0 0 0 (7) 0
   qFy   
0 0 1 0 0 0 0 0 −1 1 0  q ( 8 )   0 
   Fy   
0 0 0 0 12 4 8 0 8 0 − 1  q (9 )   0.59036145 
Fy
4 0 0 2 0 0 2 0 4 0 0   (10)  − 0.30120482
   qF y   
0 0 0 0 2 2 −2 2 0 0 0  ( S .C .)   0.15662651 
− X 3 

University of Castilla-La Mancha Draft By: Eduardo W. V. Chaves (2016)


Ciudad Real – Spain
6 LINEAR ELASTICITY 739

And the solution (nodal shear flux + shear center) is


 q F(1) 
 y
  0.04216867 
 qF y   0.10240964 
( 2)

 ( 3)   
 q F y   − 0 . 01204819 
 q ( 4)  − 0.04216867
 F(5y)   
 qFy   0.03915663 
   
 qFy  =  − 0.03915663
(6)

 (7)   0.01204819 
 qF y   
 q (8)   0.09036145 
 Fy   
 qF(9)  − 0.10240964
 (10)  − 0.09036145
y

 qFy   
− X ( S .C .)   − 1.0 
 3 
Solution due to Fz
For each flange we have obtained the following relationships between nodal shear fluxes:
q F(1z) + 0.2857143 = qF( 2z ) ⇒ − q F(1z) + q F( 2z ) = 0.2857143 (flange 1) (6.337)
q F( 7z ) = qF(3z) ⇒ qF(3z) − qF( 7z ) = 0 (flange 2) (6.338)
q F(9z ) − 0.2857143 = qF( 4z ) ⇒ qF( 4z ) − qF(9z ) = −0.2857143 (flange 3) (6.339)
q F(8z) + 0.1428571 = qF(5z ) ⇒ qF(5z ) − q F(8z) = 0.1428571 (flange 5) (6.340)
q F(5z ) = qF( 6z ) ⇒ qF( 6z ) − qF(5z ) = 0 (flange 6) (6.341)
q F( 6z ) − 0.1428571 = qF(10z ) ⇒ qF(10z ) − qF(6z ) = −0.1428571 (flange 7) (6.342)
For the two bifurcation points the following must be true
q F( 7z ) + qF(8z) = qF( 2z ) q F(3z) + qF(10z ) = qF(9z ) (6.343)
Another equation due to the total torsion moment obtained previously, (see equation
(6.326)), and since we are searching for the shear center, the following must be true
M OFz Fz − X 2( S .C .) Fz = 0 ⇒ 12q F(5z ) + 4qF( 6z ) + 8qF(7z ) + 8qF(9z ) − X 2( S .C .) = 0.952381 (6.344)

Up to now we have 9 equations and 11 unknowns (10 for shear fluxes and 1 for X 2( S .C .) ).
The two missing equations can be added by considering the following equation for each
cell:
MT s
∫ q dΓ = 2GθAt = { ∫ q dΓ = 0
Center θ=0
Shear
  →
Γ
2A Γ
=0

University of Castilla-La Mancha Draft By: Eduardo W. V. Chaves (2016)


Ciudad Real - Spain
740 SOLVING PROBLEMS BY MEANS OF CONTINUUM MECHANICS

Note that we have obtained previously the forces for each flange, (see Figure 6.123).

X3 f F(ze =1) = 4q F(1z) + 0.5714286

4 3 9 10 7 6 f F(ze = 2 ) = 2q F( 7z ) + 0.047619
f F(ze =3) = 4q F(9z ) − 0.5714286
f F(ze = 3) 3 f F(ze = 7 )
4 f F(ze = 4 ) = 2q F( 4z ) − 0.047619
6
Cell 2 f F(ze =5) = 2q F(8z ) + 0.14285771
f F(ze = 4 ) Cell 1 f F(ye=2) 2
f F(ze = 6 ) f F(ze = 6 ) = 2q F(5z ) + 0.047619
f F(ze = 7 ) = 2q F( 6z ) − 0.1428571
f F(ze =1) 7 f F(ze = 5)

1 1 2 8 5 5 X2

Figure 6.123: Forces on the flanges due to Fz .


Cell 1 ( f F(ze =1) + f F(ze = 2) + f F(ze = 3) + f F(ze = 4) = 0 )

(4qF(1z) + 0.5714286) + (2qF( 7z ) + 0.047619) + (4qF(9z ) − 0.5714286) + (2qF( 4z ) − 0.047619) = 0

⇒ 4qF(1z) + 2qF( 4z ) + 2qF(7z ) + 4qF(9z ) = 0 (6.345)

Cell 2 ( f F(ze = 5) + f F(ze = 6) + f F(ze = 7 ) − f F(ze = 2) = 0 )

(2qF(8z) + 0.14285771) + (2qF(5z ) + 0.047619) + (2qF(6z ) − 0.1428571) − (2qF(7z ) + 0.047619) = 0

⇒ 2qF(5z ) + 2qF( 6z ) − 2qF( 7z ) + 2q F(8z) = 0 (6.346)


Then, by restructuring the equations (6.337)-(6.346) in matrix form we can obtain
0   q Fz   0.28571429 
(1)
− 1 1 0 0 0 0 0 0 0 0
0 0  (2) 
 1 0 0 0 −1 0 0 0 0   qFz   0 

0 0 0 1 0 0 0 0 −1 0 0   qF(3z)  − 0.28571429
    
0 0 0 0 1 0 0 −1 0 0 0   qF( 4z )   0.14285714 
0 0 0 0 −1 1 0 0 0 0 0   qF(5z )   0 
   (6)   
0 0 0 0 0 −1 0 0 0 1 0   qF  = − 0.14285714 Solve

→
 (7 )   
z
 0 −1 0 0 0 0 1 1 0 0 0  0
   q Fz   
0 0 1 0 0 0 0 0 −1 1 0   qF  
( 8 ) 0 
   (9z )   
0 0 0 0 12 4 8 0 8 0 − 1  qF   0.95238095 
z
4 0 0 2 0 0 2 0 4 0 0   qF(10)   0 
   ( Sz.C .)   
0 0 0 0 2 2 −2 2 0 0 0  X   0 
 2 
And the solution (nodal shear flux + shear center) is

University of Castilla-La Mancha Draft By: Eduardo W. V. Chaves (2016)


Ciudad Real – Spain
6 LINEAR ELASTICITY 741

 qF(1)  − 0.14906832
 ( 2z)   
 qFz   0.13664596 
 q (3)   0.17391304 
 F( 4z )   
 qFz  − 0.14906832
 (5)   0.10559006 
 qFz   
 q Fz  =  0.10559006 
(6)

 (7 )   0.17391304 
 q Fz   
 qF(8)  − 0.03726708
 (9z )   
 qFz   0.13664596 
 q (10)  − 0.03726708
 (FSz.C .)   
 X 2   3.22153209 

NOTE: For a given Fy and Fz located at ( X 2( F ) , X 3( F ) ), (see Figure 6.124), the torsion
moment is given by:
M T = − Fy ( X 3( F ) − X 3( S .C .) ) + Fz ( X 2( F ) − X 2( S .C .) )

And we can superimpose the torsion only effect, (see Problem 6.52).

G – Area Centroid X 2 = 3.111; X 3 = 1.0 Fz


X3 S.C. – Shear Center X 2( S .C .) = 3.2215; X 3( S .C .) = 1.0 ( X 2( F ) , X 3( F ) )

Fy

Fz

MT axis of symmetry
Fy
⊗ ⊗
G S.C.

X2

Figure 6.124

University of Castilla-La Mancha Draft By: Eduardo W. V. Chaves (2016)


Ciudad Real - Spain
742 SOLVING PROBLEMS BY MEANS OF CONTINUUM MECHANICS

6.4.3 The Total Potential Energy for Torsion Problem


In Chapter 5, (see Problem 5.25), we have defined the total potential energy Π defined as
follows:
r r r r r
Π (u) = Ψ e (ε ) dV − t * ⋅ udS − ( ρb) ⋅ udV
∫ ∫ ∫ The total potential energy (6.347)
V Sσ V

where

1

U int = Ψ e (ε ) dV =
V V
∫ 2 σ : ε dV The internal potential energy (6.348)

and
r r r r
U ext = t * ⋅ udS + ( ρb) ⋅ udV
∫ ∫ The external potential energy (6.349)
Sσ V

For the torsion problem the stress state is only given by σ12 and σ13 , (see equation
(6.229)), then the internal potential energy becomes:
1 1 1 1
U int = ∫ 2 σ : ε dV = ∫ 2 σ ε ∫ 2 ( 2σ ∫ 2G (σ
2 2
ij ij dV = 12ε12 + 2σ13ε13 ) dV = 12 + σ13 ) dV (6.350)
V V V V

where we have used the definition σ12 = G 2ε12 = Gγ xy and σ13 = G 2ε13 = Gγ xz . Recall that
∂φ ∂φ
σ12 = ; σ13 = − (6.351)
∂x3 ∂x2
where φ = φ ( x2 , x3 ) is the Prandtl’s stress function, (see equation (6.244)). Then the internal
potential energy can be rewritten as follows:

1  ∂φ   ∂φ  
2 2
1 1
∫   +  ∫
  dV = (φ ,kφ ,k ) dV ∫
int 2 2
U = (σ12 + σ13 ) dV = (6.352)
2G 2G  ∂x3   ∂x2   2G
V V   V

For a position x1 the above equation becomes:

1  ∂φ   ∂φ  1  ∂φ   ∂φ 
2 2 2 2
 
∫   +   dV = x1  ∫  +   dA
int
U = (6.353)
2G  ∂x3   ∂x 2   2G  ∂x3   ∂x 2  
V   A  
The external potential energy without body forces becomes:
r r
U ext = t * ⋅ udS =∫ ∫ t u dS = ∫ (t u
*
k k
*
1 1 + t *2u 2 + t *3u 3 )dS = (σ12
*

*
u 2 + σ13 u 3 )dS
Sσ Sσ Sσ Sσ

where we have considered that for the torsion problem t 1* = 0 , t *2 = σ12


*
and t *3 = σ12
*
. Note
that that according to displacement equation (6.224), the above equation becomes:
U ext = ∫ (σ ∫ ( −σ ∫
* * * * * *
12u 2 + σ13u3 )dS = 12 x3 θ x1 + σ13 x2 θ x1 )dS = x1θ (−σ12 x3 + σ13 x2 )dS
Sσ Sσ Sσ
(6.354)
⇒ U ext = x1θ (−σ12 ∫ x2 )dS = x1θM T = 2 x1θ φ dA = x1 2θφ dA ∫ ∫
* *
x3 + σ13
Sσ A A

where M T is the torque applied.

University of Castilla-La Mancha Draft By: Eduardo W. V. Chaves (2016)


Ciudad Real – Spain
6 LINEAR ELASTICITY 743

Then, the total potential energy becomes:


Π = U int − U ext [J ]

1  ∂φ   ∂φ  
2 2

⇒ Π = x1
2G



 ∂x   ∂x   dA − x1 2θφ dA
 +
 




A  3 2
 A

1  ∂φ   ∂φ  
2 2
Π J 
⇒ =Π=
x1


 +   dA − 2θφ dA
2G  ∂x3   ∂x2   ∫ m
 
(6.355)
A   A

Problem 6.55
Considering a torsion problem in a rectangular cross section ( 2a × 2b ), (see Figure 6.125),
and by considering that the displacement field u1 is approached by the function
u1 = Kθx2 x3 , obtain the tangential stress field and the JT (polar moment of inertia).

x3

O x2
b

a a

Figure 6.125: Rectangular cross section.


Solution:
The internal potential energy U int , (see equation (6.350)), is:
1 1 1
U int = ∫ 2 σ : ε dV = ∫ 2G (σ ∫ 2G (σ
2 2 2 2
12 + σ13 ) dV = x1 12 + σ13 ) dA (6.356)
V V A

The stress field, (see equation (6.229)), can be expressed in terms of the field u1 as follows:
  ∂u1   ∂u1 
 0  − x3 θ   + x θ 
 ∂x 2 
 0 σ12 σ13    ∂x2   3 
 ∂u  
σij = σ12 0 0  = G  1 − x3 θ  0 0  (6.357)
σ13 0 0   ∂x2  
 ∂u  
 1 + x2 θ  0 0 
 ∂x3  
When u1 = Kθx2 x3 we can obtain:
 0 σ12 σ13   0 (Kθx3 − x3 θ) (Kθx2 + x2 θ)
σij = σ12 0  
0  = G  (Kθx3 − x3 θ) 0 0 
 (6.358)
σ13 0 0  (Kθx2 + x2 θ) 0 0 

University of Castilla-La Mancha Draft By: Eduardo W. V. Chaves (2016)


Ciudad Real - Spain
744 SOLVING PROBLEMS BY MEANS OF CONTINUUM MECHANICS

Then, the internal potential energy can be rewritten in terms of the constant K :
1 G2
U int ( K ) = x1 ∫ ∫
2 2
(σ12 + σ13 ) dA = x1 [( Kθx3 − x3 θ) 2 + ( Kθx2 + x2 θ) 2 ] dA
A
2G A
2G
G
=
2 A ∫
x1 [(θx3 ) 2 ( K − 1) 2 + (θx2 ) 2 ( K + 1) 2 ] dA
(6.359)
G ( K − 1) 2 θ2 Gθ2 ( K + 1) 2
=
2 A

x1 ( x3 ) 2 dA +
2 A

x1 ( x2 ) 2 dA

G ( K − 1) 2 θ2 Gθ2 ( K + 1) 2
= x1I 22 + x1I33
2 2
where Iij is the inertia tensor of area related to the system O − x2 − x3 :

 

2 2
 ( x2 + x3 )dA 0  0
I11 0 0  A 
IO( A)ij =  0 I 22 0  =  0 ∫
2
x3 dA 0 
 
 0 0 
I33   A 


0 0
A

x22 dA

(6.360)
   
I 22 + I33 0 0  I 22 + I33 0 0 
 (2a )(2b)3   4ab3 
= 0 0 = 0 0 
 12   3 
 (2b)(2a)3   4ba 3 
 0 0   0 0 
 12   3 
Taking into account the external potential energy U ext = x1θM T , the total potential energy
becomes:
Π = U int − U ext
G ( K − 1) 2 θ2 Gθ2 ( K + 1) 2 (6.361)
⇒ Π(K ) = x1I 22 + x1I33 − x1θM T
2 2
As we are looking for the stationary point, the following must be true:
∂Π ( K ) ∂  G ( K − 1) 2 θ2 Gθ2 ( K + 1) 2 
=  x I + x1I33 − x1θM T  = 0
∂K 
∂K  2
1 22
2 
⇒ G ( K − 1)θ2 x1I 22 + Gθ2 ( K + 1) x1I33 = 0 ⇒ ( K − 1)I 22 + ( K + 1)I33 = 0 (6.362)
I 22 − I33
⇒K =
I 22 + I33
Then, the displacement field u1 = Kθx2 x3 becomes:
I −I   b2 − a 2 
u1 = Kθx2 x3 =  22 33 θx2 x3 =  2 θx2 x3
2  (6.363)
 I 22 + I33  a +b 
and the stress field:

University of Castilla-La Mancha Draft By: Eduardo W. V. Chaves (2016)


Ciudad Real – Spain
6 LINEAR ELASTICITY 745

  I − I    I − I   
 0 θx3  22 33  − 1 θx2  22 33  + 1 
  I22 + I33    I22 + I33   
 0 σ12 σ13    
I 22 − I33  
σij = σ12 0   
0  = G θx3   − 1 0 0 
  I 22 + I33   
σ13 0 0   
θx  I22 − I33  + 1 0 0  (6.364)
 2  I + I   
  22 33   
 0 − x3I33 x2I22   0 − x3a 2 x2b 2 
2Gθ   2Gθ  2 
= − x3I33 0 0 = 2 − x3a 0 0 
I 22 + I33  (a + b 2 ) 
 x2I 22 0 0   x2b
2
0 0 

As we have seen previously, the net moment is obtained as follows:

∫ (I )
2Gθ 2Gθ
∫ (σ − σ12 x3 )dA = ∫ (I + I33 x3 x3 )dA = 2
MT = 13 x2 22 x2 x2 22 x2 + I33 x32 dA
A
I22 + I33 A
I 22 + I33 A

2Gθ   2Gθ
= ∫
2 2

I 22 x2 dA + I33 x3 dA = [I22I33 + I33I22 ] (6.365)
I22 + I33  A A  I 22 + I33
I 22I33 4a 3b3 16a 3b3
= 4Gθ = 4Gθ = Gθ
I22 + I33 3(a 2 + b 2 ) 3(a 2 + b 2 )
MT
Then, by using the equation JTeff = , we can obtain:

 16a 3b3 
 Gθ
M T  3(a + b ) 
2 2
16a 3b3 (6.366)
JTeff = = =
Gθ Gθ 3(a 2 + b 2 )
When b = a the above equation becomes:
MT 8 4
JTeff = = a = 2.66666667a 4 (6.367)
Gθ 3
and if we compare with the exact solution JTeff = 2.2496a 4 we can see that the error is
approximately 18.5% .

University of Castilla-La Mancha Draft By: Eduardo W. V. Chaves (2016)


Ciudad Real - Spain
746 SOLVING PROBLEMS BY MEANS OF CONTINUUM MECHANICS

6.4.4 Introduction to Finite Element for Torsion Problems

Problem 6.56
Starting from the total potential energy obtain an expression equivalent to the torsion
~ ~
problem such as [k ( e ) ]{φ ( e ) } = { f ( e ) } , where {φ ( e ) } = Gθ{φ ( e ) } is the nodal membrane
deflection vector. Consider also that the membrane deflection field is
r
φ = φ ( x2 , x3 ) = [ N ( x )]{φ ( e) } .
Solution:
We can approach the field φ = φ ( x 2 , x3 ) by means of shape functions as follows:
φ 1( e ) 
 (e) 
φ  r
φ = φ ( x2 , x3 ) = [ N1 N 2 L N n ] 2  = [ N ( x )]1× n {φ ( e) }n×1 (6.368)
 M 
φ ( e ) 
 n 
where n is the number of nodes. With that the potential Π , (see equation (6.355)), can be
written in terms of nodal values {φ (e ) } as follows:

1  ∂φ   ∂φ  
2 2

Π= ∫  +    dA − 2θφ dA
2G  ∂x3   ∂x2   ∫
A   A
(6.369)
1  ∂[ N ]{φ ( e ) }   ∂[ N ]{φ (e ) }  
2 2

(
Π {φ } =
(e)
2G
) ∫

 ∂x
 +
  ∂ x   ∫
  dA − 2θ[ N ]{φ ( e ) } dA
A  3   2   A

As we are looking for a stationary point, the following must be true:

(
∂Π {φ ( e ) } ) 
∂  1  ∂[ N ]{φ ( e ) }   ∂[ N ]{φ (e ) }  
2 2 

∫      
∫ φ (e )
= + dA − 2 θ[ N ]{ } dA  = {0}
∂{φ ( e ) } ∂{φ ( e ) }  A 2G  ∂x3  
  ∂ x  

  2
 A 
(6.370)
thus
(
∂Π {φ ( e ) } ) 1   ∂[ N ]{φ ( e ) }  ∂[ N ]T  ∂[ N ]{φ ( e ) }  ∂[ N ]T 
∫    

T
=  2  + 2   dA − 2θ[ N ] dA = {0}
∂{φ }
(e)
A
2G   ∂x3  ∂x3  ∂x2  ∂x2  A
(6.371)
or
  ∂[ N ]T ∂[ N ] ∂[ N ]T ∂[ N ]   {φ ( e ) }
   dA 
 ∫
 A  ∂x3 ∂x3
+
∂x2 ∂x2   G A

− 2θ [ N ]T dA = {0} (6.372)

Note that:

University of Castilla-La Mancha Draft By: Eduardo W. V. Chaves (2016)


Ciudad Real – Spain
6 LINEAR ELASTICITY 747

 ∂N1 ∂N1 
 ∂x3   ∂N
 ∂[ N ]   ∂x2  1 ∂N 2 ∂N n 
∂N ∂N 2   L
∂[ N ] ∂[ N ] ∂[ N ] ∂[ N ]  ∂[ N ]T
T T
∂[ N ]T   ∂x2   2 ∂x2 ∂x2 ∂x2 
+ =    =  ∂x ∂x3   ∂N 
∂x2 ∂x2 ∂x3 ∂x3  ∂x2 ∂x3   ∂[ N ]   2 ∂N 2 ∂N n 
M  
1
M L
 ∂x3   ∂N 
∂N n  3 ∂x ∂x3 ∂x3 
 n 
 ∂x2 ∂x3 
= [ B ]T [ B ]
(6.373)
where we have considered that:
 ∂N1 ∂N 2 ∂N n 
 ∂x L
∂x2 ∂x2 
[B ] =  2  (6.374)
 ∂N1 ∂N 2 ∂N n 
L
 ∂x3 ∂x3 ∂x3  2×n

Then, the equation (6.372) becomes:


  (e)
 [ B ]T [ B ] dA  {φ } − 2θ [ N ]T dA = {0}

A
∫  G
 A

  {φ (e ) }
⇒  [ B ]T [ B ] dA 
∫ = 2 [ N ]T dA ∫
  Gθ (6.375)
A  A

 ~
⇒  [ B ]T [ B ] dA {φ ( e ) } = 2 [ N ]T dA
∫ ∫
 
A  A
~
⇒ [ k ( e ) ]n× n {φ ( e ) }n×1 = { f ( e ) }n×1

~ {φ ( e ) }
where we have considered that {φ ( e ) } = . To solve the above equation we have to

~ (e)
introduce the boundary condition which is that on the boundary it must fulfill φ boundary = 0.
Once the problem is solved we can obtain:
ƒ The angle of twist θ :
~ ~
∫A

A

M T = 2Vmemb = 2 φ dA = 2 [ N ]{φ (e ) } dA = 2 [ N ]Gθ{φ ( e ) } dA = 2Gθ [ N ]{φ ( e ) } dA
A

A
(6.376)

Note also that:


MT
JTeff = (6.377)

ƒ The tangential stress can be obtained as follows:
 ∂φ   ∂[ N ]{φ }   ∂[ N ] 
(e)
 ∂N1 ∂N 2 ∂N n 
   L
σ12   ∂x3   ∂x3  ∂x3   ∂x3 ∂x3 ∂x3  ( e )
  =  ∂φ  =  (e )  =
 {φ ( e ) } =  {φ }
∂[ N ]   − ∂N1 − ∂N 2 L − ∂N n 
σ13  −  − ∂[ N ]{φ }  −
 ∂x2   ∂x2   ∂x2   ∂x2 ∂x2 ∂x2 
(6.378)
~
where {φ ( e ) } = Gθ{φ ( e ) } .

University of Castilla-La Mancha Draft By: Eduardo W. V. Chaves (2016)


Ciudad Real - Spain
748 SOLVING PROBLEMS BY MEANS OF CONTINUUM MECHANICS

NOTE 1: Let us make the same development from equation (6.369) to (6.375) by using
indicial notation:

1  ∂φ   ∂φ  
2 2
1
Π=
2G  ∫

 ∂x
 + 
  ∂x
  dA − 2θφ dA =
  2G ∫
(φ ,kφ ,k ) dA − 2θφ dA ∫ (6.379) ∫
A  3 2
 A A A

r
By considering that φ ( x ) = N iφ i(e ) we have φ ,k = N i ,kφ i(e ) . Note that i = 1,2, L , n ( n is the
number of nodes) and k = 1,2,3 . Then, the above equation becomes:
1 1
Π= ∫ 2G (φ
A
,k φ ,k ) dA − ∫ 2θφ dA = ∫
A A
2G ∫
( N i ,kφ (i e ) N p ,kφ (pe ) ) dA − 2θN iφ i(e ) dA
A
(6.380)
1
⇒Π=
A
2G ∫
( N i ,k N p ,kφ (i e )φ (pe ) ) dA − 2θN iφ (i e ) dA
A

Taking the derivative with respect to φ (je ) ( j = 1,2,L, n) we can obtain:

∂Π ∂  1 
= (e ) 
( N i ,k N p , kφ i( e )φ (pe ) ) dA − 2θN iφ i( e ) dA  = 0 j
∫ ∫
∂φ j
(e)
∂φ j  A 2G 
A 
 ∂ (φ i( e )φ (pe ) )   (e) 
1  dA − 2θN i  ∂φ i  dA = 0 j
⇒ ∫
A
2G
N i ,k N p ,k 
 ∂φ (je ) 
  A
 ∂φ (je ) 
 

1  ∂ (φ i( e ) ) ( e ) ( e ) ∂ (φ p ) 
(e)

⇒ ∫
A
2G
N i ,k N p ,k 
 ∂φ j

( e )
φ p + φ i
∂φ j 
( e )
dA − 2θN iδ ij dA = 0 j
A

⇒ ∫ 2G N
1
i ,k N p ,k (δ ij φ (pe) + φ i(e)δ pj ) dA − ∫ 2θN j dA = 0 j
A A

∫ 2G (N δ ijφ (pe) + N i ,k N p ,kφ i( e)δ pj ) dA − ∫ 2θN j dA = 0 j


1
⇒ i ,k N p ,k
A A

∫ ( )
1

A
2G
N j ,k N p ,kφ (pe ) + N i ,k N j ,kφ i( e ) dA − 2θN j dA = 0 j
A
∫ (6.381)

∫ 2G (N φ (pe ) + N p ,k N j ,kφ (pe) ) dA − ∫ 2θN j dA = 0 j


1
⇒ j ,k N p ,k
A A

∫ ( )
1

A
2G
N j ,k N p ,k + N p ,k N j ,k φ (pe ) dA − 2θN j dA = 0 j
A

 1 
⇒
 G
A
∫ ( 
 A
)
N j , k N p ,k dA φ (pe ) − 2θN j dA = 0 j ∫
  φ (pe )

A
∫(
⇒  N j ,k N p ,k dA 
 Gθ
 A
)
− 2 N j dA = 0 j ∫
φ (pe)
⇒ k (jpe )
Gθ ∫
= 2 N j dA = f j( e )
A
~ (e)
⇒ φp =
k (jpe ) f j(e )

where i, j , p = 1,2, L , n and k = 1,2,3 . Note also that neither N i nor φ vary with x1 , so
∂N i ∂φ
≡ N i ,1 = 0i and ≡ φ ,1 = 0 .
∂x1 ∂x1

University of Castilla-La Mancha Draft By: Eduardo W. V. Chaves (2016)


Ciudad Real – Spain
6 LINEAR ELASTICITY 749

Problem 6.57
By taking into account Problem 6.56 obtain the explicit formulation for the equation
~
[k (e ) ]{φ ( e ) } = { f ( e ) } , when the sub-domain (finite element) is a triangle with three nodes,
(see Figure 6.126).

φ ((3e))
( x2(3) , x3(3) )
3 Nodal value for the
membrane deflection
φ (( e2)) φ ((1e)) 
2  
( x2 , x3 ) {φ ( e ) } = φ ((e2)) 
( x2( 2) , x3( 2) ) φ ( e ) 
Ω  ( 3) 
x3 , z
1
A (e) - triangle area
φ ((1e))
( x2(1) , x3(1) )

x2 , y
Figure 6.126: Domain Ω .
NOTE: The element connectivity orientation must be counterclockwise.
Solution:
In Problem 6.40 we have obtained the shape functions for the triangle with three nodes,
and by considering the formulation on the plane x2 − x3 the shape functions can be
rewritten as follows:
 1 (2) ( 3) ( 3) ( 2) ( 2 ) ( 3) ( 2 ) ( 3)
 N1 ( x 2 , x3 ) = 2 A ( e ) [ x2 ( x3 − x3 ) + x3 ( x2 − x2 ) + ( x2 x3 − x3 x 2 )]

 1
 N 2 ( x 2 , x3 ) = (e)
[ x2 ( x3(3) − x3(1) ) + x3 ( x2(1) − x 2(3) ) + ( x2(3) x3(1) − x3(3) x2(1) )]
 2A
 1
 N 3 ( x 2 , x3 ) = [ x2 ( x3(1) − x3( 2 ) ) + x3 ( x2( 2 ) − x 2(1) ) + ( x2(1) x3( 2 ) − x3(1) x2( 2 ) )]
 2 A(e)
(6.382)
 1
 N1 ( x2 , x3 ) = 2 A (e ) [ x 2 b1 + x3 c1 + d1 ]

 1
⇒  N 2 ( x 2 , x3 ) = [ x 2 b2 + x3 c 2 + d 2 ]
 2 A(e )
 1
 N 3 ( x 2 , x3 ) = [ x 2 b3 + x3 c3 + d 3 ]
 2 A(e)

where A (e ) is the triangle area and we have considered that:


b1 = ( x3( 2) − x3(3) ) ; c1 = ( x2(3) − x2( 2 ) ) ; d1 = ( x2( 2) x3(3) − x3( 2 ) x2(3) )
b2 = ( x3(3) − x3(1) ) ; c2 = ( x2(1) − x2(3) ) ; d 2 = ( x2(3) x3(1) − x3(3) x2(1) ) (6.383)
b3 = ( x3(1) − x3( 2 ) ) ; c3 = ( x2( 2 ) − x2(1) ) ; d 3 = ( x2(1) x3( 2 ) − x3(1) x2( 2 ) )

For this case the matrix [B ] , (see equation (6.374)), becomes:

University of Castilla-La Mancha Draft By: Eduardo W. V. Chaves (2016)


Ciudad Real - Spain
750 SOLVING PROBLEMS BY MEANS OF CONTINUUM MECHANICS

 ∂N1 ∂N 2 ∂N 3 
 ∂x ∂x2 ∂x2  1 b1 b2 b2 
[B ] =  2  =  (6.384)
 ∂N1 ∂N 2 ∂N 3  2 A( e ) c1 c2 c3 
 ∂x3 ∂x3 ∂x3  2×3

As we can see, [B ] is constant, then the stiffness matrix, (see equation (6.375)), can be
obtained as follows:


A

[k ( e ) ] = [ B ]T [ B ] dA = [ B ]T [ B ] dA = A (e ) [ B ]T [ B ]
A
(6.385)

or
 b12 + c12 b1b2 + c1c 2 b1b3 + c1c3 
(e) (e) T 1  
[k ] = A [B ] [B ] = b1b2 + c1c2 b22 + c22 b2 b3 + c 2 c3  (6.386)
4 A(e) b b + c c
 1 3 1 3 b2 b3 + c 2 c3 b32 + c32 

The nodal “force” vector { f (e ) } , (see equation (6.375)), can be obtained as follows:
 N1  [ x2 b1 + x3c1 + d1 ]  1
  2   2 A( e ) 
∫ ∫ ∫
(e) T
{f } = 2 [ N ] dA = 2  N 2  dA = (e)
[ x
 2 2 b + x c
3 2 + d ]
2  dA = 1 (6.387)
A A N 
2 A A [ x b + x c + d ] 
3 1
 3  2 3 3 3 3  

The tangential stresses, (see equation (6.378)), can be obtained as follows:


 ∂φ   ∂N1 ∂N 2 ∂N3  φ (e )  φ ((1e)) 
σ12
(e)
  ∂x3   ∂x3 ∂x3 ∂x3   ( e ) 
(1)
1  c1 c2 c3   (e ) 
 ( e )  =  ∂φ  =  − ∂N  φ ( 2)  = − b − b φ ( 2 )  (6.388)
σ13  −   1 − ∂N3 − ∂N3  (e )
φ  2 A
(e )
 1 − b3   ( e ) 
φ (3) 
2
 ∂x2   ∂x2 ∂x2 ∂x2   (3) 
Note that for the triangle of three-nodes the stress field into the element is constant.
Note that the integral ∫ N1dA from (6.387) is the volume formed by the shape function N1 ,
A
(see Figure 6.127).

3
1
2
∫ N dA = V = 3 A
(e)
1

N1 1 A

Figure 6.127: Shape function N1 .

University of Castilla-La Mancha Draft By: Eduardo W. V. Chaves (2016)


Ciudad Real – Spain
6 LINEAR ELASTICITY 751

Problem 6.58
Considering the torsion problem in the squared cross section ( 2a × 2a ). Obtain the
solution for this problem by considering the discretization by using 4 finite elements, (see
Figure 6.128).

x3 Element Area Connectivity


1 a2 1− 2 − 5
4 3
2 a2 2 −3−5
3 3 a2 3− 4−5
4 a2 4 −1− 5

2a 4 2
5

1 x2
2

2a

Figure 6.128: Squared cross section.


Solution:
In order to construct the global stiffness matrix and the global nodal force vector we will
need to define for each finite element the stiffness matrix and the nodal force vector.
Element 1: Connectivity: 1 (local 1)-2(local 2)-5 (local 3),
b1 = ( x3( 2 ) − x3(3) ) = 0 ; c1 = ( x2(3) − x2( 2 ) ) = −a
b2 = ( x3(3) − x3(1) ) = a ; c2 = ( x2(1) − x2(3) ) = −a
b3 = ( x3(1) − x3( 2 ) ) = 0 ; c3 = ( x2( 2 ) − x2(1) ) = 2a
By using the equation in (6.386) we can obtain:
1 2 5 Global
 a2 a2 − 2a 2  1 1 − 2 1
1  2 1
[k (1) ] = 2  a 2 2a 2
− 2a  =  1 2 − 2 2 (6.389)
4a  4
 − 2a
2
− 2a 2 4a 2  − 2 − 2 4  5
Nodal “force” vector
Global
 f1(1)  1 1 1
(1)   2 A(1)   2a
2

{f }=  f 2(1)  = 1
 = 1 2 (6.390)
 3 3
 f 3(1)  1

1
 5

University of Castilla-La Mancha Draft By: Eduardo W. V. Chaves (2016)


Ciudad Real - Spain
752 SOLVING PROBLEMS BY MEANS OF CONTINUUM MECHANICS

Element 2: Connectivity: 2 (local 1)-3(local 2)-5 (local 3),


b1 = ( x3( 2) − x3(3) ) = a ; c1 = ( x2(3) − x2( 2 ) ) = −a
b2 = ( x3(3) − x3(1) ) = a ; c2 = ( x2(1) − x2(3) ) = a
b3 = ( x3(1) − x3( 2) ) = −2a ; c3 = ( x2( 2 ) − x2(1) ) = 0
By using equation (6.386) we can obtain:
2 3 5 Global
 2a 2
0 − 2a  2
 2 0 − 2 2
1  2 1
[k ( 2)
]= 2  0 2a 2
− 2a  =  0 2 − 2 3 (6.391)
4a  4
 − 2a
2
− 2a 2 4a 2   − 2 − 2 4  5

Nodal “force” vector


Global
 f1( 2 )  1 1 2
  2 A( 2 )   2a 2  
{ f ( 2)} =  f 2( 2 )  = 1 = 1 3 (6.392)
 3  3 
 f3( 2 )  1 1 5

Element 3: Connectivity: 3 (local 1)-4(local 2)-5 (local 3),


b1 = ( x3( 2) − x3(3) ) = a ; c1 = ( x2(3) − x2( 2 ) ) = a
b2 = ( x3(3) − x3(1) ) = −a ; c2 = ( x2(1) − x2(3) ) = a
b3 = ( x3(1) − x3( 2) ) = 0 ; c3 = ( x2( 2) − x2(1) ) = −2a
By using equation (6.386) we can obtain:
3 4 5 Global
 2a 2 − a2 − 2a 2   2 − 1 − 2 3
1  2 1
[ k ( 3) ] = 2  − a 2 2a 2 − 2a  =  − 1 2 − 2 4 (6.393)
4a  4
 − 2a
2
− 2a 2 4a 2   − 2 − 2 4  5

Nodal “force” vector


Global
 f1(3)  1 1 3
( 3)   2 A(3)   2a 2  
{f }=  f 2(3)  = 1 = 1 4 (6.394)
 3  3 
 f 3(3)  1 1 5

Element 4: Connectivity: 4 (local 1)-1(local 2)-5 (local 3),


b1 = ( x3( 2 ) − x3(3) ) = −a ; c1 = ( x2(3) − x2( 2) ) = a
b2 = ( x3(3) − x3(1) ) = −a ; c2 = ( x2(1) − x2(3) ) = −a
b3 = ( x3(1) − x3( 2 ) ) = 2a ; c3 = ( x2( 2 ) − x2(1) ) = 0

University of Castilla-La Mancha Draft By: Eduardo W. V. Chaves (2016)


Ciudad Real – Spain
6 LINEAR ELASTICITY 753

By using equation (6.386) we can obtain:


4 1 5 Global
 2a 2 0 − 2a  2
 2 0 − 2 4
1  2 1
[k ( 4) ] = 2  0 2a 2 − 2a  =  0 2 − 2 1 (6.395)
4a  4
 − 2a
2
− 2a 2 4a 2   − 2 − 2 4  5

Nodal “force” vector


Global
 f1( 4 )  1 1 4
( 4)  ( 4 )  2 A( 4 )   2a 2  
{ f } =  f2  = 1 = 1 1 (6.396)
 f (4)  3  3 
 3  1 1 5
Then, the global stiffness matrix and global nodal force vector can be obtained by:
N elem N elem

A [k ( e) ] A[ f
(e)
[K ] = ; {F } = ] (6.397)
e=1 e =1

where Astands for assemble operator. Making the contribution to the respective degree-
of-freedom we can obtain:
Global Stiffness Matrix:
(k11(1) + k22 ( 4)
) k12(1) 0 ( 4)
k21 (k13(1) + k 23
(4)
) 
 (1) (1) ( 2) (2) (1) (1) 
 k 21 (k22 + k11 ) k12 0 (k 23 + k13 ) 
[K ] =  0 (2)
k 21 ( 2)
(k22 + k11(3) ) k12(3) (2)
(k 23 + k13(3) )  (6.398)
 ( 4) ( 3) ( 3) (4) ( 3) ( 4)

 k12 0 k 21 (k22 + k11 ) (k 23 + k13 ) 
(k (1) + k ( 4) ) (k (1) + k (1) ) k ( 2 ) + k (3) (k (3) + k ( 4 ) ) (k (1) + k ( 2 ) + k (3) + k ( 4) )
 31 32 23 13 32 31 32 31 33 33 33 33 

Note that the matrix [K ] is symmetric and that det[ K ] = 0 .


Global nodal force vector:
 f1(1) + f 2( 4 )  2 
  2 
 f 2(1) + f1( 2 )  2a 2  
 
{F } =  f 2( 2 ) + f1(3) = 2  (6.399)
  3 2 
 f 2(3) + f1 (4)
  
 f 3(1) + f 3( 2) + f 3(3) + f 3( 4 )  4

With that the system can be constructed:


~
[ K ]{φ } = {F } (6.400)
~
The boundary condition is applied on the boundary in which φ boundary = 0 . Then, for the
~ ~ ~ ~
problem presented here φ 1 = φ 2 = φ 3 = φ 4 = 0 , then the system to be solved is:
~ ~ 
1 0 0 0 0 φ 1  0  φ 0
0 1 0 0 0  ~  0  ~ 1  0
~   φ 2 2  
~  = 2a   solve
φ 2  2  
 ~  2a  
[ K ]{φ } = {F } ⇒ 0 0 1 0 0 φ 3 0  → φ 3  = 0 (6.401)
  ~  3    ~  3  
0 0 0 1 0 φ 4  0
  φ 4
0
 
  ~
0 0 0 0 4 φ 5   
~
4   φ 5  1

University of Castilla-La Mancha Draft By: Eduardo W. V. Chaves (2016)


Ciudad Real - Spain
754 SOLVING PROBLEMS BY MEANS OF CONTINUUM MECHANICS

~ 2a 2
where we have considered that (1)
k33 ( 2)
+ k33 ( 3)
+ k33 ( 4)
+ k33 = 4 . Then φ5 = and
3
~ 2a 2
φ 5 = Gθφ 5 = Gθ .
3
The moment of torsion is equal to two times the membrane volume:
Nelem = 4 N nodes =3
φ (je)   2a 2 a 2   16
M T = 2Vmemb = 2 ∑ ∑ 3
A(e ) = 2 4 Gθ   = Gθa 4 (6.402)
e =1 j =1   3 3   9

and
16
Gθa 4
MT 16 (6.403)
JTeff = = 9 = a 4 ≈ 1.77778a 4
Gθ Gθ 9
and if we compare with the exact solution JTeff = 2.2496a 4 we can see that the error is
approximately 21% . To improve the result we have to discretize the domain by using more
finite elements.
Note that for this case the element slope is constant so is the tangential stress, (see
equation(6.388)):
 ∂N1 ∂N 2 ∂N 3  φ ( e )  φ 1( e ) 
σ12
(e)
  ∂x3 ∂x3 ∂x3   ( e ) 
1
1  c1 c2 c3   ( e ) 
 (e )  =  − ∂N  φ 2  = (e)  φ 2  (6.404)
σ13   1 − ∂N 2 − ∂N 3  ( e )
φ  2 A  − b1 − b2 − b3   ( e ) 
 ∂x2 ∂x2 ∂x2   3  φ 3 
For the element 1 we have:
b1 = ( x3( 2 ) − x3(3) ) = 0 ; c1 = ( x2(3) − x2( 2 ) ) = −a
b2 = ( x3(3) − x3(1) ) = a ; c2 = ( x2(1) − x2(3) ) = −a
b3 = ( x3(1) − x3( 2 ) ) = 0 ; c3 = ( x2( 2 ) − x2(1) ) = 2a

and φ 1(1) = φ 1 , φ (21) = φ 2 , φ 3(1) = φ 5 , then


 
φ 1   0 
σ12
(1)
 1  c1 c2 c3    1  − a − a 2a    2 1
 (1)  = − b  φ 2  = 2    0 2  = Gθa   (6.405)
σ13  2a 2  1 − b2 − b3    2a  0 − a 0   2a  3 0
φ 5  Gθ
3 
For the element 2 we have:
b1 = ( x3( 2) − x3(3) ) = a ; c1 = ( x2(3) − x2( 2 ) ) = − a
b2 = ( x3(3) − x3(1) ) = a ; c2 = ( x2(1) − x2(3) ) = a
b3 = ( x3(1) − x3( 2) ) = −2a ; c3 = ( x2( 2 ) − x2(1) ) = 0

and φ 1( 2) = φ 2 , φ (22) = φ 3 , φ 3( 2) = φ 5 , then


 
φ 2   0 
σ12
(2)
 1  c1 c2 c3    1 − a a 0   2 0
 (2)  = − b  φ 3  = 2    0 2  = Gθa   (6.406)
σ13  2a 2  1 − b2 − b3    2a − a − a 2a   2a  3 1
φ 5  Gθ
3 

University of Castilla-La Mancha Draft By: Eduardo W. V. Chaves (2016)


Ciudad Real – Spain
6 LINEAR ELASTICITY 755

For the element 3 we have:


b1 = ( x3( 2) − x3(3) ) = a ; c1 = ( x2(3) − x2( 2 ) ) = a
b2 = ( x3(3) − x3(1) ) = −a ; c2 = ( x2(1) − x2(3) ) = a
b3 = ( x3(1) − x3( 2) ) = 0 ; c3 = ( x2( 2) − x2(1) ) = −2a

and φ 1(3) = φ 3 , φ (23) = φ 4 , φ 3(3) = φ 5 , then


 
φ 3   0 
σ12
( 3)
 1  c1 c2 c3    1  a a − 2a    −2 1
 ( 3)  = − b  φ 4  = 2    0 2= Gθa   (6.407)
σ13  2a 2  1 − b2 − b3    2a  − a a 0   2a  3 0
φ 5  Gθ
3 
For the element 4 we have:
b1 = ( x3( 2 ) − x3(3) ) = − a ; c1 = ( x2(3) − x2( 2) ) = a
b2 = ( x3(3) − x3(1) ) = − a ; c2 = ( x2(1) − x2(3) ) = − a
b3 = ( x3(1) − x3( 2 ) ) = 2a ; c3 = ( x2( 2 ) − x2(1) ) = 0

and φ 1( 4) = φ 4 , φ (24) = φ 1 , φ 3( 4) = φ 5 , then


 
φ 4   0 
σ12
(4)
 1  c1 c2 c3    1 a − a 0   −2 0
 (4)  =   φ 1  = 2    0 2= Gθa   (6.408)
σ13  2a 2  − b1 − b2 − b3    2a a a − 2a   2a  3 1
φ 5  Gθ 3 

In Figure 6.129 we can appreciate the tangential stress distributions, which is a very poor
solution. We will need more elements to achieve a better stress distribution.

x3

4 3

3
( 3)
4 σ12

2a ( 4)
σ13 (2)
σ13
5

(1)
2
σ12
1
1 x2
2

2a

Figure 6.129: Tangential stress distribution.

University of Castilla-La Mancha Draft By: Eduardo W. V. Chaves (2016)


Ciudad Real - Spain
756 SOLVING PROBLEMS BY MEANS OF CONTINUUM MECHANICS

The membrane deflection can be appreciated in Figure 6.130. Note that for this case the
membrane deflection is a pyramid. Recall that the pyramid volume is:
1 1 2a 2 8 M
V pyr = ( Abase × hapex ) = 4a 2Gθ = Gθa 4 = T
3 3 3 9 2

2a 2
φ 5 = Gθ = hapex
3

3
x3 4
2
x2
5

1
(4) − ∂φ − 2
σ13 = = Gθa
∂x2 3
(1) ∂φ 2 hapex
σ12 = = Gθa =
∂x3 3 a

Figure 6.130: Membrane deflection for Problem 6.58.

In Figure 6.131 we can appreciate another example for the membrane deflection and in
Figure 6.132 the correspondent tangential stress.

a) Cross section b) Membrane deflection


Figure 6.131: Membrane deflection.

University of Castilla-La Mancha Draft By: Eduardo W. V. Chaves (2016)


Ciudad Real – Spain
6 LINEAR ELASTICITY 757

Figure 6.132: Tangential stress distribution.

Torsion References
LAIER, J.E. & BAREIRO, J.C., (1983). Complemento de resistência dos materiais. Publicação 073/92
São Carlos - USP - EESC.
SECHLER, E. (1952). Elasticity in Engineering. John Willey & Sons, Inc. New York.
SOKOLNIKOFF, I.S. (1956). Mathematic theory of elasticity. New York, McGraw-Hill, (1st
Edition: 1946).
UGURAL, A.C. & FENSTER, S.K. (1984). Advanced strength and applied elasticity. Edward Arnold,
London - U.K. (1st Edition: 1981).
CERVERA RUIZ, M. & BLANCO DÍAZ, E. (2001). Mecánica de Estructuras Libro 1 – Resistencia de
materiales. Edicions UPC, Barcelona. Eapaña.

University of Castilla-La Mancha Draft By: Eduardo W. V. Chaves (2016)


Ciudad Real - Spain
758 SOLVING PROBLEMS BY MEANS OF CONTINUUM MECHANICS

6.5 Introduction to One-Dimensional Elements (1D)

Problem 6.59
Consider the bar element which presents one dimension greater than the other two. Obtain
the internal forces in the cross-sectional area of the bar. The coordinate system is located at
the Area Centroid and is principal axis of inertia, (see Figure 6.133), and use engineering
notation.

Strain Stress
z diagram diagram
z ε x (z ) σ x (z )
y

y
neutral axis

a) Rod b) Cross-section

Figure 6.133: The bar.

Hypothesis:
• Small deformation regime and small rotation;
• Homogeneous, elastic, linear and isotropic material;
• To obtain the internal forces due to the stress component σ x consider that any cross-
sectional area defined by a plane remains plane after deformation.
Note that this problem was already discussed in Problem 4.22. Here we will present the
solution from another point of view.

University of Castilla-La Mancha Draft By: Eduardo W. V. Chaves (2016)


Ciudad Real – Spain
6 LINEAR ELASTICITY 759

Solution:
The internal forces are obtained by integrate over the cross-sectional area of the bar. Then,
in a generic case, on the face of the cross section (according to the system adopted) we can
appear the stresses σ x , τ xy and τ xz .
If we make a cut in the bar according to the plane defined by Π , the stress state at an
arbitrary point is the one as indicated in Figure 6.134.

z
τ xz
y
τ xy
Π
σx

A - Cross-sectional area
Π

Figure 6.134: Stress field on the cross section.

The next step is to establish how the stress field varies on the cross section.
As the material is elastic and linear, the stress varies linearly with deformation ( σ x = Eε x ).
∂u
In addition, as we are dealing with the small deformation regime the relationship ε x =
∂x
holds. Then, if the displacement field u ( y, z ) on the cross-sectional area defines a plane so
the strain and stress do.

a) We can take the following possibilities:


z u (1)
The cross-sectional area displaces according x -
direction. In this case the strain is constant on y
the cross section, as consequence the normal
stress field on cross section is also constant. By
integrate the normal stress over the area of the
x
cross section we obtain the internal force N
(the axial force) which could be positive (tensile)
or negative (compression).
Figure 6.135

University of Castilla-La Mancha Draft By: Eduardo W. V. Chaves (2016)


Ciudad Real - Spain
760 SOLVING PROBLEMS BY MEANS OF CONTINUUM MECHANICS

b) Another possibility for the displacement field on


the cross section is when the cross section rotates
about y -axis. In this case the displacement field is the
z one showed in Figure 6.136. The strain and normal
u (2) stress also vary according to a plane on the cross
y section. Note that, if we integrate the normal stress
over the area we obtain zero as result, i.e. the resultant
force is zero, but there is moment according to the y -
x direction, so, the bar is subjected to pure bending. We
denote by M y the bending moment according to the
y -direction. The displacement u ( 2) can be caused
− u ( 2) when the bar is subjected to a deflection according
to z -direction (displacement w( x) ).
Figure 6.136

c) Another possibility is when the cross − u ( 3)


section rotates about the z -axis, (see Figure z
6.137). In this case the resultant force equals y
zero, and there is a bending moment
according the z -direction which is denoted
by M z . The displacement u (3) can be caused
x
when the bar is subjected to a deflection
according to the y -direction (displacement
v( x) ). u (3)

Figure 6.137

The combination of the previous cases is also possible. In general, the normal stress field
σ x on the cross section is illustrated in Figure 6.138.
If we consider Figure 6.139, we can also express the bending moment M y as follows:

σS z σ σS

M y = σ (x2 ) zdA = ∫ ∫z
2
zdA = S dA = Iy (6.409)
A A
c c A
c

where I y = ∫ z 2 dA is the inertia moment of area about the y -direction. Taking into
A

σ S σ (x2 )
account that = we can also obtain:
c z
My
σ (x2 ) ( z ) = z (6.410)
Iy
Similarly, we can obtain:
−Mz
σ (x3) ( y ) = y (6.411)
Iz
Taking into account that σ x = Eε x , the above equations can be rewritten as follows:

University of Castilla-La Mancha Draft By: Eduardo W. V. Chaves (2016)


Ciudad Real – Spain
6 LINEAR ELASTICITY 761

My −Mz
ε (x2 ) ( z ) = z ; ε (x3) ( y ) = y (6.412)
EI y EI z

where EI xi is the modulus of flexural rigidity about xi of the bar.

z y N My M
σ x ( y, z ) = + z− z y
A Iy Iz

σ x ( y, z )

144444444444444444424444444444444444443

z z z
σ (x1) σ (x2 ) σ (x3)
y y y

+ x
+ x
x


N = σ (x1) dA
A

M y = zσ (x2 ) dA
A

M z = yσ(x3) dA
A
⇓ ⇓ ⇓

N
My Mz
144424443 144424443

∫σ
( 3)
=0
∫σ x dA
( 2)
x dA = 0
A A

Figure 6.138: The axial force and bending moments.

σS

σ x (z )
c z
b
neutral axis

Figure 6.139: Normal stress distribution on the cross section.

University of Castilla-La Mancha Draft By: Eduardo W. V. Chaves (2016)


Ciudad Real - Spain
762 SOLVING PROBLEMS BY MEANS OF CONTINUUM MECHANICS

Shearing forces and moment of torsion, (see Problem 4.22):


By integrate the tangential stresses (shearing stresses) τ xy and τ xz over the cross-sectional
area we obtain the shearing forces Q y and Q z , respectively, (see Figure 6.140):


Q y = τ xy dA
A
; ∫
Q z = τ xz dA
A
(6.413)

and the moment of torsion (torque) ( M T ):

MT = ∫ (τ
A
xz y )
− τ xy z dA (6.414)

z τ xz ( y , z ) τ xy ( y, z )
z
y y

x x

∫τ
A
xz dA ∫τ
A
xy dA

⇓ ⇓

Qz Qy

Figure 6.140: Tangential stresses – Shearing forces.

We summarize in Figure 6.141 the internal forces ( N , Q y , Qz ) and internal moments


( M x , M y , M z ) that could appear on the cross section of the bar.

z, w y, v Mz
My
Qy
Qz M x ≡ MT
N x, u

Figure 6.141: Internal forces and internal moments on the cross section of the bar.

University of Castilla-La Mancha Draft By: Eduardo W. V. Chaves (2016)


Ciudad Real – Spain
6 LINEAR ELASTICITY 763

NOTE 1: The internal forces and internal moments depend on the external actions (loads)
in which the bar is subjected, (see Figure 6.142).

1) Load 1) Internal force and moment


z, w N
[qz ] =
y, v m
qz
My
1 x, u Qz

2) Load
N 2) Internal force and moment
[qz ] =
z, w y, v m
qy Mz

2 Qy
x, u

3) Load
N 3) Internal force
[qx ] =
z, w m
y, v
qx

3 Qx ≡ N

x, u

4) Load
4) Internal moment
z, w y, v
 Nm 
mT  
 m 
4 M x ≡ MT

x, u

Figure 6.142: Some cases of external loads.

University of Castilla-La Mancha Draft By: Eduardo W. V. Chaves (2016)


Ciudad Real - Spain
764 SOLVING PROBLEMS BY MEANS OF CONTINUUM MECHANICS

NOTE 2: Warping of the Cross Section


In Problem 6.44 we have shown that only when we are dealing with a circular cross
section the section remains planar after the torsion is applied.
The warping of the cross section appears due to the non-homogeneous tangential stress
field on the cross section, (see Figure 6.144(a)). In circular section there is no warping since
the tangential stress varies linearly according to the radius as show in Figure 6.144(b).

τ xz

τ xy
y

Figure 6.143: Tangential stresses (shearing stresses) – Moment of torsion.

τ( y, z )
τ max
τ( r )
r

a) Rectangular cross section b) Circular cross section


(Warping phenomenon) (No warping phenomenon)
Figure 6.144: Distribution of the tangential stress.

NOTE 3: Deflection of the Bar


Initially, let us consider the defection only according to the z -direction (displacement
w( x) ), (see Figure 6.145). By means of Figure 6.145 we can conclude that:

du ( 2 ) d 2w
u ( 2 ) = − w, x z ⇒ ε (x2 ) ( z ) = = − 2 z ≡ − w, xx z
dx dx (6.415)
σ (x2 ) ( z ) = Eε (x2 ) = − Ew, xx z
Note that, if we compare the equations (6.416) and (6.415) they have the reversed sign
since the bending moment M z (same direction and sense as z -axis) produces the
displacement field contrary as the one presented in Figure 6.145.
Then, if we consider the defection of the bar according to the y -direction (displacement
v( x) ), (see Figure 6.146), we can obtain:

University of Castilla-La Mancha Draft By: Eduardo W. V. Chaves (2016)


Ciudad Real – Spain
6 LINEAR ELASTICITY 765

du (3) d 2v
u ( 3 ) = − v, x y ⇒ ε (x3) ( y ) = = − 2 y ≡ −v, xx y
dx dx (6.416)
σ (x3) ( y ) = Eε (x3) = − Ev, xx y

θ y = − w, x
θy , M y > 0
( 2)
u = θ y z = − w, x z

small angles: tan θ ≈ θ


(after deformation)
n
c
dw
≡ w, x
dx
w(x) m

neutral axis - w(x)


w, x < 0; w, xx < 0 θ y = − w, x
z, w
u ( 2) = − w, x z
n
c z
b
x, u
m
(before deformation)

Figure 6.145: Displacement on the transversal cross section due to M y .

∂v b) after deformation
u ( 3) = − y
∂x
y, v
m′ θz , M z > 0
∂v
p′ ∂x
.
A’
n′
θ z = v, x
v(x) ∂v
v, x > 0; v, xx > 0 ≡ v, x = θz u ( 3) = −v , x y
∂x
m
y a/2
.
A x, u
a/2
n
a) before deformation

Figure 6.146: Displacement on the transversal cross section due to M z .

University of Castilla-La Mancha Draft By: Eduardo W. V. Chaves (2016)


Ciudad Real - Spain
766 SOLVING PROBLEMS BY MEANS OF CONTINUUM MECHANICS

If we compare the equations (6.410) and (6.415) we can conclude that:


My 
σ(x2 ) ( z ) = z −My
Iy  ⇒ w, xx = (6.417)
(2)  EI y
σ x ( z ) = − Ew, xx z 
Similarly, if we compare the equations (6.411) and (6.416) we can conclude:
−Mz 
σ (x3) ( y ) = y Mz
Iz  ⇒ v, xx = (6.418)
EI z
σ x ( y ) = − Ev, xx y 
( 3)

Note that, to obtain the equations (6.417) and (6.418) we have already considered the
kinematic equations and the constitutive equations. To complete the governing equations of the
IBVP we have introduce the equilibrium equations. As in the cross section of the bar we have
lost the information of the symmetry of the Cauchy stress tensor we have to apply the
Principle of Linear Momentum (equilibrium equations or Summation of forces equal zero)
and the Principle of the Angular Momentum (summation of the moments equal zero). In
other words, we have to apply the summation of forces and moments equal zero in the
differential element of bar dx , (see Problem 6.60).
NOTE 3.1: As additional information, let us consider now that the beam element is lying
on the y -axis as indicated in Figure 6.147. Then, we can conclude that:
dv d 2w
v = − w, y z ⇒ ε y (z) = = − 2 z ≡ − w, yy z
dy dy
(6.419)
σ y ( z ) = Eε y = − Ew, yy z

∂w b) after deformation
v=− z
∂y
z, w
m′ θx , M x > 0
∂w
p′ ∂y
.
A’
n′
θx = w, y
w( y ) ∂w
w, y > 0; w, yy > 0 ≡ w, y = θ x v = − w, y z
∂y
m
z a/2
.
A y, v
a/2
n
a) before deformation

Figure 6.147: Displacement on the transversal cross section ( m - n ) due to M y .

University of Castilla-La Mancha Draft By: Eduardo W. V. Chaves (2016)


Ciudad Real – Spain
6 LINEAR ELASTICITY 767

NOTE 4: Tangential stress on the cross section of the bar


Let us suppose that we have several layers of smooth plates as indicated in Figure 6.148 (a).
As the plates can displace freely between layers after the load is applied, (see Figure 6.148
(b)), there is no tangential stress between layers. If all plates are united to form a single
monolithic piece (Figure 6.148 (c)), the displacement between layers is limited, and so the
tangential stress appears. This tangential stress is what cause the appearance of the
tangential stress on the cross section, (see Figure 6.148 (d)). Recall from Problem 4.22-
NOTE 4 that the shearing force is caused by the variation of bending moment.

a)

transversal cross section

b)

transversal cross section

τ
c) τ

transversal cross section

z
d) y
b
d τ
b
c a
d τ
x

c a

Figure 6.148: Beam subjected to bending.

University of Castilla-La Mancha Draft By: Eduardo W. V. Chaves (2016)


Ciudad Real - Spain
768 SOLVING PROBLEMS BY MEANS OF CONTINUUM MECHANICS

Problem 6.60
Obtain the governing equation for the beam in which the flexural rigidity ( EI y ) is
constant. The beam is subjected to an uniformly distributed load per unit length (q z ) , (see
Figure 6.149).
z, w
y, v N  Internal forces
qz  
m My

x, u Qz

Load dx

Figure 6.149: Beam subjected to uniformly distributed load.


Solution:
Let us consider the differential beam element dx in which the internal forces are indicated
in Figure 6.150.

y, v dx
2
z, w

qz
M y( − ) ∂M y
Equilibrium My + dx
∂x
A− x, u Point A A+ B

Qz( − ) My =0 Qz( + ) ∂Qz


M y( + ) Qz + dx
Qz = 0 ∂x
dx

Figure 6.150: Differential beam element.

By applying the equilibrium of force and moment (at point B ) in the differential beam
element we can obtain:
 ∂Qz  ∂Qz
∑F z =0 ⇒ − Qz +  Qz +
 ∂x
dx  + qz dx = 0


∂x
= −q z (6.420)

 ∂M y  dx ∂M y
∑M yB = 0 ⇒ − M y − Qz dx +  M y +
 ∂x
dx  + qz dx
 2
=0 ⇒
∂x
= Qz (6.421)

where we have considered that dxdx ≈ 0 .


To complete the governing equation we have to introduce the kinematic and constitutive
equations. Note that the equation in (6.417) is already considering these equations, then
My
w, xx = − ⇒ M y = − EI y w, xx
EI y

University of Castilla-La Mancha Draft By: Eduardo W. V. Chaves (2016)


Ciudad Real – Spain
6 LINEAR ELASTICITY 769

∂M y ∂ ∂3w

∂x
= Qz = −
∂x
( )
EI y w, xx = − EI y w, xxx ≡ − EI y 3
∂x
(6.422)
∂2M y ∂Qz ∂2 ∂ 4w
⇒ =
∂x
(
= − 2 EI y w, xx = − EI y 4 = −q z )
∂x 2 ∂x ∂x
with that the beam differential equation can be represented by:

∂Qz ∂M y
= −qz ; = Qz
∂x ∂x
(6.423)
∂2w − M y ∂ 3 w − Qz ∂ 4 w qz
= or = or =
∂x 2 EI y ∂x 3 EI y ∂x 4 EI y

NOTE 1:
Now, if we consider the beam presented in Figure 6.151 and by considering the equilibrium
we can obtain:
 ∂Q y  ∂Q y
∑F y =0 ⇒ − Q y +  Q y +
 ∂x
dx  + q y dx = 0


∂x
= −q y

 ∂M z  dx ∂M z
∑M zB = 0 ⇒ − M z + Q y dx +  M z +
 ∂x
dx  − q y dx
 2
=0 ⇒
∂x
= −Q y

and

∂ 2v M z ∂ 3v Qy ∂ 4v − q y
= or = or = (6.424)
∂x 2 EI z ∂x 3 EI z ∂x 4 EI z

a)
N Internal force and moment
Load [q z ] =
z, w y, v m
Mz
qy
Qy

x, u

dx
2
qy
b)
∂Q y
Q y( + ) A+ B Qy + dx
∂x
M z( + ) ∂M z
Mz + dx
∂x
dx

Figure 6.151: Differential beam element.

University of Castilla-La Mancha Draft By: Eduardo W. V. Chaves (2016)


Ciudad Real - Spain
770 SOLVING PROBLEMS BY MEANS OF CONTINUUM MECHANICS

NOTE 2: Tangential stress on the cross section

z z

a)
qz
A τ xz (z )
A B
b
2 z z
b
x y

a transversal cross section

QzA ∂M y
qz My + dx
∂x
My ∂Qz
Qz + dx
∂x
b)
dx
∂σ x
σ x (z ) σx + dx
A B ∂x


F2 = σ x ( z )dA

 ∂σ 
F1 =  σ x + x dx dA
A
A
∂x 

z
τ
F3 = (∫ τ( z, y)dy )dx

neutral axis

Figure 6.152:

Applying the force equilibrium according the x -direction, (see Figure 6.152 (b)), we can
obtain:
 ∂σ 
∑F x = 0 ⇒ F1 − F2 − F3 = 0 ⇒ ∫ ∂x  ∫ ∫
 σ x + x dx  dA − σ x dA − dx τ( z, y )dy = 0
A  A
(6.425)
 ∂σ ( z ) 

⇒ dx τ( z , y )dy =  x
A
∂x ∫
dx  dA

If we invoke the equations (6.410) and (6.421) we can obtain:
My ∂σ (x2) ( z ) ∂  M y  ∂M y z Q z
σ (x2 ) ( z ) = z ⇒ = z = = z
Iy ∂x 
∂x  I y  ∂x I y Iy

University of Castilla-La Mancha Draft By: Eduardo W. V. Chaves (2016)


Ciudad Real – Spain
6 LINEAR ELASTICITY 771

where we are considering that I y is constant along the beam. Taking into account the
above equation into the equation in (6.425) we can obtain:
 ∂σ ( z )  Q z  Q
∫ ∫
dx τ( z , y )dy =  x
∂x ∫  Iy  Iy A ∫
dx  dA ⇒ ⇒ dx τ( z , y )dy =  z dx dA = z dx zdA ∫
A  A 
Q
∫ ∫
⇒ τ( z , y )dy = z zdA
Iy A

with that we can conclude that:


Qz Qz χ z
∫ τ( z, y)dy = I ∫ zdA =
y A Iy
where χ z = zdA ∫
A
(6.426)

The above equation was also obtained in Problem 4.29. For the cross section a × b and if
we considering as a good approximation that


τ ave ( z )a = τ( z, y )dy

where τ ave (z ) is the average of the tangential stress at (z ) .

A τ( z, y )

Figure 6.153: Tangential stress on the cross section.


Then, we can obtain:
Qz χ z Qz χ z
τ ave ( z ) = ⇒ τ ave ( z ) =
Iy I ya
References of Mechanics of Materials
BEER, F.P.; JOHNSTON (JR.), E.R.; DEWOLF, J.T. & MAZUREK, D.F. (2012). Mechanics of
Materials. 6th Edition. McGraw-Hill, New York-USA.

Problem 6.61
Show:
a) Mohr’s First Theorem
“The change in slope of a defection curve between two points is equal to the area diagram
My
of between these two points.”
EI y
b) Mohr’s Second Theorem

University of Castilla-La Mancha Draft By: Eduardo W. V. Chaves (2016)


Ciudad Real - Spain
772 SOLVING PROBLEMS BY MEANS OF CONTINUUM MECHANICS

Let us consider a beam subjected to bending between two point, namely A and B . Let ∆d
be the distance between the point B and point D , where D is the point intercepted by the
tangent line at the point A and the vertical line through the point B , (see Figure 6.154).
The Mohr’s second theorem states:
My
“The distance ∆d is equal to the first moment of the diagram about the axis where
EI y
∆d is measured.”
a) Mohr’s first theorem
Solution:
We have seen at the end of Chapter 1 (Complementary Note 1 - curvature) that the
following relationships are true:
dψ dψ dx w, xx 1 w, xx 1
κ= = = = =
ds dx ds [1 + ( w, x ) 2 ] 1 3
r (6.427)
2 2 2 2
[1 + ( w, x ) ] [1 + ( w, x ) ]
B

∫ ∫
κds = dψ = ψ B − ψ A ≡ ∆ψ B _ A
A
(6.428)

∂w ∂2w
where w, x ≡ , w, xx ≡ 2 , in which w stands for the deflection, κ is the curvature, and
∂x ∂x
ds is the differential arc-length element.
For small curvature it fulfils:
1 ∂2w dψ dψ
κ= ≈ w′′ ≡ 2 ≡ w, xx ; ds ≈ dx ; tan ψ ≈ ψ ; cosψ ≈ 1 ; ≈
r ∂x ds dx
Taking into account the equation (6.417) we can conclude that:
−My −My
w, xx = ⇒ κ = κy =
EI y EI y
and if we apply the equation in (6.428) we can obtain:
B B B B B
My My

A

κds = dψ = ∆ψ B _ A
A
⇒ − ∫ EI
A y
ds ≈ − ∫ EI
A y

dx = dψ = ∆ψ B _ A
A
(6.429)

My My
where the expression ∫ EI y
ds is the area of the diagram defined by
EI y
, (see Figure

6.154).

University of Castilla-La Mancha Draft By: Eduardo W. V. Chaves (2016)


Ciudad Real – Spain
6 LINEAR ELASTICITY 773

∆ψ = ψ B − ψ A ∆d


A′ ds

B′
wA r wB ψB
ψA
B x
A

 My 
  My
 EI y


A A= ∫ EI y
dx  My

 EI y



 B
x
x

∆x

Figure 6.154
b) Mohr’s second theorem
By multiply the equation (6.427) by x we can obtain:
B B B B
dψ My
κx =
ds
 g → κxds = xdψ
x integratin
A A
∫ ∫ ⇒ ∫
A

EI y ∫
xdx = xdψ = ( xψ ) B − ( xψ ) A ≈ ∆d
A

 My 
B B
⇒ − ∫ dx  x = xdψ = ( xψ ) B − ( xψ ) A ≈ ∆d

 EI y 
A  A

B
My My
where ∫ EI
A y
xdx is the first moment of area of the diagram
EI y
.

University of Castilla-La Mancha Draft By: Eduardo W. V. Chaves (2016)


Ciudad Real - Spain
774 SOLVING PROBLEMS BY MEANS OF CONTINUUM MECHANICS

6.5.1 The Potential Energy for 1D Structural Element

Problem 6.62
Obtain the internal potential energy for the bar element of length L in function of forces
and moments, (see Figure 6.155).

z, w y, v Mz
My
Qy
Qz M x ≡ MT
N x, u

Figure 6.155: Forces and moments in the bar.


Solution:
The internal potential energy is given by:
1 1
U int =
2V∫σ : εdV =
2V ∫
σ ij ε ij dV

1
=
2V∫(σ11 ε 11 + σ 22 ε 22 + σ 33 ε 33 + 2σ12 ε 12 + 2σ 23 ε 23 + 2σ13 ε 13 )dV

1
=
2V∫(σ x ε x + σ y ε y + σ z ε z + τ xy γ xy + τ yz γ yz + τ xz γ xz )dV

On the transversal cross section of the bar, the normal stress σ x can be represented by
σ x = σ (x1) + σ (x2) + σ (x3) . The internal potential energy associated with the stress σ(x1) = Eε (x1) ,
(see Figure 6.138), can be expressed in function of the axial force N :
(1) 2 L L
1 1 σx 1 N2 1 N2
U int =
2V∫σ (x1) ε (x1) dV =
2V E ∫ dV =
2 ∫ EA ∫
0
2
A
dAdx =
2 ∫
0
EA
dx (6.430)

Similarly, we can obtain the internal potential energy associated with the normal stress
σ(x2) = Eε (x2 ) in terms of bending moment M y , (see equation (6.412)):

1 1
L
My My 1
L
M y2 1
L
M y2
∫ ∫ ∫ ∫ ∫ ∫ EI
int
U = σ (x2 ) ε (x2 ) dV = z zdAdx = 2
2
z dAdx = dx (6.431)
2V 2 0 A
Iy EI y 2 0
EI y A 2 0 y

Similarly, if we consider σ(x3) , (see equation (6.412)), we can obtain:


L
1 M z2
U int = ∫
2 0 EI z
dx (6.432)

The component τ xz ( z ) = Gγ xz is associated with the shearing force Q z , (see equation


(6.413)), where G is the shear modulus (also called transversal elastic modulus). In
equation (6.426) we have obtained that:

University of Castilla-La Mancha Draft By: Eduardo W. V. Chaves (2016)


Ciudad Real – Spain
6 LINEAR ELASTICITY 775

Qz Qz χ z
∫ τ( z, y)dy = I ∫ zdA =
y A Iy
where ∫
χ z = zdA
A
(6.433)

Then, the internal potential energy associated with τ( z, y ) = Gγ is given by:

1 1  
L L
1 1 τ 2xz 1 1 2
U int = ∫τγdV = ∫ dV = ∫ ∫
τ dAdx = τ 2 dydz dx
∫ ∫∫
2V 2V G 20GA 2 0 G 
 
(6.434)
 2
  2

1 1   Qz χ z  1 Qz2   χ z 
L L L
  1 ς z Qz2
= ∫ ∫
2 0 G   I y 
dz  dx = ∫
2 0 G   I y 
dz  ∫
dx =
2 0 GA ∫
dx
   
where ς z is the shape factor of the cross section along z -axis, which is given by:
2
χ 
ςz = A  z
 Iy

∫  dz


(6.435)

Similarly, we can obtain:


2
1
2
1 τ xy
2
1 ς yQy
L
χ 
∫ ∫ ∫ ∫
ςy = A  z  dz
int
U = τ xy γ xy dV = dV = dx where (6.436)
2V 2V G 2 0 GA  Iy 
 
If we consider a circular cross section, the tangential stress field in the cross section, (see
Problem 6.44-NOTE 1), is given:
MT
τ( r ) = r
JT
Then, the internal `potential energy due to the tangential stress τ( r ) = Gγ ( r ) can be
obtained by:
2 2 L L
1 1 τ 1 1  MT  1 M T2 2 1 M T2
∫ ∫  ∫ r  dV = ∫ ∫ ∫
int
U = τ( r ) γ (r )dV = dV = r dAdx = dx (6.437)
2V 2V G 2 V G  JT  2 0 GJT2 A 2 0 GJT

where JT = ∫ r 2 dA is the polar moment of inertia of the circular cross section. We can
A
obtain an equivalent polar moment of inertia for another shape of the cross section which
is denoted by J Eq . Thus, we can write:
L
1 M T2

int
U = dx (6.438)
2 0 GJ Eq

Then, the internal potential energy for the bar element can be represented as follows:

1  N 2 M y M z2 ς y Q y ς z Qz2 M T2 
L 2 2


int
U = + + + + + dx (6.439)
2 0  EA EI y EI z GA GA GJ Eq 

NOTE 1: The External Potential Energy


Next we will provide some equations related to the external potential energy due to some
external loads:
ƒ Concentrated force: U ext = Pw p , where P is the concentrated force and w p is the
deflection according to P -direction, (see Figure 6.156).

University of Castilla-La Mancha Draft By: Eduardo W. V. Chaves (2016)


Ciudad Real - Spain
776 SOLVING PROBLEMS BY MEANS OF CONTINUUM MECHANICS

y neutral axis deflection


z
wp U ext = Pw p

x
P

Figure 6.156: Concentrated force.


L
ƒ Distributed load: ∫
U ext = q z ( x) w( x)dx , where q z (x) - distributed load, w( x) -
0
deflection according to z -direction, (see Figure 6.157).

y neutral axis deflection


z
w( x)
L
x

ext
U = q z ( x) w( x)dx
0

q z ( x)

Figure 6.157: Uniformly distributed load.


L


ext
In the case that q z ( x) is uniformly distributed load we obtain U = q z w( x)dx .
0

ƒ Concentrated moment load: U ext = M yA θ yA , (see Figure 6.158), where θ stands for
rotation.

z M Ay
U ext = M yA θ yA
A θ yA x

Figure 6.158: Concentrated moment at point A .

NOTE 2: Example of application. Let us consider a beam element in which the nodal
forces (shear and moment) and nodal displacement (deflection and rotation) are indicated
in Figure 6.159.

Let us consider that each node there is two “displacements” (degrees-of-freedom):


dw d 2w
deflection ( w ) and rotation ( θ y = − ≡ − w′ ). We will use also the notation ≡ w′′ .
dx dx 2

University of Castilla-La Mancha Draft By: Eduardo W. V. Chaves (2016)


Ciudad Real – Spain
6 LINEAR ELASTICITY 777

y
z  w1 
θ 
(e)  y1 
{u } =  
θ y1 = − w1′ θ y 2 = − w′2  w2 
w1 w2 θy 2 
 

1 2 x

a) Nodal “displacements”

y
z
 Qz1 
M 
M y1 M y2  y1 
{ f (e) } =  
Qz1 Qz 2
 Qz 2 
M y 2 
 
1 2 x
L

b) Nodal “forces”

Figure 6.159: Beam element.

Let us adopt a cubic function to approach the deflection w :


w = ax 3 + bx 2 + cx + d (6.440)
The first derivative of w becomes:
dw
≡ w′ = 3ax 2 + 2bx + c (6.441)
dx
Then, at the ends (nodes) of the beam the following fulfills:
x = 0 ( w = w1 ) ⇒ w1 = d
dw (6.442)
x = 0 ( w′ = w1′ ) ⇒ = w1′ = c
dx
x = L ( w = w2 ) ⇒ w2 = aL3 + bL2 + cL + d
(6.443)
x = L ( w′ = w2′ ) ⇒ w2′ = 3aL2 + 2bL + c

By restructuring the above equations in matrix form we can obtain:

University of Castilla-La Mancha Draft By: Eduardo W. V. Chaves (2016)


Ciudad Real - Spain
778 SOLVING PROBLEMS BY MEANS OF CONTINUUM MECHANICS

 w1   0 0 0 1  a  a  2L L2 − 2L L2   w1 
 w′   0        
 1 =  0 1 0  b  Reverse  b  1  − 3L2 − 2 L3 3L2 − L3   w1′ 
  → = (6.444)
 w2   L3 L2 L 1   c   c  L4  0 L4 0 0   w2 
   2      4  
 w2′  3L 2 L 1 0  d  d   L 0 0 0   w2′ 
where the coefficients are:
2
a= 3
(w1 − w2 ) + 12 (w′2 + w1′ ) (6.445)
L L
3
b= 2
(w2 − w1 ) − 1 (w′2 + 2w1′ ) (6.446)
L L

c = w1′ (6.447)
d = w1 (6.448)
By substituting the values of a , b , c and d into the equation of the deflection (6.440), we
can obtain:
  x 3  x
2
   x 3 x 
2
 x3 2x 2   x3 x2 
w = w1 2  − 3  + 1 + w2 − 2  + 3   + w1′  2 − + x  + w′2  2 − 
  L  L    L   L   L L  L L
(6.449)
Recall that we have adopted as degree-of-freedom the rotation θ y which is in agreement
with the coordinate system adopted, (see Figure 6.159), and the above equation is in
function of w' (deflection derivative). But, they are related to each other by means of the
equation θ y = − w' . With that, the equation of the deflection becomes:

  x 3  x
2
   x 3 x 
2
 x 3 2x 2   x3 x2 
w = w1 2  − 3  + 1 + w2 − 2  + 3   − θ y1  2 − + x − θ y 2  2 − 
  L  L    L   L   L L  L L
(6.450)
and the first derivative becomes:
 6x2 6x   6x2 6x   3x 2 4 x   3x 2 2 x 
w′ = − θ y = w1  3 − 2  + w2  − 3 + 2  − θ y1  2 − + 1 − θ y 2  2 −  (6.451)
 L L   L L  L L  L L
The second derivative:
12 x 6   12 x 6   6x 4  6x 2 
w′′ = w1  3 − 2  + w2 − 3 + 2  − θ y1  2 −  − θ y 2  2 −  (6.452)
L L   L L  L L L L
It will be useful to obtain analytically the following integrals:
L
L L L2 L2

0
w( x)dx =
2
w1 + w2 −
2 12
θ y1 +
12
θ y2 (6.453)
L
13L 2 13L 2 L3 2 L3 2 9 L 11L2 13L2
∫ w 2 dx =
35
w1 +
35
w2 +
105
θ y1 +
105
θy2 +
35
w1 w2 −
105
w1 θ y1 +
210
w1 θ y 2
0
(6.454)
13L2 11L2 L3
− w2 θ y1 + w2 θ y 2 − θ y1 θ y 2
210 105 70

University of Castilla-La Mancha Draft By: Eduardo W. V. Chaves (2016)


Ciudad Real – Spain
6 LINEAR ELASTICITY 779

L
6 6 2 2 L 2 2 L 2 12 1 1
∫ w′ dx = 5L w
2 2
1 + w2 + θ y1 + θy2 − w1w2 − w1θ y1 − w1 θ y 2
5L 15 15 5L 5 5
0 (6.455)
1 1 L
+ w2 θ y1 + w2 θ y 2 − θ y1 θ y 2
5 5 15
L
12 12 2 4 2 4 2 24 12 12
∫ w′′ dx = L
2
3
w12 + 3
w2 + θ y1 + θ y 2 − 3 w1w2 − 2 w1 θ y1 − 2 w1θ y 2
L L L L L L
0 (6.456)
12 12 4
+ 2 w2 θ y1 + 2 w2 θ y 2 + θ y1θ y 2
L L L
L
3L2 7 L2 L3 L3

0
x w( x)dx =
20
w1 +
20
w2 − θ y1 +
30 20
θy2 (6.457)

Let us consider a beam element with the flexural rigidity EI y constant into the beam
element in which is under the uniformly distributed load q z , (see Figure 6.160).

y
z

qz
x
1
L 2

Figure 6.160: Beam under uniformly distributed load.


The total potential energy is given by:
L L
EI y
Π = U int − U ext = ∫
0
2 ∫
w′′ 2 dx − q z w( x)dx
0
(6.458)

As we are considering that q z is independent of x , the external potential energy becomes:


L L

∫ ∫
ext
U = q z w( x)dx = q z w( x)dx (6.459)
0 0

By considering the equation in (6.453) into the above equation we can express U ext in
terms of the nodal parameters w1 , w2 , θ y1 and θ y 2 , i.e.:
L
L L L2 L2 
0

U ext = q z w( x)dx = q z  w1 + w2 − θ y1 + θ y 2 
2 2 12 12 
(6.460)

Considering that EI y is constant in the beam element, the internal potential energy
becomes:
L
EI y
U int = ∫ w′′
2
dx (6.461)
2 0

Using the equation (6.456) the above equation can be represented as follows:

University of Castilla-La Mancha Draft By: Eduardo W. V. Chaves (2016)


Ciudad Real - Spain
780 SOLVING PROBLEMS BY MEANS OF CONTINUUM MECHANICS

L
EI y EI y  12 2 12 2 4 2 4 2 24 12
U int = ∫ w′′ dx =
2
 3 w1 + 3 w2 + θ y1 + θ y 2 − 3 w1 w2 − 2 w1 θ y1
2 2 L L L L L L
0
(6.462)
12 12 12 4 
− 2 w1 θ y 2 + 2 w2 θ y1 + 2 w2 θ y 2 + θ y1θ y 2 
L L L L 

Then, the total potential energy (6.458), Π = U int − U ext , can be written as follows:
EI y  12 2 12 2 4 2 4 2 24 12 12
Π=  3 w1 + 3 w2 + θ y1 + θ y 2 − 3 w1w2 − 2 w1θ y1 − 2 w1θ y 2
2 L L L L L L L
(6.463)
12 12 4  L L L2 L2 
+ 2 w2 θ y1 + 2 w2 θ y 2 + θ y1θ y 2  − q z  w1 + w2 − θ y1 + θ y 2 
L L L  2 2 12 12 
As we are looking for the stationary state (equilibrium) the following must hold:
∂Π EI y  24 24 12 12  L
=0 ⇒  3 w1 − 3 w2 − 2 θ y1 − 2 θ y 2  − q z = 0 (6.464)
∂w1 2 L L L L  2

∂Π EI y  8 12 12 4  L2
=0 ⇒  θ y1 − w1 + w2 + θ y2  + q z =0 (6.465)
∂θ y1 2 L L2 L2 L  12

∂Π EI y  24 24 12 12  L
=0 ⇒  3 w2 − 3 w1 + 2 θ y1 + 2 θ y 2  − q z = 0 (6.466)
∂w2 2 L L L L  2

∂Π EI y  8 12 12 4  L2
=0 ⇒  θ y2 − w1 + w2 + θ y1  − q z =0 (6.467)
∂θ y 2 2 L L2 L2 L  12

Restructuring the above set of equations in matrix form we can obtain:


 12 EI y − 6 EI y − 12 EI y − 6 EI y   qz L 
 
 L
3
L2
L 3
L  w   2 
2

 − 6 EI y 4 EI y 6 EI y 2 EI y   1   − q z L2 
θ  
 L2 L L2 L   y1  =  12 
 − 12 EI 6 EI y 12 EI y 6 EI y  w2   q z L 
(6.468)
 y
   
 L 3
L2 L3 L2  θ y 2   2 2 
 − 6 EI y 2 EI y 6 EI y 4 EI y     q z L 
   
 L2 L L2 L   12 
or:
[ Ke (1) ] {u ( e ) } = { f Eq
(e)
} (6.469)
in which:
 12 EI y − 6 EI y − 12 EI y − 6 EI y   qz L 
 3 2 3   2 
 L L L L2 
 2
 − 6 EI y 4 EI y 6 EI y 2 EI y 
 − qz L 
 2
L L2 L   
[ Ke (1) ] =  L ; (e)
{ f Eq } =  12  (6.470)
− 12 EI y 6 EI y 12 EI y 6 EI y  qz L
   
 L3 L2 L3 L2   2 
 − 6 EI y 2 EI y 6 EI y 4 EI y   q z L2 
   
 L2 L L2 L   12 

University of Castilla-La Mancha Draft By: Eduardo W. V. Chaves (2016)


Ciudad Real – Spain
6 LINEAR ELASTICITY 781

where { f Eq(e ) } is the consistent load vector and [ Ke (1) ] is the stiffness matrix for the beam
element, and note that [ Ke (1) ] has no inverse since det[ Ke (1) ] = 0 . In order to obtain the
unique solution of the set of equations (6.468) we must introduce the boundary conditions.
The same equation in (6.468) can be obtained by means of the Principle of Virtual Work.
Example 1: Let us consider that the beam is fixed at one end ( x = 0) , (see Figure 6.161).

y deflection
z Boundary conditions
w = 0
Node 1:  1
x w1′ = 0
1
2 w ≠ 0
qz Node 2:  2
w2′ ≠ 0
L

Figure 6.161: Cantilever under uniformly distributed load.


The force vector { f (e ) } is constructed by the consistent load vector ( { f Eq(e ) } ) and by the
concentrated (nodal) force vector { f 0(e ) } which is zero for the problem presented in Figure
6.161.
By applying the boundary conditions to the equation in (6.468) we can obtain:
1 0 0 0   
  w   0 
0 1 0 0   1
 0 
 12 EI y 6 EI y  θ y1   q z L  (1)
0 0   =   ⇔ [ Ke ] {u ( e ) } = { f Eq
(e)
} (6.471)
 L 3
L2  w2   2 
 
4 EI y  θ y 2   q z L 
2
0 6 EI y
0  
 L2 L   12 

By solving the above equation we can obtain:


 0 
w1   0 
   
θ y1   q z L 
4

  =  8 EI  (6.472)
w2   y 
θ   − q z L3 
 y2   
 6 EI y 

The above solution (deflection and rotation) matches the exact solution. The moment at
node 1 is given by M y1 = − EI y w1′′ . And by means of the equation in (6.452) we can obtain:

12 x 6   12 x 6   6x 4  6x 2 
w′′ = w1  3 − 2  + w2 − 3 + 2  − θ y1  2 −  − θ y 2  2 − 
 L L   L L   L L L L
6  2
⇒ w′′( x = 0) = w1′′ = w2  2  − θ y 2 − 
L   L

University of Castilla-La Mancha Draft By: Eduardo W. V. Chaves (2016)


Ciudad Real - Spain
782 SOLVING PROBLEMS BY MEANS OF CONTINUUM MECHANICS

q z L4  6  q z L3 2 5 q L
2
⇒ w1′′ = − ⇒ ⇒ w1′′ =   z
8 EI y  L2  6 EI y L
   6  2 EI y
Then, the moment becomes:
2 2
5 q L 5 q L
M y1 = − EI y w1′′ = − EI y   z = −  z (6.473)
 6  2 EI y 6 2

− q z L2
And if we compare with the exact solution M yexact
1 = the error is 16.6% .
2
NOTE 3: Note that the differential equation for the beam problem, given by equation
∂ 4w qz  qz 
(6.225), = , requires a fourth-order function if   is a constant into the beam.
∂x 4 EI y  EI y 
 
For the problem established here we have adopted a third-order function, (see equation
(6.449)). Due to this fact we have errors associated with M y and Qz . To overcome this
drawback we will establish a procedure in order to achieve accuracy for internal forces at
the beam element nodes.
Loading vector for the beam element
Once the displacements are obtained (6.472), the internal forces can be obtained by means
of the equation (6.469), i.e.:
{ f (e ) } = [ Ke (1) ] {u (e ) } (6.474)
with which we can obtain:
 12 EI y − 6 EI y − 12 EI y − 6 EI y   − qz L 
  0  
 L
3
L 2
L 3 2
L   2 
 − 6 EI 4 EI y 6 EI y 2 EI y   0   5q z L2 
y
 
L   q z L  =  12 
4

{f }=  L
(e)
2
L L2 (6.475)
− 12 EI y 6 EI y 12 EI y 6 EI y   8 EI y   q z L 
    
 L
3
L2 L3 L2   − q z L3   2 2 
 − 6 EI y 2 EI y 6 EI y 4 EI y    qL 
 6 EI y   z 
 L2 L L2 L    12 
When we are dealing with the traditional finite element technique, the internal forces of the
beam element are given by { f (e ) } , (see Figure 6.162(a)). The error can be minimized by
dividing the beam element in more elements. From structural analysis, the exact solution is
given by Figure 6.162(c), and we can verify that the reactions at the ends of the beam
element can be obtained as follows:
~
{R ( e ) } = { f (e ) } + { f (e ) } = { f ( e ) } − { f Eq
(e )
} (6.476)

where { f Eq(e ) } is given by equation (6.470). Then, the internal forces acting at the ends of
the beam can be obtained as follows:
(e)
{ f int } = −{R ( e ) } (6.477)
~
The vector { f ( e ) } = −{ f Eq( e ) } , (see equation (6.470)), is known as loading vector. Note that
the vector given in Figure 6.162(b) represents the reactions that appear when the beam is
fixed at both ends, (see Figure 6.163), (Gere&Weaver (1965)). And these reactions are the
same as the one obtained by means of the equation (6.470).

University of Castilla-La Mancha Draft By: Eduardo W. V. Chaves (2016)


Ciudad Real – Spain
6 LINEAR ELASTICITY 783

5q z L2 1 2
M y1 = q z L2
12 M y2 =
a) { f (e ) } 12
qz
− qz L qz L
Qz 1 = Qz 2 =
2 2

+
~
b) { f ( e ) } = −{ f Eq(e ) } q z L2 1 2
M y1 = − q z L2
12 M y2 =
12
qz
− qz L − qz L
Qz 1 = Qz 2 =
2 2

=
c) Exact solution q z L2 1 2
M y( −1 ) = M y( +2) = 0
2
(Reactions)
{R ( e ) } = { f ( e ) } − { f Eq
(e)
} qz
Qz(1−) = −qz L Qz( +2 ) = 0

d) Internal forces − q z L2 1 2
M y( 1+ ) = M y( −2) = 0
(e)
2
{ f int }
qz
Qz(1+) = qz L Qz( −2 ) = 0

Figure 6.162: Reaction in the beam element.

L
− qz L
Qz(1− ) = − qz L
2 Qz( +2 ) =
2

qz L2 qz − qz L2
M y( −1 ) = M y( +2) =
12 12

Figure 6.163: Reaction forces of the beam with fixed ends.

NOTE 4: Analytical solution using direct integration


In this sub-section we will obtain the analytical solution (the exact one) for the problem
described in Figure 6.164. To obtain the analytical solution we will use the direct
integration, and we will start from the moment function of the beam, (see Figure 6.164):
( L − x) − q z 2
M y ( x) = −q z ( L − x) = ( x − 2 Lx + L2 )
2 2

University of Castilla-La Mancha Draft By: Eduardo W. V. Chaves (2016)


Ciudad Real - Spain
784 SOLVING PROBLEMS BY MEANS OF CONTINUUM MECHANICS

( L − x)
My >0
2 q z ( L − x)
z 0≤ x≤L

Reactions:
qz
Fz(1− ) = − q z L x

qz L2 − qz 2
M y( 1− ) = x M y ( x) = ( x − 2 Lx + L2 )
2 L 2

Figure 6.164: Beam fixed at one end under uniformly distributed load.

Recall that the differential equation of the beam in function of the deflection ( w ), (see
equation (6.417)), is given by:
d 2w d 2 w − qz 2
M y = − EI y w′′ ≡ − EI y ⇒ − EI y = ( x − 2 Lx + L2 )
dx 2 dx 2 2
By means of direct integration we can obtain:
d 2w qz 2 integrating dw q z 3
EI y = ( x − 2 Lx + L2 )  → EI y = ( x − 3Lx 2 + 3L2 x) + C1
dx 2 2 dx 6
The constant of integration C1 can be obtained by means of the boundary condition, i.e.
dw
the rotation is zero at ( x = 0) , so, ≡ w′( x = 0) = 0 with which we can obtain C1 = 0 .
dx
Then, the first derivative of the deflection becomes:
dw
dx
q
≡ w′ = z 3L2 x − 3Lx 2 + x 3
6 EI y
( )
By integrating once more we can obtain the deflection of the beam w(x) :
 3L2 x 2 3Lx 3 x 4 
dw q
(
= z 3L2 x − 3Lx 2 + x 3
dx 6 EI y
) integratin
 g → w =
qz
6 EI y

 2 − 3 + 4  + C2

 
where the constant of integration C 2 can be obtained as follows w( x = 0) = 0 ⇒ C 2 = 0 .
Then, the deflection of the beam becomes:

w( x) =
qz x 2
24 EI y
(
6 L2 − 4 Lx + x 2 )
When x = L we can obtain:

deflection: w( x = L) =
qz x 2
24 EI y
( q L2
6 L2 − 4 Lx + x 2 = z
24 EI y
) ( q L4
6 L2 − 4 L2 + L2 = z
8 EI y
)
rotation:
dw
dx
q
( q
)
q L3
≡ w′( x = L) = z 3L2 x − 3Lx 2 + x 3 = z 3L3 − 3L3 + L3 = z = − θ y
6 EI y 6 EI y 6 EI y
( )
which matches the result in (6.472).

University of Castilla-La Mancha Draft By: Eduardo W. V. Chaves (2016)


Ciudad Real – Spain
6 LINEAR ELASTICITY 785

q z L3
z w′( x = L) = = −θy2
y 6 EI y

q z L4
w( x = L) = = w2
8EI y
1
x
2

Figure 6.165: Displacements at the free-end of the beam.

Example 2: Let us consider that the beam is fixed at one end ( x = 0) , (see Figure 6.166).

y deflection
z Boundary conditions
w = 0
Node 1:  1
x w1′ = 0
1
2 w ≠ 0
P Node 2:  2
w2′ ≠ 0
L

Figure 6.166: Cantilever under concentrated force.


This example is very similar to the previous Example 1. The only difference is the force
vector in which we must add the concentrated force vector { f 0( e ) } :
0 
 
(e) 0 
{ f0 } =   (6.478)
P 
0 
 
By applying the boundary conditions to the equation in (6.468) we can obtain:
 qz L 
1 0 0 0  w   2 
  1  2 0  0 0 
0 1 0 0    − qz L       
 12 EI y 6 EI y  θ y1   12  0  0 0 
=
   q L  +   =   +  
0 0 2  w 0
 L3 L  2   z  P    P  (6.479)
0 6 EI y 4 EI y  θ   2  0  0 0 
0  y 2   q L2  123 { 123
 L2 L  
z
 force
Concentrated Consistent Concentrated
1412
24 3
vector
( e)
load vector force vector
={ f 0 }
Consistent
load vector
( e)
={ f Eq }

Where we have considered { f Eq( e ) } = {0} . Solving the above equation we can obtain:

University of Castilla-La Mancha Draft By: Eduardo W. V. Chaves (2016)


Ciudad Real - Spain
786 SOLVING PROBLEMS BY MEANS OF CONTINUUM MECHANICS

 0 
w1   0 
   
 θ y1   PL 
3

  =  3EI  (6.480)
w2   y 
 θ   − PL2 
 y2   
 2 EI y 
Reaction Calculation
 12 EI y − 6 EI y − 12 EI y − 6 EI y 
 3 2 3  0 
 L L L L2   
 − 6 EI 4 EI y 6 EI y 2 EI y   0  − P 
y
3  

{f }=  L
(e)
2
L L2 L   PL  =  PL 
(6.481)
− 12 EI y 6 EI y 12 EI y 6 EI y   3EI y   P 
  2
  
 L
3
L2 L3 L2   − PL   0 

 − 6 EI y 2 EI y 6 EI y 4 EI y   2 EI 
 L2  y 
L L2 L 
Then, by apply the equation (6.476), {R ( e ) } = { f (e ) } − { f Eq( e ) } , we can obtain:

− P  0 − P   Fz1 
(−)

 PL  0  PL   (−) 
(e) (e) (e)        M y1 
{R } = { f } − { f Eq } =   −   =   =  ( + )  (6.482)
 P  0  P   Fz 2 
 0  0  0  M y( +1 ) 
 
And the internal forces at the extremities of the beam are:
 Fz(1+ )   P 
 (+)   
M  − PL 
{ f int } = −{R } =  (y−1)  = 
(e) (e)
 (6.483)
 Fz 2   − P 
M y( −2)   0 
   

NOTE 5: Analytical solution using direct integration


In this sub-section we will obtain the analytical solution (the exact one) for the problem
described in Figure 6.167. To obtain the analytical solution we will use the direct
integration, and we will start from the moment function of the beam, (see Figure 6.167):
M y ( x) = − P( L − x)

Recall that the differential equation of the beam in function of the deflection ( w ), (see
equation (6.417)), is given by:
d 2w d 2w
M y = − EI y w′′ ≡ − EI y ⇒ − EI y = − P( L − x)
dx 2 dx 2
By means of direct integration we can obtain:
d 2w dw  x2 
EI y = P( L − x) integratin
 g → EI y = P Lx −  + C1
dx 2 dx  2 

University of Castilla-La Mancha Draft By: Eduardo W. V. Chaves (2016)


Ciudad Real – Spain
6 LINEAR ELASTICITY 787

0≤ x≤L My >0
z ( L − x) z

P = M y( −1 )
P

P
x
L L

= Internal forces
Reactions:
Fz(1− ) = − P
z
P M y( −)
1 = PL

M y ( x) = − P ( L − x)
x

Figure 6.167: Beam fixed at one end under concentrated force.

The constant of integration C1 can be obtained by means of the boundary condition


dw
≡ w′( x = 0) = 0 with which we can obtain C1 = 0 . Then, the first derivative of the
dx
deflection becomes:
P  x2 
dw
dx
≡ w′ =  Lx −  =
EI y 
P
2  2 EI y
(
2 Lx − x 2 )
By integrating once more we can obtain the deflection of the beam w(x) :

dw P  x2  P  x 2 x3 
=  Lx −  integrating
 → w =  L −  + C2
dx EI y  2  EI y  2
 6 

where the constant of integration C 2 can be obtained as follows w( x = 0) = 0 ⇒ C 2 = 0 .


Then, the deflection of the beam becomes:
P
w( x) = (3Lx 2 − x 3 )
6 EI y
When x = L we can obtain:
P P PL3
ƒ deflection: w( x = L) = w2 = (3Lx 2 − x 3 ) = (3LL2 − L3 ) =
6 EI y 6 EI y 3EI y

ƒ rotation: dw
dx
≡ w′( x = L) = − θ y 2 =
P
2 EI y
(
2 Lx − x 2 =
P
)
2 EI y
2 LL − L2 =( PL2
2 EI y
)
which matches the solution in (6.480).

University of Castilla-La Mancha Draft By: Eduardo W. V. Chaves (2016)


Ciudad Real - Spain
788 SOLVING PROBLEMS BY MEANS OF CONTINUUM MECHANICS

Shearing Force
Taking into account that
d 2w d 3w − P
EI y = P ( L − x) derivative
 → =
dx 2 dx 3 EI y
And according to equation in (6.423) we can conclude that:
∂ 3 w − Qz − Qz − P
= ⇒ = ⇒ Qz ( x) = P (constant)
∂x 3 EI y EI y EI y
NOTE 6: When are dealing with steel structure the sign of bending moment plays no role
at the time of the design of the structure, since the steel has the same behavior when is
under either traction or compression. This scenario changes when we are dealing with
reinforced concrete structure, since the concrete has little capacity to support tensile action.
The bending moment sign convention for concrete structure is defined positive if the
lower fibers are under traction and negative if the upper fibers are in traction, (see Figure
6.168). Nevertheless, in this book we are adopting the positive values (displacements,
rotations, forces and moments) if they are in accordance with the axes orientation.

M steel reinforcement Bending moment diagram

Beam
top edge (−)

bottom edge (+)


(+)

anchorage steel reinforcement

Figure 6.168: Bending moment sign convention for reinforced concrete design.

Problem 6.63
Obtain the
explicit expression for the stiffness matrix from the equation
[ Ke ] {u } = { f (e ) } for the problem described in Problem 6.62-NOTE 2 by using the
( 3) (e)

Principle of Virtual Work, (see Problem 5.22). Obtain also the consistent mass matrix
given in Problem 5.24.
Solution:
According to Problem 6.62-NOTE 2 we have found out the deflection function, which
can also be written as follows:
  x 3  x 2    x 3  x  2   x3 2x2   x3 x 2 
w = w1  2  − 3  + 1 + w2  − 2  + 3   − θ y1  2 − + x − θ y 2  2 − 
  L  L    L   L   L L  L L
⇒ w( x) = w1 N1 + θ y1 N 2 + w2 N 3 + θ y 2 N 4

University of Castilla-La Mancha Draft By: Eduardo W. V. Chaves (2016)


Ciudad Real – Spain
6 LINEAR ELASTICITY 789

 w1 
θ 
 y1 
⇒ w( x) = [N1 N2 N3 N 4 ]  ⇒ w( x) = [ N ]{u ( e ) } (6.484)
 w2 
θ y 2 
 
where N i are the shape functions. Note that the displacement vector is made up by
displacements and rotations, and
 2 x 3 3 x 2   − x3 2 x 2   2 x 3 3x 2   − x 3 x 2 
[ N ] =  3 − 2 + 1  2 + − x   − 3 + 2   2 + 
 L
 L L   L L   L L   L 

∂[ N ]  6 x 2 6 x   − 3x 2 2 x 
 − 3x 2 4 x   6 x 2 6 x 
⇒ =  3 − 2   2 +
 L  L− 1  − 3 + 2 
 2 +  (6.485)
∂x  L L   L
 L  
  L L 
∂ 2[N ]  12 x 6   − 6 x 4   12 x 6   − 6 x 2 
⇒ 2
≡ [ BV ] =  3 − 2   2 +   − 3 + 2   2 + 
∂x  L L   L L  L L   L L 

Next we will try to relate strain and stress to the displacement vector {u (e) } . For this beam
d 2w
problem the strain is given by the equation ε (x2) ( z ) = − z ≡ − w, xx z , (see Problem 6.59-
dx 2
NOTE 3). The term w, xx can be obtained as follows:

w( x) = [ N ]{u ( e ) } ⇒
∂w( x) ∂
∂x
=
∂x
(
[ N ]{u ( e ) } =
∂x
)
∂[ N ] (e )
{u }
(6.486)
∂ 2 w( x) ∂ 2 [ N ] (e)
⇒ ≡ w, xx = {u }
∂x 2 ∂x 2
∂ 2[N ]
where is given by equation in (6.485). Then:
∂x 2
d 2w ∂ 2 [ N ] (e)
ε (x2 ) = {ε } = − z ≡ − z {u } = − z[ BV ]{u ( e) }
dx 2 ∂x 2 (6.487)
σ (x2) = Eε (x2 ) or {σ } = [ E ]{ε } = − z[ E ][ BV ]{u ( e ) }

We will adopt the same approximation for the virtual field ( ε x( 2) ), i.e.:
ε x( 2 ) = {ε } = − z[ BV ]{u ( e ) } (Virtual field) (6.488)
And according to the problem established in Problem 5.22 we can conclude that:


{F (e ) }T {u ( e ) } = {σ }T {ε } dV
V
or ∫
{u ( e ) }T {F ( e ) } = {ε }T {σ } dV
V


⇒ {u ( e ) }T {F ( e ) } = {− z[ BV ]{u ( e ) }}T {− z[ E ][ BV ]{u ( e ) }} dV
V


⇒ {u ( e ) }T {F ( e ) } = z 2 {u (e ) }T [ BV ]T [ E ][ BV ]{u ( e ) } dV
V

 
⇒ {u ( e ) }T {F ( e ) } = {u ( e ) }T  z 2 [ BV ]T [ E ][ BV ] dV {u (e ) }

 
V 
 
⇒ {F ( e ) } =  z 2 [ BV ]T [ E ][ BV ] dV {u ( e ) }

 
V 

University of Castilla-La Mancha Draft By: Eduardo W. V. Chaves (2016)


Ciudad Real - Spain
790 SOLVING PROBLEMS BY MEANS OF CONTINUUM MECHANICS

⇒ {F ( e ) } = [ Ke ]{u (e ) } (6.489)
Taking into account that [ E ] = E and I y are constant into the element, the explicit form
for the stiffness matrix is given by:

∫ ∫
[ Ke ] = z 2 [ BV ( x)]T [ E ][ BV ( x)] dV = E z 2 [ BV ( x)]T [ BV ( x)] dV
V V
L L


0

A 23

⇒ [ Ke ] = E [ BV ( x)]T [ BV ( x)] z 2 dAdx = E [ BV ( x)]T [ BV ( x)] I y dx
0 (6.490)
1
=I y
L L


⇒ [ Ke ] = EI y [ BV ( x)]T [ BV ( x)]dx ≡ EI y [ BbV ]dx
0

0

where the multiplication of matrices [ BV ( x)]T [ BV ( x)] ≡ [ BbV ] is given by


 36(2 x − L) 2 − 12( 2 x − L)(3 x − 2 L) − 36(2 x − L) 2 − 12(2 x − L)(3 x − L ) 
 
 L6 L5 L6 L5 
 4(3 x − 2 L) 2 12(2 x − L)(3 x − 2 L) 4(3 x − 2 L)(3 x − L) 
 L4 L5 L4 
[ BbV ] = 
36(2 x − L) 2 12(2 x − L)(3 x − L) 
 
 symmetric L6 L5 
 4(3 x − L) 2 
 4 
 L 
L L
36(2 x − L) 2 12 EI y
Note that the component [ Ke ]11 = EI y ∫ [ BbV ]11 dx = EI y ∫ 6
dx = , and in
0 0
L L3
the same fashion we can obtain the remaining components of the matrix [Ke ] . With that
we can obtain the same stiffness matrix as the one in Problem 6.62-NOTE 2.

In the same way we can obtain the mass matrix given in Problem 5.24 by:


[ M ( e ) ] = ρ [ N ( x)]T [ N ( x)] dV
V
(6.491)

where ρ is the mass density. For this problem the matrix [N ] is given by equation in
(6.485), and:
L L L

∫ 0
∫ A 0

[ M ( e ) ] = ρ [ N ( x)]T [ N ( x)] dAdx = ρA [ N ( x)]T [ N ( x)]dx ≡ ρA [ Nn]dx
0
∫ (6.492)
{
=A

where
 N1  ( N1 ) 2 N1 N 2 N1 N 3 N1 N 4 
N   2 
NN (N2 ) N 2 N3 N2 N4 
[ N ]T [ N ] ≡ [ Nn] =  [N1 N4 ] =  1 2
2
N2 N3
N N
(6.493)
 N3  N 2 N3 (N3 )2 N3 N 4 
   1 3 
N4   N1 N 4 N2N4 N3 N 4 ( N 4 ) 2 

The first term of [ M (e ) ] can be obtained as follows:

University of Castilla-La Mancha Draft By: Eduardo W. V. Chaves (2016)


Ciudad Real – Spain
6 LINEAR ELASTICITY 791

L L L 2
 2 x 3 3x 2  13L
∫ ∫ ∫
]11 = ρA [ Nn]11 dx = ρA ( N1 ) dx = ρA  3 − 2 + 1 dx = ρA
(e) 2
[M
0 0 0
L L  35

Then, after the integration (6.492) is carried out we can obtain:


 13L − 11L2 9L 13L2 
 
 35 2 210 70 420 
 − 11L L3 − 13L2 −L 3
L
 140 
[ M (e ) ] = ρA [ N ( x)]T [ N ( x)]dx = ρA 210
∫ 105 420 (6.494)
9L − 13L2 13L 11L2 
0  
 70 420 35 210 
 13L2 − L3 11L2 L3 
 420 105 
 140 210
The matrix in (6.494) is known as the Consistent Mass Matrix.
NOTE 1: Note that the internal potential energy due to bending moment M y can be
written as follows:
L 2 L
1 ( 2) ( 2) 1 My 1
U int =
2V ∫
σ x ε x dV =
2 0 EI y ∫
dx =
20 ∫
EI y ( w, xx ) 2 dx (6.495)

where we have considered M y = − EI y w, xx . By considering that the deflection can be


approached by the shape functions:
w( x) = [ N ]{u ( e) } ; w, xx ≡ w′′ = [ BV ]{u (e ) }
The total potential energy can be represented as follows
L L
1 1
Π = U int − U ext =
20 ∫
EI y ( w, xx ) 2 dx − U ext =
20 ∫
EI y ([ BV ]{u ( e ) })([ BV ]{u ( e ) })dx − U ext

1 L 
⇒ Π = {u }  EI y [ BV ]T [ BV ]dx {u (e ) } − U ext
(e) T

2 
 0 
As we are looking for the stationary state (equilibrium) the following must hold:
∂Π L  ext
 EI [ B ]T [ B ]dx {u ( e ) } − ∂U
=
∂{u ( e ) }  0∫y V V

 ∂{u (e ) }
= {0}

L  ∂U ext
⇒  EI y [ BV ]T [ BV ]dx {u ( e ) } =

(e)
= { f Eq }
  ∂{u (e )
}
0 
⇒ [ Ke (1) ] {u (e ) } = { f Eq
(e)
}

University of Castilla-La Mancha Draft By: Eduardo W. V. Chaves (2016)


Ciudad Real - Spain
792 SOLVING PROBLEMS BY MEANS OF CONTINUUM MECHANICS

Problem 6.64
Obtain the rotations and reaction forces at the nodes 1 and 2 for the beam described in
Figure 6.169.

z y
Boundary conditions
w = 0
qz Node 1:  1
1 2
x w1′ ≠ 0
w = 0
Node 2:  2
L w′2 ≠ 0

Figure 6.169: Beam bi-supported under uniformly distributed load.


Solution:
We will adopt the same procedure described in the previous problem, (see Problem 6.62-
NOTE 2). So, for this problem is also valid that:
 12 EI y − 6 EI y − 12 EI y − 6 EI y   qz L 
 
 L
3
L2
L3
L2  w   2 
 − 6 EI y 4 EI y 6 EI y 2 EI y   1   − q z L2 
θ  
 L2 L L2 L   y1  =  12  = { f ( e ) } (6.496)
 − 12 EI y 6 EI y 12 EI y 6 EI y  w2   q z L 
    
 L
3
L2 L3 L2  θ y 2   2 2 
 − 6 EI y 2 EI y 6 EI y 4 EI y     q z L 
 L2  12 
L L2 L   
and by applying the boundary conditions, (see Figure 6.169), we can obtain:
1 0 0 0  w = 0   0 
 2 EI y   1
4 EI y   − q z L 
2
0 0  θ
 L L   y1  =  12  (6.497)
0    
0 1 0  w2 = 0  0 2 
 2 EI y 4 EI y  θ qL
0 0   y 2   z 
 L L   12 
which is the same as
 4 EI y 2 EI y   − q z L2 
   
 L L   θ y1  =  12  (6.498)
   
 2 EI y 4 EI y  θ y 2  2
   qz L 
 L L   12 
By solving the above system we can obtain:
−1
 4 EI y 2 EI y   − q z L2 
 θ y1   L L 
  
12  = q z L − 1
3

 =  2    (6.499)
θ y 2   2 EI y 4 EI y 
 q z L  24 EI y  1 
 L L   12 
which matches the exact solution.

University of Castilla-La Mancha Draft By: Eduardo W. V. Chaves (2016)


Ciudad Real – Spain
6 LINEAR ELASTICITY 793

z deflection − q z L3
θ y1 = = − w1′
24 EI y
qz L3
θy 2 = = − w′2
θ y1 θy2 2 24 EI y
1
x

Figure 6.170: Rotations at the ends of the beam.


Reaction Calculation
 12 EI y − 6 EI y − 12 EI y − 6 EI y 
  0   
  0 2
3 2 3
 L L L L2  
2 EI y   − q z L
3

 − 6 EI y 4 EI y 6 EI y   − qz L 
{ f (e ) } =  L
2
L L2 L   24 EI y   12 
− 12 EI y 6 EI y 12 EI y 6 EI y   0 =  (6.500)
    0 
L2   q z L   q z L2 
3 3
 L L2 L3
 − 6 EI y 2 EI y 6 EI y 4 EI y   24 EI   12 
 L2  y 
L L2 L 
Then, by apply the equation (6.476), {R (e ) } = { f ( e ) } − { f Eq( e ) } , we can obtain:

 qz L 
 0

  2
 − q L   Fz(1− ) 
 − q L2   − q L2
  z   
 z  z   2  M ( −) 
(e) (e) (e)  12   12   0   y1 
{R } = { f } − { f } =  − = =  (6.501)
 0   qz L   − qz L   Fz(2+ ) 
 qz L   2
2
    
     2   (+) 
 12   q z L2   0   M y1 
 12 

And the internal forces at the ends of the beam are:

 Fz(1+ )   q z L 
 (+)   2 
M   
(e )
{ f int } = −{R ( e ) } =  (y−1)  =  q0L  (6.502)
 Fz 2   z 
M y( −2)   2 
   0 

NOTE 1: Analytical solution by using the direct integration


In this sub-section we will obtain the analytical solution (the exact one) for the problem
described in Figure 6.169. To obtain the analytical solution we will use the direct
integration, and we will start from the moment function of the beam, (see Figure 6.171):
( L − x) qz L ( L − x) qz
M y ( x) = − Fz(2− ) ( L − x) − qz ( L − x) = ( L − x) − qz ( L − x) = ( Lx − x 2 ) (6.503)
2 2 2 2

University of Castilla-La Mancha Draft By: Eduardo W. V. Chaves (2016)


Ciudad Real - Spain
794 SOLVING PROBLEMS BY MEANS OF CONTINUUM MECHANICS

( L − x)
My >0 2 q z ( L − x)
z 0≤ x≤L

qz 2
1 x
− qz L
Fz(1− ) = − qz L
2 Fz(2+ ) =
x 2
L

Figure 6.171: Beam bi-supported under uniformly distributed load.

Recall that the differential equation for the beam in function of the deflection ( w ), (see
equation (6.417)), is given by:
d 2w d 2 w qz
M y = − EI y w′′ ≡ − EI y ⇒ − EI y = ( Lx − x 2 )
dx 2 dx 2 2
By means of direct integration we can obtain:
d 2 w − qz integrating dw − qz
EI y 2
= ( Lx − x 2 )  → EI y = (3Lx 2 − 2 x 3 ) + C1
dx 2 dx 12
dw
Due to the symmetry, at the middle of the beam span the condition ≡ w′( x = L2 ) = 0
dx
holds. Then, the constant of integration C1 can be obtained as follows:

− qz   L  L 
2 3
qz L3
EI y w′( x = L
2
) = 3L  − 2   + C1 = 0 ⇒ C1 =
12   2   2   24

Then
dw − qz q L3 dw − qz
EI y = (3Lx 2 − 2 x3 ) + z ⇒ ≡ w′ = (6 Lx 2 − 4 x3 − L3 ) (6.504)
dx 12 24 dx 24 EI y
By integrating once more we obtain the deflection of the beam w(x) :
dw − qz − qz
= (6 Lx 2 − 4 x 3 − L3 ) integratin
 g → w = (2 Lx 3 − x 4 − L3 x) + C2
dx 24 EI y 24 EI y

where the constant of integration C 2 can be obtained as follows w( x = 0) = 0 ⇒ C 2 = 0 .


Then, the deflection of the beam becomes:
− qz
w( x) = (2 Lx3 − x 4 − L3 x) (6.505)
24 EI y
Then, we can obtain:
L
Deflection at x = :
2

University of Castilla-La Mancha Draft By: Eduardo W. V. Chaves (2016)


Ciudad Real – Spain
6 LINEAR ELASTICITY 795

− qz   L   L   5q L4
3 4
3 L 
w( x = L
2
) =  2 L  −   − L    = z
(6.506)
24 EI y   2   2   2  384 EI y

First derivative of the deflection, (see equation (6.504)):


− qz q L3
( x = 0) ⇒ w′( x = 0) = w1′ = (6 L(0) 2 − 4(0)3 − L3 ) = z = − θ y1
24 EI y 24 EI y
− qz − qz L3
( x = L) ⇒ w′( x = L) = w2′ = (6 L( L) 2 − 4( L)3 − L3 ) = = −θy 2
24 EI y 24 EI y

which matches the equation in (6.499).


NOTE 2: Checking the results
According to equations in (6.423) the following is true:
∂2w − M y ∂ 3 w − Qz
= and = (6.507)
∂x 2 EI y ∂x 3 EI y
By means of equation (6.505) we can obtain:
∂w( x) ∂  − q z  − qz
w′ = =  (2 Lx 3 − x 4 − L3 x) = (6 Lx 2 − 4 x 3 − L3 )
∂x ∂x  24 EI y  24 EI y
∂ 2 w( x) ∂  − q z  − qz
w′′ = =  (6 Lx 2 − 4 x 3 − L3 )  = ( Lx − x 2 )
∂x 2
∂x  24 EI y  2 EI y

∂ 3 w( x) ∂  − q z  − qz
w′′′ = =  ( Lx − x 2 ) = ( L − 2 x)
∂x 3
∂x  2 EI y  2 EI y
With that the equations (6.507) become:
∂2w − M y − qz −My qz
= ⇒ ( Lx − x 2 ) = ⇒ M y ( x) = ( Lx − x 2 ) (6.508)
∂x 2 EI y 2 EI y EI y 2
which result matches the equation in (6.503). And
 qL
Q ( x = 0) = z
∂ 3 w − Qz − Qz − qz qz  z 2
= ⇒ = ( L − 2 x) ⇒ Qz ( x) = ( L − 2 x) ⇒ 
∂x 3 EI y EI y 2 EI y 2 −
Q ( x = L) = q z L
 z 2
The functions M y (x) and Q z (x) can be appreciated in Figure 6.172.

University of Castilla-La Mancha Draft By: Eduardo W. V. Chaves (2016)


Ciudad Real - Spain
796 SOLVING PROBLEMS BY MEANS OF CONTINUUM MECHANICS

M y(− )

A− x

Qz( − )

w′( x = L2 ) = 0 deflection − qz
w( x) = (2 Lx 3 − x 4 − L3 x)
24 EI y
w

2
1 x

L L
2 2

bending moment qz
M y ( x) = ( Lx − x 2 )
My 2

qL2
(+) 8 2
1 x

Qz Shear
qz
Qz ( x) = ( L − 2 x)
2
qz L
2 (+) 2
x
(-) − qz L
1
2

Figure 6.172: Deflection and internal “forces”.

NOTE 3: Note that the function adopted to approach the displacement w(x) is given by,
(see equation (6.450)):
  x 3  x
2
   x 3 x 
2
 x 3 2x 2   x3 x2 
w = w1 2  − 3  + 1 + w2 − 2  + 3   − θ y1  2 − + x − θ y 2  2 − 
  L  L    L   L   L L  L L

 θ y1  q z L3 − 1
For this example we have that w1 = w2 = 0 , and  =   , thus the above
θ y 2  24 EI y  1 
equation becomes:
q z L3  x 3 2 x 2  q L3  x 3 x 2 
w( x) =  2− + x − z  2− 
24 EI y  L L  24 EI y  L L
At the middle of the beam the displacement is:

University of Castilla-La Mancha Draft By: Eduardo W. V. Chaves (2016)


Ciudad Real – Spain
6 LINEAR ELASTICITY 797

q z L3  ( L2 ) 2( L2 )  L  q z L3  ( L )3 ( L )2  1 q L4
3 2

w( x = L
2
) =  − +   −  22 − 2  = z
24 EI y  L2 L  2  24 EI y  L L  96 EI
 y

1 q z L4  4  5q z L4
⇒ w( x = L2 ) = = 
96 EI y  5  384 EI y
And if we compare with the exact solution, (see equation (6.506)), the error is 20% .
NOTE 4: Let us consider the problem described in Figure 6.169 in which additionally we
have concentrated moments at the nodes 1 and 2 , (see Figure 6.173). Then, by adding the
concentrated force vector to the vector { f (e ) } and by applying the boundary conditions we
can obtain:
 4 EI y 2 EI y   − q z L2 
     (1) 
 L L   θ y1  =  12  +  M y 
 2 EI y 4 EI y  θ y 2   q z L2  M y( 2 )  (6.509)
    1 3
 L L  1412  424
24 3 ={ f0( e ) }
(e)
={ f Eq }

Solving the above equation we can obtain:

  − qz L + M y L − M y L 
−1 3 (1) ( 2)
 4 EI y 2 EI y    − q z L2 
  
 θ y1   L    (1)  
L    12  +  M y   =  24 EI y 3EI y 6 EI y 
  =  2 EI 4 EI y    q z L2  M y( 2 )    q L3  (6.510)
θ y 2   y      M y(1) L M y( 2) L 
  z
− +
 L L    12    24 EI y 6 EI y 3EI y 

z y
Boundary conditions
M y( 2 )
M y(1) w = 0
qz Node 1:  1
x w1′ ≠ 0
2
1
w = 0
Node 2:  2
L w′2 ≠ 0

Figure 6.173: Beam bi-supported under uniformly distributed load and concentrated
moments at the nodes.

Then the reaction vector can be obtained as follows:


 12 EI y − 6 EI y − 12 EI y − 6 EI y   − ( M y(1) + M y( 2 ) ) 
 3
L2 L3

L2  
0   L

 − 6LEI 4 EI y 6 EI y
3 (1) (2)
2 EI y   − qz L + M y L − M y L    
 y (1) qz L2 
 2
L L2 L   24 EI y 3EI y 6 EI y   M y − 12 
{f }=  L
(e)
 = 
− 12 EI y 6 EI y 12 EI y 6 EI y  0 ( M y(1) + M y( 2) ) 
   (1) ( 2 )  
L2   q z L − M y L + M y L  
3
 L
3
L2 L3 L 
 − 6 EI y 2 EI y 6 EI y 4 EI y   24 EI y 6 EI y 3 EI   qL 2 
 y 
 M y( 2 ) + z 
 L2 L L2 L   12 
(6.511)
Then, by applying the equation (6.476), i.e. {R (e ) } = { f (e ) } − { f Eq( e ) } , we can obtain:

University of Castilla-La Mancha Draft By: Eduardo W. V. Chaves (2016)


Ciudad Real - Spain
798 SOLVING PROBLEMS BY MEANS OF CONTINUUM MECHANICS

 − ( M y(1) + M y( 2 ) )   q z L 
     − ( M y + M y ) qz L   Fz(1− ) 
(1) ( 2)
 L   2 −
    
 (1) qz L2   − qz L2   L 2   (−) 
 M y − 12   12   M
( 1)
My   y1 
{R( e ) } = { f ( e ) } − { f Eq
(e)
} =  −   =   =  
(1) ( 2) qL (1) ( 2)
 ( M y + M y )   z   ( M y + M y ) − qz L   Fz(2+ ) 
 L   2   L 2   
 qL 2   q L2
  My( 2)   M (+) 
 M y( 2 ) + z  
z
    y1 
 12   12 
(6.512)

Problem 6.65
Obtain the rotations at the extremities of the beam described in Figure 6.174, and also
obtain the deflection at the middle of the beam length. Use a fourth-order function to
approach the displacement w(x) .

z y
Boundary conditions
w = 0
qz Node 1:  1
x w1′ ≠ 0
w = 0
Node 2:  2
L w′2 ≠ 0

Figure 6.174: Beam bi-supported under uniformly distributed load.


Solution:
We adopt the function w( x) = a1 + a2 x + a3 x 2 + a4 x 3 + a5 x 4 .
Let us apply the boundary conditions to w( x) = a1 + a2 x + a3 x 2 + a4 x 3 + a5 x 4 , i.e.:
w( x = 0) = a1 + a2 0 + a3 0 2 + a4 03 + a5 0 4 = 0 ⇒ a1 = 0
2 3 4
w( x = L) = a1 + a2 L + a3 L + a4 L + a5 L = 0 ⇒ a2 + a3 L + a4 L2 + a5 L3 = 0
{
=0

⇒ a2 = −(a3 L + a4 L2 + a5 L3 )
Then the displacement function can be rewritten as follows:
w( x) = a2 x + a3 x 2 + a 4 x 3 + a5 x 4 = −(a3 L + a4 L2 + a5 L3 ) x + a3 x 2 + a4 x 3 + a5 x 4
or
w( x) = a3 ( x 2 − Lx) + a 4 ( x 3 − L2 x) + a5 ( x 4 − L2 x) (6.513)
The total potential energy is given by Π = U int − U ext , where:
Internal Potential Energy
L L
EI y EI y
U int = ∫ w′′ 2 dx = ∫ w′′ dx
2
(6.514)
0
2 2 0

where

University of Castilla-La Mancha Draft By: Eduardo W. V. Chaves (2016)


Ciudad Real – Spain
6 LINEAR ELASTICITY 799

∂w
w′( x) = = a3 (2 x − L) + a4 (3 x 2 − L2 ) + a5 (4 x 3 − L2 )
∂x
∂2w
⇒ w′′( x) = 2 = 2a3 + 6a4 x + 12a5 x 2
∂x (6.515)
⇒ ( w′′( x)) = 4a32 + 24a3 a4 x + 48a3 a5 x 2 + 36a 42 x 2 + 144a4 a5 x 3 + 144a52 x 4
2

L
144 2 5

⇒ [ w′′( x)]2 dx = 4a32 L + 12a3 a4 L2 + 16a3 a5 L3 + 12a 42 L3 + 36a 4 a5 L4 +
0
5
a5 L

thus
L
EI y EI y  2 144 2 5 
∫ w′′ 2 dx =
int
U =  4a3 L + 12a3 a4 L2 + 16a3 a5 L3 + 12a42 L3 + 36a4 a5 L4 + a5 L 
0
2 2  5 
External Potential Energy
As we are considering that q z is independent of x , the external potential energy becomes:
L L L

∫ ∫ ∫
U ext = q z w( x)dx = q z w( x)dx = q z [a3 ( x 2 − Lx) + a 4 ( x 3 − L2 x) + a5 ( x 4 − L2 x)]dx
0 0 0
(6.516)
q L q L 3
3q L  4 5
= − z a3 + z a4 + z a5 
 6 4 10 
Then, the Total Potential Energy, Π (a3 , a4 , a5 ) = U int − U ext , becomes:
EI y  2 144 2 5 
Π ( a3 , a 4 , a5 ) =  4a3 L + 12a3 a4 L2 + 16a3 a5 L3 + 12a42 L3 + 36a4 a5 L4 + a5 L 
2  5 
(6.517)
q L 3
q L 4
3q L5

+  z a3 + z a4 + z a5 
 6 4 10 
As we are looking for the stationary state the following must hold:
∂Π
∂a3
=0 ⇒
EI y
2
{8La 3 + 12 L2 a 4 + 16 L3 a5 + } q z L3
6
=0 (6.518)

∂Π
∂a 4
=0 ⇒
EI y
2
{12L a 2
3 + 24 L3 a 4 + 36 L4 a5 + } q z L4
4
=0 (6.519)

∂Π EI y  3 4 288 5  3q z L5
=0 ⇒ 16 L a3 + 36 L a4 + L a5  + =0 (6.520)
∂a5 2  5  10
Restructuring the above set of equations in matrix form we can obtain:

 − q z L3   
   0 
 4 L 6 L2 8 L3  a3   6 4   a3   
    − qz L     − qz L 
EI y 6 L2 12 L3 18L4  a4  =   Solve
→ a 4  =   (6.521)
8 L3 18 L4 144 5  
L a   4 5 a   12 EI y 
 5   5   − 3q z L   5  q 
z
 10   24 EI 
   y 

Then, by substituting the coefficients (a3 , a 4 , a5 ) into the displacement function (6.513) we
can obtain:

University of Castilla-La Mancha Draft By: Eduardo W. V. Chaves (2016)


Ciudad Real - Spain
800 SOLVING PROBLEMS BY MEANS OF CONTINUUM MECHANICS

w( x) = a3 ( x 2 − Lx) + a 4 ( x 3 − L2 x) + a5 ( x 4 − L2 x)
 − qz L  3   4
⇒ w( x) = 0( x 2 − Lx) +  ( x − L2 x) +  q z ( x − L2 x)
 12 EI y   24 EI y  (6.522)
   
qz
⇒ w( x) = ( x 4 − 2 Lx 3 + L3 x)
24 EI y
which matches the exact solution, (see equation (6.505)). Then,
qz qz
w( x) = ( x 4 − 2 Lx 3 + L3 x) ⇒ w′( x) = (4 x 3 − 6 Lx 2 + L3 ) = − θ y ( x)
24 EI y 24 EI y

 − q z L3
 θ y ( x = 0 ) ≡ θ y1 =
− qz  24 EI y
θ y ( x) = (4 x 3 − 6 Lx 2 + L3 ) ⇒ 
24 EI y θ ( x = L) ≡ θ = q z L
3

 y y 2
24 EI y

NOTE 1: Let us obtain the stiffness matrix correspondent to this case by using the
function w( x) = a1 + a2 x + a3 x 2 + a4 x 3 + a5 x 4 . To determine the coefficients ( ai ) we will
need to define 5 points (nodes), (see Figure 6.175).

z
x (1) = 0
L
x (2) =
w1 w2 w3 w4 w5 4
1 2 3 4 5 x L
x ( 3) =
2
L L L L 3 L
4 4 4 4 x (4) =
4
(5)
L x =L

Figure 6.175: Discretization of the beam (5 degrees-of-freedom).

Applying the function w(x) for each node we can obtain:


1 0 0 0 0 
 w1  1 L L2 L3 L4   a1 
w   4 16 64

256  a 
 2   L L2 L3 L4   
2

 w3  = 1 a  (6.523)
w   2 4 8 16   3 
 4  1 3L 9 L2 27 L3 81L4  a 4 

 w5   4 16 64 256  a 5 
1 L L2 L3 L4 
and its inverse is given by:

University of Castilla-La Mancha Draft By: Eduardo W. V. Chaves (2016)


Ciudad Real – Spain
6 LINEAR ELASTICITY 801

 1 0 0 0 0 
 − 25 16 − 12 16 −1 
 a1   w 
a   3 L L L 3L L  1 
− 208 − 112 
22 w2 
 2   70 76
 
a 3  =  3L
2
3L2 L2 3L2 3L2   w3  (6.524)
a   − 80 96 − 128 224 − 16  w 
 4   3L3 L3 L3 3L3 3L3   w 
4
a 5   32 − 128 64 − 128 32   5 

 3L4 3L4 L4 3L4 3L4 
Then, the function w( x) = a1 + a2 x + a3 x 2 + a4 x 3 + a5 x 4 can be written as follows:
w( x) = N 1 w1 + N 2 w2 + N 3 w3 + N 4 w4 + N 5 w5
or
 25 x 70 x 2 80 x 3 32 x 4   16 x 208 x 2 96 x 3 128 x 4 
w( x) = 1 − + 2
− 3
+ 
4 
w1 + 
 L − 3L2 + L3 − 3L4  w2 +

 3 L 3 L 3 L 3 L   
 − 12 x 76 x 128 x
2 3
64 x 
4
 16 x 112 x 2
224 x 128 x 4 
3
 + − +  w +  − + −  w4 +
 3L L2 L3 L4 
3 
3L2 3L3 3L4 
  3L
 − x 22 x 2 16 x 3 32 x 4 
 
 L + 3L2 − L3 + 3L4  w5
 
Following the same procedure used from equation (6.458) to (6.468) we can finally obtain:
 7q z L 
 90 
 494 − 1376 1444 − 736 174   w1  16q z L 
 
− 1376 4224 − 5056 2944
 − 736  w2   45 
8 EI y    2q L 
3
 1444 − 5056 7224 − 5056 1444   w3  =  z 
15L      15  (6.525)
 − 736 2944 − 5056 4224 − 1376 w4  16q z L 
 174 − 736 1444 − 1376 494  w   45 
144 444444424444444443  5   7 q 
=[ Ke ( e ) ] zL
 
1 904
42 3
={ f Eq( e ) }
Note that for this case we do not have the continuity rotation between elements, and we
cannot apply rotation equal to zero, (see Figure 6.176).

bending moment equal to zero


(free rotation)

Figure 6.176: Free rotation.

University of Castilla-La Mancha Draft By: Eduardo W. V. Chaves (2016)


Ciudad Real - Spain
802 SOLVING PROBLEMS BY MEANS OF CONTINUUM MECHANICS

NOTE 2: Note that the shape function N1 could have been obtained by means of the
procedure used in Problem 6.40 – NOTE 3, i.e.:
1 x x2 x3 x4
L L2 L3 L4
1
4 16 64 256
L L2 L3 L4
1
2 4 8 16
3L 9 L2 27 L3 81L4
1
4 16 64 256
1 L L2 L3 L4 25 x 70 x 2 80 x 3 32 x 4
N1 = =1− + − +
1 0 0 0 0 3L 3L2 3L3 3L4
L L2 L3 L4
1
4 16 64 256
L L2 L3 L4
1
2 4 8 16
3L 9 L2 27 L3 81L4
1
4 16 64 256
1 L L2 L3 L4

Problem 6.66
Obtain the consistent load vectors for the cases: a) beam presented in Figure 6.177 and b)
Figure 6.178. And the boundary conditions are: Node 1 - w1 = 0 , w1′ ≠ 0 ; Node 2 - w2 = 0 ,
w′2 ≠ 0 .

z y z y
P

a b a b 2
2 x x
1 1
M Ay
L
L
a) Concentrated force b) Concentrated moment

Figure 6.177: Concentrated load and moment.

z y

qz( 2 )
q z(1) q z ( x)
2 x
1

Figure 6.178: Linearly distributed load.

University of Castilla-La Mancha Draft By: Eduardo W. V. Chaves (2016)


Ciudad Real – Spain
6 LINEAR ELASTICITY 803

Solution:
Case a (Figure 6.177(a)): For this case the stiffness matrix is the same as the one presented
in equation (6.470). And the External Potential Energy, (see Figure 6.156), for this case
becomes:
U ext = Pwp = Pw( x = a ) (6.526)
Taking into account the deflection function w , (see equation (6.450)), when ( x = a ) :
  x 3  x  2    x 3  x  2   x3 2 x2   x3 x2 
w = w1  2  − 3  + 1 + w2 − 2  + 3   − θ y1  2 − + x − θy 2  2 − 
  L  L    L   L   L L  L L
  a 3  a  2    a 3  a  2   a 3 2a 2   a3 a 2 
⇒ wP = w1 2  − 3  + 1 + w2  − 2  + 3   − θ y1  2 − + a − θy 2  2 − 
  L  L    L   L   L L  L L
(6.527)
Then,
  2a 3 3a 2   − a 3 2a 2   2a 3 3a 2   − a3 a 2  
U ext = P w1  3 − 2 + 1 + θ y1  2 + − a  + w2  − 3 + 2  + θ y 2  2 +  
 L L  
  L L   L   L L   L
To achieve the equilibrium, the following must be true:

∂Π ∂U int ∂U ext ∂U int  2a 3 3a 2 


= − =0 ⇒ − − 2 + 1 P = 0 (6.528)
∂w1 ∂w1 ∂w1 ∂w1  L3 L 
∂Π ∂U int ∂U ext ∂U int  − a 3 2a 2 
= − =0 ⇒ − 2 + − a P = 0 (6.529)
∂θ y1 ∂θ y1 ∂θ y1 ∂ θ y1  L L 
∂Π ∂U int ∂U ext ∂U int  2a 3 3a 2 
= − =0 ⇒ − − + 2 P = 0 (6.530)
∂w2 ∂w2 ∂w2 ∂w2  L3 L 

∂Π ∂U int ∂U ext ∂U int  − a 3 a 2 


= − =0 ⇒ − + P = 0 (6.531)
∂θ y 2 ∂θ y 2 ∂θ y 2 ∂θ y 2  L2 L

Then, the consistent load vector becomes:


  2a 3 3a 2  
  3 − 2 + 1 P   Pb 2 
 L L    3 (3a + b) 
 − a 3 2a 2    L 2 
 2 + − a  P   − Pab 
 L L    
} = 
2
(e)
{ f Eq or (e)
{ f Eq } =  2 L
  (6.532)
  − 2a + 3a  P 
3 2
 Pa (a + 3b)
  L3 L2    L3 
   2
Pa b 
  − a + a  P 
3 2
 2 
  L2  L 
  L  

and the system [ Ke (1) ] {u( e ) } = { f ( e ) } becomes:

University of Castilla-La Mancha Draft By: Eduardo W. V. Chaves (2016)


Ciudad Real - Spain
804 SOLVING PROBLEMS BY MEANS OF CONTINUUM MECHANICS

  2a 3 3a 2  
 12 EI y − 6 EI y − 12 EI y − 6 EI y    3 − 2 + 1 P 
 3   L L  
 L L2 L3 L2  w  
 − a 3
2 a 2
 
 − 6 EI y
1
4 EI y 6 EI y 2 EI y     + − a P
θ  2 L  
 L2 L L2 L   y1  =  L 
 (6.533)
 − 12 EI y 6 EI y 12 EI y 
6 EI y  w2 
  − 2 a 3
3 a 2

     + 2  P 
 L
3
L2 L3 L2  θ y 2    L3 L  
 − 6 EI 2 EI y 6 EI y 4 EI y     
 L2
y
  − a 3
a 2
 
L L2 L  
  L2 + L  P 

   
To solve the problem we must apply the boundary conditions to the above system, so:

0  w   
0
1 0 0
2 EI y     − a + 2a − a  P 

1 3 2
 4 EI y
 2
0 0  θ L  
 L L   y1  =  L 
0 0 1 0  w2   0 
2 EI y 4 EI y     
θ    − a + a  P 
3 2
0 0
 L L   y 2    L2 L  
 
Solving we can obtain:
w1   0 
  (a 2 − 3aL + 2 L2 )
θ y1  − Pa  
 =   (6.534)
w2  6 LEI y  0 
θ   (a 2 − L2 ) 
 y2 
which matches the exact solution. The vector { f (e ) } can be obtained as follows:
 12 EI y − 6 EI y − 12 EI y − 6 EI y 
 
L2   
3 2 3 0
 L L L
2 EI y   − a + 2a − a  P   (2a 2 − 3aL + L2 ) 
3 2
 − 6 EI 4 EI y 6 EI y
y  2 
  Pa − L(a 2 − 2aL + L2 )
 L2 L L2 L   L L  =
 − 12 EI y 6 EI y 12 EI y 6 EI y   0  3  2 2 
    L  − (2a − 3aL + L ) 
L2    − a + a  P 
3 2
 L
3
L2 L3  − L(a − L) 
 
 − 6 EI y 2 EI y 6 EI y 4 EI y    L2 L  
 
 L2 L L2 L 
Then, by applying the equation (6.476), {R (e ) } = { f ( e ) } − { f Eq( e ) } , we can obtain the reaction
forces:
  2a 3 3a 2  
  3 − 2 + 1 P 
 L L  
 P 
 (2a − 3aL + L )   − a
2 2 3
2a 2    ( a − L)
    + 
− a P L
(e) (e) (e) Pa − L(a 2 − 2aL + L2 )  L2 L    0 
{R } = { f } − { f Eq } = 3  −
2    =  − Pa 
L  − (2a − 3aL + L )    − 2a
2 3
3a 2
 3 + 2 P   
 − L ( a − L )    L L    L 
      
 0 
   − a 3
a 2
 

  L2 + L  P 
   

University of Castilla-La Mancha Draft By: Eduardo W. V. Chaves (2016)


Ciudad Real – Spain
6 LINEAR ELASTICITY 805

Case a (Figure 6.177(b)): For this case we will only obtain the consistent load vector. The
External Potential Energy, (see Figure 6.158), due to the concentrated moment is given by:
U ext = M yA θ yA (6.535)
Taking into account the derivative of the deflection function w , (see equation (6.451)),
when ( x = a ) :
 6x2 6x   6x2 6x   3x 2 4 x   3x 2 2 x 
θ y ( x) = − w1  3 − 2  − w2  − 3 + 2  + θ y1  2 − + 1 + θ y 2  2 − 
 L L   L L  L L  L L
 6a 2 6a   6 a 2 6a   3a 2 4a   3a 2 2a 
⇒ θ y ( x = a ) = − w1  3 − 2  − w2  − 3 + 2  + θ y1  2 − + 1 + θ y 2  2 − 
 L L   L L  L L  L L
Then,
  6a 2 6a   6 a 2 6a   3a 2 4a   3a 2 2a  
U ext = M yA  − w1  3 − 2  − w2 − 3 + 2  + θ y1  2 − + 1 + θ y 2  2 −  
 L L  
  L L   L L  L  L
To achieve the equilibrium, the following must be true:

∂Π ∂U int ∂U ext ∂U int  6 a 2 6a 


= − =0 ⇒ + M yA  3 − 2  = 0
∂w1 ∂w1 ∂w1 ∂w1  L L 

∂Π ∂U int ∂U ext ∂U int  3a 2 4a 


= − =0 ⇒ − M yA  2 − + 1 = 0
∂ θ y1 ∂ θ y1 ∂θ y1 ∂θ y1 L L 
∂Π ∂U int ∂U ext ∂U int  6 a 2 6a 
= − =0 ⇒ + M yA  − 3 + 2  = 0
∂w2 ∂w2 ∂w2 ∂w2  L L 

∂Π ∂U int ∂U ext ∂U int  3a 2 2a 


= − =0 ⇒ − M yA  2 −  = 0
∂θ y 2 ∂θ y 2 ∂θ y 2 ∂θ y 2 L L

Then, the consistent load vector becomes:

 − 6a 2 6 a   6 M yA ab 
 3 + 2   3 
 L2 L   A L 
 3a − 4a + 1 My b 
 2   L2 (b − 2a ) 
(e)
{ f Eq } = M yA  L 2 L  or (e)
{ f Eq } =  A  (6.536)
 6a − 6a   − 6 M y ab 
 L3 L2   L3 
 3a 2
2a   M Aa 
 2 −   y (a − 2b)
 L L   L2 

x
Case b: For this case the distributed load can be represented by qz ( x) = qz(1) + (qz( 2) − qz(1) ) ,
L
and the external potential energy becomes:
L L
 x 
0
∫ 0

U ext = q ( x)w( x)dx = qz(1) + (qz( 2 ) − qz(1) )w( x)dx
L 
Taking into account the deflection function w , (see equation (6.450)), and after the integral
is solved we can obtain:

University of Castilla-La Mancha Draft By: Eduardo W. V. Chaves (2016)


Ciudad Real - Spain
806 SOLVING PROBLEMS BY MEANS OF CONTINUUM MECHANICS

 3Lq z( 2 ) 7 Lq z(1)   − L2 q z( 2 ) L2 q z(1)   7 Lq z( 2) 3Lq z(1)   L2 qz( 2) L2 q z(1) 


U ext =  +  w1 +  −  θ y1 +  +  w2 + 
 20 + 30  θ y 2

 20 20   30
 20   20
 20   

To achieve the equilibrium, the following must be true:

∂Π ∂U int ∂U ext ∂U int  3Lq z( 2 ) 7 Lq z(1) 


= − =0 ⇒ − + =0 (6.537)
∂w1 ∂w1 ∂w1 ∂w1  20 20 

∂Π ∂U int ∂U ext ∂U int  − L2 q z( 2 ) L2 qz(1) 


= − =0 ⇒ − − =0 (6.538)
∂θ y1 ∂θ y1 ∂θ y1 ∂θ y1  30 20 

∂Π ∂U int ∂U ext ∂U int  7 Lq z( 2 ) 3Lq z(1) 


= − =0 ⇒ − + =0 (6.539)
∂w2 ∂w2 ∂w2 ∂w2  20 20 

∂Π ∂U int ∂U ext ∂U int  L2 qz( 2) L2 qz(1) 


= − =0 ⇒ − + =0 (6.540)
∂θy 2 ∂θy 2 ∂θy 2 ∂θ y 2  20 30 

Then, the consistent load vector becomes:


 L (1) ( 2) 
 20 (7 qz + 3qz ) 
 2 
 − L (3q (1) + 2q ( 2 ) )
 z z 
(e)
{ f Eq } =  60  (6.541)
L
 (3qz(1) + 7 qz( 2 ) ) 
 20 
 L2 
 (2qz(1) + 3qz( 2 ) ) 
 60 

Note that for the particular case when q z(1) = q z( 2) = qz , the above equation must match the
equation for { f Eq(e ) } given by the equation in (6.470).

University of Castilla-La Mancha Draft By: Eduardo W. V. Chaves (2016)


Ciudad Real – Spain
6 LINEAR ELASTICITY 807

Problem 6.67
Obtain the explicit equation [ Ke ( 2) ] {u( e ) } = { f Eq( e ) } for the beam presented in Figure 6.179.

y
z  v1 
θ 
 
{u (e ) } =  z1 
θ z1 , M z1 v2 , Q y 2  v2 
θz 2 , M z 2 θz 2 
v1 , Q y1

1 2 x
 Q y1 
M 
 
{ f } =  z1 
(e)
a) Displacements and nodal forces
 Qy 2 
M z 2 
N
[q y ] =
z, w y, v m
qy

x, u b) Load
1 2
L

Figure 6.179: Beam element.


Solution:
The problem presented here is similar to the one presented in Problem 6.62 – NOTE 2.
The displacement according to y -direction can be represented by v = ax 3 + bx 2 + cx + d ,
(see equation (6.440)). Then, we can obtain a similar equation presented in (6.449), i.e.:
  x 3  x
2
   x 3 x 
2
 x 3 2x 2   x3 x2 
v = v1 2  − 3  + 1 + v 2 − 2  + 3   + v1′  2 − + x  + v 2′  2 − 
  L  L    L   L   L L  L L
(6.542)
For this case we have that θ z = v′ , (see Figure 6.146). And the derivative of the above
equation with respect to x becomes
 6 x2 6x   6 x2 6x   3x 2 4 x   3x 2 2 x 
v′ = θz = v1  3 − 2  + v2 − 3 + 2  + θz1  2 − + 1 + θz 2  2 −  (6.543)
 L L   L L  L L  L L
and
12 x 6   12 x 6   6x 4   6x 2 
v′′ = v1  3 − 2  + v2  − 3 + 2  + θ z1  2 −  + θ z 2  2 −  (6.544)
L L   L L  L L L L
It will be useful to obtain analytically the following integrals:

University of Castilla-La Mancha Draft By: Eduardo W. V. Chaves (2016)


Ciudad Real - Spain
808 SOLVING PROBLEMS BY MEANS OF CONTINUUM MECHANICS

L
L L L2 L2

0
v( x) dx =
2
v1 + v2 + θ z1 − θ z 2
2 12 12
(6.545)
L
13L 2 13L 2 L3 2 L3 2 9 L 11L2 13L2
∫ v 2 dx =
35
v1 +
35
v2 +
105
θ z1 +
105
θz2 +
35
v1v2 +
105
v1 θ z1 −
210
v1 θ z 2
0 (6.546)
13L2 11L2 L3
+ v 2 θ z1 − v 2 θ z 2 − θ z1 θ z 2
210 105 70
L
6 6 2 2 L 2 2 L 2 12 1 1
∫ v′ dx = 5L v
2 2
1 + v2 + θ z1 + θz2 − v1v2 + v1 θ z1 + v1 θ z 2
5L 15 15 5L 5 5
0 (6.547)
1 1 L
− v 2 θ z1 − v 2 θ z 2 − θ z1 θ z 2
5 5 15
L
12 12 2 4 2 4 2 24 12 12
∫ v′′ dx = L
2
3
v12 + v + θ z1 + θ z 2 − 3 v1v2 + 2 v1 θ z1 + 2 v1 θ z 2
3 2
L L L L L L
0 (6.548)
12 12 4
− 2 v 2 θ z1 − 2 v2 θ z 2 + θ z1 θ z 2
L L L
L
3L2 7 L2 L3 L3

0
x v( x)dx =
20
v1 +
20
v2 +
30
θ z1 −
20
θz 2 (6.549)

We will follow the same procedure adopted from the equation (6.450) to (6.468) in order to
obtain:
L
L L L2 L2 
0

U ext = q y v( x)dx = q y  v1 + v 2 +
2 2 12
θ z1 −
12
θ z 2 

(6.550)

Considering that EI z is constant in the beam element, the internal potential energy
becomes:
L
EI z
U int =
2 0 ∫
v′′2 dx (6.551)

and:
L
EI z EI  12 12 4 4 24 12
U int =
2 0 ∫
v′′2 dx = z  3 v12 + 3 v22 + θz21 + θz22 − 3 v1v2 + 2 v1θz1 +
2 L L L L L L
(6.552)
12 12 12 4 
v θ − 2 v2 θz1 − 2 v2 θz 2 + θz1θz 2 
2 1 z2
L L L L 
Then, the total potential energy (6.458), Π = U int − U ext , can be written as follows:
EI z  12 2 12 2 4 2 4 2 24 12 12 12
Π=  3 v1 + 3 v2 + θ z1 + θ z 2 − 3 v1v 2 + 2 v1 θ z1 + 2 v1 θ z 2 − 2 v 2 θ z1
2 L L L L L L L L
(6.553)
12 4  L L L2
L2

− 2 v2 θ z 2 + θ z1 θ z 2  − q y  v1 + v2 + θ z1 − θ z 2 
L L  2 2 12 12 
As we are looking for the stationary state the following must hold:
∂Π EI z  24 24 12 12  L
=0 ⇒  3 v1 − 3 v2 + 2 θz1 + 2 θz 2  − q y = 0 (6.554)
∂v1 2 L L L L  2

University of Castilla-La Mancha Draft By: Eduardo W. V. Chaves (2016)


Ciudad Real – Spain
6 LINEAR ELASTICITY 809

∂Π EI z  8 12 12 4  L2
=0 ⇒  θz1 + 2 v1 − 2 v2 + θz 2  − q y =0 (6.555)
∂ θz1 2 L L L L  12
∂Π EI z  24 24 12 12  L
=0 ⇒  3 v2 − 3 v1 − 2 θz1 − 2 θz 2  − q y = 0 (6.556)
∂v2 2 L L L L  2

∂Π EI z  8 12 12 4  L2
=0 ⇒  z2θ + v1 − v2 + θ z1  + q y =0 (6.557)
∂ θz 2 2 L L2 L2 L  12

Restructuring the above set of equations in matrix form we can obtain:

 12 EI z 6 EI z − 12 EI z 6 EI z   qy L 
 L3  
L2 L3 L2  v   2 2 
 6 EI 4 EI z − 6 EI z 2 EI z   1   q y L 
 z 
 L
2
L L2 L  θ z1  =  12 
 − 12 EI z − 6 EI z 12 EI z − 6 EI z  v2   q y L 
   
 L3 L2 L3 L2  θ   2  (6.558)
 6 EI 2 EI z − 6 EI z 4 EI z   z 2   − q L2 
 z
 
y

 L2 L L2 L   12 

[ Ke ( 2 ) ] {u ( e ) } = { f Eq
(e)
}
where

 12 EI z 6 EI z − 12 EI z 6 EI z   qy L 
 L3  
L2 L3 L2   22 
 6 EI − 6 EI z 2 EI z 
 z 4 EI z
  qy L 
2
L2  
[ Ke ] =  L
( 2) L L  ; (e)
{ f Eq } =  12  (6.559)
 − 12 EI z − 6 EI z 12 EI z − 6 EI z 

qy L

 L3 L2 L3 L2   2 
 6 EI 2 EI z − 6 EI z 4 EI z   − q y L2 
 2
z
  
 L L L2 L   12 
NOTE: The consistent load vector related to the load described in Figure 6.180 (a) and
(b).

z y z y
P q y ( x)
a b q (y1)
x q (y2)

1 M Az 2 1 2 x
L L

a) Concentrated force and moment b) Linearly distributed load


Figure 6.180: Load lies on plane x − y .

For the case a) the External Potential Energy due to the concentrated force is given by:
U ext = Pv p = Pv( x = a )

University of Castilla-La Mancha Draft By: Eduardo W. V. Chaves (2016)


Ciudad Real - Spain
810 SOLVING PROBLEMS BY MEANS OF CONTINUUM MECHANICS

Taking into account the deflection function v , (see equation (6.542)), when ( x = a ) :
  x 3  x  2    x 3  x  2   x3 2 x 2   x3 x 2 
v = v1 2  − 3  + 1 + v2 − 2  + 3   + v1′  2 − + x  + v2′  2 − 
  L  L    L   L   L L  L L
 2a 3 3a 2   2a 3 3a 2   a 3 2a 2   a3 a2 
⇒ vP = v1  3 − 2 + 1 + v2 − 3 + 2  + θz1  2 − + a  + θz 2  2 − 
 L L   L L  L L  L L

where we have also considered the definition θ z = v ′ . Then,


  2a 3 3a 2   2a 3 3a 2   a 3 2a 2   a3 a 2  
U ext = P v1  3 − 2 + 1 + v2 − 3 + 2  + θz1  2 − + a  + θz 2  2 −  
 L L  
  L L   L L  L  L
To achieve equilibrium, the following must be true:

∂Π ∂U int ∂U ext ∂U int  2a 3 3a 2 


= − =0 ⇒ −  3 − 2 + 1 P = 0
∂v1 ∂v1 ∂v1 ∂v1  L L 
∂Π ∂U int ∂U ext ∂U int  a 3 2a 2 
= − =0 ⇒ − 2 − + a P = 0
∂θz1 ∂θz1 ∂ θ z1 ∂ θz1  L L 
∂Π ∂U int ∂U ext ∂U int  2a 3 3a 2 
= − =0 ⇒ − − 3 + 2  P = 0
∂v2 ∂v2 ∂v2 ∂v2  L L 

∂Π ∂U int ∂U ext ∂U int  a 3 a 2 


= − =0 ⇒ −  − P = 0
∂ θz 2 ∂ θ z 2 ∂ θz 2 ∂θz 2  L2 L 

Then, the consistent load vector becomes:


 2a3 3a 2  
 3 − 2 + 1 P   Pb 2 
 L L    3 (3a + b) 
 a 3
2a 2
   L 2 
  2 − + a  P   Pab 
 L L    
} = 
2
(e)
{ f Eq  or { f Eq } =  2 L
(e)
 (6.560)
  − 2a + 3a  P 
3 2
 Pa (a + 3b)
  L3 L2    L3 
   − Pa 2b 
  a a2 
3
   
  L2 − L  P   L2 
   
The External Potential Energy due to the concentrated moment is given by:
U ext = M zA θzA (6.561)
Taking into account the derivative of the deflection function v , (see equation (6.543)),
when ( x = a ) :
 6x2 6 x   6x2 6x   3x 2 4 x   3x 2 2 x 
θz ( x) = v1  3 − 2  + v2 − 3 + 2  + θz1  2 − + 1 + θz 2  2 − 
 L L   L L  L L  L L
 6a 2 6a   6 a 2 6a   3a 2 4a   3a 2 2a 
⇒ θz ( x = a ) ≡ θzA = v1  3 − 2  + v2  − 3 + 2  + θz1  2 − + 1 + θz 2  2 − 
 L L   L L  L L   L L

Then,

University of Castilla-La Mancha Draft By: Eduardo W. V. Chaves (2016)


Ciudad Real – Spain
6 LINEAR ELASTICITY 811

  6a 2 6a   6a 2 6 a   3a 2 4a   3a 2 2a  
U ext = M zA  v1  3 − 2  + v2 − 3 + 2  + θz1  2 − + 1 + θz 2  2 −  
 L L  
  L L   L L   L  L
To achieve the equilibrium, the following must be true:

∂Π ∂U int ∂U ext ∂U int  6a 2 6a 


= − =0 ⇒ − M zA  3 − 2  = 0
∂v1 ∂v1 ∂v1 ∂v1  L L 

∂Π ∂U int ∂U ext ∂U int  3a 2 4a 


= − =0 ⇒ − M zA  2 − + 1 = 0
∂θz1 ∂θz1 ∂ θ z1 ∂ θz1 L L 
∂Π ∂U int ∂U ext ∂U int  6a 2 6 a 
= − =0 ⇒ − M zA − 3 + 2  = 0
∂v2 ∂v2 ∂v2 ∂v2  L L 

∂Π ∂U int ∂U ext ∂U int  3a 2 2a 


= − =0 ⇒ − M zA  2 −  = 0
∂ θ z 2 ∂ θz 2 ∂ θz 2 ∂ θz 2 L L

Then, the consistent load vector becomes:


 6a 2 6a   − 6 M zAab 
 3 − 2   3 
 L2 L   A L 
 3a 4 a  M z b (b − 2a ) 
− + 1
A L   2 
2
(e)
{ f Eq } = M z  L or (e)
{ f Eq }=  L (6.562)
2  A 
 − 6 a 6 a   6 M ab 
+ z
 L3 L2   L3 
 3a 2 2a   M Aa 
 2 −   z2 (a − 2b)
 L L   L 
x
For the case b) the distributed load can be represented by q y ( x) = q (y1) + (q (y2) − q (y1) ) , and
L
the External Potential Energy becomes:
L L
 x 
0
∫ 0

U ext = q y ( x)v( x)dx = q (y1) + (q (y2 ) − q (y1) )v( x)dx
L 
Taking into account the deflection function y and after the integral is solved we can
obtain:
L L2 L L2
U ext = (3q (y2 ) + 7 q (y1) )v1 + (2q (y2) + 3q (y1) ) θz1 + (7 q (y2 ) + 3q (y1) )v2 + (−3q (y2 ) − 2q (y1) ) θz 2
20 60 20 60
To achieve the equilibrium, the following must be true:
∂Π ∂U int ∂U ext ∂U int  L 
= − =0 ⇒ −  (3q (y2) + 7 q (y1) ) = 0
∂v1 ∂v1 ∂v1 ∂v1  20 
∂Π ∂U int ∂U ext ∂U int  L2 
= − =0 ⇒ −  (2q (y2 ) + 3q (y1) ) = 0
∂ θz1 ∂ θz1 ∂θz1 ∂θz1  60 
∂Π ∂U int ∂U ext ∂U int
L 
= − =0 ⇒ −  (7 q (y2 ) + 3q (y1) ) = 0
∂v2 ∂v2 ∂v2 ∂v2  20 
∂Π ∂U int ∂U ext ∂U int  L2 
= − =0 ⇒ −  ( −3q (y2 ) − 2q (y1) ) = 0
∂ θz 2 ∂ θ z 2 ∂θ z 2 ∂θz 2  60 

University of Castilla-La Mancha Draft By: Eduardo W. V. Chaves (2016)


Ciudad Real - Spain
812 SOLVING PROBLEMS BY MEANS OF CONTINUUM MECHANICS

Then, the consistent load vector becomes:


 L (1) (2) 
 20 (7 q y + 3q y ) 
 2 
 L (3q (1) + 2q ( 2) ) 
 y y 
(e)
{ f Eq } =  60  (6.563)
L
 (3q y + 7 q y ) 
(1) (2)

 20 
 − L2 (1) ( 2) 
 ( 2q y + 3q y )
 60 
In Problem 6.66 we have obtained the consistent load vector for the case when the load is
lying on the plane x − z .

University of Castilla-La Mancha Draft By: Eduardo W. V. Chaves (2016)


Ciudad Real – Spain
6 LINEAR ELASTICITY 813

Problem 6.68
Obtain the explicit equation [ Ke (3) ] {u( e ) } = { f Eq(e ) } for the beam presented in Figure 6.181.
Consider u = ax + b as the approximation for the displacement according to x -direction.

y
u 
z {u (e ) } =  1 
u1 , N x1 u2 , N x 2 u2 

x  N x1 
1 2 { f (e) } =  
N x2 

a) Displacements and nodal forces


N
[qx ] =
z, w m
y, v
qx

x, u b) Load
1 2
L

Figure 6.181: Bar element.


Solution:
By applying the displacement ( u = ax + b ) at the beam nodes we can obtain:
x = 0 (u = u1 ) ⇒ u1 = b (6.564)
x = L (u = u2 ) ⇒ u2 = aL + b (6.565)
This results in the following set of equations:
 u1   0 1 a  Reverse a  − 1  1 − 1 u1 
 =     →b  =    (6.566)
u2   L 1 b    L − L 0  u2 
where the coefficients are:
−1 1
a= u1 + u2 (6.567)
L L
b = u1 (6.568)
By substituting the values of a , b into the equation of the displacement u = ax + b , we can
obtain:
 −1 1   x x
u ( x) = ax + b =  u1 + u 2  x + u1 = 1 − u1 + u 2
 L L   L L (6.569)
(e)
u ( x) = N1u1 + N 2 u 2 = [ N ]{u }

University of Castilla-La Mancha Draft By: Eduardo W. V. Chaves (2016)


Ciudad Real - Spain
814 SOLVING PROBLEMS BY MEANS OF CONTINUUM MECHANICS

in which we have considered that


 x   x  u1 
[ N ] = [N1 N 2 ] = 1 −    and {u ( e ) } =   (6.570)
 L   L  u 2 
The total potential energy is given by:
L L
1 N2
∫ ∫
int ext
Π =U −U = dx − q x u ( x)dx (6.571)
2 0 EA 0

As we are considering that qx is independent of x , the external potential energy becomes:


L L L
 x x  qL
0
∫ 0
∫ 0 
L ∫
U ext = qxu ( x) dx = q x u ( x)dx = q x 1 − u1 + u2  dx = x (u1 + u2 )
L  2
(6.572)

The internal potential energy becomes:


L L L L 2
1 N2 1 (u′EA)2 EA 2 EA  ∂  x x 
∫ ∫ u′ dx = ∫ ∫
int
U = dx = dx =  1 − u1 + u2   dx
2 0 EA 2 0 EA 2 0 2 0  ∂x  L  L 
L 2
EA  − u1 u2  (6.573)
= ∫
2 0  L
+  dx
L
EA 2
= (u1 − 2u1u2 + u22 )
2L
∂u
where we have consider that N = σA = EεA = E A ≡ Eu′A and that EA is constant in the
∂x
beam element. Note also that the strain and the stress are constant into the element since
the displacement field is a linear function, i.e.:
∂u ( x) ∂ ([ N ( x)]{u (e ) }) ∂[ N ( x)] ( e )
ε( x) = = = {u } = [ B ( x)]{u (e ) }
∂x ∂x ∂x
(6.574)
∂  x   x  (e)  − 1   1    u1  1  u1 
⇒ ε( x) = 1 −    {u } =        = [− 1 1]  = ε
∂x  L   L   L   L   u 2  L u 2 
as we can observe the strain is independent of x as well as the stress, since

[− 1 1] 1 
E u
σ = Eε = (6.575)
L u 2 
Taking into account the equations (6.572) and (6.573), the total potential energy (6.571) can
be written as follows:
EA 2 q L
Π = U int − U ext = (u1 − 2u1u 2 + u 22 ) − x (u1 + u 2 ) (6.576)
2L 2
As we are looking for the stationary state the following must hold:
∂Π EA qL
=0 ⇒ (2u1 − 2u2 ) − x = 0 (6.577)
∂u1 2L 2
∂Π EA qL
=0 ⇒ (2u2 − 2u1 ) − x = 0 (6.578)
∂u2 2L 2

Restructuring the above set of equations in matrix form we can obtain:

University of Castilla-La Mancha Draft By: Eduardo W. V. Chaves (2016)


Ciudad Real – Spain
6 LINEAR ELASTICITY 815

 qx L 
EA  1 − 1 u1   2 
   =   ⇔ [ Ke (3) ] {u ( e ) } = { f Eq
(e )
} (6.579)
L  − 1 1  u 2   q x L 
 2 
where
 qx L 
EA  1 − 1
( 3) (e)  2 
[ Ke ] =   ; { f Eq }= q L  (6.580)
L − 1 1   x 
 2 

NOTE 1: The consistent mass matrix


In Problem 5.24 we have obtain the consistent mass matrix which is given by:


[ M (e ) ] = ρ [ N ( x)]T [ N ( x)] dV
V
(6.581)

where ρ is the mass density. For this problem the matrix [N ] is given by the equation in
(6.570), then:
L L L

∫ ∫ ∫
] = ρ [ N ( x)] [ N ( x)] dAdx = ρA [ N ( x)] [ N ( x)]dx ≡ ρA [ Nn]dx ∫
(e) T T
[M
(6.582)
0 A
{ 0 0
=A

where
N  ( N ) 2 N1 N 2 
[ N ]T [ N ] ≡ [ Nn] =  1 [N1 N2 ] =  1  (6.583)
N2   N1 N 2 (N2 )2 
Then, after the integration (6.582) is taken place we can obtain:
L L

∫ ∫
2
L  ( N 1 ) dx N 1 N 2 dx 
 

[ M (e ) ] = ρA [ N ( x)]T [ N ( x)]dx = ρA 0L L
0

0
 0 ∫
 N 1 N 2 dx
0

( N 2 ) 2 dx 

 L x
2 L
 x  x  
 
 
1 ∫

L
 dx
0

1 −  dx 
L  L  
(6.584)
⇒ [ M ( e ) ] = ρA L 0 L 2 
 1 − x  x  dx x

 0  L  L  0
L ∫
  dx 

ρAL 2 1
⇒ [ M (e) ] =  
6 1 2 
which is known as the Consistent Mass Matrix.

University of Castilla-La Mancha Draft By: Eduardo W. V. Chaves (2016)


Ciudad Real - Spain
816 SOLVING PROBLEMS BY MEANS OF CONTINUUM MECHANICS

Problem 6.69
Obtain the explicit equation [ Ke ( 4) ] {u( e ) } = { f Eq( e ) } for the beam presented in Figure 6.182.
Consider θx = ax + b as the approximation for the rotation about the x -axis.

y
θ 
z {u (e ) } =  x1 
θx1 , M x1 θx 2 , M x 2  θx 2 

1 x  M x1 
2 { f (e)} =  
M x 2 

a) “Displacements” and nodal “forces”

Nm
[mTx ] =
z, w y, v m

mTx
x, u b) Load
1 2

Figure 6.182: Bar element.


Solution:
Note that the approximation for rotation is the same as the one used to approximate the
displacement in Problem 6.68. By analogy with the previous problem we can obtain that:
 x x
θx ( x) = 1 −  θx1 + θx 2 (6.585)
 L L
As we are considering that qx is independent of x , the external potential energy becomes:
L L
mTx L
∫ ∫
ext
U = mTx θx ( x)dx = mTx θx ( x)dx = ( θx1 + θx 2 ) (6.586)
0 0
2

The internal potential energy due to M T was obtained in Problem 6.62. For torsion
problem we have obtained that M T = θxGJT , then the internal potential energy becomes:
L L L
1 M T2 1 ( θxGJT ) 2 GJ GJ
∫ ∫ ∫
int
U = dx = dx = T θx2 dx = T ( θx21 − 2 θx1θx 2 + θx22 ) (6.587)
2 0 GJT 2 0 GJT 2 0 2L

The total potential energy is given by:


L L
1 M T2
∫ ∫
int ext
Π =U −U = dx − mTx θx ( x)dx
2 0 GJT 0
(6.588)
GJ m L 
= T ( θx21 − 2 θx1θx 2 + θx22 ) −  Tx ( θx1 + θx 2 ) 
2L  2 
As we are looking for the stationary state the following must hold:

University of Castilla-La Mancha Draft By: Eduardo W. V. Chaves (2016)


Ciudad Real – Spain
6 LINEAR ELASTICITY 817

∂Π GJT m L
=0 ⇒ (2 θx1 − 2 θx 2 ) − Tx = 0 (6.589)
∂ θx1 2L 2
∂Π GJT m L
=0 ⇒ (2 θx 2 − 2 θx1 ) − Tx = 0 (6.590)
∂ θx 2 2L 2

Restructuring the above set of equations in matrix form we can obtain:


 mTx L 
GJT  1 − 1  θx1   2 
− 1 1    =  m L  ⇔ [ Ke ( 4) ] {u ( e ) } = { f Eq
(e)
} (6.591)
L   θx 2   Tx 
 2 
where
 mTx L 
( 4) GJ  1 − 1 (e)  2 
[ Ke ]= T   ; { f Eq }= m L  (6.592)
L − 1 1   Tx 
 2 

University of Castilla-La Mancha Draft By: Eduardo W. V. Chaves (2016)


Ciudad Real - Spain
818 SOLVING PROBLEMS BY MEANS OF CONTINUUM MECHANICS

Problem 6.70
(e) (e) (e)
Obtain the explicit equation [ K Local ] {uLocal } = {FEq _ L } for the beam presented in Figure

6.183. Consider as material/geometric characteristic of the transversal cross section as the


one described in Figure 6.184 and as external load the one described in Figure 6.189.

a) Degrees-of-freedom (“displacements”)
 u1  (1)
z  v  (2)
y
 1
 w1  (3)
 
(6) θ z1 (12) θ z 2  θx1  (4)
θ y1 (5) θ y 2 (11)  θ y1  (5)
 
(3) w1 v1 (2) (9) w2 v2 (8) (e)  θz1  (6)
{uLocal } =  
θ x1 (4) θ x 2 (10) x  u2  (7)
u1 (1) u 2 (7)  v2  (8)
1 2  
 w2  (9)
 θ  (10)
L  x2 
θ y 2  (11)
 θ  (12)
 z2 

b) Nodal force vector


 N x1  (1)
z Q 
y (2)
 y1 
 Q z1  (3)
 
M z1 M y1 M z2 M y2  M x1  (4)
 M y1  (5)
Q y1 Qy 2  
Qz 1 Qz 2 (e)  M z1  (6)
{ f Local } =  
M x1 M x2 x  Nx2  (7)
N x1  Qy  (8)
Nx2  2
1 2
 Qz 2  (9)
M  (10)
L  x2 
M y 2  (11)
M  (12)
 z2 

Figure 6.183: Beam element – local system.

University of Castilla-La Mancha Draft By: Eduardo W. V. Chaves (2016)


Ciudad Real – Spain
6 LINEAR ELASTICITY 819

z, x3 A − Cross-sectional area
I y − Inertia moment of area about the y -direction

I z − Inertia moment of area about z -direction


J Eq − Equivalent polar moment of inertia
y , x2 E − Young’s modulus
G − Shear modulus
Cross section ν − Poisson’s ratio
E
G=
2(1 + ν )

Figure 6.184: Material/Geometrical properties of the beam cross section.

Solution:
As we are dealing with linear elasticity we can apply the superposition principle. Let us
consider the stiffness matrices obtained previously, (see Figure 6.185, Figure 6.186, Figure
6.187 and Figure 6.188).

(3) (5) (9) (11)


1  12 EI y − 6 EI y − 12 EI y − 6 EI y  (3)
 3 
 L L2 L3 L2 
 − 6 EI y 4 EI y 6 EI y 2 EI y 
(5)
z  2
L L2 L 
(1)
[ k Local ]= L
y − 12 EI y 6 EI y 12 EI y 6 EI y  (9)
 
 L3 L2 L3 L2 
 − 6 EI y 2 EI y 6 EI y 4 EI y  (11)
 
 L2 L L2 L 
w1 , Q z 1 (3) w2 , Q z 2 (9)
θ y1 , M y 1 (5) θ y 2 , M y 2 (11)

Figure 6.185: See Problem 6.62- NOTE 2.

University of Castilla-La Mancha Draft By: Eduardo W. V. Chaves (2016)


Ciudad Real - Spain
820 SOLVING PROBLEMS BY MEANS OF CONTINUUM MECHANICS

(2) (6) (8) (12)


z 2  12 EI z 6 EI z − 12 EI z 6 EI z  (2)
 L3 L2 L3 L2 
 6 EI 4 EI z − 6 EI z 2 EI z 
y  z  (6)
2
( 2)
[ k Local ]=  L L L2 L 
 − 12 EI z − 6 EI z 12 EI z − 6 EI z 
 L3 (8)
L2 L3 L2 
 6 EI 2 EI z − 6 EI z 4 EI z 
θ z1 , M z 1 (6) θz2 , M z 2 (12)  z
 (12)
 L2 L L2 L 
v1 , Q y 1 (2) v 2 , Q y (8)
2

Figure 6.186: See Problem 6.67.

(1) (7)
z y 3
 EA − EA  (1)
( 3)  L L 
[k Local ]=  − EA EA 
  (7)
x  L L 
u1 , N x 1 (1) u 2 , N x 2 (7)

Figure 6.187: See Problem 6.68.

(4) (10)
z y 4  GJ Eq − GJ Eq  (4)
 
( 4)
[k Local ]=  L L 
 − GJ Eq GJ Eq 
(10)
x  L L 
θ x1 , M x 1 (4) θ x 2 , M x 2 (10)

Figure 6.188: See Problem 6.69.

University of Castilla-La Mancha Draft By: Eduardo W. V. Chaves (2016)


Ciudad Real – Spain
6 LINEAR ELASTICITY 821

The consistent load vectors can be appreciated in Figure 6.189.

a) External load  qz L 
  (3)
z, w N  2 
[qz ] =
y, v m  − q z L2 
(1)  12  (5)
qz { f Eq _ L} =  
 qz L  (9)
1  2 
x, u  2 
 qz L  (11)
 12 

b) External load  qy L 
  (2)
N
[q y ] =  2 
z, w y, v m  q y L2 
  (6)
qy ( 2)  12 
{ f Eq _ L} =  
 qy L  (8)
2 x, u  2 
 2 
 − qy L 
 12  (12)

c) External load
N
[qx ] =
z, w m
y, v
qx  qx L 
( 3)  2  (1)
{ f Eq _L } = q L 
3  x  (7)
 2 
x, u

d) External load
Nm
[mTx ] =
z, w y, v m
 mTx L  (4)
mT ( 4)  2 
{ f Eq _ L} = 
4 m L
 Tx  (10)
 2 
x, u

Figure 6.189: The consistent load vectors (Local system).

University of Castilla-La Mancha Draft By: Eduardo W. V. Chaves (2016)


Ciudad Real - Spain
822 SOLVING PROBLEMS BY MEANS OF CONTINUUM MECHANICS

Then, the stiffness matrix and consistent load vector can be obtained by:
4 4

A A{ f
(e ) (i ) (e ) (i )
[ K Local ]= [k Local ] ; {FLocal }= Eq _ L } (6.593)
i =1 i =1

where A
stands for assemble operator. Making the contribution to the respective degree-
of-freedom we can obtain the local stiffness matrix:
 EA − EA 
 L 0 0 0 0 0 0 0 0 0 0 
L
 12 EI z 6 EI z − 12 EI z 6 EI z 
 0 0 0 0 0 0 0 0 
 L3 L2 L3 L2 
 12 EI y − 6 EI y − 12 EI y − 6 EI y 
 0 0
L3
0
L2
0 0 0
L3
0
L2
0 
 GJ T − GJ T 
 0 0 0 0 0 0 0 0 0 0 
 L L 
 − 6 EI y 4 EI y 6 EI y 2 EI y 
 0 0
L2
0
L
0 0 0
L2
0
L
0 
 6 EI z 4 EI z − 6 EI z 2 EI z 
 0 0 0 0 0 0 0 0 
[K ]
(e)
Local =
 − EA
L2 L
EA
L2 L 

 L 0 0 0 0 0 0 0 0 0 0 
L
 − 12 EI z − 6 EI z 12 EI z − 6 EI z 
 0 0 0 0 0 0 0 0 
 L3 L2 L3 L2 
 − 12 EI y 6 EI y 12 EI y 6 EI y 
 0 0
L3
0
L2
0 0 0
L3
0
L2
0 
 − GJ T GJ T 
 0 0 0 0 0 0 0 0 0 0 
 L L 
 − 6 EI y 2 EI y 6 EI y 4 EI y 
 0 0
L2
0
L
0 0 0
L2
0
L
0 
 6 EI z 2 EI z − 6 EI z 4 EI z 
 0 0 0 0 0 0 0 0 
 L2 L L2 L 

And the consistent load vector by:


 qx L 
  (1)
 2 
 qyL  (2)
 
 q2 L 
 z  (3)
 2 
 mTx L  (4)
 2 
 2

 − qz L 
 12  (5)
 2 
 qyL 
(e )  12  (6)
{FEq _ L} =  
q L
 x  (7)
 2 
 qyL 
  (8)
 2 
 qz L 
 2  (9)
 m L 
 Tx  (10)
 2 
 q L2 
 z  (11)
 12 
 − q L2 
 y  (12)
 12 

University of Castilla-La Mancha Draft By: Eduardo W. V. Chaves (2016)


Ciudad Real – Spain
6 LINEAR ELASTICITY 823

NOTE 1: Stiffness Matrix related to the Global System


(e ) (e )
Knowing the vectors {FGlobal } and {U Global } in the global system, the transformation law
between the global and local systems is represented by the matrix [λ] , in which fulfils that
(e) (e) (e) (e)
{FLocal } = [λ]{FGlobal } and {uLocal } = [λ]{U Global } . With that we can conclude that:
(e ) (e) (e)
{FLocal } = [ K Local ]{uLocal }
(e) (e) (e)
⇒ [λ ]{FGlobal } = [ K Local ][λ]{U Global }
(e)
⇒ [λ ]T [λ]{FGlobal (e)
} = [λ]T [ K Local (e)
][λ]{U Global } (6.594)
(e)
⇒ {FGlobal (e)
} = [λ]T [ K Local (e)
][λ]{U Global }
(e) (e ) (e)
⇒ {FGlobal } = [ K Global ]{U Global }
(e ) (e)
where [ K Global ] = [λ]T [ K Local ][λ] .

z // z

k - ( X k , Yk , Z k )
y
x
z
j - ( X j ,Y j , Z j )

i
( X i , Yi , Z i )
L = ( X j − X i ) 2 + (Y j − Yi ) 2 + ( Z j − Z i ) 2

Z
( X , Y , Z ) - Global system
Y
( x, y, z ) - Local system

Figure 6.190: Beam element in 3D space.


Next, we will define the transformation matrix [λ] .
We need to define three points, namely: i − j − k
Local system x − y − z

ƒ x -direction: according to ij // x -direction.


The unit vector according to x' -direction is defined by:

University of Castilla-La Mancha Draft By: Eduardo W. V. Chaves (2016)


Ciudad Real - Spain
824 SOLVING PROBLEMS BY MEANS OF CONTINUUM MECHANICS

X j − Xi Y j − Yi Z j − Zi
a11 = ; a12 = ; a13 =
L L L
where
L = ( X j − X i ) 2 + (Y j − Yi ) 2 + ( Z j − Z i ) 2

ƒ The unit vector according to ik -direction.


Xk − Xi Yk − Yi Z k − Zi
p1 = ; p2 = ; p3 =
d (ik ) d (ik ) d (ik )
where
d (ik ) = ( X k − X i ) 2 + (Yk − Yi ) 2 + ( Z k − Z i ) 2

ƒ y -direction: perpendicular to the surface defined by ĵ = ik ∧ ij


ˆi ˆj kˆ
yˆ = p1 p2 p3 = ( p 2 a13 − p3 a12 )ˆi − ( p1 a13 − p3 a11 )ˆj + ( p1 a12 − p 2 a11 )kˆ
1442443 144 42444 3 1442443
a11 a12 a13 a21 a22 a23

ƒ z -direction: according to the convention kˆ = ˆi ∧ ˆj


ˆi ˆj kˆ
z = a11 a12 a13 = (a12 a 23 − a13 a 22 )ˆi − (a11 a 22 − a12 a 21 )ˆj + (a11 a 22 − a12 a 21 )kˆ .
1442443 144 42444 3 1442443
a 21 a 22 a 23 a31 a32 a33

The transformation matrix from the global system X − Y − Z to the local system x − y − z
is constituted by the unit vectors ˆi − ˆj − kˆ , i.e.:
 a11 a12 a13 
a ij =  a 21 a 22 a 23 
 a 31 a 32 a 33 
Then, the transformation matrix [λ] is represented as follows:
 a11 a12 a13 0 0 0 0 0 0 0 0 0 
a a 22 a 23 0 0 0 0 0 0 0 0 0 
 21
 a 31 a 32 a 33 0 0 0 0 0 0 0 0 0 
 
 0 0 0 a11 a12 a13 0 0 0 0 0 0 
 0 0 0 a 21 a 22 a 23 0 0 0 0 0 0 
 
0 0 0 a 31 a 32 a 33 0 0 0 0 0 0 
[λ] =  (6.595)
0 0 0 0 0 0 a11 a12 a13 0 0 0 
 
 0 0 0 0 0 0 a 21 a 22 a 23 0 0 0 
 
 0 0 0 0 0 0 a 31 a 32 a 33 0 0 0 
 0 0 0 0 0 0 0 0 0 a11 a12 a13 
 
 0 0 0 0 0 0 0 0 0 a 21 a 22 a 23 
 0 0 0 0 0 0 0 0 0 a 31 a 32 a 33 

University of Castilla-La Mancha Draft By: Eduardo W. V. Chaves (2016)


Ciudad Real – Spain
6 LINEAR ELASTICITY 825

NOTE 2: Note that the adopted stiffness matrix will depend on the degree-of-freedom of
the structure, which in turns depends on the external load and how the structure was
conceived. Some examples follow.
NOTE 2.1: Truss
Truss is a structural element which is subjected only by traction or compression forces and
the nodes are free to rotate, i.e. there is no moment at the nodes. The external load
(concentrated force) is considered to be applied only at the nodes. Examples for this
problem we can quote: an electricity transmission tower (electricity pylon), geodesic dome,
among others, (see Figure 6.191).

a) Geodesic dome b) Electricity transmission tower

Figure 6.191: Truss examples.

The degrees-of-freedom associated with this type of structure are only translations. For
instance, in Problem 6.68 we have defined this problem by considering one-dimensional
space (1D), in case we are dealing with two-dimensional space there will be two degrees-of-
freedom per node ( u , v ), (see Figure 6.193), and for three-dimensional space (3D) there are
3 degrees-of-freedom per node namely: u , v , and w . (see Figure 6.194).

University of Castilla-La Mancha Draft By: Eduardo W. V. Chaves (2016)


Ciudad Real - Spain
826 SOLVING PROBLEMS BY MEANS OF CONTINUUM MECHANICS

a) Displacements and nodal forces – 1D (Local system)


(e) (e) (e)
{FLocal } = [k Local ]{uLocal }
u1 , f x1 u2 , f x 2
 f x1  EA  1 − 1  u1 
1 x, X  =   
2  f x 2  L  − 1 1  u2 

b) Two-dimensional space – 2D

Y2 L = ( X 2 − X 1 ) 2 + (Y2 − Y1 ) 2
y 2
L X 2 − X1
l = cosα =
L
Y1 α Y2 − Y1
1 m = sinα =
L

X1 X2 X

c) Three-dimensional space – 3D

Z x
Y
 L = ( X − X ) 2 + (Y − Y ) 2 + ( Z − Z ) 2
2  2 1 2 1 2 1

 X 2 − X1
L l = L
Z2 
 Y2 − Y1
Y2 m =
 L
 Z 2 − Z1
1 n =
Z1  L
Y1

X1 X2
X

Figure 6.192: Bar element.


Note that the local stiffness matrix for truss element was obtained in Problem 6.68. And if
(e) (e) (e) (e)
we consider that {uLocal } = [A ]{uGlobal } and { f Local } = [A ]{ f Global } where [A ] is the
transformation matrix from the Global system to the Local system we can obtain:
(e) (e) (e) (e) (e) (e)
{ f Local } = [k Local ]{uLocal } ⇒ [A ]{ f Global } = [ k Local ][A ]{uGlobal }
(e)
⇒ [A ]T [A ]{ f Global (e)
} = [A ]T [k Local (e )
][A ]{uGlobal }
(e )
⇒ { f Global } = [A ]T [k Local
(e) (e )
][A ]{uGlobal }

University of Castilla-La Mancha Draft By: Eduardo W. V. Chaves (2016)


Ciudad Real – Spain
6 LINEAR ELASTICITY 827

(e) (e) (e) (e )


⇒ { f Global } = [kGlobal ]{uGlobal } ∴ [kGlobal ] = [A ]T [k Local
(e)
][A ] (6.596)
Two-Dimensional Space (2D)
Stiffness Matrix for 2D
For the local system the force-displacement relationship, (see Figure 6.193), is given by:
 f x1  1 0 − 1 0 u1 
 f = 0
(e) (e) (e)  y1

 EA  0 0 0 0  v1 
{ f Local } = [k Local ]{uLocal } ⇔   =   (6.597)
 f x 2  L − 1 0 1 0 u 2 
 f y 2 = 0  
  0 0 0 0 v2 

Transformation Matrix
The transformation matrix from the Global system X − Y to the Local system x − y , (see
Figure 6.192(b)), is given by:
 cosα sin α   l m
[A ] =  = (6.598)
− sin α cosα   − m l 
(e )
And the transformation matrix for the displacement vector {uLocal } becomes

 u1   l m 0 0  u X 1 
 v  − m l 0 0   vY 1 
u   l m  u Xi   1 
 → i  = 
Nodal Element
 → =
 vi   − m l   vYi   
u 2   0 0 l m  u X 2  (6.599)
v2   0 0 − m l   vY 2 
14444442444444 3
(e) (e)
{uLocal }=[ A ]{uGlobal }

Y uY 2 , fY 2 x  u1   f x1 
v   0 
v2 (e )
{uLocal
 
} =  1 ;{ f Local
(e)  
}=  
y u2 , f x 2  u 2  f x2 
v2   0 
Y2 uY 1 , fY 1
u X 1   f X1 
2 uX 2 , f X 2 v  f 
v1    
u1, f x1
(e )
{uGlobal } =  Y 1 ;{ f Global
( e)
} =  Y1 
u X 2   fX 2
Y1  vY 2   fY 2 
1 u X 1, f X 1

X1 X2 X

Figure 6.193: Bar element in two-dimensional space – degrees-of-freedom.


Then, the stiffness matrix in the Global system can be expressed as follows:
T
 l m 0 0 1 0 − 1 0  l m 0 0
 0  0 0 0 0 − m 0 
(e) EA  − m l 0  l 0
[kGlobal ] = [A ]T [k Local
(e)
][A ] =
L  0 0 l m − 1 0 1 0  0 0 l m

 0 0 −m l  
0

0 0 0  0 0 −m l 

University of Castilla-La Mancha Draft By: Eduardo W. V. Chaves (2016)


Ciudad Real - Spain
828 SOLVING PROBLEMS BY MEANS OF CONTINUUM MECHANICS

l 2 lm −l 2
− lm 
 
(e) EA  lm m2 − lm − m2
[kGlobal ]= (6.600)
L − l 2 − lm l2 lm 
 
 − lm − m2 lm m 2 
Strain and Stress (2D)
Once the global displacement vector is defined, the strain, (see equation (6.574)), can be
obtained as follows:
u X 1 
l m 0 0   vY 1 
[− 1 1] 1  = 1 [− 1 1]
1 u 1
ε=    ⇒ ε = (−u X 1l − vY 1m + u X 2 l + vY 2 m )
L u 2  L 0 0 l m  u X 2  L
 vY 2 
(6.601)
where we have considered, (see equation (6.599)), that
u X 1 
 u1  l m 0 0   vY 1 
 = (6.602)
u 2   0 0 l m  u X 2 
 vY 2 

And the stress becomes


E
σ = Eε = [(u X 2 − u X 1 )l + (vY 2 − vY 1 )m ] (6.603)
L
f x 2 EA E
Note that σ = = [(u X 2 − u X 1 )l + (vY 2 − vY 1 )m ] = [(u X 2 − u X 1 )l + (vY 2 − vY 1 )m ] .
A LA L
The Reactions and internal forces can also be obtained in the local system as follows
(e) (e) (e) (e ) (e) (e) (e)
{rLocal } = −{ f int } = −{ f Eq _ L } + { f Local } = −{ f Eq _ L } + [ k Local ]{u Local }

(e) (e) (e)


{ f Local } = [k Local ]{uLocal }
 f x1   1 0 − 1 0  u1   1 0 − 1 0  l m 0 0  u X 1 
         
 f y1 = 0  EA  0 0 0 0  v1  EA  0 0 0 0  − m l 0 0   vY 1 
  =   =
 f x 2  L  − 1 0 1 0 u2  L − 1 0 1 0  0 0 l m  u X 2 
   
 f y 2 = 0
   0 0 0 0 v2   0 0 0 0  0 0 −m l   vY 2 
 f x1  (u X 1 − u X 2 )l + (vY 1 − vY 2 )m 
   
 f y1 = 0  EA  0 
⇒ =   (6.604)
 f x 2  L (u X 2 − u X 1 )l + (vY 2 − vY 1 )m 
 f y 2 = 0  0 
 
Note that for the truss problem we do not have the consistent load vector, since the load
(e) (e)
must be applied at the nodes, so {rLocal } = −{ f int } = { f (e ) } = [k Local
(e) (e)
]{uLocal }.

It could be interesting to generate an internal global internal vector, related to Global


(e) (e) (e ) (e) (e) T (e)
system, i.e.: {rGlobal } = −{ f Eq _ G } + [ k Global ]{uGlobal } or we can use {rGlobal } = [A ] {rLocal } :

University of Castilla-La Mancha Draft By: Eduardo W. V. Chaves (2016)


Ciudad Real – Spain
6 LINEAR ELASTICITY 829

T
 l m 0 0  f x1   f x1l 
− m    
l 0 0   f y1 = 0   f x1m 
(e)
{rGlobal }=   = 
 0 0 l m  f x2   f x2l 

 0 0 −m l   f y 2 = 0  f m 
   x2 
Three-Dimensional Space (3D)
Stiffness Matrix for 3D
For the local system the force-displacement relationship is given by:
 f x1  1 0 0 − 1 0 0  u1 
 f = 0 0 0 0 0 0 0  v1 
 y1  
(e) (e) (e)
 f z1 = 0  EA  0 0 0 0 0 0  w1 
{ f Local } = [k Local ]{u Local } ⇔  =    (6.605)
 f x 2  L − 1 0 0 1 0 0  u 2 
 f y 2 = 0 0 0 0 0 0 0  v2 
    
 f z 2 = 0   0 0 0 0 0 0 w2 
Transformation Matrix
The transformation matrix from the Global system X − Y − Z to the Local system x − y − z
is given by:
 a11 a12 a13   l m n 
[A ] = a21 a22 a23  ≡ a21 a22 a23  (6.606)
 a31 a32 a33   a31 a32 a33 
(e )
And the transformation matrix for the vector {uLocal } becomes

 u1   l m n  u X 1 
   
→ v1  =  a21
Nodal
a 22 a 23   vY 1 
w   a a32 a33  wZ 1 
 1   31
 u1   l m n 0 0 0 u X 1 
 v  a a22 a 23 0 0 0   vY 1  (6.607)
 1   21
Element  w1   a31 a32 a33 0 0 0   wZ 1  (e ) (e )
 →  =  ⇒ {uLocal } = [A ]{uGlobal }
u2   0 0 0 l m n  u X 2 
 v2   0 0 0 a21 a 22 a 23   vY 2 
    
w2   0 0 0 a31 a32 a33  wZ 2 
Then, the stiffness matrix in the Global system can be expressed as follows:
1 0 0 − 1 0 0
0 0 0 0 0 0

(e) EA 0 0 0 0 0 0
[kGlobal ] = [A ]T [k Local
(e)
][A ] = [A ]T  [A ]
L − 1 0 0 1 0 0
0 0 0 0 0 0
 
 0 0 0 0 0 0
after the matrix multiplication is taken place we can obtain

University of Castilla-La Mancha Draft By: Eduardo W. V. Chaves (2016)


Ciudad Real - Spain
830 SOLVING PROBLEMS BY MEANS OF CONTINUUM MECHANICS

l 2 lm ln −l 2 − lm − ln 
 
 lm m2 mn − lm − m2 − mn 
(e) EA  ln mn n2 − ln − mn − n2 
[kGlobal ]=   (6.608)
L − l 2 − lm − ln l 2
lm ln 
 − lm − m2 − mn lm m2 mn 

 − ln − mn − n2 ln mn n 2 
Note that we do not need to define the coefficients a21 , a22 , L , a33 in order to obtain the
Global elemental stiffness matrix.
Strain and Stress (3D)
Once the global displacement vector is defined the strain, (see equation (6.574)), can be
obtained as follows:
uX 1 
v 
 Y1 
l m n 0  wZ 1 
[− 1 1] 1  = 1 [− 1 1]
1 u 0 0
ε=
L u2  L 0 0 0 l m n  u X 2 
 vY 2 
 
wZ 2 

1
⇒ε= (−u X 1l − vY 1m − wZ 1n + u X 2 l + vY 2 m + wZ 2 n ) (6.609)
L
where we have considered that
u X1 
v 
 Y1 
 u1  l m n 0 0 0  wZ 1 
 = (6.610)
u 2   0 0 0 l m n  u X 2 
 vY 2 
 
wZ 2 

And the stress becomes


E
σ = Eε = (−u X 1l − vY 1m − wZ 1n + u X 2 l + vY 2 m + wZ 2 n ) (6.611)
L
f x 2 EA E
Note that σ = = [(u X 2 − u X 1 )l + (vY 2 − vY 1 )m ] = [(u X 2 − u X 1 )l + (vY 2 − vY 1 )m ] .
A LA L
The internal forces can also be obtained in the local system as follows
(e) (e ) (e) (e) (e)
{ f Local } = [k Local ]{uLocal } = [k Local ][A ]{uGlobal }
 f x1  1 0 0 − 1 0 0  l m n 0 0 0 u X 1 
  
 f y1 = 0 
0
 0 0 0 0 0 a21 a22 a23 0 0 0   vY 1 
 f z1 = 0  EA  0 0 0 0 0 0  a31 a32 a33 0 0 0   wZ 1 
= 

 f x 2  L − 1

0 0 1 0 0  0 0 0 l m n  u X 2 
 f y 2 = 0 0 0 0 0 0 0  0 0 0 a21 a22 a23   vY 2 
     
 f z 2 = 0   0 0 0 0 0 0  0 0 0 a31 a32 a33  wZ 2 

University of Castilla-La Mancha Draft By: Eduardo W. V. Chaves (2016)


Ciudad Real – Spain
6 LINEAR ELASTICITY 831

 f x1   (u X 1 − u X 2 )l + (vY 1 − vY 2 )m + ( wZ 1 − wZ 2 )n 
   
 f y1 = 0   0 
 f z1 = 0  EA  0 
⇒ =   (6.612)
 f x 2  L − [(u X 1 − u X 2 )l + (vY 1 − vY 2 )m + ( wZ 1 − wZ 2 )n ]
 f y 2 = 0  0 
   
 f z 2 = 0   0 

Note that for the truss problem we do not have the consistent load vector, since the load
(e) (e)
must be applied at the nodes, so {rLocal } = −{ f int } = { f (e ) } = [k Local
(e ) (e)
]{uLocal }.

It could be interesting to generate an internal global internal vector, related to Global


(e) (e) (e ) (e) (e) T (e)
system, i.e.: {rGlobal } = −{ f Eq _ G } + [ k Global ]{uGlobal } or we can use {rGlobal } = [A ] {rLocal } :

T
l m n 0 0 0   f x1   f x1l 
a    
 21 a 22 a 23 0 0 0   0   f x1m 
(e)
 a31 a32 a33 0 0 0   0   f x1n 
{rGlobal }=   =
0 0 0 l m n   f x2   f x2l 

0 0 0 a 21 a22 a23   0   f m
     x2 
 0 0 0 a31 a32 a33   0   f x 2 n 

x
u Z 2 , f Z 2 uY 2 , fY 2
Z
Y

uX 2 , f X 2 uX1   f X1 
2 v  f 
uZ1 , f Z1 uY 1, fY 1  Y1   Y1 

w    f 
(e)
{uGlobal } =  Z 1  ; { f Global
(e)
} =  Z1 
u X 2   fX2
 vY 2   fY 2 
u X 1, f X 1    
1 wZ 2   f Z 2 

Figure 6.194: Bar element in three-dimensional space.


NOTE 2.2: Slab Floor compound by beams only
If we consider a Slab Floor in which only the beams are taken into account, (see Figure
6.195), the degrees-of-freedom associated with a node are characterized by 1 translation
and 2 rotations, i.e. w , θ x , θ y , (Figure 6.196).
For this case the stiffness matrix is made up by combining the problems described in
Figure 6.185 and Figure 6.188. And as external loads we will consider those described in
Figure 6.189 (a)+(d). Then, if we consider the structure characterized by the degrees-of-
freedom described in Figure 6.196 we can obtain:

University of Castilla-La Mancha Draft By: Eduardo W. V. Chaves (2016)


Ciudad Real - Spain
832 SOLVING PROBLEMS BY MEANS OF CONTINUUM MECHANICS

 12 EI y − 6 EI y − 12 EI y − 6 EI y   qz L 
 0 0 
3 2 3
L2   2 
 L L L
m L 
 0 GJ Eq − GJ Eq
0 0 0   w1   Tx 
 L L    2 
 − 6 EI y 4 EI y 6 EI y 2 EI y   θ x1   − q z L2 
 0 0  
 L
2
L L2 L   θ y1  =  12 
 − 12 EI y 6 EI y 12 EI y 6 EI y   w2   q z L  (6.613)
0 0    
 
 L
3
L2 L3 L2   θ x 2   2 
− GJ Eq GJ Eq    mTx L 
 0 0 0 0  θ y 2   2 
 L L   2 
 − 6 EI y 2 EI y 6 EI y 4 EI y   qz L 
 0 0   12 
 L2 L L2 L 
(e) (e) (e)
[ k Local ] {uLocal } = { f Eq _ L}

qZ = q z

mT
Z, z
z
y x
X

Figure 6.195: Slab floor by considering beams only.

The matrix transformation from the Global system X − Y − Z to the local system x − y − z ,
(see Figure 6.197), is given by:
 a11 a12 a13   l m 0
[A ] = a21 a22 a23  = − m l 0 (6.614)
 a31 a32 a33   0 0 1

University of Castilla-La Mancha Draft By: Eduardo W. V. Chaves (2016)


Ciudad Real – Spain
6 LINEAR ELASTICITY 833

a) Degrees-of-freedom (Displacements)
z y  w1  (1)
 θ  (2)
 x1 
 θ y1  (3)
θ y1 (3) θ y 2 (6) (e)
{uLocal }=  
 w2  (4)
(1) w1 (4) w2  θx 2  (5)
θ x1 (2) x  
θ y 2  (6)
1 2 θ x 2 (5)

z y b) Nodal force vector

M y2  Q z1  (1)
M y1 M 
 x1  (2)
Qz 1 Qz 2
M x1 M x2 x (e)  M y1  (3)
{ f Local }=  
 Qz 2  (4)
1 2 M x 2  (5)
 
M y 2  (6)
L

Figure 6.196: Slab floor beam – local system.

Z
Y
w2 = wZ 2 θY 2 x

z θy 2 θx 2
Y2
w1 = wZ 1 2 θX 2
θY 1
y L = ( X 2 − X 1 ) 2 + (Y2 − Y1 ) 2
θ y1 θx1 α
Y1 X 2 − X1
l = cosα =
1 θX 1 L
Y2 − Y1
m = sinα =
X1 X2 X L

Figure 6.197: Bar element in three-dimensional space.

University of Castilla-La Mancha Draft By: Eduardo W. V. Chaves (2016)


Ciudad Real - Spain
834 SOLVING PROBLEMS BY MEANS OF CONTINUUM MECHANICS

The transformation matrix for the nodal displacement vector is given by:
 w1  1 0 0  wZ 1 = w1 
 
→ θ x1  = 0 l
Nodal
m   θ X 1 
θ  0 − m l   θY 1 
 y1  
 w1  1 0 0 0 0 0   w1 
θ    
 x1  0 l m 0 0 0   θ X 1  (6.615)
 θ y1  0 − m l 0 0 0   θY 1 
Element
 →  =  
(e) (e)
 ⇒ {uLocal } = [A ]{uGlobal }
 w2  0 0 0 1 0 0   w2 
 θ x 2  0 0 0 0 l m  θ X 2 
    
θ y 2  0 0 0 0 −m l   θY 2 
By considering that:
 6 EI y
 f 0 −a − f 0 − a a = 2
 L
 0 b 0 0 − b 0 
  GJ Eq
− a 0 2d a 0 d  b =
(e) L
[k Local ] =   with  (6.616)
− f 0 a f 0 a   f = 12 EI y
 0 −b 0 0 b 0   L3
  
 − a 0 d a 0 2d  d = 2 EI y
 L
the stiffness matrix in the Global system, (Chaves&Mínguez(2010)), can be expressed as
follows:
 f 0 −a − f 0 − a
 0 b 0 0 − b 0 

(e)
− a 0 2d a 0 d 
[kGlobal ] = [A ]T [k Local
(e )
][A ] = [A ]T  [A ]
− f 0 a f 0 a 
 0 −b 0 0 b 0 
 
 − a 0 d a 0 2d 
thus
 f am − al −f am − al 
 am bl + 2 dm
2 2
blm − 2dlm − am − bl + dm
2 2
− blm − dlm 

(e)
 − al blm − 2dlm bm 2 + 2 dl 2 al − blm − dlm − bm 2 + dl 2 
[kGlobal ]=   (6.617)
− f − am al f − am al 
 am − bl 2 + dm 2 − blm − dlm − am bl + 2 dm 2
2
blm − 2dlm 
 
− al − blm − dlm − bm 2 + dl al blm − 2dlm bm 2 + 2dl 2 
2

University of Castilla-La Mancha Draft By: Eduardo W. V. Chaves (2016)


Ciudad Real – Spain
6 LINEAR ELASTICITY 835

The Consistent Load Vector


As external load, (see Figure 6.198), we will only consider:
 qz 
(e)  
{PLocal }= mTx  (6.618)
 0 
 

 qz 
x (e)  
z {PLocal }= mTx 
qz  0 
2  

mTx

Figure 6.198: Beam element – external load (local system).


And the consistent load vector in the Local system ( x − z ) is given by:
 qz L 
 2 
m L 
 Tx 
 2 
 − q z L2 
 
(e) 12 
{ f Eq _ L} =  (6.619)
q L 
 z 
 2 
 mTx L 
 2 
 2 
 qz L 
 12 
Then, we can obtain the consistent load vector in the Global system as follows:
(e ) (e) (e) T (e)
{ f Eq _ L } = [A ]{ f Eq _ G } ⇒ { f Eq _ G } = [A ] { f Eq _ L }

 qz L 
 qz L   2 
 2   m L 
 Tx   − q z L  
2
    l − m
m L
0   Tx   2   12  
T
1 0 0 0 0
0 l  2   
 m 0 0 0   − q L2   mTx L   − q z L2  
z  m +  l
(e )
0 − m l 0 0 0   12   2   12   (6.620)
⇒ { f Eq _G} =    = 
0 0 0 1 0 0   qz L   qz L

0 0 0 0 l m   m2 L   2 
   Tx    mTx L   q z L2  
0 0 0 0 −m l   2    l −  m 
 2  12 
 2     
 zq L   m L  q L 2
 
 12    Tx m +  z l 
  2   12  
  

University of Castilla-La Mancha Draft By: Eduardo W. V. Chaves (2016)


Ciudad Real - Spain
836 SOLVING PROBLEMS BY MEANS OF CONTINUUM MECHANICS

Internal Force in the Beam Element


Once the nodal displacements in the Global system are obtained the internal force in the
beam element can be obtained as follows:
(e) (e)
1) Obtain the nodal displacement in the local system: {u Local } = [A ]{uGlobal }
(e) (e) (e)
2) Obtain the force due to these displacements: { f Local } = [k Local ]{uLocal }
3) Then, the internal force, (see Problem 6.62- NOTE 3), can be obtained as follows:
(e) (e) (e) (e) (e) (e) (e)
{rLocal } = −{ f Eq _ L } + { f Local } = −{ f Eq _ L } + [ k Local ][A ]{uGlobal } = −{ f int } (6.621)
Explicitly we can obtain:
 qz L 
 2 
m L 
 Tx   f 0 −a − f 0 − a  1 0 0 0 0 0   w1 
 2   0 b 0 0 − b 0  0 l m 0 0 0   θ X 1 
 − q z L2  
  − a 0 2d a 0 d  0 − m l 0 0 0   θY 1 
(e)
{rLocal } = − 12  +    
q L
 z  − f 0 a f 0 a  0 0 0 1 0 0   w2 
 2   0 −b 0 0 b 0  0 0 0 0 l m  θ X 2 
 mTx L     
 2   − a 0 d a 0 2d  0 0 0 0 −m l   θY 2 
 q L2 
 z 
 12 
After the multiplication of matrices, (Chaves&Mínguez(2019)), we can obtain:
 − qz L 
 2 
− m L
 Tx   fw1 − fw2 − al ( θY 1 + θY 2 ) + am ( θ X 1 + θ X 2 ) 
 22   bl ( θ X 1 − θ X 2 ) + bm ( θY 1 − θY 2 )

 qz L   
  
− aw1 + aw2 + dl (2 θY 1 + θY 2 ) − dm (2 θ X 1 + θ X 2 )
(e)
{rLocal } =  12  +   (6.622)
− qz L
   − fw1 − + fw2 + al ( θY 1 + θY 2 ) − am ( θ X 1 + θ X 2 ) 
 2   − bl ( θ X 1 − θ X 2 ) − bm ( θY 1 − θY 2 ) 
 − mTx L   
 2  − aw1 + aw2 + dl ( θY 1 + 2 θY 2 ) − dm ( θ X 1 + 2 θ X 2 )
 q L2 
 z 
 12 
and the reaction in the global system can be obtained as follows
(e)
{rGlobal } = [A ]T {rLocal
(e)
} (6.623)

University of Castilla-La Mancha Draft By: Eduardo W. V. Chaves (2016)


Ciudad Real – Spain
6 LINEAR ELASTICITY 837

NOTE 2.3: Frame Structures


3D Frame Structures. For this type of structure at each node is associated with 6 degrees-
of-freedom, (see Figure 6.183).
2D Frame Structures. For this type of structure we will have, per node, 3 degrees-of-
freedom associated with 2 translations ( u , v ) and 1 rotation ( θ z ). Locally the internal
forces: normal force ( N x ), shear force ( Q y ) and bending moment ( M z ), (see Figure
6.200).

vY

qX
qY θZ
uX
Degrees-of-freedom

Figure 6.199: 2D Frame Structure.

If we consider the structure of the nodal displacement described in Figure 6.200, and by
considering the stiffness matrices given by Figure 6.186 and Figure 6.187 we can obtain:
 EA − EA   qx L 
 L 0 0 0 0   2 
L  q L 
 12 EI z 6 EI z − 12 EI z 6 EI z 
u1   
y
 0 0  
 L3 L2 L3 L2     2 2 
 6 EI z 4 EI z − 6 EI z 2 EI z   v1   q y L 
 0 0  
L2 L L2 L   θ z1  =  12 
 − EA EA    q L 
 0 0 0 0  u2   x  (6.624)
 L L  v   2 
 − 12 EI z − 6 EI z 12 EI z − 6 EI z   2   q y L 
 0 L3 L2
0
L3
θ 
L2   z 2   2 
 6 EI z 2 EI z − 6 EI z 4 EI z  2 
 0 0  − qy L 
 L2 L L2 L   12 
(e) (e) (e)
[ k Local ] {uLocal } = { f Eq _ L}

University of Castilla-La Mancha Draft By: Eduardo W. V. Chaves (2016)


Ciudad Real - Spain
838 SOLVING PROBLEMS BY MEANS OF CONTINUUM MECHANICS

a) Displacements

z y  u1  (1)
v  (2)
 1
 θ  (3)
(e)
{uLocal } =  z1 
(3) θ z1 (6) θ z 2
v1 (2) v2 (5)  u2  (4)
x  v2  (5)
 
u1 (1) u 2 (4) θz 2  (6)
1 2

z b) Nodal force vector


y  N x1  (1)
Q 
 y1  (2)
 M  (3)
M z1 Q y1 M z2 Qy 2 (e )
{ f Local } =  z1 
Nx2  (4)
x Qy  (5)
 2
N x1 Nx2 M z 2  (6)
1 2

Figure 6.200: 2D frame structure – local system.


Two-Dimensional Space (2D)
Stiffness Matrix for 2D
For the local system the force-displacement relationship is given by the equation in (6.624).
Transformation Matrix
The transformation matrix from the Global system X − Y to the Local system x − y , (see
Figure 6.192(b)), is given by:
 cosα sin α 0  l m 0  ui   l m 0 u Xi 
   
[A ] = − sin α cosα 0 =  − m l 
0 → vi  =  − m
Nodal
l 0  vYi  (6.625)
 0 0 1  0 0 1  θ   0 0 1  θZi 
 zi  
(e )
And the transformation matrix for the displacement vector {uLocal } becomes

 u1   l m 0 0 0 0  u X 1 
 v  − m l 0 0 0 0  v 
 1   Y1 
θ   0 0 1 0 0 0  θZ 1 
 → z1  = 
Element

 u2   0 0 0 l m 0 u X 2  (6.626)


 v2   0 0 0 − m l 0  vY 2 
    
θz1   0 0 0 0 0 1  θZ 1 
14444444244444443
(e) (e)
{ u Local }=[ A ]{uGlobal }

University of Castilla-La Mancha Draft By: Eduardo W. V. Chaves (2016)


Ciudad Real – Spain
6 LINEAR ELASTICITY 839

θZ 2 , mZ 2  u1   f x1 
Y uY 2 , fY 2 v  f 
θz 2 , m z 2 x   1  y1 

 θ 
  m 
y v2 (e )
{uLocal } =  z1 ;{ f Local
(e)
} =  z1 
u2 , f x 2 u
 2  f x2 
 v2   f y2 
Y2 uY 1 , fY 1    
θ z 2  m z 2 
2 uX 2 , f X 2
v1 u X 1   f X1 
u1, f x1 v  f 
 Y1   Y1 
Y1 (e )
 θZ 1  ( e )  mZ 1 
u X 1, f X 1 {uGlobal } =  ;{ f Global } =  
θZ 1 , mZ 1 1 u X 2   fX 2 
 vY 2   fY 2 
θ z1 , m z1    
 θZ 2  mZ 2 

X1 X2 X

Figure 6.201: Beam element in two-dimensional space – 3 DOF per node.

By considering that:
 EA
 a 0 0 −a 0 0 a = L
 0 b c 0 − b c  
 b = 12 EI z
(e)
 0 c 2d 0 − c d   L3
[k Local ]=  with  (6.627)
− a 0 0 a 0 0 c = 6 EI z
 0 −b −c 0 b − c  L2
   2 EI z
 0 c d 0 − c 2d  d =
 L
the stiffness matrix in the Global system can be expressed as follows:
 a 0 0 −a 0 0
 0 b c 0 − b c 

(e)
 0 c 2d 0 − c d 
[kGlobal ] = [A ]T [k Local
(e)
][A ] = [A ]T  [A ]
− a 0 0 a 0 0
 0 −b −c 0 b − c
 
 0 c d 0 − c 2d 
thus
 al 2 + bm 2 alm − blm − cm − al 2 − bm 2 − alm + blm − cm 
 
 alm − blm bl 2 + am 2 cl − alm + blm − bl 2 − am 2 cl 
(e)
 − cm cl 2d cm − cl d 
[kGlobal ]=   (6.628)
 − al − bm − alm + blm cm al + bm 2 alm − blm cm 
2 2 2

 − alm + blm − bl 2
− am 2 − cl alm − blm bl 2
+ am 2 − cl 
 
 − cm cl d cm − cl 2d 

University of Castilla-La Mancha Draft By: Eduardo W. V. Chaves (2016)


Ciudad Real - Spain
840 SOLVING PROBLEMS BY MEANS OF CONTINUUM MECHANICS

The Consistent Load Vector


As external load, (see Figure 6.202), we will only consider:
qx 
(e)  
{PLocal }= q y  (6.629)
0
 

y
qx
qx 
x (e)  
{PLocal }= q y 
qy 0
2  

Figure 6.202: Beam element – external load (local system).


And the consistent load vector in the local system, (see equation (6.624)), is given by:
 qx L 
 2 
 q L 
 y   f ( Eq _ L ) 
 2   1( Eq _ L ) 
 q y L2   f 2 
   f ( Eq _ L ) 
(e) 12
{ f Eq _ L} =  = 3 (6.630)
q x L   f ( Eq _ L ) 
   4 
 2   f ( Eq _ L ) 
 q y L   5( Eq _ L ) 
   f 6 
 2 2
− qy L 
 12 

Then, we can obtain the consistent load vector in the global system as follows:
(e) (e) (e) T (e)
{ f Eq _ L } = [A ]{ f Eq _ G } ⇒ { f Eq _ G } = [A ] { f Eq _ L }
T
 l m 0 0 0 0  f1( Eq _ L )   f1( Eq _ L ) l − f 2( Eq _ L ) m 
 ( Eq _ L )   ( Eq _ L )
− m
 l 0 0 0 0  f2   f1 m + f 2( Eq _ L) l 
(e)
 0 0 1 0 0 0  f ( Eq _ L )   f 3( Eq _ L )  (6.631)
⇒ { f Eq _G} =    ( Eq _ L )  =  ( Eq _ L )
3

 0 0 0 l m 0  f4   f4 l − f 5( Eq _ L) m 
 0 0 0 −m l 0  f ( Eq _ L )   f ( Eq _ L ) m + f ( Eq _ L ) l 
   5( Eq _ L )   4 5

 0 0 0 0 0 1  f 6   f6 ( Eq _ L )


or more explicitly

University of Castilla-La Mancha Draft By: Eduardo W. V. Chaves (2016)


Ciudad Real – Spain
6 LINEAR ELASTICITY 841

 q x L   qy L  
 l −  m 
 2   2  
 q x L   qy L  
 f1( Eq _ L )
l − f2 ( Eq _ L )
m   2   2 l 
   m +
 ( Eq _ L )
 f1 m + f 2( Eq _ L ) l   q y L2 
 ( Eq _ L )   
(e)  f3   12 
⇒ { f Eq } = = (6.632)
_G 
 f4
( Eq _ L )
l − f5 ( Eq _ L )
m   q x L l −  q y L m 
 
   2  
 f ( Eq _ L ) m + f ( Eq _ L ) l   2   
 4 5
  
  x m +  y l 
 f6 ( Eq _ L )
  q L  q L
 2   2  
   
2
 − q y L 
 12 

Internal Force in the Beam Element


Once the nodal displacements in the Global system are obtained the internal force in the
beam element can be obtained as follows:
(e) (e)
1) Obtain the nodal displacement in the local system: {u Local } = [A ]{uGlobal }
(e) (e) (e)
2) Obtain the force due to these displacements: { f Local } = [k Local ]{uLocal }
3) Then, the internal force related to local system, (see Problem 6.62-NOTE 3), can be
obtained as follows:
(e) (e ) (e) (e) (e) (e) (e)
{rLocal } = −{ f Eq _ L } + { f Local } = −{ f Eq _ L } + [ k Local ][A ]{uGlobal } = −{ f int } (6.633)
Explicitly we can obtain:
 f1( Eq _ L )   a 0 0 −a 0 0  l m 0 0 0 0  u X 1 
 ( Eq _ L )  
 f2   0 b c 0 − b c  − m l 0 0 0 0  vY 1 
(e)
 f ( Eq _ L )   0 c 2d 0 − c d   0 0 1 0 0 0  θZ 1 
{rLocal } = − 3( Eq _ L )  +    
 f4  − a 0 0 a 0 0  0 0 0 ml 0 u X 2 
 f ( Eq _ L )   0 − b − c 0 b − c  0 0 0 −m l 0  vY 2 
 5( Eq _ L )     
 f 6   0 c d 0 − c 2d   0 0 0 0 0 1   θZ 1 

After the multiplication of matrices we can obtain:


 f x1   f1( Eq _ L )   al (u X 1 − u X 2 ) + am (vY 1 − vY 2 ) 
   ( Eq _ L )   
 f y1   f2   bl (vY 1 − vY 2 ) + bm (u X 2 − u X 1 ) + c( θZ 1 + θZ 2 ) 
(e) (e)
 m   f ( Eq _ L )   cl (vY 1 − vY 2 ) + cm (u X 2 − u X 1 ) + d (2 θZ 1 + θZ 2 ) 
{rLocal } = −{ f int } =  z1  = − 3( Eq _ L )  +  
 f x2   f4   − al (u X 1 − u X 2 ) − am (vY 1 − vY 2 ) 
f   f ( Eq _ L )  − bl (vY 1 − vY 2 ) − bm (u X 2 − u X 1 ) − c( θZ 1 + θZ 2 ) 
 y2   5( Eq _ L )   
m z 2   f 6   cl (vY 1 − vY 2 ) + cm (u X 2 − u X 1 ) + d ( θZ 1 + 2 θZ 2 ) 
(6.634)

University of Castilla-La Mancha Draft By: Eduardo W. V. Chaves (2016)


Ciudad Real - Spain
842 SOLVING PROBLEMS BY MEANS OF CONTINUUM MECHANICS

More explicitly
 qx L 
 2 
 q L 
 f x1   y   al (u X 1 − u X 2 ) + am (vY 1 − vY 2 ) 
   2   
 f y1   q y L2   bl (vY 1 − vY 2 ) + bm (u X 2 − u X 1 ) + c( θZ 1 + θZ 2 ) 
 m     cl (v − v ) + cm (u − u ) + d (2 θ + θ ) 
   Z2 
(e)
{rLocal (e)
} = −{ f int } =  z1  = − 12  +  Y1 Y2 X2 X1 Z1

 f x2 
q L
 x   − al (u X 1 − u X 2 ) − am (vY 1 − vY 2 ) 
f   2  − bl (vY 1 − vY 2 ) − bm (u X 2 − u X 1 ) − c( θZ 1 + θZ 2 )
 y2   qy L   
m z 2     cl (vY 1 − vY 2 ) + cm (u X 2 − u X 1 ) + d ( θZ 1 + 2 θZ 2 ) 
 2 2
− qy L 
 12 
(6.635)
(e)
The reaction forces in the Global system can be obtained by {rGlobal } = [A ]T {rLocal
(e )
}:
T
 l m 0 0 0 0  f x1   f x1l − f y1m 
− m    
 l 0 0 0 0  f y1   f x1m + f y1l 
(e)
 0 0 1 0 0 0  m   m z1 
{rGlobal }=   
z1
=  (6.636)
 0 0 0 l m 0  f x2   f x2 l − f y2m 
 0 0 0 −m l 0  f  f m + f l 
   y 2   x 2 (int ) y 2 
 0 0 0 0 0 1 m z 2   m z 2 

University of Castilla-La Mancha Draft By: Eduardo W. V. Chaves (2016)


Ciudad Real – Spain
6 LINEAR ELASTICITY 843

Problem 6.71
Consider the structure described in Figure 6.203, (Timoshenko (1940)). Obtain the
displacement of the node 1 according to Y -direction.

Y Element connectivity
P (e = 1) ⇒ 3 − 1
1 ( e = 2) ⇒ 2 − 1
( Ltanβ , L)
(e = 3) ⇒ 4 − 1
Element properties
β β (e = 1) ⇒ ( EA)(1)
2 3 L
(e = 2) ⇒ ( EA)( 2 )
1
(e = 3) ⇒ ( EA)( 2 )

O 2 3 4
X
(0,0) ( Ltanβ ,0) (2 Ltanβ ,0)

Figure 6.203: 2D truss structure.


Solution:
By considering 2D truss structure we have two degrees-of freedom per node, so the Global
displacement vector for the structure has 8 degrees-of-freedom (Number of nodes (4)
times Number of degrees-of-freedom per node (2)), (see Figure 6.204).

vY (2) Degrees-of-freedom
P
Y
U2 u X (1) U1  0 
U  P
1  2  
U 3  0 
U1    
U 4  0 
{U Global } =   ; {FGlobal } =  
U 5  0 
2 1 3 U 6  0 
   
U 7  0 
U4 U6 U8 U  0 
 8  
2 3 4
O
U3 U5 U7 X

Figure 6.204: Degrees-of-freedom – Global system.


Next we will construct the system {FGlobal } = [ K Global ]{U Global } where [ K Global ] is the Global
stiffness matrix of the structure, and {FGlobal } is the Global nodal force vector.
Construction of the Global Stiffness Matrix - [ K Global ]
The matrix [ K Global ] can be constructed by assembling the individual bar elements, i.e.:

University of Castilla-La Mancha Draft By: Eduardo W. V. Chaves (2016)


Ciudad Real - Spain
844 SOLVING PROBLEMS BY MEANS OF CONTINUUM MECHANICS

A[k
(e)
[ K Global ] = Global ] (6.637)
e =1

The stiffness matrix for the 2D truss element can be obtained by using the equation in
(6.600), i.e.:
l 2 lm −l 2
− lm 
 
 lm m2 − lm − m2
(e)
(e) ( EA)
[k Global ]= (6.638)
L(e ) − l 2 − lm l2 lm 
 
 − lm − m2 lm m 2 
For each element we have
Connectivi ty : N1 → N2
Element Node1 : ( X 1 , Y1 ) L(e ) l ( e)
=
X 2 − X1
m (e ) = Y2 −(e)Y1 ( EA) (e )
N1 → N2 L( e) L
Node2 : ( X 2 , Y2 )
3 : ( Ltanβ ,0)
1 3 → 1 L l (1)
=0 m (1) = 1 (EA) (1)
1 : ( Ltanβ , L)
2 : (0,0)
2 → 1
L l ( 2)
= sin β m ( 2) = cos β
2 (EA) ( 2 )
1 : ( Ltanβ , L) cos β l ( 2)
=s m ( 2) = c
2 : (2 Ltanβ ,0) l L ( 3)
= − sin β m (3) = cos β
3 4 → 1 ( EA) ( 2 )
1 : ( Ltanβ , L) l ( 3) = − s cos β m ( 3) = c
And we will consider that l ( 2) = sin β = s = −l (3) , m ( 2) = m ( 2) = cos β = c
Element 1: ( l (1)
= 0 , m (1) = 1 )

5 6 1 2 Global
l 2
lm −l 2
− lm  0 0 0 0 5
  0 1
(1) ( EA) (1)
 lm m2 − lm − m 2  ( EA) (1)  0 − 1 6
[k Global ]= =
L(1) − l 2 − lm l2 lm  L 0 0 0 0 1
   
 − lm − m2 lm m  0 − 1
2
0 1 2

Element 2: ( l ( 2)
m ( 2) = c )
= s, 3 4 1 2
Global
l 2 lm − l 2 − lm   s 2
sc − s − sc  3
2

   
( 2) ( EA) ( 2 )  lm m 2 − lm − m 2  c( EA) ( 2 )  sc c 2 − sc − c 2  4
[k Global ]= =
L( 2 )  − l 2 − lm l2 lm  L − s 2 − sc s 2 sc  1
   
 − lm − m
2
lm m 
2
 − sc − c
2
sc c 2  2

Element 3 ( l ( 3)
= − s , m ( 3) = c ) 7 8 1 2
Global
l 2
lm −l 2
− lm   s 2
− sc − s sc  72

   
( 3) ( EA) ( 2)
 lm m2 − lm − m 2  c( EA) ( 2 )  − sc c
2
sc − c 2  8
[k Global ]= =
L(3) − l 2 − lm l2 lm  L − s 2 sc s 2 − sc  1
   
 − lm −m2 lm m   sc − c − sc c  2
2 2 2

University of Castilla-La Mancha Draft By: Eduardo W. V. Chaves (2016)


Ciudad Real – Spain
6 LINEAR ELASTICITY 845

U 2 , F2
{FGlobal } = [ K Global ]{U Global }
1
U1 , F1

x x
(4) x
(4) (4)
2
(3) 2 (3)
(3) 2
2 3
1
(2)
(1) (2)
(2)
1 (1) 1 (1)
1

U 4 , F4 U 6 , F6 U 8 , F8
3
U 3 , F3 4
2 U 5 , F5 U 7 , F7

Figure 6.205: Discrete system.

 k33
(1) ( 2)
+ k33 ( 3)
+ k33 (1)
k34 (2)
+ k34 ( 3)
+ k34 (2)
k31 (2)
k32 (1)
k31 (1)
k32 ( 3)
k31 ( 3)
k32 
 (1) ( 2) ( 3) (1) (2) ( 3) (2) (2) (1) (1) ( 3) ( 3) 
 k43 + k43 + k43 k44 + k44 + k44 k 41 k 42 k41 k42 k41 k42 
 k13( 2 ) k14( 2 ) k11( 2 ) k12( 2 ) 0 0 0 0 
3
 ( 2) ( 2) (2) (2)

 k23 k24 k 21 k 22 0 0 0 0 
A
(e)
[ K Global ] = [kGlobal ]= 
k13(1) k14(1) 0 0 k11(1) k12(1) 0 0 
e =1
 
(1) (1) (1) (1)
 k 23 k24 0 0 k21 k22 0 0 
 
 k13(3) k14(3) 0 0 0 0 k11(3) k12(3) 
 ( 3)
k23 ( 3)
k24 0 0 0 0 ( 3)
k21 ( 3) 
k22 

Construction of the Global Nodal Force Vector - {FGlobal }


0 
P
 
0 
 3   
  0 
A
(e)
{FGlobal } =  { f Eq _ L } + {F0 } =  
 e=1  0 
14 4244 3 0 
={0 }
 
0 
0 
 

University of Castilla-La Mancha Draft By: Eduardo W. V. Chaves (2016)


Ciudad Real - Spain
846 SOLVING PROBLEMS BY MEANS OF CONTINUUM MECHANICS

Applying the Boundary Conditions


Note that the degrees-of-freedom related to the nodes 2 , 3 , and 4 cannot move due to
the boundary conditions, then the system {FGlobal } = [ K Global ]{U Global } after impose the
boundary conditions becomes:
k 33
(1) ( 2)
+ k33 ( 3)
+ k33 (1)
k 34 ( 2)
+ k34 ( 3)
+ k34 0 0 0 0 0 0 U1   0 
 (1) ( 2) ( 3) (1) ( 2) ( 3)    
k 43 + k 43 + k 43 k 44 + k 44 + k 44 0 0 0 0 0 0 U 2  P 
 0 0 1 0 0 0 0 0 U 3   0 
    
 0 0 0 1 0 0 0 0 U 4   0 
  = 
0 0 0 0 1 0 0 0 U 5   0 
 
 0 0 0 0 0 1 0 0 U 6   0 
    
 0 0 0 0 0 0 1 0 U 7   0 
 0 0 0 0 0 0 0 1  U 8   0 
By substituting the values we can obtain:
2c( EA) ( 2 ) s 2 0 0 0 0 0 0 0 U1   0 
 (1) (2) 3    
 0 ( EA) + 2( EA) c 0 0 0 0 0 0 U 2   P 
 0 0 1 0 0 0 0 0 U 3   0 
    
1 0 0 0 1 0 0 0 0 U 4   0 
  = 
L 0 0 0 0 1 0 0 0 U 5   0 
 
 0 0 0 0 0 1 0 0 U 6   0 
    
 0 0 0 0 0 0 1 0 U 7   0 
 0 0 0 0 0 0 0 1 U 8   0 

Solving the System


 0 
U1   
U   LP 
 2   (1)

( 2) 3 
U 3   ( EA) + 2( EA) c  
   0 
U 4   
 = 0 
U 5   0 
U 6   
   0 
U 7   
U   0 
 8  
 0
Internal and Reaction Forces
(e) (e) (e)
We calculate the vector { f Local } = [ k Local ]{uLocal } , (see equation (6.604)):

 f x(1e )  (u (Xe1) − u (Xe2) )l (e)


+ (vY(e1) − vY(e2) )m ( e ) 
 (e)  (e)  
 f y1  ( EA)  0 
 (e)  =  (e) (e)  (6.639)
L( e ) (u X 2 − u X 1 )l + (vY 2 − vY 1 )m 
(e) (e) (e) (e)
 f x2 
 f y(2e )   0 
   
and the reaction vector in the Local and Global system can be obtained as follows

University of Castilla-La Mancha Draft By: Eduardo W. V. Chaves (2016)


Ciudad Real – Spain
6 LINEAR ELASTICITY 847

(e) (e )
{rLocal } = −{ f int } = { f (e ) } = [k Local
(e) (e)
]{uLocal } ⇒ (e)
{rGlobal } = [A ]T {rLocal
(e)
}

where
T
 l m 0 0  f x1   f x1l 
− m    
l 0 0   f y1 = 0   f x1m 
{rGlobal } = 
(e)
 = 
 0 0 l m  f x 2   f x 2l 

 0 0 −m l   f y 2 = 0  f m 
   x2 
Element 1: l (1)
=0, m (1) = 1 , ( EA) (1) , L(1) = L , (u (X11) = U 5 = 0) , (vY(11) = U 6 = 0) ,
 LP 
(u (X1)2 = U1 = 0) ,  vY(12) = U 2 = (1)
 , then
( 2) 3 
 ( EA) + 2( EA) c 

 − LP 
 f x(11)  (u (X11) − u (X1)2 )l (1)
+ (vY(11) − vY(12) )m (1)   (1) ( 2 ) 3 

 (1)     ( EA) + 2( EA) c 
(1) (1)
 f  ( EA)  0  ( EA)  0 
{ f } =  y(11)  =
(1)
 (1)  = 
 f x2  L(1) (u X 2 − u X 1 )l
(1) (1)
+ (vY(12) − vY(11) )m (1)  L  LP 

 f y(12)   0   ( EA)(1) + 2( EA)( 2 ) c 3 
     
 0 
(1)
= −{ f int }
and
 − LP  Global
 ( EA)(1) + 2( EA)( 2) c 3 l 
 f x1l     0  5
 − LP   − ( EA ) (1)
P 
  m 6
(1)  f x1m  ( EA)
(1) (1) ( 2 ) 3 
 ( EA) + 2( EA) c   ( EA) + 2( EA) c 
(1) ( 2 ) 3
{rGlobal } =  =  = 
LP 0
 f x 2l  L  l   (1)  1
 f m (1) ( 2) 3
 ( EA) + 2( EA) c   ( EA ) P  2
 x2     ( EA)(1) + 2( EA)( 2) c 3 
LP
 (1) ( 2) 3
m
 ( EA) + 2( EA) c 
L L
Element 2: l (2) = s , m ( 2) = c , ( EA) ( 2 ) , L( 2 ) = = , (u (X21) = U 3 = 0) , (vY( 21) = U 4 = 0) ,
cos β c
 LP 
(u X( 22) = U 1 = 0) ,  vY( 22) = U 2 = (1)
 , then
(2) 3 
 ( EA) + 2( EA) c 

 − c 2 ( EA)( 2 ) P 
 
  LP    ( EA)(1) + 2( EA)( 2 ) c 3 

 f x(12 )  −  (1) (2) 3  
c  
 (2)    ( EA) + 2( EA) c    
 f  c( EA)
( 2)
 0   0 
{ f } =  y(12 )  =
( 2)
 = 
 f x2  L  LP    
c c 2 ( EA)( 2 ) P
 f y(22 )    ( EA)(1) + 2( EA)( 2) c 3    (1)

( 2 ) 3 
     ( EA) + 2 ( EA ) c 
 0   
 0 
(2)
= −{ f int }
and

University of Castilla-La Mancha Draft By: Eduardo W. V. Chaves (2016)


Ciudad Real - Spain
848 SOLVING PROBLEMS BY MEANS OF CONTINUUM MECHANICS

Global
 2
− c ( EA) P ( 2)
  2 ( 2)

 (1)
l  −(c1) ( EA) P( 2) 3 s  3
( 2) 3 
 ( EA) + 2( EA) c   ( EA) + 2( EA) c 
 f x1l   − c 2 ( EA)( 2 ) P   − c 2 ( EA)( 2 ) P 
   m   c 4
( 2) 3 
( 2)  f m   ( EA) + 2( EA) c   ( EA) + 2( EA) c 
(1) (2) 3 (1)
{rGlobal } =  x1  =  = 
 f x 2l   c 2 ( EA)( 2 ) P
l   c 2 ( EA)( 2 ) P
s 1
 f m   ( EA)(1) + 2( EA)( 2 ) c 3   ( EA)(1) + 2( EA)( 2 ) c 3 
 x2     
 c 2 ( EA)( 2 ) P   c 2 ( EA)( 2 ) P 2
m c
 ( EA)(1) + 2( EA)( 2 ) c3   ( EA)(1) + 2( EA)( 2 ) c 3 

L L
Element 3: l ( 3) = − s , m ( 2 ) = c , (EA) ( 2 ) , L(3) = = , (u (X31) = U 3 = 0) , (vY(31) = U 4 = 0) ,
cos β c
 LP 
(u (X32) = U1 = 0) ,  vY(32) = U 2 = (1)
 , then
(2) 3 
 ( EA) + 2( EA) c 

 − c 2 ( EA)( 2 ) P 
 
  LP    ( EA)(1) + 2( EA)( 2 ) c 3 
 f x(13)  −
   (1) (2) 3  
 c  
 ( 3)    ( EA) + 2( EA) c    
 f  c( EA)
(2)
 0   0 
{ f } =  y(13)  =
( 3)
  = 
 f x2  L  LP    2 ( 2)
 
c c ( EA ) P
 f y2 
( 3 )   ( EA)(1) + 2( EA)( 2) c 3    (1)

( 2 ) 3 
   0   ( EA) + 2( EA) c 
   
 0 
( 3)
= −{ f int }
and
Global
 2
− c ( EA) P ( 2)
  2 ( 2)

 (1)
l  c(1)( EA) P( 2) 3 s  7
( 2) 3 
 ( EA) + 2( EA) c   ( EA) + 2( EA) c 
 f x1l   − c 2 ( EA)( 2 ) P   − c 2 ( EA)( 2 ) P 
   m   c 8
( 2) 3 
( 3)  f m   ( EA) + 2( EA) c   ( EA) + 2( EA) c 
} =  x1  = 
(1) (2) 3 (1)
{rGlobal = 
 f x 2l   c 2 ( EA)( 2 ) P 2 ( 2)
l   − c ( EA) P s  1
 f m   ( EA)(1) + 2( EA)( 2 ) c 3   ( EA)(1) + 2( EA)( 2 ) c 3 
 x2     
 c 2 ( EA)( 2 ) P   c 2 ( EA)( 2 ) P 2
m c
 ( EA)(1) + 2( EA)( 2 ) c3   ( EA)(1) + 2( EA)( 2 ) c 3 

University of Castilla-La Mancha Draft By: Eduardo W. V. Chaves (2016)


Ciudad Real – Spain
6 LINEAR ELASTICITY 849

The Global reaction vector can be obtained by the contribution of each element:

Global
 2
c ( EA) P (2) 2 ( 2)
− c ( EA) P 
 0+ (1) (2) 3
s+ (1) (2) 3
s  1
 ( EA) + 2( EA) c ( EA) + 2( EA) c 
 ( EA) (1) P c 2 ( EA) ( 2 ) P c 2 ( EA) ( 2 ) P 
 (1) ( 2) 3
+ (1) ( 2) 3
c+ (1)
c
(2) 3 
2
 ( EA) + 2( EA) c ( EA) + 2( EA) c
2 ( 2)
( EA) + 2( EA) c 
 − c ( EA) P 
 (1) ( 2) 3
s  3
( EA) + 2( EA) c
 2 (2) 
 − c ( EA) P 
3
 c  4
A
(1) ( 2) 3
(e )
{rGlobal } =  ( EA) + 2( EA) c 
e =1  0 
  5
 − ( EA) (1) P 
 (1) (2) 3  6
 ( EA) + 2( EA) c 
2 ( 2)
 c ( EA) P 
 (1) ( 2) 3
s  7
( EA) + 2( EA) c
 
 − c 2 ( EA) ( 2 ) P 
c 8
 ( EA ) (1)
+ 2 ( EA ) ( 2) 3
c 
 
which results
 0   0 
 P   0 
 2 ( 2)   
 − c ( EA) P  0  
2
− c ( EA) P (2)

s s
 ( EA) (1) + 2( EA) ( 2) c 3     ( EA) (1) + 2( EA) ( 2 ) c 3 
  P  
 − c 2 ( EA) ( 2) P  0   − c 2 ( EA) ( 2 ) P 
c c
 ( EA) (1) + 2( EA) ( 2 ) c 3     ( EA) (1) + 2( EA) ( 2 ) c 3 
3
  0   
A
(e)
{R} = {rGlobal } − {F0 } =  0 0
 − 0  =  
e =1  − ( EA) P(1)     − ( EA) P(1) 
 (1) (2) 3
+  0   (1) ( 2) 3
+
 ( EA) + 2( EA) c     ( EA) + 2( EA) c 
 c 2 ( EA) ( 2 ) P  0   c 2 ( EA) ( 2 ) P 
 s     ( 2) 3 
s
 ( EA) + 2( EA) c   0   ( EA) + 2( EA) c 
(1) ( 2) 3 (1)

 − c 2 ( EA) ( 2) P   − c 2 ( EA) ( 2 ) P 
 (1) (2) 3 
c  c
(2) 3 
 ( EA) + 2( EA) c 
(1)
 ( EA) + 2( EA) c 

University of Castilla-La Mancha Draft By: Eduardo W. V. Chaves (2016)


Ciudad Real - Spain
850 SOLVING PROBLEMS BY MEANS OF CONTINUUM MECHANICS

Problem 6.72
Consider the 2D Framed structure described in Figure 6.206, where L = 5m and the
mechanical - geometrical properties for the beams are EI z = 2 × 105 kNm 2 and EA = 107 kN .
Obtain the displacements of the node 1 and 2 according to Y -direction when
qY = −20kN / m .

Y Element connectivity
(e = 1) ⇒ 1 − 2
qY 2 (e = 2) ⇒ 3 − 1
1
1 (e = 3) ⇒ 2 − 4

2 3 vY
L

θZ
O 3 4 uX
X Degrees-of-freedom
(0,0) (L,0)

Figure 6.206: 2D Framed structure.


Solution:
By considering 2D framed we have three degrees-of-freedom per node, so the Global
displacement vector for the structure has 12 degrees-of-freedom (Number of nodes (4)
times Number of degrees-of-freedom per node (3)), (see Figure 6.207).

Y  U1 
U 
U5  2
U2 U3 
U6  
U1 U 4 
U4 U 5 
1 U3 1 2  
U 6 
{U Global } =  
3 U 7 
2 U8 
 
U 9 
U8 U 11 U 
 10 
3 U7 U 12 U 10 U11 
U 
X  12 
U9 4

Figure 6.207: Degrees-of-freedom – Global system.


Next we will construct the system {FGlobal } = [ K Global ]{U Global } where [ K Global ] is the Global
stiffness matrix of the structure, and {FGlobal } is the Global nodal force vector.

University of Castilla-La Mancha Draft By: Eduardo W. V. Chaves (2016)


Ciudad Real – Spain
6 LINEAR ELASTICITY 851

Construction of the Global Stiffness Matrix - [ K Global ]


The matrix [ K Global ] can be constructed by assembling the individual beam elements, i.e.:
3

A[k
(e)
[ K Global ]12×12 = Global ] (6.640)
e =1

The stiffness matrix for the 2D frame element can be obtained by using the equation in
(6.628), i.e.:
 al 2 + bm 2 alm − blm − cm − al 2 − bm 2 − alm + blm − cm 
 
 alm − blm bl 2 + am 2 cl − alm + blm − bl 2 − am 2 cl 
(e)
 − cm cl 2d cm − cl d 
[kGlobal ]=   (6.641)
 − al − bm − alm + blm cm al 2 + bm 2 alm − blm cm 
2 2

 − alm + blm − bl 2 − am 2 − cl alm − blm bl 2 + am 2 − cl 


 
 − cm cl d cm − cl 2d 
For each element we have
Connectivi ty : N1 → N2
Element Node1 : ( X 1 , Y1 ) L(e ) l (e )
=
X 2 − X1
m (e) = Y2 −(e)Y1
N1 → N2  L(e ) L
Node2 : ( X 2 , Y2 )
1 : (0, L)
1 1 → 2 L l (1) = 1 m (1) = 0
2 : ( L , L )
3 : (0,0)
2 3 → 1 L l ( 2) = 0 m ( 2) = 1
1 : (0, L)
2 : ( L , L )
3 2 → 4 L l ( 3) = 0 m (3) = −1
 4 : ( L ,0 )
Element 1: 1 2 3 Global 4 5 6
2 0 0 −2 0 0 1
0 0.0192 0.048 0 − 0.0192 0.048  2

(1)
0 0.048 0.16 0 − 0.048 0.08  3
[kGlobal ] = 106 ×  
− 2 0 0 2 0 0 4
 0 − 0.0192 − 0.048 0 0.0192 − 0.048 5
 
 0 0.048 0.08 0 − 0.048 0.16  6
Element 2:
7 8 9 Global 1 2 3
 0.0192 0 − 0.048 − 0.0192 0 − 0.048 7
 0 2 0 0 −2 0  8

( 2)
 − 0.048 0 0.16 0.048 0 0.08  9
[kGlobal ] = 106 ×  
− 0.0192 0 0.048 0.0192 0 0.048  1
 0 − 2 − 0.048 0 2 0 2
 
 − 0.048 0 0.08 0.048 0 0.16  3

University of Castilla-La Mancha Draft By: Eduardo W. V. Chaves (2016)


Ciudad Real - Spain
852 SOLVING PROBLEMS BY MEANS OF CONTINUUM MECHANICS

Element 3
4 5 6 10 11 12 Global
 0.0192 0 0.048 − 0.0192 0 0.048  4
 0 2 0 0 −2 0  5

( 3)
 0.048 0 0.16 − 0.048 0 0.08  6
[kGlobal ] = 106 ×  
− 0.0192 0 − 0.048 0.0192 0 − 0.048 10
 0 − 2 − 0.048 0 2 0  11
 
 0.048 0 0.08 − 0.048 0 0.16  12
Once we have assembled the contribution of all elements the stiffness matrix
3

A[k
(e)
[ K Global ] = Global ]
e =1

we will obtain the stiffness of the structure.

Construction of the Global Nodal Force Vector - {FGlobal } , (see equation (6.632))
 3 
 
A
(e )
{FGlobal } =  { f Eq _G  + {
} F0 }
 e =1  {
  ={0 }

Element 1:
Global
6 L[q x l + q y m ]  0  1
6 L[q m + q l ]   2
 x y   − 50 
(1)

1  q y L2

   − 41 . 666667  3
{ f Eq _G} =  = 
12 6 L[q x l + q y m ]  0  4
6 L[q x m + q y l ]  − 50  5
 2   
 − qy L   41.666667  6

Element 2 and Element 3 { f Eq( 2)_ G } = { f Eq(3)_ G } = {0} .


Then,
 0 
 − 50 
 
− 41.666667 
 
{FGlobal }12×1 =  0 
 − 50 
 
 41.666667 
 
 {0}6×1 

Applying the Boundary Conditions


Note that the nodes 3 and 4 cannot move, so U 7 = U 8 = U 9 = 0 , U10 = U11 = U12 = 0 and after
applying these boundary conditions the global stiffness matrix will have the following
aspect:

University of Castilla-La Mancha Draft By: Eduardo W. V. Chaves (2016)


Ciudad Real – Spain
6 LINEAR ELASTICITY 853

Global
 K11 K12 K13 K14 K15 K16 0 0 0 0 0 0 1
K K 22 K 23 K 24 K 25 K 26 0 0 0 0 0 0 2
 12
 K13 K 23 K 33 K 34 K 35 K 36 0 0 0 0 0 0 3
 
 K14 K 24 K 34 K 44 K 45 K 46 0 0 0 0 0 0 4
 K15 K 25 K 35 K 45 K 55 K 56 0 0 0 0 0 0 5
 
K K 26 K 36 K 46 K 56 K 66 0 0 0 0 0 0 6
[ K Global ]12×12 =  16
0 0 0 0 0 0 1 0 0 0 0 0 7
 
 0 0 0 0 0 0 0 1 0 0 0 0 8
 
 0 0 0 0 0 0 0 0 1 0 0 0 9
 0 0 0 0 0 0 0 0 0 1 0 0 10
 
 0 0 0 0 0 0 0 0 0 0 1 0 11
 0 0 0 0 0 0 0 0 0 0 0 1 12

Then, for the sake of simplicity we will only express here the sub-matrix 6 × 6 , which is
 k11(1) + k 44
( 2)
k12(1) + k 45
(2)
k13(1) + k 46
( 2)
k14(1) k15(1) k16(1) 
 (1) ( 2) (1) (2) (1) ( 2) (1) (1) (1) 
 k 21 + k54 k 22 + k55 k 23 + k56 k 24 k 25 k 26 
 k (1) + k ( 2 ) (1)
k32 (2)
+ k 65 (1)
k33 + k66( 2) (1)
k34 (1)
k35 (1)
k36 
[ K Global ]6×6 =  31 (1) 64 (1) (1) (1)

 k 41 k 42 k 43 k 44 + k11(3) (1)
k 45 + k12(3) (1)
k 46 + k13(3) 
 k (1) (1)
k52 (1)
k53 (1)
k54 ( 3)
+ k 21 (1)
k55 ( 3)
+ k 22 (1)
k56 ( 3) 
+ k 23
 51 
(1) (1) (1) (1) ( 3) (1) ( 3) (1) ( 3)
 k61 k62 k63 k64 + k31 k 65 + k32 k66 + k33 

which results in
 2.0192 0 0.048 −2 0 0 
 0 2.0192 0.048 0 − 0.0192 0.048 

 0.048 0.048 0.32 0 − 0.048 0.08  6
[ K Global ]6×6 =  × 10
 − 2 0 0 2 . 0192 0 0 . 048 
 0 − 0.0192 − 0.048 0 2.0192 − 0.048
 
 0 0.048 0.08 0.048 − 0.048 0.32 
Solving the System
Then, we have to solve the system
2.0192 0 0.048 −2 0 0  U1   0 
 0  U   
 2.0192 0.048 0 − 0.0192 0.048   2  − 50 
 0.048 0.048 0.32 0 − 0.048 0.08    − 41.666667 
6 U 3 
  × 10   =  
 −2 0 0 2.0192 0 0.048  U 4   0 
 0 − 0.0192 − 0.048 0 2.0192 − 0.048 U 5   − 50 
     
 0 0.048 0.08 0.048 − 0.048 0.32  U 6   41.666667 

and the solution is

University of Castilla-La Mancha Draft By: Eduardo W. V. Chaves (2016)


Ciudad Real - Spain
854 SOLVING PROBLEMS BY MEANS OF CONTINUUM MECHANICS

U 1   2.078345 
U   
 2  − 25 
U 3  − 174.02678 −6
 =  × 10
U 4   − 2.078345 
U 5   − 25 
   
U 6   174.02678 

Internal and Reaction Forces


(e) (e) (e ) (e)
The internal and reaction forces can be obtained by {rLocal } = −{ f Eq _ L } + { f Local } = −{ f int } ,

(see equation (6.635)):


 qx L 
 2 
 q L 
 f x1   y   al (u X 1 − u X 2 ) + am (vY 1 − vY 2 ) 
   2   
 f y1   q y L   bl (vY 1 − vY 2 ) + bm (u X 2 − u X 1 ) + c( θZ 1 + θZ 2 ) 
2

 m     cl (v − v ) + cm (u − u ) + d (2 θ + θ ) 
   Z2 
} =  z1  = − 12  + 
(e) (e ) Y1 Y2 X2 X1 Z1
{rLocal } = −{ f int 
 f x2 
qL
 x   − al (u X 1 − u X 2 ) − am (vY 1 − vY 2 ) 
f   2  − bl (vY 1 − vY 2 ) − bm (u X 2 − u X 1 ) − c( θZ 1 + θZ 2 )
 y2   qy L   
mz 2     cl (vY 1 − vY 2 ) + cm (u X 2 − u X 1 ) + d ( θZ 1 + 2 θZ 2 ) 
 2 2
 − qy L 
 12 
(e)
and the reaction vector in the Global system {rGlobal } = [A ]T {rLocal
(e)
} becomes:
T
 l m 0 0 0 0  f x1   f x1l − f y1m 
− m    
 l 0 0 0 0  f y1   f x1m + f y1l 
(e)
 0 0 1 0 0 0  m   m (int ) 
{rGlobal }=   
z1
=
z1

 0 0 0 l m 0 f
 x2   x2 f l − f y 2m 
 0 0 0 −m l 0  f  f m+ f l 
   y2   x2 y2

 0 0 0 0 0 1 mz 2   mz 2 
Element 1:
Internal force (Local system)
 f x1   0   8.31338122   8.31338122 
       
 f y1   − 50   0   50 
(1) (1)
 m  − 41.666667   − 13.922142   27.74452425 
{rLocal } = −{ f int } =   = −
z1
+ = 
f
  x 2  0   − 8 . 31338122   − 8.31338122 
f   − 50   0   50 
 y2       
mz 2   41.666667   13.922142  − 27.74452425

University of Castilla-La Mancha Draft By: Eduardo W. V. Chaves (2016)


Ciudad Real – Spain
6 LINEAR ELASTICITY 855

and in the global system becomes: Global


T
 l m 0 0 0 0  f x1   f x1l − f y1m   8.31338122  1
− m     
 l 0 0 0 0  f y1   f x1m + f y1l   50  2

(1)
 0 0 1 0 0 0  m   m z1   27.74452425  3
{rGlobal } =   
z1
= = 
 0 0 0 l m 0  f x 2   f x 2 l − f y 2 m   − 8.31338122  4
 0 0 0 −m l 0  f  f m + f l   50  5
   y2   x2 y2
  
 0 0 0 0 0 1 m z 2   mz 2  − 27.74452425 6

Element 2:
Internal force (Local system)
 f x1  0  50   50 
  0  − 8.313812   − 8.313812 
 f y1       
( 2) (2)

 m 
 
 0 
 
 − 13 . 8223818
 
 − 13. 8223818
{rLocal } = −{ f int } =  z1  = −   +  = 
 f x2  0  − 50   − 50 
f  0  8.3133812   8.3133812 
 y2       
mz 2  0  − 27.744524   − 27.744524 

and in the global system becomes: Global


 f x1l − f y1m   8.31338122  7
   
 f x1m + f y1l   50  8
( 2)
 m z1   − 13.8223818  9
{rGlobal } =  = 
 f x 2 l − f y 2 m   − 8.31338122  1
f m + f l   − 50  2
 x2 y2
  
 mz 2  − 27.74452425 3
Element 3:
Internal force (Local system)
 f x1  0  50   50 
  0  8.313812   8.313812 
 f y1       
( 3) ( 3)
 m  
 0 
 
 27 . 744524 
 
 27 .744524 
{rLocal } = −{ f int } =  z1  = −  +  = 
 f x2  0  − 50   − 50 
f  0 − 8.3133812 − 8.3133812
 y2       
mz 2  0  13.8223818   13.8223818 

and in the global system becomes:


Global
 f x1l − f y1m   8.31338122  4
   
 f x1m + f y1l   − 50  5
( 3)

 m 
 
 27 . 74452425  6
{rGlobal }=  z1
= 
 f x 2 l − f y 2 m  − 8.31338122 10
f m + f l   50  11
 x2 y2
  
 mz 2   13.8223818  12

University of Castilla-La Mancha Draft By: Eduardo W. V. Chaves (2016)


Ciudad Real - Spain
856 SOLVING PROBLEMS BY MEANS OF CONTINUUM MECHANICS

The Global reaction vector, (see Figure 6.208), becomes:


Global
 8.31338122 + ( −8.31338122)   0 1
 50 + (−50)   0 2
   
27.74452425 + (−27.74452425)  0 3
   
 − 8.31338122 + 8.31338122   0 4
 50 + (−50)   0 5
   
 − 27.74452425 + 27.74452425   6
3
0
A
(e)
{R} = {rGlobal } =  = 
e =1  8.31338122   8.31338122 7
 50   50 8
   
 − 13.8223818   − 13.8223818 9
 − 8.31338122  − 8.3133812210
   
 50   50 11
   
 13.8223818   13.8223818 12
and

Y
qY = −20

2
1
1

2 3

50 50
13.822 13.822

8.313 8.313 X
3 4

Figure 6.208: Reactions.

University of Castilla-La Mancha Draft By: Eduardo W. V. Chaves (2016)


Ciudad Real – Spain
6 LINEAR ELASTICITY 857

Problem 6.73
Consider the 2D Framed structure described in Figure 6.209, where the mechanical -
geometrical properties for the beams are EI z = 104 and EA = 3 × 104 . Obtain the
displacements of the node 1 (rotation) and 2 .

Y Coordinates
P = −6 Nodes
X Y
3 1 0 4
4
M z = −5
2 7 4
3 7 0
1
1 2 Element Connectivity
e i→ j
2 1 1→ 2
2 2→3

3
O qY = −5 X

Figure 6.209: 2D Framed structure.


Solution:
By considering 2D framed we have three degrees-of freedom per node, so the Global
displacement vector for the structure has 12 degrees-of-freedom (Number of nodes (4)
times Number of degrees-of-freedom per node (3)), (see Figure 6.210).

U 1 
U 
Y  2
U 3 
U2 U5  
U3 U 4 
 
U1 U6 U4 {U Global } = U 5 
U 
1 1 2  6
U 7 
 
2 U 8 
U 9 
U8
U9
U7
O X
3

Figure 6.210: Degrees-of-freedom – Global system.


Next we will construct the system {FGlobal } = [ K Global ]{U Global } where [ K Global ] is the Global
stiffness matrix of the structure, and {FGlobal } is the Global nodal force vector.
Construction of the Global Stiffness Matrix - [ K Global ]
The matrix [ K Global ] can be constructed by assembling the beam elements, i.e.:

University of Castilla-La Mancha Draft By: Eduardo W. V. Chaves (2016)


Ciudad Real - Spain
858 SOLVING PROBLEMS BY MEANS OF CONTINUUM MECHANICS

A[k
(e)
[ K Global ]9×9 = Global ] (6.642)
e =1

The stiffness matrix for the 2D framed element can be obtained by using the equation in
(6.628). For each element we have
Connectivi ty : N1 → N2
Element Node1 : ( X 1 , Y1 ) L(e ) l (e )
=
X 2 − X1
m (e) = Y2 −(e)Y1
N1 → N2  L(e ) L
Node2 : ( X 2 , Y2 )
1 : (0,4)
1 1 → 2 7 l (1) = 1 m (1) = 0
 2 : ( 7 , 4)
3 : (7,4)
2 3 → 12 4 l ( 2) = 0 m ( 2) = −1
1 : (7,0)
Element 1: 1 2 3 4 5 6 Global
 4.28571 0 0 − 4.28571 0 0  1
 0 0.34985 1.22449 0 − 0.34985 1.22449  2

(1)
 0 1.22449 5.71429 0 − 1.22449 2.85714  3
[k Global ] = 10 3 ×  
 − 4.28571 0 0 4.28571 0 0  4
 0 − 0.34985 − 1.22449 0 0.34985 − 1.22449 5
 
 0 1.22449 2.85714 0 − 1.22449 5.71429  6
Element 2:
4 5 6 7 8 9 Global
 1.875 0 3.75 − 1.875 0 3.75  4
 0 7 .5 0 0 − 7 .5 0  5

( 2)
 3.75 0 10 − 3.75 0 5  6
[kGlobal ] = kij( 2 ) = 103 ×  
− 1.875 0 − 3.75 1.875 0 − 3.75 7
 0 − 7 .5 0 0 7.5 0  8
 
 3.75 0 5 − 3.75 0 10  9
After the contribution of all elements we have assembled we can obtain
2

A[k
(e)
[ K Global ] = Global ]
e =1

1 2 3 4 5 6 7 8 9 Global
k11(1) k12(1) k13(1) k14(1) k15(1) k16(1) 0 0 0  1
 (1) 
k 21
(1)
k 22 (1)
k 23 (1)
k 24 (1)
k 25 (1)
k 26 0 0 0  2
k (1) (1)
k 32 (1)
k 33 (1)
k 34 (1)
k 35 (1)
k 36 0 0 0  3
 31 
(1)
k 41
(1)
k 42 (1)
k 43 (1)
k 44 + k11( 2 ) (1)
k 45 + k12( 2 ) (1)
k 46 + k13( 2 ) k14( 2 ) k15( 2 ) k16( 2)  4
[ K Global ] = k (1) (1)
k 52 (1)
k 53 (1)
k 54 + k 21( 2) (1)
k 55 + k 22(2) (1)
k 56 + k 23( 2) (2)
k 24 ( 2)
k 25 ( 2) 
k 26 5
 51 
(1)
k 61 (1)
k 62 (1)
k 63 (1)
k 64 + k 31( 2) (1)
k 65 + k 32(2) (1)
k 66 + k 33( 2) (2)
k 34 ( 2)
k 35 k 36  6
( 2)

 (2) ( 2) (2) (2) ( 2) ( 2) 


 0 0 0 k 41 k 42 k 43 k 44 k 45 k 46  7
 0 0 0 (2)
k 51 ( 2)
k 52 (2)
k 53 (2)
k 54 ( 2)
k 55 ( 2) 
k 56 8
 (2) ( 2) (2) (2) ( 2) ( 2) 
 0 0 0 k 61 k 62 k 63 k 64 k 65 k 66  9

University of Castilla-La Mancha Draft By: Eduardo W. V. Chaves (2016)


Ciudad Real – Spain
6 LINEAR ELASTICITY 859

By substituting the numerical values we can obtain:


 4.286 0 0 − 4.286 0 0 0 0 0 
 0 0.35 1.224 0 − 0.35 1.224 0 0 0 

 0 1.224 5.714 0 − 1.224 2.875 0 0 0 
 
− 4.286 0 0 6.161 0 3.75 − 1.875 0 3.75 
[ K Global ] =  0 − 0.35 − 1.224 0 7.85 − 1.224 0 − 7.5 0  ×103
 
 0 1.224 2.875 3.75 − 1.224 15.714 − 3.75 0 5 
 0 0 0 − 1.875 0 − 3.75 1.875 0 − 3.75
 
 0 0 0 0 − 7.5 0 0 7.5 0 
 
 0 0 0 3.75 0 5 − 3.75 0 10 

Construction of the Global Nodal Force Vector - {FGlobal } , (see equation (6.632))
 2 
 
A
(e)
{FGlobal } =  { f Eq _ G } + {F0 }
 e =1 
 
Element 1: Concentrated force, (see Problem 6.67 –NOTE), ( P = −6 , a P = 4 , L = 7 ):
 0 
  2aP3 3aP2  
  3 − 2 + 1 P 
 L L    0   f1( Eq _ L ) 
 a 3 2a 2    
  − 2.361516  f 2( Eq _ L ) 

 2 −
P P 
+ aP  P 
 ( Eq _ L ) 
(1)  L
{ f Eq _ L } = 
L   = − 4.408163 =  f 3
0     ( Eq _ L ) 
   0   f4 
  − 2aP + 3aP  P  − 3.638484  f ( Eq _ L ) 
3 2

  L3 L2      5( Eq _ L ) 
   5 . 877551   f 6 
  aP − aP  P 
3 2

  L2 L  
   
And by means of the equation { f Eq(1)_ G } = [A ]T { f Eq(1)_ L } we can obtain the vector in the
Global system, (see equation (6.631)): Global
 f1 ( Eq _ G ) _(1)
  f1( Eq _ L )
l − f2 ( Eq _ L )
m  0 1
 ( Eq _ G ) _(1)   ( Eq _ L ) 
 f2   f1

m + f 2( Eq _ L ) l − 2.361516 2
(1)
 f ( Eq _ G ) _(1)   f 3( Eq _ L ) − 4.408163 3
_ G } =  ( Eq _ G ) _(1)  =  ( Eq _ L )
3
{ f Eq
 f4   f4 l − f 5( Eq _ L ) m  0 
4
 f ( Eq _ G ) _(1)   f ( Eq _ L ) m + f ( Eq _ L ) l − 3.638484 5
 5( Eq _ G ) _(1)   4 5
 
 f 6   f 6( Eq _ L )  5.877551  6

Element 2: Linearly distributed load, (see Problem 6.67 –NOTE), ( q (y1) = 0 , q (y2) = −5 ,
L = 4 ):

University of Castilla-La Mancha Draft By: Eduardo W. V. Chaves (2016)


Ciudad Real - Spain
860 SOLVING PROBLEMS BY MEANS OF CONTINUUM MECHANICS

 L (2) 
 20 (7 q y + 3q y )  
(1)
0   f1( Eq _ L ) 
 2   −3   ( Eq _ L ) 
 L    f2 
(3q (y1) + 2q (y2) )  − 2.666667  f ( Eq _ L ) 
( 2)  60     
{ f Eq _ L} =   =  =  3( Eq _ L ) 
L 0
 (3q (y1) + 7 q (y2 ) )     f4 
 20   − 7   f ( Eq _ L ) 
 − L2 (1) (2)     5( Eq _ L ) 
 ( 2 q y + 3q y )   4   f 6 
 60 
And by means of the equation { f Eq( 2)_ G } = [A ]T { f Eq( 2)_ L } we can obtain the vector in the
Global system, (see equation (6.631)): Global
 f1 ( Eq _ G ) _( 2 )
  f1( Eq _ L )
l − f2 ( Eq _ L )
m  − 3  4
 ( Eq _ G ) _( 2 )   ( Eq _ L ) ( Eq _ L ) 
 f2   f1 m + f2 l   0  5

( 2)
 f ( Eq _ G ) _( 2 )   f 3( Eq _ L )  − 2.666667 6
{ f Eq _ G } =  ( Eq _ G ) _( 2 )  =  ( Eq _ L )
3
=
 f4   f4 l − f 5( Eq _ L) m   − 7  7
 f ( Eq _ G ) _( 2 )   f ( Eq _ L ) m + f ( Eq _ L ) l   0  8
 5( Eq _ G ) _( 2 )   4 5
  
 f 6   f 6( Eq _ L )   4  9

Then,
Global
 f1( Eq _ G ) _(1)   0  1
   
  − 2.361516  2
( Eq _ G ) _(1)
 f2
 f ( Eq _ G ) _(1)  − 4.4081633 3
 ( Eq _ G ) _(3 1)   
2  f 4 + f 1
( Eq _ G ) _( 2 )
  −3  4
 ( Eq _ G ) _(1) ( Eq _ G ) _( 2 )   
A
(e)
{ f Eq _G } =  f5 + f2 =
  − 3 . 638484  5
e =1  f ( Eq _ G ) _(1) + f ( Eq _ G ) _( 2 )   3.210884  6
 6 3   
 f 4( Eq _ G ) _( 2 )   −7  7
   
 f5 ( Eq _ G ) _( 2 )
  0  8
 f 6( Eq _ G ) _( 2 )   4  9
And
 0  0  0 
 − 2.361516   0   − 2.361516 
     
− 4.4081633  0  − 4.4081633
     
 2   −3  0  −3 
       
A
(e)
{FGlobal } =  { f Eq }
_G  + {F0 } =  − 3. 638484 + 0
    = − 3 . 638484 
 e =1   3.210884  − 5  − 1.789116 
 
     
 −7  0  −7 
     
 0  0  0 
 4   4   4 

Applying the Boundary Conditions


Note that there are restrictions in motion for the following degrees-of-freedom: U 1 = 0 ,
U 2 = 0 , U 7 = 0 , U 8 = 0 , U 9 = 0 and after applying these boundary conditions the set of
equation {FGlobal } = [ K Global ]{U Global } will have the following aspect:

University of Castilla-La Mancha Draft By: Eduardo W. V. Chaves (2016)


Ciudad Real – Spain
6 LINEAR ELASTICITY 861

1 0 0 0 0 0 0 0 U 1   0 0 
0  U   
 1 0 0 0 0 0 0  2   0 0 
0 0 5.714 0 0 0 0 U 3  − 4.4081633
− 1.224 2.875
    
0 0 0 6.161 0 3.75 0 0 0 U 4   −3 
   
10 × 0
3
0 − 1.224 0 7.85 
− 1.224 0 0 0 U 5  =  − 3.638484 
 
0 0 2.875 3.75 − 1.224 15.714 0 0 0 U 6   − 1.789116 
   
0 0 0 0 0 0 1 0 0 U 7   0 
    
0 0 0 0 0 0 0 1 0 U 8   0 
 
0 0 0 0 0 0 0 0 1 U 9   0 

Solving the System


U1   0 
U   
 2  0 
U 3  − 9.7643
   
U 4  − 5.8224
    −4
U 5  =  − 5.914  × 10
U   1.5654 
 6  
U 7   0 
   
U 8   0 
U 9   0 

Internal and Reaction Forces


(e) (e) (e ) (e)
The internal and reaction forces can be obtained by {rLocal } = −{ f Eq _ L } + { f Local } = −{ f int } ,
(e)
(see equation (6.634)). And the reaction vector in the Global system {rGlobal } = [A ]T {rLocal
(e)
},
(see equation (6.636)).
Element 1:
Internal force (Local system)
 f x1   0   2.495322   2.495322 
  − 2.361516  − 0.797039   1.564477 
 f y1       
(1) (1)

 m 
 
 − 4 . 408163
 
 − 4 . 408163 
 
 0 
{rLocal } = −{ f int } =  z1  = −  + = 
 f x2   0   − 2.495355  − 2.495355
f  − 3.638484  0.797039   4.435523 
 y2       
mz 2   5.877551  − 1.1711112 − 7.048662
144244 3 144244 3
(1 ) (1 )
={ f Eq _ L} ={ f Local }

and in the global system becomes: Global


 rG(1)1   f x1l − f y1m   2.495322  1
 (1)     
rG 2   f x1m + f y1l   1.564477  2
r (1)   m z1   0  3
(1) G 3
{rGlobal }=  (1)  = = 
r
 G 4  f x 2 l − f y 2 m  − 2.495355 4
rG(1)   f m + f l   4.435523  5
 (1) 5   x2 y2
  
rG 6   mz 2  − 7.048662 6

University of Castilla-La Mancha Draft By: Eduardo W. V. Chaves (2016)


Ciudad Real - Spain
862 SOLVING PROBLEMS BY MEANS OF CONTINUUM MECHANICS

Element 2:
Internal force (Local system)
 f x1   0   4.4355232   4.4355232 
    − 0.5046776  2.495322 
 f y1   −3     
( 2) ( 2)

 m 
 
 − 2 . 666667 
 
 − 0. 6180044 
 
 2. 048662 
{rLocal } = −{ f int } =  z1  = −  + = 
 f x2   0  − 4.4355232 − 4.4355232
f   −7   0.5046776   7.504678 
 y2       
mz 2   4   − 1.4007058   − 5.400706 
144244 3 1442443
(2) (2)
={ f Eq _ L} ={ f Local }

and in the global system becomes: Global


 rG( 2 )1   f x1l − f y1m   2.49532245  7
 ( 2)     
rG 2   f x1m + f y1l  − 4.43552318 8
r ( 2 )   m z1   2.04866225  9
(2) G 3
{rGlobal }=  ( 2)  = = 
rG 4   f x 2 l − f y 2 m   7.50467755  1
rG( 2 )   f m + f l   4.43552318  2
 ( 2) 5   x2 y2
  
rG 6   mz 2  − 5.40070578 3
The Global reaction vector becomes:
 rG(1)1   2.4953224 
 (1)   
 rG 2   1.5644768 
 r (1)   0 
 (1) G 3 ( 2 )   
2  rG 4 + rG 1   0 
   
A
(e)
{rGlobal } = rG(1) 5 + rG( 2 ) 2  =  0 
e =1 r (1) + r ( 2 )   − 5 
 G 6 G 3  
 rG( 2) 4   7.5046776 
 ( 2)   
 rG 5   4.4355232 
 rG( 2 )  − 5.4007058
 6 
And Global
 2.4953224   0   2.4953224  1
 1.5644768   0   1.5644768  2
     
 0  0  0  3
     
2  0  0  0  4
     
A
(e)
{R} = {rGlobal } − {F0 } =  0 − 0 = 0  5
e =1  −5   −5   0  6
     
 7.5046776   0   7.5046776  7
      8
 4.4355232   0   4.4355232 
− 5.4007058  0  − 5.4007058 9

University of Castilla-La Mancha Draft By: Eduardo W. V. Chaves (2016)


Ciudad Real – Spain
6 LINEAR ELASTICITY 863

Problem 6.74
Obtain the explicit equation [ K (e ) ] {u ( e ) } = {F ( e ) } for the beam presented in Figure 6.211,
in which the beam is supported by an elastic foundation.

z
y
qz < 0

w( x) < 0

K zf

Figure 6.211: Beam element on elastic foundation.


Solution:
In this situation, the total potential energy is given by:
Π = U int − U ext = (U beam + U spring ) − U ext (6.643)
According to the stationary principle of the total potential energy we can conclude:

=0 ⇒
d
(U beam + U spring − U ext = 0 ⇒) dU beam
+
dU spring

dU ext
=0
d {u (e ) } d {u (e ) } d {u ( e ) } d {u (e ) } d {u ( e ) }
(6.644)
or
dU beam dU spring dU ext dU spring
+ − =0 ⇒ [ke (1) ]{u (e ) } + = { f (e) } (6.645)
d {u ( e ) } d {u ( e ) } d {u ( e ) } d {u } (e)

dU spring
where [ke (1) ] is the same matrix showed in Problem 6.62. Then, the term can be
d {u (e ) }
obtained as follows. The internal energy for the spring, (see Figure 6.212), is given by:
L
1
∫2K
f
U spring = z w 2 dx (6.646)
0

University of Castilla-La Mancha Draft By: Eduardo W. V. Chaves (2016)


Ciudad Real - Spain
864 SOLVING PROBLEMS BY MEANS OF CONTINUUM MECHANICS

F F stored energy
1 1
wF = wK zf w
w 2 2
K zf
K zf 1

w w

Figure 6.212: Spring element.

Considering that the spring coefficient K zf is constant in the beam element the equation
(6.646) becomes:
L
K zf
∫ w dx
2
U spring = (6.647)
2 0

Moreover, by means of equation, (see Problem 6.62 – NOTE 2):


L
13L 2 13L 2 L3 2 L3 2 9L 11L2 13L2

0
w 2 dx =
35
w1 +
35
w2 +
105
θ y1 +
105
θy2 +
35
w1 w2 −
105
w1 θ y1 +
210
w1 θ y 2

13L2 11L2 L3
− w2 θ y1 + w2 θ y 2 − θ y1 θ y 2
210 105 70
we can obtain:
K f  13L 2 13L 2 L3 2 L3 2 9 L 11L2
U spring =  w + w + θ + θ + w w − w1 θ y1 +
2  35
1 2 y1 y 2 1 2
35 105 105 35 105
13L2 13L2 11L2 L3 
w1 θ y 2 − w2 θ y1 + w2 θ y 2 − θ y1 θ y 2 
210 210 105 70 
(6.648)
Then
∂U spring K zf  26 L 9L 11L2 13L2 
=  w1 + w2 − θ y1 + θy2  (6.649)
∂w1 2  35 35 105 210 
∂U spring K zf  2 L3 11L2 13L2 L3 
=  θ y1 − w1 − w2 − θy 2  (6.650)
∂ θ y1 2 105 105 210 70 
∂U spring K zf  26 L 9L 13L2 11L2 
=  w2 + w1 − θ y1 + θy 2  (6.651)
∂w2 2  35 35 210 105 
∂U spring K zf  2 L3 13L2 11L2 L3 
=  θ y2 + w1 + w2 − θ y1  (6.652)
∂θ y 2 2 105 210 105 70 

Restructuring the above set of equations in matrix form we can obtain:

University of Castilla-La Mancha Draft By: Eduardo W. V. Chaves (2016)


Ciudad Real – Spain
6 LINEAR ELASTICITY 865

 13 − 11L 9 13L 
 35 210 70 420  w 
  1
 − 11 L L2 − 13L − L2   
dU spring θ
= K zf L  210 105 420 140   y1 
d {u ( e ) }  9 − 13L 13 11L  w2 

 70  
 420 35 210  θ  (6.653)
 13L − L2 11L L2   y 2 
 420 140 210 105 
dU spring
(e)
= [ke ( Spring _ z ) ]{u ( e ) }
d {u }
where

 13L − 11L2 9L 13L2 


 
 35 2 210 70 420 
 − 11L L3 − 13L2 − L3 
 140 
[ke ( Spring _ z ) ] = K zf  210 105 420 (6.654)
9L − 13L2 13l 11L2 
 
 70 420 35 210 
 13L2 − L3 11L2 L3 
 
 420 140 210 105 

Then, the equation (6.645) becomes


dU spring
[ke (1) ]{u (e ) } + (e)
= { f (e) } ⇒ [ ke (1) ]{u ( e ) } + [ke ( Spring _ z ) ]{u (e ) } = { f ( e ) }
d {u } (6.655)
[ (1)
⇒ [ke ] + [ ke ( Spring _ z )
] (e)
] {u } = { f (e)
}

NOTE 1: Note that if the elastic foundation is orientated according to the plane x − y ,
(see Problem 6.67), the internal energy becomes:
L
K yf K yf  13L 2 13L 2 L3 2 L3 2 9 L
U spring =
2 ∫
0
v 2 dx = 
2  35
v1 +
35
v 2 +
105
θ z1 +
105
θz 2 +
35
v1v2 +

11L2 13L2 13L2 11L2 L3 


v1θz1 − v1θz 2 + v2 θz1 − v2 θz 2 − θz1θz 2 
105 210 210 105 70 
(6.656)
And the stiffness matrix becomes:

 13L 11L2 9L − 13L2 


 
 352 210 70 420 
 11L L3 13L2 − L3 
 140 
[ke ( Spring _ y ) ] = K yf  210 105 420 (6.657)
9L 13L2 13L − 11L2 
 
 70 420 35 210 
 − 13L2 − L3 − 11L2 L3 
 
 420 140 210 105 

University of Castilla-La Mancha Draft By: Eduardo W. V. Chaves (2016)


Ciudad Real - Spain
866 SOLVING PROBLEMS BY MEANS OF CONTINUUM MECHANICS

Problem 6.75
Consider the beam describe in Figure 6.213. Obtain the displacements at the node 2 and
the rotations at the nodes 1 and 3 .

z
1 .5 m 1 .5 m EI y = 3360kN m 2
K zf = 200kN / m 2

q z = −10kN / m

1 2 2 x
1 3

K zf

Figure 6.213: Beam and elastic foundation.


Solution:
For this problem the global degrees-of-freedom are described in Figure 6.214.

w (1)
U 1 
z θ y (2) U 
 2
Degrees-of-freedom U 3 
{U } =  
U1 U3 U5 U 4 
U2 y U4 U6 U 5 
 
U 6 
1 2 2 x
1 3

Figure 6.214: Global degrees-of-freedom.


Construction of the Global Stiffness Matrix - [ K Global ]
The matrix [ K Global ] can be constructed by assembling the individual beam elements, i.e.:
2

A[k
(e)
[ K Global ]6×6 = Global ] (6.658)
e =1

The stiffness matrix for the adopted system was obtained in Problem 6.62. And the elastic
foundation stiffness matrix was obtained in (6.654).
For each element we have

University of Castilla-La Mancha Draft By: Eduardo W. V. Chaves (2016)


Ciudad Real – Spain
6 LINEAR ELASTICITY 867

Element 1:
1 2
Global 3 4
 1.194667 − 0.896 − 1.194667 − 0.896 1
 − 0.896 0.896 0.896 0.448  2
(1)
[k1Global (1)
] = [k1Local ] = 10 4 × 
− 1.194667 0.896 1.194667 0.896  3
 
 − 0.896 0.448 0.896 0.896  4

1 2 3 4 Global
 111.42857 − 23.57143 38.57143 13.92857  1
− 23.57143 6.42857 − 13.92857 − 4.82143 2
( Spring _ z )(1)
[k Global ( Spring _ z )(1)
] = [k Local ]= 
 38.57143 − 13.92857 111.42857 23.57143  3
 
 13.92857 − 4.82143 23.57143 6.42857  4
Then
(1) (1) ( Spring _ z )(1)
[k Global ] = [k1Global ] + [k Global ]

1 2 Global 3 4
 1.20581 − 0.89836 − 1.19081 − 0.89461 1
− 0.89836 0.89664 0.89461 0.44752  2
k ij(1) (1) 4
≡ [k Global ] ≈ 10 × 
 − 1.19081 0.89461 1.20581 0.89836  3
 
 − 0.89461 0.44752 0.89836 6.42857  4
(2) (1) ( Spring _ z )( 2 ) ( Spring _ z )(1)
Element 2: Note that [k1Global ] = [k1Global ] , [k Global ] = [k Global ] , then

3 4Global 5 6
 1.20581 − 0.89836 − 1.19081 − 0.89461 3
 − 0.89836 0.89664 0.89461 0.44752  4
k ij( 2 ) ( 2)
≡ [k Global (1)
] = [k Global ] ≈ 10 4 × 
 − 1.19081 0.89461 1.20581 0.89836  5
 
 − 0.89461 0.44752 0.89836 6.42857  6
After all elements have been assembled we can obtain
2

A[k
(e)
[ K Global ] = Global ]
e =1

1 2 3 4 5 6 Global
 k11(1) k12(1) k13(1) k14(1) 0 0  1
 (1) (1) (1) (1)  2
 k 21 k 22 k 23 k 24 0 0 
 k (1) (1)
k 32 (1)
k 33 + k11( 2 ) (1)
k 44 + k12( 2 ) k13( 2 ) k14( 2 )  3
[ K Global ] =  31 
(1)
 k 41
(1)
k 42 (1)
k 43 + k 21( 2) (1)
k 44 + k 22( 2) (2)
k 23 ( 2)
k 24  4
 0 0 ( 2)
k 31 ( 2)
k 32 (2)
k 33 ( 2) 
k 34 5
 
 0 0 ( 2)
k 41 ( 2)
k 42 (2)
k 43 ( 2)
k 44  6

By substituting the numerical values we can obtain:

University of Castilla-La Mancha Draft By: Eduardo W. V. Chaves (2016)


Ciudad Real - Spain
868 SOLVING PROBLEMS BY MEANS OF CONTINUUM MECHANICS

1 2 3 4 5 6 Global
 1.20581 − 0.89836 − 1.19081 − 0.89461 0 0  1
− 0.89836 0.89664 0. 89461 0 .44752 0 0  2
 
4
 − 1.19081 0.89461 2.41162 0 − 1.19081 − 0.89461 3
[ K Global ] = 10 ×  
 − 0.89461 0.44752 0 1.79329 0.89461 0.44752  4
 0 0 − 1.19081 0.89461 1.20581 0.89836  5
 
 0 0 − 0.89461 0.44752 0.89836 0.89664  6
Construction of the Global Nodal Force Vector - {FGlobal }
 2 
 
A
(e)
{FGlobal } =  { f Eq _ G } + {F0 }
 e =1  {
  ={0 }

Element 1: uniformly distributed load, ( q z = −10 , L = 1.5 ):


 qz L  Global
 2 
 2  − 7 .5  1
 − qz L   
(1) (1)  12   1.875  2
{ f Eq _ G } = { f Eq _ L } =  =
qL   
 z   − 7 .5  3
 2  − 1.875 4
 q z L2 
 
 12 
Element 2: { f Eq(1)_ G } = { f Eq( 2)_ G }
Then,
Global
 (1)
{ f Eq _ G }1
  − 7 .5  1
 (1)    2
 { f Eq _ G }2   1.875 
 2  { f (1) } + { f ( 2) }   − 15  3
   Eq _ G 3 Eq _ G 1   
A
(e)
{FGlobal }6×1 =  { f Eq _G  = 
} = 
 e =1
(1) ( 2)
 { f Eq _ G }4 + { f Eq _ G }2   0  4
 
 ( 2)
{ f Eq _ G }3
  − 7 .5  5
   
 − 1.875

( 2)
{ f Eq _ G }4 6

Applying the Boundary Conditions


Note that there are restrictions to move for the following degrees-of-freedom: U 1 = 0 ,
U 5 = 0 , and after applying these boundary conditions the set of equation will have the
following aspect:
1 0 0 0 0 0  U 1   0 
0 0.89664 0.89461 0.44752 0 0  U   1.875 
  2   
0 0.89461 2.41162 0 0 − 0.89461 U 3   − 15 
10 4 ×    =  
0 0.44752 0 1.79329 0 0.44752  U 4   0 
0 0 0 0 1 0  U 5   0 
    
0 0 − 0.89461 0.44752 0 0.89664  U 6  − 1.875

Solving the System

University of Castilla-La Mancha Draft By: Eduardo W. V. Chaves (2016)


Ciudad Real – Spain
6 LINEAR ELASTICITY 869

U 1   0 
U   
 2   3.1939374 
U 3  − 2.9916161 −3
 =  × 10
U 4   0 
U 5   0 
   
U 6  − 3.1939374

Internal and Reaction Forces


The internal and reaction forces can be obtained as follows
(e) (e ) (e) (e) (e) (e) (e)
{rLocal } = −{ f Eq _ L } + { f Local } = −{ f Eq _ L } + [ k Local ][A ]{uGlobal } = −{ f int }

(e)
And the reaction vector in the Global system {rGlobal } = [A ]T {rLocal
(e)
} . Note also that for this
problem the Global and Local systems have the same orientation, so [A ] = [1] .
Element 1:
Internal force (Local system=Global system)
 f z1   − 7.5   6.931485   14.431485 
   1.875   1.875   
m       0 
(1)
{rLocal (1)
} = −{ f int } =  y1  = −  +
  =
 
(1)
 = {rGlobal }
f
 z2   − 7 . 5   − 7 .5   0 
m y 2  − 1.875 − 12.581957  − 10.706957 
  1424 3 144244 3
(1 ) ( 1)
={ f Eq _ L} ={ f Local }

Element 2:
Internal force (Local system=Global system)
 f z1   − 7.5   − 7.5   0 
       
m   1.875  12.581957  10.706957
( 2)
{rLocal ( 2)
} = −{ f int } =  y1  = −  +
  =
 
(2)
 = {rGlobal }
f
 z2   − 7 . 5   6 . 931485   14 . 431485 
m y 2  − 1.875  − 1.875   0 
  1424 3 142 4 43 4
(2) (2)
={ f Eq _ L} ={ f Local }

The Global reaction vector becomes:


Global
 rG(1)1  14.431485 1
 (1)    2
 rG 2   0 
2 
 r (1)
+ r ( 2) 
   0  3
A
(e)
{R} = {rGlobal } − {F0 } =  G(1) 3 G( 2 ) 1  =  
{ 4
e =1 ={0 } rG 4 + rG 2   0 
 rG( 2 )  14.431485 5
 3
  
 rG 4 (2)
  0  6
References
GERE, J.M. & WEAVER JR., W. (1965). Analysis of Framed Structures. Van Nostrand Reinhold,
U.S.
SECHLER, E.E. (1952). Elasticity in engineering. John Wiley & Sons, Inc., New York.
LAIER, J.E.; BAREIRO, J.C., (1983). Complemento de resistência dos materiais. Publicação 073/92
São Carlos - USP - EESC.

University of Castilla-La Mancha Draft By: Eduardo W. V. Chaves (2016)


Ciudad Real - Spain
870 SOLVING PROBLEMS BY MEANS OF CONTINUUM MECHANICS

UGURAL, A.C.; FENSTER, S.K., (1984). Advanced strength and applied elasticity - The SI version.
Elsevier Science Publishing Co. Inc., New York.

Problem 6.76
Consider a beam of length L where the internal forces are schematically described in
Figure 6.215. Apply the Principle of Complementary Virtual Work to the beam, (see
Problem 5.22 – NOTE 4).

z, w y, v Mz
My
Qy
Qz M x ≡ MT
N x, u

Figure 6.215: Internal forces in the beam.


Obs.: Consider the elastic problem without body forces, and also consider that the beam is
only subjected by concentrated loads.
Solution:
In Problem 5.22 – NOTE 4 we have shown that:
r
loc r loc Principle of Complementary
F4
1 2⋅ u43 = ∫ σ : ε dV Virtual Work (static case
Total external complementary virtual work V
14243 (6.659)
(due to concentrated forces)
Total internal
without body forces and with
complementary virtual work concentrated forces)
r
with σ ⋅ nˆ = t * on S σ .
The total internal complementary virtual work:

∫ ∫ ∫
Wint = σ : ε dV = σ ij ε ij dV = ( σ11ε 11 + σ 22 ε 22 + σ 33 ε 33 + 2 σ12 ε 12 + 2σ 23 ε 23 + 2σ13 ε 13 )dV
V V V


= ( σ x ε x + σ y ε y + σ z ε z + τ xy γ xy + τ yz γ yz + τ xz γ xz )dV
V

In the previous problem we have expressed the stresses in terms of internal forces, (see
Problem 6.62):
σ x = σ (x1) + σ (x2 ) + σ (x3) :
L L
σ (x1) N N N

V
σ x(1) ε (x1) dV = σ x(1)∫
V
E
dV = ∫
0
A EA A ∫
dAdx = N
0
EA
dx ∫
L L L
My My My My
∫ σ x( 2) ε (x2 ) dV ∫ ∫ ∫ ∫ ∫
2
= z zdAdx = M y z dAdx = M y dx
V 0 A
Iy EI y 0
EI y2 A 0
EI y
L
Mz

V
σ x(3) ε (x3) dV = M z ∫ 0
EI z
dx

University of Castilla-La Mancha Draft By: Eduardo W. V. Chaves (2016)


Ciudad Real – Spain
6 LINEAR ELASTICITY 871

The component τ xz ( z ) = Gγ xz is related to the shearing force Q z , (see equation (6.216)),


where G is the shear modulus. In equation (6.228) we have obtained:
Qz Qz χ z
∫ τ( z, y)dy = I ∫ zdA = y A Iy
where ∫
χ z = zdA (6.660)
A

Then, the strain energy associated with τ xz ( z ) = Gγ xz is given by:

1  
L L
τ xz 1
∫ τγdV = ∫
2
τ xz dV = ∫ ∫
τ τ dAdx = τ τ dydz  dx
∫ ∫∫
G GA G 
V V 0 0  
(6.661)
Q z   χ z  
2
1   Q z χ z  Q z χ z  
L L L
 ς Q
= ∫ ∫   
G   I y  I y   
dz dx = Q z  
G   Iy  ∫ 
dz 

dx =∫ Q z z z dx
GA ∫
0   0   0

where ς z is the shape factor which is given by:


2
χ 
ςz = A  z
 Iy

∫  dz


(6.662)

In the same way we can obtain, (see Problem 6.62):


L 2
τ xy ς yQ y  χy 
∫τ
V
xy γ xy dV
V

= τ xy
G
dV = Qy ∫
0
GA
dx where ∫
ς y = A   dA
I
A z 
(6.663)

The internal complementary virtual work due to the torsion moment becomes:
τ
1  M T  M T 
L
MT
L
M
∫ ∫  r  ∫ ∫ ∫ ∫
2
τ(r ) γ (r )dV = τ dV =  r dV = M T r dAdx = M T T dx
V V
G V
G  J  J  0
2
GJ A 0
GJ
(6.664)
where J = ∫ r 2 dA is the polar inertia moment. When we are dealing with rectangular cross
A
section we can express the equivalent internal complementary virtual work as follows:
L
MT
∫M
0
T
GJ Eq
dx (6.665)

where J Eq is the equivalent inertia moment.


The total internal complementary virtual work:


Wint = ( σ x ε x + σ y ε y + σ z ε z + τ xy γ xy + τ yz γ yz + τ xz γ xz )dV
V

 N
L
My M ς Q ς yQy MT 
= N
∫ +My + M z z + Qz z z + Q y + MT dx
 EA EI y EI z GA GA GJ Eq 
0 
And the Principle of Complementary Virtual Work becomes:

University of Castilla-La Mancha Draft By: Eduardo W. V. Chaves (2016)


Ciudad Real - Spain
872 SOLVING PROBLEMS BY MEANS OF CONTINUUM MECHANICS

r
loc r loc
F
142 ⋅ u43 = ∫ σ : ε dV
Total external complementary virtual work V
(due to concentrated forces) 14243
Total internal
complementary virtual work

r r L
 N My M ς Q ς yQy MT 
⇒ F loc ⋅ uloc =  N
∫ +My + M z z + Qz z z + Q y + MT dx
 EA EI y EI z GA GA GJ Eq 
0 

NOTE: For the next example we will apply the principle of complementary virtual work,
in which we consider the beam fixed at one end, (see Figure 6.216).

My >0
z

q
1
x
x 2
qx 2
M y ( x) =
2

Figure 6.216: Beam fixed at one end under uniformly distributed load.
In this situation the principle of the complementary virtual work becomes:
r r L
My
F loc ⋅ u loc = M y∫ dx
0
EI y

If we want to know the deflection of the beam at x = 0 (node 1) we apply a concentrated


virtual load F (1) = 1 at this point. The moment diagram for real and virtual states are
presented in Figure 6.217.

REAL VIRTUAL

qx 2 z
M y ( x) =
z 2

F (1) = 1
M y ( x) = x
1 1

x 2 x 2

Figure 6.217: Beam fixed at one end under uniformly distributed load.

University of Castilla-La Mancha Draft By: Eduardo W. V. Chaves (2016)


Ciudad Real – Spain
6 LINEAR ELASTICITY 873

With that we can state that:


qx 2
r L
My L
My L L
loc r loc q
F ⋅u = M y ∫ dx ⇒ F (1)U(1) ∫
= My dx = x 2

dx = x3dx ∫
0
EI y 0
EI y 0
EI y 2 EI y 0

Taking into account that F (1) = 1 and by solving the integral we can obtain:
L
q qL4

(1)
U = x 3 dx =
2 EI y 0
8 EI y

which value matches the result in Problem 6.62- NOTE 4.


If we want to know the rotation of the beam at x = 0 we apply a concentrated virtual load
(moment) m (1) = 1 at this point. The moment diagram for real and virtual states are
presented in Figure 6.218.

REAL VIRTUAL

qx 2 z
M y ( x) =
z 2

M y ( x) = 1
m (1) = 1
1 1

x x 2
2

Figure 6.218: Beam fixed at one end under uniformly distributed load.

With that we can state that:


qx 2
r L
My L
My L L
loc r loc q
F ⋅u = M y ∫ dx ⇒ m (1)θ(1) ∫
= My dx = 1 2

dx = x 2 dx ∫
0
EI y 0
EI y 0
EI y 2 EI y 0

Taking into account that m (1) = 1 and by solving the integral we can obtain:
L
q qL3

(1)
θ = x 2 dx =
2 EI y 0 6 EI y

which value matches the result in Problem 6.62- NOTE 4.


Problem 6.77
Consider Figure 6.216 and apply the first and second Mohr Theorem, (see Problem 6.61),
to obtain the deflection and rotation at the free-end of the beam (node 1 in Figure 6.216).
Solution:

University of Castilla-La Mancha Draft By: Eduardo W. V. Chaves (2016)


Ciudad Real - Spain
874 SOLVING PROBLEMS BY MEANS OF CONTINUUM MECHANICS

The Mohr’s first theorem states: “The change in slope of a deflection curve between two
My
points is equal the area diagram of between these two points.”
EI y

The slope at the point 2, (see Figure 6.219), is zero ( ψ ( 2) = 0 ). Then,


(2) ( 2) ( 2)
My My My
∆ψ ( 2) _(1) = − ∫ EI
(1) y
dx ⇒ ψ ( 2) − ψ (1) = − ∫
(1)
EI y
dx ⇒ ψ (1) = ∫ EI
(1) y
dx

Moment diagram Moment diagram ÷ EI y

qx 2 My qx 2
M y ( x) = = (parabola)
z 2 z EI y 2 EI y
qL2
b=
2 EI y
A ⊕
1 2 1
2
x 3
L x= L (centroid)
4

bL qL3
A ≡ Area = =
3 6 EI y

∆ψ ( 2) _(1) = ψ ( 2 ) − ψ (1) = −ψ (1) deflection of the beam

∆d

2
1
L

Figure 6.219: Beam fixed at one end under uniformly distributed load.

According to Figure 6.219 we can conclude that:


( 2)
My qL3
ψ (1) = ∫ EI
(1) y
dx = A =
6 EI y

University of Castilla-La Mancha Draft By: Eduardo W. V. Chaves (2016)


Ciudad Real – Spain
6 LINEAR ELASTICITY 875

The Mohr’s second theorem states:


My
“The distance ∆d is equal to the first moment of the diagram about the axis where
EI y
∆d is measured.”
According to Figure 6.219 we can conclude that:
qL3 3 qL4
∆d = Ax = L=
6 EI y 4 8EI y

Problem 6.78
Consider a bar of length L , which has a squared cross section of side a . The elastic
constants of the material is assumed to be known ( E and ν = 0.25 ).
a) In the case of Figure 6.220(a), calculate the stored energy (strain energy density) in the
bar due to the deformation, and also obtain the total strain energy;
b) Determine the stored energy corresponds to the change of volume and to the change of
shape;
c) The same question described in paragraph (a) but considering the case of Figure
6.220(b).

A -cross section area

M M
P P

L a
a cross
M section
a) a

b)

Figure 6.220
Solution:
Considering a one dimensional case:
σx P
σ x = Eε x ⇒ ε x = with σx = (6.666)
E A
We know that the strain energy per unit volume is given by:
1 1 1 σx 1 P2
Ψ e = σ : ε    →Ψ e = σ x ε x = σ x
one - dimensional
= (6.667)
2 2 2 E 2 EA 2
Then, the total energy U is given by:
P2
Ψ ex × (volume) = L × A × ⇒
2 EA 2
(6.668)
P2L
⇒U =
2 EA
The strain energy density (per unit volume) can also be expressed as follows:

University of Castilla-La Mancha Draft By: Eduardo W. V. Chaves (2016)


Ciudad Real - Spain
876 SOLVING PROBLEMS BY MEANS OF CONTINUUM MECHANICS

1 1
Ψe = I σ2 − II dev
6(3λ + 2 µ ) 2µ σ (6.669)
144244 3 1 424 3
Ψ e
vol Ψ e
shape

Considering:
σ x 0 0
P
σij =  0 0 0 → I σ = σ x = (6.670)
A
 0 0 0

Calculation of II σ dev :
2
1 I2 σ
II σ dev = (3 II σ − I σ2 ) = − σ = − x (6.671)
3 3 3
Then, the strain energy density associated with the change of volume is:
1 (1 − 2ν ) 2 (1 − 2ν ) 2
Ψ evol = I σ2 = Iσ = σx (6.672)
6(3λ + 2 µ ) 6E 6E

(1 − 2ν ) P 2
Ψ evol = (per unit volume) (6.673)
6 E A2
The strain energy density associated with the change of shape is:
1 1 2(1 + ν )
Ψ e shape = − II σ dev = − II σ dev
2µ 2 E
(6.674)
(1 + ν )  σ x 
2
=− − 
E  3 

(1 + ν ) σ x (1 + ν ) P 2
2
Ψ e shape = = (6.675)
E 3 3E A2
Checking:
(1 − 2ν ) P 2 (1 + ν ) P 2 P2
Ψ e vol + Ψ e shape = + = [(1 − 2ν ) + 2(1 + ν )]
6E A2 3E A 2 6 EA 2
P2 P2
= [1 − 2ν + 2 + 2ν ] = =Ψ e
6 EA 2 2 EA 2
For the case of bending, we consider the following relationships:
M y a4
σy = where I=
I 12
12 M y
σy =
a4
σy
σ y = Eε y ⇒ ε y =
E
The strain energy density becomes:
1 1  12 M y σ y  1  12 M y 12 M y  72 M 2 y 2
Ψ e = σ y ε y =  4  =  4 = (6.676)
2 2 a E  2 a Ea 4  Ea 8

University of Castilla-La Mancha Draft By: Eduardo W. V. Chaves (2016)


Ciudad Real – Spain
6 LINEAR ELASTICITY 877

6.5.2 Beam with Varying Cross Section

Problem 6.79
Obtain the Finite Element Formulation for the beam with variable cross section, (see
Figure 6.221).

Quadratic Linear w (1)


a ( x) = a x 2 + b x + c
( a0 − a f ) a =0 θ y (2)
a=
L2 − ( a0 − a f )
b= Degrees-of-freedom
z − 2( a 0 − a f ) L
b=
L c = a0
c = a0

af
a ( x) x
a0

L
b0

Figure 6.221: Beam with variable cross-section.


Solution:
The internal potential energy is given by
L 2L
EI y  d 2 w  E
∫  

I y ( w′′) 2 dx
int
U =  2 
dx ≡ (6.677)
0
2  dx  20

where the moment of inertia I y is given by

b0 [a ( x)]3 b0 [ a x 2 + b x + c ]3
Iy = = (6.678)
12 12
Then, the internal potential energy becomes
L L
E Eb
U int =
20∫ 24 0 ∫
I y ( w′′) 2 dx = 0 [a x 2 + b x + c ]3 ( w′′) 2 dx (6.679)

In Problem 6.62 we have obtain an expression for (w′′) in terms of the degrees-of-
freedom (nodal values of deflection and rotation):
12 x 6   − 6x 4  12 x 6   − 6x 2 
w′′ = w1  3 − 2  + θ y1  2 +  + w2 − 3 + 2  + θ y 2  2 + 
L L   L L  L L   L L
Then, the internal potential energy becomes

University of Castilla-La Mancha Draft By: Eduardo W. V. Chaves (2016)


Ciudad Real - Spain
878 SOLVING PROBLEMS BY MEANS OF CONTINUUM MECHANICS

2
 12 x 6   − 6 x 2  
L
Eb  − 6x 4  12 x 6 

int
U = 0 [a x 2 + b x + c ]3 w1  3 − 2  + θ y1  2 +  + w2 − 3 + 2  + θ y 2  2 +   dx
24 0   L L   L L  L L   L L  

By using the computer software Mathematica we can solve the above integral and express it
in terms of w1 , θ y1 , w2 and θ y 2 , with that the total potential energy can be represented as
follows
Π ( w1 , θ y1, w2 , θ y 2 ) = U int ( w1 , θ y1, w2 , θ y 2 ) − U ext ( w1 , θ y1 , w2 , θ y 2 ) =

And the stationary point is given by


∂Π ∂U int ∂U ext ∂U ext
= − = k11w1 + k12 θ y1 + k13 w2 + k14 θ y 2 − =0
∂w1 ∂w1 ∂w1 ∂w1
∂Π ∂U int ∂U ext ∂U ext
= − = k 21w1 + k 22 θ y1 + k 23 w2 + k 24 θ y 2 − =0
∂θ y1 ∂θ y1 ∂θ y1 ∂θ y1
∂Π ∂U int ∂U ext ∂U ext
= − = k31w1 + k32 θ y1 + k33 w2 + k34 θ y 2 − =0
∂w2 ∂w2 ∂w2 ∂w2
∂Π ∂U int ∂U ext ∂U ext
= − = k 41w1 + k 42 θ y1 + k 43 w2 + k 44 θ y 2 − =0
∂θ y 2 ∂θ y 2 ∂θ y 2 ∂θ y 2
By restructuring the above equations in matrix form we can obtain
 ∂U ext 
 
 ∂w1 
 k11 k12 k13 k14   w1   ∂U  ext

k  
 21 k 22 k 23 k 24   θ y1   ∂θ y1  ∂U ext
 =  ⇔ [ k (e ) ]{u (e ) } = (e)
= { f Eq } (6.680)
 k31 k32 k33 k34   w2   ∂U ext  (e)
∂{u }
 
k 41 k 42 k 43 k 44  θ y 2   ∂w2 
 ∂U ext 
 
 ∂θ y 2 

where the components of the stiffness matrix are given by


First row
Eb0
k11 = (792a 2 c L4 + 792a b 2 L4 + 294b 3 L3 + 1764a b c L3 + 840c 3 + 1008b 2 c L2 + 720a 2b L5
840 L3
+ 1260b c 2 L + 220a 3 L6 + 1008a c 2 L2 )
− Eb0
k12 = 2
(504a b c L3 + 228a b 2 L4 + 420c 3 + 210a 2b L5 + 228a 2 c L4 + 65a 3 L6
840 L
+ 294a c 2 L2 + 420b c 2 L + 84b 3 L3 + 294b 2 c L2 )
− Eb0
k13 = 3
(792a 2 c L4 + 792a b 2 L4 + 294b 3 L3 + 1764a b c L3 + 840c 3 + 1008b 2 c L2 + 720a 2b L5
840 L
+ 1260b c 2 L + 220a 3 L6 + 1008a c 2 L2 ) = −k11
− Eb0
k14 = (1260a b c L3 + 564a b 2 L4 + 420c 3 + 510a 2 b L5 + 564a 2 c L4 + 155a 3 L6
840 L2
+ 714a c 2 L2 + 840b c 2 L + 210b 3 L3 + 714b 2 c L2 )

University of Castilla-La Mancha Draft By: Eduardo W. V. Chaves (2016)


Ciudad Real – Spain
6 LINEAR ELASTICITY 879

Second row
k 21 = k12
Eb0
k 22 = (65a 2b L5 + 168a b c L3 + 112b 2 c L2 + 72a b 2 L4 + 72a 2 c L4 + 20a 3 L6
840 L
+ 210b c 2 L + 112a c 2 L2 + 28b 3 L3 + 280c 3 )
k 23 = −k12
Eb0
k 24 = (156a 2 c L4 + 140c 3 + 45a 3 L6 + 182b 2 c L2 + 156a b 2 L4 + 336a b c L3
840 L
+ 210b c 2 L + 182a c 2 L2 + 56b 3 L3 + 145a 2b L5 )
Third row
k31 = k13 , k32 = k23 , k33 = k11 , k34 = −k14
Fourth row
k 41 = k14 , k 42 = k 24 , k 43 = k34 ,
Eb0
k 44 = (408a b 2 L4 + 365a 2b L5 + 110a 3 L6 + 532b 2 c L2 + 532a c 2 L2 + 630b c 2 L
840 L
+ 924a b c L3 + 408a 2 c L4 + 154b 3 L3 + 280c 3 )
And the parameters a , b and c are given by Figure 6.221.
NOTE: If we are considering the deflection v(x) instead of w(x) , (see Problem 6.67), the
total potential becomes
L
E

I z (v′′) 2 dx − U ext
int ext
Π (v1 , θz1 , v2 , θz 2 ) = U −U =
20

where the moment of inertia I y is given by

a ( x)b03 [a x 2 + b x + c ]b03
Iz = = (6.681)
12 12
Then,
L L
E E [a x 2 + b x + c ]b03
Π (v1 , θ z1 , v2 , θz 2 ) =∫2 0
I z (v′′) 2 dx − U ext =
20 ∫ 12
(v′′) 2 dx − U ext (6.682)

In Problem 6.67 we have obtained an expression for (v′′) in terms of the degrees-of-
freedom (nodal values of deflection and rotation):
12 x 6   12 x 6   6x 4   6x 2 
v′′ = v1  3 − 2  + v2 − 3 + 2  + θz1  2 −  + θz 2  2 − 
 L L   L L  L L   L L
And the total potential energy becomes
L 2
Eb3  12 x 6   12 x 6   6x 4   6x 2 
24 0 ∫
Π = 0 [a x 2 + b x + c ]v1  3 − 2  + v2  − 3 + 2  + θz1  2 −  + θz 2  2 −   dx − U ext
  L L   L L   L L  L L 
By using the computer software Mathematica we can solve the above integral and express
the total potential energy in terms of v1 , θ z1 , v2 and θ z 2 . And the stationary point is given
by

University of Castilla-La Mancha Draft By: Eduardo W. V. Chaves (2016)


Ciudad Real - Spain
880 SOLVING PROBLEMS BY MEANS OF CONTINUUM MECHANICS

∂Π ∂U ext
= k11v1 + k12 θ z1 + k13v2 + k14 θ z 2 − =0
∂v1 ∂v1
∂Π ∂U ext
= k 21v1 + k 22 θ z1 + k 23v2 + k 24 θ z 2 − =0
∂ θ z1 ∂θ z1
∂Π ∂U ext
= k31v1 + k32 θ z1 + k 33v2 + k 34 θ z 2 − =0
∂v2 ∂v2
∂Π ∂U ext
= k 41v1 + k 42 θ z1 + k 43v2 + k 44 θ z 2 − =0
∂θz 2 ∂θz 2
with that we can obtain
 ∂U ext 
 
 ∂vext
1 
 k11 k12 k13 k14   v1   ∂U 
k k 22 k 23 k 24   θ z1   ∂θ z1 
 21  =  ⇔ [k ( e ) ]{u ( e ) } = { f Eq
(e)
} (6.683)
 k31 k32 k33 k34   v2   ∂U ext 
 
k 41 k 42 k 43 k 44  θz 2   ∂v2 
 ∂U ext 
 
 ∂θz 2 
where the components of the stiffness matrix are given by
First row
Eb03 2 Eb03
k11 = ( 4 a L + 5b L + 10c ) , k12 = (7 a L2 + 10b L + 30c )
10 L3 60 L2
− Eb03 2 Eb03
k13 = 3
( 4 a L + 5b L + 10 c ) , k14 = 2
(17 a L2 + 20b L + 30c )
10 L 60 L
Second row
Eb03 Eb03
k 21 = k12 , k 22 = (8a L2 + 15b L + 60c ) , k 23 = − k12 , k 24 = (13a L2 + 15b L + 30c )
180 L 180 L
Third row
k31 = k13 , k32 = k23 , k33 = k11 , k34 = − k14
Fourth row
Eb03
k 41 = k14 , k 42 = k 24 , k 43 = k34 , k 44 = (38a L2 + 45b L + 60c )
180 L
And the parameters a , b and c are given by Figure 6.221.

University of Castilla-La Mancha Draft By: Eduardo W. V. Chaves (2016)


Ciudad Real – Spain
6 LINEAR ELASTICITY 881

Problem 6.80
By means of Problem 6.79 obtain the rotation at node 2 and the moment at node 1 for
the problem described in Figure 6.222).
NOTE 0: Although the data are in SI unit, computationally is better to work with the units
kN and cm , with that we avoid to deal with large numbers.

Data (load) Data (material) Data (geometry)


q z(1) = −30000 N / m E = 2.1 × 1011 Pa L = 5m
q z( 2 ) = −10000 N / m H = 0 .6 m
z h = 0.2m
b0 = 0.2m

q z(1)
q z( 2 )

x
a0 af

L
b0

Figure 6.222: Beam with variable cross-section (Linear variation).


Solution:
By using the finite element formulation given by (6.680), where we will adopt 1 finite
element, (Figure 6.223).

w (1)
U1 
z θ y (2) U 
 
Degrees-of-freedom {U } =  2 
U 3 
U1 U3 U 4 
U2 y U4

1 2 x
1

Figure 6.223: Global degrees-of-freedom.


The consistent load vector was obtained in Problem 6.66:

University of Castilla-La Mancha Draft By: Eduardo W. V. Chaves (2016)


Ciudad Real - Spain
882 SOLVING PROBLEMS BY MEANS OF CONTINUUM MECHANICS

 L (1) ( 2) 
 20 (7q z + 3q z ) 
 2   − 6 × 10 4 
 − L
(3q z + 2q z ) 4.583333 ×10 4 
(1) ( 2 )
   
(e)
{ f Eq } =  60 =  (6.684)
L 4
 (3q z + 7 q z )   − 4 × 10
(1) ( 2)

 20   − 3.75 × 10 4 
 L2  
(1) (2) 
 ( 2 q z + 3q z ) 
 60 
The stiffness matrix given by (6.680), and for linear variation of the height, (see Figure
(a f − a0 )
6.221), we have a f = 0.2 , a0 = 0.6 , a = 0 , b = = −0.08 and c = a0 = 0.6 , thus
L
 k11 k12 k13 k14   0.311808 − 1.06176 − 0.311808 − 0.49728
k k 24   − 1.06176
k 22 k23 3.92 1.06176 1.3888 
[k ( e) ] = 
21
= × 108
 k31 k32 k33 k34  − 0.311808 1.06176 0.311808 0.49728 
   
k 41 k 42 k43 k 44   − 0.49728 1.3888 0.49728 1.0976 

Then by applying the boundary conditions U1 = 0 , U 2 = 0 and U 3 = 0


[k ( e) ]{u (e ) } = { f Eq
(e)
}
1 0 0   w1  
0 0   w1   0 
0  θ    θ   
 1 0 0   y1  =  0   y1   0 
 Solve
→  = 
0 0 1 0   w2   0   w2   0 
 
0 0 0 1.0976 × 108  θ y 2  − 3.75 × 10 4  θ y 2  − 3.416545 × 10 −4 
 
Internal and Reaction Forces
The internal and reaction forces can be obtained as follows
(e) (e ) (e) (e) (e) (e) (e)
{rLocal } = −{ f Eq _ L } + { f Local } = −{ f Eq _ L } + [ k Local ][A ]{uGlobal } = −{ f int }

(e)
And the reaction vector in the Global system {rGlobal } = [A ]T {rLocal
(e)
} . Note also that for this
problem the Global and Local systems have the same orientation, so [A ] = [1] .
Element 1:
Internal force (Local system=Global system)
(e) (e) (e)
{ f Local } = [k Local ]{uGlobal }
 0.311808 − 1.06176 − 0.311808 − 0.49728  0 
 − 1.06176 3.92 1.06176 
1.3888    0 

= 108 ×   
− 0.311808 1.06176 0.311808 0.49728   0 
  −4 
 − 0 . 49728 1 . 3888 0. 49728 1 . 0976  − 3 . 416545 × 10 
 1.6989796 
 − 4.744898 
  4
=  ×10
 − 1 . 6989796 
 − 3.75 

University of Castilla-La Mancha Draft By: Eduardo W. V. Chaves (2016)


Ciudad Real – Spain
6 LINEAR ELASTICITY 883

 f z1   −6   1.6989796   7.6989796 
      − 9.32823129
(1) (1)  m y1  4.583333 4  − 4.744898  4   4
{rLocal } = −{ f int } =   = −  × 10 +   × 10 =   × 10
 fz2   −4  − 1.6989796  2.30102041 
m y 2   − 3.75   − 3.75   0 
  144 42444 3 144424443 
(1 ) (1 )
={ f Eq _ L} ={ f Local }

So, the moment at node 1 is M y(1) = {rLocal


(1)
}2 = −93282.3129 .

NOTE 1:
For 1 finite element we have obtained: Rotation at node 2 : − 0.3416545 × 10 −3 ; and moment
at node 1 : M y(1) = −93282.3129
If we discretize into 2 finite elements we can obtain
Rotation at node 3 : − 0.331712829486 × 10 -3
Deflection at node 2( x = 2.5) : − 0.218494069993 × 10 −3
Moment at node 1 : M y(1) = −93531.0701
If we discretize into 4 finite elements we can obtain
Rotation at node 5 : − 0.3094426299 × 10 -3
Deflection at node 3( x = 2.5) : − 0.2230288945 × 10 −3
Moment at node 1 : M y(1) = −94106.583974

University of Castilla-La Mancha Draft By: Eduardo W. V. Chaves (2016)


Ciudad Real - Spain
884 SOLVING PROBLEMS BY MEANS OF CONTINUUM MECHANICS

6.5.3 Introduction to Linear Buckling Problems

Problem 6.81
Obtain the total potential energy Π = U int − U ext in terms of the deflection v for the
problem described in Figure 6.224, (Laier&Barreiro (1983)), and for the internal potential
energy consider only the effect due to the flexural moment M z . Consider small
deformation and small rotation regime. Consider also that the column has cross section
constant.

P>0
x, u x, u
dv
−P −P
dx − dx cos θz du
θz dx
dx cos θ z dx

L θz
du = −(dx − dx cos θ z )

y, v y, v

z z θz

Figure 6.224: Column under compression.


Solution:
The internal potential energy for the deflection problem was established in Problem 6.67,
which is
L 2 L
EI  d 2v  EI
∫ ∫
 2  dx ≡ z (v′′) 2 dx
int
U = z  dx  (6.685)
2 0  2 0

Note that since we are dealing with small rotations the following is true
dv
≡ v′ = tan θ z ≈ sin θz ≈ θ z (small rotation) (6.686)
dx
The differential of the external potential energy can be represented as follows
dU ext = − Pdu = P (dx − dx cos θz ) = Pdx(1 − cos θz ) (6.687)
Let us try to express the above equation in terms of sine of the angle. By considering the
θz
following trigonometric relation cos(α ± β ) = cos α cos β m sin α sin β , when α = β =
2

University of Castilla-La Mancha Draft By: Eduardo W. V. Chaves (2016)


Ciudad Real – Spain
6 LINEAR ELASTICITY 885

 θz  θ  θ  θ 
we can obtain cos( θz ) = cos 2   − sin 2  z  , and recall also that cos 2  z  + sin 2  z  = 1
 2   2   2   2 
holds, then the equation in (6.687) can be rewritten as follows
 θ  θ   2  θz  θ  
dU ext = Pdx(1 − cos θz ) = Pdx cos 2  z  + sin 2  z  − cos   − sin 2  z  
  2   2    2   2  

θ 
⇒ dU ext = 2 P sin 2  z  dx (6.688)
 2 
By considering small rotations the following is true
2 2
2
θ   θz  θ (v′) 2
sin  z  ≈   = z =
 2   2  4 4
With that the equation (6.688) can be rewritten as follows
θ  P (v′) 2
dU ext = 2 P sin 2  z dx ⇒ dU ext = dx (6.689)
 2  2
And the external potential energy becomes:
L L
P (v ′) 2 P

U ext = dU ext = ∫
0
2
dx =
20 ∫
(v ′) 2 dx (6.690)

Then, the Total potential Energy can be expressed as follows


L L
1 P

EI z (v′′) 2 dx − (v′) 2 dx∫
int ext
Π =U −U = (6.691)
20 20

NOTE 1: If we are dealing with the deflection w = w(x) , the total potential energy is given
by:
L L
1 P
∫EI y ( w′′) 2 dx − ∫
( w′) 2 dx
int ext
Π =U −U = (6.692)
20 20

NOTE 2: The value for P in which Π = 0 is called critic load ( Pcr ), then according to
equation (6.691) we can conclude that
L

∫ EI (v′′)
2
L L z dx
1 P
Π=
20 ∫
EI z (v′′) 2 dx −
20 ∫
(v′) 2 dx = 0 ⇒ Pcr = 0
L

∫ (v′)
2
dx
0

For a exact value for the deflection v(x) we have the exact value for the critical load
Pcr(exact ) . And also note that the approximated value for Pcr is always greater than Pcr(exact ) .
Problem 6.82
Consider the problem established in Problem 6.81, (see Figure 6.224), and also consider
the following approximations for the deflection ( v = v(x) ) of the column:
a) v( x) = C2 ( x 2 − Lx) ; b) v( x) = C 2 ( x 2 − Lx) + C3 ( x 3 − L2 x) ; c) v( x) = C4 ( x 4 − 2 Lx 3 + L3 x)

University of Castilla-La Mancha Draft By: Eduardo W. V. Chaves (2016)


Ciudad Real - Spain
886 SOLVING PROBLEMS BY MEANS OF CONTINUUM MECHANICS

Obtain the critical load Pcr for each case.


NOTE: The exact value for the critical load for the problem describe in Figure 6.224,
(Sechler (1952)), is
π 2 EI z EI
Pcr(exact ) = 2
≈ 9.8696044 2z
L L
Solution:
Case a): Taking the derivatives of the deflection v( x) = C2 ( x 2 − Lx) we can obtain:
d d
v( x) = C 2 ( x 2 − Lx) →
dx v ′( x) = C 2 (2 x − L) →
dx v′′ = 2C2
The total potential energy becomes
L L
EI z P
Π = U int − U ext =
2 0 ∫
(v′′) 2 dx −
20
(v′) 2 dx∫
L L
EI z P P  P 
⇒Π=
2 0∫(2C 2 ) 2 dx −
20 ∫
[C 2 ( 2 x − L )]2 dx = 2 EI z LC 22 − L3C 22 =  2 EI z L − L3 C 22
6  6 

The inflection point is given by


∂Π ∂  P 3 2  P 3 C ≠0 12 EI z
=  2 EI z L − 6 L C2  = 2 2 EI z L − 6 L C2 = 0 
2
→ Pcr =
∂C2 ∂C2      L2

Note that Pcr > Pcr(exact ) and if we compare with the exact value the error is 21.59% .
Case b): Taking the derivatives of the deflection v( x) = C 2 ( x 2 − Lx) + C3 ( x 3 − L2 x) we can
obtain:
d d
2
v = C 2 ( x − Lx) + C3 ( x − L 3 2
x) → v′( x)
dx 2 2
= C 2 ( 2 x − L ) + C 3 (3 x − L ) → v′′
dx = 2C 2 + 6C3 x
The total potential energy becomes
L L
EI z P
Π = U int − U ext =
2 0 ∫
(v′′) 2 dx −
20
(v′) 2 dx ∫
L L
EI P
2 0 ∫
⇒ Π = z ( 2C 2 + 6C3 x) 2 dx −
20 ∫
[C 2 (2 x − L) + C3 (3x 2 − L2 )]2 dx

PL3
⇒ Π (C 2 , C3 ) = 2 EI z L(C 22 + 3LC 2 C3 + 3L2 C32 ) − (5C 22 + 15LC 2 C3 + 12 L2 C32 )
30
Then
∂Π ∂  2 2 2 PL3 
=  2 EI z L ( C 2 + 3 LC C
2 3 + 3 L C 3 ) − (5C 22 + 15LC 2 C3 + 12 L2 C32 ) = 0
∂C 2 ∂K 2  30 
PL3
= 2 EI z L(2C 2 + 3LC3 ) − (10C 2 + 15LC 3 ) = 0
30
∂Π ∂  2 2 2 PL3 
=  2 EI z L (C 2 + 3 LC C
2 3 + 3 L C 3 ) − (5C 22 + 15LC 2 C3 + 12 L2 C32 ) = 0
∂C3 ∂C3  30 
PL3
= 2 EI z L(3LC 2 + 6 L2 C3 ) − (15 LC 2 + 24 L2 C3 ) = 0
30

University of Castilla-La Mancha Draft By: Eduardo W. V. Chaves (2016)


Ciudad Real – Spain
6 LINEAR ELASTICITY 887

in matrix form becomes


 L3 L4 
 4 EI z L 6 EI z L  C 2   
2  C 2  = 0
2
3
 2 3   − P 4 5     (6.693)
6 EI z L 12 EI z L  C3   L 4 L  C3  0
 2 5 
Note that
−1
 L3 L4   48 − 30 
   3
 34 2  = L L4 
L 4 L5  − 30 20 
 4 
 2 5   L L5 
And the equation in (6.693) can also be expressed as follows:
−1 −1
 L3 L4   L3 L4   L3 L4 
     
2   4 EI z L 6 EI z L  C 2  − P  3 2  C 2  = 0
2
 34 5  2 3   4
2 
5  34 5    
L 4 L  6 EI z L 12 EI z L  C3  L 4L  L 4 L  C3  0
 2 5   2 5   2 5 
12 EI z − 72 EI z 
 2 L  C 2  − P 1 0 C 2  = 0
⇒ L
60 EI z  C3  0 1 C  0
  3   
 0 
 L2 
 12 EI z − 72 EI z  
 2   C
   2  =   with C 2  0
L 1 0 0
⇒  L  − P   ≠ 
 60 EI z 0 1 C3  0 C3  0
 0  
 L 2 
 
⇒ ([ A] − P[1]){C } = {0}
Note that the above system has non-trivial solution, i.e. {C} ≠ {0} , if and only if the
determinant of ([ A] − P[1]) is zero, in other words, we are dealing with the eigenvalue-
eigenvector problem, where the eigenvalues are Pi and the eigenvectors are {C} , and the
critical value is the smallest value of Pi . After solving the above problem we obtain the
following eigenvalues
 12 EI z 
 12 EI z  60 EI   P1   L2 
det ([ A] − P[1]) = 0 ⇒  2 − P  2 z − P  = 0 ⇒   =  60 EI 
 L  L   P2   z
2
 L 
12 EI z
⇒ Pcr = P1 =
L2
which matches the solution for the case (a) in which we have considered a quadratic
L
function for the deflection. Note that at x = the derivative of the deflection is zero, so
2
L  L   L2 
v′( x = ) = C 2 (2 x − L) + C3 (3 x 2 − L2 ) = C 2  2 − L  + C3  3 − L2  = 0 ⇒ C3 = 0
2  2   4 
In other words, the total potential does not depend on C3 .
Case c): Taking the derivatives of the deflection v( x) = C4 ( x 4 − 2 Lx 3 + L3 x) we can obtain:

University of Castilla-La Mancha Draft By: Eduardo W. V. Chaves (2016)


Ciudad Real - Spain
888 SOLVING PROBLEMS BY MEANS OF CONTINUUM MECHANICS

d d
v( x) = C 4 ( x 4 − 2 Lx 3 + L3 x) →
dx v′( x) = C 4 (4 x 3 − 6 Lx 2 + L3 ) →
dx v′′ = C4 (12 x 2 − 12 Lx)
The total potential energy becomes
L L
EI P
∫ (v′′) dx − ∫
(v′) 2 dx
int ext 2
Π =U −U = z
2 0
2 0

L L
EI z P
⇒Π=
2 0 ∫
[C 4 (12 x 2 − 12 Lx )]2 dx −
20 ∫
[C 4 (4 x 3 − 6 Lx 2 + L3 )]2 dx

12 EI z L5 2 17 PL7 2
⇒ Π (C 4 ) = C4 − C4
5 70
The inflection point is given by
∂Π ∂ 12 EI z L5 2 17 PL7 2   24 EI z L5 17 PL7 
⇒ =  C4 − C 4  =  − C 4 = 0

∂C 4 ∂C 4  5 70   5 35 
C ≠0 168EI z EI

4
→ Pcr = 2
≈ 9.88235 2z > Pcr( exact )
17 L L
which is a good solution since the error is 0.1292% .

6.5.3.1 Introduction to Finite Element for Column Problems

Problem 6.83
a) Obtain Finite Element Formulation for the problem established in Problem 6.81.
Consider the degree-of-freedom (beam element) as the one described in Figure 6.225. b)
Use this required formulation to solve the problem described in Figure 6.225(a).

π 2 EI z x, u x, u
Pcr(exact ) = P>0
L2
−P
v2 , f y 2
2

θ z 2 , mz 2
 v1 
θ 
L  
1 {u (e ) } =  z1 
 v2 
θ z 2 

y, v y v1 , f y1
1

z z θz1, mz1

a) Column under compression b) Degree-of-freedom and nodal forces

Figure 6.225: Column under compression.

University of Castilla-La Mancha Draft By: Eduardo W. V. Chaves (2016)


Ciudad Real – Spain
6 LINEAR ELASTICITY 889

Solution:
a) Let us express the total potential energy in terms of the degrees-of-freedom {u (e ) } . In
L L

∫ v′2 dx and ∫ v′′ dx


2
Problem 6.67 we have obtained the integrals in terms of {u (e ) } . Then,
0 0
by considering the total potential energy:
L L
EI z P
Π = U int − U ext =
2 0 ∫
(v′′) 2 dx −
20 ∫
(v′)2 dx (6.694)

we can obtain
EI z  12 2 12 2 4 2 4 2 24 12 12 12 12
Π=  v + 3 v2 + θz1 + θ z 2 − 3 v1v2 + 2 v1 θ z1 + 2 v1 θz 2 − 2 v2 θz1 − 2 v2 θz 2
3 1
2 L L L L L L L L L
4  P 6 6 2 2 L 2 2 L 2 12 1 1
+ θz1 θz 2  −  v12 + v2 + θ z1 + θz 2 − v1v2 + v1 θz1 + v1 θz 2
L  2  5L 5L 15 15 5L 5 5
1 1 L 
− v2 θz1 − v2 θz 2 − θ z1 θ z 2 
5 5 15 
As we are looking for the stationary state the following must hold:
∂Π EI z  24 12 24 12  P  12 1 12 1 
=0 ⇒  3 v1 + 2 θz1 − 3 v2 + 2 θz 2  −  v1 + θz1 − v2 + θz 2  = 0
∂v1 2 L L L L  2  5L 5 5L 5 
∂Π EI z 12 8 12 4  P 1 4L 1 L 
=0 ⇒  2 v1 + θz1 − 2 v2 + θz 2  −  v1 + θz1 − v2 − θz 2  = 0
∂θz1 2 L L L L  2 5 15 5 15 
∂Π EI z  − 24 12 24 12  P  − 12 1 12 1 
=0 ⇒  3 v1 − 2 θz1 + 3 v2 − 2 θz 2  −  v1 − θz1 + v2 − θz 2  = 0
∂v2 2  L L L L  2  5L 5 5L 5 
∂Π EI z 12 4 12 8  P 1 L 1 4L 
=0 ⇒  2 v1 + θz1 − 2 v2 + θz 2  −  v1 − θz1 − v2 + θz 2  = 0
∂ θz 2 2 L L L L  2 5 15 5 15 
Restructuring the above set of equations in matrix form we can obtain:
  12 6 − 12 6   6 1 −6 1 
  L3 
 L2 L3 L2   5L 10 5L 10   v

 6 4 −6 2   1 2L −1 − L    1  0
 2       
  L L L2 L  − P  10 15 10 30    θ z1  = 0
 EI z  − 12 −6 12 − 6 − 6 −1 6 − 1    v2  0
     (6.695)
 L3 L2 L3 L 2   5L 10 5L 10   θ  0


 6 2 −6 4   1 −L −1 2 L   z 2   
  2   
  L L L2 L   10 30 10 15  
⇒ ([k ( e ) ] − P[kp (ye ) ]){u ( e ) } = {0}
where [kp (ey ) ] is called Geometric Matrix.
b) As we are using 1 finite element we can apply directly the above equation and apply the
boundary conditions: v1 = 0 , v2 = 0 , then the above equation becomes
 1 0 0 0  1 0 0 0 
 4 EI z 2 EI z   2L − L    v1  0
 0 0  0 0    
 L L − P 15 30    θ z1  = 0 ⇔ ([k (e ) ] − P[ kp (e ) ]){u (e ) } = {0}
 0 0 1 0  0 0 1 0    v2  0 y

2 EI z 4 EI z   −L 2L     
 0 0  0 0  θ  0
 15    z 2   
 L L   30

University of Castilla-La Mancha Draft By: Eduardo W. V. Chaves (2016)


Ciudad Real - Spain
890 SOLVING PROBLEMS BY MEANS OF CONTINUUM MECHANICS

By applying the solution for eigenvalue-eigenvector problem we can obtain


(e) (e) 1 (720( EI z ) 2 − 72 EI z L2 P + L4 P 2 )
det ([k ] − P[kp y ]) = 0 ⇒ (1 − P ) 2 =0
60 L2
We have 4 values for P whose values are
 1 
 P1   
P   1 
 2   12 EI z 
 = 2 
 P3   L 
 P4   60 EI z 
 L2 

The solutions P1 = P2 = 1 are related to the boundary conditions and can be discarded. The
12 EI z
critical value is the smallest one among the other two solutions, i.e. Pcr = . The
L2
solution can be improved by discretizing the column into more finite elements.

NOTE: If we are dealing with the deflection w = w(x) , (see Problem 6.62-NOTE 1), the
following is true
L
P P 6 6 2 2 L 2 2 L 2 12 1 1
∫w′ 2 dx =  w12 +
ext
U = w2 + θ y1 + θy2 − w1w2 − w1θ y1 − w1 θ y 2
20 2  5L 5L 15 15 5L 5 5
1 1 L 
+ w2 θ y1 + w2 θ y 2 − θ y1 θ y 2 
5 5 15 
then
∂Π EI y  24 12 24 12  P  12 1 12 1 
=0⇒  w1 − 2 θ y1 − 3 w2 − 2 θ y 2  −  w1 − θ y1 − w2 − θ y 2  = 0
∂w1 2  L3 L L L  2  5 L 5 5 L 5 
∂Π EI y  − 12 8 12 4  P −1 4L 1 L 
=0⇒  2 w1 + θ y1 + 2 w2 + θ y 2  −  w1 + θ y1 + w2 − θ y 2  = 0
∂ θ y1 2  L L L L  25 15 5 15 
∂Π EI y  − 24 12 24 12  P  − 12 1 12 1 
=0⇒  3 w1 + 2 θ y1 + 3 w2 + 2 θ y 2  −  w1 + θ y1 + w2 + θ y 2  = 0
∂w2 2  L L L L  2  5 L 5 5 L 5 
∂Π EI y  − 12 4 12 8  P −1 L 1 4L 
=0⇒  w1 + θ y1 + 2 w2 + θ y 2  −  w1 − θ y1 + w2 + θy2  = 0
∂θ y 2 2  L2 L L L  2  5 15 5 15 
Restructuring the above set of equations in matrix form we can obtain:
  12 −6 − 12 − 6  6 −1 −6 −1 
  L3 2   5L 
 L2 L3 L 10 5L 10   w 

 −6 4 6 2   −1 2L 1 − L   1  0
 2     
  L L L2 L  − P  10 15 10 30   θ y1  = 0
 EI y  − 12 6 12 6  − 6 1 6 1   w2  0
  L3 2   5L    
 L2 L3 L 10 5L 10  θ y 2  0 (6.696)

 −6 2 6 4   −1 −L 1 2 L     
  2   
  L L L2 L   10 30 10 15  

⇒ ([k ( e) ] − P[kp z( e) ]){u (e ) } = {0}

University of Castilla-La Mancha Draft By: Eduardo W. V. Chaves (2016)


Ciudad Real – Spain
6 LINEAR ELASTICITY 891

Problem 6.84
Obtain the critical load for the cases described in Figure 6.226(a), (b) and (c), by using the
finite element formulation, (see Problem 6.83).

π 2 EI z π 2 EI z π 2 EI z
Pcr(exact ) = Pcr( exact ) = Pcr(exact ) =
( 2 L) 2 ( 0 .7 L ) 2 ( 0 .5 L ) 2

−P P>0 −P P>0 −P P>0

L L L

y, v x, u y, v x, u y, v x, u

z z z
a) b) c)

Figure 6.226: Column under compression.


Solution:
This problem is the same one described in Problem 6.83, i.e.:
  12 6 − 12 6   6 1 −6 1 
  L3 2   5L 
 L2 L3 L 10 5L 10   v

 6 4 −6 2   1 2L −1 − L    1  0
 2       

EI  L L L2 L  − P  10 15 10 30    θz1  = 0
 z  − 12 −6 12 − 6 − 6 −1 6 − 1    v2  0
  L3 2   5L    
 L2 L3 L 10 5L 10   θ  0

 6 2 −6 4   1 −L −1 2 L   z 2   
  2   
  L L L2 L   10 30 10 15  

Case a): For case a) the boundary conditions are v1 = 0 , θz1 = 0 , then the above equation
becomes:
 1 0 0 0  1 0 0 0 
  0   v1  0
 0 1 0 0 
 1 0 0      
 12 EI z − 6 EI z 
0 6 − 1    θ z1  0
 0 0 − P 0  = 
L3 L2   5L 10    v2  0
 − 6 EI z 4 EI z   −1 2 L   θ  0
 0 0 0 0  z 2   
 L2 L 
 10 15  
(e) (e)
([k ] − P[kp y ]){u ( e ) } = {0}

the above equation has non-trivial solution if and only if

University of Castilla-La Mancha Draft By: Eduardo W. V. Chaves (2016)


Ciudad Real - Spain
892 SOLVING PROBLEMS BY MEANS OF CONTINUUM MECHANICS

(e) (e) 1 (240( EI z ) 2 − 104 EI z L2 P + 3L4 P 2 )


det ([k ] − P[kp y ]) = 0 ⇒ ( P − 1) 2 =0
20 L4
We have 4 values for P whose values are
 1 
 P1   
P   1
 2  EI z 
   = 32 . 180705 
 P3   L2 
 P4  2.4859617 EI z 
 L2 
The solutions P1 = P2 = 1 are related to the boundary conditions and can be discarded. The
critical value is the smallest one among the others, i.e.
EI z EI
Pcr = 2.4859617 2
> Pcr(exact ) = 2.4674011 2z .
L L
which error is about 0.752% .
Case b)
For this case the boundary conditions are v1 = 0 , θz1 = 0 and v2 = 0 , then the system to
solved is
 1 0 0 0  1 0 0 0    v  0
  0  1
 0 1 0 0   1 0 0    θ  0
 0 0  − P 0  z1
 0 1 0 1 0    v  = 0
 0 4 EI z   2 L   2   
0 0  0 0 0   θ  0
 L   15    z 2   
(e) (e)
([k ] − P[kp y ]){u ( e ) } = {0}
Then

 P1   1 
P   1 
(e) (e) 2 ( L2 P − 30 EI z )  2  
det ([k ] − P[kp y ]) = 0 ⇒ ( P − 1) 3 =0 ⇒  = 1 
15 L  P3   EI z 
 P4  30 2 
 L 
EI z EI
⇒ Pcr = 30 2
> Pcr(exact ) = 20.1420498 2z .
L L
which error is about 48.94% . The error can be minimized by considering more finite
EI z
elements. For instance, if we are adopting two elements we obtain Pcr = 20.7088 , and
L2
the error associated with it is about 2.81% .
Case c)
For the case c) we cannot use 1 finite element due to the boundary conditions, so we will
adopt two finite elements for the discretization, (see Figure 6.227).

University of Castilla-La Mancha Draft By: Eduardo W. V. Chaves (2016)


Ciudad Real – Spain
6 LINEAR ELASTICITY 893

x, u x, u
v
P>0
−P θz
U5 Degrees-of-freedom
3 per node

L U6
2
L( 2 ) =
2 U 1 
U3 U 
2
 2
U 3 
L {U } =  
L(1) = U4
1 U 4 
2
U 5 
y  
1 U 6 
y, v U1
z z U2

a) Column under compression b) Global degree-of-freedom

Figure 6.227: Column under compression.


Solution:
Construction of the Global Matrices - [ K Global ] and [ KpGlobal ]
We need to construct the following system
[ [ K Global ] − P[ KpGlobal ] ]{U } = {0}
The matrices [ K Global ] and [ KpGlobal ] can be constructed by assembling the individual beam
elements, i.e.:
2 2

A A[kp
(e) (e )
[ K Global ]6×6 = [k Global ] [ KpGlobal ]6×6 = Global ] (6.697)
e =1 e =1

where
 12 6 − 12 6   6 1 −6 1 
 L3 L2 L3 L2   5L 10 5L 10 
 6 4 −6 2   1 2L −1 − L
 2   
(e)
[kGlobal ] = EI z  L L L2 L  ; (e)
[kpGlobal ] =  10 15 10 30  (6.698)
 − 12 −6 12 − 6 − 6 −1 6 −1 
 L3 L2 L3 L2   5L 10 5L 10 
 6 2 −6 4   1 −L −1 2L 
 2   
 L L L2 L   10 30 10 15 
For this problem the stiffness matrices for both elements are the same.
L
Element 1 and 2: ( L) (1) = ( L) ( 2) = , ( EI z ) (1) = ( EI z ) ( 2) = EI z and by substituting these
2
values into the equations in (6.698) we can obtain:

University of Castilla-La Mancha Draft By: Eduardo W. V. Chaves (2016)


Ciudad Real - Spain
894 SOLVING PROBLEMS BY MEANS OF CONTINUUM MECHANICS

1 2 3 4 <= Global => 1 2 3 4


 96 24 − 96 24   12 1 − 12 1 
 L3 L2 L3 L2   5L 10 5L 10 
 24 8 − 24 4   1 L −1 − L
 2   
(1)
[kGlobal ] = EI z  L L L2 L  ; (1)
[kpGlobal ] =  10 15 10 60 
 − 96 − 24 96 − 24   − 12 −1 12 −1 
 L3 L2 L3 L2   5L 10 5L 10 
 24 4 − 24 8   1 −L −1 L 
 2   
 L L L2 L   10 60 10 15 

k11(1) k12(1) k13(1) k14(1) 0 0 


 (1) (1) (1) (1) 
k 21 k 22 k 23 k 24 0 0 
2 k (1) (1)
k 32 (1)
k 33 + k11( 2 ) (1)
k 34 + k12( 2 ) k13( 2 ) k14( 2) 
A
(e )
[ K Global ]6×6 = [k Global ] =  31
(1) (1) (1) ( 2) (1) (2) ( 2) ( 2)

e =1 k 41 k 42 k 43 + k 21 k 44 + k 22 k 23 k 24 
 0 0 ( 2)
k 31 ( 2)
k 32 ( 2)
k 33 ( 2) 
k 34
 
( 2) ( 2) ( 2) ( 2)
 0 0 k 41 k 42 k 43 k 44 

After the values are substituted we can obtain


 96 24 − 96 24 
 L3 0 0 
L2 L3 L2
 24 8 − 24 4 
 2 0 0 
 L L L2 L 
 − 96 − 24 192 − 96 24 
 3 0
[ K Global ]6×6 = EI z  L L2 L3 L3 L2 
24 4 16 − 24 4 
 2 0 
 L L L L2 L 
 0 − 96 − 24 96 − 24 
 0
L3 L2 L3 L2 
 24 4 − 24 8 
 0 0 
 L2 L L2 L 
And
1 2 3 4 5 6 Global
kp11
(1) (1)
kp12 (1)
kp13 (1)
kp14 0 0 
 (1) (1) (1) (1) 
kp21 kp22 kp23 kp24 0 0 
2 kp (1) (1)
kp32 (1)
kp33 ( 2)
+ kp11 (1)
kp34 + kp12( 2) (2)
kp13 ( 2) 
kp14
A
(e)
[ KpGlobal ]6×6 = [kpGlobal ] =  31(1) (1) (1) ( 2) (1) ( 2) (2) ( 2)

e =1 kp41 kp42 kp43 + kp21 kp44 + kp22 kp23 kp24 
 0 0 (2)
kp31 ( 2)
kp32 (2)
kp33 ( 2) 
kp34
 
(2) ( 2) (2) ( 2)
 0 0 kp41 kp42 kp43 kp44 

After the values are substituted we can obtain

University of Castilla-La Mancha Draft By: Eduardo W. V. Chaves (2016)


Ciudad Real – Spain
6 LINEAR ELASTICITY 895

 12 1 − 12 1 
 5L 0 0 
10 5L 10
 1 L −1 −L 
 0 0 
 10 15 10 60 
 − 12 −1 24
0
− 12 1 
 10 
[ KpGlobal ]6× 6 =  5L 10 5L 5L
1 −L 2L −1 − L
 0 
 10 60 15 10 60 
 0 − 12 −1 12 −1 
0
 5L 10 5L 10 
 1 −L −1 L 
 0 0 
 10 60 10 15 

Applying the Boundary Conditions


Note that there are restrictions to move for the following degrees-of-freedom: U 1 = 0 ,
U 2 = 0 , U 5 = 0 and U 6 = 0 , thus

[ [ K Global ] − P[ KpGlobal ] ]{U } = {0} Boundary


Conditions

→[ [ K Global ] − P[ KpGlobal ] ]{U } = {0}

 1 0 0 0 0 0 1 0 0 0 0 0 
  0  U1  0
 0 1 0 0 0 0  1 0 0 0 0     
 192 EI z 24  U 2  0
 0 0 0 0 0 0 0 0 0 0  U   
L3  − P 5L   3  = 0
 16 EI z   2L     
 0 0 0 0 0 0 0 0 0 0  U 4  0
 L 15  U  0
 0 0 0 0 1 0   5   
 0 0 0 0 1 0
   U  0
 0 0 1 0 1   6   
 0 0 0 0 0 0 0

[ [ K Global ] − P[ Kp Global ] ]{U } = {0}

The solution for the above system, i.e. the eigenvalues of det[ [ K Global ] − P[ Kp Global ] ] = 0
are:
16 ( PL2 − 120 EI z )
( P − 1) 4 ( PL2 − 40 EI z ) =0
25 L4
EI z EI z
P1 = P2 = P3 = P4 = 1; P5 = 40 ; P5 = 120
L2 L2
And the critical value is
EI z EI
Pcr = 40 2
> Pcr( exact ) ≈ 39.478 2z
L L

University of Castilla-La Mancha Draft By: Eduardo W. V. Chaves (2016)


Ciudad Real - Spain
896 SOLVING PROBLEMS BY MEANS OF CONTINUUM MECHANICS

Problem 6.85
a) Consider the column described in Figure 6.228 in order to obtain the critical load by
considering two finite elements, (see Figure 6.228(b)).

x, u x, u
v
P>0
−P θz
U5 Degrees-of-freedom
3 per node

( EI z )( 2 ) = EI z 2L U6
L( 2 ) = 2 U 1 
3
U 
U3  2
2 U 3 
{U } =  
( EI z )(1) = 16 EI z L U4 U 4 
L(1) = 1 U 5 
3 y  
U1 1 U 6 
y, v
z z U2

a) Column under compression b) Global degree-of-freedom

Figure 6.228: Column under compression.


Solution:
Construction of the Global Matrices - [ K Global ] and [ KpGlobal ]
We need to construct the following system
[ [ K Global ] − P[ KpGlobal ] ]{U } = {0}
The matrices [ K Global ] and [ KpGlobal ] can be constructed by assembling the individual beam
elements, i.e.:
2 2

A A[kp
(e) (e )
[ K Global ]6×6 = [k Global ] [ KpGlobal ]6×6 = Global ] (6.699)
e =1 e =1

where
 12 6 − 12 6   6 1 −6 1 
 L3 L2 L3 L2   5L 10 5L 10 
 6 4 −6 2   1 2L −1 − L
 2   
(e)
[kGlobal ] = EI z  L L L2 L  ; (e)
[kpGlobal ] =  10 15 10 30  (6.700)
 − 12 −6 12 − 6 − 6 −1 6 −1 
 L3 L2 L3 L2   5L 10 5L 10 
 6 2 −6 4   1 −L −1 2L 
 2   
 L L L2 L   10 30 10 15 
L
Element 1: ( L) (1) = , ( EI z ) (1) = 16 EI z and by substituting these values into the equations
3
in (6.700) we can obtain:

University of Castilla-La Mancha Draft By: Eduardo W. V. Chaves (2016)


Ciudad Real – Spain
6 LINEAR ELASTICITY 897

1 2 3 4 <= Global => 1 2 3 4


 5184 864 − 5184 864   18 1 − 18 1 
 L3 L2 L3 L2   5L 10 5L 10 
 864 192 − 864 96   1 2L −1 − L
 2
  
(1)
[kGlobal ] = EI z  L L L2 L  ; (1)
[kpGlobal ] =  10 45 10 90 
 − 5184 − 864 5184 − 864   − 18 −1 18 −1 
 L3 L2 L3 L2   5L 10 5L 10 
 864 96 − 864 192   1 −L −1 2L 
   
 L2 L L2 L   10 90 10 45 
2L
Element 2: ( L) ( 2) = , ( EI z ) ( 2) = EI z and by substituting these values into the equations
3
in (6.700) we can obtain:

3 4 5 6 <= Global => 3 4 5 6


 81 27 − 81 27   9 1 −9 1 
 2 L3 2 L2 2 L3 2 L2   5L 10 5L 10 
 27 6 − 27 3   1 4L −1 − L
 2   
( 2)
[kGlobal ] = EI z  2 L L 2 L2 L  ; (2)
[kpGlobal ] =  10 45 10 45 
 − 81 − 27 81 − 27  − 9 −1 9 −1 
 2 L3 2 L2 2 L3 2 L2   5L 10 5L 10 
 27 3 − 27 6   1 −L −1 4L 
 2   
 2L L 2 L2 L   10 45 10 45 
Then, the equations in (6.699) become

1 2 3 4 5 6 Global
k11(1) k12(1) k13(1) k14(1) 0 0 
 (1) (1) (1) (1) 
k 21 k 22 k 23 k 24 0 0 
2 k (1) (1)
k 32 (1)
k 33 + k11( 2 ) (1)
k 34 + k12( 2 ) k13( 2 ) k14( 2) 
A
(e )
[ K Global ]6×6 = [k Global ]=  31
(1) (1) (1) ( 2) (1) (2) ( 2) ( 2)

e =1 k 41 k 42 k 43 + k 21 k 44 + k 22 k 23 k 24 
 0 0 ( 2)
k 31 ( 2)
k 32 ( 2)
k 33 ( 2) 
k 34
 
( 2) ( 2) ( 2) ( 2)
 0 0 k 41 k 42 k 43 k 44 
After the values are substituted we can obtain
 5184 864 − 5184 864 
 L3 0 0 
L2 L3 L2
 864 192 − 864 96 
 0 0 
 L2 L L2 L 
 − 5184 − 864 10449 − 1701 − 81 27 
 3
L2 2 L3 2 L2 2 L3 2 L2 
[ K Global ]6×6 = EI z  L
864 96 − 1701 198 − 27 3 
 
 L2 L 2 L2 L 2 L2 L 
 − 81 − 27 81 − 27 
 0 0
2 L3 2 L2 2 L3 2 L2 
 27 3 − 27 6 
 0 0 
 2 L2 L 2 L2 L 

University of Castilla-La Mancha Draft By: Eduardo W. V. Chaves (2016)


Ciudad Real - Spain
898 SOLVING PROBLEMS BY MEANS OF CONTINUUM MECHANICS

And

1 2 3 4 5 6 Global
kp11
(1) (1)
kp12 (1)
kp13 (1)
kp14 0 0 
 (1) (1) (1) (1) 
kp21 kp22 kp23 kp24 0 0 
2 kp (1) (1)
kp32 (1)
kp33 ( 2)
+ kp11 (1)
kp34 + kp12( 2) (2)
kp13 ( 2) 
kp14
A
(e)
[ KpGlobal ]6×6 = [kpGlobal ]=  31(1) (1) (1) ( 2) (1) ( 2) (2) ( 2)

e =1 kp41 kp42 kp43 + kp21 kp44 + kp22 kp23 kp24 
 0 0 (2)
kp31 ( 2)
kp32 (2)
kp33 ( 2) 
kp34
 
(2) ( 2) (2) ( 2)
 0 0 kp41 kp42 kp43 kp44 

After the values are substituted we can obtain


 18 1 − 18 1 
 5L 0 0 
10 5L 10
 1 2L −1 −L 
 0 0 
 10 45 10 90 
 − 18 −1 27
0
−9 1 
 10 
[ KpGlobal ]6× 6 =  5L 10 5L 5L
1 −L 2L −1 − L
 0 
 10 90 15 10 45 
 0 −9 −1 9 −1 
0
 5L 10 5L 10 
 1 −L −1 4L 
 0 0 
 10 45 10 45 

Applying the Boundary Conditions


Note that there are restrictions to move for the following degrees-of-freedom: U 1 = 0 and
U 5 = 0 , thus

[ [ K Global ] − P[ KpGlobal ] ]{U } = {0} Boundary


Conditions

→[ [ K Global ] − P[ KpGlobal ] ]{U } = {0}

  1  
  EI 0 0 0 0 0  
  z  1 0 0 0 0 0 
  0 192 − 864 96   2L −1 −1 
 0 0 0 0 0  U1  0
 
 L L2 L 45 10 90 U   

 − 864 10449 − 1701 27   −1 27 1    2  0 
 0 0 2
0 0 0 
 EI  L2 2 L3 2 L2 2L  − P  10 5L 10  U 3  = 0
 z 96 − 1701 198 3  0 −1 2L − L  U 4  0
 0 0 0 0    
 L 2 L2 L L   90 15 45  U  0
  1  0 0 0 0 1 0   5   
  0 0 0 0 0   1 −L 4 L  U 6  0
  EI z  0 0 0 
  0 27 3 6   10 45 45  
 0 0 
  2 L2 L L  

[ [ K Global ] − P[ KpGlobal ] ]{U } = {0}


Let us assume that L = 1 and EI z = 1 and the solution for the above system, i.e. the
eigenvalues of det[ [ K Global ] − P[ Kp Global ] ] = 0 are:

University of Castilla-La Mancha Draft By: Eduardo W. V. Chaves (2016)


Ciudad Real – Spain
6 LINEAR ELASTICITY 899

5.7331997382 × 10 3 
 3
1.0931697830 ×10 
 12.13650390 
Pi =  
 79.4939748 
 1 
 
 1 
Note that the solutions P = 1 are associated with the boundary conditions and can be
discarded. Then, the smallest value is Pcr = 12.13650390 . If we compare with the exact
solution for this problem which is
EI z
Pcr(exact ) = 11.1
L2
the error is about 9.34% . The eigenvector associated with the eigenvalue Pcr = 12.1365 is:
 0 
− 0.490914
 
− 0.161349
Ui =  
 − 0.470371
 0 
 
 0.715347 

Problem 6.86
Consider the structure described in Figure 6.229. Obtain the displacements (translation and
rotation) at the node 2.
Approach: Do not consider the energy due to the axial force ( EA → ∞ ).

v (1)

Degrees-of-freedom
y
2 .0 m
θz (2)
qy

−P
EI z = 4500kN m 2
1 2 x
1 K yf = 200kN / m 2
z K yf q y = −10kN / m
P = 1125kN

Figure 6.229: Beam and elastic foundation.

University of Castilla-La Mancha Draft By: Eduardo W. V. Chaves (2016)


Ciudad Real - Spain
900 SOLVING PROBLEMS BY MEANS OF CONTINUUM MECHANICS

Solution:
In Problem 6.83, by considering the plane x − y , we have shown that the following is
true
 12 6 − 12 6   6 1 −6 1 
 L3 L2 L3 L2   5L 10 5L 10 
 6 4 −6 2   1 2L −1 − L
 2   
[k ] = EI z  L
(e) L L2 L  ; P[kp y ] = P  10
(e) 15 10 30 
 − 12 −6 12 − 6 − 6 −1 6 −1 
 L3 L2 L3 L2   5L 10 5L 10 
 6 2 −6 4   1 −L −1 2L 
 2   
 L L L2 L   10 30 10 15 
And in Problem 6.74 we have shown that the stiffness due to the elastic base is given by
 13 11L 9 − 13L 
 35 210 70 420 
 
 11L L2 13L − L2 
[ke ( Spring _ y ) ] = K yf L  210 105 420 140 
 9 13L 13 − 11L 
 70 420 35 210 

 − 13L − L2 − 11L L2 
 420 140 210 105 
Then, the complete system is represented as follows:
[ [k (e ) ] + [ke ( Spring _ y ) ] − P[kp (ye ) ] ]{u ( e ) } = { f ( e ) }

The consistent load vector, (see Problem 6.67), is given by


 qy L 
 
 22   − 10 
 qy L   
(e)  12  − 3.33333333
{f }=  =
q L   − 10

 y   
 2   3.33333333 
 − q y L2 
 
 12 
After the assemble is complete and the numerical variables are considered we can obtain
 6.223571 6.679405 − 6.023571 6.612738   v1   − 10 
 6.679405 8.715238 − 6.612738 4.563571   θ z1  − 3.33333333
   
3
10 ×   = 
− 6.023571 − 6.612738 6.223571 − 6.679405  v2   − 10 
    
 6 . 612738 4 . 563571 − 6 . 679405 8. 715238 θ
 z2   3 .33333333

Applying the boundary conditions, i.e. v1 = 0 , θ z1 = 0 :


1 0 0 0   v1   0   v1   0 
0      θ   0 
10 3 × 
1 0 0   θ z1  =  0     
→ z1  = 
Solve 
 × 10
−3
0 0 6.223571 
− 6.679405  v2   − 10   v2   − 6.741
      θ z 2  − 4.784
0 0 − 6.679405 8.715238   θ z 2  3.3333

University of Castilla-La Mancha Draft By: Eduardo W. V. Chaves (2016)


Ciudad Real – Spain
6 LINEAR ELASTICITY 901

Problem 6.87
Consider the column described in Figure 6.230 in which the load is only due its own
weight, i.e. q x = ρgA , where ρ is the mass density, g is the gravity acceleration and A is
the cross-section area, and also consider that all these variables are constant along the
kg m 2 kgm 1 N
column. Note that the unit of q x is [q x ] = [ ρgA] = m = 2 = .
m3 s 2 s m m
a) By considering the deflection v = C3 ( x 3 − 3Lx 2 ) obtaining the critical value for q x ;
b) Obtain the finite element formulation for this problem.

x, u
x, u ( exact ) EI z
q x Cr = 7.83
L3 v2 , f y 2
2

θ z 2 , mz 2
 v1 
θ 
L  
qx 1 {u (e ) } =  z1 
 v2 
θ z 2 

y v1 , f y1
y, v 1

z z θz1, mz1

a) Column under compression b) Degree-of-freedom and nodal forces


Figure 6.230: Column under compression due to mass density.
Solution:
P(v′) 2
In Problem 6.81 we have obtained that dU ext = dx , now the force is varying along
2
P ( x)(v′) 2 q ( L − x)(v′) 2
the column, then we can state that dU ext = dx = x dx , then
2 2
L

∫ [(L − x)(v′) ]dx


L
 q ( L − x)  q
∫ ∫ (v′) 2  dx = x
ext ext 2
U = dU =  x
0
2  2 0

dP(v′) 2
Note also that we can consider that dU ext = dx and
2

dP
x
q x
L
 1 
L x


(v′) 2 dx ∫ ∫ ∫
(v′) dx  dx = ∫
q x (v′) dx  dx ∫
ext ext ext 2 2
dU ( x) = ⇒ U = dU =  x
2 0 2 0
0   2 0  0 
where we have considered dP = q x dx .
Case a) The derivatives of the deflections are:
d d
v( x) = C3 ( x 3 − 3Lx 2 ) →
dx v′( x) = C3 (3 x 2 − 6 Lx) →
dx v′′ = C3 (6 x − 6 L)

University of Castilla-La Mancha Draft By: Eduardo W. V. Chaves (2016)


Ciudad Real - Spain
902 SOLVING PROBLEMS BY MEANS OF CONTINUUM MECHANICS

The total potential energy becomes


EI z
L
1
L
 x 
Π = U int − U ext = ∫ (v′′) 2 dx − ∫ ∫
 q x (v′) dx  dx
2
2 0
2 0 
 0 

EI z
L
1
L
 x 
∫[C3 (6 x − 6 L)]2 dx − ∫ ∫
2 2
⇒Π= q x [C3 (3 x − 6 Lx )] dx  dx
2 0 2 0 
 0 
L L
EI z 1  9 
⇒Π=
2 0 ∫
[C3 (6 x − 6 L)]2 dx − 
20 5 ∫
q x C32  x 5 − 9 Lx 4 + 12 L2 x 3   dx


3q x L6 2
⇒ Π (C3 ) = 6 EI z L3C32 − C3
4
The inflection point is given by
∂Π ∂  3q x L6 2   3q x L6 
⇒ =  6 EI L3 2
C − C 3 =  12 EI L3
− C3 = 0
∂C3 ∂C3
z 3
4  z
2 
  
C ≠0 EI z ( exact ) EI

3
→ q x Cr = 8 3
> q x Cr = 7.83 3z
L L
The error is 2.17% .
Case b) In Problem 6.67 we have obtained that
 6x2 6x   6x2 6x   3x 2 4 x   3x 2 2 x 
v′ = θz = v1  3 − 2  + v2 − 3 + 2  + θ z1  2 − + 1 + θ z 2  2 − 
 L L   L L  L L  L L
L

∫ [( L − x)(v′) ]dx
qx
Then, the integral U ext = 2
can be expressed in terms of nodal values as
2 0
follows

∫ [(L − x)(v′) ]dx = q  10 v θ


L
qx − L L L2 L2 2 3 2
U ext = 2
x 2 z2 + v1 θ z 2 − θ z1 θ z 2 + θ z1 + v2 +
2 0  10 60 20 10
L2 2 3 3 
θ z 2 + v12 − v1v2 
60 10 5 
And by considering the total potential energy:

∫ [(L − x)(v′) ]dx


L L
EI z qx
Π = U int − U ext = ∫ (v′′) 2 dx − 2
(6.701)
2 0
2 0

we can obtain
EI z 12 2 12 2 4 2 4 2 24 12 12 12 12
Π=  v + 3 v2 + θz1 + θz 2 − 3 v1v2 + 2 v1 θz1 + 2 v1θ z 2 − 2 v2 θz1 − 2 v2 θz 2
3 1
2 L L L L L L L L L
4  − L L L2 L2 2 3 2
+ θ z1 θ z 2  − q x  v2 θz 2 + v1 θz 2 − θz1 θz 2 + θ z1 + v 2 +
L   10 10 60 20 10
L2 2 3 3 
θz 2 + v12 − v1v2 
60 10 5 
As we are looking for the stationary state the following must hold:

University of Castilla-La Mancha Draft By: Eduardo W. V. Chaves (2016)


Ciudad Real – Spain
6 LINEAR ELASTICITY 903

∂Π EI z  24 12 24 12  3 3 L 
=0 ⇒  3 v1 + 2 θ z1 − 3 v2 + 2 θ z 2  − q x  v1 − v2 + θ z 2  = 0
∂v1 2 L L L L  5 5 10 
∂Π EI z 12 8 12 4   L2 L2 
=0 ⇒  2 v1 + θ z1 − 2 v2 + θ z 2  − q x  θ z1 − θz 2  = 0
∂θ z1 2 L L L L  10 60 
∂Π EI z  − 24 12 24 12  − 3 3 L 
=0 ⇒  v1 − 2 θ z1 + 3 v2 − 2 θ z 2  − q x  v1 + v2 − θ z 2  = 0
∂v2 2  L3 L L L   5 5 10 
∂Π EI z 12 4 12 8  L L2 L L2 
=0 ⇒  2 v1 + θz1 − 2 v2 + θz 2  − qx  v1 − θz1 − v2 + θz 2  = 0
∂ θz 2 2 L L L L  10 60 10 30 
Restructuring the above set of equations in matrix form we can obtain:
  12 6 − 12 6   3 −3 L 
  5 0 
  L3 L2 L3 L 2  5 10  
  v
  6 4 −6 2 
 0 L2 − L2    1  0
  2  0    
L L2 L −q  10 60    θz1  = 0
 z  − 12
EI L
 −6 12 − 6 x − 3 3 − L    v2  0
  5 0    
  L3 L2 L3 L2  5 10   θ  0 (6.702)
 6 −6     z2   
 2 4
 L −L 2
−L L2  
  2  
  L L L2 L   10 60 10 30  

⇒ ([k ( e ) ] − q x [kq (e ) ]){u (e ) } = {0}

Note that we are not considering the strain energy due to the axial force, so if the domain
is discretized by several elements we have to transfer the concentrated load indirectly, for
instance, if the domain is dicretized into 3 finite elements we also have to consider the
effect of the concentrated load as the one described in Figure 6.231(b). For each element
we have to consider ([k ( e ) ] − q x [ [kq (e ) ] + L(je ) [kp y ( e ) ] ]){u( e ) } = {0} .

x, u
Pj(=e =4 3) = 0 ( L(j3) = 0)
4
qx 3

3  2 
Pj(=e 3= 2 ) = q x  L − L 
L 142343
qx qx 2 = L (j2 )

2  1 
Pj(=e 2=1) = q x  L − L 
142343
y, v qx 1 = L (j1)

z 1

a) Column under compression b) Discretization (3 finite elements)


Figure 6.231: Column under compression due to mass density – 3 finite elements.

University of Castilla-La Mancha Draft By: Eduardo W. V. Chaves (2016)


Ciudad Real - Spain
904 SOLVING PROBLEMS BY MEANS OF CONTINUUM MECHANICS

Let us consider 1 finite element, then by applying the boundary conditions v1 = 0 , θz1 = 0 ,
the equation in (6.702) becomes
 1 0 0 0  1 0 0 0 
 0  v  0
 0 1 0 0  
 1 0 0   1   
 12 EI z − 6 EI z   3 − L   θz1  0 (e) (e) (e)
 0 0 − q x 0 0    =   ⇒ ([k ] − q x [ kq ]){u } = {0}
L3 L2   5 10   2   
v 0
  
 0
− 6 EI z 4 EI z  −L L2  θ  0
0
L2  0 0   z 2   
 L   10 30  
and
 1 
 1 
(e)  EI  EI z
det ([k (e ) ] − q x [kq ]) = 0 ⇒ q x i = 152.111026 3z  ⇒ q x Cr = 7.888974
 L  L3
 7.888974 EI z 
 L3 
EI z
if we are adopting 2 elements the result is q x Cr = 7.857 , and by adopting 3 elements the
L3
EI z
critical value is q x Cr = 7.8421 .
L3

Problem 6.88
Obtain the critical load for the problem described in Figure 6.232. As academic problem
consider that L = 1m , K yf = 19.75kN / m 2 , EI z = 10kN / m 2 .

v (1) Degrees-of-freedom

x θz (2)

−P x, u
v2 , f y 2
2

θ z 2 , mz 2

L K yf 1

y y v1, f y1
1
z
z θz1, mz1
a) Column supported laterally by elastic restraint b) Degree-of-freedom and nodal forces
Figure 6.232: Beam and elastic base.

University of Castilla-La Mancha Draft By: Eduardo W. V. Chaves (2016)


Ciudad Real – Spain
6 LINEAR ELASTICITY 905

Solution:
In Problem 6.86 we have established that
[ [k (e ) ] + [ ke ( Spring _ y ) ] − P[kp (ye ) ] ]{u ( e ) } = {0}

where
 12 6 − 12 6   6 1 −6 1 
 L3 L2 L3 L2   5L 10 5L 10 
 6 4 −6 2   1 2L −1 − L
 2   
[k ] = EI z  L
(e) L L2 L  ; P[kp y ] = P  10
(e) 15 10 30 
 − 12 −6 12 − 6 − 6 −1 6 −1 
 L3 L2 L3 L2   5L 10 5L 10 
 6 2 −6 4   1 −L −1 2L 
 2   
 L L L2 L   10 30 10 15 
and
 13 11L 9 − 13L 
 35 210 70 420 
 
 11L L2 13L − L2 
[ke ( Spring _ y ) ] = K yf L  210 105 420 140 
 9 13L 13 − 11L 
 70 420 35 210 

 − 13L − L2 − 11L L2 
 420 140 210 105 
Then, after applying the boundary conditions ( v1 = 0 , v2 = 0 ) to the system
( Spring _ y ) (e)
[ [k ( e ) ] + [ke ] − P[kp y ] ]{u (e ) } = {0}

 1 0 0 0  1 0 0 0  

2 EI z L K y  − L   v1  0
3 f 3 f
 0 4 EI z L K y  2L
+ 0 −  0 0    
 L 105 L 140  − P  15 30   θ z1  = 0
 0 0 1 0 
0 0 1 0   v2  0
 3
2 EI z L K y
f 3 f
4 EI z L K y 
 −L 2 L     
 0 − + 0 0  θ  0
 L 140
0
L 105   30 15   z 2   
( Spring _ y ) (e)
and the solution for det[ [k ( e ) ] + [ke ] − P[kp y ] ] = 0 is

 1 
   1 
 4 f 1   
 L K y + 2520 EI z   1 
Pi =  =
  3
 1.2 × 10 
2
 4 f42 L
 L K y + 120 EI z  241.975
 
 10 L2 
And the critical load is Pcr = 241.975 , which is a very poor approximation with an error
approximately 50% . In order to obtain a better solution more finite element is needed.

University of Castilla-La Mancha Draft By: Eduardo W. V. Chaves (2016)


Ciudad Real - Spain
906 SOLVING PROBLEMS BY MEANS OF CONTINUUM MECHANICS

Problem 6.89
By using the finite element formulation, obtain the critical load for the problem described
in Figure 6.233.

x, u x, u
v
P>0
θz
−P
U5 Degrees-of-freedom
3 per node

U6
L( 2 ) = L 2
U 1 
U3 U 
2
 2
U 3 
L(1) = L {U } =  
U4 1 U 4 
U 5 
y  
U1 1 U 6 
y, v
z z U2

a) Column under compression b) Global degree-of-freedom

Figure 6.233: Column under compression.


Solution:
Construction of the Global Stiffness Matrices - [ K Global ] and [ KpGlobal ]
We need to construct the following system
[ [ K Global ] − P[ KpGlobal ] ]{U } = {0}
The matrices [ K Global ] and [ KpGlobal ] can be constructed by assembling the individual beam
elements, i.e.:
2 2

A A[kp
(e) (e )
[ K Global ]6×6 = [k Global ] [ KpGlobal ]6×6 = Global ] (6.703)
e =1 e =1

where
 12 6 − 12 6   6 1 −6 1 
 L3 L2 L3 L2   5L 10 5L 10 
 6 4 −6 2   1 2L −1 − L
 2   
(e)
[kGlobal ] = EI z  L L L2 L  ; (e)
[kpGlobal ] =  10 15 10 30  (6.704)
 − 12 −6 12 − 6 − 6 −1 6 −1 
 L3 L2 L3 L2   5L 10 5L 10 
 6 2 −6 4   1 −L −1 2L 
 2   
 L L L2 L   10 30 10 15 
For this problem the stiffness matrices for both elements are the same.
For both elements the stiffness matrices are the same, the

University of Castilla-La Mancha Draft By: Eduardo W. V. Chaves (2016)


Ciudad Real – Spain
6 LINEAR ELASTICITY 907

k11(1) k12(1) k13(1) k14(1) 0 0 


 (1) (1) (1) (1) 
k 21 k 22 k 23 k 24 0 0 
2 k (1) (1)
k 32 (1)
k 33 + k11( 2 ) (1)
k 34 + k12( 2 ) k13( 2 ) k14( 2) 
A
(e )
[ K Global ]6×6 = [k Global ] =  31
(1) (1) (1) ( 2) (1) (2) ( 2) ( 2)

e =1 k 41 k 42 k 43 + k 21 k 44 + k 22 k 23 k 24 
 0 0 ( 2)
k 31 ( 2)
k 32 ( 2)
k 33 ( 2) 
k 34
 
( 2) ( 2) ( 2) ( 2)
 0 0 k 41 k 42 k 43 k 44 

After the values are substituted we can obtain


 12 6 − 12 6 
 L3 0 0 
L2 L3 L2
 6 4 −6 2 
 2 0 0 
 L L L2 L 
 − 12 −6 24
0
− 12 6 
 3 L2 L3 L3 L2 
[ K Global ]6× 6 = EI z  L
6 2 8 −6 2 
 2 0 
 L L L L2 L 
 0 − 12 −6 12 − 6
0
 L3 L2 L3 L2 
 6 2 −6 4 
 0 0 
 L2 L L2 L 
And

1 2 3 4 5 6 Global
kp11
(1) (1)
kp12 (1)
kp13 (1)
kp14 0 0 
 (1) (1) (1) (1) 
kp21 kp22 kp23 kp24 0 0 
2 kp (1) (1)
kp32 (1)
kp33 ( 2)
+ kp11 (1)
kp34 + kp12( 2) (2)
kp13 ( 2) 
kp14
A
(e)
[ KpGlobal ]6×6 = [kpGlobal ] =  31(1) (1) (1) ( 2) (1) ( 2) (2) ( 2)

e =1 kp41 kp42 kp43 + kp21 kp44 + kp22 kp23 kp24 
 0 0 (2)
kp31 ( 2)
kp32 (2)
kp33 ( 2) 
kp34
 
(2) ( 2) (2) ( 2)
 0 0 kp41 kp42 kp43 kp44 

After the values are substituted we can obtain


 6 1 −6 1 
 5L 0 0 
10 5L 10
 1 2L −1 −L 
 0 0 
 10 15 10 30 
− 6 −1 12
0
−6 1 
 10 
[ KpGlobal ]6× 6 =  5L 10 5L 5L
1 −L 4L −1 − L
 0 
 10 30 15 10 30 
 0 −6 −1 6 −1 
0
 5L 10 5L 10 
 1 −L −1 2L 
 0 0 
 10 30 10 15 

Applying the Boundary Conditions


Note that there are restrictions to move for the following degrees-of-freedom: U 1 = 0 ,
U 3 = 0 , thus

[ [ K Global ] − P[ KpGlobal ] ]{U } = {0} Boundary


Conditions

→[ [ K Global ] − P[ KpGlobal ] ]{U } = {0}

University of Castilla-La Mancha Draft By: Eduardo W. V. Chaves (2016)


Ciudad Real - Spain
908 SOLVING PROBLEMS BY MEANS OF CONTINUUM MECHANICS

 1 0 0 0 0 0  1 0 0 0 0 0 
 4 EI z 2 EI z   2L −L 
 0 0 0 0  0 0 0 0  U1  0
 L L
 0
15 30 U  0
 0 0 1 0 0 0 0 1 0 0 0   2   
2 EI z 8 EI z − 6 EI z 2 EI z   −L 4L −1 − L  U 3  0
 0 0  − P 0 0  =
 L L L2 L   30 15 10 30  U 4  0
 − 6 EI z 12 EI z − 6 EI z  −1 6 −1      
0 U 5  0
 0 0 0 0 0 
 L2 L3 L2   10 5L 10     
 0 2 EI z − 6 EI z 4 EI z   −L −1 2 L  U 6  0
 0 0  0 0 0 
 L L2 L   30 10 15  

[ [ K Global ] − P[ Kp Global ] ]{U } = {0}

The solution for the above system, i.e. the eigenvalues of det[ [ K Global ] − P[ Kp Global ] ] = 0
are:
( P − 1) 2 (17 L8 P 4 − 1776 L6 EI z P 3 + 52704 L4 ( EI z ) 2 P 2 − 449280 L2 ( EI z )3 P + 518400( EI z ) 4 )
=0
3600 L6
And the solution is
 EI z 
 60 L2 
 EI z 
 12 2 
 L 
31.10915 EI z  EI z EI
Pi =   ⇒ Pcr = 1.36143 > Pcr(exact ) ≈ 1.359 2z
 L2  L2
L
 1.36143 EI z 
 L2 
 1 
 
 1 

University of Castilla-La Mancha Draft By: Eduardo W. V. Chaves (2016)


Ciudad Real – Spain
6 LINEAR ELASTICITY 909

6.6 Introduction to Flexural Plates

Problem 6.90
Obtain expressions for the following fields: displacement, strain and stress for the problem
represented in Figure 6.234 in which w = w( x, y ) represents the deflection according to z -
direction; θx and θ y stand for the rotations according to the directions x and y
respectively; and t is the plate thickness.
Hint: In order to obtain the displacement field consider the Euler-Bernoulli beam theory
(the classical beam theory), i.e. by combining Figure 6.145 and Figure 6.147.

z, x3
a
y , x2

b qz

t neutral surface

θy
x, x1
θx

Figure 6.234: Flexural plate.


Solution:
According to Figure 6.145 and Figure 6.147 the displacement field, (see Figure 6.235), can
be represented as follows:
− ∂w − ∂w
w = w( x, y ) ; u = u ( x, y , z ) = z ≡ − w, x z ; v = v ( x, y , z ) = z ≡ − w, y z (6.705)
∂x ∂y

z y θ y = − w, x
w θy
x u = − w, x z

z w
y
θ x = w, y

x v = − w, y z
θx

Figure 6.235: Classical beam theory.

University of Castilla-La Mancha Draft By: Eduardo W. V. Chaves (2016)


Ciudad Real - Spain
910 SOLVING PROBLEMS BY MEANS OF CONTINUUM MECHANICS

The Displacement Field


The displacement field vector, (see Figure 6.236), can be represented as follows:
u1   u  − w, x z   θ y z   βx z   z 0 0  βx 
           
{u( x )} = u 2  =  v  = − w, y z  = − θx z  = β y z  = 0 z 0 βy 
r
(6.706)
u  w  w   w   w  0 0 1  w 
 3           
where we have introduced the variables βx = θ y = − w, x and β y = − θ x = − w, y . The above
equation in indicial notation becomes
− ∂u3
u3 = u3 ( x1 , x2 ) ; ui = x3 ≡ −u3,i x3 (i = 1,2) (6.707)
∂xi

βx = θ y = − w, x βx
w
v z u ( z = t / 2)
β y = − θ x = − w, y y
u
u (z )
z t
w βx 2 z
βy x
t t
2 2

x u( z)
t tan βx ≈ βx =
2 z

a) Displacement and rotation b) u -displacement


Figure 6.236: Displacement and rotations – Kirchhoff-Love plate theory.

The Strain Field


For small deformation regime the strain and displacement, (see Problem 5.8), are related
to each other by using Voigt notation as follows:

∂  ∂   ∂( βx z ) 
 ∂x 0 0  ∂x 0 0  ∂x 
     
∂ ∂  ∂( βy z ) 
ε11   ε x   0 0 0 0  
     ∂y   ∂y  ∂ y
 
ε 22   ε y   ∂ u   ∂   β z   ∂( w( x, y )) 
ε33   ε z   0 0
∂z   v  = 
0 0
∂z  β z  = 
x
∂z 
{ε } =  = = ∂ ∂   ∂ ∂  y   ∂(β z) ∂(β z) 
 2 ε12  γ xy   0   w  0  w   x
+
y

2ε 23  γ yz   ∂y ∂x     ∂y ∂x     ∂y ∂x 
     ∂ ∂  ∂ ∂  ∂ ( βy z ) ∂ ( w) 
2ε13   γ xz   0  0   + 
 ∂z ∂y   ∂z ∂y 
 ∂z ∂y 
∂ 0
∂ ∂ 0
∂  ∂ ( βx z ) ∂ ( w) 
 ∂z ∂x   ∂z ∂x   + 
 ∂z ∂x 
thus

University of Castilla-La Mancha Draft By: Eduardo W. V. Chaves (2016)


Ciudad Real – Spain
6 LINEAR ELASTICITY 911

ε11   ε x   βx , x z   z βx , x   − zw, xx 
         
ε 22   ε y   βy , y z   z βy , y   − zw, yy 
ε 33   ε z   0   0   0 
⇒ {ε } =  = = = =  (6.708)
2ε12  γ xy  βx , y z + βy , x z  2 z βx , y  − 2 zw, xy 
2ε 23  γ yz   βy + w, y   0   0 
         
2ε13   γ xz   βx + w, x   0   0 

Note that βx = θ y = − w, x and β y = − θ x = − w, y , then the derivatives are βx , y = θ y , y = − w, xy


and β y , x = − θ x , x = − w, yx , so that we can conclude βx , y = − w, xy = βy , x .
The strain field (6.708) could have been obtained by means of indicial notation:
r 1 − ∂u3
ε ij = [(∇u) sym ]ij = (ui , j + u j ,i ) = ui , j = x3 ≡ −u3,ij x3 (i = 1,2)
2 ∂xi ∂x j (6.709)
ε 3i = 0 (i = 1,2,3)

where we have considered that


− ∂u3 − ∂u3
ui = x3 ≡ −u3,i x3 ⇒ ui , j = x3 ≡ −u3,ij x3 = u j ,i (i = 1,2)
∂xi ∂xi ∂x j
The Stress Field
To obtain the stress-strain relationship we assume that the stress according to z -direction
is zero, i.e. σ 3i = σ i 3 = 0 since we have assume that ε 3i = 0 , in other words we are dealing
with the state of plane stress. And the stress-strain relationship for the state of plane stress
in Voigt notation, (see Problem 6.24), is given by:
   
1 ν 1 ν
σx  0   εx  0   − zw, xx 
  E   E  
σ y  = 1 − ν 2 ν 1 0  εy  = ν 1 0   − zw, yy 
1−ν    1−ν
2
τ xy    1−ν   
   0 0  γ 0 0  − 2 zw, xy 
2   2 
xy
  (6.710)
σx   w, xx + ν w, yy 
  − Ez  
⇒ σ y  = ν w, xx + w, yy 
τ xy  1 − ν
2
 (1 − ν ) w, xy 
   
where E stands for Young’s modulus and ν is the Poisson’s ratio. The stress-strain
relationship in indicial notation, (see Problem 6.24), can be obtained as follows:
 νE E
σij = (1 − ν 2 ) Tr (ε )δ ij + (1 + ν ) ε ij ; (i, j = 1,2) with Tr (ε ) = ε11 + ε 22
 (6.711)
ε = − ν Tr (σ )δ + (1 + ν ) σ (i, j = 1,2,3) (the same as 3D)
 ij E
ij
E
ij

Taking into account that ε ij = −u3,ij x3 and Tr (ε ) = Tr (−u3,ij x3 ) = −u3,kk x3 the above
equation for stress becomes
− E x3
σ ij = [ν u3,kkδ ij + (1 − ν )u3,ij ] ; (i, j = 1,2) (6.712)
(1 − ν 2 )

The above equation can also be expressed more explicitly as follows:

University of Castilla-La Mancha Draft By: Eduardo W. V. Chaves (2016)


Ciudad Real - Spain
912 SOLVING PROBLEMS BY MEANS OF CONTINUUM MECHANICS

− E x3
σij = [ν u3,kkδ ij + (1 − ν )u3,ij ] ; (i, j = 1,2)
(1 − ν 2 )
− E x3
⇒ σij = [ν (u3,11 + u3, 22 )δ ij + (1 − ν )u3,ij ]
(1 − ν 2 )
− E x3  ν (u3,11 + u3, 22 ) 0   (1 − ν )u3,11 (1 − ν )u3,12  

⇒ σij =  +
(1 − ν 2 )   0 ν (u3,11 + u3, 22 )  (1 − ν )u3, 21 (1 − ν )u3, 22  
 ∂ 2u3 ∂ 2u3 ∂ 2u3 
+ ν (1 − ν )
− E x3 ν u3, 22 + u3,11 (1 − ν )u3,12  − E x3  ∂x12 ∂x22

∂x1∂x2 
⇒ σij =   = 
1 − ν 2  (1 − ν )u3,12 ν u3,11 + u3, 22  1 − ν 2  ∂ 2u3 ∂ 2u3 ∂ 2u3 
 (1 − ν ) ∂x ∂x + ν
 1 2 ∂x22 ∂x12 
which matches the equation in (6.710).
NOTE 1: The problem established here is called Kirchhoff-Love Plate Theory, and is used to
solve flexural plates when the thickness is very small.
NOTE 2: Note that the stresses σ 3i = σ i 3 ≠ 0 are not zero, (see Figure 6.237), and we
cannot obtain σ 3i from the constitutive equations since we have assumed that ε 3i = 0 . In
order to obtain the equations for σ 3i we have to consider the equilibrium equations
σ ij , j = 0 i (without body forces and static case).

σy
y
τ xy
σx τ yx
τ zy
t
2
x τ zx
t
2

Figure 6.237: Stress distribution – Kirchhoff-Love plate theory.

NOTE 3: The Resultant Moments/Forces


By considering the infinitesimal element described in Figure 6.238 the resultant moment
can be expressed as follows
t t t
2 2 2
mx = ∫ zσ dz
−t
x ; my = ∫ zσ dz
−t
y ; mxy = ∫ zτ
−t
xy dz (i, j = 1,2) (6.713)
2 2 2

note that τ xy = τ yx hence mxy = m yx . The above equations in indicial notation can be
represented as follows:

University of Castilla-La Mancha Draft By: Eduardo W. V. Chaves (2016)


Ciudad Real – Spain
6 LINEAR ELASTICITY 913

t
2
mij = ∫ x σ dx
−t
3 ij 3 (i, j = 1,2) (6.714)
2

The resultant forces


t t
2 2
Q31 = Qx = ∫τ
−t
zx dz ; Q32 = Q y = ∫τ
−t
zy dz (i, j = 1,2) (6.715)
2 2

or in indicial notation
t
2


Q3i = σi 3 dx3
t
(i, j = 1,2) (6.716)
2

Note that
t t t t
2 2 2 2

∫ ∫ ∫
mij , j = ( x3σ ij ) , j dx3 = ( x3, j σij + x3σij , j )dx3 = ( x3, j σij )dx3 + ( x3σ ij , j )dx3
−t −t −t

−t
2 2 2 2
t t t
2 2 2

−t
∫ { {
−t
{ ∫
⇒ mij , j = ( x3,1 σi1 + x3, 2 σ i 2 + x3,3 σ i 3 )dx3 + ( x3 σ ij , j )dx3 = σi 3 dx3 = Q3i
{
−t

=0 =0 =1 =0i
2 2 2

Flexural plate differential equation


mij , j − Q3i = 0i (6.717)
(in terms of moments)

where we have applied the equilibrium equations σij , j = 0i . Taking into account the
equation for σij given by the equation in (6.712), the moments become
t t
2 2
−E
mij = ∫ x σ dx
−t
3 ij 3 =
(1 − ν )
2
[ν u3,kkδ ij + (1 − ν )u3,ij ] x32 dx3
−t
∫ (i, j = 1,2)

2 2
3
− Et
⇒ mij = [ν u3,kkδ ij + (1 − ν )u3,ij ]
12(1 − ν 2 )

 ∂ 2u3 ∂ 2u3 ∂ 2u3 


 2 +ν (1 − ν ) 
∂x ∂x22 ∂x1∂x2 
⇒ mij = − D  1 (6.718)
 2
∂ u3 ∂ 2u3 ∂ 2u3 
 (1 − ν ) ∂x ∂x + ν
 1 2 ∂x22 ∂x12 

Et 3
where we have introduced the parameter D = which is called the bending stiffness
12(1 − ν 2 )
of the plate or flexural rigidity of the plate. The above equation in Voigt notation can be
expressed as follows

University of Castilla-La Mancha Draft By: Eduardo W. V. Chaves (2016)


Ciudad Real - Spain
914 SOLVING PROBLEMS BY MEANS OF CONTINUUM MECHANICS

 ∂ 2u 3 
 
   ∂x12 
 m11   m x   D ν D 0   ∂ 2u 
   
{m} = m22  =  m y  = − νD D 0  3
 Et 3
 m  m   2
D(1 − ν )   ∂x 2  with D = (6.719)
 12   xy  0 0   ∂ 2u  12(1 − ν 2 )
 2  2 3
 
 ∂x 1 ∂ x 2

⇒ {m} = {DK }{εˆ}


or in Engineering notation:
 ∂2w   ∂2w ∂2w
   − D 2 − Dν 2 
   ∂x 2   ∂x ∂y 
 mx 
   D νD 0  ∂ 2 w   ∂2w ∂2w
 m y  = − νD D 0 
2 
= − Dν 2 − D 2  (6.720)
m   D (1 − ν )   ∂y   ∂x ∂y 
 xy  0 0  ∂ 2 w   ∂ w 
2
 2  2
   − D(1 − ν ) 
 ∂x∂y   ∂x∂y 

Integration over z

a) Stresses b) Resultant
Moments/Forces
z z

mx
Qy m yx
τ zy t
2 y
σy y my
Qx
m xy my
τ zx τ xy t
τ yx 2 m xy m yx
σx mx Qy
σy
τ zx τ xy
x τ zy x Qx
τ yx
σx

Figure 6.238: Infinitesimal element of plate.

NOTE 4: Plate Differential Equation


Now if we consider the differential element of plate, (see Figure 6.239), and by applying the
equilibrium of force according to z -direction we can obtain:

∑F z =0
 ∂Q   ∂Q y 
⇒ q z dxdy − Q y dx − Qx dy +  Qx + x dx dy +  Q y + dy  dx = 0
 ∂x   ∂y  (6.721)
∂Qx ∂Qy
⇒ + + qz = 0
∂x ∂y
or in indicial notation
Q3i ,i + q z = 0 (i, j = 1,2) (6.722)

University of Castilla-La Mancha Draft By: Eduardo W. V. Chaves (2016)


Ciudad Real – Spain
6 LINEAR ELASTICITY 915

Equilibrium of moments
Total moment about x -direction:
∑M x =0
 ∂m yx   ∂Q x  dy
⇒ m y dx + m yx dy −  m yx + dx dy +  Q x + dx  dy +
 ∂x   ∂x  2
 ∂m y   ∂Q y  dy dy
−  m y + dy  dx +  Q y + dy dxdy − Q x dy + q z dxdy =0
 ∂y   ∂y  2 2
− ∂m yx ∂Q x dy ∂m y ∂Q y dy
⇒ dxdy + dxdy − dxdy + Q y dxdy + dxdydy + q z dxdy =0
∂x ∂x 2 ∂y ∂y 2
 ∂m yx ∂m y   ∂Q ∂Q y  dy ∂Q y dy
⇒ − + − Q y  dxdy +  x + + q z dxdy + dxdy =0
 ∂x ∂y  1∂4
x ∂y
442444 3  2 ∂y 2
=0

 ∂m yx ∂m y  ∂Q y dy
⇒ − + − Q y dxdy + dxdy =0
 ∂x ∂y  ∂y 2
And by discarding the term related to dxdydy ≈ 0 we can obtain:
∂m yx ∂m y
+ − Qy = 0 (6.723)
∂x ∂y
Total moment about y -direction:

∑M y =0
 ∂m xy   ∂m x 
⇒ − m x dy − m xy dx +  m xy + dy  dx +  m x + dx  dy +
 ∂y   ∂x 
dx  ∂Q x   ∂Q y  dx dx
+ Q y dx −  Q x + dx dydx −  Q y + dy dx − q z dxdy =0
2  ∂x   ∂y  2 2
∂m xy ∂m x ∂Q x ∂Q y dx dx
⇒ dydx + dxdy − Q x dydx − dxdydx − dydx − q z dxdy =0
∂y ∂x ∂x ∂y 2 2
 ∂m xy ∂m x   ∂Q ∂Q y  dx ∂Q x dx
⇒  + − Q x dxdy −  x + + q z dydx + dxdy =0
 ∂y ∂x  1∂4
x ∂y
442444 3  2 ∂x 2
=0

 ∂m xy ∂m x  ∂Q x dx
⇒  + − Q x dxdy + dxdy =0
 ∂y ∂x  ∂x 2
And by discarding the term related to dxdydx ≈ 0 we can obtain:
∂m xy ∂m x
+ − Qx = 0 (6.724)
∂y ∂x

University of Castilla-La Mancha Draft By: Eduardo W. V. Chaves (2016)


Ciudad Real - Spain
916 SOLVING PROBLEMS BY MEANS OF CONTINUUM MECHANICS

∂Qy
Qy + dy
∂y
∂mxy ∂my
mxy + dy my + dy
∂y ∂y

Qx
myx qz
mx ∂mx
dy mx + dx
∂x
∂Qx
Qx + dx
∂x
z y my m xy ∂myx
myx + dx
∂y
x
Qy

dx

Figure 6.239: Differential element of plate.

According to equation (6.720) we have


 ∂2w   ∂2w ∂2w
 − D − D ν 
   ∂x 2   ∂x 2 ∂y 2 
 mx 
   D νD 0  ∂ 2 w   ∂2w ∂2w
 m y  = − νD D 0 
2  =  − D ν − D 
m   D (1 − ν )   ∂y   ∂x 2 ∂y 2 
 xy  0 0  ∂ 2 w   ∂2w 
 2  2 − D (1 − ν )
   
 ∂x∂y   ∂x∂y 

Then, the equation (6.724) can be rewritten as follows


∂m xy ∂m x ∂  ∂2w  ∂  ∂2w ∂2w 
+ − Q x = 0 ⇒ Q x =  − D (1 − ν )  +  − D 2 − Dν 2 
∂y ∂x ∂y  ∂x∂y  ∂x  ∂x ∂y 
(6.725)
∂  ∂2w ∂2w 
⇒ Q x = − D  2 + 2 
∂x  ∂x ∂y 

And the equation in (6.723) can be rewritten as follows


∂m yx ∂m y ∂  ∂2w  ∂  ∂2w ∂2w 
+ − Qy = 0 ⇒ Qy =  − D(1 − ν )  +  − Dν 2 − D 2 
∂x ∂y ∂x  ∂x∂y  ∂y  ∂x ∂y 
(6.726)
∂  ∂2w ∂2w 
⇒ Q y = − D  2 + 2 
∂y  ∂x ∂y 

Then, by substituting the equations (6.725) and (6.726) into the force equilibrium equation
(6.721) we can obtain:

University of Castilla-La Mancha Draft By: Eduardo W. V. Chaves (2016)


Ciudad Real – Spain
6 LINEAR ELASTICITY 917

∂Q x ∂Q y
+ + qz = 0
∂x ∂y
∂  ∂  ∂2w ∂2w  ∂  ∂  ∂2w ∂2w 
⇒ − D  2 + 2   +  − D  2 + 2   + q z = 0
∂x  ∂x  ∂x ∂y   ∂y  ∂y  ∂x ∂y  
(6.727)
∂2 ∂2w ∂2w  ∂2  ∂2w ∂2w 
⇒ −D 2 2 + 2 −D 2  2 + 2  + qz = 0
∂x ∂x ∂y  ∂y  ∂x ∂y 
 
4 4 4
∂ w ∂ w ∂ w q
⇒ 4 +2 2 2 + 4 = z
∂x ∂x ∂y ∂y D
which is the flexural plate differential equation in terms of w -displacement (deflection).
The above equation can also be written as follows:

∂4w ∂4w ∂ 4w qz
+ 2 + =
∂x 4 ∂x 2 ∂y 2 ∂y 4 D
Flexural plate differential equation
or (6.728)
(in terms of deflection w )
qz qz
∇ 2xr (∇ 2xr w) = ⇔ ∇ 4xr w =
D D

where ∇ 2xr ≡ ∇ xr ⋅ (∇ xr ) is the Laplacian operator and ∇ 4xr is the bi-Laplacian operator.
Using indicial notation:
By starting from the equation (6.717) we can obtain:
mij , j − Q3i = 0 i (i, j = 1,2)
 − Et 3 
⇒  [ν u 3,kkδ ij + (1 − ν )u 3,ij ]  − Q3i = 0 i
 12(1 − ν )
2
, j
− Et 3
⇒ [ν u 3,kkjδ ij + (1 − ν )u 3,ijj ] − Q3i = 0 i
12(1 − ν 2 )
⇒ − D[ν u 3,kki + (1 − ν )u 3,ikk ] − Q3i = 0 i
⇒ − Du3,kki − Q3i = 0 i (6.729)
Taking the derivative with respect to xi we can obtain
⇒ − Du 3,kkii − Q3i ,i = 0 i ,i = 0
⇒ − Du 3,kkii + q z = 0
(6.730)
q
⇒ u 3,kkii = z (i, k = 1,2)
D
which is the same as the equation in (6.728).
NOTE 5: The stresses σ 3i
Once the problem is solved the stresses σ 3i , (see Figure 6.237), can be obtained. We start
from the equilibrium equation
σ ij , j = 0 i ⇒ σ i1,1 + σ i 2, 2 + σ i 3,3 = 0 i ⇒ σ i 3,3 = −(σ i1,1 + σ i 2, 2 )

which is the same as


σ i 3,3 = −(σ i1,1 + σ i 2, 2 ) ⇒ σ i 3,3 = −σ ik ,k (i = 1,2,3; k = 1,2) (6.731)

University of Castilla-La Mancha Draft By: Eduardo W. V. Chaves (2016)


Ciudad Real - Spain
918 SOLVING PROBLEMS BY MEANS OF CONTINUUM MECHANICS

According to the equation (6.712) we can obtain


− E x3
σ ik = [ν u 3,ttδ ik + (1 − ν )u 3,ik ] ; (i, k , t = 1,2)
(1 − ν 2 )
− E x3 − E x3
⇒ σ ik ,k = [ν u 3,ttδ ik + (1 − ν )u 3,ik ],k = [ν u 3,ttkδ ik + (1 − ν )u 3,ikk ]
(1 − ν )2
(1 − ν 2 )
− E x3 − E x3 (6.732)
⇒ σ ik ,k = [ν u 3,tti + (1 − ν )u 3,ikk ] = [ν u 3,kki + (1 − ν )u 3,ikk ]
(1 − ν )2 { (1 − ν 2 ) {
=u3 , kki =u3 , kki

− E x3
⇒ σ ik ,k = u 3,kki
(1 − ν 2 )
Taking into account the equation in (6.729) the above equation can be written as follows:
− E x3 − E x3  − Q3i  E Q3i x3 12(1 − ν 2 )
σ ik ,k = u 3, kki =  =
(1 − ν 2 ) (1 − ν 2 )  D  (1 − ν 2 ) Et 3
(6.733)
12Q
⇒ σ ik ,k = 3 3i x3 (i, k = 1,2)
t
Et 3
where we considered D = . By substituting the above equation into the
12(1 − ν 2 )
equilibrium equation (6.731) we can obtain
∂σ i 3 − 12Q3i
σ i 3,3 ≡ = −σ ik ,k = x3 (i, k = 1,2) (6.734)
∂x3 t3
By integrating the above equation over x3 we can obtain
 − 12Q3i  − 12Q3i 2
σi3 = 
 t
∫ 3
x3 ∂x3 =
 2t 3
x3 + K (6.735)

t
The constant of integration can be obtained by the condition: x3 = ± ⇒ σ i 3 = 0 :
2
2
− 12Q3i 2 − 12Q3i  ± t  3Q3i
σi 3 = 3
x3 + K =   +K =0 ⇒ K=
2t 2t 3  2  2t
Then
− 12Q3i 2 − 6Q3i 2 3Q3i 3Q3i   2x  2 
σi3 = 3
x3 + K = x3 + = 1 −  3   (6.736)
2t t3 2t 2t   t  
From the equilibrium equation (6.731) we can also obtain σ 33,3
∂σ 33  − 6Q3k 2 3Q3k 
σ 33,3 ≡ = −σ 3k ,k = − x3 +  (k = 1,2)
∂x3  t
3
2t  ,k
(6.737)
∂σ 6Q3k ,k 2 3Q3k ,k integrating over x3 6Q3k ,k x33 3Q3k ,k
⇒ 33 = x 3 −      → σ 33 = − x3 + K
∂x3 t3 2t t3 3 2t
−t
The constant of integration can be obtained by means x3 = ⇒ σ 33 = 0 , then
2

University of Castilla-La Mancha Draft By: Eduardo W. V. Chaves (2016)


Ciudad Real – Spain
6 LINEAR ELASTICITY 919

3
−t 
 
6Q3k ,k  2  − 3Q3k ,k −t  − Q3k ,k
σ 33 = 3  +K =0 ⇒ K=
t 3 2t  2  2
thus
2Q3k ,k 3Q3k ,k Q3k ,k 3  2 2x 1  2x 3 
σ 33 = x33 − x3 − = − Q3k ,k  + 3 −  3   (6.738)
t3 2t 2 4  3 t 3  t  

By means of equation in (6.722), the above equation can also be written as follows:

3  2 2 x3 1  2 x 3  
3
σ 33 = qz  + −    (6.739)
4  3 t 3  t  

t
And note that when x3 = we can obtain
2

3 2 2  t  1  2  t  
3

σ 33 = q z  +   −      = q z
4 3 t  2  3  t  2  
 

University of Castilla-La Mancha Draft By: Eduardo W. V. Chaves (2016)


Ciudad Real - Spain
920 SOLVING PROBLEMS BY MEANS OF CONTINUUM MECHANICS

University of Castilla-La Mancha Draft By: Eduardo W. V. Chaves (2016)


Ciudad Real – Spain
11 Introduction to Fluids

Problem 11.1
Demonstrate whether the following statements are true or false:
a) If the velocity field is steady, then the acceleration field is also;
b) If the velocity field is homogeneous, the acceleration field is always equal to zero;
c) If the velocity field is steady and the medium is incompressible, the acceleration is always
zero.
Solution:
r r
∂v ( x , t) r
a) In a steady velocity field we have = 0 whereby the acceleration field becomes:
∂t
r
∂vi ( x , t )
a i = v&i = + vi ,k v k = v i ,k v k
142 ∂t 43
=0i
r r
r r& ∂v ( x ) r r r r r r r r
a=v = + ∇ xr v ( x ) ⋅ v ( x ) = ∇ xr v ( x ) ⋅ v ( x )
∂t 1 4 4244 3
Independent of time

Then, assumption (a) is TRUE.


r r r
b) A homogeneous velocity field implies that v ( x , t ) = v (t ) , whereby:
r r r r
r r ∂v ( x , t ) r r r r ∂v ( x , t )
a = v& = + ∇ xr v ( x , t ) ⋅ v ( x, t ) =
∂t 1424 3 ∂t
=0

Then, assumption (b) is FALSE.


r r r r
c) A steady velocity field implies that v ( x , t ) = v ( x ) and an incompressible medium means
r r
that ∇ xr ⋅ v ( x , t ) = 0 , so, we can conclude that:
r r
r r& ∂v ( x ) r r r r r r r r
a=v = + ∇ xr v ( x ) ⋅ v ( x ) = ∇ xr v ( x ) ⋅ v ( x )
∂t
Then, assumption (c) is FALSE.
800 SOLVING PROBLEMS BY MEANS OF CONTINUUM MECHANICS

Problem 11.2
Show the Navier-Stokes-Duhem equations of motion:
ρv&i = ρbi − p,i + (λ* + µ * )v j , ji + µ *vi , jj Navier-Stokes-Duhem
r r r r equations of motion (11.1)
ρv& = ρb − ∇ xr p + (λ* + µ * )[∇ xr (∇ xr ⋅ v )] + µ *∇ 2xr v
Solution:
The Navier-Stokes-Duhem equations of motion are a combination of the equations of
r r
motion ∇ xr ⋅ σ + ρb = ρv& , (σ ij , j + ρ b i = ρv&i ) , and the constitutive equations:

σ = − p1 + λ* Tr (D)1 + 2 µ *D σij = − pδ ij + λ*δ ijD kk + 2 µ *Dij (11.2)

The Cauchy stress tensor divergence ( ∇ xr ⋅ σ ) can be evaluated as follows:


σij , j = ( − pδ ij + λ*δ ijD kk + 2 µ *Dij ) j = − p, jδ ij + λ* (δ ijD kk ), j + 2 µ *Dij , j
(11.3)
= − p,i + λ*δ ijD kk , j + 2 µ *Dij , j = − p,i + λ*D kk ,i + 2 µ *Dij , j

Note that, in this formulation, we are considering that the material is homogeneous, i.e.
λ*, j = µ *, j = 0 j .
In addition, by considering 2D ij = vi , j + v j ,i and 2D kk = v k ,k + v k , k = 2v k ,k , we can obtain:
⇒ 2D ij , j = vi , jj + v j ,ij = vi , jj + v j , ji ⇒ D kk , j = vk ,kj (11.4)
whereby the equation in (11.2) becomes:
σij , j = − p,i + λ*D jj ,i + 2 µ *Dij , j = − p,i + λ*v j , ji + µ * (vi , jj + v j , ji )
(11.5)
= − p,i + (λ* + µ * )v j , ji + µ *vi , jj

Then, by substituting the equation in (11.5) into the equations of motion ( σ ij , j + ρ b i = ρ v&i ),
we obtain the Navier-Stokes-Duhem equations of motion for homogeneous materials.
NOTE 1: The explicit form of the equation (11.1) is presented as follows:
(λ * + µ * )v j , ji + µvi , jj + ρb i − p,i = ρv&i
⇒ (λ * + µ * )(v1,1i + v2, 2i + v3,3i ) + µ * (vi ,11 + vi , 22 + vi ,33 ) + ρb i − p,i = ρv&i

(λ * + µ * )(v1,11 + v2, 21 + v3,31 ) + µ * (v1,11 + v1, 22 + v1,33 ) − p,1 + ρb1 = ρv&1
 *
(λ + µ )(v1,12 + v2, 22 + v3,32 ) + µ (v2,11 + u 2, 22 + v2,33 ) − p, 2 + ρb 2 = ρv&2
* *

 *
(λ + µ )(v1,13 + v2, 23 + v3,33 ) + µ (v3,11 + v3, 22 + v3,33 ) − p,3 + ρb 3 = ρv&3
* *

or:

University of Castilla-La Mancha Draft By: Eduardo W. V. Chaves (2016)


Ciudad Real – Spain
11 INTRODUCTION TO FLUIDS 801

 * ∂  ∂v1 ∂v2 ∂v3   ∂ 2v ∂ 2v ∂ 2v 


(λ + µ )
*
 + +  + µ *  21 + 21 + 21  + ρb1 − p,1 = ρv&1
∂x1  ∂x1 ∂x2 ∂x3   ∂x ∂x2 ∂x3 
  1

 * ∂  ∂v1 ∂v2 ∂v3   ∂ 2v ∂ 2 v2 ∂ 2 v2 
(λ + µ )
*
 + +  + µ *  22 + +  + ρb 2 − p, 2 = ρv&2
∂x2  ∂x1 ∂x2 ∂x3   ∂x ∂x22 ∂x32 
  1

(λ * + µ * ) ∂  ∂v1 + ∂v2 + ∂v3  + µ *  ∂ v3 + ∂ v3 + ∂ v3  + ρb − p = ρv&
2 2 2

 ∂x3  ∂x1 ∂x2 ∂x3   ∂x 2 ∂x32 


3 ,3 3
  1 ∂x22

NOTE 2: We have proven in Problem 1.106 (Chapter 1) that the following is true:
r r r r r
∇ xr ∧ (∇ xr ∧ a) = ∇ xr (∇ xr ⋅ a) − ∇ 2xr a indicial
 → ilq  qjk ak , jl = a j , ji − ai , jj
Then, we can obtain
r r r r r r
∇ xr ⋅ (∇ xr v ) ≡ ∇ 2xr v = ∇ xr (∇ xr ⋅ v ) − ∇ xr ∧ (∇ xr ∧ v ) indicial
 → vi , jj = v j , ji − ilq  qjk vk , jl
with which the equation (11.1) can also be written as follows:
(λ* + µ * )v j , ji + µ *vi , jj + ρbi − p,i = ρv&i
⇒ (λ* + µ * )v j , ji + µ * (v j , ji − ilq  qjk vk , jl ) + ρbi − p, i = ρv&i
⇒ (λ* + 2 µ * )v j , ji − µ *ilq  qjk vk , jl + ρbi − p, i = ρv&i
and the equivalent in tensorial notation:
r r r r
(λ* + µ * )[∇ xr (∇ xr ⋅ v )] + µ * [∇ xr ⋅ (∇ xr v )] + ρb − ∇ xr p = ρv&
[ ]
r r r r r r r
⇒ (λ* + µ * )[∇ xr (∇ xr ⋅ v )] + µ * ∇ xr (∇ xr ⋅ v ) − ∇ xr ∧ (∇ xr ∧ v ) + ρb − ∇ xr p = ρv&
[ ]
r r r r r r
⇒ (λ* + 2 µ * )[∇ xr (∇ xr ⋅ v )] − µ * ∇ xr ∧ (∇ xr ∧ v ) + ρb − ∇ xr p = ρv&
In the Cartesian System we have:
r
v = vi eˆ i = v1eˆ 1 + v2 eˆ 2 + v3eˆ 3
r r r r  ∂v ∂v   ∂v ∂v   ∂v ∂v 
(∇ xr ∧ v ) ≡ rot (v ) = (rot (v ) )i eˆ i =  3 − 2 eˆ 1 +  1 − 3 eˆ 2 +  2 − 1 eˆ 3
1∂42
x2 ∂x3 
4 r43
4 1∂42
x3 ∂x1 
43 1∂42
x1 ∂x2 
4 r43
4
r
=(rot (u) )1 =(rot (u) )2 =(rot (u) )3
r r
r r  ∂ (rot (v ) ) ∂ (rot (v ) )  r r r r
r  ∂ (rot (v ) )1 ∂ (rot (v ) )3   ∂ (rot (v ) )2 ∂ (rot (v ) )1 
∇ ∧ (∇ ∧ v ) =  3
− 2 ˆ
e
 1 +  −  ˆ
e +
 2   − eˆ 3
 ∂x2 ∂x3   ∂x3 ∂x1   ∂x1 ∂x2 
r r
 ∂ (rot (v ) )3 ∂ (rot (v ) )2   ∂  ∂v2 − ∂v1  − ∂  ∂v1 − ∂v3 
 −   ∂x2  ∂x1 ∂x2  ∂x3  ∂x3 ∂x1 
 ∂x ∂x  
r  
2 3
r 
r r r  ∂ (rot (v ) )1 ∂ (rot (v ) )3   ∂  ∂v3 ∂v2  ∂  ∂v2 ∂v1  
[∇ x ∧ (∇ x ∧ v )]i = 
r r − =  −  −  −  
 ∂x3 ∂x1   ∂x3  ∂x2 ∂x3  ∂x1  ∂x1 ∂x2  
 ∂ (rot (vr ) )2 ∂ (rot (vr ) )1   
 −   ∂  ∂v1 − ∂v3  − ∂  ∂v3 − ∂v2  
 ∂x1 ∂x2   ∂x  ∂x   
 1  3 ∂x1  ∂x2  ∂x2 ∂x3  

NOTE 3: If we are dealing with heterogeneous material, we have:


σij = − pδ ij + λ*δ ijD kk + 2 µ *D ij
⇒ σij , j = (− pδ ij + λ*δ ijD kk + 2 µ *D ij ) , j = − p, jδ ij + (λ*D kk ), j δ ij + 2( µ *D ij ), j
⇒ σij , j = − p,i + (λ*D kk ) ,i + 2( µ *Dij ), j
Taking into account that 2D ij = vi , j + v j ,i and D kk = vk ,k , the above equation becomes:
[
σij , j = − p,i + (λ*D kk ),i + 2( µ *D ij ) , j = − p,i + (λ*vk ,k ),i + µ * (vi , j + v j ,i ) , j ]
whereby

University of Castilla-La Mancha Draft By: Eduardo W. V. Chaves (2016)


Ciudad Real – Spain
802 SOLVING PROBLEMS BY MEANS OF CONTINUUM MECHANICS

σij , j + ρbi = ρv&i ⇒ − p,i + (λ*vk , k ),i + [ µ * (vi , j + v j ,i )], j + ρbi = ρv&i (11.6)
Note that
r r
vk , k = Tr (∇ xr v ) = (∇ xr ⋅ v )
Dv&i ∂vi ∂vi ∂v ∂v ∂v ∂v
v&i = = + v j = i + i v1 + i v2 + i v3 , and its explicit components are:
Dt ∂t ∂x j ∂t ∂x1 ∂x2 ∂x3
 ∂v1 ∂v1 ∂v ∂v 
 + v1 + 1 v2 + 1 v3 
 ∂t ∂x1 ∂x2 ∂x3 
 ∂v ∂v ∂v ∂v 
ai = v&i =  2 + 2 v1 + 2 v2 + 2 v3 
 ∂t ∂x1 ∂x2 ∂x3 
 ∂v3 ∂v3 ∂v ∂v 
 + v1 + 3 v2 + 3 v3 
 ∂t ∂x1 ∂x2 ∂x3 

[ µ * (vi , j + v j ,i )], j = [ µ * (vi , j + v j ,i )]
∂x j
∂ ∂ ∂
= [ µ * (vi ,1 + v1,i )] + [ µ * (vi , 2 + v2,i )] + [ µ * (vi ,3 + v3,i )]
∂x1 ∂x2 ∂x3
 ∂ ∂ ∂ 
 [2 µ (v1,1 )] + [ µ * (v1, 2 + v2,1 )] + [ µ * (v1,3 + v3,1 )] 
*

 ∂x1 ∂x2 ∂x3 



 ∂ ∂ ∂ 
[ µ (vi , j + v j ,i )], j =  [ µ (v2,1 + v1, 2 )] +
* *
[2 µ (v2, 2 )] +
*
[ µ (v2,3 + v3, 2 )]
*

 ∂x1 ∂x2 ∂x3 


 ∂ ∂ ∂ 
 [ µ (v3,1 + v1,3 )] + [ µ (v3, 2 + v2,3 )] + [2 µ (v3,3 )] 
* * *
 ∂x1 ∂x2 ∂x3 
The three equations in (11.6), ( i = 1,2,3 ), are explicitly given by:

 ∂ * r ∂ ∂ ∂
 [λ (∇ xr ⋅ v )] + [2 µ * (v1,1 )] + [ µ * (v1, 2 + v2,1 )] + [ µ * (v1,3 + v3,1 )] + ρb1 − p,1 = ρv&1
 ∂x1 ∂x1 ∂x2 ∂x3
 ∂ * r ∂ ∂ ∂
 [λ (∇ xr ⋅ v )] + [ µ * (v2,1 + v1, 2 )] + [2 µ * (v2, 2 )] + [ µ * (v2,3 + v3, 2 )] + ρb2 − p, 2 = ρv&2
 ∂x2 ∂x1 ∂x2 ∂x3
 ∂ * r ∂ ∂ ∂
 [λ (∇ xr ⋅ v )] + [ µ * (v3,1 + v1,3 )] + [ µ * (v3, 2 + v2,3 )] + [2 µ *v3,3 ] + ρb3 − p,3 = ρv&3
 ∂x3 ∂x1 ∂x2 ∂x3
(11.7)
or

 ∂ * r ∂ ∂
 [λ (∇ xr ⋅ v ) + 2 µ (v1,1 )] + [ µ * (v1, 2 + v2,1 )] + [ µ * (v1,3 + v3,1 )] + ρb1 − p,1 = ρv&1
*

 1 ∂x ∂ x2 ∂x 3
 ∂ * r ∂ ∂
 [λ (∇ xr ⋅ v ) + 2 µ * (v2, 2 )] + [ µ * (v2,1 + v1, 2 )] + [ µ * (v2,3 + v3, 2 )] + ρb 2 − p, 2 = ρv&2
 ∂x2 ∂x1 ∂x3
 ∂ * r ∂ ∂
 [λ (∇ xr ⋅ v ) + 2 µ (v3,3 )] + [ µ * (v3,1 + v1,3 )] + [ µ * (v3, 2 + v2,3 )] + ρb3 − p,3 = ρv&3
*

 ∂x3 ∂x1 ∂x2


(11.8)

University of Castilla-La Mancha Draft By: Eduardo W. V. Chaves (2016)


Ciudad Real – Spain
11 INTRODUCTION TO FLUIDS 803

NOTE 4: To obtain the dynamic equations of motion in a rotating fluid on the sphere, we
have to consider the rotation of the Earth and the curvature, and the acceleration for a
fixed system was obtained in Problem 4.38:
r r r r r r r r
a f = a r + 2(ω ∧ v r ) + Ω T ⋅ v r + ω ∧ (ω ∧ x )
r r r r r r r r (11.9)
⇒ a r = a f − 2(ω ∧ v r ) − Ω T ⋅ v r − ω ∧ (ω ∧ x )
r r r r r r
where (2(ω ∧ v r )) is the Coriolis term, (Ω T ⋅ v r ) is the curvature term, ω ∧ (ω ∧ x ) is the
centrifugal term, and the components of these terms are:
2[ω3vr 3 cos(φ ) − ω3vr 2 sin(φ )]
r r  
2(ω ∧ v r ) i =  2[ω3 vr1 sin(φ )] 
 − 2[ω3vr1 cos(φ )] 
 
− vr1vr 2 tan(φ ) + vr1vr 3  (11.10)
T r 1  2 
(Ω ⋅ v r ) i = r  vr1 tan(φ ) + vr 2 vr 3 
x  
 − vr21 − vr22 
r r r r 2r
(ω ∧ (ω ∧ r )) i = − ω r
r
Then, if we want to consider these terms we replace a r into the Navier-Stokes-Duhem
equations.

Problem 11.3
Consider

ρv&i = ρbi − p,i + (λ* + µ * )v j , ji + µ *vi , jj Navier-Stokes-Duhem


r r r r equations of motion (11.11)
ρv& = ρb − ∇ xr p + (λ* + µ * )[∇ xr (∇ xr ⋅ v )] + µ *∇ 2xr v

Show the equation of vorticity:


r
∂ω r r µ* r r
+ 2∇ xr ⋅ [(ω ⊗ v ) skew ] − ∇ 2xr ω = 0 The equation of vorticity (11.12)
∂t ρ
r r r r r
where ω is vorticity vector and is given by ω ≡ rot (v ) ≡ (∇ xr ∧ v ) .
Solution:
Taking into account the material time derivative of the Eulerian velocity we obtain:
r r
Dvi ∂vi ∂v i ∂v r r ∂v ( x , t ) r r r r
v&i ≡ = + v j = i + vi , j v j a = v& = + ∇ xr v ( x , t ) ⋅ v ( x , t ) (11.13)
Dt ∂t ∂x j ∂t ∂t
The resulting components of the algebraic operation vi , j v j are the components of
r r
(∇ xr v ) ⋅ v , (see Problem 1.120 in Chapter 1), and it was shown that:
r r r r r 1 r r
(∇ xr v ) ⋅ v = (∇ xr ∧ v ) ∧ v + ∇ xr (v ⋅ v )
2
r r r 1
= (∇ xr ∧ v ) ∧ v + ∇ xr (v 2 ) (11.14)
2
r r 1
= ω ∧ v + ∇ xr (v 2 )
2

University of Castilla-La Mancha Draft By: Eduardo W. V. Chaves (2016)


Ciudad Real – Spain
804 SOLVING PROBLEMS BY MEANS OF CONTINUUM MECHANICS

Then, the Eulerian acceleration can also be represented by:


r r
r r r& ∂v ( x , t ) r r 1 r 2
a ( x, t ) = v = + ω ∧ v + ∇ x (v ) (11.15)
∂t 2
Taking into account (11.13) and (11.14), the equation in (11.11) becomes:
ρv&i = ρbi − p,i + (λ* + µ * )v j , ji + µ *vi , jj
r r r r
ρv& = ρb − ∇ xr p + (λ* + µ * )∇ xr (∇ xr ⋅ v ) + µ *∇ 2xr v
r r
 ∂v r r 1 r 2  r r
⇒ ρ  + ω ∧ v + ∇ x (v )  = ρb − ∇ xr p + (λ* + µ * )∇ xr (∇ xr ⋅ v ) + µ *∇ 2xr v (11.16)
 ∂t 2 
r r
∂v r r 1 r 2 1 (λ* + µ * ) r r r µ * 2r r r
⇒ + ω ∧ v + ∇ x (v ) − b + ∇ xr p − ∇ x (∇ x ⋅ v ) − ∇ x v = 0
∂t 2 ρ ρ ρ
Then we take the curl of the above equation:
r r 1
r  ∂v r r 1 (λ* + µ * ) r r r µ * 2r r  r
∇ x ∧  + ω ∧ v + ∇ xr (v 2 ) − b + ∇ xr p −
r ∇ x (∇ x ⋅ v ) − ∇ x v  = 0 (11.17)
 ∂t 2 ρ ρ ρ 
Note that the following relationships hold:
r r r r r r r
ƒ ∇ xr ∧ [∇ xr (v 2 )] = 0 , ∇ xr ∧ [∇ xr p ] = 0 , ∇ xr ∧ [∇ xr (∇ xr ⋅ v )] = 0 ;
r r r r r r r r r r r r r
∇ xr ∧ [(∇ xr ∧ v ) ∧ v ] = (∇ xr ⋅ v )(∇ xr ∧ v ) + [∇ xr (∇ xr ∧ v )] ⋅ v − (∇ xr v ) ⋅ (∇ xr ∧ v )
ƒ r r r r r r r r r ;
⇒ ∇ xr ∧ [ω ∧ v ] = (∇ xr ⋅ v )ω + [∇ xr ω] ⋅ v − (∇ xr v ) ⋅ ω
r r r r r r r r r
ƒ ∇ xr ∧ [∇ 2xr v ] = −∇ xr ∧ [∇ xr ∧ (∇ xr ∧ v )] = ∇ 2xr [∇ xr ∧ v ] = ∇ 2xr ω ;
r r
r  ∂ v  ∂ r r ∂ ω
ƒ ∇ xr ∧   = [∇ xr ∧ v ] = ;
 ∂t  ∂t ∂t
r
ƒ Considering that the field b is conservative, and considering that the curl of any
r r
r
conservative vector field is always zero we obtain ∇ xr ∧ b = 0 .
Considering the previous expression, the equation (11.17) becomes:
r
∂ω r r r r r r µ* r r
+ (∇ xr ⋅ v )ω + (∇ xr ω) ⋅ v − (∇ xr v ) ⋅ ω − ∇ 2xr ω = 0 (11.18)
∂t ρ
Note that the following relationships hold:
(v i ω j ), i = vi , i ω j + vi ω j , i ⇒ v i , i ω j = (vi ω j ), i −vi ω j , i
(11.19)
(v i ω j ), j = v i , j ω j + vi ω j , j ⇒ vi , j ω j = (v i ω j ), j −v i ω j , j = (vi ω j ), j
The above equations in tensorial notation are represented by:
r r r r r r
(∇ xr ⋅ v )ω = ∇ xr ⋅ [ω ⊗ v ] − (∇ xr ω) ⋅ v
r r r r r r r r (11.20)
(∇ xr v ) ⋅ ω = ∇ xr ⋅ [v ⊗ ω] − (∇ xr ⋅ ω)v = ∇ xr ⋅ [v ⊗ ω]
where we have applied the definition that the divergence of the curl of a vector is zero, i.e.
r r r
∇ xr ⋅ ω = ∇ xr ⋅ (∇ xr ∧ v ) = 0 . Taking into account (11.20), the equation (11.18) becomes:

University of Castilla-La Mancha Draft By: Eduardo W. V. Chaves (2016)


Ciudad Real – Spain
11 INTRODUCTION TO FLUIDS 805

r
∂ω r r r r r r µ * 2r r r
+ (∇ xr ⋅ v )ω + (∇ xr ω) ⋅ v − (∇ xr v ) ⋅ ω − ∇ ω=0
∂t ρ x
r
∂ω r r r r r r r r µ* r r
⇒ + ∇ xr ⋅ [ω ⊗ v ] − (∇ xr ω) ⋅ v + (∇ xr ω) ⋅ v − ∇ xr ⋅ [v ⊗ ω] − ∇ 2xr ω = 0
∂t ρ
r
∂ω r r r r µ 2 r r *
⇒ + ∇ xr ⋅ [ω ⊗ v ] − ∇ xr ⋅ [v ⊗ ω] − ∇r ω = 0 (11.21)
∂t ρ x
r
∂ω r r r r µ * 2r r r
⇒ + ∇ xr ⋅ [ω ⊗ v − v ⊗ ω] − ∇ ω=0
∂t ρ x
r
∂ω r r µ* r r
⇒ + 2∇ xr ⋅ [(ω ⊗ v ) skew ] − ∇ 2xr ω = 0
∂t ρ
With that we prove the equation of vorticity given by the equation in (11.12).

Problem 11.4
r
Let us consider a body immersed in a Newtonian fluid. Find the total traction force E
acting on the closed surface S which delimits the volume V . Consider that the bulk
viscosity coefficient to be zero.


n̂ t (n)

Figure 11.1

Solution: We know that the following holds:


ˆ
dE i = t i(n) dS
The total traction force is given by the following integral:

∫ ∫ ∫
ˆ
E i = t i(n ) dS = σ ij n̂ j dS = σ ij , j dV
S S V

where we have used the relationship σ ij nˆ j = t i(nˆ ) .


2
Then, if the bulk viscosity coefficient is zero, we have: κ * = 0 ⇒ λ * = − µ * (Stokes’
3
condition).

University of Castilla-La Mancha Draft By: Eduardo W. V. Chaves (2016)


Ciudad Real – Spain
806 SOLVING PROBLEMS BY MEANS OF CONTINUUM MECHANICS

Next, by considering the stress constitutive equation for Newtonian fluids, we obtain:
2 *  D 
σ ij = − pδ ij + λ * δ ij D kk + 2µ *D ij = − pδ ij − µ δ ij D kk + 2µ *D ij = − pδ ij + 2µ *  D ij − kk δ ij 
3 14423443
Dijdev

σ ij = − pδ ij + 2µ *D ijdev

Then


E i = ( − pδ ij + 2µ *D ijdev )nˆ j dS
S

and by applying the Gauss’ theorem, we obtain:

∫ ∫ ∫
Ei = ( − pδ ij + 2 µ *D ijdev ) , j dV = ( − p, jδ ij + 2 µ *D ijdev, j )dV = ( − p,i + 2 µ *D ijdev, j )dV
V V V

where we have considered that µ *,j = 0 j , i.e. µ * is a homogenous scalar field (homogenous
material). Then, the above equation in tensorial notation becomes:
r
E = [−∇ xr p + 2 µ * (∇ xr ⋅ Ddev )]dV
∫ (11.22)
V

Problem 11.5
Let us consider a fluid at rest which has the mass density ρ f . Prove Archimedes’
Principle: “Any body immersed in a fluid at rest experiences an upward buoyant force equal to the
weight of the volume fluid displaced by the body”.
If mass density in the body is equal to ρ s and the body force (per unit mass) is given by
b i = − gδ i 3 , obtain the resultant force and acceleration acting on the body.
Solution:
r
In Problem 11.4 we showed that E = ∫ [−∇ xr p + 2 µ * (∇ xr ⋅ Ddev )]dV . If the fluid is at rest

D dev = 0 holds, and the thermodynamic pressure is equal to the hydrostatic pressure, i.e.
p = p 0 whereby we have:

V

E = [ −∇ xr p0 ]dV (11.23)

The weight of the fluid volume displaced by the body is given by:
r r

= ρ f bdV
f
W (11.24)
V

Then, by applying the equilibrium equations we have:

University of Castilla-La Mancha Draft By: Eduardo W. V. Chaves (2016)


Ciudad Real – Spain
11 INTRODUCTION TO FLUIDS 807

r r
∇ xr ⋅ σ + ρ f b = 0 σ ij , j + ρ f b i = 0 i
r
⇒ ∇ xr ⋅ σ = −ρ f b ⇒ σ ij , j = −ρ f b i
r (11.25)
⇒ ∇ xr ⋅ (− p 0 1) = −ρ f b ⇒ (− p 0 δ ij ), j = −ρ f b i
r
⇒ ∇ xr p 0 = ρ f b ⇒ p 0,i = ρ f b i

Next, by considering both (11.23) and (11.24), we can conclude that:


r r r
∫ ∫
= ρ f bdV = ∇ xr p 0 dV = − E
f
W (11.26)
V V

which thereby proves Archimedes’ principle.

r
E

V -volume
p0 n̂

r
x3 Ws

x2

x1

Figure 11.2

Now, the body weight, with mass density ρ s , can be obtained as follows:
r r

W s = ρ s bdV
V

and the resultant force acting on the body is given by:


r r r r r r
∫ ∫
R = E + W s = − ρ f bdV + ρ s bdV = (ρ s − ρ f )bdV
V V

V

whose components are:


 0 
 
 

V
∫ V

Ri = ( ρ s − ρ f )bi dV = − g ( ρ s − ρ f )δ i 3dV = 

0 

V

 g ( ρ f − ρ s )dV 


University of Castilla-La Mancha Draft By: Eduardo W. V. Chaves (2016)


Ciudad Real – Spain
808 SOLVING PROBLEMS BY MEANS OF CONTINUUM MECHANICS

thereby verifying that: if the body has a mass density lower than fluid mass density, e.g. if
r r
the body is a gas, the body rises, i.e. ρ f > ρ s ⇒ R > 0 , and if not the body falls. Moreover,
r r
if we consider that R = m s a , where m s is the total mass of the submerged body, we can
obtain the acceleration of the body as:
ρs g (ρ f
−ρ s)
∫ g (ρ − ρ s )dV ∫ g (ρ −ρs) ∫ρ
f f s
dV dV
R3 ρs ρ s
g (ρ f
−ρs)
a3 = =V =V = V
=
ms ms ms ms ρs
NOTE: It is interesting to note that if the medium ( f ) is such that ρ f = 0 we have
a 3 = − g , i.e. the acceleration is independent of the mass. Here we have clearly seen, as did
Galileo, by means of a simple experiment, that a freely falling body was independent of the
mass. For example, on the moon where we can consider that the mass density of air is
equal to zero, two bodies with different masses in free fall, e.g. a feather and a hammer, will
have the same acceleration and will reach the moon surface at the same time.
Problem 11.6
Prove that the Cauchy deviatoric stress tensor σ dev is equal to τ dev , where
σ ij = − pδ ij + τ ij .

Solution
If we consider that σ kk = −3 p + τ kk we can obtain:
σ kk ( −3 p + τ kk ) τ
σ ijdev = σ ij − δ ij = − pδ ij + τij − δ ij = τ ij − kk δ ij = τ ijdev
3 3 3

Problem 11.7
Obtain the one-dimensional mass continuity equation for a non-viscous incompressible
fluid flow through a pipeline. Then, consider the volume V between the two arbitrary
cross sections A and B .

B
n̂ B
r
vB

n̂ A
V
r
vA
Figure 11.3
Solution:

In an incompressible medium, the mass density is independent of time ≡ ρ& = 0 .
Dt r
Moreover, here we can consider the mass continuity equation ρ& + ρ v k ,k = ρ (∇ xr ⋅ v ) = 0 ,
r
where ∇ xr ⋅ v = 0 or v k , k = 0 holds. Then, by considering the entire volume we obtain:

University of Castilla-La Mancha Draft By: Eduardo W. V. Chaves (2016)


Ciudad Real – Spain
11 INTRODUCTION TO FLUIDS 809

r
∫∇ r
x ⋅ v dV =0 ∫v k , k dV =0 (11.27)
V V

and by applying the divergence theorem (Gauss’ theorem) we can obtain:


r
∫ v ⋅ n̂ dS = 0
S
∫v
S
ˆ k dS
kn =0 (11.28)

Thus:
r r
∫v A ⋅ nˆ A dS + ∫v B ⋅ nˆ B dS = 0
SA SB

Next, the velocities at the cross sections SA and SB can be expressed as follows:
r r
v A = −v A nˆ A ; v B = v B nˆ B
and by substituting the velocity into the integral we can obtain:
− v A nˆ A ⋅ nˆ A dS + v B nˆ B ⋅ nˆ B dS = 0
∫ ∫
SA SB

v AS A = vB S B

Problem 11.8
The velocity field of a gas in motion through a pipeline, whose prismatic axis is x 2 , is
defined by its components as follows:
v1 = 0 ; v 2 = 0.02 x 2 + 0.05 ; v3 = 0
kg
At x 2 = 0 the mass density ρ is equal to 1.5 . Find ρ at x 2 = 5m .
m3
Solution:
r r r
Note that the velocity field is stationary, i.e. v = v ( x ) . From the mass continuity equation
we can obtain:
∂ρ r r
+ ∇ xr ⋅ (ρ v ) = 0 ⇒ ∇ xr ⋅ (ρ v ) = 0
∂t
r
Then, we can conclude that ρ v is constant along x 2 -direction, so:
(ρ v 2 ) x2 = 0
= (ρ v 2 ) x2 =5

v 2 ( x 2 = 0) = 0.02 × 0 + 0.05 = 0.05 and v 2 ( x 2 = 5) = 0.02 × 5 + 0.05 = 0.15 , thus:


(ρ v 2 ) x2 = 0
= (ρ v 2 ) x2 =5

kg
1.5 × 0.05 = ρ 0.15 ⇒ ρ ( x 2 = 5) = 0.5
m3
Alternative solution:
r
∇ xr ⋅ (ρ v ) = 0 indicial
 → ( ρ v i ) ,i = ρ ,i v i + ρ v i ,i = 0

∂ρ ∂v  ∂ρ ∂ρ ∂ρ   ∂v ∂v ∂v 
vi + ρ i =  v1 + v2 + v3  + ρ  1 + 2 + 3  = 0
∂x i ∂xi  ∂x1 ∂x 2 ∂x 3   ∂x1 ∂x 2 ∂x 3 

University of Castilla-La Mancha Draft By: Eduardo W. V. Chaves (2016)


Ciudad Real – Spain
810 SOLVING PROBLEMS BY MEANS OF CONTINUUM MECHANICS

Thus:
 ∂ρ  ∂ρ 0.02
 (0.02 x2 + 0.05)  + ρ (0.02 ) = 0 ⇒ =− ∂x
 ∂x2  ρ (0.02 x2 + 0.05) 2
By integrating the above equation, we obtain:
 C 
Lnρ = Ln(0.02 x2 + 0.05) + LnC ⇒ Lnρ = Ln 
 (0.02 x2 + 0.05) 
C
⇒ρ=
(0.02 x2 + 0.05)
The constant of integration can be obtained by applying the boundary condition, i.e. at
x 2 = 0 ⇒ ρ = 1.5 , with that we obtain C = 0.075 :
0.075 x2 =5 0.075 kg
ρ=  → ρ= = 0.5 3
(0.02 x 2 + 0.05) (0.02 × 5 + 0.05) m

Problem 11.9
The Cauchy stress tensor components at one point of a Newtonian fluid, in which the bulk
viscosity coefficient is zero, are given by:
 − 6 2 − 1
σ ij =  2 − 9 4  Pa
 − 1 4 − 3

Obtain the viscous stress tensor components.


Solution:
In the case in which the bulk viscosity coefficient is zero (Stokes’ condition) we have
p = p = p 0 , and, in addition, we can obtain:

σ ij = − pδ ij + τij ; κ* = λ * +
2 *
µ =0 ; σii = −3 p ; p=−
σii
=−
(− 6 − 9 − 3) = 6
3 3 3
Then:
 − 6 2 − 1 6 0 0   0 2 − 1
    
τij = σ ij + pδ ij =  2 − 9 4  + 0 6 0  =  2 − 3 4  Pa
 − 1 4 − 3 0 0 6   − 1 4 3 

Problem 11.10
σ kk
Determine the conditions needed for mean normal pressure p = − = −σ m to be equal
3
to thermodynamic pressure p for a Newtonian fluid.
Solution:
It was deduced that:

University of Castilla-La Mancha Draft By: Eduardo W. V. Chaves (2016)


Ciudad Real – Spain
11 INTRODUCTION TO FLUIDS 811

1 σ kk
σ ijdev = 2µ *D ijdev ; σ kk = − p + k *D ii ; = − p = − p + κ *D kk
3 {3
−p

Thus, p = p holds when the following is satisfied:


D ii = 0 2
κ* = 0 or  or λ* = − µ *
 Tr (D) = 0 3

Problem 11.11
p
A baratropic perfect fluid has as equation of state ρ = ρ 0 + , where k is constant. Obtain
k
the pressure field for a quasi-static regime (zero acceleration), under the action of the
gravitational field bi = [0 0 − g]T .
Solution:
The constitutive equation in stress for a perfect fluid is given by:
σ = − p1
The equations of motion become:
r r
∇ xr ⋅ σ + ρb = ρv& indicial
 → σij , j + ρbi = ρv&i = 0i
{
= 0i
(− pδ ij ), j + ρbi = 0i (11.29)
− p, j δ ij + ρbi = 0i
r r
− ∇ xr p + ρb = 0 tensorial
←   − p ,i + ρ b i = 0 i
Considering the body force vector bi = [0 0 − g ] we obtain:
 ∂p ∂p
(i = 1) ⇒ − + ρ b1 = 0 ⇒ = 0 ⇒ p = p ( x1 , x 2 , x3 )
 ∂x1 ∂x1
 ∂p ∂p
− p, i +ρ b i = 0 i ⇒ (i = 2) ⇒ − + ρb2 = 0 ⇒ = 0 ⇒ p = p ( x1 , x 2 , x3 ) (11.30)
 ∂ x 2 ∂ x2
 ∂p dp ( x3 )
(i = 3) ⇒ − ∂x + ρ b 3 = 0 ⇒ dx + ρ g = 0
 3 3

With that we can conclude that the pressure field is only a function of the coordinate x 3 ,
i.e. p = p( x3 ) .
By the fact we are dealing with a barotropic fluid, this implies that the mass density is only
a function of pressure ρ = ρ ( p ) . This relationship is precisely the equation of state of the
problem statement:
p
ρ = ρ ( p) ⇒ ρ =ρ0 +
k
Then:
dp( x3 ) dp ( x3 )  p dp ( x3 ) g
+ ρg = 0 ⇒ +  ρ 0 + g = 0 ⇒ + p = −ρ 0g (11.31)
dx3 dx3  k dx3 k
The solution of the above differential equation is the sum of a particular solution and a
homogeneous solution:

University of Castilla-La Mancha Draft By: Eduardo W. V. Chaves (2016)


Ciudad Real – Spain
812 SOLVING PROBLEMS BY MEANS OF CONTINUUM MECHANICS

−g
dp( x 3 ) g ( x3 )
Homogeneous solution: + p = 0 ⇒ p = Cexp k
dx3 k
Particular solution: p = −kρ 0
Thus:
−g
( x )
p = C exp k 3 − kρ 0

Problem 11.12
A perfect gas is an ideal and incompressible fluid in which in the absence of heat sources
the pressure is proportional to ρ γ (barotropic motion), where γ is a constant and γ > 1 .
Show that when r = 0 (no internal heat source), the specific internal energy for a ideal gas
is given by:
1 p
u= + constant
( γ − 1) ρ
Solution:
For the proposed problem, the energy equation becomes:
r
ρ u& = σ : D − ∇ xr ⋅ q + ρr = σ : D ⇒ ρ u& − σ : D = 0
For a perfect gas the stress tensor is a spherical tensor and is given by:
σ ( p ) = − p1
where p is the thermodynamic pressure. Then, the energy equation becomes:
ρ u& − σ : D = 0
⇒ ρ u& + p1 : D = 0
⇒ ρ u& + pTr (D) = 0
r
⇒ ρ u& + p (∇ xr ⋅ v ) = 0
For a barotropic motion, the specific internal energy is a function of the mass density,
u = u ( ρ ) , thus:
r ∂u r
ρ u& + p(∇ xr ⋅ v ) = 0 ⇒ ρ ρ& + p(∇ xr ⋅ v ) = 0
∂ρ
Dρ r r
Considering the mass continuity equation + ρ (∇ xr ⋅ v ) = 0 ⇒ ρ& = − ρ (∇ xr ⋅ v ) , the energy
Dt
equation becomes:
∂u r
ρ ρ& + p(∇ xr ⋅ v ) = 0
∂ρ
∂u r r
⇒ −ρ ρ (∇ xr ⋅ v ) + p(∇ xr ⋅ v ) = 0
∂ρ
 ∂u  r
⇒  − ρ 2 + p ∇ xr ⋅ v = 0
 ∂ρ  12 3
≠0

with that the following holds:

University of Castilla-La Mancha Draft By: Eduardo W. V. Chaves (2016)


Ciudad Real – Spain
11 INTRODUCTION TO FLUIDS 813

∂u ∂u p
− ρ2 + p=0 ⇒ = 2
∂ρ ∂ρ ρ

As pressure is proportional to ρ γ , we can state that p = p (ρ ) = kρ γ , where k is a


proportionality constant, then:
∂u ∂u kρ γ
− ρ2 + p=0 ⇒ = 2 = kρ ( γ − 2 )
∂ρ ∂ρ ρ
By integrating the above equation we obtain:
k ργ
u= + constant
( γ − 1) ρ
1 p
= + constant
( γ − 1) ρ

Problem 11.13
r
A fluid moves with velocity v around a sphere of radius R , where the velocity
components in spherical coordinates ( r , θ, φ ) are given by:
 R 3 3R   R 3 3R 
vr = c 3 − + 1 cos(θ) ; vθ = c 3 + − 1 sin(θ) ; vφ = 0 (11.32)
 2r 2r   4r 4r 
where c is a positive constant.
Check whether we are dealing with an isochoric motion or not.
r
Note: Given a vector u , the divergence of this vector in spherical coordinates is:
r r ∂u 1 ∂u θ 1 ∂u φ cot(θ) 2
div u ≡ ∇ xr ⋅ u = r + + + uθ + u r
∂r r ∂θ r sin(θ) ∂φ r r
Solution:
r
To demonstrate that a motion is isochoric, we must show that ∇ xr ⋅ v = 0 .
From the velocity field we can obtain the following derivatives:
∂v r ∂   R 3 3R    − 3R 3 3R 
= c 3 − + 1 cos(θ) = c + 2 + 1 cos(θ)
∂r ∂r   2r 2r    2r
4
2r 

∂v θ ∂   R 3 3R    R 3 3R 
= c 3 + − 1 sin(θ) = c 3 + − 1 cos(θ)
∂θ ∂θ   4r 4r    4r 4r 
With that it is possible to obtain the divergence of the velocity field:
r ∂v 1 ∂v θ 1 ∂v φ cot(θ) 2
∇ xr ⋅ v = r + + + vθ + v r
∂φ
∂r r ∂θ r sin(θ) { r r
=0

 − 3R3
3R  1  R 3 3R 
= c + + 1  cos(θ) + c
 + − 1 cos(θ) +
 2r
4
2r 2  r  4r 3 4r 
cos(θ) 1   R 3 3R   2   R 3 3R  
+ c 
  3 + − 1  sin( θ)  +  c  − + 1  cos(θ)
sin(θ) r   4r 4r   3 2r 
  r   2r  

University of Castilla-La Mancha Draft By: Eduardo W. V. Chaves (2016)


Ciudad Real – Spain
814 SOLVING PROBLEMS BY MEANS OF CONTINUUM MECHANICS

By simplifying the above equation we can conclude that:


r
∇ xr ⋅ v = 0
So, we are dealing with an isochoric motion.

Problem 11.14
A barotropic fluid flows through a pipeline as shown in Figure 11.4, and said fluid has as
equation of state:
 ρ 
p = β Ln  ; (β and ρ 0 are constants )
ρ0 
where p is pressure, and ρ is the mass density.
Calculate, in steady state, the output pressure p ( 2) in terms of variables presented in Figure
11.4.

p -pressure;
v -velocity;
S -cross section area
p (1)
v (1)
S (1) p( 2)
v( 2)
S (2)

Figure 11.4
Solution:
According to the principle of conservation of mass we have:
D
Dt V ∫
ρ dV = 0
r
and given a property Φ ( x , t ) , it fulfills that:
r
D r  DΦ ( x , t ) r DdV 
Dt V∫Φ ( x , t )dV = 
V
∫Dt
dV + Φ ( x , t )
Dt 

r
 DΦ ( x , t ) r r r 
=  ∫ dV + Φ ( x , t )∇ xr ⋅ v ( x, t )dV 
V
Dt 
r
 DΦ ( x , t ) r r r 
=  ∫ + Φ ( x , t )∇ xr ⋅ v ( x, t )  dV
V
Dt 
r
 ∂Φ ( x , t ) r r r r r r 
=  ∫ + ∇ xr Φ ( x , t ) ⋅ v ( x , t ) + Φ ( x , t )∇ xr ⋅ v ( x , t )  dV
V
∂t 

University of Castilla-La Mancha Draft By: Eduardo W. V. Chaves (2016)


Ciudad Real – Spain
11 INTRODUCTION TO FLUIDS 815

r
D r  ∂Φ ( x , t ) r r r r r r 

Φ ( x , t )dV =  ∫ + ∇ xr Φ ( x , t ) ⋅ v ( x , t ) + Φ ( x , t )∇ xr ⋅ v ( x , t )  dV
Dt V V
∂t 
r
 ∂Φ ( x , t ) r r r 
=  ∫ + ∇ xr ⋅ (Φ ( x , t )v ( x , t ) ) dV
V
∂t 
r (11.33)
∂Φ ( x , t ) r r r
= ∫ dV + ∇ xr ⋅ [Φ ( x , t )v ( x , t )]dV

V
∂t V
r
∂Φ ( x , t ) r r r
= ∫ dV + [Φ ( x , t )v ( x , t )] ⋅ nˆ dS = 0

V
∂t S
r r
by denoting Φ ( x , t ) = ρ ( x , t ) the above equation becomes:
r
D r ∂ρ ( x , t ) r r r

ρ ( x , t )dV = dV + [ρ ( x , t )v ( x , t )] ⋅ nˆ dS = 0
∫ ∫
Dt V V
∂t S
r
∂ρ ( x , t )
By applying the steady state condition, i.e. = 0 , we obtain:
∂t
[ρ ( xr )vr ( xr )]⋅ nˆ dS = 0 ⇒
∫ [ρ ( xr )vr ( xr )]⋅ nˆ dS + [ρ ( xr )vr ( xr )]⋅ nˆ dS = 0
∫ ∫
S S (1) S( 2)

⇒ ∫− ρ
S (1)
(1) v(1) dS + ∫ρ
S(2)
( 2 ) v( 2 ) dS =0 ⇒ − ρ (1) v(1) S (1) + ρ ( 2 )v( 2 ) S( 2) = 0

Thus:

ρ (1) v (1) S (1) = ρ ( 2) v ( 2) S ( 2) (11.34)

r r r kg
Remember that q = ρ v is the mass flux, and the SI unit is [q] = .
m2 s
By means of the equation of state we can obtain an expression for mass density:
 p  p
 ρ  p  ρ     ρ   
p = β Ln  ⇒ = Ln  ⇒ exp  β  =   ⇒ ρ ( x ) = ρ 0 exp  β 
ρ0  β ρ0  ρ0 
Then:
 p (1 )   p( 2 ) 
   
 β   β 
ρ (1)v(1) S(1) = ρ ( 2)v( 2) S( 2) ⇒ ρ 0exp  
v(1) S(1) = ρ 0exp  
v( 2 ) S( 2 )
 p ( 2 ) − p ( 1) 



 v(1) S(1) p( 2 ) − p(1)  v(1) S(1) 
= Ln 
β
⇒ exp  
= ⇒ (11.35)
v( 2 ) S( 2) β  v( 2 ) S( 2 ) 
 
 v(1) S(1)   v(1) S(1) 
⇒ p( 2 ) − p(1) = β Ln  ⇒ p( 2 ) = p(1) + β Ln 
 v( 2 ) S( 2 )   v( 2 ) S( 2 ) 
   
NOTE: The volumetric flow rate, (also known as volume flow rate), often represented by
Q , is the specific total flow, i.e.:
r r r r
q ⋅ dS ρ v ⋅ dS r r  m3 
Q= ∫ =∫ = ∫ v ⋅ dS Volumetric flow rate   (11.36)
S
ρ S
ρ S  s 

University of Castilla-La Mancha Draft By: Eduardo W. V. Chaves (2016)


Ciudad Real – Spain
816 SOLVING PROBLEMS BY MEANS OF CONTINUUM MECHANICS

 q r r
⋅ dS = kg m3 m 2 = m3 . In this example, we have obtained
We check the SI unit [Q] =   ∫
 S ρ  m s kg
2
s
ρ (1) v (1) S (1) = ρ ( 2) v ( 2) S ( 2) , which can be rewritten as:
ρ (1) v(1) S(1) = ρ ( 2) v( 2) S( 2) ⇒ ρ (1)Q(1) = ρ ( 2)Q( 2)
For the particular case of an incompressible medium we have ρ (1) = ρ ( 2) , then:
v (1) S (1) = v ( 2) S ( 2) ⇒ Q(1) = Q( 2 ) (see Problem 11.7)

Problem 11.15
Consider the water flow described in Figure 11.5 in which the jet is deflected by the curved
vane. Obtain the total force applied to the curved vane.
Hypotheses (approximations): Consider that we are dealing with: a) the incompressible
fluid; b) a steady flow; c) no body forces; d) the atmosphere pressure can be discarded.

x2

x2′ v( 2 ) , S ( 2 ) x1′

ê′2 ê1′

n̂ θ
ê 2
n̂ curved vane


v(1) , S (1) ê1 x1

Figure 11.5: Jet deflected by the curved vane.


Solution:
By considering the homogeneous mass density field we can conclude
r
Dρ ∂ρ ∂ρ ∂x ∂ρ ∂ρ r Dρ ∂ρ
= + r⋅ = + r ⋅v Homogeneou
 s mass density field
   → =
Dt ∂t ∂x ∂t ∂t ∂x
{ Dt ∂t
r
=0
( Homogeneous)
r
∂v r
Since we are dealing with steady flow the following is true = 0 , and in addition for an
∂t

incompressible fluid the mass density it fulfills = 0 . Then, for this problem the
Dt
r
∂ ( ρv ) r
equation = 0 holds.
∂t
Recall from the textbook (Chapter 5 – Chaves (2013)) that the “Principle of Conservation
of Linear Momentum” states

University of Castilla-La Mancha Draft By: Eduardo W. V. Chaves (2016)


Ciudad Real – Spain
11 INTRODUCTION TO FLUIDS 817

r r r D r


F = t * dS + ρ b dV =
V
Dt V ∫
ρ v dV ∫ (11.37)
142r 4
3
=0
r
where F is the total force of the system. The above material time derivative can be
expressed as follows, (see equation (11.33)):
r
D r ∂( ρ v ) r r r r

ρ v dV = ∫dV + ( ρ v )v ⋅ nˆ dS = ( ρ v )v ⋅ nˆ dS
∫ ∫
Dt V V
∂t (11.38)
142r 43 S S
=0

Note that we are adopting the Eulerian formulation and the control volume and surface
control can be appreciated in Figure 11.6.

S = S (1) + S ( 2 ) + S (3) + S ( 4 ) - Control surface


x2
x2′ S ( 2) x1′
nˆ = eˆ 1′
ê′2 ê1′
S ( 3) θ

ê 2
Control volume S ( 4)
nˆ = −eˆ 1

S (1)
ê1 x1

Figure 11.6: Control surface and control volume.


Then the equation (11.37) can be rewritten as follows
r r D r r r

F = t * dS = ρ v dV = ( ρ v )v ⋅ nˆ dS
∫ ∫

Dt V S
r r r r r r r r r (11.39)
⇒ F = ( ρ v )v ⋅ nˆ dS + ( ρ v )v ⋅ nˆ dS + ( ρ v )v ⋅ nˆ dS +
∫ ∫ ∫ ∫ ( ρ v )v ⋅ nˆ dS
S ( 1) S( 2 ) S( 3 ) S
1442r 44
3 1
(4)
442r 44
3
=0 =0

Note that on the control surface S (3) and S ( 4) the velocity is always perpendicular to n̂ ,
r r
then v ⋅ nˆ = 0 . On surface S (1) v = v(1) ê1 and nˆ = −eˆ 1 with that
we have
r r
v ⋅ nˆ = (v(1) eˆ 1 ) ⋅ (−eˆ 1 ) = −v(1) , on the surface S ( 2) we have v = v( 2) ê1′ and nˆ = eˆ 1′ , with that
r r
v ⋅ nˆ = (v( 2) eˆ 1′ ) ⋅ (eˆ 1′ ) = v( 2) . Note also that the transformation law from the system x to the
r
system x ′ is given by
 eˆ 1′   cos θ sin θ   eˆ 1   cos θ eˆ 1 + sin θ eˆ 2 
ˆ  =   = 
e′2   − sin θ cos θ  eˆ 2  − sin θ eˆ 1 + cos θ eˆ 2 

Then, the velocity on the control surface S ( 2) can also be expressed as follows

University of Castilla-La Mancha Draft By: Eduardo W. V. Chaves (2016)


Ciudad Real – Spain
818 SOLVING PROBLEMS BY MEANS OF CONTINUUM MECHANICS

r
v = v( 2 )eˆ 1′ = v( 2 ) (cosθ eˆ 1 + sin θ eˆ 2 )

By taking into account all the previous considerations the equation (11.39) becomes:
r r r r r
F= ∫ ( ρ v )v ⋅ nˆ dS + ∫ ( ρ v )v ⋅ nˆ dS
S (1 ) S( 2 )
r
⇒F = ∫ − (ρ v
S ( 1)
ˆ
(1) e1 )v(1) dS ∫
+ [ ρ v( 2 ) (cosθ eˆ 1 + sin θ eˆ 2 )]v( 2 ) dS
S( 2 )
(11.40)
r
⇒ F = − ρ v(1) v(1) S (1) eˆ 1 + ρ v( 2 ) v( 2 ) cosθ S ( 2 ) eˆ 1 + ρ v( 2 ) v( 2 ) sin θ S ( 2 ) eˆ 2
r
⇒ F = ρ [v(22 ) cosθ S ( 2) − v(21) S (1) ]eˆ 1 + ρ v(22 ) sin θ S ( 2 ) eˆ 2

For the particular case when v(1) = v( 2) = v0 , S (1) = S ( 2) = S 0 , the above equation becomes
r
F = ρ [v(22 ) cosθ S ( 2) − v(21) S (1) ]eˆ 1 + ρ v(22 ) sin θ S ( 2 ) eˆ 2
r  Fx = ρ v02 S 0 [cosθ − 1] (11.41)
⇒ F = ρ v02 S 0 [cosθ − 1]eˆ 1 + ρ v02 S 0 sin θ eˆ 2 ⇒ 
1

 x2 = ρ v0 S 0 sin θ
2
F
r r
And the reaction force in the curved vane is R = − F .

Problem 11.16
Consider the water flow described in Figure 11.7. Obtain the total force applied to the
plate.
Hypotheses (approximations): Consider that we are dealing with: a) the incompressible
fluid; b) a steady flow; c) no body forces; d) the atmosphere pressure can be discarded.

ê′2′

v0 , S( 2 ) ê1′′
velocities
r
S (1) ⇒ v = v0 eˆ 1′ x2
r
S ( 2) ⇒ v = v0 eˆ ′2′
r
S (3) ⇒ v = −v0 eˆ 1′′′
ê 2
θ

ê1 x1

ê′2 ê1′′′
ê1′

v0 , S(1)
ê′2′′
v0 , S(3)

Figure 11.7

University of Castilla-La Mancha Draft By: Eduardo W. V. Chaves (2016)


Ciudad Real – Spain
11 INTRODUCTION TO FLUIDS 819

Solution:
Note that this problem is similar to the previous problem in which we have established that
the total force, (see equation (11.39)), is given by
r r D r r r

F = t * dS = ρ v dV = ( ρ v )v ⋅ nˆ dS
∫ ∫ (11.42)
S
Dt V S
σ

According to Figure 11.8 we can obtain:


r r
Transformation law from the system x to the system x ′ , (angle θ ):
 eˆ 1′   cosθ
sin θ   eˆ 1   cosθ eˆ 1 + sin θ eˆ 2 
ˆ  =   = 
cosθ  eˆ 2  − sin θ eˆ 1 + cosθ eˆ 2 
e′2  − sin θ
r r
Transformation law from the system x to the system x ′′ , (angle θ = 0º ):
 eˆ 1′′   cos 0º sin 0º   eˆ 1   eˆ 1 
ˆ  =   = 
e′2′  − sin 0º cos 0º  eˆ 2   eˆ 2 
r r
Transformation law from the system x to the system x ′′′ , (angle θ = 90º ):
 eˆ 1′′′  cos 90º sin 90º   eˆ 1   eˆ 2 
ˆ  =   = 
e′2′′ − sin 90º cos 90º  eˆ 2   − eˆ 1 

ê′2′
Control surface v0 , S( 2 )
S = S (1) + S ( 2) + S (3) + S ( 4) + S (5) + S (6 ) ê1′′

x2

ê 2
θ
S( 4)

ê1 x1

S( 6 )

ê′2 ê1′′′
ê1′ n̂ n̂
v0 , S(1) S( 5 )

n̂ ê′2′′ S(3) , v0

Figure 11.8
Then, on the control surface S (1) :
r
v = v0 eˆ 1′ = −v0 (cosθ eˆ 1 + sin θ eˆ 2 ) r
S (1) ⇒ ⇒ v ⋅ nˆ = v0
nˆ = −eˆ 1′ = −(cosθ eˆ 1 + sin θ eˆ 2 )

University of Castilla-La Mancha Draft By: Eduardo W. V. Chaves (2016)


Ciudad Real – Spain
820 SOLVING PROBLEMS BY MEANS OF CONTINUUM MECHANICS

On the control surface S ( 2) :


r
v = v0 eˆ ′2′ = v0 eˆ 2 r
S ( 2) ⇒ ⇒ v ⋅ nˆ = v0
nˆ = eˆ ′2′ = eˆ 2

On the control surface S (3) :


r
v = −v0 eˆ 1′′′ = −v0 eˆ 2 r
S ( 3) ⇒  ⇒ v ⋅ nˆ = v0
nˆ = −eˆ 1′′′ = −eˆ 2
r
Note that for the control surfaces S ( 4) , S (5) and S (6) the equation v ⋅ nˆ = 0 holds, since the
outward normal is always perpendicular to the velocity. Then, the equation in (11.42)
becomes
r r r
F = ( ρ v )v ⋅ nˆ dS

S
r r r r r r r r r
⇒F = ∫ ( ρ v )v ⋅ nˆ dS + ∫ ( ρ v )v ⋅ nˆ dS + ∫ ( ρ v )v ⋅ nˆ dS +
S ( 1) S( 2 ) S( 3) S +S
∫ ( ρ v )v ⋅ nˆ dS
+S
1
(4)
4(442
5) (6)
r
444
3
=0
r
⇒F = ∫ ρ [−v (cosθ eˆ
S ( 1)
0 1 + sin θ eˆ 2 )]v0 dS + ∫ ρ (v
S( 2 )
0 eˆ 2 )v0 dS + ∫ ρ ( −v
S( 3 )
0 eˆ 2 )v0 dS

r
⇒ F = − ρv02 (cosθ eˆ 1 + sin θ eˆ 2 ) S (1) + ρv02 S ( 2 ) eˆ 2 − ρv02 S (3) eˆ 2
r
⇒ F = − ρv02 S (1) cosθ eˆ 1 + ρv02 [− S (1) sin θ + S ( 2 ) − S (3) ] eˆ 2

Problem 11.17
Starting from the Navier-Stokes-Duhem equations of motion, obtain the Bernoulli’s
equation:

p v2
gh + + = constant Bernoulli’s equation (11.43)
ρ 2

Hypothesis: incompressible and non-viscous fluid. Consider that the velocity field is steady
and irrotational.
Solution:
r
Considering an incompressible medium (∇ xr ⋅ v ) = 0 , and a non-viscous fluid (λ * = µ * = 0) ,
the Navier-Stokes-Duhem equations of motion become:
ρv&i = ρbi − p,i + (λ* + µ * )v j , ji + µ *vi , jj
r r r r
ρv& = ρb − ∇ xr p + (λ* + µ * )∇ xr (∇ xr ⋅ v ) + µ *∇ 2xr v (11.44)
r r
⇒ ρv& = ρb − ∇ xr p
r r
Note that the ρ v& = ρ b − ∇ xr p are the Euler equations of motion. The material time derivative
of the velocity, (see equation (11.15)), becomes:
r
r& ∂v r r 1 r 2 1
v= + ω ∧ v + ∇ x (v ) = ∇ xr (v 2 )
∂t 2 2

University of Castilla-La Mancha Draft By: Eduardo W. V. Chaves (2016)


Ciudad Real – Spain
11 INTRODUCTION TO FLUIDS 821

r
∂v r
where we have considered that the steady velocity field = 0 , and irrotational
∂t
r r r r r
∇ xr ∧ v = rot v = ω = 0 . With that the equation (11.44) can be rewritten as follows:
ρ r 1 r 2 r 1 r
∇ xr (v 2 ) = ρ b − ∇ xr p ⇒ ∇ x (v ) − b + ∇ xr p = 0 (11.45)
2 2 ρ
r
Considering that the body force (conservative field) can be represented by b = −∇ xr ϕ ,
where ϕ is a potential, and also by considering that the mass density field is homogeneous
 p 1
the relationship ∇ xr   = ∇ xr p holds. Then, the equation in (11.45) becomes:
ρ  ρ
 p v2  p v2
∇ xr  ϕ + +  = 0i
 ⇒ ϕ+ + = constant (11.46)
 ρ 2  ρ 2
Considering that the potential can be represented by ϕ = gh , where g is the acceleration
of gravity and h is the piezometric height, we obtain the Bernoulli’s equation:
p v2
gh + + = constant
ρ 2

 v 2   p  N m3 Nm J m 2
We check the SI unit: [ gh] =  = = 2 = = = , which is the unit of
 2   ρ  m kg kg kg s 2
specific energy, i.e. unit of energy per unit mass.
Note that the Bernoulli’s equation is the application of the conservation of energy, i.e. in
the system there is no energy dissipation, (Figure 11.9).

constant energy
energy at A energy at B

p
ρ A
p
ρ
v2 B

2 A
v2
gh A A 2 B

h B gh B

Figure 11.9

University of Castilla-La Mancha Draft By: Eduardo W. V. Chaves (2016)


Ciudad Real – Spain
822 SOLVING PROBLEMS BY MEANS OF CONTINUUM MECHANICS

Problem 11.18
Let us consider a perfect and incompressible fluid in steady regime that is flowing through
the channel as shown in Figure 11.10. Obtain the value of H .
Hypothesis: No energy dissipation is considered.

v (1) = 1m / s

h(1) v( 2 ) = 2 m / s

H
h( 2)

Figure 11.10

Solution:
The mass continuity equation:
v (1) 1
v (1) h(1) = v ( 2 ) h( 2) ⇒ h( 2 ) = h(1) = h(1)
v( 2) 2
The Bernoulli’s equation:
v (21) 
( H + h(1) ) + 0 + 
2g  v (22 ) − v (21) − h(1) 3
 ⇒ H = h( 2 ) − h(1) + ⇒ H= +
v (22)  2g 2 2g
h( 2 ) + 0 + 
2g 

Problem 11.19
A large diameter circular tank is filled with water. The water pours through a small orifice
located at a height H below the water level of the reservoir, (see Figure 11.11). If the
volumetric flow rate is Q , obtain the orifice diameter D .
Hypothesis: Consider that H does not vary with time (steady state). Consider that in the
section BB ′ , the flow pressure is equal to atmospheric pressure, (see Figure 11.11).

University of Castilla-La Mancha Draft By: Eduardo W. V. Chaves (2016)


Ciudad Real – Spain
11 INTRODUCTION TO FLUIDS 823

H
B

( p − patm ) patm

C′ B′

C
B Pressure Pressure

Cross section CC ′ Cross section BB′

Figure 11.11

Solution:
The water can be considered as an incompressible perfect fluid. For this problem, we will
consider the Bernoulli’s equation:
p v2
z+ + = const.
ρ g 2g
where it fulfills that:
p atm 
Point A ⇒ H+ +0 
ρg 
2 
⇒ v ( B ) = 2 gH
p v( B ) 
Point B ⇒ 0 + atm +
ρg 2 g 
Considering that the volumetric flow rate is given by Q = v ( B ) S ( B ) , we can conclude that:

πD 2 4Q
Q = v ( B ) S ( B ) = 2 gH ⇒ D=
4 π 2 gH

University of Castilla-La Mancha Draft By: Eduardo W. V. Chaves (2016)


Ciudad Real – Spain
824 SOLVING PROBLEMS BY MEANS OF CONTINUUM MECHANICS

Problem 11.20
Consider a pipeline which has been introduced a pitot tube as shown in Figure 11.12.
Obtain the velocity at the point 1 in terms of h(1) and h( 2) . Consider that there is no energy
dissipation in the system.

Pitot tube

Pipeline
h( 2)

h(1)

v(1) v( 2) = 0
1  2 
h  p(1)  p( 2 )

Figure 11.12: Pitot tube.


Solution:
By applying the Bernoulli’s equation between the point 1 and 2 , we obtain:
p (1) v (21) p( 2) v (22 )
gh + + = gh + +
ρ 2 ρ 2
p (1) v (21) p ( 2)
⇒ + =
ρ 2 ρ
2( p ( 2 ) − p (1) )
⇒ v (1) =
ρ
The pressure values at the points 1 and 2 are given, respectively, by:
p (1) = ρ gh(1) ; p ( 2 ) = ρ gh( 2 )

with that the velocity v (1) is obtained as follows:

2( p ( 2) − p (1) ) 2(ρ gh( 2 ) − ρ gh(1) )


v (1) = = = 2 g (h( 2 ) − h(1) )
ρ ρ

University of Castilla-La Mancha Draft By: Eduardo W. V. Chaves (2016)


Ciudad Real – Spain
11 INTRODUCTION TO FLUIDS 825

Problem 11.21
Consider an incompressible non-viscous fluid, which has a steady velocity field and
irrotational. Consider also that the velocity field is independent of x3 -direction. Obtain the
governing equations for the proposed problem in terms of the velocity potential φ and
streamlines ψ .
Solution:
Velocity potential: In this example we can represent the velocity field by means of a potential
r
φ , i.e. v = ∇ xr φ . With that we are considering that the velocity field is conservative, hence
r r r r r
the curl of the velocity field is zero, i.e. ∇ xr ∧ v = rot v = ω = 0 . Remember that a field
whose curl is zero it does not necessarily imply that the field is conservative, but for a
conservative field the curl is always equals zero.
Note that the velocity has the same direction as ∇ xr φ , i.e. it is normal to the isosurfaces
φ = const .
Streamline: Given a spatial velocity field at time t , we can define a streamline ( ψ ) to the
curve in which the tangent at each point has the same direction as the velocity, (see Figure
11.13). In general, the streamline and trajectory do not coincide, but in steady state motion
they do. Two streamlines cannot intersect.

ψ (5) = const.

r r ψ ( 4) = const.
Control volume v ( x)
t Streamlines

ψ ( 2) = const.
∇ xr ψ
ψ (1) = const.

r
x

Figure 11.13

Based on the definition of differential dψ (the total derivative), and the definition of
r
gradient ∇ xr ψ we obtain the relationship dψ = ∇ xr ψ ⋅ dx .
r
Note that it holds that (∇ xrψ ) ⋅ (∇ xrφ ) = 0 , (Figure 11.14). The differential dx in the
streamline, at a point, has the same direction as the velocity at this point. Hence, the
following is fulfilled:
r r r
dx ∧ v = 0

University of Castilla-La Mancha Draft By: Eduardo W. V. Chaves (2016)


Ciudad Real – Spain
826 SOLVING PROBLEMS BY MEANS OF CONTINUUM MECHANICS

r
v = ∇ xr φ
ψ + dψ
∇ xr ψ

ψ = const. ⇒ dψ = 0
in the streamline

Figure 11.14

r r r
In the Cartesian system, dx ∧ v = 0 is represented by:
eˆ 1 eˆ 2 eˆ 3
r r r
dx ∧ v = dx1 dx 2 dx3 = 0
v1 v2 v3
r
= (v3 dx 2 − v 2 dx3 )eˆ 1 + (v 3 dx1 − v1 dx3 )eˆ 2 + (v 2 dx1 − v1 dx 2 )eˆ 3 = 0
Components:
(v3 dx 2 − v 2 dx3 ) 0
(dx ∧ v ) i =  (v 3 dx1 − v1 dx3 )  = 0
r r

 (v 2 dx1 − v1 dx 2 )  0

For this example the velocity field is independent of x3 , i.e. the problem is stated on the
plane x1 − x 2 (2D-case). With that we can conclude that:
 0  0 
r r 
( dx ∧ v ) i =  0  = 0 
  
(v 2 dx1 − v1 dx 2 ) 0

⇒ v 2 dx1 − v1 dx 2 = 0 (11.47)
Note that in a streamline it holds that ψ = const. ⇒ dψ = 0 and also by applying the
r
definition dψ = ∇ xr ψ ⋅ dx , we can obtain:
r
dψ = ∇ xr ψ ⋅ dx indicial
→ dψ = ψ ,i dxi = 0
⇒ dψ = ψ ,1 dx1 + ψ , 2 dx 2 + ψ ,3 dx3 = 0
∂ψ ∂ψ ∂ψ
⇒ dψ = dx1 + dx 2 + dx3 = 0
∂x1 ∂x 2 ∂x3
For the 2D-case (two-dimensional case) we have:
∂ψ ∂ψ
dx1 + dx 2 = 0 (11.48)
∂x1 ∂x 2
If we compare the equations (11.47) and (11.48) we can conclude that:
∂ψ ∂ψ
v1 = − ; v2 = (11.49)
∂x 2 ∂x1

University of Castilla-La Mancha Draft By: Eduardo W. V. Chaves (2016)


Ciudad Real – Spain
11 INTRODUCTION TO FLUIDS 827

r
1) Starting from an incompressible fluid: (∇ xr ⋅ v ) = 0 we can obtain:
∂v1 ∂v 2 ∂v3 2D ∂v1 ∂v 2
v i ,i = + + =0 → + =0
∂x1 ∂x 2 ∂x3 ∂x1 ∂x 2
r
Considering that v = ∇ xr φ , we obtain:
∂v1 ∂v2 ∂ 2φ ∂ 2φ
+ =0 ⇒ + = 0 ⇒ ∇ 2xrφ = 0 (11.50)
∂x1 ∂x2 ∂x12 ∂x22
r r r r r
2) Based on the fact that the fluid is irrotational ∇ xr ∧ v ≡ rot v = ω = 0 :
eˆ 1 eˆ 2 eˆ 3
r r r ∂ ∂ ∂ r
rot (v ) = ∇ xr ∧ v = = ijk v k , j eˆ i = 0
∂x1 ∂x2 ∂x3
v1 v2 v3 (11.51)
 ∂v ∂v   ∂v ∂v   ∂v ∂v  r
=  3 − 2 eˆ 1 +  1 − 3 eˆ 2 +  2 − 1 eˆ 3 = 0
 ∂x2 ∂x3   ∂x3 ∂x1   ∂x1 ∂x2 
Then:
 ∂v 3 ∂v 2 
 − 
  ∂x 2 ∂x 3   0 
  ∂v1 ∂v3    
  −   = 0
 
  ∂x3 ∂x1   0
 ∂v ∂v1   
 2
− 
 ∂x1 ∂x 2 
As we are dealing with a 2D-case, the above equations reduce to:
∂v 2 ∂v1
− =0
∂x1 ∂x 2
Taking into account the equations in (11.49) we can conclude that:
∂v 2 ∂v1 ∂ 2ψ ∂ 2ψ
− =0 ⇒ + =0 ⇒ ∇ 2xr ψ = 0
∂x1 ∂x 2 ∂x1 ∂x 2
With which the problem is stated by the equations:

∇ 2xr φ = 0 ; ∇ 2xr ψ = 0 (11.52)

University of Castilla-La Mancha Draft By: Eduardo W. V. Chaves (2016)


Ciudad Real – Spain
828 SOLVING PROBLEMS BY MEANS OF CONTINUUM MECHANICS

University of Castilla-La Mancha Draft By: Eduardo W. V. Chaves (2016)


Ciudad Real – Spain
A. Numerical Integration over Time

Annex A
Numerical Integration
over Time

A.1 Introduction

Before raising the case for multiple degrees of freedom we will analysis the numerical
solution for the problem:
dy ( x, t )
y ′( x, t ) = (A.1)
dt
which goal is to find the function y ( x, t ) .
The partial differential exact solution for the most engineering problems cannot be
obtained due to the problem complexity. But, for a better understanding of the procedure
numerical solution we will adopt the very simply differential equation, (see
CHAPRA&CANALE (1988)):
dy (t )
y′(t ) = = −2t 3 + 12t 2 − 20t + 8.5 (A.2)
dt
which is time dependent only. The exact solution can be obtained by integrate the above
equation over time, i.e.:
y = −0.5t 4 + 4t 3 − 10t 2 + 8.5t + C (A.3)
where C is the constant of integration. Note that there are infinite solutions, since C
could assume infinite values, (see Figure A.1). The solution is unique if the initial condition
is known. For this example (A.2) which is only time dependent the function value at t = 0
is known and given by y (t = 0) ≡ y 0 = 1 ⇒ C = 1 . Then, the solution is unique:
y = −0.5t 4 + 4t 3 − 10t 2 + 8.5t + 1 (A.4)
At t = 0 we know the following parameters:
826 SOLVING PROBLEMS BY MEANS OF CONTINUUM MECHANICS

 y0 = 1
 (A.5)
 y0′ = 8.5

C =3

C=2

C =1

C =0 t

C = −1

C = −2

Figure A.1: Infinites solutions.

y0′ = 8.5

y (t )

y0 = 1

K ∆t ∆t K

t1 t2 t3 K ti −1 ti ti +1 K tn t

Figure A.2: Time discretization.

University of Castilla-La Mancha Draft By: Eduardo W. V. Chaves (2015)


Ciudad Real – Spain
ANNEX A: NUMERICAL INTEGRATION OVER TIME 827

One of the most techniques used for numerical integration over time is the Finite
Difference Method in which the “domain” is discretized by the finite value ∆t (time
increment). Next we will discuss some of these methods.

A.2 Euler’s Method

Knowing the value of the curve slope at time t , i.e. y i′ , we can obtain the next
approximated value for y i +1 by means of a lineal approach:
y i +1 − y i
y i′ = ⇒ y i +1 = y i + y i′∆t (A.6)
∆t
The above approach is the same as forward finite difference.

y yi +1 = yi + yi′∆t
approximated value for y

} Error
yi +1

exact value for y


yi

∆t

ti ti +1 t

Figure A.3: Forward Finite Difference.

By means of Figure A.3 we can guess that when the time increment tends to zero we
approach the exact value of the function. For problems with several unknowns working
with very small time increment it can result in a high computational cost, so, to overcome
this drawback some effective methods have been developed in order to guarantee result
accuracy even when the time increment is big.
In previous example we have applied the forward finite difference using y i′ to obtain yi +1 .
We can also use the following approach: we situate at yi +1 and we apply the backward
finite difference, i.e.:
y i +1 − y i
y i′+1 = ⇒ y i +1 = y i + y i′+1 ∆t (A.7)
∆t
This method is known as backward Euler’s method (implicit method). With that we can
summarize that:
y i +1 = y i + y i′∆t (Explicit method) (A.8)

University of Castilla-La Mancha Draft By: Eduardo W. V. Chaves (2015)


Ciudad Real – Spain
828 SOLVING PROBLEMS BY MEANS OF CONTINUUM MECHANICS

y i +1 = y i + y i′+1 ∆t (Implicit method) (A.9)

Another approach we can adopt is by consider the curve slope as the average between y i′
and y i′+1 , i.e.:
 y ′ + y i′+1 
y i +1 = y i +  i ∆t (A.10)
 2 
which method is more precise than forward/backward finite difference. This method is
called Crank-Nicolson’s method.

A.3 Alfa Method

We can generalize the above method in a single expression. To do this we consider the
following approaches to the functions y (t ) and y ′(t ) , (see Figure A.4).

y (t ) y ′(t )
yi +1 y i′+1
yi +1 y i′+1

y i +α y i′+α

yi y i′
yi y i′
α 1− α α 1− α

t t
ti t i +α ti +1 ti t i +α ti +1

α ∈ [0;1]
∆t ∆t

a) b)

Figure A.4: Alfa method.

By means of linearization for the functions y (t ) and y ′(t ) , we can obtain:


y i +1 − y i
y i′+α = (A.11)
∆t
with that we can conclude that:
y i +1 = y i + y i′+ α ∆t (A.12)
Using similarity triangle, (see Figure A.4(b)), it is possible to express y i′+α as follows:

University of Castilla-La Mancha Draft By: Eduardo W. V. Chaves (2015)


Ciudad Real – Spain
ANNEX A: NUMERICAL INTEGRATION OVER TIME 829

y i′+1 − y i′ y i′+α − y i′
= ⇒ y i′+α = y i′ + α( y i′+1 − y i′ ) (A.13)
1 α
or:
y i′+ α = α y i′+1 + (1 − α ) y i′ (A.14)

The we summarize the Alfa method as follows:


 y i +1 = y i + y i′+α ∆t
 (A.15)
 y i′+ α = α y i′+1 + (1 − α ) y i′
in which depending on the α value we obtain::
ƒ α = 0 (Explicit)
 y i +1 = y i + y i′+ α ∆t
 ⇒ y i +1 = y i + y i′∆t (A.16)
 y i′+ α = (1) y i′
ƒ α = 1 (Implicit)
 y i +1 = y i + y i′+ α ∆t
 ⇒ y i +1 = y i + y i′+1 ∆t (A.17)
 y i′+ α = y i′+1
1
ƒ α= (Crank-Nicolson)
2
 y i +1 = y i + y ′ 1 ∆t
 i+
2  y ′ + y i′ 
 y ′ + y i′ ⇒ y i +1 = y i +  i +1  ∆t (A.18)
1  1  2 
 y ′ 1 = y i′+1 + 1 −  y i′ = i +1
 i + 2 2  2 2

The Crank-Nicolson’s method is also called Heun’s method. Geometrically we can


interpret as indicated in Figure A.5. At t i we make a prediction for y i0+1 = y i + y i′∆t and in
turn we can obtain y i′+01 . Then we find the value for the new curve slope:
y i′+01 + y i′
y i′ = (A.19)
2
Then we obtain once again the new value for y i +1 by considering the slope y i′ :
 y ′ 0 + y i′ 
y i +1 = y i +  i +1 ∆t
 (A.20)
 2 

University of Castilla-La Mancha Draft By: Eduardo W. V. Chaves (2015)


Ciudad Real – Spain
830 SOLVING PROBLEMS BY MEANS OF CONTINUUM MECHANICS

yi′
y i′ + y i′+01
yi′ =
2
y Predictor - y i0+1

y i′+01
yi0+1 = yi + yi′∆t
Corrector

yi +1

yi

∆t

ti ti +1 t

Figure A.5: Heun’s method (predictor-corrector).

Returning to our example initially raised whose differential equations is:


dy
y′ = = −2t 3 + 12t 2 − 20t + 8.5 (A.21)
dt
with the initial condition:
t=0 ; y0 = 1 ⇒ y0′ = 8.5 (A.22)
We will aplly the Euler’s method and the Heun’s method with time increment ∆t = 0.5s .
For the first time step (t = 0.5) the exact value of the function can be obtained by means of
(A.4), i.e.:
y (t = 0.5) = −0.5 × 0.54 + 4 × 0.53 − 10 × 0.52 + 8.5 × 0.5 + 1 = 3.21875 (A.23)
The numerical procedure follows:
Predictor i + 1
y10 = y0 + y0′ ∆t = 1 + 8.5 × 0.5 = 5.25 (We stop here if we are using the Euler’s method)
Corrector
y′(t = 0.5) = y1′0 = −2t 3 + 12t 2 − 20t + 8.5 = −2(0.5)3 + 12(0.5) 2 − 20(0.5) + 8.5 = 1.25

yi′+01 + yi′ 1.25 + 8.5


yi′ = = = 4.875
2 2
y1 = y0 + y0′ ∆t = 1 + 4.875 × 0.5 = 3.4375

University of Castilla-La Mancha Draft By: Eduardo W. V. Chaves (2015)


Ciudad Real – Spain
ANNEX A: NUMERICAL INTEGRATION OVER TIME 831

A.4 Modified Euler’s Method

In the modified Euler’s method we use the Euler’s method to predict the value function in
the middle of the interval:
∆t
y 1 = y i + y i′ (A.24)
i+
2
2

Next, we obtain the slope y ′ 1 at this point, which is used to obtain y i +1 :


i+
2

y i +1 = y i + y ′ 1 ∆t
i+ (A.25)
2

Apply this methodology to the proposed example (A.21) we can obtain:


∆t 0.5
y 1 = yi + yi′ = 1 + 8 .5 × = 3.125
i+
2
2 2
Slope calculation at the middle point:
y′(t = 0.25) = y′ 1 = −2t 3 + 12t 2 − 20t + 8.5 = −2(0.25)3 + 12(0.25) 2 − 20(0.25) + 8.5 = 4.21875
0+
2

y1 = y0 + y′1 ∆t = 1 + 4.21875 × 0.5 = 3.1093


2

In Figure A.6 we can appreciate this procedure to calculate the function,


dy (t )
y′(t ) = = −2t 3 + 12t 2 − 20t + 8.5 with y ′(0) = 1 , in which we have used the Euler’s
dt
method, modified Euler’s method and the Heun’s method with time increment ∆t = 0.5 .

Figure A.6: Comparative responses between some methods.

University of Castilla-La Mancha Draft By: Eduardo W. V. Chaves (2015)


Ciudad Real – Spain
832 SOLVING PROBLEMS BY MEANS OF CONTINUUM MECHANICS

As we can see the Heun’s method (H) always overestimates the function value while the
Modified Euler’s method (ME) underestimates the function value. We can make the
following approximation for y i +1 :

yi+1 =
1 H
3
(
yi +1 + 2 yiME
+1 ) (A.26)

Using the equations (A.20) and (A.25) in the above equation we can obtain:
1 H
yi+1 =
3
(
yi+1 + 2 yiME
+1 )
1  y′0 + yi′   
=  yi +  i+1 ∆t + 2 yi + y ′ 1 ∆t 
3  2   i+ 
   2 
(A.27)
1 y ′0 y′ 
=  yi + i+1 ∆t + i ∆t + 2 yi + 2 y′ 1 ∆t 
3  2 2 i+
2 
1 y ′0 y′ 
= 3 yi + i +1 ∆t + i ∆t + 2 y′ 1 ∆t 
3  2 2 i+
2 
thus:
∆t  
 y i′ + 4 y ′ 1 + y i′+1 
0
y i +1 = y i + (A.28)
6  i+
2 
whose equation is known as Runge-Kutta’s integration method of third order (see
CHAPRA&CANALE (1988)), which is a good approximation as we can see in Figure A.7.

Figure A.7: Comparative responses between some methods (Hunge-Kutta).

University of Castilla-La Mancha Draft By: Eduardo W. V. Chaves (2015)


Ciudad Real – Spain
ANNEX A: NUMERICAL INTEGRATION OVER TIME 833

A.5 Unsteady Case with Multiply Degree-of-


Freedom

Let us consider the following set of equations:


D T& + K T = F (A.29)
For the thermal problem, D stands for capacitance matrix, K is the conductivity matrix,
and T represents nodal temperature values.
By considering that
T − Tt
T& = t +1 (A.30)
∆t
and by apply the Alfa method:
Tα = αTt +1 + (1 − α )Tt
(A.31)
Fα = αFt +1 + (1 − α ) Ft
By substituting the equations (A.30) and (A.31) into the equation (A.29), we can obtain:
T −T 
D  t +1 t  + K Tα = Fα (A.32)
 ∆t 

T −T 
D  t +1 t  + K [αTα +1 + (1 − α )Tα ] = αFα +1 + (1 − α ) Fα (A.33)
 ∆t 
then:
D  D 
 ∆t + αK Tt +1 = α Ft +1 + (1 − α ) Ft +  ∆t − (1 − α ) K Tt (A.34)
   
K eff Tt +1 = F eff (A.35)

where
D D 
K eff = + αK ; F eff = α Ft +1 + (1 − α ) Ft +  − (1 − α ) K Tt (A.36)
∆t  ∆t 

University of Castilla-La Mancha Draft By: Eduardo W. V. Chaves (2015)


Ciudad Real – Spain
834 SOLVING PROBLEMS BY MEANS OF CONTINUUM MECHANICS

A.6 Dynamic Analysis with Numerical


Integration

The more general approach to the solution of the dynamic response for structures is the
direct numerical integration of the dynamic equilibrium:
MU&& + DU& + KU = F (A.37)
whose equations must fulfill for all time t , then it is also valid at time t + ∆t :
MU&&t + ∆t + DU& t + ∆t + KU t +∆t = Ft + ∆t (A.38)
where M is the mass matrix, D is the damping matrix, K is the stiffness matrix, F is the
nodal external force vector, and U , U& , U&& are displacement, velocity and acceleration,
respectively. The absence of subscript time step in the matrices M , D and K indicates a
linear problem, i.e. they do not depend on U , U& and U&& . In the case in which the structure
presents a material non-linearity the matrix K depends on U .
For a system without damping ( D = 0 ) the energy is conserved (system without energy
dissipation), and the sum of the internal energy ( 12 U& T MU& ) plus the strain energy
( 12 U T KU ) is constant at any time step:

2 E = U& tT MU& t + U tT KU t = U& tT+ ∆t MU& t + ∆t + U tT+ ∆t KU t + ∆t (A.39)


The numerical analysis of the dynamic system (A.37) can be inefficient where D is
responsible for the damping (energy dissipation) in the structure. Therefore some methods
were developed in order to introduce a numerical damping (artificial) which generally is
controlled by a parameter. For example, we can replace the damping matrix D by a linear
combination between K and M (Rayleigh damping):
D = α K + βM (A.40)
Then, we will study some methods in which a numerical damping is introduced albeit the
equation has no matrix D .
Several numerical techniques have been developed for solving the set of equations (A.38).
We can classify these techniques as Explicit, Implicit, or Mixed.
The Explicit methods do not require information at the time step t + ∆t to predict the
response at time t + ∆t , i.e.:
(
U t + ∆t = f U t , U& t , U&&t , U& t − ∆t , L ) (A.41)
These methods are conditionally stable, which implies that the time step size (∆t ) must be less
than a critical value (∆t cr ) , otherwise the solution is not stable, i.e. the solution diverges.
The Implicit methods use the information at time step t + ∆t to predict the structural
response at time t + ∆t , i.e.:
( )
U t + ∆t = f U t , U& t + ∆t , U&& t + ∆t , L (A.42)
With these methods it is possible to use larger time steps than those used in the explicit
method. The implicit method can be unconditionally or conditionally stable. In general, the
implicit methods are unconditionally stable, and the only restriction for time step size is the
solution accuracy.

University of Castilla-La Mancha Draft By: Eduardo W. V. Chaves (2015)


Ciudad Real – Spain
ANNEX A: NUMERICAL INTEGRATION OVER TIME 835

A.6.1 Newmark’s Family of Methods


Newmark in 1959 introduced a family of integration methods for solving dynamic
structural problems. To illustrate these methods we will star from the following set of
equations:
MU&&t + ∆t + DU& t + ∆t + KU t +∆t = Ft + ∆t (A.43)
We can apply the Taylor series to approximate the functions U and U& :
∆t 2 && ∆t 3 &&&
U t + ∆t = U t + ∆tU& t + Ut + Ut + L (A.44)
2! 3!
∆t 2 &&&
U& t + ∆t = U& t + ∆tU&&t + Ut + L (A.45)
2!
Newmark truncated the previous equations as follows:
∆t 2 &&
U t + ∆t ≈ U t + ∆tU& t + U t + β ∆t 3U
&&&
t
(A.46)
2
U& t + ∆t ≈ U& t + ∆tU&&t + γ∆t 2 U
&&&
t (A.47)
Assuming that the acceleration varies linearly within the range [t , t + ∆t ] , we can apply the
&&& , i.e.:
finite difference to approach U t

&& &&
&&& = U t + ∆t − U t
U (A.48)
t
∆t
Substituting the equation (A.48) into the equations (A.47) and (A.46) we can obtain:
∆t 2 && U&& − U&&t
U t + ∆t ≈ U t + ∆tU& t + U t + β ∆t 3 t + ∆t
2 ∆t
2
∆t &&
≈ U t + ∆tU& t + U t + β ∆t 2 U&& t + ∆t − β ∆t 2U&&t (A.49)
2
1 
≈ U t + ∆tU& t +  − β ∆t 2 U&& t + β ∆t 2 U&& t + ∆t
 2 
U&& − U&& t
U& t + ∆t ≈ U& t + ∆tU&&t + γ∆t 2 t + ∆t
∆t
(A.50)
≈ U t + ∆tU t + γ∆t U t + ∆t − γ∆t U&&t
& && &&
≈ U& + (1 − γ )∆tU&& + γ∆t U&&
t t t + ∆t

Then, we summarize the approaches used by Newmark for displacement, velocity and
acceleration:
1 
U t +∆t = U t + ∆tU& t +  − β  ∆t 2U&&t + β∆t 2U&&t +∆t
2 
U& = U& + (1 − γ ) ∆tU&& + γ∆t U&&
Newmark’s method (A.51)
t + ∆t t t t + ∆t

MU&&t + ∆t + DU& t + ∆t + KU t + ∆t = Ft + ∆t

This method is unconditionally stable when:


1
2β ≥ γ ≥ (A.52)
2

University of Castilla-La Mancha Draft By: Eduardo W. V. Chaves (2015)


Ciudad Real – Spain
836 SOLVING PROBLEMS BY MEANS OF CONTINUUM MECHANICS

Solving for U&& t + ∆t by using the displacement given by the equation (A.51) we can obtain:

β ∆t 2U&& t + ∆t = U t + ∆t − U t − ∆tU& t −  − β ∆t 2U&& t


1
2 
(A.53)
1 1 &  1  &&
⇒ U&& t + ∆t = (U t + ∆t − Ut ) − U t + 1 − U t
β ∆t 2 β ∆t  2β 
Substituting the equation (A.53) into the velocity equation (A.51) we can obtain:
U& t + ∆t = U& t + (1 − γ )∆tU&& t + γ∆t U&& t + ∆t
 1 
= U& t + (1 − γ )∆tU&& t + γ∆t  (U t + ∆t − U t ) − 1 U& t + 1 − 1 U&&t 
 β ∆t
2
β ∆t  2β   (A.54)
γ
= (U t + ∆t − U t ) + 1 − γ U& t + 1 − γ ∆tU&&t
β ∆t  β  2β 
Thus
 &&
U t + ∆t =
1
(U t + ∆t − U t ) − 1 U& t + 1 − 1 U&&t
 β ∆t 2
β ∆t  2β 
 (A.55)
γ
U& ( )  γ  &  γ  &&
 t + ∆t = β ∆t U t + ∆t − U t + 1 − β U t + 1 − 2β  ∆tU t
    
By substituting the equations given by (A.55) into the equation (A.43) we can obtain:
K eff U t + ∆t = F eff (A.56)
where
 1 γ 
K eff =  M+ D + K
 β∆t
2
β∆t 
 M γ   M  γ   1   γ  
F eff = Ft + ∆t +  + D U t +  − 1 −  D U& t − 1 −  M + 1 − ∆tD U&&t
 β ∆t
2
β ∆t   β ∆t  β    2 β   2β  
(A.57)
It is also possible to express the above equations as follows:
K eff = [b1 M + b4 D + K ]
(A.58)
[ ] [
F eff = Ft +∆t + M b1U t − b2U& t − b3U&&t + D b4U t − b5U& t − b6U&&t ]
where
1 1 1
b1 = ; b2 = − ; b3 = 1 −
β ∆t 2 β ∆t 2β (A.59)
b4 = γ∆tb1 ; b5 = 1 + γ∆tb2 ; b6 = ∆t [1 − γ + γb3 ]
The velocity and displacement fields can also be expressed in terms of the parameters
(A.59):
U&&t + ∆t = b1 (U t + ∆t − U t ) + b2 U& t + b3U&&t
& (A.60)
U t + ∆t = b4 (U t + ∆t − U t ) + b5U t + b6U t
& &&

Next we will apply the same methodology to solve the following system:

University of Castilla-La Mancha Draft By: Eduardo W. V. Chaves (2015)


Ciudad Real – Spain
ANNEX A: NUMERICAL INTEGRATION OVER TIME 837

DU& t + ∆t + KU t + ∆t = Ft +∆t (A.61)


Given the vector U t + ∆t and its approach by using Taylor series (A.44), in which we truncate
until second order term:
U t + ∆t = U t + ∆tU& t + α∆t 2 U&& t (A.62)
Considering the following approach for U&&t :
U& − U& t
U&&t = t + ∆t (A.63)
∆t
Then, the vector U t + ∆t can be rewritten as follows:
U& − U& t
U t + ∆t = U t + ∆tU& t + α∆t 2 t + ∆t (A.64)
∆t
And the vector U& t + ∆t becomes:
1
U& t + ∆t = (U t + ∆t − U t ) + 1 (α − 1)U& t (A.65)
α∆t α
By substituting the equation (A.65) into the equation (A.61) we can obtain:
 1
D (U t +∆t − U t ) + 1 (α − 1)U& t  + KU t +∆t = Ft +∆t
α ∆t α 
 1   1 1 
⇒ D + K U t + ∆t = Ft + ∆t + D  U t − (α − 1)U& t  (A.66)
α ∆ t   α ∆t α 
1  1
⇒  D + αK U t + ∆t = αFt + ∆t + DU t + (1 − α )DU& t
 ∆t  ∆t
And the vector U& t can be expressed as follows:
DU& t + KU t = Ft ⇒ U& t = D −1 (Ft − KU t ) (A.67)
Substituting (A.67) into (A.66) we can obtain:
1  1
 ∆t D + αK U t + ∆t = αFt + ∆t + ∆t DU t + (1 − α )DU t
&
 
1  1
⇒  D + αK U t +∆t = αFt +∆t + DU t + (1 − α )DD −1 (Ft − KU t )
 ∆t  ∆t
(A.68)
1  1
⇒  D + αK U t +∆t = αFt +∆t + DU t + (1 − α )Ft − (1 − α )KU t
 ∆t  ∆t
1  1 
⇒  D + αK U t +∆t = αFt +∆t + (1 − α )Ft +  D − (1 − α )K U t
 ∆t   ∆t 
or:
K eff U t + ∆t = F eff (A.69)
where
1 1 
K eff = D + αK ; F eff = αFt + ∆t + (1 − α )Ft +  D − (1 − α )K U t (A.70)
∆t  ∆t 
We can verify that the equation (A.69) is the same equation obtained by means of Alfa
method employed for unsteady temperature problem, (see equation (A.34)).

University of Castilla-La Mancha Draft By: Eduardo W. V. Chaves (2015)


Ciudad Real – Spain
838 SOLVING PROBLEMS BY MEANS OF CONTINUUM MECHANICS

A.6.1.1 Newmark’s Method Scheme

I. Initial Parameters
I.1. Construction of matrices M , D , K .
I.2. Obtain the parameters:
1 1 1
b1 = ; b2 = − ; b3 = 1 −
β ∆t 2 β ∆t 2β
b4 = γ∆tb1 ; b5 = 1 + γ∆tb2 ; b6 = ∆t [1 − γ + γb3 ]

I.3. Construction of K eff :


K eff = [b1 M + b4 D + K ]

I.4. Given the boundary conditions U 0 , U& 0 , obtain U&& 0 :


[
U&&0 = M −1 F0 − DU& 0 − KU 0 ]
I.5. Update the variables:
Ut ← U0 ; U& t ← U& 0 ; U&& t ← U&& 0
II. For each time step t + ∆t do
II.1. Obtain the effective force vector:
[ ] [
F eff = Ft +∆t + M b1U t − b2U& t − b3U&&t + D b4U t − b5U& t − b6U&&t ]
II.2. Solve the system:
K eff U t + ∆t = F eff

II.3. Calculate the vectors U& t + ∆t and U&& t + ∆t :


U&& t + ∆t = b1 (U t + ∆t − U t ) + b2 U& t + b3U&&t
&
U t + ∆t = b4 (U t + ∆t − U t ) + b5U& t + b6 U&&t
II.4. Update the variables:
U t ← U t + ∆t ; U& t ← U& t + ∆t ; U&& t ← U&& t + ∆t
If it is the case Ft + ∆t ← F (t + ∆t , U t + ∆t ,U& t + ∆t , U&& t + ∆t ,...)
Go to step II.1 with t + ∆t .

University of Castilla-La Mancha Draft By: Eduardo W. V. Chaves (2015)


Ciudad Real – Spain
ANNEX A: NUMERICAL INTEGRATION OVER TIME 839

A.6.2 Average Acceleration Method


The average acceleration method is identical to the trapezoidal rule, (see Figure A.8), in
which we take the acceleration approach as follows:
U&& + U&& t + ∆t
U&&t + τ (τ) = t ; 0 ≤ τ ≤ ∆t (A.71)
2

U&& (t )

U&&t + ∆t
U&& + U&& t + ∆t
U&&t (τ ) = t
2

U&&t

t t + ∆t t

Figure A.8: Average acceleration.

Integrating the equation (A.71) we can obtain:


 U&& + U&&t + ∆t   U&& + U&&t + ∆t 
U& t +τ (τ ) = τ  t  + C1 = U& t + τ  t
 

 (A.72)
 2   2 
where the constant of integration C1 was obtained by means of the initial condition
U& t +τ (τ = 0) = U& t ⇒ C1 = U& t . The displacement can be obtain by means of integration of
the equation (A.72) over time, then:
τ 2
 U&&t + U&&t + ∆t  τ 2  U&& t + U&& t + ∆t 
U t +τ (τ ) = τU& t +   + C 2 = U t + τU& t +  (A.73)
2  2  2  2 
  
Once again we use the initial condition to obtain the constant of integration
U t +τ (τ = 0) = U t ⇒ C1 = U t .
For τ = ∆t , the displacement and velocity vectors become:
∆t 2 && ∆t 2 &&
U t + ∆t = U t + ∆tU& t + Ut + U t + ∆t
4 4 (A.74)
∆t && ∆t &&
U& t + ∆t = U& t + Ut + U t + ∆t
2 2
The equations given by (A.74) are the same equations given by the Newmark’s Method
1 1
when γ = and β = , (see equations (A.49) and (A.50)).
2 4

University of Castilla-La Mancha Draft By: Eduardo W. V. Chaves (2015)


Ciudad Real – Spain
840 SOLVING PROBLEMS BY MEANS OF CONTINUUM MECHANICS

By means of the equations (A.74) we can obtain the vectors U& t + ∆t and U&& t + ∆t in terms of
U t + ∆t and U t , U& t , U&&t . By means of displacement vector given by (A.74) it is possible to
obtain U&& t + ∆t as follows:
4 4
U&&t + ∆t = 2 (U t + ∆t − U t ) − U& t − U&& t (A.75)
∆t ∆t
In turn we substitute the equation (A.75) into the velocity given by (A.74), thus:
∆t ∆t  4
U& t + ∆t = U& t + U&&t +  (U t + ∆t − U t ) − 4 U& t − U&&t 
2 2  ∆t 2
∆t  (A.76)
2
= (U t + ∆t − U t ) − U& t
∆t
By substituting the velocity vector (A.76) and the acceleration vector (A.75) into the
dynamic equilibrium equation at time t + ∆t we can obtain:
MU&&t + ∆t + DU& t + ∆t + KU t + ∆t = Ft +∆t
 4 4  2  (A.77)
M  2 (U t + ∆t − U t ) − U& t − U&&t  + D  (U t +∆t − U t ) − U& t  + KU t + ∆t = Ft + ∆t
 ∆t ∆t   ∆t 
By restructuring the above equation we can obtain
 4 2   4 2  4 & &&
 ∆t 2 M + ∆t D + K U t +∆t = Ft + ∆t +  ∆t 2 M + ∆t D U t +  ∆t M + D U t + MU t (A.78)
     
or
K eff U t + ∆t = Ft eff
+ ∆t (A.79)
where we have considered that:
4 2
K eff = M+ D+K
∆t 2
∆t
(A.80)
 4 2  4 
Ft eff
+ ∆t = Ft + ∆t +  2 M + D U t +  M + D U& t + MU&&t
 ∆t ∆ t   ∆t 

University of Castilla-La Mancha Draft By: Eduardo W. V. Chaves (2015)


Ciudad Real – Spain
ANNEX A: NUMERICAL INTEGRATION OVER TIME 841

A.6.2.1 Average Acceleration Method Scheme

I. Initial Parameters
I.1. Construction of M , D , K .
I.2. Given the boundary conditions U 0 , U& 0 , obtain U&& 0 :
(
U&&0 = M −1 F0 − DU& 0 − KU 0 )
I.3. Construction of K eff :
4 2
K eff = M+ D+K
∆t 2
∆t
I.4. Update the variables:
Ut ← U0 ; U& t ← U& 0 ; U&& t ← U&& 0
II. For each time step t + ∆t do
II.1. Obtain the effective force vector:
 4 2  4 
F eff = Ft +∆t +  2 M + D U t +  M + D U& t + MU&&t
 ∆t ∆ 
t ∆ t 
II.2. Solve the system:
K eff U t + ∆t = F eff

II.3. Calculate the vectors U& t + ∆t , U&& t + ∆t :


& 2
U t + ∆t = ∆t (U t + ∆t − U t ) − U t
&

U&& =
2 &
U
 t + ∆t ∆t t + ∆t
[ − U& t − U&& t]
II.4. Update the variables:
U t ← U t + ∆t ; U& t ← U& t + ∆t ; U&& t ← U&& t + ∆t

If it is the case Ft + ∆t ← F (t + ∆t , U t + ∆t ,U& t + ∆t , U&& t + ∆t ,...)


Go to step II.1 with t + ∆t .

University of Castilla-La Mancha Draft By: Eduardo W. V. Chaves (2015)


Ciudad Real – Spain
842 SOLVING PROBLEMS BY MEANS OF CONTINUUM MECHANICS

A.6.3 Linear Acceleration Method


For this method we consider a linear variation for the acceleration field within the range
[t , t + ∆t ] , (see Figure A.9):
τ &&
U&&t +τ (τ ) = U&& t +
∆t
(
U t + ∆t − U&& t ) (A.81)

U&& (t )

U&&t + ∆t

U&&t + τ (τ) = U&& t +


∆t
(
τ &&
U t + ∆t − U&& t )
U&&t

t t + ∆t t
∆t

Figure A.9: Linear acceleration method.

The velocity vector can be obtained by integrate over time the equation (A.81), thus:
τ 2 &&
U& t +τ (τ ) = τU&&t +
2∆t
(
U t + ∆t − U&& t + C1 )
(A.82)
τ 2 &&
= U& t + τU&&t +
2∆t
U t + ∆t − U&& t ( )
where we have apply the initial condition in order to obtain the constant of integration, i.e.
at τ = 0 we have that U& t +τ (τ = 0) = U& t ⇒ C1 = U& t .
Then, integrate over time the equation (A.82) we can obtain the displacement vector:
τ 2 && τ 3 &&
U t + τ (τ) = τU& t +
2
Ut +
6 ∆t
(
U t + ∆t − U&& t + C 2 ) ∴ U t + τ ( τ = 0) = U t ⇒ C 2 = U t
(A.83)
τ2 τ 3 &&
= U t + τU& t + U&&t +
2 6 ∆t
(
U t + ∆t − U&& t )
When τ = ∆t we can obtain:
∆t 2 && ∆t 2 &&
U t + ∆t = U t + ∆tU& t +
2
Ut +
6
U t + ∆t − U&& t ( ) (A.84)

U& t + ∆t = U& t + ∆tU&& t +


∆t &&
2
(
U t + ∆t − U&& t = U& t +
∆t &&
2
)
U t + ∆t + U&& t ( ) (A.85)

By means of the equation (A.84) it is possible to solve for U&& : t + ∆t

University of Castilla-La Mancha Draft By: Eduardo W. V. Chaves (2015)


Ciudad Real – Spain
ANNEX A: NUMERICAL INTEGRATION OVER TIME 843

6 6
U&&t + ∆t = 2 (U t + ∆t − U t ) − U& t − 2U&& t (A.86)
∆t ∆t
In turn, by substituting (A.86) into (A.85) we can obtain:

U& t + ∆t =
3
(U t + ∆t − U t ) − 2U& t − ∆t U&&t (A.87)
∆t 2
Then, by substituting the velocity vector (A.87) and the acceleration vector (A.86) into the
dynamic equilibrium equation we can obtain:
MU&&t + ∆t + DU& t + ∆t + KU t + ∆t = Ft +∆t
 6 6 
M  2 (U t + ∆t − U t ) − U& t − 2U&&t 
 ∆t ∆t  (A.88)
3 ∆t 
+ D  (U t + ∆t − U t ) − 2U& t − U&&t  + KU t +∆t = Ft + ∆t
 ∆t 2 
The above equation can be restructured as follows:
K eff U t + ∆t = Ft eff
+ ∆t (A.89)
where
6 3
K eff = M+ D+K
∆t 2
∆t
(A.90)
 6 3  6   ∆t  &&
Ft eff = Ft + ∆t +  2 M + D U t +  M + 2 D U& t + 2 M + D Ut
2 
+ ∆t
 ∆t ∆t   ∆t  
1 1
The linear acceleration method is the same as Newmark’s method when γ = and β = .
2 6
1 1
This fact can be verified by substituting γ = and β = into the equations in (A.51), with
2 6
which the displacement and velocity vectors become:
1 1 1 ∆t 2 && ∆t 2 &&
U t + ∆t = U t + ∆tU& t +  − ∆t 2 U&&t + ∆t 2U&&t + ∆t = U t + ∆tU& t + Ut + U t + ∆t
 2 6 6 3 6
(A.91)
 1 1
U& t + ∆t = U& t + 1 −  ∆tU&&t + ∆t U&&t + ∆t = U& t +
2 2
∆t &&
2
[
U t + U&& t + ∆t ]

which match the equations obtained by using the linear acceleration method, (see equations
(A.84) and (A.85)).

University of Castilla-La Mancha Draft By: Eduardo W. V. Chaves (2015)


Ciudad Real – Spain
844 SOLVING PROBLEMS BY MEANS OF CONTINUUM MECHANICS

A.6.3.1 Linear Acceleration Method Scheme

I. Initial Parameters
I.1. Construction of M , D , K .
I.2. Given the boundary conditions U 0 , U& 0 , obtain U&& 0 :
(
U&&0 = M −1 F0 − DU& 0 − KU 0 )
I.3. Construction of K eff :
6 3
K eff = M+ D+K
∆t 2
∆t
I.4. Update the variables:
Ut ← U0 ; U& t ← U& 0 ; U&& t ← U&& 0
II. For each time step t + ∆t do
II.1. Obtain the effective force vector:
 6 3  6   ∆t 
F eff = Ft + ∆t +  2 M + D U t +  M + 2 D U& t +  2 M + D U&&t
 ∆t ∆t   ∆t   2 
II.2. Solve the system:
K eff U t + ∆t = F eff

II.3. Calculate the vectors U& t + ∆t , U&& t + ∆t :


 && 6 6 &
U t + ∆t = ∆t 2 (U t + ∆t − U t ) − ∆t U t − 2U t
&&

U&
 t + ∆t
= U& t +
∆t &&
2
(
U t + ∆t + U&& t )
II.4. Update the variables:
U t ← U t + ∆t ; U& t ← U& t + ∆t ; U&& t ← U&& t + ∆t

If it is the case Ft + ∆t ← F (t + ∆t , U t + ∆t ,U& t + ∆t , U&& t + ∆t ,...)


Go to step II.1 with t + ∆t .

University of Castilla-La Mancha Draft By: Eduardo W. V. Chaves (2015)


Ciudad Real – Spain
ANNEX A: NUMERICAL INTEGRATION OVER TIME 845

A.6.4 Central Finite Difference Method


This explicit method assumes that the velocity and acceleration vectors are obtained by
using th central finite difference to approach the first and second derivatives:
U − U t − ∆t
U& t = t + ∆t
2∆t
(A.92)
U − 2U t + U t − ∆t
U&&t = t + ∆t
∆t 2
By substituting the equations in (A.92) into the dynamic equilibrium equation at time t ,
MU&&t + DU& t + KU t = Ft , we can obtain:
U t + ∆t − 2U t + U t −∆t U − U t −∆t
M + D t + ∆t + KU t = Ft
∆t 2 2∆t
M D   2M   D M  (A.93)
⇒ 2 + U t + ∆t = Ft +  2 − K U t +  − 2 U t −∆t
 ∆t 2∆t   ∆t   2∆t ∆t 
⇒ K eff U t + ∆t = F eff
where
M D
K eff = +
∆t 2
2 ∆t
(A.94)
eff  2M   D M 
F = Ft +  2 − K U t +  − 2 U t −∆t
 ∆t   2∆t ∆t 
At time t = 0 we have to calculate U 0− ∆t , which value can be obtained by means of the
equation (A.92):
U − U 0 − ∆t
U& 0 = 0 + ∆t ⇒ U 0 + ∆t = 2∆tU& 0 + U 0 − ∆t
2∆t
(A.95)
U − 2U 0 + U 0 − ∆t
U&& 0 = 0 + ∆t ⇒ U 0 + ∆t = ∆t 2 U&& 0 + 2U 0 − U 0 − ∆t
∆t 2
From the two above equations in (A.95) we can obtain:
∆t 2 U&& 0 + 2U 0 − U 0 − ∆t = 2∆tU& 0 + U 0 − ∆t
⇒ 2U = ∆t 2U&& + 2U − 2∆tU&
0 − ∆t 0 0 0
(A.96)
2
∆t &&
⇒ U 0 − ∆t = U 0 − ∆tU& 0 + U0
2

University of Castilla-La Mancha Draft By: Eduardo W. V. Chaves (2015)


Ciudad Real – Spain
846 SOLVING PROBLEMS BY MEANS OF CONTINUUM MECHANICS

A.6.4.1 Central Finite Difference Method Scheme

I. Initial Parameters
I.1. Construction of M , D , K .
I.2. Given the boundary conditions U 0 , U& 0 , obtain U&& 0 :
(
U&&0 = M −1 F0 − DU& 0 − KU 0 )
I.3. Construction of K eff :
M D
K eff = +
∆t 2
2∆t
I.4. Calculate U 0− ∆t :
∆t 2 &&
U 0 − ∆t = U 0 − ∆tU& 0 + U0
2
I.4. Update the variables:
U t − ∆t ← U 0− ∆t ; Ut ← U0 ; Ft = F0
II. For each time step t do
II.1. Obtain the effective force vector:
 2M   D M 
F eff = Ft +  2 − K U t +  − 2 U t −∆t
 ∆t   2 ∆t ∆t 
II.2. Solve the system:
K eff U t + ∆t = F eff

II.3. Calculate the vectors U& t , U&&t :


& U t + ∆t − U t − ∆t
U t = 2∆t

U&& = t + ∆t − 2U t + U t − ∆t
U
 t ∆t 2
II.4. Update the variables:
U t − ∆t ← U t ; U t ← U t + ∆t

If it is the case Ft ← F (t + ∆t , U t + ∆t ,U& t + ∆t , U&&t + ∆t ,...)


Go to step II.1 with t + ∆t .

University of Castilla-La Mancha Draft By: Eduardo W. V. Chaves (2015)


Ciudad Real – Spain
ANNEX A: NUMERICAL INTEGRATION OVER TIME 847

The central finite difference is a method with explicit integration and conditionally stable
and requires time step ∆t less than the critical value:
Tmin 2
∆t ≤ ∆t cr = = (A.97)
π ω max
where Tmin stands for the smallest natural period, and ω max is the maximum frequency of
the discrete system, which is greater eigenvalue of the characteristic determinant:
det ( K + ω 2 M ) ≡ K + ω 2 M = K + λM = 0 (A.98)

A.6.5 Wilson- θ Method


The Wilson- θ method (1968) is an extension of the linear acceleration method
(Newmark’s method when γ = 12 and β = 16 ). The acceleration vector is approached within
the range 0 ≤ τ ≤ θ ∆t as showed in Figure A.10, where θ ≥ 1 . When θ = 1 we fall back into
the linear acceleration method.

U&& (t )

U&&t +θ∆t
U&&t + τ (τ) = U&&t + U(
τ &&
θ ∆t t + θ ∆ t
− U&& t )
U&&t + ∆t

U&&t

t ∆t t + ∆t t + θ ∆t t

θ ∆t

Figure A.10: Acceleration approach – Wilson- θ method.

By means of Figure A.10 the acceleration vector becomes:

U t +τ
&& + τ U
&& (τ) = U
t
&& ( &&
t + θ ∆t − U t
θ ∆t
) (A.99)

By integrate over time the above equation we can obtain the velocity vector:
τ 2 &&
U& t + τ (τ) = U& t + τU&&t +
2θ ∆t
( &&
U t + θ ∆t − U t ) (A.100)

In turn, by integrate the above equation (A.100) we can obtain the displacement vector:
τ2 τ 3 &&
U t + τ (τ) = U t + τU& t + U&&t +
2 6θ ∆t
( &&
U t + θ ∆t − U t ) (A.101)

By considering τ = θ ∆t the velocity vector becomes:

University of Castilla-La Mancha Draft By: Eduardo W. V. Chaves (2015)


Ciudad Real – Spain
848 SOLVING PROBLEMS BY MEANS OF CONTINUUM MECHANICS

(θ ∆t ) 2 &&
U& t + θ ∆t = U& t + θ ∆tU&&t +
2θ ∆t
( &&
U t + θ ∆t − U t )
(A.102)
θ ∆t &&
= U& t +
2
( &&
U t +θ ∆t + U t )
and the displacement vector:
(θ ∆t ) 2 && (θ ∆t ) 3 &&
U t + θ ∆ = U t + θ ∆tU& t +
2
Ut +
6θ ∆t
( &&
U t + θ ∆t − U t )
(A.103)
θ 2 ∆t 2 &&
= U t + θ ∆tU& t +
6
(
U t + θ ∆t + 2U&&
t )
Then, by solving for U&& t +θ∆t :
6
&&
U t + θ ∆t = (U t +θ∆ − U t ) − 6 U& t − 2U&&t (A.104)
θ ∆t22
θ ∆t
By substituting the equation (A.104) into the velocity equation (A.102) we can obtain:

U& t + θ ∆t =
3
(U t +θ ∆ − U t ) − 2U& t − θ ∆t U&&t (A.105)
θ ∆t 2
Taking into account the dynamic equilibrium equation at time t + θ ∆t :
MU&&t +θ∆t + DU& t +θ∆t + KU t +θ∆t = Fˆt +θ∆t (A.106)
where Fˆ t +θ∆t = θ Ft +θ∆t + (1 − θ ) Ft , and by substituting the values for U& t +θ∆t and U&&t +θ∆t given
respectively by the equations (A.105) and (A.104), we can obtain the following set of
equations:
K eff U t + θ ∆t = F eff (A.107)
where
6M 3D
K eff = + +K
θ ∆t θ∆t
2 2
(A.108)
 6M 3D   6M   θ∆t  &&
F eff = θFt +θ∆t + (1 − θ ) Ft +  2 2 + U t +  + 2 D U& t +  2 M + D U t
 θ ∆t θ ∆t   θ ∆t   2 
After the system (A.107) is solved, U t + θ∆t is determined and is possible to calculate U t + ∆t ,
U& t + ∆t and U&& t + ∆t . To do this, we consider the equation (A.99) when τ = ∆t , thus:

&&
U && 1 U
t + ∆t = U t +
&&
θ
( &&
t + θ ∆t − U t ) (A.109)

By substituting the value of U&&t + θ∆t given by the equation (A.104), the above equation
becomes:
&&
U && 1 U
t + ∆t = U t +
&&
θ
( &&
t + θ ∆t − U t )
&& + 1  6 (U 6 & && 
t +θ ∆ − U t ) −
=U U t − 2U && − U
t  t t (A.110)
θ  θ ∆t
2 2
θ ∆t 
6 6  3  &&
= 3 2 (U t + θ ∆ − U t ) − 2 U& t + 1 − U t
θ ∆t θ ∆t  θ
To obtain U& t + ∆t we use the equation in (A.100) by assuming τ = ∆t , thus:

University of Castilla-La Mancha Draft By: Eduardo W. V. Chaves (2015)


Ciudad Real – Spain
ANNEX A: NUMERICAL INTEGRATION OVER TIME 849

U& t + ∆t = U& t + ∆tU&&t +


∆t &&

( &&
U t +θ ∆t − U t ) (A.111)

Taking into account the equation (A.109), the relationship


1 &&
θ
(
U t +θ ∆t − U t)
&& = U
&& &&
t + ∆t − U t

holds, and substituting into the above equation we can obtain:

U& t + ∆t = U& t + ∆tU&&t +


∆t &&
2
( &&
U t + ∆t − U t )
(A.112)
= U& t +
2
(
∆t && &&
U t + ∆t + U t )
In order to obtain the displacement U t + ∆t it is enough to enforce τ = ∆t in the equation
(A.101), thus:

U t + ∆t = U t + ∆tU& t +
∆t 2
2
U&& t +
∆t 2

(U&& t + θ ∆t
&&
−U t )
= U t + ∆tU& t +
∆t 2
2
U&& t +
∆t 2
6
(U&& t + ∆t
&&
−U t ) (A.113)

= U t + ∆tU& t +
∆t 2
6
(U&& t + ∆t + 2U&& t )

University of Castilla-La Mancha Draft By: Eduardo W. V. Chaves (2015)


Ciudad Real – Spain
850 SOLVING PROBLEMS BY MEANS OF CONTINUUM MECHANICS

A.6.5.1 Wilson- θ Method Scheme

I. Initial Parameters
I.1. Construction of M , D , K .
I.2. Given the boundary conditions U 0 , U& 0 , obtain U&& 0 :
(
U&&0 = M −1 F0 − DU& 0 − KU 0 )
I.3. Calculate K eff :
6M 3D
K eff = + +K
θ ∆t θ∆t
2 2

I.4. Update the variables:


Ut ← U0 ; U& t ← U& 0 ; U&& t ← U&& 0
II. For each time step t + ∆t do
II.1. Obtain the effective force vector:
 6M 3D   6M   θ∆t  &&
F eff = θFt +θ∆t + (1 − θ ) Ft +  2 2 + U t +  + 2 D U& t +  2 M + D U t
 θ ∆t θ ∆t   θ ∆t   2 
II.2. Solve the system:
K eff U t + θ ∆t = F eff

II.3. Calculate the vectors U&& t + ∆t , U& t + ∆t y U t + ∆t :


 && 6 6 &  3  &&
U t + ∆t = 3 2 (U t + θ ∆ − U t ) − 2 U t + 1 − θ U t
 θ ∆t θ ∆t  
&
U t + ∆t = U& t +

2
t
U (
&& &&
t + ∆t + U t )

 ∆t 2 &&
U t + ∆t = U t + ∆tU& t +
6
(
U t + ∆t + 2U&& t )

II.4. Update the variables:
U t ← U t + ∆t ; U& t ← U& t + ∆t ; U&& t ← U&& t + ∆t
If it is the case Ft +θ∆t ← F (t + θ ∆t , U t +θ ∆t ,U& t +θ∆t , U&&t +θ∆t ,...)
Go to step II.1 with t + ∆t .

University of Castilla-La Mancha Draft By: Eduardo W. V. Chaves (2015)


Ciudad Real – Spain
ANNEX A: NUMERICAL INTEGRATION OVER TIME 851

A.6.6 Houbolt’s Method


Houbolt’s method (1950) is a mixed method, and approaches the displacement by a cubic
function within the range between t − 2∆t and t + ∆t , according to the function
U t +τ (τ ) = aτ 3 + bτ 2 + cτ + d (A.114)
where the coefficients a , b , c and d are given by:
1
a= 3
[− U t − 2∆t + 3U t −∆t − 3U t + U t + ∆t ] ; b = 1 2 [U t − ∆t − 2U t + U t + ∆t ]
6∆t 2∆t
(A.115)
1
c= [U t − 2∆t − 6U t −∆t + 3U t + 2U t + ∆t ] ; d = U t
6∆t
The first and second derivatives in (A.114) are given by:
U& t + τ (τ) = 3aτ 2 + 2bτ + c ; U&&t + τ (τ) = 6aτ + 2b (A.116)
Assuming τ = ∆t we can obtain:
U& t + ∆t = 3a∆t 2 + 2b∆t + c ; U&&t + ∆t = 6a∆t + 2b (A.117)

By substituting the values for a , b and c given by (A.115) into the equations in (A.117)
we can obtain:
1
U& t + ∆t = [11U t + ∆t − 18U t + 9U t −∆t − 2U t −2∆t ]
6∆t
(A.118)
1
U&&t + ∆t = 2 [2U t + ∆t − 5U t + 4U t − ∆t − U t − 2 ∆t ]
∆t
NOTE: This method is unconditionally stable but provides artificial damping (numerical
damping) which is very high for a low-frequency response.
By considering the dynamic equation at t + ∆t , i.e. MU&&t +∆t + DU& t +∆t + KU t +∆t = Ft +∆t , and
by substituting the values for U& t + ∆t and U&&t + ∆t given by (A.118), we can obtain:
K eff U t + ∆t = F eff (A.119)
where
2 11
K eff = M+ D+K
∆t 2
6∆t
(A.120)
 5 M 3D   4 M 3D   M D 
F eff = Ft +∆t +  2 + U t −  2 + U t −∆t +  2 + U t −2 ∆t
 ∆t ∆t   ∆t 2∆t   ∆t 3∆t 
Similarly to the central finite difference, the Houbolt’s method needs a pretreatment at
t = 0 in order to obtain the values for U 0 − ∆t and U 0 −2 ∆t . Then, we express the values for
U& t + τ (τ) and U&&t + τ (τ) , (see equations in (A.116)), at time τ = 0 , thus
1
U& t + 0 (τ = 0) = c = [U t −2∆t − 6U t −∆t + 3U t + 2U t + ∆t ]
6∆t
(A.121)
1
U&&t + 0 (τ = 0) = 2b = 2 [U t − ∆t − 2U t + U t + ∆t ]
∆t
Thatt is, we apply in the third point of integration, (see Figure A.11).

University of Castilla-La Mancha Draft By: Eduardo W. V. Chaves (2015)


Ciudad Real – Spain
852 SOLVING PROBLEMS BY MEANS OF CONTINUUM MECHANICS

U (t )

U 0 + ∆t

U0
U 0 −2 ∆t U 0 − ∆t

− 2 ∆t − ∆t 0 ∆t t

Figure A.11: Houbolt’s method parameters at t = 0 .

Assuming the equations (A.121), at time t = 0 , we can obtain:


1 1
U& 0 = [U 0−2∆t − 6U 0−∆t + 3U 0 + 2U 0+ ∆t ] ; U&&0 = 2 [U 0 − ∆t − 2U 0 + U 0+ ∆t ] (A.122)
6∆t ∆t
The value for U 0− ∆t can be obtained by means of the acceleration equation given by
(A.122), i.e.:
U 0 − ∆t = ∆t 2U&&0 + 2U 0 − U 0+ ∆t (A.123)
Then, by substituting the value U 0− ∆t into the velocity equation ( U& 0 ), given by (A.122), it is
possible to obtain the value for U 0− 2 ∆t , i.e.:
U 0− 2 ∆t = 6∆tU& 0 + 6∆t 2U&&0 − 8U 0+ ∆t + 9U 0 (A.124)
At time t = 0 the set of equations (A.119) becomes:
K eff U 0 + ∆t = F eff (A.125)
where
2 11
K eff = M+ D+K
∆t 2
6 ∆t
(A.126)
eff  5 M 3D   4 M 3D   M D 
F = F0+∆t +  2 + U 0 −  2 + U 0−∆t +  2 + U 0−2 ∆t
 ∆t ∆t   ∆t 2∆t   ∆t 3∆t 
Note that U 0− ∆t and U 0−2 ∆t are also functions of the unknown U 0+ ∆t . By substituting the
values for U 0− ∆t and U 0−2 ∆t given by (A.123) and (A.124) into the set of equations (A.125)
and after restructuring we can obtain:
Kˆ eff U 0 + ∆t = Fˆ eff (A.127)
6 3
Kˆ eff = 2 M + D+K
∆t ∆t
(A.128)
 6 M 3D   6M   ∆t  &&
Fˆ eff = F0+∆t +  2 + U 0 +  + 2 D U& 0 +  2 M + D U 0
 ∆t ∆t   ∆t   2 
The value for U 0+ ∆t is obtained after the system (A.127) is solved, with which the values
for U 0− ∆t and U 0−2 ∆t can be obtained by means of the equations (A.123) and (A.124),
respectively. Note that the equations in (A.128) are the same equations obtained by linear
acceleration method, (see equations in (A.90)).

University of Castilla-La Mancha Draft By: Eduardo W. V. Chaves (2015)


Ciudad Real – Spain
ANNEX A: NUMERICAL INTEGRATION OVER TIME 853

A.6.6.1 Houbolt’s Method Scheme

I. Initial Parameters
I.1. Construction of M , D , K .
I.2. Given the boundary conditions U 0 , U& 0 , obtain U&& 0 :
(
U&&0 = M −1 F0 − DU& 0 − KU 0 )
I.3. Calculate the matrices:
6 3
Kˆ eff = 2 M + D+K
∆t ∆t
 6 M 3D   6M   ∆t  &&
Fˆ eff = F0+∆t +  2 + U 0 +  + 2 D U& 0 +  2 M + D U 0
 ∆t ∆t   ∆t   2 
I.4. Solve the system:
Kˆ eff U 0 + ∆t = Fˆ eff
I.5. Calculate the vectors U 0− ∆t , U 0−2 ∆t and U& 0+ ∆t , U&& 0+ ∆t :
U 0− ∆t = ∆t 2U&& 0 + 2U 0 − U 0+ ∆t y U 0− 2 ∆t = 6∆tU& 0 + 6∆t 2U&& 0 − 8U 0+ ∆t + 9U 0
U 0−∆t = ∆t 2U&&0 + 2U 0 − U 0+ ∆t ; U 0−2 ∆t = 6 ∆tU& 0 + 6∆t 2U&&0 − 8U 0+ ∆t + 9U 0
1
U& 0+ ∆t = [11U 0+∆t − 18U 0 + 9U 0−∆t − 2U 0−2∆t ] ; U&&0+∆t = 1 2 [2U 0+∆t − 5U 0 + 4U 0−∆t − U 0−2 ∆t ]
6 ∆t ∆t
I.6. Calculate the effective matrix K eff :
2 11
K eff = M+ D+K
∆t 2
6∆t
I.7. Update the variables:
U t − 2 ∆t ← U 0 − ∆t ; U t − ∆t ← U 0 ; U t ← U 0 + ∆t
II. For each time step t + ∆t do
II.1. Obtain the effective force vector:
 5 M 3D   4 M 3D   M D 
F eff = Ft +∆t +  2 + U t −  2 + U t −∆t +  2 + U t −2 ∆t
 ∆t ∆t   ∆t 2 ∆t   ∆t 3∆t 
II.2. Solve the system:
K eff U t + ∆t = F eff
II.3. Calculate the vectors U& t + ∆t , U&& t + ∆t :
1
U& t + ∆t = [11U t +∆t − 18U t + 9U t −∆t − 2U t −2∆t ] ; U&&t +∆t = 1 2 [2U t +∆t − 5U t + 4U t −∆t − U t −2∆t ]
6 ∆t ∆t
II.4. Update tje variables:
U t − 2 ∆t ← U t − ∆t ; U t − ∆t ← U t ; U t ← U t + ∆t
If it is the case Ft + ∆t ← F (t + ∆t , U t + ∆t ,U& t + ∆t , U&&t + ∆t ,...)
Go to step II.1 with t + ∆t .

University of Castilla-La Mancha Draft By: Eduardo W. V. Chaves (2015)


Ciudad Real – Spain
854 SOLVING PROBLEMS BY MEANS OF CONTINUUM MECHANICS

A.6.7 Hilber-Hughes-Taylor’s Method (HHT)


The HHT method adopts the same displacement and velocity approximations used by
Newmark’s method. The difference between these two methods is how they treat the
dynamic equilibrium equation:
1 
U t +∆t = U t + ∆tU& t +  − β H ∆t 2U&&t + β H ∆t 2U&&t + ∆t Hilber-
2  Hughes-
U& = U& + (1 − γ ) ∆tU&& + γ ∆t U&&
(A.129)
t + ∆t t H t H t + ∆t Taylor’s
MU&&t + ∆t + (1 + α H ) DU& t + ∆t − α H DU& t + (1 + α H )KU t + ∆t − α H KU t = Ft + ∆t Method

with α H < 0 .
By means of the displacement U t + ∆t , given by the equation (A.129), we can obtain U&& t + ∆t as
follows:
1  
U&&t + ∆t = (U t + ∆t − U t ) − 1 U& t −  1 − 1U&&t (A.130)
β H ∆t 2
β H ∆t  2β H 
By substituting the equation (A.130) ino the velocity equation U& t + ∆t , given by (A.129), we
can obtain:
γ  γ &  γ  &&
U& t + ∆t = H (U t + ∆t − U t ) + 1 − H U t + 1 − H  ∆tU t (A.131)
β H ∆t  βH   2β H 
By substituting (A.130) and (A.131) into the dynamic equilibrium equation (A.129), we can
obtain:
K eff U t + ∆t = F eff (A.132)
where
1 (1 + α H ) γ H
K eff = M+ D + (1 + α H ) K
β H ∆t 2
β H ∆t
 M (1 + α H ) γ H 
F eff = Ft + ∆t +  + D + α H K U t + (A.133)
 β H ∆t
2
β H ∆t 
 M  γ (1 + α H )   &  1   γ  
+ − 1 − H  D U t +  − 1 M − (1 + α H )1 − H ∆tD U&&t
 β H ∆t  βH    2 β H   2β H  
For th particular case when α H = 0 we fall back into the Newmark’s method. Besides, this
method has second-order accuracy and is unconditionally stable when:
−1 1 1
≤ α H ≤ 0 ; γ H = (1 − 2α H ) ; β H = (1 − α H ) 2 (A.134)
3 2 4
The smaller α H greater the numerical damping is. The numerical damping is small for a
low frequency response and will be high for high frequency response.

University of Castilla-La Mancha Draft By: Eduardo W. V. Chaves (2015)


Ciudad Real – Spain
ANNEX A: NUMERICAL INTEGRATION OVER TIME 855

A.6.8 Bossak’s Method


The Bossak’s method uses the same displacement and velocity approaches used by
Newmark’s method. And for the Bossak’s method the dynamic equilibrium equation is
treated as follows:
1 
U t + ∆t = U t + ∆tU& t +  − β B  ∆t 2U&&t + β B ∆t 2U&&t + ∆t
 2 
&
U = U + (1 − γ )∆tU + γ ∆t U&&
& && Bossak’s method (A.135)
t + ∆t t B t B t + ∆t

(1 − α B )MU&&t +∆t + α B MU&&t + DU& t +∆t + KU t +∆t = Ft +∆t


When α B = 0 we fall back into the Newmark’s method.
The stability condition is met when:
1 γB 1 1
αB ≤ ; βB ≥ ≥ ; αB + β B ≥ (A.136)
2 2 4 2
Using the displacement vector U t + ∆t , given by (A.135), we can obtain the acceleration
vector U&& t + ∆t :
1  
U&&t + ∆t = (U t + ∆t − U t ) − 1 U& t −  1 − 1U&&t (A.137)
β B ∆t 2
β B ∆t  2β B 
By substituting the equation (A.137) into the velocity equation U& t + ∆t , given by (A.135), we
can obtain:
γ  γ &  γ  &&
U& t + ∆t = B (U t + ∆t − U t ) + 1 − B U t + 1 − B  ∆tU t (A.138)
β B ∆t  βB   2β B 
By substituting (A.137) and (A.138) into the dynamic equilibrium equation (A.135), we can
obtain:
K eff U t + ∆t = F eff (A.139)
where
(1 − α B ) γ
K eff = M + B D+K
β B ∆t 2
β B ∆t
 (1 − α B ) γ 
F eff = Ft + ∆t +  M + B D U t + (A.140)
 β B ∆t
2
β B ∆t 
 (1 − α B )  γ    1 − α B   γ  
+ M − 1 − B  D U& t +  − 1 M − 1 − B  ∆tD U&&t
 β B ∆t  β B    2 β B   2β B  

A.6.9 Generalized α Method


The generalized α method was introduced by Chung&Hulbert (1993), and considers the
following dynamic equilibrium equation:
MU&&t + ∆t −α m + DU& t +∆t −α + KU t +∆t −α = Ft + ∆t −α (t ) Generalized α method (A.141)
f f f

University of Castilla-La Mancha Draft By: Eduardo W. V. Chaves (2015)


Ciudad Real – Spain
856 SOLVING PROBLEMS BY MEANS OF CONTINUUM MECHANICS

where
U t + ∆t −α = (1 − α f )U t + ∆t + α f U t (A.142)
f

U& t + ∆t −α = (1 − α f )U& t + ∆t + α f U& t (A.143)


f

U&&t + ∆t −α m = (1 − α m )U&&t + ∆t + α m U&& t (A.144)


t = (1 − α f )(t + ∆t ) + α f t (A.145)
with
1 
U t + ∆t = U t + ∆tU& t +  − β  ∆t 2 U&& t + β ∆t 2 U&& t + ∆t (A.146)
2 
U t + ∆t = U t + (1 − γ )∆tU&& t + γ∆tU&&t + ∆t
& & (A.147)
Note that the displacement and velocity vectors are the same used for Bossak and HHT
methods. Similarly for these methods we can express U&& t + ∆t and U& t + ∆t as follows:

U&&t + ∆t =
1
(U t + ∆t − U t ) − 1 U& t −  1 − 1U&&t (A.148)
β ∆t 2
β ∆t  2β 
γ
U& t + ∆t = (U t + ∆t − U t ) + 1 − γ U& t + 1 − γ ∆tU&&t (A.149)
β ∆t  β  2β 
By substituting (A.142), (A.143), (A.144), and (A.148), (A.149) into the equation (A.141) we
can obtain:
K eff U t + ∆t = F eff (A.150)
where
(1 − α m ) (1 − α f ) γ
K eff = M+ D + (1 − α f ) K
β∆t 2
β∆t
 (1 − α m ) (1 − α f ) γ 
F eff = Ft +α +  M+ D − α f K U t +
f
 β∆t
2
β∆t 
 (1 − α m )  γ (1 − α f )   &  (1 − α m )   γ  
+ M − 1 −  D U t +  − 1 M − (1 − α f )1 − ∆tD U&&t
 β ∆t  β    2 β   2 β  
(A.151)
We can verify that when α f = α m = 0 we fall back into the equations obtained for
Newmark’s method, (see equations in (A.57)). When α f = 0 and α m = α B we fall back
into the Bossak’s method, (see equations in (A.140)). When α m = 0 we fall back into the
HHT method.

A.6.10 Park-Housner’s Method


The Park-Housner’s method is a semi-implicit method. The mass matrix ( M ) is a diagonal
matrix and the stiffness matrix K is split into triangular matrices, i.e. K = K L + K U , in
which K L = K UT . The scheme for this method is presented next:
Construction of K and the diagonal mass matrix M ;

University of Castilla-La Mancha Draft By: Eduardo W. V. Chaves (2015)


Ciudad Real – Spain
ANNEX A: NUMERICAL INTEGRATION OVER TIME 857

Construction of the matrices K = K L + K U


Obtain the matrices:
L = M (1 + αβ ∆t 2 M −1 K L ) Q = 1 + αβ ∆t 2 M −1 K U
;
g t + ∆t = αβ ∆t 2 [β ft + ∆t + (1 − β ) ft ] + M (U t + β ∆tU& t )
Solve the system:
Lyt + ∆t = g t + ∆t ; QU t*+ ∆t = yt + ∆t
Update the variables

U t + ∆t ←
1
β
[U *
t + ∆t − (1 − β )U t ]
U& t + ∆t ← [U t + ∆t − U t ] − (1 − α) U& t
α∆t α

A.6.11 Trujillo’s Method


Trujillo (1977) presented a semi-implicit method, and applied to solve a linear structural
dynamic problem. The Trujillo’s method separates the stiffness and damping matrices into
two upper and lower matrices:
K = K L + KU ; D = DL + DU (A.152)
The mass matrix ( M ) is a diagonal matrix.
Trujillo’s Method Scheme:
Backward substitution
 ∆t ∆t 2  ∆t  ∆t ∆t 2 
U 1 = K (−11)  M + DL −

K U U t +  M + ( DL − DU )U& t + [Ft +∆t + Ft ]
t+
2  2 8  2  4  16 
with
 ∆t ∆t 2 
K (1) =  M + DL + KL
 2 8 

4  
U& 1 = U 1 − U t  − U& t
t+
2
∆t  t + 2 
Forward Substitution
 ∆t ∆t 2  ∆t  ∆t & ∆t 2 
U t +1 = K (−21)  M + DU − K L U 1 +  M + ( DU − D L ) U
 t+ 1 + [Ft + ∆t + Ft ] 
 2 8  2 2
t +  4  2 16 
with
 ∆t ∆t 2 
K ( 2) =  M + DU + KU 
 2 8 

4  
U& t +1 = U t +1 − U 1  − U& 1
∆t  t+

2
t+
2

University of Castilla-La Mancha Draft By: Eduardo W. V. Chaves (2015)


Ciudad Real – Spain
858 SOLVING PROBLEMS BY MEANS OF CONTINUUM MECHANICS

A.7 Examples

Let us consider the mechanical model with on degree-of-freedom, (see Figure A.12). This
mechanical model is made up by a mass body m which is connected by two devices,
namely: spring (Structural), which is characterized by spring constant k ; and by a dashpot
with viscosity d (Damping), which is responsible for the system energy dissipation. The
syste is conservative if d = 0 .

u , u& , u&&
k
FS = ku

FI = mu&& F (t )
m
FD = du&
c

Figure A.12: Mechanical model.

The mechanical problem proposed here has three forces, namely: The inertial force
FI = mu&& , where u&& ≡ a is the acceleration; FD is the damping force; and FS is the spring
force associated with the structural stiffness.
The governing equation for this problem, (see Figure A.12), is obtained by force
equilibrium:
FI + FD + FS = F (t ) (A.153)
or:
mu&& + du& + ku = F (t )
d 2u du (A.154)
m 2
+d + ku = F (t )
dt dt
If the body is free of external forces F (t ) = 0 , the governing equation is called free vibration.
The equation in (A.154) can also be expressed as follows:
d k F (t )
u&& + u& + u =
m m m
(A.155)
d F (t )
u&& + u& + ω2 u =
m m
where we have defined the parameter:

University of Castilla-La Mancha Draft By: Eduardo W. V. Chaves (2015)


Ciudad Real – Spain
ANNEX A: NUMERICAL INTEGRATION OVER TIME 859

k
ω= [ω] ≡ rad / s Natural circular frequency (A.156)
m

A.7.1 Oscillatory Motion


The mechanical model that represents an oscillatory motion is made up by a body mass m
connected to the spring with constant k , (see Figure A.13).

Figure A.13: Mechanical model for an oscillatory problem.

Oscillatory motion is a conservative system, since there is no energy dissipation. The


governing equation is represented mathematically by:
mu&& + ku = F = 0 (A.157)
with F = 0 (free vibration).
Consider as example, m = 26 , k = 21000 , and F = 0 . As boundary and initial conditions we
have:
u (t = 0) ≡ u 0 = 2 ; u& (t = 0) ≡ u& 0 = −3 (A.158)
The exact solution is given by the following harmonic function:
u& 0 k
u (t ) = sin(ωt ) + u 0 cos(ωt ) where ω= (A.159)
ω m
We also define the following parameters for the model:
Natural frequency:
k
ω Natural frequency (A.160)
f = = m ≈ 2.52 Hz
2π 2π
Natural period:
1
T= ≈ 0.22108 sec Natural period (A.161)
f

By means of numerical integration we present the results using the time increment
∆t = 0.01 .

University of Castilla-La Mancha Draft By: Eduardo W. V. Chaves (2015)


Ciudad Real – Spain
860 SOLVING PROBLEMS BY MEANS OF CONTINUUM MECHANICS

see Figure A.15 see Figure A.16

Figure A.14: Displacement vs. time curve ( ∆t = 0.01 ).

Exact

Trujillo

Newmark γ = 1
2 ;β = 1
6

Trapecio

Figure A.15: Displacement vs. time curve[0.84:0.93], ( ∆t = 0.01 ).

University of Castilla-La Mancha Draft By: Eduardo W. V. Chaves (2015)


Ciudad Real – Spain
ANNEX A: NUMERICAL INTEGRATION OVER TIME 861

Exact

Trujillo-Newmark

Trujillo

Newmark γ = 1
2 ;β = 1
6

Trapecio

Figure A.16: Displacement vs. time curve [1.72:1.82], ( ∆t = 0.01 ).

A.7.2 Free Vibration with Damping


Let us consider a mechanical model, (see Figure A.17), which represents the free vibration
problem ( F (t ) = 0 ) with damping ( d ≠ 0 ).

d k

Figure A.17: Mechanical model for free vibration with damping.

As previously seen the governing equation is given by:


d k
mu&& + du& + ku = 0 ⇒ u&& + u& + u = 0 (A.162)
m m
By assuming that u (t ) = C1e st , the above equation becomes:

University of Castilla-La Mancha Draft By: Eduardo W. V. Chaves (2015)


Ciudad Real – Spain
862 SOLVING PROBLEMS BY MEANS OF CONTINUUM MECHANICS

d k
s2 + s+ =0 (A.163)
m m
whose solutions are given by:
2
d  d  k (A.164)
s1, 2 = − ±   −
2m  2 m  m
We could have three possibilities, namely: radicand equals zero (two identical solutions);
radicand greater than zero (two different solutions); radicand less than zero (two complex
solutions).
ƒ Radicand equals zero (Critical damping)
In this case we have:
2
 d  k d k
  − =0 ⇒ = =ω ⇒ d = 2mω (A.165)
 2m  m 2m m
In this situation the damping coefficient is called critical damping coefficient:

d c = 2mω critical damping coefficient (A.166)

In general, we define a parameter called damping factor ( ζ ) which is used to indicate


whether the system is underdamping ( ζ < 1 , subcritical damping) or overdamping ( ζ > 1 ,
supercritical damping). This damping factor is defined by:

d d
ζ= = Damping factor (A.167)
d c 2mω

Note that the equation in (A.163) can be rewritten as follows:


s 2 + 2ωζs + ω 2 = 0 (A.168)
whose solutions are given by:

s1, 2 =  − ζ ± ζ 2 − 1 ω (A.169)


 
For the critical damping case, i.e ζ = 1 , we have that s1, 2 = −ζω .
The exact solution for the differential equation (A.162) is given by:
u (t ) = e − ωt {[u& 0 + ωu 0 ] t + u 0 } with (u& 0 ≠ 0; u 0 ≠ 0) (A.170)
ƒ Overcritical damping ζ > 1
In this situation we have two different solutions (and real numbers) given by(A.169). And
the solution for the differential equation (A.162) becomes:
− s1t − s2t
u (t ) = Ae + Be (A.171)
where

u&0 +  ζ + ζ 2 − 1 ωu0 − u&0 −  ζ − ζ 2 − 1 ωu0


A=   ; B=   (A.172)
2 2
2ω ζ − 1 2ω ζ − 1

University of Castilla-La Mancha Draft By: Eduardo W. V. Chaves (2015)


Ciudad Real – Spain
ANNEX A: NUMERICAL INTEGRATION OVER TIME 863

ƒ Subcritical damping ζ < 1


In this case the solution for the equation (A.169) is given by:

s1, 2 =  − ζ ± i 1 − ζ 2 ω (A.173)


 
And the solution for the differential equation (A.162) becomes:

u (t ) = e −ζω t  A sin  ωt 1 − ζ 2  + B cos ωt 1 − ζ 2   (A.174)


     
or:
 u& + ζωu 0 
u (t ) = e −ζω t  0 sin (ω d t ) + u 0 cos(ω d t ) (A.175)
 ωd 

where ω d = ω 1 − ζ 2

A.7.2.1 Free Vibration with Damping Example

As example, consider that m = 0.0052 , d = 0.1 , k = 12 , and boundary and initial conditions:
u 0 = 1 .5 ; u&0 = 0 (A.176)
By means of numerical integration we present the results using the time increment
∆t = 0.017 .

see Figure A.19

Figure A.18: Displacement vs. time curve, ( ∆t = 0.017 ).

University of Castilla-La Mancha Draft By: Eduardo W. V. Chaves (2015)


Ciudad Real – Spain
864 SOLVING PROBLEMS BY MEANS OF CONTINUUM MECHANICS

Exact

Trujillo-Newmark

Trujillo

Newmark γ = 1
2 ;β = 1
6

Trapecio

Figure A.19: Displacement vs. time curve [0.2:0.4], ( ∆t = 0.017 ).

University of Castilla-La Mancha Draft By: Eduardo W. V. Chaves (2015)


Ciudad Real – Spain
ANNEX A: NUMERICAL INTEGRATION OVER TIME 865

A.7.3 Miscellaneous Examples

A.7.3.1 Pendulum

Let us consider the pendulum problem, (see Figure A.20).

L θ

u& = Lθ&

mg

Figure A.20: Pendulum.

The governing equation for this problem is given by:


mL2&θ& + mgL sin(θ) + dLθ& = 0 (A.177)
The above equation can be rewritten as follows:
d mg
m&θ& + θ& = − sin(θ) = F (t , θ) (A.178)
L L
We consider the parameter values: L = 10.0 , m = 1.0 , d = 2.0 , g = 10 , and boundary and
initial conditions:
θ(t = 0) = 0 ; θ& (t = 0) = 3 (A.179)
By means of numerical integration we present the results using the time increment
∆t = 0.05 .

University of Castilla-La Mancha Draft By: Eduardo W. V. Chaves (2015)


Ciudad Real – Spain
866 SOLVING PROBLEMS BY MEANS OF CONTINUUM MECHANICS

see Figure A.22

Figure A.21: Displacement vs. time curve, ( ∆t = 0.05 ).

Exact

Trujillo-Newmark

Newmark γ = 1
2 ;β = 1
6

Figure A.22: Displacement vs. time curve [15:20], ( ∆t = 0.05 ).

University of Castilla-La Mancha Draft By: Eduardo W. V. Chaves (2015)


Ciudad Real – Spain
ANNEX A: NUMERICAL INTEGRATION OVER TIME 867

Figure A.23: Velocity vs. time curve, ( ∆t = 0.05 ).

University of Castilla-La Mancha Draft By: Eduardo W. V. Chaves (2015)


Ciudad Real – Spain
868 SOLVING PROBLEMS BY MEANS OF CONTINUUM MECHANICS

A.7.3.2 Dynamics of the van der Pol Equation

Let us consider the differential equation:


&ν& + α(ν 2 − 1)ν& + ω 2 ν = 0 (A.180)
which is known as van der Pol oscillator. The above equation can be rewritten as follows:
&ν& − αν& + ω 2 ν = −αν 2 ν& = F (t , ν, ν& ) (A.181)
By considering the dynamic equilibrium equation:
d k F
mu&& + du& + ku = F ⇒ u&& + u& + u = (A.182)
m m m
d
and by comparing the equations (A.181) and (A.182) we can conclude that: = −α ,
m
k F (t , ν, ν& )
= ω2 , = −αν 2 ν& . Consider as example the values: m = 1 , d = −α = −8 ,
m m
k = ω 2 = 0.25 , and boundary and initial conditions:
ν (t = 0) ≡ ν 0 = 1 ; ν& (t = 0) ≡ ν&0 = 3 (A.183)
By means of numerical integration we present the results using the time increment
∆t = 0.01 .

see Figure A.25

Figure A.24: “Displacement” vs. time curve, ( ∆t = 0.01 ).

University of Castilla-La Mancha Draft By: Eduardo W. V. Chaves (2015)


Ciudad Real – Spain
ANNEX A: NUMERICAL INTEGRATION OVER TIME 869

Exact

Trujillo-Newmark

Newmark γ = 1
2 ;β = 1
6

Figure A.25: “Displacement” vs. time curve [50:70], ( ∆t = 0.01 ).

University of Castilla-La Mancha Draft By: Eduardo W. V. Chaves (2015)


Ciudad Real – Spain
870 SOLVING PROBLEMS BY MEANS OF CONTINUUM MECHANICS

see Figure A.27

Figure A.26: “Velocity” vs. time curve, ( ∆t = 0.01 ).

Exact

Trujillo-Newmark

Newmark γ = 1
2 ;β = 1
6

Figure A.27: “Velocity” vs. time [50:60], ( ∆t = 0.01 ).

University of Castilla-La Mancha Draft By: Eduardo W. V. Chaves (2015)


Ciudad Real – Spain
ANNEX A: NUMERICAL INTEGRATION OVER TIME 871

(see Figure A.29)

Figure A.28: “Acceleration” vs. time curve, ( ∆t = 0.01 ).

Exact

Trujillo-Newmark

Newmark γ = 1
2 ;β = 1
6

Figure A.29: “Acceleration” vs. time curve [53:56], ( ∆t = 0.01 ).

University of Castilla-La Mancha Draft By: Eduardo W. V. Chaves (2015)


Ciudad Real – Spain
872 SOLVING PROBLEMS BY MEANS OF CONTINUUM MECHANICS

A continuación presentamos resultados utilizando integración directa con un incremento


de tiempo igual a ∆t = 0.03 .

see Figure A.31

Figure A.30: “Displacement” vs. time curve, ( ∆t = 0.03 ).

Trujillo

Trujillo-Newmark

Exact

Newmark γ = 1
2 ;β = 1
6

Figure A.31: “Displacement” vs. time curve [53:60], ( ∆t = 0.03 ).

University of Castilla-La Mancha Draft By: Eduardo W. V. Chaves (2015)


Ciudad Real – Spain
ANNEX A: NUMERICAL INTEGRATION OVER TIME 873

By means of numerical integration we present the results using the time increment
∆t = 0.05 .

see Figure A.33

Figure A.32: “Displacement” vs. time curve, ( ∆t = 0.05 ).

Trujillo

Trujillo-Newmark

Exact

Newmark γ = 1
2 ;β = 1
6

Figure A.33: “Displacement” vs. time curve [53:60], ( ∆t = 0.05 ).

University of Castilla-La Mancha Draft By: Eduardo W. V. Chaves (2015)


Ciudad Real – Spain
874 SOLVING PROBLEMS BY MEANS OF CONTINUUM MECHANICS

In Figure A.34 we show the “displacement” vs. time curve for different values for α .

α = 0 .1

α = 0 .6 ≈ ω

α = 20

Figure A.34: “Displacement” vs. time curve.

University of Castilla-La Mancha Draft By: Eduardo W. V. Chaves (2015)


Ciudad Real – Spain
ANNEX A: NUMERICAL INTEGRATION OVER TIME 875

A.7.3.3 Forced Harmonic Response without Damping

Consider the differential equation:


mu&& + ku = F0 sin(Ωt ) (A.184)
where Ω is the excitation frequency. When Ω = ω , resonance phenomena appears.
As example consider that: m = 4.5 , k = 3500 , F0 = 100 , Ω = 18 , and boundary and initial
conditions:
u (t = 0) = 15 ; u& (t = 0) = 150 (A.185)
By means of numerical integration we present the results using the time increment
∆t = 0.01 .

see Figure A.36

Figure A.35: Displacement vs. time curve, ( ∆t = 0.01 ).

University of Castilla-La Mancha Draft By: Eduardo W. V. Chaves (2015)


Ciudad Real – Spain
876 SOLVING PROBLEMS BY MEANS OF CONTINUUM MECHANICS

Trujillo Exact

Trujillo-Newmark

Newmark γ = 1
2 ;β = 1
6

Figure A.36: Displacement vs. time curve [3:3.5], ( ∆t = 0.01 ).

Now consider that:


u (t = 0) = 0 ; u& (t = 0) = 0 (A.186)
besides we consider that Ω = ω . With these conditions we can note that the system enter in
resonance, (see Figure A.37).

Figure A.37: Displacement vs. time curve.

University of Castilla-La Mancha Draft By: Eduardo W. V. Chaves (2015)


Ciudad Real – Spain
ANNEX A: NUMERICAL INTEGRATION OVER TIME 877

Dynamics Structures References

CHAPRA, S.C. & CANALE, R.P. (1988). Numerical Method for engineering. McGraw-Hill.
CHUNG. J. & HULBERT (1993). A time integration algorithm for structural dynamics with
improved numerical dissipation: The Generalized- α method. Journal of Applied Mechanics.
Vol. 60. pp. 371-375.
COOK, R.D.; MALKUS, D.S. & PLESHA, M.E. (1989). Concepts and applications of finite element
analysis. John Wiley & Sons.
CRISFIELD, M.A.(1997). Non-linear finite element analysis of solids and structures. Vol.2. John
Wiley & Sons, England.
HILBER, H.; HUGHES, T.J.R. & TAYLOR, R. L. (1977). Improved numerical dissipation for
time integration algorithms in structural dynamics. Earthquake Engineering and Structural
Dynamics. Nº5, pp. 283-292.
PARK, K. & HOUSNER, J.M. (1982). Semi-implicit transient analysis procedures for
structural dynamics analysis. Int. J. Num. Meth. Eng. Vol. 18, pp. 609-622.
TEDESCO, J.M.; MCDOUGAL, W.G. & ROSS, C.A.(1998). Structural dynamics: theory and
applications. Addison Wesley Longman, Inc.
TRUJILLO, D.M. (1977). An unconditionally stable explicit algorithm for structural
dynamics. Int. J. Num. Meth. Eng. Vol. 11. pp. 1579-1592.
WOOD, W.L.; BOSSAK, M. & ZIENKIEWICZ, O.C. (1981). An alpha modification of
Newmark’s methods. Int. J. Num. Meth. Eng. Vol. 15. pp. 1562-1566.

University of Castilla-La Mancha Draft By: Eduardo W. V. Chaves (2015)


Ciudad Real – Spain
878 SOLVING PROBLEMS BY MEANS OF CONTINUUM MECHANICS

University of Castilla-La Mancha Draft By: Eduardo W. V. Chaves (2015)


Ciudad Real – Spain
B. Introduction to finite differences

Annex B
Introduction to
Finite Differences
B.1 Introduction

The finite difference method was the first technique that emerged to solve numerically partial
differential equations related to practical engineering problems. Today this technique is now
obsolete with respect to solving partial differential equations, for instance, to solve problems
related to beams, plates, flux, etc. But the finite difference technique is widely spread when we
are dealing with numerical integration over time (see Annex A).

B.2 The Finite Difference Method

Let us consider the function y = y (x) , and the derivative of y with respect to x is defined by:
dy ∆y y (x + ∆x ) − y ( x )
y′ ≡ = lim = lim (B.1)
dx ∆ x → 0 ∆x ∆x → 0 ∆x
where y ′ indicates the slope of the function at the point x , (see Figure B.1).

y ( x) yi yi′

xi x

Figure B.1: Derivative of a function.


642 SOLVING PROBLEMS BY MEANS OF CONTINUUM MECHANICS

When the term ∆x does not tend to zero but to a finite value, (see Figure B.2), the derivative
at the point xi can be defined in several ways. If we use the left neighbor point ( y i −1 ), left finite
difference, the first derivative can be approached as follows:
 ∆y  y − yi −1
yi′ L =   = i (B.2)
 ∆x i ∆x
Otherwise, if we used information of the right neighbor point ( y i +1 ), right finite difference, the
first derivative can be represented as follows:
 ∆y  y − y
yi′ R =   = i +1 i (B.3)
 ∆x i ∆x
where we have adopted the nomenclature y ( x i −1 ) = y i −1 , y ( xi ) = y i , y ( x i +1 ) = y i +1 . As we can
appreciate in Figure B.2, by using these techniques the first derivate is approximated to its
exact value, i.e. there is an error associated with it. When ∆x → 0 , the exact value for the first
derivative is achieved.

y yi′L -approximated (Left)

yi yi′ -exact value


y i −1
y i +1
y i′ C -approximated.
(Central)
y ( x)
yi′ R -approximated (Right)

∆x ∆x

xi −1 xi xi +1
x

Figure B.2: Derivative of a function by means of finite differences.

We can still raise another possibility to approach the first derivative of the function at the
point xi by using simultaneously the left and the right points, (central finite difference):
 ∆y  y − y i −1
y i′ C =   = i +1 (B.4)
 ∆x  i 2∆x
As we can appreciate in Figure B.2, the central finite difference approach is closer to the exact
value. Note that the central finite difference, for the first derivative, is the average of left and
right techniques:
 ∆y  yi′ R + yi′ L yi +1 − yi −1
  = = (B.5)
 ∆x  i 2 2∆x
Similarly we can obtain the derivatives for higher order, for example for the second derivative:

University of Castilla-La Mancha Draft By: Eduardo W. V. Chaves (2016)


Ciudad Real – Spain
ANNEX B: INTRODUCTION TO FINITE DIFFERENCES 643

y ( x + ∆x ) − y ( x ) y ( x ) − y ( x + ∆x )

d2y ∆  ∆y  ∆x ∆x (B.6)
= lim   = lim
dx 2 ∆x →0 ∆x  ∆x  ∆x → 0 ∆x
The left finite derivative:
L
 ∆2 y  ∆  ∆y  ∆  yi − yi −1  1  ∆yi ∆yi −1 
 2 =
 ∆x  ∆x  ∆x  = ∆x  ∆x  = ∆x  ∆x − ∆x 
 i
1  yi − yi −1 yi −1 − yi − 2  (B.7)
=  − 
∆x  ∆x ∆x 
yi − 2 yi −1 + yi − 2
=
∆x 2
The right finite derivative:
R
 ∆2 y  ∆  ∆y  ∆  yi +1 − yi  1  ∆yi +1 ∆yi 
 2 =
 ∆x  ∆x  ∆x  = ∆x  ∆x  = ∆x  ∆x − ∆x 
 
 i (B.8)
1  yi + 2 − yi +1 yi +1 − yi  yi + 2 − 2 yi +1 + yi
=  − =
∆x  ∆x ∆x  ∆x 2
And by means of the central finite difference we can approach the second derivative as
follows:
y i +1 − y i y i − y i −1
 ∆2 y  −
∆x ∆x y − 2 y i + y i −1 (B.9)
  = i +1
 ∆x 2  = ∆x ∆x 2
 i

B.2.1 Left Finite Differences

Next we will define an automatic way in order to obtain the operators ∆y, ∆2 y, L when we
use the left finite difference. As we have seen previously, for the first derivative we have
∆y = y i − y i −1 , (see equation (B.2)). If we want to obtain the operator for the second derivative
we use the points located at the left side of the point xi :
 ∆2 y  ∆  ∆y  ∆  y i − y i −1  ∆y i − ∆y i −1
 
 ∆x 2  = ∆x  ∆x  = ∆x  ∆x  = ∆x 2
(B.10)
 i  
By applying once more the left derivative definition we get ∆y i = y i − y i −1 and
∆y i −1 = y i −1 − y i − 2 and by substituting into the above equation we can obtain:

 ∆2 y  ∆y i − ∆y i −1 ( y i − y i −1 ) − ( y i −1 − y i − 2 ) ( y i − 2 y i −1 + y i − 2 )
 
 ∆x 2  = ∆x 2
=
∆x 2
=
∆x 2
(B.11)
 i
Then, we define the operator ∆2 y = y i − 2 y i −1 + y i − 2 for the left finite difference case. In
Figure B.3 we constructed a figure in order to obtain automatically the operators for higher
order operators.

University of Castilla-La Mancha Draft By: Eduardo W. V. Chaves (2016)


Ciudad Real – Spain
644 SOLVING PROBLEMS BY MEANS OF CONTINUUM MECHANICS

∆y ∆2 y ∆3 y ∆4 y

yi

∆y i

y i −1 ∆2 y i

∆y i −1 ∆3 y i
(−) y i −2 ∆2 y i −1 ∆4 y i

∆y i − 2 ∆3 y i −1

y i −3 ∆2 y i − 2

∆y i −3

yi −4

Figure B.3: Left finite differences.

For example, in order to obtain the operator ∆4 y by means of Figure B.3 we localize the term
∆4 y i and we subtract the values as follows:

( ) ( )
∆4 y = ∆3 y i − ∆3 y i −1 = ∆2 y i − ∆2 y i −1 − ∆2 y i −1 − ∆2 y i − 2 = ∆2 y i − 2∆2 y i −1 + ∆2 y i − 2
= (∆y i − ∆y i −1 ) − 2(∆y i −1 − ∆y i − 2 ) + (∆y i − 2 − ∆y i −3 )
= ∆y i − 3∆y i −1 + 3∆y i − 2 − ∆y i −3 (B.12)
= ( y i − y i −1 ) − 3( y i −1 − y i − 2 ) + 3( y i − 2 − y i −3 ) − ( y i −3 − y i − 4 )
= y i − 4 y i −1 + 6 y i − 2 − 4 y i −3 + y i − 4
With that we can define the fourth derivative by means of left finite difference as follows:
 ∆4 y  y i − 4 y i −1 + 6 y i − 2 − 4 y i −3 + y i − 4
 
 ∆x 4  = ∆x 4
(B.13)
 i

B.2.2 Right Finite Differences

Next we will define an automatic way in order to obtain the operators ∆y, ∆2 y, L when we
use the right finite difference. As we have seen previously, for the first derivative we have
∆y = y i +1 − y i , (see equation (B.3)). If we want to obtain the operator for the second derivative
we use the points located at the right side of the point xi :

University of Castilla-La Mancha Draft By: Eduardo W. V. Chaves (2016)


Ciudad Real – Spain
ANNEX B: INTRODUCTION TO FINITE DIFFERENCES 645

 ∆2 y  ∆  ∆y  ∆  y i +1 − y i  ∆y i +1 − ∆y i
 
 ∆x 2  = ∆x  ∆x  = ∆x  ∆x  = ∆x 2
(B.14)
 i  
By applying once more the right derivative definition we get ∆y i +1 = y i + 2 − y i +1 and
∆y i = y i +1 − y i and by substituting into the above equation we can obtain:

 ∆2 y  ∆y i +1 − ∆y i ( y i + 2 − y i +1 ) − ( y i +1 − y i ) ( y i + 2 − 2 y i +1 + y i )
 
 ∆x 2  = ∆x 2
=
∆x 2
=
∆x 2
(B.15)
 i
Then, we define the operator ∆2 y = y i + 2 − 2 y i +1 + y i for the right finite difference case. Note
that we only use the points on the right of the point xi . In Figure B.4 we constructed a figure
in order to obtain automatically the operators for higher order operators.

∆y ∆2 y ∆3 y ∆4 y

yi

∆y i

y i +1 ∆2 y i

∆y i +1 ∆3 y i
(−) yi +2 ∆2 y i +1 ∆4 y i

∆y i + 2 ∆3 y i +1

y i +3 ∆2 y i + 2

∆y i +3

yi +4

Figure B.4: Right finite differences.

For example, in order to obtain the operator ∆3 y by means of Figure B.4 it is enough to do:
∆3 y = ∆2 y i +1 − ∆2 y i = (∆y i + 2 − ∆y i +1 ) − (∆y i +1 − ∆y i ) = ∆y i + 2 − 2∆y i +1 + ∆y i
= ( y i +3 − y i + 2 ) − 2( y i + 2 − y i +1 ) + ( y i +1 − y i ) (B.16)
= y i +3 − 3 y i + 2 + 3 y i +1 − y i
With that we can define the third derivative by means of right finite difference as follows:
 ∆3 y  y i +3 − 3 y i + 2 + 3 y i +1 − y i
 
 ∆x 3  = ∆x 3
(B.17)
 i

University of Castilla-La Mancha Draft By: Eduardo W. V. Chaves (2016)


Ciudad Real – Spain
646 SOLVING PROBLEMS BY MEANS OF CONTINUUM MECHANICS

B.2.3 Central Finite Differences


The central finite difference uses simultaneously the points on the left and on the right.
According to Figure B.4 we can define an automatic way in order to obtain the operators
∆y, ∆2 y, L when we use the central finite difference.

∆y ∆2 y ∆3 y ∆4 y

yi+2

∆y i + 3 / 2

yi +1 ∆yi +1 ∆2 y i +1

∆y i +1 / 2 ∆3 y i +1 / 2

yi ∆yi ∆2 y i ∆3 yi +1 ∆4 y i

∆y i −1 / 2 ∆3 y i −1 / 2

y i −1 ∆yi −1 ∆2 y i −1

∆y i −3 / 2

y i −2

Figure B.5: Central finite differences.

In Figure B.5 the term ∆y i + 3 / 2 characterizes finite difference taking at the point between xi +1
and xi + 2 . For example, to obtain the first derivative, we localize the term ∆y i in Figure B.5,
such term is between ∆y i +1 / 2 and ∆y i −1 / 2 and we take the average:
∆y i +1 / 2 + ∆y i − 2 ( y i +1 − y i ) + ( y i − y i −1 ) y i +1 − y i −1
∆y i = = =
2 2 2
(B.18)
 ∆y  y − y i −1
⇒   = i +1
 ∆x  i 2∆x

According to Figure B.5 we can represent the second derivative ∆2 y i = ∆y i +1 / 2 − ∆y i − 2 , then:


∆2 y i = ∆y i +1 / 2 − ∆y i − 2 = ( y i +1 − y i ) − ( y i − y i −1 ) = y i +1 − 2 y i + y i −1
 ∆2 y  y − 2 y i + y i −1 (B.19)
⇒  2  = i +1
 ∆x  i ∆x 2
In the same way the third derivative can be represented as follows:

University of Castilla-La Mancha Draft By: Eduardo W. V. Chaves (2016)


Ciudad Real – Spain
ANNEX B: INTRODUCTION TO FINITE DIFFERENCES 647

∆3 y i +1 / 2 + ∆3 y i − 2 (∆2 y i +1 − ∆2 y i ) + (∆2 y i − ∆2 y i −1 )
∆3 y i = =
2 2
∆ y i +1 − ∆ y i −1 [∆y i +3 / 2 − ∆y i +1 / 2 ] − [∆y i −1 / 2 − ∆y i −3 / 2 ]
2 2
= =
2 2 (B.20)
[( y − y i +1 ) − ( y i +1 − y i )] − [( y i − y i−1 ) − ( yi −1 − y i −2 )]
= i+2
2
y i + 2 − 2 y i +1 + 2 y i −1 − y i − 2
=
2
thus
 ∆3 y  y i + 2 − 2 y i +1 + 2 y i −1 − y i − 2
 
 ∆x 3  = 2∆x 3
(B.21)
 i
Notice that when we are applying the central finite differences for derivatives of odd order it
appears 2 in the denominator.
NOTE: For the finite differences of even order, e.g. ∆2 y, ∆4 y, ∆6 y, L , the coefficients are the
same as those for the binomial expression (a − b) n , for instance
(a − b) 2 = 1a 2 − 2ab + 1b 2 (B.22)
and the coefficients are (1,−2,1) . Another example
(a − b) 4 = 1a 4 − 4a 3 b + 6a 2 b 2 − 4ab 3 + 1b 4 (B.23)
and the coefficients are (1,−4,6,−4,1)

B.3 Finite Differences to Partial Derivatives

Let us consider now the function z = z ( x, y ) . The partial derivatives can be approached by
means of Central Finite Differences as follows:
 ∂z  z i +1, j − z i −1, j ∂ 2 z z i +1, j − 2 z i , j + z i −1, j
  ≈ ; ≈
 ∂x  i , j 2∆x ∂x 2 ∆x 2
(B.24)
 ∂z  z i , j +1 − z i , j −1 ∂ 2 z z i , j +1 − 2 z i , j + z i , j −1
  ≈ ; ≈
 ∂y  i , j 2∆y ∂y 2 ∆y 2

with that we can also obtain:


 ∂2z  ∂ ∂z ∂  z i +1, j − z i −1, j  1 ∂ ∂ 
  =   ≈
 ∂y∂x    =  z i +1, j − (
z i −1, j) ( )
  i , j ∂y  ∂x  ∂y  2∆x  2∆x  ∂y ∂y 
(B.25)
 ∂2z 
 
1
(
 ∂y∂x  ≈ 4∆x∆y z i +1, j +1 − z i +1, j −1 − z i −1, j +1 + z i −1, j −1 )
  i, j
If we consider the domain discretized into points, (see Figure B.6), we can represent the
 ∂2z 
partial derivative   in operator form as indicated in Figure B.7, in which ∆x = h ,
∂y∂x 
  i, j
∆y = k .

University of Castilla-La Mancha Draft By: Eduardo W. V. Chaves (2016)


Ciudad Real – Spain
648 SOLVING PROBLEMS BY MEANS OF CONTINUUM MECHANICS

y
i−2 i −1 i i +1 i+2

j+2
∆y

j +1
i, j + 1
∆y

j
i − 1, j i, j i + 1, j
∆y
j −1
i, j − 1
∆y

j−2
∆y

x
∆x ∆x ∆x ∆x ∆x

Figure B.6: Domain discretization.

−1 0 1

 ∂2 z 
4hk   =
 0 0 0
 ∂y∂x i, j i, j

1 0 −1

Figure B.7

In the same fashion we can represent


 ∂4z  ∂ 2 z  ∂ 2  z i +1, j − 2 z i , j + z i −1, j 
 ∂2
 =
 
 ∂y 2 ∂x 2  ∂x 2  = ∂y 2 
 ∂y 2 h2

 
   
(B.26)
 ∂ z 
4
1  z i +1, j +1 − 2 z i +1, j + z i +1, j −1 − 2 z i , j +1 
⇒  2 2  = 2 2  
 + 4 z i , j − 2 z i , j −1 + z i −1, j +1 − 2 z i −1, j + z i −1, j −1 
 ∂y ∂x  h k 

University of Castilla-La Mancha Draft By: Eduardo W. V. Chaves (2016)


Ciudad Real – Spain
ANNEX B: INTRODUCTION TO FINITE DIFFERENCES 649

The above equation can be represented if form of operator as the one indicated in Figure B.8.

1 −2 1

 ∂4z 
4h 2 k 2  2 2  = −2 4 −2
 ∂y ∂x  i , j i, j

1 −2 1

Figure B.8
 ∆2 y  y i +1 − 2 y i + y i −1
As we have seen previously the following is true  2  = , then, the partial
 ∆x  i ∆x 2
derivative can be written as follows:
 ∂2z  z − 2 z i , j + z i −1, j
  = i +1, j (B.27)
 ∂x 2  ∆x 2
  i, j
In the same fashion we can obtain
 ∂2z  z i , j +1 − 2 z i , j + z i , j −1
 
 ∂y 2  = ∆y 2
(B.28)
  i, j

with that, the Laplacian ∇ 2 z becomes:


 ∂2z   ∂2z  z i , j +1 − 2 z i , j + z i , j −1 z i +1, j − 2 z i , j + z i −1, j
∇ 2 z =  2  +  2  ≈ + (B.29)
 ∂x  i , j  ∂y  i , j ∆y 2 ∆x 2

Example

Let us consider the following partial differential equation


p
∇2 z = − (B.30)
S
where z represents the membrane deflection under the pressure p , in which the membrane
deflection on the boundary is zero. The square cross section has side b = 6h as indicated in
Figure B.9. Obtain the membrane deflection z in the cross section.

University of Castilla-La Mancha Draft By: Eduardo W. V. Chaves (2016)


Ciudad Real – Spain
650 SOLVING PROBLEMS BY MEANS OF CONTINUUM MECHANICS

Solution:
We can use the symmetry of the cross section and analyze only a quarter of the section.
Besides, in this quarter there are points with the same displacement, with that we will need to
analyze only the half of the quarter, (see Figure B.9).

z=0 z=0 z=0


z=0
h
1 2 3 2
z=0
h
2 4 5 5
z=0
h
3 5 6 5
z=0
h
5
h

h h h h h h

Figure B.9: Discretization of the domain.

As seen previously, the Laplacian can be approached by means of the finite difference:
− h2 p
∇ 2 z ≈ z i , j +1 + z i , j −1 + z i +1, j + z i −1, j − 4 z i , j = (B.31)
S
where we have considered ∆x 2 = ∆y 2 = h 2 . The operator can be appreciated in Figure B.10.

 ∂4z  p
 2 2 = 1 −4 1 = −h 2
 ∂y ∂x  S
 i , j i, j

Figure B.10

University of Castilla-La Mancha Draft By: Eduardo W. V. Chaves (2016)


Ciudad Real – Spain
ANNEX B: INTRODUCTION TO FINITE DIFFERENCES 651

By applying the operator described in Figure B.10 to the points of the mesh ( 1,2, L ,6 ), (see
Figure B.9), we can construct the following set of equations:
 − 4 z1 + 2z2  1
 z − 4z 2 + z3 + z4  1
 1  
 + 2z2 − 4z3 + z5  − h 2 p 1
 =  (B.32)
 + 2z2 − 4z4 + 2z5  S 1
 z3 + 2z 4 − 4z5 + z6  1
  
 4z5 − 4 z 6  1

By restructuring the above set of equations we can obtain:


− 4 2 0 0 0 0   z1  1
 1 −4 1 1 0  
0 z 2   1
 
 0 2 −4 0 1 0   z 3  − h 2 p 1
   =  (B.33)
 0 2 0 −4 2 0 z 4  S 1
 0 0 1 2 − 4 1  z5  1
   
 0 0 0 0 4 − 4  z 6  1

By solving the above set of equations we can obtain:


h2 p h2 p h2 p
z1 = 0,95192 ; z 2 = 1,4035 ; z 3 = 1,53846
S S S
2 2
(B.34)
h p h p h2 p
z 4 = 2,1250 ; z 5 = 2,34615 ; z 6 = 2,59615
S S S

University of Castilla-La Mancha Draft By: Eduardo W. V. Chaves (2016)


Ciudad Real – Spain
652 SOLVING PROBLEMS BY MEANS OF CONTINUUM MECHANICS

University of Castilla-La Mancha Draft By: Eduardo W. V. Chaves (2016)


Ciudad Real – Spain
C. Incremental-Iterative strategy solution

Annex C
Incremental-Iterative
Strategy Solution

C.1 Introduction

For a better understanding of the behavior of structures and materials, it has increased the
demand to formulate algorithms that are able to realistically simulate the complete behavior
of the structure/material. Among one of the applications we can mention the simulation of
a structure to its complete destruction. In this way it allows us to design more efficiently
the structure in order to face a disaster (earthquakes, explosions, etc.). To achieve this
objective we must take into account non-linearity behavior. Basically we can highlight two
types of non-linearity:
ƒ Material Non-Linearity;
ƒ Geometric Non-Linearity.
The material non-linearity appears when the stress-strain relationship is non-linear. The
geometric non-linearity occurs when deformed configuration (current state) has great
influence in the outcome. When we are dealing with the Finite Element Formulation the
r
strain field ε ( x ) is related to the nodal displacements {u}( e ) by means the matrix which
contains the derivatives of the shape functions [B] , which in small deformation regime is
r
only a function of initial geometric parameters, ε( x ) = [ B]{u}(e ) . In the case of finite
deformation regime the matrix [B] is also a function of the displacement field, i.e.
r
ε ( x ) = [ B (u)]{u}( e ) . And as consequence the stiffness matrix is also a function of the
displacement field.
In general, the geometric non-linearity is a consequence of the large displacement that
undergoes the structure. Then, the measure of strain adopted must be able to capture the
real displacement of the structure. Several measures of strain have been established, e.g.
Green-Lagrange strain tensor, Almansi strain tensor, Logarithmic strain tensor, etc.
654 SOLVING PROBLEMS BY MEANS OF CONTINUUM MECHANICS

ε
F

Figure C.1: Material non-linearity.

e e
F F F
F
M = Fe M = F (e + u )
e
u

≡ ≡

a) Small deformation a) Large displacement

Figure C.2: Boundary conditions non-linearity.

C.1.1 Solution Strategies


To achieve the previous objectives, various solution techniques have been proposed. The
choice of numerical algorithm depends on the given problem. For example, if the structure
response is represented by a curve Force vs. Displacement, (see Figure C.3), the aim is to
obtain the complete curve of the graph in question. As we will see later, we can use a
strategy that is force increment (force control). But there may be a point, for instance, point A
of the graph, where the force control procedure will get a no desired point as solution
(point F ) or even a divergence of the solution. Another strategy used is through
incremental displacement (displacement control), which can also have undesirable solution if
we are at point B and to a further increment in displacement we reach the point D of the
graph missing all curve from B to D . Another strategy which can be employed is a
combination of the two above methodologies, i.e. force control and displacement control
simultaneously, which is known as Arc-length control.

University of Castilla-La Mancha Draft By: Eduardo W. V. Chaves (2016)


Ciudad Real – Spain
ANNEX C: INCREMENTAL-ITERATIVE STRATEGY SOLUTION 655

Force limit point

F Displacement limit point

A
Snap through F

B
Snap back

D
E
u

Figure C.3: Force-displacement curve.

Sometimes when we are using an incremental strategy it has an error associated with it or
even the solution can diverge, (see Figure C.4). To overcome this drawback we must use an
incremental-iterative scheme. Among the iterative methods we can mention the Newton-
Raphson’s method.

F obtained solution desired solution

u
Figure C.4: Diverting of the solution

University of Castilla-La Mancha Draft By: Eduardo W. V. Chaves (2016)


Ciudad Real – Spain
656 SOLVING PROBLEMS BY MEANS OF CONTINUUM MECHANICS

C.2 Total Potential Energy

The total potential energy ( Π ) of an elastic system is composed by two parts, namely:
ƒ Internal potential energy (strain energy potential ( U int ));
ƒ External potential energy ( U ext ):
Π ( z ) = U int ( z ) − U ext ( z ) (C.1)
If the equation (C.1) represents the stationary condition for the total potential energy, it can
be shown that the value of z is an extreme of Π ( z ) . This is the Principle of stationary of the
total potential energy. Then, if we looking for the extremes (maximums and minimums) of the
function, (see Figure C.5), the following must hold:
dΠ ( z )
=0 (C.2)
dz

Π( z)

dΠ ( z )
=0
dz
Π min

z
Figure C.5: Minimum of a function
Let us suppose a one-dimensional case in which U ext (u ) = Wz . With that the condition
(C.2) becomes:
dΠ ( z ) dU int ( z ) dU ext ( z ) dU int ( z )
= − = − W = B( z ) − W = 0
dz dz dz dz (C.3)
⇒ B( z ) = W

dU int ( z )
where we have considered B( z ) = . We can rewrite the above equation as follows:
dz
 B( z ) 
W =  z = K Sec z (C.4)
 z 
where K Sec is the secant of the curve W × z at the point z , (see Figure C.6).
Let us suppose that for a give increment ∆z we have:
B ( z + ∆z ) = W + ∆W (C.5)
∂B ( z )
In addition, by means of Taylor series the equation B( z + ∆z ) = B( z ) + ∆z + ... holds,
∂z
in which we have only considered linear terms. Then, the equation (C.5) can be rewritten as
follows:

University of Castilla-La Mancha Draft By: Eduardo W. V. Chaves (2016)


Ciudad Real – Spain
ANNEX C: INCREMENTAL-ITERATIVE STRATEGY SOLUTION 657

∂B ( z )
B ( z + ∆z ) = B ( z ) + ∆z = W + ∆W
∂z
∂B( z )
⇒ B( z ) − W + ∆z = ∆W
1424 3 ∂z (C.6)
=0

∂B ( z )
⇒ ∆z = ∆W
∂z
⇒ K T ∆z = ∆W
∂B ( z ) ∂ 2U int ( z )
where K tan = = is the tangent of the curve W × z at the point z , (see
∂z ∂z 2
Figure C.6).

W
K Sec
K tan
1 1
∆W W = K Sec z
W
∆z ∆W = K tan ∆z

z z + ∆z z

Figure C.6: Tangent vs. Secant.

C.2.1 Extension to n Dimensions


Suppose that the total potential energy is a function of n variables, i.e. Π = Π (u i ) for
i = 1,2,3, L , n . Then, we can obtain that
dΠ (u) dU int (u) dU ext ( u) dU int (u)
= − = − Fi = Bi (u) − Fi = 0
dui dui dui dui (C.7)
⇒ Bi (u) = Fi

With that we can defined the Secant stiffness matrix K Sec as follows:
 B (u)  Sec
Fi =  i  u j = K ij u j (C.8)
u
 j 
Note that we are using indicial notation. Similarly to obtain the equations in (C.6) we can
define the Tangent stiffness matrix K tan as follows:
∂Bi (u)
∆u j = ∆Fi ⇒ K ijtan ∆u j = ∆Fi (C.9)
∂u j

University of Castilla-La Mancha Draft By: Eduardo W. V. Chaves (2016)


Ciudad Real – Spain
658 SOLVING PROBLEMS BY MEANS OF CONTINUUM MECHANICS

where
∂Bi (u) ∂ 2U int (u)
K ijtan = = Tangent stiffness matrix (C.10)
∂u j ∂ui ∂u j

C.3 Example with 1 Degree-of-Freedom

In this subsection we will adopt the example in the book Crisfield (1991), (see Figure C.7),
where A stands for cross-section area of the bar and E represents the Young’s modulus.

Initial Configuration
L0

KS z0
θ0

Deformed Configuration
(current configuration) L z

KS z0
θ

Figure C.7: Example with 1 degree-of-freedom, (Crisfield(1991)).

Making the vertical equilibrium at the node in which the force W is applied, (see Figure
C.8), we can obtain:
( z 0 + z) ( z + z)
W = N sin θ + K S z = N + KS z ≅ N 0 + KS z (C.11)
L L0
where N is the axial force of the bar and K S is the spring coefficient. In Crisfield (1991)
more detail about this formulation is provided.

University of Castilla-La Mancha Draft By: Eduardo W. V. Chaves (2016)


Ciudad Real – Spain
ANNEX C: INCREMENTAL-ITERATIVE STRATEGY SOLUTION 659

W
W

Bar θ N cos θ θ
N
= N
N sin θ
KS z
KS z

Figure C.8: Equilibrium at the node.

Additionally we can state that:


 z  z  1 z  
2

N = σA = EAε = EA 0   +    (C.12)


 L0  L0  2  L0  

and

W = W ( z) =
EA 2
3
L0
[ ]
z 0 z + 32 z 0 z 2 + 12 z 3 + K S z (C.13)

For a given value of z we can calculate the tangent of the curve W = W (z ) as follows:

dW d  EA 2  EA 2
K tan = = ( 2 1 3
) (
3 2
 3 z 0 z + 2 z 0 z + 2 z + K S z  = 3 z 0 + 3z 0 z + 2 z + K S
dz dz  L0
3
) (C.14)
 L 0

We can also express K tan in terms of axial force as follows:


dW ( z 0 + z ) dN N
K tan = = + + KS
dz L0 dz L0
2
EA  z 0 + z  N
=   + + KS (C.15)
L0  L 0  L0
EA N
= 3
( z 0 + z) 2 + + KS
L0 L0
Next we will adopt values for z < 0 in order to draw the curve W × z , (see Figure C.9),
which represents the exact value of the function W = W (z ) .

University of Castilla-La Mancha Draft By: Eduardo W. V. Chaves (2016)


Ciudad Real – Spain
660 SOLVING PROBLEMS BY MEANS OF CONTINUUM MECHANICS

−W

W=
EA 2
L30
[ ]
z 0 z + 32 z 0 z 2 + 12 z 3 + K S z

Data
EA = 5.0 × 10 7
z 0 = 25
L0 = 2500
K S = 1.35

−z

Figure C.9: Function W ( z ) .

Now let us assume that we do not know the function W ( z ) , but instead we know its first
dW
derivative K tan = . In order to obtain the function W ( z ) we can adopt the Euler’s
dz
method, (see Annex A). With this method we can control the force W ( z ) throughout
increment of ∆W or we can control the displacement z throughout increments of ∆z ,
(see Figure C.10).

University of Castilla-La Mancha Draft By: Eduardo W. V. Chaves (2016)


Ciudad Real – Spain
ANNEX C: INCREMENTAL-ITERATIVE STRATEGY SOLUTION 661

Wn +1 = Wn + K ntan ∆z K ntan
W
Approximated value of W
Wn +1
} Error
∆W = K ntan ∆z
Exact value of W
Wn

∆z

zn z n +1 z

Figure C.10: Euler’s method (Implicit method).

For increments of ∆z we can apply:


Wn +1 − Wn
K ntan = ⇒ Wn +1 = Wn + K ntan ∆z (C.16)
∆z
And for increments of ∆W we can apply:

K ntan =
Wn +1 − Wn ∆W
∆z
=
∆z
⇒ (
∆z = K ntan )−1
∆W (C.17)

We will apply these two methodologies in order to obtain numerically the curve W × z . For
the first case we will impose a displacement increment of ∆z = 5.0 and for the second case
an increment in force equal to ∆W = 7.0 , (see Figure C.11). If we want a more accurate
solution we will need to adopt very small increments, but this procedure could cause a
time-consuming from a computational point of view when we are dealing with several
degrees-of-freedom. To overcome this drawback we can adopt the incremental-iterative
scheme, in which even with big increments we can obtain good results.
The procedure in FORTRAN code can be found at the link: NON_LINEAR1D.FOR,
(starting at label 20 for Displacement control and starting at label 30 for Load control).

When we are using force control the solution diverges when the force starts to decrease.
To illustrate this behavior let us consider same example, but changing the value of K S by
K S = −0.35 , (see Figure C.9). In this situation we can verify that when we are adopting the
force control the curve diverge, (see Figure C.12). For this example we have adopt a force
increment equal to ∆W = 2 .

University of Castilla-La Mancha Draft By: Eduardo W. V. Chaves (2016)


Ciudad Real – Spain
662 SOLVING PROBLEMS BY MEANS OF CONTINUUM MECHANICS

−W

Displacement control
∆z = 5 . 0

Force control
∆W = 7 . 0

Data
EA = 5.0 × 10 7
exact z 0 = 25
L0 = 2500
K S = 1.35

−z

Figure C.11: Incremental solution.

−W

force control ∆W = 2 (diverge)

displacement control ∆z = 5

exact
K S = −0.35

−z

Figure C.12: Incremental solution.

University of Castilla-La Mancha Draft By: Eduardo W. V. Chaves (2016)


Ciudad Real – Spain
ANNEX C: INCREMENTAL-ITERATIVE STRATEGY SOLUTION 663

C.3.1 Total Potential Energy


For the example given by Section C.3 the total potential energy is given by:
 1 L0 N 2 1 
Π( z) = U int
( z) − U ext
(z) =  ∫ dx + K S z 2  − (Wz ) (C.18)
 2 EA 2 
 0 
which is the same as:
L0 1
Π( z) = N 2 + K S z 2 − Wz (C.19)
2 EA 2
Note that the axial force depends on z , i.e. N = N ( z ) .
By applying the Principle of stationary of the total potential energy we can obtain:
dΠ ( z ) d  L0 1 
=  N 2 + K S z 2 − Wz  = 0
dz dz  2 EA 2 
L0 dN (C.20)
⇒ N + KS z − {W =0
EA44dz
1 2443 dU ext =
dU int dz
=
dz

By considering the equation in (C.12) we can obtain:


dN EA
= 2 [z 0 + z ] (C.21)
dz L0
And by substituting the equation (C.21) into (C.20) we can obtain:
L0 dN L0 EA
N + KS z − W = 0 ⇒ N 2 [z 0 + z ] + K S z − W = 0
EA dz EA L0
N
⇒ [z0 + z ] + K S z − W = 0 (C.22)
L0
N
⇒W = [z0 + z ] + K S z
L0
As expected, the above equation is the same as the one given by the equation in (C.11).
The tangent matrix can be obtained by means of:
d 2U int ( z ) d  N 
2
=  ( z0 + z ) + K S z 
dz dz  L0 
1 dN N
= ( z0 + z) + + KS (C.23)
L0 dz L0
1 EA
= 2
[z0 + z ]( z0 + z ) + N + K S
L0 L0 L0
which results in:
d 2U int ( z ) EA N
2
= K tan = 3 ( z0 + z ) 2 + + KS (C.24)
dz L0 L0
which matches the equation in (C.15).
Next, we will draw the curve Π × z . To do so, let us adopt the following values for the load
W , namely:

University of Castilla-La Mancha Draft By: Eduardo W. V. Chaves (2016)


Ciudad Real – Spain
664 SOLVING PROBLEMS BY MEANS OF CONTINUUM MECHANICS

z1 = −2.5 ⇒ W1 = −7.65
z 2 = −4.0 ⇒ W2 = −11.5824 (C.25)
z 3 = −5.0 ⇒ W3 = −13.95
where we have used the data of Figure C.9. By means of the total potential energy given by
equation (C.19) and with fixed values of W1 , W2 and W3 we can obtain, respectively, three
curves Π 1 , Π 2 and Π 3 , (see Figure C.13). As we can verify, for each potential function
the extreme (minimum) corresponds to the displacement associated with Wi .

Π 2 (W2 = −11.5824)

Π 1 (W1 = −7.65)

z1 = −2.5
dΠ1
=0
dz
Π 3 (W3 = −13.95)

dΠ 3
=0
dz

−z
z = −5
Figure C.13: Total potential ( K S = 1.35 ).

Let us now turn our attention to the example in Figure C.12, where we have considered
that K S = −0.35 . And we draw the total potential when the load is equal to W = −2 , (see
Figure C.15). If we look at Figure C.14 we can verify that for the value W = −2 we have
three solutions, namely z1 = −1.34047 , z 2 = −16.24045 and z 3 = −57.41908 , (see Figure
C.14), whose values are the roots of the cubic equation:
EA 2
L30
[ ]
z 0 z + 32 z 0 z 2 + 12 z 3 + K S z − W = 0


These roots correspond exactly to the extremes of Π , i.e. when = 0 , (see Figure C.15).
dz
We also emphasize that at z1 and at z 3 the tangent matrix is positive definite, while at z 2
the tangent matrix is negative definite, in this case indicating that the solution diverges for
force increment, but not for an increment of displacement (displacement control).

University of Castilla-La Mancha Draft By: Eduardo W. V. Chaves (2016)


Ciudad Real – Spain
ANNEX C: INCREMENTAL-ITERATIVE STRATEGY SOLUTION 665

−W

2
z1 z2 z3

−z

Figure C.14: Function W when K S = −0.35 .

Π (W = −2)

d 2 Π( z) dΠ ( z )
>0 =0
dz 2 dz

z3 −z

dΠ ( z )
Π =0
dz

d 2Π ( z )
<0
dz 2

d 2 Π( z)
>0 dΠ ( z )
dz 2 =0
dz

z1 −z
z2

Figure C.15: Total potential K S = −0.35 .

University of Castilla-La Mancha Draft By: Eduardo W. V. Chaves (2016)


Ciudad Real – Spain
666 SOLVING PROBLEMS BY MEANS OF CONTINUUM MECHANICS

C.4 Analyzing the Total Potential Energy

For the next analysis let us consider that we are at the equilibrium point and we add an
increment of force ∆W and we search for the next equilibrium point due to this new
increment. We know that the solution lies on an extreme point (minimum or maximum).
r
Let us consider the known displacement vector z 0 in n dimensions, which is represented
by its components z 0 i (i = 1,2,..., n) . Let us suppose that z i = z 0 i + β d i where d i
represents a vector, supposedly known, that minimizes or maximizes the total potential
energy. With that total potential energy is a function of β , i.e. Π ( z i ) = Π (β ) .
We apply the chain rule in order to obtain:
dΠ (β ) ∂Π (β ) ∂z i ∂Π (β )
= = di (C.26)
dβ ∂z i ∂β ∂z i
or in tensorial notation:
r
dΠ (β ) ∂Π (β ) ∂z ∂ Π (β ) r r
= r ⋅ = r ⋅ d = ∇ zr Π (β ) ⋅ d (C.27)
dβ ∂z ∂β ∂z
r ∂Π (β ) r
where g = r ≡ ∇ zr Π (β ) represents the gradient of Π (β ) in the space defined by z .
∂z
We rewrite the above equation as follows:
dΠ ( β ) r r dΠ ( β ) dΠ ( β )
= g ⋅d indicial
 → = gi di matrix
 form
→ = { g}T {d } (C.28)
dβ dβ dβ
r r r
where g = g (z ) is the residue vector.
Taking once again the derivative of the equation (C.26) we can obtain:
 
d2 Π ( β ) d  ∂Π ( β )   d ∂Π ( β ) ∂ Π ( β ) dd

=  d i  =  di + i 
dβ 2
dβ  ∂zi   dβ ∂z i ∂z i { dβ 
 (C.29)
 = 0i 

∂  dΠ ( β )  ∂  ∂Π ( β )  ∂ 2Π(β )
=   d i = d j di = di d j = d i S ij d j
∂z i  dβ  ∂z i  ∂z j 
 ∂zi ∂z j

or in tensorial notation:
d2 Π ( β ) r ∂ 2 Π ( β ) r r r d2 Π ( β )
= d ⋅ r r ⋅ d = d ⋅ S ⋅ d matrix
 form
→ = {d }T [ S ]{d } (C.30)
dβ 2 ∂z ⊗ ∂z dβ 2

where the matrix S is the Hessian of Π (β ) , and within the scope of structural analysis is
called tangent stiffness matrix.
Next, we will express Π ( z 0 i + β d i ) my means of Taylor series:
r r
r ∂Π ( z 0 ) 1 ∂ 2 Π(z0 ) 2
Π(z0i + βd i ) = Π(z0 ) + β+ β + ... (C.31)
∂β 2! ∂β 2

d Π (β ) d 2 Π (β )
and taking into account = g i d i and = d i S ij d j , the above equation becomes
dβ dβ 2

University of Castilla-La Mancha Draft By: Eduardo W. V. Chaves (2016)


Ciudad Real – Spain
ANNEX C: INCREMENTAL-ITERATIVE STRATEGY SOLUTION 667

r r r
Π ( z 0 + β d ) = Π ( z 0 ) + g i d i β + 0.5d i S ij d j β 2
r r r (C.32)
⇒ Π ( z 0 + β d ) − Π ( z 0 ) = ∆ Π = β ( g i d i + 0.5β d i S ij d j )
r r r r
tensorial
 → = ∆ Π = β ( g ⋅ d + 0 .5 β d ⋅ S ⋅ d )
(C.33)
matrix
 form
→ (
= ∆ Π = β { g}T {d } + 0.5 β {d }T [ S ]{d } )
We can deal with two scenarios:
Minimum
r r r r r r
Π (z0 + β d ) < Π (z 0 ) ⇒ Π ( z 0 + β d ) − Π ( z 0 ) = β ( g i d i + 0.5β d i S ij d j ) < 0
r r r r r r r r r r (C.34)
⇒ Π ( z 0 + β d ) − Π ( z 0 ) = β ( g ⋅ d + 0.5β d ⋅ S ⋅ d ) = β ( g + 0.5β d ⋅ S ) ⋅ d < 0
In this situation, S is positive definite, i.e. all its eigenvalues are positive, and as
consequence d i S ij d j > 0 . with that we can conclude that g i d i < 0 and is the predominate
term for small values of β > 0 :
r r
gi di < 0 tensorial
 → g⋅d < 0 (C.35)
Maximum
r r r r r r
Π(z0 + β d ) > Π(z0 ) ⇒ Π ( z 0 + β d ) − Π ( z 0 ) = β ( g i d i + 0.5β d i S ij d j ) > 0 (C.36)
In this case, S is definite negative, i.e. all its eigenvalues are negative, and as consequence
d i S ij d j < 0 . With that we can conclude that g i d i > 0 and it is the predominate term for
small values of β > 0 :
r r
gi di > 0 tensorial
 → g ⋅d > 0 (C.37)
For better illustration of the previous development, we will make an analogy where we
have a mountain and depression in which they are represented by its level curves, (see
Figure C.16). Recall that the gradient always points towards the growing sense of the
function and is normal to the level curves.
r r
If the parameters g and d are fixed, the value of β that minimize or maximize the
function ∆Π (β ) , (see equation (C.34)), is given by:
r r r r r r r
Π ( z0 + β d ) − Π ( z 0 ) = ∆Π = β ( g ⋅ d + 0.5β d ⋅ S ⋅ d )
d∆ Π r r r r
⇒ = g ⋅ d + βd ⋅ S ⋅ d = 0
dβ (C.38)
r r
− g ⋅d
⇒β = r r
d ⋅ S ⋅d
r r
Given a known state z 0 , now the question is: Which is the value of d in which we must
adopt in order to guarantee that we are reaching an extreme?
If we are looking for a minimum, it is enough to adopt a matrix A which is positive
r r r
definite, with that the equation g ⋅ A ⋅ g > 0 holds. Then, the vector d could be defined
such as:
r r
d = −A ⋅ g (C.39)
In this way we guarantee that:
r r r r
g ⋅ d = −g ⋅ A ⋅ g < 0 (C.40)

University of Castilla-La Mancha Draft By: Eduardo W. V. Chaves (2016)


Ciudad Real – Spain
668 SOLVING PROBLEMS BY MEANS OF CONTINUUM MECHANICS

The choice for the matrix A is what given origin for several iterative methods.
Taking into account that given a matrix which is positive definite its inverse is also definite
r r r r r
positive, we adopt A = S −1 , with that d = − A ⋅ g = − S −1 ⋅ g ⇒ g = − S ⋅ d . Then, we can
conclude that:
r r r r r r
− g⋅d − (− S ⋅ d ) ⋅ d d ⋅ S ⋅ d
β= r r= r r = r r =1 (C.41)
d ⋅ S ⋅d d ⋅ S ⋅d d ⋅ S ⋅d
With these conditions we obtain the Newton-Raphson’s method, in which the matrix A is
the inverse of the Hessian matrix (tangent stiffness matrix) A = S −1 .
r r r r
− g (k ) ⋅ d (k ) − g (k ) ⋅ d (k )
β = r ( k ) ( k ) r ( k ) = r ( k ) r ( k +1) r ( k )
(k )
(C.42)
d ⋅ S ⋅d d ⋅ (g −g )
r ( k ) r ( k +1) r ( k )
Note that S ( k ) ⋅d = g −g .

elevation
mountain

depression

Level curve
r
d r
d
∇φ ∇φ

r
r ∇φ = 0 maximum ∇φ ⋅ d > 0
minimum ∇φ ⋅ d < 0

Figure C.16: Level curves.

University of Castilla-La Mancha Draft By: Eduardo W. V. Chaves (2016)


Ciudad Real – Spain
ANNEX C: INCREMENTAL-ITERATIVE STRATEGY SOLUTION 669

r r r r r
g (1) = g ( z 0 + β d ) βd

r
g (1)

Min. r r
d (0) ∇φ( z 0 )

r
g (0)
r
minimum ∇φ ⋅ d < 0

Figure C.17

r
∇ φ( z ( 3 ) )

r
d ( 3)
r
Min. d (2) r
∇ φ( z ( 2 ) )
r
d (1)
r
g (1)
r
d (0) r
∇φ( z (1) )
r
minimum ∇φ ⋅ d < 0 r
g (0)
r
∇φ( z 0 )

Figure C.18: Iterative process.

University of Castilla-La Mancha Draft By: Eduardo W. V. Chaves (2016)


Ciudad Real – Spain
670 SOLVING PROBLEMS BY MEANS OF CONTINUUM MECHANICS

C.5 Classical Formulation of the Newton-Raphson’s Method

The Newton-Raphson’s is employed to obtain the roots of a function, i.e. f ( x) = 0 ,


(Chapra&Canale(1988)). Newton in 1669 has obtained a version of the method and
Raphson in 1690 has generalized the method.
The formulation of the Newton-Raphson’s method can be obtained by means of Taylor
series. For example, given the function f (x) we can approach the value of the function
f ( x + ∆x) by Taylor series as follows:

1 ∂f ( x 0 ) 1 ∂ 2 f ( x0 ) 2
f ( x + ∆x) = f ( x 0 ) + ∆x + ∆x + ...
1! ∂x 2! ∂x 2 (C.43)
≈ f ( x 0 ) + f ′( x 0 )∆x
where we have considered until linear terms. Let us adopt the following nomenclature:
xi +1 = xi + ∆x , ∆x = x i +1 − xi and for the application point xi = x 0 . With that the above
equation can be rewritten as follows:
f ( xi +1 ) = f ( xi ) + f ′( x i )∆x = f ( x i ) + f ′( xi )( x i +1 − x i ) (C.44)
Here the index (i) does not indicate indicial notation.
As we are searching for the roots of the function f ( xi +1 ) = 0 we can obtain:
f ( xi +1 ) = f ( xi ) + f ′( xi )( xi +1 − xi ) = 0
⇒ f ( xi ) + xi +1 f ′( xi ) − xi f ′( xi ) = 0
⇒ xi +1 f ′( xi ) = xi f ′( xi ) − f ( xi ) (C.45)
f ( xi )
⇒ xi +1 = xi −
f ′( xi )

Once xi +1 is obtained the values of the function ( f ( xi +1 ) ) and its derivative ( f ′( xi +1 ) ) at


the point can be obtained. This procedure is repeated until f ( xi +n ) ≈ 0 is reached. It can be
shown that the error associated with it is proportionally to the square of the previous error,
i.e. it has quadratic convergence. For more details the lector is referred to Chapra&Canale
(1988).

University of Castilla-La Mancha Draft By: Eduardo W. V. Chaves (2016)


Ciudad Real – Spain
ANNEX C: INCREMENTAL-ITERATIVE STRATEGY SOLUTION 671

f ( x) f ′( xi )

f ( xi )
tan(α) = f ′( xi ) =
∆x
f (x)

f ( xi )

xi +1 xi x

∆x

Figure C.19: Newton-Raphson’s method.

In general, in structure analysis it is taken as a direct solution procedure to solve the set of
equations. For a non-linear problem an incremental/iterative strategy is adopted. In this
way we apply and increment of load (load step) and in each load step we apply iterative
procedure in order to achieve the equilibrium point.
r r
Let us consider that the function is the residue g (u) and we know its value and its
r r
∂g (u0 ) r
derivative, r , at the application point u0 . Then, by means of Taylor series we can
∂u
obtain:
r r
r r r r ∂g (u0 ) r
g (u) = g (u0 ) + r ⋅ ∆u (C.46)
∂u
or in indicial notation:
r r
r r ∂g i (u0 ) r ∂g i (u0 )
g i (u) = g i (u0 ) + ∆u j = g i ( u0 ) + (u j − u 0 j )
∂u j ∂u j (C.47)
r tan
= g i (u0 ) − K ij (u j − u 0 j )
r
∂g i (u0 ) r
where = − K ijtan (u0 ) is the Jacobian matrix which in structural analysis ambit
∂u j
r
represents the tangent stiffness matrix at the application point u0 . As we are looking for
r r r
the value in which g (u) = 0 , we can state that:
r r r
g i (u) = g i (u0 ) − K ijtan (u j − u 0 j ) = 0 i ⇒ − g i (u0 ) = − K ijtan (u j − u 0 j )
r
⇒ −( K kitan ) −1 g i (u0 ) = −( K kitan ) −1 K ijtan (u j − u 0 j )
r (C.48)
⇒ ( K kitan ) −1 g i ( u0 ) = δ kj (u j − u 0 j ) = (u k − u 0 k )
r
⇒ u k = u 0 k + ( K kitan ) −1 g i (u0 )
With that we can conclude:

University of Castilla-La Mancha Draft By: Eduardo W. V. Chaves (2016)


Ciudad Real – Spain
672 SOLVING PROBLEMS BY MEANS OF CONTINUUM MECHANICS

r
u i = u 0 i + ( K ijtan ) −1 g j (u0 ) = u 0 i + ∆u i (C.49)

g (u ) g ′(u i )

g (u )
g (u i )

u0 i ui u

∆u i

Figure C.20: Residue function.

We can generalized the equation in (C.46) such as:


r r
r r r r ∂g (u0 ) r
g ( u) = g ( u 0 ) + r ⋅ ∆u
∂u
r r
r r r r  ∂g (u0 )  r
⇒ g (u) − g (u0 ) =  r  ⋅ ∆u
 ∂u  (C.50)
r r (k )
r  ∂g (u0 )  r
⇒ ∆g ( k ) =  r  ⋅ ∆u ( k )
 ∂u 
r (k ) r
⇒ ∆g = H ( k ) ⋅ ∆u ( k )

where (k ) means iterations. When H (k ) represents the tangent matrix and changes for
each iteration we fallback into the classical Newton-Raphson’s method. When the matrix
H (k ) does not change during the iterations we are dealing with the so-called Modified
Newton-Raphson’s method. We can also state that:
r r
∆g ( k ) = H ( k ) ⋅ ∆u ( k )
r r (C.51)
⇒ ∆g ( k ) = ( H ( k −1) + E ( k ) ) ⋅ ∆u ( k )

where E (k ) is a correction matrix. In a generic way we can adopt:


r r r r
E (k ) = αa ⊗ a + β b ⊗ b (C.52)
r r r r
where we can adopt a = u (k ) , b = H ( k −1) ⋅ g ( k ) . With that the above equation becomes:
r r r r
E ( k ) = αu ( k ) ⊗ u ( k ) + β ( H ( k −1) ⋅ g ( k ) ) ⊗ ( H ( k −1) ⋅ g ( k ) ) (C.53)
The values for α and β can be determined in a such a way that the equation in (C.51) is
fulfilled. For example:
r r 1 1
αa ⋅ g ( k ) = 1 ⇒ α = r r (k ) = r (k ) r (k ) (C.54)
a⋅g u ⋅g

University of Castilla-La Mancha Draft By: Eduardo W. V. Chaves (2016)


Ciudad Real – Spain
ANNEX C: INCREMENTAL-ITERATIVE STRATEGY SOLUTION 673

r r −1 −1 −1
β b ⋅ g ( k ) = −1 ⇒ β = r r (k ) = r (k ) r (k ) = r (k ) r (C.55)
b⋅g (H ( k −1)
⋅ g )⋅ g g ⋅ H ( k −1) ⋅ g ( k )
In this way we can obtain:
r r r r
u ( k ) ⊗ u ( k ) ( H ( k −1) ⋅ g ( k ) ) ⊗ ( H ( k −1) ⋅ g ( k ) )
E (k ) = r (k ) r (k ) − r r (C.56)
u ⋅g g ( k ) ⋅ H ( k −1) ⋅ g ( k )

And in turn we define the matrix H (k ) as follows:


H ( k ) = H ( k −1) + E ( k )
r r r r
u ( k ) ⊗ u ( k ) ( H ( k −1) ⋅ g ( k ) ) ⊗ ( H ( k −1) ⋅ g ( k ) ) (C.57)
⇒H =H (k ) ( k −1)
+ r (k ) r (k ) − r r
u ⋅g g ( k ) ⋅ H ( k −1) ⋅ g ( k )
This method is called BFGS method (Broyden-Fletcher-Goldfarb-Shanno). Note that if we
know the inverse of H ( k −1) , the inverse of H (k ) can be easily obtained by means of
Problem 1.87.

C.6 Newton-Raphson’ Method

Let us return to our initial problem proposed in Section C.2 and let us assume that we
know the values Wn and z n . For the force increment ∆Wn +1 we can state that
Wn +1 = Wn + ∆Wn +1 . For the next load step, represented by n + 1 , we have that for the first
iteration (i) :
PREDICTION ( i = 0 )
Tangent matrix calculation:
 z  z i  1  z i  2   z  z  1  z  2 
N ni +1  0  n +1 
= EA   +  n +1   if
 → N n+1 ← N n = EA 0  n  +  n  
i =1 i

 L0  L0  2  L0    L0  L0  2  L0  
   
(C.58)

EA N ni +1
( K tan ) in+1 = ( z 0 + z i
n +1 ) 2
+ + KS
L30 L0
(C.59)
EA N
if i =1⇒ ( K tan ) in +1 ← (K tan
) n = 3 ( z0 + z n ) 2 + n + K S
L0 L0
Solve the system:

( )
−1
∆z ni +1 =  K ntan 
i i si i =1 i tan −1
+1  Rn +1 → ∆z n +1 = K n ∆Wn +1 (C.60)
 
where Rni +1 is the residue. Once the above equation is solved, we can obtain:

z ni +1 = z n + ∆z ni +1 (C.61)
And in turn we can calculate the internal forces for this iteration:

University of Castilla-La Mancha Draft By: Eduardo W. V. Chaves (2016)


Ciudad Real – Spain
674 SOLVING PROBLEMS BY MEANS OF CONTINUUM MECHANICS

 z  z ni +1  1  z ni +1 
2

N ni +1 = EA 0 

+ 
 


 (C.62)
 L0  L0  2  L0  
 
and
( z 0 + z ni +1 )
Wni+1 = N ni +1 + K S z ni +1 (C.63)
L0
and the residue can be obtained:
Rni +=12 = Wni+=11 − Wn +1 (C.64)
Next, we obtain a norm in order to check the convergence. For our one-dimensional
example the residue is a scalar, and we check whether this norm is less than a tolerance. If
so, we go to the next load step (n ← n + 1) . Otherwise, we have to do a new iteration
( i ← i + 1 ) until the convergence is achieved, (see Figure C.21).
The procedure in FORTRAN code can be found at the link: NON_LINEAR1D.FOR,
(starting at label 40).

i =1
W K ntan
+1 = K ntan
i=2
K ntan
+1

W n +1
Rni =+13
Rni =+12

Rni +=11
∆W n +1

∆z ni =+11 ∆z ni =+21 ∆z ni =+31


Wn

zn z ni =+11 = z n + ∆z ni =+11 z ni =+21 z ni =+31 z

Figure C.21: Newton-Raphson iterative procedure.

For our example we will consider an load increment equal to ∆W = 30 . With that we can
obtain the graph describe in Figure C.22. And in Figure C.23 the curve for several load
steps.
As we can see in Figure C.22, for each iteration, we need to calculate the tangent matrix.
This is a drawback of the Newton-Raphson’s method, since for a system with a large
number of variables it could be costly, from a computation point of view, to calculate the
stiffness matrix for each iteration.

University of Castilla-La Mancha Draft By: Eduardo W. V. Chaves (2016)


Ciudad Real – Spain
ANNEX C: INCREMENTAL-ITERATIVE STRATEGY SOLUTION 675

2nd load step

1st load step

see figure below

primer paso de carga ∆W = 30 iterations


∆W = 30 (load step)

convergence

Figure C.22: Newton-Raphson iterative procedure.

University of Castilla-La Mancha Draft By: Eduardo W. V. Chaves (2016)


Ciudad Real – Spain
676 SOLVING PROBLEMS BY MEANS OF CONTINUUM MECHANICS

Figure C.23: Example of Newton-Raphson iterative procedure.

C.7 Modified Newton-Raphson Method

Several methods have been formulated based on the Newton-Raphson’s method, e.g. the
modified Newton-Raphson method, which basically consists in adopt the same tangent
matrix for each iteration, (see Figure C.24). This method requires more iterations than the
Full Newton-Raphson method and beside has no convergence if we are dealing with
inflection point as the one describe in Figure C.22. Just to illustrate this method, we apply
the load increment equal to ∆W = 30 which is before the inflection point, (see Figure
C.25). As we can see the modified Newton-Raphson method needs more iterations to
achieve convergence.
The procedure in FORTRAN code can be found at the link: NON_LINEAR1D.FOR,
(starting at label 50).

University of Castilla-La Mancha Draft By: Eduardo W. V. Chaves (2016)


Ciudad Real – Spain
ANNEX C: INCREMENTAL-ITERATIVE STRATEGY SOLUTION 677

tan i =1 tan
K n +1 = K n
6444444444 474444444444
8
W

W n +1
Rni =+12

Rni +=11
∆W n +1

∆z ni =+11 ∆z ni =+21 ∆z ni =+31


Wn ... ...

zn z ni =+11 z ni =+21 z ni =+31 ... z

Figure C.24: Modified Newton-Raphson iterative procedure.

Full Newton-Raphson

Modified Newton-Raphson

exact

Figure C.25: Full Newton-Raphson vs. Modified Newton-Raphson.

University of Castilla-La Mancha Draft By: Eduardo W. V. Chaves (2016)


Ciudad Real – Spain
678 SOLVING PROBLEMS BY MEANS OF CONTINUUM MECHANICS

Incremental-Iterative Solution References

CHAPRA, S.C. & CANALE, R.P. (1988). Métodos Numéricos para Ingenieros. Com aplicaciones en
computadores personales. McGraw-Hill/Interamericana de México, S.A. de C.V.
GANABA, T.H. (1985). Nonlinear Finite Element Analysis of Plates and Slabs. Ph.D. Thesis. The
University of Warwick, Coventry, UK.
POSADA, L.M. (2007). Stability Analysis of Two-dimensional Truss Structures. MSc, Universität
Stuttgart.
CRISFIELD, M.A.(1991). Non-linear finite element analysis of solids and structures. Vol.1. John
Wiley & Sons, England.
CRISFIELD, M.A.(1997). Non-linear finite element analysis of solids and structures. Vol.2. John
Wiley & Sons, England.
RUBERT, J.B. (1993). Estudo do desempenho de algoritmos numéricos na solução de sistemas não-lenares
de estruturas formadas por barras de treliça. Dissertação de Mestrado, USP-Sao Carlos

University of Castilla-La Mancha Draft By: Eduardo W. V. Chaves (2016)


Ciudad Real – Spain
Bibliography

Basic Bibliography

ASARO, R.J. & LUBARDA, V.A. (2006). Mechanics of solids and materials. Cambridge University
Press, New York, USA.
BATRA, R. C. (2006). Elements of Continuum Mechanics. John Wiley & Sons Ltd., United
Kingdom.
CASANOVA, J.C. (1993). Ejercicios de elasticidad. Editorial UPV.
CHADWICK, P. (1976). Continuum mechanics concise theory and problems. George Allen & Unwin
Ltd.Great Britain.
CHAVES, E.W.V. (2013). Notes on Continuum Mechanics. Springer/CIMNE, Barcelona, Spain.
GOICOLEA, J.M. Mecánica del medio continuo web: http://w3.mecanica.upm.es/mmc-ig/
HOLZAPFEL, G.A. (2000). Nonlinear solid mechanics. John Wiley & Sons Ltd. England.
MASE, G.E. (1977). Teoría y problemas de mecánica del medio continuo. McGraw-Hill, USA.
OLIVER, X. & AGELET DE SARACÍBAR, C. (2000). Mecánica de medios continuos para ingenieros.
Ediciones UPC, Barcelona, España.
OLIVER, X. & AGELET DE SARACÍBAR, C. (2000). Cuestiones y problemas de mecánica de medios
continuos. Ediciones UPC, Barcelona, España.
ORTIZ BERROCAL, L. (1985). Elasticidad. E.T.S. de Ingenieros Industriales. Litoprint. U.P.
Madrid.
PARKER, D.F. (2003). Fields, Flows and Waves: An introduction to continuum models. Springer-
Verlag London, UK.
TAYLOR, J.R. (2005). Classical Mechanics. University Science Books. USA.
880 SOLVING PROBLEMS BY MEANS OF CONTINUUM MECHANICS

Complementary Bibliography

ALEXANDER, H. (1968). A constitutive relation for rubber-like material. Int. J. Eng. Sci., Vol.
6, pp. 549-563.
ANTMAN, S.S. (1995). Nonlinear Problems of Elasticity. Springer-Verlag, New York.
ARRUDA, E.M. & BOYCE, M.C. (1993). A three-dimensional constitutive model for the large
stretch behavior of rubber elastic materials. J. Mech. Phys. Solids, Vol. 41, Nº. 2, pp. 389-
412.
ASARO, R.J. & LUBARDA, V.A. (2006). Mechanics of solids and materials. Cambridge University
Press, New York, USA.
BAşAR, Y. & WEICHERT (2000). Nonlinear continuum mechanics of solids: fundamental concepts and
perspectives. Springer Verlag. Berlin.
BATHE, K-J (1996). Finite Element Procedures. Prentice Hall, New Jersey, USA.
BATRA, R. C. (2006). Elements of Continuum Mechanics. John Wiley & Sons Ltd., United
Kingdom.
BAŽANT, Z.P. & KIM, S.-S. (1979). Plastic-Fracturing Theory for Concrete. J. Engng. mech.
Div. ASCE, 105, 407.
BAŽANT, Z.P. & PLANAS, J. (1997). Fracture and size effect in concrete and other quasibrittle
materials, CRC Press LLC, USA.
BEER, F.P. & JOHNSTON, E.R: (1987). Vector Mechanics for Engineers: Dynamics. Seventh
Edition. 2 Volumes. McGraw-Hill Science/Engineering/Math; 4 edition.
BEER, F.P.; JOHNSTON, E.R & CLAUSEN, W.E. (2004). Instructor's and Solutions Manual to
Accompany Vector Mechanics for Engineers - Dynamics. Seventh Edition. 2 Volumes.
McGraw Hill Higher Education; Seventh edition (2004)
BIGONI, D. (2000). Bifurcation and instability of non-associative elastoplastic solids. CISM
Lecture Notes. Material Instabilities in elastic an plastic Solids, H. Petryk (IPPT, Warsaw)
Coordinator.
BIOT, M.A. (1954). Theory of stress-strain relations in anisotropic viscoelasticity and
relaxation phenomena. Jour. Appl. Phys., pp. 1385-1391.
BIOT, M.A. (1955). Variational principles in irreversible thermodynamics with application to
viscoelasticity. The Physical Reviwe 97, pp. 1463-1469.
BIOT, M.A. (1956). Thermoelasticity and irreversible thermodynamics. Jour. Appl. Phys., pp.
240-253.
BLATZ, P.J. & KO, W.L. (1962). Application of finite elasticity theory to the deformation of
rubbery material. Transactions of the Society of Rheology, Vol. VI, pp. 223-251.
BONET, J. & WOOD, R.D. (1997). Nonlinear continuum mechanics for finite element
analysis. Cambridge University Press, USA.
BUCHANAN, G.R. (1988). Mechanics of Materials. Holt, Rinehart and Winston, Inc., New
York, USA.
BUECHE, F. (1960). Molecular basis for the Mullins effect. J. Appl. Poly. Sci., 4(10), 107-114.
BUECHE, F. (1961). Mullins effect and rubber-filler interaction. J. Appl. Poly. Sci., 5(15), 271-
281.
BIBLIOGRAPHY 881

CAROL, I. & WILLAM, K. (1997) Application of analytical solutions in elasto-plasticity to


locaization analysis of damage models. In Owen,. D.R.J., Oñate, E., and Hinton, E.
(Eds.), Computational Plasticity (COMPLAS V), pp. 714-719, Barcelona. Pineridge Press.
CAROL, I.; RIZZI, E. & WILLAM, K. (1998) On the formulation of isotropic and anisotropic
damage. Computational Modelling of Concrete Structures. de borst, Bićanić, Mang & Meschke
(eds.). Balkema, Rotterdam, ISBN 9054109467. pp. 183-192.
CASANOVA, J.C. (1993). Ejercicios de elasticidad. Editorial UPV.
CERVERA RUIZ, M. & BLANCO DÍAZ, E. (2001). Mecánica de Estructuras Libro 1 – Resistencia de
materiales. Edicions UPC, Barcelona. Eapaña.
CHABOCHE, J.L. (1979). Le concept de contrainte effective appliqué à l’élasticité et à la
viscoplasticité en presence d’un endommagement anitrope. Colloque EUROMECH
115, Grenoble Edition du CNRS.
CHADWICK, P. (1976). Continuum mechanics concise theory and problems. George Allen & Unwin
Ltd.Great Britain.
CHANDRASEKHARAIAH, D.S. & DEBNATH, L. (1994). Continuum mechanics. Academic Press,
San Diego (CA, U.S.A.).
CHAVES, E.W.V. (2013). Notes on Continuum Mechanics. Springer/CIMNE, Barcelona, Spain.
CHAVES, E.W.V. & MÍNGUEZ, R. (2009). Mecánica Computacional en la Ingeniería con Aplicaciones
en MATLAB. Editorial UCLM, ISBN:978-84-692-8273-1. (in Spanish).
CHEN, A. & CHEN, W.F.(1975). Constitutive relations for concrete. J. Eng. Mech.-ASCE,
101:465-481.
CHEN, W.F. & HAN, D.J.(1988). Plasticity for Structural Engineers. Springer-Verlag New Yor
Inc.
CHEN, W.F. (1982). Plasticity in reinforced concrete. McGraw-Hill, Inc. USA.
CHUNG, T.J. & KIM, J.Y. (1984). Two-dimensional, combined-mode heat transfer by
conduction, convective and radiation in emitting, Absorbing, and scattering media.
ASME Trans. J. Heat Transfer 106, pp.:448-452.
CHUNG, T.J. (1996). Applied continuum mechanics, Cambridge University Press.
COLEMAN, B.D. & GURTIN, M.E. (1967). Thermodynamics with internal sate variables. J.
Chem. Phys. 47, pp. 597-613.
COLEMAN, B.D. (1964). Thermodynamics of materials with memory. Arch. Rat. Mech. Anal.
17, pp. 1-46.
CRISFIELD, M.A. (1997). Non-Linear Finite Element Analysis of Solids and Structures, volume 1,2.
John Wiley and Sons, New York, USA.
CRISTENSEN, R. M. (1982). Theory of Viscoelasticity, second edition. Dover Publications, Inc.,
New York, USA.
DESAI, C.S. & SIRIWARDANE, H.J. (1984). Constitutive laws for engineering materials with emphasis
on geological materials. Prentice-Hall, Inc.USA.
DÍAZ DEL VALLE, J. (1984). Mecánica de los Medios Continuos I. Servicio de publicaciones
E.T.S. de Ingenieros de Caminos, C. y P. Santander.
DOBLARÉ, M. & ALARCÓN, M. (1983). Elementos de Plasticidad. Sevicio de publicaciones de la
E.T.S.I. Industriales. Madrid.
882 SOLVING PROBLEMS BY MEANS OF CONTINUUM MECHANICS

DRAGON, A. & MRÓZ, Z. (1979). A Continuum Model for Plastic-Brittle Behavior of Rock
and Concrete. Int. J. Engng. Sci., 17, 121.
DRUCKER, D.C. (1950). Stress-strain relations in the plastic range – A survey of the theory
and experiment. Report to the Office of Naval Research, under contact N7-onr-358,
Division of Applied Mathematics, Brown University, Providence, RI.
FARIA, R. & OLIVER, X. (1993). A rate dependent plastic-damage constitutive model for
large scale computations in concrete structures. Monograph CIMNE Nº17, International
Center for Numerical Methods in Engineering, Barcelona, Spain.
FELIPPA,C.A. (2002). Introduction to Finite Element Methods. Course Notes, see World Wide
Web.
FINDLEY, W.N.; LAI, J.S. & ONARAN, K.(1989). Creep and relaxation of nonlinear viscoelastic
materials: with an introduction to linear viscoelasticity. Dover Publications, Inc. NY.
FUNG, Y.C. (1965). Foundations of solids mechanics. Prentice Hall Inc., New Jersey.
FUNG, Y.C. (1977). A first course in continuum mechanics. Prentice Hall Inc., New Jersey.
GENT, A.N. (1996). A new constitutive relation for rubber. Rubber Chem. Technol., Vol. 69.
pp. 59-61.
GERE, J.M. & WEAVER JR., W. (1965). Analysis of Framed Structures. Van Nostrand Reinhold,
U.S.
GREEN, A.E. & NAGHDI, P.M. (1965). A general theory of an elasto-plastic continuum.
Arch. Rat. Mech. Anal., 18.
GREEN, A.E. & NAGHDI, P.M. (1971). Some remarks on elastic-plastic deformation at finite
strain. Int. J. Engng. Sci., Vol. 9, pp. 1219-1229.
GURTIN, M. E. (1996). An introduction to continuum mechanics. NY: Academic Press, Inc.
GURTIN, M.E. & FRANCIS, E.C. (1981). Simple rate-independent model for damage. J.
Spacecraft, 18(3) pages: 285-286.
HAUPT, P. (2002). Continuum mechanics and theory of materials. Springer-Verlag, Gemany.
HILL, R. (1950). The mathematical theory of plasticity. Oxford University Press.
HILL, R. (1959). Some Basic Principles in the Mechanics of Solids without a natural Time. J.
Mech. Phys. Solids, 7, 209.
HOLZAPFEL, G.A. (2000). Nonlinear solid mechanics. John Wiley & Sons Ltd. England.
IORDACHE, M.-M. (1996). Failure Analysis of Classical and Micropolar Elastoplastic Materials.
Ph.D. Dissertation, University of Colorado at Bolder.
JIRÁSEK, M. (1998). Finite elements with embedded cracks. LSC Internal Report 98/01,
April.
JU, J.W. (1989). Energy-based coupled elastoplastic damage models at finite strains. Journal
of Engineering Mechanics, Vol.115, No. 11, pp-2507-2525.
KACHANOV, L. M. (1986). Introduction to Continuum, Damage Mechanics. Nijhoff, Dordrecht,
The Netherlands.
KOITER, W.T. (1953). Stress-strain relations, uniqueness and variational theorems for
elastic-plastic material with singular yield surface. Q. Appl. Math. 11, 350-54.
LAI, W.M.; RUBIN, D. & KREMPL, E.(1978). Introduction to Continuum Mechanics. Pergamon
Press.
BIBLIOGRAPHY 883

LAIER, J.E.; BAREIRO, J.C., (1983). Complemento de resistência dos materiais. Publicação 073/92
São Carlos - USP - EESC.
LANCZOS, C. (1970). The variational principles of mechanics. Dover Publications, Inc., New
York.
LEE, E.H. (1969). Elastic-Plastic deformation at finite strains. Journal of Applied Mechanics,
Vol. 36, pp. 1-6.
LEE, E.H. (1981). Some comments on elastic-plastic analysis. Int. J. Solids Structures, Vol. 17,
pp. 859-872.
LEMAITRE, J. & CHABOCHE, J.-L. (1990). Mechanics of Solids materials. Cambridge University
Press, Cambridge.
LINDER, C. (2003). An arbritary lagrangian-Eulerian finite element formulation for dynamics and finite
strain plasticity models. Master’s Thesis. University Stuttgart.
LOVE, A.E.H. (1944). A treatise on the Mathematical Theory of Elasticity. Cambridge University
Press, London.
LU, S.C.H. & PISTER, K.S. (1975). Descomposition of deformation and representation of
the free energy function for isotropic solids, Int. J. of Solids and Structures, 11, pages: 927-
934.
LUBARDA, V.A. & BENSON D.J. (2001). On the partitioning of the rate of deformation
gradient in phenomenological plasticity. Int. J. of Solids and Structures, 38 pages: 6805-
6814.
LUBARDA, V.A. & KRAJCINOVIC, D. (1995). Some fundamental issues in rate theory of
damage-elastoplasticity. Int. J. of Plasticity, Vol. 11, Nº 7, pp. 763-797.
LUBARDA, V.A. (2004). Constitutive theories based on the multiplicative decomposition of
deformation gradient: Thermoelasticity, elastoplasticity, and biomechanics. Appl. Mech.
Rev., Vol. 57, no 2, March.
LUBLINER, J. (1990). Plasticity Theory. Macmillan Publishing Company, New York.
MALVERN, L.E. (1969). Introduction to the mechanics of a continuous medium. Prentice-Hall, Inc.
Englewood Cliffs, New Jersey.
MARSDEN, J. E. & HUGHES, T. J.R. (1983). Mathematical foundations of elasticity. Dover
Publications, Inc., New York.
MASE, G.E. (1977). Mecánica del Medio Continuo. McGraw-Hill, USA.
MAZARS, J. & LEMAITRE, J. (1984). Application of continuous damage mechanics to strain
and fracture behavior of concrete. Advances of Fracture Mechanics to Cementitious
Composites, NATO Advanced Research Workshop, 4-7 September 1984, Northwestern
University (Edited by S.P. Shah), pp. 375-388.
MAZARS, J. (1982). Mechanical damage and fracture of concrete structures. Advances in
Fracture Research (Fracture 81), Vol. 4, pp. 1499-1506. Pergamon Press, Oxford.
MENDELSON, A. (1968). Plasticity: Theory and application. New York, Robert E. Krieger
Publishing Company.
MOONEY, M. (1940). A theory of large elastic deformation. Journal of Applied Physics, Vol.
11, pp. 582-92.
MORMAN, K.N. (1986). The generalized strain measure with application to
nonhomegeneous deformation in rubber-like solids. J. Appl. Mech., Vol. 53, pp. 726-
728.
884 SOLVING PROBLEMS BY MEANS OF CONTINUUM MECHANICS

MORZ, Z. (1967). On the description of anisotropic work hardening. J. Mech. Phys. Solids,
15, 163.
MULLINS, L. (1969). Softening of rubber by deformation. Rubber Chemistry and Tecnology, 42,
339-351.
NAGHDI, P.M. (1960). Stress-strain relations in plasticity and thermoplasticity. in E.H. Lee
& P. Symonds, eds., Plasticity. Pergamon, Oxford, pp. 121-69.
NEMAT-NASSER, S. (1982). On finite deformation elasto-plasticity. Int. J. Solids Structures,
Vol. 18, pp. 857-872.
NOWACKI, W. (1967). Problems of thermoelasticity. Progress in Thermoelasticity, VIIIth
European Mechanics Colloquium, Warszawa.
OGDEN, R.W. (1984). Non-linear elastic deformations. Dover Publications, Inc., New York.
OLIVER, J. & AGELET DE SARACÍBAR, C. (2000). Mecánica de medios continuos para ingenieros.
Ediciones UPC, Barcelona, España.
OLIVER, X. & AGELET DE SARACÍBAR, C. (2000). Cuestiones y problemas de mecánica de medios
continuos. Ediciones UPC, Barcelona, España.
OLIVER, J. (2000). On the discrete constitutive models induced by strong discontinuity
kinematics and continuum constitutive equations. Int. J. Solids Struct., 37:7207-7229.
OLIVER, J. (2002). Topics on Failure Mechanics. Monograph CIMNE Nº 68, International
Center for Numerical Methods in Engineering, Barcelona, Spain.
OLIVER, J.O. ; CERVERA, M. ; OLLER, S. & LUBLINER, J. (1990). Isotropic damage models
and smeared crack analysis of concrete, SCI-C Second Int. Conf. On Computer Aided Design
of Concrete Structure, Zell am See, Austria. Pg – 945 – 957.
OLLER, S. (1988). Un modelo de daño continuo para materiales friccionales, Ph.D. thesis.
Universidad Politécnica de Cataluña, Barcelona, España.
OLLER, S. (2001). Fractura mecánica. Un enfoque global. CIMNE, Barcelona, España.
OLLER, S.; OÑATE, E.; OLIVER, J. & LUBLINER, J. (1990). Finite element non-linear analysis
of concrete structures using a plastic-damage model. Engineering Fracture Mechanics Vol.
35, pp. 219-231.
OÑATE, E. (1992). Cálculo de estructuras por el método de elementos finitos análisis estático lineal.
Centro Internacional de Métodos Numéricos en Ingeniería. Barcelona- España.
ORTIZ BERROCAL, L. (1985). Elasticidad. E.T.S. de Ingenieros Industriales. Litoprint. U.P.
Madrid.
ORTIZ, M. (1985). A Constitutive theory for inelastic behavior of concrete. Mech. Mater.,
Vol. 4, 67.
PABST, W. (2005). The linear theory of thermoelasticity from the viewpoint of rational
thermomechanics. Ceramics - Silikáty, 49 (4) 242-251.
PARKER, D.F. (2003). Fields, Flows and Waves: An introduction to continuum models. Springer-
Verlag London, UK.
PILKEY, W. & WUNDERLICH, W. (1992). Mechanics of structures. Variational and computational
methods. CRC Press, Inc., Florida, USA.
POWERS, J.M. (2004). On the necessity of positive semi-definite conductivity and Onsager
reciprocity in modeling heat conduction in anisotropic media. Journal of Heat Transfer,
Vol.126, pp. 670-675.
BIBLIOGRAPHY 885

PRAGER, W. (1945). Strain hardening under combined stress. J. Appl. Phys. 16, 837-40.
PRAGER, W. (1955). The theory of plasticity: A survey of recent achievements. Proc. Inst.
Mech. Eng. 169:41.
PRANDTL, L. (1924). Spannungverteilung in plastischen korpen, in Proceedings of the First
International. Congress of Applied Mechanics, Delft, The Netherlands, Vol. 43.
RABOTNOV, Y.N. (1963). On the equations of state for creep. Progress in Applied Mechanics,
Prager Anniversary Volume, page 307, New York, MacMillan.
RANKINE, W.J.M. (1951). Laws of the elasticity of solids bodies. Cambridge Dublin Math.
J. 6, 41-80.
REUSS, A. (1930). Berrucksichtingung der elastischen formanderungen in der
plastizitatstheorie, Z. Angew. Math. Mech. 10, 266.
ROMANO, A.; LANCELLOTA, R. & MARASCO, A. (2006). Continuum Mechanics using
Mathematica: fundamentals, applications, and scientific computing. Birkhauser Boston. USA.
RUNESSON, K. & MROZ, Z. (1989). A note on nonassociated plastic flow rules. Int. J.
Plasticity, 5, 639-658.
RUNESSON, K. & OTTOSEN, N. (1991). Discontinuity bifurcation of elastic-plastic solutions
at plane stress and plane strain. Int. J. Plasticity, 7:99-121.
SANSOUR, C. (1998). Large strain deformations of elastic shell. Constitutive modeling and
finite element analysis. Comp. Mech. Appl. Mech. Engrg. 161, pp1-18.
SANSOUR, C.; FEIH, S. & WAGNER, W. (2003). On the performance of enhanced strain
finite elements in large strain deformations of elastic shells. Comparison of two classes
of constitutive models for rubber materials. Report Universitat Karlsruhe, Institut für
Baustatik
SECHLER, E. (1952). Elasticity in Engineering. John Willey & Sons, Inc. new York.
ŠILHAVÝ, M. (1997). The mechanics and thermodynamics of continuous media. Springer-
Verlag, Germany.
SIMO, J. & HUGHES, T.J.R. (1998). Computational Inelasticity. Springer-Verlag, New York.
SIMO, J.C. & JU, J.W. (1987a). Strain and Stress – Based Continuum damage Models – I.
Formulation. International Journal Solids Structures, Vol. 23, pp. 821-840.
SIMO, J.C. & JU, J.W. (1987b). Strain and Stress – Based Continuum damage Models – II.
Computational aspects. International Journal Solids Structures, Vol. 23, pp. 841-869.
SIMO, J.C. (1988a). A framework for finite strain elastoplasticity based on maximum plastic
dissipation: Part I. Computer Methods in Applied Mechanics and Engineering, Vol. 66, pp. 199-
219.
SIMO, J.C. (1988b). A framework for finite strain elastoplasticity based on maximum plastic
dissipation: Part II. Computer Methods in Applied Mechanics and Engineering, Vol. 68, pp. 1-
31.
SIMO, J.C. (1992). Algorithms for static and dynamic multiplicative plasticity that preserve
the classical return mapping schemes of the infinitesimal theory, Computer Methods in
Applied Mechanics and Engineering, Vol. 99, pp. 61-112.
SOKOLNIKOFF, I.S. (1956). Mathematic theory of elasticity. New York, McGraw-Hill.
SOUZA NETO, E.A.; PERIĆ, D. & OWEN, D.R.J. (1998). A phenomenological three-
dimensional rate-independent continuum damage model for highly filled polymers:
Formulation and computational aspects. J. Mech. Phys. Solids. 42(10), pp. 1533-1550.
886 SOLVING PROBLEMS BY MEANS OF CONTINUUM MECHANICS

SOUZA NETO, E.A.; PERIĆ, D. & OWEN, D.R.J. (1998). Continuum Modelling and
Numerical Simulation of Material Damage at Finite Strains, Archives of Computational
Methods in Engineering, Vol.5,4, pp. 311-384.
SPENCER, A.J.M. (1980). Continuum Mechanics, Longmans, Hong-Kong.
TAYLOR, J.R. (2005). Classical Mechanics. University Science Books. USA.
TIMOSHENKO, S. & GOODIER, J.N. (1951). Theory of elasticity, 2nd edition, McGraw-Hill.
TRELOAR, L.R.G. (1944). Stress-strain data for vulcanized rubber under various types of
deformation. Proc. of the Faraday Soc, 40:59-70.
TRELOAR, L.R.G. (1975). The physics of rubber elasticity. Clarendon Press, Oxford.
TRESCA, H. (1864). Mémire sur l’Ecoulement des corps solids soumis a de fortes pressions
comptes rendua academie de sciences. Paris, France, Vol.59, p.754..
TRUESDELL, C.A. & NOLL, W. (1965). The non-linear field theories of mechanics, in
Handuch der Physik, Vol. III/3, S. Flügge (Ed.), Springer-Verlag, Berlin.
UGURAL, A.C. (1981). Stress in Plates and Shells. McGraw Hill.
UGURAL, A.C. & FENSTER, S.K. (1981). Advanced strength and applied elasticity. Edward Arnold,
London - U.K.
VON MISES, R. (1930). Über die bisherigen Ansätze in der lassischen MEchanik der
Kontinua. in Proceedings of the Third Intenational Congress for Applied Mechanics.
Vol.2, pp1-9.
VUJOŠEVIĆ, L. & LUBARDA, V.A. (2002). Finite-Strain thermoelasticity based on
multiplicative decomposition of deformation gradient. Theoretical and Applied Mechanics,
vol. 28-29, pp. 379-399, Belgrade.
WILLAM, K. (2000). Constitutive models for materials: Encyclopedia of Physical Science & Technology,
3rd edition. Academic Press.
YEOH, O.H. (1993). Some forms of the strain energy function for rubber. Rubber Chem.
Technol., Vol. 66, pp. 754-71.
ZIEGLER, H. (1959). A modification of Prager’s hardening rule. Q. Appl. Math. 17, 55-64.
ZIENKIEWICZ, O.C. & TAYLOR, R.L. (1994a). El método de los elementos finitos. Volumen 1:
Formulación básica y problemas lineales. CIMNE, Barcelona, 4ª edición.
ZIENKIEWICZ, O.C. & TAYLOR, R.L. (1994b). El método de los elementos finitos. Volumen 2:
Mecánica de sólidos y fluidos. Dinámica y no linealidad. CIMNE, Barcelona, 4ª edición.

You might also like